You are on page 1of 351

About Thomson Peterson’s

Thomson Peterson’s (www.petersons.com) is a leading provider of education information and advice, with books
and online resources focusing on education search, test preparation, and financial aid. Its Web site offers
searchable databases and interactive tools for contacting educational institutions, online practice tests and
instruction, and planning tools for securing financial aid. Thomson Peterson’s serves 110 million education
consumers annually.

Petersons.com/publishing
Check out our Web site at www.petersons.com/publishing to see if there is any new information regarding the test
and any revisions or corrections to the content of this book. We’ve made sure the information in this book is
accurate and up-to-date; however, the test format or content may have changed since the time of publication.

For more information, contact Thomson Peterson’s, 2000 Lenox Drive, Lawrenceville, NJ 08648;
800-338-3282; or find us on the World Wide Web at www.petersons.com/about.

© 2004 Thomson Peterson’s, a part of The Thomson Corporation


Thomson Learning™ is a trademark used herein under license.

Chapter 5: GRE Word List is excerpted from The Insider’s Guide to the GRE CAT.
2nd Edition, by Karl Weber. © Petersons, 2002.

Editor: Mandie Rosenberg; Production Editor: Megan Hellerman; Manufacturing


Manager: Ray Golaszewski; Composition Manager: Gary Rozmierski; Cover and
Interior Design: Allison Sullivan; CD Producer: Carol Aickley; CD Quality
Assurance: Jeff Pagano.

Contributing Writer: Keith Cox

ALL RIGHTS RESERVED. No part of this work covered by the copyright herein
may be reproduced or used in any form or by any means—graphic, electronic, or
mechanical, including photocopying, recording, taping, Web distribution, or
information storage and retrieval systems—without the prior written permission of
the publisher.

For permission to use material from this text or product, submit a request online
at www.thomsonrights.com
Any additional questions about permissions can be submitted by e-mail to
thomsonrights@thomson.com

ISBN 0-7689-1432-9

Printed in the United States of America

10 9 8 7 6 5 4 3 2 1 06 05 04

First Edition
Contents
123456789012345678901234567890121234567890123456789012
12345678901234567890123456789012123456789012345678901 2
12345678901234567890123456789012123456789012345678901 2
12345678901234567890123456789012123456789012345678901 2
12345678901234567890123456789012123456789012345678901 V 2
12345678901234567890123456789012123456789012345678901
ABOUT THIS TOOL KIT. . . . . . . . . . . . . . . . . . . . . . . 2
12345678901234567890123456789012123456789012345678901 2
12345678901234567890123456789012123456789012345678901
Your Tools . . . . . . . . . . . . . . . . . . . . . . . . . . . . . . . . . .
2345678901234567890123456789012123456789012345678901
v 2
2
1
12345678901234567890123456789012123456789012345678901
Structure of the GRE CAT . . . . . . . . . . . . . . . . . . . . . vii 2
12345678901234567890123456789012123456789012345678901 2
12345678901234567890123456789012123456789012345678901 2
12345678901234567890123456789012123456789012345678901
2345678901234567890123456789012123456789012345678901
2
1
12345678901234567890123456789012123456789012345678901
INTRODUCTION .............................. 1 22
12345678901234567890123456789012123456789012345678901 2
12345678901234567890123456789012123456789012345678901
Does This CAT Have Claws? . . . . . . . . . . . . . . . . . . . . . . 1 2
12345678901234567890123456789012123456789012345678901 2
12345678901234567890123456789012123456789012345678901
The Unique Peterson’s Approach—Adaptive 2
12345678901234567890123456789012123456789012345678901 2
12345678901234567890123456789012123456789012345678901
Preparation for an Adaptive Test. . . . . . . . . . . . . . . . 3 2
12345678901234567890123456789012123456789012345678901 2
12345678901234567890123456789012123456789012345678901
2345678901234567890123456789012123456789012345678901
2
2
1
12345678901234567890123456789012123456789012345678901 4 2
12345678901234567890123456789012123456789012345678901
HOW TO USE THIS BOOK . . . . . . . . . . . . . . . . . . . . 2
12345678901234567890123456789012123456789012345678901 2
12345678901234567890123456789012123456789012345678901 2
12345678901234567890123456789012123456789012345678901 22
2345678901234567890123456789012123456789012345678901
1
PART I: UNDERSTANDING THE GRE
12345678901234567890123456789012123456789012345678901 2
12345678901234567890123456789012123456789012345678901 2
12345678901234567890123456789012123456789012345678901
Chapter 1: Everything You Need to Know about 2
12345678901234567890123456789012123456789012345678901 22
2345678901234567890123456789012123456789012345678901
1 the GRE CAT . . . . . . . . . . . . . . . . . . . . . . . . . . . . 6
12345678901234567890123456789012123456789012345678901 2
12345678901234567890123456789012123456789012345678901
Chapter 2: Section Strategies . . . . . . . . . . . . . . . . . 19 2
12345678901234567890123456789012123456789012345678901 2
12345678901234567890123456789012123456789012345678901 22
2345678901234567890123456789012123456789012345678901
12345678901234567890123456789012123456789012345678901 2
12345678901234567890123456789012123456789012345678901 2
12345678901234567890123456789012123456789012345678901
PART II: DIAGNOSING STRENGTHS AND
12345678901234567890123456789012123456789012345678901 2
12345678901234567890123456789012123456789012345678901
WEAKNESSES 2
12345678901234567890123456789012123456789012345678901 2
12345678901234567890123456789012123456789012345678901 2
12345678901234567890123456789012123456789012345678901 22
Chapter 3: Diagnostic: Dipping Your Big Toe in
1
12345678901234567890123456789012123456789012345678901
the GRE Waters . . . . . . . . . . . . . . . . . . . . . . . . . . 32 2
12345678901234567890123456789012123456789012345678901 2
12345678901234567890123456789012123456789012345678901 2
12345678901234567890123456789012123456789012345678901 2
12345678901234567890123456789012123456789012345678901 2
12345678901234567890123456789012123456789012345678901
PART III: SECTION REVIEW 2
12345678901234567890123456789012123456789012345678901 2
12345678901234567890123456789012123456789012345678901
Chapter 4: Quantitative Review. . . . . . . . . . . . . . . 48 2
12345678901234567890123456789012123456789012345678901 2
12345678901234567890123456789012123456789012345678901
Take It to the Next Level. . . . . . . . . . . . . . . . . . . 106 2
12345678901234567890123456789012123456789012345678901 22
2345678901234567890123456789012123456789012345678901
12345678901234567890123456789012123456789012345678901
1 Chapter 5: Verbal Review . . . . . . . . . . . . . . . . . . . . 134 2
12345678901234567890123456789012123456789012345678901 2
12345678901234567890123456789012123456789012345678901
Take It to the Next Level. . . . . . . . . . . . . . . . . . . 161 2
12345678901234567890123456789012123456789012345678901 2
12345678901234567890123456789012123456789012345678901
Chapter 6: Analytical Writing Review. . . . . . . . . . 227 2
12345678901234567890123456789012123456789012345678901 2
12345678901234567890123456789012123456789012345678901
Take It to the Next Level. . . . . . . . . . . . . . . . . . . 245 2
12345678901234567890123456789012123456789012345678901 2
12345678901234567890123456789012123456789012345678901 22
2345678901234567890123456789012123456789012345678901
12345678901234567890123456789012123456789012345678901 2
12345678901234567890123456789012123456789012345678901 2
12345678901234567890123456789012123456789012345678901 2
1
PART IV: THREE PRACTICE GREs
12345678901234567890123456789012123456789012345678901 2
12345678901234567890123456789012123456789012345678901
Chapter 7: Preface to Practice Tests. . . . . . . . . . . . 256 2
12345678901234567890123456789012123456789012345678901 2
12345678901234567890123456789012123456789012345678901
Practice Test 1 . . . . . . . . . . . . . . . . . . . . . . . . . . . . . . 258 2
12345678901234567890123456789012123456789012345678901 2
2345678901234567890123456789012123456789012345678901
Answers and Explanations . . . . . . . . . . . . . . . . . 271 2
12345678901234567890123456789012123456789012345678901
1 2
12345678901234567890123456789012123456789012345678901 2
12345678901234567890123456789012123456789012345678901
Practice Test 2 . . . . . . . . . . . . . . . . . . . . . . . . . . . . . . 282 2
12345678901234567890123456789012123456789012345678901
2345678901234567890123456789012123456789012345678901
2
1 Answers and Explanations . . . . . . . . . . . . . . . . . 294 2
12345678901234567890123456789012123456789012345678901 2
12345678901234567890123456789012123456789012345678901 22
12345678901234567890123456789012123456789012345678901
123456789012345678901234567890121234567890123456789012
iii
123456789012345678901234567890121234567890123456789012
12345678901234567890123456789012123456789012345678901 2
12345678901234567890123456789012123456789012345678901 2
12345678901234567890123456789012123456789012345678901
Practice Test 3 . . . . . . . . . . . . . . . . . . . . . . . . . . . . . . 305 2
12345678901234567890123456789012123456789012345678901 2
12345678901234567890123456789012123456789012345678901 2
12345678901234567890123456789012123456789012345678901 2
Answers and Explanations . . . . . . . . . . . . . . . . . 318
12345678901234567890123456789012123456789012345678901 2
12345678901234567890123456789012123456789012345678901
Chapter 8: Closing Remarks. . . . . . . . . . . . . . . . . . 329 2
12345678901234567890123456789012123456789012345678901 2
12345678901234567890123456789012123456789012345678901 2
12345678901234567890123456789012123456789012345678901 2
12345678901234567890123456789012123456789012345678901
2345678901234567890123456789012123456789012345678901
2
2
1
PART V: APPENDIX
12345678901234567890123456789012123456789012345678901 2
12345678901234567890123456789012123456789012345678901
About the CAT . . . . . . . . . . . . . . . . . . . . . . . . . . . . . 332 2
12345678901234567890123456789012123456789012345678901 2
12345678901234567890123456789012123456789012345678901
2345678901234567890123456789012123456789012345678901
2
2
1
12345678901234567890123456789012123456789012345678901 2
12345678901234567890123456789012123456789012345678901 2
12345678901234567890123456789012123456789012345678901 2
12345678901234567890123456789012123456789012345678901
2345678901234567890123456789012123456789012345678901
2
2
1
12345678901234567890123456789012123456789012345678901 2
12345678901234567890123456789012123456789012345678901 2
12345678901234567890123456789012123456789012345678901 2
12345678901234567890123456789012123456789012345678901 2
12345678901234567890123456789012123456789012345678901 2
2345678901234567890123456789012123456789012345678901
12345678901234567890123456789012123456789012345678901 2
12345678901234567890123456789012123456789012345678901 2
12345678901234567890123456789012123456789012345678901 2
12345678901234567890123456789012123456789012345678901 2
12345678901234567890123456789012123456789012345678901 2 2
1
12345678901234567890123456789012123456789012345678901 2
12345678901234567890123456789012123456789012345678901 2
12345678901234567890123456789012123456789012345678901 2
12345678901234567890123456789012123456789012345678901 2
2345678901234567890123456789012123456789012345678901
12345678901234567890123456789012123456789012345678901 2
12345678901234567890123456789012123456789012345678901 2
12345678901234567890123456789012123456789012345678901 2
12345678901234567890123456789012123456789012345678901 2
12345678901234567890123456789012123456789012345678901 2
12345678901234567890123456789012123456789012345678901 2
12345678901234567890123456789012123456789012345678901 2
12345678901234567890123456789012123456789012345678901 2 2
1
12345678901234567890123456789012123456789012345678901 2
12345678901234567890123456789012123456789012345678901 2
12345678901234567890123456789012123456789012345678901 2
12345678901234567890123456789012123456789012345678901 2
12345678901234567890123456789012123456789012345678901 2
12345678901234567890123456789012123456789012345678901 2
12345678901234567890123456789012123456789012345678901 2
12345678901234567890123456789012123456789012345678901 2
12345678901234567890123456789012123456789012345678901 2
12345678901234567890123456789012123456789012345678901 2
12345678901234567890123456789012123456789012345678901 2
2345678901234567890123456789012123456789012345678901
12345678901234567890123456789012123456789012345678901 2 2
1
12345678901234567890123456789012123456789012345678901 2
12345678901234567890123456789012123456789012345678901 2
12345678901234567890123456789012123456789012345678901 2
12345678901234567890123456789012123456789012345678901 2
12345678901234567890123456789012123456789012345678901 2
12345678901234567890123456789012123456789012345678901 2
12345678901234567890123456789012123456789012345678901 2
12345678901234567890123456789012123456789012345678901 2
2345678901234567890123456789012123456789012345678901
12345678901234567890123456789012123456789012345678901 2
12345678901234567890123456789012123456789012345678901 2
12345678901234567890123456789012123456789012345678901 2 2
1
12345678901234567890123456789012123456789012345678901 2
12345678901234567890123456789012123456789012345678901 2
12345678901234567890123456789012123456789012345678901 2
12345678901234567890123456789012123456789012345678901 2
12345678901234567890123456789012123456789012345678901 2
12345678901234567890123456789012123456789012345678901 2
2345678901234567890123456789012123456789012345678901
2
1
12345678901234567890123456789012123456789012345678901 2
12345678901234567890123456789012123456789012345678901 2
12345678901234567890123456789012123456789012345678901 2
12345678901234567890123456789012123456789012345678901 2
12345678901234567890123456789012123456789012345678901 2
12345678901234567890123456789012123456789012345678901 2
12345678901234567890123456789012123456789012345678901 2
123456789012345678901234567890121234567890123456789012
iv

www.petersons.com
About This Tool Kit

123456789012345678901234567890121234567890123456789012
12345678901234567890123456789012123456789012345678901 2
12345678901234567890123456789012123456789012345678901
2345678901234567890123456789012123456789012345678901
2
2
1 Tools
Your
12345678901234567890123456789012123456789012345678901 2
12345678901234567890123456789012123456789012345678901 2
12345678901234567890123456789012123456789012345678901
Peterson’s Ultimate GRE Tool Kit provides the complete package you 2
12345678901234567890123456789012123456789012345678901 2
12345678901234567890123456789012123456789012345678901 22
2345678901234567890123456789012123456789012345678901
need to score your personal best on the GRE and get into your top-choice
12345678901234567890123456789012123456789012345678901
program. Unlike any book previously published, this tool kit contains 2
12345678901234567890123456789012123456789012345678901
many features that used to be available only to those who purchased 2
12345678901234567890123456789012123456789012345678901
12345678901234567890123456789012123456789012345678901 2
12345678901234567890123456789012123456789012345678901 2
1 expensive test-prep classes. 2
12345678901234567890123456789012123456789012345678901 2
12345678901234567890123456789012123456789012345678901 2
12345678901234567890123456789012123456789012345678901 2
2345678901234567890123456789012123456789012345678901
12345678901234567890123456789012123456789012345678901 2
12345678901234567890123456789012123456789012345678901
e-Tutoring 2
12345678901234567890123456789012123456789012345678901
Use the CD to go on line to register for one-on-one math help from a live 2
2
12345678901234567890123456789012123456789012345678901
12345678901234567890123456789012123456789012345678901
expert whenever you need it. Tutoring is offered using an on-line 2
2
12345678901234567890123456789012123456789012345678901
12345678901234567890123456789012123456789012345678901
whiteboard shared by you and the tutor, which allows you to communi- 2
2
12345678901234567890123456789012123456789012345678901
12345678901234567890123456789012123456789012345678901 2
1 cate with one another in real time. However, if you prefer, you can submit 2
2345678901234567890123456789012123456789012345678901
your math question in writing instead and receive a written answer within 2
12345678901234567890123456789012123456789012345678901
12345678901234567890123456789012123456789012345678901 2
1 24 hours. 2
12345678901234567890123456789012123456789012345678901 2
12345678901234567890123456789012123456789012345678901 2
12345678901234567890123456789012123456789012345678901
The free tutoring offered with this product is limited to 30 minutes, 2
12345678901234567890123456789012123456789012345678901 2
12345678901234567890123456789012123456789012345678901 2
12345678901234567890123456789012123456789012345678901
although you may purchase more time if you need it. A written response to 2
12345678901234567890123456789012123456789012345678901 2
12345678901234567890123456789012123456789012345678901 22
2345678901234567890123456789012123456789012345678901
a submitted question counts as 20 minutes. The tutoring service is
12345678901234567890123456789012123456789012345678901
1 available for six months from the date you register on line. Note: The 2
12345678901234567890123456789012123456789012345678901
e-tutoring service offered to purchasers of this 2005 edition will expire on 2
12345678901234567890123456789012123456789012345678901 2
2345678901234567890123456789012123456789012345678901
12345678901234567890123456789012123456789012345678901
January 1, 2006. 2
12345678901234567890123456789012123456789012345678901 2
12345678901234567890123456789012123456789012345678901
This service is available 24 hours a day, seven days a week for most of the 2
2
12345678901234567890123456789012123456789012345678901
1 year. During the summer the service is available from 9 a.m. to 1 a.m. 2
12345678901234567890123456789012123456789012345678901 2
12345678901234567890123456789012123456789012345678901 2
12345678901234567890123456789012123456789012345678901
Eastern Standard Time. Due to low demand, the service is not available 2
12345678901234567890123456789012123456789012345678901
during several holiday periods, including Thanksgiving, Christmas, Easter, 2
12345678901234567890123456789012123456789012345678901 2
2345678901234567890123456789012123456789012345678901
12345678901234567890123456789012123456789012345678901 2
12345678901234567890123456789012123456789012345678901
Labor Day, and Memorial Day. 2
12345678901234567890123456789012123456789012345678901 22
12345678901234567890123456789012123456789012345678901
1 Remember, to register for this service, you will need the CD that 2
12345678901234567890123456789012123456789012345678901
accompanies this book. You will also need to refer to this book to provide 2
12345678901234567890123456789012123456789012345678901 2
12345678901234567890123456789012123456789012345678901 2
12345678901234567890123456789012123456789012345678901 2
the access code when prompted.
12345678901234567890123456789012123456789012345678901 2
12345678901234567890123456789012123456789012345678901 2
12345678901234567890123456789012123456789012345678901 2
12345678901234567890123456789012123456789012345678901 2
12345678901234567890123456789012123456789012345678901 2
123456789012345678901234567890121234567890123456789012
v
About This Tool Kit

123456789012345678901234567890121234567890123456789012
12345678901234567890123456789012123456789012345678901 2
12345678901234567890123456789012123456789012345678901 2
12345678901234567890123456789012123456789012345678901
Essay Scoring 2
12345678901234567890123456789012123456789012345678901 2
12345678901234567890123456789012123456789012345678901 2
12345678901234567890123456789012123456789012345678901 2
The CD that accompanies this book allows you to write 2 practice test
12345678901234567890123456789012123456789012345678901
essays on line and receive a score for each of them, which approximates 2
12345678901234567890123456789012123456789012345678901 2
12345678901234567890123456789012123456789012345678901
your performance on the essays in the actual GRE. In addition to a score, 2
12345678901234567890123456789012123456789012345678901 2
12345678901234567890123456789012123456789012345678901
you will also receive constructive feedback on your essays, including tips 2
12345678901234567890123456789012123456789012345678901 2
12345678901234567890123456789012123456789012345678901
to improve your score. 2
12345678901234567890123456789012123456789012345678901 2
12345678901234567890123456789012123456789012345678901 2
12345678901234567890123456789012123456789012345678901
With this tool kit, you get scoring for 2 practice test essays—one analysis 2
12345678901234567890123456789012123456789012345678901 2
12345678901234567890123456789012123456789012345678901
of an issue and one analysis of an argument. If you wish, for a fee, you may 2
12345678901234567890123456789012123456789012345678901
obtain scoring information and feedback for additional practice essays 2
12345678901234567890123456789012123456789012345678901 2
12345678901234567890123456789012123456789012345678901 2
12345678901234567890123456789012123456789012345678901
you write on line. 2
1 Note: Your practice test essays will be scored by a computer, but on the 2
2345678901234567890123456789012123456789012345678901
12345678901234567890123456789012123456789012345678901 2
12345678901234567890123456789012123456789012345678901 2
12345678901234567890123456789012123456789012345678901
actual GRE test, the essays are scored by a combination of a computer 2
12345678901234567890123456789012123456789012345678901 2
12345678901234567890123456789012123456789012345678901 2
2345678901234567890123456789012123456789012345678901
program and a human reader.
12345678901234567890123456789012123456789012345678901 2
To register for the essay-scoring service you need the CD that accompanies 2
12345678901234567890123456789012123456789012345678901
12345678901234567890123456789012123456789012345678901 2
this book. In addition, you will need to refer to this book to provide the 2
12345678901234567890123456789012123456789012345678901
12345678901234567890123456789012123456789012345678901 2
1 access code when prompted. Note: The scoring of 2 essays is offered free 2
12345678901234567890123456789012123456789012345678901 2
12345678901234567890123456789012123456789012345678901
to purchasers of the 2005 edition and this offer will expire on December 2
12345678901234567890123456789012123456789012345678901 2
12345678901234567890123456789012123456789012345678901
31, 2005. 2
12345678901234567890123456789012123456789012345678901 2
12345678901234567890123456789012123456789012345678901 2
12345678901234567890123456789012123456789012345678901 2
12345678901234567890123456789012123456789012345678901 2
2345678901234567890123456789012123456789012345678901
12345678901234567890123456789012123456789012345678901 2
12345678901234567890123456789012123456789012345678901
Computer-Adaptive Tests on CD 2
12345678901234567890123456789012123456789012345678901 2
12345678901234567890123456789012123456789012345678901 2
The actual GRE is a computer-adaptive test (CAT)—the test questions you
1 see depend on your performance on previous questions. On the CD-ROM, 2
12345678901234567890123456789012123456789012345678901 2
12345678901234567890123456789012123456789012345678901 2
12345678901234567890123456789012123456789012345678901 2
which accompanies this book, you receive 3 practice GMAT CATs that
12345678901234567890123456789012123456789012345678901
carefully reproduce the experience you can expect on test day. The 2
12345678901234567890123456789012123456789012345678901
2345678901234567890123456789012123456789012345678901
2
2
12345678901234567890123456789012123456789012345678901
12345678901234567890123456789012123456789012345678901
additional 3 practice tests in the book can be used for further practice. 2
1 2
12345678901234567890123456789012123456789012345678901 2
12345678901234567890123456789012123456789012345678901 2
12345678901234567890123456789012123456789012345678901 2
2345678901234567890123456789012123456789012345678901
12345678901234567890123456789012123456789012345678901 2 2
1
12345678901234567890123456789012123456789012345678901 2
12345678901234567890123456789012123456789012345678901 2
12345678901234567890123456789012123456789012345678901 2
12345678901234567890123456789012123456789012345678901 2
12345678901234567890123456789012123456789012345678901 2
12345678901234567890123456789012123456789012345678901 2
12345678901234567890123456789012123456789012345678901 2
12345678901234567890123456789012123456789012345678901 2
2345678901234567890123456789012123456789012345678901
12345678901234567890123456789012123456789012345678901 2
12345678901234567890123456789012123456789012345678901 2
12345678901234567890123456789012123456789012345678901 2 2
1
12345678901234567890123456789012123456789012345678901 2
12345678901234567890123456789012123456789012345678901 2
12345678901234567890123456789012123456789012345678901 2
12345678901234567890123456789012123456789012345678901 2
12345678901234567890123456789012123456789012345678901 2
12345678901234567890123456789012123456789012345678901 2
2345678901234567890123456789012123456789012345678901
2
1
12345678901234567890123456789012123456789012345678901 2
12345678901234567890123456789012123456789012345678901 2
12345678901234567890123456789012123456789012345678901 2
12345678901234567890123456789012123456789012345678901 2
12345678901234567890123456789012123456789012345678901 2
12345678901234567890123456789012123456789012345678901 2 2
vi 12345678901234567890123456789012123456789012345678901
123456789012345678901234567890121234567890123456789012

www.petersons.com
About This Tool Kit

123456789012345678901234567890121234567890123456789012
12345678901234567890123456789012123456789012345678901 2
12345678901234567890123456789012123456789012345678901 2
12345678901234567890123456789012123456789012345678901 2
12345678901234567890123456789012123456789012345678901
STRUCTURE OF THE GRE CAT 2
12345678901234567890123456789012123456789012345678901 2
12345678901234567890123456789012123456789012345678901 2
12345678901234567890123456789012123456789012345678901
Test Section Number of Questions Time
2345678901234567890123456789012123456789012345678901
Allotted 2
2
1
12345678901234567890123456789012123456789012345678901 2
12345678901234567890123456789012123456789012345678901 2
12345678901234567890123456789012123456789012345678901
Computer-Based General Test 2
12345678901234567890123456789012123456789012345678901
2345678901234567890123456789012123456789012345678901
2
2
1 Analytical Writing 2 writing tasks
12345678901234567890123456789012123456789012345678901
75-minute section 2
12345678901234567890123456789012123456789012345678901 2
12345678901234567890123456789012123456789012345678901
Present Your Perspective 45 minutes 2
12345678901234567890123456789012123456789012345678901
2345678901234567890123456789012123456789012345678901
2
2
1 on an Issue
12345678901234567890123456789012123456789012345678901 2
12345678901234567890123456789012123456789012345678901 2
12345678901234567890123456789012123456789012345678901
Analyze an Argument 30 minutes 2
12345678901234567890123456789012123456789012345678901
2345678901234567890123456789012123456789012345678901
2
2
1 Verbal 30 questions 30-minute section
12345678901234567890123456789012123456789012345678901 2
12345678901234567890123456789012123456789012345678901 2
12345678901234567890123456789012123456789012345678901
Quantitative 28 questions 45-minute section 2
12345678901234567890123456789012123456789012345678901 2
12345678901234567890123456789012123456789012345678901 22
2345678901234567890123456789012123456789012345678901
12345678901234567890123456789012123456789012345678901 2
12345678901234567890123456789012123456789012345678901
12345678901234567890123456789012123456789012345678901
Paper-Based General Test 2
12345678901234567890123456789012123456789012345678901 2
12345678901234567890123456789012123456789012345678901 22
Analytical Writing 2 writing tasks 75-minute section
1
12345678901234567890123456789012123456789012345678901 2
12345678901234567890123456789012123456789012345678901
Present Your Perspective 45 minutes 2
12345678901234567890123456789012123456789012345678901
on an Issue 2
12345678901234567890123456789012123456789012345678901 22
2345678901234567890123456789012123456789012345678901
12345678901234567890123456789012123456789012345678901
12345678901234567890123456789012123456789012345678901
Analyze an Argument 30 minutes 2
12345678901234567890123456789012123456789012345678901 22
12345678901234567890123456789012123456789012345678901
Verbal 30 questions 30-minute section
12345678901234567890123456789012123456789012345678901 2
12345678901234567890123456789012123456789012345678901 2
12345678901234567890123456789012123456789012345678901
Quantitative 28 questions 45-minute section 2
12345678901234567890123456789012123456789012345678901 22
1
12345678901234567890123456789012123456789012345678901 2
12345678901234567890123456789012123456789012345678901 2
12345678901234567890123456789012123456789012345678901 2
12345678901234567890123456789012123456789012345678901 2
12345678901234567890123456789012123456789012345678901 2
12345678901234567890123456789012123456789012345678901 2
12345678901234567890123456789012123456789012345678901 2
12345678901234567890123456789012123456789012345678901 2
12345678901234567890123456789012123456789012345678901 2
12345678901234567890123456789012123456789012345678901 2
12345678901234567890123456789012123456789012345678901 22
2345678901234567890123456789012123456789012345678901
12345678901234567890123456789012123456789012345678901 2
1
12345678901234567890123456789012123456789012345678901 2
12345678901234567890123456789012123456789012345678901 2
12345678901234567890123456789012123456789012345678901 2
12345678901234567890123456789012123456789012345678901 2
12345678901234567890123456789012123456789012345678901 2
12345678901234567890123456789012123456789012345678901 2
12345678901234567890123456789012123456789012345678901 2
12345678901234567890123456789012123456789012345678901 22
2345678901234567890123456789012123456789012345678901
12345678901234567890123456789012123456789012345678901 2
12345678901234567890123456789012123456789012345678901 2
12345678901234567890123456789012123456789012345678901 2
1
12345678901234567890123456789012123456789012345678901 2
12345678901234567890123456789012123456789012345678901 2
12345678901234567890123456789012123456789012345678901 2
12345678901234567890123456789012123456789012345678901 2
12345678901234567890123456789012123456789012345678901 2
12345678901234567890123456789012123456789012345678901 22
2345678901234567890123456789012123456789012345678901
1
12345678901234567890123456789012123456789012345678901 2
12345678901234567890123456789012123456789012345678901 2
12345678901234567890123456789012123456789012345678901 2
12345678901234567890123456789012123456789012345678901 2
12345678901234567890123456789012123456789012345678901 2
12345678901234567890123456789012123456789012345678901 2
12345678901234567890123456789012123456789012345678901 2 vii
123456789012345678901234567890121234567890123456789012
Introduction

123456789012345678901234567890121234567890123456789012
12345678901234567890123456789012123456789012345678901 2
12345678901234567890123456789012123456789012345678901
2345678901234567890123456789012123456789012345678901
2
2
1
Does This CAT Have Claws?
12345678901234567890123456789012123456789012345678901 2
12345678901234567890123456789012123456789012345678901 2
12345678901234567890123456789012123456789012345678901
Unless you’ve been living as a hermit in a hut for the past twenty years, 2
12345678901234567890123456789012123456789012345678901 2
12345678901234567890123456789012123456789012345678901 22
2345678901234567890123456789012123456789012345678901
chances are good you have already taken at least one standardized test in
12345678901234567890123456789012123456789012345678901
your lifetime. And we bet the list of tests you might have taken reads like a 2
12345678901234567890123456789012123456789012345678901
bowl of alphabet soup: SAT, ACT, ITBS, TAAS, MAT, and the PSAT/ 2
12345678901234567890123456789012123456789012345678901
12345678901234567890123456789012123456789012345678901 2
12345678901234567890123456789012123456789012345678901 2
1 NMSQT, to name just a few. Standardized tests such as these have become 2
12345678901234567890123456789012123456789012345678901
a fixture of America’s educational system, and like them or not, they are 2
12345678901234567890123456789012123456789012345678901 2
12345678901234567890123456789012123456789012345678901
not going away anytime soon. Students encounter them at every stage of 2
12345678901234567890123456789012123456789012345678901 22
2345678901234567890123456789012123456789012345678901
12345678901234567890123456789012123456789012345678901
their education—like toll booths on the academic highway. Students in
12345678901234567890123456789012123456789012345678901 2
grade school get the ITBS or a similar exam. Teenagers hoping to enter 2
12345678901234567890123456789012123456789012345678901
12345678901234567890123456789012123456789012345678901 22
12345678901234567890123456789012123456789012345678901
college must pass through the SAT or ACT Assessment toll booth. For
12345678901234567890123456789012123456789012345678901
those brave folks who wish to enter graduate school, the exit is labeled 2
2
12345678901234567890123456789012123456789012345678901
12345678901234567890123456789012123456789012345678901 2
1 GRE (Graduate Record Examination). 2
12345678901234567890123456789012123456789012345678901 22
2345678901234567890123456789012123456789012345678901
12345678901234567890123456789012123456789012345678901
1 If you have taken the SAT, you will already be somewhat familiar with the 2
12345678901234567890123456789012123456789012345678901
GRE. Both tests are administered by the same company. Each test is a 2
12345678901234567890123456789012123456789012345678901
2345678901234567890123456789012123456789012345678901
2
timed, multiple-choice exam that schools use as a factor when making 2
12345678901234567890123456789012123456789012345678901
12345678901234567890123456789012123456789012345678901 2
1 enrollment decisions. Like the SAT, the GRE has a Quantitative (math) 2
12345678901234567890123456789012123456789012345678901 2
12345678901234567890123456789012123456789012345678901 2
12345678901234567890123456789012123456789012345678901 22
2345678901234567890123456789012123456789012345678901
section and a Verbal section, and many of the questions in these SAT and
12345678901234567890123456789012123456789012345678901
1 GRE sections are similar in style. For example, there are analogy, sentence 2
12345678901234567890123456789012123456789012345678901
completions, and reading comprehension problems in the Verbal section of 2
12345678901234567890123456789012123456789012345678901 2
2345678901234567890123456789012123456789012345678901
both tests. Many of the math topics covered on the SAT—geometry, 2
12345678901234567890123456789012123456789012345678901
12345678901234567890123456789012123456789012345678901 2
algebra, understanding charts and graphs—make an encore appearance on 2
12345678901234567890123456789012123456789012345678901
12345678901234567890123456789012123456789012345678901 22
1 the GRE.
12345678901234567890123456789012123456789012345678901 2
12345678901234567890123456789012123456789012345678901 2
12345678901234567890123456789012123456789012345678901
While these similarities exist, don’t be fooled into thinking the GRE is 2
12345678901234567890123456789012123456789012345678901 2
12345678901234567890123456789012123456789012345678901
simply another SAT. The GRE is more like the SAT’s weird older brother. 2
2345678901234567890123456789012123456789012345678901
12345678901234567890123456789012123456789012345678901
Although you can see some family resemblance, some big differences are 2
2
12345678901234567890123456789012123456789012345678901
pretty obvious as well. For example, both tests have analogy and sentence 2
12345678901234567890123456789012123456789012345678901
12345678901234567890123456789012123456789012345678901 2
1 completions problems, but GRE questions are typically much harder, since 2
12345678901234567890123456789012123456789012345678901 22
2345678901234567890123456789012123456789012345678901
1 they employ a more advanced vocabulary. This is because the GRE
12345678901234567890123456789012123456789012345678901 2
12345678901234567890123456789012123456789012345678901 2
12345678901234567890123456789012123456789012345678901
problems assume a college-level vocabulary, while the SAT questions are 2
1 set at a lower, high school level. This is just one reason why the GRE can be 2
2345678901234567890123456789012123456789012345678901
12345678901234567890123456789012123456789012345678901 2
12345678901234567890123456789012123456789012345678901 2
12345678901234567890123456789012123456789012345678901 2
123456789012345678901234567890121234567890123456789012
1
Introduction

123456789012345678901234567890121234567890123456789012
12345678901234567890123456789012123456789012345678901 2
12345678901234567890123456789012123456789012345678901 2
12345678901234567890123456789012123456789012345678901
seen as an older brother, since it is geared toward an older, college- 2
12345678901234567890123456789012123456789012345678901 2
12345678901234567890123456789012123456789012345678901
educated audience. The GRE’s weirdness comes from its unusual format. 2
12345678901234567890123456789012123456789012345678901 2
12345678901234567890123456789012123456789012345678901
The SAT remains a pencil-and-paper test, meaning students across the 2
1 country are still using chewed-up number 2 pencils to fiercely darken ovals 2
2345678901234567890123456789012123456789012345678901
12345678901234567890123456789012123456789012345678901 2
12345678901234567890123456789012123456789012345678901 2
12345678901234567890123456789012123456789012345678901
on a score sheet. In contrast, the GRE is now primarily a Computer 2
12345678901234567890123456789012123456789012345678901
2345678901234567890123456789012123456789012345678901
2
2
1 Adaptive Test (CAT), so all questions appear onscreen and are answered
12345678901234567890123456789012123456789012345678901 2
12345678901234567890123456789012123456789012345678901
by using a mouse to click on an answer choice. 2
12345678901234567890123456789012123456789012345678901 2
12345678901234567890123456789012123456789012345678901
2345678901234567890123456789012123456789012345678901
2
2
1
12345678901234567890123456789012123456789012345678901 2

X-Ref
12345678901234567890123456789012123456789012345678901
There are still some places, mostly outside of the United States, that offer 2
12345678901234567890123456789012123456789012345678901 2
12345678901234567890123456789012123456789012345678901
pencil-and-paper versions of the GRE. For more information about these 2
1 exams, and about registering for the GRE in general, turn to the 2
2345678901234567890123456789012123456789012345678901
12345678901234567890123456789012123456789012345678901 2
12345678901234567890123456789012123456789012345678901 2
12345678901234567890123456789012123456789012345678901
Appendix. 2
12345678901234567890123456789012123456789012345678901 2
12345678901234567890123456789012123456789012345678901 2
2345678901234567890123456789012123456789012345678901
12345678901234567890123456789012123456789012345678901 2
The GRE CAT is more than just a pencil-and-paper test transposed onto a 2
12345678901234567890123456789012123456789012345678901
12345678901234567890123456789012123456789012345678901 2
computer screen, and unless you have taken the GRE previously, it is 2
12345678901234567890123456789012123456789012345678901
12345678901234567890123456789012123456789012345678901 2
1 unlikely you have ever experienced an exam like it. For one thing, the 2
12345678901234567890123456789012123456789012345678901 2
12345678901234567890123456789012123456789012345678901
method used to score the GRE CAT is radically different from most tests. 2
12345678901234567890123456789012123456789012345678901 2
12345678901234567890123456789012123456789012345678901
Most exams start with easy questions, and then gradually progress to 2
12345678901234567890123456789012123456789012345678901 2
12345678901234567890123456789012123456789012345678901
medium and harder problems. People taking this type of test might get the 2
12345678901234567890123456789012123456789012345678901 2
12345678901234567890123456789012123456789012345678901
easiest problem wrong but still manage to answer the toughest problem 2
2345678901234567890123456789012123456789012345678901
12345678901234567890123456789012123456789012345678901
correctly. In the end, the number of right answers is added up (regardless 2
2
12345678901234567890123456789012123456789012345678901
12345678901234567890123456789012123456789012345678901
of which question was answered right or wrong), and a score is eventually 2
2
12345678901234567890123456789012123456789012345678901 2
1 determined.
12345678901234567890123456789012123456789012345678901 2
12345678901234567890123456789012123456789012345678901 2
12345678901234567890123456789012123456789012345678901 2
12345678901234567890123456789012123456789012345678901 2
On the GRE CAT, things are a bit different. How well (or not well) you
12345678901234567890123456789012123456789012345678901
answer each question determines the next question. In each section of the 2
12345678901234567890123456789012123456789012345678901 2
12345678901234567890123456789012123456789012345678901 2
12345678901234567890123456789012123456789012345678901 2
test, you start off with a question that has a medium difficulty rating. If
12345678901234567890123456789012123456789012345678901
you get it right, the computer says, “Okay, Brainiac, you handled that one, 2
12345678901234567890123456789012123456789012345678901 2
2345678901234567890123456789012123456789012345678901
now try this tougher problem.” If you get the next problem right, you get 2
12345678901234567890123456789012123456789012345678901
12345678901234567890123456789012123456789012345678901 2
1 a harder question, and if you get that one correct, an even tougher 2
12345678901234567890123456789012123456789012345678901 2
12345678901234567890123456789012123456789012345678901
question comes next. Depending on how you answer each question, the 2
12345678901234567890123456789012123456789012345678901 2
12345678901234567890123456789012123456789012345678901
computer adapts the next question to make it either harder or easier than 2
12345678901234567890123456789012123456789012345678901 2
12345678901234567890123456789012123456789012345678901
the next one. The way the computer adapts your GRE experience will be 2
12345678901234567890123456789012123456789012345678901 2
12345678901234567890123456789012123456789012345678901
different from what other GRE test-takers experience, so even though 2
12345678901234567890123456789012123456789012345678901 2
12345678901234567890123456789012123456789012345678901
you’re all taking the same GRE CAT, you’re not seeing the same problems 2
12345678901234567890123456789012123456789012345678901 2
12345678901234567890123456789012123456789012345678901
as everyone else. In the same way, if you get the first question wrong, you’ll 2
2345678901234567890123456789012123456789012345678901
12345678901234567890123456789012123456789012345678901
get an easier one. And if you get the easier questions wrong, you’ll see an 2
2
12345678901234567890123456789012123456789012345678901 2
12345678901234567890123456789012123456789012345678901 2
12345678901234567890123456789012123456789012345678901
easier question.
2
1
2345678901234567890123456789012123456789012345678901
12345678901234567890123456789012123456789012345678901 2
1 One consequence of the computer adaptive format is that getting a high 2
12345678901234567890123456789012123456789012345678901
score means handling only medium and hard questions. The more difficult 2
12345678901234567890123456789012123456789012345678901 2
12345678901234567890123456789012123456789012345678901
2345678901234567890123456789012123456789012345678901
2
2
1 your questions are, the higher your score potential, and vice versa. And
12345678901234567890123456789012123456789012345678901 2
12345678901234567890123456789012123456789012345678901 2
there are no easy questions at the start of a section to help you get your feet
12345678901234567890123456789012123456789012345678901 2
2 123456789012345678901234567890121234567890123456789012

www.petersons.com
Introduction

123456789012345678901234567890121234567890123456789012
12345678901234567890123456789012123456789012345678901 2
12345678901234567890123456789012123456789012345678901 2
12345678901234567890123456789012123456789012345678901
wet. The first question will be somewhat difficult, and it may well be the 2
12345678901234567890123456789012123456789012345678901 2
12345678901234567890123456789012123456789012345678901
easiest problem you see on that section. In terms of your score, seeing only 2
12345678901234567890123456789012123456789012345678901 2
12345678901234567890123456789012123456789012345678901
medium and tough questions tells you that you are getting problems right, 2
1 which is good news. However, having to answer one challenging question 2
2345678901234567890123456789012123456789012345678901
12345678901234567890123456789012123456789012345678901 2
12345678901234567890123456789012123456789012345678901 2
12345678901234567890123456789012123456789012345678901
after another makes the testing experience a really nerve-wracking, 2
12345678901234567890123456789012123456789012345678901
2345678901234567890123456789012123456789012345678901
2
2
1 unpleasant affair, kind of like visiting a dentist who holds a personal
12345678901234567890123456789012123456789012345678901 2
12345678901234567890123456789012123456789012345678901
grudge against your gums. 2
12345678901234567890123456789012123456789012345678901 2
12345678901234567890123456789012123456789012345678901
2345678901234567890123456789012123456789012345678901
2
2
1
12345678901234567890123456789012123456789012345678901 2
12345678901234567890123456789012123456789012345678901
The Unique Peterson’s Approach—Adaptive 2
12345678901234567890123456789012123456789012345678901 2
12345678901234567890123456789012123456789012345678901
2345678901234567890123456789012123456789012345678901
2
2
1
Preparation for an Adaptive Test
12345678901234567890123456789012123456789012345678901 2
12345678901234567890123456789012123456789012345678901
Test preparation is not a “one-size-fits-all” proposition. Each person has 2
12345678901234567890123456789012123456789012345678901 2
12345678901234567890123456789012123456789012345678901
different strengths and weaknesses, different quirks and anxieties. We 2
2345678901234567890123456789012123456789012345678901
12345678901234567890123456789012123456789012345678901
designed Peterson’s Ultimate GRE Tool Kit with those differences in 2
2
12345678901234567890123456789012123456789012345678901
mind. We provide a two-tiered approach that lets you decide how best to 2
12345678901234567890123456789012123456789012345678901
12345678901234567890123456789012123456789012345678901 2
12345678901234567890123456789012123456789012345678901 2
12345678901234567890123456789012123456789012345678901 22
organize your program of study to suit your needs. The first tier is the
1 general instruction in this book, and it provides skills and strategies you
12345678901234567890123456789012123456789012345678901 2
12345678901234567890123456789012123456789012345678901 2
12345678901234567890123456789012123456789012345678901
will need to handle medium-level questions on each section. You then have 2
2345678901234567890123456789012123456789012345678901
an opportunity to Take It to the Next Level, and learn skills and strategies 2
12345678901234567890123456789012123456789012345678901
12345678901234567890123456789012123456789012345678901 2
12345678901234567890123456789012123456789012345678901 2
12345678901234567890123456789012123456789012345678901 22
for the hardest questions on the GRE. No other GRE test-preparation
12345678901234567890123456789012123456789012345678901
12345678901234567890123456789012123456789012345678901
book does this. It’s the next best thing to having a personal tutor. 2
12345678901234567890123456789012123456789012345678901 22
12345678901234567890123456789012123456789012345678901 2
12345678901234567890123456789012123456789012345678901 2
1
12345678901234567890123456789012123456789012345678901 2
12345678901234567890123456789012123456789012345678901 2
12345678901234567890123456789012123456789012345678901 2
12345678901234567890123456789012123456789012345678901 2
12345678901234567890123456789012123456789012345678901 2
12345678901234567890123456789012123456789012345678901 2
12345678901234567890123456789012123456789012345678901 2
12345678901234567890123456789012123456789012345678901 2
12345678901234567890123456789012123456789012345678901 2
12345678901234567890123456789012123456789012345678901 2
12345678901234567890123456789012123456789012345678901 22
2345678901234567890123456789012123456789012345678901
12345678901234567890123456789012123456789012345678901 2
1
12345678901234567890123456789012123456789012345678901 2
12345678901234567890123456789012123456789012345678901 2
12345678901234567890123456789012123456789012345678901 2
12345678901234567890123456789012123456789012345678901 2
12345678901234567890123456789012123456789012345678901 2
12345678901234567890123456789012123456789012345678901 2
12345678901234567890123456789012123456789012345678901 2
12345678901234567890123456789012123456789012345678901 22
2345678901234567890123456789012123456789012345678901
12345678901234567890123456789012123456789012345678901 2
12345678901234567890123456789012123456789012345678901 2
12345678901234567890123456789012123456789012345678901 2
1
12345678901234567890123456789012123456789012345678901 2
12345678901234567890123456789012123456789012345678901 2
12345678901234567890123456789012123456789012345678901 2
12345678901234567890123456789012123456789012345678901 2
12345678901234567890123456789012123456789012345678901 2
12345678901234567890123456789012123456789012345678901 22
2345678901234567890123456789012123456789012345678901
1
12345678901234567890123456789012123456789012345678901 2
12345678901234567890123456789012123456789012345678901 2
12345678901234567890123456789012123456789012345678901 2
12345678901234567890123456789012123456789012345678901 2
12345678901234567890123456789012123456789012345678901 2
12345678901234567890123456789012123456789012345678901 2
12345678901234567890123456789012123456789012345678901 2
123456789012345678901234567890121234567890123456789012 3
How to Use This Book

123456789012345678901234567890121234567890123456789012
12345678901234567890123456789012123456789012345678901 2
12345678901234567890123456789012123456789012345678901
2345678901234567890123456789012123456789012345678901
2
2
1 How do you know where to begin? We give you a roadmap. After you read
12345678901234567890123456789012123456789012345678901 2
12345678901234567890123456789012123456789012345678901
Chapters 1 and 2— required reading for all GRE CAT takers, no matter 2
12345678901234567890123456789012123456789012345678901 2
12345678901234567890123456789012123456789012345678901
what—take the diagnostic pretest in Chapter 3. Use the scoring discussion 2
2345678901234567890123456789012123456789012345678901
12345678901234567890123456789012123456789012345678901
provided on pages 32–33 to assess your current GRE CAT testing level. 2
2
12345678901234567890123456789012123456789012345678901
You can then either follow the suggested course of study we provide, or 2
12345678901234567890123456789012123456789012345678901
12345678901234567890123456789012123456789012345678901 2
12345678901234567890123456789012123456789012345678901 2
12345678901234567890123456789012123456789012345678901 2
design your own.
2
1
12345678901234567890123456789012123456789012345678901 2
12345678901234567890123456789012123456789012345678901
Your personalized Peterson’s GRE course of study will help you 2
12345678901234567890123456789012123456789012345678901 2
12345678901234567890123456789012123456789012345678901 2
2345678901234567890123456789012123456789012345678901
accomplish four things:
12345678901234567890123456789012123456789012345678901 2
1. You will become thoroughly familiar with the computer adaptive 2
12345678901234567890123456789012123456789012345678901
12345678901234567890123456789012123456789012345678901 2
12345678901234567890123456789012123456789012345678901
test format. 2
12345678901234567890123456789012123456789012345678901 2
12345678901234567890123456789012123456789012345678901
2. You will be comfortable with all question types you will 2
2
12345678901234567890123456789012123456789012345678901
12345678901234567890123456789012123456789012345678901 2
1 encounter on the three sections of the test. 2
12345678901234567890123456789012123456789012345678901 2
2345678901234567890123456789012123456789012345678901
12345678901234567890123456789012123456789012345678901 2
1 3. You will brush up on any weak subject areas. 2
12345678901234567890123456789012123456789012345678901 2
12345678901234567890123456789012123456789012345678901
4. You will learn and practice specific test-taking strategies to help 2
12345678901234567890123456789012123456789012345678901 2
12345678901234567890123456789012123456789012345678901
you achieve your target score. 2
12345678901234567890123456789012123456789012345678901 2
12345678901234567890123456789012123456789012345678901 2
12345678901234567890123456789012123456789012345678901
If you give yourself enough time to work through your personalized course 2
2345678901234567890123456789012123456789012345678901
12345678901234567890123456789012123456789012345678901 2
12345678901234567890123456789012123456789012345678901
of study at a measured pace, you will accomplish all of these things. 2
1 2
12345678901234567890123456789012123456789012345678901 2
12345678901234567890123456789012123456789012345678901 2
Combining relevant factual knowledge with a sound strategic approach is
2345678901234567890123456789012123456789012345678901
the best way to score well on any standardized test, not just the GRE. This 2
12345678901234567890123456789012123456789012345678901
12345678901234567890123456789012123456789012345678901 2
12345678901234567890123456789012123456789012345678901 2
12345678901234567890123456789012123456789012345678901 2
book will provide you with the facts and strategies you need to succeed on
1 the GRE. The first step is to learn about the computer adaptive format, 2
12345678901234567890123456789012123456789012345678901 2
12345678901234567890123456789012123456789012345678901 2
12345678901234567890123456789012123456789012345678901
covered in the next chapter. So, let’s get started. 2
12345678901234567890123456789012123456789012345678901 2
12345678901234567890123456789012123456789012345678901 2
12345678901234567890123456789012123456789012345678901 2
2345678901234567890123456789012123456789012345678901
12345678901234567890123456789012123456789012345678901 2
12345678901234567890123456789012123456789012345678901 2
12345678901234567890123456789012123456789012345678901 2 2
1
12345678901234567890123456789012123456789012345678901 2
12345678901234567890123456789012123456789012345678901 2
12345678901234567890123456789012123456789012345678901 2
12345678901234567890123456789012123456789012345678901 2
12345678901234567890123456789012123456789012345678901 2
12345678901234567890123456789012123456789012345678901 2
12345678901234567890123456789012123456789012345678901 2
12345678901234567890123456789012123456789012345678901 2
12345678901234567890123456789012123456789012345678901 2
123456789012345678901234567890121234567890123456789012
4
PART

I
Understanding the GRE
Everything You Need to Know
about the GRE CAT 6
Section Strategies 19

PART I
Chapter

Everything You Need to


1
Know about the GRE CAT
123456789012345678901234567890121234567890123456789012
12345678901234567890123456789012123456789012345678901 2
12345678901234567890123456789012123456789012345678901 2
12345678901234567890123456789012123456789012345678901
Once upon a time, people who wanted to take the GRE (Graduate Record 2
2345678901234567890123456789012123456789012345678901
12345678901234567890123456789012123456789012345678901
Examination) and go to graduate school had to take a paper-and-pencil 2
2
12345678901234567890123456789012123456789012345678901
test much like the SAT they had to take to get into college. They had to 2
12345678901234567890123456789012123456789012345678901
12345678901234567890123456789012123456789012345678901 2
12345678901234567890123456789012123456789012345678901 2
12345678901234567890123456789012123456789012345678901 2
show up at a test center early on a Saturday morning with dozens of other
1 prospective grad students, all with their number 2 pencils sharpened, ready 2
12345678901234567890123456789012123456789012345678901 2
12345678901234567890123456789012123456789012345678901 2
12345678901234567890123456789012123456789012345678901
to fill in a lot of answer sheet bubbles and spend pretty much the entire 2
2345678901234567890123456789012123456789012345678901
12345678901234567890123456789012123456789012345678901 2
12345678901234567890123456789012123456789012345678901
morning slogging through the test. 2
12345678901234567890123456789012123456789012345678901 2
That was before the CAT (Computer Adaptive Test) came along and 2
12345678901234567890123456789012123456789012345678901
2
12345678901234567890123456789012123456789012345678901
12345678901234567890123456789012123456789012345678901
changed everything. . . . 2
12345678901234567890123456789012123456789012345678901 2
12345678901234567890123456789012123456789012345678901
No more number 2 pencils, no more answer bubbles; the GRE world 2
2
12345678901234567890123456789012123456789012345678901
1 changed. Like anything new, of course, the GRE CAT takes some getting 2
12345678901234567890123456789012123456789012345678901 2
12345678901234567890123456789012123456789012345678901 2
12345678901234567890123456789012123456789012345678901
used to. Most students have taken countless paper-and-pencil tests, but 2
12345678901234567890123456789012123456789012345678901 2
12345678901234567890123456789012123456789012345678901
few have taken a high-stakes standardized test in a computerized format. 2
2345678901234567890123456789012123456789012345678901
In this chapter, we’ll help you get a grip on the world of computer adaptive 2
12345678901234567890123456789012123456789012345678901
12345678901234567890123456789012123456789012345678901 2
1 testing. 2
12345678901234567890123456789012123456789012345678901 2
12345678901234567890123456789012123456789012345678901 2
12345678901234567890123456789012123456789012345678901 2
2345678901234567890123456789012123456789012345678901
12345678901234567890123456789012123456789012345678901 2 2
1
12345678901234567890123456789012123456789012345678901
Why the GRE CAT Rocks! 2
12345678901234567890123456789012123456789012345678901 2
2345678901234567890123456789012123456789012345678901
No one likes taking a big test and having to wait, and wait, and wait for 2
12345678901234567890123456789012123456789012345678901
12345678901234567890123456789012123456789012345678901 2
12345678901234567890123456789012123456789012345678901 2
12345678901234567890123456789012123456789012345678901 2
the results. The time between finishing the test and getting your score fills
1 you with conflicting emotions. You might be confident one moment and 2
12345678901234567890123456789012123456789012345678901 2
12345678901234567890123456789012123456789012345678901 2
12345678901234567890123456789012123456789012345678901
filled with despair the next. The doubt and uncertainty won’t go away 2
12345678901234567890123456789012123456789012345678901 2
12345678901234567890123456789012123456789012345678901 2
until you finally get your scores in the mail, so the issue goes unresolved for
2345678901234567890123456789012123456789012345678901
12345678901234567890123456789012123456789012345678901
about a month. 2
12345678901234567890123456789012123456789012345678901 2
12345678901234567890123456789012123456789012345678901 2
the GRE CAT, waiting for your score is a thing of the past. Once you 2
12345678901234567890123456789012123456789012345678901
1 With 2
2345678901234567890123456789012123456789012345678901
your test, you have the option to cancel your score. If you decide not 2
12345678901234567890123456789012123456789012345678901
finish
1 to cancel, your Quantitative and Verbal scores appear on the screen. Voila! 2
12345678901234567890123456789012123456789012345678901 2
12345678901234567890123456789012123456789012345678901 2
12345678901234567890123456789012123456789012345678901
A wait of six weeks reduced to the time it
2345678901234567890123456789012123456789012345678901
takes to click one button. 2
2
1
12345678901234567890123456789012123456789012345678901 2
12345678901234567890123456789012123456789012345678901 2
12345678901234567890123456789012123456789012345678901 2
123456789012345678901234567890121234567890123456789012
6
Chapter 1: Everything You Need to Know about the GRE CAT

123456789012345678901234567890121234567890123456789012
12345678901234567890123456789012123456789012345678901 2
12345678901234567890123456789012123456789012345678901 2
12345678901234567890123456789012123456789012345678901 2
12345678901234567890123456789012123456789012345678901 2
12345678901234567890123456789012123456789012345678901 2

Note
12345678901234567890123456789012123456789012345678901 2
You don’t receive your Analytical writing score immediately, since the
12345678901234567890123456789012123456789012345678901 2
12345678901234567890123456789012123456789012345678901
two essays must be graded by professional readers. You should get your 2
12345678901234567890123456789012123456789012345678901 2
12345678901234567890123456789012123456789012345678901
Analytical results back in about three to four weeks. 2
12345678901234567890123456789012123456789012345678901 2
12345678901234567890123456789012123456789012345678901
2345678901234567890123456789012123456789012345678901
2
2
1
12345678901234567890123456789012123456789012345678901 2
12345678901234567890123456789012123456789012345678901
This immediacy is the prime advantage of the GRE CAT. It lets you know 2
12345678901234567890123456789012123456789012345678901 2
12345678901234567890123456789012123456789012345678901
where you stand in terms of the application process and allows you to plan 2
1 accordingly. Most schools release information about the average GRE 2
2345678901234567890123456789012123456789012345678901
12345678901234567890123456789012123456789012345678901 2
12345678901234567890123456789012123456789012345678901 2
12345678901234567890123456789012123456789012345678901
scores of their incoming graduate students, and some also include an idea 2
12345678901234567890123456789012123456789012345678901
about the range of scores by releasing
2345678901234567890123456789012123456789012345678901 factoids like, “Over 90% of the 2
2
1
12345678901234567890123456789012123456789012345678901
incoming class of psychiatry students scored more than 600 on the GRE.” 2
12345678901234567890123456789012123456789012345678901 2
12345678901234567890123456789012123456789012345678901
If you wanted to go to that school, and you just received a combined 2
12345678901234567890123456789012123456789012345678901 2
12345678901234567890123456789012123456789012345678901
Quantitative and Verbal score of 750, then you know your GRE score is 2
12345678901234567890123456789012123456789012345678901 2
12345678901234567890123456789012123456789012345678901
good enough to get into that program. 2
12345678901234567890123456789012123456789012345678901 2
12345678901234567890123456789012123456789012345678901 2
12345678901234567890123456789012123456789012345678901 22
2345678901234567890123456789012123456789012345678901
If your GRE scores are not as high as you wanted, the immediate feedback
1 gives you a chance to shift to Plan B right then and there. You might decide
12345678901234567890123456789012123456789012345678901 2
12345678901234567890123456789012123456789012345678901 2
12345678901234567890123456789012123456789012345678901
to take the exam again, although you won’t be able to do so for at least one 2
2345678901234567890123456789012123456789012345678901
calendar month. (You can take the GRE up to five times in a single year.) If 2
12345678901234567890123456789012123456789012345678901
12345678901234567890123456789012123456789012345678901 2
12345678901234567890123456789012123456789012345678901 2
12345678901234567890123456789012123456789012345678901 22
you were only considering extremely competitive schools, you might
12345678901234567890123456789012123456789012345678901
decide to broaden your search and look at some schools with more 2
12345678901234567890123456789012123456789012345678901
inclusive admissions standards. When your Analytical scores arrive, you 2
12345678901234567890123456789012123456789012345678901
12345678901234567890123456789012123456789012345678901 2
12345678901234567890123456789012123456789012345678901 2
1 will also receive a form you can use to report your scores to additional 2
2345678901234567890123456789012123456789012345678901
schools. (You can download this from the GRE Web site.) Knowing your 2
12345678901234567890123456789012123456789012345678901
12345678901234567890123456789012123456789012345678901 2
1 scores right away gives you a chance to react to them right away. 2
12345678901234567890123456789012123456789012345678901 2
12345678901234567890123456789012123456789012345678901 2
12345678901234567890123456789012123456789012345678901
The second big advantage to the GRE CAT is that you can schedule it for 2
12345678901234567890123456789012123456789012345678901 2
12345678901234567890123456789012123456789012345678901
a convenient time. Other big standardized tests, like the business school 2
12345678901234567890123456789012123456789012345678901 2
12345678901234567890123456789012123456789012345678901
exam (GMAT) and the law school entrance exam (LSAT), are given at 2
2345678901234567890123456789012123456789012345678901
12345678901234567890123456789012123456789012345678901
fixed times on fixed dates at fixed locations. If any one of these three 2
2
12345678901234567890123456789012123456789012345678901 2
1 factors poses a problem for you, tough luck.
12345678901234567890123456789012123456789012345678901 2
12345678901234567890123456789012123456789012345678901 2
12345678901234567890123456789012123456789012345678901
This rigidity is not a problem for the GRE CAT. Most test centers are open 2
12345678901234567890123456789012123456789012345678901 2
12345678901234567890123456789012123456789012345678901 2
12345678901234567890123456789012123456789012345678901
during regular business hours, and all you have to do is call one up and 2
12345678901234567890123456789012123456789012345678901 2
12345678901234567890123456789012123456789012345678901 22
2345678901234567890123456789012123456789012345678901
decide on a time that works well for you.
12345678901234567890123456789012123456789012345678901 2
12345678901234567890123456789012123456789012345678901 2
12345678901234567890123456789012123456789012345678901 2
1 Before you schedule your actual GRE CAT, make sure you complete your
12345678901234567890123456789012123456789012345678901 2
12345678901234567890123456789012123456789012345678901
personalized course of study with this book and take all the practice exams 2
12345678901234567890123456789012123456789012345678901 2
Tip

12345678901234567890123456789012123456789012345678901
we provide. At that point, schedule an exam for two weeks in the future. 2
12345678901234567890123456789012123456789012345678901 2
12345678901234567890123456789012123456789012345678901 22
2345678901234567890123456789012123456789012345678901
Use the two-week interval to take practice CAT exams and write practice
1 esays using the CD-ROM included. Your skills will stay fresh and you will
12345678901234567890123456789012123456789012345678901 2
12345678901234567890123456789012123456789012345678901
be accustomed to taking the test on a computer. 2
12345678901234567890123456789012123456789012345678901
2345678901234567890123456789012123456789012345678901
2
2
1
12345678901234567890123456789012123456789012345678901 2
12345678901234567890123456789012123456789012345678901 2
12345678901234567890123456789012123456789012345678901 2
123456789012345678901234567890121234567890123456789012 7
Part I: Understanding the GRE

123456789012345678901234567890121234567890123456789012
12345678901234567890123456789012123456789012345678901 2
12345678901234567890123456789012123456789012345678901 2
12345678901234567890123456789012123456789012345678901
In addition to the flexible schedule and immediacy of the test results, you 2
12345678901234567890123456789012123456789012345678901 2
12345678901234567890123456789012123456789012345678901
can also send your scores to schools automatically by filling out the proper 2
12345678901234567890123456789012123456789012345678901 2
12345678901234567890123456789012123456789012345678901
computer form before the test.
2345678901234567890123456789012123456789012345678901
2
2
1
12345678901234567890123456789012123456789012345678901 2
12345678901234567890123456789012123456789012345678901 2
12345678901234567890123456789012123456789012345678901 2
12345678901234567890123456789012123456789012345678901
Why the GRE CAT Doesn’t Rock
2345678901234567890123456789012123456789012345678901
2
2
1
12345678901234567890123456789012123456789012345678901 2
12345678901234567890123456789012123456789012345678901
No matter what the standardized test is, one fundamental truth always 2
12345678901234567890123456789012123456789012345678901
holds true: The more familiar you are with the test format, the better your 2
12345678901234567890123456789012123456789012345678901 2
1 score will be. Familiarity with the test design allows you to focus more 2
2345678901234567890123456789012123456789012345678901
12345678901234567890123456789012123456789012345678901 2
12345678901234567890123456789012123456789012345678901 2
12345678901234567890123456789012123456789012345678901
attention on the questions themselves and expend less mental effort just 2
12345678901234567890123456789012123456789012345678901
2345678901234567890123456789012123456789012345678901
2
2
1 figuring out the test itself. This axiom holds true whether you are taking
12345678901234567890123456789012123456789012345678901 2
12345678901234567890123456789012123456789012345678901
the MCAT (medical school exam) or a third grade math and reading exam. 2
12345678901234567890123456789012123456789012345678901 2
12345678901234567890123456789012123456789012345678901
Now, answer this question: How many computer adaptive tests have you 2
12345678901234567890123456789012123456789012345678901 2
2345678901234567890123456789012123456789012345678901
12345678901234567890123456789012123456789012345678901
taken in your lifetime? 2
12345678901234567890123456789012123456789012345678901 2
12345678901234567890123456789012123456789012345678901
Most people would answer “none” or “one or two” to the above question, 2
2
12345678901234567890123456789012123456789012345678901
12345678901234567890123456789012123456789012345678901 2
1 but the real answer is, “not enough to be comfortable with them.” The 2
12345678901234567890123456789012123456789012345678901 2
12345678901234567890123456789012123456789012345678901
adaptive format is unusual, and this can work against your score. Most 2
12345678901234567890123456789012123456789012345678901 2
12345678901234567890123456789012123456789012345678901
people are already a little nervous about taking the GRE, and the added 2
12345678901234567890123456789012123456789012345678901
stress of having to tap dance with the three-legged wonder that is the GRE 2
12345678901234567890123456789012123456789012345678901 2
12345678901234567890123456789012123456789012345678901 2
12345678901234567890123456789012123456789012345678901
CAT only makes matters worse. 2
12345678901234567890123456789012123456789012345678901 2
2345678901234567890123456789012123456789012345678901
In addition to being stranger than a paper-and-pencil test, the GRE CAT is 2
12345678901234567890123456789012123456789012345678901
12345678901234567890123456789012123456789012345678901 2
12345678901234567890123456789012123456789012345678901 2
1 riskier. Here’s why: 2
12345678901234567890123456789012123456789012345678901 2
2345678901234567890123456789012123456789012345678901
12345678901234567890123456789012123456789012345678901 2 2
1
12345678901234567890123456789012123456789012345678901
Typical Test GRE CAT What It Means 2
12345678901234567890123456789012123456789012345678901
2345678901234567890123456789012123456789012345678901
2
2
12345678901234567890123456789012123456789012345678901
12345678901234567890123456789012123456789012345678901
Every question Earlier questions are Minor mistakes at 2
1 2
12345678901234567890123456789012123456789012345678901
counts the same “weighted” more the beginning have 2
12345678901234567890123456789012123456789012345678901 2
12345678901234567890123456789012123456789012345678901 2
2345678901234567890123456789012123456789012345678901
toward your score. than later ones. greater negative
12345678901234567890123456789012123456789012345678901 2
1 impact. 2
12345678901234567890123456789012123456789012345678901 2
12345678901234567890123456789012123456789012345678901 2
You can skip a Must answer each Limits your options
2345678901234567890123456789012123456789012345678901
12345678901234567890123456789012123456789012345678901 2
12345678901234567890123456789012123456789012345678901
question that you question presented. regarding a tough 2
12345678901234567890123456789012123456789012345678901 2
12345678901234567890123456789012123456789012345678901 2
don’t understand. You can’t proceed problem you don’t
2
1
12345678901234567890123456789012123456789012345678901
to the next question understand—you 2
12345678901234567890123456789012123456789012345678901 2
12345678901234567890123456789012123456789012345678901
until you have sub- must guess. 2
12345678901234567890123456789012123456789012345678901 2
12345678901234567890123456789012123456789012345678901 2
mitted an answer to
2345678901234567890123456789012123456789012345678901
12345678901234567890123456789012123456789012345678901 2
12345678901234567890123456789012123456789012345678901
the current one. 2
12345678901234567890123456789012123456789012345678901 2
12345678901234567890123456789012123456789012345678901
Can go back and Once the question is Adaptive format 2
1 2
12345678901234567890123456789012123456789012345678901 2
2345678901234567890123456789012123456789012345678901
check work for answered and veri- eliminates the abil-
2
1
12345678901234567890123456789012123456789012345678901
careless errors. fied, it’s out of your ity to go back and 2
12345678901234567890123456789012123456789012345678901 2
12345678901234567890123456789012123456789012345678901
hands.
2345678901234567890123456789012123456789012345678901
catch any mistakes. 2
2
1
12345678901234567890123456789012123456789012345678901 2
12345678901234567890123456789012123456789012345678901 2
12345678901234567890123456789012123456789012345678901 2
8 123456789012345678901234567890121234567890123456789012

www.petersons.com
Chapter 1: Everything You Need to Know about the GRE CAT

123456789012345678901234567890121234567890123456789012
12345678901234567890123456789012123456789012345678901 2
12345678901234567890123456789012123456789012345678901 2
12345678901234567890123456789012123456789012345678901
If the GRE CAT were a pencil-and-paper test, you could begin with the 2
12345678901234567890123456789012123456789012345678901 2
12345678901234567890123456789012123456789012345678901
easy questions and skip over any questions that were too tough for you on 2
12345678901234567890123456789012123456789012345678901 2
12345678901234567890123456789012123456789012345678901
the first pass. On a second pass, you could review your work to catch any 2
1 careless mistakes, and devote time on those problems you skipped. This 2
2345678901234567890123456789012123456789012345678901
12345678901234567890123456789012123456789012345678901 2
12345678901234567890123456789012123456789012345678901 2
12345678901234567890123456789012123456789012345678901
sound strategy is no longer available to you, because of the adaptive nature 2
12345678901234567890123456789012123456789012345678901
2345678901234567890123456789012123456789012345678901
2
2
1 of the test. Instead, you are thrown into the middle of the fray, and how
12345678901234567890123456789012123456789012345678901 2
12345678901234567890123456789012123456789012345678901
well you answer the first half of the questions posed to you goes a long way 2
12345678901234567890123456789012123456789012345678901 2
12345678901234567890123456789012123456789012345678901
toward determining your score.
2345678901234567890123456789012123456789012345678901
2
2
1
12345678901234567890123456789012123456789012345678901 2
12345678901234567890123456789012123456789012345678901
This doesn’t mean the adaptive nature will invariably lead to an unfairly 2
12345678901234567890123456789012123456789012345678901
low score. You could be given an initial set of questions that plays right 2
12345678901234567890123456789012123456789012345678901 2
1 into your hands, like a hard antonym that you just happen to know the 2
2345678901234567890123456789012123456789012345678901
12345678901234567890123456789012123456789012345678901 2
12345678901234567890123456789012123456789012345678901
definition of or a particular type of math problem you’ve always been 2
12345678901234567890123456789012123456789012345678901 2
12345678901234567890123456789012123456789012345678901
good at. If that’s the case, the weighted nature of the GRE CAT works to 2
12345678901234567890123456789012123456789012345678901 22
2345678901234567890123456789012123456789012345678901
12345678901234567890123456789012123456789012345678901
your advantage. Either way, the GRE CAT is a riskier test because many of
12345678901234567890123456789012123456789012345678901
the fail-safes you have on other standardized tests—getting easy problems 2
2
12345678901234567890123456789012123456789012345678901
12345678901234567890123456789012123456789012345678901
first, being able to go back over the entire section and not just one 2
2
12345678901234567890123456789012123456789012345678901 2
1 question—are not present on the GRE CAT.
12345678901234567890123456789012123456789012345678901 2
12345678901234567890123456789012123456789012345678901 2
12345678901234567890123456789012123456789012345678901 2
12345678901234567890123456789012123456789012345678901 22
2345678901234567890123456789012123456789012345678901
So, yes, you get your score back super early, but if you’re not ready for the
12345678901234567890123456789012123456789012345678901
adaptive format, it might not be the kind of score you want to get back 2
12345678901234567890123456789012123456789012345678901
12345678901234567890123456789012123456789012345678901 2
12345678901234567890123456789012123456789012345678901
super early. 2
12345678901234567890123456789012123456789012345678901 22
12345678901234567890123456789012123456789012345678901
Learning about the computer adaptive format will give you the opportu- 2
12345678901234567890123456789012123456789012345678901
12345678901234567890123456789012123456789012345678901 2
1 nity to eliminate many of the negative aspects of the CAT while keeping 2
2345678901234567890123456789012123456789012345678901
12345678901234567890123456789012123456789012345678901
the good ones, so the rest of this chapter is devoted to just that. 2
12345678901234567890123456789012123456789012345678901 2
1 2
12345678901234567890123456789012123456789012345678901 2
12345678901234567890123456789012123456789012345678901 2
12345678901234567890123456789012123456789012345678901 2
12345678901234567890123456789012123456789012345678901
Putting the “Computer” into the Computer 2
12345678901234567890123456789012123456789012345678901 2
12345678901234567890123456789012123456789012345678901
Adaptive Test 2
12345678901234567890123456789012123456789012345678901 2
2345678901234567890123456789012123456789012345678901
12345678901234567890123456789012123456789012345678901 2
12345678901234567890123456789012123456789012345678901
Take a moment and give an honest answer to the following question: 2
1 2
12345678901234567890123456789012123456789012345678901
How would you describe your comfort level with computers? 2
12345678901234567890123456789012123456789012345678901 2
2345678901234567890123456789012123456789012345678901
12345678901234567890123456789012123456789012345678901
In other words, do you feel your trusty laptop is like an extension of your 2
2
12345678901234567890123456789012123456789012345678901 2
12345678901234567890123456789012123456789012345678901
body, or do you believe that computers are little more than mutant 2
12345678901234567890123456789012123456789012345678901 2
1 typewriters determined to destroy the world? Take your time.
12345678901234567890123456789012123456789012345678901 2
12345678901234567890123456789012123456789012345678901 2
12345678901234567890123456789012123456789012345678901 2
12345678901234567890123456789012123456789012345678901
Whichever category you start out in, by the time you take the GRE CAT 2
12345678901234567890123456789012123456789012345678901 2
12345678901234567890123456789012123456789012345678901 22
2345678901234567890123456789012123456789012345678901
you need to be completely comfortable navigating through a computer
12345678901234567890123456789012123456789012345678901
test. If you start out already feeling good about computers, then it won’t 2
12345678901234567890123456789012123456789012345678901 2
12345678901234567890123456789012123456789012345678901
1 take much time to get used to the commands used when taking the GRE 2
2345678901234567890123456789012123456789012345678901
12345678901234567890123456789012123456789012345678901 2
1 CAT. If you’re closer to the “mutant typewriter” faction, then the amount 2
12345678901234567890123456789012123456789012345678901
of time needed to acclimate yourself will probably be much longer. Even if 2
12345678901234567890123456789012123456789012345678901 2
12345678901234567890123456789012123456789012345678901
that is the case, it will be time well spent,
2345678901234567890123456789012123456789012345678901 since the day you need to take the 2
2
1
12345678901234567890123456789012123456789012345678901 2
12345678901234567890123456789012123456789012345678901 2
12345678901234567890123456789012123456789012345678901 2
123456789012345678901234567890121234567890123456789012 9
Part I: Understanding the GRE

123456789012345678901234567890121234567890123456789012
12345678901234567890123456789012123456789012345678901 2
12345678901234567890123456789012123456789012345678901 2
12345678901234567890123456789012123456789012345678901
test should be exclusively about answering GRE questions correctly and 2
12345678901234567890123456789012123456789012345678901 2
12345678901234567890123456789012123456789012345678901
not about learning how to move through a computer adaptive test. 2
12345678901234567890123456789012123456789012345678901 2
12345678901234567890123456789012123456789012345678901
Before starting the GRE CAT, you go through a computer tutorial, which 2
12345678901234567890123456789012123456789012345678901 2
12345678901234567890123456789012123456789012345678901 2
12345678901234567890123456789012123456789012345678901
goes over every facet of the onscreen instructions in excruciating detail. 2
12345678901234567890123456789012123456789012345678901 2
12345678901234567890123456789012123456789012345678901
You could wait until test day and then spend 15–45 minutes in the tutorial 2
1 familiarizing yourself with all the different instructions, but wouldn’t you 2
2345678901234567890123456789012123456789012345678901
12345678901234567890123456789012123456789012345678901 2
12345678901234567890123456789012123456789012345678901 2
12345678901234567890123456789012123456789012345678901
rather learn the instructions beforehand so you can skip the boring tutorial 2
12345678901234567890123456789012123456789012345678901
2345678901234567890123456789012123456789012345678901
2
2
1 and get right to the actual test? The goal on that day is to use all your
12345678901234567890123456789012123456789012345678901 2
12345678901234567890123456789012123456789012345678901
brainpower to take a GRE CAT, not waste time and mental energy 2
12345678901234567890123456789012123456789012345678901
learning about how to take a GRE CAT. 2
12345678901234567890123456789012123456789012345678901
2345678901234567890123456789012123456789012345678901
2
2
1
12345678901234567890123456789012123456789012345678901
Be sure you go over this tutorial before taking any tests on that disk. 2
12345678901234567890123456789012123456789012345678901 2
12345678901234567890123456789012123456789012345678901
Learning about the computer commands ahead of time is a valuable skill. 2
12345678901234567890123456789012123456789012345678901 2
12345678901234567890123456789012123456789012345678901
A tutorial similar to the one you’ll see on test day is contained on the CD 2
12345678901234567890123456789012123456789012345678901 2
12345678901234567890123456789012123456789012345678901
that came with this book. Or, check out the Appendix on page 332, which 2
12345678901234567890123456789012123456789012345678901 2
12345678901234567890123456789012123456789012345678901
goes over all the major screen commands, what they look like, what each 2
2345678901234567890123456789012123456789012345678901
12345678901234567890123456789012123456789012345678901 2
1 command means, and when they will appear. Be sure you understand all 2
12345678901234567890123456789012123456789012345678901 2
12345678901234567890123456789012123456789012345678901
the different commands before you take the actual GRE CAT. 2
12345678901234567890123456789012123456789012345678901 2
2345678901234567890123456789012123456789012345678901
Now that we’ve talked about the instructions you’ll see on the computer, 2
12345678901234567890123456789012123456789012345678901
12345678901234567890123456789012123456789012345678901 2
12345678901234567890123456789012123456789012345678901 2
12345678901234567890123456789012123456789012345678901 2
let’s talk about the computers themselves. You know those computers you
see on every TV show, the ones with monstrous flat screens, dynamic 2
12345678901234567890123456789012123456789012345678901
12345678901234567890123456789012123456789012345678901 2
12345678901234567890123456789012123456789012345678901 2
12345678901234567890123456789012123456789012345678901 2
graphics, and blazing computing speed? Those are NOT the computers
12345678901234567890123456789012123456789012345678901 2
1 you will use to take the GRE CAT. Instead, the computer you will probably 2
2345678901234567890123456789012123456789012345678901
use resembles the kind you find in a public library. It will be a no-frills, 2
12345678901234567890123456789012123456789012345678901
12345678901234567890123456789012123456789012345678901 2
1 functional PC with an average keyboard, average mouse, average screen, 2
12345678901234567890123456789012123456789012345678901 2
12345678901234567890123456789012123456789012345678901
average everything, really. 2
12345678901234567890123456789012123456789012345678901 2
12345678901234567890123456789012123456789012345678901 2
12345678901234567890123456789012123456789012345678901
Many people who work on computers for a long time make modifications 2
12345678901234567890123456789012123456789012345678901
to their computers to maximize comfort. They might replace the regular 2
12345678901234567890123456789012123456789012345678901 2
2345678901234567890123456789012123456789012345678901
mouse with a wireless trackball version designed to reduce carpal tunnel 2
12345678901234567890123456789012123456789012345678901
12345678901234567890123456789012123456789012345678901 2
1 problems and place ultra-fine netting over their screen to reduce the glare. 2
12345678901234567890123456789012123456789012345678901 2
12345678901234567890123456789012123456789012345678901
None of these modifications will be in place on the GRE computer, so if 2
12345678901234567890123456789012123456789012345678901 2
12345678901234567890123456789012123456789012345678901 2
12345678901234567890123456789012123456789012345678901
you are someone who uses modified accessories, you have two options: 2
12345678901234567890123456789012123456789012345678901 2
12345678901234567890123456789012123456789012345678901 2
12345678901234567890123456789012123456789012345678901 2
2345678901234567890123456789012123456789012345678901
1. You can replace your nice gear with the basic equipment in order
2
12345678901234567890123456789012123456789012345678901
to simulate the real GRE experience.
2
12345678901234567890123456789012123456789012345678901
12345678901234567890123456789012123456789012345678901 2
1 2. You can use your nice gear at home but be aware that during the 2
12345678901234567890123456789012123456789012345678901 2
12345678901234567890123456789012123456789012345678901
real test none of it will be there for you. 2
12345678901234567890123456789012123456789012345678901 2
12345678901234567890123456789012123456789012345678901 2
12345678901234567890123456789012123456789012345678901 2
Both options have advantages. Option 1 gives you greater verisimilitude (a
2345678901234567890123456789012123456789012345678901
12345678901234567890123456789012123456789012345678901 2
1 good GRE vocabulary word, by the way). If you want your computer 2
12345678901234567890123456789012123456789012345678901
practice to be as close to the real thing as possible, this is the plan for you. 2
12345678901234567890123456789012123456789012345678901 2
12345678901234567890123456789012123456789012345678901
2345678901234567890123456789012123456789012345678901
2
2
1 On the other hand, if you understand ahead of time what the real thing
12345678901234567890123456789012123456789012345678901 2
12345678901234567890123456789012123456789012345678901 2
will be like, there’s no reason not to practice in a way that’s more
12345678901234567890123456789012123456789012345678901 2
10 123456789012345678901234567890121234567890123456789012

www.petersons.com
Chapter 1: Everything You Need to Know about the GRE CAT

123456789012345678901234567890121234567890123456789012
12345678901234567890123456789012123456789012345678901 2
12345678901234567890123456789012123456789012345678901 2
12345678901234567890123456789012123456789012345678901
physically or aesthetically comfortable. Ultimately, the type of computer 2
12345678901234567890123456789012123456789012345678901 2
12345678901234567890123456789012123456789012345678901
mouse you use won’t be the deciding factor in your score, so this is not a 2
12345678901234567890123456789012123456789012345678901 2
12345678901234567890123456789012123456789012345678901
major point, but every little thing counts. Knowing what to expect and 2
1 planning for it is a good way to boost your confidence level going into 2
2345678901234567890123456789012123456789012345678901
12345678901234567890123456789012123456789012345678901 2
12345678901234567890123456789012123456789012345678901 2
12345678901234567890123456789012123456789012345678901
the exam. 2
12345678901234567890123456789012123456789012345678901
2345678901234567890123456789012123456789012345678901
2
2
1
12345678901234567890123456789012123456789012345678901 2
12345678901234567890123456789012123456789012345678901
Before you register for the test by phone or computer, try to visit the 2
12345678901234567890123456789012123456789012345678901 2
12345678901234567890123456789012123456789012345678901
actual testing site ahead of time—before you plunk down your money. 2
Alert!

2345678901234567890123456789012123456789012345678901
1 Most testing sites are fine, but what if the testing site gives you a bad vibe 2
12345678901234567890123456789012123456789012345678901 2
12345678901234567890123456789012123456789012345678901 2
12345678901234567890123456789012123456789012345678901
or something? Don’t go there! You need to be at your best on test day, so 2
12345678901234567890123456789012123456789012345678901
2345678901234567890123456789012123456789012345678901
2
2
1 make sure the site is adequate before you sign up. Once you call and get a
12345678901234567890123456789012123456789012345678901 2
12345678901234567890123456789012123456789012345678901
time, you’re locked in, and even if you cancel the appointment, you lose a 2
12345678901234567890123456789012123456789012345678901 2
12345678901234567890123456789012123456789012345678901
large chunk of change. 2
12345678901234567890123456789012123456789012345678901 22
2345678901234567890123456789012123456789012345678901
12345678901234567890123456789012123456789012345678901 2
12345678901234567890123456789012123456789012345678901
12345678901234567890123456789012123456789012345678901
The “Adaptive” 2
12345678901234567890123456789012123456789012345678901 22
12345678901234567890123456789012123456789012345678901
1 To get a taste of what the computer adaptive format is like (without taking 2
12345678901234567890123456789012123456789012345678901
a test), rent a movie you know nothing about. Don’t look at the cover or 2
12345678901234567890123456789012123456789012345678901 2
12345678901234567890123456789012123456789012345678901
read any summary of the events of the film. Fast forward to the middle of 2
2345678901234567890123456789012123456789012345678901
12345678901234567890123456789012123456789012345678901
the film and begin watching at that point. After a few minutes, if you think 2
2
12345678901234567890123456789012123456789012345678901
12345678901234567890123456789012123456789012345678901
you understand what’s going on, keep watching. If you start watching the 2
2
12345678901234567890123456789012123456789012345678901
12345678901234567890123456789012123456789012345678901
middle of the film and have no idea what’s going on, go back about 20 2
2
12345678901234567890123456789012123456789012345678901
minutes and start watching from that point. Move forward when the 2
12345678901234567890123456789012123456789012345678901
12345678901234567890123456789012123456789012345678901 2
12345678901234567890123456789012123456789012345678901 2
1 movie makes sense, and rewind when it doesn’t. Keep watching until you 2
2345678901234567890123456789012123456789012345678901
12345678901234567890123456789012123456789012345678901 2
12345678901234567890123456789012123456789012345678901
reach the end of the film. 2
1 2
12345678901234567890123456789012123456789012345678901 2
12345678901234567890123456789012123456789012345678901
You have just watched your first adaptive
2345678901234567890123456789012123456789012345678901
movie. Was it fun? How was the 2
2
12345678901234567890123456789012123456789012345678901
12345678901234567890123456789012123456789012345678901
popcorn? 2
1 2
12345678901234567890123456789012123456789012345678901
You probably don’t watch movies this way on a regular basis, and you 2
12345678901234567890123456789012123456789012345678901 2
12345678901234567890123456789012123456789012345678901
don’t take computer adaptive tests like the GRE every day, either. The 2
12345678901234567890123456789012123456789012345678901 2
12345678901234567890123456789012123456789012345678901
danger is that the unusual format leads you to take the wrong approach to 2
12345678901234567890123456789012123456789012345678901 2
12345678901234567890123456789012123456789012345678901
the test. The chart on the following page lists some common mistakes 2
2345678901234567890123456789012123456789012345678901
12345678901234567890123456789012123456789012345678901 2
12345678901234567890123456789012123456789012345678901
people make when taking the CAT. 2
12345678901234567890123456789012123456789012345678901 22
12345678901234567890123456789012123456789012345678901 2
1
12345678901234567890123456789012123456789012345678901 2
12345678901234567890123456789012123456789012345678901 2
12345678901234567890123456789012123456789012345678901 2
12345678901234567890123456789012123456789012345678901 2
12345678901234567890123456789012123456789012345678901 2
12345678901234567890123456789012123456789012345678901 22
2345678901234567890123456789012123456789012345678901
12345678901234567890123456789012123456789012345678901 2
12345678901234567890123456789012123456789012345678901 2
12345678901234567890123456789012123456789012345678901 2
1
12345678901234567890123456789012123456789012345678901 2
12345678901234567890123456789012123456789012345678901 2
12345678901234567890123456789012123456789012345678901 2
12345678901234567890123456789012123456789012345678901 2
12345678901234567890123456789012123456789012345678901 2
12345678901234567890123456789012123456789012345678901 2
12345678901234567890123456789012123456789012345678901 2
12345678901234567890123456789012123456789012345678901 2
12345678901234567890123456789012123456789012345678901 2 11
123456789012345678901234567890121234567890123456789012
Part I: Understanding the GRE

123456789012345678901234567890121234567890123456789012
12345678901234567890123456789012123456789012345678901 2
12345678901234567890123456789012123456789012345678901 2
12345678901234567890123456789012123456789012345678901 2
12345678901234567890123456789012123456789012345678901
Common Thinking But Really . . . 2
12345678901234567890123456789012123456789012345678901 2
12345678901234567890123456789012123456789012345678901 2
12345678901234567890123456789012123456789012345678901
Tests go from easy to GRE CAT
2345678901234567890123456789012123456789012345678901
problems start in the middle, 2
2
1
12345678901234567890123456789012123456789012345678901
hard problems. and then go all over the place from there 2
12345678901234567890123456789012123456789012345678901
depending on how you score. 2
12345678901234567890123456789012123456789012345678901 2
12345678901234567890123456789012123456789012345678901
2345678901234567890123456789012123456789012345678901
2
2
1 Work at a consistent Early problems on the GRE play a greater
12345678901234567890123456789012123456789012345678901 2
12345678901234567890123456789012123456789012345678901
pace to have a role in determining your score than later 2
12345678901234567890123456789012123456789012345678901 2
12345678901234567890123456789012123456789012345678901
chance to answer ones, so you
2345678901234567890123456789012123456789012345678901
should make sure to spend 2
2
1
12345678901234567890123456789012123456789012345678901
every problem. more time on the first 10–15 problems. 2
12345678901234567890123456789012123456789012345678901
Even if this means you end up guessing on 2
12345678901234567890123456789012123456789012345678901 2
12345678901234567890123456789012123456789012345678901
the last five problems in a section because
2345678901234567890123456789012123456789012345678901
2
2
1
12345678901234567890123456789012123456789012345678901
you have no time left, your score will still 2
12345678901234567890123456789012123456789012345678901 2
12345678901234567890123456789012123456789012345678901
be better than if you rushed through the 2
12345678901234567890123456789012123456789012345678901 2
12345678901234567890123456789012123456789012345678901 2
2345678901234567890123456789012123456789012345678901
first questions. Remember, even if you
12345678901234567890123456789012123456789012345678901 2
12345678901234567890123456789012123456789012345678901
don’t have time for the last few problems, 2
12345678901234567890123456789012123456789012345678901 2
12345678901234567890123456789012123456789012345678901 2
be sure to guess.
12345678901234567890123456789012123456789012345678901 2
1 If I study hard If you have the necessary knowledge, then 2
12345678901234567890123456789012123456789012345678901 2
12345678901234567890123456789012123456789012345678901
enough, the test you’ll quickly find yourself mired in the 2
12345678901234567890123456789012123456789012345678901 2
12345678901234567890123456789012123456789012345678901 2
2345678901234567890123456789012123456789012345678901
should be easy. hardest math and verbal questions the
12345678901234567890123456789012123456789012345678901 2
12345678901234567890123456789012123456789012345678901
computer can dredge up. This means you’re 2
12345678901234567890123456789012123456789012345678901 2
12345678901234567890123456789012123456789012345678901 2
heading toward a good score, but it doesn’t
12345678901234567890123456789012123456789012345678901
mean the test will be easy. Unlike a pencil- 2
12345678901234567890123456789012123456789012345678901 2
12345678901234567890123456789012123456789012345678901
and-paper test with set easy, medium, and 2
12345678901234567890123456789012123456789012345678901 2 2
1 hard problems, the computer will adapt to
12345678901234567890123456789012123456789012345678901 2
12345678901234567890123456789012123456789012345678901
throw you the nastiest, toughest, most 2
12345678901234567890123456789012123456789012345678901 2
12345678901234567890123456789012123456789012345678901 2
convoluted questions it has to offer.
12345678901234567890123456789012123456789012345678901 2
12345678901234567890123456789012123456789012345678901 2
12345678901234567890123456789012123456789012345678901 2
12345678901234567890123456789012123456789012345678901 2
On many standardized tests, people rush through the initial easy questions
12345678901234567890123456789012123456789012345678901
because they want to answer every problem. This impatience often leads to 2
12345678901234567890123456789012123456789012345678901 2
2345678901234567890123456789012123456789012345678901
careless errors at the beginning. This impatience is harmful on a regular test, 2
12345678901234567890123456789012123456789012345678901
12345678901234567890123456789012123456789012345678901 2
1 but it’s deadly on the GRE CAT. The first 10–15 questions you face on each 2
12345678901234567890123456789012123456789012345678901 2
12345678901234567890123456789012123456789012345678901
section are the most crucial, and you should slow down and take as much as 2
12345678901234567890123456789012123456789012345678901 2
12345678901234567890123456789012123456789012345678901
2 minutes apiece if it will help you answer them correctly. As for the last 5 2
12345678901234567890123456789012123456789012345678901 2
12345678901234567890123456789012123456789012345678901
questions? If you don’t have time for them, just guess. They are not as critical 2
12345678901234567890123456789012123456789012345678901 2
12345678901234567890123456789012123456789012345678901
to your score. If you can get to them, it’s a bonus, but the real way to improve 2
12345678901234567890123456789012123456789012345678901 2
12345678901234567890123456789012123456789012345678901
your score is to focus on the earlier questions to the best of your ability. 2
12345678901234567890123456789012123456789012345678901 2
12345678901234567890123456789012123456789012345678901 2
12345678901234567890123456789012123456789012345678901 2
2345678901234567890123456789012123456789012345678901
Now that the “Slow down and get the early question right” mantra has
been beaten into your skull yet again, it might be a good time to clarify just 2
12345678901234567890123456789012123456789012345678901
12345678901234567890123456789012123456789012345678901 2
12345678901234567890123456789012123456789012345678901 2
1 what an easy, medium, and hard question looks like. Officially, a hard 2
2345678901234567890123456789012123456789012345678901
12345678901234567890123456789012123456789012345678901 2
1 problem is one that previous test-takers have seen and found to be very 2
12345678901234567890123456789012123456789012345678901
difficult to answer correctly. The official derivation of “easy” and 2
12345678901234567890123456789012123456789012345678901 2
12345678901234567890123456789012123456789012345678901
2345678901234567890123456789012123456789012345678901
2
2
1 “medium” also comes from how well previous test-takers did when trying
12345678901234567890123456789012123456789012345678901 2
12345678901234567890123456789012123456789012345678901 2
to answer these problems.
12345678901234567890123456789012123456789012345678901 2
12 123456789012345678901234567890121234567890123456789012

www.petersons.com
Chapter 1: Everything You Need to Know about the GRE CAT

123456789012345678901234567890121234567890123456789012
12345678901234567890123456789012123456789012345678901 2
12345678901234567890123456789012123456789012345678901 2
12345678901234567890123456789012123456789012345678901
Yet these labels are all subjective, since everyone carries around a unique 2
12345678901234567890123456789012123456789012345678901 2
12345678901234567890123456789012123456789012345678901
knowledge set. You might have a knack for geometry, so a medium 2
12345678901234567890123456789012123456789012345678901 2
12345678901234567890123456789012123456789012345678901
geometry problem appears easy to you. The best way to get a feel for a 2
1 problem’s difficulty level is to look over samples of easy, medium, and 2
2345678901234567890123456789012123456789012345678901
12345678901234567890123456789012123456789012345678901 2
12345678901234567890123456789012123456789012345678901 2
12345678901234567890123456789012123456789012345678901
hard questions. You can find these throughout this book, and you can also 2
12345678901234567890123456789012123456789012345678901
2345678901234567890123456789012123456789012345678901
2
2
1 go right to the source and purchase The Official Guide to the GRE, 10 th

12345678901234567890123456789012123456789012345678901 2
12345678901234567890123456789012123456789012345678901
Edition, by the Educational Testing Service. This roughly $20 book 2
12345678901234567890123456789012123456789012345678901 2
12345678901234567890123456789012123456789012345678901
contains a host of old pencil-and-paper GREs with questions arranged in 2
1 old-timey easy/medium/hard order of difficulty. The exams are all over a 2
2345678901234567890123456789012123456789012345678901
12345678901234567890123456789012123456789012345678901 2
12345678901234567890123456789012123456789012345678901 2
12345678901234567890123456789012123456789012345678901
decade old, but they do give you an idea of what constitutes an easy, 2
12345678901234567890123456789012123456789012345678901
2345678901234567890123456789012123456789012345678901
2
2
1 medium, and hard problem.
12345678901234567890123456789012123456789012345678901 2
12345678901234567890123456789012123456789012345678901 2
12345678901234567890123456789012123456789012345678901 2
12345678901234567890123456789012123456789012345678901 2
X-Ref

12345678901234567890123456789012123456789012345678901 22
2345678901234567890123456789012123456789012345678901
The Official Guide is a good place to work on sample problems, but the
12345678901234567890123456789012123456789012345678901
format has changed a lot since these test were issued, so don’t bother
12345678901234567890123456789012123456789012345678901 2
12345678901234567890123456789012123456789012345678901
working the sections under timed conditions. Use the sample tests at the 2
12345678901234567890123456789012123456789012345678901 22
12345678901234567890123456789012123456789012345678901
1 back of this book for that, as they are geared toward the new exam. 2
12345678901234567890123456789012123456789012345678901 2
12345678901234567890123456789012123456789012345678901 2
12345678901234567890123456789012123456789012345678901 2
2345678901234567890123456789012123456789012345678901
The ability to determine easy, medium, and hard problems is a nice skill to 2
12345678901234567890123456789012123456789012345678901
12345678901234567890123456789012123456789012345678901
have on the CAT, but it is not a crucial one. Don’t spend any amount of 2
2
12345678901234567890123456789012123456789012345678901
12345678901234567890123456789012123456789012345678901
time looking at a problem and debating whether or not it’s a medium or 2
2
12345678901234567890123456789012123456789012345678901
12345678901234567890123456789012123456789012345678901
hard one. Either way, it’s in front of you, so you have to work it one way or 2
2
12345678901234567890123456789012123456789012345678901
12345678901234567890123456789012123456789012345678901
another. But understanding what kind of problem you are looking at can 2
2
12345678901234567890123456789012123456789012345678901
1 help you avoid incorrect answer choices. For example, if you are taking the 2
12345678901234567890123456789012123456789012345678901 2
12345678901234567890123456789012123456789012345678901
math section and doing pretty well, you’ll notice the problems are getting 2
12345678901234567890123456789012123456789012345678901 2
12345678901234567890123456789012123456789012345678901 2
12345678901234567890123456789012123456789012345678901
more difficult. Suddenly, you get the following probability question: 2
12345678901234567890123456789012123456789012345678901 22
2345678901234567890123456789012123456789012345678901
12345678901234567890123456789012123456789012345678901 2
1 There are seven tokens in a bag, labeled 8 through 14 consecutively. The
12345678901234567890123456789012123456789012345678901 2
12345678901234567890123456789012123456789012345678901
first number pulled out of the bag is 10. What are the odds that the next 2
12345678901234567890123456789012123456789012345678901 22
2345678901234567890123456789012123456789012345678901
12345678901234567890123456789012123456789012345678901
two numbers pulled out of the bag will BOTH be less than 10?
2
1
12345678901234567890123456789012123456789012345678901 2
12345678901234567890123456789012123456789012345678901
1 2
2345678901234567890123456789012123456789012345678901
12345678901234567890123456789012123456789012345678901
A. 2
12345678901234567890123456789012123456789012345678901
15 2
12345678901234567890123456789012123456789012345678901 22
12345678901234567890123456789012123456789012345678901
3
2
1
2345678901234567890123456789012123456789012345678901
12345678901234567890123456789012123456789012345678901
B. 2
12345678901234567890123456789012123456789012345678901
14 2
12345678901234567890123456789012123456789012345678901 2
12345678901234567890123456789012123456789012345678901
2 2
1 C. 2
12345678901234567890123456789012123456789012345678901 22
2345678901234567890123456789012123456789012345678901
7
12345678901234567890123456789012123456789012345678901 2
12345678901234567890123456789012123456789012345678901
1
2
12345678901234567890123456789012123456789012345678901
D.
2
1 3
12345678901234567890123456789012123456789012345678901 22
2345678901234567890123456789012123456789012345678901
1
12345678901234567890123456789012123456789012345678901
2 2
12345678901234567890123456789012123456789012345678901
E. 2
12345678901234567890123456789012123456789012345678901
5
2345678901234567890123456789012123456789012345678901
2
2
1
12345678901234567890123456789012123456789012345678901 2
12345678901234567890123456789012123456789012345678901 2
12345678901234567890123456789012123456789012345678901 2 13
123456789012345678901234567890121234567890123456789012
Part I: Understanding the GRE

123456789012345678901234567890121234567890123456789012
12345678901234567890123456789012123456789012345678901 2
12345678901234567890123456789012123456789012345678901 2
12345678901234567890123456789012123456789012345678901 2
12345678901234567890123456789012123456789012345678901
On the CAT exam, letters will not be used to designate choices, as each 2
12345678901234567890123456789012123456789012345678901 2
12345678901234567890123456789012123456789012345678901 2
12345678901234567890123456789012123456789012345678901
choice will simply appear on the screen with a hollow oval next to it. 2
1 Obviously, the test-maker did not care that this lack of lettering would 2
2345678901234567890123456789012123456789012345678901

Note
12345678901234567890123456789012123456789012345678901 2
12345678901234567890123456789012123456789012345678901
make it hard on the poor souls who make a living writing test-prep 2
12345678901234567890123456789012123456789012345678901 2
12345678901234567890123456789012123456789012345678901
material. Throughout this book, all answer choices will have letters next 2
2345678901234567890123456789012123456789012345678901 2
1
12345678901234567890123456789012123456789012345678901
to them so that they can be discussed without having to resort to phrases 2
12345678901234567890123456789012123456789012345678901 2
12345678901234567890123456789012123456789012345678901
like “the second oval from the top” or “you know, the answer that’s got 2
12345678901234567890123456789012123456789012345678901 2
12345678901234567890123456789012123456789012345678901
the word ‘hamster’ in it.” 2
12345678901234567890123456789012123456789012345678901 2
12345678901234567890123456789012123456789012345678901 2
12345678901234567890123456789012123456789012345678901 2
12345678901234567890123456789012123456789012345678901
This is a medium → difficult problem, something to keep in mind when 2
1 you look over the answer choices. If you try to figure this problem in your 2
2345678901234567890123456789012123456789012345678901
12345678901234567890123456789012123456789012345678901 2
12345678901234567890123456789012123456789012345678901 2
12345678901234567890123456789012123456789012345678901
head, you might think that there are two numbers less than 10 (8 and 9) 2
12345678901234567890123456789012123456789012345678901 2
12345678901234567890123456789012123456789012345678901
and 7 tokens total, or 6 tokens now that the 10 has been removed. Choice 2
12345678901234567890123456789012123456789012345678901 2
12345678901234567890123456789012123456789012345678901
2 2
12345678901234567890123456789012123456789012345678901
(C) is (which is two tokens over 7 tokens, right), while choice (D) has 2
12345678901234567890123456789012123456789012345678901
7 2
12345678901234567890123456789012123456789012345678901
2 tokens 2 1 2
12345678901234567890123456789012123456789012345678901 2
12345678901234567890123456789012123456789012345678901
5 5 . 2
12345678901234567890123456789012123456789012345678901
6 tokens 6 3 2
12345678901234567890123456789012123456789012345678901 2
12345678901234567890123456789012123456789012345678901 2
2345678901234567890123456789012123456789012345678901
Both of these answers use the wrong math, but they are also too easy. If
you know it is a medium → difficult problem, how could the answer be so 2
12345678901234567890123456789012123456789012345678901
12345678901234567890123456789012123456789012345678901 2
12345678901234567890123456789012123456789012345678901 2
12345678901234567890123456789012123456789012345678901 2
simple that you could just do the math in your head? Finding choice (C)
takes only 15 seconds and choice (D) takes only a bit longer than that. 2
12345678901234567890123456789012123456789012345678901
12345678901234567890123456789012123456789012345678901 2
12345678901234567890123456789012123456789012345678901 2
12345678901234567890123456789012123456789012345678901 2
When you find the answer to a tough problem that easily, your GRE “sixth
1 sense” should start tingling wildly, telling you that you are picking an 2
12345678901234567890123456789012123456789012345678901 2
12345678901234567890123456789012123456789012345678901 2
12345678901234567890123456789012123456789012345678901 2
answer that is so easy it is probably incorrect.
12345678901234567890123456789012123456789012345678901 2
12345678901234567890123456789012123456789012345678901
For this particular problem, eliminating incorrect answers is just as 2
12345678901234567890123456789012123456789012345678901 2
12345678901234567890123456789012123456789012345678901
important as finding the correct choice. If you don’t know how to do the 2
12345678901234567890123456789012123456789012345678901 2
12345678901234567890123456789012123456789012345678901
math, eliminate choices (C) and (D) and take a guess. This gives you a 1 in 2
12345678901234567890123456789012123456789012345678901 2
2345678901234567890123456789012123456789012345678901
3 shot of choosing the right answer, which happens to be choice (A). Even 2
12345678901234567890123456789012123456789012345678901
12345678901234567890123456789012123456789012345678901 2
1 if you guessed wrong that time, you gave yourself a fighting chance by 2
12345678901234567890123456789012123456789012345678901 2
12345678901234567890123456789012123456789012345678901
understanding the GRE CAT well enough to know which answer choices 2
2345678901234567890123456789012123456789012345678901
12345678901234567890123456789012123456789012345678901 2
12345678901234567890123456789012123456789012345678901
were traps. 2
12345678901234567890123456789012123456789012345678901 2
12345678901234567890123456789012123456789012345678901 2 2
1
12345678901234567890123456789012123456789012345678901 2
X-Ref

12345678901234567890123456789012123456789012345678901 2
12345678901234567890123456789012123456789012345678901 2
12345678901234567890123456789012123456789012345678901
The math regarding this problem is covered on pages 100–101, the 2
12345678901234567890123456789012123456789012345678901 2
12345678901234567890123456789012123456789012345678901 2
2345678901234567890123456789012123456789012345678901
section on probabilities.
12345678901234567890123456789012123456789012345678901 2
12345678901234567890123456789012123456789012345678901 2
12345678901234567890123456789012123456789012345678901 2 2
1
12345678901234567890123456789012123456789012345678901 2
12345678901234567890123456789012123456789012345678901 2
12345678901234567890123456789012123456789012345678901 2
12345678901234567890123456789012123456789012345678901 2
12345678901234567890123456789012123456789012345678901 2
12345678901234567890123456789012123456789012345678901 2
12345678901234567890123456789012123456789012345678901 2
12345678901234567890123456789012123456789012345678901 2
12345678901234567890123456789012123456789012345678901 2
14 123456789012345678901234567890121234567890123456789012

www.petersons.com
Chapter 1: Everything You Need to Know about the GRE CAT

123456789012345678901234567890121234567890123456789012
12345678901234567890123456789012123456789012345678901 2
12345678901234567890123456789012123456789012345678901 2
12345678901234567890123456789012123456789012345678901
The “Test” 2
12345678901234567890123456789012123456789012345678901 2
12345678901234567890123456789012123456789012345678901 2
12345678901234567890123456789012123456789012345678901 2
Use the following checklist to prepare for the day you take the actual GRE:
12345678901234567890123456789012123456789012345678901 2
12345678901234567890123456789012123456789012345678901 2
12345678901234567890123456789012123456789012345678901 2
12345678901234567890123456789012123456789012345678901 2
12345678901234567890123456789012123456789012345678901
Items to Bring to the GRE: 2
12345678901234567890123456789012123456789012345678901
2345678901234567890123456789012123456789012345678901
2
2
1
12345678901234567890123456789012123456789012345678901
1. Good old picture I.D., preferably something legitimate like a 2
12345678901234567890123456789012123456789012345678901
driver’s license and not a badly laminated “Honorary Member of 2
12345678901234567890123456789012123456789012345678901 2
12345678901234567890123456789012123456789012345678901
2345678901234567890123456789012123456789012345678901
2
2
1 the Justice League” card.
12345678901234567890123456789012123456789012345678901 2
12345678901234567890123456789012123456789012345678901 2
12345678901234567890123456789012123456789012345678901
2. Fruit snack or something light, yet filling, like nuts. You don’t 2
12345678901234567890123456789012123456789012345678901
2345678901234567890123456789012123456789012345678901
2
2
1 want to eat something so heavy you find yourself dying for a nap.
12345678901234567890123456789012123456789012345678901 2
12345678901234567890123456789012123456789012345678901
3. A bottle of water. Some would rather drown out the entire 2
12345678901234567890123456789012123456789012345678901 2
12345678901234567890123456789012123456789012345678901
experience, but this practice is frowned upon for a multitude of 2
12345678901234567890123456789012123456789012345678901 22
2345678901234567890123456789012123456789012345678901
12345678901234567890123456789012123456789012345678901
reasons.
12345678901234567890123456789012123456789012345678901 2
12345678901234567890123456789012123456789012345678901 2
12345678901234567890123456789012123456789012345678901 22
4. Overshirt of some kind. When you check out the site initially,
12345678901234567890123456789012123456789012345678901
you should get an idea of what the ambient temperature is like. 2
1
12345678901234567890123456789012123456789012345678901
Dress accordingly. The testing room is often rather small, just a 2
12345678901234567890123456789012123456789012345678901 2
12345678901234567890123456789012123456789012345678901 2
12345678901234567890123456789012123456789012345678901 22
2345678901234567890123456789012123456789012345678901
cubicle farm of 6–10 spaces. Depending on how many people
12345678901234567890123456789012123456789012345678901
are taking tests next to you, the room can be either cold or 2
12345678901234567890123456789012123456789012345678901
warm, so the best advice is to start with light clothing with the 2
12345678901234567890123456789012123456789012345678901
12345678901234567890123456789012123456789012345678901 2
12345678901234567890123456789012123456789012345678901 2
12345678901234567890123456789012123456789012345678901
option of bundling up. 2
12345678901234567890123456789012123456789012345678901 22
12345678901234567890123456789012123456789012345678901
5. A positive attitude. If that’s asking too much, just don’t bring a
2
1
2345678901234567890123456789012123456789012345678901
negative one. Shoot for feeling relaxed and confident, since this 2
12345678901234567890123456789012123456789012345678901
12345678901234567890123456789012123456789012345678901 2
1 disposition will help you stay sharp and focused on the task 2
12345678901234567890123456789012123456789012345678901 2
12345678901234567890123456789012123456789012345678901
at hand. 2
12345678901234567890123456789012123456789012345678901 2
12345678901234567890123456789012123456789012345678901 2
12345678901234567890123456789012123456789012345678901 2
12345678901234567890123456789012123456789012345678901 2
Note

12345678901234567890123456789012123456789012345678901
Many test centers have a set of lockers just inside the testing room for you 2
12345678901234567890123456789012123456789012345678901 22
2345678901234567890123456789012123456789012345678901
12345678901234567890123456789012123456789012345678901
1 to keep valuables and whatever else you want. This is another thing you 2
12345678901234567890123456789012123456789012345678901
should look for when you are scouting out a GRE location. 2
12345678901234567890123456789012123456789012345678901 2
12345678901234567890123456789012123456789012345678901 22
2345678901234567890123456789012123456789012345678901
12345678901234567890123456789012123456789012345678901 2
12345678901234567890123456789012123456789012345678901 2
12345678901234567890123456789012123456789012345678901
1 The test’s sign-up process is a mixture of registering for a seminar and 2
12345678901234567890123456789012123456789012345678901
getting arrested. You have to write out a mini-essay claiming you’ll be a 2
12345678901234567890123456789012123456789012345678901 2
12345678901234567890123456789012123456789012345678901
good person and not sell your GRE secrets to a cheesy newspaper 2
12345678901234567890123456789012123456789012345678901 2
12345678901234567890123456789012123456789012345678901
tabloid—or anyone else for that matter. Then you sign in again, have 2
2345678901234567890123456789012123456789012345678901
12345678901234567890123456789012123456789012345678901
your identification checked, and get a mug shot taken . . . just in case. In 2
2
12345678901234567890123456789012123456789012345678901 2
12345678901234567890123456789012123456789012345678901
many locations, you are also filmed throughout the testing procedure. 2
12345678901234567890123456789012123456789012345678901
1 These actions are all part of the underlying theme of, “Don’t try 2
12345678901234567890123456789012123456789012345678901 2
12345678901234567890123456789012123456789012345678901 2
12345678901234567890123456789012123456789012345678901
anything funny, bub.” 2
12345678901234567890123456789012123456789012345678901 2
12345678901234567890123456789012123456789012345678901
You have the option of wearing noise-blocking headphones while taking 2
12345678901234567890123456789012123456789012345678901 2
12345678901234567890123456789012123456789012345678901 2
12345678901234567890123456789012123456789012345678901 22
the GRE, which we don’t recommended unless you studied for the GRE
12345678901234567890123456789012123456789012345678901
123456789012345678901234567890121234567890123456789012 15
Part I: Understanding the GRE

123456789012345678901234567890121234567890123456789012
12345678901234567890123456789012123456789012345678901 2
12345678901234567890123456789012123456789012345678901 2
12345678901234567890123456789012123456789012345678901
wearing noise-blocking headphones. A little sound—the whir of the 2
12345678901234567890123456789012123456789012345678901 2
12345678901234567890123456789012123456789012345678901
computer’s fans, the hum of air-conditioning—should be the type of 2
12345678901234567890123456789012123456789012345678901 2
12345678901234567890123456789012123456789012345678901
background noise you’re used to, and total silence can be a little freaky. 2
1 Unless there’s someone trying to cough up a lung next to you in the room, 2
2345678901234567890123456789012123456789012345678901
12345678901234567890123456789012123456789012345678901 2
12345678901234567890123456789012123456789012345678901 2
12345678901234567890123456789012123456789012345678901
skip the ‘phones. 2
12345678901234567890123456789012123456789012345678901 2
1 At long last, you get in front of the computer, ready to get your GRE 2
2345678901234567890123456789012123456789012345678901
12345678901234567890123456789012123456789012345678901 2
12345678901234567890123456789012123456789012345678901 2
12345678901234567890123456789012123456789012345678901
groove on. Unfortunately, the meaty portions of the GRE, i.e., the sections 2
12345678901234567890123456789012123456789012345678901
2345678901234567890123456789012123456789012345678901
2
2
1 that count, are still a long way off. First you have to fill out a basic form
12345678901234567890123456789012123456789012345678901 2
12345678901234567890123456789012123456789012345678901
for what seems like the umpteenth time, then you get a market research 2
12345678901234567890123456789012123456789012345678901
section where the computer asks you a lot of basic questions like, “Why 2
12345678901234567890123456789012123456789012345678901 2
1 are you taking this test?” and “Have you taken other standardized tests 2
2345678901234567890123456789012123456789012345678901
12345678901234567890123456789012123456789012345678901 2
12345678901234567890123456789012123456789012345678901
before?” Is the computer trying to hit on you, or is it just being nosy? It’s 2
12345678901234567890123456789012123456789012345678901 2
12345678901234567890123456789012123456789012345678901
up to you to decide, but the fact is, you’re stuck in that chair and you have 2
12345678901234567890123456789012123456789012345678901 2
2345678901234567890123456789012123456789012345678901
12345678901234567890123456789012123456789012345678901
to answer all the questions before you can start taking the exam. 2
12345678901234567890123456789012123456789012345678901 2
12345678901234567890123456789012123456789012345678901 2
12345678901234567890123456789012123456789012345678901 2
After the market research section, you get another section asking you
12345678901234567890123456789012123456789012345678901 2
1 where you want the scores sent. You should have an idea of where you 2
12345678901234567890123456789012123456789012345678901
want to send your scores. If not, this section can take a while. Even if you 2
12345678901234567890123456789012123456789012345678901 2
12345678901234567890123456789012123456789012345678901 2
12345678901234567890123456789012123456789012345678901 2
2345678901234567890123456789012123456789012345678901
do know where you want scores sent, scrolling around takes awhile, so be
12345678901234567890123456789012123456789012345678901 2
12345678901234567890123456789012123456789012345678901
prepared to spend about 5–10 minutes futzing around in this section. 2
12345678901234567890123456789012123456789012345678901 2
Once you finish leaping through these hoops, the GRE begins . . . almost. 2
12345678901234567890123456789012123456789012345678901
12345678901234567890123456789012123456789012345678901 2
12345678901234567890123456789012123456789012345678901 2
12345678901234567890123456789012123456789012345678901 2
You now get a chance to go through the tutorial showing how to navigate
12345678901234567890123456789012123456789012345678901 2
1 the GRE CAT. Hopefully, you will already be very familiar with these 2
2345678901234567890123456789012123456789012345678901
commands, so you can just skip this section. Go through one of the 2
12345678901234567890123456789012123456789012345678901
12345678901234567890123456789012123456789012345678901 2
1 tutorial lessons if you feel like it to reassure yourself, but this should not be 2
12345678901234567890123456789012123456789012345678901 2
12345678901234567890123456789012123456789012345678901
necessary. Ideally, you can skip this entirely and get to what’s important, 2
12345678901234567890123456789012123456789012345678901 2
12345678901234567890123456789012123456789012345678901
the test itself. 2
12345678901234567890123456789012123456789012345678901 2
12345678901234567890123456789012123456789012345678901 2
12345678901234567890123456789012123456789012345678901 2
2345678901234567890123456789012123456789012345678901
The GRE is like an awards show. Most people who watch awards shows 2
12345678901234567890123456789012123456789012345678901
12345678901234567890123456789012123456789012345678901 2
1 are primarily interested in who the big winners will be. To find this out, 2
12345678901234567890123456789012123456789012345678901 2
12345678901234567890123456789012123456789012345678901
though, they have to endure endless music performances, skits, sentimen- 2
Alert!

2345678901234567890123456789012123456789012345678901
12345678901234567890123456789012123456789012345678901
tal retrospectives, and all sorts of other fodder before the big payoff. For 2
2
12345678901234567890123456789012123456789012345678901 2
12345678901234567890123456789012123456789012345678901
the GRE, these distractions come in the form of market research 2
12345678901234567890123456789012123456789012345678901
1 questions, tutorials, and basic questions about who you are, and so on. 2
12345678901234567890123456789012123456789012345678901 2
12345678901234567890123456789012123456789012345678901
All you really want to do is take the sections that count toward your 2
12345678901234567890123456789012123456789012345678901 2
12345678901234567890123456789012123456789012345678901 2
12345678901234567890123456789012123456789012345678901 2
score, but to do so you have to endure the GRE equivalent of a dance
2345678901234567890123456789012123456789012345678901
12345678901234567890123456789012123456789012345678901 2
12345678901234567890123456789012123456789012345678901
medley. 2
12345678901234567890123456789012123456789012345678901 2
12345678901234567890123456789012123456789012345678901 2
1 The test begins! It starts with the Analytical Writing portion, and you have 2
12345678901234567890123456789012123456789012345678901 2
12345678901234567890123456789012123456789012345678901 2
12345678901234567890123456789012123456789012345678901
75 minutes to respond to 2 essay questions. For many test-takers, the wait 2
12345678901234567890123456789012123456789012345678901 2
12345678901234567890123456789012123456789012345678901
is over and it’s time to get cracking. However, some people apply to 2
1 programs that do not “care” about the Analytical score. If this is the case 2
2345678901234567890123456789012123456789012345678901
12345678901234567890123456789012123456789012345678901 2
12345678901234567890123456789012123456789012345678901 2
12345678901234567890123456789012123456789012345678901 2
16 123456789012345678901234567890121234567890123456789012

www.petersons.com
Chapter 1: Everything You Need to Know about the GRE CAT

123456789012345678901234567890121234567890123456789012
12345678901234567890123456789012123456789012345678901 2
12345678901234567890123456789012123456789012345678901 2
12345678901234567890123456789012123456789012345678901
for you—and be sure this is the case before following the next piece of 2
12345678901234567890123456789012123456789012345678901 2
12345678901234567890123456789012123456789012345678901
advice—then you can be more relaxed in your approach to the essays. You 2
12345678901234567890123456789012123456789012345678901 2
12345678901234567890123456789012123456789012345678901
have to answer them, since a zero might raise eyebrows, but if the schools 2
1 aren’t going to give the Analytical score any weight, why should you? Save 2
2345678901234567890123456789012123456789012345678901
12345678901234567890123456789012123456789012345678901 2
12345678901234567890123456789012123456789012345678901 2
12345678901234567890123456789012123456789012345678901
your brainpower for the sections of the test that do matter. 2
12345678901234567890123456789012123456789012345678901 2
1 Once you’ve finished clacking out your essays, the computer adaptive part 2
2345678901234567890123456789012123456789012345678901
12345678901234567890123456789012123456789012345678901 2
12345678901234567890123456789012123456789012345678901 2
12345678901234567890123456789012123456789012345678901
of the GRE kicks in. You will see a Quantitative section, a Verbal section, 2
12345678901234567890123456789012123456789012345678901
2345678901234567890123456789012123456789012345678901
2
2
1 and an Experimental section. ETS, the company that administers the GRE,
12345678901234567890123456789012123456789012345678901 2
12345678901234567890123456789012123456789012345678901
uses your answers on the Experimental section to fashion future GRE 2
12345678901234567890123456789012123456789012345678901
CATs. In this way, ETS seeks to guarantee that future GRE CATs are 2
12345678901234567890123456789012123456789012345678901 2
1 similar to current GRE CATs in terms of question similarity. This is all well 2
2345678901234567890123456789012123456789012345678901
12345678901234567890123456789012123456789012345678901 2
12345678901234567890123456789012123456789012345678901
and good in terms of fairness, but what it boils down to is that you have to 2
12345678901234567890123456789012123456789012345678901 2
12345678901234567890123456789012123456789012345678901
take an entire test section that doesn’t count toward your score. For 2
12345678901234567890123456789012123456789012345678901 22
2345678901234567890123456789012123456789012345678901
12345678901234567890123456789012123456789012345678901
obvious reasons, you won’t know which section is the Experimental
12345678901234567890123456789012123456789012345678901 2
12345678901234567890123456789012123456789012345678901
section ahead of time. 2
12345678901234567890123456789012123456789012345678901 22
12345678901234567890123456789012123456789012345678901
1 However, you can sometimes get an idea which section is the 2
12345678901234567890123456789012123456789012345678901
experimental one. It all depends on how adept you get at understanding 2
12345678901234567890123456789012123456789012345678901 2
12345678901234567890123456789012123456789012345678901 2
12345678901234567890123456789012123456789012345678901 22
2345678901234567890123456789012123456789012345678901
the difference between easy, medium, and hard problems. Let’s say
12345678901234567890123456789012123456789012345678901
you’ve taken one Quantitative section and one Verbal Section. The third 2
12345678901234567890123456789012123456789012345678901
section is another Quantitative section, so you know that either this 2
12345678901234567890123456789012123456789012345678901
12345678901234567890123456789012123456789012345678901 2
section or the previous Quant one is the experimental. You start off the 2
12345678901234567890123456789012123456789012345678901
12345678901234567890123456789012123456789012345678901 2
12345678901234567890123456789012123456789012345678901 2
12345678901234567890123456789012123456789012345678901 22
third section with four questions that you know you got correct. You’re
1 certain you nailed them. Yet the questions never seem to change in
12345678901234567890123456789012123456789012345678901 2
12345678901234567890123456789012123456789012345678901
difficulty level, not like they did on the previous Quant section. There 2
12345678901234567890123456789012123456789012345678901 2
12345678901234567890123456789012123456789012345678901
are two explanations for this: 2
12345678901234567890123456789012123456789012345678901
2345678901234567890123456789012123456789012345678901
2
2
12345678901234567890123456789012123456789012345678901
12345678901234567890123456789012123456789012345678901
1. You botched those questions and just don’t realize it. 2
1 2
12345678901234567890123456789012123456789012345678901 2
12345678901234567890123456789012123456789012345678901
2. You are working on the experimental section. 2
12345678901234567890123456789012123456789012345678901 22
2345678901234567890123456789012123456789012345678901
12345678901234567890123456789012123456789012345678901
1 There is no way to be 100 percent certain which section is Experimental, 2
12345678901234567890123456789012123456789012345678901
so the smart move is to treat all sections as if they count. If you guess 2
12345678901234567890123456789012123456789012345678901 2
2345678901234567890123456789012123456789012345678901
wrong about a section, your score goes down the drain, and you would 2
12345678901234567890123456789012123456789012345678901
12345678901234567890123456789012123456789012345678901 2
12345678901234567890123456789012123456789012345678901 2
12345678901234567890123456789012123456789012345678901 22
have achieved nothing. Still, it can be reassuring to have a feeling about
1 which section is Experimental. If you get this feeling, then you should
12345678901234567890123456789012123456789012345678901 2
12345678901234567890123456789012123456789012345678901
realize that you are pretty GRE savvy, and that the intricacies of the test 2
12345678901234567890123456789012123456789012345678901 2
12345678901234567890123456789012123456789012345678901
are not so mysterious after all. 2
12345678901234567890123456789012123456789012345678901 2
12345678901234567890123456789012123456789012345678901 22
2345678901234567890123456789012123456789012345678901
12345678901234567890123456789012123456789012345678901 2
1
. .112345678901234567890123456789012123456789012345678901
2345678901234567890123456789012123456789012345678901
. And Then Some 2
2
12345678901234567890123456789012123456789012345678901 22
2345678901234567890123456789012123456789012345678901
1 You should have an idea from this chapter that the GRE CAT is a kind of
12345678901234567890123456789012123456789012345678901 2
12345678901234567890123456789012123456789012345678901
odd test. It is unlike most of the major tests in another way, and that is 2
12345678901234567890123456789012123456789012345678901 2
12345678901234567890123456789012123456789012345678901
overall disposition of the proctors. On most big tests (standardized exams 2
12345678901234567890123456789012123456789012345678901 2
12345678901234567890123456789012123456789012345678901
or final exams in the college class), the proctors giving tests have a serious, 2
12345678901234567890123456789012123456789012345678901 2 17
123456789012345678901234567890121234567890123456789012
Part I: Understanding the GRE

123456789012345678901234567890121234567890123456789012
12345678901234567890123456789012123456789012345678901 2
12345678901234567890123456789012123456789012345678901 2
12345678901234567890123456789012123456789012345678901
formal air about them. Students are hushed, people avoid eye contact, 2
12345678901234567890123456789012123456789012345678901 2
12345678901234567890123456789012123456789012345678901
everyone is a little nervous. It’s sort of an “I’m a 9-year-old sitting with my 2
12345678901234567890123456789012123456789012345678901 2
12345678901234567890123456789012123456789012345678901
entire adult family at the Thanksgiving dinner table” kind of vibe, 2
1 nervousness and seriousness and formality all rolled into one. The proctors 2
2345678901234567890123456789012123456789012345678901
12345678901234567890123456789012123456789012345678901 2
12345678901234567890123456789012123456789012345678901 2
12345678901234567890123456789012123456789012345678901
are the same way, acting officious and ever-diligent in order to make sure 2
12345678901234567890123456789012123456789012345678901
2345678901234567890123456789012123456789012345678901
2
2
1 that nothing goes wrong.
12345678901234567890123456789012123456789012345678901 2
12345678901234567890123456789012123456789012345678901 2
12345678901234567890123456789012123456789012345678901
The GRE CAT is, as you might expect, different. Here the test is given all 2
12345678901234567890123456789012123456789012345678901
2345678901234567890123456789012123456789012345678901
2
2
1 the time, every day and sometimes even at night during most of the week.
12345678901234567890123456789012123456789012345678901 2
12345678901234567890123456789012123456789012345678901
There’s nothing exceptional about a single GRE administration. It means a 2
12345678901234567890123456789012123456789012345678901
lot to you, but the people at the test centers see people like you every day of 2
12345678901234567890123456789012123456789012345678901
2345678901234567890123456789012123456789012345678901
2
2
1 the week. For them it’s just a day’s work.
12345678901234567890123456789012123456789012345678901 2
12345678901234567890123456789012123456789012345678901 2
12345678901234567890123456789012123456789012345678901
This does not mean to imply that the proctors are lazy, uncaring, or 2
12345678901234567890123456789012123456789012345678901 2
12345678901234567890123456789012123456789012345678901
anything bad. Nothing of the sort is true. It’s just that they aren’t going to 2
12345678901234567890123456789012123456789012345678901 2
12345678901234567890123456789012123456789012345678901
have that same “Thanksgiving dinner table” attitude, since they are just 2
12345678901234567890123456789012123456789012345678901 2
12345678901234567890123456789012123456789012345678901
doing a routine day of work. They will do it well and be courteous and 2
2345678901234567890123456789012123456789012345678901
12345678901234567890123456789012123456789012345678901 2
1 helpful, but they won’t have the sense of “Wow, this is an important 2
12345678901234567890123456789012123456789012345678901 2
12345678901234567890123456789012123456789012345678901
moment” that you will. 2
12345678901234567890123456789012123456789012345678901 2
2345678901234567890123456789012123456789012345678901
They also won’t be at the front of the classroom eagerly scanning the room 2
12345678901234567890123456789012123456789012345678901
12345678901234567890123456789012123456789012345678901 2
12345678901234567890123456789012123456789012345678901 2
12345678901234567890123456789012123456789012345678901 2
to see if anyone’s cheating or if anyone needs help. The former is not
needed because you can’t look at another person’s GRE since it won’t have 2
12345678901234567890123456789012123456789012345678901
12345678901234567890123456789012123456789012345678901 2
12345678901234567890123456789012123456789012345678901 2
12345678901234567890123456789012123456789012345678901 2
the same questions. If you tried to bring something in to cheat with it
12345678901234567890123456789012123456789012345678901 2
1 would be spotted on camera. If you need help you can raise your hand, but 2
2345678901234567890123456789012123456789012345678901
there’s no guarantee that it will be spotted immediately. There’s no person 2
12345678901234567890123456789012123456789012345678901
12345678901234567890123456789012123456789012345678901 2
1 watching the testing room with eagle eyes since that mindset is just not 2
12345678901234567890123456789012123456789012345678901 2
12345678901234567890123456789012123456789012345678901
prevalent. The overall point is that you should not expect the proctor to 2
12345678901234567890123456789012123456789012345678901 2
12345678901234567890123456789012123456789012345678901
come rapidly if you need something during the middle of the section. 2
12345678901234567890123456789012123456789012345678901 2
12345678901234567890123456789012123456789012345678901
Therefore, make sure you have all the scratch paper you need during a 2
12345678901234567890123456789012123456789012345678901 2
12345678901234567890123456789012123456789012345678901
break and that you have enough pencils as well. There’s no way to 2
12345678901234567890123456789012123456789012345678901 2
12345678901234567890123456789012123456789012345678901
guarantee someone will come speedily to your aid while the test is in 2
12345678901234567890123456789012123456789012345678901 2
12345678901234567890123456789012123456789012345678901
progress. 2
12345678901234567890123456789012123456789012345678901 2
2345678901234567890123456789012123456789012345678901
There! You now have a host of strategies to help you comprehend the 2
12345678901234567890123456789012123456789012345678901
12345678901234567890123456789012123456789012345678901 2
12345678901234567890123456789012123456789012345678901 2
1 computer adaptive aspect of the GRE. For some additional test-taking 2
2345678901234567890123456789012123456789012345678901
strategies on how to handle each section of the GRE—Quantitative, 2
12345678901234567890123456789012123456789012345678901
12345678901234567890123456789012123456789012345678901 2
12345678901234567890123456789012123456789012345678901 2
12345678901234567890123456789012123456789012345678901 2
Verbal, and Analytical—all you have to do is turn the page.
2
1
12345678901234567890123456789012123456789012345678901 2
12345678901234567890123456789012123456789012345678901 2
12345678901234567890123456789012123456789012345678901 2
12345678901234567890123456789012123456789012345678901 2
12345678901234567890123456789012123456789012345678901 2
12345678901234567890123456789012123456789012345678901 2
2345678901234567890123456789012123456789012345678901
2
1
12345678901234567890123456789012123456789012345678901 2
12345678901234567890123456789012123456789012345678901 2
12345678901234567890123456789012123456789012345678901 2
12345678901234567890123456789012123456789012345678901 2
12345678901234567890123456789012123456789012345678901 2
12345678901234567890123456789012123456789012345678901 2
12345678901234567890123456789012123456789012345678901 2
18 123456789012345678901234567890121234567890123456789012

www.petersons.com
Chapter

2
Section Strategies
123456789012345678901234567890121234567890123456789012
12345678901234567890123456789012123456789012345678901 2
12345678901234567890123456789012123456789012345678901 2
12345678901234567890123456789012123456789012345678901
In the last chapter, we familiarized you with the computerized aspect of the 2
2345678901234567890123456789012123456789012345678901
12345678901234567890123456789012123456789012345678901
GRE. In this chapter, we’ll acquaint you with the three GRE sections. Our 2
2
12345678901234567890123456789012123456789012345678901
goal is to eliminate any element of surprise you might have while taking 2
12345678901234567890123456789012123456789012345678901
12345678901234567890123456789012123456789012345678901 2
12345678901234567890123456789012123456789012345678901 2
12345678901234567890123456789012123456789012345678901 22
the test, since unexpected problem-types lead to confusion, uncertainty,
1 and an overall negativity that can hinder your exam performance. We’ll
12345678901234567890123456789012123456789012345678901 2
12345678901234567890123456789012123456789012345678901 2
12345678901234567890123456789012123456789012345678901
tell you about the makeup of the GRE and how it is administered, and 2
2345678901234567890123456789012123456789012345678901
your anxiety will diminish as the test becomes less of a mystery. While you 2
12345678901234567890123456789012123456789012345678901
12345678901234567890123456789012123456789012345678901 2
12345678901234567890123456789012123456789012345678901 2
12345678901234567890123456789012123456789012345678901 22
might still run across some individual problems that throw you for a loop,
12345678901234567890123456789012123456789012345678901
we’ll make sure you won’t run into any question types that leave you 2
12345678901234567890123456789012123456789012345678901
12345678901234567890123456789012123456789012345678901 2
befuddled.
12345678901234567890123456789012123456789012345678901 2
12345678901234567890123456789012123456789012345678901 22
1
12345678901234567890123456789012123456789012345678901 2
12345678901234567890123456789012123456789012345678901 2
12345678901234567890123456789012123456789012345678901 2
Quantitative
12345678901234567890123456789012123456789012345678901 2
12345678901234567890123456789012123456789012345678901
The GRE section devoted to math goes by the mouthy word “Quantita- 2
12345678901234567890123456789012123456789012345678901 2
12345678901234567890123456789012123456789012345678901
tive.” Half of the problems in this section are regular multiple-choice 2
12345678901234567890123456789012123456789012345678901 2
12345678901234567890123456789012123456789012345678901
problems, but the other half are presented in a format known as 2
12345678901234567890123456789012123456789012345678901 2
2345678901234567890123456789012123456789012345678901
12345678901234567890123456789012123456789012345678901
Quantitative Comparison (QC for short). 2
12345678901234567890123456789012123456789012345678901 22
1
12345678901234567890123456789012123456789012345678901 2
12345678901234567890123456789012123456789012345678901 2
12345678901234567890123456789012123456789012345678901 2
12345678901234567890123456789012123456789012345678901
QC Problems 2
12345678901234567890123456789012123456789012345678901 2
12345678901234567890123456789012123456789012345678901
The directions for Quantitative Comparisons look something like this: 2
12345678901234567890123456789012123456789012345678901 2
12345678901234567890123456789012123456789012345678901 22
2345678901234567890123456789012123456789012345678901
12345678901234567890123456789012123456789012345678901 2
12345678901234567890123456789012123456789012345678901
Directions: Column A and Column B will have one quantity each.
2
12345678901234567890123456789012123456789012345678901 2
1 Your goal is to compare the quantities in each column and choose
12345678901234567890123456789012123456789012345678901 22
2345678901234567890123456789012123456789012345678901
12345678901234567890123456789012123456789012345678901 2
12345678901234567890123456789012123456789012345678901
A if the quantity in Column A is greater
2
12345678901234567890123456789012123456789012345678901 2
1 B if the quantity in Column B is greater
2345678901234567890123456789012123456789012345678901
12345678901234567890123456789012123456789012345678901 2
1 C if the two quantities are equal 2
12345678901234567890123456789012123456789012345678901 2
D if the relationship between the
12345678901234567890123456789012123456789012345678901 2 two quantities cannot be de-
12345678901234567890123456789012123456789012345678901
termined from the information given 2
12345678901234567890123456789012123456789012345678901 2
12345678901234567890123456789012123456789012345678901 2
12345678901234567890123456789012123456789012345678901 2
12345678901234567890123456789012123456789012345678901 2
123456789012345678901234567890121234567890123456789012
19
Part I: Understanding the GRE

123456789012345678901234567890121234567890123456789012
12345678901234567890123456789012123456789012345678901 2
12345678901234567890123456789012123456789012345678901 2
12345678901234567890123456789012123456789012345678901
Here’s a wildly simple example of a QC problem. 2
12345678901234567890123456789012123456789012345678901 2
12345678901234567890123456789012123456789012345678901 2
12345678901234567890123456789012123456789012345678901
Column A Column B 2
12345678901234567890123456789012123456789012345678901
2345678901234567890123456789012123456789012345678901
2
2
1
12345678901234567890123456789012123456789012345678901
h52 2
12345678901234567890123456789012123456789012345678901 2
12345678901234567890123456789012123456789012345678901 2
12345678901234567890123456789012123456789012345678901
h 1
2345678901234567890123456789012123456789012345678901
2
2
1
12345678901234567890123456789012123456789012345678901 2
12345678901234567890123456789012123456789012345678901
You have two columns, A and B, and there is a “quantity” underneath 2
12345678901234567890123456789012123456789012345678901 2
12345678901234567890123456789012123456789012345678901
each one. This quantity might be a variable (like Column A), a number 2
1 (like Column B), or some mixture of the two with a little extra thrown in 2
2345678901234567890123456789012123456789012345678901
12345678901234567890123456789012123456789012345678901 2
12345678901234567890123456789012123456789012345678901 2
12345678901234567890123456789012123456789012345678901
for good measure. On occasion, floating above this columnar stew is some 2
12345678901234567890123456789012123456789012345678901 2
12345678901234567890123456789012123456789012345678901
additional information that applies to the quantity in both columns, 2
12345678901234567890123456789012123456789012345678901 2
12345678901234567890123456789012123456789012345678901
like “h 5 2.” 2
12345678901234567890123456789012123456789012345678901 2
12345678901234567890123456789012123456789012345678901 2
12345678901234567890123456789012123456789012345678901 2
2345678901234567890123456789012123456789012345678901
Like the example above, the items in the two columns often can’t be
compared to one another, because they are in different formats. In our 2
12345678901234567890123456789012123456789012345678901
12345678901234567890123456789012123456789012345678901 2
12345678901234567890123456789012123456789012345678901 2
12345678901234567890123456789012123456789012345678901 2
example, one column has a variable while the other has a number. You
12345678901234567890123456789012123456789012345678901 2
1 would need to convert the variable into a number in order to compare 2
12345678901234567890123456789012123456789012345678901
them. Since the centered information states h 5 2, you know that the 2
12345678901234567890123456789012123456789012345678901 2
12345678901234567890123456789012123456789012345678901
quantity, or value, of the letter h in Column A is 2. Under Column B, the 2
12345678901234567890123456789012123456789012345678901 2
2345678901234567890123456789012123456789012345678901
value is 1. There are no other possible interpretations or solutions using 2
12345678901234567890123456789012123456789012345678901
12345678901234567890123456789012123456789012345678901 2
the centered information. We know that 1 is less than 2, so the correct 2
12345678901234567890123456789012123456789012345678901
12345678901234567890123456789012123456789012345678901 2
12345678901234567890123456789012123456789012345678901
answer must be (A). 2
12345678901234567890123456789012123456789012345678901 2
12345678901234567890123456789012123456789012345678901 2
12345678901234567890123456789012123456789012345678901 2 2
1
Note
12345678901234567890123456789012123456789012345678901
Converting the information in both columns into easily comparable 2
12345678901234567890123456789012123456789012345678901 2
12345678901234567890123456789012123456789012345678901
forms is a good strategy for many QC problems, especially if you find that 2
12345678901234567890123456789012123456789012345678901 2
12345678901234567890123456789012123456789012345678901
you are stuck. 2
12345678901234567890123456789012123456789012345678901 2
12345678901234567890123456789012123456789012345678901 2
12345678901234567890123456789012123456789012345678901 2
12345678901234567890123456789012123456789012345678901 2
12345678901234567890123456789012123456789012345678901
Note the qualifying phrase “There are no other possibile interpretations or 2
12345678901234567890123456789012123456789012345678901 2
2345678901234567890123456789012123456789012345678901
12345678901234567890123456789012123456789012345678901 2
1 solutions” in the previous sentence. Exploring alternate possible answers 2
12345678901234567890123456789012123456789012345678901
is critical on QC problems, because there are often hidden outcomes that 2
12345678901234567890123456789012123456789012345678901 2
2345678901234567890123456789012123456789012345678901
can affect the answer on a problem. Here’s a QC problem that looks 2
12345678901234567890123456789012123456789012345678901
12345678901234567890123456789012123456789012345678901 2
12345678901234567890123456789012123456789012345678901
similar to the previous one. 2
12345678901234567890123456789012123456789012345678901 2 2
1
12345678901234567890123456789012123456789012345678901
Column A Column B 2
12345678901234567890123456789012123456789012345678901 2
12345678901234567890123456789012123456789012345678901 2
12345678901234567890123456789012123456789012345678901 2
2
h 5 4
12345678901234567890123456789012123456789012345678901 2
2345678901234567890123456789012123456789012345678901
12345678901234567890123456789012123456789012345678901 2
12345678901234567890123456789012123456789012345678901
h 1 2
12345678901234567890123456789012123456789012345678901 2
12345678901234567890123456789012123456789012345678901 2
1 Again, you want to convert Column A into a number so you can compare 2
12345678901234567890123456789012123456789012345678901 2
12345678901234567890123456789012123456789012345678901
it to the number in Column B. The obvious answer is that h 5 2, since 2
12345678901234567890123456789012123456789012345678901 2
12345678901234567890123456789012123456789012345678901
(2)(2) 5 4. This would mean (A) is the answer . . . if there are no other 2
12345678901234567890123456789012123456789012345678901 2
12345678901234567890123456789012123456789012345678901
possibilities, that is. In this case, you should see that h could also equal 22, 2
12345678901234567890123456789012123456789012345678901 2
12345678901234567890123456789012123456789012345678901 2
12345678901234567890123456789012123456789012345678901 2
20 123456789012345678901234567890121234567890123456789012

www.petersons.com
Chapter 2: Section Strategies

123456789012345678901234567890121234567890123456789012
12345678901234567890123456789012123456789012345678901 2
12345678901234567890123456789012123456789012345678901 2
12345678901234567890123456789012123456789012345678901
since (22)( 22) 5 4. If you compare 22 with Column B’s 1, Column B’s 2
12345678901234567890123456789012123456789012345678901 2
12345678901234567890123456789012123456789012345678901
quantity is larger, so (B) is the answer in this scenario. 2
12345678901234567890123456789012123456789012345678901 2
12345678901234567890123456789012123456789012345678901
One possible value for h gives you the answer (A), while another gives you 2
12345678901234567890123456789012123456789012345678901 2
12345678901234567890123456789012123456789012345678901 2
12345678901234567890123456789012123456789012345678901
an answer of (B). Whenever you have a conflict like this, the correct 2
12345678901234567890123456789012123456789012345678901 2
12345678901234567890123456789012123456789012345678901
response is choice (D), “the answer cannot be determined by the 2
1 information given.” A more precise statement for choice (D) would be that 2
2345678901234567890123456789012123456789012345678901
12345678901234567890123456789012123456789012345678901 2
12345678901234567890123456789012123456789012345678901 2
12345678901234567890123456789012123456789012345678901
“more than one answer can be determined, and there’s no way to tell 2
12345678901234567890123456789012123456789012345678901
2345678901234567890123456789012123456789012345678901
2
2
1 which answer is correct.” It’s not that you can’t determine an answer, it’s
12345678901234567890123456789012123456789012345678901 2
12345678901234567890123456789012123456789012345678901
just that you don’t know which answer is correct. 2
12345678901234567890123456789012123456789012345678901 2
12345678901234567890123456789012123456789012345678901
To properly explore all possibilities on QC problems involving variables, 2
2345678901234567890123456789012123456789012345678901 2
1
12345678901234567890123456789012123456789012345678901
don’t just use easy numbers like 2, 3, and 4 when giving values to 2
12345678901234567890123456789012123456789012345678901 2
12345678901234567890123456789012123456789012345678901
variables. Be sure to insert some weird numbers like those on the 2
12345678901234567890123456789012123456789012345678901 2
12345678901234567890123456789012123456789012345678901
following list: 2
12345678901234567890123456789012123456789012345678901 2
12345678901234567890123456789012123456789012345678901 2
12345678901234567890123456789012123456789012345678901 2
12345678901234567890123456789012123456789012345678901 2
12345678901234567890123456789012123456789012345678901 22
2345678901234567890123456789012123456789012345678901
Weird Numbers
1
12345678901234567890123456789012123456789012345678901
The Number Why It’s Weird 2
12345678901234567890123456789012123456789012345678901 2
12345678901234567890123456789012123456789012345678901 2
12345678901234567890123456789012123456789012345678901 22
2345678901234567890123456789012123456789012345678901
0 Any number multiplied by 0 equals zero.
12345678901234567890123456789012123456789012345678901
12345678901234567890123456789012123456789012345678901
Also, any number to the zero power equals 2
12345678901234567890123456789012123456789012345678901
0 2
12345678901234567890123456789012123456789012345678901 22
1, or x 5 1.
12345678901234567890123456789012123456789012345678901
12345678901234567890123456789012123456789012345678901
1 Any number multiplied by 1 does not 2
12345678901234567890123456789012123456789012345678901 2
12345678901234567890123456789012123456789012345678901
change. 2
1 2
2345678901234567890123456789012123456789012345678901
12345678901234567890123456789012123456789012345678901 2
12345678901234567890123456789012123456789012345678901
Fractions between Normally, when you multiply two numbers 2
1 0 and 1 together, the end result is larger. When you 2
12345678901234567890123456789012123456789012345678901 2
12345678901234567890123456789012123456789012345678901
multiply two fractions less than one to- 2
12345678901234567890123456789012123456789012345678901 2
12345678901234567890123456789012123456789012345678901
gether, the result is less than the original 2
12345678901234567890123456789012123456789012345678901 2
12345678901234567890123456789012123456789012345678901
two numbers. 2
12345678901234567890123456789012123456789012345678901 2
2345678901234567890123456789012123456789012345678901
12345678901234567890123456789012123456789012345678901 2
12345678901234567890123456789012123456789012345678901
Negative numbers Two negatives makes a positive when 2
1 2
12345678901234567890123456789012123456789012345678901 2
multiplying or dividing, and 22 is greater
12345678901234567890123456789012123456789012345678901 2
12345678901234567890123456789012123456789012345678901
than 24 even though our minds like to 2
12345678901234567890123456789012123456789012345678901 2
12345678901234567890123456789012123456789012345678901
think that 4 is bigger than 2. 2
12345678901234567890123456789012123456789012345678901 2
12345678901234567890123456789012123456789012345678901 2
2345678901234567890123456789012123456789012345678901
12345678901234567890123456789012123456789012345678901
On our second sample question, realizing the existence of 22 (the weird 2
12345678901234567890123456789012123456789012345678901 2
12345678901234567890123456789012123456789012345678901
negative number) led us to the correct answer of (D). 2
12345678901234567890123456789012123456789012345678901 22
1 Many incorrect QC answers are chosen in the name of speed, as people
12345678901234567890123456789012123456789012345678901 2
12345678901234567890123456789012123456789012345678901 2
12345678901234567890123456789012123456789012345678901
look at a problem and “see” the answer that must be the correct one. If the 2
12345678901234567890123456789012123456789012345678901
answer is so obvious, why not pick it quickly and zip on to the next 2
12345678901234567890123456789012123456789012345678901 2
2345678901234567890123456789012123456789012345678901
12345678901234567890123456789012123456789012345678901 2
1 question? The response to this hypothetical question is that answers that 2
12345678901234567890123456789012123456789012345678901
look correct are often traps designed to catch those who do not actually 2
12345678901234567890123456789012123456789012345678901 2
12345678901234567890123456789012123456789012345678901
2345678901234567890123456789012123456789012345678901
2
2
1 work the problem. Here’s a classic QC trap on a medium-difficulty
12345678901234567890123456789012123456789012345678901 2
12345678901234567890123456789012123456789012345678901 22
problem.
12345678901234567890123456789012123456789012345678901
123456789012345678901234567890121234567890123456789012 21
Part I: Understanding the GRE

123456789012345678901234567890121234567890123456789012
12345678901234567890123456789012123456789012345678901 2
12345678901234567890123456789012123456789012345678901 2
12345678901234567890123456789012123456789012345678901
Column A Column B 2
12345678901234567890123456789012123456789012345678901 2
12345678901234567890123456789012123456789012345678901 2
12345678901234567890123456789012123456789012345678901
4(g 1 i) 5 76 2
12345678901234567890123456789012123456789012345678901
2345678901234567890123456789012123456789012345678901
2
2
1 3i 5 57
12345678901234567890123456789012123456789012345678901 2
12345678901234567890123456789012123456789012345678901 2
12345678901234567890123456789012123456789012345678901 2
g g
6 8
12345678901234567890123456789012123456789012345678901
2345678901234567890123456789012123456789012345678901
2
2
1
12345678901234567890123456789012123456789012345678901
If g is a regular, Joe Schmo kind of number, the answer seems pretty 2
12345678901234567890123456789012123456789012345678901 2
12345678901234567890123456789012123456789012345678901
obvious. Column B is going to be greater since 8 is bigger than 6 and both 2
12345678901234567890123456789012123456789012345678901 2
12345678901234567890123456789012123456789012345678901
numbers are being taken to the same exponential power, g. An experienced 2
12345678901234567890123456789012123456789012345678901 2
12345678901234567890123456789012123456789012345678901
GRE taker—which you will be after reading this book—should have 2
12345678901234567890123456789012123456789012345678901 2
12345678901234567890123456789012123456789012345678901
internal alarms going off at the sight of this problem. It just looks too 2
1 simple, and there has to be something more to this problem in order to 2
2345678901234567890123456789012123456789012345678901
12345678901234567890123456789012123456789012345678901 2
12345678901234567890123456789012123456789012345678901 2
12345678901234567890123456789012123456789012345678901
justify the fact that it’s a medium problem. 2
12345678901234567890123456789012123456789012345678901 2
2345678901234567890123456789012123456789012345678901
Take the time to solve for both g and i, and you’ll discover the trap. From 2
12345678901234567890123456789012123456789012345678901
12345678901234567890123456789012123456789012345678901 2
12345678901234567890123456789012123456789012345678901 2
12345678901234567890123456789012123456789012345678901 2
the second equation, you will discover that i 5 19. Placing this value into
12345678901234567890123456789012123456789012345678901 2
12345678901234567890123456789012123456789012345678901
the first equation, you will find: 2
1 2
12345678901234567890123456789012123456789012345678901 2
12345678901234567890123456789012123456789012345678901
4(g 1 i) 5 76 2
12345678901234567890123456789012123456789012345678901 2
12345678901234567890123456789012123456789012345678901 2
2345678901234567890123456789012123456789012345678901
4g 1 4i 5 76
12345678901234567890123456789012123456789012345678901 2
12345678901234567890123456789012123456789012345678901
4g 1 (4)(19) 5 76 2
12345678901234567890123456789012123456789012345678901 2
12345678901234567890123456789012123456789012345678901 2
12345678901234567890123456789012123456789012345678901 2
4g 1 76 5 76
12345678901234567890123456789012123456789012345678901 2
12345678901234567890123456789012123456789012345678901
4g 1 76 2 76 5 76 2 76 2
12345678901234567890123456789012123456789012345678901 2 2
1 4g 5 0
12345678901234567890123456789012123456789012345678901 2
12345678901234567890123456789012123456789012345678901 2
12345678901234567890123456789012123456789012345678901 2
g 5 0
12345678901234567890123456789012123456789012345678901 2
12345678901234567890123456789012123456789012345678901
If g equals zero, then Column A’s 6g 5 60 5 1 and Column B’s 8g 5 80 5 2
12345678901234567890123456789012123456789012345678901 2
2345678901234567890123456789012123456789012345678901
12345678901234567890123456789012123456789012345678901 2
1 1. Both columns are equal, and there’s no other possible value for g, so the 2
12345678901234567890123456789012123456789012345678901
correct answer must be (C). 2
12345678901234567890123456789012123456789012345678901 2
2345678901234567890123456789012123456789012345678901
12345678901234567890123456789012123456789012345678901
You will see a lot of QC problems throughout this book, so use the 2
2
12345678901234567890123456789012123456789012345678901
1 following checklist as a way of remembering the key strategies to use when 2
12345678901234567890123456789012123456789012345678901 2
12345678901234567890123456789012123456789012345678901 2
12345678901234567890123456789012123456789012345678901
handling these problems. 2
12345678901234567890123456789012123456789012345678901 2
12345678901234567890123456789012123456789012345678901 2
12345678901234567890123456789012123456789012345678901 2
12345678901234567890123456789012123456789012345678901 2
2345678901234567890123456789012123456789012345678901
12345678901234567890123456789012123456789012345678901
QC Approach Check List 2
12345678901234567890123456789012123456789012345678901 2
12345678901234567890123456789012123456789012345678901 2
12345678901234567890123456789012123456789012345678901 2
1. For the answer choices, the word always is critical. You can only pick
1 (A) if the quantity in Column A is always greater, and you can only 2
12345678901234567890123456789012123456789012345678901 2
12345678901234567890123456789012123456789012345678901
pick (B) if Column B’s quantity is always greater. Choice (C) is the 2
12345678901234567890123456789012123456789012345678901 2
12345678901234567890123456789012123456789012345678901
answer if the two columns are always equal, no matter what weird 2
12345678901234567890123456789012123456789012345678901 2
12345678901234567890123456789012123456789012345678901
numbers are punched in. Whenever you find a QC problem that has 2
12345678901234567890123456789012123456789012345678901 2
12345678901234567890123456789012123456789012345678901
more than one possible outcome, (A), (B) and/or (C), you have (D) as 2
12345678901234567890123456789012123456789012345678901 2
12345678901234567890123456789012123456789012345678901
your answer.
2345678901234567890123456789012123456789012345678901
2
2
1
12345678901234567890123456789012123456789012345678901 2
12345678901234567890123456789012123456789012345678901 2
12345678901234567890123456789012123456789012345678901 2
22 123456789012345678901234567890121234567890123456789012

www.petersons.com
Chapter 2: Section Strategies

123456789012345678901234567890121234567890123456789012
12345678901234567890123456789012123456789012345678901 2
12345678901234567890123456789012123456789012345678901 2
12345678901234567890123456789012123456789012345678901
2. The goal is to compare the two problems, so you should take 2
12345678901234567890123456789012123456789012345678901 2
12345678901234567890123456789012123456789012345678901
information and try to rearrange it in a way that makes comparison 2
12345678901234567890123456789012123456789012345678901 2
12345678901234567890123456789012123456789012345678901
easier. If you have a problem with a lot of variables, you should 2
2345678901234567890123456789012123456789012345678901 2
1
12345678901234567890123456789012123456789012345678901
realize the goal will be to get comparable variables into both 2
12345678901234567890123456789012123456789012345678901
columns. If one column only has numbers in it, the key to that 2
12345678901234567890123456789012123456789012345678901 2
12345678901234567890123456789012123456789012345678901
2345678901234567890123456789012123456789012345678901
2
2
1 problem is finding a way to convert the other column into a number
12345678901234567890123456789012123456789012345678901 2
12345678901234567890123456789012123456789012345678901
than can be compared. 2
12345678901234567890123456789012123456789012345678901 2
12345678901234567890123456789012123456789012345678901
2345678901234567890123456789012123456789012345678901
2
2
1 3. When variables are involved, don’t be content to plug in one value for
12345678901234567890123456789012123456789012345678901 2
12345678901234567890123456789012123456789012345678901
the variable and then use that as the result. Use a “weird” number to 2
12345678901234567890123456789012123456789012345678901
challenge your initial finding. If you find a weird number that leads to 2
12345678901234567890123456789012123456789012345678901
2345678901234567890123456789012123456789012345678901
2
1 a different result, you know the correct choice on that problem is (D). 2
12345678901234567890123456789012123456789012345678901 2
12345678901234567890123456789012123456789012345678901
(See the table on page 21 of this chapter) 2
12345678901234567890123456789012123456789012345678901 2
12345678901234567890123456789012123456789012345678901 2
12345678901234567890123456789012123456789012345678901 22
2345678901234567890123456789012123456789012345678901
4. Many QC problems are designed to look like there is one obvious
12345678901234567890123456789012123456789012345678901
answer. Never jump to this answer unless you have done all of the 2
12345678901234567890123456789012123456789012345678901
12345678901234567890123456789012123456789012345678901
actual math that the problem demands. The obvious answer is often 2
2
12345678901234567890123456789012123456789012345678901
12345678901234567890123456789012123456789012345678901 2
1 an incorrect choice set there to trap hurried test-takers. 2
12345678901234567890123456789012123456789012345678901 2
12345678901234567890123456789012123456789012345678901 2
12345678901234567890123456789012123456789012345678901 2
12345678901234567890123456789012123456789012345678901 22
2345678901234567890123456789012123456789012345678901
12345678901234567890123456789012123456789012345678901
How Much Does Your Brain Weigh?
12345678901234567890123456789012123456789012345678901 2
12345678901234567890123456789012123456789012345678901 2
12345678901234567890123456789012123456789012345678901 22
There are several ways you can answer this question. You could take your
12345678901234567890123456789012123456789012345678901
brain out and place it on a scale, although this is not recommended. You 2
12345678901234567890123456789012123456789012345678901
could find out what the average brain weighs by looking it up in a 2
12345678901234567890123456789012123456789012345678901
12345678901234567890123456789012123456789012345678901 2
1 reference book or surfing the Net. You could place both your hands 2
12345678901234567890123456789012123456789012345678901 22
2345678901234567890123456789012123456789012345678901
12345678901234567890123456789012123456789012345678901
1 around your head, lean over, and use your arms as a scale. 2
12345678901234567890123456789012123456789012345678901 2
12345678901234567890123456789012123456789012345678901
Or you could just guess. You won’t be completely accurate, but you should 2
2345678901234567890123456789012123456789012345678901 2
12345678901234567890123456789012123456789012345678901
12345678901234567890123456789012123456789012345678901
be pretty close. Look at the following choices, and then take a guess: 2
1 2
12345678901234567890123456789012123456789012345678901 2
12345678901234567890123456789012123456789012345678901 2
12345678901234567890123456789012123456789012345678901 22
2345678901234567890123456789012123456789012345678901
Which of the following is the most likely weight of your brain?
12345678901234567890123456789012123456789012345678901 2
1 A. 1 pound
12345678901234567890123456789012123456789012345678901 2
12345678901234567890123456789012123456789012345678901
B. 3 pounds 2
12345678901234567890123456789012123456789012345678901 22
2345678901234567890123456789012123456789012345678901
12345678901234567890123456789012123456789012345678901
C. 6 pounds
12345678901234567890123456789012123456789012345678901 22
12345678901234567890123456789012123456789012345678901
D. 10 pounds
2
1
2345678901234567890123456789012123456789012345678901
12345678901234567890123456789012123456789012345678901
E. 15 pounds 2
12345678901234567890123456789012123456789012345678901 22
12345678901234567890123456789012123456789012345678901
The correct answer is B. Estimating an answer can sometimes lead you 2
12345678901234567890123456789012123456789012345678901
1 directly to the right answer, but this does not happen very often. More 2
12345678901234567890123456789012123456789012345678901 2
12345678901234567890123456789012123456789012345678901 2
12345678901234567890123456789012123456789012345678901
important than determining the correct answer, guessing can often help 2
12345678901234567890123456789012123456789012345678901 2
12345678901234567890123456789012123456789012345678901 2
you eliminate wrong answers, which increases your odds of getting a
2345678901234567890123456789012123456789012345678901
12345678901234567890123456789012123456789012345678901 2
1 problem right. If your brain weighed 10 or 15 pounds, you would need 2
12345678901234567890123456789012123456789012345678901
metal girders in your neck to help support its weight. So, (D) and (E) can 2
12345678901234567890123456789012123456789012345678901 2
12345678901234567890123456789012123456789012345678901
2345678901234567890123456789012123456789012345678901
2
2
1 be eliminated, leaving you three choices. Choice (A) is a little light, and
12345678901234567890123456789012123456789012345678901 2
12345678901234567890123456789012123456789012345678901 22
although this answer provides a fertile ground for a cutting remark, it’s not
12345678901234567890123456789012123456789012345678901
123456789012345678901234567890121234567890123456789012 23
Part I: Understanding the GRE

123456789012345678901234567890121234567890123456789012
12345678901234567890123456789012123456789012345678901 2
12345678901234567890123456789012123456789012345678901 2
12345678901234567890123456789012123456789012345678901
correct. This leaves (B) and (C), and if you thought 6 pounds was a bit 2
12345678901234567890123456789012123456789012345678901 2
12345678901234567890123456789012123456789012345678901
portly for a brain, you were right. The answer is (B). 2
12345678901234567890123456789012123456789012345678901 2
12345678901234567890123456789012123456789012345678901
Some problems are hard because the math required to answer them is 2
12345678901234567890123456789012123456789012345678901 2
12345678901234567890123456789012123456789012345678901 2
12345678901234567890123456789012123456789012345678901
hard. If you can use your eyeballs and your 3-pound brain to estimate 2
12345678901234567890123456789012123456789012345678901 2
12345678901234567890123456789012123456789012345678901
what the answer should be, you will increase your chances of getting a 2
1 problem correct. It might not lead you directly to the right answer, but it 2
2345678901234567890123456789012123456789012345678901
12345678901234567890123456789012123456789012345678901 2
12345678901234567890123456789012123456789012345678901 2
12345678901234567890123456789012123456789012345678901
can steer you clear of answer choices that are clearly wrong. 2
12345678901234567890123456789012123456789012345678901 2
1 Geometry problems involving angles, lines, figures, or measurements are 2
2345678901234567890123456789012123456789012345678901
12345678901234567890123456789012123456789012345678901 2
12345678901234567890123456789012123456789012345678901
the most susceptible to estimating, although the fact that the images are 2
12345678901234567890123456789012123456789012345678901 2
12345678901234567890123456789012123456789012345678901
displayed on a computer screen makes it a little problematic. Whenever 2
2345678901234567890123456789012123456789012345678901 2
1
12345678901234567890123456789012123456789012345678901
you are given a figure on the screen, redraw it on paper in front of you as 2
12345678901234567890123456789012123456789012345678901 2
12345678901234567890123456789012123456789012345678901
accurately as possible, and then work with that figure. For example: 2
12345678901234567890123456789012123456789012345678901 2
12345678901234567890123456789012123456789012345678901 2
2345678901234567890123456789012123456789012345678901
12345678901234567890123456789012123456789012345678901 2
12345678901234567890123456789012123456789012345678901
The shortest side of a triangle is n centimeters long. Another side is 2
12345678901234567890123456789012123456789012345678901 2
12345678901234567890123456789012123456789012345678901 2
three times this length, while the third side is 22 centimeters greater
1 than n. If this is the case, which of the following is the only possible 2
12345678901234567890123456789012123456789012345678901 2
12345678901234567890123456789012123456789012345678901 2
12345678901234567890123456789012123456789012345678901
value of n? 2
12345678901234567890123456789012123456789012345678901 2
2345678901234567890123456789012123456789012345678901
12345678901234567890123456789012123456789012345678901
A. 2 centimeters 2
12345678901234567890123456789012123456789012345678901 2
12345678901234567890123456789012123456789012345678901
B. 4 centimeters 2
12345678901234567890123456789012123456789012345678901 2
12345678901234567890123456789012123456789012345678901
C. 5 centimeters 2
12345678901234567890123456789012123456789012345678901 2
12345678901234567890123456789012123456789012345678901
D. 6 centimeters 2
12345678901234567890123456789012123456789012345678901 2
12345678901234567890123456789012123456789012345678901
E. 8 centimeters 2
1 2
2345678901234567890123456789012123456789012345678901
There’s an equation you can use to solve this problem, but you can also 2
12345678901234567890123456789012123456789012345678901
12345678901234567890123456789012123456789012345678901 2
1 attack it with just a number 2 pencil and some scratch paper. Start with 2
12345678901234567890123456789012123456789012345678901 2
12345678901234567890123456789012123456789012345678901
choice (A), 2 centimeters. The second side would be (2)(3) 5 6 cm and the 2
12345678901234567890123456789012123456789012345678901 2
12345678901234567890123456789012123456789012345678901 2
12345678901234567890123456789012123456789012345678901 2
third side is 2 1 22 5 24 cm. Now, try to accurately draw a triangle with
12345678901234567890123456789012123456789012345678901
sides of 2, 6, and 24. With one side 24 cm, how are the sides of 2 cm and 2
12345678901234567890123456789012123456789012345678901 2
2345678901234567890123456789012123456789012345678901
12345678901234567890123456789012123456789012345678901 2
12345678901234567890123456789012123456789012345678901
6 cm actually going to connect? 2
1 2
12345678901234567890123456789012123456789012345678901 2
12345678901234567890123456789012123456789012345678901 2
12345678901234567890123456789012123456789012345678901 2
12345678901234567890123456789012123456789012345678901 2
12345678901234567890123456789012123456789012345678901 2
12345678901234567890123456789012123456789012345678901 2
12345678901234567890123456789012123456789012345678901 2
12345678901234567890123456789012123456789012345678901 2
2345678901234567890123456789012123456789012345678901
Since (A) fails spectacularly, you might want to head to the larger values
like choice (E). If n 5 8, then the other two sides are 24 cm and 30 cm. If 2
12345678901234567890123456789012123456789012345678901
12345678901234567890123456789012123456789012345678901 2
you freehand this, it seems to work. At this point, knowing the math 2
12345678901234567890123456789012123456789012345678901
1 2
12345678901234567890123456789012123456789012345678901
would come in handy, but if you don’t know it, guess (E) and move on. 2
12345678901234567890123456789012123456789012345678901 2
12345678901234567890123456789012123456789012345678901 2
12345678901234567890123456789012123456789012345678901 2
12345678901234567890123456789012123456789012345678901 2
Note

2345678901234567890123456789012123456789012345678901
12345678901234567890123456789012123456789012345678901 2
1 In a triangle, the length of every side must be less than the sum of the other 2
12345678901234567890123456789012123456789012345678901
two sides. Choice (E) fulfills this, since even the length of the longest side, 2
12345678901234567890123456789012123456789012345678901 2
12345678901234567890123456789012123456789012345678901
30, is less than the sum of 8 and 24.
2345678901234567890123456789012123456789012345678901
2
2
1
12345678901234567890123456789012123456789012345678901 2
12345678901234567890123456789012123456789012345678901 2
12345678901234567890123456789012123456789012345678901 2
24 123456789012345678901234567890121234567890123456789012

www.petersons.com
Chapter 2: Section Strategies

123456789012345678901234567890121234567890123456789012
12345678901234567890123456789012123456789012345678901 2
12345678901234567890123456789012123456789012345678901 2
12345678901234567890123456789012123456789012345678901
Techniques, strategy, practice, and factual knowledge can be combined to 2
12345678901234567890123456789012123456789012345678901 2
12345678901234567890123456789012123456789012345678901
give you a good score, but it doesn’t guarantee a perfect score. Inevitably, 2
12345678901234567890123456789012123456789012345678901 2
12345678901234567890123456789012123456789012345678901
there will be one or two questions that seem to defy explanation, and these 2
1 often appear as time is winding down on the section. If you can’t take an 2
2345678901234567890123456789012123456789012345678901
12345678901234567890123456789012123456789012345678901 2
12345678901234567890123456789012123456789012345678901 2
12345678901234567890123456789012123456789012345678901
educated guess, just take a plain guess and move on to the next problem. 2
12345678901234567890123456789012123456789012345678901
2345678901234567890123456789012123456789012345678901
2
2
1 On the computer, you must place an answer down on every problem, even
12345678901234567890123456789012123456789012345678901 2
12345678901234567890123456789012123456789012345678901
if it that means you’re simply filling in (E) on the last couple of questions. 2
12345678901234567890123456789012123456789012345678901 2
12345678901234567890123456789012123456789012345678901
Keep in mind that these questions are not worth as much as the earliest 2
1 questions. On a 28-question test, you can randomly fill in the last 6 2
2345678901234567890123456789012123456789012345678901
12345678901234567890123456789012123456789012345678901 2
12345678901234567890123456789012123456789012345678901 2
12345678901234567890123456789012123456789012345678901
questions and still get a score in the high 700s. The point is that guessing 2
12345678901234567890123456789012123456789012345678901
2345678901234567890123456789012123456789012345678901
2
2
1 late in the test won’t hurt you as much as you might think. If you have to
12345678901234567890123456789012123456789012345678901 2
12345678901234567890123456789012123456789012345678901
guess early in the math section, try to use estimating and common sense to 2
12345678901234567890123456789012123456789012345678901
eliminate some answers, and then just take your best shot. 2
12345678901234567890123456789012123456789012345678901 2
12345678901234567890123456789012123456789012345678901 22
2345678901234567890123456789012123456789012345678901
12345678901234567890123456789012123456789012345678901 2
12345678901234567890123456789012123456789012345678901 2
12345678901234567890123456789012123456789012345678901 2
12345678901234567890123456789012123456789012345678901 2
Verbal
12345678901234567890123456789012123456789012345678901 2
1 The Verbal portion of the GRE will be eerily familiar to anyone who took
12345678901234567890123456789012123456789012345678901 2
12345678901234567890123456789012123456789012345678901
the SAT when applying for college. All four question types have made an 2
12345678901234567890123456789012123456789012345678901 2
12345678901234567890123456789012123456789012345678901
appearance on the SAT at one time or another, and they’ve even starred in 2
12345678901234567890123456789012123456789012345678901 2
12345678901234567890123456789012123456789012345678901
other standardized tests as well. 2
12345678901234567890123456789012123456789012345678901 2
12345678901234567890123456789012123456789012345678901 2
12345678901234567890123456789012123456789012345678901 22
2345678901234567890123456789012123456789012345678901
12345678901234567890123456789012123456789012345678901
12345678901234567890123456789012123456789012345678901
Question Types on the GRE Verbal 2
12345678901234567890123456789012123456789012345678901 22
1 Question Type Description
12345678901234567890123456789012123456789012345678901 2
12345678901234567890123456789012123456789012345678901 2
12345678901234567890123456789012123456789012345678901 2
1. Antonyms You are given a single word and then must
12345678901234567890123456789012123456789012345678901 2
12345678901234567890123456789012123456789012345678901
find its opposite in the answer choices. 2
12345678901234567890123456789012123456789012345678901 22
2345678901234567890123456789012123456789012345678901
12345678901234567890123456789012123456789012345678901
2. Analogies You’ll see two words separated by a colon
2
1
12345678901234567890123456789012123456789012345678901
at the beginning of each problem. These 2
12345678901234567890123456789012123456789012345678901 2
12345678901234567890123456789012123456789012345678901 22
2345678901234567890123456789012123456789012345678901
are called the “stem” words, and the stem
12345678901234567890123456789012123456789012345678901 2
1 words have some sort of relationship with
12345678901234567890123456789012123456789012345678901 2
12345678901234567890123456789012123456789012345678901 2
each other. Your job is to find the paired
2345678901234567890123456789012123456789012345678901
12345678901234567890123456789012123456789012345678901 2
12345678901234567890123456789012123456789012345678901
words in the answer choices that have the 2
12345678901234567890123456789012123456789012345678901 2
12345678901234567890123456789012123456789012345678901 22
same, analogous relationship as the stem
1
12345678901234567890123456789012123456789012345678901
words. 2
12345678901234567890123456789012123456789012345678901 2
12345678901234567890123456789012123456789012345678901
3. Sentence A good old fill-in-the-blank exercise where 2
12345678901234567890123456789012123456789012345678901 2
12345678901234567890123456789012123456789012345678901
Completions you must add the correct missing word or 2
2345678901234567890123456789012123456789012345678901
12345678901234567890123456789012123456789012345678901 2
12345678901234567890123456789012123456789012345678901
words to an initial sentence. 2
12345678901234567890123456789012123456789012345678901 22
12345678901234567890123456789012123456789012345678901 2
1 4. Reading Read some text and then answer questions
12345678901234567890123456789012123456789012345678901 2
12345678901234567890123456789012123456789012345678901
Comprehension about it. The staple of standardized verbal 2
12345678901234567890123456789012123456789012345678901 2
12345678901234567890123456789012123456789012345678901 2
(RC) exams everywhere.
12345678901234567890123456789012123456789012345678901 2
12345678901234567890123456789012123456789012345678901 2
12345678901234567890123456789012123456789012345678901 2
12345678901234567890123456789012123456789012345678901 2
12345678901234567890123456789012123456789012345678901 2 25
123456789012345678901234567890121234567890123456789012
Part I: Understanding the GRE

123456789012345678901234567890121234567890123456789012
12345678901234567890123456789012123456789012345678901 2
12345678901234567890123456789012123456789012345678901 2
12345678901234567890123456789012123456789012345678901
There are 30 problems on the Verbal section and four different question 2
12345678901234567890123456789012123456789012345678901 2
12345678901234567890123456789012123456789012345678901
types, so you might think that there will about 7–8 of each question type 2
12345678901234567890123456789012123456789012345678901 2
12345678901234567890123456789012123456789012345678901
on the test. It rarely works out that way. Although every CAT is different, 2
1 for the most part, Sentence Completion problems get short shrift on the 2
2345678901234567890123456789012123456789012345678901
12345678901234567890123456789012123456789012345678901 2
12345678901234567890123456789012123456789012345678901 2
12345678901234567890123456789012123456789012345678901
GRE CAT, and this is especially true if you are answering a majority of the 2
12345678901234567890123456789012123456789012345678901
2345678901234567890123456789012123456789012345678901
2
2
1 questions correctly. If you’re doing well, you’ll find yourself bombarded by
12345678901234567890123456789012123456789012345678901 2
12345678901234567890123456789012123456789012345678901
antonyms and analogies. These two question types are very vocabulary- 2
12345678901234567890123456789012123456789012345678901 2
12345678901234567890123456789012123456789012345678901
centric, since knowledge of the words used is a large factor in getting the 2
1 problems right. The better you do on the GRE CAT Verbal, the more the 2
2345678901234567890123456789012123456789012345678901
12345678901234567890123456789012123456789012345678901 2
12345678901234567890123456789012123456789012345678901 2
12345678901234567890123456789012123456789012345678901
section resembles a vocabulary quiz by the end. 2
12345678901234567890123456789012123456789012345678901
2345678901234567890123456789012123456789012345678901
2
2
1
12345678901234567890123456789012123456789012345678901 2
12345678901234567890123456789012123456789012345678901 2

X-Ref
12345678901234567890123456789012123456789012345678901 2
12345678901234567890123456789012123456789012345678901 2
12345678901234567890123456789012123456789012345678901 2
2345678901234567890123456789012123456789012345678901
In-depth discussion and suggestions on how to tackle each of the four
12345678901234567890123456789012123456789012345678901 2
Verbal question types is covered starting on page 25.
12345678901234567890123456789012123456789012345678901 2
12345678901234567890123456789012123456789012345678901 2
12345678901234567890123456789012123456789012345678901 2
12345678901234567890123456789012123456789012345678901 2 2
1
12345678901234567890123456789012123456789012345678901 2
12345678901234567890123456789012123456789012345678901
The Verbal section also starts out like a vocabulary quiz, since the first 3–6 2
12345678901234567890123456789012123456789012345678901 2
12345678901234567890123456789012123456789012345678901
questions are usually antonyms and some analogies. This means that on 2
12345678901234567890123456789012123456789012345678901 2
12345678901234567890123456789012123456789012345678901
the initial part of the Verbal test (when questions count the most), a lot 2
12345678901234567890123456789012123456789012345678901 2
12345678901234567890123456789012123456789012345678901
depends on whether you know the particular word given in a question. 2
2345678901234567890123456789012123456789012345678901
12345678901234567890123456789012123456789012345678901
This makes the beginning of the Verbal section a little arbitrary. People 2
2
12345678901234567890123456789012123456789012345678901
12345678901234567890123456789012123456789012345678901
with large vocabularies might get an unfamiliar word, while someone with 2
2
12345678901234567890123456789012123456789012345678901
1 a less-developed vocabulary will get a tough word whose meaning they 2
12345678901234567890123456789012123456789012345678901 2
12345678901234567890123456789012123456789012345678901
just happen to know. All in all, the start of the section is a bit of a crap 2
12345678901234567890123456789012123456789012345678901 2
12345678901234567890123456789012123456789012345678901 2
12345678901234567890123456789012123456789012345678901
shoot. Getting the right word or words
2345678901234567890123456789012123456789012345678901
can help your score, especially if 2
2
12345678901234567890123456789012123456789012345678901
12345678901234567890123456789012123456789012345678901
you don’t consider yourself someone with a large vocabulary. 2
1 2
12345678901234567890123456789012123456789012345678901 2
12345678901234567890123456789012123456789012345678901
The Verbal section might seem less fair than the Math, but there’s no real 2
2345678901234567890123456789012123456789012345678901
difference. If you’re strong in geometry, and your first two math questions 2
12345678901234567890123456789012123456789012345678901
12345678901234567890123456789012123456789012345678901 2
1 concern geometry, then you’re in luck. If geometry is your worst subject, 2
12345678901234567890123456789012123456789012345678901 2
12345678901234567890123456789012123456789012345678901
then you’re out of luck. The adaptive format, with early questions 2
12345678901234567890123456789012123456789012345678901 2
12345678901234567890123456789012123456789012345678901
counting for more, is the culprit (or helper) in both cases. 2
12345678901234567890123456789012123456789012345678901 2
12345678901234567890123456789012123456789012345678901 2
12345678901234567890123456789012123456789012345678901 2
In the end, just knowing how the Verbal section will begin gives you a
2345678901234567890123456789012123456789012345678901
12345678901234567890123456789012123456789012345678901
mental advantage over someone who just shows up and starts answering 2
2
12345678901234567890123456789012123456789012345678901 2
12345678901234567890123456789012123456789012345678901 2
12345678901234567890123456789012123456789012345678901
questions blindly.
2
1
2345678901234567890123456789012123456789012345678901
A reading passage will show up sooner or later, with 2–4 questions 2
12345678901234567890123456789012123456789012345678901
12345678901234567890123456789012123456789012345678901 2
12345678901234567890123456789012123456789012345678901 2
12345678901234567890123456789012123456789012345678901 2
attached to it. If you do a lot of Web site reading, then you won’t have
1 much problem with the computer format. There’s a scroll bar to help you 2
12345678901234567890123456789012123456789012345678901 2
12345678901234567890123456789012123456789012345678901 2
12345678901234567890123456789012123456789012345678901
move up and down the text, but the actual text box is ridiculously small. It 2
12345678901234567890123456789012123456789012345678901
only holds two or three sentences at a time, so no matter how short the 2
12345678901234567890123456789012123456789012345678901 2
1 reading passage is, there’s always going to be some scrolling to do. To give 2
2345678901234567890123456789012123456789012345678901
12345678901234567890123456789012123456789012345678901 2
12345678901234567890123456789012123456789012345678901 2
12345678901234567890123456789012123456789012345678901 2
26 123456789012345678901234567890121234567890123456789012

www.petersons.com
Chapter 2: Section Strategies

123456789012345678901234567890121234567890123456789012
12345678901234567890123456789012123456789012345678901 2
12345678901234567890123456789012123456789012345678901 2
12345678901234567890123456789012123456789012345678901
yourself a preview of what it will be like, open up some text on a computer, 2
12345678901234567890123456789012123456789012345678901 2
12345678901234567890123456789012123456789012345678901
and place it in a box about 3 inches by 4 inches. Give yourself some big 2
12345678901234567890123456789012123456789012345678901 2
12345678901234567890123456789012123456789012345678901
letters, and allow for a nice amount of space between each line.
2345678901234567890123456789012123456789012345678901
2
2
1
12345678901234567890123456789012123456789012345678901 2
12345678901234567890123456789012123456789012345678901 2
12345678901234567890123456789012123456789012345678901
On RC problems, the passage takes up the upper left portion of the 2
12345678901234567890123456789012123456789012345678901 2
Tip
12345678901234567890123456789012123456789012345678901
screen, while questions appear on the right side. The bottom of the screen 2
12345678901234567890123456789012123456789012345678901 2
12345678901234567890123456789012123456789012345678901
contains a lot of basic instructions like “Time on/off” and “Wanna quit, 2
12345678901234567890123456789012123456789012345678901 2
12345678901234567890123456789012123456789012345678901
fraidy cat?” stuff that you will rarely, if
2345678901234567890123456789012123456789012345678901
ever, use during the exam. 2
2
1
12345678901234567890123456789012123456789012345678901 2
12345678901234567890123456789012123456789012345678901 2
12345678901234567890123456789012123456789012345678901
If the above instructions are too tech-savvy for you, don’t sweat it. The 2
12345678901234567890123456789012123456789012345678901
2345678901234567890123456789012123456789012345678901
2
2
1 point is that you will have to move up and down to read some text, so be
12345678901234567890123456789012123456789012345678901 2
12345678901234567890123456789012123456789012345678901
prepared to do just that. 2
12345678901234567890123456789012123456789012345678901 2
12345678901234567890123456789012123456789012345678901 2
12345678901234567890123456789012123456789012345678901 22
2345678901234567890123456789012123456789012345678901
The last problem type, Sentence Completion, translates very well to the
12345678901234567890123456789012123456789012345678901
computer. Of the four Verbal question types, these questions are probably 2
12345678901234567890123456789012123456789012345678901
the easiest to prepare for and improve on, which might be why they show 2
12345678901234567890123456789012123456789012345678901
12345678901234567890123456789012123456789012345678901 2
12345678901234567890123456789012123456789012345678901 2
1 up the least. Even so, when they do pop up on your screen, lick your chops 2
12345678901234567890123456789012123456789012345678901
and dive right in. 2
12345678901234567890123456789012123456789012345678901 2
12345678901234567890123456789012123456789012345678901 2
12345678901234567890123456789012123456789012345678901 22
2345678901234567890123456789012123456789012345678901
12345678901234567890123456789012123456789012345678901 2
12345678901234567890123456789012123456789012345678901
12345678901234567890123456789012123456789012345678901
. . . And You’re Done 2
12345678901234567890123456789012123456789012345678901 2
12345678901234567890123456789012123456789012345678901 22
For most people, time is not as big an issue on the Verbal section as it is on
12345678901234567890123456789012123456789012345678901
the Math section. Many hard Math problems are complex, multi-step 2
12345678901234567890123456789012123456789012345678901
12345678901234567890123456789012123456789012345678901 2
1 questions that require a lot of computations in order to unravel the right 2
2345678901234567890123456789012123456789012345678901
answer. This takes time. In contrast, hard questions on the Verbal section 2
12345678901234567890123456789012123456789012345678901
12345678901234567890123456789012123456789012345678901 2
1 consist mainly of hard vocabulary words. You still need time to go through 2
12345678901234567890123456789012123456789012345678901 2
12345678901234567890123456789012123456789012345678901
the answer choices and use some strategies, but this doesn’t take as long as 2
12345678901234567890123456789012123456789012345678901 2
12345678901234567890123456789012123456789012345678901 2
S DS D
12345678901234567890123456789012123456789012345678901 2
2
12345678901234567890123456789012123456789012345678901
12345678901234567890123456789012123456789012345678901
9 15
7
2
2
2345678901234567890123456789012123456789012345678901
12345678901234567890123456789012123456789012345678901
figuring out what the value of is. 2
1 S DS D
12345678901234567890123456789012123456789012345678901
8 14
12345678901234567890123456789012123456789012345678901
25 33
2
2
2
12345678901234567890123456789012123456789012345678901 2
2345678901234567890123456789012123456789012345678901
There’s no need to rush through any questions on the Verbal section, since 2
12345678901234567890123456789012123456789012345678901
12345678901234567890123456789012123456789012345678901 2
12345678901234567890123456789012123456789012345678901
lack of time is rarely a problem. On average, you have a minute per 2
12345678901234567890123456789012123456789012345678901 22
1 question, but some questions will take less time. For example, if there’s an
12345678901234567890123456789012123456789012345678901 2
12345678901234567890123456789012123456789012345678901
antonym question with a word that you know, finding the right answer 2
12345678901234567890123456789012123456789012345678901 2
12345678901234567890123456789012123456789012345678901
can take under 15 seconds. Don’t spend less than15 seconds, because you 2
12345678901234567890123456789012123456789012345678901 2
12345678901234567890123456789012123456789012345678901
want to make sure not to make a careless error by speeding. That gives you 2
12345678901234567890123456789012123456789012345678901 2
12345678901234567890123456789012123456789012345678901
a lot of extra time to spend on another problem, such as a RC question. 2
12345678901234567890123456789012123456789012345678901 2
12345678901234567890123456789012123456789012345678901
These typically take more time since you want to check your answer with 2
2345678901234567890123456789012123456789012345678901
12345678901234567890123456789012123456789012345678901 2
1 the information in the passage. 2
12345678901234567890123456789012123456789012345678901 2
12345678901234567890123456789012123456789012345678901 2
12345678901234567890123456789012123456789012345678901
If you are running out of time on a practice Verbal section, then you are 2
1 probably staring at some questions and dredging your brain to come up 2
2345678901234567890123456789012123456789012345678901
12345678901234567890123456789012123456789012345678901 2
12345678901234567890123456789012123456789012345678901 2
12345678901234567890123456789012123456789012345678901 2 27
123456789012345678901234567890121234567890123456789012
Part I: Understanding the GRE

123456789012345678901234567890121234567890123456789012
12345678901234567890123456789012123456789012345678901 2
12345678901234567890123456789012123456789012345678901 2
12345678901234567890123456789012123456789012345678901
with a definition of a word. This is not a good use of your time. The 2
12345678901234567890123456789012123456789012345678901 2
12345678901234567890123456789012123456789012345678901
vocabulary on the GRE can be tough, so there will be some words whose 2
12345678901234567890123456789012123456789012345678901 2
12345678901234567890123456789012123456789012345678901
meaning you don’t know. Accept this, use any elimination strategies you 2
2345678901234567890123456789012123456789012345678901 2
1 can on those problems, pick an answer, and move on.
12345678901234567890123456789012123456789012345678901 2
12345678901234567890123456789012123456789012345678901 2
12345678901234567890123456789012123456789012345678901 2
12345678901234567890123456789012123456789012345678901
2345678901234567890123456789012123456789012345678901
2
2
1
12345678901234567890123456789012123456789012345678901
Analytical Writing 2
12345678901234567890123456789012123456789012345678901 2
12345678901234567890123456789012123456789012345678901 2
12345678901234567890123456789012123456789012345678901
The first essay prompt you see poses an issue, and then gives you 45 2
1 minutes to analyze and discuss that issue. This is your standard, “There’s 2
2345678901234567890123456789012123456789012345678901
12345678901234567890123456789012123456789012345678901 2
12345678901234567890123456789012123456789012345678901 2
12345678901234567890123456789012123456789012345678901
no right or wrong side, all that matters is how well you present your side of 2
12345678901234567890123456789012123456789012345678901
2345678901234567890123456789012123456789012345678901
2
2
1 the case” kind of essay. You are often given two topics to choose from, so
12345678901234567890123456789012123456789012345678901 2
12345678901234567890123456789012123456789012345678901
pick the topic you feel comfortable—um, talking about. 2
12345678901234567890123456789012123456789012345678901 2
12345678901234567890123456789012123456789012345678901
For the second essay, analysis of an argument, you only have 30 minutes to 2
12345678901234567890123456789012123456789012345678901 2
2345678901234567890123456789012123456789012345678901
write. This time, you are first given some text, in which someone makes an 2
12345678901234567890123456789012123456789012345678901
12345678901234567890123456789012123456789012345678901 2
argument about something. This argument is lame in some way or another. 2
12345678901234567890123456789012123456789012345678901
12345678901234567890123456789012123456789012345678901 2
12345678901234567890123456789012123456789012345678901 2
1 Your goal here is to evaluate the argument, i.e., explain ways in which the 2
12345678901234567890123456789012123456789012345678901
argument’s lame and ways in which it works. 2
12345678901234567890123456789012123456789012345678901 2
12345678901234567890123456789012123456789012345678901 2
12345678901234567890123456789012123456789012345678901 2
2345678901234567890123456789012123456789012345678901
Combined, the two essays take over an hour to start at the beginning of
every GRE CAT. That’s a lot of time spent typing, and the key word here is 2
12345678901234567890123456789012123456789012345678901
12345678901234567890123456789012123456789012345678901 2
12345678901234567890123456789012123456789012345678901 2
12345678901234567890123456789012123456789012345678901 2
typing. How fast do you type? Answering this question will determine
your approach to these essays. Many English aficionados will talk 2
12345678901234567890123456789012123456789012345678901
12345678901234567890123456789012123456789012345678901 2
12345678901234567890123456789012123456789012345678901 2
12345678901234567890123456789012123456789012345678901 2
gushingly about the standard five-paragraph essay approach that virtually
1 guarantees you a top score of 6 on the Analytical portion. Yet, this 2
12345678901234567890123456789012123456789012345678901 2
12345678901234567890123456789012123456789012345678901 2
12345678901234567890123456789012123456789012345678901 2
approach doesn’t work if you’re such a slow typist that you can’t type five
12345678901234567890123456789012123456789012345678901 2
12345678901234567890123456789012123456789012345678901
paragraphs in the time allotted. 2
12345678901234567890123456789012123456789012345678901 2
2345678901234567890123456789012123456789012345678901
12345678901234567890123456789012123456789012345678901 2 2
1
12345678901234567890123456789012123456789012345678901 2
Alert!
As stated in the previous chapter,
12345678901234567890123456789012123456789012345678901if you’re applying for a graduate 2
2345678901234567890123456789012123456789012345678901
program in astrophysics, your Analytical score might not be of highest 2
12345678901234567890123456789012123456789012345678901
12345678901234567890123456789012123456789012345678901 2
1 importance. Do your best, but spend your time studying the section that 2
12345678901234567890123456789012123456789012345678901 2
12345678901234567890123456789012123456789012345678901
matters most, which is Math. 2
12345678901234567890123456789012123456789012345678901 2
12345678901234567890123456789012123456789012345678901 2
12345678901234567890123456789012123456789012345678901 2
12345678901234567890123456789012123456789012345678901 2
12345678901234567890123456789012123456789012345678901
Before anything else, find out how fast you can type. Sit down in front of a 2
2345678901234567890123456789012123456789012345678901
12345678901234567890123456789012123456789012345678901
computer with some word processing program. Start timing, and then type 2
2
12345678901234567890123456789012123456789012345678901 2
12345678901234567890123456789012123456789012345678901
something like, “The quick brown fox jumps over the lazy dog” over and 2
12345678901234567890123456789012123456789012345678901
1 over again until one minute is up. If you make a mistake while typing, stop 2
12345678901234567890123456789012123456789012345678901 2
12345678901234567890123456789012123456789012345678901 2
12345678901234567890123456789012123456789012345678901
and correct your error while the clock is running. Now count up the 2
12345678901234567890123456789012123456789012345678901 2
12345678901234567890123456789012123456789012345678901 2
number of words you typed correctly in one minute. Take this number and
2345678901234567890123456789012123456789012345678901
12345678901234567890123456789012123456789012345678901 2
1 use it to answer the following question: 2
12345678901234567890123456789012123456789012345678901 2
12345678901234567890123456789012123456789012345678901 2
12345678901234567890123456789012123456789012345678901 2
12345678901234567890123456789012123456789012345678901 2
12345678901234567890123456789012123456789012345678901 2
12345678901234567890123456789012123456789012345678901 2
12345678901234567890123456789012123456789012345678901 2
28 123456789012345678901234567890121234567890123456789012

www.petersons.com
Chapter 2: Section Strategies

123456789012345678901234567890121234567890123456789012
12345678901234567890123456789012123456789012345678901 2
12345678901234567890123456789012123456789012345678901 2
12345678901234567890123456789012123456789012345678901 2
12345678901234567890123456789012123456789012345678901
Question How long should my essays be? 2
12345678901234567890123456789012123456789012345678901 2
12345678901234567890123456789012123456789012345678901 2
12345678901234567890123456789012123456789012345678901
Step 1 Take the amount of time
2345678901234567890123456789012123456789012345678901
given for an essay. For essay 1, 2
2
1
12345678901234567890123456789012123456789012345678901
this means 45 minutes. 2
12345678901234567890123456789012123456789012345678901 2
12345678901234567890123456789012123456789012345678901
Step 2 Subtract 5 minutes. This time will be used to decide 2
12345678901234567890123456789012123456789012345678901
2345678901234567890123456789012123456789012345678901
2
2
1 what you want to write in your essay. You shouldn’t
12345678901234567890123456789012123456789012345678901 2
12345678901234567890123456789012123456789012345678901
just blaze off at the start; it helps to plan out your essay 2
12345678901234567890123456789012123456789012345678901 2
12345678901234567890123456789012123456789012345678901
a bit.
2345678901234567890123456789012123456789012345678901
2
2
1
12345678901234567890123456789012123456789012345678901
Step 3 40 minutes remain. Take the number of words you type 2
12345678901234567890123456789012123456789012345678901 2
12345678901234567890123456789012123456789012345678901
per minute and multiply by 40. If you can type 20 2
12345678901234567890123456789012123456789012345678901
2345678901234567890123456789012123456789012345678901
2
2
1 words per minute, then (20)(40) 5 800.
12345678901234567890123456789012123456789012345678901 2
12345678901234567890123456789012123456789012345678901 2
12345678901234567890123456789012123456789012345678901
Step 4 This is the number of words you could type if you knew 2
12345678901234567890123456789012123456789012345678901 2
12345678901234567890123456789012123456789012345678901 22
2345678901234567890123456789012123456789012345678901
exactly what you were going to say and typed nonstop.
12345678901234567890123456789012123456789012345678901
12345678901234567890123456789012123456789012345678901
Since there will be many points in your essay where you 2
12345678901234567890123456789012123456789012345678901 2
12345678901234567890123456789012123456789012345678901 22
pause for a moment to come up with the right phrase or
12345678901234567890123456789012123456789012345678901 2
1 sentence, 800 is not really achievable. Halve this
12345678901234567890123456789012123456789012345678901 2
12345678901234567890123456789012123456789012345678901
number for a more realistic assessment, giving you 400 2
12345678901234567890123456789012123456789012345678901
words. 2
12345678901234567890123456789012123456789012345678901 22
2345678901234567890123456789012123456789012345678901
12345678901234567890123456789012123456789012345678901
12345678901234567890123456789012123456789012345678901
Step 5 If you assume a standard paragraph is around 100 2
12345678901234567890123456789012123456789012345678901 2
12345678901234567890123456789012123456789012345678901 22
words, you can see that your essay will not make the
12345678901234567890123456789012123456789012345678901
12345678901234567890123456789012123456789012345678901
five-paragraph format due to technical reasons (i.e. 2
12345678901234567890123456789012123456789012345678901 2
12345678901234567890123456789012123456789012345678901 22
your slow typing.) You only have time for four
1
12345678901234567890123456789012123456789012345678901
paragraphs. You should therefore state your case clearly 2
12345678901234567890123456789012123456789012345678901 2
12345678901234567890123456789012123456789012345678901 2
in Paragraph 1, back it up with claims in the next two
12345678901234567890123456789012123456789012345678901 2
12345678901234567890123456789012123456789012345678901
paragraphs, and then wrap things up in Paragraph 4 as
2345678901234567890123456789012123456789012345678901
2
2
12345678901234567890123456789012123456789012345678901
12345678901234567890123456789012123456789012345678901
time expires on you and your nondexterous fingers. 2
1 2
12345678901234567890123456789012123456789012345678901 2
12345678901234567890123456789012123456789012345678901
Unless you practice typing or type a lot, you might be appalled at how 2
2345678901234567890123456789012123456789012345678901
12345678901234567890123456789012123456789012345678901
slow you are. Even with fast fingers, there’s no way to compose a flawless, 2
2
12345678901234567890123456789012123456789012345678901
1 five-paragraph essay on a previously unknown topic in 45 minutes. 2
12345678901234567890123456789012123456789012345678901 2
12345678901234567890123456789012123456789012345678901 2
12345678901234567890123456789012123456789012345678901
Flawless essays require time to map out, draft, and then rewrite. Maybe a 2
12345678901234567890123456789012123456789012345678901 2
12345678901234567890123456789012123456789012345678901
professional essay writer—if there are any still around these days—could 2
12345678901234567890123456789012123456789012345678901
do it, but folks taking the GRE can’t, and it isn’t expected of them. What is 2
12345678901234567890123456789012123456789012345678901 2
2345678901234567890123456789012123456789012345678901
expected is that the thoughts you do punch into the computer are good, 2
12345678901234567890123456789012123456789012345678901
12345678901234567890123456789012123456789012345678901 2
12345678901234567890123456789012123456789012345678901 2
12345678901234567890123456789012123456789012345678901 22
and that the overall structure of your essay is sound.
1
12345678901234567890123456789012123456789012345678901
On both essays, don’t sacrifice quality for quantity. Two body (middle) 2
12345678901234567890123456789012123456789012345678901 2
12345678901234567890123456789012123456789012345678901 2
12345678901234567890123456789012123456789012345678901
paragraphs with 1–3 compelling sentences that back up your initial claim 2
12345678901234567890123456789012123456789012345678901
are all that is needed to get a decent score on the Analytical section. Some 2
12345678901234567890123456789012123456789012345678901 22
2345678901234567890123456789012123456789012345678901
1 people have even peppered their essays with jokes—relevant to the
12345678901234567890123456789012123456789012345678901 2
12345678901234567890123456789012123456789012345678901
discussion, of course—and received a 6, so you don’t have to kill yourself 2
12345678901234567890123456789012123456789012345678901 2
12345678901234567890123456789012123456789012345678901
to place words on the screen. Just make sure that the words you do put 2
12345678901234567890123456789012123456789012345678901 2
12345678901234567890123456789012123456789012345678901
down help support your position in the essay. 2
12345678901234567890123456789012123456789012345678901 2 29
123456789012345678901234567890121234567890123456789012
Part I: Understanding the GRE

123456789012345678901234567890121234567890123456789012
12345678901234567890123456789012123456789012345678901 2
12345678901234567890123456789012123456789012345678901 2
12345678901234567890123456789012123456789012345678901
If you want to increase your typing skills, get a dictionary, a computer, and 2
12345678901234567890123456789012123456789012345678901 2
12345678901234567890123456789012123456789012345678901
get ready to kill two birds with one stone. Purchase a book that lists 2
12345678901234567890123456789012123456789012345678901 2
12345678901234567890123456789012123456789012345678901
college-level vocabulary words, and then spend time typing in each word 2
1 and its definition into a word processing document. By doing this, you will 2
2345678901234567890123456789012123456789012345678901
12345678901234567890123456789012123456789012345678901 2
12345678901234567890123456789012123456789012345678901 2
12345678901234567890123456789012123456789012345678901
improve your typing skills and create a vocabulary study guide at the 2
12345678901234567890123456789012123456789012345678901
2345678901234567890123456789012123456789012345678901
2
2
1 same time.
12345678901234567890123456789012123456789012345678901 2
12345678901234567890123456789012123456789012345678901 2
12345678901234567890123456789012123456789012345678901 2
12345678901234567890123456789012123456789012345678901
2345678901234567890123456789012123456789012345678901
2
2
1
12345678901234567890123456789012123456789012345678901 2
12345678901234567890123456789012123456789012345678901 2
12345678901234567890123456789012123456789012345678901 2
12345678901234567890123456789012123456789012345678901
2345678901234567890123456789012123456789012345678901
2
2
1
12345678901234567890123456789012123456789012345678901 2
12345678901234567890123456789012123456789012345678901 2
12345678901234567890123456789012123456789012345678901 2
12345678901234567890123456789012123456789012345678901 2
12345678901234567890123456789012123456789012345678901 2
2345678901234567890123456789012123456789012345678901
12345678901234567890123456789012123456789012345678901 2
12345678901234567890123456789012123456789012345678901 2
12345678901234567890123456789012123456789012345678901 2
12345678901234567890123456789012123456789012345678901 2
12345678901234567890123456789012123456789012345678901 2 2
1
12345678901234567890123456789012123456789012345678901 2
12345678901234567890123456789012123456789012345678901 2
12345678901234567890123456789012123456789012345678901 2
12345678901234567890123456789012123456789012345678901 2
2345678901234567890123456789012123456789012345678901
12345678901234567890123456789012123456789012345678901 2
12345678901234567890123456789012123456789012345678901 2
12345678901234567890123456789012123456789012345678901 2
12345678901234567890123456789012123456789012345678901 2
12345678901234567890123456789012123456789012345678901 2
12345678901234567890123456789012123456789012345678901 2
12345678901234567890123456789012123456789012345678901 2
12345678901234567890123456789012123456789012345678901 2 2
1
12345678901234567890123456789012123456789012345678901 2
12345678901234567890123456789012123456789012345678901 2
12345678901234567890123456789012123456789012345678901 2
12345678901234567890123456789012123456789012345678901 2
12345678901234567890123456789012123456789012345678901 2
12345678901234567890123456789012123456789012345678901 2
12345678901234567890123456789012123456789012345678901 2
12345678901234567890123456789012123456789012345678901 2
12345678901234567890123456789012123456789012345678901 2
12345678901234567890123456789012123456789012345678901 2
12345678901234567890123456789012123456789012345678901 2
2345678901234567890123456789012123456789012345678901
12345678901234567890123456789012123456789012345678901 2 2
1
12345678901234567890123456789012123456789012345678901 2
12345678901234567890123456789012123456789012345678901 2
12345678901234567890123456789012123456789012345678901 2
12345678901234567890123456789012123456789012345678901 2
12345678901234567890123456789012123456789012345678901 2
12345678901234567890123456789012123456789012345678901 2
12345678901234567890123456789012123456789012345678901 2
12345678901234567890123456789012123456789012345678901 2
2345678901234567890123456789012123456789012345678901
12345678901234567890123456789012123456789012345678901 2
12345678901234567890123456789012123456789012345678901 2
12345678901234567890123456789012123456789012345678901 2 2
1
12345678901234567890123456789012123456789012345678901 2
12345678901234567890123456789012123456789012345678901 2
12345678901234567890123456789012123456789012345678901 2
12345678901234567890123456789012123456789012345678901 2
12345678901234567890123456789012123456789012345678901 2
12345678901234567890123456789012123456789012345678901 2
2345678901234567890123456789012123456789012345678901
2
1
12345678901234567890123456789012123456789012345678901 2
12345678901234567890123456789012123456789012345678901 2
12345678901234567890123456789012123456789012345678901 2
12345678901234567890123456789012123456789012345678901 2
12345678901234567890123456789012123456789012345678901 2
12345678901234567890123456789012123456789012345678901 2
12345678901234567890123456789012123456789012345678901 2
30 123456789012345678901234567890121234567890123456789012

www.petersons.com
PART

II
Diagnosing Strengths
and Weaknesses
Diagnostic:
Dipping Your Big Toe in the GRE Waters 32

PART II
Chapter

Diagnostic: Dipping Your Big Toe


3
in the GRE Waters
1234567890123456789012345678901212345678901234567890123456789012123456
12 3456789012345678901234567890121234567890123456789012345678901212345 6
12 3456789012345678901234567890121234567890123456789012345678901212345 6
12 3456789012345678901234567890121234567890123456789012345678901212345
Now that you have some section-specific techniques, it’s time to put them to use. Trying a 6
2 3456789012345678901234567890121234567890123456789012345678901212345
123456789012345678901234567890121234567890123456789012345678901212345 6
123456789012345678901234567890121234567890123456789012345678901212345
sample set of math and verbal questions has the following benefits: 6
123456789012345678901234567890121234567890123456789012345678901212345 66
123456789012345678901234567890121234567890123456789012345678901212345
1. It gives you an idea of how you might do if you were to take the real GRE this very 6
123456789012345678901234567890121234567890123456789012345678901212345
123456789012345678901234567890121234567890123456789012345678901212345 6
1 moment. 6
12 3456789012345678901234567890121234567890123456789012345678901212345 6
12 3456789012345678901234567890121234567890123456789012345678901212345
2. It lets you see what areas of study you’re strongest in, and what areas need 6
12 3456789012345678901234567890121234567890123456789012345678901212345 6
2 3456789012345678901234567890121234567890123456789012345678901212345
improvement. This knowledge can be used to fine-tune your study plans for the GRE. 6
123456789012345678901234567890121234567890123456789012345678901212345
123456789012345678901234567890121234567890123456789012345678901212345 6
123456789012345678901234567890121234567890123456789012345678901212345 6
123456789012345678901234567890121234567890123456789012345678901212345
3. No matter how you do on the diagnostic, it doesn’t count. 6
123456789012345678901234567890121234567890123456789012345678901212345 66
123456789012345678901234567890121234567890123456789012345678901212345
The third point is perhaps the most important. If you answer every sample question correctly, 6
123456789012345678901234567890121234567890123456789012345678901212345
then bellow out a proud “Huzzah!” You can’t send that score to universities, but you can feel 6
123456789012345678901234567890121234567890123456789012345678901212345
123456789012345678901234567890121234567890123456789012345678901212345 6
1 6
123456789012345678901234567890121234567890123456789012345678901212345 66
2 3456789012345678901234567890121234567890123456789012345678901212345
good about the fact that you’ve demonstrated the ability to do well on GRE-type questions. On
123456789012345678901234567890121234567890123456789012345678901212345 6
1 the other hand, if you miss a lot of diagnostic questions, don’t sweat it.
123456789012345678901234567890121234567890123456789012345678901212345 6
12 3456789012345678901234567890121234567890123456789012345678901212345 6
12 3456789012345678901234567890121234567890123456789012345678901212345 6
12 3456789012345678901234567890121234567890123456789012345678901212345 6
Alert!

123456789012345678901234567890121234567890123456789012345678901212345
If you waited until the last moment and are taking this diagnostic the night before 6
12 3456789012345678901234567890121234567890123456789012345678901212345 6
12 3456789012345678901234567890121234567890123456789012345678901212345 6
the real GRE CAT, you can sweat a little. Hopefully, you are taking this
2 3456789012345678901234567890121234567890123456789012345678901212345
123456789012345678901234567890121234567890123456789012345678901212345 6
123456789012345678901234567890121234567890123456789012345678901212345
diagnostic at least a month in advance of the real test, giving you ample time to go 6
1 6
123456789012345678901234567890121234567890123456789012345678901212345 6
through the rest of the book at a measured pace.
123456789012345678901234567890121234567890123456789012345678901212345 6
12 3456789012345678901234567890121234567890123456789012345678901212345 6
12 3456789012345678901234567890121234567890123456789012345678901212345 6
12 3456789012345678901234567890121234567890123456789012345678901212345
The whole point of the diagnostic is to determine what subjects you’re good at and what 6
12 3456789012345678901234567890121234567890123456789012345678901212345 6
123456789012345678901234567890121234567890123456789012345678901212345 6
123456789012345678901234567890121234567890123456789012345678901212345 66
2 3456789012345678901234567890121234567890123456789012345678901212345
subjects you need help in. You can then use this information to devise a study plan tailored to
123456789012345678901234567890121234567890123456789012345678901212345 6
123456789012345678901234567890121234567890123456789012345678901212345
your strengths and weaknesses.
123456789012345678901234567890121234567890123456789012345678901212345 66
1
12 3456789012345678901234567890121234567890123456789012345678901212345 6
12 3456789012345678901234567890121234567890123456789012345678901212345 6
12 3456789012345678901234567890121234567890123456789012345678901212345
Should You Take It to the Next Level? 6
12 3456789012345678901234567890121234567890123456789012345678901212345 6
123456789012345678901234567890121234567890123456789012345678901212345
As stated earlier, the adaptive nature of the GRE makes it likely that someone shooting for a 6
123456789012345678901234567890121234567890123456789012345678901212345 66
2 3456789012345678901234567890121234567890123456789012345678901212345
1 good score will only see medium and hard questions. Since you probably didn’t purchase this
12 3456789012345678901234567890121234567890123456789012345678901212345 6
123456789012345678901234567890121234567890123456789012345678901212345
book to get a lousy score, it makes sense to concentrate on medium and hard GRE questions. 6
12 3456789012345678901234567890121234567890123456789012345678901212345 6
123456789012345678901234567890121234567890123456789012345678901212345
The bulk of this book does just that. The first part of each chapter in the subject review section 6
123456789012345678901234567890121234567890123456789012345678901212345 6
123456789012345678901234567890121234567890123456789012345678901212345
focuses on how medium questions will appear on the math and verbal sections, and it provides 6
123456789012345678901234567890121234567890123456789012345678901212345 6
1234567890123456789012345678901212345678901234567890123456789012123456
32
DIAGNOSTIC
1234567890123456789012345678901212345678901234567890123456789012123456
123456789012345678901234567890121234567890123456789012345678901212345 6
12 3456789012345678901234567890121234567890123456789012345678901212345 6
123456789012345678901234567890121234567890123456789012345678901212345
you with the facts and strategies needed to tackle these problems. The general instruction for 6
12 3456789012345678901234567890121234567890123456789012345678901212345 6
12 3456789012345678901234567890121234567890123456789012345678901212345
the Analytical section goes over the basic structure of the essay, giving you the knowledge 6
123456789012345678901234567890121234567890123456789012345678901212345 6
12
2
3456789012345678901234567890121234567890123456789012345678901212345
needed to gain a respectable score on the written portion. For most students, starting with the 6
3456789012345678901234567890121234567890123456789012345678901212345 6
1
12 3456789012345678901234567890121234567890123456789012345678901212345
medium-level material is the wisest course of action. 6
12 3456789012345678901234567890121234567890123456789012345678901212345 6
12 3456789012345678901234567890121234567890123456789012345678901212345
However, suppose you feel very confident about your math abilities. A high score on the math 6
12
2
3456789012345678901234567890121234567890123456789012345678901212345
3456789012345678901234567890121234567890123456789012345678901212345
6
1
12 portion of the diagnostic proves that this confidence is well-founded. If that’s the case, you may 6
3456789012345678901234567890121234567890123456789012345678901212345 6
12 3456789012345678901234567890121234567890123456789012345678901212345 6
12 3456789012345678901234567890121234567890123456789012345678901212345 6
choose to skip over the general study material on math and go straight to the Take It to the Next
12 3456789012345678901234567890121234567890123456789012345678901212345
Level material. This material focuses on more advanced topics—involved probabilities and 6
123456789012345678901234567890121234567890123456789012345678901212345 6
12 3456789012345678901234567890121234567890123456789012345678901212345 6
12 3456789012345678901234567890121234567890123456789012345678901212345 6
number sets, for example—and it shows how these topics appear on hard problems. The Next
12 3456789012345678901234567890121234567890123456789012345678901212345
Level Verbal and Analytical sections cover advanced topics and problems in a similar fashion. 6
12
2
3456789012345678901234567890121234567890123456789012345678901212345
3456789012345678901234567890121234567890123456789012345678901212345
6
6
1
12 3456789012345678901234567890121234567890123456789012345678901212345
So what’s the best study plan? Not to get too mystical or anything, but the answer to this 6
12 3456789012345678901234567890121234567890123456789012345678901212345 6
12 3456789012345678901234567890121234567890123456789012345678901212345
question can only be found inside you. By working through the entire book, you give yourself a 6
12 3456789012345678901234567890121234567890123456789012345678901212345 6
2 3456789012345678901234567890121234567890123456789012345678901212345
look at everything this book has to offer. Yet this approach takes time, and time is a scarce 6
123456789012345678901234567890121234567890123456789012345678901212345
123456789012345678901234567890121234567890123456789012345678901212345
commodity in many people’s lives. Therefore, you might wish to Take It to the Next Level in 6
6
123456789012345678901234567890121234567890123456789012345678901212345
123456789012345678901234567890121234567890123456789012345678901212345
subjects you feel comfortable with in order to use the time you have to concentrate on your 6
6
123456789012345678901234567890121234567890123456789012345678901212345
123456789012345678901234567890121234567890123456789012345678901212345 6
1 weakest areas. 6
12 3456789012345678901234567890121234567890123456789012345678901212345 6
12 3456789012345678901234567890121234567890123456789012345678901212345
The requirements of your graduate program might let you tailor your approach in another way. 6
12 3456789012345678901234567890121234567890123456789012345678901212345 6
2 3456789012345678901234567890121234567890123456789012345678901212345
For example, some fine arts programs require students to take the GRE, but these programs are 6
123456789012345678901234567890121234567890123456789012345678901212345
123456789012345678901234567890121234567890123456789012345678901212345 6
123456789012345678901234567890121234567890123456789012345678901212345 6
123456789012345678901234567890121234567890123456789012345678901212345 66
not concerned with a student’s Quantitative (math) score. If this is the case for you, you could
123456789012345678901234567890121234567890123456789012345678901212345
decide to skip the Next Level math, since a high score in math will not help you one way or 6
123456789012345678901234567890121234567890123456789012345678901212345
another. It would still be a good idea to go over the basic study material—you don’t want to 6
123456789012345678901234567890121234567890123456789012345678901212345
123456789012345678901234567890121234567890123456789012345678901212345 6
123456789012345678901234567890121234567890123456789012345678901212345
completely blow the math section—but a high math score is just not a relevant concern. 6
1 6
123456789012345678901234567890121234567890123456789012345678901212345 66
2 3456789012345678901234567890121234567890123456789012345678901212345
123456789012345678901234567890121234567890123456789012345678901212345
Before making any decision, take the diagnostic and let the results guide you. There are 28
6
1
123456789012345678901234567890121234567890123456789012345678901212345
Quantitative questions, roughly divided so that the first half contains medium questions while 6
12 3456789012345678901234567890121234567890123456789012345678901212345 6
12 3456789012345678901234567890121234567890123456789012345678901212345
the second half contains hard questions. If you ace 201 Quantitative problems in allotted time, 6
12 3456789012345678901234567890121234567890123456789012345678901212345 6
123456789012345678901234567890121234567890123456789012345678901212345
then you are a good candidate for skipping the general material if you want. If you miss 6
12 3456789012345678901234567890121234567890123456789012345678901212345
questions randomly throughout the section, it would be a good idea to look over both the 6
12 3456789012345678901234567890121234567890123456789012345678901212345 6
2 3456789012345678901234567890121234567890123456789012345678901212345
general and Next Level materials, since there’s no clear pattern as to why you got some 6
123456789012345678901234567890121234567890123456789012345678901212345
123456789012345678901234567890121234567890123456789012345678901212345 6
1 6
123456789012345678901234567890121234567890123456789012345678901212345 6
questions right while missing others. It could mean you were weak in a specific subject—like
123456789012345678901234567890121234567890123456789012345678901212345
determining mean/median/mode, for instance—in which case a comprehensive review of a wide 6
12 3456789012345678901234567890121234567890123456789012345678901212345 6
12 3456789012345678901234567890121234567890123456789012345678901212345 6
12 3456789012345678901234567890121234567890123456789012345678901212345 6
range of topics would be beneficial. If you miss a bunch of medium questions but get all the
12 3456789012345678901234567890121234567890123456789012345678901212345
harder problems right, chances are your problem lies in pacing. You rushed through the 6
123456789012345678901234567890121234567890123456789012345678901212345 6
2 3456789012345678901234567890121234567890123456789012345678901212345
medium questions, making careless errors, in order to spend more time on the harder problems. 6
123456789012345678901234567890121234567890123456789012345678901212345
123456789012345678901234567890121234567890123456789012345678901212345 6
123456789012345678901234567890121234567890123456789012345678901212345 6
123456789012345678901234567890121234567890123456789012345678901212345 66
If this is the case, slow down, Sparky!
1
12 3456789012345678901234567890121234567890123456789012345678901212345 6
12 3456789012345678901234567890121234567890123456789012345678901212345 6
12 3456789012345678901234567890121234567890123456789012345678901212345 6
12 3456789012345678901234567890121234567890123456789012345678901212345 6
123456789012345678901234567890121234567890123456789012345678901212345 6
123456789012345678901234567890121234567890123456789012345678901212345
2 6
1 3456789012345678901234567890121234567890123456789012345678901212345 6
12 3456789012345678901234567890121234567890123456789012345678901212345 6
123456789012345678901234567890121234567890123456789012345678901212345 6
12 3456789012345678901234567890121234567890123456789012345678901212345 6
123456789012345678901234567890121234567890123456789012345678901212345 6
123456789012345678901234567890121234567890123456789012345678901212345 6
123456789012345678901234567890121234567890123456789012345678901212345 6
123456789012345678901234567890121234567890123456789012345678901212345 6 33
1234567890123456789012345678901212345678901234567890123456789012123456
Part II: Diagnosing Strengths and Weaknesses

1234567890123456789012345678901212345678901234567890123456789012123456
123456789012345678901234567890121234567890123456789012345678901212345 6
12 3456789012345678901234567890121234567890123456789012345678901212345 6
123456789012345678901234567890121234567890123456789012345678901212345 6
12 3456789012345678901234567890121234567890123456789012345678901212345 6
12 3456789012345678901234567890121234567890123456789012345678901212345
Note 6
123456789012345678901234567890121234567890123456789012345678901212345
In our diagnostic the Quantitative section starts off with 14 QC questions, and then 6
12
2
3456789012345678901234567890121234567890123456789012345678901212345
3456789012345678901234567890121234567890123456789012345678901212345
6
6
1 follows with 14 regular math problems. The 14 QC problems are arranged in order
12 3456789012345678901234567890121234567890123456789012345678901212345 6
12 3456789012345678901234567890121234567890123456789012345678901212345 6
of difficulty, and the 14 regular questions are also in their own order of difficulty.
12 3456789012345678901234567890121234567890123456789012345678901212345 6
12 3456789012345678901234567890121234567890123456789012345678901212345 6
1 3456789012345678901234567890121234567890123456789012345678901212345 6
2
12 3456789012345678901234567890121234567890123456789012345678901212345
Things are a little different for the Verbal section. There are 30 questions, still divided into half 6
12 3456789012345678901234567890121234567890123456789012345678901212345 6
12 3456789012345678901234567890121234567890123456789012345678901212345 6
12
2
3456789012345678901234567890121234567890123456789012345678901212345 6
medium/half hard, but there are four distinct question
3456789012345678901234567890121234567890123456789012345678901212345
types:
6
1
12 3456789012345678901234567890121234567890123456789012345678901212345
• Antonyms 6
12 3456789012345678901234567890121234567890123456789012345678901212345 6
12 3456789012345678901234567890121234567890123456789012345678901212345
• Analogies 6
12
2
3456789012345678901234567890121234567890123456789012345678901212345
3456789012345678901234567890121234567890123456789012345678901212345
6
6
1 • Sentence Completions
12 3456789012345678901234567890121234567890123456789012345678901212345 6
12 3456789012345678901234567890121234567890123456789012345678901212345 6
12 3456789012345678901234567890121234567890123456789012345678901212345 6
• Reading Comprehension
12 3456789012345678901234567890121234567890123456789012345678901212345 6
12 3456789012345678901234567890121234567890123456789012345678901212345
Questions within each question type are arranged in order of difficulty, so the first antonym you 6
12 3456789012345678901234567890121234567890123456789012345678901212345 6
12 3456789012345678901234567890121234567890123456789012345678901212345
see is (theoretically) the easiest, while the last is the hardest. After that section, you proceed to 6
12 3456789012345678901234567890121234567890123456789012345678901212345 6
12 3456789012345678901234567890121234567890123456789012345678901212345
analogies, with all analogies arranged in their own order of difficulty. For the Reading 6
123456789012345678901234567890121234567890123456789012345678901212345
2 6
1 3456789012345678901234567890121234567890123456789012345678901212345
Comprehension section, the first passage is easier than the second one. 6
12 3456789012345678901234567890121234567890123456789012345678901212345 6
12 3456789012345678901234567890121234567890123456789012345678901212345 6
12 3456789012345678901234567890121234567890123456789012345678901212345
You can fine-tune your approach depending on how you do within each question type. Suppose 6
2 3456789012345678901234567890121234567890123456789012345678901212345
you answer every analogy correctly but miss half of the Reading Comprehension problems. If 6
123456789012345678901234567890121234567890123456789012345678901212345
123456789012345678901234567890121234567890123456789012345678901212345 6
123456789012345678901234567890121234567890123456789012345678901212345 6
123456789012345678901234567890121234567890123456789012345678901212345 66
this occurs, a voice inside your head should start hollering, “Focus on Reading Comprehension
123456789012345678901234567890121234567890123456789012345678901212345
123456789012345678901234567890121234567890123456789012345678901212345
in both levels, and don’t worry about analogies as much.” 6
123456789012345678901234567890121234567890123456789012345678901212345 66
123456789012345678901234567890121234567890123456789012345678901212345
This book has been designed to give you a variety of study plan options. You want to take 6
123456789012345678901234567890121234567890123456789012345678901212345 6
1
12 3456789012345678901234567890121234567890123456789012345678901212345
charge of the GRE, and a good way to start doing this is to take charge of your study plan. 6
12 3456789012345678901234567890121234567890123456789012345678901212345 6
123456789012345678901234567890121234567890123456789012345678901212345 6
123456789012345678901234567890121234567890123456789012345678901212345 6
12 3456789012345678901234567890121234567890123456789012345678901212345
Optimal Diagnostic-Taking Conditions 6
12 3456789012345678901234567890121234567890123456789012345678901212345 6
12 3456789012345678901234567890121234567890123456789012345678901212345
As much as you might like to, you should not take the diagnostic while watching television, 6
123456789012345678901234567890121234567890123456789012345678901212345 6
12 3456789012345678901234567890121234567890123456789012345678901212345
answering questions only during commercial breaks. Seal yourself away in some solitary 6
12 3456789012345678901234567890121234567890123456789012345678901212345 6
2 3456789012345678901234567890121234567890123456789012345678901212345
location with a heap of scrap paper and a handful of number 2 pencils as your companions. 6
123456789012345678901234567890121234567890123456789012345678901212345
123456789012345678901234567890121234567890123456789012345678901212345
Give yourself 45 minutes for the Quantitative section and 30 minutes for the Verbal section. 6
6
1
123456789012345678901234567890121234567890123456789012345678901212345 6
123456789012345678901234567890121234567890123456789012345678901212345
Take both sections consecutively with a 1-minute break in between. Don’t rush, but try to 6
2 3456789012345678901234567890121234567890123456789012345678901212345
123456789012345678901234567890121234567890123456789012345678901212345 6
123456789012345678901234567890121234567890123456789012345678901212345
answer as many of the questions as you can. 6
123456789012345678901234567890121234567890123456789012345678901212345 66
123456789012345678901234567890121234567890123456789012345678901212345 6
1
12 3456789012345678901234567890121234567890123456789012345678901212345 6
12 3456789012345678901234567890121234567890123456789012345678901212345
You can write your answers in the book, but make sure to do all work on scratch 6
12 3456789012345678901234567890121234567890123456789012345678901212345 6
Tip

12 3456789012345678901234567890121234567890123456789012345678901212345 6
paper since that is what you have to do for the GRE CAT. Afterward, check to see
123456789012345678901234567890121234567890123456789012345678901212345
how much scratch paper you used on each section. This will help give you an idea 6
12 3456789012345678901234567890121234567890123456789012345678901212345 6
12 3456789012345678901234567890121234567890123456789012345678901212345
of how much you’ll need on the real GRE. 6
12 3456789012345678901234567890121234567890123456789012345678901212345 6
12 3456789012345678901234567890121234567890123456789012345678901212345 6
123456789012345678901234567890121234567890123456789012345678901212345 6
123456789012345678901234567890121234567890123456789012345678901212345
2 Once finished, give yourself a pat of the back. You will have just finished a GRE-like test. It 6
1 3456789012345678901234567890121234567890123456789012345678901212345 6
12 3456789012345678901234567890121234567890123456789012345678901212345
wasn’t a computer adaptive test, but it was a GRE-like test nevertheless. 6
123456789012345678901234567890121234567890123456789012345678901212345 6
12 3456789012345678901234567890121234567890123456789012345678901212345 6
123456789012345678901234567890121234567890123456789012345678901212345
Good luck! 6
123456789012345678901234567890121234567890123456789012345678901212345 6
123456789012345678901234567890121234567890123456789012345678901212345 6
123456789012345678901234567890121234567890123456789012345678901212345 6
34 1234567890123456789012345678901212345678901234567890123456789012123456

www.petersons.com
DIAGNOSTIC
1234567890123456789012345678901212345678901234567890123456789012123456
123456789012345678901234567890121234567890123456789012345678901212345 6
12 3456789012345678901234567890121234567890123456789012345678901212345 6
123456789012345678901234567890121234567890123456789012345678901212345
The Test 6
12 3456789012345678901234567890121234567890123456789012345678901212345 6
12 3456789012345678901234567890121234567890123456789012345678901212345 6
123456789012345678901234567890121234567890123456789012345678901212345
Quantitative 6
12
2
3456789012345678901234567890121234567890123456789012345678901212345
3456789012345678901234567890121234567890123456789012345678901212345
6
6
1
12 3456789012345678901234567890121234567890123456789012345678901212345
General Information 6
12 3456789012345678901234567890121234567890123456789012345678901212345 6
12 3456789012345678901234567890121234567890123456789012345678901212345 6
12
2
3456789012345678901234567890121234567890123456789012345678901212345
1. The test has 28 questions. You have 45 minutes to complete the section.
3456789012345678901234567890121234567890123456789012345678901212345
6
6
1
12 3456789012345678901234567890121234567890123456789012345678901212345
2. All numbers used are real numbers. 6
12 3456789012345678901234567890121234567890123456789012345678901212345 6
12 3456789012345678901234567890121234567890123456789012345678901212345 6
12
2
3456789012345678901234567890121234567890123456789012345678901212345
3. All angle measurements can be assumed to be positive unless otherwise noted.
3456789012345678901234567890121234567890123456789012345678901212345
6
6
1
12 3456789012345678901234567890121234567890123456789012345678901212345 6
12 3456789012345678901234567890121234567890123456789012345678901212345 6
4. All figures lie in the same plane unless otherwise noted.
12 3456789012345678901234567890121234567890123456789012345678901212345 6
12 3456789012345678901234567890121234567890123456789012345678901212345
5. Drawings that accompany questions are intended to provide information useful in 6
1 3456789012345678901234567890121234567890123456789012345678901212345
2 6
12 3456789012345678901234567890121234567890123456789012345678901212345 6
12 3456789012345678901234567890121234567890123456789012345678901212345 6
answering the question. The figures are drawn closely to scale unless otherwise noted.
12 3456789012345678901234567890121234567890123456789012345678901212345 6
12 3456789012345678901234567890121234567890123456789012345678901212345 6
123456789012345678901234567890121234567890123456789012345678901212345
2 6
123456789012345678901234567890121234567890123456789012345678901212345
Directions: For questions 1–14, Column A and Column B will have one quantity each. 6
123456789012345678901234567890121234567890123456789012345678901212345 6
123456789012345678901234567890121234567890123456789012345678901212345
Your goal is to compare the quantities in each column and choose 6
123456789012345678901234567890121234567890123456789012345678901212345 6
3456789012345678901234567890121234567890123456789012345678901212345
123456789012345678901234567890121234567890123456789012345678901212345
A if the quantity in Column A is greater 6
1 6
12 3456789012345678901234567890121234567890123456789012345678901212345
B if the quantity in Column B is greater 6
12 3456789012345678901234567890121234567890123456789012345678901212345 6
12 3456789012345678901234567890121234567890123456789012345678901212345
C if the two quantities are equal 6
12 3456789012345678901234567890121234567890123456789012345678901212345 6
12 3456789012345678901234567890121234567890123456789012345678901212345
D if the relationship between the two quantities cannot be determined from the 6
12 3456789012345678901234567890121234567890123456789012345678901212345 6
12 3456789012345678901234567890121234567890123456789012345678901212345
information in the problem 6
12 3456789012345678901234567890121234567890123456789012345678901212345 6
2 3456789012345678901234567890121234567890123456789012345678901212345
123456789012345678901234567890121234567890123456789012345678901212345 6
123456789012345678901234567890121234567890123456789012345678901212345
On some questions, information about one or both quantities will be given. This 6
123456789012345678901234567890121234567890123456789012345678901212345 6
123456789012345678901234567890121234567890123456789012345678901212345 66
information will be centered above the two columns.
1
12 3456789012345678901234567890121234567890123456789012345678901212345 6
12 3456789012345678901234567890121234567890123456789012345678901212345 6
123456789012345678901234567890121234567890123456789012345678901212345 6
123456789012345678901234567890121234567890123456789012345678901212345 6
12 3456789012345678901234567890121234567890123456789012345678901212345
Column A Column B Column A Column B 6
123456789012345678901234567890121234567890123456789012345678901212345 66
2 3456789012345678901234567890121234567890123456789012345678901212345
123456789012345678901234567890121234567890123456789012345678901212345 6
1 1. 4 692
12 3456789012345678901234567890121234567890123456789012345678901212345 6
12 3456789012345678901234567890121234567890123456789012345678901212345 6
15 1,781
123456789012345678901234567890121234567890123456789012345678901212345 66
2 3456789012345678901234567890121234567890123456789012345678901212345
123456789012345678901234567890121234567890123456789012345678901212345 6
1 3.
123456789012345678901234567890121234567890123456789012345678901212345 6
123456789012345678901234567890121234567890123456789012345678901212345 6
12 3456789012345678901234567890121234567890123456789012345678901212345 6
12 3456789012345678901234567890121234567890123456789012345678901212345 6
12 3456789012345678901234567890121234567890123456789012345678901212345 6
12 3456789012345678901234567890121234567890123456789012345678901212345 6
123456789012345678901234567890121234567890123456789012345678901212345 6
123456789012345678901234567890121234567890123456789012345678901212345
2 a2 (c2b) 6
123456789012345678901234567890121234567890123456789012345678901212345 6
123456789012345678901234567890121234567890123456789012345678901212345
= = 6
123456789012345678901234567890121234567890123456789012345678901212345
2. 40 1 9 7 1 63 6
1 3456789012345678901234567890121234567890123456789012345678901212345 6
123456789012345678901234567890121234567890123456789012345678901212345 66
2 3456789012345678901234567890121234567890123456789012345678901212345
123456789012345678901234567890121234567890123456789012345678901212345 6
123456789012345678901234567890121234567890123456789012345678901212345 6
123456789012345678901234567890121234567890123456789012345678901212345 6
1
12 3456789012345678901234567890121234567890123456789012345678901212345 6
123456789012345678901234567890121234567890123456789012345678901212345 6
12 3456789012345678901234567890121234567890123456789012345678901212345 6
123456789012345678901234567890121234567890123456789012345678901212345 6
12 3456789012345678901234567890121234567890123456789012345678901212345 6
123456789012345678901234567890121234567890123456789012345678901212345 6
123456789012345678901234567890121234567890123456789012345678901212345 6
123456789012345678901234567890121234567890123456789012345678901212345 6
123456789012345678901234567890121234567890123456789012345678901212345 6 35
1234567890123456789012345678901212345678901234567890123456789012123456
Part II: Diagnosing Strengths and Weaknesses

1234567890123456789012345678901212345678901234567890123456789012123456
123456789012345678901234567890121234567890123456789012345678901212345 6
12 3456789012345678901234567890121234567890123456789012345678901212345 6
123456789012345678901234567890121234567890123456789012345678901212345
Column A Column B Column A Column B 6
12 3456789012345678901234567890121234567890123456789012345678901212345 6
12 3456789012345678901234567890121234567890123456789012345678901212345 6
123456789012345678901234567890121234567890123456789012345678901212345
1
@ 50 1 2~2 1 3! #2 8. u5x y 2
xÞ0Þy 6
12
2
3456789012345678901234567890121234567890123456789012345678901212345
4. @4~2 1 3! #2 2
3456789012345678901234567890121234567890123456789012345678901212345
6
6
1 10
12 3456789012345678901234567890121234567890123456789012345678901212345 x 6
12 3456789012345678901234567890121234567890123456789012345678901212345
2
u 5x y 4 2 3
xu 6
12 3456789012345678901234567890121234567890123456789012345678901212345 y 6
12
2
3456789012345678901234567890121234567890123456789012345678901212345 6
3456789012345678901234567890121234567890123456789012345678901212345 6
1
12 3456789012345678901234567890121234567890123456789012345678901212345 6
12 3456789012345678901234567890121234567890123456789012345678901212345 6
12 3456789012345678901234567890121234567890123456789012345678901212345 6
12 3456789012345678901234567890121234567890123456789012345678901212345 6
1 3456789012345678901234567890121234567890123456789012345678901212345 6
2
12 3456789012345678901234567890121234567890123456789012345678901212345 6
12 3456789012345678901234567890121234567890123456789012345678901212345 6
12 3456789012345678901234567890121234567890123456789012345678901212345 6
12
2
3456789012345678901234567890121234567890123456789012345678901212345
5. A circle has a radius q.
3456789012345678901234567890121234567890123456789012345678901212345
6
6
1
12 3456789012345678901234567890121234567890123456789012345678901212345
Twice the area of The circumference 9. x is a positive integer. 6
12 3456789012345678901234567890121234567890123456789012345678901212345 6
12 3456789012345678901234567890121234567890123456789012345678901212345
the circle divided of the circle 6
12 3456789012345678901234567890121234567890123456789012345678901212345
(x2 2 3x 1 2) x2 2 4 6
123456789012345678901234567890121234567890123456789012345678901212345 66
2 3456789012345678901234567890121234567890123456789012345678901212345
by the radius
123456789012345678901234567890121234567890123456789012345678901212345 6
123456789012345678901234567890121234567890123456789012345678901212345 6
123456789012345678901234567890121234567890123456789012345678901212345 6
123456789012345678901234567890121234567890123456789012345678901212345 6
123456789012345678901234567890121234567890123456789012345678901212345 6
1
12 3456789012345678901234567890121234567890123456789012345678901212345 6
12 3456789012345678901234567890121234567890123456789012345678901212345 6
12 3456789012345678901234567890121234567890123456789012345678901212345 6
123456789012345678901234567890121234567890123456789012345678901212345
2 6
123456789012345678901234567890121234567890123456789012345678901212345 6
123456789012345678901234567890121234567890123456789012345678901212345 6
123456789012345678901234567890121234567890123456789012345678901212345
6. The area of a The area of a 6
123456789012345678901234567890121234567890123456789012345678901212345 6
3456789012345678901234567890121234567890123456789012345678901212345
123456789012345678901234567890121234567890123456789012345678901212345
square with a rectangle with a 6
123456789012345678901234567890121234567890123456789012345678901212345 6
123456789012345678901234567890121234567890123456789012345678901212345
perimeter of 20 perimeter 20 6
123456789012345678901234567890121234567890123456789012345678901212345 66
1
12 3456789012345678901234567890121234567890123456789012345678901212345 6
12 3456789012345678901234567890121234567890123456789012345678901212345 6
123456789012345678901234567890121234567890123456789012345678901212345 6
123456789012345678901234567890121234567890123456789012345678901212345 6
12 3456789012345678901234567890121234567890123456789012345678901212345 6
12 3456789012345678901234567890121234567890123456789012345678901212345 6
12 3456789012345678901234567890121234567890123456789012345678901212345 6
123456789012345678901234567890121234567890123456789012345678901212345 6
12 3456789012345678901234567890121234567890123456789012345678901212345
10. 6
12 3456789012345678901234567890121234567890123456789012345678901212345 6
2 3456789012345678901234567890121234567890123456789012345678901212345
123456789012345678901234567890121234567890123456789012345678901212345 6
123456789012345678901234567890121234567890123456789012345678901212345
7.
1 1 1 2 3 1 2 1 1 2 6
1 2 1 1 2 2 1 1 2 1 6
123456789012345678901234567890121234567890123456789012345678901212345 6
3 2 7 5 4 4 5 7 5 3
123456789012345678901234567890121234567890123456789012345678901212345 6
12 3456789012345678901234567890121234567890123456789012345678901212345 6
12 3456789012345678901234567890121234567890123456789012345678901212345 6
12 3456789012345678901234567890121234567890123456789012345678901212345 6
12 3456789012345678901234567890121234567890123456789012345678901212345 6
123456789012345678901234567890121234567890123456789012345678901212345 6
123456789012345678901234567890121234567890123456789012345678901212345
2 6
123456789012345678901234567890121234567890123456789012345678901212345 6
123456789012345678901234567890121234567890123456789012345678901212345
Point C is at the center of the circle. 6
123456789012345678901234567890121234567890123456789012345678901212345 6
1 3456789012345678901234567890121234567890123456789012345678901212345 6 AB 5 DE
123456789012345678901234567890121234567890123456789012345678901212345 66
2 3456789012345678901234567890121234567890123456789012345678901212345
123456789012345678901234567890121234567890123456789012345678901212345
∠CBD ∠CEA 6
123456789012345678901234567890121234567890123456789012345678901212345
123456789012345678901234567890121234567890123456789012345678901212345 66
1
12 3456789012345678901234567890121234567890123456789012345678901212345 6
123456789012345678901234567890121234567890123456789012345678901212345 6
12 3456789012345678901234567890121234567890123456789012345678901212345 6
123456789012345678901234567890121234567890123456789012345678901212345 6
12 3456789012345678901234567890121234567890123456789012345678901212345 6
123456789012345678901234567890121234567890123456789012345678901212345 6
123456789012345678901234567890121234567890123456789012345678901212345 6
123456789012345678901234567890121234567890123456789012345678901212345 6
123456789012345678901234567890121234567890123456789012345678901212345 6
36 1234567890123456789012345678901212345678901234567890123456789012123456

www.petersons.com
DIAGNOSTIC
1234567890123456789012345678901212345678901234567890123456789012123456
123456789012345678901234567890121234567890123456789012345678901212345 6
12 3456789012345678901234567890121234567890123456789012345678901212345 6
123456789012345678901234567890121234567890123456789012345678901212345
Column A Column B 6
12 3456789012345678901234567890121234567890123456789012345678901212345 6
12 3456789012345678901234567890121234567890123456789012345678901212345
Directions: Select the best answer for each 6
123456789012345678901234567890121234567890123456789012345678901212345
11. The sum of all even The sum of all odd of the following questions. 6
12
2
3456789012345678901234567890121234567890123456789012345678901212345
3456789012345678901234567890121234567890123456789012345678901212345
6
6
1 integers one integers one
12 3456789012345678901234567890121234567890123456789012345678901212345 6
12 3456789012345678901234567890121234567890123456789012345678901212345
through one through one 6
12 3456789012345678901234567890121234567890123456789012345678901212345
15. There are 13 marbles in a bag. One is red, 6
12
2
3456789012345678901234567890121234567890123456789012345678901212345
hundred hundred
3456789012345678901234567890121234567890123456789012345678901212345
6
6
1 4 are blue, and 8 are yellow. If a blue
12 3456789012345678901234567890121234567890123456789012345678901212345 6
12 3456789012345678901234567890121234567890123456789012345678901212345 6
marble is drawn from the bag, what is the
12 3456789012345678901234567890121234567890123456789012345678901212345 6
12
2
3456789012345678901234567890121234567890123456789012345678901212345
probability that a red or yellow marble
3456789012345678901234567890121234567890123456789012345678901212345
6
6
1
12 3456789012345678901234567890121234567890123456789012345678901212345
will be drawn next? 6
12 3456789012345678901234567890121234567890123456789012345678901212345 6
12 3456789012345678901234567890121234567890123456789012345678901212345
1 6
12
2
3456789012345678901234567890121234567890123456789012345678901212345
A.
3456789012345678901234567890121234567890123456789012345678901212345
6
6
1 4
12 3456789012345678901234567890121234567890123456789012345678901212345 6
12 3456789012345678901234567890121234567890123456789012345678901212345
12. The cube of an The square of an odd 6
12 3456789012345678901234567890121234567890123456789012345678901212345
2 6
12 3456789012345678901234567890121234567890123456789012345678901212345
even prime number prime number B. 6
123456789012345678901234567890121234567890123456789012345678901212345 66
2 3456789012345678901234567890121234567890123456789012345678901212345
3
123456789012345678901234567890121234567890123456789012345678901212345
123456789012345678901234567890121234567890123456789012345678901212345
8 6
123456789012345678901234567890121234567890123456789012345678901212345
C. 6
123456789012345678901234567890121234567890123456789012345678901212345
13 6
123456789012345678901234567890121234567890123456789012345678901212345 66
1
12 3456789012345678901234567890121234567890123456789012345678901212345
3 6
12 3456789012345678901234567890121234567890123456789012345678901212345
D. 6
12 3456789012345678901234567890121234567890123456789012345678901212345 6
4
2 3456789012345678901234567890121234567890123456789012345678901212345
123456789012345678901234567890121234567890123456789012345678901212345 6
123456789012345678901234567890121234567890123456789012345678901212345
9 6
123456789012345678901234567890121234567890123456789012345678901212345
13. x,0 E. 6
123456789012345678901234567890121234567890123456789012345678901212345
13 6
123456789012345678901234567890121234567890123456789012345678901212345
2 6
123456789012345678901234567890121234567890123456789012345678901212345
x 12x 4 6
2
2 2 x
123456789012345678901234567890121234567890123456789012345678901212345
16. Bananas at Max’s Grocer went on sale for 6
123456789012345678901234567890121234567890123456789012345678901212345
2x 2 3 2 x 1 1 6
123456789012345678901234567890121234567890123456789012345678901212345
50 percent off. They were then marked 6
1 6
123456789012345678901234567890121234567890123456789012345678901212345 66
2 3456789012345678901234567890121234567890123456789012345678901212345
down again by 30 percent. If x is the
123456789012345678901234567890121234567890123456789012345678901212345
original price of bananas per pound, how 6
1
123456789012345678901234567890121234567890123456789012345678901212345 6
12
2
3456789012345678901234567890121234567890123456789012345678901212345
much does it cost to buy 2.5 pounds of
3456789012345678901234567890121234567890123456789012345678901212345
6
6
123456789012345678901234567890121234567890123456789012345678901212345
123456789012345678901234567890121234567890123456789012345678901212345
bananas? 6
1 6
12 3456789012345678901234567890121234567890123456789012345678901212345 6
12 3456789012345678901234567890121234567890123456789012345678901212345 6
14. xµ 5 (x)(x 1 1) A. 0.15x
2 3456789012345678901234567890121234567890123456789012345678901212345
123456789012345678901234567890121234567890123456789012345678901212345 6
123456789012345678901234567890121234567890123456789012345678901212345
B. 0.375x 6
1 (3µ)µ (3µ)(3µ) 6
123456789012345678901234567890121234567890123456789012345678901212345 6
C. 0.42x
123456789012345678901234567890121234567890123456789012345678901212345
D. 0.50x 6
12 3456789012345678901234567890121234567890123456789012345678901212345 6
12 3456789012345678901234567890121234567890123456789012345678901212345
E. 0.875x 6
12 3456789012345678901234567890121234567890123456789012345678901212345 6
12 3456789012345678901234567890121234567890123456789012345678901212345 6
123456789012345678901234567890121234567890123456789012345678901212345 6
123456789012345678901234567890121234567890123456789012345678901212345
2 6
123456789012345678901234567890121234567890123456789012345678901212345 6
123456789012345678901234567890121234567890123456789012345678901212345 6
123456789012345678901234567890121234567890123456789012345678901212345 6
1 3456789012345678901234567890121234567890123456789012345678901212345 6
123456789012345678901234567890121234567890123456789012345678901212345 66
2 3456789012345678901234567890121234567890123456789012345678901212345
123456789012345678901234567890121234567890123456789012345678901212345 6
123456789012345678901234567890121234567890123456789012345678901212345 6
123456789012345678901234567890121234567890123456789012345678901212345 6
1
12 3456789012345678901234567890121234567890123456789012345678901212345 6
123456789012345678901234567890121234567890123456789012345678901212345 6
12 3456789012345678901234567890121234567890123456789012345678901212345 6
123456789012345678901234567890121234567890123456789012345678901212345 6
12 3456789012345678901234567890121234567890123456789012345678901212345 6
123456789012345678901234567890121234567890123456789012345678901212345 6
123456789012345678901234567890121234567890123456789012345678901212345 6
123456789012345678901234567890121234567890123456789012345678901212345 6
123456789012345678901234567890121234567890123456789012345678901212345 6 37
1234567890123456789012345678901212345678901234567890123456789012123456
Part II: Diagnosing Strengths and Weaknesses

1234567890123456789012345678901212345678901234567890123456789012123456
123456789012345678901234567890121234567890123456789012345678901212345 6
12 3456789012345678901234567890121234567890123456789012345678901212345 6
123456789012345678901234567890121234567890123456789012345678901212345
17. Questions 20–21 refer to the following graphs. 6
12 3456789012345678901234567890121234567890123456789012345678901212345 6
12 3456789012345678901234567890121234567890123456789012345678901212345 6
123456789012345678901234567890121234567890123456789012345678901212345 6
12 3456789012345678901234567890121234567890123456789012345678901212345 6
123456789012345678901234567890121234567890123456789012345678901212345 6
12 3456789012345678901234567890121234567890123456789012345678901212345 6
12 3456789012345678901234567890121234567890123456789012345678901212345 6
12 3456789012345678901234567890121234567890123456789012345678901212345 6
12 3456789012345678901234567890121234567890123456789012345678901212345 6
1 3456789012345678901234567890121234567890123456789012345678901212345 6
2
12 3456789012345678901234567890121234567890123456789012345678901212345 6
12 3456789012345678901234567890121234567890123456789012345678901212345 6
12 3456789012345678901234567890121234567890123456789012345678901212345 6
12
2
3456789012345678901234567890121234567890123456789012345678901212345
If BE bisects AC, what is the length of
3456789012345678901234567890121234567890123456789012345678901212345
6
6
1
12 3456789012345678901234567890121234567890123456789012345678901212345
EC? 6
12 3456789012345678901234567890121234567890123456789012345678901212345 6
12 3456789012345678901234567890121234567890123456789012345678901212345 6
12 3456789012345678901234567890121234567890123456789012345678901212345
A. 2=3 6
1 3456789012345678901234567890121234567890123456789012345678901212345
2 6
12 3456789012345678901234567890121234567890123456789012345678901212345 6
12 3456789012345678901234567890121234567890123456789012345678901212345 6
B. 4
12 3456789012345678901234567890121234567890123456789012345678901212345
C. 4=3 6
12 3456789012345678901234567890121234567890123456789012345678901212345 6
123456789012345678901234567890121234567890123456789012345678901212345 66
2 3456789012345678901234567890121234567890123456789012345678901212345
123456789012345678901234567890121234567890123456789012345678901212345
D. 8
123456789012345678901234567890121234567890123456789012345678901212345 6
123456789012345678901234567890121234567890123456789012345678901212345
E. 8=3 6
123456789012345678901234567890121234567890123456789012345678901212345 66
123456789012345678901234567890121234567890123456789012345678901212345 6
1 18. If x is an odd integer, in which one of the
12 3456789012345678901234567890121234567890123456789012345678901212345 6
12 3456789012345678901234567890121234567890123456789012345678901212345 6
12 3456789012345678901234567890121234567890123456789012345678901212345
following expressions must y be even? 6
123456789012345678901234567890121234567890123456789012345678901212345 66
2 3456789012345678901234567890121234567890123456789012345678901212345
123456789012345678901234567890121234567890123456789012345678901212345
A. x 1 y 5 12
123456789012345678901234567890121234567890123456789012345678901212345 6
123456789012345678901234567890121234567890123456789012345678901212345
B. x2 1 y 5 16
20. Which of the following has the greatest 6
123456789012345678901234567890121234567890123456789012345678901212345
value? 6
123456789012345678901234567890121234567890123456789012345678901212345
C. xy 5 25 6
123456789012345678901234567890121234567890123456789012345678901212345 6
123456789012345678901234567890121234567890123456789012345678901212345
2
D. xy 5 49 A. Segment of Manchester population 6
123456789012345678901234567890121234567890123456789012345678901212345 6
1 E. xy 5 98 0–25 in 1920 6
2 3456789012345678901234567890121234567890123456789012345678901212345
123456789012345678901234567890121234567890123456789012345678901212345 6
123456789012345678901234567890121234567890123456789012345678901212345
B. Byson population in 1900 6
1 19. Export Auto charges $30 for an oil change 6
123456789012345678901234567890121234567890123456789012345678901212345
C. Byson population in 1915 6
12 3456789012345678901234567890121234567890123456789012345678901212345 6
123456789012345678901234567890121234567890123456789012345678901212345 66
2 3456789012345678901234567890121234567890123456789012345678901212345
and $40 for a brake job. On Tuesday, they D. Segment of Manchester population
123456789012345678901234567890121234567890123456789012345678901212345 6
1 charged a total of $320 for all oil changes 25–50 in 1920
12 3456789012345678901234567890121234567890123456789012345678901212345 6
12 3456789012345678901234567890121234567890123456789012345678901212345 6
and brake checks. They performed a fourth E. Segment of Manchester population
2 3456789012345678901234567890121234567890123456789012345678901212345
123456789012345678901234567890121234567890123456789012345678901212345 6
123456789012345678901234567890121234567890123456789012345678901212345
as many brake checks as oil changes. How 50–100 in 1920 6
1 6
123456789012345678901234567890121234567890123456789012345678901212345 6
many oil changes did they perform on
123456789012345678901234567890121234567890123456789012345678901212345
21. Which town in which decade had the 6
12 3456789012345678901234567890121234567890123456789012345678901212345
Tuesday? 6
12 3456789012345678901234567890121234567890123456789012345678901212345
greatest percentage increase in population? 6
12 3456789012345678901234567890121234567890123456789012345678901212345
A. 2 6
12 3456789012345678901234567890121234567890123456789012345678901212345 6
123456789012345678901234567890121234567890123456789012345678901212345
B. 4 A. Manchester 1900–1910 6
2 3456789012345678901234567890121234567890123456789012345678901212345
123456789012345678901234567890121234567890123456789012345678901212345 6
123456789012345678901234567890121234567890123456789012345678901212345
C. 6 B. Ledbetter 1920–1930 6
123456789012345678901234567890121234567890123456789012345678901212345 6
123456789012345678901234567890121234567890123456789012345678901212345
D. 8 C. Ledbetter 1930–1940 6
1 6
123456789012345678901234567890121234567890123456789012345678901212345 66
2 3456789012345678901234567890121234567890123456789012345678901212345
E. 10 D. Byson 1920–1930
123456789012345678901234567890121234567890123456789012345678901212345 6
123456789012345678901234567890121234567890123456789012345678901212345
E. Manchester 1930–1940
123456789012345678901234567890121234567890123456789012345678901212345 66
1
12 3456789012345678901234567890121234567890123456789012345678901212345 6
123456789012345678901234567890121234567890123456789012345678901212345 6
12 3456789012345678901234567890121234567890123456789012345678901212345 6
123456789012345678901234567890121234567890123456789012345678901212345 6
12 3456789012345678901234567890121234567890123456789012345678901212345 6
123456789012345678901234567890121234567890123456789012345678901212345 6
123456789012345678901234567890121234567890123456789012345678901212345 6
123456789012345678901234567890121234567890123456789012345678901212345 6
123456789012345678901234567890121234567890123456789012345678901212345 6
38 1234567890123456789012345678901212345678901234567890123456789012123456

www.petersons.com
DIAGNOSTIC
1234567890123456789012345678901212345678901234567890123456789012123456
123456789012345678901234567890121234567890123456789012345678901212345 6
12
2
3456789012345678901234567890121234567890123456789012345678901212345
@~6a!a 2 ~4a 1 7a!#
3456789012345678901234567890121234567890123456789012345678901212345
6
6
1 22. 2 7a 1 5 25. If x is an odd integer, what is the value of half
12 3456789012345678901234567890121234567890123456789012345678901212345 6
J
a
12 3456789012345678901234567890121234567890123456789012345678901212345
the sum of x and the next three odd integers? 6
123456789012345678901234567890121234567890123456789012345678901212345
2
6a 2 18a 6
12
2
3456789012345678901234567890121234567890123456789012345678901212345
3456789012345678901234567890121234567890123456789012345678901212345
6
6
1 A. 1 J A. 2(x 1 3)
12 3456789012345678901234567890121234567890123456789012345678901212345
a 6
12 3456789012345678901234567890121234567890123456789012345678901212345
B. 3(x 1 3) 6
12 3456789012345678901234567890121234567890123456789012345678901212345 6
2
6a 2 4a
12
2
3456789012345678901234567890121234567890123456789012345678901212345
B. 1 J C. 4(x 1 3)
3456789012345678901234567890121234567890123456789012345678901212345
6
6
1 a
12 3456789012345678901234567890121234567890123456789012345678901212345
D. 5(x 1 3) 6
12 3456789012345678901234567890121234567890123456789012345678901212345
C. 7 2 a 1 J 6
12 3456789012345678901234567890121234567890123456789012345678901212345 6
E. 6(x 1 3)
12 3456789012345678901234567890121234567890123456789012345678901212345 6
123456789012345678901234567890121234567890123456789012345678901212345
D. 211 2 a 1 J 6
12 3456789012345678901234567890121234567890123456789012345678901212345 6
12 3456789012345678901234567890121234567890123456789012345678901212345
E. 13a 2 11 1 J Questions 26–28 refer to the following graphs. 6
12 3456789012345678901234567890121234567890123456789012345678901212345 6
12 3456789012345678901234567890121234567890123456789012345678901212345 6
1 3456789012345678901234567890121234567890123456789012345678901212345
2 6
12 3456789012345678901234567890121234567890123456789012345678901212345
23. 6
12 3456789012345678901234567890121234567890123456789012345678901212345 6
12 3456789012345678901234567890121234567890123456789012345678901212345 6
12 3456789012345678901234567890121234567890123456789012345678901212345 6
123456789012345678901234567890121234567890123456789012345678901212345
2 6
123456789012345678901234567890121234567890123456789012345678901212345 6
123456789012345678901234567890121234567890123456789012345678901212345 6
123456789012345678901234567890121234567890123456789012345678901212345 6
123456789012345678901234567890121234567890123456789012345678901212345 6
123456789012345678901234567890121234567890123456789012345678901212345 66
3456789012345678901234567890121234567890123456789012345678901212345
1
12 3456789012345678901234567890121234567890123456789012345678901212345 6
12 3456789012345678901234567890121234567890123456789012345678901212345
B is the midpoint of AC, and E is the 6
12 3456789012345678901234567890121234567890123456789012345678901212345 6
2 3456789012345678901234567890121234567890123456789012345678901212345
123456789012345678901234567890121234567890123456789012345678901212345
midpoint of FD. If the area of rectangle 6
123456789012345678901234567890121234567890123456789012345678901212345 6
123456789012345678901234567890121234567890123456789012345678901212345
BCDE is 12, what is the perimeter of 6
123456789012345678901234567890121234567890123456789012345678901212345 66
123456789012345678901234567890121234567890123456789012345678901212345
rectangle ACDF?
123456789012345678901234567890121234567890123456789012345678901212345 66
123456789012345678901234567890121234567890123456789012345678901212345
123456789012345678901234567890121234567890123456789012345678901212345
A. 18 6
123456789012345678901234567890121234567890123456789012345678901212345 6
1 B. 20 6
2 3456789012345678901234567890121234567890123456789012345678901212345
123456789012345678901234567890121234567890123456789012345678901212345 6
123456789012345678901234567890121234567890123456789012345678901212345
C. 22 6
1 6
123456789012345678901234567890121234567890123456789012345678901212345 6
D. 24
12 3456789012345678901234567890121234567890123456789012345678901212345 6
12 3456789012345678901234567890121234567890123456789012345678901212345
E. 26 6
12 3456789012345678901234567890121234567890123456789012345678901212345 6
123456789012345678901234567890121234567890123456789012345678901212345 6
12 3456789012345678901234567890121234567890123456789012345678901212345
24. 6
12 3456789012345678901234567890121234567890123456789012345678901212345 6
123456789012345678901234567890121234567890123456789012345678901212345 66
2 3456789012345678901234567890121234567890123456789012345678901212345
123456789012345678901234567890121234567890123456789012345678901212345 6
1
123456789012345678901234567890121234567890123456789012345678901212345 6
123456789012345678901234567890121234567890123456789012345678901212345 6
12 3456789012345678901234567890121234567890123456789012345678901212345 6
12 3456789012345678901234567890121234567890123456789012345678901212345 6
12 3456789012345678901234567890121234567890123456789012345678901212345 6
12 3456789012345678901234567890121234567890123456789012345678901212345
What is the perimeter of DABC? 6
123456789012345678901234567890121234567890123456789012345678901212345 6
2 3456789012345678901234567890121234567890123456789012345678901212345
123456789012345678901234567890121234567890123456789012345678901212345 6
123456789012345678901234567890121234567890123456789012345678901212345
A. 2 1 =2 6
123456789012345678901234567890121234567890123456789012345678901212345 66
123456789012345678901234567890121234567890123456789012345678901212345
B. 2 1 2=3 6
1
12 3456789012345678901234567890121234567890123456789012345678901212345 6
12 3456789012345678901234567890121234567890123456789012345678901212345
= 15 20 25 30 35 40 45 6
12 3456789012345678901234567890121234567890123456789012345678901212345 6
C. 4 1 2 2
12 3456789012345678901234567890121234567890123456789012345678901212345 6
123456789012345678901234567890121234567890123456789012345678901212345
D. 4 1 2=3 6
123456789012345678901234567890121234567890123456789012345678901212345
2 6
1 3456789012345678901234567890121234567890123456789012345678901212345 6
12 3456789012345678901234567890121234567890123456789012345678901212345
E. 4 1 4=3 6
123456789012345678901234567890121234567890123456789012345678901212345 6
12 3456789012345678901234567890121234567890123456789012345678901212345 6
123456789012345678901234567890121234567890123456789012345678901212345 6
123456789012345678901234567890121234567890123456789012345678901212345 6
123456789012345678901234567890121234567890123456789012345678901212345 6
123456789012345678901234567890121234567890123456789012345678901212345 6 39
1234567890123456789012345678901212345678901234567890123456789012123456
Part II: Diagnosing Strengths and Weaknesses

1234567890123456789012345678901212345678901234567890123456789012123456
123456789012345678901234567890121234567890123456789012345678901212345 6
12 3456789012345678901234567890121234567890123456789012345678901212345 6
123456789012345678901234567890121234567890123456789012345678901212345
26. Whose annual savings/annual spending 28. Over what 5-year span did percentage of 6
12 3456789012345678901234567890121234567890123456789012345678901212345 6
12 3456789012345678901234567890121234567890123456789012345678901212345
ratio was the smallest? spending increase the most? 6
123456789012345678901234567890121234567890123456789012345678901212345 6
12 3456789012345678901234567890121234567890123456789012345678901212345 6
123456789012345678901234567890121234567890123456789012345678901212345
A. 15 A. 15–20 6
12 3456789012345678901234567890121234567890123456789012345678901212345 6
12 3456789012345678901234567890121234567890123456789012345678901212345 6
B. 20 B. 25–30
12 3456789012345678901234567890121234567890123456789012345678901212345 6
12
2
3456789012345678901234567890121234567890123456789012345678901212345
C. 30 C. 30–35
3456789012345678901234567890121234567890123456789012345678901212345
6
6
1
12 3456789012345678901234567890121234567890123456789012345678901212345
D. 35 D. 35–40 6
12 3456789012345678901234567890121234567890123456789012345678901212345 6
12 3456789012345678901234567890121234567890123456789012345678901212345 6
E. 45 E. 40–45
12 3456789012345678901234567890121234567890123456789012345678901212345 6
123456789012345678901234567890121234567890123456789012345678901212345 6
12 3456789012345678901234567890121234567890123456789012345678901212345
27. What category of sales had the greatest 6
12 3456789012345678901234567890121234567890123456789012345678901212345 6
12 3456789012345678901234567890121234567890123456789012345678901212345
amount of money spent on it if you only 6
12
2
3456789012345678901234567890121234567890123456789012345678901212345
3456789012345678901234567890121234567890123456789012345678901212345
6
6
1 consider 15-, 30-, and 45-year-olds?
12 3456789012345678901234567890121234567890123456789012345678901212345 6
12 3456789012345678901234567890121234567890123456789012345678901212345 6
12 3456789012345678901234567890121234567890123456789012345678901212345
A. Home Entertainment 6
12 3456789012345678901234567890121234567890123456789012345678901212345 6
123456789012345678901234567890121234567890123456789012345678901212345 66
2 3456789012345678901234567890121234567890123456789012345678901212345
B. Clothes
123456789012345678901234567890121234567890123456789012345678901212345
123456789012345678901234567890121234567890123456789012345678901212345
C. Lawn Care 6
123456789012345678901234567890121234567890123456789012345678901212345 6
123456789012345678901234567890121234567890123456789012345678901212345 66
D. Sporting Goods
123456789012345678901234567890121234567890123456789012345678901212345 6
1 E. Apparel
12 3456789012345678901234567890121234567890123456789012345678901212345 6
12 3456789012345678901234567890121234567890123456789012345678901212345 6
12 3456789012345678901234567890121234567890123456789012345678901212345 6
123456789012345678901234567890121234567890123456789012345678901212345
2 Verbal 6
123456789012345678901234567890121234567890123456789012345678901212345 6
123456789012345678901234567890121234567890123456789012345678901212345 6
123456789012345678901234567890121234567890123456789012345678901212345
General Information 6
123456789012345678901234567890121234567890123456789012345678901212345 6
123456789012345678901234567890121234567890123456789012345678901212345 66
3456789012345678901234567890121234567890123456789012345678901212345
1. The test has 30 questions. You have 30 minutes to complete the section.
123456789012345678901234567890121234567890123456789012345678901212345 6
123456789012345678901234567890121234567890123456789012345678901212345 6
123456789012345678901234567890121234567890123456789012345678901212345 6
1 Directions: Each question contains a word printed in capital letters, followed by five
12 3456789012345678901234567890121234567890123456789012345678901212345 6
12 3456789012345678901234567890121234567890123456789012345678901212345
answer choices. Choose the answer choice that contains the word or phrase most nearly 6
123456789012345678901234567890121234567890123456789012345678901212345 6
123456789012345678901234567890121234567890123456789012345678901212345
OPPOSITE in meaning to the word in capital letters. 6
12 3456789012345678901234567890121234567890123456789012345678901212345 6
123456789012345678901234567890121234567890123456789012345678901212345 66
2 3456789012345678901234567890121234567890123456789012345678901212345
123456789012345678901234567890121234567890123456789012345678901212345 6
1
12 3456789012345678901234567890121234567890123456789012345678901212345 6
12 3456789012345678901234567890121234567890123456789012345678901212345
1. PRISTINE : 3. LUCID : 6
123456789012345678901234567890121234567890123456789012345678901212345 66
2 3456789012345678901234567890121234567890123456789012345678901212345
123456789012345678901234567890121234567890123456789012345678901212345 6
1 A. dull A. translucent
123456789012345678901234567890121234567890123456789012345678901212345 6
123456789012345678901234567890121234567890123456789012345678901212345 6
B. hopeless B. myopic
2 3456789012345678901234567890121234567890123456789012345678901212345
123456789012345678901234567890121234567890123456789012345678901212345 6
123456789012345678901234567890121234567890123456789012345678901212345
C. congested C. putrid 6
123456789012345678901234567890121234567890123456789012345678901212345 6
123456789012345678901234567890121234567890123456789012345678901212345 66
D. majestic D. luminescent
1
12 3456789012345678901234567890121234567890123456789012345678901212345
E. polluted E. opaque 6
12 3456789012345678901234567890121234567890123456789012345678901212345 6
12 3456789012345678901234567890121234567890123456789012345678901212345 6
12 3456789012345678901234567890121234567890123456789012345678901212345
2. IMPREGNABLE : 4. MALLEABLE : 6
123456789012345678901234567890121234567890123456789012345678901212345 6
2 3456789012345678901234567890121234567890123456789012345678901212345
123456789012345678901234567890121234567890123456789012345678901212345 6
123456789012345678901234567890121234567890123456789012345678901212345
A. penetrable A. brittle 6
123456789012345678901234567890121234567890123456789012345678901212345 6
123456789012345678901234567890121234567890123456789012345678901212345 66
B. tangible B. ductile
1
12 3456789012345678901234567890121234567890123456789012345678901212345
C. intractable C. pliant 6
123456789012345678901234567890121234567890123456789012345678901212345 6
12 3456789012345678901234567890121234567890123456789012345678901212345 6
D. tolerable D. flaccid
123456789012345678901234567890121234567890123456789012345678901212345 6
12
2
3456789012345678901234567890121234567890123456789012345678901212345
E. reliable E. verdant
3456789012345678901234567890121234567890123456789012345678901212345
6
6
1
123456789012345678901234567890121234567890123456789012345678901212345 6
123456789012345678901234567890121234567890123456789012345678901212345 6
123456789012345678901234567890121234567890123456789012345678901212345 6
40 1234567890123456789012345678901212345678901234567890123456789012123456

www.petersons.com
DIAGNOSTIC
1234567890123456789012345678901212345678901234567890123456789012123456
123456789012345678901234567890121234567890123456789012345678901212345 6
12 3456789012345678901234567890121234567890123456789012345678901212345 6
123456789012345678901234567890121234567890123456789012345678901212345
5. CASTIGATE : 8. TRUCULENT : 6
12 3456789012345678901234567890121234567890123456789012345678901212345 6
12 3456789012345678901234567890121234567890123456789012345678901212345 6
123456789012345678901234567890121234567890123456789012345678901212345
A. criticize A. pugnacious 6
12
2
3456789012345678901234567890121234567890123456789012345678901212345
3456789012345678901234567890121234567890123456789012345678901212345
6
6
1 B. laud B. boorish
12 3456789012345678901234567890121234567890123456789012345678901212345 6
12 3456789012345678901234567890121234567890123456789012345678901212345 6
C. ignore C. gentle
12 3456789012345678901234567890121234567890123456789012345678901212345 6
12
2
3456789012345678901234567890121234567890123456789012345678901212345
D. mollify D. beneficent
3456789012345678901234567890121234567890123456789012345678901212345
6
6
1
12 3456789012345678901234567890121234567890123456789012345678901212345
E. obfuscate E. altruistic 6
12 3456789012345678901234567890121234567890123456789012345678901212345 6
12 3456789012345678901234567890121234567890123456789012345678901212345 6
12 3456789012345678901234567890121234567890123456789012345678901212345
6. APPROBATION : 9. SALUBRIOUS : 6
1 3456789012345678901234567890121234567890123456789012345678901212345
2 6
12 3456789012345678901234567890121234567890123456789012345678901212345 6
12 3456789012345678901234567890121234567890123456789012345678901212345
A. disapproval A. salutary 6
12 3456789012345678901234567890121234567890123456789012345678901212345 6
12
2
3456789012345678901234567890121234567890123456789012345678901212345 6
B. fanfare B. deleterious
3456789012345678901234567890121234567890123456789012345678901212345 6
1
12 3456789012345678901234567890121234567890123456789012345678901212345
C. calamity C. beneficial 6
12 3456789012345678901234567890121234567890123456789012345678901212345 6
12 3456789012345678901234567890121234567890123456789012345678901212345 6
D. enigma D. acrid
12 3456789012345678901234567890121234567890123456789012345678901212345 6
12 3456789012345678901234567890121234567890123456789012345678901212345
E. rejoinder E. ignominious 6
12 3456789012345678901234567890121234567890123456789012345678901212345 6
12 3456789012345678901234567890121234567890123456789012345678901212345 6
12 3456789012345678901234567890121234567890123456789012345678901212345
7. MENDACITY : 6
12 3456789012345678901234567890121234567890123456789012345678901212345 6
123456789012345678901234567890121234567890123456789012345678901212345 66
2 3456789012345678901234567890121234567890123456789012345678901212345
1 A. evil
12 3456789012345678901234567890121234567890123456789012345678901212345 6
12 3456789012345678901234567890121234567890123456789012345678901212345
B. wisdom 6
12 3456789012345678901234567890121234567890123456789012345678901212345 6
123456789012345678901234567890121234567890123456789012345678901212345 66
2 3456789012345678901234567890121234567890123456789012345678901212345
C. perfidy
123456789012345678901234567890121234567890123456789012345678901212345
123456789012345678901234567890121234567890123456789012345678901212345
D. honesty 6
123456789012345678901234567890121234567890123456789012345678901212345 6
123456789012345678901234567890121234567890123456789012345678901212345 66
E. irregularity
123456789012345678901234567890121234567890123456789012345678901212345 6
123456789012345678901234567890121234567890123456789012345678901212345 6
123456789012345678901234567890121234567890123456789012345678901212345 6
123456789012345678901234567890121234567890123456789012345678901212345 6
1 Directions: In each question, there is an initial pair of words or phrases that are related in
12 3456789012345678901234567890121234567890123456789012345678901212345 6
12 3456789012345678901234567890121234567890123456789012345678901212345 6
123456789012345678901234567890121234567890123456789012345678901212345
some manner. Select the answer choice containing the lettered pair that best expresses a 6
123456789012345678901234567890121234567890123456789012345678901212345 6
12 3456789012345678901234567890121234567890123456789012345678901212345 6
relationship similar to the initial pair.
123456789012345678901234567890121234567890123456789012345678901212345 66
2 3456789012345678901234567890121234567890123456789012345678901212345
123456789012345678901234567890121234567890123456789012345678901212345 6
1
12 3456789012345678901234567890121234567890123456789012345678901212345 6
12 3456789012345678901234567890121234567890123456789012345678901212345 6
123456789012345678901234567890121234567890123456789012345678901212345
2 10. POTTER : CLAY :: 12. HOMOSAPIEN : MAMMAL :: 6
123456789012345678901234567890121234567890123456789012345678901212345 6
1 3456789012345678901234567890121234567890123456789012345678901212345
A. bull : rider A. lung : organ 6
123456789012345678901234567890121234567890123456789012345678901212345 6
123456789012345678901234567890121234567890123456789012345678901212345
B. conductor : instrument B. beach : ocean 6
12 3456789012345678901234567890121234567890123456789012345678901212345 6
12 3456789012345678901234567890121234567890123456789012345678901212345
C. poet : words C. canine : feline 6
12 3456789012345678901234567890121234567890123456789012345678901212345 6
12 3456789012345678901234567890121234567890123456789012345678901212345
D. doctor : patient D. mystery : novel 6
123456789012345678901234567890121234567890123456789012345678901212345 6
123456789012345678901234567890121234567890123456789012345678901212345 66
2 3456789012345678901234567890121234567890123456789012345678901212345
E. lady : gentleman E. bovine : husbandry
123456789012345678901234567890121234567890123456789012345678901212345 6
123456789012345678901234567890121234567890123456789012345678901212345 6
123456789012345678901234567890121234567890123456789012345678901212345 6
1 11. MINISCULE : SMALL :: 13. IMMUTABLE : CHANGE ::
12 3456789012345678901234567890121234567890123456789012345678901212345 6
12 3456789012345678901234567890121234567890123456789012345678901212345
A. large : gigantic A. political : mastermind 6
12 3456789012345678901234567890121234567890123456789012345678901212345 6
12 3456789012345678901234567890121234567890123456789012345678901212345
B. exuberant : happy B. contingent : necessity 6
123456789012345678901234567890121234567890123456789012345678901212345 6
2 3456789012345678901234567890121234567890123456789012345678901212345
123456789012345678901234567890121234567890123456789012345678901212345
C. downtrodden : unjust C. ecstatic : joy 6
1 6
12 3456789012345678901234567890121234567890123456789012345678901212345
D. humorless : hapless D. imminent : proximity 6
123456789012345678901234567890121234567890123456789012345678901212345 6
12 3456789012345678901234567890121234567890123456789012345678901212345
E. tense : relaxed E. humble : hubris 6
123456789012345678901234567890121234567890123456789012345678901212345 6
123456789012345678901234567890121234567890123456789012345678901212345 6
123456789012345678901234567890121234567890123456789012345678901212345 6
123456789012345678901234567890121234567890123456789012345678901212345 6 41
1234567890123456789012345678901212345678901234567890123456789012123456
Part II: Diagnosing Strengths and Weaknesses

1234567890123456789012345678901212345678901234567890123456789012123456
123456789012345678901234567890121234567890123456789012345678901212345 6
12 3456789012345678901234567890121234567890123456789012345678901212345 6
123456789012345678901234567890121234567890123456789012345678901212345
14. VITUPERATE : BERATE :: 16. LACONIC : VERBOSE :: 6
12 3456789012345678901234567890121234567890123456789012345678901212345 6
12 3456789012345678901234567890121234567890123456789012345678901212345 6
123456789012345678901234567890121234567890123456789012345678901212345
A. destroy : rebuild A. terse : taciturn 6
12
2
3456789012345678901234567890121234567890123456789012345678901212345
3456789012345678901234567890121234567890123456789012345678901212345
6
6
1 B. vacillate : waver B. delicious : delectable
12 3456789012345678901234567890121234567890123456789012345678901212345 6
12 3456789012345678901234567890121234567890123456789012345678901212345 6
C. synthesize : raze C. passive : lethargic
12 3456789012345678901234567890121234567890123456789012345678901212345 6
12
2
3456789012345678901234567890121234567890123456789012345678901212345
D. confuse : clarify D. guileless : deceitful
3456789012345678901234567890121234567890123456789012345678901212345
6
6
1
12 3456789012345678901234567890121234567890123456789012345678901212345
E. pacify : enrage E. fastidious : studious 6
12 3456789012345678901234567890121234567890123456789012345678901212345 6
12 3456789012345678901234567890121234567890123456789012345678901212345 6
12 3456789012345678901234567890121234567890123456789012345678901212345
15. BUCOLIC : PASTORAL :: 17. MALEVOLENT : BONHOMIE :: 6
1 3456789012345678901234567890121234567890123456789012345678901212345
2 6
12 3456789012345678901234567890121234567890123456789012345678901212345 6
12 3456789012345678901234567890121234567890123456789012345678901212345
A. frigid : lukewarm A. hateful : disgust 6
12 3456789012345678901234567890121234567890123456789012345678901212345 6
12
2
3456789012345678901234567890121234567890123456789012345678901212345 6
B. basic : advanced B. calm
3456789012345678901234567890121234567890123456789012345678901212345
: anxiety
6
1
12 3456789012345678901234567890121234567890123456789012345678901212345
C. irascible : temperamental C. dangerous : peril 6
12 3456789012345678901234567890121234567890123456789012345678901212345 6
12 3456789012345678901234567890121234567890123456789012345678901212345 6
D. haughty : selfish D. hungry : want
12 3456789012345678901234567890121234567890123456789012345678901212345 6
12 3456789012345678901234567890121234567890123456789012345678901212345
E. pugnacious : optimistic E. facetious : sarcasm 6
12 3456789012345678901234567890121234567890123456789012345678901212345 6
12 3456789012345678901234567890121234567890123456789012345678901212345 6
12 3456789012345678901234567890121234567890123456789012345678901212345 6
12 3456789012345678901234567890121234567890123456789012345678901212345 6
123456789012345678901234567890121234567890123456789012345678901212345
2 6
1 3456789012345678901234567890121234567890123456789012345678901212345
Directions: Each sentence below contains one or two blanks, with each blank 6
12 3456789012345678901234567890121234567890123456789012345678901212345
corresponding to an omitted word. Find the answer choice that has the word or set or 6
12 3456789012345678901234567890121234567890123456789012345678901212345 6
12 3456789012345678901234567890121234567890123456789012345678901212345
words that best fits the meaning of the sentence as a whole. 6
12 3456789012345678901234567890121234567890123456789012345678901212345 6
12 3456789012345678901234567890121234567890123456789012345678901212345 6
12 3456789012345678901234567890121234567890123456789012345678901212345 6
12 3456789012345678901234567890121234567890123456789012345678901212345 6
12 3456789012345678901234567890121234567890123456789012345678901212345 6
123456789012345678901234567890121234567890123456789012345678901212345
2 18. The mayor’s unpopularity was dramati- 20. Normally the press secretary is fairly 6
123456789012345678901234567890121234567890123456789012345678901212345 6
123456789012345678901234567890121234567890123456789012345678901212345
cally demonstrated by his __________ __________, but in today’s briefing her 6
123456789012345678901234567890121234567890123456789012345678901212345 6
1 3456789012345678901234567890121234567890123456789012345678901212345
performance in the citywide straw poll. face was quite ___________ and her 6
123456789012345678901234567890121234567890123456789012345678901212345 66
2 3456789012345678901234567890121234567890123456789012345678901212345
123456789012345678901234567890121234567890123456789012345678901212345
responses lively.
6
1 A. sanguine
123456789012345678901234567890121234567890123456789012345678901212345 6
12
2
3456789012345678901234567890121234567890123456789012345678901212345 6
B. dismal A. peppy.
3456789012345678901234567890121234567890123456789012345678901212345
.morose
6
123456789012345678901234567890121234567890123456789012345678901212345
123456789012345678901234567890121234567890123456789012345678901212345
C. fiery B. subjugated. .active 6
1 6
12 3456789012345678901234567890121234567890123456789012345678901212345 6
D. impoverished C. noxious. .expressive
12 3456789012345678901234567890121234567890123456789012345678901212345 6
12 3456789012345678901234567890121234567890123456789012345678901212345
E. discouraging D. concise. .perturbed 6
12 3456789012345678901234567890121234567890123456789012345678901212345 6
123456789012345678901234567890121234567890123456789012345678901212345 6
E. subdued. .animated
123456789012345678901234567890121234567890123456789012345678901212345
19. Country X will never stabilize its economy 6
123456789012345678901234567890121234567890123456789012345678901212345 6
2 3456789012345678901234567890121234567890123456789012345678901212345
21. A slight ___________ of the soil’s settling 6
123456789012345678901234567890121234567890123456789012345678901212345
if it does not ___________ its disastrous
123456789012345678901234567890121234567890123456789012345678901212345 6
123456789012345678901234567890121234567890123456789012345678901212345
inflationary spending practices. rate caused the building’s foundation to 6
123456789012345678901234567890121234567890123456789012345678901212345 66
1 ___________, which led to the building
12 3456789012345678901234567890121234567890123456789012345678901212345
A. exacerbate 6
12 3456789012345678901234567890121234567890123456789012345678901212345
being deemed unusable. 6
12 3456789012345678901234567890121234567890123456789012345678901212345
B. curtail 6
12 3456789012345678901234567890121234567890123456789012345678901212345 6
123456789012345678901234567890121234567890123456789012345678901212345 6
C. strengthen A. estimation. .settle
2 3456789012345678901234567890121234567890123456789012345678901212345
123456789012345678901234567890121234567890123456789012345678901212345 6
123456789012345678901234567890121234567890123456789012345678901212345
D. maximize B. deviation. .bulge 6
123456789012345678901234567890121234567890123456789012345678901212345 6
123456789012345678901234567890121234567890123456789012345678901212345 66
E. weaken C. miscalculation. .shift
1
12 3456789012345678901234567890121234567890123456789012345678901212345
D. permutation. .stabilize 6
123456789012345678901234567890121234567890123456789012345678901212345 6
12 3456789012345678901234567890121234567890123456789012345678901212345 6
E. escalation. .liquefy
123456789012345678901234567890121234567890123456789012345678901212345 6
12 3456789012345678901234567890121234567890123456789012345678901212345 6
123456789012345678901234567890121234567890123456789012345678901212345 6
123456789012345678901234567890121234567890123456789012345678901212345 6
123456789012345678901234567890121234567890123456789012345678901212345 6
123456789012345678901234567890121234567890123456789012345678901212345 6
42 1234567890123456789012345678901212345678901234567890123456789012123456

www.petersons.com
DIAGNOSTIC
1234567890123456789012345678901212345678901234567890123456789012123456
123456789012345678901234567890121234567890123456789012345678901212345 6
12 3456789012345678901234567890121234567890123456789012345678901212345 6
123456789012345678901234567890121234567890123456789012345678901212345
22. Prison reform in Victorian England, 23. For three centuries, classical mechanics 6
12 3456789012345678901234567890121234567890123456789012345678901212345 6
12 3456789012345678901234567890121234567890123456789012345678901212345
though ___________, was not far-reaching was the __________ paradigm in physics 6
123456789012345678901234567890121234567890123456789012345678901212345 6
12
2
3456789012345678901234567890121234567890123456789012345678901212345 6
enough to eliminate all the ___________ until
3456789012345678901234567890121234567890123456789012345678901212345
developments in quantum mechanics
6
1
12 3456789012345678901234567890121234567890123456789012345678901212345
vestiges of harsher times. began to call __________ assumptions of 6
12 3456789012345678901234567890121234567890123456789012345678901212345 6
12 3456789012345678901234567890121234567890123456789012345678901212345
classical mechanics into question. 6
12
2
3456789012345678901234567890121234567890123456789012345678901212345 6
A. appreciable. .miscellaneous
3456789012345678901234567890121234567890123456789012345678901212345 6
1
12 3456789012345678901234567890121234567890123456789012345678901212345
B. significant. .cruel A. strongest. .tangential 6
12 3456789012345678901234567890121234567890123456789012345678901212345 6
12 3456789012345678901234567890121234567890123456789012345678901212345 6
C. dogged. .basic B. supreme. .superficial
12 3456789012345678901234567890121234567890123456789012345678901212345 6
123456789012345678901234567890121234567890123456789012345678901212345
D. laudable. .benign C. reigning. .fundamental 6
12 3456789012345678901234567890121234567890123456789012345678901212345 6
12 3456789012345678901234567890121234567890123456789012345678901212345 6
E. wrongheaded. .base D. secondary. .basic
12 3456789012345678901234567890121234567890123456789012345678901212345
E. unchallenged. .facile 6
12
2
3456789012345678901234567890121234567890123456789012345678901212345
3456789012345678901234567890121234567890123456789012345678901212345
6
6
1
12 3456789012345678901234567890121234567890123456789012345678901212345 6
12 3456789012345678901234567890121234567890123456789012345678901212345 6
12 3456789012345678901234567890121234567890123456789012345678901212345 6
12 3456789012345678901234567890121234567890123456789012345678901212345
Directions: Questions follow each passage. After reading the passage, determine the best 6
2 3456789012345678901234567890121234567890123456789012345678901212345
123456789012345678901234567890121234567890123456789012345678901212345 6
123456789012345678901234567890121234567890123456789012345678901212345
answer for each question based on the passage’s content. 6
123456789012345678901234567890121234567890123456789012345678901212345 66
123456789012345678901234567890121234567890123456789012345678901212345 6
123456789012345678901234567890121234567890123456789012345678901212345 6
123456789012345678901234567890121234567890123456789012345678901212345 6
1 Passage 1
12 3456789012345678901234567890121234567890123456789012345678901212345imperial family. Even when the Yamato 6
12 3456789012345678901234567890121234567890123456789012345678901212345 6
12 3456789012345678901234567890121234567890123456789012345678901212345clan had nominally laid claim to the title 6
2 3456789012345678901234567890121234567890123456789012345678901212345
123456789012345678901234567890121234567890123456789012345678901212345
Line Though the founding myth of the 6
123456789012345678901234567890121234567890123456789012345678901212345 of the imperial throne, their power was 6
123456789012345678901234567890121234567890123456789012345678901212345
Yamato clan sets the beginning of their 6
123456789012345678901234567890121234567890123456789012345678901212345 still not as absolute as their titular 6
123456789012345678901234567890121234567890123456789012345678901212345
dynasty at the dawn of time, the Yamato 6
123456789012345678901234567890121234567890123456789012345678901212345
(30) pretensions suggested. The aristocratic 6
123456789012345678901234567890121234567890123456789012345678901212345
clan did not unify Japan under their 6
123456789012345678901234567890121234567890123456789012345678901212345 clan chieftains still maintained significant 6
123456789012345678901234567890121234567890123456789012345678901212345
(5) monarchical rule until the sixth century 6
1 power bases in the regions historically 6
2 3456789012345678901234567890121234567890123456789012345678901212345
123456789012345678901234567890121234567890123456789012345678901212345
B.C.E. It is difficult to designate a specific
6
123456789012345678901234567890121234567890123456789012345678901212345 linked with their clans. 6
1 date for the beginning of the Yamato 6
123456789012345678901234567890121234567890123456789012345678901212345
monarchy because the process of political
This scenario of power diffusion 6
12
2
3456789012345678901234567890121234567890123456789012345678901212345
3456789012345678901234567890121234567890123456789012345678901212345
6
6
123456789012345678901234567890121234567890123456789012345678901212345
evolution in ancient Japan was not
(35) under the political rubric of monarchy is
123456789012345678901234567890121234567890123456789012345678901212345 6
1 (10) marked by clearly defined stages. Rather, not peculiar to ancient Japan. In fact, it is 6
12 3456789012345678901234567890121234567890123456789012345678901212345a common theme in the narrative of 6
12 3456789012345678901234567890121234567890123456789012345678901212345
the political evolution from aristocratic 6
2 3456789012345678901234567890121234567890123456789012345678901212345
123456789012345678901234567890121234567890123456789012345678901212345 kingdom formation in Medieval Europe. 6
123456789012345678901234567890121234567890123456789012345678901212345
rule centered on a number of clan 6
1 Many monarchs, with the notable 6
123456789012345678901234567890121234567890123456789012345678901212345
families; the monarchical rule dominated 6
123456789012345678901234567890121234567890123456789012345678901212345
(40) exception of Charlemagne, had great 6
123456789012345678901234567890121234567890123456789012345678901212345 66
2 3456789012345678901234567890121234567890123456789012345678901212345
by one family was a gradual process.
123456789012345678901234567890121234567890123456789012345678901212345 difficulty consolidating their kingdom’s
6
123456789012345678901234567890121234567890123456789012345678901212345
(15) The slow pace of the transformation
resources under their authority given the 6
123456789012345678901234567890121234567890123456789012345678901212345 6
1 was mirrored by the means employed to
12 3456789012345678901234567890121234567890123456789012345678901212345prominent and powerful positions of 6
12 3456789012345678901234567890121234567890123456789012345678901212345
achieve the result. The Yamato clan did 6
12 3456789012345678901234567890121234567890123456789012345678901212345dukes, lords, and wealthy merchants that 6
12 3456789012345678901234567890121234567890123456789012345678901212345
not militarily subjugate the competing 6
123456789012345678901234567890121234567890123456789012345678901212345
(45) populated their lands. 6
123456789012345678901234567890121234567890123456789012345678901212345 66
2 3456789012345678901234567890121234567890123456789012345678901212345
field of clans, but instead gained ascen-
123456789012345678901234567890121234567890123456789012345678901212345 6
123456789012345678901234567890121234567890123456789012345678901212345
(20) dancy through political, economic, and
6
123456789012345678901234567890121234567890123456789012345678901212345 6
1 diplomatic means. Since such means
2 3456789012345678901234567890121234567890123456789012345678901212345
123456789012345678901234567890121234567890123456789012345678901212345 6
1 generally preclude decisive shifts in 6
12 3456789012345678901234567890121234567890123456789012345678901212345 6
123456789012345678901234567890121234567890123456789012345678901212345 6
power, the Yamato clan had to patiently
12 3456789012345678901234567890121234567890123456789012345678901212345 6
123456789012345678901234567890121234567890123456789012345678901212345
leverage their advantages until they were 6
123456789012345678901234567890121234567890123456789012345678901212345
able to demand recognition as the 6
123456789012345678901234567890121234567890123456789012345678901212345 66
(25)
123456789012345678901234567890121234567890123456789012345678901212345
1234567890123456789012345678901212345678901234567890123456789012123456 43
Part II: Diagnosing Strengths and Weaknesses

1234567890123456789012345678901212345678901234567890123456789012123456
123456789012345678901234567890121234567890123456789012345678901212345 6
12 3456789012345678901234567890121234567890123456789012345678901212345 6
123456789012345678901234567890121234567890123456789012345678901212345
24. In line 34, the word diffusion could most Passage 2 6
12 3456789012345678901234567890121234567890123456789012345678901212345 6
12 3456789012345678901234567890121234567890123456789012345678901212345
reasonably be replaced by which of the 6
123456789012345678901234567890121234567890123456789012345678901212345
Line Grasping the interrelationship between 6
12
2
3456789012345678901234567890121234567890123456789012345678901212345
following?
3456789012345678901234567890121234567890123456789012345678901212345
6
6
1 structure and function is essential to
12 3456789012345678901234567890121234567890123456789012345678901212345 6
12 3456789012345678901234567890121234567890123456789012345678901212345 6
A. Assimilation gaining a basic understanding of molecu-
12 3456789012345678901234567890121234567890123456789012345678901212345 6
12
2
3456789012345678901234567890121234567890123456789012345678901212345
B. Sharing lar biology. All biological entities are
3456789012345678901234567890121234567890123456789012345678901212345
6
6
1
12 3456789012345678901234567890121234567890123456789012345678901212345
C. Hegemony (5) composed of molecules, and every 6
12 3456789012345678901234567890121234567890123456789012345678901212345 6
12 3456789012345678901234567890121234567890123456789012345678901212345 6
D. Mongering molecule has a specific shape. A hydro-
12 3456789012345678901234567890121234567890123456789012345678901212345 6
123456789012345678901234567890121234567890123456789012345678901212345
E. Imbalances gen molecule, for instance, has a simple 6
12 3456789012345678901234567890121234567890123456789012345678901212345 6
12 3456789012345678901234567890121234567890123456789012345678901212345 6
linear shape, while DNA, the molecule
12 3456789012345678901234567890121234567890123456789012345678901212345
25. What is the main point of the third that stores most of a cell’s genetic 6
12
2
3456789012345678901234567890121234567890123456789012345678901212345
3456789012345678901234567890121234567890123456789012345678901212345
6
1 paragraph? information, has a complex double helix 6
12 3456789012345678901234567890121234567890123456789012345678901212345
(10)
6
12 3456789012345678901234567890121234567890123456789012345678901212345
shape. The shape of a molecule, com- 6
12 3456789012345678901234567890121234567890123456789012345678901212345 6
A. Ancient Japan’s political evolution
12 3456789012345678901234567890121234567890123456789012345678901212345
bined with the configuration of its 6
12 3456789012345678901234567890121234567890123456789012345678901212345
was dissimilar to Medieval European 6
12 3456789012345678901234567890121234567890123456789012345678901212345
valence electrons (the electrons available 6
12 3456789012345678901234567890121234567890123456789012345678901212345
political evolution. 6
12 3456789012345678901234567890121234567890123456789012345678901212345
for bonding with other molecules), 6
12 3456789012345678901234567890121234567890123456789012345678901212345
B. Charlemagne was a powerful mon- 6
2 3456789012345678901234567890121234567890123456789012345678901212345
123456789012345678901234567890121234567890123456789012345678901212345
(15) determine the structure of a molecule. 6
1 arch comparable to the great emper- 6
12 3456789012345678901234567890121234567890123456789012345678901212345
Since this structure determines the 6
12 3456789012345678901234567890121234567890123456789012345678901212345
ors of ancient Japan. 6
12 3456789012345678901234567890121234567890123456789012345678901212345
biological functionality of the molecule, 6
2 3456789012345678901234567890121234567890123456789012345678901212345
123456789012345678901234567890121234567890123456789012345678901212345
C. Both Medieval European monarchs and 6
123456789012345678901234567890121234567890123456789012345678901212345
it is of paramount importance. 6
123456789012345678901234567890121234567890123456789012345678901212345
Japanese emperors had difficulties in 6
123456789012345678901234567890121234567890123456789012345678901212345
unifying the lands under their power. The chemistry of the brain illustrates 6
123456789012345678901234567890121234567890123456789012345678901212345 6
123456789012345678901234567890121234567890123456789012345678901212345
D. Dukes, lords, wealthy merchants, and (20) the interaction between molecular 6
123456789012345678901234567890121234567890123456789012345678901212345 6
123456789012345678901234567890121234567890123456789012345678901212345
clan chieftains were politically obtuse structure and biological function. One 6
123456789012345678901234567890121234567890123456789012345678901212345 6
1 throughout the world. nerve cell in the brain communicates with 6
2 3456789012345678901234567890121234567890123456789012345678901212345
123456789012345678901234567890121234567890123456789012345678901212345 6
123456789012345678901234567890121234567890123456789012345678901212345
E. Clan chieftains of ancient Japan another nerve cell by the emission and 6
1 6
123456789012345678901234567890121234567890123456789012345678901212345
resembled wealthy merchants in uptake of molecular signals. A transmit- 6
12 3456789012345678901234567890121234567890123456789012345678901212345 6
123456789012345678901234567890121234567890123456789012345678901212345 66
2 3456789012345678901234567890121234567890123456789012345678901212345
Medieval Europe with respect to the (25) ting nerve cell initiates the communica-
123456789012345678901234567890121234567890123456789012345678901212345
tion with a second nerve cell known as a 6
1 political role each played in their
12 3456789012345678901234567890121234567890123456789012345678901212345
receiving cell. The transmitting cell 6
12 3456789012345678901234567890121234567890123456789012345678901212345
respective eras and locales. 6
2 3456789012345678901234567890121234567890123456789012345678901212345
123456789012345678901234567890121234567890123456789012345678901212345
releases signal molecules that are 6
123456789012345678901234567890121234567890123456789012345678901212345 6
1 26. What is the author’s attitude toward the structured specifically to bond with 6
123456789012345678901234567890121234567890123456789012345678901212345 6
123456789012345678901234567890121234567890123456789012345678901212345
(30) receptor molecules located at specific 6
12 3456789012345678901234567890121234567890123456789012345678901212345
notion of absolute monarchical rule in 6
12 3456789012345678901234567890121234567890123456789012345678901212345
ancient Japan and Medieval Europe? sites on the receiving cell. The signal and 6
12 3456789012345678901234567890121234567890123456789012345678901212345 6
12 3456789012345678901234567890121234567890123456789012345678901212345
receptor molecules have complementary 6
123456789012345678901234567890121234567890123456789012345678901212345
A. Harsh criticism of the weak monarchs 6
12 3456789012345678901234567890121234567890123456789012345678901212345
molecular structures which allow for 6
12 3456789012345678901234567890121234567890123456789012345678901212345
B. Measured skepticism about the 6
12 3456789012345678901234567890121234567890123456789012345678901212345
their bonding ability. You can say the 6
12 3456789012345678901234567890121234567890123456789012345678901212345
absoluteness of the monarch’s rule 6
123456789012345678901234567890121234567890123456789012345678901212345
(35) signal molecule functions like a key while 6
2 3456789012345678901234567890121234567890123456789012345678901212345
Strenuous objection to the monarch’s
the receptor molecule is the lock. In this 6
123456789012345678901234567890121234567890123456789012345678901212345
C.
123456789012345678901234567890121234567890123456789012345678901212345 6
123456789012345678901234567890121234567890123456789012345678901212345
political practices 6
123456789012345678901234567890121234567890123456789012345678901212345
analogy, the molecular structure can be 6
1 D. Moderate praise of the monarch’s 6
2 3456789012345678901234567890121234567890123456789012345678901212345
123456789012345678901234567890121234567890123456789012345678901212345
thought of as the nodes on the key and 6
1 restraint 6
12 3456789012345678901234567890121234567890123456789012345678901212345
within the lock that complement each 6
123456789012345678901234567890121234567890123456789012345678901212345
E. Fulsome praise of the monarch’s 6
12
2
3456789012345678901234567890121234567890123456789012345678901212345
(40) other
3456789012345678901234567890121234567890123456789012345678901212345
and allow for the key to turn the 6
6
1 policies
123456789012345678901234567890121234567890123456789012345678901212345
lock. The bonding of the signal and 6
123456789012345678901234567890121234567890123456789012345678901212345 66
123456789012345678901234567890121234567890123456789012345678901212345
44 1234567890123456789012345678901212345678901234567890123456789012123456

www.petersons.com
DIAGNOSTIC
1234567890123456789012345678901212345678901234567890123456789012123456
123456789012345678901234567890121234567890123456789012345678901212345 6
12 3456789012345678901234567890121234567890123456789012345678901212345 6
123456789012345678901234567890121234567890123456789012345678901212345
receptor molecules sets in motion a series 29. What is the function of the second 6
12 3456789012345678901234567890121234567890123456789012345678901212345 6
12 3456789012345678901234567890121234567890123456789012345678901212345
of chemical reactions within the receiving paragraph in relation to the passage as a 6
123456789012345678901234567890121234567890123456789012345678901212345 6
12
2
3456789012345678901234567890121234567890123456789012345678901212345 6
cell. Thus, through the biological whole?
3456789012345678901234567890121234567890123456789012345678901212345 6
1 (45) mechanism of complementary molecular
12 3456789012345678901234567890121234567890123456789012345678901212345 6
12 3456789012345678901234567890121234567890123456789012345678901212345
A. It relates an anecdote that demon- 6
12 3456789012345678901234567890121234567890123456789012345678901212345
structure, one brain cell is able to 6
12
2
3456789012345678901234567890121234567890123456789012345678901212345
strates
3456789012345678901234567890121234567890123456789012345678901212345
the principle described in the 6
6
1 communicate and affect another
12 3456789012345678901234567890121234567890123456789012345678901212345
first paragraph. 6
12 3456789012345678901234567890121234567890123456789012345678901212345
brain cell. 6
12 3456789012345678901234567890121234567890123456789012345678901212345 6
B. It gives further explanation to the
12 3456789012345678901234567890121234567890123456789012345678901212345 6
123456789012345678901234567890121234567890123456789012345678901212345
theoretical hypothesis made in the 6
12 3456789012345678901234567890121234567890123456789012345678901212345
27. The primary purpose of this passage is to 6
12 3456789012345678901234567890121234567890123456789012345678901212345 6
first paragraph.
12 3456789012345678901234567890121234567890123456789012345678901212345
C. It provides an example that illustrates 6
12
2
3456789012345678901234567890121234567890123456789012345678901212345
A. demonstrate the importance of
3456789012345678901234567890121234567890123456789012345678901212345
6
6
1 the relationship discussed in the first
12 3456789012345678901234567890121234567890123456789012345678901212345
biological functionality regarding 6
12 3456789012345678901234567890121234567890123456789012345678901212345
paragraph. 6
12 3456789012345678901234567890121234567890123456789012345678901212345 6
brain functioning capacities.
12 3456789012345678901234567890121234567890123456789012345678901212345
D. It presents a second viewpoint on the 6
12 3456789012345678901234567890121234567890123456789012345678901212345
B. criticize the contemporary theory that 6
12 3456789012345678901234567890121234567890123456789012345678901212345
issues raised in the first paragraph. 6
12 3456789012345678901234567890121234567890123456789012345678901212345
molecular structure and biological 6
12 3456789012345678901234567890121234567890123456789012345678901212345
E. It debunks a common misconception 6
12 3456789012345678901234567890121234567890123456789012345678901212345
functionality are interrelated. 6
12 3456789012345678901234567890121234567890123456789012345678901212345
about the principle discussed in the 6
123456789012345678901234567890121234567890123456789012345678901212345
C. present competing views on the nature 6
12 3456789012345678901234567890121234567890123456789012345678901212345
first paragraph. 6
12 3456789012345678901234567890121234567890123456789012345678901212345 6
of the interrelationship between brain
12 3456789012345678901234567890121234567890123456789012345678901212345 6
12 3456789012345678901234567890121234567890123456789012345678901212345
chemistry and molecular structure. 6
12 3456789012345678901234567890121234567890123456789012345678901212345
30. Based on the passage, which of the 6
12 3456789012345678901234567890121234567890123456789012345678901212345
D. explain the contemporary theory of 6
12 3456789012345678901234567890121234567890123456789012345678901212345
the interrelationship between molecu-
following statements would the author 6
12 3456789012345678901234567890121234567890123456789012345678901212345 6
2 3456789012345678901234567890121234567890123456789012345678901212345
123456789012345678901234567890121234567890123456789012345678901212345
lar structure and biological function-
most likely agree with? 6
123456789012345678901234567890121234567890123456789012345678901212345 6
123456789012345678901234567890121234567890123456789012345678901212345
ality. A. The study of molecular functionality 6
123456789012345678901234567890121234567890123456789012345678901212345 66
1 E. argue that molecular structure has is an important field in molecular
12 3456789012345678901234567890121234567890123456789012345678901212345 6
12 3456789012345678901234567890121234567890123456789012345678901212345
greater explanatory force than does chemistry. 6
123456789012345678901234567890121234567890123456789012345678901212345 6
123456789012345678901234567890121234567890123456789012345678901212345
biological functionality. B. The advancement of understanding in 6
12 3456789012345678901234567890121234567890123456789012345678901212345 6
12 3456789012345678901234567890121234567890123456789012345678901212345
molecular biology is contingent upon 6
12 3456789012345678901234567890121234567890123456789012345678901212345 6
123456789012345678901234567890121234567890123456789012345678901212345
28. Why does the author mention a hydrogen further research in how molecules are 6
12 3456789012345678901234567890121234567890123456789012345678901212345 6
12 3456789012345678901234567890121234567890123456789012345678901212345
molecule and DNA in lines 6–11 in the structured. 6
2 3456789012345678901234567890121234567890123456789012345678901212345
C. Brain chemistry will not ultimately be 6
123456789012345678901234567890121234567890123456789012345678901212345
123456789012345678901234567890121234567890123456789012345678901212345
first paragraph? 6
1 6
123456789012345678901234567890121234567890123456789012345678901212345
A. To contrast a simply shaped molecule understood until neurotransmitters 6
123456789012345678901234567890121234567890123456789012345678901212345 6
2 3456789012345678901234567890121234567890123456789012345678901212345
123456789012345678901234567890121234567890123456789012345678901212345
with a complexly shaped molecule are fully conceptualized. 6
123456789012345678901234567890121234567890123456789012345678901212345 6
123456789012345678901234567890121234567890123456789012345678901212345
B. To demonstrate that DNA is similarly D. Biological functionality is not 6
123456789012345678901234567890121234567890123456789012345678901212345 6
1 shaped to hydrogen dependent on molecular shape. 6
2 3456789012345678901234567890121234567890123456789012345678901212345
123456789012345678901234567890121234567890123456789012345678901212345 6
123456789012345678901234567890121234567890123456789012345678901212345
C. To illustrate that biological function- E. DNA is the most complex molecule 6
123456789012345678901234567890121234567890123456789012345678901212345 6
123456789012345678901234567890121234567890123456789012345678901212345
ality is not directly related to molecu- known to biology. 6
1 6
123456789012345678901234567890121234567890123456789012345678901212345 66
2 3456789012345678901234567890121234567890123456789012345678901212345
lar shape
123456789012345678901234567890121234567890123456789012345678901212345 6
123456789012345678901234567890121234567890123456789012345678901212345
D. To give examples of the molecular
6
123456789012345678901234567890121234567890123456789012345678901212345 6
1 structure of two different molecules
2 3456789012345678901234567890121234567890123456789012345678901212345
123456789012345678901234567890121234567890123456789012345678901212345 6
1 E. To undermine the assertion that 6
12 3456789012345678901234567890121234567890123456789012345678901212345 6
123456789012345678901234567890121234567890123456789012345678901212345 6
hydrogen is more simply structured
12 3456789012345678901234567890121234567890123456789012345678901212345 6
123456789012345678901234567890121234567890123456789012345678901212345
than DNA 6
123456789012345678901234567890121234567890123456789012345678901212345 6
123456789012345678901234567890121234567890123456789012345678901212345 6
123456789012345678901234567890121234567890123456789012345678901212345 6 45
1234567890123456789012345678901212345678901234567890123456789012123456
Part II: Diagnosing Strengths and Weaknesses

1234567890123456789012345678901212345678901234567890123456789012123456
123456789012345678901234567890121234567890123456789012345678901212345 6
12 3456789012345678901234567890121234567890123456789012345678901212345 6
123456789012345678901234567890121234567890123456789012345678901212345 6
12 3456789012345678901234567890121234567890123456789012345678901212345
Answer Key 6
12 3456789012345678901234567890121234567890123456789012345678901212345 6
123456789012345678901234567890121234567890123456789012345678901212345 6
12
2
3456789012345678901234567890121234567890123456789012345678901212345 6
Quantitative Verbal
3456789012345678901234567890121234567890123456789012345678901212345 6
1
12 3456789012345678901234567890121234567890123456789012345678901212345 6
12 3456789012345678901234567890121234567890123456789012345678901212345
1. B 15. D 1. E 16. D 6
12 3456789012345678901234567890121234567890123456789012345678901212345 6
12
2
3456789012345678901234567890121234567890123456789012345678901212345
2. A 16. E 2. A 17. B
3456789012345678901234567890121234567890123456789012345678901212345
6
6
1
12 3456789012345678901234567890121234567890123456789012345678901212345
3. A 17. B 3. E 18. B 6
12 3456789012345678901234567890121234567890123456789012345678901212345 6
12 3456789012345678901234567890121234567890123456789012345678901212345
4. C 18. E 4. A 19. B 6
12
2
3456789012345678901234567890121234567890123456789012345678901212345
3456789012345678901234567890121234567890123456789012345678901212345
6
6
1 5. C 19. D 5. B 20. E
12 3456789012345678901234567890121234567890123456789012345678901212345 6
12 3456789012345678901234567890121234567890123456789012345678901212345
6. D 20. E 6. A 21. C 6
12 3456789012345678901234567890121234567890123456789012345678901212345 6
12
2
3456789012345678901234567890121234567890123456789012345678901212345 6
7. B 21. D 7. D 22. B
3456789012345678901234567890121234567890123456789012345678901212345 6
1
12 3456789012345678901234567890121234567890123456789012345678901212345
8. D 22. D 8. C 23. C 6
12 3456789012345678901234567890121234567890123456789012345678901212345 6
12 3456789012345678901234567890121234567890123456789012345678901212345 6
9. D 23. C 9. D 24. B
12 3456789012345678901234567890121234567890123456789012345678901212345 6
12 3456789012345678901234567890121234567890123456789012345678901212345
10. C 24. C 10. C 25. C 6
12 3456789012345678901234567890121234567890123456789012345678901212345 6
12 3456789012345678901234567890121234567890123456789012345678901212345 6
11. A 25. A 11. B 26. B
12 3456789012345678901234567890121234567890123456789012345678901212345 6
12 3456789012345678901234567890121234567890123456789012345678901212345
12. B 26. E 12. A 27. D 6
2 3456789012345678901234567890121234567890123456789012345678901212345
123456789012345678901234567890121234567890123456789012345678901212345 6
1 13. D 27. A 13. E 28. A 6
12 3456789012345678901234567890121234567890123456789012345678901212345 6
12 3456789012345678901234567890121234567890123456789012345678901212345 6
14. A 28. B 14. B 29. C
12 3456789012345678901234567890121234567890123456789012345678901212345
15. C 30. B 6
12 3456789012345678901234567890121234567890123456789012345678901212345 6
12 3456789012345678901234567890121234567890123456789012345678901212345 6
12 3456789012345678901234567890121234567890123456789012345678901212345 6
12 3456789012345678901234567890121234567890123456789012345678901212345 6
12 3456789012345678901234567890121234567890123456789012345678901212345 6
123456789012345678901234567890121234567890123456789012345678901212345
2 6
123456789012345678901234567890121234567890123456789012345678901212345 6
123456789012345678901234567890121234567890123456789012345678901212345 6
123456789012345678901234567890121234567890123456789012345678901212345 6
1 3456789012345678901234567890121234567890123456789012345678901212345 6
123456789012345678901234567890121234567890123456789012345678901212345 66
2 3456789012345678901234567890121234567890123456789012345678901212345
123456789012345678901234567890121234567890123456789012345678901212345 6
1
123456789012345678901234567890121234567890123456789012345678901212345 6
12 3456789012345678901234567890121234567890123456789012345678901212345 6
12 3456789012345678901234567890121234567890123456789012345678901212345 6
12 3456789012345678901234567890121234567890123456789012345678901212345 6
123456789012345678901234567890121234567890123456789012345678901212345 6
12 3456789012345678901234567890121234567890123456789012345678901212345 6
12 3456789012345678901234567890121234567890123456789012345678901212345 6
123456789012345678901234567890121234567890123456789012345678901212345
2 6
123456789012345678901234567890121234567890123456789012345678901212345 6
1 3456789012345678901234567890121234567890123456789012345678901212345 6
123456789012345678901234567890121234567890123456789012345678901212345 6
123456789012345678901234567890121234567890123456789012345678901212345 6
12 3456789012345678901234567890121234567890123456789012345678901212345 6
12 3456789012345678901234567890121234567890123456789012345678901212345 6
12 3456789012345678901234567890121234567890123456789012345678901212345 6
12 3456789012345678901234567890121234567890123456789012345678901212345 6
123456789012345678901234567890121234567890123456789012345678901212345 6
123456789012345678901234567890121234567890123456789012345678901212345
2 6
123456789012345678901234567890121234567890123456789012345678901212345 6
123456789012345678901234567890121234567890123456789012345678901212345 6
123456789012345678901234567890121234567890123456789012345678901212345 6
1 3456789012345678901234567890121234567890123456789012345678901212345 6
123456789012345678901234567890121234567890123456789012345678901212345 66
2 3456789012345678901234567890121234567890123456789012345678901212345
123456789012345678901234567890121234567890123456789012345678901212345 6
123456789012345678901234567890121234567890123456789012345678901212345 6
123456789012345678901234567890121234567890123456789012345678901212345 6
1
12 3456789012345678901234567890121234567890123456789012345678901212345 6
123456789012345678901234567890121234567890123456789012345678901212345 6
12 3456789012345678901234567890121234567890123456789012345678901212345 6
123456789012345678901234567890121234567890123456789012345678901212345 6
12 3456789012345678901234567890121234567890123456789012345678901212345 6
123456789012345678901234567890121234567890123456789012345678901212345 6
123456789012345678901234567890121234567890123456789012345678901212345 6
123456789012345678901234567890121234567890123456789012345678901212345 6
123456789012345678901234567890121234567890123456789012345678901212345 6
46 1234567890123456789012345678901212345678901234567890123456789012123456

www.petersons.com
PART

III
Section Review
Quantitative Review 48

Verbal Review 134

Analytical Writing Review 227

PART III
Chapter

4
Quantitative Review
123456789012345678901234567890121234567890123456789012
12345678901234567890123456789012123456789012345678901 2
12345678901234567890123456789012123456789012345678901 2
12345678901234567890123456789012123456789012345678901
All the Math You Left Behind 2
12345678901234567890123456789012123456789012345678901 2
2345678901234567890123456789012123456789012345678901
For those who have not seen a triangle since high school geometry, it’s 2
12345678901234567890123456789012123456789012345678901
12345678901234567890123456789012123456789012345678901 2
12345678901234567890123456789012123456789012345678901 2
12345678901234567890123456789012123456789012345678901 2
scary to think that your graduate school aspirations partially rest on your
12345678901234567890123456789012123456789012345678901 2
1 ability to work with the math concepts of your youth. Those of you who 2
12345678901234567890123456789012123456789012345678901
studied math extensively in college aren’t off the hook either, since the 2
12345678901234567890123456789012123456789012345678901 2
12345678901234567890123456789012123456789012345678901
concepts tested on the GRE are not the ones studied by math majors or 2
12345678901234567890123456789012123456789012345678901 2
2345678901234567890123456789012123456789012345678901
engineering students. Brainiacs who are used to working with orthogonal 2
12345678901234567890123456789012123456789012345678901
12345678901234567890123456789012123456789012345678901 2
12345678901234567890123456789012123456789012345678901
matrices might not remember dividing with remainders. 2
12345678901234567890123456789012123456789012345678901 2
12345678901234567890123456789012123456789012345678901
That’s the “half-empty” news. The “half-full” news is that the GRE math 2
2
12345678901234567890123456789012123456789012345678901
12345678901234567890123456789012123456789012345678901
section tests a limited number of math concepts, so the math content of the 2
2
12345678901234567890123456789012123456789012345678901
1 GRE can be mastered. With a little bit—okay, for some of you, a lot—of 2
12345678901234567890123456789012123456789012345678901 2
12345678901234567890123456789012123456789012345678901
studying, you can be confident that you will not be tested on an unfamiliar 2
12345678901234567890123456789012123456789012345678901 2
12345678901234567890123456789012123456789012345678901 2
12345678901234567890123456789012123456789012345678901
math concept. The reason for this comes
2345678901234567890123456789012123456789012345678901
partly from the fact that the 2
material covered in the GRE math section is not very difficult. Instead, the 2
12345678901234567890123456789012123456789012345678901
12345678901234567890123456789012123456789012345678901 2
1 difficulty comes when the test demands you apply this basic set of GRE 2
12345678901234567890123456789012123456789012345678901 2
12345678901234567890123456789012123456789012345678901 2
12345678901234567890123456789012123456789012345678901 2
2345678901234567890123456789012123456789012345678901
math knowledge in creative ways to solve problems.
12345678901234567890123456789012123456789012345678901 2
1 Mastering the basic math knowledge of the GRE becomes the first goal, so 2
12345678901234567890123456789012123456789012345678901 2
12345678901234567890123456789012123456789012345678901
this is what we’ll focus on for now (without such mastery, the math will 2
12345678901234567890123456789012123456789012345678901 2
12345678901234567890123456789012123456789012345678901
indeed be difficult). Once this is done, the second step is learning to 2
12345678901234567890123456789012123456789012345678901 2
12345678901234567890123456789012123456789012345678901
recognize how and when to apply the basic math knowledge on the GRE. 2
12345678901234567890123456789012123456789012345678901 2
12345678901234567890123456789012123456789012345678901
This will take more time than the first part, but you will have more than 2
12345678901234567890123456789012123456789012345678901 2
12345678901234567890123456789012123456789012345678901
enough questions to practice on before you have to exercise this skill on 2
12345678901234567890123456789012123456789012345678901 2
12345678901234567890123456789012123456789012345678901
test day. 2
12345678901234567890123456789012123456789012345678901 2
2345678901234567890123456789012123456789012345678901
On test day, you can expect to see a lot of math jargon in the Quantitative 2
12345678901234567890123456789012123456789012345678901
12345678901234567890123456789012123456789012345678901 2
12345678901234567890123456789012123456789012345678901 2
1 section. But don’t fear; you don’t have to be intimidated by math speak. In 2
2345678901234567890123456789012123456789012345678901
12345678901234567890123456789012123456789012345678901 2
1 this section, we will define all the terms that you need to know for the test, 2
12345678901234567890123456789012123456789012345678901
and we will work with them so that you can get comfortable with phrases 2
12345678901234567890123456789012123456789012345678901 2
12345678901234567890123456789012123456789012345678901
2345678901234567890123456789012123456789012345678901
2
2
1 like “positive even integers divisible by 4.” As you learn (or refresh your
12345678901234567890123456789012123456789012345678901 2
12345678901234567890123456789012123456789012345678901 2
memory about) this math jargon, remember that learning a math term is
12345678901234567890123456789012123456789012345678901 2
123456789012345678901234567890121234567890123456789012
48
Chapter 4: Quantitative Review

123456789012345678901234567890121234567890123456789012
12345678901234567890123456789012123456789012345678901 2
12345678901234567890123456789012123456789012345678901 2
12345678901234567890123456789012123456789012345678901
just like learning a new word. Don’t let the jargon-ness (jargonosity?) of it 2
12345678901234567890123456789012123456789012345678901 2
12345678901234567890123456789012123456789012345678901
throw you. A math term simply refers to a specific math notion, just like 2
12345678901234567890123456789012123456789012345678901 2
12345678901234567890123456789012123456789012345678901
the word chair refers to a specific idea we have of “furniture you sit on.” 2
1 When a math term is introduced, read the definition, consider the 2
2345678901234567890123456789012123456789012345678901
12345678901234567890123456789012123456789012345678901 2
12345678901234567890123456789012123456789012345678901 2
12345678901234567890123456789012123456789012345678901
example, and think of it as your new word for the day. 2
12345678901234567890123456789012123456789012345678901 2
1 So, don’t let the GRE math section bully you, or your feelings, around. It 2
2345678901234567890123456789012123456789012345678901
12345678901234567890123456789012123456789012345678901 2
12345678901234567890123456789012123456789012345678901 2
12345678901234567890123456789012123456789012345678901
can be conquered with a little effort. Roll up your sleeves, get to work, and 2
12345678901234567890123456789012123456789012345678901
2345678901234567890123456789012123456789012345678901
2
2
1 comfort yourself with the thought that once you finish the math section on
12345678901234567890123456789012123456789012345678901 2
12345678901234567890123456789012123456789012345678901
the GRE, you will probably never have to be concerned about the area of a 2
12345678901234567890123456789012123456789012345678901
parallelogram again (unless you decide to go to business school and need 2
12345678901234567890123456789012123456789012345678901
2345678901234567890123456789012123456789012345678901
2
2
1 to take the GMAT, that is).
12345678901234567890123456789012123456789012345678901 2
12345678901234567890123456789012123456789012345678901 2
12345678901234567890123456789012123456789012345678901 2
12345678901234567890123456789012123456789012345678901 2
2345678901234567890123456789012123456789012345678901
12345678901234567890123456789012123456789012345678901 2
12345678901234567890123456789012123456789012345678901
Arithmetic, or Fun with Numbers 2
12345678901234567890123456789012123456789012345678901 22
12345678901234567890123456789012123456789012345678901
Most people hear the word arithmetic and think “add-subtract-multiply-
12345678901234567890123456789012123456789012345678901 2
12345678901234567890123456789012123456789012345678901 2
1 divide,” but arithmetic actually covers many basic math concepts and 2
12345678901234567890123456789012123456789012345678901
terms. GRE math loves to pile one math topic on top of another, so there’s 2
12345678901234567890123456789012123456789012345678901 2
12345678901234567890123456789012123456789012345678901
a lot of arithmetic in the Quant section. Take your time and don’t just skim 2
2345678901234567890123456789012123456789012345678901
12345678901234567890123456789012123456789012345678901
over this portion to get to a juicier topic like Geometry, since Arithmetic 2
2
12345678901234567890123456789012123456789012345678901
12345678901234567890123456789012123456789012345678901
covers the fundamental terms that will be prevalent on almost every 2
2
12345678901234567890123456789012123456789012345678901
12345678901234567890123456789012123456789012345678901 2
12345678901234567890123456789012123456789012345678901
question, in one form or another. 2
12345678901234567890123456789012123456789012345678901 22
12345678901234567890123456789012123456789012345678901 2
12345678901234567890123456789012123456789012345678901 2
1
12345678901234567890123456789012123456789012345678901
Integers, or Pretty Much Everything That’s Not 2
12345678901234567890123456789012123456789012345678901 2
12345678901234567890123456789012123456789012345678901
a Fraction 2
12345678901234567890123456789012123456789012345678901 2
12345678901234567890123456789012123456789012345678901
Integers are the numbers that you count with (1, 2, 3 and so on). They can 2
12345678901234567890123456789012123456789012345678901 2
12345678901234567890123456789012123456789012345678901 2
12345678901234567890123456789012123456789012345678901 2
also be negative (21, 22, 23 . . .). Zero is also an integer. The numbers
12345678901234567890123456789012123456789012345678901
23, 22, 21, 0, 1, 2, and 3 are all integers. Pretty much any whole number 2
12345678901234567890123456789012123456789012345678901 2
2345678901234567890123456789012123456789012345678901
you can think of—any number that is not a fraction or a decimal—is an 2
12345678901234567890123456789012123456789012345678901
12345678901234567890123456789012123456789012345678901 2
1 integer, and if you put a negative in front of that number, it also is an 2
12345678901234567890123456789012123456789012345678901 2
12345678901234567890123456789012123456789012345678901
integer. 2
12345678901234567890123456789012123456789012345678901 2
12345678901234567890123456789012123456789012345678901 2
12345678901234567890123456789012123456789012345678901 2
12345678901234567890123456789012123456789012345678901
In the real world, the terms integer and number are fairly synonymous, 2
12345678901234567890123456789012123456789012345678901 2
Alert!

2345678901234567890123456789012123456789012345678901
12345678901234567890123456789012123456789012345678901
since when any real person says “pick a number,” they don’t expect you 2
12345678901234567890123456789012123456789012345678901 2
12345678901234567890123456789012123456789012345678901
13 2
12345678901234567890123456789012123456789012345678901
to pick a fraction like . However, integer is a more precise term, and 2
1 8 2
2345678901234567890123456789012123456789012345678901
12345678901234567890123456789012123456789012345678901 2
12345678901234567890123456789012123456789012345678901
since it’s also something people are unfamiliar with, it makes many 2
12345678901234567890123456789012123456789012345678901 2
12345678901234567890123456789012123456789012345678901 22
appearances on the GRE.
1
12345678901234567890123456789012123456789012345678901 2
12345678901234567890123456789012123456789012345678901 2
12345678901234567890123456789012123456789012345678901
Besides zero, all integers are positive or negative, and they are also even or 2
12345678901234567890123456789012123456789012345678901 2
12345678901234567890123456789012123456789012345678901
odd. Positive integers are greater than
2345678901234567890123456789012123456789012345678901
zero (like 2, 15, or 309), and 2
2
1
12345678901234567890123456789012123456789012345678901 2
12345678901234567890123456789012123456789012345678901 2
12345678901234567890123456789012123456789012345678901 2 49
123456789012345678901234567890121234567890123456789012
Part III: Section Review

123456789012345678901234567890121234567890123456789012
12345678901234567890123456789012123456789012345678901 2
12345678901234567890123456789012123456789012345678901 2
12345678901234567890123456789012123456789012345678901
negative integers are less than zero (21, 223, 290). Think of the integers 2
12345678901234567890123456789012123456789012345678901 2
12345678901234567890123456789012123456789012345678901
as being along a line with zero at the center. 2
12345678901234567890123456789012123456789012345678901 2
12345678901234567890123456789012123456789012345678901 2
12345678901234567890123456789012123456789012345678901 2
12345678901234567890123456789012123456789012345678901 2
12345678901234567890123456789012123456789012345678901 2
12345678901234567890123456789012123456789012345678901 2
12345678901234567890123456789012123456789012345678901
This figure above is called a number line,
2345678901234567890123456789012123456789012345678901and seeing it may provoke a 2
2
1
12345678901234567890123456789012123456789012345678901
jarring flashback to the eighth grade. As you go from left to right on a 2
12345678901234567890123456789012123456789012345678901 2
12345678901234567890123456789012123456789012345678901
number line, the value of the number increases. Notice that this means that 2
12345678901234567890123456789012123456789012345678901 2
12345678901234567890123456789012123456789012345678901
25 is greater than 210. 2
12345678901234567890123456789012123456789012345678901 2
12345678901234567890123456789012123456789012345678901 2
12345678901234567890123456789012123456789012345678901
The mathematical definitions for even and odd values use integers, stating 2
12345678901234567890123456789012123456789012345678901 2
12345678901234567890123456789012123456789012345678901
that an integer is even if you can divide it by 2 and have no remainder 2
12345678901234567890123456789012123456789012345678901
(nothing left over). For example, 24 and 10 are even since two evenly 2
12345678901234567890123456789012123456789012345678901 2
12345678901234567890123456789012123456789012345678901 2
12345678901234567890123456789012123456789012345678901
divides into both. 11 and 227, on the other hand, are odd since 2 does not 2
2345678901234567890123456789012123456789012345678901
12345678901234567890123456789012123456789012345678901 2
12345678901234567890123456789012123456789012345678901
evenly divide into them. 2
12345678901234567890123456789012123456789012345678901 2
12345678901234567890123456789012123456789012345678901
123456789012345678901234 2
4123456789012345678901234
12345678901234567890123456789012123456789012345678901
12345678901234567890123 12345678901234567890123 4 2
4
12345678901234567890123456789012123456789012345678901
12345678901234567890123 4 2
12345678901234567890123412345678901234567890123
even 1 even 5 even e.g., 12 1 8 5 20
1 123456789012345678901234 2
12345678901234567890123456789012123456789012345678901
even 1 odd 5 odd e.g., 6 1 13 5 19 2
12345678901234567890123456789012123456789012345678901 2
12345678901234567890123456789012123456789012345678901 2
12345678901234567890123456789012123456789012345678901 2
2345678901234567890123456789012123456789012345678901
odd 1 odd 5 even e.g., 7 1 9 5 16
12345678901234567890123456789012123456789012345678901 2
12345678901234567890123456789012123456789012345678901 2
12345678901234567890123456789012123456789012345678901
even 3 even 5 even e.g., 4 3 6 5 24 2
12345678901234567890123456789012123456789012345678901 2
12345678901234567890123456789012123456789012345678901 2
12345678901234567890123456789012123456789012345678901
even 3 odd 5 even e.g., 3 3 8 5 24 2
12345678901234567890123456789012123456789012345678901 2
12345678901234567890123456789012123456789012345678901
odd 3 odd 5 odd e.g., 1 3 9 5 9 2
1 2
12345678901234567890123456789012123456789012345678901 2
2345678901234567890123456789012123456789012345678901
12345678901234567890123456789012123456789012345678901
(Remember, if you forget one of these, like what an even times an 2
1 2
12345678901234567890123456789012123456789012345678901
odd is, just pick an even and an odd number and multiply them. 2
12345678901234567890123456789012123456789012345678901 2
12345678901234567890123456789012123456789012345678901 2
2345678901234567890123456789012123456789012345678901
Their product will remind you.)
12345678901234567890123456789012123456789012345678901 2 2
1
12345678901234567890123456789012123456789012345678901 2
12345678901234567890123456789012123456789012345678901
You might not remember how basic math operations work with that pesky 2
2345678901234567890123456789012123456789012345678901
12345678901234567890123456789012123456789012345678901 2
12345678901234567890123456789012123456789012345678901
negative sign, so see the quick refresher on the following page. 2
1 2
12345678901234567890123456789012123456789012345678901 2
12345678901234567890123456789012123456789012345678901 2
12345678901234567890123456789012123456789012345678901 2
12345678901234567890123456789012123456789012345678901 2
12345678901234567890123456789012123456789012345678901 2
12345678901234567890123456789012123456789012345678901 2
12345678901234567890123456789012123456789012345678901 2
12345678901234567890123456789012123456789012345678901 2
2345678901234567890123456789012123456789012345678901
12345678901234567890123456789012123456789012345678901 2
12345678901234567890123456789012123456789012345678901 2
12345678901234567890123456789012123456789012345678901 2 2
1
12345678901234567890123456789012123456789012345678901 2
12345678901234567890123456789012123456789012345678901 2
12345678901234567890123456789012123456789012345678901 2
12345678901234567890123456789012123456789012345678901 2
12345678901234567890123456789012123456789012345678901 2
12345678901234567890123456789012123456789012345678901 2
2345678901234567890123456789012123456789012345678901
2
1
12345678901234567890123456789012123456789012345678901 2
12345678901234567890123456789012123456789012345678901 2
12345678901234567890123456789012123456789012345678901 2
12345678901234567890123456789012123456789012345678901 2
12345678901234567890123456789012123456789012345678901 2
12345678901234567890123456789012123456789012345678901 2
12345678901234567890123456789012123456789012345678901 2
50 123456789012345678901234567890121234567890123456789012

www.petersons.com
Chapter 4: Quantitative Review

123456789012345678901234567890121234567890123456789012
12345678901234567890123456789012123456789012345678901 2
12345678901234567890123456789012123456789012345678901 2
12345678901234567890123456789012123456789012345678901
Working with Negative Numbers 2
12345678901234567890123456789012123456789012345678901 2
12345678901234567890123456789012123456789012345678901
12345678901234567890123456 1234567890123456789012345 2
12345678901234567890123456789012123456789012345678901
1234567890123456789012345
(negative) 1 (negative) 6123456789012345678901234
e.g., (24) 1 (25) 5 29 5 2
12345678901234567890123456789012123456789012345678901
1234567890123456789012345 6 123456789012345678901234 5 2
12345678901234567890123456789012123456789012345678901
1234567890123456789012345
5 negative 6
123456789012345678901234 5 2
12345678901234567890123456789012123456789012345678901
12345678901234567890123456 1234567890123456789012345 2
12345678901234567890123456789012123456789012345678901 2
12345678901234567890123456789012123456789012345678901
(positive) 1 (negative) e.g., (10) 1 (212) 5 10 2 12 2
12345678901234567890123456789012123456789012345678901
2345678901234567890123456789012123456789012345678901
2
2
1 5 (positive) 2 (positive) 5 22
12345678901234567890123456789012123456789012345678901 2
12345678901234567890123456789012123456789012345678901
or 2
12345678901234567890123456789012123456789012345678901 2
12345678901234567890123456789012123456789012345678901
(26)
2345678901234567890123456789012123456789012345678901
1 (8) 2
2
1
12345678901234567890123456789012123456789012345678901
5826 2
12345678901234567890123456789012123456789012345678901 2
12345678901234567890123456789012123456789012345678901
5 2 2
12345678901234567890123456789012123456789012345678901
2345678901234567890123456789012123456789012345678901
2
2
1 (positive) 2 (negative) e.g., 12 2 (23) 5 12 1 3 5 15
12345678901234567890123456789012123456789012345678901 2
12345678901234567890123456789012123456789012345678901
5 positive 1 positive 2
12345678901234567890123456789012123456789012345678901 2
12345678901234567890123456789012123456789012345678901 2
12345678901234567890123456789012123456789012345678901 22
2345678901234567890123456789012123456789012345678901
(positive) 3 (negative) 5 negative e.g., 26 3 3 5 218
12345678901234567890123456789012123456789012345678901 2
12345678901234567890123456789012123456789012345678901
12345678901234567890123456789012123456789012345678901
(negative) 3 (negative) 5 positive e.g., (27) 3 (24) 5 28 2
12345678901234567890123456789012123456789012345678901 22
12345678901234567890123456789012123456789012345678901 2
1 Zero is not negative, but it is e.g., 0 3 (210) 5 0
12345678901234567890123456789012123456789012345678901 2
12345678901234567890123456789012123456789012345678901
good to remember that 2
12345678901234567890123456789012123456789012345678901 2
12345678901234567890123456789012123456789012345678901 22
2345678901234567890123456789012123456789012345678901
0 3 (anything) 5 0
12345678901234567890123456789012123456789012345678901 2
12345678901234567890123456789012123456789012345678901 2
12345678901234567890123456789012123456789012345678901
12345678901234567890123456789012123456789012345678901
How many integers are there from 25 to 7 inclusive? 2
12345678901234567890123456789012123456789012345678901 22
12345678901234567890123456789012123456789012345678901
12345678901234567890123456789012123456789012345678901
A. 14 2
12345678901234567890123456789012123456789012345678901 2
1 B. 13 2
2345678901234567890123456789012123456789012345678901
12345678901234567890123456789012123456789012345678901 2
12345678901234567890123456789012123456789012345678901
C. 12 2
1 2
12345678901234567890123456789012123456789012345678901 2
D. 2
12345678901234567890123456789012123456789012345678901 2
12345678901234567890123456789012123456789012345678901
E. 212 2
12345678901234567890123456789012123456789012345678901 2
12345678901234567890123456789012123456789012345678901
Here’s a simple math question with two well-traveled math terms, integer 2
12345678901234567890123456789012123456789012345678901 2
12345678901234567890123456789012123456789012345678901
and inclusive. You know the first term, so if you figure out what inclusive 2
2345678901234567890123456789012123456789012345678901
12345678901234567890123456789012123456789012345678901
means, you should be able to solve this problem. Inclusive means you 2
2
12345678901234567890123456789012123456789012345678901
1 include 25 and 7 when you are counting the number of integers. How 2
12345678901234567890123456789012123456789012345678901 2
12345678901234567890123456789012123456789012345678901 2
12345678901234567890123456789012123456789012345678901
many negative numbers are there from 25 to 7? Five. How many positive 2
12345678901234567890123456789012123456789012345678901 2
12345678901234567890123456789012123456789012345678901
numbers are there from 25 to 7? Seven. Test-takers in a hurry may add 2
12345678901234567890123456789012123456789012345678901
these numbers, 5 and 7, and then pick choice (C). Yet this would be too 2
12345678901234567890123456789012123456789012345678901 2
2345678901234567890123456789012123456789012345678901
hasty, for if you recall, zero is also an integer. That makes the correct 2
12345678901234567890123456789012123456789012345678901
12345678901234567890123456789012123456789012345678901 2
12345678901234567890123456789012123456789012345678901 2
12345678901234567890123456789012123456789012345678901 22
answer for this problem 13, choice (B).
1
12345678901234567890123456789012123456789012345678901
This problem typifies the GRE Quant section. The question contains two 2
12345678901234567890123456789012123456789012345678901 2
12345678901234567890123456789012123456789012345678901 2
12345678901234567890123456789012123456789012345678901
basic math terms and in many ways looks deceptively simple. You can 2
12345678901234567890123456789012123456789012345678901
come up with a quick answer, 12, that’s just begging for you to choose it. 2
12345678901234567890123456789012123456789012345678901 22
2345678901234567890123456789012123456789012345678901
1 However, if you take time and use a strong understanding of the basic
12345678901234567890123456789012123456789012345678901 2
12345678901234567890123456789012123456789012345678901
terms integer and inclusive, you will not forget the presence of zero. 2
12345678901234567890123456789012123456789012345678901
2345678901234567890123456789012123456789012345678901
2
2
1
12345678901234567890123456789012123456789012345678901 2
12345678901234567890123456789012123456789012345678901 2
12345678901234567890123456789012123456789012345678901 2 51
123456789012345678901234567890121234567890123456789012
Part III: Section Review

123456789012345678901234567890121234567890123456789012
12345678901234567890123456789012123456789012345678901 2
12345678901234567890123456789012123456789012345678901 2
12345678901234567890123456789012123456789012345678901
The terms divisible and product are often used along with integers. 2
12345678901234567890123456789012123456789012345678901 2
12345678901234567890123456789012123456789012345678901
Divisible is the math term for when an integer can be divided by another 2
12345678901234567890123456789012123456789012345678901 2
12345678901234567890123456789012123456789012345678901
without there being a remainder. For instance, 12 is divisible by 3 since 2
1 12 4 3 5 4, and there is no remainder. In this instance, 3 also has a name, 2
2345678901234567890123456789012123456789012345678901
12345678901234567890123456789012123456789012345678901 2
12345678901234567890123456789012123456789012345678901 2
12345678901234567890123456789012123456789012345678901
divisor, since it evenly divides into 12. When you multiply two numbers 2
12345678901234567890123456789012123456789012345678901
2345678901234567890123456789012123456789012345678901
2
2
1 together, the answer is the product. So 12 is the product of 3 times 4. In
12345678901234567890123456789012123456789012345678901 2
12345678901234567890123456789012123456789012345678901
this case, 3 and 4 are called factors of 12 because their product is 12. 2
12345678901234567890123456789012123456789012345678901 2
12345678901234567890123456789012123456789012345678901
2345678901234567890123456789012123456789012345678901
2
2
1 The factors of 12 are the integers that multiply together to give 12: 1 3 12,
12345678901234567890123456789012123456789012345678901 2
12345678901234567890123456789012123456789012345678901
2 3 6, 3 3 4. Notice that the factors of 12 and the divisors of 12 are the 2
12345678901234567890123456789012123456789012345678901
same. They just involve using a different math operation. Think of it as 2
12345678901234567890123456789012123456789012345678901 2
1 going up a set of stairs or heading down a set of stairs. Either way, it’s the 2
2345678901234567890123456789012123456789012345678901
12345678901234567890123456789012123456789012345678901 2
12345678901234567890123456789012123456789012345678901
same set of stairs. 2
12345678901234567890123456789012123456789012345678901 2
12345678901234567890123456789012123456789012345678901 2
12345678901234567890123456789012123456789012345678901 2
2345678901234567890123456789012123456789012345678901
There is another related term, multiple. The multiple of an integer is 2
12345678901234567890123456789012123456789012345678901
12345678901234567890123456789012123456789012345678901 2

Note
that integer times counting numbers like 1, 2, 3, and up. So the 2
12345678901234567890123456789012123456789012345678901
12345678901234567890123456789012123456789012345678901 2
12345678901234567890123456789012123456789012345678901 2
1 multiples of 3 are 3(3 3 1), 6(3 3 2), 9(3 3 3), 12(3 3 4), 15(3 3 5), 2
12345678901234567890123456789012123456789012345678901
18(3 3 6) . . . and so on. The integer 12(3 3 4) is a multiple of 3, and 2
12345678901234567890123456789012123456789012345678901 2
12345678901234567890123456789012123456789012345678901
it’s also a multiple of 4. 2
12345678901234567890123456789012123456789012345678901 2
2345678901234567890123456789012123456789012345678901
12345678901234567890123456789012123456789012345678901 2
12345678901234567890123456789012123456789012345678901 2
Keeping all these terms in your head can be a bit tricky. It’s like having 2
12345678901234567890123456789012123456789012345678901
12345678901234567890123456789012123456789012345678901
identical triplet cousins whose names are Tad, Todd, and Ted. Don’t lose 2
2
12345678901234567890123456789012123456789012345678901
12345678901234567890123456789012123456789012345678901
heart in this flurry of terms. None of them individually is difficult to grasp, 2
2
12345678901234567890123456789012123456789012345678901
12345678901234567890123456789012123456789012345678901 2
1 but they can be a little confusing if they are all heaped together at the same 2
2345678901234567890123456789012123456789012345678901
12345678901234567890123456789012123456789012345678901 2
12345678901234567890123456789012123456789012345678901
time like in the following problem. 2
1 2
12345678901234567890123456789012123456789012345678901 2
12345678901234567890123456789012123456789012345678901
Column A Column B 2
12345678901234567890123456789012123456789012345678901 2
2345678901234567890123456789012123456789012345678901
12345678901234567890123456789012123456789012345678901 2
1 The greatest The first even 2
12345678901234567890123456789012123456789012345678901 2
12345678901234567890123456789012123456789012345678901
common factor of integer greater than 2
2345678901234567890123456789012123456789012345678901
12345678901234567890123456789012123456789012345678901 2
12345678901234567890123456789012123456789012345678901
18 and 36 434 2
1 2
12345678901234567890123456789012123456789012345678901 2
12345678901234567890123456789012123456789012345678901
The greatest common factor, or GCF, is the largest number that is a factor 2
2345678901234567890123456789012123456789012345678901
12345678901234567890123456789012123456789012345678901
of both 18 and 36. You might notice right off the bat that 36 5 2 3 18, 2
2
12345678901234567890123456789012123456789012345678901 2
12345678901234567890123456789012123456789012345678901
and so 18 must be the GCF. If you don’t catch this, don’t sweat it. Just start 2
12345678901234567890123456789012123456789012345678901 2
1 writing down the factors of 18 and 36.
12345678901234567890123456789012123456789012345678901 2
12345678901234567890123456789012123456789012345678901 2
12345678901234567890123456789012123456789012345678901 2
12345678901234567890123456789012123456789012345678901
Factors of 18: 1, 2, 3, 6, 9, 18 2
12345678901234567890123456789012123456789012345678901 2
2345678901234567890123456789012123456789012345678901
12345678901234567890123456789012123456789012345678901
Factors of 36: 1, 2, 3, 4, 6, 9, 12, 18, 36 2
12345678901234567890123456789012123456789012345678901 2
12345678901234567890123456789012123456789012345678901 2 2
12345678901234567890123456789012123456789012345678901
1 18 is the largest number they share, so the integer value of Column A is 18. 2
12345678901234567890123456789012123456789012345678901 2
2345678901234567890123456789012123456789012345678901
Looking at Column B, you just have to unpack the terms. The first integer
1 greater than 16 (4 3 4) is 17, and the first even integer greater than 16 is 2
12345678901234567890123456789012123456789012345678901 2
12345678901234567890123456789012123456789012345678901 2
12345678901234567890123456789012123456789012345678901
18. Column B also equals 18, so (C) is the
2345678901234567890123456789012123456789012345678901
correct answer. 2
2
1
12345678901234567890123456789012123456789012345678901 2
12345678901234567890123456789012123456789012345678901 2
12345678901234567890123456789012123456789012345678901 2
52 123456789012345678901234567890121234567890123456789012

www.petersons.com
Chapter 4: Quantitative Review

123456789012345678901234567890121234567890123456789012
12345678901234567890123456789012123456789012345678901 2
12345678901234567890123456789012123456789012345678901 2
12345678901234567890123456789012123456789012345678901 2
12345678901234567890123456789012123456789012345678901 2
12345678901234567890123456789012123456789012345678901 2

Tip
12345678901234567890123456789012123456789012345678901 2
Notice that both Columns A and B describe specific numbers that will
12345678901234567890123456789012123456789012345678901 2
12345678901234567890123456789012123456789012345678901
have a definite relationship. Therefore, (D) cannot be the correct answer. 2
12345678901234567890123456789012123456789012345678901 2
12345678901234567890123456789012123456789012345678901 2
12345678901234567890123456789012123456789012345678901 2
12345678901234567890123456789012123456789012345678901
2345678901234567890123456789012123456789012345678901
2
2
1 One final integer-related term to cover concerns prime numbers. Prime
12345678901234567890123456789012123456789012345678901
numbers are divisible by only two integers, themselves and one. Another 2
12345678901234567890123456789012123456789012345678901 2
12345678901234567890123456789012123456789012345678901
way of saying this is to state “a prime number only has two multiples: Itself 2
12345678901234567890123456789012123456789012345678901 2
12345678901234567890123456789012123456789012345678901
and 1.” 11 is a prime number since the only integers that evenly divide into 2
12345678901234567890123456789012123456789012345678901 2
12345678901234567890123456789012123456789012345678901
11 are 11 and 1. Notice that 1 is not a prime number since it has one 2
12345678901234567890123456789012123456789012345678901 2
12345678901234567890123456789012123456789012345678901
multiple (1 3 1) and not two.
2345678901234567890123456789012123456789012345678901
2
2
1
12345678901234567890123456789012123456789012345678901 2
12345678901234567890123456789012123456789012345678901
It is a good idea to be familiar with the smaller prime numbers because you 2
12345678901234567890123456789012123456789012345678901
might need them at your fingertips for a problem. Here’s the first ten: 2, 3, 2
12345678901234567890123456789012123456789012345678901 2
2345678901234567890123456789012123456789012345678901
5, 7, 11, 13, 17, 19, 23, and 29. If you want any more, you’ll have to figure 2
12345678901234567890123456789012123456789012345678901
12345678901234567890123456789012123456789012345678901 2
12345678901234567890123456789012123456789012345678901 2
12345678901234567890123456789012123456789012345678901 22
them out yourself or pay extra.
12345678901234567890123456789012123456789012345678901 2
12345678901234567890123456789012123456789012345678901 2
1 If n equals the number of prime numbers greater than 10 and less
12345678901234567890123456789012123456789012345678901 2
12345678901234567890123456789012123456789012345678901
than 20, then 3n 5 2
12345678901234567890123456789012123456789012345678901 2
2345678901234567890123456789012123456789012345678901
12345678901234567890123456789012123456789012345678901 2
12345678901234567890123456789012123456789012345678901
A. 3 2
12345678901234567890123456789012123456789012345678901 2
12345678901234567890123456789012123456789012345678901
B. 6 2
12345678901234567890123456789012123456789012345678901 2
12345678901234567890123456789012123456789012345678901 22
C. 9
12345678901234567890123456789012123456789012345678901
12345678901234567890123456789012123456789012345678901
D. 12 2
12345678901234567890123456789012123456789012345678901 2
1 E. 15 2
12345678901234567890123456789012123456789012345678901 22
2345678901234567890123456789012123456789012345678901
12345678901234567890123456789012123456789012345678901
1 Before you can find 3n, you must figure out what n is. Looking at the list of 2
12345678901234567890123456789012123456789012345678901
prime numbers in the paragraph above, you can see that there are four 2
12345678901234567890123456789012123456789012345678901
2345678901234567890123456789012123456789012345678901
2
prime numbers between 10 and 20: 11, 13, 17, 19. If n 5 4, then 3n will be 2
12345678901234567890123456789012123456789012345678901
12345678901234567890123456789012123456789012345678901 2
1 (3)(4) 5 12. Therefore, (D) is the correct answer. 2
12345678901234567890123456789012123456789012345678901 2
12345678901234567890123456789012123456789012345678901 2
12345678901234567890123456789012123456789012345678901 22
2345678901234567890123456789012123456789012345678901
12345678901234567890123456789012123456789012345678901 2
1Fractions, or Pretty Much Everything That’s Not
12345678901234567890123456789012123456789012345678901 2
12345678901234567890123456789012123456789012345678901 2
12345678901234567890123456789012123456789012345678901
an Integer 2
12345678901234567890123456789012123456789012345678901 2
12345678901234567890123456789012123456789012345678901 a 2
12345678901234567890123456789012123456789012345678901
A fraction is a number made up of two integers and it looks like this: . 2
12345678901234567890123456789012123456789012345678901 b 2
12345678901234567890123456789012123456789012345678901
The number on top, a, is called the numerator. The number on the bottom, 2
12345678901234567890123456789012123456789012345678901 2
12345678901234567890123456789012123456789012345678901 2
12345678901234567890123456789012123456789012345678901
b, is called the denominator. (If you can come up with a clever way to help 2
12345678901234567890123456789012123456789012345678901 2
12345678901234567890123456789012123456789012345678901 22
2345678901234567890123456789012123456789012345678901
yourself remember that the numerator is on top while the denominator is
12345678901234567890123456789012123456789012345678901
on bottom, you should do so.) A fraction means you are dividing the top 2
12345678901234567890123456789012123456789012345678901 2
12345678901234567890123456789012123456789012345678901
1 by the bottom, so a is the same thing as a 4 b. Since a 5 a 4 b, b cannot 2
2345678901234567890123456789012123456789012345678901
12345678901234567890123456789012123456789012345678901 2
1 equal zero becauseb mathematicians don’t think it makes b 2
12345678901234567890123456789012123456789012345678901 any sense to 2
12345678901234567890123456789012123456789012345678901 2
12345678901234567890123456789012123456789012345678901
divide by 0. I don’t have the exact reasons for this, but in general, 2
2345678901234567890123456789012123456789012345678901 2
1
12345678901234567890123456789012123456789012345678901 2
12345678901234567890123456789012123456789012345678901 2
12345678901234567890123456789012123456789012345678901 2 53
123456789012345678901234567890121234567890123456789012
Part III: Section Review

123456789012345678901234567890121234567890123456789012
12345678901234567890123456789012123456789012345678901 2
12345678901234567890123456789012123456789012345678901 2
12345678901234567890123456789012123456789012345678901
mathematicians are a serious lot and I don’t think they would say this just 2
12345678901234567890123456789012123456789012345678901 2
12345678901234567890123456789012123456789012345678901
to pull our leg. 2
12345678901234567890123456789012123456789012345678901 2
12345678901234567890123456789012123456789012345678901
The GRE math section is going to expect you to do things like add and 2
12345678901234567890123456789012123456789012345678901 2
12345678901234567890123456789012123456789012345678901 2
12345678901234567890123456789012123456789012345678901
divide fractions, so here is a quick review of how to perform these math 2
12345678901234567890123456789012123456789012345678901 2
12345678901234567890123456789012123456789012345678901
operations with fractions.
2345678901234567890123456789012123456789012345678901
2
2
1
12345678901234567890123456789012123456789012345678901 2
12345678901234567890123456789012123456789012345678901 2
12345678901234567890123456789012123456789012345678901
1. Addition 2
12345678901234567890123456789012123456789012345678901
2345678901234567890123456789012123456789012345678901
2
2
1 You can only add fractions that have the same denominator. If you are
12345678901234567890123456789012123456789012345678901 2
12345678901234567890123456789012123456789012345678901
3 2 2
12345678901234567890123456789012123456789012345678901
asked what 1 equals, you have to make the bottoms look the same. In 2
12345678901234567890123456789012123456789012345678901
4 3 2
12345678901234567890123456789012123456789012345678901
math speak, this is called “finding a common denominator.” The common 2
12345678901234567890123456789012123456789012345678901 2
12345678901234567890123456789012123456789012345678901 2
12345678901234567890123456789012123456789012345678901
denominator will be a number that is a multiple of both 4 and 3. 12 is the 2
12345678901234567890123456789012123456789012345678901 2
12345678901234567890123456789012123456789012345678901 2
2345678901234567890123456789012123456789012345678901
smallest common multiple (in math speak, the least common multiple or
LCM). To get 12 as your denominator for both fractions, you multiply the 2
12345678901234567890123456789012123456789012345678901
12345678901234567890123456789012123456789012345678901 2
first denominator by 3 to get 12 and the second denominator by 4 2
12345678901234567890123456789012123456789012345678901
12345678901234567890123456789012123456789012345678901 2
12345678901234567890123456789012123456789012345678901 2
1 to get 12. 2
12345678901234567890123456789012123456789012345678901 2
12345678901234567890123456789012123456789012345678901
However, you can’t just multiply only the denominator by whatever 2
12345678901234567890123456789012123456789012345678901 2
12345678901234567890123456789012123456789012345678901
number you please. You must also multiply the numerator by that same 2
12345678901234567890123456789012123456789012345678901 2
12345678901234567890123456789012123456789012345678901
3 4 2
12345678901234567890123456789012123456789012345678901
number. Multiply the first fraction by and the second one by . 2
12345678901234567890123456789012123456789012345678901
3 4 2
12345678901234567890123456789012123456789012345678901 2
2345678901234567890123456789012123456789012345678901
12345678901234567890123456789012123456789012345678901
This gives you: 2
12345678901234567890123456789012123456789012345678901 2
12345678901234567890123456789012123456789012345678901 2
1 3 3 9 2
2345678901234567890123456789012123456789012345678901
12345678901234567890123456789012123456789012345678901
3 5 2
12345678901234567890123456789012123456789012345678901
4 3 12 2
1 2
12345678901234567890123456789012123456789012345678901 2
12345678901234567890123456789012123456789012345678901
and
2345678901234567890123456789012123456789012345678901
2
2
12345678901234567890123456789012123456789012345678901
12345678901234567890123456789012123456789012345678901
2 4 8 2
1 3 5 2
12345678901234567890123456789012123456789012345678901
3 4 12 2
12345678901234567890123456789012123456789012345678901 2
2345678901234567890123456789012123456789012345678901
12345678901234567890123456789012123456789012345678901 2
1 SD SD
12345678901234567890123456789012123456789012345678901
3 3
3
1
2 4
3 4
5
9
12
1
8
12345678901234567890123456789012123456789012345678901 2
4 12
2
2
12345678901234567890123456789012123456789012345678901 2
12345678901234567890123456789012123456789012345678901 2
12345678901234567890123456789012123456789012345678901
Now that the denominators are the same you can add the numerators, 2
12345678901234567890123456789012123456789012345678901 2
12345678901234567890123456789012123456789012345678901 2
12345678901234567890123456789012123456789012345678901
9 8 8 1 9 17 2
2345678901234567890123456789012123456789012345678901
12345678901234567890123456789012123456789012345678901
1 5 5 2
12345678901234567890123456789012123456789012345678901
12 12 12 12 2
12345678901234567890123456789012123456789012345678901 2
12345678901234567890123456789012123456789012345678901 2 2
1 2. Subtraction
12345678901234567890123456789012123456789012345678901 2
12345678901234567890123456789012123456789012345678901 2
12345678901234567890123456789012123456789012345678901
Like addition, you have to have the same denominator in order to subtract 2
12345678901234567890123456789012123456789012345678901 2
12345678901234567890123456789012123456789012345678901 2
fractions. Once you have that, just subtract the two numbers on top (the
2345678901234567890123456789012123456789012345678901
12345678901234567890123456789012123456789012345678901 2
1 numerators). So, 2
12345678901234567890123456789012123456789012345678901 2
12345678901234567890123456789012123456789012345678901
3 2 9 8 928 1 2
12345678901234567890123456789012123456789012345678901
2345678901234567890123456789012123456789012345678901
2
2
1 2 5 2 5 5
4 3 12 12 12 12
12345678901234567890123456789012123456789012345678901 2
12345678901234567890123456789012123456789012345678901 2
12345678901234567890123456789012123456789012345678901 2
54 123456789012345678901234567890121234567890123456789012

www.petersons.com
Chapter 4: Quantitative Review

123456789012345678901234567890121234567890123456789012
12345678901234567890123456789012123456789012345678901 2
12345678901234567890123456789012123456789012345678901 2
12345678901234567890123456789012123456789012345678901
3. Multiplication 2
12345678901234567890123456789012123456789012345678901 2
12345678901234567890123456789012123456789012345678901
This is a lot easier. Multiply the numerators together and multiply the 2
12345678901234567890123456789012123456789012345678901 2
12345678901234567890123456789012123456789012345678901
denominators together. For example,
2345678901234567890123456789012123456789012345678901
2
2
1
12345678901234567890123456789012123456789012345678901 2
12345678901234567890123456789012123456789012345678901
210 3 230 2
12345678901234567890123456789012123456789012345678901
3 5 2
12345678901234567890123456789012123456789012345678901
4 5 20
2345678901234567890123456789012123456789012345678901
2
2
1
12345678901234567890123456789012123456789012345678901
Fractions in answer choices on the GRE will usually be in their simplest 2
12345678901234567890123456789012123456789012345678901 2
12345678901234567890123456789012123456789012345678901
230 2
12345678901234567890123456789012123456789012345678901
form, and so would not be an answer on the GRE. It would be 2
2345678901234567890123456789012123456789012345678901 2
1 20
12345678901234567890123456789012123456789012345678901 2
12345678901234567890123456789012123456789012345678901
simplified, 2
12345678901234567890123456789012123456789012345678901 2
12345678901234567890123456789012123456789012345678901
10 10
2345678901234567890123456789012123456789012345678901 3 1 2
2
1 230 23 3 23 23
12345678901234567890123456789012123456789012345678901
5 5 3 5 3 1 5 2 or 21 2
12345678901234567890123456789012123456789012345678901
20 2 3 10 2 10 2 2 2 2
12345678901234567890123456789012123456789012345678901 2
12345678901234567890123456789012123456789012345678901 2
2345678901234567890123456789012123456789012345678901
12345678901234567890123456789012123456789012345678901 2
12345678901234567890123456789012123456789012345678901
4. Division 2
12345678901234567890123456789012123456789012345678901
2 7 2
12345678901234567890123456789012123456789012345678901 2
12345678901234567890123456789012123456789012345678901
Dividing two fractions can look like this, 4 , or it can look like this, 2
12345678901234567890123456789012123456789012345678901 22
5 8
1
12345678901234567890123456789012123456789012345678901
2 2
12345678901234567890123456789012123456789012345678901 2
12345678901234567890123456789012123456789012345678901
5 2
2345678901234567890123456789012123456789012345678901
12345678901234567890123456789012123456789012345678901 2
12345678901234567890123456789012123456789012345678901
7 2
12345678901234567890123456789012123456789012345678901 22
12345678901234567890123456789012123456789012345678901
8
12345678901234567890123456789012123456789012345678901 2
These two expressions are the same. You solve them with a two-step 2
12345678901234567890123456789012123456789012345678901
12345678901234567890123456789012123456789012345678901 2
12345678901234567890123456789012123456789012345678901 2
12345678901234567890123456789012123456789012345678901 7

8 2
1 process. First, flip the fraction you are dividing by, so that . In math 2
2345678901234567890123456789012123456789012345678901
12345678901234567890123456789012123456789012345678901 8 7
speak, this flipping is called “finding the reciprocal.” Now multiply the 2
2
12345678901234567890123456789012123456789012345678901 2
1 flipped fraction and the other fraction. So,
12345678901234567890123456789012123456789012345678901 2
12345678901234567890123456789012123456789012345678901 2
12345678901234567890123456789012123456789012345678901 2
12345678901234567890123456789012123456789012345678901
2 2
12345678901234567890123456789012123456789012345678901 2
12345678901234567890123456789012123456789012345678901
5 2 8 16 2
12345678901234567890123456789012123456789012345678901
5 3 5 2
12345678901234567890123456789012123456789012345678901 22
2345678901234567890123456789012123456789012345678901
7 5 7 35
12345678901234567890123456789012123456789012345678901 2
1 8
12345678901234567890123456789012123456789012345678901 2
12345678901234567890123456789012123456789012345678901
A catchy phrase to remember the dividing procedure is, “When dividing, 2
12345678901234567890123456789012123456789012345678901 2
12345678901234567890123456789012123456789012345678901
don’t ask why; just flip it over and multiply.” If you forget it, any 2
12345678901234567890123456789012123456789012345678901 2
12345678901234567890123456789012123456789012345678901
14-year-old kid can remind you of this phrase. 2
12345678901234567890123456789012123456789012345678901 2
2345678901234567890123456789012123456789012345678901
12345678901234567890123456789012123456789012345678901
Fractions also come in hybrid form. They’re called mixed fractions, and 2
2
12345678901234567890123456789012123456789012345678901 2
12345678901234567890123456789012123456789012345678901 2
12345678901234567890123456789012123456789012345678901
2 2
1 they look like 37, which means 3 1 7. In order to do any arithmetic 2
12345678901234567890123456789012123456789012345678901 22
2345678901234567890123456789012123456789012345678901
12345678901234567890123456789012123456789012345678901
operation like the ones discussed above, mixed fractions should be 2
12345678901234567890123456789012123456789012345678901 2
12345678901234567890123456789012123456789012345678901
1 converted into pure fractions. You might be able to multiply the mixed 2
2345678901234567890123456789012123456789012345678901
12345678901234567890123456789012123456789012345678901 2
1 fractions 32 with 43, but it would be pretty messy. 2
12345678901234567890123456789012123456789012345678901 2
12345678901234567890123456789012123456789012345678901 2
7 5
12345678901234567890123456789012123456789012345678901 2
12345678901234567890123456789012123456789012345678901 2
12345678901234567890123456789012123456789012345678901 2
12345678901234567890123456789012123456789012345678901 2
12345678901234567890123456789012123456789012345678901 2 55
123456789012345678901234567890121234567890123456789012
Part III: Section Review

123456789012345678901234567890121234567890123456789012
12345678901234567890123456789012123456789012345678901 2
12345678901234567890123456789012123456789012345678901 2
12345678901234567890123456789012123456789012345678901
To convert a mixed fraction into a pure fraction, first multiply the integer 2
12345678901234567890123456789012123456789012345678901 2
12345678901234567890123456789012123456789012345678901
out front by the denominator, then add that product to the numerator. 2
12345678901234567890123456789012123456789012345678901 2
12345678901234567890123456789012123456789012345678901 2
12345678901234567890123456789012123456789012345678901
2 2
12345678901234567890123456789012123456789012345678901
3 2
12345678901234567890123456789012123456789012345678901
7 2
12345678901234567890123456789012123456789012345678901 2
12345678901234567890123456789012123456789012345678901
2345678901234567890123456789012123456789012345678901
2
2
1
12345678901234567890123456789012123456789012345678901
3 3 7 5 21 (multiplying the integer 3 with the denominator 7) 2
12345678901234567890123456789012123456789012345678901 2
12345678901234567890123456789012123456789012345678901
21 1 2 5 23 (adding the product to the numerator 2) 2
12345678901234567890123456789012123456789012345678901
2345678901234567890123456789012123456789012345678901
2
2
1 23
12345678901234567890123456789012123456789012345678901
5 2
12345678901234567890123456789012123456789012345678901
7 2
12345678901234567890123456789012123456789012345678901 2
12345678901234567890123456789012123456789012345678901
2345678901234567890123456789012123456789012345678901
2
2
1 Voila, you have it.
12345678901234567890123456789012123456789012345678901 2
12345678901234567890123456789012123456789012345678901 2
12345678901234567890123456789012123456789012345678901 2
12345678901234567890123456789012123456789012345678901 2

Note
12345678901234567890123456789012123456789012345678901 2
2345678901234567890123456789012123456789012345678901
All integers can be expressed as a fraction. Place the integer in the
numerator and a 1 in the denominator. For instance, 7 in fractional form 2
12345678901234567890123456789012123456789012345678901
12345678901234567890123456789012123456789012345678901 2
12345678901234567890123456789012123456789012345678901
7 7 2
12345678901234567890123456789012123456789012345678901 2
is . If you think about it, 7 1
12345678901234567890123456789012123456789012345678901
5 4 5 7 should make sense to you. 2
1 1 1 2
12345678901234567890123456789012123456789012345678901 2
12345678901234567890123456789012123456789012345678901 2
12345678901234567890123456789012123456789012345678901
3x 3 2 4 2
2345678901234567890123456789012123456789012345678901
12345678901234567890123456789012123456789012345678901
If 4 5 4 then 3 5 2
12345678901234567890123456789012123456789012345678901
2 8 3 4x 2
12345678901234567890123456789012123456789012345678901 2
12345678901234567890123456789012123456789012345678901
2 2
12345678901234567890123456789012123456789012345678901 2
12345678901234567890123456789012123456789012345678901
A. 2
12345678901234567890123456789012123456789012345678901
5 2
12345678901234567890123456789012123456789012345678901 2
12345678901234567890123456789012123456789012345678901
2 2
1 B. 2
12345678901234567890123456789012123456789012345678901 2
2345678901234567890123456789012123456789012345678901
3
12345678901234567890123456789012123456789012345678901 2 2
1 C. 1
12345678901234567890123456789012123456789012345678901 2
12345678901234567890123456789012123456789012345678901
2345678901234567890123456789012123456789012345678901
2
2
12345678901234567890123456789012123456789012345678901
8
12345678901234567890123456789012123456789012345678901
D. 2
1 3 2
12345678901234567890123456789012123456789012345678901 2
12345678901234567890123456789012123456789012345678901 2
E. 4
12345678901234567890123456789012123456789012345678901 2
2345678901234567890123456789012123456789012345678901
12345678901234567890123456789012123456789012345678901 2
1 Here’s a basic GRE question that seeks to overwhelm you with a bunch of 2
12345678901234567890123456789012123456789012345678901 2
12345678901234567890123456789012123456789012345678901
fractions and arithmetic flitting about. You can’t solve the second equation 2
2345678901234567890123456789012123456789012345678901
12345678901234567890123456789012123456789012345678901
until you learn the value of x, which can be found by solving the first 2
2
12345678901234567890123456789012123456789012345678901 2
12345678901234567890123456789012123456789012345678901 2
12345678901234567890123456789012123456789012345678901
equation:
2
1
12345678901234567890123456789012123456789012345678901 2
2345678901234567890123456789012123456789012345678901
2
12345678901234567890123456789012123456789012345678901
3x 3
2
12345678901234567890123456789012123456789012345678901
4 54
2
12345678901234567890123456789012123456789012345678901
2 8
2
1
2345678901234567890123456789012123456789012345678901
12345678901234567890123456789012123456789012345678901 2
12345678901234567890123456789012123456789012345678901
3x 3 3x 8 24x 2
12345678901234567890123456789012123456789012345678901
4 5 3 5 5 4x (This solves the left side of the 2
12345678901234567890123456789012123456789012345678901
2 8 2 3 6 2
1 equation.) 2
2345678901234567890123456789012123456789012345678901
12345678901234567890123456789012123456789012345678901 2
1 4x 5 4 2
12345678901234567890123456789012123456789012345678901 2
12345678901234567890123456789012123456789012345678901
x51 2
12345678901234567890123456789012123456789012345678901
2345678901234567890123456789012123456789012345678901
2
2
1
12345678901234567890123456789012123456789012345678901 2
12345678901234567890123456789012123456789012345678901 2
12345678901234567890123456789012123456789012345678901 2
56 123456789012345678901234567890121234567890123456789012

www.petersons.com
Chapter 4: Quantitative Review

123456789012345678901234567890121234567890123456789012
12345678901234567890123456789012123456789012345678901 2
12345678901234567890123456789012123456789012345678901 2
12345678901234567890123456789012123456789012345678901
Plugging x 5 1 into the second equation will get you: 2
12345678901234567890123456789012123456789012345678901 2
12345678901234567890123456789012123456789012345678901 2
12345678901234567890123456789012123456789012345678901
2 4 2
12345678901234567890123456789012123456789012345678901
2345678901234567890123456789012123456789012345678901
2
2
1 3
3 4x
5
12345678901234567890123456789012123456789012345678901 2
12345678901234567890123456789012123456789012345678901 2
12345678901234567890123456789012123456789012345678901
or 2
12345678901234567890123456789012123456789012345678901
2345678901234567890123456789012123456789012345678901
2
2
1
12345678901234567890123456789012123456789012345678901
2 4 8 2 2
12345678901234567890123456789012123456789012345678901
3
~ !
5 5 2
12345678901234567890123456789012123456789012345678901
3 4 1 12 3 2
12345678901234567890123456789012123456789012345678901
2345678901234567890123456789012123456789012345678901
2
2
1 4
12345678901234567890123456789012123456789012345678901
You might have noticed that 5 1, so multiplying out the second equation 2
12345678901234567890123456789012123456789012345678901
4 2
12345678901234567890123456789012123456789012345678901 2
12345678901234567890123456789012123456789012345678901
wasn’t necessary. Either way, you end up with
2345678901234567890123456789012123456789012345678901
(B), the correct answer. 2
2
1
12345678901234567890123456789012123456789012345678901 2
12345678901234567890123456789012123456789012345678901 2
12345678901234567890123456789012123456789012345678901 2
12345678901234567890123456789012123456789012345678901
Decimals (Fractions in Disguise!) 2
12345678901234567890123456789012123456789012345678901 22
2345678901234567890123456789012123456789012345678901
12345678901234567890123456789012123456789012345678901
Decimal form is another way to write a fraction. Decimals can easily be 2
12345678901234567890123456789012123456789012345678901
converted into fractions, since every decimal has a denominator that is a 2
12345678901234567890123456789012123456789012345678901
12345678901234567890123456789012123456789012345678901 2
12345678901234567890123456789012123456789012345678901 2
1 multiple of 10. The decimal 0.7 is the fraction 7 , while the decimal 0.765 2
12345678901234567890123456789012123456789012345678901 2
12345678901234567890123456789012123456789012345678901 10 2
12345678901234567890123456789012123456789012345678901
765 2
2345678901234567890123456789012123456789012345678901
12345678901234567890123456789012123456789012345678901
is . You take the number and make it the numerator, then you look 2
12345678901234567890123456789012123456789012345678901
1,000 2
12345678901234567890123456789012123456789012345678901 2
12345678901234567890123456789012123456789012345678901 22
and count how many spaces there are to the right of the decimal point.
12345678901234567890123456789012123456789012345678901
This number equals the number of zeroes you need to place after a 1 in the 2
12345678901234567890123456789012123456789012345678901
denominator. 0.7 has only one space to the right, so you put only one zero 2
12345678901234567890123456789012123456789012345678901
12345678901234567890123456789012123456789012345678901 2
after the 1, giving you 10 in the denominator. The decimal 0.765 has three 2
12345678901234567890123456789012123456789012345678901
1 2
12345678901234567890123456789012123456789012345678901
spaces, so you put three zeroes, giving you 1,000 in the denominator. 2
12345678901234567890123456789012123456789012345678901 2
12345678901234567890123456789012123456789012345678901 2
12345678901234567890123456789012123456789012345678901
All the numbers to the right of the decimal point have place values that end 2
12345678901234567890123456789012123456789012345678901 2
12345678901234567890123456789012123456789012345678901
in the suffix “-th.” The decimal 0.7 is expressed “seven tenths,” while 0.25 2
12345678901234567890123456789012123456789012345678901 2
12345678901234567890123456789012123456789012345678901
is “twenty-five hundredths.” Place values on either side of the decimal 2
12345678901234567890123456789012123456789012345678901 2
12345678901234567890123456789012123456789012345678901
point might crop up, so it’s good to have a quick review: 2
12345678901234567890123456789012123456789012345678901 22
2345678901234567890123456789012123456789012345678901
12345678901234567890123456789012123456789012345678901 2
1 Take the number 934.167:
12345678901234567890123456789012123456789012345678901 2
12345678901234567890123456789012123456789012345678901
12345678901234567890123456789011234567 2
12345678901234567890123456789012123456789012345678901
9 is in the hundreds place, so the 9 means1
123456789012345678901234567890 11234567
900 2
12345678901234567890123456789012123456789012345678901
123456789012345678901234567890 1234567 2
12345678901234567890123456789012123456789012345678901
12345678901234567890123456789011234567 2
12345678901234567890123456789012123456789012345678901
3 is in the tens place, so the 3 means 30 2
12345678901234567890123456789012123456789012345678901 2
2345678901234567890123456789012123456789012345678901
12345678901234567890123456789012123456789012345678901 2
12345678901234567890123456789012123456789012345678901
4 is in the units place, so the 4 means 4 2
12345678901234567890123456789012123456789012345678901 22
12345678901234567890123456789012123456789012345678901 2
1 1 is in the tenths place, so the 1 means 1
12345678901234567890123456789012123456789012345678901 2
12345678901234567890123456789012123456789012345678901 10 2
12345678901234567890123456789012123456789012345678901 2
12345678901234567890123456789012123456789012345678901 6 2
12345678901234567890123456789012123456789012345678901 2
6 is in the hundredths place, so the 6 means
2345678901234567890123456789012123456789012345678901
12345678901234567890123456789012123456789012345678901 2
1 100 2
12345678901234567890123456789012123456789012345678901 2
12345678901234567890123456789012123456789012345678901 7 2
12345678901234567890123456789012123456789012345678901
7 is in the thousands place, so the 7
2345678901234567890123456789012123456789012345678901
means 2
2
1 1,000
12345678901234567890123456789012123456789012345678901 2
12345678901234567890123456789012123456789012345678901 22
12345678901234567890123456789012123456789012345678901
123456789012345678901234567890121234567890123456789012 57
Part III: Section Review

123456789012345678901234567890121234567890123456789012
12345678901234567890123456789012123456789012345678901 2
12345678901234567890123456789012123456789012345678901 2
12345678901234567890123456789012123456789012345678901 2
12345678901234567890123456789012123456789012345678901 2
12345678901234567890123456789012123456789012345678901 2

Alert!
A tricky part about place values and decimals is that there is no “unit-ths”
12345678901234567890123456789012123456789012345678901 2
12345678901234567890123456789012123456789012345678901
value. In this example, the 4 is “four units” and the number to the left of 2
12345678901234567890123456789012123456789012345678901 2
12345678901234567890123456789012123456789012345678901
it is the tens. However, the number immediately to the right of 4—over 2
12345678901234567890123456789012123456789012345678901 2
12345678901234567890123456789012123456789012345678901
the decimal point divide—is the tenths, not unit-ths. 2
12345678901234567890123456789012123456789012345678901
2345678901234567890123456789012123456789012345678901
2
2
1
12345678901234567890123456789012123456789012345678901 2
12345678901234567890123456789012123456789012345678901 2
12345678901234567890123456789012123456789012345678901
When you add and subtract with decimals you need to make sure you line 2
12345678901234567890123456789012123456789012345678901 2
12345678901234567890123456789012123456789012345678901
up the decimal points. For instance, 345.4 minus 202.45 is: 2
12345678901234567890123456789012123456789012345678901 2
12345678901234567890123456789012123456789012345678901 2
12345678901234567890123456789012123456789012345678901
345.4 2
12345678901234567890123456789012123456789012345678901
2345678901234567890123456789012123456789012345678901
2
2
1 2 202.45
12345678901234567890123456789012123456789012345678901 2
12345678901234567890123456789012123456789012345678901 2
12345678901234567890123456789012123456789012345678901
When multiplying decimals, do the multiplication as if the decimal points 2
12345678901234567890123456789012123456789012345678901 2
12345678901234567890123456789012123456789012345678901
weren’t there. Just ignore them for starters. Once you have the product, 2
12345678901234567890123456789012123456789012345678901 2
12345678901234567890123456789012123456789012345678901
count how many decimal places you have behind the decimal in both 2
12345678901234567890123456789012123456789012345678901 2
12345678901234567890123456789012123456789012345678901
factors, and then put the decimal that many places into the product. If that 2
2345678901234567890123456789012123456789012345678901
12345678901234567890123456789012123456789012345678901 2
1 sounds confusing, in application it’s not. For instance, 3.5 3 0.5 → 175 2
12345678901234567890123456789012123456789012345678901
(multiplying forgetting the decimals). Or, there are two places behind the 2
12345678901234567890123456789012123456789012345678901 2
12345678901234567890123456789012123456789012345678901 2
12345678901234567890123456789012123456789012345678901 2
2345678901234567890123456789012123456789012345678901
decimal in the two factors, one for each factor. Putting two numbers
behind the decimal in the product gives you 1.75. If you change the initial 2
12345678901234567890123456789012123456789012345678901
12345678901234567890123456789012123456789012345678901 2
12345678901234567890123456789012123456789012345678901 2
12345678901234567890123456789012123456789012345678901 2
numbers to 0.35 and 0.005, you still multiply and get 175 to start. But
now there are five spaces to the left of the decimal point (two in 0.35 and 2
12345678901234567890123456789012123456789012345678901
12345678901234567890123456789012123456789012345678901 2
12345678901234567890123456789012123456789012345678901 2
12345678901234567890123456789012123456789012345678901 2
three in 0.005), so your final answer will be 0.00175 for that problem.
2
1
12345678901234567890123456789012123456789012345678901
To divide a decimal by a decimal, first move the decimal in the divisor until 2
12345678901234567890123456789012123456789012345678901 2
12345678901234567890123456789012123456789012345678901
it is a whole number, and then move the decimal point as many spaces in 2
12345678901234567890123456789012123456789012345678901 2
12345678901234567890123456789012123456789012345678901
the number being divided. The expression 2.3 4 0.5 would look like this 2
12345678901234567890123456789012123456789012345678901 2
12345678901234567890123456789012123456789012345678901
= 2
12345678901234567890123456789012123456789012345678901 2
5 23.0 in long division form. You would have to move the decimal one
12345678901234567890123456789012123456789012345678901
space to the left to transform 2.3 into the integer 23, so you must move 2
12345678901234567890123456789012123456789012345678901 2
2345678901234567890123456789012123456789012345678901
the divisor one space to the left as well, making 0.5 into 5. Now do the 2
12345678901234567890123456789012123456789012345678901
12345678901234567890123456789012123456789012345678901 2
1 division, and put the decimal in the quotient right where it lies in the 2
12345678901234567890123456789012123456789012345678901 2
12345678901234567890123456789012123456789012345678901
4.6 2
12345678901234567890123456789012123456789012345678901 2
12345678901234567890123456789012123456789012345678901 2
12345678901234567890123456789012123456789012345678901
number being divided, 5q23.0. 2
12345678901234567890123456789012123456789012345678901 2
12345678901234567890123456789012123456789012345678901 2
12345678901234567890123456789012123456789012345678901 2
2345678901234567890123456789012123456789012345678901
12345678901234567890123456789012123456789012345678901 2
12345678901234567890123456789012123456789012345678901 2
12345678901234567890123456789012123456789012345678901 2 2
1
12345678901234567890123456789012123456789012345678901 2
12345678901234567890123456789012123456789012345678901 2
12345678901234567890123456789012123456789012345678901 2
12345678901234567890123456789012123456789012345678901 2
12345678901234567890123456789012123456789012345678901 2
12345678901234567890123456789012123456789012345678901 2
2345678901234567890123456789012123456789012345678901
2
1
12345678901234567890123456789012123456789012345678901 2
12345678901234567890123456789012123456789012345678901 2
12345678901234567890123456789012123456789012345678901 2
12345678901234567890123456789012123456789012345678901 2
12345678901234567890123456789012123456789012345678901 2
12345678901234567890123456789012123456789012345678901 2
12345678901234567890123456789012123456789012345678901 2
58 123456789012345678901234567890121234567890123456789012

www.petersons.com
Chapter 4: Quantitative Review

123456789012345678901234567890121234567890123456789012
12345678901234567890123456789012123456789012345678901 2
12345678901234567890123456789012123456789012345678901 2
12345678901234567890123456789012123456789012345678901 a 2
12345678901234567890123456789012123456789012345678901
If you need to convert a fraction into a decimal, remember that means 2
12345678901234567890123456789012123456789012345678901 b 2
12345678901234567890123456789012123456789012345678901 2
12345678901234567890123456789012123456789012345678901
a 4 b. So if you are given a fraction and you need to know its value in 2
1 decimal form, divide the top by the bottom, and you have it. For example, 2
2345678901234567890123456789012123456789012345678901
12345678901234567890123456789012123456789012345678901 2
12345678901234567890123456789012123456789012345678901 2
12345678901234567890123456789012123456789012345678901
2 2
12345678901234567890123456789012123456789012345678901
if you need to know what is in decimal form,
2345678901234567890123456789012123456789012345678901
divide 2 by 7, 2
2
1 7
12345678901234567890123456789012123456789012345678901 2
12345678901234567890123456789012123456789012345678901
0.28 2
12345678901234567890123456789012123456789012345678901 2
12345678901234567890123456789012123456789012345678901
2345678901234567890123456789012123456789012345678901
2
2
1 7 q 2.00
12345678901234567890123456789012123456789012345678901 2
12345678901234567890123456789012123456789012345678901
14 2
12345678901234567890123456789012123456789012345678901 2
12345678901234567890123456789012123456789012345678901
2345678901234567890123456789012123456789012345678901
2
2
1 60
12345678901234567890123456789012123456789012345678901 2
12345678901234567890123456789012123456789012345678901
56 2
12345678901234567890123456789012123456789012345678901 2
12345678901234567890123456789012123456789012345678901
4 2
12345678901234567890123456789012123456789012345678901 22
2345678901234567890123456789012123456789012345678901
12345678901234567890123456789012123456789012345678901
2
12345678901234567890123456789012123456789012345678901 2
12345678901234567890123456789012123456789012345678901
' 0.28 2
12345678901234567890123456789012123456789012345678901 22
7
12345678901234567890123456789012123456789012345678901 2
1
12345678901234567890123456789012123456789012345678901
Column A Column B
2
12345678901234567890123456789012123456789012345678901 2
12345678901234567890123456789012123456789012345678901
d, c, e are distinct prime numbers and 2
2345678901234567890123456789012123456789012345678901
12345678901234567890123456789012123456789012345678901 2
12345678901234567890123456789012123456789012345678901
d 1 e 5 c. They are also part of the six 2
12345678901234567890123456789012123456789012345678901 2
12345678901234567890123456789012123456789012345678901
digit number 54de.3c. 2
12345678901234567890123456789012123456789012345678901 22
12345678901234567890123456789012123456789012345678901
12345678901234567890123456789012123456789012345678901
The value in the The value in the 2
12345678901234567890123456789012123456789012345678901 2
12345678901234567890123456789012123456789012345678901 22
tens place in the tenths place in the
1
12345678901234567890123456789012123456789012345678901
6-digit number 6-digit number 2
12345678901234567890123456789012123456789012345678901 2
12345678901234567890123456789012123456789012345678901 2
12345678901234567890123456789012123456789012345678901
The test is trying to confuse you with math speak. Don’t let it. Distinct 2
12345678901234567890123456789012123456789012345678901
2345678901234567890123456789012123456789012345678901
2
prime numbers are prime numbers that are different from each other, so 2
12345678901234567890123456789012123456789012345678901
12345678901234567890123456789012123456789012345678901 2
1 d Þ e Þ c. You can also infer that d, c, e must be 2, 3, 5, or 7 since those are 2
12345678901234567890123456789012123456789012345678901 2
12345678901234567890123456789012123456789012345678901
all the single digit prime numbers. 2
12345678901234567890123456789012123456789012345678901 22
2345678901234567890123456789012123456789012345678901
12345678901234567890123456789012123456789012345678901
1 Let’s look at Column A and try to pin it down. The value in the tens place 2
12345678901234567890123456789012123456789012345678901
is d. With what you know, d 1 e 5 c could be 2 1 3 5 5 or 2 1 5 5 7, but 2
12345678901234567890123456789012123456789012345678901 2
2345678901234567890123456789012123456789012345678901
there’s no way to tell which equation is valid. Therefore, d could be 2, 3, 2
12345678901234567890123456789012123456789012345678901
12345678901234567890123456789012123456789012345678901 2
12345678901234567890123456789012123456789012345678901 2
12345678901234567890123456789012123456789012345678901 22
or 5. That narrows it down as much as possible, so it’s time to turn to
1 Column B. The tenths place is the one just to the right of the decimal,
12345678901234567890123456789012123456789012345678901 2
12345678901234567890123456789012123456789012345678901 2
12345678901234567890123456789012123456789012345678901
which is 3. 2
12345678901234567890123456789012123456789012345678901 2
12345678901234567890123456789012123456789012345678901
This gives us three possibilities. Depending on d’s value, it could be less 2
12345678901234567890123456789012123456789012345678901 22
2345678901234567890123456789012123456789012345678901
12345678901234567890123456789012123456789012345678901
than, equal to, or greater than 3. This makes Column A less than, equal to,
2
12345678901234567890123456789012123456789012345678901
or greater than Column B, depending on what value you use for d. 2
12345678901234567890123456789012123456789012345678901
1 2
12345678901234567890123456789012123456789012345678901
Therefore, choice (D) is the correct answer. 2
12345678901234567890123456789012123456789012345678901 2
12345678901234567890123456789012123456789012345678901 2
12345678901234567890123456789012123456789012345678901 2
12345678901234567890123456789012123456789012345678901 2
12345678901234567890123456789012123456789012345678901 2
12345678901234567890123456789012123456789012345678901 2
12345678901234567890123456789012123456789012345678901 2
12345678901234567890123456789012123456789012345678901 2 59
123456789012345678901234567890121234567890123456789012
Part III: Section Review

123456789012345678901234567890121234567890123456789012
12345678901234567890123456789012123456789012345678901 2
12345678901234567890123456789012123456789012345678901 2
12345678901234567890123456789012123456789012345678901
Exponents(Those Tiny Raised Numbers) and Roots 2
12345678901234567890123456789012123456789012345678901 2
12345678901234567890123456789012123456789012345678901 2
12345678901234567890123456789012123456789012345678901
Exponents can be a little weird, and that’s putting it nicely. A basic 2
12345678901234567890123456789012123456789012345678901
looks like 4 2
2345678901234567890123456789012123456789012345678901
exponent , with the integer 4 called the base and the number 2 2
2
1
12345678901234567890123456789012123456789012345678901
called the power, or exponent. The expression 4 is math shorthand for 2
2
12345678901234567890123456789012123456789012345678901 2
12345678901234567890123456789012123456789012345678901
4 3 4, which might make you think people have to be in a real hurry to use 2
12345678901234567890123456789012123456789012345678901 2
12345678901234567890123456789012123456789012345678901
that shorthand. Exponents come in handy when you deal with numbers 2
12345678901234567890123456789012123456789012345678901 2
12345678901234567890123456789012123456789012345678901
like 4350, since writing out 350 fours is a bit time-consuming. 2
12345678901234567890123456789012123456789012345678901 2
12345678901234567890123456789012123456789012345678901
2345678901234567890123456789012123456789012345678901
2
2
1 While you might be using some arithmetic—adding, subtracting, fractions,
12345678901234567890123456789012123456789012345678901
and decimals—in your everyday life, you’re usually not walking around 2
12345678901234567890123456789012123456789012345678901 2
12345678901234567890123456789012123456789012345678901 2
12345678901234567890123456789012123456789012345678901
raising values to the n th
power. Here is a condensed version of how 2
1 exponents work. Look it over, play with them (plug in numbers and such), 2
2345678901234567890123456789012123456789012345678901
12345678901234567890123456789012123456789012345678901 2
12345678901234567890123456789012123456789012345678901 2
12345678901234567890123456789012123456789012345678901
and get comfortable with them. 2
12345678901234567890123456789012123456789012345678901 2
12345678901234567890123456789012123456789012345678901 2
2345678901234567890123456789012123456789012345678901
12345678901234567890123456789012123456789012345678901 2
12345678901234567890123456789012123456789012345678901
Exponent Refresher 2
12345678901234567890123456789012123456789012345678901 2
12345678901234567890123456789012123456789012345678901
0 2
12345678901234567890123456789012123456789012345678901
x 5 1 2
1 1 2
12345678901234567890123456789012123456789012345678901
x 5 x 2
12345678901234567890123456789012123456789012345678901 2
12345678901234567890123456789012123456789012345678901
2
x 5x3x 2
2345678901234567890123456789012123456789012345678901
12345678901234567890123456789012123456789012345678901 2
12345678901234567890123456789012123456789012345678901
x3 5 x 3 x 3 x 2
12345678901234567890123456789012123456789012345678901 2
12345678901234567890123456789012123456789012345678901 2
12345678901234567890123456789012123456789012345678901
You get the picture. But just for good measure, here’s one more: 2
12345678901234567890123456789012123456789012345678901 2
12345678901234567890123456789012123456789012345678901 2
12345678901234567890123456789012123456789012345678901 2
3 4
xx 5 x
12345678901234567890123456789012123456789012345678901 2 2
1 Exponents can also be negative.
12345678901234567890123456789012123456789012345678901 2
12345678901234567890123456789012123456789012345678901 2
12345678901234567890123456789012123456789012345678901
1 2
12345678901234567890123456789012123456789012345678901
x 21
5 2
12345678901234567890123456789012123456789012345678901
x 2
12345678901234567890123456789012123456789012345678901 2
2345678901234567890123456789012123456789012345678901
12345678901234567890123456789012123456789012345678901 2
1 22 1 1 2
12345678901234567890123456789012123456789012345678901
x 5 5 2
12345678901234567890123456789012123456789012345678901 2
2 x 3 x
x
12345678901234567890123456789012123456789012345678901 2
2345678901234567890123456789012123456789012345678901
12345678901234567890123456789012123456789012345678901 2
1 You can add and subtract numbers with exponents if they have the same 2
12345678901234567890123456789012123456789012345678901 2
12345678901234567890123456789012123456789012345678901
base and are raised to the same power. 2
2345678901234567890123456789012123456789012345678901
12345678901234567890123456789012123456789012345678901 2
12345678901234567890123456789012123456789012345678901
2 2 2 2
12345678901234567890123456789012123456789012345678901 2
3x 1 2x 5 5x
2
12345678901234567890123456789012123456789012345678901 2
1 2x2 2 x2 5 x2
2345678901234567890123456789012123456789012345678901
12345678901234567890123456789012123456789012345678901 2
12345678901234567890123456789012123456789012345678901 2
12345678901234567890123456789012123456789012345678901 2
3 2 3
but 3x 1 3x Þ 6x since they are not raised to the same power.
12345678901234567890123456789012123456789012345678901 2 2
1 You can also multiply and divide numbers if they have the same base.
12345678901234567890123456789012123456789012345678901 2
12345678901234567890123456789012123456789012345678901 2
12345678901234567890123456789012123456789012345678901
a b
x x 5x a1b
2
12345678901234567890123456789012123456789012345678901 2
12345678901234567890123456789012123456789012345678901 2
12345678901234567890123456789012123456789012345678901 2
2345678901234567890123456789012123456789012345678901
For instance, x 2
3 x 3
5 x 5
. This makes sense if you see that x 2
3 x 3
5
2
1 (x 3 x)(x 3 x 3 x) 5 x5.
12345678901234567890123456789012123456789012345678901 2
12345678901234567890123456789012123456789012345678901 2
12345678901234567890123456789012123456789012345678901
a
2345678901234567890123456789012123456789012345678901
2
2
1 x a 2 b
12345678901234567890123456789012123456789012345678901
5x 2
12345678901234567890123456789012123456789012345678901
x b
2
12345678901234567890123456789012123456789012345678901 2
60 123456789012345678901234567890121234567890123456789012

www.petersons.com
Chapter 4: Quantitative Review

123456789012345678901234567890121234567890123456789012
12345678901234567890123456789012123456789012345678901 2
12345678901234567890123456789012123456789012345678901 2
12345678901234567890123456789012123456789012345678901
For example, 2
12345678901234567890123456789012123456789012345678901 2
12345678901234567890123456789012123456789012345678901 2
12345678901234567890123456789012123456789012345678901
3 2
12345678901234567890123456789012123456789012345678901
x 3 2
2345678901234567890123456789012123456789012345678901
2
2
1
2
5 x 5 x
12345678901234567890123456789012123456789012345678901
x2 2
12345678901234567890123456789012123456789012345678901 2
12345678901234567890123456789012123456789012345678901 2
12345678901234567890123456789012123456789012345678901
This also makes sense if you see that
2345678901234567890123456789012123456789012345678901
2
2
1
12345678901234567890123456789012123456789012345678901 2
12345678901234567890123456789012123456789012345678901
x 3
xxx 2
12345678901234567890123456789012123456789012345678901
5 5x 2
12345678901234567890123456789012123456789012345678901
x 2 xx
2345678901234567890123456789012123456789012345678901
2
2
1
12345678901234567890123456789012123456789012345678901 2
12345678901234567890123456789012123456789012345678901
You can also raise a number with an exponent to a power. In gambling 2
12345678901234567890123456789012123456789012345678901 2
12345678901234567890123456789012123456789012345678901
terms, this would be like saying, “I’ll see your exponent, and raise you 2
2345678901234567890123456789012123456789012345678901 2
1 another.”
12345678901234567890123456789012123456789012345678901 2
12345678901234567890123456789012123456789012345678901 2
12345678901234567890123456789012123456789012345678901 2
12345678901234567890123456789012123456789012345678901 2
a b ab
(x ) 5 x
12345678901234567890123456789012123456789012345678901 22
2345678901234567890123456789012123456789012345678901
12345678901234567890123456789012123456789012345678901
12345678901234567890123456789012123456789012345678901
So, 2
12345678901234567890123456789012123456789012345678901 2
12345678901234567890123456789012123456789012345678901
2 3 6 23 3 29 1 2
12345678901234567890123456789012123456789012345678901
(x ) 5 x or (x ) 5 x 5 9 2
1 x 2
12345678901234567890123456789012123456789012345678901 2
12345678901234567890123456789012123456789012345678901
Column A Column B 2
12345678901234567890123456789012123456789012345678901 2
2345678901234567890123456789012123456789012345678901
12345678901234567890123456789012123456789012345678901 2
12345678901234567890123456789012123456789012345678901
a and b are positive distinct integers. 2
12345678901234567890123456789012123456789012345678901 2
12345678901234567890123456789012123456789012345678901
~ 2
! 22
~ ! ~ 2
! 2 2
12345678901234567890123456789012123456789012345678901
2a 4b 10 b 2
12345678901234567890123456789012123456789012345678901 2
12345678901234567890123456789012123456789012345678901 2
2
a2b23 5~2a! a2a2
12345678901234567890123456789012123456789012345678901 22
12345678901234567890123456789012123456789012345678901
1 Here is another problem filled with complex-looking math things, 2
12345678901234567890123456789012123456789012345678901
standard issue for the GRE math problem visual. At first, don’t worry 2
12345678901234567890123456789012123456789012345678901 2
12345678901234567890123456789012123456789012345678901
about the positive distinct integer bit. Just simplify the expressions step 2
12345678901234567890123456789012123456789012345678901 2
12345678901234567890123456789012123456789012345678901
by step. 2
12345678901234567890123456789012123456789012345678901 2
12345678901234567890123456789012123456789012345678901 2
12345678901234567890123456789012123456789012345678901
Column A Column B 2
12345678901234567890123456789012123456789012345678901 2
12345678901234567890123456789012123456789012345678901 2
2345678901234567890123456789012123456789012345678901
12345678901234567890123456789012123456789012345678901
22
~2a ! ~4b!
2
10(b ) 2 2 2
12345678901234567890123456789012123456789012345678901 2
1 2 23 2 2 2 2
12345678901234567890123456789012123456789012345678901 2
a b 5(2a) a a
12345678901234567890123456789012123456789012345678901 2
12345678901234567890123456789012123456789012345678901 2
12345678901234567890123456789012123456789012345678901
4b~b3! 10b4 2
12345678901234567890123456789012123456789012345678901
5 2 5 2
12345678901234567890123456789012123456789012345678901
~ 2
! 2
~ 2
! 4 2
12345678901234567890123456789012123456789012345678901 2
a 2a 5 4a a
12345678901234567890123456789012123456789012345678901 22
2345678901234567890123456789012123456789012345678901
12345678901234567890123456789012123456789012345678901 2
12345678901234567890123456789012123456789012345678901
4b4 10b4
2
12345678901234567890123456789012123456789012345678901
5 5
2
1 4a 6
20a 6
12345678901234567890123456789012123456789012345678901 22
2345678901234567890123456789012123456789012345678901
12345678901234567890123456789012123456789012345678901 2
12345678901234567890123456789012123456789012345678901
b4 b4 2
12345678901234567890123456789012123456789012345678901 2
1 5 5
12345678901234567890123456789012123456789012345678901 22
2345678901234567890123456789012123456789012345678901
a6
2a 6
1
12345678901234567890123456789012123456789012345678901 2
12345678901234567890123456789012123456789012345678901 2
12345678901234567890123456789012123456789012345678901
Now that you’ve done the work, you can go back to the “positive distinct 2
12345678901234567890123456789012123456789012345678901 2
12345678901234567890123456789012123456789012345678901 2
12345678901234567890123456789012123456789012345678901 22
integers” phrase. Since the two variables are positive integers, the
12345678901234567890123456789012123456789012345678901
123456789012345678901234567890121234567890123456789012 61
Part III: Section Review

123456789012345678901234567890121234567890123456789012
12345678901234567890123456789012123456789012345678901 2
12345678901234567890123456789012123456789012345678901 2
12345678901234567890123456789012123456789012345678901
denominator in Column A is smaller than the denominator in Column B, 2
12345678901234567890123456789012123456789012345678901 2
12345678901234567890123456789012123456789012345678901
while the numerators are equal. Try substituting some small positive 2
12345678901234567890123456789012123456789012345678901 2
12345678901234567890123456789012123456789012345678901
integers into the equation if you like. If the denominator in Column B is 2
2345678901234567890123456789012123456789012345678901 2
1
12345678901234567890123456789012123456789012345678901 1 1 2
12345678901234567890123456789012123456789012345678901
greater, this means that Column A is greater in value, since . . Using 2
12345678901234567890123456789012123456789012345678901 2 8 2
12345678901234567890123456789012123456789012345678901
2345678901234567890123456789012123456789012345678901
2
2
1 these simpler values, you can see how the numerators are equal and the
12345678901234567890123456789012123456789012345678901 2
12345678901234567890123456789012123456789012345678901
fraction with the smaller denominator is the largest value. 2
12345678901234567890123456789012123456789012345678901 2
12345678901234567890123456789012123456789012345678901
2345678901234567890123456789012123456789012345678901
2
2
1
12345678901234567890123456789012123456789012345678901 2

Tip
12345678901234567890123456789012123456789012345678901
If the variables weren’t “distinct” they could both equal 1, and the two 2
12345678901234567890123456789012123456789012345678901 2
12345678901234567890123456789012123456789012345678901
fractions would be equal.
2345678901234567890123456789012123456789012345678901
2
2
1
12345678901234567890123456789012123456789012345678901 2
12345678901234567890123456789012123456789012345678901 2
12345678901234567890123456789012123456789012345678901
4 2
12345678901234567890123456789012123456789012345678901
2 2
2345678901234567890123456789012123456789012345678901
12345678901234567890123456789012123456789012345678901
What does equal? 2
12345678901234567890123456789012123456789012345678901
4243 2
12345678901234567890123456789012123456789012345678901 2
12345678901234567890123456789012123456789012345678901
A. 2 26 2
12345678901234567890123456789012123456789012345678901 2
12345678901234567890123456789012123456789012345678901
B. 2 23 2
1 2
12345678901234567890123456789012123456789012345678901
C. 2 21
2
12345678901234567890123456789012123456789012345678901 2
12345678901234567890123456789012123456789012345678901
D. 4 2
2
2345678901234567890123456789012123456789012345678901
12345678901234567890123456789012123456789012345678901 2
12345678901234567890123456789012123456789012345678901
E. 25 2
12345678901234567890123456789012123456789012345678901 2
12345678901234567890123456789012123456789012345678901
One way to do this is to raise each number to its power, multiply where 2
2
12345678901234567890123456789012123456789012345678901
needed, and then simplify. You might be able to work this method on this 2
12345678901234567890123456789012123456789012345678901
12345678901234567890123456789012123456789012345678901 2
12345678901234567890123456789012123456789012345678901 2
12345678901234567890123456789012123456789012345678901 2
problem, but if the numbers are raised to higher powers, this method
1 becomes very difficult very quickly. The other way is to try to make the 2
12345678901234567890123456789012123456789012345678901 2
12345678901234567890123456789012123456789012345678901 2
12345678901234567890123456789012123456789012345678901 2
numbers in the numerator and the denominator have the same base so that
12345678901234567890123456789012123456789012345678901 2
12345678901234567890123456789012123456789012345678901
you can simplify directly. Since 4 5 (2)(2) then
2345678901234567890123456789012123456789012345678901
2
2
12345678901234567890123456789012123456789012345678901
12345678901234567890123456789012123456789012345678901
2 2
4 5 [(2)(2)] 5 2 2 5 2 , 2 2 4 2
1 2
12345678901234567890123456789012123456789012345678901 2
12345678901234567890123456789012123456789012345678901
and 2
12345678901234567890123456789012123456789012345678901 2
2345678901234567890123456789012123456789012345678901
12345678901234567890123456789012123456789012345678901 2
1 43 5 [(2)(2)]3 5 2323 5 26 2
12345678901234567890123456789012123456789012345678901 2
12345678901234567890123456789012123456789012345678901 2
12345678901234567890123456789012123456789012345678901
So we can rewrite the fraction like this, 2
12345678901234567890123456789012123456789012345678901 2
12345678901234567890123456789012123456789012345678901
4 4 4 2
12345678901234567890123456789012123456789012345678901
2 2 2 2
12345678901234567890123456789012123456789012345678901
5 5 2
12345678901234567890123456789012123456789012345678901 2
2345678901234567890123456789012123456789012345678901
2 3 4 6 10
4 4 2 2 2
12345678901234567890123456789012123456789012345678901 2 2
12345678901234567890123456789012123456789012345678901
This can be simplified straightforwardly,
2
12345678901234567890123456789012123456789012345678901 2
1
2345678901234567890123456789012123456789012345678901
12345678901234567890123456789012123456789012345678901
2 4 2
12345678901234567890123456789012123456789012345678901
4 210 26 2
10
5 2 2
12345678901234567890123456789012123456789012345678901
5 2 2
12345678901234567890123456789012123456789012345678901 2
2
1 You could also have transformed the numerator into a base of 4. Either 2
12345678901234567890123456789012123456789012345678901 2
12345678901234567890123456789012123456789012345678901 2
12345678901234567890123456789012123456789012345678901
method would lead to the right answer, although since a majority of the 2
12345678901234567890123456789012123456789012345678901 2
12345678901234567890123456789012123456789012345678901
answer choices have a base of 2, transforming everything into that base 2
2345678901234567890123456789012123456789012345678901 2
1
12345678901234567890123456789012123456789012345678901 2
12345678901234567890123456789012123456789012345678901 2
12345678901234567890123456789012123456789012345678901 2
62 123456789012345678901234567890121234567890123456789012

www.petersons.com
Chapter 4: Quantitative Review

123456789012345678901234567890121234567890123456789012
12345678901234567890123456789012123456789012345678901 2
12345678901234567890123456789012123456789012345678901 2
12345678901234567890123456789012123456789012345678901
increases your chances of simply coming up with the right answer without 2
12345678901234567890123456789012123456789012345678901 2
12345678901234567890123456789012123456789012345678901
having to do one last conversion. 2
12345678901234567890123456789012123456789012345678901 2
12345678901234567890123456789012123456789012345678901
The annoying relative of the exponent is the square root. It looks like this 2
12345678901234567890123456789012123456789012345678901 2
12345678901234567890123456789012123456789012345678901 2
12345678901234567890123456789012123456789012345678901
= , and it’s the opposite of squaring a number. When you square 4, you 2
12345678901234567890123456789012123456789012345678901 2
12345678901234567890123456789012123456789012345678901
multiply it by itself to get 16, but when you take the square root of 4, you 2
2345678901234567890123456789012123456789012345678901 2
1
12345678901234567890123456789012123456789012345678901 = 2
12345678901234567890123456789012123456789012345678901
get the number that times itself gives 4(2 3 2) 5 4, so 4 52. Square 2
12345678901234567890123456789012123456789012345678901 1 2
12345678901234567890123456789012123456789012345678901
roots can also be written as exponents, =x 5 x .
2345678901234567890123456789012123456789012345678901
2 2
2
1
12345678901234567890123456789012123456789012345678901 2
12345678901234567890123456789012123456789012345678901
Even more annoying than the square root is the cube root. Just look at it, 2
12345678901234567890123456789012123456789012345678901 2
12345678901234567890123456789012123456789012345678901
= 3
. Something like this practically screams “convoluted math process 2
2345678901234567890123456789012123456789012345678901 2
1
12345678901234567890123456789012123456789012345678901
up ahead.” When you cube something, you multiply it by itself twice (so 2
12345678901234567890123456789012123456789012345678901 2
12345678901234567890123456789012123456789012345678901
the cube of 3 is 33 5 3 3 3 3 3 5 27). The cube root is the opposite, 2
12345678901234567890123456789012123456789012345678901 2
12345678901234567890123456789012123456789012345678901
= 3
27 5 3 or =8 5 2 because 2 3 2 3 2 5 8. Cube roots are written as 2
3
12345678901234567890123456789012123456789012345678901 2
12345678901234567890123456789012123456789012345678901
1 2
12345678901234567890123456789012123456789012345678901 2
3
exponents like this, =x 5 x3.
12345678901234567890123456789012123456789012345678901 2
2345678901234567890123456789012123456789012345678901
12345678901234567890123456789012123456789012345678901 2
1 2 25 2
12345678901234567890123456789012123456789012345678901
@ ~ 2
!~ 3!~x12!# 2
12345678901234567890123456789012123456789012345678901
2x x 2
12345678901234567890123456789012123456789012345678901
If x 5 3 then 3 2
2345678901234567890123456789012123456789012345678901
4~x4!
12345678901234567890123456789012123456789012345678901 2
12345678901234567890123456789012123456789012345678901 2
12345678901234567890123456789012123456789012345678901 2
12345678901234567890123456789012123456789012345678901 22
A. 1
12345678901234567890123456789012123456789012345678901
12345678901234567890123456789012123456789012345678901
B. =3 2
12345678901234567890123456789012123456789012345678901 2
12345678901234567890123456789012123456789012345678901
C. 3 2
12345678901234567890123456789012123456789012345678901 2
1 D. 27 2
2345678901234567890123456789012123456789012345678901
12345678901234567890123456789012123456789012345678901 2
12345678901234567890123456789012123456789012345678901
E. 81 2
1 2
12345678901234567890123456789012123456789012345678901 2
12345678901234567890123456789012123456789012345678901
There is good news and bad news about
2345678901234567890123456789012123456789012345678901
this problem. The good news is 2
that with a problem like this, there are no hidden tricks. The bad news is 2
12345678901234567890123456789012123456789012345678901
12345678901234567890123456789012123456789012345678901 2
1 that you have to work this problem out step by step, but you know 2
12345678901234567890123456789012123456789012345678901 2
12345678901234567890123456789012123456789012345678901 2
12345678901234567890123456789012123456789012345678901 22
2345678901234567890123456789012123456789012345678901
everything you need to know to do that. To the solving, then!
12345678901234567890123456789012123456789012345678901 2
1 2 2 25
12345678901234567890123456789012123456789012345678901
~ 2x ! ~ x 3!~x12! 2 25 3 2
12345678901234567890123456789012123456789012345678901
2 2 2
2345678901234567890123456789012123456789012345678901
12345678901234567890123456789012123456789012345678901
3 5 x x 3x12x 4
2
~ !
12345678901234567890123456789012123456789012345678901
4 x
12345678901234567890123456789012123456789012345678901 22
4 2
12345678901234567890123456789012123456789012345678901
1 The four in the denominator is cancelled out by the 2 that gets squared. 2
12345678901234567890123456789012123456789012345678901 2
12345678901234567890123456789012123456789012345678901
Now we need to find a common denominator for the exponents in order to 2
12345678901234567890123456789012123456789012345678901 2
12345678901234567890123456789012123456789012345678901
add and subtract these fractions. This process was discussed earlier on 2
12345678901234567890123456789012123456789012345678901
page 62, and as you can see, straightforward concepts have a way of 2
12345678901234567890123456789012123456789012345678901 2
12345678901234567890123456789012123456789012345678901 2
12345678901234567890123456789012123456789012345678901
showing up in odd places on a GRE math question. 12 is an easy choice. 2
12345678901234567890123456789012123456789012345678901 2
12345678901234567890123456789012123456789012345678901 2
2345678901234567890123456789012123456789012345678901
12345678901234567890123456789012123456789012345678901
2
2 25 3 24 1 8 1 25 2 9 48
4 2
1 2
2
x x3x12x 4 5 x 12 5 x12 5 x
12345678901234567890123456789012123456789012345678901 2
12345678901234567890123456789012123456789012345678901 2
12345678901234567890123456789012123456789012345678901
If x 5 3 then the answer is 81, or (E).
2345678901234567890123456789012123456789012345678901
2
2
1
12345678901234567890123456789012123456789012345678901 2
12345678901234567890123456789012123456789012345678901 2
12345678901234567890123456789012123456789012345678901 2 63
123456789012345678901234567890121234567890123456789012
Part III: Section Review

123456789012345678901234567890121234567890123456789012
12345678901234567890123456789012123456789012345678901 2
12345678901234567890123456789012123456789012345678901 2
12345678901234567890123456789012123456789012345678901 2
12345678901234567890123456789012123456789012345678901 2
12345678901234567890123456789012123456789012345678901
Common Exponents to Know 2
12345678901234567890123456789012123456789012345678901 2
12345678901234567890123456789012123456789012345678901 2

Note
3 4 5
12345678901234567890123456789012123456789012345678901
2 5 8, 2 5 16, 2 5 32 2
12345678901234567890123456789012123456789012345678901 2
12345678901234567890123456789012123456789012345678901
3
3 5 27, 3 5 81 4
2
12345678901234567890123456789012123456789012345678901 2
12345678901234567890123456789012123456789012345678901
3
4 5 64
2345678901234567890123456789012123456789012345678901
2
2
1
12345678901234567890123456789012123456789012345678901
3 2
12345678901234567890123456789012123456789012345678901
5 5 125 2
12345678901234567890123456789012123456789012345678901 2
12345678901234567890123456789012123456789012345678901 2
1 One more tip when dealing with exponents: Always remember that the 2
2345678901234567890123456789012123456789012345678901
12345678901234567890123456789012123456789012345678901 2
12345678901234567890123456789012123456789012345678901 2 2
12345678901234567890123456789012123456789012345678901
base can be a negative number. If a problem states, “x 5 36,” it’s 2
12345678901234567890123456789012123456789012345678901
common to assume that x 6. This is
2345678901234567890123456789012123456789012345678901a tried-and-true test trap. Without 2
2
1 5
12345678901234567890123456789012123456789012345678901
any more information given in the problem, all you know is that x could be 2
12345678901234567890123456789012123456789012345678901 2
12345678901234567890123456789012123456789012345678901
6 or 26. Any number raised to an even power will yield a positive number, 2
12345678901234567890123456789012123456789012345678901 2
12345678901234567890123456789012123456789012345678901
but the base could be positive or negative. When the exponent is odd, the 2
12345678901234567890123456789012123456789012345678901 2
12345678901234567890123456789012123456789012345678901
base is the same sign as what it equals. 2
12345678901234567890123456789012123456789012345678901 2
12345678901234567890123456789012123456789012345678901 2
12345678901234567890123456789012123456789012345678901 2
2345678901234567890123456789012123456789012345678901
For instance, if x 5
5 232 then x 5 22.
2
1
12345678901234567890123456789012123456789012345678901 2
12345678901234567890123456789012123456789012345678901 2
12345678901234567890123456789012123456789012345678901 2
2345678901234567890123456789012123456789012345678901
12345678901234567890123456789012123456789012345678901
Percents Are about 4.6% of This Entire Chapter 2
12345678901234567890123456789012123456789012345678901 2
12345678901234567890123456789012123456789012345678901 2
12345678901234567890123456789012123456789012345678901 2
Percents are not an alien concept, but the GRE expects a greater
understanding of percents than what you need to navigate the local 2
12345678901234567890123456789012123456789012345678901
12345678901234567890123456789012123456789012345678901 2
12345678901234567890123456789012123456789012345678901 2
12345678901234567890123456789012123456789012345678901 2
grocery store. It isn’t a huge amount of additional information that you
12345678901234567890123456789012123456789012345678901 2
1 need to understand, but a few things probably need to be ironed out in 2
2345678901234567890123456789012123456789012345678901
12345678901234567890123456789012123456789012345678901
your mind about percents. Let’s get to ironing. 2
12345678901234567890123456789012123456789012345678901 2
1 Percent just means “per hundred” or “divided by one hundred.” 2
12345678901234567890123456789012123456789012345678901 2
12345678901234567890123456789012123456789012345678901 2
12345678901234567890123456789012123456789012345678901 2
12345678901234567890123456789012123456789012345678901
27 2
12345678901234567890123456789012123456789012345678901
So 27% 5 , or 2
12345678901234567890123456789012123456789012345678901
100 2
12345678901234567890123456789012123456789012345678901 2
2345678901234567890123456789012123456789012345678901
12345678901234567890123456789012123456789012345678901
0.6 6 2
12345678901234567890123456789012123456789012345678901
0.6% 5 5 2
1 100 1,000 2
12345678901234567890123456789012123456789012345678901 2
12345678901234567890123456789012123456789012345678901
If bananas are 40% off at the store, and their normal price is $3/pound, 2
12345678901234567890123456789012123456789012345678901 2
12345678901234567890123456789012123456789012345678901 2
12345678901234567890123456789012123456789012345678901
then their sale price is 2
12345678901234567890123456789012123456789012345678901 2
12345678901234567890123456789012123456789012345678901 2
12345678901234567890123456789012123456789012345678901 2
2345678901234567890123456789012123456789012345678901
(100% 2 40%)($3 pound) 5 (60%) ($3 pound)
2
12345678901234567890123456789012123456789012345678901
1 S D
12345678901234567890123456789012123456789012345678901
5
60
12345678901234567890123456789012123456789012345678901
100
12345678901234567890123456789012123456789012345678901 2
2345678901234567890123456789012123456789012345678901
~ $3 pound!
2
2
2
12345678901234567890123456789012123456789012345678901 2
12345678901234567890123456789012123456789012345678901
5 (0.6)($3 pound) 2
12345678901234567890123456789012123456789012345678901 2
1 5 $1.80 pound 2
12345678901234567890123456789012123456789012345678901 2
2345678901234567890123456789012123456789012345678901
2
1
12345678901234567890123456789012123456789012345678901 2
12345678901234567890123456789012123456789012345678901 2
12345678901234567890123456789012123456789012345678901 2
12345678901234567890123456789012123456789012345678901 2
12345678901234567890123456789012123456789012345678901 2
12345678901234567890123456789012123456789012345678901 2
12345678901234567890123456789012123456789012345678901 2
64 123456789012345678901234567890121234567890123456789012

www.petersons.com
Chapter 4: Quantitative Review

123456789012345678901234567890121234567890123456789012
12345678901234567890123456789012123456789012345678901 2
12345678901234567890123456789012123456789012345678901 2
12345678901234567890123456789012123456789012345678901
Notice that we multiplied the price times 60% because the sales price was 2
12345678901234567890123456789012123456789012345678901 2
12345678901234567890123456789012123456789012345678901
40% off. You could also have found out how much the bananas were 2
12345678901234567890123456789012123456789012345678901 2
12345678901234567890123456789012123456789012345678901
discounted due to the sale, and then subtracted that number from the 2
2345678901234567890123456789012123456789012345678901 2
1 original price. This would give you:
12345678901234567890123456789012123456789012345678901 2
12345678901234567890123456789012123456789012345678901 2
12345678901234567890123456789012123456789012345678901 2
1 (40 sale)($3 pound) 5S D
12345678901234567890123456789012123456789012345678901
40
2345678901234567890123456789012123456789012345678901
100
~$3 pound!
12345678901234567890123456789012123456789012345678901
12345678901234567890123456789012123456789012345678901
2
2
2
2
12345678901234567890123456789012123456789012345678901 2
12345678901234567890123456789012123456789012345678901
5 (0.4)($3 pound)
2345678901234567890123456789012123456789012345678901
2
2
1
12345678901234567890123456789012123456789012345678901
5 $1.20 discount 2
12345678901234567890123456789012123456789012345678901 2
12345678901234567890123456789012123456789012345678901 2
12345678901234567890123456789012123456789012345678901
Subtract $1.20, the discount, from the original
2345678901234567890123456789012123456789012345678901
price of $3/pound and you 2
2
1 get $1.80/pound, the same answer as before.
12345678901234567890123456789012123456789012345678901 2
12345678901234567890123456789012123456789012345678901 2
12345678901234567890123456789012123456789012345678901
A straightforward use of percent is when you are given two numbers and 2
12345678901234567890123456789012123456789012345678901 2
12345678901234567890123456789012123456789012345678901
asked to figure out a percentage value. A simple question of this kind 2
12345678901234567890123456789012123456789012345678901 2
12345678901234567890123456789012123456789012345678901
might ask, “What percentage of 33 is 3?” To answer this problem, it helps 2
12345678901234567890123456789012123456789012345678901 2
12345678901234567890123456789012123456789012345678901
to travel back in time to the fifth grade and dredge up the skill known as 2
2345678901234567890123456789012123456789012345678901
12345678901234567890123456789012123456789012345678901 2
1 cross-multiplying. Given two fractions equal to each other, you can 2
12345678901234567890123456789012123456789012345678901 2
12345678901234567890123456789012123456789012345678901
multiply the numerator of the first fraction with the denominator of the 2
12345678901234567890123456789012123456789012345678901
second fraction. Then you can multiply the numerator of the second 2
12345678901234567890123456789012123456789012345678901 2
12345678901234567890123456789012123456789012345678901
fraction with the denominator of the first fraction, and set both products 2
12345678901234567890123456789012123456789012345678901 2
12345678901234567890123456789012123456789012345678901 2
12345678901234567890123456789012123456789012345678901
equal to each other. Like many basic math concepts, the explanation is 2
2345678901234567890123456789012123456789012345678901
12345678901234567890123456789012123456789012345678901 2
12345678901234567890123456789012123456789012345678901
tougher than the actual cross-multiplying. 2
12345678901234567890123456789012123456789012345678901 22
12345678901234567890123456789012123456789012345678901
1 One way to set up this problem is 3 5 x , because you want to know 2
12345678901234567890123456789012123456789012345678901 2
12345678901234567890123456789012123456789012345678901
33 100 2
12345678901234567890123456789012123456789012345678901
what percentage the original fraction equals. Solving for x by cross- 2
12345678901234567890123456789012123456789012345678901 2
12345678901234567890123456789012123456789012345678901
multiplying yields 2
12345678901234567890123456789012123456789012345678901 22
2345678901234567890123456789012123456789012345678901
12345678901234567890123456789012123456789012345678901 2
1 3 x
12345678901234567890123456789012123456789012345678901
5 → 2
12345678901234567890123456789012123456789012345678901
33 100 2
12345678901234567890123456789012123456789012345678901 22
2345678901234567890123456789012123456789012345678901
12345678901234567890123456789012123456789012345678901
~33!~x! 5 ~3!~100! → (the second step is the 2
1
12345678901234567890123456789012123456789012345678901 2
12345678901234567890123456789012123456789012345678901 2
33x 5 300 → cross-multiplying one)
12345678901234567890123456789012123456789012345678901 22
2345678901234567890123456789012123456789012345678901
12345678901234567890123456789012123456789012345678901
300
2
12345678901234567890123456789012123456789012345678901
x5 '9
2
12345678901234567890123456789012123456789012345678901
33
2
1
2345678901234567890123456789012123456789012345678901
12345678901234567890123456789012123456789012345678901
With no calculator, 9 is a good estimate. Therefore, 3 is about 9% of 33. 2
2
12345678901234567890123456789012123456789012345678901
12345678901234567890123456789012123456789012345678901 22
12345678901234567890123456789012123456789012345678901 2
1
12345678901234567890123456789012123456789012345678901 2
X-Ref

12345678901234567890123456789012123456789012345678901 2
12345678901234567890123456789012123456789012345678901 3 1 2
12345678901234567890123456789012123456789012345678901
The shortcut on this problem was to see that 5 , and using 2
12345678901234567890123456789012123456789012345678901 33 11 2
12345678901234567890123456789012123456789012345678901 22
2345678901234567890123456789012123456789012345678901
1
1 fraction-decimal conversions, 5 0.09 5 9%.
12345678901234567890123456789012123456789012345678901
11 2
12345678901234567890123456789012123456789012345678901 2
12345678901234567890123456789012123456789012345678901
2345678901234567890123456789012123456789012345678901
2
2
1
12345678901234567890123456789012123456789012345678901 2
12345678901234567890123456789012123456789012345678901 2
12345678901234567890123456789012123456789012345678901 2 65
123456789012345678901234567890121234567890123456789012
Part III: Section Review

123456789012345678901234567890121234567890123456789012
12345678901234567890123456789012123456789012345678901 2
12345678901234567890123456789012123456789012345678901 2
12345678901234567890123456789012123456789012345678901
Those are the user-friendly percent questions. Percent increase and 2
12345678901234567890123456789012123456789012345678901 2
12345678901234567890123456789012123456789012345678901
decrease problems are a tad bit trickier. With these problems, the critical 2
12345678901234567890123456789012123456789012345678901 2
12345678901234567890123456789012123456789012345678901
point to remember is that any increase or decrease is relative to the original 2
1 value (what is increasing or decreasing). You need to use the original value 2
2345678901234567890123456789012123456789012345678901
12345678901234567890123456789012123456789012345678901 2
12345678901234567890123456789012123456789012345678901 2
12345678901234567890123456789012123456789012345678901
if you are trying to find the percent change. Suppose a problem tells you 2
12345678901234567890123456789012123456789012345678901
2345678901234567890123456789012123456789012345678901
2
2
1 that a certain stock started the year priced at $40/share and ended the year
12345678901234567890123456789012123456789012345678901 2
12345678901234567890123456789012123456789012345678901
at $56/share. What was the percentage change? 2
12345678901234567890123456789012123456789012345678901 2
12345678901234567890123456789012123456789012345678901
2345678901234567890123456789012123456789012345678901
2
2
1 This percentage change is relative to the original value of $40/share. To
12345678901234567890123456789012123456789012345678901 2
12345678901234567890123456789012123456789012345678901
find the percentage change, you find how much the stock price changed 2
12345678901234567890123456789012123456789012345678901 $16 share 4 2
12345678901234567890123456789012123456789012345678901
2345678901234567890123456789012123456789012345678901
2
2
1 ($16/share) and divide it by the original value, 5 5 0.4
12345678901234567890123456789012123456789012345678901 $40 share 10 2
12345678901234567890123456789012123456789012345678901
5 40%. Since the change was an increase, the stock price increased 40%. 2
12345678901234567890123456789012123456789012345678901 2
12345678901234567890123456789012123456789012345678901
The same method would be used to find a percentage decrease. 2
12345678901234567890123456789012123456789012345678901 2
2345678901234567890123456789012123456789012345678901
You might see a percentage change problem like this but be given the 2
12345678901234567890123456789012123456789012345678901
12345678901234567890123456789012123456789012345678901 2
12345678901234567890123456789012123456789012345678901 2
12345678901234567890123456789012123456789012345678901 2
percentage change and the final value of the item and asked to find the
12345678901234567890123456789012123456789012345678901 2
1 original value. In that case, you are doing the same problem you just did in 2
12345678901234567890123456789012123456789012345678901
reverse. Here is the formula you can use to solve either kind of problem, 2
12345678901234567890123456789012123456789012345678901 2
12345678901234567890123456789012123456789012345678901
|x y| 2
2345678901234567890123456789012123456789012345678901
5 z, where x is the original value, y is the second value, and z is the 2
12345678901234567890123456789012123456789012345678901
2
12345678901234567890123456789012123456789012345678901 2
12345678901234567890123456789012123456789012345678901
x
percentage change. (If the lines around the numerator are like alien 2
2
12345678901234567890123456789012123456789012345678901
12345678901234567890123456789012123456789012345678901
scribblings to you, look forward to the section on absolute value for an 2
2
12345678901234567890123456789012123456789012345678901
12345678901234567890123456789012123456789012345678901 2
12345678901234567890123456789012123456789012345678901
explanation of them.) 2
12345678901234567890123456789012123456789012345678901 2 2
1
12345678901234567890123456789012123456789012345678901 2
12345678901234567890123456789012123456789012345678901
The price of a house increased 15%, and then decreased 20%. 2
12345678901234567890123456789012123456789012345678901 2
12345678901234567890123456789012123456789012345678901 2
Compared to the original price, the final price was
12345678901234567890123456789012123456789012345678901 2
12345678901234567890123456789012123456789012345678901
A. an 8% decrease. 2
12345678901234567890123456789012123456789012345678901 2
12345678901234567890123456789012123456789012345678901
B. a 5% decrease. 2
12345678901234567890123456789012123456789012345678901 2
12345678901234567890123456789012123456789012345678901
C. the same. 2
2345678901234567890123456789012123456789012345678901
12345678901234567890123456789012123456789012345678901 2
12345678901234567890123456789012123456789012345678901
D. a 3% increase. 2
1 2
12345678901234567890123456789012123456789012345678901
E. a 5% increase. 2
12345678901234567890123456789012123456789012345678901 2
12345678901234567890123456789012123456789012345678901 2
2345678901234567890123456789012123456789012345678901
2
12345678901234567890123456789012123456789012345678901
Choices (B) and (E) are the big traps here. It seems so inviting to just add 2
12345678901234567890123456789012123456789012345678901 2
12345678901234567890123456789012123456789012345678901
1 or subtract percentages together, but this is almost always wrong. This is 2
2345678901234567890123456789012123456789012345678901
12345678901234567890123456789012123456789012345678901 2
Note

because the two percents are often reflections of different numerical 2


12345678901234567890123456789012123456789012345678901
values. In other words, you could add 50% of 100 and 25% of 100 2
12345678901234567890123456789012123456789012345678901
12345678901234567890123456789012123456789012345678901 2
1 together only because they have the same base value, 100. But you can’t 2
12345678901234567890123456789012123456789012345678901 2
2345678901234567890123456789012123456789012345678901
just add up 50% of 2 and 25% of 100 because these are percentages of 2
12345678901234567890123456789012123456789012345678901
12345678901234567890123456789012123456789012345678901 2
12345678901234567890123456789012123456789012345678901 2
1 different numbers, 2 and 100. 2
12345678901234567890123456789012123456789012345678901 2
2345678901234567890123456789012123456789012345678901
2
1
12345678901234567890123456789012123456789012345678901 2
12345678901234567890123456789012123456789012345678901 2
There’s a simple trick that makes this kind of problem a lot easier. If you’re
12345678901234567890123456789012123456789012345678901
not told what the original price of the house is, pick a value and work the 2
12345678901234567890123456789012123456789012345678901 2
12345678901234567890123456789012123456789012345678901 2
12345678901234567890123456789012123456789012345678901 2
problem with it. You want to make things easy, so don’t pick a tough
12345678901234567890123456789012123456789012345678901 2
66 123456789012345678901234567890121234567890123456789012

www.petersons.com
Chapter 4: Quantitative Review

123456789012345678901234567890121234567890123456789012
12345678901234567890123456789012123456789012345678901 2
12345678901234567890123456789012123456789012345678901 2
12345678901234567890123456789012123456789012345678901
number like 17.53. Using the value of 100 is a much better choice, as this 2
12345678901234567890123456789012123456789012345678901 2
12345678901234567890123456789012123456789012345678901
makes the first percent change easy to figure. A 15% increase on 100 is 15, 2
12345678901234567890123456789012123456789012345678901 2
12345678901234567890123456789012123456789012345678901
and so after the first increase the price of the house is 115.
2345678901234567890123456789012123456789012345678901
2
2
1
12345678901234567890123456789012123456789012345678901 2
12345678901234567890123456789012123456789012345678901
You now have to calculate a 20% decrease on a house valued at 115. If 2
12345678901234567890123456789012123456789012345678901 2
12345678901234567890123456789012123456789012345678901
you recall those fraction-decimal conversions
2345678901234567890123456789012123456789012345678901
on page 57, 20% 5 0.20 2
2
1
12345678901234567890123456789012123456789012345678901
1 2
12345678901234567890123456789012123456789012345678901
5 . Therefore, to find the value of the 20% decrease, multiply 115 by 2
12345678901234567890123456789012123456789012345678901
5 2
12345678901234567890123456789012123456789012345678901
1 115 115
2345678901234567890123456789012123456789012345678901
2
1 one fifth, 3 5 5 23. The price of the house decreased 23 from 2
12345678901234567890123456789012123456789012345678901
5 1 5 2
12345678901234567890123456789012123456789012345678901 2
12345678901234567890123456789012123456789012345678901
115, which means the final value of the house is 92(115 2 23). What is the 2
12345678901234567890123456789012123456789012345678901
2345678901234567890123456789012123456789012345678901
2
2
1 percentage change from 100 to 92? It’s an 8% decrease, choice (A).
12345678901234567890123456789012123456789012345678901 2
12345678901234567890123456789012123456789012345678901 2
12345678901234567890123456789012123456789012345678901 2
12345678901234567890123456789012123456789012345678901 2
2345678901234567890123456789012123456789012345678901
12345678901234567890123456789012123456789012345678901
Ratios, Where You Can Compare Apples to 2
12345678901234567890123456789012123456789012345678901 2
12345678901234567890123456789012123456789012345678901
Oranges 2
12345678901234567890123456789012123456789012345678901 22
12345678901234567890123456789012123456789012345678901
Ratio problems on standardized tests are like affable guests on a talk 2
12345678901234567890123456789012123456789012345678901
1 show: They make a lot of appearances. Ratios can be tested in lots of 2
12345678901234567890123456789012123456789012345678901 2
12345678901234567890123456789012123456789012345678901
different kinds of problems, which is one reason for their common 2
12345678901234567890123456789012123456789012345678901 2
12345678901234567890123456789012123456789012345678901
occurrence. You might see ratios in a word problem or a probability 2
12345678901234567890123456789012123456789012345678901 2
12345678901234567890123456789012123456789012345678901
problem, or maybe in just a straightforward ratio problem. 2
12345678901234567890123456789012123456789012345678901 2
12345678901234567890123456789012123456789012345678901 2
12345678901234567890123456789012123456789012345678901 22
2345678901234567890123456789012123456789012345678901
Ratios are a fancy way of writing fractions. There are a couple of different
12345678901234567890123456789012123456789012345678901
forms of ratios that you might see on a test, but they are all equivalent to 2
12345678901234567890123456789012123456789012345678901
12345678901234567890123456789012123456789012345678901 2
1 each other and they are also all equivalent to the fractional form of the 2
2345678901234567890123456789012123456789012345678901
ratio. In a word problem, a ratio would look like this: “9 bananas were 2
12345678901234567890123456789012123456789012345678901
12345678901234567890123456789012123456789012345678901 2
1 sold for every 4 apples that were sold.” The ratio of bananas to apples sold 2
12345678901234567890123456789012123456789012345678901 2
12345678901234567890123456789012123456789012345678901
is 9 to 4, or 9:4. These three expressions are equivalent, and they are 2
12345678901234567890123456789012123456789012345678901 2
12345678901234567890123456789012123456789012345678901 2
12345678901234567890123456789012123456789012345678901
9 2
12345678901234567890123456789012123456789012345678901
equivalent to the fraction (the first number in the ratio is the numerator 2
12345678901234567890123456789012123456789012345678901
4 2
2345678901234567890123456789012123456789012345678901
12345678901234567890123456789012123456789012345678901
and the second the denominator). 2
12345678901234567890123456789012123456789012345678901 2
1 2
12345678901234567890123456789012123456789012345678901 2
12345678901234567890123456789012123456789012345678901
From a precise mathematical standpoint, ratios and fractions are not 2
Tip

12345678901234567890123456789012123456789012345678901 2
12345678901234567890123456789012123456789012345678901
completely interchangeable. However, the precise distinction between 2
12345678901234567890123456789012123456789012345678901 2
12345678901234567890123456789012123456789012345678901
ratios and fractions is not relevant for the GRE, so don’t sweat it. 2
12345678901234567890123456789012123456789012345678901 2
12345678901234567890123456789012123456789012345678901 22
2345678901234567890123456789012123456789012345678901
12345678901234567890123456789012123456789012345678901 2
12345678901234567890123456789012123456789012345678901
Like fractions, ratios can be reduced to lowest terms. If the ratio had been
2
12345678901234567890123456789012123456789012345678901
1 “9 bananas sold for every 3 apples,” the ratio would be 9:3 or 3:1. These 2
2345678901234567890123456789012123456789012345678901
12345678901234567890123456789012123456789012345678901 2
12345678901234567890123456789012123456789012345678901
9 3 2
12345678901234567890123456789012123456789012345678901
ratios are the same, just as is the same thing as . 2
12345678901234567890123456789012123456789012345678901
3 1 2
1 2
2345678901234567890123456789012123456789012345678901
12345678901234567890123456789012123456789012345678901 2
1 Since you can compare fractions and say that one is greater than another, 2
12345678901234567890123456789012123456789012345678901
you can do the same thing with ratios. For instance, which ratio is greater, 2
12345678901234567890123456789012123456789012345678901 2
12345678901234567890123456789012123456789012345678901 4 2
1 4:3 or 5:4? Convert both to fractions and compare. Is greater or smaller 2
2345678901234567890123456789012123456789012345678901
12345678901234567890123456789012123456789012345678901 3 2
12345678901234567890123456789012123456789012345678901 2
12345678901234567890123456789012123456789012345678901 2 67
123456789012345678901234567890121234567890123456789012
Part III: Section Review

123456789012345678901234567890121234567890123456789012
12345678901234567890123456789012123456789012345678901 2
12345678901234567890123456789012123456789012345678901 2
12345678901234567890123456789012123456789012345678901
5 2
12345678901234567890123456789012123456789012345678901
than ? It is greater because a third is greater than a fourth (bigger 2
12345678901234567890123456789012123456789012345678901
4 2
12345678901234567890123456789012123456789012345678901 2
12345678901234567890123456789012123456789012345678901
denominator means smaller value, just like
2345678901234567890123456789012123456789012345678901
earlier), and four thirds is a 2
2
1 third greater than one, while five fourths is a fourth greater than one.
12345678901234567890123456789012123456789012345678901 2
12345678901234567890123456789012123456789012345678901 2
12345678901234567890123456789012123456789012345678901 2
12345678901234567890123456789012123456789012345678901
Column A Column
2345678901234567890123456789012123456789012345678901
B 2
2
1
12345678901234567890123456789012123456789012345678901 2
12345678901234567890123456789012123456789012345678901
Before 4 girls entered the class, there were
2
12345678901234567890123456789012123456789012345678901
18 boys and 16 girls in the fourth grade 2
12345678901234567890123456789012123456789012345678901
2345678901234567890123456789012123456789012345678901
2
2
1 class at Santiago Elementary School.
12345678901234567890123456789012123456789012345678901 2
12345678901234567890123456789012123456789012345678901 2
12345678901234567890123456789012123456789012345678901
The original ratio of The final ratio of 2
12345678901234567890123456789012123456789012345678901
2345678901234567890123456789012123456789012345678901
2
2
1 boys to girls in girls to boys in
12345678901234567890123456789012123456789012345678901 2
12345678901234567890123456789012123456789012345678901
the class the class 2
12345678901234567890123456789012123456789012345678901 2
12345678901234567890123456789012123456789012345678901 2
2345678901234567890123456789012123456789012345678901
First, you know that you can determine the two ratios, which means you 2
12345678901234567890123456789012123456789012345678901
12345678901234567890123456789012123456789012345678901 2
12345678901234567890123456789012123456789012345678901
can compare them. Since there will be a definite relationship between 2
12345678901234567890123456789012123456789012345678901 2
Column A and Column B, choice (D) is out. The original ratio of boys to 2
12345678901234567890123456789012123456789012345678901
12345678901234567890123456789012123456789012345678901 2
1 18 9 20 10 2
12345678901234567890123456789012123456789012345678901
girls is 18:16 or 5 . The final ratio of girls to boys is 20:18 or 5 . 2
12345678901234567890123456789012123456789012345678901
16 8 18 9 2
12345678901234567890123456789012123456789012345678901 2
12345678901234567890123456789012123456789012345678901 2
2345678901234567890123456789012123456789012345678901
Column A is one eighth greater than one, and Column B is one ninth
12345678901234567890123456789012123456789012345678901 2
12345678901234567890123456789012123456789012345678901
greater than one, so (A) is greater. 2
12345678901234567890123456789012123456789012345678901 2
12345678901234567890123456789012123456789012345678901 2
12345678901234567890123456789012123456789012345678901 2
12345678901234567890123456789012123456789012345678901
Absolute Value (not a vodka-related concept) 2
12345678901234567890123456789012123456789012345678901 2
12345678901234567890123456789012123456789012345678901 2
1 Do you remember the number line on page 50 of this chapter? The concept 2
2345678901234567890123456789012123456789012345678901
of absolute value is related to the number line. The absolute value of a 2
12345678901234567890123456789012123456789012345678901
12345678901234567890123456789012123456789012345678901 2
1 number is how many numbers (how far) it is away from zero on the 2
12345678901234567890123456789012123456789012345678901 2
12345678901234567890123456789012123456789012345678901
number line. 214 is 14 numbers away from zero on the number line, so the 2
12345678901234567890123456789012123456789012345678901 2
12345678901234567890123456789012123456789012345678901
absolute value of 214 is 14. Of course, the powers-that-be came up with a 2
12345678901234567890123456789012123456789012345678901 2
12345678901234567890123456789012123456789012345678901
cute little way to denote absolute value. For our example, it’s like this: 2
12345678901234567890123456789012123456789012345678901 2
2345678901234567890123456789012123456789012345678901
12345678901234567890123456789012123456789012345678901
|214| 5 14. 2
12345678901234567890123456789012123456789012345678901 2
1 2
12345678901234567890123456789012123456789012345678901 2
12345678901234567890123456789012123456789012345678901 2
12345678901234567890123456789012123456789012345678901
Algebra 2
12345678901234567890123456789012123456789012345678901 2
12345678901234567890123456789012123456789012345678901
The word algebra usually conjures up memories of numbers and letters 2
12345678901234567890123456789012123456789012345678901 2
12345678901234567890123456789012123456789012345678901 3 2
2345678901234567890123456789012123456789012345678901
mashed together in strings like “9y 1 3x 5 2 .” In simplest terms, algebra 2
12345678901234567890123456789012123456789012345678901
12345678901234567890123456789012123456789012345678901 4 2
12345678901234567890123456789012123456789012345678901 2
12345678901234567890123456789012123456789012345678901 2
uses placeholders, called variables, as substitutes for a known value. Using
1 variables in this manner often allows someone to set up an equation and 2
12345678901234567890123456789012123456789012345678901 2
12345678901234567890123456789012123456789012345678901 2
12345678901234567890123456789012123456789012345678901
then solve this equation to find the actual value of the variable. 2
12345678901234567890123456789012123456789012345678901 2
12345678901234567890123456789012123456789012345678901 2
12345678901234567890123456789012123456789012345678901 2
2345678901234567890123456789012123456789012345678901
Setting up equations with variables and manipulating these equations are
1 standard fare for all standardized math tests, and the GRE is no exception. 2
12345678901234567890123456789012123456789012345678901 2
12345678901234567890123456789012123456789012345678901
Problems filled with variables will be easily recognizable algebra prob- 2
12345678901234567890123456789012123456789012345678901 2
12345678901234567890123456789012123456789012345678901
lems, but these are not the only ones you will encounter. On others, the key 2
12345678901234567890123456789012123456789012345678901 2
12345678901234567890123456789012123456789012345678901
will be to create the correct equation on your own, although this might not 2
12345678901234567890123456789012123456789012345678901 2
68 123456789012345678901234567890121234567890123456789012

www.petersons.com
Chapter 4: Quantitative Review

123456789012345678901234567890121234567890123456789012
12345678901234567890123456789012123456789012345678901 2
12345678901234567890123456789012123456789012345678901 2
12345678901234567890123456789012123456789012345678901
be the most obvious step required. The end point is that you should get 2
12345678901234567890123456789012123456789012345678901 2
12345678901234567890123456789012123456789012345678901
comfortable with the idea of using variables and making equations, since 2
12345678901234567890123456789012123456789012345678901 2
12345678901234567890123456789012123456789012345678901
this is yet another tool you can use to crack open and solve GRE 2
2345678901234567890123456789012123456789012345678901 2
1 Quantitative questions.
12345678901234567890123456789012123456789012345678901 2
12345678901234567890123456789012123456789012345678901 2
12345678901234567890123456789012123456789012345678901 2
12345678901234567890123456789012123456789012345678901
2345678901234567890123456789012123456789012345678901
2
2
1 Many algebra problems have a geometry component, and others are filled
Alert!
12345678901234567890123456789012123456789012345678901 2
12345678901234567890123456789012123456789012345678901 2
12345678901234567890123456789012123456789012345678901
with the sort of math jargon you just finished studying. As you work 2
12345678901234567890123456789012123456789012345678901
2345678901234567890123456789012123456789012345678901
2
2
1 through the rest of this chapter, you will see how a firm foundation of
12345678901234567890123456789012123456789012345678901 2
12345678901234567890123456789012123456789012345678901
arithmetic aids you on all types of problems. 2
12345678901234567890123456789012123456789012345678901 2
12345678901234567890123456789012123456789012345678901
2345678901234567890123456789012123456789012345678901
2
2
1
12345678901234567890123456789012123456789012345678901 2
12345678901234567890123456789012123456789012345678901
Equations 2
12345678901234567890123456789012123456789012345678901 2
12345678901234567890123456789012123456789012345678901
Equations are like sentences written in math. Unlike sentences written in 2
2345678901234567890123456789012123456789012345678901
English, sentences written in math always follow the same form and rules: 2
12345678901234567890123456789012123456789012345678901
12345678901234567890123456789012123456789012345678901 2
12345678901234567890123456789012123456789012345678901 22
12345678901234567890123456789012123456789012345678901
1. There is stuff on the right side of the equation, and it equals the 2
12345678901234567890123456789012123456789012345678901
12345678901234567890123456789012123456789012345678901 2
1 stuff on the left side of the equation. 2
12345678901234567890123456789012123456789012345678901 2
12345678901234567890123456789012123456789012345678901
2. If you do something to the right side, you have to do the exact 2
12345678901234567890123456789012123456789012345678901 2
12345678901234567890123456789012123456789012345678901
same thing to the left side, and vice versa. (That is how you can 2
12345678901234567890123456789012123456789012345678901 2
12345678901234567890123456789012123456789012345678901
keep the equal sign between them. If you do the same thing to 2
12345678901234567890123456789012123456789012345678901 2
12345678901234567890123456789012123456789012345678901
each side, they remain equal.) 2
12345678901234567890123456789012123456789012345678901 22
2345678901234567890123456789012123456789012345678901
12345678901234567890123456789012123456789012345678901
3. It doesn’t matter what you do to one side, so long as you follow 2
12345678901234567890123456789012123456789012345678901
12345678901234567890123456789012123456789012345678901 2
1 Rule #2. 2
12345678901234567890123456789012123456789012345678901 22
2345678901234567890123456789012123456789012345678901
12345678901234567890123456789012123456789012345678901
1 Burn Rules #2 and #3 into your head, since you’ll often be given an 2
12345678901234567890123456789012123456789012345678901
equation to start with and asked to do something to it. Anything’s fair 2
12345678901234567890123456789012123456789012345678901
2345678901234567890123456789012123456789012345678901
2
game—multiplying, dividing, raising to a power, cubing the root, adding 2
12345678901234567890123456789012123456789012345678901
12345678901234567890123456789012123456789012345678901 2
1 43,000,000,000—so long as it’s done to both sides. 2
12345678901234567890123456789012123456789012345678901 2
12345678901234567890123456789012123456789012345678901 2
2345678901234567890123456789012123456789012345678901
12345678901234567890123456789012123456789012345678901
If 3x 2 12 5 33, then what does x equal? 2
12345678901234567890123456789012123456789012345678901 2
1 This is a simple equation. To find out what x equals, you need to get x all 2
12345678901234567890123456789012123456789012345678901 2
12345678901234567890123456789012123456789012345678901
by itself on one side of the equation. The variable x will then equal 2
12345678901234567890123456789012123456789012345678901 2
12345678901234567890123456789012123456789012345678901 2
12345678901234567890123456789012123456789012345678901
everything on the other side of the equation. 2
12345678901234567890123456789012123456789012345678901 2
12345678901234567890123456789012123456789012345678901 2
12345678901234567890123456789012123456789012345678901 22
2345678901234567890123456789012123456789012345678901
3x 2 12 5 33
12345678901234567890123456789012123456789012345678901 2
12345678901234567890123456789012123456789012345678901
3x 2 12 1 12 5 33 1 12 By adding twelve to both sides, we can
2
12345678901234567890123456789012123456789012345678901 2
1 get rid of the 212 on the left side.
12345678901234567890123456789012123456789012345678901 22
2345678901234567890123456789012123456789012345678901
12345678901234567890123456789012123456789012345678901 2
12345678901234567890123456789012123456789012345678901
3x 5 45
2
12345678901234567890123456789012123456789012345678901
Dividing both sides by three will get rid 2
1 3x 4 3 5 45 4 3
2345678901234567890123456789012123456789012345678901
12345678901234567890123456789012123456789012345678901
of the 3 in front of the x. 2
1 x 5 15 2
12345678901234567890123456789012123456789012345678901 2
12345678901234567890123456789012123456789012345678901 2
12345678901234567890123456789012123456789012345678901
There you have it.
2345678901234567890123456789012123456789012345678901
2
2
1
12345678901234567890123456789012123456789012345678901 2
12345678901234567890123456789012123456789012345678901 2
12345678901234567890123456789012123456789012345678901 2 69
123456789012345678901234567890121234567890123456789012
Part III: Section Review

123456789012345678901234567890121234567890123456789012
12345678901234567890123456789012123456789012345678901 2
12345678901234567890123456789012123456789012345678901 2
12345678901234567890123456789012123456789012345678901
Notice that in the example there was only one equation and one variable. 2
12345678901234567890123456789012123456789012345678901 2
12345678901234567890123456789012123456789012345678901
The next step up is a problem where you are given two equations and two 2
12345678901234567890123456789012123456789012345678901 2
12345678901234567890123456789012123456789012345678901
unknowns. It’s all still good, because if you have two equations you can 2
1 solve for two unknowns. It just takes a little bit longer. (The general math 2
2345678901234567890123456789012123456789012345678901
12345678901234567890123456789012123456789012345678901 2
12345678901234567890123456789012123456789012345678901 2
12345678901234567890123456789012123456789012345678901
rule is that you can solve for as many unknowns as you have distinct 2
12345678901234567890123456789012123456789012345678901
2345678901234567890123456789012123456789012345678901
2
2
1 equations.)
12345678901234567890123456789012123456789012345678901 2
12345678901234567890123456789012123456789012345678901 2
12345678901234567890123456789012123456789012345678901 2
12345678901234567890123456789012123456789012345678901
If 4x 1 3y 5 13 and x 1 2y 5 2, then
2345678901234567890123456789012123456789012345678901
what is the value of 27xy? 2
2
1
12345678901234567890123456789012123456789012345678901
A. 224 2
12345678901234567890123456789012123456789012345678901 2
12345678901234567890123456789012123456789012345678901
B. 24 2
12345678901234567890123456789012123456789012345678901
2345678901234567890123456789012123456789012345678901
2
2
1 C. 21
12345678901234567890123456789012123456789012345678901 2
12345678901234567890123456789012123456789012345678901
D. 4 2
12345678901234567890123456789012123456789012345678901 2
12345678901234567890123456789012123456789012345678901
E. 28 2
12345678901234567890123456789012123456789012345678901 2
2345678901234567890123456789012123456789012345678901
There are two different ways to try to solve this problem. The first way is 2
12345678901234567890123456789012123456789012345678901
12345678901234567890123456789012123456789012345678901 2
12345678901234567890123456789012123456789012345678901 2
12345678901234567890123456789012123456789012345678901 2
the “solve and plug.” You solve for one variable in one equation and then
12345678901234567890123456789012123456789012345678901 2
1 plug it into the other equation. It looks like this: 2
12345678901234567890123456789012123456789012345678901 2
12345678901234567890123456789012123456789012345678901 2
12345678901234567890123456789012123456789012345678901
x 1 2y 5 2 → x 5 2 2 2y (Plug this in for x in the other 2
2345678901234567890123456789012123456789012345678901
12345678901234567890123456789012123456789012345678901 2
equation.)
12345678901234567890123456789012123456789012345678901 2
12345678901234567890123456789012123456789012345678901 2
12345678901234567890123456789012123456789012345678901
4~2 2 2y! 1 3y 5 13 (Now we have one equation with 2
12345678901234567890123456789012123456789012345678901 2
12345678901234567890123456789012123456789012345678901 2
one unknown, so we can solve
12345678901234567890123456789012123456789012345678901 2
12345678901234567890123456789012123456789012345678901
for y.) 2
12345678901234567890123456789012123456789012345678901 2 2
1
12345678901234567890123456789012123456789012345678901
8 2 8y 1 3y 5 13 2
12345678901234567890123456789012123456789012345678901 2
12345678901234567890123456789012123456789012345678901
25y 5 5 2
12345678901234567890123456789012123456789012345678901 2
12345678901234567890123456789012123456789012345678901
y 5 21 (We can
2345678901234567890123456789012123456789012345678901
take this information and 2
2
12345678901234567890123456789012123456789012345678901
12345678901234567890123456789012123456789012345678901
plug it back into our equation 2
1 x 5 2 2 2y 2
12345678901234567890123456789012123456789012345678901
with x.) 2
12345678901234567890123456789012123456789012345678901
5 2 2 2(21) 2
2345678901234567890123456789012123456789012345678901
12345678901234567890123456789012123456789012345678901 2
12345678901234567890123456789012123456789012345678901
54 2
1 2
12345678901234567890123456789012123456789012345678901 2
12345678901234567890123456789012123456789012345678901 2
Once you have the values for both variables, you can determine what
2345678901234567890123456789012123456789012345678901
12345678901234567890123456789012123456789012345678901 2
12345678901234567890123456789012123456789012345678901
27xy equals. 2
12345678901234567890123456789012123456789012345678901 2 2
12345678901234567890123456789012123456789012345678901 2
1 7xy 5
12345678901234567890123456789012123456789012345678901 2
2345678901234567890123456789012123456789012345678901
2
12345678901234567890123456789012123456789012345678901
7~4!~21! 5 228
12345678901234567890123456789012123456789012345678901 2
12345678901234567890123456789012123456789012345678901 2 2
1
12345678901234567890123456789012123456789012345678901 2
12345678901234567890123456789012123456789012345678901 2
12345678901234567890123456789012123456789012345678901 2
12345678901234567890123456789012123456789012345678901 2
12345678901234567890123456789012123456789012345678901 2
12345678901234567890123456789012123456789012345678901 2
2345678901234567890123456789012123456789012345678901
2
1
12345678901234567890123456789012123456789012345678901 2
12345678901234567890123456789012123456789012345678901 2
12345678901234567890123456789012123456789012345678901 2
12345678901234567890123456789012123456789012345678901 2
12345678901234567890123456789012123456789012345678901 2
12345678901234567890123456789012123456789012345678901 2
12345678901234567890123456789012123456789012345678901 2
70 123456789012345678901234567890121234567890123456789012

www.petersons.com
Chapter 4: Quantitative Review

123456789012345678901234567890121234567890123456789012
12345678901234567890123456789012123456789012345678901 2
12345678901234567890123456789012123456789012345678901 2
12345678901234567890123456789012123456789012345678901
That was the first way. The second way is a little more inventive and can be 2
12345678901234567890123456789012123456789012345678901 2
12345678901234567890123456789012123456789012345678901
quicker. To do it, you want to make the coefficients of one of the variables 2
12345678901234567890123456789012123456789012345678901 2
12345678901234567890123456789012123456789012345678901
the same in both equations. Then you can subtract one equation from the 2
1 other and so eliminate one of the variables. In our example, it is easiest to 2
2345678901234567890123456789012123456789012345678901
12345678901234567890123456789012123456789012345678901 2
12345678901234567890123456789012123456789012345678901 2
12345678901234567890123456789012123456789012345678901
make both of the x’s have a coefficient of 4. You can get this by 2
12345678901234567890123456789012123456789012345678901
2345678901234567890123456789012123456789012345678901
2
2
1 multiplying the second equation by 4:
12345678901234567890123456789012123456789012345678901 2
12345678901234567890123456789012123456789012345678901 2
12345678901234567890123456789012123456789012345678901
x 1 2y 5 2 → 2
12345678901234567890123456789012123456789012345678901
2345678901234567890123456789012123456789012345678901
2
2
1 4~x 1 2y! 5 4~2! → Rule #2 at work!
12345678901234567890123456789012123456789012345678901 2
12345678901234567890123456789012123456789012345678901 2
12345678901234567890123456789012123456789012345678901
4x 1 2y 5 8 2
12345678901234567890123456789012123456789012345678901
2345678901234567890123456789012123456789012345678901
2
2
1 Line the two equations up and subtract one from the other:
12345678901234567890123456789012123456789012345678901 2
12345678901234567890123456789012123456789012345678901 2
12345678901234567890123456789012123456789012345678901 2
12345678901234567890123456789012123456789012345678901
4x 1 3y 5 13 2
2345678901234567890123456789012123456789012345678901
12345678901234567890123456789012123456789012345678901 2
12345678901234567890123456789012123456789012345678901
2 4x 1 8y 5 8 2
12345678901234567890123456789012123456789012345678901 2
12345678901234567890123456789012123456789012345678901
25y 5 5 2
12345678901234567890123456789012123456789012345678901 22
12345678901234567890123456789012123456789012345678901
y 5 21
2
1
12345678901234567890123456789012123456789012345678901 2
12345678901234567890123456789012123456789012345678901
Plugging this back in, you will find that x 5 4. As you would hope, this is 2
12345678901234567890123456789012123456789012345678901 2
12345678901234567890123456789012123456789012345678901
the same result that we got doing the problem the first way. 2
12345678901234567890123456789012123456789012345678901 2
12345678901234567890123456789012123456789012345678901 2
12345678901234567890123456789012123456789012345678901 2
12345678901234567890123456789012123456789012345678901 2
2345678901234567890123456789012123456789012345678901
12345678901234567890123456789012123456789012345678901
More Complicated Equations 2
12345678901234567890123456789012123456789012345678901 2
12345678901234567890123456789012123456789012345678901 2
12345678901234567890123456789012123456789012345678901 22
Equations on the GRE don’t get much more complicated than the previous
1 ones. There are, however, quadratic equations occasionally lurking around
12345678901234567890123456789012123456789012345678901 2
12345678901234567890123456789012123456789012345678901 2
12345678901234567890123456789012123456789012345678901 2
the test. These sound intimidating, but if you got through high school
12345678901234567890123456789012123456789012345678901
algebra, you have already mastered these imps once. You’ll just need a 2
12345678901234567890123456789012123456789012345678901
2345678901234567890123456789012123456789012345678901
2
2
12345678901234567890123456789012123456789012345678901
little refresher so you can master them again.
12345678901234567890123456789012123456789012345678901 2
1 2
12345678901234567890123456789012123456789012345678901
A quadratic equation means that one variable will be raised to a power like 2
12345678901234567890123456789012123456789012345678901 2
12345678901234567890123456789012123456789012345678901
this, x2. In order to solve these buggers, you often have to factor. 2
12345678901234567890123456789012123456789012345678901 2
12345678901234567890123456789012123456789012345678901 2
12345678901234567890123456789012123456789012345678901 2
12345678901234567890123456789012123456789012345678901 2
12345678901234567890123456789012123456789012345678901 2
12345678901234567890123456789012123456789012345678901
Factor Refresher 2
12345678901234567890123456789012123456789012345678901 2
12345678901234567890123456789012123456789012345678901 2
12345678901234567890123456789012123456789012345678901 2
ab 1 ac 5 a ~ b 1 c!
2345678901234567890123456789012123456789012345678901
12345678901234567890123456789012123456789012345678901 2
12345678901234567890123456789012123456789012345678901
~a 1 b!~c 1 d!5 ac 1 ad 1 bc 1 bd 2
12345678901234567890123456789012123456789012345678901 2
12345678901234567890123456789012123456789012345678901
2 2
a 1 2ab 1 b 5 ~a 1 b! 2
2
1 2
2345678901234567890123456789012123456789012345678901
12345678901234567890123456789012123456789012345678901
2 2
a 2 2ab 1 b 5 ~a 2 b! 2 2
12345678901234567890123456789012123456789012345678901 2
12345678901234567890123456789012123456789012345678901 2
12345678901234567890123456789012123456789012345678901 22
2 2
a 2 b 5 ~ a 2 b!~ a 1 b!
1
12345678901234567890123456789012123456789012345678901 2
12345678901234567890123456789012123456789012345678901 2
12345678901234567890123456789012123456789012345678901 2
12345678901234567890123456789012123456789012345678901 2
12345678901234567890123456789012123456789012345678901 2
12345678901234567890123456789012123456789012345678901 2
12345678901234567890123456789012123456789012345678901 2
12345678901234567890123456789012123456789012345678901 2
12345678901234567890123456789012123456789012345678901 2 71
123456789012345678901234567890121234567890123456789012
Part III: Section Review

123456789012345678901234567890121234567890123456789012
12345678901234567890123456789012123456789012345678901 2
12345678901234567890123456789012123456789012345678901 2
12345678901234567890123456789012123456789012345678901
Think back, way back, to Algebra I. You might recall the memory aid 2
12345678901234567890123456789012123456789012345678901 2
12345678901234567890123456789012123456789012345678901
called “FOIL.” FOIL is used when you are multiplying two groups of two 2
12345678901234567890123456789012123456789012345678901 2
12345678901234567890123456789012123456789012345678901
variables that look like this: (a 1 b)(c 1 d). The expression stands for, 2
2345678901234567890123456789012123456789012345678901 2
1 “First, Outside, Inside, Last.”
12345678901234567890123456789012123456789012345678901 2
12345678901234567890123456789012123456789012345678901 2
12345678901234567890123456789012123456789012345678901 2
12345678901234567890123456789012123456789012345678901
You multiply the First terms (a 3 c),
2345678901234567890123456789012123456789012345678901
2
2
1
12345678901234567890123456789012123456789012345678901 2
12345678901234567890123456789012123456789012345678901
the Outside terms (a 3 d), 2
12345678901234567890123456789012123456789012345678901 2
12345678901234567890123456789012123456789012345678901
the Inside terms (b 3 c),
2345678901234567890123456789012123456789012345678901
2
2
1
12345678901234567890123456789012123456789012345678901 2
12345678901234567890123456789012123456789012345678901
the Last terms (b 3 d), 2
12345678901234567890123456789012123456789012345678901 2
12345678901234567890123456789012123456789012345678901
2345678901234567890123456789012123456789012345678901
2
2
1 and then you add them all together.
12345678901234567890123456789012123456789012345678901 2
12345678901234567890123456789012123456789012345678901 2
12345678901234567890123456789012123456789012345678901
If you are asked to “expand”—that’s the term often used—the expression, 2
12345678901234567890123456789012123456789012345678901 2
12345678901234567890123456789012123456789012345678901 2
2345678901234567890123456789012123456789012345678901
just follow FOIL:
12345678901234567890123456789012123456789012345678901 2
12345678901234567890123456789012123456789012345678901 2
12345678901234567890123456789012123456789012345678901
~2x 1 3!~x 2 4! 5 2
12345678901234567890123456789012123456789012345678901 2
12345678901234567890123456789012123456789012345678901 2
2x 2
2 8x 1 3x2 12 5
2
1
12345678901234567890123456789012123456789012345678901
2x2 2 5x 2 12 5 2
12345678901234567890123456789012123456789012345678901 2
12345678901234567890123456789012123456789012345678901 2
12345678901234567890123456789012123456789012345678901 2
2345678901234567890123456789012123456789012345678901
Factoring is the reverse of FOIL. Factoring is when you make the math
12345678901234567890123456789012123456789012345678901
expression have parentheses instead of the other way around, so you take 2
2
12345678901234567890123456789012123456789012345678901
something like 2x2 2 5x 2 12 and must factor it into two parentheses like 2
12345678901234567890123456789012123456789012345678901
12345678901234567890123456789012123456789012345678901 2
12345678901234567890123456789012123456789012345678901 2
12345678901234567890123456789012123456789012345678901
the (2x 1 3)(x 2 4) you started with above. 2
12345678901234567890123456789012123456789012345678901 2
12345678901234567890123456789012123456789012345678901 2
1 Factoring is 2often not as straightforward as doing FOIL. Let’s use the 2
2345678901234567890123456789012123456789012345678901
expression x 2 2x 2 15 as an example of what needs to be factored. First, 2
12345678901234567890123456789012123456789012345678901
12345678901234567890123456789012123456789012345678901 2
1 if there is no coefficient in front of the squared term—x2 in our 2
12345678901234567890123456789012123456789012345678901 2
12345678901234567890123456789012123456789012345678901
example—then you can put an x in the first place of each parentheses, 2
12345678901234567890123456789012123456789012345678901 2
12345678901234567890123456789012123456789012345678901
(x )(x ). Next, notice that the 15 has a minus in front of it. This 2
12345678901234567890123456789012123456789012345678901 2
12345678901234567890123456789012123456789012345678901
could only be the case if one of the last terms in the parentheses is negative 2
12345678901234567890123456789012123456789012345678901 2
2345678901234567890123456789012123456789012345678901
12345678901234567890123456789012123456789012345678901
and the other last term is positive, which means (x 1 )(x 2 ). 2
12345678901234567890123456789012123456789012345678901 2
1 The product of the last two terms is 15. What factors give fifteen? 1 3 15 2
12345678901234567890123456789012123456789012345678901 2
12345678901234567890123456789012123456789012345678901 2
12345678901234567890123456789012123456789012345678901
and 3 3 5 answers this problem. One of these factored pairs is the one 2
12345678901234567890123456789012123456789012345678901
needed to complete the parentheses, but figuring out which one is correct 2
12345678901234567890123456789012123456789012345678901 2
12345678901234567890123456789012123456789012345678901 2
12345678901234567890123456789012123456789012345678901 2
can take some trial and error. The difference between the factors must be 2
2345678901234567890123456789012123456789012345678901
since that is the coefficient of the x-term, so 3 3 5 has to be what goes in 2
12345678901234567890123456789012123456789012345678901
12345678901234567890123456789012123456789012345678901 2
12345678901234567890123456789012123456789012345678901 2
12345678901234567890123456789012123456789012345678901 2
the blanks of our parentheses. But which goes where? Since the x-term is
1 negative (22), the larger number, 5, should go in front of the minus sign. 2
12345678901234567890123456789012123456789012345678901 2
12345678901234567890123456789012123456789012345678901 2
12345678901234567890123456789012123456789012345678901
This gives us (x 1 3)(x 2 5). 2
12345678901234567890123456789012123456789012345678901 2
12345678901234567890123456789012123456789012345678901 2
12345678901234567890123456789012123456789012345678901 2
2345678901234567890123456789012123456789012345678901
2
1
12345678901234567890123456789012123456789012345678901 2
12345678901234567890123456789012123456789012345678901 2
12345678901234567890123456789012123456789012345678901 2
12345678901234567890123456789012123456789012345678901 2
12345678901234567890123456789012123456789012345678901 2
12345678901234567890123456789012123456789012345678901 2
12345678901234567890123456789012123456789012345678901 2
72 123456789012345678901234567890121234567890123456789012

www.petersons.com
Chapter 4: Quantitative Review

123456789012345678901234567890121234567890123456789012
12345678901234567890123456789012123456789012345678901 2
12345678901234567890123456789012123456789012345678901 2
12345678901234567890123456789012123456789012345678901 2
12345678901234567890123456789012123456789012345678901 2
12345678901234567890123456789012123456789012345678901 2

Note
You can always check to see if you have factored correctly by FOILing to
12345678901234567890123456789012123456789012345678901 2
12345678901234567890123456789012123456789012345678901
see if you get what you started with:
2345678901234567890123456789012123456789012345678901
2
2
1
12345678901234567890123456789012123456789012345678901 2
12345678901234567890123456789012123456789012345678901
~ 2
x 1 3!~x 2 5! 5 x 2 5x 1 3x 2 15 5 x 2 2x 2 15 2
2
12345678901234567890123456789012123456789012345678901 2
12345678901234567890123456789012123456789012345678901
2345678901234567890123456789012123456789012345678901
2
2
1
12345678901234567890123456789012123456789012345678901 2
12345678901234567890123456789012123456789012345678901
You want to be comfortable using FOIL and factoring, since many 2
12345678901234567890123456789012123456789012345678901 2
12345678901234567890123456789012123456789012345678901
problems will require these math operations. If this one example of 2
1 factoring didn’t get all those old factoring juices flowing, then it is a good 2
2345678901234567890123456789012123456789012345678901
12345678901234567890123456789012123456789012345678901 2
12345678901234567890123456789012123456789012345678901 2
12345678901234567890123456789012123456789012345678901
idea to play around with factoring to get more comfortable with it. Make 2
12345678901234567890123456789012123456789012345678901
up a few expressions like (x 5)(x
2345678901234567890123456789012123456789012345678901 2) or (x 8)(x 2), expand them, 2
2
1 2 2 1 1
12345678901234567890123456789012123456789012345678901
and then factor them. With practice, factoring can become as straightfor- 2
12345678901234567890123456789012123456789012345678901 2
12345678901234567890123456789012123456789012345678901
ward as expanding. 2
12345678901234567890123456789012123456789012345678901 2
12345678901234567890123456789012123456789012345678901 22
2345678901234567890123456789012123456789012345678901
12345678901234567890123456789012123456789012345678901
Column A Column B
12345678901234567890123456789012123456789012345678901 22
12345678901234567890123456789012123456789012345678901
12345678901234567890123456789012123456789012345678901
c is a positive integer. 2
12345678901234567890123456789012123456789012345678901 2
1 2
12345678901234567890123456789012123456789012345678901 2
2 2
c 2 4 2c 2 c 2 6
12345678901234567890123456789012123456789012345678901 2
12345678901234567890123456789012123456789012345678901 2
2345678901234567890123456789012123456789012345678901
When you see a problem like this, you should immediately suspect that you 2
12345678901234567890123456789012123456789012345678901
12345678901234567890123456789012123456789012345678901 2
can factor these two expressions and then compare them directly. The GRE 2
12345678901234567890123456789012123456789012345678901
12345678901234567890123456789012123456789012345678901 22
12345678901234567890123456789012123456789012345678901
isn’t going to give a problem where the only way to solve it is just plug in
12345678901234567890123456789012123456789012345678901 2
numbers randomly; that’s just not how it’s designed. Column A is one of the 2
12345678901234567890123456789012123456789012345678901
12345678901234567890123456789012123456789012345678901
classic factors from the chart, although you may have to look hard to see 2
2
12345678901234567890123456789012123456789012345678901
1 a2 2 b2 5 (a 2 b)(a 1 b). It’s there, though, and c2 2 4 5 (c 1 2)(c 2 2). 2
12345678901234567890123456789012123456789012345678901 2
12345678901234567890123456789012123456789012345678901 2
12345678901234567890123456789012123456789012345678901 2
12345678901234567890123456789012123456789012345678901 2
Keep these results in mind when factoring Column B. You should be on the
12345678901234567890123456789012123456789012345678901
look out for a (c 1 2) or a (c 2 2) since that would make it easier to directly 2
12345678901234567890123456789012123456789012345678901 2
12345678901234567890123456789012123456789012345678901 2
12345678901234567890123456789012123456789012345678901 2
compare Column A and B. In factoring Column B, the first terms can only
12345678901234567890123456789012123456789012345678901
). If you don’t see that, play with the numbers and it 2
12345678901234567890123456789012123456789012345678901
be: (2c )(c 2
2345678901234567890123456789012123456789012345678901
should become clear. Since there is a negative sign in front of the 6, there 2
12345678901234567890123456789012123456789012345678901
12345678901234567890123456789012123456789012345678901 2
1 must be one plus and one minus sign in the parentheses. The factor pairs of 2
12345678901234567890123456789012123456789012345678901 2
12345678901234567890123456789012123456789012345678901
6 are 1 3 6 and 2 3 3. Since the middle c-term has a coefficient of one, 2 2
12345678901234567890123456789012123456789012345678901 2
12345678901234567890123456789012123456789012345678901
and 3 are probably our factors, but with the 2 in front of the first c, we 2
12345678901234567890123456789012123456789012345678901 2
12345678901234567890123456789012123456789012345678901
can’t be sure. 2
12345678901234567890123456789012123456789012345678901 2
2345678901234567890123456789012123456789012345678901
12345678901234567890123456789012123456789012345678901
Play with the numbers to see if 2 and 3 can make it work. After some 2
2
12345678901234567890123456789012123456789012345678901 2
12345678901234567890123456789012123456789012345678901
unavoidable trial and error, you should hit upon (2c 1 3)(c 2 2). Since 2
12345678901234567890123456789012123456789012345678901
1 both columns have a (c 2 2) term in them, you can disregard that term in 2
12345678901234567890123456789012123456789012345678901 2
12345678901234567890123456789012123456789012345678901 2
12345678901234567890123456789012123456789012345678901
each column and focus on the other terms. You are now left comparing (c 2
12345678901234567890123456789012123456789012345678901
1 2) and (2c 1 3). Since c is a positive integer, it seems likely that (2c 1 3) 2
12345678901234567890123456789012123456789012345678901 2
2345678901234567890123456789012123456789012345678901
12345678901234567890123456789012123456789012345678901 2
1 is greater, but plug in a few numbers to confirm this if you aren’t sure. 2
12345678901234567890123456789012123456789012345678901
Make sure to plug in c 5 1, since 1 is the lowest positive integer and 2
12345678901234567890123456789012123456789012345678901 2
12345678901234567890123456789012123456789012345678901
2345678901234567890123456789012123456789012345678901
2
2
1 therefore the most likely candidate for making (A) greater or equal to (B).
12345678901234567890123456789012123456789012345678901 2
12345678901234567890123456789012123456789012345678901 22
It turns out that (B) is always greater than (A), so the correct answer is (B).
12345678901234567890123456789012123456789012345678901
123456789012345678901234567890121234567890123456789012 73
Part III: Section Review

123456789012345678901234567890121234567890123456789012
12345678901234567890123456789012123456789012345678901 2
12345678901234567890123456789012123456789012345678901 2
12345678901234567890123456789012123456789012345678901
When factoring quadratic equations, the first thing to be done is to get all 2
12345678901234567890123456789012123456789012345678901 2
12345678901234567890123456789012123456789012345678901
the numbers and variables on one side and have the other side of the 2
12345678901234567890123456789012123456789012345678901 2
12345678901234567890123456789012123456789012345678901
equation equal zero. Suppose you were given the equation x2 1 x 2 6 5 6, 2
2345678901234567890123456789012123456789012345678901 2
1 and asked to determine the value(s) of x. The first step is:
12345678901234567890123456789012123456789012345678901 2
12345678901234567890123456789012123456789012345678901 2
12345678901234567890123456789012123456789012345678901 2
12345678901234567890123456789012123456789012345678901
2
x 1x2656→
2345678901234567890123456789012123456789012345678901
2
2
1
12345678901234567890123456789012123456789012345678901
x 2
x 6 6 6 6 → 2
12345678901234567890123456789012123456789012345678901
1 2 2 5 2 2
12345678901234567890123456789012123456789012345678901
2 2
12345678901234567890123456789012123456789012345678901
x 1 x 2 12 5 0
2345678901234567890123456789012123456789012345678901
2
2
1
12345678901234567890123456789012123456789012345678901 2
12345678901234567890123456789012123456789012345678901 2
12345678901234567890123456789012123456789012345678901 2

Note
12345678901234567890123456789012123456789012345678901
2345678901234567890123456789012123456789012345678901
2
2
1 There is a good reason why you do this, but you don’t need to know it
12345678901234567890123456789012123456789012345678901 2
12345678901234567890123456789012123456789012345678901
because GRE math tests the how’s and not the why’s of algebra. 2
12345678901234567890123456789012123456789012345678901 2
12345678901234567890123456789012123456789012345678901 2
12345678901234567890123456789012123456789012345678901 2
2345678901234567890123456789012123456789012345678901
12345678901234567890123456789012123456789012345678901 2
Now it’s time to factor, and with some work you will get: x2 1 x 2 12 5 2
12345678901234567890123456789012123456789012345678901
12345678901234567890123456789012123456789012345678901 2
12345678901234567890123456789012123456789012345678901 2
12345678901234567890123456789012123456789012345678901 2
(x 1 4)(x 2 3). To determine the value(s) of x, look at the two sets of
1 parentheses and remember that any number multiplied by zero is zero. You 2
12345678901234567890123456789012123456789012345678901 2
12345678901234567890123456789012123456789012345678901 2
12345678901234567890123456789012123456789012345678901
can deduce that either (x 1 4) 5 0 or (x 2 3) 5 0 equals zero, although at 2
2345678901234567890123456789012123456789012345678901
this point you don’t know which. When you are dealing with quadratic 2
12345678901234567890123456789012123456789012345678901
12345678901234567890123456789012123456789012345678901 2
12345678901234567890123456789012123456789012345678901 2
12345678901234567890123456789012123456789012345678901 2
equations (equations that have squared terms in them), there will often be
two answers. In this example, x equals 24 or 3. Keep the “two answers are 2
12345678901234567890123456789012123456789012345678901
12345678901234567890123456789012123456789012345678901 2
12345678901234567890123456789012123456789012345678901
correct” point in mind when you see quadratic equations, since that’s the 2
12345678901234567890123456789012123456789012345678901 2
12345678901234567890123456789012123456789012345678901 2
1 kind of funky math knowledge that the test-makers love to test. 2
12345678901234567890123456789012123456789012345678901 2
2345678901234567890123456789012123456789012345678901
12345678901234567890123456789012123456789012345678901 2
1 2
3x 1 11x 1 6 2
12345678901234567890123456789012123456789012345678901
If 2 5 0, then x 5 2
12345678901234567890123456789012123456789012345678901
x 1 6x 1 9 2
12345678901234567890123456789012123456789012345678901 2
2345678901234567890123456789012123456789012345678901
12345678901234567890123456789012123456789012345678901
2 2
1 2
12345678901234567890123456789012123456789012345678901
A. 2
3 2
12345678901234567890123456789012123456789012345678901 2
2345678901234567890123456789012123456789012345678901
12345678901234567890123456789012123456789012345678901 2
12345678901234567890123456789012123456789012345678901
B. 21 2
1 2
12345678901234567890123456789012123456789012345678901 2
12345678901234567890123456789012123456789012345678901
3 2
2345678901234567890123456789012123456789012345678901
12345678901234567890123456789012123456789012345678901
C. 2
12345678901234567890123456789012123456789012345678901
5 2
12345678901234567890123456789012123456789012345678901 2
12345678901234567890123456789012123456789012345678901
D. 2 2
1 2
2345678901234567890123456789012123456789012345678901
12345678901234567890123456789012123456789012345678901
E. 4 2
12345678901234567890123456789012123456789012345678901 2
12345678901234567890123456789012123456789012345678901
As always, don’t be intimidated by how complicated that algebra 2
2
12345678901234567890123456789012123456789012345678901
1 expression looks. If you see something like this, you should know that the 2
12345678901234567890123456789012123456789012345678901 2
12345678901234567890123456789012123456789012345678901 2
12345678901234567890123456789012123456789012345678901
winning strategy on this type of problem is to find a way to simplify it. The 2
12345678901234567890123456789012123456789012345678901 2
12345678901234567890123456789012123456789012345678901
denominator looks less intricate, so let’s start there: 2
2345678901234567890123456789012123456789012345678901
12345678901234567890123456789012123456789012345678901 2
1 2 2
12345678901234567890123456789012123456789012345678901
x 1 6x 1 9 5 (x 1 3)(x 1 3) 2
12345678901234567890123456789012123456789012345678901 2
12345678901234567890123456789012123456789012345678901 2
12345678901234567890123456789012123456789012345678901 2
12345678901234567890123456789012123456789012345678901 2
12345678901234567890123456789012123456789012345678901 2
12345678901234567890123456789012123456789012345678901 2
74 123456789012345678901234567890121234567890123456789012

www.petersons.com
Chapter 4: Quantitative Review

123456789012345678901234567890121234567890123456789012
12345678901234567890123456789012123456789012345678901 2
12345678901234567890123456789012123456789012345678901 2
12345678901234567890123456789012123456789012345678901
It is a good bet that in factoring the numerator, you will find a (x 1 3) 2
12345678901234567890123456789012123456789012345678901 2
12345678901234567890123456789012123456789012345678901
term. Start with that premise and see where it leads you: 2
12345678901234567890123456789012123456789012345678901 2
12345678901234567890123456789012123456789012345678901 2
12345678901234567890123456789012123456789012345678901
3x2 1 11x 1 6 5 (x 1 3)(3x 1 2) 2
12345678901234567890123456789012123456789012345678901 2
12345678901234567890123456789012123456789012345678901 2
12345678901234567890123456789012123456789012345678901
There it is. It’s not a coincidence, either. Putting it all together, you have: 2
12345678901234567890123456789012123456789012345678901
2345678901234567890123456789012123456789012345678901
2
2
1
12345678901234567890123456789012123456789012345678901 2
2
3x 1 11x 1 6 ~x 1 3!~3x 1 2! ~3x 1 2!
12345678901234567890123456789012123456789012345678901
5 5 50 2
12345678901234567890123456789012123456789012345678901
x 2
1 6x 1 9 ~ x 1 3!~x 1 3! ~ x 1 3 ! 2
12345678901234567890123456789012123456789012345678901 2
1 Remember that a fraction equals zero if the numerator equals zero, and 2
2345678901234567890123456789012123456789012345678901
12345678901234567890123456789012123456789012345678901 2
12345678901234567890123456789012123456789012345678901 2
12345678901234567890123456789012123456789012345678901
2 2
12345678901234567890123456789012123456789012345678901
(3x 1 2) 5 0 if x 5 2 , which is the correct
2345678901234567890123456789012123456789012345678901
answer, choice (A). 2
2
1 3
12345678901234567890123456789012123456789012345678901 2
12345678901234567890123456789012123456789012345678901 2
12345678901234567890123456789012123456789012345678901 2
12345678901234567890123456789012123456789012345678901 2
12345678901234567890123456789012123456789012345678901 22
2345678901234567890123456789012123456789012345678901
Not Equations But Inequalities
12345678901234567890123456789012123456789012345678901
Inequalities are like equations except that there is not an equal sign between 2
12345678901234567890123456789012123456789012345678901
12345678901234567890123456789012123456789012345678901 2
12345678901234567890123456789012123456789012345678901
the left and right side. Instead you might find one of the following: 2
12345678901234567890123456789012123456789012345678901 22
1
12345678901234567890123456789012123456789012345678901
Various Inequality Signs 2
12345678901234567890123456789012123456789012345678901 2
12345678901234567890123456789012123456789012345678901
Þ not equal to 2
2345678901234567890123456789012123456789012345678901
12345678901234567890123456789012123456789012345678901 2
12345678901234567890123456789012123456789012345678901
, less than 2
12345678901234567890123456789012123456789012345678901 22
12345678901234567890123456789012123456789012345678901
≤ less than or equal to
12345678901234567890123456789012123456789012345678901 2
12345678901234567890123456789012123456789012345678901 2
12345678901234567890123456789012123456789012345678901
. greater than 2
12345678901234567890123456789012123456789012345678901 2
12345678901234567890123456789012123456789012345678901
≥ greater than or equal to 2
1 2
2345678901234567890123456789012123456789012345678901
If 4x 1 2 , 12, then 4x 1 2 is less than 12. If you needed to solve for x in 2
12345678901234567890123456789012123456789012345678901
12345678901234567890123456789012123456789012345678901 2
1 this inequality, the rules are the same as for equations. Whatever you do to 2
12345678901234567890123456789012123456789012345678901 2
12345678901234567890123456789012123456789012345678901 2
12345678901234567890123456789012123456789012345678901
one side, you have to do to the other. For example: 2
12345678901234567890123456789012123456789012345678901 2
12345678901234567890123456789012123456789012345678901 2
12345678901234567890123456789012123456789012345678901
4x 1 2 , 12 → 2
12345678901234567890123456789012123456789012345678901 2
12345678901234567890123456789012123456789012345678901 22
2345678901234567890123456789012123456789012345678901
4x 1 2 2 2 , 12 2 2 →
12345678901234567890123456789012123456789012345678901 2
1 4x , 10 →
12345678901234567890123456789012123456789012345678901 2
12345678901234567890123456789012123456789012345678901
→ 2
12345678901234567890123456789012123456789012345678901 22
2345678901234567890123456789012123456789012345678901
4x 4 4 , 10 4 4
12345678901234567890123456789012123456789012345678901 2
12345678901234567890123456789012123456789012345678901
5
2
12345678901234567890123456789012123456789012345678901
x,
2
1 2
12345678901234567890123456789012123456789012345678901 22
2345678901234567890123456789012123456789012345678901
12345678901234567890123456789012123456789012345678901
There is one caveat to working with inequalities. If you multiply or divide 2
12345678901234567890123456789012123456789012345678901 2
12345678901234567890123456789012123456789012345678901
1 by a negative number, the direction of the inequality sign switches. Freaky, 2
2345678901234567890123456789012123456789012345678901
eh? Suppose you need to solve the inequality 4 1 2g . 8 2 4g. First, collect 2
12345678901234567890123456789012123456789012345678901
12345678901234567890123456789012123456789012345678901 2
12345678901234567890123456789012123456789012345678901 2
12345678901234567890123456789012123456789012345678901 22
all the g’s on one side:
1
12345678901234567890123456789012123456789012345678901 2
12345678901234567890123456789012123456789012345678901
4 1 2g . 8 2 4g → 2
12345678901234567890123456789012123456789012345678901 2
12345678901234567890123456789012123456789012345678901
4 2 8 1 2g 2 2 g . 8 2 8 2 4 g 2 2 g → 2
12345678901234567890123456789012123456789012345678901
2345678901234567890123456789012123456789012345678901
2
2
1 2 4 . 2 6g
12345678901234567890123456789012123456789012345678901 2
12345678901234567890123456789012123456789012345678901 22
12345678901234567890123456789012123456789012345678901
123456789012345678901234567890121234567890123456789012 75
Part III: Section Review

123456789012345678901234567890121234567890123456789012
12345678901234567890123456789012123456789012345678901 2
12345678901234567890123456789012123456789012345678901 2
12345678901234567890123456789012123456789012345678901
Divide each side by 26 and remember to switch the inequality. You get 2
12345678901234567890123456789012123456789012345678901 2
12345678901234567890123456789012123456789012345678901
2 2
12345678901234567890123456789012123456789012345678901
, g. 2
12345678901234567890123456789012123456789012345678901
3
2345678901234567890123456789012123456789012345678901
2
2
1
12345678901234567890123456789012123456789012345678901 2
12345678901234567890123456789012123456789012345678901
Column A Column B 2
12345678901234567890123456789012123456789012345678901 2
12345678901234567890123456789012123456789012345678901
2345678901234567890123456789012123456789012345678901
2
2
1 10 . 2 2x 1 3 . 4
12345678901234567890123456789012123456789012345678901 2
12345678901234567890123456789012123456789012345678901
x 23 2
12345678901234567890123456789012123456789012345678901 2
12345678901234567890123456789012123456789012345678901 2
1 If 10 . 22x 1 3 . 4 looks a little overwhelming, just think of it as two 2
2345678901234567890123456789012123456789012345678901
12345678901234567890123456789012123456789012345678901 2
12345678901234567890123456789012123456789012345678901 2
12345678901234567890123456789012123456789012345678901
separate inequalities (10 . 22x 13 and 22x 1 3 . 4) and solve them 2
12345678901234567890123456789012123456789012345678901
2345678901234567890123456789012123456789012345678901
2
2
1 separately.
12345678901234567890123456789012123456789012345678901 2
12345678901234567890123456789012123456789012345678901 2
12345678901234567890123456789012123456789012345678901
22x 1 3 . 4 → 2
12345678901234567890123456789012123456789012345678901 2
12345678901234567890123456789012123456789012345678901 2
2345678901234567890123456789012123456789012345678901
22x 1 3 2 3 . 4 2 3 →
12345678901234567890123456789012123456789012345678901 2
12345678901234567890123456789012123456789012345678901
22x . 1 → 2
12345678901234567890123456789012123456789012345678901 2
12345678901234567890123456789012123456789012345678901
→ 2
12345678901234567890123456789012123456789012345678901 2
22x 4 22 . 1 4 22
2
1
12345678901234567890123456789012123456789012345678901
1 2
12345678901234567890123456789012123456789012345678901
x,2 2
12345678901234567890123456789012123456789012345678901
2 2
2345678901234567890123456789012123456789012345678901
12345678901234567890123456789012123456789012345678901 2
12345678901234567890123456789012123456789012345678901
and 2
12345678901234567890123456789012123456789012345678901 2
12345678901234567890123456789012123456789012345678901 2
12345678901234567890123456789012123456789012345678901
10 . 22x 1 3 → 2
12345678901234567890123456789012123456789012345678901 2
12345678901234567890123456789012123456789012345678901
10 2 3 . 22x 1 3 2 3 → 2
12345678901234567890123456789012123456789012345678901 2
12345678901234567890123456789012123456789012345678901
→ 2
1 7 . 22x 2
2345678901234567890123456789012123456789012345678901
12345678901234567890123456789012123456789012345678901 2
12345678901234567890123456789012123456789012345678901
7 4 22 . 22x 4 22 → 2
1 2
12345678901234567890123456789012123456789012345678901
7 2
12345678901234567890123456789012123456789012345678901
2 ,x
2345678901234567890123456789012123456789012345678901
2
2
12345678901234567890123456789012123456789012345678901
2
12345678901234567890123456789012123456789012345678901 2
1 Column B is an actual number, 23. As for Column A, all you know is that 2
12345678901234567890123456789012123456789012345678901 2
12345678901234567890123456789012123456789012345678901 2
12345678901234567890123456789012123456789012345678901 2
2345678901234567890123456789012123456789012345678901
x is between negative seven halves (23.5) and negative one and one half.
12345678901234567890123456789012123456789012345678901 2
1 This means that x could be smaller than, equal to, or greater than 23, 2
12345678901234567890123456789012123456789012345678901 2
12345678901234567890123456789012123456789012345678901
depending on where it is within its range. The correct answer is (D). 2
12345678901234567890123456789012123456789012345678901 2
2345678901234567890123456789012123456789012345678901
12345678901234567890123456789012123456789012345678901 2
12345678901234567890123456789012123456789012345678901 2
12345678901234567890123456789012123456789012345678901 2 2
1A Statistics Excursus (Look It Up!)
12345678901234567890123456789012123456789012345678901 2
12345678901234567890123456789012123456789012345678901 2
12345678901234567890123456789012123456789012345678901
The next three statistical concepts are kind of like the Marx Brothers. 2
12345678901234567890123456789012123456789012345678901 2
12345678901234567890123456789012123456789012345678901 2
There are three main ones, and then another that’s sort of the Zeppo of the
2345678901234567890123456789012123456789012345678901
bunch. Some combination of mean, median, and mode will make an 2
12345678901234567890123456789012123456789012345678901
12345678901234567890123456789012123456789012345678901 2
12345678901234567890123456789012123456789012345678901
appearance on your GRE math section. Test-makers like to make 2
12345678901234567890123456789012123456789012345678901 2
1 questions using these statistical concepts because they test your ability to 2
12345678901234567890123456789012123456789012345678901 2
12345678901234567890123456789012123456789012345678901
apply a concept to a math scenario. 2
12345678901234567890123456789012123456789012345678901 2
12345678901234567890123456789012123456789012345678901 2
12345678901234567890123456789012123456789012345678901 2
12345678901234567890123456789012123456789012345678901 2
12345678901234567890123456789012123456789012345678901 2
12345678901234567890123456789012123456789012345678901 2
12345678901234567890123456789012123456789012345678901 2
76 123456789012345678901234567890121234567890123456789012

www.petersons.com
Chapter 4: Quantitative Review

123456789012345678901234567890121234567890123456789012
12345678901234567890123456789012123456789012345678901 2
12345678901234567890123456789012123456789012345678901 2
12345678901234567890123456789012123456789012345678901 2
12345678901234567890123456789012123456789012345678901
Statistics Refresher 2
12345678901234567890123456789012123456789012345678901 2
12345678901234567890123456789012123456789012345678901 2
12345678901234567890123456789012123456789012345678901
arithmetic mean, mean, or
2345678901234567890123456789012123456789012345678901
sum of numbers divided by the 2
2
1
12345678901234567890123456789012123456789012345678901
average number of items in set 2
12345678901234567890123456789012123456789012345678901 2
12345678901234567890123456789012123456789012345678901
median middle number when set is 2
12345678901234567890123456789012123456789012345678901
2345678901234567890123456789012123456789012345678901
2
2
1 lined up in numerical order
12345678901234567890123456789012123456789012345678901 2
12345678901234567890123456789012123456789012345678901 2
12345678901234567890123456789012123456789012345678901
mode number that appears most often 2
12345678901234567890123456789012123456789012345678901
2345678901234567890123456789012123456789012345678901
2
2
1 in a set
12345678901234567890123456789012123456789012345678901 2
12345678901234567890123456789012123456789012345678901
range difference between the greatest 2
12345678901234567890123456789012123456789012345678901 2
12345678901234567890123456789012123456789012345678901
2345678901234567890123456789012123456789012345678901
and smallest values in set 2
2
1
12345678901234567890123456789012123456789012345678901 2
12345678901234567890123456789012123456789012345678901 2
12345678901234567890123456789012123456789012345678901
To compare all the terms, consider the number set, {14, 12, 8, 15, 5, 8}. To 2
12345678901234567890123456789012123456789012345678901 2
12345678901234567890123456789012123456789012345678901 22
2345678901234567890123456789012123456789012345678901
determine the mean, you add up the numbers in the set (14 1 12 1 8 1 15
12345678901234567890123456789012123456789012345678901
1 5 1 8 5 62), then divide the sum by number of items in set. There are six 2
12345678901234567890123456789012123456789012345678901
12345678901234567890123456789012123456789012345678901
62 1 2
12345678901234567890123456789012123456789012345678901 2
12345678901234567890123456789012123456789012345678901
items, or numbers, total, so 5 10 . This is the average mean of the set. 2
1 6 3 2
12345678901234567890123456789012123456789012345678901 2
12345678901234567890123456789012123456789012345678901
When people use the word average in everyday speech, they are referring 2
12345678901234567890123456789012123456789012345678901 2
12345678901234567890123456789012123456789012345678901 22
2345678901234567890123456789012123456789012345678901
to mean. But there are two other ways to find what can be considered an
12345678901234567890123456789012123456789012345678901
average of a group and these are median and mode. To find the median, 2
12345678901234567890123456789012123456789012345678901
you must first line up the numbers in numerical order, {5, 8, 8, 12, 14, 15}. 2
12345678901234567890123456789012123456789012345678901
12345678901234567890123456789012123456789012345678901 2
You then pick the one in the middle, and that’s the median. This works 2
12345678901234567890123456789012123456789012345678901
12345678901234567890123456789012123456789012345678901 2
12345678901234567890123456789012123456789012345678901 2
12345678901234567890123456789012123456789012345678901 22
quickly if there is an odd number of items in the set, but it takes a teensy bit
1 of work if there is an even number of items in the set. If that’s the case, the
12345678901234567890123456789012123456789012345678901 2
12345678901234567890123456789012123456789012345678901
there is not one single number in the middle. Instead, there are two in the 2
12345678901234567890123456789012123456789012345678901 2
12345678901234567890123456789012123456789012345678901
middle, 8 and 12. To find the median of this set, you must find the mean of 2
12345678901234567890123456789012123456789012345678901
2345678901234567890123456789012123456789012345678901
2
2
12345678901234567890123456789012123456789012345678901
8 1 12 20
12345678901234567890123456789012123456789012345678901
the two middle numbers. 5 5 10. The median of this group of 2
1 2 2 2
12345678901234567890123456789012123456789012345678901 2
12345678901234567890123456789012123456789012345678901
numbers is 10, even though 10 is not one of the numbers listed in the set. 2
12345678901234567890123456789012123456789012345678901 22
2345678901234567890123456789012123456789012345678901
12345678901234567890123456789012123456789012345678901
1 The mode of a set is the number that appears most often. It is 8 for the 2
12345678901234567890123456789012123456789012345678901
number set in question, since this is the only number that occurs more than 2
12345678901234567890123456789012123456789012345678901 2
2345678901234567890123456789012123456789012345678901
once. To find the range (the Zeppo of this group), you subtract the smallest 2
12345678901234567890123456789012123456789012345678901
12345678901234567890123456789012123456789012345678901 2
12345678901234567890123456789012123456789012345678901
number from the greatest number, 15 5 10. 2
12345678901234567890123456789012123456789012345678901 22
2 5
1
12345678901234567890123456789012123456789012345678901
The test, though, won’t ask you questions as easy as, “What is the 2
12345678901234567890123456789012123456789012345678901 2
12345678901234567890123456789012123456789012345678901
average?” It will expect you to be able to find these different values and 2
12345678901234567890123456789012123456789012345678901 2
12345678901234567890123456789012123456789012345678901
then work a problem from there. For example, with the number set above, 2
2345678901234567890123456789012123456789012345678901
12345678901234567890123456789012123456789012345678901 2
12345678901234567890123456789012123456789012345678901
you might be asked a QC question like: 2
12345678901234567890123456789012123456789012345678901 22
12345678901234567890123456789012123456789012345678901 2
1 Column A Column B
12345678901234567890123456789012123456789012345678901 2
12345678901234567890123456789012123456789012345678901 2
12345678901234567890123456789012123456789012345678901
{14, 12, 8, 15, 5, 8} 2
12345678901234567890123456789012123456789012345678901 2
12345678901234567890123456789012123456789012345678901 2
12345678901234567890123456789012123456789012345678901
The median of The range of the set 2
12345678901234567890123456789012123456789012345678901 2
12345678901234567890123456789012123456789012345678901 22
the set
12345678901234567890123456789012123456789012345678901
123456789012345678901234567890121234567890123456789012 77
Part III: Section Review

123456789012345678901234567890121234567890123456789012
12345678901234567890123456789012123456789012345678901 2
12345678901234567890123456789012123456789012345678901 2
12345678901234567890123456789012123456789012345678901
You have already found the value of 10 for both these concepts, so the 2
12345678901234567890123456789012123456789012345678901 2
12345678901234567890123456789012123456789012345678901
answer to this question would be (C). This problem is easy for anyone 2
12345678901234567890123456789012123456789012345678901 2
12345678901234567890123456789012123456789012345678901
knowledgeable with the terms, but to someone who doesn’t know what is 2
2345678901234567890123456789012123456789012345678901 2
1 being asked, it would be time for a one-in-four guess.
12345678901234567890123456789012123456789012345678901 2
12345678901234567890123456789012123456789012345678901 2
12345678901234567890123456789012123456789012345678901 2
12345678901234567890123456789012123456789012345678901
2345678901234567890123456789012123456789012345678901
2
2
1 Gina has received the following four scores on tests: 87, 92, 90, and
12345678901234567890123456789012123456789012345678901 2
12345678901234567890123456789012123456789012345678901
85. If after a fifth test Gina’s test average was 91, what was her 2
12345678901234567890123456789012123456789012345678901 2
12345678901234567890123456789012123456789012345678901
score on the fifth test?
2345678901234567890123456789012123456789012345678901
2
2
1
12345678901234567890123456789012123456789012345678901
A. 85 2
12345678901234567890123456789012123456789012345678901 2
12345678901234567890123456789012123456789012345678901
B. 88 2
12345678901234567890123456789012123456789012345678901
2345678901234567890123456789012123456789012345678901
2
2
1 C. 95
12345678901234567890123456789012123456789012345678901 2
12345678901234567890123456789012123456789012345678901
D. 98 2
12345678901234567890123456789012123456789012345678901 2
12345678901234567890123456789012123456789012345678901
E. 101 2
12345678901234567890123456789012123456789012345678901 2
2345678901234567890123456789012123456789012345678901
There are two ways to attack this problem. The first is to set up an 2
12345678901234567890123456789012123456789012345678901
12345678901234567890123456789012123456789012345678901 2
12345678901234567890123456789012123456789012345678901 2
12345678901234567890123456789012123456789012345678901 2
equation using the average formula and place a variable for the unknown
12345678901234567890123456789012123456789012345678901 2
1 fifth test score, 87 1 92 1 90 1 85 1 x 5 91. You then solve for x. The 2
12345678901234567890123456789012123456789012345678901 2
12345678901234567890123456789012123456789012345678901
5 2
12345678901234567890123456789012123456789012345678901
second way of attack is to take the likely answer choices and use them as 2
2345678901234567890123456789012123456789012345678901
12345678901234567890123456789012123456789012345678901 2
the fifth test score and see if the set averages to 91. With a little mental 2
12345678901234567890123456789012123456789012345678901
12345678901234567890123456789012123456789012345678901
work, you can guess that neither 85 nor 88 is the correct answer since the 2
2
12345678901234567890123456789012123456789012345678901
12345678901234567890123456789012123456789012345678901
fifth test will need to be above 91 to bring the average up to that. 2
2
12345678901234567890123456789012123456789012345678901
12345678901234567890123456789012123456789012345678901
Proceeding with this approach, you should start with 98, the middle of the 2
2
12345678901234567890123456789012123456789012345678901
12345678901234567890123456789012123456789012345678901 2
1 three remaining numbers. This way, if 98 is not the answer, you should be 2
2345678901234567890123456789012123456789012345678901
able to figure out what the right answer is. Placing 98 as the fifth score, 2
12345678901234567890123456789012123456789012345678901
12345678901234567890123456789012123456789012345678901 2
1 you get 87 1 92 1 90 1 85 1 98 5 452. However, since 91 3 5 5 455, 2
12345678901234567890123456789012123456789012345678901 2
12345678901234567890123456789012123456789012345678901
you know this answer choice is 3 points too low. You need a higher score, 2
12345678901234567890123456789012123456789012345678901 2
12345678901234567890123456789012123456789012345678901 2
12345678901234567890123456789012123456789012345678901 2
one preferably higher by three points. Looking at your answer choices,
12345678901234567890123456789012123456789012345678901
there is only one possible choice that’s higher, choice (E). It’s the correct 2
12345678901234567890123456789012123456789012345678901 2
2345678901234567890123456789012123456789012345678901
12345678901234567890123456789012123456789012345678901 2
12345678901234567890123456789012123456789012345678901
answer. 2
1 2
12345678901234567890123456789012123456789012345678901
This approach might seem unconventional or non-mathy, but that’s good. 2
12345678901234567890123456789012123456789012345678901 2
12345678901234567890123456789012123456789012345678901
You don’t want to limit yourself to a single method for all problems. The 2
12345678901234567890123456789012123456789012345678901 2
12345678901234567890123456789012123456789012345678901
more creative you get in approaching problems, the more flexibility you 2
12345678901234567890123456789012123456789012345678901 2
12345678901234567890123456789012123456789012345678901
give yourself to find the right answer. 2
12345678901234567890123456789012123456789012345678901 2
2345678901234567890123456789012123456789012345678901
12345678901234567890123456789012123456789012345678901 2
12345678901234567890123456789012123456789012345678901 2
12345678901234567890123456789012123456789012345678901 2 2
1Word Problems
12345678901234567890123456789012123456789012345678901 2
12345678901234567890123456789012123456789012345678901 2
12345678901234567890123456789012123456789012345678901
A train leaves Pittsburgh at 12 p.m. (EST) traveling due east at a speed of 2
12345678901234567890123456789012123456789012345678901 2
12345678901234567890123456789012123456789012345678901 2
45 mph. A second train leaves San Francisco at 2 p.m. (PST) traveling at 50
2345678901234567890123456789012123456789012345678901
12345678901234567890123456789012123456789012345678901 2
1 mph heading due north. When will the two trains cross paths? 2
12345678901234567890123456789012123456789012345678901 2
12345678901234567890123456789012123456789012345678901 2
12345678901234567890123456789012123456789012345678901 2
12345678901234567890123456789012123456789012345678901 2
12345678901234567890123456789012123456789012345678901 2
12345678901234567890123456789012123456789012345678901 2
12345678901234567890123456789012123456789012345678901 2
78 123456789012345678901234567890121234567890123456789012

www.petersons.com
Chapter 4: Quantitative Review

123456789012345678901234567890121234567890123456789012
12345678901234567890123456789012123456789012345678901 2
12345678901234567890123456789012123456789012345678901 2
12345678901234567890123456789012123456789012345678901 2
12345678901234567890123456789012123456789012345678901 2
12345678901234567890123456789012123456789012345678901 2

Alert!
12345678901234567890123456789012123456789012345678901
The trains will never meet. The Pittsburgh locomotive will soon run into 2
12345678901234567890123456789012123456789012345678901
2345678901234567890123456789012123456789012345678901
2
2
1 the Atlantic Ocean, while the San Fran train will take a while before
12345678901234567890123456789012123456789012345678901 2
12345678901234567890123456789012123456789012345678901
sliding off into the Pacific. 2
12345678901234567890123456789012123456789012345678901 2
12345678901234567890123456789012123456789012345678901
2345678901234567890123456789012123456789012345678901
2
2
1
12345678901234567890123456789012123456789012345678901 2
12345678901234567890123456789012123456789012345678901 2
12345678901234567890123456789012123456789012345678901
Admittedly, you probably won’t see that word problem on test day, but it 2
12345678901234567890123456789012123456789012345678901 2
12345678901234567890123456789012123456789012345678901
does illustrate that word problems are questions that contain a set of math 2
12345678901234567890123456789012123456789012345678901
conditions in written form. Many word problems on the GRE involve 2
12345678901234567890123456789012123456789012345678901 2
12345678901234567890123456789012123456789012345678901 2
12345678901234567890123456789012123456789012345678901
converting a statement in English into an algebra equation, and then 2
1 solving the equation for the desired variable. Suppose you are told that 2
2345678901234567890123456789012123456789012345678901
12345678901234567890123456789012123456789012345678901 2
12345678901234567890123456789012123456789012345678901 2
12345678901234567890123456789012123456789012345678901
two doughnuts and three coffees cost $2.70 at the local bakery. At the 2
12345678901234567890123456789012123456789012345678901 2
12345678901234567890123456789012123456789012345678901 22
2345678901234567890123456789012123456789012345678901
same bakery, 2 doughnuts and a coffee cost $1.50. You can translate the
12345678901234567890123456789012123456789012345678901
12345678901234567890123456789012123456789012345678901
first sentence into algebra like this: 2
12345678901234567890123456789012123456789012345678901 22
12345678901234567890123456789012123456789012345678901
12345678901234567890123456789012123456789012345678901
2 doughnuts 1 3 coffees 5 $2.70 2
1 2
12345678901234567890123456789012123456789012345678901
2d 1 3c 5 $2.70 2
12345678901234567890123456789012123456789012345678901 2
12345678901234567890123456789012123456789012345678901 2
12345678901234567890123456789012123456789012345678901 22
2345678901234567890123456789012123456789012345678901
With this one equation, you don’t have enough information to determine
12345678901234567890123456789012123456789012345678901
how much a coffee or a doughnut costs. But if you translate the second 2
12345678901234567890123456789012123456789012345678901
statement into an equation, then you will have two unknowns and two 2
12345678901234567890123456789012123456789012345678901
12345678901234567890123456789012123456789012345678901 2
equations. This will give you the same set-up as seen on page 70 earlier, 2
12345678901234567890123456789012123456789012345678901
12345678901234567890123456789012123456789012345678901 2
12345678901234567890123456789012123456789012345678901 2
12345678901234567890123456789012123456789012345678901 22
when you solve for two variables using two equations.
1
12345678901234567890123456789012123456789012345678901
The second equation is 2d 1 c 5 $1.50. If you subtract the second 2
12345678901234567890123456789012123456789012345678901 2
12345678901234567890123456789012123456789012345678901
equation from the first, the 2d terms cancels out and you are left with 2
12345678901234567890123456789012123456789012345678901 2
12345678901234567890123456789012123456789012345678901
2c 5 $1.20 → c 5 $0.60. Plugging in the price for coffee into the second 2
12345678901234567890123456789012123456789012345678901 2
12345678901234567890123456789012123456789012345678901
equation, you can solve for d, which is 0.45. Ta da! You used algebra to 2
12345678901234567890123456789012123456789012345678901 2
12345678901234567890123456789012123456789012345678901
decipher a word problem and then manipulated two equations to find the 2
12345678901234567890123456789012123456789012345678901 2
12345678901234567890123456789012123456789012345678901
values of two unknown variables. You’ll be an expert GRE mathematician 2
12345678901234567890123456789012123456789012345678901 2
12345678901234567890123456789012123456789012345678901
yet! 2
12345678901234567890123456789012123456789012345678901 2
12345678901234567890123456789012123456789012345678901
The key to translating English into algebra is to do it piece by piece. 2
12345678901234567890123456789012123456789012345678901 2
12345678901234567890123456789012123456789012345678901 2
12345678901234567890123456789012123456789012345678901
Suppose a question states, “The difference between Tom and John’s age is 2
12345678901234567890123456789012123456789012345678901 2
12345678901234567890123456789012123456789012345678901 2
13, and Tom is older.” The term difference means you are subtracting, and
2345678901234567890123456789012123456789012345678901
the word is means “equals,” so our translation would be T 2 J 5 13 (it is 2
12345678901234567890123456789012123456789012345678901
12345678901234567890123456789012123456789012345678901 2
12345678901234567890123456789012123456789012345678901 2
12345678901234567890123456789012123456789012345678901 22
T minus J because T is older).
1
12345678901234567890123456789012123456789012345678901
There are a number of key words and phrases often used when translating 2
12345678901234567890123456789012123456789012345678901 2
12345678901234567890123456789012123456789012345678901
English to algebra, as you can see from the chart on the next page. 2
12345678901234567890123456789012123456789012345678901 2
12345678901234567890123456789012123456789012345678901 2
12345678901234567890123456789012123456789012345678901 22
2345678901234567890123456789012123456789012345678901
1
12345678901234567890123456789012123456789012345678901 2
12345678901234567890123456789012123456789012345678901 2
12345678901234567890123456789012123456789012345678901 2
12345678901234567890123456789012123456789012345678901 2
12345678901234567890123456789012123456789012345678901 2
12345678901234567890123456789012123456789012345678901 2
12345678901234567890123456789012123456789012345678901 2 79
123456789012345678901234567890121234567890123456789012
Part III: Section Review

123456789012345678901234567890121234567890123456789012
12345678901234567890123456789012123456789012345678901 2
12345678901234567890123456789012123456789012345678901 2
12345678901234567890123456789012123456789012345678901 2
12345678901234567890123456789012123456789012345678901
Translation Key 2
12345678901234567890123456789012123456789012345678901 2
12345678901234567890123456789012123456789012345678901 2
12345678901234567890123456789012123456789012345678901
If You See This
2345678901234567890123456789012123456789012345678901
Do This 2
2
1
12345678901234567890123456789012123456789012345678901
sum, plus, added to, combined with, greater Add 2
12345678901234567890123456789012123456789012345678901 2
12345678901234567890123456789012123456789012345678901
than, increased by 2
12345678901234567890123456789012123456789012345678901
2345678901234567890123456789012123456789012345678901
2
2
1
12345678901234567890123456789012123456789012345678901
difference between, minus, subtracted from, Subtract 2
12345678901234567890123456789012123456789012345678901 2
12345678901234567890123456789012123456789012345678901
less than, reduced by 2
12345678901234567890123456789012123456789012345678901
2345678901234567890123456789012123456789012345678901
2
2
1 product, times, multiplied by, twice, doubled, Multiply
12345678901234567890123456789012123456789012345678901 2
12345678901234567890123456789012123456789012345678901
half 2
12345678901234567890123456789012123456789012345678901 2
12345678901234567890123456789012123456789012345678901
2345678901234567890123456789012123456789012345678901
quotient, divided by, per, ratio of blank to Divide 2
2
1
12345678901234567890123456789012123456789012345678901
blank, out of 2
12345678901234567890123456789012123456789012345678901 2
12345678901234567890123456789012123456789012345678901 2
12345678901234567890123456789012123456789012345678901
equals, will be, was, is the same as, costs, Equals 2
2345678901234567890123456789012123456789012345678901
12345678901234567890123456789012123456789012345678901 2
12345678901234567890123456789012123456789012345678901
adds up to, results in 2
12345678901234567890123456789012123456789012345678901 2
12345678901234567890123456789012123456789012345678901 2
12345678901234567890123456789012123456789012345678901 2
12345678901234567890123456789012123456789012345678901
A mixture of 14 ounces of water and oil is 50% water (by weight). 2
1 2
12345678901234567890123456789012123456789012345678901
How many ounces of oil must be added to the mixture to produce a 2
12345678901234567890123456789012123456789012345678901 2
12345678901234567890123456789012123456789012345678901
new mixture that is only 25% water? 2
12345678901234567890123456789012123456789012345678901 2
2345678901234567890123456789012123456789012345678901
12345678901234567890123456789012123456789012345678901 2
12345678901234567890123456789012123456789012345678901
A. 10 2
12345678901234567890123456789012123456789012345678901 2
12345678901234567890123456789012123456789012345678901 2
B. 14
12345678901234567890123456789012123456789012345678901 2
12345678901234567890123456789012123456789012345678901
C. 16 2
12345678901234567890123456789012123456789012345678901 2
12345678901234567890123456789012123456789012345678901 2
D. 20
2
1
12345678901234567890123456789012123456789012345678901
E. 28 2
12345678901234567890123456789012123456789012345678901 2
12345678901234567890123456789012123456789012345678901
This is a little tricky, so we have a little unpacking to do. In the first 2
12345678901234567890123456789012123456789012345678901 2
12345678901234567890123456789012123456789012345678901
mixture, 50% of the 14 ounces is water, which we could represent by 2
12345678901234567890123456789012123456789012345678901 2
12345678901234567890123456789012123456789012345678901
14~0.5! 2
12345678901234567890123456789012123456789012345678901
5 0.5. The numerator is the number of ounces of water, and the 2
12345678901234567890123456789012123456789012345678901
14 2
12345678901234567890123456789012123456789012345678901 2
12345678901234567890123456789012123456789012345678901 2
2345678901234567890123456789012123456789012345678901
denominator is the total number of ounces. That might seem obvious, but
12345678901234567890123456789012123456789012345678901 2
1 it is helpful for being able to write down the second mixture, which is 2
12345678901234567890123456789012123456789012345678901
going to have ounces of oil added to it (added to the denominator in this 2
12345678901234567890123456789012123456789012345678901 2
2345678901234567890123456789012123456789012345678901
case) until the percentage equals 0.25. This can be written down as 2
12345678901234567890123456789012123456789012345678901
12345678901234567890123456789012123456789012345678901 2
12345678901234567890123456789012123456789012345678901
~ ! 2
12345678901234567890123456789012123456789012345678901
14 0.5 2
1 14 1 x 5 0.25. All that remains is to solve for x, which after some 2
12345678901234567890123456789012123456789012345678901 2
2345678901234567890123456789012123456789012345678901
working will get you x 5 14, choice (B). The alternate way to do this 2
12345678901234567890123456789012123456789012345678901
12345678901234567890123456789012123456789012345678901
problem is to start with the answer choices and work backward. Always 2
2
12345678901234567890123456789012123456789012345678901
1 start with the middle value so that if it does not yield the right answer, you 2
12345678901234567890123456789012123456789012345678901 2
12345678901234567890123456789012123456789012345678901 2
12345678901234567890123456789012123456789012345678901
will know if you need to go higher or lower (and so eliminate the answer 2
12345678901234567890123456789012123456789012345678901 2
12345678901234567890123456789012123456789012345678901
choices in the opposite direction). If you add 16 ounces of oil, then you 2
2345678901234567890123456789012123456789012345678901
12345678901234567890123456789012123456789012345678901 2
1 will have a total of 30 ounces with 7 ounces of water. That is a little under 2
12345678901234567890123456789012123456789012345678901
25% water, so 16 is a little too much to add. 14 then would be a good 2
12345678901234567890123456789012123456789012345678901 2
12345678901234567890123456789012123456789012345678901
2345678901234567890123456789012123456789012345678901
2
2
1 guess if you are crunched for time.
12345678901234567890123456789012123456789012345678901 2
12345678901234567890123456789012123456789012345678901 2
12345678901234567890123456789012123456789012345678901 2
80 123456789012345678901234567890121234567890123456789012

www.petersons.com
Chapter 4: Quantitative Review

123456789012345678901234567890121234567890123456789012
12345678901234567890123456789012123456789012345678901 2
12345678901234567890123456789012123456789012345678901 2
12345678901234567890123456789012123456789012345678901 2
Geometry
12345678901234567890123456789012123456789012345678901 2
12345678901234567890123456789012123456789012345678901 2
12345678901234567890123456789012123456789012345678901 2
Fortunately, there will be no proofs and no mention of Euclid in the GRE,
12345678901234567890123456789012123456789012345678901
so you won’t have to revisit those painful experiences from high school. 2
12345678901234567890123456789012123456789012345678901 2
12345678901234567890123456789012123456789012345678901 2
12345678901234567890123456789012123456789012345678901
There is a limited set of basic geometry that you will be expected to 2
12345678901234567890123456789012123456789012345678901 2
12345678901234567890123456789012123456789012345678901
understand and apply to solve problems,
2345678901234567890123456789012123456789012345678901
so the first order of business is 2
2
1 the review of this material.
12345678901234567890123456789012123456789012345678901 2
12345678901234567890123456789012123456789012345678901 2
12345678901234567890123456789012123456789012345678901 2
12345678901234567890123456789012123456789012345678901
2345678901234567890123456789012123456789012345678901
2
2
1Lines and Angles
12345678901234567890123456789012123456789012345678901 2
12345678901234567890123456789012123456789012345678901 2
12345678901234567890123456789012123456789012345678901
It doesn’t get much more basic than the line. Most of the geometric figures 2
12345678901234567890123456789012123456789012345678901 2
12345678901234567890123456789012123456789012345678901
that you see on test day will be composed of lines or segments of lines, with 2
12345678901234567890123456789012123456789012345678901 2
12345678901234567890123456789012123456789012345678901
circles being the major exception. 2
12345678901234567890123456789012123456789012345678901 2
12345678901234567890123456789012123456789012345678901 2
12345678901234567890123456789012123456789012345678901 22
2345678901234567890123456789012123456789012345678901
12345678901234567890123456789012123456789012345678901 2
12345678901234567890123456789012123456789012345678901 2
12345678901234567890123456789012123456789012345678901
The test will use the notation AB sometimes to refer to the line segment 2
12345678901234567890123456789012123456789012345678901
12345678901234567890123456789012123456789012345678901 2
1 AB. This notation will also refer to the length of the line segment AB. You 2
12345678901234567890123456789012123456789012345678901 2
12345678901234567890123456789012123456789012345678901
will be able to tell from the context which is intended. 2
12345678901234567890123456789012123456789012345678901 2
2345678901234567890123456789012123456789012345678901
12345678901234567890123456789012123456789012345678901 2
12345678901234567890123456789012123456789012345678901
The angle is one step up in complexity. It will include at least two lines. 2
12345678901234567890123456789012123456789012345678901 22
12345678901234567890123456789012123456789012345678901 2
12345678901234567890123456789012123456789012345678901 2
12345678901234567890123456789012123456789012345678901 2
12345678901234567890123456789012123456789012345678901 2
12345678901234567890123456789012123456789012345678901 2
12345678901234567890123456789012123456789012345678901 2
1
12345678901234567890123456789012123456789012345678901 2
12345678901234567890123456789012123456789012345678901 2
12345678901234567890123456789012123456789012345678901 2
12345678901234567890123456789012123456789012345678901 2
12345678901234567890123456789012123456789012345678901 2
12345678901234567890123456789012123456789012345678901
When two lines intersect as in the above figure, the angles across from each 2
12345678901234567890123456789012123456789012345678901 2
12345678901234567890123456789012123456789012345678901
other are called vertical angles, and they are equal (so a° 5 c° and b° 5 d°). 2
12345678901234567890123456789012123456789012345678901 2
12345678901234567890123456789012123456789012345678901
The sum of the four angles is 360 degrees, while the sum of any of the 2
2345678901234567890123456789012123456789012345678901
angles adjacent to each other is 180 degrees ( a° 1 b° or d° 1 c°, for 2
12345678901234567890123456789012123456789012345678901
12345678901234567890123456789012123456789012345678901 2
1 example). The adjacent angles sum to 180 degrees because two adjacent 2
12345678901234567890123456789012123456789012345678901 2
12345678901234567890123456789012123456789012345678901
angles in our example define the angle measure of a straight line, and the 2
12345678901234567890123456789012123456789012345678901 2
12345678901234567890123456789012123456789012345678901 2
12345678901234567890123456789012123456789012345678901
measure of a straight line is 180 degrees. If a° 5 90, then you could deduce 2
12345678901234567890123456789012123456789012345678901 2
12345678901234567890123456789012123456789012345678901 2
that all four of the angles are 90 degrees because if a° 5 90 then c° 5 90,
2345678901234567890123456789012123456789012345678901
and if c° 5 90 then d° 5 90 because d and c must sum to 180 degrees, and 2
12345678901234567890123456789012123456789012345678901
12345678901234567890123456789012123456789012345678901 2
12345678901234567890123456789012123456789012345678901 2
12345678901234567890123456789012123456789012345678901 22
so on around the diagram. Ninety-degree angles also go by the special
1 name of right angles. They have a special symbol as well, the .
12345678901234567890123456789012123456789012345678901 2
12345678901234567890123456789012123456789012345678901 2
12345678901234567890123456789012123456789012345678901 2
12345678901234567890123456789012123456789012345678901 2
12345678901234567890123456789012123456789012345678901 2
Tip

12345678901234567890123456789012123456789012345678901 22
2345678901234567890123456789012123456789012345678901
Angles that add up to 180 degrees are called supplementary angles.
1 Angles that add up to 90 degrees are called complementary angles.
12345678901234567890123456789012123456789012345678901 2
12345678901234567890123456789012123456789012345678901 2
12345678901234567890123456789012123456789012345678901 2
12345678901234567890123456789012123456789012345678901 2
12345678901234567890123456789012123456789012345678901 2
12345678901234567890123456789012123456789012345678901 2
12345678901234567890123456789012123456789012345678901 2 81
123456789012345678901234567890121234567890123456789012
Part III: Section Review

123456789012345678901234567890121234567890123456789012
12345678901234567890123456789012123456789012345678901 2
12345678901234567890123456789012123456789012345678901 2
12345678901234567890123456789012123456789012345678901 2
12345678901234567890123456789012123456789012345678901 2
12345678901234567890123456789012123456789012345678901 2
12345678901234567890123456789012123456789012345678901 2
12345678901234567890123456789012123456789012345678901 2
12345678901234567890123456789012123456789012345678901 2
12345678901234567890123456789012123456789012345678901 2
12345678901234567890123456789012123456789012345678901 2
12345678901234567890123456789012123456789012345678901
If two lines intersect and the angles formed by their intersection are right 2
12345678901234567890123456789012123456789012345678901 2
12345678901234567890123456789012123456789012345678901
angles, the lines are said to be perpendicular. In our example, if the angles 2
12345678901234567890123456789012123456789012345678901 2
12345678901234567890123456789012123456789012345678901
are right angles then l1 is perpendicular to l2, which is denoted as l1 ⊥ l2. 2
12345678901234567890123456789012123456789012345678901 2
12345678901234567890123456789012123456789012345678901
2345678901234567890123456789012123456789012345678901
2
2
1 Two lines can have one other kind of special relationship, which is known
12345678901234567890123456789012123456789012345678901
as parallel and denoted by two close-together straight lines such as l1 i l2. 2
12345678901234567890123456789012123456789012345678901 2
12345678901234567890123456789012123456789012345678901 2
12345678901234567890123456789012123456789012345678901
Like train tracks, parallel lines never intersect. Two parallel lines are not 2
1 that interesting, but if you cross two parallel lines with a third line, 2
2345678901234567890123456789012123456789012345678901
12345678901234567890123456789012123456789012345678901 2
12345678901234567890123456789012123456789012345678901 2
12345678901234567890123456789012123456789012345678901
hoo boy! 2
12345678901234567890123456789012123456789012345678901 2
12345678901234567890123456789012123456789012345678901 2
2345678901234567890123456789012123456789012345678901
12345678901234567890123456789012123456789012345678901 2
12345678901234567890123456789012123456789012345678901 2
12345678901234567890123456789012123456789012345678901 2
12345678901234567890123456789012123456789012345678901 2
12345678901234567890123456789012123456789012345678901 2 2
1
12345678901234567890123456789012123456789012345678901 2
12345678901234567890123456789012123456789012345678901 2
12345678901234567890123456789012123456789012345678901 2
12345678901234567890123456789012123456789012345678901 2
2345678901234567890123456789012123456789012345678901
12345678901234567890123456789012123456789012345678901 2
12345678901234567890123456789012123456789012345678901 2
Since they are vertical angles, 1 equals 4, 2 equals 3, 5 equals 8, and 6 2
12345678901234567890123456789012123456789012345678901
12345678901234567890123456789012123456789012345678901 2
equals 7. Also, angles 2 and 4, 1 and 3, 5 and 6, 5 and 7, and a host of 2
12345678901234567890123456789012123456789012345678901
12345678901234567890123456789012123456789012345678901 2
12345678901234567890123456789012123456789012345678901
others are all supplementary angles. That’s just the start. Alternate interior 2
12345678901234567890123456789012123456789012345678901 2
1 angles are also equal. To unpack that term, consider that interior refers to 2
12345678901234567890123456789012123456789012345678901 2
12345678901234567890123456789012123456789012345678901
an angle on the inside (3, 4, 5, 6) of the parallel lines. Alternate means the 2
12345678901234567890123456789012123456789012345678901 2
12345678901234567890123456789012123456789012345678901
angles that are on different sides of the non-parallel line (3 and 6, 4 and 5). 2
12345678901234567890123456789012123456789012345678901
2345678901234567890123456789012123456789012345678901
2
So 3 5 6 and 4 5 5. Notice this means we can deduce a lot more about the 2
12345678901234567890123456789012123456789012345678901
12345678901234567890123456789012123456789012345678901 2
1 relationships of the different angles (1 5 8, 2 5 7). 2
12345678901234567890123456789012123456789012345678901 2
12345678901234567890123456789012123456789012345678901 2
12345678901234567890123456789012123456789012345678901 2
2345678901234567890123456789012123456789012345678901
Column A Column B
12345678901234567890123456789012123456789012345678901 2 2
1
12345678901234567890123456789012123456789012345678901 2
12345678901234567890123456789012123456789012345678901 2
12345678901234567890123456789012123456789012345678901 2
12345678901234567890123456789012123456789012345678901 2
12345678901234567890123456789012123456789012345678901 2
12345678901234567890123456789012123456789012345678901 2
12345678901234567890123456789012123456789012345678901 2
12345678901234567890123456789012123456789012345678901 2
2345678901234567890123456789012123456789012345678901
12345678901234567890123456789012123456789012345678901 2
12345678901234567890123456789012123456789012345678901 2 2
12345678901234567890123456789012123456789012345678901
l1 i l2
2
1
2345678901234567890123456789012123456789012345678901
12345678901234567890123456789012123456789012345678901 2
12345678901234567890123456789012123456789012345678901
a b 2
12345678901234567890123456789012123456789012345678901 2
12345678901234567890123456789012123456789012345678901 2
1 This looks a little different than the figure we were just working with, but 2
12345678901234567890123456789012123456789012345678901 2
12345678901234567890123456789012123456789012345678901
if you rotate it ninety degrees, you can see that it is basically the same. 2
12345678901234567890123456789012123456789012345678901 2
12345678901234567890123456789012123456789012345678901
b 5 90 because it and the right angle must sum to 180 (supplementary 2
12345678901234567890123456789012123456789012345678901 2
12345678901234567890123456789012123456789012345678901
angles). At this point there are two ways to get the answer. You could 2
12345678901234567890123456789012123456789012345678901 2
12345678901234567890123456789012123456789012345678901
reason, “Two lines that are parallel will intersect a third line at the same 2
12345678901234567890123456789012123456789012345678901 2
82 123456789012345678901234567890121234567890123456789012

www.petersons.com
Chapter 4: Quantitative Review

123456789012345678901234567890121234567890123456789012
12345678901234567890123456789012123456789012345678901 2
12345678901234567890123456789012123456789012345678901 2
12345678901234567890123456789012123456789012345678901
angle, which would mean that a is a right angle.” If you didn’t see that, 2
12345678901234567890123456789012123456789012345678901 2
12345678901234567890123456789012123456789012345678901
don’t worry, since there is another way to get the answer. b is equal to the 2
12345678901234567890123456789012123456789012345678901 2
12345678901234567890123456789012123456789012345678901
angle across from it (vertical angles), and so b is also equal to the angle 2
1 across from a (alternate interior angles). So a and b are equal since they are 2
2345678901234567890123456789012123456789012345678901
12345678901234567890123456789012123456789012345678901 2
12345678901234567890123456789012123456789012345678901 2
12345678901234567890123456789012123456789012345678901
equal to the same angle. The correct answer is (C). 2
12345678901234567890123456789012123456789012345678901
2345678901234567890123456789012123456789012345678901
2
2
1
12345678901234567890123456789012123456789012345678901 2
12345678901234567890123456789012123456789012345678901 2
12345678901234567890123456789012123456789012345678901
Triangles 2
12345678901234567890123456789012123456789012345678901 2
1 Triangles are tri-angled geometric figures that have three angles and three 2
2345678901234567890123456789012123456789012345678901
12345678901234567890123456789012123456789012345678901 2
12345678901234567890123456789012123456789012345678901 2
12345678901234567890123456789012123456789012345678901
sides. There are many different kinds of triangles that all have different 2
12345678901234567890123456789012123456789012345678901
names, but we don’t need to concern ourselves
2345678901234567890123456789012123456789012345678901 with those here. The GRE 2
2
1
12345678901234567890123456789012123456789012345678901
will test some general principles about triangles and will also focus on a 2
12345678901234567890123456789012123456789012345678901 2
12345678901234567890123456789012123456789012345678901
few special kinds of triangles. The wisest course of action is to review these 2
12345678901234567890123456789012123456789012345678901 2
12345678901234567890123456789012123456789012345678901
general principles and then get really familiar with the triangles that are 2
12345678901234567890123456789012123456789012345678901 2
12345678901234567890123456789012123456789012345678901
the GRE’s favorites. 2
12345678901234567890123456789012123456789012345678901 2
12345678901234567890123456789012123456789012345678901 2
12345678901234567890123456789012123456789012345678901 22
2345678901234567890123456789012123456789012345678901
A rule that holds true for all triangles is that the three angles sum to 180
1 degrees. This means that if you know two angles of a triangle, then you can
12345678901234567890123456789012123456789012345678901 2
12345678901234567890123456789012123456789012345678901 2
12345678901234567890123456789012123456789012345678901
find the third. So if a triangle has two angles that are equal and third angle 2
2345678901234567890123456789012123456789012345678901
12345678901234567890123456789012123456789012345678901 2
12345678901234567890123456789012123456789012345678901
is 50 degrees, what is the measure of the other two angles? 2
12345678901234567890123456789012123456789012345678901 22
12345678901234567890123456789012123456789012345678901
To answer this, let’s go back in this chapter and employ some algebra. 2
12345678901234567890123456789012123456789012345678901
12345678901234567890123456789012123456789012345678901 2
12345678901234567890123456789012123456789012345678901
Calling the two equal angles x, you get 2
12345678901234567890123456789012123456789012345678901 22
12345678901234567890123456789012123456789012345678901
2x 1 50 5 180 → 2
1
12345678901234567890123456789012123456789012345678901 2
12345678901234567890123456789012123456789012345678901
2x 1 50 2 50 5 180 2 50 → 2
12345678901234567890123456789012123456789012345678901 2
12345678901234567890123456789012123456789012345678901
2x 5 130 → 2
12345678901234567890123456789012123456789012345678901
2345678901234567890123456789012123456789012345678901
2
2
12345678901234567890123456789012123456789012345678901
2x 4 2 5 130 4 2 →
12345678901234567890123456789012123456789012345678901 2
1 2
12345678901234567890123456789012123456789012345678901
x 5 65 2
12345678901234567890123456789012123456789012345678901 2
2345678901234567890123456789012123456789012345678901
Another general rule for triangles—referenced earlier in the book—is that 2
12345678901234567890123456789012123456789012345678901
12345678901234567890123456789012123456789012345678901 2
1 any two sides of a triangle must be longer than the third side. Otherwise, 2
12345678901234567890123456789012123456789012345678901 2
12345678901234567890123456789012123456789012345678901
the sides would not meet. You might need this information on a QC where 2
12345678901234567890123456789012123456789012345678901 2
12345678901234567890123456789012123456789012345678901
one column is the length of the longest side of the triangle and the other 2
12345678901234567890123456789012123456789012345678901 2
12345678901234567890123456789012123456789012345678901
column is the maximum length of the other shorter sides combined. In that 2
12345678901234567890123456789012123456789012345678901 2
12345678901234567890123456789012123456789012345678901
case, the other two sides must be longer for the figure to actually be a 2
12345678901234567890123456789012123456789012345678901 2
12345678901234567890123456789012123456789012345678901
triangle. 2
12345678901234567890123456789012123456789012345678901 2
12345678901234567890123456789012123456789012345678901 2
12345678901234567890123456789012123456789012345678901 22
2345678901234567890123456789012123456789012345678901
Another principle that holds for all triangles is the formula for the area of
12345678901234567890123456789012123456789012345678901 2
12345678901234567890123456789012123456789012345678901
1
a triangle, A 5 ~base!~height!. Using this formula would be simple if you 2
12345678901234567890123456789012123456789012345678901 2
1 2
12345678901234567890123456789012123456789012345678901
knew what the base and height of the triangle are, but most problems 2
12345678901234567890123456789012123456789012345678901 2
12345678901234567890123456789012123456789012345678901
aren’t going to tell you that directly. To find them, you must know that the 2
12345678901234567890123456789012123456789012345678901 2
12345678901234567890123456789012123456789012345678901
height is the measure of any line segment that begins at one of the vertices 2
12345678901234567890123456789012123456789012345678901 2
12345678901234567890123456789012123456789012345678901
and ends at the opposite side, forming a right angle with that side, and that 2
12345678901234567890123456789012123456789012345678901 2
12345678901234567890123456789012123456789012345678901 2 83
123456789012345678901234567890121234567890123456789012
Part III: Section Review

123456789012345678901234567890121234567890123456789012
12345678901234567890123456789012123456789012345678901 2
12345678901234567890123456789012123456789012345678901 2
12345678901234567890123456789012123456789012345678901
the base is the length of the line that the height forms the right angle with. 2
12345678901234567890123456789012123456789012345678901 2
12345678901234567890123456789012123456789012345678901
Look at this figure if that explanation didn’t clear everything up for you. 2
12345678901234567890123456789012123456789012345678901 2
12345678901234567890123456789012123456789012345678901 2
12345678901234567890123456789012123456789012345678901 2
12345678901234567890123456789012123456789012345678901 2
12345678901234567890123456789012123456789012345678901 2
12345678901234567890123456789012123456789012345678901 2
12345678901234567890123456789012123456789012345678901
2345678901234567890123456789012123456789012345678901
2
2
1
12345678901234567890123456789012123456789012345678901 2
12345678901234567890123456789012123456789012345678901 2
12345678901234567890123456789012123456789012345678901 2
12345678901234567890123456789012123456789012345678901
2345678901234567890123456789012123456789012345678901
2
2
1
12345678901234567890123456789012123456789012345678901 2
12345678901234567890123456789012123456789012345678901
One thing to keep in mind is that the base and the height of a triangle are 2
12345678901234567890123456789012123456789012345678901
not fixed values. In the figure above, we could have drawn a line from ∠C 2
12345678901234567890123456789012123456789012345678901 2
1 and perpendicular with AB. In that case, the height would be the 2
2345678901234567890123456789012123456789012345678901
12345678901234567890123456789012123456789012345678901 2
12345678901234567890123456789012123456789012345678901
perpendicular line going from ∠C to AB, and the base would be AB. To 2
12345678901234567890123456789012123456789012345678901 2
12345678901234567890123456789012123456789012345678901
decide what to make the height and what to make the base, follow this rule 2
12345678901234567890123456789012123456789012345678901 2
2345678901234567890123456789012123456789012345678901
of thumb: find the base and height wherever it is easiest. Generally it is 2
12345678901234567890123456789012123456789012345678901
12345678901234567890123456789012123456789012345678901 2
harder to find the height—unless you can draw the height so that it is one 2
12345678901234567890123456789012123456789012345678901
12345678901234567890123456789012123456789012345678901
of the triangle sides—so approach an area problem looking for the easiest 2
2
12345678901234567890123456789012123456789012345678901 2
1 way to find the height.
12345678901234567890123456789012123456789012345678901 2
12345678901234567890123456789012123456789012345678901 2
12345678901234567890123456789012123456789012345678901 2
12345678901234567890123456789012123456789012345678901 2
2345678901234567890123456789012123456789012345678901
12345678901234567890123456789012123456789012345678901 2
12345678901234567890123456789012123456789012345678901 2
12345678901234567890123456789012123456789012345678901 2
12345678901234567890123456789012123456789012345678901 2
12345678901234567890123456789012123456789012345678901 2
12345678901234567890123456789012123456789012345678901 2
12345678901234567890123456789012123456789012345678901 2
Sometimes you have to get really creative in finding the height of a 2
12345678901234567890123456789012123456789012345678901
1 triangle. That is what the figure above shows. The height doesn’t have to 2
12345678901234567890123456789012123456789012345678901 2
12345678901234567890123456789012123456789012345678901 2
12345678901234567890123456789012123456789012345678901
be a line inside the triangle. The height is the length of the line from one 2
12345678901234567890123456789012123456789012345678901 2
12345678901234567890123456789012123456789012345678901
angle to its opposite side if that line is perpendicular to the opposite side. 2
12345678901234567890123456789012123456789012345678901 2
2345678901234567890123456789012123456789012345678901
12345678901234567890123456789012123456789012345678901 2 2
1
12345678901234567890123456789012123456789012345678901 2
12345678901234567890123456789012123456789012345678901 2
12345678901234567890123456789012123456789012345678901 2
2345678901234567890123456789012123456789012345678901
12345678901234567890123456789012123456789012345678901 2 2
1
12345678901234567890123456789012123456789012345678901 2
12345678901234567890123456789012123456789012345678901 2
12345678901234567890123456789012123456789012345678901 2
12345678901234567890123456789012123456789012345678901 2
12345678901234567890123456789012123456789012345678901
What is the area of the above figure? 2
12345678901234567890123456789012123456789012345678901 2
12345678901234567890123456789012123456789012345678901 2
2345678901234567890123456789012123456789012345678901
12345678901234567890123456789012123456789012345678901
A. 3 2
12345678901234567890123456789012123456789012345678901 2
12345678901234567890123456789012123456789012345678901
B. 6 2
12345678901234567890123456789012123456789012345678901 2
1 C. 8 2
2345678901234567890123456789012123456789012345678901
12345678901234567890123456789012123456789012345678901 2
12345678901234567890123456789012123456789012345678901
D. 9 2
12345678901234567890123456789012123456789012345678901 2
12345678901234567890123456789012123456789012345678901
E. 12 2
1 2
12345678901234567890123456789012123456789012345678901 2
2345678901234567890123456789012123456789012345678901
2
1
12345678901234567890123456789012123456789012345678901 2
12345678901234567890123456789012123456789012345678901 2
12345678901234567890123456789012123456789012345678901 2
12345678901234567890123456789012123456789012345678901 2
12345678901234567890123456789012123456789012345678901 2
12345678901234567890123456789012123456789012345678901 2
12345678901234567890123456789012123456789012345678901 2
84 123456789012345678901234567890121234567890123456789012

www.petersons.com
Chapter 4: Quantitative Review

123456789012345678901234567890121234567890123456789012
12345678901234567890123456789012123456789012345678901 2
12345678901234567890123456789012123456789012345678901 2
12345678901234567890123456789012123456789012345678901
With the triangle-ometry that we have covered thus far, there is only one 2
12345678901234567890123456789012123456789012345678901 2
12345678901234567890123456789012123456789012345678901
way to pick the height and base so that you can solve this problem. 2
12345678901234567890123456789012123456789012345678901 2
12345678901234567890123456789012123456789012345678901 2
12345678901234567890123456789012123456789012345678901 2
12345678901234567890123456789012123456789012345678901 2
12345678901234567890123456789012123456789012345678901 2
12345678901234567890123456789012123456789012345678901 2
12345678901234567890123456789012123456789012345678901
2345678901234567890123456789012123456789012345678901
2
2
1
12345678901234567890123456789012123456789012345678901 2
12345678901234567890123456789012123456789012345678901 2
12345678901234567890123456789012123456789012345678901 2
12345678901234567890123456789012123456789012345678901 2
1 You can pick one of the sides to be the height since it forms a right angle 2
2345678901234567890123456789012123456789012345678901
12345678901234567890123456789012123456789012345678901 2
12345678901234567890123456789012123456789012345678901
1 2
12345678901234567890123456789012123456789012345678901
with the side it intersects. So A 5 ~3!~4! 5 6. Therefore, the correct 2
12345678901234567890123456789012123456789012345678901
2 2
12345678901234567890123456789012123456789012345678901 2
12345678901234567890123456789012123456789012345678901
answer is (B). 2
12345678901234567890123456789012123456789012345678901 2
12345678901234567890123456789012123456789012345678901 2
12345678901234567890123456789012123456789012345678901 2
12345678901234567890123456789012123456789012345678901 22
2345678901234567890123456789012123456789012345678901
12345678901234567890123456789012123456789012345678901
Special Triangles
12345678901234567890123456789012123456789012345678901 22
12345678901234567890123456789012123456789012345678901
The important thing about special triangles is that the GRE likes to use
12345678901234567890123456789012123456789012345678901 2
12345678901234567890123456789012123456789012345678901 2
1 them. When one pops up on the test, you need to be able to identify it and 2
12345678901234567890123456789012123456789012345678901
then use its special properties to solve the problem. 2
12345678901234567890123456789012123456789012345678901 2
12345678901234567890123456789012123456789012345678901 2
12345678901234567890123456789012123456789012345678901 22
2345678901234567890123456789012123456789012345678901
The first kind of special triangle to discuss is usually pretty easy to spot. A
12345678901234567890123456789012123456789012345678901
right triangle is a triangle that has one 90-degree angle in it. Visually, this 2
12345678901234567890123456789012123456789012345678901
angle might be denoted by one of those box symbols in it. Recalling that all 2
12345678901234567890123456789012123456789012345678901
12345678901234567890123456789012123456789012345678901 2
three angles in a triangle always equal 180, you know that in a right 2
12345678901234567890123456789012123456789012345678901
12345678901234567890123456789012123456789012345678901 2
12345678901234567890123456789012123456789012345678901 2
12345678901234567890123456789012123456789012345678901 22
triangle, the other two angles always sum to 90.
1
12345678901234567890123456789012123456789012345678901
The Pythagorean theorem is a special equation that only applies to right 2
12345678901234567890123456789012123456789012345678901 2
12345678901234567890123456789012123456789012345678901 2 2 2 2
12345678901234567890123456789012123456789012345678901 2
triangles. Call it the “Big P,” and its equation is a 1 b 5 c , where c is the
12345678901234567890123456789012123456789012345678901
length of the side opposite the right angle (called the hypotenuse by the 2
12345678901234567890123456789012123456789012345678901 2
12345678901234567890123456789012123456789012345678901
math establishment) and a and b are the lengths of the other two sides. 2
12345678901234567890123456789012123456789012345678901 2
12345678901234567890123456789012123456789012345678901
With this equation, if you know two sides of a right triangle, you can 2
12345678901234567890123456789012123456789012345678901 2
2345678901234567890123456789012123456789012345678901
always find the third. That is a basic move the test will expect you to be 2
12345678901234567890123456789012123456789012345678901
12345678901234567890123456789012123456789012345678901 2
1 able to make. A problem won’t be that easy, but that will be one necessary 2
12345678901234567890123456789012123456789012345678901 2
12345678901234567890123456789012123456789012345678901
step for solving a problem. 2
12345678901234567890123456789012123456789012345678901 22
2345678901234567890123456789012123456789012345678901
12345678901234567890123456789012123456789012345678901 2
12345678901234567890123456789012123456789012345678901
Column A Column B
12345678901234567890123456789012123456789012345678901 22
1
12345678901234567890123456789012123456789012345678901 2
12345678901234567890123456789012123456789012345678901 2
12345678901234567890123456789012123456789012345678901 2
12345678901234567890123456789012123456789012345678901 2
12345678901234567890123456789012123456789012345678901 2
12345678901234567890123456789012123456789012345678901 22
2345678901234567890123456789012123456789012345678901
12345678901234567890123456789012123456789012345678901 2
12345678901234567890123456789012123456789012345678901 2
12345678901234567890123456789012123456789012345678901
Area of DABC Area of DDEF
2
1
12345678901234567890123456789012123456789012345678901 22
2345678901234567890123456789012123456789012345678901
1 To answer this question, you need to be able to determine the base and the
12345678901234567890123456789012123456789012345678901 2
12345678901234567890123456789012123456789012345678901
height of each of the triangles. Can you do that? You can with the right 2
12345678901234567890123456789012123456789012345678901 2
12345678901234567890123456789012123456789012345678901
triangle, but you can’t with DDEF. Choice (D) is the correct answer. 2
12345678901234567890123456789012123456789012345678901 2
12345678901234567890123456789012123456789012345678901 2
12345678901234567890123456789012123456789012345678901 2 85
123456789012345678901234567890121234567890123456789012
Part III: Section Review

123456789012345678901234567890121234567890123456789012
12345678901234567890123456789012123456789012345678901 2
12345678901234567890123456789012123456789012345678901 2
12345678901234567890123456789012123456789012345678901
A second kind of special triangle is the isosceles triangle. It has a funny 2
12345678901234567890123456789012123456789012345678901 2
12345678901234567890123456789012123456789012345678901
name but a simple meaning: Two of the sides of the triangle are equal. A 2
12345678901234567890123456789012123456789012345678901 2
12345678901234567890123456789012123456789012345678901
problem will either tell you that a triangle is isosceles and expect you to 2
2345678901234567890123456789012123456789012345678901 2
1 know what that means, or you might see a figure like this:
12345678901234567890123456789012123456789012345678901 2
12345678901234567890123456789012123456789012345678901 2
12345678901234567890123456789012123456789012345678901 2
12345678901234567890123456789012123456789012345678901
2345678901234567890123456789012123456789012345678901
2
2
1
12345678901234567890123456789012123456789012345678901 2
12345678901234567890123456789012123456789012345678901 2
12345678901234567890123456789012123456789012345678901 2
12345678901234567890123456789012123456789012345678901
2345678901234567890123456789012123456789012345678901
2
2
1
12345678901234567890123456789012123456789012345678901 2
12345678901234567890123456789012123456789012345678901 2
12345678901234567890123456789012123456789012345678901 2
12345678901234567890123456789012123456789012345678901
The lines through two of the sides of the triangle
2345678901234567890123456789012123456789012345678901
mean that those two sides 2
2
1 are equal.
12345678901234567890123456789012123456789012345678901 2
12345678901234567890123456789012123456789012345678901 2
12345678901234567890123456789012123456789012345678901 2
12345678901234567890123456789012123456789012345678901
Now let’s think about triangles for a second. If one angle of a triangle is 2
2345678901234567890123456789012123456789012345678901
large—say 120 degrees—will the side opposite that angle be large or small 2
12345678901234567890123456789012123456789012345678901
12345678901234567890123456789012123456789012345678901 2
12345678901234567890123456789012123456789012345678901 2
12345678901234567890123456789012123456789012345678901 2
in comparison to the other angles? Draw out a triangle with this angle. The
side opposite the big angle will be big in comparison to the other sides. The 2
12345678901234567890123456789012123456789012345678901
12345678901234567890123456789012123456789012345678901 2
1 converse is also true. The side opposite the small angle will be small in 2
12345678901234567890123456789012123456789012345678901 2
12345678901234567890123456789012123456789012345678901
comparison to the other sides. 2
12345678901234567890123456789012123456789012345678901 2
2345678901234567890123456789012123456789012345678901
12345678901234567890123456789012123456789012345678901
With these illustrations, you can see that the size of an angle in a triangle is 2
2
12345678901234567890123456789012123456789012345678901
12345678901234567890123456789012123456789012345678901
related to the length of the side opposite that angle. Again, the converse is 2
2
12345678901234567890123456789012123456789012345678901
12345678901234567890123456789012123456789012345678901
true; the length of a side of a triangle is related to the size of the angle 2
2
12345678901234567890123456789012123456789012345678901
opposite it. Now think about an isosceles triangle, where two sides are the 2
12345678901234567890123456789012123456789012345678901
12345678901234567890123456789012123456789012345678901 2
12345678901234567890123456789012123456789012345678901 2
1 same length. Since the sides are equal, the angles opposite those sides are 2
2345678901234567890123456789012123456789012345678901
12345678901234567890123456789012123456789012345678901 2
12345678901234567890123456789012123456789012345678901
also equal. In the isosceles triangle above, ∠A 5 ∠B. 2
1 2
12345678901234567890123456789012123456789012345678901 2
12345678901234567890123456789012123456789012345678901
A wrinkle on the isosceles triangle is
2345678901234567890123456789012123456789012345678901
the right isosceles triangle. It’s 2
nothing more than a right triangle with two sides that are equal. You know 2
12345678901234567890123456789012123456789012345678901
12345678901234567890123456789012123456789012345678901 2
1 one angle equals 90 degrees, and that the other two sum to 90 degrees. 2
12345678901234567890123456789012123456789012345678901 2
12345678901234567890123456789012123456789012345678901
Since you also know the other two angles are equal, it can be stated 2
12345678901234567890123456789012123456789012345678901 2
2345678901234567890123456789012123456789012345678901
12345678901234567890123456789012123456789012345678901 2
1 algebraically that 2
12345678901234567890123456789012123456789012345678901 2
12345678901234567890123456789012123456789012345678901
x 1 x 5 90 2
12345678901234567890123456789012123456789012345678901 2
12345678901234567890123456789012123456789012345678901 2
12345678901234567890123456789012123456789012345678901
2x 5 90 Here, x is the angle for the two equal sides. 2
12345678901234567890123456789012123456789012345678901 2
12345678901234567890123456789012123456789012345678901 2
x 5 45
12345678901234567890123456789012123456789012345678901 2
2345678901234567890123456789012123456789012345678901
The other two angles are 45 degrees. If the right triangle is a special 2
12345678901234567890123456789012123456789012345678901
12345678901234567890123456789012123456789012345678901 2
12345678901234567890123456789012123456789012345678901 2
1 triangle, then the 45-45-90 triangle is a “super special” triangle because it 2
2345678901234567890123456789012123456789012345678901
12345678901234567890123456789012123456789012345678901 2
12345678901234567890123456789012123456789012345678901
is a special kind of right triangle. 2
12345678901234567890123456789012123456789012345678901 2
12345678901234567890123456789012123456789012345678901 2 2
1
12345678901234567890123456789012123456789012345678901 2
12345678901234567890123456789012123456789012345678901 2
12345678901234567890123456789012123456789012345678901 2
12345678901234567890123456789012123456789012345678901 2
12345678901234567890123456789012123456789012345678901 2
12345678901234567890123456789012123456789012345678901 2
12345678901234567890123456789012123456789012345678901 2
12345678901234567890123456789012123456789012345678901 2
12345678901234567890123456789012123456789012345678901 2
86 123456789012345678901234567890121234567890123456789012

www.petersons.com
Chapter 4: Quantitative Review

123456789012345678901234567890121234567890123456789012
12345678901234567890123456789012123456789012345678901 2
12345678901234567890123456789012123456789012345678901 2
12345678901234567890123456789012123456789012345678901
As you can see, the relationships between the sides and angles of a 2
12345678901234567890123456789012123456789012345678901 2
12345678901234567890123456789012123456789012345678901
45-45-90 triangle are all nailed down. The test-makers like this feature of 2
12345678901234567890123456789012123456789012345678901 2
12345678901234567890123456789012123456789012345678901
the triangle because they figure that if they give you enough information to 2
1 identify a triangle as a 45-45-90 triangle, you should be able to determine 2
2345678901234567890123456789012123456789012345678901
12345678901234567890123456789012123456789012345678901 2
12345678901234567890123456789012123456789012345678901 2
12345678901234567890123456789012123456789012345678901
most everything about that triangle. 2
12345678901234567890123456789012123456789012345678901
2345678901234567890123456789012123456789012345678901
2
2
1
12345678901234567890123456789012123456789012345678901 2
12345678901234567890123456789012123456789012345678901 2
Note
12345678901234567890123456789012123456789012345678901 2
12345678901234567890123456789012123456789012345678901
If you want to know how the side ratios are determined, just use the
2345678901234567890123456789012123456789012345678901
2
2
1 Pythagorean theorem on the 45-45-90.
12345678901234567890123456789012123456789012345678901 2
12345678901234567890123456789012123456789012345678901 2
12345678901234567890123456789012123456789012345678901 2
12345678901234567890123456789012123456789012345678901
2345678901234567890123456789012123456789012345678901
2
2
1
12345678901234567890123456789012123456789012345678901 2
12345678901234567890123456789012123456789012345678901 2
12345678901234567890123456789012123456789012345678901 2
12345678901234567890123456789012123456789012345678901 2
12345678901234567890123456789012123456789012345678901 22
2345678901234567890123456789012123456789012345678901
12345678901234567890123456789012123456789012345678901 2
12345678901234567890123456789012123456789012345678901 2
12345678901234567890123456789012123456789012345678901 2
12345678901234567890123456789012123456789012345678901 2
12345678901234567890123456789012123456789012345678901 2
1 If a 5 c, then x 5
12345678901234567890123456789012123456789012345678901 2
12345678901234567890123456789012123456789012345678901 2
12345678901234567890123456789012123456789012345678901
A. 2 2
2345678901234567890123456789012123456789012345678901
12345678901234567890123456789012123456789012345678901 2
12345678901234567890123456789012123456789012345678901
B. =3 2
12345678901234567890123456789012123456789012345678901 22
12345678901234567890123456789012123456789012345678901
C. =5
12345678901234567890123456789012123456789012345678901 2
12345678901234567890123456789012123456789012345678901 2
12345678901234567890123456789012123456789012345678901
D. =6 2
12345678901234567890123456789012123456789012345678901 2
12345678901234567890123456789012123456789012345678901
E. 4 2
1 2
2345678901234567890123456789012123456789012345678901
This is a busy little diagram, but you’ll get some of those. If your test radar 2
12345678901234567890123456789012123456789012345678901
12345678901234567890123456789012123456789012345678901 2
1 is working, you should suspect that there is a special triangle in here 2
12345678901234567890123456789012123456789012345678901 2
12345678901234567890123456789012123456789012345678901
somewhere. Notice that b must be 90 for the angles in the top triangle to 2
12345678901234567890123456789012123456789012345678901 2
12345678901234567890123456789012123456789012345678901 2
12345678901234567890123456789012123456789012345678901 2
sum to 180 degrees, and since a 5 c, they both must be 45 because a 1 b 1
12345678901234567890123456789012123456789012345678901
c 5 180° (it is a straight line). This means that the triangle on the right is a 2
12345678901234567890123456789012123456789012345678901 2
2345678901234567890123456789012123456789012345678901
45-45-90 triangle, and since we know the length of one side we can find 2
12345678901234567890123456789012123456789012345678901
12345678901234567890123456789012123456789012345678901 2
1 the length of all sides, including x, which is: 2
12345678901234567890123456789012123456789012345678901 2
12345678901234567890123456789012123456789012345678901 2
12345678901234567890123456789012123456789012345678901
x 5 (length of one side of the 45-45-90 triangle)(=2) 2
12345678901234567890123456789012123456789012345678901 2
12345678901234567890123456789012123456789012345678901 2
12345678901234567890123456789012123456789012345678901
= = = 2
12345678901234567890123456789012123456789012345678901 2
x 5 3 2 5 6 or choice (D)
12345678901234567890123456789012123456789012345678901 22
2345678901234567890123456789012123456789012345678901
12345678901234567890123456789012123456789012345678901 2
12345678901234567890123456789012123456789012345678901 2
12345678901234567890123456789012123456789012345678901 2
1
12345678901234567890123456789012123456789012345678901 2
12345678901234567890123456789012123456789012345678901 2
12345678901234567890123456789012123456789012345678901 2
12345678901234567890123456789012123456789012345678901 2
12345678901234567890123456789012123456789012345678901 2
12345678901234567890123456789012123456789012345678901 22
2345678901234567890123456789012123456789012345678901
1
12345678901234567890123456789012123456789012345678901 2
12345678901234567890123456789012123456789012345678901 2
12345678901234567890123456789012123456789012345678901 2
12345678901234567890123456789012123456789012345678901 2
12345678901234567890123456789012123456789012345678901 2
12345678901234567890123456789012123456789012345678901 2
12345678901234567890123456789012123456789012345678901 2 87
123456789012345678901234567890121234567890123456789012
Part III: Section Review

123456789012345678901234567890121234567890123456789012
12345678901234567890123456789012123456789012345678901 2
12345678901234567890123456789012123456789012345678901 2
12345678901234567890123456789012123456789012345678901
The next kind of super special triangle is the 30-60-90 triangle. There is 2
12345678901234567890123456789012123456789012345678901 2
12345678901234567890123456789012123456789012345678901
not a special name for it, but they appear on the test with great frequency 2
12345678901234567890123456789012123456789012345678901 2
12345678901234567890123456789012123456789012345678901
for the same reason that the 45-45-90 triangle does. You are expected to 2
1 identify the 30-60-90 triangle and then use its properties to solve 2
2345678901234567890123456789012123456789012345678901
12345678901234567890123456789012123456789012345678901 2
12345678901234567890123456789012123456789012345678901 2
12345678901234567890123456789012123456789012345678901
problems. 2
12345678901234567890123456789012123456789012345678901
2345678901234567890123456789012123456789012345678901
2
2
1
12345678901234567890123456789012123456789012345678901 2
12345678901234567890123456789012123456789012345678901 2
12345678901234567890123456789012123456789012345678901 2
12345678901234567890123456789012123456789012345678901
2345678901234567890123456789012123456789012345678901
2
2
1
12345678901234567890123456789012123456789012345678901 2
12345678901234567890123456789012123456789012345678901 2
12345678901234567890123456789012123456789012345678901 2
12345678901234567890123456789012123456789012345678901
2345678901234567890123456789012123456789012345678901
2
2
1
12345678901234567890123456789012123456789012345678901 2
12345678901234567890123456789012123456789012345678901
Call the smallest side length x. This will be the side opposite the 30-degree 2
12345678901234567890123456789012123456789012345678901 2
12345678901234567890123456789012123456789012345678901 = 2
12345678901234567890123456789012123456789012345678901 2
2345678901234567890123456789012123456789012345678901
angle. The side opposite the 60-degree angle is x 3, and the length of the
12345678901234567890123456789012123456789012345678901 2
12345678901234567890123456789012123456789012345678901
hypotenuse is always 2x. 2
12345678901234567890123456789012123456789012345678901 2
The equilateral triangle, the triangle whose sides are equal, is related to the 2
12345678901234567890123456789012123456789012345678901
12345678901234567890123456789012123456789012345678901 2
1 30-60-90 triangle. An equilateral triangle is like two 30-60-90 triangles 2
12345678901234567890123456789012123456789012345678901 2
12345678901234567890123456789012123456789012345678901
back-to-back. 2
12345678901234567890123456789012123456789012345678901 2
12345678901234567890123456789012123456789012345678901 2
2345678901234567890123456789012123456789012345678901
12345678901234567890123456789012123456789012345678901 2
12345678901234567890123456789012123456789012345678901 2
12345678901234567890123456789012123456789012345678901 2
12345678901234567890123456789012123456789012345678901 2
12345678901234567890123456789012123456789012345678901 2
12345678901234567890123456789012123456789012345678901 2
12345678901234567890123456789012123456789012345678901 2
12345678901234567890123456789012123456789012345678901 2 2
1
12345678901234567890123456789012123456789012345678901
Since the sides of an equilateral triangle are all equal, what must the 2
12345678901234567890123456789012123456789012345678901 2
12345678901234567890123456789012123456789012345678901 2
12345678901234567890123456789012123456789012345678901 2
relationship of the angles be? Equal. This means that 3 sides 5 180.
12345678901234567890123456789012123456789012345678901 2
12345678901234567890123456789012123456789012345678901 2
12345678901234567890123456789012123456789012345678901
So, 2
12345678901234567890123456789012123456789012345678901 2
12345678901234567890123456789012123456789012345678901 2
12345678901234567890123456789012123456789012345678901
3s 5 180 2
2345678901234567890123456789012123456789012345678901
12345678901234567890123456789012123456789012345678901 2
12345678901234567890123456789012123456789012345678901
s 5 60 2
1 2
12345678901234567890123456789012123456789012345678901 2
12345678901234567890123456789012123456789012345678901 2
For every equilateral triangle, each angle is 60 degrees. The lengths of all 3
2345678901234567890123456789012123456789012345678901
12345678901234567890123456789012123456789012345678901 2
12345678901234567890123456789012123456789012345678901
sides in an equilateral triangle are of equal lengths. 2
12345678901234567890123456789012123456789012345678901 2 2
12345678901234567890123456789012123456789012345678901
1 There are two other special right triangles that appear regularly on the 2
2345678901234567890123456789012123456789012345678901
12345678901234567890123456789012123456789012345678901
GRE. Both are defined by the relationships of the sides. 2
12345678901234567890123456789012123456789012345678901 2
12345678901234567890123456789012123456789012345678901 2
12345678901234567890123456789012123456789012345678901 2 2
1
12345678901234567890123456789012123456789012345678901 2
12345678901234567890123456789012123456789012345678901 2
12345678901234567890123456789012123456789012345678901 2
12345678901234567890123456789012123456789012345678901 2
12345678901234567890123456789012123456789012345678901 2
12345678901234567890123456789012123456789012345678901 2
2345678901234567890123456789012123456789012345678901
2
1
12345678901234567890123456789012123456789012345678901
If you recognize one of these triangles in a problem, and you know two 2
12345678901234567890123456789012123456789012345678901 2
12345678901234567890123456789012123456789012345678901
sides of the triangle, you can simply read off the length of the third side. 2
12345678901234567890123456789012123456789012345678901 2
12345678901234567890123456789012123456789012345678901 2
12345678901234567890123456789012123456789012345678901 2
12345678901234567890123456789012123456789012345678901 2
88 123456789012345678901234567890121234567890123456789012

www.petersons.com
Chapter 4: Quantitative Review

123456789012345678901234567890121234567890123456789012
12345678901234567890123456789012123456789012345678901 2
12345678901234567890123456789012123456789012345678901 2
12345678901234567890123456789012123456789012345678901 2
12345678901234567890123456789012123456789012345678901 2
12345678901234567890123456789012123456789012345678901 2
12345678901234567890123456789012123456789012345678901 2
12345678901234567890123456789012123456789012345678901 2
12345678901234567890123456789012123456789012345678901 2
12345678901234567890123456789012123456789012345678901 2
12345678901234567890123456789012123456789012345678901 2
12345678901234567890123456789012123456789012345678901 2
12345678901234567890123456789012123456789012345678901
2345678901234567890123456789012123456789012345678901
2
2
1
12345678901234567890123456789012123456789012345678901 2
12345678901234567890123456789012123456789012345678901 2
12345678901234567890123456789012123456789012345678901
If y 5 3, then x 5 2
12345678901234567890123456789012123456789012345678901
2345678901234567890123456789012123456789012345678901
2
2
1
12345678901234567890123456789012123456789012345678901
A. 21 2
12345678901234567890123456789012123456789012345678901 2
12345678901234567890123456789012123456789012345678901
B. 0 2
12345678901234567890123456789012123456789012345678901
2345678901234567890123456789012123456789012345678901
2
2
1 C. 1
12345678901234567890123456789012123456789012345678901 2
12345678901234567890123456789012123456789012345678901
D. 2 2
12345678901234567890123456789012123456789012345678901 2
12345678901234567890123456789012123456789012345678901
E. 3 2
12345678901234567890123456789012123456789012345678901 22
2345678901234567890123456789012123456789012345678901
12345678901234567890123456789012123456789012345678901
When you see the 30-degree angle, you should immediately start looking 2
12345678901234567890123456789012123456789012345678901
to see if you have 30-60-90 triangle. The top triangle is a right triangle 2
12345678901234567890123456789012123456789012345678901
12345678901234567890123456789012123456789012345678901 2
12345678901234567890123456789012123456789012345678901 2
1 since the bottom right angle is vertical with the right angle. This means the 2
12345678901234567890123456789012123456789012345678901
top triangle is a 30-60-90 triangle. So 2(x 1 y) 5 6, which means that if y 2
12345678901234567890123456789012123456789012345678901 2
12345678901234567890123456789012123456789012345678901
5 3 then x 5 0. 2
12345678901234567890123456789012123456789012345678901 22
2345678901234567890123456789012123456789012345678901
12345678901234567890123456789012123456789012345678901 2
12345678901234567890123456789012123456789012345678901 2
12345678901234567890123456789012123456789012345678901 2
12345678901234567890123456789012123456789012345678901
12345678901234567890123456789012123456789012345678901
Triangles: Similar and Congruent 2
When you say that a Cheeto looks similar to an orange caterpillar, you 2
12345678901234567890123456789012123456789012345678901
12345678901234567890123456789012123456789012345678901 2
12345678901234567890123456789012123456789012345678901 2
1 probably aren’t using an exact definition of similar. When mathematicians 2
2345678901234567890123456789012123456789012345678901
use the term similar, they are being much more precise. To a mathemati- 2
12345678901234567890123456789012123456789012345678901
12345678901234567890123456789012123456789012345678901 2
1 cian, two triangles are similar if all corresponding angles are equal and all 2
12345678901234567890123456789012123456789012345678901 2
12345678901234567890123456789012123456789012345678901
corresponding sides are proportional. Similar triangles look the same, but 2
12345678901234567890123456789012123456789012345678901 2
12345678901234567890123456789012123456789012345678901 2
12345678901234567890123456789012123456789012345678901 2
one can be bigger than the other.
12345678901234567890123456789012123456789012345678901 2
12345678901234567890123456789012123456789012345678901 2
12345678901234567890123456789012123456789012345678901 22
2345678901234567890123456789012123456789012345678901
12345678901234567890123456789012123456789012345678901 2
1
12345678901234567890123456789012123456789012345678901 2
12345678901234567890123456789012123456789012345678901 2
12345678901234567890123456789012123456789012345678901 2
12345678901234567890123456789012123456789012345678901 2
12345678901234567890123456789012123456789012345678901 2
12345678901234567890123456789012123456789012345678901 2
12345678901234567890123456789012123456789012345678901
Triangle M has sides that are three times as large as triangle N, but all 2
12345678901234567890123456789012123456789012345678901 22
2345678901234567890123456789012123456789012345678901
12345678901234567890123456789012123456789012345678901
angles are equal. It doesn’t matter that triangle N is flipped around in
2
12345678901234567890123456789012123456789012345678901
relation to M. Since the angles are equal and the sides proportional, the 2
12345678901234567890123456789012123456789012345678901
1 two triangles are congruent. 2
12345678901234567890123456789012123456789012345678901 2
12345678901234567890123456789012123456789012345678901 2
12345678901234567890123456789012123456789012345678901 2
12345678901234567890123456789012123456789012345678901
If you were to increase the size of triangle N’s sides by a factor of 3, then 2
12345678901234567890123456789012123456789012345678901 2
12345678901234567890123456789012123456789012345678901 22
2345678901234567890123456789012123456789012345678901
you would have two triangles with equal sides and angles. This is the
1 definition of congruent triangles.
12345678901234567890123456789012123456789012345678901 2
12345678901234567890123456789012123456789012345678901 2
12345678901234567890123456789012123456789012345678901 2
12345678901234567890123456789012123456789012345678901 2
12345678901234567890123456789012123456789012345678901 2
12345678901234567890123456789012123456789012345678901 2
12345678901234567890123456789012123456789012345678901 2 89
123456789012345678901234567890121234567890123456789012
Part III: Section Review

123456789012345678901234567890121234567890123456789012
12345678901234567890123456789012123456789012345678901 2
12345678901234567890123456789012123456789012345678901 2
12345678901234567890123456789012123456789012345678901
Other Polygons 2
12345678901234567890123456789012123456789012345678901 2
12345678901234567890123456789012123456789012345678901 2
12345678901234567890123456789012123456789012345678901 2
Polygon is the fancy geometric word for shapes that have three or more
12345678901234567890123456789012123456789012345678901
sides. Lots of shapes are polygons like triangles, squares, pentagons, and 2
12345678901234567890123456789012123456789012345678901 2
12345678901234567890123456789012123456789012345678901
octagons, to name just a few. 2
12345678901234567890123456789012123456789012345678901 2
12345678901234567890123456789012123456789012345678901 2
12345678901234567890123456789012123456789012345678901
The one general rule to remember about polygons comes in the form of a 2
1 formula. The interior angles of an n-sided polygon sum to (n 2 2)(180). 2
2345678901234567890123456789012123456789012345678901
12345678901234567890123456789012123456789012345678901 2
12345678901234567890123456789012123456789012345678901 2
12345678901234567890123456789012123456789012345678901
You can try this formula out on a triangle and get (3 2 2)(180) 5 180. This 2
12345678901234567890123456789012123456789012345678901 2
12345678901234567890123456789012123456789012345678901
restates the old “all three angles of a triangle sum to 180” rule that should 2
12345678901234567890123456789012123456789012345678901
be hammered about halfway into your skull by now. You don’t need the 2
12345678901234567890123456789012123456789012345678901 2
12345678901234567890123456789012123456789012345678901 2
12345678901234567890123456789012123456789012345678901
(n 2 2)(180) formula to find out the sum of the interior angles of a 2
1 triangle, but you might need it if you need to find the sum of the interior 2
2345678901234567890123456789012123456789012345678901
12345678901234567890123456789012123456789012345678901 2
12345678901234567890123456789012123456789012345678901 2
12345678901234567890123456789012123456789012345678901
angles of a hexagon. 2
12345678901234567890123456789012123456789012345678901 2
2345678901234567890123456789012123456789012345678901
The rectangle is a polygon. It has four sides, so the sum of its interior 2
12345678901234567890123456789012123456789012345678901
12345678901234567890123456789012123456789012345678901 2
12345678901234567890123456789012123456789012345678901 2
12345678901234567890123456789012123456789012345678901 2
angles must be (4 2 2)(180) 5 360. There are two important properties of
12345678901234567890123456789012123456789012345678901
rectangle: 2
12345678901234567890123456789012123456789012345678901 2
1 2
12345678901234567890123456789012123456789012345678901
1. All interior angles are 90 degrees. 2
12345678901234567890123456789012123456789012345678901 2
12345678901234567890123456789012123456789012345678901 2
12345678901234567890123456789012123456789012345678901 2
2345678901234567890123456789012123456789012345678901
2. Opposite sides are parallel and equal in length.
12345678901234567890123456789012123456789012345678901 2
12345678901234567890123456789012123456789012345678901 2
12345678901234567890123456789012123456789012345678901 2
12345678901234567890123456789012123456789012345678901 2
12345678901234567890123456789012123456789012345678901 2
12345678901234567890123456789012123456789012345678901 2
12345678901234567890123456789012123456789012345678901 2
12345678901234567890123456789012123456789012345678901 2 2
1
12345678901234567890123456789012123456789012345678901 2
12345678901234567890123456789012123456789012345678901 2
12345678901234567890123456789012123456789012345678901 2
12345678901234567890123456789012123456789012345678901 2
12345678901234567890123456789012123456789012345678901 2
12345678901234567890123456789012123456789012345678901 2
12345678901234567890123456789012123456789012345678901 2
12345678901234567890123456789012123456789012345678901 2
12345678901234567890123456789012123456789012345678901
Note that only opposite sides of a rectangle need to be equal in length. If all 2
12345678901234567890123456789012123456789012345678901 2
2345678901234567890123456789012123456789012345678901
12345678901234567890123456789012123456789012345678901
sides of a rectangle are equal, you have the figure known as a square. 2
12345678901234567890123456789012123456789012345678901 2 2
1
12345678901234567890123456789012123456789012345678901 2
12345678901234567890123456789012123456789012345678901 2
Note

12345678901234567890123456789012123456789012345678901
The perimeter of a rectangle can be found by the formula P 5 2(l 1 w) 2
12345678901234567890123456789012123456789012345678901 2
12345678901234567890123456789012123456789012345678901
and its area is A 5 l 3 w. For a square, the equations are pretty basic: 2
12345678901234567890123456789012123456789012345678901 2
12345678901234567890123456789012123456789012345678901
P 5 4s and A 5 s2. 2
12345678901234567890123456789012123456789012345678901 2
2345678901234567890123456789012123456789012345678901
12345678901234567890123456789012123456789012345678901 2
12345678901234567890123456789012123456789012345678901 2
12345678901234567890123456789012123456789012345678901 2 2
1
12345678901234567890123456789012123456789012345678901 2
12345678901234567890123456789012123456789012345678901 2
12345678901234567890123456789012123456789012345678901 2
12345678901234567890123456789012123456789012345678901 2
12345678901234567890123456789012123456789012345678901 2
12345678901234567890123456789012123456789012345678901 2
2345678901234567890123456789012123456789012345678901
2
1
12345678901234567890123456789012123456789012345678901 2
12345678901234567890123456789012123456789012345678901 2
12345678901234567890123456789012123456789012345678901 2
12345678901234567890123456789012123456789012345678901 2
12345678901234567890123456789012123456789012345678901 2
12345678901234567890123456789012123456789012345678901 2
12345678901234567890123456789012123456789012345678901 2
90 123456789012345678901234567890121234567890123456789012

www.petersons.com
Chapter 4: Quantitative Review

123456789012345678901234567890121234567890123456789012
12345678901234567890123456789012123456789012345678901 2
12345678901234567890123456789012123456789012345678901 2
12345678901234567890123456789012123456789012345678901
Column A Column B 2
12345678901234567890123456789012123456789012345678901 2
12345678901234567890123456789012123456789012345678901 2
12345678901234567890123456789012123456789012345678901 2
12345678901234567890123456789012123456789012345678901 2
12345678901234567890123456789012123456789012345678901 2
12345678901234567890123456789012123456789012345678901 2
12345678901234567890123456789012123456789012345678901 2
12345678901234567890123456789012123456789012345678901 2
12345678901234567890123456789012123456789012345678901
2345678901234567890123456789012123456789012345678901
2
2
1
12345678901234567890123456789012123456789012345678901 2
12345678901234567890123456789012123456789012345678901 2
12345678901234567890123456789012123456789012345678901
In the figure above, ACDF is a rectangle, 2
12345678901234567890123456789012123456789012345678901
2345678901234567890123456789012123456789012345678901
2
2
1 and ABEF and BCDE are squares.
12345678901234567890123456789012123456789012345678901 2
12345678901234567890123456789012123456789012345678901 2
12345678901234567890123456789012123456789012345678901
32 Area of ACDF 2
12345678901234567890123456789012123456789012345678901 2
1 You have to find the area of ACDF to compare the two columns, which 2
2345678901234567890123456789012123456789012345678901
12345678901234567890123456789012123456789012345678901 2
12345678901234567890123456789012123456789012345678901 2
12345678901234567890123456789012123456789012345678901
means you need to find the length and the width of the rectangle. The 2
12345678901234567890123456789012123456789012345678901 2
12345678901234567890123456789012123456789012345678901
diagonal in the square bisects two right angles, creating your new friend the 2
12345678901234567890123456789012123456789012345678901 2
12345678901234567890123456789012123456789012345678901
45-45-90 triangle. This makes =32 the hypotenuse. From this, you can 2
12345678901234567890123456789012123456789012345678901 2
12345678901234567890123456789012123456789012345678901
= 2
2345678901234567890123456789012123456789012345678901
12345678901234567890123456789012123456789012345678901
32
5 =16 5 4. Line segment EF 2
1 read off the side length of the square, 2
12345678901234567890123456789012123456789012345678901
= 2 2
12345678901234567890123456789012123456789012345678901 2
12345678901234567890123456789012123456789012345678901
5 4, as does ED, AF, AB, BC, and BE. This makes the length of the rect- 2
2345678901234567890123456789012123456789012345678901
angle 8 (4 1 4), while the width the rectangle is 4. Placing these values into 2
12345678901234567890123456789012123456789012345678901
12345678901234567890123456789012123456789012345678901 2
12345678901234567890123456789012123456789012345678901 2
12345678901234567890123456789012123456789012345678901 22
the formula for the area of a rectangle gives you 4 3 8 5 32. Choice (C) is
12345678901234567890123456789012123456789012345678901
12345678901234567890123456789012123456789012345678901
the correct answer. 2
12345678901234567890123456789012123456789012345678901 22
12345678901234567890123456789012123456789012345678901
The parallelogram and the trapezoid are two other 4-sided polygons to
12345678901234567890123456789012123456789012345678901 2
1 know. A parallelogram, like a rectangle, has opposite sides that are parallel 2
12345678901234567890123456789012123456789012345678901 22
2345678901234567890123456789012123456789012345678901
12345678901234567890123456789012123456789012345678901
1 and equal. Unlike a rectangle, the interior angles of parallelograms are not
2
12345678901234567890123456789012123456789012345678901
right angles. The opposite angles of a parallelogram are equal and they 2
12345678901234567890123456789012123456789012345678901 2
12345678901234567890123456789012123456789012345678901
sum to 360, which makes sense if you remember that n-sided polygon 2
12345678901234567890123456789012123456789012345678901 2
12345678901234567890123456789012123456789012345678901
formula. 2
12345678901234567890123456789012123456789012345678901 2
12345678901234567890123456789012123456789012345678901 2
12345678901234567890123456789012123456789012345678901 22
2345678901234567890123456789012123456789012345678901
12345678901234567890123456789012123456789012345678901 2
1
12345678901234567890123456789012123456789012345678901 2
12345678901234567890123456789012123456789012345678901 2
12345678901234567890123456789012123456789012345678901 2
12345678901234567890123456789012123456789012345678901 2
12345678901234567890123456789012123456789012345678901 2
12345678901234567890123456789012123456789012345678901 2
12345678901234567890123456789012123456789012345678901 2
12345678901234567890123456789012123456789012345678901 22
2345678901234567890123456789012123456789012345678901
12345678901234567890123456789012123456789012345678901
The area of a parallelogram is the base times the height. These values are
2
12345678901234567890123456789012123456789012345678901
determined similarly to the way you determine the base and height of a 2
12345678901234567890123456789012123456789012345678901
1 2
12345678901234567890123456789012123456789012345678901
triangle. Pick the base, then draw a line perpendicular to it that intersects 2
12345678901234567890123456789012123456789012345678901 2
12345678901234567890123456789012123456789012345678901
with the side across from it. That line is the height. The one difference is 2
12345678901234567890123456789012123456789012345678901 2
12345678901234567890123456789012123456789012345678901
that the height of a parallelogram doesn’t have to intersect with an angle. 2
12345678901234567890123456789012123456789012345678901 22
2345678901234567890123456789012123456789012345678901
1 As you can see, trapezoids do not have equal opposite sides or angles. In
12345678901234567890123456789012123456789012345678901 2
12345678901234567890123456789012123456789012345678901 2
12345678901234567890123456789012123456789012345678901
fact, to be a trapezoid, all you need is 4
2345678901234567890123456789012123456789012345678901
sides, where 2 of those sides are 2
2
1 parallel. In the trapezoid shown, the top and bottom sides are parallel.
12345678901234567890123456789012123456789012345678901 2
12345678901234567890123456789012123456789012345678901 2
12345678901234567890123456789012123456789012345678901 2 91
123456789012345678901234567890121234567890123456789012
Part III: Section Review

123456789012345678901234567890121234567890123456789012
12345678901234567890123456789012123456789012345678901 2
12345678901234567890123456789012123456789012345678901 2
12345678901234567890123456789012123456789012345678901 2
12345678901234567890123456789012123456789012345678901 2
12345678901234567890123456789012123456789012345678901 2
12345678901234567890123456789012123456789012345678901 2
12345678901234567890123456789012123456789012345678901 2
12345678901234567890123456789012123456789012345678901 2
12345678901234567890123456789012123456789012345678901 2
12345678901234567890123456789012123456789012345678901 2
12345678901234567890123456789012123456789012345678901
What is the area of the above figure? 2
12345678901234567890123456789012123456789012345678901
2345678901234567890123456789012123456789012345678901
2
2
1
12345678901234567890123456789012123456789012345678901
A. 20 2
12345678901234567890123456789012123456789012345678901 2
12345678901234567890123456789012123456789012345678901
B. 28 2
12345678901234567890123456789012123456789012345678901
C. 32
2345678901234567890123456789012123456789012345678901
2
2
1
12345678901234567890123456789012123456789012345678901
D. 34 2
12345678901234567890123456789012123456789012345678901 2
12345678901234567890123456789012123456789012345678901
E. 40 2
12345678901234567890123456789012123456789012345678901 2
1 If you have no idea how to do this problem, you can at least eliminate 2
2345678901234567890123456789012123456789012345678901
12345678901234567890123456789012123456789012345678901 2
12345678901234567890123456789012123456789012345678901 2
12345678901234567890123456789012123456789012345678901
choices (A) and (B), since you know that this trapezoid will be bigger than 2
12345678901234567890123456789012123456789012345678901 2
12345678901234567890123456789012123456789012345678901
a rectangle with length 7 and width 4. That’s because you can see how this 2
12345678901234567890123456789012123456789012345678901
trapezoid is like a rectangle with length 7 and width 4 with a triangle 2
12345678901234567890123456789012123456789012345678901 2
12345678901234567890123456789012123456789012345678901 2
12345678901234567890123456789012123456789012345678901
attached to it. Viewing it this way is the key to the problem. To find the 2
2345678901234567890123456789012123456789012345678901
12345678901234567890123456789012123456789012345678901 2
1 area of a trapezoid, first draw a dotted line to turn the trapezoid into a 2
12345678901234567890123456789012123456789012345678901 2
12345678901234567890123456789012123456789012345678901
rectangle and a triangle. 2
12345678901234567890123456789012123456789012345678901 2
12345678901234567890123456789012123456789012345678901 2
2345678901234567890123456789012123456789012345678901
12345678901234567890123456789012123456789012345678901 2
12345678901234567890123456789012123456789012345678901 2
12345678901234567890123456789012123456789012345678901 2
12345678901234567890123456789012123456789012345678901 2
12345678901234567890123456789012123456789012345678901 2
12345678901234567890123456789012123456789012345678901 2
12345678901234567890123456789012123456789012345678901 2
12345678901234567890123456789012123456789012345678901 2
1 You can read off the third side of the triangle if you recognize that it is a 2
12345678901234567890123456789012123456789012345678901 2
12345678901234567890123456789012123456789012345678901
3-4-5 triangle. The area of the rectangle is 28, and the area of the triangle 2
12345678901234567890123456789012123456789012345678901 2
12345678901234567890123456789012123456789012345678901 2
12345678901234567890123456789012123456789012345678901
1
2345678901234567890123456789012123456789012345678901
2
2
12345678901234567890123456789012123456789012345678901
is ~3 !~4! 5 6. So the area of the trapezoid is 34, choice (D). You found
12345678901234567890123456789012123456789012345678901
2 2
1 the answer, and you also saw a way in which special triangles can be 2
12345678901234567890123456789012123456789012345678901 2
12345678901234567890123456789012123456789012345678901
cropped and provide the key to a problem. 2
12345678901234567890123456789012123456789012345678901 2
2345678901234567890123456789012123456789012345678901
12345678901234567890123456789012123456789012345678901 2 2
1
12345678901234567890123456789012123456789012345678901 2
12345678901234567890123456789012123456789012345678901 2
12345678901234567890123456789012123456789012345678901 2
12345678901234567890123456789012123456789012345678901 2
12345678901234567890123456789012123456789012345678901 2
12345678901234567890123456789012123456789012345678901 2
12345678901234567890123456789012123456789012345678901 2
12345678901234567890123456789012123456789012345678901 2
2345678901234567890123456789012123456789012345678901
12345678901234567890123456789012123456789012345678901 2
12345678901234567890123456789012123456789012345678901 2
12345678901234567890123456789012123456789012345678901 2 2
1
12345678901234567890123456789012123456789012345678901 2
12345678901234567890123456789012123456789012345678901 2
12345678901234567890123456789012123456789012345678901 2
12345678901234567890123456789012123456789012345678901 2
12345678901234567890123456789012123456789012345678901 2
12345678901234567890123456789012123456789012345678901 2
2345678901234567890123456789012123456789012345678901
2
1
12345678901234567890123456789012123456789012345678901 2
12345678901234567890123456789012123456789012345678901 2
12345678901234567890123456789012123456789012345678901 2
12345678901234567890123456789012123456789012345678901 2
12345678901234567890123456789012123456789012345678901 2
12345678901234567890123456789012123456789012345678901 2
12345678901234567890123456789012123456789012345678901 2
92 123456789012345678901234567890121234567890123456789012

www.petersons.com
Chapter 4: Quantitative Review

123456789012345678901234567890121234567890123456789012
12345678901234567890123456789012123456789012345678901 2
12345678901234567890123456789012123456789012345678901 2
12345678901234567890123456789012123456789012345678901
Circles 2
12345678901234567890123456789012123456789012345678901 2
12345678901234567890123456789012123456789012345678901 2
12345678901234567890123456789012123456789012345678901 2
For many, circles evoke ideas like elegance and eternity. Sadly, elegance is
12345678901234567890123456789012123456789012345678901
not one of standards commonly tested on the GRE. Instead, the more 2
12345678901234567890123456789012123456789012345678901 2
12345678901234567890123456789012123456789012345678901
mundane properties of the circle are focused on, so any musings about 2
12345678901234567890123456789012123456789012345678901 2
12345678901234567890123456789012123456789012345678901
eternity must take a back seat to the definition of circumference. 2
12345678901234567890123456789012123456789012345678901
2345678901234567890123456789012123456789012345678901
2
2
1
12345678901234567890123456789012123456789012345678901 2
12345678901234567890123456789012123456789012345678901 2
12345678901234567890123456789012123456789012345678901 2
12345678901234567890123456789012123456789012345678901
2345678901234567890123456789012123456789012345678901
2
2
1
12345678901234567890123456789012123456789012345678901 2
12345678901234567890123456789012123456789012345678901 2
12345678901234567890123456789012123456789012345678901 2
12345678901234567890123456789012123456789012345678901
2345678901234567890123456789012123456789012345678901
2
2
1
12345678901234567890123456789012123456789012345678901 2
12345678901234567890123456789012123456789012345678901 2
12345678901234567890123456789012123456789012345678901 2
12345678901234567890123456789012123456789012345678901 2
12345678901234567890123456789012123456789012345678901 22
2345678901234567890123456789012123456789012345678901
12345678901234567890123456789012123456789012345678901 2
12345678901234567890123456789012123456789012345678901 2
12345678901234567890123456789012123456789012345678901 2
12345678901234567890123456789012123456789012345678901
O is called the center of the circle, and all the points along the edge of the 2
12345678901234567890123456789012123456789012345678901
1 circle are the same distance from O. That distance, r, is called the radius. 2
12345678901234567890123456789012123456789012345678901 2
12345678901234567890123456789012123456789012345678901 2
12345678901234567890123456789012123456789012345678901
The radius is any line that goes from the center to the edge of the circle. If 2
2345678901234567890123456789012123456789012345678901
a line starts at the edge of the circle, runs through O, and continues on to 2
12345678901234567890123456789012123456789012345678901
12345678901234567890123456789012123456789012345678901 2
12345678901234567890123456789012123456789012345678901
the opposite edge, it is called the diameter. By definition, every diameter is 2
12345678901234567890123456789012123456789012345678901 22
12345678901234567890123456789012123456789012345678901
made up of two radii, so diameter 5 2(radius). Any line whose endpoints
12345678901234567890123456789012123456789012345678901 2
are on the edge of the circle is called a chord, like MN. The diameter is one 2
12345678901234567890123456789012123456789012345678901
12345678901234567890123456789012123456789012345678901 22
12345678901234567890123456789012123456789012345678901
1 particular chord. 2
12345678901234567890123456789012123456789012345678901 2
12345678901234567890123456789012123456789012345678901
If you were to walk along the edge of a circle once, the distance you would 2
12345678901234567890123456789012123456789012345678901 2
12345678901234567890123456789012123456789012345678901
have traveled is called the circumference. The circumference is like the 2
12345678901234567890123456789012123456789012345678901 2
12345678901234567890123456789012123456789012345678901
perimeter of a polygon except that you have to be a little more 2
12345678901234567890123456789012123456789012345678901
sophisticated in calculating it because the circumference isn’t composed of 2
12345678901234567890123456789012123456789012345678901 2
12345678901234567890123456789012123456789012345678901 2
12345678901234567890123456789012123456789012345678901
straight lines. This is the beauty of the concept of pi, often shown by the 2
2345678901234567890123456789012123456789012345678901
symbol p. Mathematicians way back when came up with this concept, and 2
12345678901234567890123456789012123456789012345678901
12345678901234567890123456789012123456789012345678901 2
1 boy, does it help. 2
12345678901234567890123456789012123456789012345678901 2
12345678901234567890123456789012123456789012345678901 2
12345678901234567890123456789012123456789012345678901
To dispel the mystery around it, p is simply defined as the circumference of 2
12345678901234567890123456789012123456789012345678901 2
12345678901234567890123456789012123456789012345678901
a circle divided by its diameter: 2
12345678901234567890123456789012123456789012345678901 2
12345678901234567890123456789012123456789012345678901 2
2345678901234567890123456789012123456789012345678901
12345678901234567890123456789012123456789012345678901 2
12345678901234567890123456789012123456789012345678901
circumference 2
12345678901234567890123456789012123456789012345678901
p 5 2
12345678901234567890123456789012123456789012345678901 22
diameter
1
12345678901234567890123456789012123456789012345678901
The neat thing about p is that its value remains constant regardless of the 2
12345678901234567890123456789012123456789012345678901 2
12345678901234567890123456789012123456789012345678901
size of the circle. The ancient mathematicians figured this value to be 2
12345678901234567890123456789012123456789012345678901 2
12345678901234567890123456789012123456789012345678901
3.14159 . . . and a bunch of other numbers, since pi goes on for a long, 2
12345678901234567890123456789012123456789012345678901 22
2345678901234567890123456789012123456789012345678901
1 long time. For our purposes, just remember that it is a little bigger than 3,
12345678901234567890123456789012123456789012345678901 2
12345678901234567890123456789012123456789012345678901
about 3.14. 2
12345678901234567890123456789012123456789012345678901
2345678901234567890123456789012123456789012345678901
2
2
1
12345678901234567890123456789012123456789012345678901 2
12345678901234567890123456789012123456789012345678901 2
12345678901234567890123456789012123456789012345678901 2 93
123456789012345678901234567890121234567890123456789012
Part III: Section Review

123456789012345678901234567890121234567890123456789012
12345678901234567890123456789012123456789012345678901 2
12345678901234567890123456789012123456789012345678901 2
12345678901234567890123456789012123456789012345678901
From the definition of p, you can determine the formula for circumfer- 2
12345678901234567890123456789012123456789012345678901 2
12345678901234567890123456789012123456789012345678901
ence: 2
12345678901234567890123456789012123456789012345678901 2
12345678901234567890123456789012123456789012345678901 2
12345678901234567890123456789012123456789012345678901
circumference 2
12345678901234567890123456789012123456789012345678901
p5 → 2
12345678901234567890123456789012123456789012345678901
diameter 2
12345678901234567890123456789012123456789012345678901 2
12345678901234567890123456789012123456789012345678901
2345678901234567890123456789012123456789012345678901
2
2
1 p5 →
c
12345678901234567890123456789012123456789012345678901 2
12345678901234567890123456789012123456789012345678901
d 2
12345678901234567890123456789012123456789012345678901 2
12345678901234567890123456789012123456789012345678901
c
2345678901234567890123456789012123456789012345678901
2
2
1 p 3 d 5 3 d →
12345678901234567890123456789012123456789012345678901
d 2
12345678901234567890123456789012123456789012345678901 2
12345678901234567890123456789012123456789012345678901
c 2
12345678901234567890123456789012123456789012345678901
pd 5
2345678901234567890123456789012123456789012345678901
2
2
1
12345678901234567890123456789012123456789012345678901
p 3 diameter 5 circumference 2
12345678901234567890123456789012123456789012345678901 2
12345678901234567890123456789012123456789012345678901 2
12345678901234567890123456789012123456789012345678901
or c 5 pd for short. Remember that since the diameter is twice the radius, 2
2345678901234567890123456789012123456789012345678901
12345678901234567890123456789012123456789012345678901
you can also state: 2
12345678901234567890123456789012123456789012345678901 2
12345678901234567890123456789012123456789012345678901 2
12345678901234567890123456789012123456789012345678901
c 5 pd 5 2pr 2
12345678901234567890123456789012123456789012345678901 2
12345678901234567890123456789012123456789012345678901 2
1 Let’s go back to that mythical walk around the circle. If you only walked 2
12345678901234567890123456789012123456789012345678901
partway around the circle, the distance that you walked would be called an 2
12345678901234567890123456789012123456789012345678901 2
12345678901234567890123456789012123456789012345678901 2
12345678901234567890123456789012123456789012345678901 2
2345678901234567890123456789012123456789012345678901
arc of the circle. Imagine you walked from A through B to C in the figure
12345678901234567890123456789012123456789012345678901 2
12345678901234567890123456789012123456789012345678901
below: 2
12345678901234567890123456789012123456789012345678901 2
12345678901234567890123456789012123456789012345678901 2
12345678901234567890123456789012123456789012345678901 2
12345678901234567890123456789012123456789012345678901 2
12345678901234567890123456789012123456789012345678901 2
12345678901234567890123456789012123456789012345678901 2 2
1
12345678901234567890123456789012123456789012345678901 2
12345678901234567890123456789012123456789012345678901 2
12345678901234567890123456789012123456789012345678901 2
12345678901234567890123456789012123456789012345678901 2
12345678901234567890123456789012123456789012345678901 2
12345678901234567890123456789012123456789012345678901 2
12345678901234567890123456789012123456789012345678901 2
12345678901234567890123456789012123456789012345678901 2
12345678901234567890123456789012123456789012345678901 2
12345678901234567890123456789012123456789012345678901 2
12345678901234567890123456789012123456789012345678901 2
2345678901234567890123456789012123456789012345678901
12345678901234567890123456789012123456789012345678901 2 2
1
12345678901234567890123456789012123456789012345678901 2
12345678901234567890123456789012123456789012345678901 2
Note

12345678901234567890123456789012123456789012345678901 2
12345678901234567890123456789012123456789012345678901
This arc would be referred to as arc ABC, since it starts at A, goes through 2
12345678901234567890123456789012123456789012345678901 2
12345678901234567890123456789012123456789012345678901
B, and ends at C. 2
12345678901234567890123456789012123456789012345678901 2
12345678901234567890123456789012123456789012345678901 2
2345678901234567890123456789012123456789012345678901
12345678901234567890123456789012123456789012345678901 2
12345678901234567890123456789012123456789012345678901 2
12345678901234567890123456789012123456789012345678901 2
1 The distance you walked is part of the circumference, but what part is it? 2
12345678901234567890123456789012123456789012345678901 2
12345678901234567890123456789012123456789012345678901
To answer this, you must recall that there are 360 degrees in a circle. (If 2
12345678901234567890123456789012123456789012345678901 2
12345678901234567890123456789012123456789012345678901
you think this is a foolish number, blame the Babylonians.) The angle of 2
12345678901234567890123456789012123456789012345678901 2
12345678901234567890123456789012123456789012345678901
the arc is 60 degrees, and since there are 360 degrees in a circle, the arc 2
12345678901234567890123456789012123456789012345678901 2
S D
12345678901234567890123456789012123456789012345678901
12345678901234567890123456789012123456789012345678901
angle is one sixth of the degrees in the circle
12345678901234567890123456789012123456789012345678901
12345678901234567890123456789012123456789012345678901
60
360
5 .
1
6
2
2
2
2
12345678901234567890123456789012123456789012345678901 2
12345678901234567890123456789012123456789012345678901 2
12345678901234567890123456789012123456789012345678901 2
94 123456789012345678901234567890121234567890123456789012

www.petersons.com
Chapter 4: Quantitative Review

123456789012345678901234567890121234567890123456789012
12345678901234567890123456789012123456789012345678901 2
12345678901234567890123456789012123456789012345678901 2
12345678901234567890123456789012123456789012345678901
The arc length will have the same relationship with the circumference as 2
12345678901234567890123456789012123456789012345678901 2
12345678901234567890123456789012123456789012345678901
the arc angle does with the total number of degrees in a circle. Therefore, 2
12345678901234567890123456789012123456789012345678901 2
12345678901234567890123456789012123456789012345678901
1
2345678901234567890123456789012123456789012345678901
2
2
1 arcABC 5 c. All this can be encapsulated in a formula, the arc length 5
12345678901234567890123456789012123456789012345678901
6 2
12345678901234567890123456789012123456789012345678901 2
12345678901234567890123456789012123456789012345678901
n 2
12345678901234567890123456789012123456789012345678901
3 2pr, where n is the measure of
2345678901234567890123456789012123456789012345678901
the arc angle. 2
2
1 360
12345678901234567890123456789012123456789012345678901 2
12345678901234567890123456789012123456789012345678901
The area of a circle is pr . This means that if you know the radius (or the 2
2
12345678901234567890123456789012123456789012345678901 2
12345678901234567890123456789012123456789012345678901
diameter) of a circle, you can find the area and the circumference of that 2
1 circle. A common test question is to give enough information to find the 2
2345678901234567890123456789012123456789012345678901
12345678901234567890123456789012123456789012345678901 2
12345678901234567890123456789012123456789012345678901 2
12345678901234567890123456789012123456789012345678901
radius of a circle and ask for the area or circumference of a circle. 2
12345678901234567890123456789012123456789012345678901
2345678901234567890123456789012123456789012345678901
2
2
1
12345678901234567890123456789012123456789012345678901
Column A Column B 2
12345678901234567890123456789012123456789012345678901 2
12345678901234567890123456789012123456789012345678901 2
12345678901234567890123456789012123456789012345678901
The area of a circle is py. 2
2345678901234567890123456789012123456789012345678901
12345678901234567890123456789012123456789012345678901 2
12345678901234567890123456789012123456789012345678901
The diameter of the 3 2
12345678901234567890123456789012123456789012345678901 y2 2
12345678901234567890123456789012123456789012345678901
circle 2
12345678901234567890123456789012123456789012345678901 y 2
12345678901234567890123456789012123456789012345678901 22
1
12345678901234567890123456789012123456789012345678901 2
12345678901234567890123456789012123456789012345678901 2
12345678901234567890123456789012123456789012345678901 2
12345678901234567890123456789012123456789012345678901 22
2345678901234567890123456789012123456789012345678901
If this question is a mystery to you, you can at least reason that Column A
12345678901234567890123456789012123456789012345678901
will have a y in it. This means that the two columns will probably have a 2
12345678901234567890123456789012123456789012345678901
12345678901234567890123456789012123456789012345678901
definite relationship, so (D) is out. The trick is to make Column A look like 2
2
12345678901234567890123456789012123456789012345678901
something you can compare to Column B, which you can do if you figure 2
12345678901234567890123456789012123456789012345678901
12345678901234567890123456789012123456789012345678901 2
12345678901234567890123456789012123456789012345678901 2
12345678901234567890123456789012123456789012345678901 22
out what the diameter of the circle is:
1
12345678901234567890123456789012123456789012345678901
Area of a circle 5 pr2 2
12345678901234567890123456789012123456789012345678901 2
12345678901234567890123456789012123456789012345678901 2
12345678901234567890123456789012123456789012345678901
2
→ 2
12345678901234567890123456789012123456789012345678901
py 5 pr
2345678901234567890123456789012123456789012345678901
2
2
12345678901234567890123456789012123456789012345678901
12345678901234567890123456789012123456789012345678901
py 4 p 5 pr2 4 p → 2
1 2
12345678901234567890123456789012123456789012345678901
y5r → 2
2
12345678901234567890123456789012123456789012345678901 2
2345678901234567890123456789012123456789012345678901
12345678901234567890123456789012123456789012345678901 2
12345678901234567890123456789012123456789012345678901
= y5r 2
1 2
12345678901234567890123456789012123456789012345678901 2
12345678901234567890123456789012123456789012345678901
If that’s the radius, then the diameter is twice that, or d 5 2=y. Compare 2
12345678901234567890123456789012123456789012345678901 2
12345678901234567890123456789012123456789012345678901
this with Column B after you sort it all out: 2
12345678901234567890123456789012123456789012345678901 2
12345678901234567890123456789012123456789012345678901 2
12345678901234567890123456789012123456789012345678901
3
2
2345678901234567890123456789012123456789012345678901
12345678901234567890123456789012123456789012345678901
y2 3 2 1 2
12345678901234567890123456789012123456789012345678901
5 y2 2 2 5 y2 5 =y 2
12345678901234567890123456789012123456789012345678901
y 2
12345678901234567890123456789012123456789012345678901 22
1 (A) is greater.
12345678901234567890123456789012123456789012345678901 2
12345678901234567890123456789012123456789012345678901 2
12345678901234567890123456789012123456789012345678901 2
12345678901234567890123456789012123456789012345678901 2
12345678901234567890123456789012123456789012345678901 2
12345678901234567890123456789012123456789012345678901 22
2345678901234567890123456789012123456789012345678901
1
12345678901234567890123456789012123456789012345678901 2
12345678901234567890123456789012123456789012345678901 2
12345678901234567890123456789012123456789012345678901 2
12345678901234567890123456789012123456789012345678901 2
12345678901234567890123456789012123456789012345678901 2
12345678901234567890123456789012123456789012345678901 2
12345678901234567890123456789012123456789012345678901 2 95
123456789012345678901234567890121234567890123456789012
Part III: Section Review

123456789012345678901234567890121234567890123456789012
12345678901234567890123456789012123456789012345678901 2
12345678901234567890123456789012123456789012345678901 2
12345678901234567890123456789012123456789012345678901 2
12345678901234567890123456789012123456789012345678901 2
12345678901234567890123456789012123456789012345678901 2

Alert!
12345678901234567890123456789012123456789012345678901 2
12345678901234567890123456789012123456789012345678901
If you didn’t know that the y had to be positive since it is a distance, you 2
12345678901234567890123456789012123456789012345678901 2
12345678901234567890123456789012123456789012345678901
would not know which column was greater. 2
12345678901234567890123456789012123456789012345678901 2
12345678901234567890123456789012123456789012345678901 2
12345678901234567890123456789012123456789012345678901
2345678901234567890123456789012123456789012345678901
2
2
1
12345678901234567890123456789012123456789012345678901 2
12345678901234567890123456789012123456789012345678901 2
12345678901234567890123456789012123456789012345678901
A tangent to a circle is a line that has one point in common with a circle. 2
12345678901234567890123456789012123456789012345678901 2
12345678901234567890123456789012123456789012345678901
The figure below is an illustration of a circle and one tangent line. Notice 2
12345678901234567890123456789012123456789012345678901
that the tangent line forms a right angle with the radius that shares a point 2
12345678901234567890123456789012123456789012345678901 2
12345678901234567890123456789012123456789012345678901 2
12345678901234567890123456789012123456789012345678901
with it. Tuck that fact away in your head
2345678901234567890123456789012123456789012345678901
because you might need it later. 2
2
1
12345678901234567890123456789012123456789012345678901 2
12345678901234567890123456789012123456789012345678901 2
12345678901234567890123456789012123456789012345678901 2
12345678901234567890123456789012123456789012345678901 2
12345678901234567890123456789012123456789012345678901 2
2345678901234567890123456789012123456789012345678901
12345678901234567890123456789012123456789012345678901 2
12345678901234567890123456789012123456789012345678901 2
Another word that might come up with circles or other shapes is inscribed. 2
12345678901234567890123456789012123456789012345678901
12345678901234567890123456789012123456789012345678901 2
12345678901234567890123456789012123456789012345678901 2
1 A polygon is inscribed (in-“scribed,” drawn in) in a circle if each of its 2
12345678901234567890123456789012123456789012345678901
vertices lies on the circle. A circle can also be inscribed in a polygon if each 2
12345678901234567890123456789012123456789012345678901 2
12345678901234567890123456789012123456789012345678901
of the polygon’s sides is tangent to the circle. Circumscribed is related to 2
12345678901234567890123456789012123456789012345678901 2
2345678901234567890123456789012123456789012345678901
12345678901234567890123456789012123456789012345678901
inscribed in an antonym-like way. 2
12345678901234567890123456789012123456789012345678901 2
12345678901234567890123456789012123456789012345678901 2
12345678901234567890123456789012123456789012345678901 2
12345678901234567890123456789012123456789012345678901 2
12345678901234567890123456789012123456789012345678901 2
12345678901234567890123456789012123456789012345678901 2
12345678901234567890123456789012123456789012345678901 2 2
1
12345678901234567890123456789012123456789012345678901 2
12345678901234567890123456789012123456789012345678901 2
12345678901234567890123456789012123456789012345678901 2
12345678901234567890123456789012123456789012345678901 2
12345678901234567890123456789012123456789012345678901 2
12345678901234567890123456789012123456789012345678901
Notice that when figures are inscribed and circumscribed, they share 2
12345678901234567890123456789012123456789012345678901 2
12345678901234567890123456789012123456789012345678901 2
12345678901234567890123456789012123456789012345678901
certain features. With the circle inscribed above in the square, the diameter 2
12345678901234567890123456789012123456789012345678901 2
12345678901234567890123456789012123456789012345678901 2
2345678901234567890123456789012123456789012345678901
of the circle is also the length of one side of the square. These sort of
12345678901234567890123456789012123456789012345678901 2
1 overlapping features are what the GRE focuses on when there are multiple 2
12345678901234567890123456789012123456789012345678901
figures involved in a problem. Be aware of what features figures share 2
12345678901234567890123456789012123456789012345678901 2
2345678901234567890123456789012123456789012345678901
12345678901234567890123456789012123456789012345678901
because that often is the clue to the answer. 2
12345678901234567890123456789012123456789012345678901 2
12345678901234567890123456789012123456789012345678901 2
12345678901234567890123456789012123456789012345678901
Column A Column B 2
1 2
12345678901234567890123456789012123456789012345678901 2
2345678901234567890123456789012123456789012345678901
12345678901234567890123456789012123456789012345678901 2
12345678901234567890123456789012123456789012345678901 2
12345678901234567890123456789012123456789012345678901 2 2
1
12345678901234567890123456789012123456789012345678901 2
12345678901234567890123456789012123456789012345678901 2
12345678901234567890123456789012123456789012345678901 2
12345678901234567890123456789012123456789012345678901
The area of the above circle is 16p. 2
12345678901234567890123456789012123456789012345678901 2
2345678901234567890123456789012123456789012345678901
12345678901234567890123456789012123456789012345678901 2
1 The area of the 6l 2
12345678901234567890123456789012123456789012345678901 2
12345678901234567890123456789012123456789012345678901 2
rectangle
12345678901234567890123456789012123456789012345678901 2
12345678901234567890123456789012123456789012345678901 2
12345678901234567890123456789012123456789012345678901 2
12345678901234567890123456789012123456789012345678901 2
12345678901234567890123456789012123456789012345678901 2
96 123456789012345678901234567890121234567890123456789012

www.petersons.com
Chapter 4: Quantitative Review

123456789012345678901234567890121234567890123456789012
12345678901234567890123456789012123456789012345678901 2
12345678901234567890123456789012123456789012345678901 2
12345678901234567890123456789012123456789012345678901
Look at the features the two figures share. Notice that the diameter of the 2
12345678901234567890123456789012123456789012345678901 2
12345678901234567890123456789012123456789012345678901
circle is the width of the rectangle. If you can find the diameter, you can 2
12345678901234567890123456789012123456789012345678901 2
12345678901234567890123456789012123456789012345678901
compare the two columns:
2345678901234567890123456789012123456789012345678901
2
2
1
12345678901234567890123456789012123456789012345678901 2
12345678901234567890123456789012123456789012345678901 2
2
A 5 pr
12345678901234567890123456789012123456789012345678901 2
12345678901234567890123456789012123456789012345678901
16p 5 pr 2
2345678901234567890123456789012123456789012345678901
2
2
1
12345678901234567890123456789012123456789012345678901
16 5 r 2 2
12345678901234567890123456789012123456789012345678901 2
12345678901234567890123456789012123456789012345678901 2
12345678901234567890123456789012123456789012345678901
r 5 4
2345678901234567890123456789012123456789012345678901
2
2
1
12345678901234567890123456789012123456789012345678901
If the radius is 4, then the diameter is twice that, so d 5 8. The area of the 2
12345678901234567890123456789012123456789012345678901 2
12345678901234567890123456789012123456789012345678901
rectangle is 8l, which means that (A) is greater. 2
12345678901234567890123456789012123456789012345678901 2
1 One last point: Concentric circles have the same center but do not have the 2
2345678901234567890123456789012123456789012345678901
12345678901234567890123456789012123456789012345678901 2
12345678901234567890123456789012123456789012345678901 2
12345678901234567890123456789012123456789012345678901
same radius. 2
12345678901234567890123456789012123456789012345678901 2
12345678901234567890123456789012123456789012345678901 22
2345678901234567890123456789012123456789012345678901
12345678901234567890123456789012123456789012345678901 2
12345678901234567890123456789012123456789012345678901 2
12345678901234567890123456789012123456789012345678901 2
12345678901234567890123456789012123456789012345678901 2
12345678901234567890123456789012123456789012345678901 2
1
12345678901234567890123456789012123456789012345678901 2
12345678901234567890123456789012123456789012345678901 2
12345678901234567890123456789012123456789012345678901 2
12345678901234567890123456789012123456789012345678901 22
2345678901234567890123456789012123456789012345678901
12345678901234567890123456789012123456789012345678901 2
12345678901234567890123456789012123456789012345678901 2
12345678901234567890123456789012123456789012345678901 2
12345678901234567890123456789012123456789012345678901 2
12345678901234567890123456789012123456789012345678901 2
12345678901234567890123456789012123456789012345678901 2
12345678901234567890123456789012123456789012345678901 2
12345678901234567890123456789012123456789012345678901 2
1Coordinate Geometry
12345678901234567890123456789012123456789012345678901
The above phrase refers to geometry done on a coordinate plane. When 2
12345678901234567890123456789012123456789012345678901 2
12345678901234567890123456789012123456789012345678901 2
12345678901234567890123456789012123456789012345678901 2
you see the following visual, you’ll either remember coordinate geometry
12345678901234567890123456789012123456789012345678901 2
12345678901234567890123456789012123456789012345678901
or playing the game Battleship! as a child. “B . . . 4 . . . Miss!” 2
12345678901234567890123456789012123456789012345678901 2
12345678901234567890123456789012123456789012345678901 2
12345678901234567890123456789012123456789012345678901 2
12345678901234567890123456789012123456789012345678901 2
12345678901234567890123456789012123456789012345678901 22
2345678901234567890123456789012123456789012345678901
12345678901234567890123456789012123456789012345678901 2
1
12345678901234567890123456789012123456789012345678901 2
12345678901234567890123456789012123456789012345678901 2
12345678901234567890123456789012123456789012345678901 2
12345678901234567890123456789012123456789012345678901 2
12345678901234567890123456789012123456789012345678901 2
12345678901234567890123456789012123456789012345678901 2
12345678901234567890123456789012123456789012345678901 2
12345678901234567890123456789012123456789012345678901 22
2345678901234567890123456789012123456789012345678901
12345678901234567890123456789012123456789012345678901 2
12345678901234567890123456789012123456789012345678901 2
12345678901234567890123456789012123456789012345678901 2
1
12345678901234567890123456789012123456789012345678901 2
12345678901234567890123456789012123456789012345678901 2
12345678901234567890123456789012123456789012345678901 2
12345678901234567890123456789012123456789012345678901 2
12345678901234567890123456789012123456789012345678901 2
12345678901234567890123456789012123456789012345678901 22
2345678901234567890123456789012123456789012345678901
1 A coordinate plane is just two number lines perpendicular to each other.
12345678901234567890123456789012123456789012345678901 2
12345678901234567890123456789012123456789012345678901 2
12345678901234567890123456789012123456789012345678901
The one going up and down is called the y-axis, and the one going left and 2
1 right is called the x-axis. Any point on the plane gets two coordinates, an 2
2345678901234567890123456789012123456789012345678901
12345678901234567890123456789012123456789012345678901 2
12345678901234567890123456789012123456789012345678901 2
12345678901234567890123456789012123456789012345678901 2 97
123456789012345678901234567890121234567890123456789012
Part III: Section Review

123456789012345678901234567890121234567890123456789012
12345678901234567890123456789012123456789012345678901 2
12345678901234567890123456789012123456789012345678901 2
12345678901234567890123456789012123456789012345678901
x one and a y one, and they tell you where the point goes on the plane. The 2
12345678901234567890123456789012123456789012345678901 2
12345678901234567890123456789012123456789012345678901
notation for this looks like this (x,y), with the x-coordinate always first. 2
12345678901234567890123456789012123456789012345678901 2
12345678901234567890123456789012123456789012345678901
The number in the x spot tells you where along the x-axis the point goes, 2
12345678901234567890123456789012123456789012345678901 2
12345678901234567890123456789012123456789012345678901 2
12345678901234567890123456789012123456789012345678901
and the number in the y spot tells you where along the y-axis the point 2
12345678901234567890123456789012123456789012345678901 2
12345678901234567890123456789012123456789012345678901
goes. The points (24,2) and (0,3) would
2345678901234567890123456789012123456789012345678901
be here: 2
2
1
12345678901234567890123456789012123456789012345678901 2
12345678901234567890123456789012123456789012345678901 2
12345678901234567890123456789012123456789012345678901 2
12345678901234567890123456789012123456789012345678901
2345678901234567890123456789012123456789012345678901
2
2
1
12345678901234567890123456789012123456789012345678901 2
12345678901234567890123456789012123456789012345678901 2
12345678901234567890123456789012123456789012345678901 2
12345678901234567890123456789012123456789012345678901
2345678901234567890123456789012123456789012345678901
2
2
1
12345678901234567890123456789012123456789012345678901 2
12345678901234567890123456789012123456789012345678901 2
12345678901234567890123456789012123456789012345678901 2
12345678901234567890123456789012123456789012345678901 2
12345678901234567890123456789012123456789012345678901 2
2345678901234567890123456789012123456789012345678901
12345678901234567890123456789012123456789012345678901 2
12345678901234567890123456789012123456789012345678901 2
12345678901234567890123456789012123456789012345678901 2
12345678901234567890123456789012123456789012345678901 2
12345678901234567890123456789012123456789012345678901 2
1 Say that a line was drawn between those two points. Would the slope be 2
12345678901234567890123456789012123456789012345678901
steep or flat? Looking at it, the slope would rise if you were going from left 2
12345678901234567890123456789012123456789012345678901 2
12345678901234567890123456789012123456789012345678901 2
12345678901234567890123456789012123456789012345678901 2
2345678901234567890123456789012123456789012345678901
to right, and that is the direction that geometrists are always going on
coordinate planes. Yet you can say more than “the slope is going up.” You 2
12345678901234567890123456789012123456789012345678901
12345678901234567890123456789012123456789012345678901 2
12345678901234567890123456789012123456789012345678901 2
12345678901234567890123456789012123456789012345678901 2
can give it a value since there is a mathematical definition of slope. It is the
change in the y values over the change in the x values, also called the rise 2
12345678901234567890123456789012123456789012345678901
12345678901234567890123456789012123456789012345678901 2
12345678901234567890123456789012123456789012345678901 2
12345678901234567890123456789012123456789012345678901 2
over the run.
2
1
12345678901234567890123456789012123456789012345678901
In this case, the slope is 2
12345678901234567890123456789012123456789012345678901 2
12345678901234567890123456789012123456789012345678901 2
12345678901234567890123456789012123456789012345678901 2
12345678901234567890123456789012123456789012345678901
Dy y1 2 y2 322 1
2345678901234567890123456789012123456789012345678901
2
2
12345678901234567890123456789012123456789012345678901
5 5
Dx x1 2 x2 0 2 ~24! 4
5
12345678901234567890123456789012123456789012345678901 2
1 2
12345678901234567890123456789012123456789012345678901
Always subtract the coordinate values of the point on the right from the 2
12345678901234567890123456789012123456789012345678901 2
12345678901234567890123456789012123456789012345678901
coordinate values from the point on the left. The slope is positive 2
12345678901234567890123456789012123456789012345678901 2
12345678901234567890123456789012123456789012345678901
one fourth, which means that the line goes up one in the y-direction for 2
12345678901234567890123456789012123456789012345678901 2
12345678901234567890123456789012123456789012345678901
every four it goes over in the x-direction (again, if you are traveling left to 2
2345678901234567890123456789012123456789012345678901
12345678901234567890123456789012123456789012345678901
right). A negative slope means that the line goes down from left to right 2
2
12345678901234567890123456789012123456789012345678901 2
12345678901234567890123456789012123456789012345678901 2
12345678901234567890123456789012123456789012345678901
instead of up.
2
1
2345678901234567890123456789012123456789012345678901
The geometry part of coordinate geometry comes in when geometrical 2
12345678901234567890123456789012123456789012345678901
12345678901234567890123456789012123456789012345678901 2
12345678901234567890123456789012123456789012345678901 2
12345678901234567890123456789012123456789012345678901 2
figures are put on the coordinate plane. In one way, this makes the
1 geometry easier because you have the coordinate plane to help do the 2
12345678901234567890123456789012123456789012345678901 2
12345678901234567890123456789012123456789012345678901 2
12345678901234567890123456789012123456789012345678901
geometry. In another way, though, the test will up the complexity of a 2
12345678901234567890123456789012123456789012345678901 2
12345678901234567890123456789012123456789012345678901 2
problem by expecting you to make inferences about the geometrical
2345678901234567890123456789012123456789012345678901
12345678901234567890123456789012123456789012345678901 2
1 figures based on your knowledge of coordinate geometry. 2
12345678901234567890123456789012123456789012345678901 2
12345678901234567890123456789012123456789012345678901 2
12345678901234567890123456789012123456789012345678901 2
12345678901234567890123456789012123456789012345678901 2
12345678901234567890123456789012123456789012345678901 2
12345678901234567890123456789012123456789012345678901 2
12345678901234567890123456789012123456789012345678901 2
98 123456789012345678901234567890121234567890123456789012

www.petersons.com
Chapter 4: Quantitative Review

123456789012345678901234567890121234567890123456789012
12345678901234567890123456789012123456789012345678901 2
12345678901234567890123456789012123456789012345678901 2
12345678901234567890123456789012123456789012345678901
Column A Column B 2
12345678901234567890123456789012123456789012345678901 2
12345678901234567890123456789012123456789012345678901 2
12345678901234567890123456789012123456789012345678901
A figure is defined by these points (1,1), 2
12345678901234567890123456789012123456789012345678901
2345678901234567890123456789012123456789012345678901
2
2
1 (3,1), and (1,22) on a coordinate plane.
12345678901234567890123456789012123456789012345678901 2
12345678901234567890123456789012123456789012345678901 2
12345678901234567890123456789012123456789012345678901
The area of the The perimeter of 2
12345678901234567890123456789012123456789012345678901
2345678901234567890123456789012123456789012345678901
2
2
1 figure the figure
12345678901234567890123456789012123456789012345678901 2
12345678901234567890123456789012123456789012345678901 2
12345678901234567890123456789012123456789012345678901
If this question mystifies you, or you are short on time, realize that the 2
12345678901234567890123456789012123456789012345678901 2
12345678901234567890123456789012123456789012345678901
perimeter of a figure and the area of a figure will have a definite 2
12345678901234567890123456789012123456789012345678901 2
12345678901234567890123456789012123456789012345678901
relationship, and so the answer is probably not (D). That makes guessing a 2
12345678901234567890123456789012123456789012345678901 2
12345678901234567890123456789012123456789012345678901
one in three chance. If you have the time to do the problem, you should 2
1 suspect that this figure is a triangle since it is defined by three points. 2
2345678901234567890123456789012123456789012345678901
12345678901234567890123456789012123456789012345678901 2
12345678901234567890123456789012123456789012345678901 2
12345678901234567890123456789012123456789012345678901
Regardless of what you suspect, though, the way to attack this problem is 2
12345678901234567890123456789012123456789012345678901 2
12345678901234567890123456789012123456789012345678901
to quickly plot the points on a coordinate plane 2
12345678901234567890123456789012123456789012345678901 2
12345678901234567890123456789012123456789012345678901 2
12345678901234567890123456789012123456789012345678901 2
12345678901234567890123456789012123456789012345678901 2
12345678901234567890123456789012123456789012345678901 22
2345678901234567890123456789012123456789012345678901
1
12345678901234567890123456789012123456789012345678901 2
12345678901234567890123456789012123456789012345678901 2
12345678901234567890123456789012123456789012345678901 2
12345678901234567890123456789012123456789012345678901 22
2345678901234567890123456789012123456789012345678901
12345678901234567890123456789012123456789012345678901 2
12345678901234567890123456789012123456789012345678901 2
12345678901234567890123456789012123456789012345678901 2
12345678901234567890123456789012123456789012345678901 2
12345678901234567890123456789012123456789012345678901 2
12345678901234567890123456789012123456789012345678901 2
12345678901234567890123456789012123456789012345678901 2
12345678901234567890123456789012123456789012345678901 2
1 It is a right triangle, and you can use this fact to find both the area and the
12345678901234567890123456789012123456789012345678901 2
12345678901234567890123456789012123456789012345678901 2
12345678901234567890123456789012123456789012345678901
perimeter. Let’s do the area first. The distance between (1,1) and (1,22) is 2
12345678901234567890123456789012123456789012345678901 2
12345678901234567890123456789012123456789012345678901
three units, so the height is three. The distance between (1,1) and (3,1) is 2
2345678901234567890123456789012123456789012345678901 2
12345678901234567890123456789012123456789012345678901
12345678901234567890123456789012123456789012345678901
two units, making it the base. 2
1 2
12345678901234567890123456789012123456789012345678901 2
12345678901234567890123456789012123456789012345678901
1 2
2345678901234567890123456789012123456789012345678901
12345678901234567890123456789012123456789012345678901
A 5 bh 2
12345678901234567890123456789012123456789012345678901
2 2
1 2
12345678901234567890123456789012123456789012345678901
1 2
12345678901234567890123456789012123456789012345678901
~ !~ ! 2
2345678901234567890123456789012123456789012345678901
12345678901234567890123456789012123456789012345678901
A 5 2 3 5 3 2
12345678901234567890123456789012123456789012345678901
2 2
12345678901234567890123456789012123456789012345678901 22
12345678901234567890123456789012123456789012345678901
1 The perimeter is the sum of the sides. You have 2
12345678901234567890123456789012123456789012345678901 2
12345678901234567890123456789012123456789012345678901
Perimeter 2 3 hypotenuse 2
12345678901234567890123456789012123456789012345678901
5 1 1 2
12345678901234567890123456789012123456789012345678901
1 2
12345678901234567890123456789012123456789012345678901
~ 2 3
! 2
12345678901234567890123456789012123456789012345678901 22
2345678901234567890123456789012123456789012345678901
Perimeter 5 5 1 2 1 3 2
12345678901234567890123456789012123456789012345678901 2
12345678901234567890123456789012123456789012345678901
The Pythagorean theorem was used for the hypotenuse. At this point, you
2
12345678901234567890123456789012123456789012345678901
1 don’t have to finish the problem since this sum is clearly greater than 3. 2
2345678901234567890123456789012123456789012345678901
12345678901234567890123456789012123456789012345678901 2
1 Therefore, choice (B) is the correct answer. 2
12345678901234567890123456789012123456789012345678901 2
12345678901234567890123456789012123456789012345678901 2
12345678901234567890123456789012123456789012345678901 2
12345678901234567890123456789012123456789012345678901 2
12345678901234567890123456789012123456789012345678901 2
12345678901234567890123456789012123456789012345678901 2
12345678901234567890123456789012123456789012345678901 2 99
123456789012345678901234567890121234567890123456789012
Part III: Section Review

123456789012345678901234567890121234567890123456789012
12345678901234567890123456789012123456789012345678901 2
12345678901234567890123456789012123456789012345678901 2
12345678901234567890123456789012123456789012345678901
Data Analysis 2
12345678901234567890123456789012123456789012345678901 2
12345678901234567890123456789012123456789012345678901 2
12345678901234567890123456789012123456789012345678901 2
If you read USA Today on a regular basis, you’ll have experience with data
12345678901234567890123456789012123456789012345678901
analysis. The “data” that you are expected to analyze usually comes in the 2
12345678901234567890123456789012123456789012345678901 2
12345678901234567890123456789012123456789012345678901 2
12345678901234567890123456789012123456789012345678901
form of chart, graph, or table, three things that national newspapers love 2
12345678901234567890123456789012123456789012345678901 2
12345678901234567890123456789012123456789012345678901
to print. It’s visual information that needs
2345678901234567890123456789012123456789012345678901
to be understood and then used 2
2
1 in some manner.
12345678901234567890123456789012123456789012345678901 2
12345678901234567890123456789012123456789012345678901 2
12345678901234567890123456789012123456789012345678901
Before jumping into all the different graphs, there is one other topic often 2
12345678901234567890123456789012123456789012345678901 2
12345678901234567890123456789012123456789012345678901
placed under the Data Analysis umbrella. What are the odds you can guess 2
12345678901234567890123456789012123456789012345678901 2
12345678901234567890123456789012123456789012345678901
what it is? 2
12345678901234567890123456789012123456789012345678901 2
12345678901234567890123456789012123456789012345678901
2345678901234567890123456789012123456789012345678901
2
2
1
12345678901234567890123456789012123456789012345678901 2
12345678901234567890123456789012123456789012345678901 2
12345678901234567890123456789012123456789012345678901
Probability 2
12345678901234567890123456789012123456789012345678901 2
12345678901234567890123456789012123456789012345678901 2
2345678901234567890123456789012123456789012345678901
If you flip a coin, what is the chance that it will land on heads? Pretty much
everyone knows the answer is “one out of two” or “fifty-fifty.” This is 2
12345678901234567890123456789012123456789012345678901
12345678901234567890123456789012123456789012345678901 2
12345678901234567890123456789012123456789012345678901 2
12345678901234567890123456789012123456789012345678901 2
because there are two possible outcomes that are equally probable
12345678901234567890123456789012123456789012345678901 2
1 (landing heads, landing tails), so the chance the coin will land on heads is 2
12345678901234567890123456789012123456789012345678901 2
12345678901234567890123456789012123456789012345678901
one out of two. 2
12345678901234567890123456789012123456789012345678901 2
2345678901234567890123456789012123456789012345678901
That’s kid stuff, but you just covered the basics of probability theory. In 2
12345678901234567890123456789012123456789012345678901
12345678901234567890123456789012123456789012345678901 2
any probability scenario, the probability of A occurring is the number of 2
12345678901234567890123456789012123456789012345678901
12345678901234567890123456789012123456789012345678901 2
ways A can occur divided by the total number of possible outcomes. In our 2
12345678901234567890123456789012123456789012345678901
12345678901234567890123456789012123456789012345678901 2
case, A is the coin landing on heads, and that can only happen one way. 2
12345678901234567890123456789012123456789012345678901
12345678901234567890123456789012123456789012345678901 2
12345678901234567890123456789012123456789012345678901 1 2
1 There are two possible outcomes, heads or tails, so the probability is . 2
2345678901234567890123456789012123456789012345678901
12345678901234567890123456789012123456789012345678901 2 2
12345678901234567890123456789012123456789012345678901 2 2
1
12345678901234567890123456789012123456789012345678901
number of desired outcomes 2
12345678901234567890123456789012123456789012345678901
probability 5 2
12345678901234567890123456789012123456789012345678901 2
2345678901234567890123456789012123456789012345678901
total number of possible outcomes
12345678901234567890123456789012123456789012345678901 2
1 Things get a little more interesting when you start talking about multiple 2
12345678901234567890123456789012123456789012345678901 2
12345678901234567890123456789012123456789012345678901 2
12345678901234567890123456789012123456789012345678901 2
2345678901234567890123456789012123456789012345678901
event probability, but that’s nothing you can’t handle if you followed
12345678901234567890123456789012123456789012345678901 2
1 things thus far. Say you flip the coin again. What are the chances that it will 2
12345678901234567890123456789012123456789012345678901 2
12345678901234567890123456789012123456789012345678901
1 2
2345678901234567890123456789012123456789012345678901
12345678901234567890123456789012123456789012345678901
land on heads this time? It’s still . If you thought otherwise, remember 2
12345678901234567890123456789012123456789012345678901
2
that each coin toss is independent and unaffected by previous coin tosses. 2
2
12345678901234567890123456789012123456789012345678901
12345678901234567890123456789012123456789012345678901 2
1 Moving up one more notch, what is the probability of tossing the coin twice 2
12345678901234567890123456789012123456789012345678901 2
12345678901234567890123456789012123456789012345678901
and it landing on heads twice? It is the probability of the first event times 2
12345678901234567890123456789012123456789012345678901 2
12345678901234567890123456789012123456789012345678901 2
12345678901234567890123456789012123456789012345678901
2345678901234567890123456789012123456789012345678901
12345678901234567890123456789012123456789012345678901
the probability of the second event. Therefore,
12345678901234567890123456789012123456789012345678901
probability S DS D
5
1
2 2
1
5
1
4
. 2
2
2
That’s the basics of multiple-event probability. The probability of multiple 2
12345678901234567890123456789012123456789012345678901
12345678901234567890123456789012123456789012345678901 2 2
1 events occurring is the product of the individual probabilities involved.
12345678901234567890123456789012123456789012345678901 2
12345678901234567890123456789012123456789012345678901 2
12345678901234567890123456789012123456789012345678901 2
12345678901234567890123456789012123456789012345678901 2
12345678901234567890123456789012123456789012345678901 2
12345678901234567890123456789012123456789012345678901 2
12345678901234567890123456789012123456789012345678901 2
12345678901234567890123456789012123456789012345678901 2 2
100 12345678901234567890123456789012123456789012345678901
123456789012345678901234567890121234567890123456789012

www.petersons.com
Chapter 4: Quantitative Review

123456789012345678901234567890121234567890123456789012
12345678901234567890123456789012123456789012345678901 2
12345678901234567890123456789012123456789012345678901 2
12345678901234567890123456789012123456789012345678901
There are 5 boys and 7 girls in the fifth grade class at Saddleback 2
12345678901234567890123456789012123456789012345678901 2
12345678901234567890123456789012123456789012345678901
Elementary. What is the probability that if 2 students are chosen at 2
12345678901234567890123456789012123456789012345678901 2
12345678901234567890123456789012123456789012345678901
random from the class, that both students
2345678901234567890123456789012123456789012345678901
will be girls? 2
2
1
12345678901234567890123456789012123456789012345678901 2
12345678901234567890123456789012123456789012345678901
7 2
12345678901234567890123456789012123456789012345678901
A. 2
12345678901234567890123456789012123456789012345678901
24
2345678901234567890123456789012123456789012345678901
2
2
1
12345678901234567890123456789012123456789012345678901
7 2
12345678901234567890123456789012123456789012345678901
B. 2
12345678901234567890123456789012123456789012345678901
22 2
12345678901234567890123456789012123456789012345678901
2345678901234567890123456789012123456789012345678901
2
2
1
12345678901234567890123456789012123456789012345678901
1 2
12345678901234567890123456789012123456789012345678901
C. 2
12345678901234567890123456789012123456789012345678901
2 2
12345678901234567890123456789012123456789012345678901
2345678901234567890123456789012123456789012345678901
2
2
1 7
12345678901234567890123456789012123456789012345678901
D. 2
12345678901234567890123456789012123456789012345678901
12 2
12345678901234567890123456789012123456789012345678901 2
12345678901234567890123456789012123456789012345678901
11 2
2345678901234567890123456789012123456789012345678901
12345678901234567890123456789012123456789012345678901 2
12345678901234567890123456789012123456789012345678901
E. 2
12345678901234567890123456789012123456789012345678901 22
12
12345678901234567890123456789012123456789012345678901
This probability problem involves two events, and you first need to 2
12345678901234567890123456789012123456789012345678901
12345678901234567890123456789012123456789012345678901 2
1 determine the probability of each event. A girl getting picked first has a 2
12345678901234567890123456789012123456789012345678901 2
12345678901234567890123456789012123456789012345678901
7 2
12345678901234567890123456789012123456789012345678901
probability of since there are 7 girls and a total of 12 students in the 2
12345678901234567890123456789012123456789012345678901 22
2345678901234567890123456789012123456789012345678901
12
12345678901234567890123456789012123456789012345678901 6
class. The probability of a girl getting picked the second time is . Why? 2
12345678901234567890123456789012123456789012345678901
12345678901234567890123456789012123456789012345678901 11 2
12345678901234567890123456789012123456789012345678901 2
12345678901234567890123456789012123456789012345678901 22
Because one girl has been picked already, leaving only 6 girls left in the
12345678901234567890123456789012123456789012345678901
class of 11 students. Multiplying the two probabilities together gives you: 2
12345678901234567890123456789012123456789012345678901
12345678901234567890123456789012123456789012345678901 22
1
12345678901234567890123456789012123456789012345678901
7 6 2
12345678901234567890123456789012123456789012345678901
3 5 2
12345678901234567890123456789012123456789012345678901
12 11 2
12345678901234567890123456789012123456789012345678901 2
12345678901234567890123456789012123456789012345678901
~7 3 6! 5 2
12345678901234567890123456789012123456789012345678901 2
12345678901234567890123456789012123456789012345678901 2
12345678901234567890123456789012123456789012345678901
~ 12 3 11! 2
12345678901234567890123456789012123456789012345678901 2
12345678901234567890123456789012123456789012345678901 2
2345678901234567890123456789012123456789012345678901
12345678901234567890123456789012123456789012345678901
7 2
12345678901234567890123456789012123456789012345678901
5 2
1 22 2
12345678901234567890123456789012123456789012345678901 2
12345678901234567890123456789012123456789012345678901
That’s choice (B). 2
12345678901234567890123456789012123456789012345678901 2
12345678901234567890123456789012123456789012345678901 2
12345678901234567890123456789012123456789012345678901 2
12345678901234567890123456789012123456789012345678901 2
12345678901234567890123456789012123456789012345678901 2
12345678901234567890123456789012123456789012345678901 22
2345678901234567890123456789012123456789012345678901
12345678901234567890123456789012123456789012345678901 2
12345678901234567890123456789012123456789012345678901 2
12345678901234567890123456789012123456789012345678901 2
1
12345678901234567890123456789012123456789012345678901 2
12345678901234567890123456789012123456789012345678901 2
12345678901234567890123456789012123456789012345678901 2
12345678901234567890123456789012123456789012345678901 2
12345678901234567890123456789012123456789012345678901 2
12345678901234567890123456789012123456789012345678901 22
2345678901234567890123456789012123456789012345678901
1
12345678901234567890123456789012123456789012345678901 2
12345678901234567890123456789012123456789012345678901 2
12345678901234567890123456789012123456789012345678901 2
12345678901234567890123456789012123456789012345678901 2
12345678901234567890123456789012123456789012345678901 2
12345678901234567890123456789012123456789012345678901 2
12345678901234567890123456789012123456789012345678901 2 101
123456789012345678901234567890121234567890123456789012
Part III: Section Review

123456789012345678901234567890121234567890123456789012
12345678901234567890123456789012123456789012345678901 2
12345678901234567890123456789012123456789012345678901 2
12345678901234567890123456789012123456789012345678901
Workin’ the Data 2
12345678901234567890123456789012123456789012345678901 2
12345678901234567890123456789012123456789012345678901 2
12345678901234567890123456789012123456789012345678901 2
Data or information can come to you in lots of different forms. You are
12345678901234567890123456789012123456789012345678901
getting text information right now, for instance, as you read this sentence. 2
12345678901234567890123456789012123456789012345678901 2
12345678901234567890123456789012123456789012345678901
Earlier you might have looked at the table of contents of this book to get 2
12345678901234567890123456789012123456789012345678901 2
12345678901234567890123456789012123456789012345678901
some information about where to start reading. The GRE likes to test your 2
12345678901234567890123456789012123456789012345678901 2
12345678901234567890123456789012123456789012345678901
ability to take in information and answer questions about that informa- 2
12345678901234567890123456789012123456789012345678901 2
12345678901234567890123456789012123456789012345678901
tion. Specifically, the GRE math section will present information or data in 2
12345678901234567890123456789012123456789012345678901 2
12345678901234567890123456789012123456789012345678901
the form of a line graph, a circle graph, a bar graph, a table, or some 2
1 combination of all of these, and then expect you to answer questions about 2
2345678901234567890123456789012123456789012345678901
12345678901234567890123456789012123456789012345678901 2
12345678901234567890123456789012123456789012345678901 2
12345678901234567890123456789012123456789012345678901
the information presented. You might think this sounds too easy, and in a 2
12345678901234567890123456789012123456789012345678901 2
12345678901234567890123456789012123456789012345678901
sense it is. All the answers will be right in front of you in a graph or table. 2
12345678901234567890123456789012123456789012345678901
The hard part is that the GRE is pretty creative in coming up with 2
12345678901234567890123456789012123456789012345678901 2
12345678901234567890123456789012123456789012345678901 2
12345678901234567890123456789012123456789012345678901
complicated tables and charts and graphs, which makes understanding 2
2345678901234567890123456789012123456789012345678901
12345678901234567890123456789012123456789012345678901 2
12345678901234567890123456789012123456789012345678901
and using them a challenge. 2
12345678901234567890123456789012123456789012345678901 2
12345678901234567890123456789012123456789012345678901 2
12345678901234567890123456789012123456789012345678901
Usually you will see one or two sets of graphs and tables on the GRE math 2
2
12345678901234567890123456789012123456789012345678901

Note
1 section. Each set will most likely have multiple parts, like a circle graph 2
12345678901234567890123456789012123456789012345678901 2
12345678901234567890123456789012123456789012345678901 2
12345678901234567890123456789012123456789012345678901
combined with a table or a diagram coupled with a bar graph. Each set 2
2345678901234567890123456789012123456789012345678901
12345678901234567890123456789012123456789012345678901 2
12345678901234567890123456789012123456789012345678901
will have 2–4 questions that refer to it. 2
12345678901234567890123456789012123456789012345678901 2
12345678901234567890123456789012123456789012345678901 2
12345678901234567890123456789012123456789012345678901
The first thing to do when you come upon a set of graphs and charts is read 2
2
12345678901234567890123456789012123456789012345678901
the title. It sounds simplistic, but do it. The simplest steps can make the 2
12345678901234567890123456789012123456789012345678901
12345678901234567890123456789012123456789012345678901 2
12345678901234567890123456789012123456789012345678901 2
1 great difference, so: 2
12345678901234567890123456789012123456789012345678901 2
2345678901234567890123456789012123456789012345678901
12345678901234567890123456789012123456789012345678901 2
1 1. Read the title. 2
12345678901234567890123456789012123456789012345678901 2
12345678901234567890123456789012123456789012345678901
2. Read whatever key there is. 2
12345678901234567890123456789012123456789012345678901 2
12345678901234567890123456789012123456789012345678901 2
12345678901234567890123456789012123456789012345678901
3. Read down any graph axis to see what information is being 2
12345678901234567890123456789012123456789012345678901 2
12345678901234567890123456789012123456789012345678901
presented. 2
12345678901234567890123456789012123456789012345678901 2
2345678901234567890123456789012123456789012345678901
1 4. If there are two graphs or charts, look to see how they relate to 2
12345678901234567890123456789012123456789012345678901 2
12345678901234567890123456789012123456789012345678901 2
12345678901234567890123456789012123456789012345678901 2
each other.
12345678901234567890123456789012123456789012345678901 2
2345678901234567890123456789012123456789012345678901
Basically, spend 20 to 30 seconds getting familiar with the layout of 2
12345678901234567890123456789012123456789012345678901
12345678901234567890123456789012123456789012345678901 2
12345678901234567890123456789012123456789012345678901 2
1 everything, then proceed to the first question. This will not be a waste of 2
2345678901234567890123456789012123456789012345678901
time, as you will be able to answer each of the following questions more 2
12345678901234567890123456789012123456789012345678901
12345678901234567890123456789012123456789012345678901 2
quickly. These kinds of questions try to trap you with an overload of 2
12345678901234567890123456789012123456789012345678901
12345678901234567890123456789012123456789012345678901 2
1 information you have to wade through to get to the answer. If you don’t 2
12345678901234567890123456789012123456789012345678901 2
12345678901234567890123456789012123456789012345678901
have a good sense of what you are wading through, you will probably get 2
12345678901234567890123456789012123456789012345678901 2
12345678901234567890123456789012123456789012345678901
bogged down and frustrated. That is why getting a sense of the 2
12345678901234567890123456789012123456789012345678901 2
12345678901234567890123456789012123456789012345678901
information being presented is task one. 2
12345678901234567890123456789012123456789012345678901 2
12345678901234567890123456789012123456789012345678901 2
12345678901234567890123456789012123456789012345678901 2
12345678901234567890123456789012123456789012345678901 2
12345678901234567890123456789012123456789012345678901 2
12345678901234567890123456789012123456789012345678901 2
12345678901234567890123456789012123456789012345678901 2 2
102 12345678901234567890123456789012123456789012345678901
123456789012345678901234567890121234567890123456789012

www.petersons.com
Chapter 4: Quantitative Review

123456789012345678901234567890121234567890123456789012
12345678901234567890123456789012123456789012345678901 2
12345678901234567890123456789012123456789012345678901 2
12345678901234567890123456789012123456789012345678901 2
12345678901234567890123456789012123456789012345678901 2
12345678901234567890123456789012123456789012345678901 2
12345678901234567890123456789012123456789012345678901 2
12345678901234567890123456789012123456789012345678901 2
12345678901234567890123456789012123456789012345678901 2
12345678901234567890123456789012123456789012345678901 2
12345678901234567890123456789012123456789012345678901 2
12345678901234567890123456789012123456789012345678901 2
12345678901234567890123456789012123456789012345678901
2345678901234567890123456789012123456789012345678901
2
2
1
12345678901234567890123456789012123456789012345678901 2
12345678901234567890123456789012123456789012345678901 2
12345678901234567890123456789012123456789012345678901 2
12345678901234567890123456789012123456789012345678901
2345678901234567890123456789012123456789012345678901
2
2
1
12345678901234567890123456789012123456789012345678901 2
12345678901234567890123456789012123456789012345678901 2
12345678901234567890123456789012123456789012345678901 2
12345678901234567890123456789012123456789012345678901
2345678901234567890123456789012123456789012345678901
2
2
1
12345678901234567890123456789012123456789012345678901 2
12345678901234567890123456789012123456789012345678901 2
12345678901234567890123456789012123456789012345678901 2
12345678901234567890123456789012123456789012345678901 2
2345678901234567890123456789012123456789012345678901
12345678901234567890123456789012123456789012345678901
For which year was the ratio of exports to imports the greatest? 2
12345678901234567890123456789012123456789012345678901 2
12345678901234567890123456789012123456789012345678901 2
12345678901234567890123456789012123456789012345678901
A. 1978 2
12345678901234567890123456789012123456789012345678901 2
12345678901234567890123456789012123456789012345678901 22
B. 1982
1
12345678901234567890123456789012123456789012345678901
C. 1984 2
12345678901234567890123456789012123456789012345678901 2
12345678901234567890123456789012123456789012345678901
D. 1987 2
2345678901234567890123456789012123456789012345678901
12345678901234567890123456789012123456789012345678901 2
12345678901234567890123456789012123456789012345678901
E. 1980 2
12345678901234567890123456789012123456789012345678901 22
12345678901234567890123456789012123456789012345678901
First, determine which line is for exports and which is for imports. Once 2
12345678901234567890123456789012123456789012345678901
you have that straight, you have to pull out the trusty concept of ratios. 2
12345678901234567890123456789012123456789012345678901
12345678901234567890123456789012123456789012345678901 2
12345678901234567890123456789012123456789012345678901
Remember that ratios can be expressed as fractions, and here the exports 2
12345678901234567890123456789012123456789012345678901 22
1 will be the numerator and the imports the denominator. To get a big ratio,
12345678901234567890123456789012123456789012345678901 2
12345678901234567890123456789012123456789012345678901
you are looking for years when the exports were high and the imports were 2
12345678901234567890123456789012123456789012345678901 2
12345678901234567890123456789012123456789012345678901
low. Eyeballing the graph, which years are candidates? In 1984 and 1987 2
12345678901234567890123456789012123456789012345678901
2345678901234567890123456789012123456789012345678901
2
the export-import lines are farthest apart, so these are the best candidates, 2
12345678901234567890123456789012123456789012345678901
12345678901234567890123456789012123456789012345678901 2
1 and they are both answer choices. In 1984 the ratio was approximately 2
12345678901234567890123456789012123456789012345678901 2
12345678901234567890123456789012123456789012345678901
14 15.5 2
2345678901234567890123456789012123456789012345678901
12345678901234567890123456789012123456789012345678901
. 3, and in 1987 the ratio was approximately , 3. Choice (C), 2
12345678901234567890123456789012123456789012345678901
4 5.5 2
1 1984, is the correct answer. 2
12345678901234567890123456789012123456789012345678901 2
12345678901234567890123456789012123456789012345678901 2
12345678901234567890123456789012123456789012345678901 2
12345678901234567890123456789012123456789012345678901 2
12345678901234567890123456789012123456789012345678901 2
Tip

12345678901234567890123456789012123456789012345678901
This question shows how basic concepts like ratios are used in other areas 2
12345678901234567890123456789012123456789012345678901 2
12345678901234567890123456789012123456789012345678901 22
2345678901234567890123456789012123456789012345678901
to make questions more difficult.
12345678901234567890123456789012123456789012345678901 2
12345678901234567890123456789012123456789012345678901 2
12345678901234567890123456789012123456789012345678901 2
1
12345678901234567890123456789012123456789012345678901
What was the greatest consecutive two-year total of exports? 2
12345678901234567890123456789012123456789012345678901 2
12345678901234567890123456789012123456789012345678901
A. 14 billion 2
12345678901234567890123456789012123456789012345678901 2
12345678901234567890123456789012123456789012345678901
B. 18 billion 2
12345678901234567890123456789012123456789012345678901 22
2345678901234567890123456789012123456789012345678901
1 C. 24 billion
12345678901234567890123456789012123456789012345678901 2
12345678901234567890123456789012123456789012345678901
D. 26 billion 2
12345678901234567890123456789012123456789012345678901
2345678901234567890123456789012123456789012345678901
2
2
1 E. 28 billion
12345678901234567890123456789012123456789012345678901 2
12345678901234567890123456789012123456789012345678901 2
12345678901234567890123456789012123456789012345678901 2 103
123456789012345678901234567890121234567890123456789012
Part III: Section Review

123456789012345678901234567890121234567890123456789012
12345678901234567890123456789012123456789012345678901 2
12345678901234567890123456789012123456789012345678901 2
12345678901234567890123456789012123456789012345678901
You’re looking for exports, so you’re looking at the dotted line. The 2
12345678901234567890123456789012123456789012345678901 2
12345678901234567890123456789012123456789012345678901
greatest consecutive two-year total will be the two consecutive years 2
12345678901234567890123456789012123456789012345678901 2
12345678901234567890123456789012123456789012345678901
whose sum is the greatest. Eyeballing the chart, the sets of years that are 2
1 the best candidates will be the ones where the export line spikes upward. 2
2345678901234567890123456789012123456789012345678901
12345678901234567890123456789012123456789012345678901 2
12345678901234567890123456789012123456789012345678901 2
12345678901234567890123456789012123456789012345678901
You have two good candidates: 83–84 and 87–88. Summing each, you will 2
12345678901234567890123456789012123456789012345678901
2345678901234567890123456789012123456789012345678901
2
2
1 find 83–84 is a little more than 26 billion, and 87–88 is about 76 billion.
12345678901234567890123456789012123456789012345678901 2
12345678901234567890123456789012123456789012345678901
Therefore, choice (D) is the correct answer. 2
12345678901234567890123456789012123456789012345678901 2
12345678901234567890123456789012123456789012345678901
2345678901234567890123456789012123456789012345678901
2
2
1 The previous example involved one line graph. Things can get a little more
12345678901234567890123456789012123456789012345678901 2
12345678901234567890123456789012123456789012345678901
complicated when there are multiple graphs involved. Again, take a few 2
12345678901234567890123456789012123456789012345678901
seconds to get familiar with the graphs involved, and especially take note 2
12345678901234567890123456789012123456789012345678901
2345678901234567890123456789012123456789012345678901
2
2
1 of how the graphs are related.
12345678901234567890123456789012123456789012345678901 2
12345678901234567890123456789012123456789012345678901 2
12345678901234567890123456789012123456789012345678901 2
12345678901234567890123456789012123456789012345678901
Student Enrollment Part-Time or 2
12345678901234567890123456789012123456789012345678901 2
2345678901234567890123456789012123456789012345678901
Full-Time Employed at College T: 1976–1980
12345678901234567890123456789012123456789012345678901 2
12345678901234567890123456789012123456789012345678901 2
12345678901234567890123456789012123456789012345678901 2
12345678901234567890123456789012123456789012345678901 Breakdown of Student
2
12345678901234567890123456789012123456789012345678901 2 Majors at College T in 1979
2
1
12345678901234567890123456789012123456789012345678901 2
12345678901234567890123456789012123456789012345678901 2
12345678901234567890123456789012123456789012345678901 2
12345678901234567890123456789012123456789012345678901 2
2345678901234567890123456789012123456789012345678901
12345678901234567890123456789012123456789012345678901 2
12345678901234567890123456789012123456789012345678901 2
12345678901234567890123456789012123456789012345678901 2
12345678901234567890123456789012123456789012345678901 2
12345678901234567890123456789012123456789012345678901 2
12345678901234567890123456789012123456789012345678901 2
12345678901234567890123456789012123456789012345678901 2
12345678901234567890123456789012123456789012345678901 2 2
1
12345678901234567890123456789012123456789012345678901 2
12345678901234567890123456789012123456789012345678901 2
12345678901234567890123456789012123456789012345678901 2
12345678901234567890123456789012123456789012345678901 2
12345678901234567890123456789012123456789012345678901 2
12345678901234567890123456789012123456789012345678901 2
12345678901234567890123456789012123456789012345678901
In what years did the percentage of part-time employed students at 2
12345678901234567890123456789012123456789012345678901 2
12345678901234567890123456789012123456789012345678901
college T increase most dramatically? 2
12345678901234567890123456789012123456789012345678901 2
2345678901234567890123456789012123456789012345678901
12345678901234567890123456789012123456789012345678901 2
12345678901234567890123456789012123456789012345678901
A. 1976–1977 2
1 2
12345678901234567890123456789012123456789012345678901
B. 1977–1978 2
12345678901234567890123456789012123456789012345678901 2
12345678901234567890123456789012123456789012345678901 2
2345678901234567890123456789012123456789012345678901
C. 1978–1979
2
12345678901234567890123456789012123456789012345678901 2
12345678901234567890123456789012123456789012345678901
D. 1979–1980
2
12345678901234567890123456789012123456789012345678901 2
1 E. 1977–1979
12345678901234567890123456789012123456789012345678901 2
2345678901234567890123456789012123456789012345678901
It might take a moment to determine whether the bar graph or the pie 2
12345678901234567890123456789012123456789012345678901
12345678901234567890123456789012123456789012345678901 2
12345678901234567890123456789012123456789012345678901 2
1 chart will provide the answer. Since part-time students are listed on the bar 2
2345678901234567890123456789012123456789012345678901
graph, that’s where you should look. Specifically, you are looking for two 2
12345678901234567890123456789012123456789012345678901
12345678901234567890123456789012123456789012345678901 2
12345678901234567890123456789012123456789012345678901
years where the striped lines had the greatest percentage increase. The 2
12345678901234567890123456789012123456789012345678901 2
1 actual numbers won’t be as important—indeed, they will be wrong 2
12345678901234567890123456789012123456789012345678901 2
12345678901234567890123456789012123456789012345678901
answers—as the percentages you derive from them. 2
12345678901234567890123456789012123456789012345678901 2
12345678901234567890123456789012123456789012345678901 2
12345678901234567890123456789012123456789012345678901
1978 to 1979 is out because it is a decrease. If time was running out, you 2
1 could eliminate this choice and take a guess. Since the question concerns 2
2345678901234567890123456789012123456789012345678901
12345678901234567890123456789012123456789012345678901 2
12345678901234567890123456789012123456789012345678901 2 2
104 12345678901234567890123456789012123456789012345678901
123456789012345678901234567890121234567890123456789012

www.petersons.com
Chapter 4: Quantitative Review

123456789012345678901234567890121234567890123456789012
12345678901234567890123456789012123456789012345678901 2
12345678901234567890123456789012123456789012345678901 2
12345678901234567890123456789012123456789012345678901
percentage change, take a close look at 1976 to 1977 because 1976 is 2
12345678901234567890123456789012123456789012345678901 2
12345678901234567890123456789012123456789012345678901
small in comparison to the other years. 100 to 150 is a 50% increase. How 2
12345678901234567890123456789012123456789012345678901 2
12345678901234567890123456789012123456789012345678901
about the other years? 1977 to 1978 have the raw numbers 150 to 250, 2
1 which is a 66% increase (100 is 66% or two thirds of 150). 1979 to 1980 2
2345678901234567890123456789012123456789012345678901
12345678901234567890123456789012123456789012345678901 2
12345678901234567890123456789012123456789012345678901 2
12345678901234567890123456789012123456789012345678901
starts at 200 and goes to 350, which is a 75% increase (150 is 75% of 2
12345678901234567890123456789012123456789012345678901
2345678901234567890123456789012123456789012345678901
2
2
1 200). This is the biggest increase, so (D) is the correct answer.
12345678901234567890123456789012123456789012345678901 2
12345678901234567890123456789012123456789012345678901 2
12345678901234567890123456789012123456789012345678901 2
12345678901234567890123456789012123456789012345678901
Percentage increases are greater when the
2345678901234567890123456789012123456789012345678901
starting numbers are very small, 2
2
1 which is why a company like General Electric (GE) might improve its
12345678901234567890123456789012123456789012345678901 2
12345678901234567890123456789012123456789012345678901 2
Note

12345678901234567890123456789012123456789012345678901
profits by 30 billion dollars and only have a 1% increase. The fact is that 2
12345678901234567890123456789012123456789012345678901
2345678901234567890123456789012123456789012345678901
2
2
1 GE was already making so much cash, the additional amount is a small
12345678901234567890123456789012123456789012345678901 2
12345678901234567890123456789012123456789012345678901
percentage increase. In contrast, if a kid sets up a lemonade stand and 2
12345678901234567890123456789012123456789012345678901 2
12345678901234567890123456789012123456789012345678901
makes $1 on Saturday and $3 on Sunday, his/her profits increased by 2
2345678901234567890123456789012123456789012345678901
12345678901234567890123456789012123456789012345678901 2
12345678901234567890123456789012123456789012345678901
300% from one day to the next. 2
12345678901234567890123456789012123456789012345678901 22
12345678901234567890123456789012123456789012345678901 2
12345678901234567890123456789012123456789012345678901
12345678901234567890123456789012123456789012345678901
Which of the following represents the greatest number of students? 2
1 2
12345678901234567890123456789012123456789012345678901
A. Part-time employed in 1976 2
12345678901234567890123456789012123456789012345678901 2
12345678901234567890123456789012123456789012345678901
B. Full-time employed in 1976 2
12345678901234567890123456789012123456789012345678901 22
2345678901234567890123456789012123456789012345678901
12345678901234567890123456789012123456789012345678901
C. Humanities majors in 1979
12345678901234567890123456789012123456789012345678901 2
12345678901234567890123456789012123456789012345678901
D. Engineering majors in 1979 2
12345678901234567890123456789012123456789012345678901 2
12345678901234567890123456789012123456789012345678901 22
E. Full-time employed in 1978
12345678901234567890123456789012123456789012345678901 2
12345678901234567890123456789012123456789012345678901
This question involves the bar graph and the pie chart. Look down the left 2
12345678901234567890123456789012123456789012345678901
1 side of the bar graph to get a sense of the numbers that the graph is 2
12345678901234567890123456789012123456789012345678901 2
12345678901234567890123456789012123456789012345678901 2
12345678901234567890123456789012123456789012345678901 2
displaying. Then, try to get the same sense with the pie chart.
12345678901234567890123456789012123456789012345678901 2
12345678901234567890123456789012123456789012345678901
Now attack the answer choices. You can eliminate (A) because all the 2
12345678901234567890123456789012123456789012345678901 2
12345678901234567890123456789012123456789012345678901 2
12345678901234567890123456789012123456789012345678901 2
full-time numbers are greater than all the part-time numbers. Reading (B)
12345678901234567890123456789012123456789012345678901
off the bar graph gets you 300. Looking at choices (C) and (D), you can see 2
12345678901234567890123456789012123456789012345678901 2
2345678901234567890123456789012123456789012345678901
that the percentage of Humanities (23%) is less than the percentage of 2
12345678901234567890123456789012123456789012345678901
12345678901234567890123456789012123456789012345678901 2
1 Engineering students, at 24%. (D) has to be greater than (C), so you don’t 2
12345678901234567890123456789012123456789012345678901 2
12345678901234567890123456789012123456789012345678901
even need to figure out the exact number for it. To calculate the number of 2
12345678901234567890123456789012123456789012345678901 2
12345678901234567890123456789012123456789012345678901
Engineering majors in 1979, you’ll need to establish the total number of 2
12345678901234567890123456789012123456789012345678901 2
12345678901234567890123456789012123456789012345678901
students in that year. That number is the total height of the bars in the bar 2
12345678901234567890123456789012123456789012345678901 2
12345678901234567890123456789012123456789012345678901
graph for 1979: 700 students. For Engineering majors, you must 2
12345678901234567890123456789012123456789012345678901 2
12345678901234567890123456789012123456789012345678901
determine what is 24% of 700. It works out to a little less than 280. 2
12345678901234567890123456789012123456789012345678901 2
12345678901234567890123456789012123456789012345678901
Reading (E) off the bar graph, it is 750. Choice (B) is the answer. 2
12345678901234567890123456789012123456789012345678901 22
2345678901234567890123456789012123456789012345678901
12345678901234567890123456789012123456789012345678901 2
12345678901234567890123456789012123456789012345678901 2
12345678901234567890123456789012123456789012345678901 2
1
12345678901234567890123456789012123456789012345678901 2
12345678901234567890123456789012123456789012345678901 2
12345678901234567890123456789012123456789012345678901 2
12345678901234567890123456789012123456789012345678901 2
12345678901234567890123456789012123456789012345678901 2
12345678901234567890123456789012123456789012345678901 2
12345678901234567890123456789012123456789012345678901 2
12345678901234567890123456789012123456789012345678901 2
12345678901234567890123456789012123456789012345678901 2 105
123456789012345678901234567890121234567890123456789012
Take It to
the Next Level
123456789012345678901234567890121234567890123456789012
12345678901234567890123456789012123456789012345678901 2
12345678901234567890123456789012123456789012345678901
2345678901234567890123456789012123456789012345678901
2
2
1
Math: The Multi-Step Complicated Stuff
12345678901234567890123456789012123456789012345678901 2
12345678901234567890123456789012123456789012345678901 2
12345678901234567890123456789012123456789012345678901
The harder questions on the GRE do not require you to understand more 2
12345678901234567890123456789012123456789012345678901 2
12345678901234567890123456789012123456789012345678901 2
2345678901234567890123456789012123456789012345678901
sophisticated math. The math concepts required to answer a 500-level
problem and a 750-level problem are, for the most part, the same. The 2
12345678901234567890123456789012123456789012345678901
12345678901234567890123456789012123456789012345678901 2
12345678901234567890123456789012123456789012345678901 2
12345678901234567890123456789012123456789012345678901 2
difference is that the 750-level problem will challenge you to be more
12345678901234567890123456789012123456789012345678901 2
1 clever, creative, and inventive in your approach. Showing you how to 2
12345678901234567890123456789012123456789012345678901
“out-clever” the test will be a major focus of this chapter. 2
12345678901234567890123456789012123456789012345678901 2
12345678901234567890123456789012123456789012345678901 2
2345678901234567890123456789012123456789012345678901
Before that starts, there are a few math concepts you are probably only 2
12345678901234567890123456789012123456789012345678901
12345678901234567890123456789012123456789012345678901 2
going to see on harder questions. These harder concepts are really just 2
12345678901234567890123456789012123456789012345678901
12345678901234567890123456789012123456789012345678901
extensions of what we’ve already covered in this chapter, so it’s not as if 2
2
12345678901234567890123456789012123456789012345678901
12345678901234567890123456789012123456789012345678901
the easier questions are algebra and the harder ones are calculus. Basically, 2
2
12345678901234567890123456789012123456789012345678901
12345678901234567890123456789012123456789012345678901
the harder questions contain the basics revved up to make problems more 2
2
12345678901234567890123456789012123456789012345678901 2
1 interesting and difficult.
12345678901234567890123456789012123456789012345678901 2
12345678901234567890123456789012123456789012345678901 2
12345678901234567890123456789012123456789012345678901 2
12345678901234567890123456789012123456789012345678901 2
If a math concept comes up that you are not familiar with, or you can’t
12345678901234567890123456789012123456789012345678901
remember exactly what it is, look back to the beginning of the chapter to 2
12345678901234567890123456789012123456789012345678901 2
12345678901234567890123456789012123456789012345678901 2
12345678901234567890123456789012123456789012345678901 2
see if it is covered there. If you’ve skipped the beginning of the chapter, it’s
12345678901234567890123456789012123456789012345678901
not a bad idea to leaf through it anyway and make sure that all the math 2
12345678901234567890123456789012123456789012345678901 2
2345678901234567890123456789012123456789012345678901
content is fresh in your mind. Once that’s done, you can Take It to the 2
12345678901234567890123456789012123456789012345678901
12345678901234567890123456789012123456789012345678901 2
1 Next Level and start honing your clever skills. 2
12345678901234567890123456789012123456789012345678901 2
12345678901234567890123456789012123456789012345678901 2
12345678901234567890123456789012123456789012345678901 2
12345678901234567890123456789012123456789012345678901
2345678901234567890123456789012123456789012345678901
2
2
1
12345678901234567890123456789012123456789012345678901
Advanced Arithmetic 2
12345678901234567890123456789012123456789012345678901 2
2345678901234567890123456789012123456789012345678901
Hard arithmetic problems will use jargon to make problems more 2
12345678901234567890123456789012123456789012345678901
12345678901234567890123456789012123456789012345678901 2
complex. Problems will include multiple phrases like “the sum of the first 2
12345678901234567890123456789012123456789012345678901
12345678901234567890123456789012123456789012345678901 2
1 three prime numbers” or “the product of two consecutive integers.” The 2
12345678901234567890123456789012123456789012345678901 2
12345678901234567890123456789012123456789012345678901
way to decode such math speak is to attack it piece by piece and phrase by 2
12345678901234567890123456789012123456789012345678901 2
12345678901234567890123456789012123456789012345678901
phrase. Unless you are a math wunderkind, trying to conceptualize whole 2
12345678901234567890123456789012123456789012345678901 2
12345678901234567890123456789012123456789012345678901
sentences of math speak at one time is impossible. At the least, it is 2
12345678901234567890123456789012123456789012345678901 2
12345678901234567890123456789012123456789012345678901
foolhardy. 2
12345678901234567890123456789012123456789012345678901 2
12345678901234567890123456789012123456789012345678901
Let’s decode these two phrases. The first three prime numbers are 2, 3, and 2
12345678901234567890123456789012123456789012345678901 2
12345678901234567890123456789012123456789012345678901 2
12345678901234567890123456789012123456789012345678901 2
5, and their sum is 10. In the second phrase, since we don’t know what the
12345678901234567890123456789012123456789012345678901 2
123456789012345678901234567890121234567890123456789012
106
Chapter 4: Quantitative Review

123456789012345678901234567890121234567890123456789012
12345678901234567890123456789012123456789012345678901 2
12345678901234567890123456789012123456789012345678901 2
12345678901234567890123456789012123456789012345678901
first integer is, let’s call it x. The product of x and the next integer is 2
12345678901234567890123456789012123456789012345678901 2
12345678901234567890123456789012123456789012345678901
(x)(x 1 1). None of that was too difficult, but suppose a problem reads, “If 2
12345678901234567890123456789012123456789012345678901 2
12345678901234567890123456789012123456789012345678901
the sum of the first three prime numbers plus 2 equals the product of two 2
2345678901234567890123456789012123456789012345678901 2
1 consecutive integers, what is the value of the first integer?”
12345678901234567890123456789012123456789012345678901 2
12345678901234567890123456789012123456789012345678901 2
12345678901234567890123456789012123456789012345678901 2
12345678901234567890123456789012123456789012345678901
2345678901234567890123456789012123456789012345678901
2
2
1
Alert!
12345678901234567890123456789012123456789012345678901 2
12345678901234567890123456789012123456789012345678901
Using algebra in the form of the variable x to help describe an arithmetic 2
12345678901234567890123456789012123456789012345678901 2
12345678901234567890123456789012123456789012345678901
expression is fairly common practice. Don’t
2345678901234567890123456789012123456789012345678901
hesitate to draw information 2
2
1 from any field of math if you believe it will help you solve a problem.
12345678901234567890123456789012123456789012345678901 2
12345678901234567890123456789012123456789012345678901 2
12345678901234567890123456789012123456789012345678901 2
12345678901234567890123456789012123456789012345678901
2345678901234567890123456789012123456789012345678901
2
2
1
12345678901234567890123456789012123456789012345678901 2
12345678901234567890123456789012123456789012345678901
If you decode this jargon piece by piece, it is manageable. The sum of the 2
12345678901234567890123456789012123456789012345678901 2
12345678901234567890123456789012123456789012345678901
first three prime numbers plus 2 is 12, and you already know what the 2
2345678901234567890123456789012123456789012345678901
product of two consecutive integers is. Therefore, you can set up the 2
12345678901234567890123456789012123456789012345678901
12345678901234567890123456789012123456789012345678901 2

Take It to the Next Level


12345678901234567890123456789012123456789012345678901
equation: 2
12345678901234567890123456789012123456789012345678901 22
12345678901234567890123456789012123456789012345678901 2
12345678901234567890123456789012123456789012345678901
~x!~x 1 1! 5 12 → 2
1
12345678901234567890123456789012123456789012345678901 2
12345678901234567890123456789012123456789012345678901 2
2
x 1 x 5 12 →
12345678901234567890123456789012123456789012345678901 2
12345678901234567890123456789012123456789012345678901 22
2345678901234567890123456789012123456789012345678901
x2
1 x 2 12 5 12 2 12 →
12345678901234567890123456789012123456789012345678901
12345678901234567890123456789012123456789012345678901
x2 1 x 2 12 5 0 → 2
12345678901234567890123456789012123456789012345678901 2
12345678901234567890123456789012123456789012345678901
→ 2
12345678901234567890123456789012123456789012345678901 22
~x 1 4 !~ x 2 3 ! 5 0
12345678901234567890123456789012123456789012345678901
12345678901234567890123456789012123456789012345678901
x 5 24,3 2
12345678901234567890123456789012123456789012345678901 22
1 There’s a bit of everything in the above problem, and that’s not unusual
12345678901234567890123456789012123456789012345678901 2
12345678901234567890123456789012123456789012345678901 2
12345678901234567890123456789012123456789012345678901 2
for hard problems. You had to use arithmetic to decipher the problem and
12345678901234567890123456789012123456789012345678901
algebra to set up an equation. Then you had to do some factoring to split 2
12345678901234567890123456789012123456789012345678901
2345678901234567890123456789012123456789012345678901
2
the equation x2 1 x 2 12 5 0 in order to find the values for x. If you 2
12345678901234567890123456789012123456789012345678901
12345678901234567890123456789012123456789012345678901 2
1 handle each step, though, the answer falls into your lap. 2
12345678901234567890123456789012123456789012345678901 2
12345678901234567890123456789012123456789012345678901 2
2345678901234567890123456789012123456789012345678901
12345678901234567890123456789012123456789012345678901
Column A Column B 2
12345678901234567890123456789012123456789012345678901 2
1 2
12345678901234567890123456789012123456789012345678901
a c e 2
12345678901234567890123456789012123456789012345678901 2
2345678901234567890123456789012123456789012345678901
1 1 . . .
12345678901234567890123456789012123456789012345678901 2
12345678901234567890123456789012123456789012345678901
b d f 2
12345678901234567890123456789012123456789012345678901 2
12345678901234567890123456789012123456789012345678901 22
is a sum in which all the variables are
1
12345678901234567890123456789012123456789012345678901
positive consecutive integers greater than 1 2
12345678901234567890123456789012123456789012345678901 2
12345678901234567890123456789012123456789012345678901
that increase in alphabetical order.
2
12345678901234567890123456789012123456789012345678901 2
12345678901234567890123456789012123456789012345678901
The third term in a 2 2
2345678901234567890123456789012123456789012345678901
123456789012345678901234567890121234567890123456789012 2
12345678901234567890123456789012123456789012345678901
the sum 2
12345678901234567890123456789012123456789012345678901 22
a 2 1
12345678901234567890123456789012123456789012345678901 2
1
12345678901234567890123456789012123456789012345678901 2
12345678901234567890123456789012123456789012345678901
First, you must decode. Positive consecutive integers are things like 4, 5, 2
12345678901234567890123456789012123456789012345678901 2
12345678901234567890123456789012123456789012345678901
and 6, and these increase in alphabetical order, which means that b 5 a 1 1 2
12345678901234567890123456789012123456789012345678901 2
12345678901234567890123456789012123456789012345678901 2
12345678901234567890123456789012123456789012345678901 2
12345678901234567890123456789012123456789012345678901 2
12345678901234567890123456789012123456789012345678901 2 107
123456789012345678901234567890121234567890123456789012
Part III: Section Review

123456789012345678901234567890121234567890123456789012
12345678901234567890123456789012123456789012345678901 2
12345678901234567890123456789012123456789012345678901 2
12345678901234567890123456789012123456789012345678901
and c 5 b 1 1. Notice that since b 5 a 1 1, then c 5 (a 1 1) 1 1 5 a 1 2. 2
12345678901234567890123456789012123456789012345678901 2
12345678901234567890123456789012123456789012345678901
This means that the sum can be rewritten 2
12345678901234567890123456789012123456789012345678901 2
12345678901234567890123456789012123456789012345678901 2
12345678901234567890123456789012123456789012345678901
a a12 a14 2
12345678901234567890123456789012123456789012345678901
1 1 .... 2
12345678901234567890123456789012123456789012345678901
a11 a13 a15 2
12345678901234567890123456789012123456789012345678901 2
12345678901234567890123456789012123456789012345678901
2345678901234567890123456789012123456789012345678901
2
2
1 This makes the third term a 1 4
12345678901234567890123456789012123456789012345678901
. That’s Column A, so far. 2
12345678901234567890123456789012123456789012345678901
a15 2
12345678901234567890123456789012123456789012345678901 2
12345678901234567890123456789012123456789012345678901
2345678901234567890123456789012123456789012345678901
2
2
1 Compare this to the fraction in Column B. The smallest value that a can
12345678901234567890123456789012123456789012345678901
have is 2, so it will be interesting to start with this number and see what 2
12345678901234567890123456789012123456789012345678901 2
12345678901234567890123456789012123456789012345678901
6 2
12345678901234567890123456789012123456789012345678901
you get. Column A becomes , while
2345678901234567890123456789012123456789012345678901 Column B is 2
2
1 7
12345678901234567890123456789012123456789012345678901 2
12345678901234567890123456789012123456789012345678901 2
12345678901234567890123456789012123456789012345678901
a 2 2 2 2 2 0 2
12345678901234567890123456789012123456789012345678901
5 5 5 0. 2
12345678901234567890123456789012123456789012345678901 2
2345678901234567890123456789012123456789012345678901
a 2 1 2 2 1 1
12345678901234567890123456789012123456789012345678901 2
12345678901234567890123456789012123456789012345678901
Column A is greater is this instance, but to play it safe, it helps to try 2
2
12345678901234567890123456789012123456789012345678901
12345678901234567890123456789012123456789012345678901
another number. A small number was used the first time, so a large number 2
2
12345678901234567890123456789012123456789012345678901 2
1
12345678901234567890123456789012123456789012345678901 14 2
12345678901234567890123456789012123456789012345678901
can be plugged in this time for variety. Making a 5 10, you get in 2
12345678901234567890123456789012123456789012345678901 15 2
2345678901234567890123456789012123456789012345678901
12345678901234567890123456789012123456789012345678901
8 2
12345678901234567890123456789012123456789012345678901 2
12345678901234567890123456789012123456789012345678901
Column A and in Column B. Column A is again greater, and placing an 2
12345678901234567890123456789012123456789012345678901
9 2
12345678901234567890123456789012123456789012345678901 2
even larger number will probably not make a difference. The correct
12345678901234567890123456789012123456789012345678901 2
12345678901234567890123456789012123456789012345678901
answer is (A). 2
12345678901234567890123456789012123456789012345678901 2
12345678901234567890123456789012123456789012345678901 2
1 Ratios are another arithmetic topic that can be made interesting. Suppose 2
2345678901234567890123456789012123456789012345678901
you are told the ratio of a to b is 4:7 and the ratio of b to c is 8:7. You are 2
12345678901234567890123456789012123456789012345678901
12345678901234567890123456789012123456789012345678901 2
1 then asked, “What is the ratio of a to c?” 2
12345678901234567890123456789012123456789012345678901 2
12345678901234567890123456789012123456789012345678901 2
12345678901234567890123456789012123456789012345678901
The easiest way to approach such a problem is to make the common 2
12345678901234567890123456789012123456789012345678901 2
12345678901234567890123456789012123456789012345678901
element in both ratios, b, the same so that you can directly compare a and 2
12345678901234567890123456789012123456789012345678901 2
12345678901234567890123456789012123456789012345678901
c. To do that, find a common multiple of 7 and 8, preferably the least 2
12345678901234567890123456789012123456789012345678901 2
2345678901234567890123456789012123456789012345678901
12345678901234567890123456789012123456789012345678901 2
1 common multiple. 56 works, since it’s 7 3 8. 2
12345678901234567890123456789012123456789012345678901 2
12345678901234567890123456789012123456789012345678901 2
First ratio: a to b 5 4:7. Multiplied by 8, this becomes
12345678901234567890123456789012123456789012345678901 2
2345678901234567890123456789012123456789012345678901
2
12345678901234567890123456789012123456789012345678901 2
12345678901234567890123456789012123456789012345678901
a to b 5 32:56
12345678901234567890123456789012123456789012345678901 2 2
1 Second ratio: b to c 5 8:7. Multiplied by 7, this becomes
12345678901234567890123456789012123456789012345678901 2
12345678901234567890123456789012123456789012345678901 2
12345678901234567890123456789012123456789012345678901
b to c 5 56:49 2
12345678901234567890123456789012123456789012345678901 2
12345678901234567890123456789012123456789012345678901 2
12345678901234567890123456789012123456789012345678901 2
2345678901234567890123456789012123456789012345678901
For both ratios, b is now 56. This allows you write to both ratios as a
2
12345678901234567890123456789012123456789012345678901 2
12345678901234567890123456789012123456789012345678901
single long ratio:
12345678901234567890123456789012123456789012345678901 2 2
1
12345678901234567890123456789012123456789012345678901
a to b to c 5 32:56:49 2
12345678901234567890123456789012123456789012345678901 2
12345678901234567890123456789012123456789012345678901
You can now see that a : c 5 32 : 49. If these numbers in the ratio had any 2
12345678901234567890123456789012123456789012345678901 2
12345678901234567890123456789012123456789012345678901
2345678901234567890123456789012123456789012345678901
2
2
1 common multiples, then the ratio should be simplified.
12345678901234567890123456789012123456789012345678901 2
12345678901234567890123456789012123456789012345678901 2 2
108 12345678901234567890123456789012123456789012345678901
123456789012345678901234567890121234567890123456789012

www.petersons.com
Chapter 4: Quantitative Review

123456789012345678901234567890121234567890123456789012
12345678901234567890123456789012123456789012345678901 2
12345678901234567890123456789012123456789012345678901 2
12345678901234567890123456789012123456789012345678901 2
12345678901234567890123456789012123456789012345678901 2
12345678901234567890123456789012123456789012345678901 2

X-Ref
12345678901234567890123456789012123456789012345678901
Finding a common element for these ratios is similar to the practice of 2
12345678901234567890123456789012123456789012345678901
2345678901234567890123456789012123456789012345678901
2
2
1 finding a common denominator before adding and subtracting fractions.
12345678901234567890123456789012123456789012345678901 2
12345678901234567890123456789012123456789012345678901
See page 54 of this chapter. 2
12345678901234567890123456789012123456789012345678901 2
12345678901234567890123456789012123456789012345678901
2345678901234567890123456789012123456789012345678901
2
2
1
12345678901234567890123456789012123456789012345678901 2
12345678901234567890123456789012123456789012345678901 2
12345678901234567890123456789012123456789012345678901 2
If m and n are integers and (m 1 n) is even, then which of the 2
12345678901234567890123456789012123456789012345678901
2345678901234567890123456789012123456789012345678901
2
2
1 following must be true?
12345678901234567890123456789012123456789012345678901 2
12345678901234567890123456789012123456789012345678901 2
12345678901234567890123456789012123456789012345678901
I. Both integers are even. 2
12345678901234567890123456789012123456789012345678901
2345678901234567890123456789012123456789012345678901
2
2
1 II. Both integers are odd.
12345678901234567890123456789012123456789012345678901 2
12345678901234567890123456789012123456789012345678901
III. One integer is even and one integer is odd. 2
12345678901234567890123456789012123456789012345678901 2
12345678901234567890123456789012123456789012345678901 2
12345678901234567890123456789012123456789012345678901 22
2345678901234567890123456789012123456789012345678901
A. I only
12345678901234567890123456789012123456789012345678901

Take It to the Next Level


B. II only
12345678901234567890123456789012123456789012345678901 2
12345678901234567890123456789012123456789012345678901
C. III only 2
12345678901234567890123456789012123456789012345678901 22
12345678901234567890123456789012123456789012345678901
D. I and II only
2
1
12345678901234567890123456789012123456789012345678901
E. I and III only 2
12345678901234567890123456789012123456789012345678901 2
12345678901234567890123456789012123456789012345678901 2
12345678901234567890123456789012123456789012345678901 22
2345678901234567890123456789012123456789012345678901
To attack this one, you have to work through each option to see if it must
12345678901234567890123456789012123456789012345678901
be true. Using FOIL, you know that if (m 1 n)2 is even, then m2 1 mn 1 n2 2
12345678901234567890123456789012123456789012345678901
12345678901234567890123456789012123456789012345678901 2
12345678901234567890123456789012123456789012345678901
is also even. 2
12345678901234567890123456789012123456789012345678901 2
12345678901234567890123456789012123456789012345678901
2 2
→ 2
12345678901234567890123456789012123456789012345678901 22
I. must be true because m 1 2mn 1 n even 1 even 1 even 5 even.
12345678901234567890123456789012123456789012345678901
That eliminates choices (B) and (C). If you’re pressed for time, you 2
1
12345678901234567890123456789012123456789012345678901 2
12345678901234567890123456789012123456789012345678901
could guess now with one in three odds of guessing right. 2
12345678901234567890123456789012123456789012345678901 2
12345678901234567890123456789012123456789012345678901
II. can be written as: m 1 2mn 1 n → odd 1 even 1 odd 5 even 1 2
2 2
12345678901234567890123456789012123456789012345678901 2
12345678901234567890123456789012123456789012345678901
even 5 even. This does work, so (B) is out. 2
12345678901234567890123456789012123456789012345678901 2
12345678901234567890123456789012123456789012345678901 2
12345678901234567890123456789012123456789012345678901
III. can be written as: (m 1 2mn 1 n → even 1 even 1 odd 5 even 1 2
2 2
12345678901234567890123456789012123456789012345678901 2
12345678901234567890123456789012123456789012345678901
odd 5 odd. This doesn’t work, so (D) is the correct answer. 2
12345678901234567890123456789012123456789012345678901 2
12345678901234567890123456789012123456789012345678901 2
12345678901234567890123456789012123456789012345678901 2
This approach uses basic facts about how even and odd numbers
12345678901234567890123456789012123456789012345678901
interact when multiplied and added together. If this approach is too 2
12345678901234567890123456789012123456789012345678901 2
12345678901234567890123456789012123456789012345678901 2
12345678901234567890123456789012123456789012345678901
abstract for you, just pick two numbers for m and n according to each 2
12345678901234567890123456789012123456789012345678901 2
12345678901234567890123456789012123456789012345678901 2
scenario and work the problem through using those numbers. If you
2345678901234567890123456789012123456789012345678901
don’t make a careless math error, you should get the same answer, (D). 2
12345678901234567890123456789012123456789012345678901
12345678901234567890123456789012123456789012345678901 2
12345678901234567890123456789012123456789012345678901 22
12345678901234567890123456789012123456789012345678901 2
1
12345678901234567890123456789012123456789012345678901 2
12345678901234567890123456789012123456789012345678901
Harder Algebra 2
12345678901234567890123456789012123456789012345678901 2
12345678901234567890123456789012123456789012345678901
There are three primary ways the GRE will increase the difficulty on 2
12345678901234567890123456789012123456789012345678901 2
2345678901234567890123456789012123456789012345678901
12345678901234567890123456789012123456789012345678901 2
1 algebra problems. The first relies on shock value. On this track, you are 2
12345678901234567890123456789012123456789012345678901
presented with one or more monstrous-looking algebraic expressions that 2
12345678901234567890123456789012123456789012345678901 2
12345678901234567890123456789012123456789012345678901
are supposed to shock you into some form of problem-solving paralysis. If 2
2345678901234567890123456789012123456789012345678901 2
1
12345678901234567890123456789012123456789012345678901
you overcome this initial shock, keep in mind that almost every ugly 2
12345678901234567890123456789012123456789012345678901 2
12345678901234567890123456789012123456789012345678901 2 109
123456789012345678901234567890121234567890123456789012
Part III: Section Review

123456789012345678901234567890121234567890123456789012
12345678901234567890123456789012123456789012345678901 2
12345678901234567890123456789012123456789012345678901 2
12345678901234567890123456789012123456789012345678901
algebraic expression will have a way to simplify it quickly. These problems 2
12345678901234567890123456789012123456789012345678901 2
12345678901234567890123456789012123456789012345678901
are designed to shock, but they almost always can be simplified quickly. 2
12345678901234567890123456789012123456789012345678901 2
12345678901234567890123456789012123456789012345678901
2345678901234567890123456789012123456789012345678901
2
1 If you see an expression like , you should immediately 2
2
4x 1 14x 1 6
12345678901234567890123456789012123456789012345678901 2
12345678901234567890123456789012123456789012345678901
4x 1 2 2
12345678901234567890123456789012123456789012345678901
think, “There must be a quick way to simplify it.” With algebra in 2
12345678901234567890123456789012123456789012345678901 2
1 fractions, always be on the lookout for canceling terms that are in both the 2
2345678901234567890123456789012123456789012345678901
12345678901234567890123456789012123456789012345678901 2
12345678901234567890123456789012123456789012345678901
numerator and the denominator. Since the denominator is simpler, start 2
12345678901234567890123456789012123456789012345678901 2
12345678901234567890123456789012123456789012345678901
there: 4x 1 2 5 2(2x 1 1). Now look for a (2x 1 1) in the numerator. Seek 2
2345678901234567890123456789012123456789012345678901 2
1
12345678901234567890123456789012123456789012345678901
and ye shall find: 2
12345678901234567890123456789012123456789012345678901 2
12345678901234567890123456789012123456789012345678901 2
12345678901234567890123456789012123456789012345678901
2 2
4x 1 14x 1 6 5 2~2x 1 7x 1 3!
2345678901234567890123456789012123456789012345678901
2
2
1
12345678901234567890123456789012123456789012345678901 2
12345678901234567890123456789012123456789012345678901
5 2~2x 1 1!~ x 1 3 ! 2
12345678901234567890123456789012123456789012345678901 2
12345678901234567890123456789012123456789012345678901
Therefore, 2
12345678901234567890123456789012123456789012345678901 2
2345678901234567890123456789012123456789012345678901
12345678901234567890123456789012123456789012345678901 2
12345678901234567890123456789012123456789012345678901
2 2
12345678901234567890123456789012123456789012345678901
4x 1 14x 1 6 2(2x 1 1)(x 1 3) 2
12345678901234567890123456789012123456789012345678901
5 5 x 1 3 2
12345678901234567890123456789012123456789012345678901 2
4x 1 2 2(2x 1 1)
2
1
12345678901234567890123456789012123456789012345678901
That whole mess boils down to a simple x 1 3. 2
12345678901234567890123456789012123456789012345678901 2
12345678901234567890123456789012123456789012345678901 2
12345678901234567890123456789012123456789012345678901 2
2345678901234567890123456789012123456789012345678901
For an idea about where to start simplifying, think of these guidelines:
12345678901234567890123456789012123456789012345678901 2
12345678901234567890123456789012123456789012345678901
1. Look at the numerator and denominator and start with which- 2
2
12345678901234567890123456789012123456789012345678901
12345678901234567890123456789012123456789012345678901 2
12345678901234567890123456789012123456789012345678901
ever one looks simpler. 2
12345678901234567890123456789012123456789012345678901 2
2. Once you factor the simpler term, look for those factors in the 2
12345678901234567890123456789012123456789012345678901
12345678901234567890123456789012123456789012345678901 2
1 2
12345678901234567890123456789012123456789012345678901 2
2345678901234567890123456789012123456789012345678901
harder term. They should be there in some form or another.
12345678901234567890123456789012123456789012345678901 2
1 3. Cancel out any similar terms you can, and enjoy your new 2
12345678901234567890123456789012123456789012345678901 2
12345678901234567890123456789012123456789012345678901
simplified equation. 2
12345678901234567890123456789012123456789012345678901 2
2345678901234567890123456789012123456789012345678901
12345678901234567890123456789012123456789012345678901 2
1 Expecting cleverness is the second way that the GRE will try to make 2
12345678901234567890123456789012123456789012345678901 2
12345678901234567890123456789012123456789012345678901
algebra difficult. A problem will expect you to make a clever algebra 2
2345678901234567890123456789012123456789012345678901
12345678901234567890123456789012123456789012345678901
maneuver to finish the problem. You are especially liable to see this on 2
2
12345678901234567890123456789012123456789012345678901 2
1 QCs. The following problem illustrates this idea.
12345678901234567890123456789012123456789012345678901 2
12345678901234567890123456789012123456789012345678901 2
12345678901234567890123456789012123456789012345678901 2
12345678901234567890123456789012123456789012345678901
Column A Column B 2
12345678901234567890123456789012123456789012345678901 2
12345678901234567890123456789012123456789012345678901 2
12345678901234567890123456789012123456789012345678901 2
a and b are positive constants
2345678901234567890123456789012123456789012345678901
12345678901234567890123456789012123456789012345678901 2
12345678901234567890123456789012123456789012345678901
and 0 , x , y. 2
12345678901234567890123456789012123456789012345678901 2
12345678901234567890123456789012123456789012345678901
2 2
1 x (x1y) 2
2345678901234567890123456789012123456789012345678901
12345678901234567890123456789012123456789012345678901 2
12345678901234567890123456789012123456789012345678901
a ab 2
12345678901234567890123456789012123456789012345678901 2
12345678901234567890123456789012123456789012345678901 2
b
2
1
12345678901234567890123456789012123456789012345678901
y 2
12345678901234567890123456789012123456789012345678901 2
12345678901234567890123456789012123456789012345678901 2
12345678901234567890123456789012123456789012345678901 2
12345678901234567890123456789012123456789012345678901 2
12345678901234567890123456789012123456789012345678901 2
12345678901234567890123456789012123456789012345678901 2
12345678901234567890123456789012123456789012345678901 2 2
110 12345678901234567890123456789012123456789012345678901
123456789012345678901234567890121234567890123456789012

www.petersons.com
Chapter 4: Quantitative Review

123456789012345678901234567890121234567890123456789012
12345678901234567890123456789012123456789012345678901 2
12345678901234567890123456789012123456789012345678901 2
12345678901234567890123456789012123456789012345678901
First simplify Column A. To do this, you have to divide each fraction 2
12345678901234567890123456789012123456789012345678901 2
12345678901234567890123456789012123456789012345678901
before multiplying. 2
12345678901234567890123456789012123456789012345678901 2
12345678901234567890123456789012123456789012345678901 2
12345678901234567890123456789012123456789012345678901
x y 2
12345678901234567890123456789012123456789012345678901 2
12345678901234567890123456789012123456789012345678901 2
1
a b
b y
S D S D
x b
y
3
y y
12345678901234567890123456789012123456789012345678901
12345678901234567890123456789012123456789012345678901
3 5 4
b
4
2345678901234567890123456789012123456789012345678901
a b
2
2
2
12345678901234567890123456789012123456789012345678901 2
12345678901234567890123456789012123456789012345678901
y b 2
12345678901234567890123456789012123456789012345678901 2
1 S D S D
12345678901234567890123456789012123456789012345678901
5
x
3
a b
y
3
y
3
b
2345678901234567890123456789012123456789012345678901
12345678901234567890123456789012123456789012345678901
b y
2
2
2
12345678901234567890123456789012123456789012345678901 2
12345678901234567890123456789012123456789012345678901
xy 2
12345678901234567890123456789012123456789012345678901
2345678901234567890123456789012123456789012345678901
2
2
1 5
ab
12345678901234567890123456789012123456789012345678901 2
12345678901234567890123456789012123456789012345678901 2
12345678901234567890123456789012123456789012345678901
As you can see, the second term cancels itself out, leaving only the single 2
12345678901234567890123456789012123456789012345678901 2
12345678901234567890123456789012123456789012345678901
xy 2
12345678901234567890123456789012123456789012345678901 2

Take It to the Next Level


fraction . Column B can be rewritten as:
12345678901234567890123456789012123456789012345678901
ab 2
12345678901234567890123456789012123456789012345678901 2
12345678901234567890123456789012123456789012345678901 2
2345678901234567890123456789012123456789012345678901
12345678901234567890123456789012123456789012345678901
~ x 1 y! 2
x2
1 2xy 1 y2
2
1 5 2
12345678901234567890123456789012123456789012345678901
ab ab 2
12345678901234567890123456789012123456789012345678901 2
12345678901234567890123456789012123456789012345678901 2
12345678901234567890123456789012123456789012345678901 22
2345678901234567890123456789012123456789012345678901
You can disregard the denominators since they are the same in each
12345678901234567890123456789012123456789012345678901
column. Comparing the numerators, you can see a 2xy in Column B 2
12345678901234567890123456789012123456789012345678901
(among other values) compared to a single xy in Column A. Since the 2
12345678901234567890123456789012123456789012345678901
12345678901234567890123456789012123456789012345678901 2
statement says that both integers are positive, it is clear that (B) is greater. 2
12345678901234567890123456789012123456789012345678901
12345678901234567890123456789012123456789012345678901 2
12345678901234567890123456789012123456789012345678901 22
12345678901234567890123456789012123456789012345678901
1 It didn’t take earthshaking genius to solve the last problem. Instead, all 2
12345678901234567890123456789012123456789012345678901
you needed was confidence that the two columns could be readily 2
12345678901234567890123456789012123456789012345678901 2
12345678901234567890123456789012123456789012345678901
compared and the ability to jump in and start performing mathematical 2
12345678901234567890123456789012123456789012345678901 2
12345678901234567890123456789012123456789012345678901
operations to help compare the two. 2
12345678901234567890123456789012123456789012345678901 2
12345678901234567890123456789012123456789012345678901 2
12345678901234567890123456789012123456789012345678901
Embedding algebra in a coordinate plane is the third way that the GRE 2
12345678901234567890123456789012123456789012345678901
will make algebra more challenging. You might encounter a polygon 2
12345678901234567890123456789012123456789012345678901 2
2345678901234567890123456789012123456789012345678901
whose side lengths are given in terms of algebraic expressions, as opposed 2
12345678901234567890123456789012123456789012345678901
12345678901234567890123456789012123456789012345678901 2
1 to simply numbers or constants. The key to solving such problems is to not 2
12345678901234567890123456789012123456789012345678901 2
12345678901234567890123456789012123456789012345678901
be unnerved by the combination of geometry and algebra. Normally you 2
12345678901234567890123456789012123456789012345678901 2
12345678901234567890123456789012123456789012345678901 2
12345678901234567890123456789012123456789012345678901
will need to start with the geometry and see what you can infer from it, and 2
12345678901234567890123456789012123456789012345678901 2
12345678901234567890123456789012123456789012345678901 2
then examine the algebra.
12345678901234567890123456789012123456789012345678901 22
2345678901234567890123456789012123456789012345678901
12345678901234567890123456789012123456789012345678901 2
12345678901234567890123456789012123456789012345678901 2
12345678901234567890123456789012123456789012345678901 2
1
12345678901234567890123456789012123456789012345678901 2
12345678901234567890123456789012123456789012345678901 2
12345678901234567890123456789012123456789012345678901 2
12345678901234567890123456789012123456789012345678901 2
12345678901234567890123456789012123456789012345678901 2
12345678901234567890123456789012123456789012345678901 22
2345678901234567890123456789012123456789012345678901
1
12345678901234567890123456789012123456789012345678901 2
12345678901234567890123456789012123456789012345678901 2
12345678901234567890123456789012123456789012345678901 2
12345678901234567890123456789012123456789012345678901 2
12345678901234567890123456789012123456789012345678901 2
12345678901234567890123456789012123456789012345678901 2
12345678901234567890123456789012123456789012345678901 2 111
123456789012345678901234567890121234567890123456789012
Part III: Section Review

123456789012345678901234567890121234567890123456789012
12345678901234567890123456789012123456789012345678901 2
12345678901234567890123456789012123456789012345678901 2
12345678901234567890123456789012123456789012345678901
If you are given this figure and then asked to solve for x, start with the 2
12345678901234567890123456789012123456789012345678901 2
12345678901234567890123456789012123456789012345678901
geometry. You have a right triangle, since the angle at the origin (0, 0) is 90 2
12345678901234567890123456789012123456789012345678901 2
12345678901234567890123456789012123456789012345678901
degrees. The side along the y-axis is length 3, and the side along the x-axis 2
1 is length 4. This means you have an old friend, the 3:4:5 right triangle, 2
2345678901234567890123456789012123456789012345678901
12345678901234567890123456789012123456789012345678901 2
12345678901234567890123456789012123456789012345678901 2
12345678901234567890123456789012123456789012345678901
guest starring in a coordinate grid problem. 2
12345678901234567890123456789012123456789012345678901 2
1 Now turn to the algebra. The diagram contains an expression, 3x 1 y, for 2
2345678901234567890123456789012123456789012345678901
12345678901234567890123456789012123456789012345678901 2
12345678901234567890123456789012123456789012345678901 2
12345678901234567890123456789012123456789012345678901
the hypotenuse, but you know that the length is 5 since it’s a 3: 4: 5 right 2
12345678901234567890123456789012123456789012345678901
2345678901234567890123456789012123456789012345678901
2
2
1 triangle. This gives you the equation 3x 1 y 5 5. From the side of length 3,
12345678901234567890123456789012123456789012345678901 2
12345678901234567890123456789012123456789012345678901
you can get x 1 y 5 3. Once you have two equations (x 1 y 5 3 and 2
12345678901234567890123456789012123456789012345678901
3x y 5) and two variables, you can subtract the former equation from 2
12345678901234567890123456789012123456789012345678901
1 5
2345678901234567890123456789012123456789012345678901
2
2
1 the latter and eliminate y. 2x 5 2 → x 5 1
12345678901234567890123456789012123456789012345678901 2
12345678901234567890123456789012123456789012345678901 2
12345678901234567890123456789012123456789012345678901
No one single step in this problem was difficult, but the problem was 2
12345678901234567890123456789012123456789012345678901 2
12345678901234567890123456789012123456789012345678901
involved since it was a messy pile of heaped-together geometry and algebra 2
12345678901234567890123456789012123456789012345678901 2
12345678901234567890123456789012123456789012345678901
concepts. If you deal with them separately, and take each in its turn, the 2
12345678901234567890123456789012123456789012345678901 2
12345678901234567890123456789012123456789012345678901
problem becomes quite manageable. 2
12345678901234567890123456789012123456789012345678901 2
2345678901234567890123456789012123456789012345678901
1 A symbol that might come up in an algebra problem is the “!”. In English 2
12345678901234567890123456789012123456789012345678901 2
12345678901234567890123456789012123456789012345678901 2
12345678901234567890123456789012123456789012345678901
grammar, this is that excited bastion of punctuation, the exclamation 2
2345678901234567890123456789012123456789012345678901
point. In math, however, the “!” is the symbol for factorials. With a 2
12345678901234567890123456789012123456789012345678901
12345678901234567890123456789012123456789012345678901 2
12345678901234567890123456789012123456789012345678901 2
12345678901234567890123456789012123456789012345678901 2
factorial, you take the initial number (say 5!) and multiply it by all integers
12345678901234567890123456789012123456789012345678901 2
12345678901234567890123456789012123456789012345678901
between 5 and 1. 2
12345678901234567890123456789012123456789012345678901 2
12345678901234567890123456789012123456789012345678901
Sample factorials: 2
12345678901234567890123456789012123456789012345678901 2
1 2
12345678901234567890123456789012123456789012345678901 2
2345678901234567890123456789012123456789012345678901
5! 5 5 3 4 3 3 3 2 3 1 5 120
12345678901234567890123456789012123456789012345678901 2
1 3! 5 3 3 2 3 1 5 6 2
12345678901234567890123456789012123456789012345678901 2
12345678901234567890123456789012123456789012345678901
or
2345678901234567890123456789012123456789012345678901
2
x! 5 x 3 (x 2 1) 3 (x 2 2) 3 . . . (It goes on until the last term is 1.) 2
12345678901234567890123456789012123456789012345678901
12345678901234567890123456789012123456789012345678901 2
1 2
12345678901234567890123456789012123456789012345678901 2
12345678901234567890123456789012123456789012345678901 2
12345678901234567890123456789012123456789012345678901 2
2345678901234567890123456789012123456789012345678901
Note

12345678901234567890123456789012123456789012345678901 2 2
1 You can only take the factorial of an integer greater than 1.
12345678901234567890123456789012123456789012345678901 2
12345678901234567890123456789012123456789012345678901 2
12345678901234567890123456789012123456789012345678901 2
12345678901234567890123456789012123456789012345678901 2
12345678901234567890123456789012123456789012345678901 2
12345678901234567890123456789012123456789012345678901 2
12345678901234567890123456789012123456789012345678901 2
2345678901234567890123456789012123456789012345678901
12345678901234567890123456789012123456789012345678901 x! 2
12345678901234567890123456789012123456789012345678901
If x is two greater than y, how can you simplify ? 2
12345678901234567890123456789012123456789012345678901 y! 2
12345678901234567890123456789012123456789012345678901 2
1 A. y 3
1 3y 2
1 2y 2
12345678901234567890123456789012123456789012345678901 2
2345678901234567890123456789012123456789012345678901
2
12345678901234567890123456789012123456789012345678901 2
12345678901234567890123456789012123456789012345678901
y 2
2
12345678901234567890123456789012123456789012345678901
B.
2
1 x 2
2345678901234567890123456789012123456789012345678901
12345678901234567890123456789012123456789012345678901 2
1 2 2 2
12345678901234567890123456789012123456789012345678901
C. y 1 3xy 1 2xy 2
12345678901234567890123456789012123456789012345678901 2
12345678901234567890123456789012123456789012345678901
D. (y 1 2)(y 1 1)
2345678901234567890123456789012123456789012345678901
2
2
1
12345678901234567890123456789012123456789012345678901
E. (y 2 2)(y 2 1) 2
12345678901234567890123456789012123456789012345678901 2 2
112 12345678901234567890123456789012123456789012345678901
123456789012345678901234567890121234567890123456789012

www.petersons.com
Chapter 4: Quantitative Review

123456789012345678901234567890121234567890123456789012
12345678901234567890123456789012123456789012345678901 2
12345678901234567890123456789012123456789012345678901 2
12345678901234567890123456789012123456789012345678901
Since x 5 y 1 2, then 2
12345678901234567890123456789012123456789012345678901 2
12345678901234567890123456789012123456789012345678901 2
12345678901234567890123456789012123456789012345678901
x! ~y 1 2!! ~y 1 2!~y 1 1!~y!! 2
12345678901234567890123456789012123456789012345678901
5 ~y 1 2!~y 1 1!.
2345678901234567890123456789012123456789012345678901
2
2
1 y!
5
y!
5
y!
12345678901234567890123456789012123456789012345678901 2
12345678901234567890123456789012123456789012345678901 2
12345678901234567890123456789012123456789012345678901
Factorials problems are pretty simple once you get over the freak-out 2
12345678901234567890123456789012123456789012345678901 2
12345678901234567890123456789012123456789012345678901
element of seeing an exclamation point. 2
12345678901234567890123456789012123456789012345678901 2
12345678901234567890123456789012123456789012345678901 2
12345678901234567890123456789012123456789012345678901 2
12345678901234567890123456789012123456789012345678901
2345678901234567890123456789012123456789012345678901
2
2
1Word Problems
12345678901234567890123456789012123456789012345678901 2
12345678901234567890123456789012123456789012345678901 2
12345678901234567890123456789012123456789012345678901
Word problems are common two-tier questions that prompt you to first 2
12345678901234567890123456789012123456789012345678901
2345678901234567890123456789012123456789012345678901
2
2
1 translate English into algebra and then solve the algebra. The simplest way
12345678901234567890123456789012123456789012345678901 2
12345678901234567890123456789012123456789012345678901
to make a word problem harder is to make the translation from English to 2
12345678901234567890123456789012123456789012345678901 2
12345678901234567890123456789012123456789012345678901
algebra more difficult. The actual algebra that you will have to do on a 2
2345678901234567890123456789012123456789012345678901
500-level word problem and 700-level word problem is going to be similar, 2
12345678901234567890123456789012123456789012345678901
12345678901234567890123456789012123456789012345678901 2

Take It to the Next Level


12345678901234567890123456789012123456789012345678901
but getting to the algebra becomes more of a challenge. 2
12345678901234567890123456789012123456789012345678901 22
12345678901234567890123456789012123456789012345678901 2
12345678901234567890123456789012123456789012345678901
1 Here’s a classic word problem set-up: 2
12345678901234567890123456789012123456789012345678901 2
12345678901234567890123456789012123456789012345678901
How many liters of a solution that is 15 percent water by volume have 2
12345678901234567890123456789012123456789012345678901
to be added to 3 liters of a solution that is 30 percent water by volume 2
12345678901234567890123456789012123456789012345678901 2
12345678901234567890123456789012123456789012345678901 2
12345678901234567890123456789012123456789012345678901
to create a solution that is 20 percent water by volume? 2
12345678901234567890123456789012123456789012345678901 2
12345678901234567890123456789012123456789012345678901 2
12345678901234567890123456789012123456789012345678901 22
2345678901234567890123456789012123456789012345678901
All that was a quite a tangle. To untangle it, there is a basic form to follow
12345678901234567890123456789012123456789012345678901
for mixture problems. It’s called the balance method since you are 2
12345678901234567890123456789012123456789012345678901
12345678901234567890123456789012123456789012345678901 2
1 balancing the left and right side of the equation. On the left side, put the 2
2345678901234567890123456789012123456789012345678901
12345678901234567890123456789012123456789012345678901
original solution: 2
12345678901234567890123456789012123456789012345678901 2
1 2
12345678901234567890123456789012123456789012345678901
Number of original liters (original% 2 final%) 5 3(30 2 20) 2
12345678901234567890123456789012123456789012345678901
2345678901234567890123456789012123456789012345678901
2
2
12345678901234567890123456789012123456789012345678901
12345678901234567890123456789012123456789012345678901
And on the right side put the new solution, 2
1 2
12345678901234567890123456789012123456789012345678901 2
12345678901234567890123456789012123456789012345678901
n(final% 2 added%) 5 n(20 2 15) 2
12345678901234567890123456789012123456789012345678901 22
2345678901234567890123456789012123456789012345678901
12345678901234567890123456789012123456789012345678901
1 where n is the number of liters of the 15% solution. Now just solve for n: 2
12345678901234567890123456789012123456789012345678901 2
12345678901234567890123456789012123456789012345678901
3~30 2 20! 5 n~20 2 15! → 2
12345678901234567890123456789012123456789012345678901 2
12345678901234567890123456789012123456789012345678901 2
12345678901234567890123456789012123456789012345678901
90 2 60 5 n~5! → 2
12345678901234567890123456789012123456789012345678901 2
12345678901234567890123456789012123456789012345678901
30 5 5n 2
12345678901234567890123456789012123456789012345678901 22
2345678901234567890123456789012123456789012345678901
12345678901234567890123456789012123456789012345678901
30 4 5 5 5n 4 5 → 2
12345678901234567890123456789012123456789012345678901 2
12345678901234567890123456789012123456789012345678901 2
1 6 5 n
12345678901234567890123456789012123456789012345678901 22
2345678901234567890123456789012123456789012345678901
12345678901234567890123456789012123456789012345678901
Take your time to muck through the English of a word problem and you 2
12345678901234567890123456789012123456789012345678901 2
12345678901234567890123456789012123456789012345678901 2
1 will end up with a perfectly solvable algebraic equation.
12345678901234567890123456789012123456789012345678901 22
2345678901234567890123456789012123456789012345678901
1
12345678901234567890123456789012123456789012345678901 2
12345678901234567890123456789012123456789012345678901 2
12345678901234567890123456789012123456789012345678901 2
12345678901234567890123456789012123456789012345678901 2
12345678901234567890123456789012123456789012345678901 2
12345678901234567890123456789012123456789012345678901 2
12345678901234567890123456789012123456789012345678901 2 113
123456789012345678901234567890121234567890123456789012
Part III: Section Review

123456789012345678901234567890121234567890123456789012
12345678901234567890123456789012123456789012345678901 2
12345678901234567890123456789012123456789012345678901 2
12345678901234567890123456789012123456789012345678901
Advanced Statistics 2
12345678901234567890123456789012123456789012345678901 2
12345678901234567890123456789012123456789012345678901 2
12345678901234567890123456789012123456789012345678901 2
There are some interesting ways to manipulate statistical notions like
12345678901234567890123456789012123456789012345678901
average or median. Harder questions on the GRE math section will expect 2
12345678901234567890123456789012123456789012345678901 2
12345678901234567890123456789012123456789012345678901
you to know the basic set of statistic notions—covered on pages 76–78 in 2
12345678901234567890123456789012123456789012345678901 2
12345678901234567890123456789012123456789012345678901
this chapter—and how to use them creatively to problem solve. 2
12345678901234567890123456789012123456789012345678901 2
1 For example, you commonly employ the concept of mean to find the 2
2345678901234567890123456789012123456789012345678901
12345678901234567890123456789012123456789012345678901 2
12345678901234567890123456789012123456789012345678901 2
12345678901234567890123456789012123456789012345678901
average of a set of numbers, but suppose you were asked for the sum of 2
12345678901234567890123456789012123456789012345678901 2
12345678901234567890123456789012123456789012345678901
the set of numbers instead? If 18 is the average of a set of numbers that 2
12345678901234567890123456789012123456789012345678901
has 10 members, then can you find the sum of the set of numbers? To do 2
12345678901234567890123456789012123456789012345678901 2
12345678901234567890123456789012123456789012345678901 2
12345678901234567890123456789012123456789012345678901
this, you have to multiply the mean average
2345678901234567890123456789012123456789012345678901
by the number of terms, 2
2
1 18 3 10 5 180.
12345678901234567890123456789012123456789012345678901 2
12345678901234567890123456789012123456789012345678901 2
12345678901234567890123456789012123456789012345678901 2
12345678901234567890123456789012123456789012345678901
That’s simple enough, but it gets trickier. Suppose you are asked, “What is 2
2345678901234567890123456789012123456789012345678901
the sum of the integers 20 to 50 inclusive?” Actually adding all the terms 2
12345678901234567890123456789012123456789012345678901
12345678901234567890123456789012123456789012345678901 2
12345678901234567890123456789012123456789012345678901 2
12345678901234567890123456789012123456789012345678901 2
up can’t be the best way to solve this problem because that would take all
day. You could do it, but there has to be a simpler way. The shortcut 2
12345678901234567890123456789012123456789012345678901
12345678901234567890123456789012123456789012345678901 2
1 method here is that the sum of consecutive numbers is equal to the average 2
12345678901234567890123456789012123456789012345678901 2
12345678901234567890123456789012123456789012345678901
of the numbers times the number of terms: 2
12345678901234567890123456789012123456789012345678901 2
2345678901234567890123456789012123456789012345678901
12345678901234567890123456789012123456789012345678901 2
12345678901234567890123456789012123456789012345678901
sum 5 (average) 3 (number of terms) 2
12345678901234567890123456789012123456789012345678901 2
12345678901234567890123456789012123456789012345678901 2
12345678901234567890123456789012123456789012345678901
In our problem, the average is 35 (halfway between 20 and 50), so 2
12345678901234567890123456789012123456789012345678901 2
12345678901234567890123456789012123456789012345678901 2
12345678901234567890123456789012123456789012345678901
sum 5 (35) 3 (31) 5 1,085 2
12345678901234567890123456789012123456789012345678901 2
1 The notion of weighted averages can make finding the average of a 2
12345678901234567890123456789012123456789012345678901 2
12345678901234567890123456789012123456789012345678901
number set a little more interesting. A weighted average is when certain 2
12345678901234567890123456789012123456789012345678901 2
12345678901234567890123456789012123456789012345678901
terms in a set are counted more (weighted more) than other terms in 2
12345678901234567890123456789012123456789012345678901
2345678901234567890123456789012123456789012345678901
2
2
12345678901234567890123456789012123456789012345678901
the set.
12345678901234567890123456789012123456789012345678901 2 2
1
12345678901234567890123456789012123456789012345678901 2
12345678901234567890123456789012123456789012345678901
In a certain class, the average boys’ score is 50 and the average girls’ 2
2345678901234567890123456789012123456789012345678901
12345678901234567890123456789012123456789012345678901 2
12345678901234567890123456789012123456789012345678901
score is 47. If there are twice as many girls in the class as boys, 2
1 2
12345678901234567890123456789012123456789012345678901 2
what is the class average?
12345678901234567890123456789012123456789012345678901 2
12345678901234567890123456789012123456789012345678901
A. 49.5 2
12345678901234567890123456789012123456789012345678901 2
12345678901234567890123456789012123456789012345678901
B. 49 2
12345678901234567890123456789012123456789012345678901 2
12345678901234567890123456789012123456789012345678901
C. 48.5 2
12345678901234567890123456789012123456789012345678901 2
2345678901234567890123456789012123456789012345678901
2
12345678901234567890123456789012123456789012345678901
D. 48
12345678901234567890123456789012123456789012345678901 2 2
12345678901234567890123456789012123456789012345678901
E. 47.5
2
1
2345678901234567890123456789012123456789012345678901
12345678901234567890123456789012123456789012345678901
There are not equal amounts of girls and boys, so you have to weight their 2
2
12345678901234567890123456789012123456789012345678901
averages to get to the overall average. Since there are twice as many girls as 2
12345678901234567890123456789012123456789012345678901
12345678901234567890123456789012123456789012345678901 2
1 boys, there are twice as many terms that go into the 47 average. The 2
12345678901234567890123456789012123456789012345678901 2
2345678901234567890123456789012123456789012345678901
2
1
12345678901234567890123456789012123456789012345678901 2
12345678901234567890123456789012123456789012345678901 2
12345678901234567890123456789012123456789012345678901 2
12345678901234567890123456789012123456789012345678901 2
12345678901234567890123456789012123456789012345678901 2
12345678901234567890123456789012123456789012345678901 2 2
114 12345678901234567890123456789012123456789012345678901
123456789012345678901234567890121234567890123456789012

www.petersons.com
Chapter 4: Quantitative Review

123456789012345678901234567890121234567890123456789012
12345678901234567890123456789012123456789012345678901 2
12345678901234567890123456789012123456789012345678901 2
12345678901234567890123456789012123456789012345678901
simplest way is to assume a three-person class where there is one boy and 2
12345678901234567890123456789012123456789012345678901 2
12345678901234567890123456789012123456789012345678901
two girls. The boy scored a 47 and the girls scored 50s, so 2
12345678901234567890123456789012123456789012345678901 2
12345678901234567890123456789012123456789012345678901 2
12345678901234567890123456789012123456789012345678901
1 3 50 1 2 3 47 144 2
12345678901234567890123456789012123456789012345678901
average 5 5 5 48 2
12345678901234567890123456789012123456789012345678901
3 3 2
12345678901234567890123456789012123456789012345678901 2
12345678901234567890123456789012123456789012345678901
2345678901234567890123456789012123456789012345678901
2
2
1 The denominator is 3 because there are three terms in this number set (50,
12345678901234567890123456789012123456789012345678901 2
12345678901234567890123456789012123456789012345678901
47, and 47) to reflect the mythical three people in the classroom. 2
12345678901234567890123456789012123456789012345678901 2
12345678901234567890123456789012123456789012345678901
2345678901234567890123456789012123456789012345678901
2
2
1 Another variation on the average is when you start with a number set, add
12345678901234567890123456789012123456789012345678901 2
12345678901234567890123456789012123456789012345678901
or subtract a term, and then must find the new average. 2
12345678901234567890123456789012123456789012345678901 2
12345678901234567890123456789012123456789012345678901
2345678901234567890123456789012123456789012345678901
2
2
1 Column A Column B
12345678901234567890123456789012123456789012345678901 2
12345678901234567890123456789012123456789012345678901 2
12345678901234567890123456789012123456789012345678901
The mean average of 4 numbers is 26.
2
12345678901234567890123456789012123456789012345678901 2
12345678901234567890123456789012123456789012345678901 22
2345678901234567890123456789012123456789012345678901
One number is taken from the set,
12345678901234567890123456789012123456789012345678901

Take It to the Next Level


and the new mean is 23.
12345678901234567890123456789012123456789012345678901 22
12345678901234567890123456789012123456789012345678901
12345678901234567890123456789012123456789012345678901
The number taken The largest number 2
12345678901234567890123456789012123456789012345678901 2
1 from the set left in the set 2
12345678901234567890123456789012123456789012345678901 2
12345678901234567890123456789012123456789012345678901 2
12345678901234567890123456789012123456789012345678901
Start with Column A unless you see something that makes you feel 2
2345678901234567890123456789012123456789012345678901
Column B is simple. For Column A, the number taken must be the 2
12345678901234567890123456789012123456789012345678901
12345678901234567890123456789012123456789012345678901 2
12345678901234567890123456789012123456789012345678901 2
12345678901234567890123456789012123456789012345678901 22
difference between the sums of the two sets:
12345678901234567890123456789012123456789012345678901 2
12345678901234567890123456789012123456789012345678901
(original sum) 2 (new sum)
12345678901234567890123456789012123456789012345678901 2
12345678901234567890123456789012123456789012345678901 2
12345678901234567890123456789012123456789012345678901
(26 3 4) 2 (23 3 3) 5 2 15 2
1 2
2345678901234567890123456789012123456789012345678901
For Column A, you know that 215 was taken from the number set. Think 2
12345678901234567890123456789012123456789012345678901
12345678901234567890123456789012123456789012345678901 2
1 about how this number is much lower than the average after it left and 2
12345678901234567890123456789012123456789012345678901 2
12345678901234567890123456789012123456789012345678901
before it left. 215 was obviously one of the lower numbers in the set, and 2
12345678901234567890123456789012123456789012345678901 2
12345678901234567890123456789012123456789012345678901 2
12345678901234567890123456789012123456789012345678901 2
the higher numbers brought the average up to 26 and then up to 23. It
12345678901234567890123456789012123456789012345678901
makes sense that Column B definitely contains a number larger than 215, 2
12345678901234567890123456789012123456789012345678901 2
2345678901234567890123456789012123456789012345678901
12345678901234567890123456789012123456789012345678901 2
12345678901234567890123456789012123456789012345678901
so choice (B) is the correct answer for this average problem. 2
1 2
12345678901234567890123456789012123456789012345678901
Compound interest problems involve a loaded math concept. Simple 2
12345678901234567890123456789012123456789012345678901 2
12345678901234567890123456789012123456789012345678901
interest is when the interest applies only to the principal over a given 2
12345678901234567890123456789012123456789012345678901 2
12345678901234567890123456789012123456789012345678901
period. If a $10,000 account has a 5 percent simple interest rate 2
12345678901234567890123456789012123456789012345678901 2
12345678901234567890123456789012123456789012345678901
compounded annually, then after six months the interest will be: 2
12345678901234567890123456789012123456789012345678901 22
2345678901234567890123456789012123456789012345678901
12345678901234567890123456789012123456789012345678901 2
12345678901234567890123456789012123456789012345678901
principal 3 rate 3 time 5 10,000 3 0.05 3 0.5
2
12345678901234567890123456789012123456789012345678901 2
1 5 5,000 3 0.05
2345678901234567890123456789012123456789012345678901
12345678901234567890123456789012123456789012345678901 2
12345678901234567890123456789012123456789012345678901
5 250 2
12345678901234567890123456789012123456789012345678901 22
12345678901234567890123456789012123456789012345678901
1 The first number is the cash, the second value is the percent interest 2
12345678901234567890123456789012123456789012345678901
expressed in decimal form as 0.05, and the 0.5 at the end shows that you 2
12345678901234567890123456789012123456789012345678901 2
12345678901234567890123456789012123456789012345678901
are halfway (six months) through the annual interest period. 2
12345678901234567890123456789012123456789012345678901 2
12345678901234567890123456789012123456789012345678901 2
12345678901234567890123456789012123456789012345678901
Things get a little more complicated with compounding interest. Under 2
12345678901234567890123456789012123456789012345678901 2
12345678901234567890123456789012123456789012345678901
this format, the interest rate is applied to the principal and to the 2
12345678901234567890123456789012123456789012345678901 2 115
123456789012345678901234567890121234567890123456789012
Part III: Section Review

123456789012345678901234567890121234567890123456789012
12345678901234567890123456789012123456789012345678901 2
12345678901234567890123456789012123456789012345678901 2
12345678901234567890123456789012123456789012345678901
accumulated interest. There is a complicated formula for compound 2
12345678901234567890123456789012123456789012345678901 2
12345678901234567890123456789012123456789012345678901
interest, but you can do without it. If you come across a compound interest 2
12345678901234567890123456789012123456789012345678901 2
12345678901234567890123456789012123456789012345678901
problem, simply calculate the simple interest for the scenario, and the 2
1 compound interest will be a little more than this. There is typically a small 2
2345678901234567890123456789012123456789012345678901
12345678901234567890123456789012123456789012345678901 2
12345678901234567890123456789012123456789012345678901 2
12345678901234567890123456789012123456789012345678901
difference between the two because the interest accrued on the interest will 2
12345678901234567890123456789012123456789012345678901
2345678901234567890123456789012123456789012345678901
2
2
1 be small.
12345678901234567890123456789012123456789012345678901 2
12345678901234567890123456789012123456789012345678901 2
12345678901234567890123456789012123456789012345678901 2
12345678901234567890123456789012123456789012345678901 2

Alert!
12345678901234567890123456789012123456789012345678901
Supposedly, many savings accounts at banks work on the concept of 2
12345678901234567890123456789012123456789012345678901 2
12345678901234567890123456789012123456789012345678901
compound interest, but like many Americans, the author has never 2
12345678901234567890123456789012123456789012345678901 2
12345678901234567890123456789012123456789012345678901
experienced this fact firsthand. The author
2345678901234567890123456789012123456789012345678901
is, however, well aware of the 2
2
1 interest formulas used by credit card companies.
12345678901234567890123456789012123456789012345678901 2
12345678901234567890123456789012123456789012345678901 2
12345678901234567890123456789012123456789012345678901 2
12345678901234567890123456789012123456789012345678901 2
12345678901234567890123456789012123456789012345678901 2
2345678901234567890123456789012123456789012345678901
12345678901234567890123456789012123456789012345678901 2
Geometry
12345678901234567890123456789012123456789012345678901
To double the geometric fun (and the difficulty), harder geometry 2
2
12345678901234567890123456789012123456789012345678901
12345678901234567890123456789012123456789012345678901
problems will include multiple geometric shapes. A square might be 2
2
12345678901234567890123456789012123456789012345678901
1 inscribed in a circle. A triangle might be located inside a trapezoid. 2
12345678901234567890123456789012123456789012345678901 2
12345678901234567890123456789012123456789012345678901 2
12345678901234567890123456789012123456789012345678901
Regardless of the particulars, the idea behind increasing the difficulty is the 2
2345678901234567890123456789012123456789012345678901
same. Things look cluttered when there are multiple figures, and that 2
12345678901234567890123456789012123456789012345678901
12345678901234567890123456789012123456789012345678901 2
12345678901234567890123456789012123456789012345678901 2
12345678901234567890123456789012123456789012345678901 2
clutter can distract you from problem-solving tasks. You combat the
12345678901234567890123456789012123456789012345678901 2
12345678901234567890123456789012123456789012345678901
clutter by: 2
12345678901234567890123456789012123456789012345678901 2
12345678901234567890123456789012123456789012345678901 2
1. First determining exactly what you are trying to find
12345678901234567890123456789012123456789012345678901 2
1 2
2345678901234567890123456789012123456789012345678901
12345678901234567890123456789012123456789012345678901
2. Considering each of the figures separately 2
12345678901234567890123456789012123456789012345678901 2
1 2
12345678901234567890123456789012123456789012345678901
3. Trying to find the relationships between the figures that will help 2
12345678901234567890123456789012123456789012345678901 2
12345678901234567890123456789012123456789012345678901
solve the problem 2
12345678901234567890123456789012123456789012345678901 2
12345678901234567890123456789012123456789012345678901 2
12345678901234567890123456789012123456789012345678901
All three steps are important, but the third step is the one that holds the 2
12345678901234567890123456789012123456789012345678901 2
12345678901234567890123456789012123456789012345678901 2
2345678901234567890123456789012123456789012345678901
key to most of these questions. To try out this methodology, consider a
12345678901234567890123456789012123456789012345678901 2
1 square inscribed in a circle like the following: 2
12345678901234567890123456789012123456789012345678901 2
12345678901234567890123456789012123456789012345678901 2
12345678901234567890123456789012123456789012345678901 2
12345678901234567890123456789012123456789012345678901 2
12345678901234567890123456789012123456789012345678901 2
12345678901234567890123456789012123456789012345678901 2
12345678901234567890123456789012123456789012345678901 2
12345678901234567890123456789012123456789012345678901 2
2345678901234567890123456789012123456789012345678901
12345678901234567890123456789012123456789012345678901 2
12345678901234567890123456789012123456789012345678901 2
12345678901234567890123456789012123456789012345678901 2 2
1
12345678901234567890123456789012123456789012345678901
Your goal is to find the radius of the circle, but all you have been given is 2
12345678901234567890123456789012123456789012345678901 2
12345678901234567890123456789012123456789012345678901
the area of the square. First, locate the radius of the circle. If you think 2
12345678901234567890123456789012123456789012345678901 2
12345678901234567890123456789012123456789012345678901
about it, you can see that the diagonal of the square is also the diameter of 2
12345678901234567890123456789012123456789012345678901 2
2345678901234567890123456789012123456789012345678901
1 the circle, and the radius is always half a circle’s diameter. So half of the 2
12345678901234567890123456789012123456789012345678901 2
12345678901234567890123456789012123456789012345678901
diagonal of the square will equal the radius. Second, you can use the area 2
12345678901234567890123456789012123456789012345678901 2
12345678901234567890123456789012123456789012345678901
formula of a square to figure out the length of the sides. If area is 9, then 2
12345678901234567890123456789012123456789012345678901 2
12345678901234567890123456789012123456789012345678901
the sides are all length 3. 2
2
116 12345678901234567890123456789012123456789012345678901
123456789012345678901234567890121234567890123456789012

www.petersons.com
Chapter 4: Quantitative Review

123456789012345678901234567890121234567890123456789012
12345678901234567890123456789012123456789012345678901 2
12345678901234567890123456789012123456789012345678901 2
12345678901234567890123456789012123456789012345678901
Now imagine the diagonal. If you draw it out, you would see yourself 2
12345678901234567890123456789012123456789012345678901 2
12345678901234567890123456789012123456789012345678901
making two 45-45-90 right triangles out of the square. Finding the 2
12345678901234567890123456789012123456789012345678901 2
12345678901234567890123456789012123456789012345678901
hypotenuse of one these triangles is equal to finding the square’s diagonal, 2
1 and that is equal to the circle’s diameter. Follow all the steps (try it on your 2
2345678901234567890123456789012123456789012345678901
12345678901234567890123456789012123456789012345678901 2
12345678901234567890123456789012123456789012345678901 2
12345678901234567890123456789012123456789012345678901 3=2 2
12345678901234567890123456789012123456789012345678901
own), and you should end up with a radius
2345678901234567890123456789012123456789012345678901 of . 2
2
1 2
12345678901234567890123456789012123456789012345678901 2
12345678901234567890123456789012123456789012345678901 2
12345678901234567890123456789012123456789012345678901
To help you recognize the relationship that two or more figures share, ask 2
12345678901234567890123456789012123456789012345678901
2345678901234567890123456789012123456789012345678901
2
2
1 yourself questions like, “Do they share a side, or an angle?” or “What is
12345678901234567890123456789012123456789012345678901 2
12345678901234567890123456789012123456789012345678901
the relationship between their areas?” Once you have picked out the 2
12345678901234567890123456789012123456789012345678901 2
12345678901234567890123456789012123456789012345678901
relationship that links the two figures together,
2345678901234567890123456789012123456789012345678901
the actual geometry is often 2
2
1 no harder than what is covered in the first section.
12345678901234567890123456789012123456789012345678901 2
12345678901234567890123456789012123456789012345678901 2
12345678901234567890123456789012123456789012345678901 2
12345678901234567890123456789012123456789012345678901
Column A Column B 2
12345678901234567890123456789012123456789012345678901 22
2345678901234567890123456789012123456789012345678901
12345678901234567890123456789012123456789012345678901 2

Take It to the Next Level


12345678901234567890123456789012123456789012345678901 2
12345678901234567890123456789012123456789012345678901 2
12345678901234567890123456789012123456789012345678901 2
12345678901234567890123456789012123456789012345678901 2
1
12345678901234567890123456789012123456789012345678901 2
12345678901234567890123456789012123456789012345678901 2
12345678901234567890123456789012123456789012345678901 2
12345678901234567890123456789012123456789012345678901 22
2345678901234567890123456789012123456789012345678901
12345678901234567890123456789012123456789012345678901 2
12345678901234567890123456789012123456789012345678901 2
12345678901234567890123456789012123456789012345678901 2
12345678901234567890123456789012123456789012345678901 2
12345678901234567890123456789012123456789012345678901 2
12345678901234567890123456789012123456789012345678901 2
12345678901234567890123456789012123456789012345678901
The difference of the areas of trapezoid
12345678901234567890123456789012123456789012345678901 2
1 ABDH and trapezoid FECJ is 15. 2
12345678901234567890123456789012123456789012345678901 22
2345678901234567890123456789012123456789012345678901
12345678901234567890123456789012123456789012345678901 2
1 The area of the The area of the
12345678901234567890123456789012123456789012345678901 2
12345678901234567890123456789012123456789012345678901
dotted region striped region 2
12345678901234567890123456789012123456789012345678901 22
2345678901234567890123456789012123456789012345678901
12345678901234567890123456789012123456789012345678901
1 Locate the two trapezoids. How are they geometrically related? Both share 2
12345678901234567890123456789012123456789012345678901
the striped region and they have the same height, 5. Now you can try to 2
12345678901234567890123456789012123456789012345678901 2
2345678901234567890123456789012123456789012345678901
compare the striped region and the dotted region. Can you determine the 2
12345678901234567890123456789012123456789012345678901
12345678901234567890123456789012123456789012345678901 2
1 area of the dotted region? Yes, since both are rectangles with length and 2
12345678901234567890123456789012123456789012345678901 2
12345678901234567890123456789012123456789012345678901
width readily obvious. However, you cannot determine the area of the 2
12345678901234567890123456789012123456789012345678901 2
12345678901234567890123456789012123456789012345678901
striped region, since there’s no way to figure out its width. The difference 2
12345678901234567890123456789012123456789012345678901 2
12345678901234567890123456789012123456789012345678901
in areas of the trapezoids does not help us, for when the areas of the 2
12345678901234567890123456789012123456789012345678901 2
12345678901234567890123456789012123456789012345678901
trapezoid are subtracted, the area of the striped region in each just cancels 2
12345678901234567890123456789012123456789012345678901 2
12345678901234567890123456789012123456789012345678901
out. Those triangles on both ends of the figure hold an unknown amount 2
12345678901234567890123456789012123456789012345678901 2
12345678901234567890123456789012123456789012345678901
of area for both trapezoids, making any attempt to subtract one 2
2345678901234567890123456789012123456789012345678901
12345678901234567890123456789012123456789012345678901
trapezoid’s area from the other impossible. Therefore, (D) is the correct 2
2
12345678901234567890123456789012123456789012345678901 2
12345678901234567890123456789012123456789012345678901 2
12345678901234567890123456789012123456789012345678901
answer.
2
1
12345678901234567890123456789012123456789012345678901 22
2345678901234567890123456789012123456789012345678901
1
12345678901234567890123456789012123456789012345678901 2
12345678901234567890123456789012123456789012345678901 2
12345678901234567890123456789012123456789012345678901 2
12345678901234567890123456789012123456789012345678901 2
12345678901234567890123456789012123456789012345678901 2
12345678901234567890123456789012123456789012345678901 2
12345678901234567890123456789012123456789012345678901 2 117
123456789012345678901234567890121234567890123456789012
Part III: Section Review

123456789012345678901234567890121234567890123456789012
12345678901234567890123456789012123456789012345678901 2
12345678901234567890123456789012123456789012345678901 2
12345678901234567890123456789012123456789012345678901
Coordinate Geometry 2
12345678901234567890123456789012123456789012345678901 2
12345678901234567890123456789012123456789012345678901 2
12345678901234567890123456789012123456789012345678901 2
Reach back into your past and dust off the equation y 5 mx 1 b. This
12345678901234567890123456789012123456789012345678901
formula can be used to describe a straight line in a coordinate plane. The y 2
12345678901234567890123456789012123456789012345678901 2
12345678901234567890123456789012123456789012345678901
and the x variables stand the value of y and x, respectively, of any point on 2
12345678901234567890123456789012123456789012345678901 2
12345678901234567890123456789012123456789012345678901
a particular line. m is the slope of the line and b is the y-intercept (where 2
12345678901234567890123456789012123456789012345678901 2
12345678901234567890123456789012123456789012345678901
the line crosses the y-axis). The beauty of this equation is that if you are 2
12345678901234567890123456789012123456789012345678901 2
12345678901234567890123456789012123456789012345678901
given some of the parts—such as the slope and one point on the line—there 2
12345678901234567890123456789012123456789012345678901 2
12345678901234567890123456789012123456789012345678901
is a way to figure out all the other parts of the equation.
2345678901234567890123456789012123456789012345678901
2
2
1
12345678901234567890123456789012123456789012345678901
To be able to use this formula and just read off the slope or the y-intercept, 2
12345678901234567890123456789012123456789012345678901 2
12345678901234567890123456789012123456789012345678901 2
12345678901234567890123456789012123456789012345678901
the equation has to be in the y 5 mx 1 b form, with y by itself on one side, 2
1 and the x on the other side summed with b. If you are given a linear 2
2345678901234567890123456789012123456789012345678901
12345678901234567890123456789012123456789012345678901 2
12345678901234567890123456789012123456789012345678901 2
12345678901234567890123456789012123456789012345678901
equation of a line in the form 2y 1 5x 5 b, you would have to do some 2
12345678901234567890123456789012123456789012345678901 2
12345678901234567890123456789012123456789012345678901 2
2345678901234567890123456789012123456789012345678901
manipulating:
12345678901234567890123456789012123456789012345678901 2
12345678901234567890123456789012123456789012345678901 2
12345678901234567890123456789012123456789012345678901
2y 1 5x 5 23 → 2
12345678901234567890123456789012123456789012345678901 2
12345678901234567890123456789012123456789012345678901 2
2y 1 5x 2 5x 5 23 2 5x →
2
1
12345678901234567890123456789012123456789012345678901
2y 3 → 2
12345678901234567890123456789012123456789012345678901
5 25x 2 2
12345678901234567890123456789012123456789012345678901
→ 2
12345678901234567890123456789012123456789012345678901 2
2345678901234567890123456789012123456789012345678901
~ 2y ! 4 2 5 ~ 25x 2 3 ! 4 2
12345678901234567890123456789012123456789012345678901 2
12345678901234567890123456789012123456789012345678901
5 3 2
12345678901234567890123456789012123456789012345678901
y52 x2 2
12345678901234567890123456789012123456789012345678901
2 2 2
12345678901234567890123456789012123456789012345678901 2
12345678901234567890123456789012123456789012345678901 2
1
halves, S D
12345678901234567890123456789012123456789012345678901
12345678901234567890123456789012123456789012345678901
The slope is negative five 2
5
2
, and the y-intercept is negative 2
2
2
2345678901234567890123456789012123456789012345678901
12345678901234567890123456789012123456789012345678901 2
three halves, S D
2
3
12345678901234567890123456789012123456789012345678901
1 .
12345678901234567890123456789012123456789012345678901 2
2
2
2
12345678901234567890123456789012123456789012345678901
The slope equation works for a line—but nothing else. You won’t see 2
12345678901234567890123456789012123456789012345678901 2
12345678901234567890123456789012123456789012345678901 2
12345678901234567890123456789012123456789012345678901
many things graphed besides lines on the GRE, but you might see a 2
12345678901234567890123456789012123456789012345678901 2
12345678901234567890123456789012123456789012345678901 2
2
quadratic equation (an equation that has a squared term like y 5 x ). To
2345678901234567890123456789012123456789012345678901
make things more confusing, the y may be exchanged for an f(x) so it looks 2
12345678901234567890123456789012123456789012345678901
12345678901234567890123456789012123456789012345678901 2
1 like this: f(x) 5 x2. Then the whole thing is renamed a function. It is 2
12345678901234567890123456789012123456789012345678901 2
12345678901234567890123456789012123456789012345678901
simplest to just think of the f(x) as a more complicated way to write y. As 2
12345678901234567890123456789012123456789012345678901 2
12345678901234567890123456789012123456789012345678901 2
12345678901234567890123456789012123456789012345678901
you might imagine, the graph of this quadratic is not as simple as the graph 2
12345678901234567890123456789012123456789012345678901
of a line, but it has a basic form that simplifies it. 2
12345678901234567890123456789012123456789012345678901 2
12345678901234567890123456789012123456789012345678901 2
2345678901234567890123456789012123456789012345678901
12345678901234567890123456789012123456789012345678901 2
12345678901234567890123456789012123456789012345678901 2
12345678901234567890123456789012123456789012345678901 2 2
1
12345678901234567890123456789012123456789012345678901 2
12345678901234567890123456789012123456789012345678901 2
12345678901234567890123456789012123456789012345678901 2
12345678901234567890123456789012123456789012345678901 2
12345678901234567890123456789012123456789012345678901 2
12345678901234567890123456789012123456789012345678901 2
2345678901234567890123456789012123456789012345678901
2
1
12345678901234567890123456789012123456789012345678901 2
12345678901234567890123456789012123456789012345678901 2
12345678901234567890123456789012123456789012345678901 2
12345678901234567890123456789012123456789012345678901 2
12345678901234567890123456789012123456789012345678901 2
12345678901234567890123456789012123456789012345678901 2 2
118 12345678901234567890123456789012123456789012345678901
123456789012345678901234567890121234567890123456789012

www.petersons.com
Chapter 4: Quantitative Review

123456789012345678901234567890121234567890123456789012
12345678901234567890123456789012123456789012345678901 2
12345678901234567890123456789012123456789012345678901 2
12345678901234567890123456789012123456789012345678901 2
12345678901234567890123456789012123456789012345678901 2
12345678901234567890123456789012123456789012345678901 2
12345678901234567890123456789012123456789012345678901 2
12345678901234567890123456789012123456789012345678901 2
12345678901234567890123456789012123456789012345678901 2
12345678901234567890123456789012123456789012345678901 2
12345678901234567890123456789012123456789012345678901 2
12345678901234567890123456789012123456789012345678901 2
12345678901234567890123456789012123456789012345678901
2345678901234567890123456789012123456789012345678901
2
2
1
12345678901234567890123456789012123456789012345678901 2
12345678901234567890123456789012123456789012345678901 2
12345678901234567890123456789012123456789012345678901 2
12345678901234567890123456789012123456789012345678901
2345678901234567890123456789012123456789012345678901
2
2
1
12345678901234567890123456789012123456789012345678901 2
12345678901234567890123456789012123456789012345678901 2
12345678901234567890123456789012123456789012345678901 2
12345678901234567890123456789012123456789012345678901
2345678901234567890123456789012123456789012345678901
2
2
1
12345678901234567890123456789012123456789012345678901 2
12345678901234567890123456789012123456789012345678901 2
12345678901234567890123456789012123456789012345678901 2
12345678901234567890123456789012123456789012345678901 2
12345678901234567890123456789012123456789012345678901 22
2345678901234567890123456789012123456789012345678901
12345678901234567890123456789012123456789012345678901 2

Take It to the Next Level


12345678901234567890123456789012123456789012345678901 2
12345678901234567890123456789012123456789012345678901 2
12345678901234567890123456789012123456789012345678901 2
12345678901234567890123456789012123456789012345678901 2
1
12345678901234567890123456789012123456789012345678901 2
12345678901234567890123456789012123456789012345678901 2
12345678901234567890123456789012123456789012345678901
Each of these graphs shares a basic shape called a parabola. Examine the 2
2345678901234567890123456789012123456789012345678901
different graphs and you should see the logic of how they are graphed. For 2
12345678901234567890123456789012123456789012345678901
12345678901234567890123456789012123456789012345678901 2
12345678901234567890123456789012123456789012345678901
example, a negative squared term like f(x) 2
1 creates an upside- 2
12345678901234567890123456789012123456789012345678901 22
5 2x 1
12345678901234567890123456789012123456789012345678901
down hump. If there is an integer after the squared value, then this number
12345678901234567890123456789012123456789012345678901 2
is the value of y where the crest of the hump occurs. You could see this with 2
12345678901234567890123456789012123456789012345678901
12345678901234567890123456789012123456789012345678901 2
f(x) 5 2x2 1 1 with a crest at 1 and f(x) 5 x2 2 2 with a crest at y 5 22. 2
12345678901234567890123456789012123456789012345678901
1 2
12345678901234567890123456789012123456789012345678901 2
12345678901234567890123456789012123456789012345678901
The last coordinate geometry concept to cover is the distance formula. It 2
12345678901234567890123456789012123456789012345678901 2
12345678901234567890123456789012123456789012345678901
allows you to find the distance between two points on the coordinate plane 2
12345678901234567890123456789012123456789012345678901 2
12345678901234567890123456789012123456789012345678901
using the following formula: 2
12345678901234567890123456789012123456789012345678901 2
12345678901234567890123456789012123456789012345678901 2
12345678901234567890123456789012123456789012345678901
= ~ ! 2
~ ! 2 2
12345678901234567890123456789012123456789012345678901
d 5 x1 2 x2 1 y1 2 y2 2
12345678901234567890123456789012123456789012345678901 22
2345678901234567890123456789012123456789012345678901
12345678901234567890123456789012123456789012345678901
If you know two points, just plug in the x values and the y values into the
2
1
12345678901234567890123456789012123456789012345678901
equation and then solve for the distance. It’s that simple. 2
12345678901234567890123456789012123456789012345678901 2
12345678901234567890123456789012123456789012345678901 22
2345678901234567890123456789012123456789012345678901
12345678901234567890123456789012123456789012345678901 2
12345678901234567890123456789012123456789012345678901 2
Note

12345678901234567890123456789012123456789012345678901 2
1 If the distance formula reminds you of the Pythagorean theorem, you’re
12345678901234567890123456789012123456789012345678901 2
12345678901234567890123456789012123456789012345678901
quite observant, because the distance formula is based on it. 2
12345678901234567890123456789012123456789012345678901 2
12345678901234567890123456789012123456789012345678901 2
12345678901234567890123456789012123456789012345678901 2
12345678901234567890123456789012123456789012345678901 22
2345678901234567890123456789012123456789012345678901
12345678901234567890123456789012123456789012345678901 2
12345678901234567890123456789012123456789012345678901 2
12345678901234567890123456789012123456789012345678901 2
1
12345678901234567890123456789012123456789012345678901 2
12345678901234567890123456789012123456789012345678901 2
12345678901234567890123456789012123456789012345678901 2
12345678901234567890123456789012123456789012345678901 2
12345678901234567890123456789012123456789012345678901 2
12345678901234567890123456789012123456789012345678901 2
12345678901234567890123456789012123456789012345678901 2
12345678901234567890123456789012123456789012345678901 2
12345678901234567890123456789012123456789012345678901 2 119
123456789012345678901234567890121234567890123456789012
Part III: Section Review

123456789012345678901234567890121234567890123456789012
12345678901234567890123456789012123456789012345678901 2
12345678901234567890123456789012123456789012345678901 2
12345678901234567890123456789012123456789012345678901
If A(2, 24) and B(3, 6) are two points on a rectangular coordinate 2
12345678901234567890123456789012123456789012345678901 2
12345678901234567890123456789012123456789012345678901
system, what is the distance between A and B? 2
12345678901234567890123456789012123456789012345678901 2
12345678901234567890123456789012123456789012345678901 2
12345678901234567890123456789012123456789012345678901
A. 1 2
12345678901234567890123456789012123456789012345678901 2
12345678901234567890123456789012123456789012345678901
B. 10 2
12345678901234567890123456789012123456789012345678901 2
12345678901234567890123456789012123456789012345678901
C. = 101
2345678901234567890123456789012123456789012345678901
2
2
1
12345678901234567890123456789012123456789012345678901
D. 11 2
12345678901234567890123456789012123456789012345678901 2
12345678901234567890123456789012123456789012345678901
E. 15 2
12345678901234567890123456789012123456789012345678901
2345678901234567890123456789012123456789012345678901
2
2
1 Hauling out the distance formula:
12345678901234567890123456789012123456789012345678901 2
12345678901234567890123456789012123456789012345678901 2
12345678901234567890123456789012123456789012345678901 2
12345678901234567890123456789012123456789012345678901
d 5 =~x1 2 x2! 1 ~y1 2 y2! 5 =~2 2 3! 1 ~24 2 6!
2 2
2345678901234567890123456789012123456789012345678901
2 2 2
2
1
12345678901234567890123456789012123456789012345678901 2
12345678901234567890123456789012123456789012345678901
5 =1 1 100 2
12345678901234567890123456789012123456789012345678901 2
12345678901234567890123456789012123456789012345678901
= 2
12345678901234567890123456789012123456789012345678901 2
2345678901234567890123456789012123456789012345678901
5 101
12345678901234567890123456789012123456789012345678901 2
12345678901234567890123456789012123456789012345678901
This is choice (C). 2
12345678901234567890123456789012123456789012345678901 2
That might look like a big number, but it actually is just a little bit bigger 2
12345678901234567890123456789012123456789012345678901
12345678901234567890123456789012123456789012345678901 2
1 than 10. If you visualize where the two points are, this answer makes 2
12345678901234567890123456789012123456789012345678901 2
12345678901234567890123456789012123456789012345678901 2
12345678901234567890123456789012123456789012345678901
sense. 2
12345678901234567890123456789012123456789012345678901 2
2345678901234567890123456789012123456789012345678901
12345678901234567890123456789012123456789012345678901 2
12345678901234567890123456789012123456789012345678901 2
12345678901234567890123456789012123456789012345678901 2
12345678901234567890123456789012123456789012345678901
3-D Figures 2
12345678901234567890123456789012123456789012345678901
Another way to up the geometry ante is by upping the number of 2
2
12345678901234567890123456789012123456789012345678901
12345678901234567890123456789012123456789012345678901
dimensions from two to three. The 3-D figures that you will come across 2
2
12345678901234567890123456789012123456789012345678901
1 on the GRE are all built upon the figures that were covered in the first 2
12345678901234567890123456789012123456789012345678901 2
12345678901234567890123456789012123456789012345678901
section (rectangles, squares, circles, triangles). The 2-D figures are given 2
12345678901234567890123456789012123456789012345678901 2
12345678901234567890123456789012123456789012345678901 2
12345678901234567890123456789012123456789012345678901
height, so instead of dealing just with a square
2345678901234567890123456789012123456789012345678901
you now get a cube, a 3-D 2
2
12345678901234567890123456789012123456789012345678901
12345678901234567890123456789012123456789012345678901
figure composed of six squares. 2
1 2
12345678901234567890123456789012123456789012345678901 2
12345678901234567890123456789012123456789012345678901
If you need a refresher on the 2-D geometry, buzz through the geometry 2
2345678901234567890123456789012123456789012345678901
section earlier in this chapter. If not, go ahead and dive into the 3-D 2
12345678901234567890123456789012123456789012345678901
12345678901234567890123456789012123456789012345678901 2
1 geometry. 2
12345678901234567890123456789012123456789012345678901 2
12345678901234567890123456789012123456789012345678901 2
12345678901234567890123456789012123456789012345678901 2
12345678901234567890123456789012123456789012345678901 2
12345678901234567890123456789012123456789012345678901 2
12345678901234567890123456789012123456789012345678901 2
12345678901234567890123456789012123456789012345678901 2
12345678901234567890123456789012123456789012345678901 2
2345678901234567890123456789012123456789012345678901
12345678901234567890123456789012123456789012345678901 2
12345678901234567890123456789012123456789012345678901 2
12345678901234567890123456789012123456789012345678901 2 2
1
12345678901234567890123456789012123456789012345678901 2
12345678901234567890123456789012123456789012345678901 2
12345678901234567890123456789012123456789012345678901 2
12345678901234567890123456789012123456789012345678901 2
12345678901234567890123456789012123456789012345678901 2
12345678901234567890123456789012123456789012345678901 2
2345678901234567890123456789012123456789012345678901
2
1
12345678901234567890123456789012123456789012345678901 2
12345678901234567890123456789012123456789012345678901 2
12345678901234567890123456789012123456789012345678901 2
12345678901234567890123456789012123456789012345678901 2
12345678901234567890123456789012123456789012345678901 2
12345678901234567890123456789012123456789012345678901 2 2
120 12345678901234567890123456789012123456789012345678901
123456789012345678901234567890121234567890123456789012

www.petersons.com
Chapter 4: Quantitative Review

123456789012345678901234567890121234567890123456789012
12345678901234567890123456789012123456789012345678901 2
12345678901234567890123456789012123456789012345678901 2
12345678901234567890123456789012123456789012345678901 2
12345678901234567890123456789012123456789012345678901 2
12345678901234567890123456789012123456789012345678901 2
12345678901234567890123456789012123456789012345678901 2
12345678901234567890123456789012123456789012345678901 2
12345678901234567890123456789012123456789012345678901 2
12345678901234567890123456789012123456789012345678901 2
12345678901234567890123456789012123456789012345678901 2
12345678901234567890123456789012123456789012345678901 2
12345678901234567890123456789012123456789012345678901
2345678901234567890123456789012123456789012345678901
2
2
1
12345678901234567890123456789012123456789012345678901 2
12345678901234567890123456789012123456789012345678901 2
12345678901234567890123456789012123456789012345678901 2
12345678901234567890123456789012123456789012345678901
2345678901234567890123456789012123456789012345678901
2
2
1
12345678901234567890123456789012123456789012345678901 2
12345678901234567890123456789012123456789012345678901 2
12345678901234567890123456789012123456789012345678901 2
12345678901234567890123456789012123456789012345678901
2345678901234567890123456789012123456789012345678901
2
2
1
12345678901234567890123456789012123456789012345678901 2
12345678901234567890123456789012123456789012345678901 2
12345678901234567890123456789012123456789012345678901
The above figure is a rectangular solid. What is the shortest distance 2
12345678901234567890123456789012123456789012345678901 2
12345678901234567890123456789012123456789012345678901 22
2345678901234567890123456789012123456789012345678901
from A to F?
12345678901234567890123456789012123456789012345678901 2

Take It to the Next Level


12345678901234567890123456789012123456789012345678901
A. =18
12345678901234567890123456789012123456789012345678901 2
12345678901234567890123456789012123456789012345678901 22
12345678901234567890123456789012123456789012345678901
B. =20
2
1
12345678901234567890123456789012123456789012345678901 2
12345678901234567890123456789012123456789012345678901
C. =28 2
12345678901234567890123456789012123456789012345678901 2
2345678901234567890123456789012123456789012345678901
12345678901234567890123456789012123456789012345678901 2
12345678901234567890123456789012123456789012345678901
D. =34 2
12345678901234567890123456789012123456789012345678901 22
12345678901234567890123456789012123456789012345678901
12345678901234567890123456789012123456789012345678901
E. 6 2
12345678901234567890123456789012123456789012345678901 22
12345678901234567890123456789012123456789012345678901
The shortest distance between two points is a line, so the question is asking
12345678901234567890123456789012123456789012345678901 22
12345678901234567890123456789012123456789012345678901
1 you to find the distance of the line AF. If you want a hint about how to find
2
2345678901234567890123456789012123456789012345678901
12345678901234567890123456789012123456789012345678901
it, think triangles. 2
12345678901234567890123456789012123456789012345678901 22
1 Notice that AF is the hypotenuse of DACF, which is a right triangle. If you
12345678901234567890123456789012123456789012345678901 2
12345678901234567890123456789012123456789012345678901
find AC, you can use it along with CF in the Pythagorean theorem to 2
12345678901234567890123456789012123456789012345678901 2
12345678901234567890123456789012123456789012345678901 2
12345678901234567890123456789012123456789012345678901 2
find AF.
12345678901234567890123456789012123456789012345678901 2
12345678901234567890123456789012123456789012345678901 2
12345678901234567890123456789012123456789012345678901 22
2345678901234567890123456789012123456789012345678901
12345678901234567890123456789012123456789012345678901 2
1
12345678901234567890123456789012123456789012345678901 2
12345678901234567890123456789012123456789012345678901 2
12345678901234567890123456789012123456789012345678901 2
12345678901234567890123456789012123456789012345678901 2
12345678901234567890123456789012123456789012345678901 2
12345678901234567890123456789012123456789012345678901 2
12345678901234567890123456789012123456789012345678901 2
12345678901234567890123456789012123456789012345678901 22
2345678901234567890123456789012123456789012345678901
12345678901234567890123456789012123456789012345678901 2
12345678901234567890123456789012123456789012345678901 2
12345678901234567890123456789012123456789012345678901 2
1
12345678901234567890123456789012123456789012345678901 2
12345678901234567890123456789012123456789012345678901 2
12345678901234567890123456789012123456789012345678901 2
12345678901234567890123456789012123456789012345678901 2
12345678901234567890123456789012123456789012345678901 2
12345678901234567890123456789012123456789012345678901 22
2345678901234567890123456789012123456789012345678901
1
12345678901234567890123456789012123456789012345678901
Look at the top surface of the figure. You should see a square with sides of 2
12345678901234567890123456789012123456789012345678901 2
12345678901234567890123456789012123456789012345678901
length 3, and the diagonal is the hypotenuse of a 45-45-90 right triangle, 2
2345678901234567890123456789012123456789012345678901 2
1
12345678901234567890123456789012123456789012345678901 2
12345678901234567890123456789012123456789012345678901 2
12345678901234567890123456789012123456789012345678901 2 121
123456789012345678901234567890121234567890123456789012
Part III: Section Review

123456789012345678901234567890121234567890123456789012
12345678901234567890123456789012123456789012345678901 2
12345678901234567890123456789012123456789012345678901 2
12345678901234567890123456789012123456789012345678901
just like it was earlier in this chapter. This makes AC 5 3=2, since it is 2
12345678901234567890123456789012123456789012345678901 2
12345678901234567890123456789012123456789012345678901 2
12345678901234567890123456789012123456789012345678901 2
the hypotenuse of a 45-45-90 triangle with smaller sides of length 3.
12345678901234567890123456789012123456789012345678901 2
12345678901234567890123456789012123456789012345678901
Put that value into the next Pythagorean theorem as follows: 2
12345678901234567890123456789012123456789012345678901 2
12345678901234567890123456789012123456789012345678901 2
12345678901234567890123456789012123456789012345678901
~ AF ! 2
~ AC ! 2
~ CF! 2 2
12345678901234567890123456789012123456789012345678901
5 1
2345678901234567890123456789012123456789012345678901
2
2
1 2 2
12345678901234567890123456789012123456789012345678901
~AF! 5 ~3=2! 1 16 5 18 1 16 2
12345678901234567890123456789012123456789012345678901 2
12345678901234567890123456789012123456789012345678901
~ ! 2 2
AF 5 34
12345678901234567890123456789012123456789012345678901
2345678901234567890123456789012123456789012345678901
2
2
1
12345678901234567890123456789012123456789012345678901
AF 5 =34 2
12345678901234567890123456789012123456789012345678901 2
12345678901234567890123456789012123456789012345678901 2
12345678901234567890123456789012123456789012345678901
2345678901234567890123456789012123456789012345678901
2
2
1
12345678901234567890123456789012123456789012345678901 2
12345678901234567890123456789012123456789012345678901 2
12345678901234567890123456789012123456789012345678901 2
12345678901234567890123456789012123456789012345678901 2
12345678901234567890123456789012123456789012345678901 2
2345678901234567890123456789012123456789012345678901
12345678901234567890123456789012123456789012345678901 2
12345678901234567890123456789012123456789012345678901 2
12345678901234567890123456789012123456789012345678901 2
12345678901234567890123456789012123456789012345678901 2
12345678901234567890123456789012123456789012345678901 2 2
1
12345678901234567890123456789012123456789012345678901 2
12345678901234567890123456789012123456789012345678901 2
12345678901234567890123456789012123456789012345678901 2
12345678901234567890123456789012123456789012345678901 2
2345678901234567890123456789012123456789012345678901
12345678901234567890123456789012123456789012345678901 2
12345678901234567890123456789012123456789012345678901 2
12345678901234567890123456789012123456789012345678901 2
12345678901234567890123456789012123456789012345678901 2
12345678901234567890123456789012123456789012345678901 2
12345678901234567890123456789012123456789012345678901 2
12345678901234567890123456789012123456789012345678901 2
12345678901234567890123456789012123456789012345678901 2 2
1
12345678901234567890123456789012123456789012345678901 2
12345678901234567890123456789012123456789012345678901 2
12345678901234567890123456789012123456789012345678901 2
12345678901234567890123456789012123456789012345678901 2
12345678901234567890123456789012123456789012345678901 2
12345678901234567890123456789012123456789012345678901 2
12345678901234567890123456789012123456789012345678901 2
12345678901234567890123456789012123456789012345678901 2
12345678901234567890123456789012123456789012345678901 2
12345678901234567890123456789012123456789012345678901 2
2345678901234567890123456789012123456789012345678901
12345678901234567890123456789012123456789012345678901
If a sphere has a radius of r, what is the greatest distance along the 2
12345678901234567890123456789012123456789012345678901 2
1 surface of the sphere that two points could be from each other? 2
12345678901234567890123456789012123456789012345678901 2
12345678901234567890123456789012123456789012345678901 2
12345678901234567890123456789012123456789012345678901
A. 2r 2
12345678901234567890123456789012123456789012345678901 2
12345678901234567890123456789012123456789012345678901
B. pr 2
12345678901234567890123456789012123456789012345678901 2
12345678901234567890123456789012123456789012345678901 2
C. 2pr
12345678901234567890123456789012123456789012345678901 2
2345678901234567890123456789012123456789012345678901
2
12345678901234567890123456789012123456789012345678901 2
12345678901234567890123456789012123456789012345678901
4
2
12345678901234567890123456789012123456789012345678901
D. pr
2
1 3
2345678901234567890123456789012123456789012345678901
12345678901234567890123456789012123456789012345678901 2
12345678901234567890123456789012123456789012345678901
4 2 2
12345678901234567890123456789012123456789012345678901
E. pr 2
12345678901234567890123456789012123456789012345678901
3 2
1 2
2345678901234567890123456789012123456789012345678901
12345678901234567890123456789012123456789012345678901 2
1 Sometimes coming up with a real-life situation will help you understand an 2
12345678901234567890123456789012123456789012345678901
abstract geometrical one. For our purposes, think of the earth as a sphere. 2
12345678901234567890123456789012123456789012345678901 2
12345678901234567890123456789012123456789012345678901
In real terms, the questions is asking, “If
2345678901234567890123456789012123456789012345678901 you are at the North Pole, what 2
2
1
12345678901234567890123456789012123456789012345678901 2
12345678901234567890123456789012123456789012345678901 2 2
122 12345678901234567890123456789012123456789012345678901
123456789012345678901234567890121234567890123456789012

www.petersons.com
Chapter 4: Quantitative Review

123456789012345678901234567890121234567890123456789012
12345678901234567890123456789012123456789012345678901 2
12345678901234567890123456789012123456789012345678901 2
12345678901234567890123456789012123456789012345678901
point on the earth is farthest from you if you had to travel along the 2
12345678901234567890123456789012123456789012345678901 2
12345678901234567890123456789012123456789012345678901
earth’s surface?” 2
12345678901234567890123456789012123456789012345678901 2
12345678901234567890123456789012123456789012345678901 2
12345678901234567890123456789012123456789012345678901 2
12345678901234567890123456789012123456789012345678901 2
Alert!12345678901234567890123456789012123456789012345678901
12345678901234567890123456789012123456789012345678901
2
2
12345678901234567890123456789012123456789012345678901
Technically, the earth is an oblate spheroid,
2345678901234567890123456789012123456789012345678901
not a sphere. Thankfully, 2
2
1 oblate spheroids are NOT on the GRE.
12345678901234567890123456789012123456789012345678901 2
12345678901234567890123456789012123456789012345678901 2
12345678901234567890123456789012123456789012345678901 2
12345678901234567890123456789012123456789012345678901
2345678901234567890123456789012123456789012345678901
2
2
1
12345678901234567890123456789012123456789012345678901 2
12345678901234567890123456789012123456789012345678901 2
12345678901234567890123456789012123456789012345678901
Viewed in this way, the answer, “the South Pole,” should be obvious. The 2
12345678901234567890123456789012123456789012345678901
2345678901234567890123456789012123456789012345678901
2
2
1 same is true for the abstract sphere question. To determine that distance,
12345678901234567890123456789012123456789012345678901 2
12345678901234567890123456789012123456789012345678901
imagine drawing a circle on the surface of the sphere that runs through its 2
12345678901234567890123456789012123456789012345678901 2
12345678901234567890123456789012123456789012345678901
North and South Pole. You can see then that those two points are a half 2
2345678901234567890123456789012123456789012345678901
circumference of that circle from each other. The radius of that circle is r, 2
12345678901234567890123456789012123456789012345678901
12345678901234567890123456789012123456789012345678901 2

Take It to the Next Level


the same as the radius of the sphere. This makes the circumference 2pr, 2
12345678901234567890123456789012123456789012345678901
12345678901234567890123456789012123456789012345678901 22
12345678901234567890123456789012123456789012345678901
and half of that is pr. That’s choice (B).
12345678901234567890123456789012123456789012345678901 2
1 2
12345678901234567890123456789012123456789012345678901 2
12345678901234567890123456789012123456789012345678901 2
12345678901234567890123456789012123456789012345678901 2
12345678901234567890123456789012123456789012345678901 22
2345678901234567890123456789012123456789012345678901
Data Analysis
12345678901234567890123456789012123456789012345678901 2
12345678901234567890123456789012123456789012345678901
12345678901234567890123456789012123456789012345678901
Measures of Central Location and Dispersion (or 2
12345678901234567890123456789012123456789012345678901 2
12345678901234567890123456789012123456789012345678901 22
How Close and Far Apart Stuff Is)
12345678901234567890123456789012123456789012345678901 2
12345678901234567890123456789012123456789012345678901
Mean, median, and mode were covered earlier in this chapter. These are
12345678901234567890123456789012123456789012345678901 2
1 notions from statistics that give different ways to measure the central 2
2345678901234567890123456789012123456789012345678901
12345678901234567890123456789012123456789012345678901
location of a number set. Basically, they are different ways to say where the 2
2
12345678901234567890123456789012123456789012345678901 2
1 middle of a bunch of numbers is.
12345678901234567890123456789012123456789012345678901 2
12345678901234567890123456789012123456789012345678901 2
12345678901234567890123456789012123456789012345678901
The dispersion of a number set is the opposite of the “middleness” or 2
12345678901234567890123456789012123456789012345678901 2
12345678901234567890123456789012123456789012345678901 2
12345678901234567890123456789012123456789012345678901
“clusteredness” of a number set. The range is one measure of the 2
12345678901234567890123456789012123456789012345678901 2
12345678901234567890123456789012123456789012345678901 22
2345678901234567890123456789012123456789012345678901
dispersion since it tells you how far apart the extremes are. Standard
12345678901234567890123456789012123456789012345678901
1 deviation is another measure of dispersion. It measures on average how 2
12345678901234567890123456789012123456789012345678901
much numbers of a set differ from the middle of that set (basically how 2
12345678901234567890123456789012123456789012345678901 2
2345678901234567890123456789012123456789012345678901
spread out the numbers are). There is an involved way to calculate the 2
12345678901234567890123456789012123456789012345678901
12345678901234567890123456789012123456789012345678901 2
standard deviation, but you don’t need to bother with that. Standard 2
12345678901234567890123456789012123456789012345678901
12345678901234567890123456789012123456789012345678901 22
1 deviation, though, might appear on a question, and so you will need to
12345678901234567890123456789012123456789012345678901 2
12345678901234567890123456789012123456789012345678901
grasp the concept. To that end, consider the following problem. 2
12345678901234567890123456789012123456789012345678901 2
12345678901234567890123456789012123456789012345678901 2
12345678901234567890123456789012123456789012345678901
Column A Column B 2
12345678901234567890123456789012123456789012345678901 22
2345678901234567890123456789012123456789012345678901
12345678901234567890123456789012123456789012345678901 2
12345678901234567890123456789012123456789012345678901
Standard Deviation Standard Deviation
2
12345678901234567890123456789012123456789012345678901 2
1 of this number set of this number set
2345678901234567890123456789012123456789012345678901
12345678901234567890123456789012123456789012345678901 2
1 [5, 10, 2, 4] [1, 14, 3, 22] 2
12345678901234567890123456789012123456789012345678901 2
12345678901234567890123456789012123456789012345678901 2
12345678901234567890123456789012123456789012345678901
You don’t need to actually calculate the two standard deviations to answer 2
1 this question. Estimate the mean of the number set in Column A. It’s 2
2345678901234567890123456789012123456789012345678901
12345678901234567890123456789012123456789012345678901 2
12345678901234567890123456789012123456789012345678901 2
12345678901234567890123456789012123456789012345678901 2 123
123456789012345678901234567890121234567890123456789012
Part III: Section Review

123456789012345678901234567890121234567890123456789012
12345678901234567890123456789012123456789012345678901 2
12345678901234567890123456789012123456789012345678901 2
12345678901234567890123456789012123456789012345678901
probably around 4 or 5, and the numbers in the set deviate from this by, at 2
12345678901234567890123456789012123456789012345678901 2
12345678901234567890123456789012123456789012345678901
most, 6. So, the standard deviation will be something less than 6. Now 2
12345678901234567890123456789012123456789012345678901 2
12345678901234567890123456789012123456789012345678901
estimate the mean for Column B. It’s probably around 11, and the number 2
1 set deviates from this by 10 or more. The standard deviation is going to be 2
2345678901234567890123456789012123456789012345678901
12345678901234567890123456789012123456789012345678901 2
12345678901234567890123456789012123456789012345678901 2
12345678901234567890123456789012123456789012345678901
greater in Column B. The answer will be Column B, in part because the 2
12345678901234567890123456789012123456789012345678901
2345678901234567890123456789012123456789012345678901
2
2
1 numbers in Column B are more spread out than the numbers in Column A.
12345678901234567890123456789012123456789012345678901 2
12345678901234567890123456789012123456789012345678901 2
12345678901234567890123456789012123456789012345678901
Standard deviation is not the only way to make these statistics concepts 2
12345678901234567890123456789012123456789012345678901
2345678901234567890123456789012123456789012345678901
2
2
1 interesting. The test-writers can craft difficult problems with median,
12345678901234567890123456789012123456789012345678901 2
12345678901234567890123456789012123456789012345678901
mode, range, and average. 2
12345678901234567890123456789012123456789012345678901 2
12345678901234567890123456789012123456789012345678901
2345678901234567890123456789012123456789012345678901
2
2
1 Column A Column B
12345678901234567890123456789012123456789012345678901 2
12345678901234567890123456789012123456789012345678901
A number set is composed of five non- 2
12345678901234567890123456789012123456789012345678901 2
12345678901234567890123456789012123456789012345678901
negative integers, and its average is 20. 2
12345678901234567890123456789012123456789012345678901 2
2345678901234567890123456789012123456789012345678901
12345678901234567890123456789012123456789012345678901 2
12345678901234567890123456789012123456789012345678901
The greatest The sum of the 2
12345678901234567890123456789012123456789012345678901 2
12345678901234567890123456789012123456789012345678901 2
possible range of number set
12345678901234567890123456789012123456789012345678901 2
1 the number set 2
12345678901234567890123456789012123456789012345678901 2
12345678901234567890123456789012123456789012345678901 2
12345678901234567890123456789012123456789012345678901
Start with Column B. The sum of the number set is the average times the 2
12345678901234567890123456789012123456789012345678901 2
2345678901234567890123456789012123456789012345678901
12345678901234567890123456789012123456789012345678901
number of elements: 2
12345678901234567890123456789012123456789012345678901 2
12345678901234567890123456789012123456789012345678901 2
12345678901234567890123456789012123456789012345678901
sum 2
12345678901234567890123456789012123456789012345678901
5 average → sum 5 5 3 20 5 100 2
12345678901234567890123456789012123456789012345678901
5 2
12345678901234567890123456789012123456789012345678901 2
12345678901234567890123456789012123456789012345678901 2
1 Column B is 100. 2
12345678901234567890123456789012123456789012345678901 2
2345678901234567890123456789012123456789012345678901
As for Column A, the greatest possible range will maximize the highest 2
12345678901234567890123456789012123456789012345678901
1 2
12345678901234567890123456789012123456789012345678901
element and minimize the lowest element. The set is composed of 2
12345678901234567890123456789012123456789012345678901 2
12345678901234567890123456789012123456789012345678901
non-negative integers, so the smallest element possible is 0. There is no 2
12345678901234567890123456789012123456789012345678901 2
12345678901234567890123456789012123456789012345678901
mention that the elements are distinct, so four of the elements could be 2
12345678901234567890123456789012123456789012345678901 2
12345678901234567890123456789012123456789012345678901
zero. In that case, the fifth element would have to be 100 for the sum to be 2
2345678901234567890123456789012123456789012345678901
12345678901234567890123456789012123456789012345678901
100. 100 is actually the greatest that a number in the set could be, since the 2
2
12345678901234567890123456789012123456789012345678901
1 sum cannot exceed 100 and there are no negative numbers to add. So the 2
12345678901234567890123456789012123456789012345678901 2
12345678901234567890123456789012123456789012345678901
greatest possible range is 100 2 0 5 100. Column A and B are equal, 2
12345678901234567890123456789012123456789012345678901 2
12345678901234567890123456789012123456789012345678901 2
12345678901234567890123456789012123456789012345678901
making (C) the correct answer. 2
12345678901234567890123456789012123456789012345678901 2
12345678901234567890123456789012123456789012345678901 2
12345678901234567890123456789012123456789012345678901 2
2345678901234567890123456789012123456789012345678901
12345678901234567890123456789012123456789012345678901 2 2
12345678901234567890123456789012123456789012345678901
Counting Stuff 2
12345678901234567890123456789012123456789012345678901
1 There are a few different kinds of word problems on the GRE that involve 2
12345678901234567890123456789012123456789012345678901 2
2345678901234567890123456789012123456789012345678901
counting possible outcomes or permutations of a set of items. Each has a 2
12345678901234567890123456789012123456789012345678901
12345678901234567890123456789012123456789012345678901 2
12345678901234567890123456789012123456789012345678901
different kind of way to solve them, so we’ll review them one by one. 2
1 2
12345678901234567890123456789012123456789012345678901 2
2345678901234567890123456789012123456789012345678901
2
1
12345678901234567890123456789012123456789012345678901 2
12345678901234567890123456789012123456789012345678901 2
Different Ways to Count Stuff
12345678901234567890123456789012123456789012345678901
1. If one task has n possible outcomes and a second one has m possible 2
12345678901234567890123456789012123456789012345678901 2
12345678901234567890123456789012123456789012345678901 2
12345678901234567890123456789012123456789012345678901 2
outcomes, then the joint occurrence of the two tasks has n 3 m possible
2
124 12345678901234567890123456789012123456789012345678901
123456789012345678901234567890121234567890123456789012

www.petersons.com
Chapter 4: Quantitative Review

123456789012345678901234567890121234567890123456789012
12345678901234567890123456789012123456789012345678901 2
12345678901234567890123456789012123456789012345678901 2
12345678901234567890123456789012123456789012345678901
outcomes. That postulate was a bit vague, so consider an example. If 2
12345678901234567890123456789012123456789012345678901 2
12345678901234567890123456789012123456789012345678901
between Town A and Town B there are four roads, and between Town B 2
12345678901234567890123456789012123456789012345678901 2
12345678901234567890123456789012123456789012345678901
and Town C there are five roads, how many different routes are there from 2
1 Town A to Town C that travel through Town B? One way to attack this 2
2345678901234567890123456789012123456789012345678901
12345678901234567890123456789012123456789012345678901 2
12345678901234567890123456789012123456789012345678901 2
12345678901234567890123456789012123456789012345678901
problem is to sketch out the scenario and start counting up the 2
12345678901234567890123456789012123456789012345678901
2345678901234567890123456789012123456789012345678901
2
2
1 possibilities. The other way is to use our nifty little formula, n 3 m 5
12345678901234567890123456789012123456789012345678901 2
12345678901234567890123456789012123456789012345678901
4 3 5 5 20. 2
12345678901234567890123456789012123456789012345678901 2
12345678901234567890123456789012123456789012345678901
2345678901234567890123456789012123456789012345678901
2
2
1 2. Permutation problems prompt you to determine how many different
12345678901234567890123456789012123456789012345678901 2
12345678901234567890123456789012123456789012345678901
permutations, or outcomes, are possible. A permutation word problem 2
12345678901234567890123456789012123456789012345678901
might ask, “If there are six car spots at the Destin Apartments and six 2
12345678901234567890123456789012123456789012345678901 2
1 different cars owned by the renters, how many different arrangements of 2
2345678901234567890123456789012123456789012345678901
12345678901234567890123456789012123456789012345678901 2
12345678901234567890123456789012123456789012345678901
cars are possible in the parking lot?” You solve this problem by 2
12345678901234567890123456789012123456789012345678901 2
12345678901234567890123456789012123456789012345678901
multiplying the number of possibilities for the first spot (6 cars can park 2
12345678901234567890123456789012123456789012345678901 22
2345678901234567890123456789012123456789012345678901
12345678901234567890123456789012123456789012345678901

Take It to the Next Level


there) times the number of remaining possibilities for the second spot (5
12345678901234567890123456789012123456789012345678901
cars can if the first spot is filled) times the number of remaining 2
2
12345678901234567890123456789012123456789012345678901
12345678901234567890123456789012123456789012345678901
possibilities for the third spot (4) and so on. In other words, 6 3 5 3 4 3 2
2
12345678901234567890123456789012123456789012345678901
1 3 3 2 3 1. This is the same thing as 6!, our good friend the factorial. In 2
12345678901234567890123456789012123456789012345678901 2
12345678901234567890123456789012123456789012345678901 2
12345678901234567890123456789012123456789012345678901
general, when you come across a permutation problem like this one, you 2
2345678901234567890123456789012123456789012345678901
can use this factorial shortcut rather than going through all the logic. The 2
12345678901234567890123456789012123456789012345678901
12345678901234567890123456789012123456789012345678901 2
12345678901234567890123456789012123456789012345678901 2
12345678901234567890123456789012123456789012345678901 22
basic formula is n!; the number of ways that n different objects can be
12345678901234567890123456789012123456789012345678901
12345678901234567890123456789012123456789012345678901
ordered is n!. 2
12345678901234567890123456789012123456789012345678901 22
12345678901234567890123456789012123456789012345678901
3. The last kind of counting problem involves determining the number of 2
12345678901234567890123456789012123456789012345678901
1 different ways that sets of items can be selected from a larger set of items. 2
12345678901234567890123456789012123456789012345678901 2
12345678901234567890123456789012123456789012345678901
A good example of this is selecting committees of three people from a 2
12345678901234567890123456789012123456789012345678901 2
12345678901234567890123456789012123456789012345678901
group of five people. How many different ways can this be done? You 2
12345678901234567890123456789012123456789012345678901
2345678901234567890123456789012123456789012345678901
2
could try to work out the possibilities with a systematic use of logic, or you 2
12345678901234567890123456789012123456789012345678901
12345678901234567890123456789012123456789012345678901 2
1 could use the handy-dandy formula that has all the logic packed into it. 2
12345678901234567890123456789012123456789012345678901 2
12345678901234567890123456789012123456789012345678901
n! 2
2345678901234567890123456789012123456789012345678901
, where n is the size of the larger set, and r is the 2
12345678901234567890123456789012123456789012345678901
The formula is
12345678901234567890123456789012123456789012345678901
~ n 2 r! !r! 2
1 size of the groups 2
12345678901234567890123456789012123456789012345678901 2
being selected. In our example, the formula would be
12345678901234567890123456789012123456789012345678901 2
12345678901234567890123456789012123456789012345678901
applied like this: 2
12345678901234567890123456789012123456789012345678901 2
12345678901234567890123456789012123456789012345678901 2
12345678901234567890123456789012123456789012345678901
5! 5 3 4 3 3! 20 2
12345678901234567890123456789012123456789012345678901
5 5 5 10 2
12345678901234567890123456789012123456789012345678901 22
2345678901234567890123456789012123456789012345678901
~ 5 2 3! !3! 2!3! 2!
12345678901234567890123456789012123456789012345678901 2
12345678901234567890123456789012123456789012345678901 2
12345678901234567890123456789012123456789012345678901 2
1
12345678901234567890123456789012123456789012345678901
Probability 2
12345678901234567890123456789012123456789012345678901 2
12345678901234567890123456789012123456789012345678901 2
12345678901234567890123456789012123456789012345678901
The basics of single and multi-event probability were covered in the first 2
12345678901234567890123456789012123456789012345678901 2
12345678901234567890123456789012123456789012345678901 22
2345678901234567890123456789012123456789012345678901
section. Yet there are some multi-event probability scenarios that go
1 beyond the basic. If two events are independent, then the probability of
12345678901234567890123456789012123456789012345678901 2
12345678901234567890123456789012123456789012345678901
both events occurring is the product of the two individual probabilities. 2
12345678901234567890123456789012123456789012345678901 2
12345678901234567890123456789012123456789012345678901
Now suppose you know the probabilities of event A and event B 2
12345678901234567890123456789012123456789012345678901 2
12345678901234567890123456789012123456789012345678901 22
12345678901234567890123456789012123456789012345678901 125
123456789012345678901234567890121234567890123456789012
Part III: Section Review

123456789012345678901234567890121234567890123456789012
12345678901234567890123456789012123456789012345678901 2
12345678901234567890123456789012123456789012345678901 2
12345678901234567890123456789012123456789012345678901
individually and want to find the probabilities of either A or B occurring 2
12345678901234567890123456789012123456789012345678901 2
12345678901234567890123456789012123456789012345678901
(as opposed to the probability of A and B occurring). 2
12345678901234567890123456789012123456789012345678901 2
12345678901234567890123456789012123456789012345678901 2
12345678901234567890123456789012123456789012345678901
A fundamental rule of probability is: 2
12345678901234567890123456789012123456789012345678901 2
12345678901234567890123456789012123456789012345678901 2
12345678901234567890123456789012123456789012345678901
P(A or B) 5 P(A) 1 P(B) 2 P(A)P(B) 2
12345678901234567890123456789012123456789012345678901 2
1 Suppose there is a one-in-two chance of event A occurring and a 2
2345678901234567890123456789012123456789012345678901
12345678901234567890123456789012123456789012345678901 2
12345678901234567890123456789012123456789012345678901
two-in-five chance for event B. Expressed in decimal terms, this makes the 2
12345678901234567890123456789012123456789012345678901 2
12345678901234567890123456789012123456789012345678901 2
SD1
2345678901234567890123456789012123456789012345678901
1 probability of A 5 0.5
2
and probability of B 5 0.4
12345678901234567890123456789012123456789012345678901
12345678901234567890123456789012123456789012345678901
SD 2
5
. Putting these 2
2
2
12345678901234567890123456789012123456789012345678901
values into the formula gets you: 2
12345678901234567890123456789012123456789012345678901
2345678901234567890123456789012123456789012345678901
2
2
1
12345678901234567890123456789012123456789012345678901
P ~A or B! 5 P~A! 1 P~B! 2 P~A!P~B! 2
12345678901234567890123456789012123456789012345678901 2
12345678901234567890123456789012123456789012345678901
P ~A or B! 5 0.5 1 0.4 2 ~0.5!~0.4! 2
12345678901234567890123456789012123456789012345678901 2
2345678901234567890123456789012123456789012345678901
12345678901234567890123456789012123456789012345678901
P ~A or B! 5 0.9 2 0.2 5 0.7 2
12345678901234567890123456789012123456789012345678901 2
12345678901234567890123456789012123456789012345678901 2
0.7 translates to a seven-in-ten chance, showing that the probability of 2
12345678901234567890123456789012123456789012345678901
12345678901234567890123456789012123456789012345678901 2
12345678901234567890123456789012123456789012345678901 2
1 A or B occurring is larger than the probability of either single event 2
12345678901234567890123456789012123456789012345678901
occurring. 2
12345678901234567890123456789012123456789012345678901 2
12345678901234567890123456789012123456789012345678901 2
2345678901234567890123456789012123456789012345678901
12345678901234567890123456789012123456789012345678901 2
12345678901234567890123456789012123456789012345678901
A bag of marbles contains 12 red marbles, 8 blue marbles, and 5 2
12345678901234567890123456789012123456789012345678901 2
12345678901234567890123456789012123456789012345678901 2
green marbles. What is the probability that a green and then a blue
12345678901234567890123456789012123456789012345678901 2
12345678901234567890123456789012123456789012345678901
marble will be chosen in two attempts? 2
12345678901234567890123456789012123456789012345678901 2
12345678901234567890123456789012123456789012345678901
1 2
12345678901234567890123456789012123456789012345678901 2
1 A. 2
12345678901234567890123456789012123456789012345678901 2
2345678901234567890123456789012123456789012345678901
15
12345678901234567890123456789012123456789012345678901 2
1 1 2
12345678901234567890123456789012123456789012345678901
B. 2
12345678901234567890123456789012123456789012345678901
10 2
12345678901234567890123456789012123456789012345678901 2
2345678901234567890123456789012123456789012345678901
12345678901234567890123456789012123456789012345678901
1 2
1 C. 2
12345678901234567890123456789012123456789012345678901
5 2
12345678901234567890123456789012123456789012345678901 2
2345678901234567890123456789012123456789012345678901
12345678901234567890123456789012123456789012345678901 2
12345678901234567890123456789012123456789012345678901
12 2
1 D. 2
12345678901234567890123456789012123456789012345678901 2
25
12345678901234567890123456789012123456789012345678901 2
12345678901234567890123456789012123456789012345678901
13 2
12345678901234567890123456789012123456789012345678901
E. 2
12345678901234567890123456789012123456789012345678901
25 2
12345678901234567890123456789012123456789012345678901 2
12345678901234567890123456789012123456789012345678901 2
12345678901234567890123456789012123456789012345678901 2
2345678901234567890123456789012123456789012345678901
Determine the probability of each event individually. The probability of
2
12345678901234567890123456789012123456789012345678901
the first event is: 2
12345678901234567890123456789012123456789012345678901
12345678901234567890123456789012123456789012345678901 2 2
1
12345678901234567890123456789012123456789012345678901
5 5 1 2
12345678901234567890123456789012123456789012345678901
5 5 2
12345678901234567890123456789012123456789012345678901
12 1 8 1 5 25 5 2
12345678901234567890123456789012123456789012345678901 2
12345678901234567890123456789012123456789012345678901
The numerator, 5, is the number of green marbles divided by the total 2
12345678901234567890123456789012123456789012345678901 2
2345678901234567890123456789012123456789012345678901
1 number of marbles. For the second probability, you must remember to 2
12345678901234567890123456789012123456789012345678901 2
12345678901234567890123456789012123456789012345678901
take the first transaction into account. This means there are only 24 total 2
12345678901234567890123456789012123456789012345678901 2
12345678901234567890123456789012123456789012345678901
marbles and 4 green marbles. 2
12345678901234567890123456789012123456789012345678901 2
12345678901234567890123456789012123456789012345678901 2 2
126 12345678901234567890123456789012123456789012345678901
123456789012345678901234567890121234567890123456789012

www.petersons.com
Chapter 4: Quantitative Review

123456789012345678901234567890121234567890123456789012
12345678901234567890123456789012123456789012345678901 2
12345678901234567890123456789012123456789012345678901 2
12345678901234567890123456789012123456789012345678901
This makes the blue marble equation: 2
12345678901234567890123456789012123456789012345678901 2
12345678901234567890123456789012123456789012345678901 2
12345678901234567890123456789012123456789012345678901
8 8 1 2
12345678901234567890123456789012123456789012345678901
2345678901234567890123456789012123456789012345678901
2
2
1 12 1 8 1 4 24 3
5 5
12345678901234567890123456789012123456789012345678901 2
12345678901234567890123456789012123456789012345678901 2
12345678901234567890123456789012123456789012345678901
The probability of both occurring is the product of the individual 2
12345678901234567890123456789012123456789012345678901 2
12345678901234567890123456789012123456789012345678901
probabilities: 2
12345678901234567890123456789012123456789012345678901 2
12345678901234567890123456789012123456789012345678901 2
12345678901234567890123456789012123456789012345678901
1 1 1 2
12345678901234567890123456789012123456789012345678901
3 5
2345678901234567890123456789012123456789012345678901
2
2
1 3 5 15
12345678901234567890123456789012123456789012345678901 2
12345678901234567890123456789012123456789012345678901 2
12345678901234567890123456789012123456789012345678901
The correct answer is (A). 2
12345678901234567890123456789012123456789012345678901
2345678901234567890123456789012123456789012345678901
2
2
1
12345678901234567890123456789012123456789012345678901 2
12345678901234567890123456789012123456789012345678901 2
12345678901234567890123456789012123456789012345678901
Given the same scenario, what would the probability of picking a green 2
12345678901234567890123456789012123456789012345678901 2
12345678901234567890123456789012123456789012345678901 22
2345678901234567890123456789012123456789012345678901
marble first or a blue marble second?
12345678901234567890123456789012123456789012345678901 2

Take It to the Next Level


12345678901234567890123456789012123456789012345678901
12345678901234567890123456789012123456789012345678901
P ~A or B! 5 P~A! 1 P~B! 2 P~A!P~B! 2
12345678901234567890123456789012123456789012345678901 22
12345678901234567890123456789012123456789012345678901
Note

1 1 1
1 2
12345678901234567890123456789012123456789012345678901
5 1 2
5 3 15 2
12345678901234567890123456789012123456789012345678901 2
12345678901234567890123456789012123456789012345678901 2
2345678901234567890123456789012123456789012345678901
12345678901234567890123456789012123456789012345678901
8 1 2
12345678901234567890123456789012123456789012345678901
5 2 2
12345678901234567890123456789012123456789012345678901 22
15 15
12345678901234567890123456789012123456789012345678901
12345678901234567890123456789012123456789012345678901
7 2
12345678901234567890123456789012123456789012345678901
5 2
12345678901234567890123456789012123456789012345678901
15 2
12345678901234567890123456789012123456789012345678901 22
12345678901234567890123456789012123456789012345678901
1 . 2
12345678901234567890123456789012123456789012345678901 2
12345678901234567890123456789012123456789012345678901 2
12345678901234567890123456789012123456789012345678901
Column A Column B 2
12345678901234567890123456789012123456789012345678901 2
12345678901234567890123456789012123456789012345678901 2
12345678901234567890123456789012123456789012345678901
There are 24 students in a class. Half are 2
12345678901234567890123456789012123456789012345678901 2
12345678901234567890123456789012123456789012345678901
male and half are female. At random, 2
12345678901234567890123456789012123456789012345678901 2
12345678901234567890123456789012123456789012345678901
3 students are chosen successively 2
2345678901234567890123456789012123456789012345678901
12345678901234567890123456789012123456789012345678901 2
12345678901234567890123456789012123456789012345678901
from the class. 2
1 2
12345678901234567890123456789012123456789012345678901 2
12345678901234567890123456789012123456789012345678901 2
The probability of a The probability of a
2345678901234567890123456789012123456789012345678901
12345678901234567890123456789012123456789012345678901 2
12345678901234567890123456789012123456789012345678901
boy, then a girl, girl, then a boy, 2
12345678901234567890123456789012123456789012345678901 2
12345678901234567890123456789012123456789012345678901 22
then a girl, then a girl
1
12345678901234567890123456789012123456789012345678901
being chosen being chosen 2
12345678901234567890123456789012123456789012345678901 2
12345678901234567890123456789012123456789012345678901
These are both and situations, not or situations. The probability of either 2
12345678901234567890123456789012123456789012345678901 2
12345678901234567890123456789012123456789012345678901
first event is 0.5 since the class is evenly split. The second event in both 2
2345678901234567890123456789012123456789012345678901
12345678901234567890123456789012123456789012345678901
columns then switches genders, so again the probability would be the 2
2
12345678901234567890123456789012123456789012345678901 2
12345678901234567890123456789012123456789012345678901 2
12345678901234567890123456789012123456789012345678901
12
2
1 same, . For the third event, in both columns a girl and a boy have
12345678901234567890123456789012123456789012345678901 22
2345678901234567890123456789012123456789012345678901
23
1 already been chosen, so the probability of picking a male or female is again
12345678901234567890123456789012123456789012345678901 2
12345678901234567890123456789012123456789012345678901 2
12345678901234567890123456789012123456789012345678901
the same. So the third events also have the
2345678901234567890123456789012123456789012345678901
same probability. The products 2
2
1 of equal factors are equal, so (C) is the correct answer.
12345678901234567890123456789012123456789012345678901 2
12345678901234567890123456789012123456789012345678901 2
12345678901234567890123456789012123456789012345678901 2 127
123456789012345678901234567890121234567890123456789012
Part III: Section Review

123456789012345678901234567890121234567890123456789012
12345678901234567890123456789012123456789012345678901 2
12345678901234567890123456789012123456789012345678901 2
12345678901234567890123456789012123456789012345678901
Get Graphical with the Data 2
12345678901234567890123456789012123456789012345678901 2
12345678901234567890123456789012123456789012345678901 2
12345678901234567890123456789012123456789012345678901 2
You will see one or two sets of graphs and charts on the GRE math section.
12345678901234567890123456789012123456789012345678901
Each set will have 2 to 4 questions, so understanding and interpreting the 2
12345678901234567890123456789012123456789012345678901 2
12345678901234567890123456789012123456789012345678901
graph sets are important. As was said in the first section, there is good 2
12345678901234567890123456789012123456789012345678901 2
12345678901234567890123456789012123456789012345678901
news and bad news for this. The good news is that the answers for the 2
12345678901234567890123456789012123456789012345678901 2
12345678901234567890123456789012123456789012345678901
graph set questions are always right in front of you. The bad news is that 2
12345678901234567890123456789012123456789012345678901 2
12345678901234567890123456789012123456789012345678901
the GRE can come up with some pretty complex graph sets that make 2
12345678901234567890123456789012123456789012345678901 2
12345678901234567890123456789012123456789012345678901
finding the answers a challenge. The way to combat this “complexifying” 2
1 is to spend 10 to 20 seconds familiarizing yourself with a graph set when it 2
2345678901234567890123456789012123456789012345678901
12345678901234567890123456789012123456789012345678901 2
12345678901234567890123456789012123456789012345678901 2
12345678901234567890123456789012123456789012345678901
comes up. Scroll all the way down so that you see all the pertinent 2
12345678901234567890123456789012123456789012345678901 2
12345678901234567890123456789012123456789012345678901
information. Look at any key. Read the titles of everything. Inspect graph 2
12345678901234567890123456789012123456789012345678901
axes so that you know with what units the information is being presented. 2
12345678901234567890123456789012123456789012345678901 2
12345678901234567890123456789012123456789012345678901 2
12345678901234567890123456789012123456789012345678901
Basically, get comfortable with the graph sets. This way, when you start on 2
2345678901234567890123456789012123456789012345678901
the questions you will be ready to hone in on whatever information a 2
12345678901234567890123456789012123456789012345678901
12345678901234567890123456789012123456789012345678901 2
12345678901234567890123456789012123456789012345678901 2
12345678901234567890123456789012123456789012345678901 2
question asks for.
12345678901234567890123456789012123456789012345678901 2
12345678901234567890123456789012123456789012345678901 2
1 There are two ways that the GRE increases the difficulty of these kinds of 2
12345678901234567890123456789012123456789012345678901
problems. The first is to give you wackier or more unconventional graph 2
12345678901234567890123456789012123456789012345678901 2
12345678901234567890123456789012123456789012345678901
sets. Instead of getting just a line graph and a bar graph, you might get a 2
12345678901234567890123456789012123456789012345678901 2
2345678901234567890123456789012123456789012345678901
line graph that has a right and left axis with different figures graphed, and 2
12345678901234567890123456789012123456789012345678901
12345678901234567890123456789012123456789012345678901 2
also a vertical pie chart. The second way the test-makers increase the 2
12345678901234567890123456789012123456789012345678901
12345678901234567890123456789012123456789012345678901 2
difficulty is by increasing the number of steps needed to find the answer. 2
12345678901234567890123456789012123456789012345678901
12345678901234567890123456789012123456789012345678901
Instead of just having to read the answer off a bar graph, you might have 2
2
12345678901234567890123456789012123456789012345678901
12345678901234567890123456789012123456789012345678901 2
1 to take information from both graphs and then do a little math. Although 2
2345678901234567890123456789012123456789012345678901
12345678901234567890123456789012123456789012345678901
these multi-stage problems take time, graph set questions will not involve 2
2
12345678901234567890123456789012123456789012345678901
1 more than two or three steps. The GRE tests your ability to quickly and 2
12345678901234567890123456789012123456789012345678901 2
12345678901234567890123456789012123456789012345678901
correctly interpret graph sets, not your ability to do sophisticated analysis 2
12345678901234567890123456789012123456789012345678901 2
12345678901234567890123456789012123456789012345678901 2
12345678901234567890123456789012123456789012345678901
of the information presented. 2
12345678901234567890123456789012123456789012345678901 2
12345678901234567890123456789012123456789012345678901 2
12345678901234567890123456789012123456789012345678901 2
2345678901234567890123456789012123456789012345678901
12345678901234567890123456789012123456789012345678901 2 2
1
12345678901234567890123456789012123456789012345678901 2
12345678901234567890123456789012123456789012345678901 2
12345678901234567890123456789012123456789012345678901 2
12345678901234567890123456789012123456789012345678901 2
12345678901234567890123456789012123456789012345678901 2
12345678901234567890123456789012123456789012345678901 2
12345678901234567890123456789012123456789012345678901 2
12345678901234567890123456789012123456789012345678901 2
2345678901234567890123456789012123456789012345678901
12345678901234567890123456789012123456789012345678901 2
12345678901234567890123456789012123456789012345678901 2
12345678901234567890123456789012123456789012345678901 2 2
1
12345678901234567890123456789012123456789012345678901 2
12345678901234567890123456789012123456789012345678901 2
12345678901234567890123456789012123456789012345678901 2
12345678901234567890123456789012123456789012345678901 2
12345678901234567890123456789012123456789012345678901 2
12345678901234567890123456789012123456789012345678901 2
2345678901234567890123456789012123456789012345678901
2
1
12345678901234567890123456789012123456789012345678901 2
12345678901234567890123456789012123456789012345678901 2
12345678901234567890123456789012123456789012345678901 2
12345678901234567890123456789012123456789012345678901 2
12345678901234567890123456789012123456789012345678901 2
12345678901234567890123456789012123456789012345678901 2 2
128 12345678901234567890123456789012123456789012345678901
123456789012345678901234567890121234567890123456789012

www.petersons.com
Chapter 4: Quantitative Review

123456789012345678901234567890121234567890123456789012
12345678901234567890123456789012123456789012345678901 2
12345678901234567890123456789012123456789012345678901 2
12345678901234567890123456789012123456789012345678901 2
12345678901234567890123456789012123456789012345678901 2
12345678901234567890123456789012123456789012345678901 2
12345678901234567890123456789012123456789012345678901 2
12345678901234567890123456789012123456789012345678901
165
2345678901234567890123456789012123456789012345678901
2
2
1
12345678901234567890123456789012123456789012345678901 2
12345678901234567890123456789012123456789012345678901 2
12345678901234567890123456789012123456789012345678901 2
12345678901234567890123456789012123456789012345678901
2345678901234567890123456789012123456789012345678901
2
2
1
12345678901234567890123456789012123456789012345678901 2
12345678901234567890123456789012123456789012345678901 2
12345678901234567890123456789012123456789012345678901 2
12345678901234567890123456789012123456789012345678901
2345678901234567890123456789012123456789012345678901
2
2
1
12345678901234567890123456789012123456789012345678901 2
12345678901234567890123456789012123456789012345678901 2
12345678901234567890123456789012123456789012345678901 2
12345678901234567890123456789012123456789012345678901
2345678901234567890123456789012123456789012345678901
2
2
1
12345678901234567890123456789012123456789012345678901 2
12345678901234567890123456789012123456789012345678901 2
12345678901234567890123456789012123456789012345678901 2
12345678901234567890123456789012123456789012345678901 2
12345678901234567890123456789012123456789012345678901 22
2345678901234567890123456789012123456789012345678901
12345678901234567890123456789012123456789012345678901 2

Take It to the Next Level


12345678901234567890123456789012123456789012345678901 2
12345678901234567890123456789012123456789012345678901 2
12345678901234567890123456789012123456789012345678901 2
12345678901234567890123456789012123456789012345678901 2
1
12345678901234567890123456789012123456789012345678901 2
12345678901234567890123456789012123456789012345678901 2
12345678901234567890123456789012123456789012345678901 2
12345678901234567890123456789012123456789012345678901 22
2345678901234567890123456789012123456789012345678901
12345678901234567890123456789012123456789012345678901 2
12345678901234567890123456789012123456789012345678901 2
12345678901234567890123456789012123456789012345678901 2
12345678901234567890123456789012123456789012345678901 2
12345678901234567890123456789012123456789012345678901
12345678901234567890123456789012123456789012345678901
What percentage of the European Travelers surveyed did NOT 2
12345678901234567890123456789012123456789012345678901 2
12345678901234567890123456789012123456789012345678901 22
travel to Africa?
1
12345678901234567890123456789012123456789012345678901
A. 34% 2
12345678901234567890123456789012123456789012345678901 2
12345678901234567890123456789012123456789012345678901
B. 43% 2
12345678901234567890123456789012123456789012345678901 2
12345678901234567890123456789012123456789012345678901
C. 57% 2
12345678901234567890123456789012123456789012345678901 2
12345678901234567890123456789012123456789012345678901
D. 63% 2
12345678901234567890123456789012123456789012345678901 2
12345678901234567890123456789012123456789012345678901
E. 70% 2
12345678901234567890123456789012123456789012345678901 2
2345678901234567890123456789012123456789012345678901
This question only involves the Venn diagram, so focus on it. You will find 2
12345678901234567890123456789012123456789012345678901
12345678901234567890123456789012123456789012345678901 2
1 that the total number of European travelers is 250 (165 African travelers 2
12345678901234567890123456789012123456789012345678901 2
12345678901234567890123456789012123456789012345678901
1 155 Asian travelers). You will note that by looking closely at the Venn 2
12345678901234567890123456789012123456789012345678901 2
12345678901234567890123456789012123456789012345678901 2
12345678901234567890123456789012123456789012345678901
diagram that there is a shaded region shared by both Africa and Asia 2
12345678901234567890123456789012123456789012345678901 2
12345678901234567890123456789012123456789012345678901 2
(represented by 70).
12345678901234567890123456789012123456789012345678901 22
2345678901234567890123456789012123456789012345678901
12345678901234567890123456789012123456789012345678901
You’ll calculate the travelers represented in the non-shaded portion of the
2
12345678901234567890123456789012123456789012345678901
Venn diagram by subtracting 70 from both sides. So, for Africa 165 2 70 2
12345678901234567890123456789012123456789012345678901
1 2
12345678901234567890123456789012123456789012345678901
5 95 travelers. And Asia, 155 2 70 5 85 travelers. 2
12345678901234567890123456789012123456789012345678901 2
12345678901234567890123456789012123456789012345678901 2
12345678901234567890123456789012123456789012345678901
Remember our total amount of European travelers is 250. Therefore, for 2
12345678901234567890123456789012123456789012345678901 2
12345678901234567890123456789012123456789012345678901
95 85 2
12345678901234567890123456789012123456789012345678901
Africa (or approximately 38 percent) and for Asia (or approxi- 2
12345678901234567890123456789012123456789012345678901
250 250 2
12345678901234567890123456789012123456789012345678901 2
12345678901234567890123456789012123456789012345678901
mately 34 percent). The question only asks for
2345678901234567890123456789012123456789012345678901
the percentage that did 2
2
1 NOT travel to Africa, so the correct answer is (A), 34 percent.
12345678901234567890123456789012123456789012345678901 2
12345678901234567890123456789012123456789012345678901 2
12345678901234567890123456789012123456789012345678901 2 129
123456789012345678901234567890121234567890123456789012
Part III: Section Review

123456789012345678901234567890121234567890123456789012
12345678901234567890123456789012123456789012345678901 2
12345678901234567890123456789012123456789012345678901 2
12345678901234567890123456789012123456789012345678901
How many of the European travelers surveyed are over 50 and have 2
12345678901234567890123456789012123456789012345678901 2
12345678901234567890123456789012123456789012345678901
traveled to Asia? 2
12345678901234567890123456789012123456789012345678901 2
12345678901234567890123456789012123456789012345678901 2
12345678901234567890123456789012123456789012345678901
A. 28 2
12345678901234567890123456789012123456789012345678901 2
12345678901234567890123456789012123456789012345678901
B. 38 2
12345678901234567890123456789012123456789012345678901 2
12345678901234567890123456789012123456789012345678901
C. 42
2345678901234567890123456789012123456789012345678901
2
2
1
12345678901234567890123456789012123456789012345678901
D. 50 2
12345678901234567890123456789012123456789012345678901 2
12345678901234567890123456789012123456789012345678901
E. 54 2
12345678901234567890123456789012123456789012345678901 2
1 This question involves both figures. The Venn diagram indicates there are 2
2345678901234567890123456789012123456789012345678901
12345678901234567890123456789012123456789012345678901 2
12345678901234567890123456789012123456789012345678901
155 travelers to Asia, so the percentages in the bar graph are percentages 2
12345678901234567890123456789012123456789012345678901 2
12345678901234567890123456789012123456789012345678901
of 155. Find the 51–100 category and read the value of the lightly-shaded 2
2345678901234567890123456789012123456789012345678901 2
1
12345678901234567890123456789012123456789012345678901
column. It is about midway between 25% and 30%, so estimate it at 2
12345678901234567890123456789012123456789012345678901 2
12345678901234567890123456789012123456789012345678901
27.5%. 2
12345678901234567890123456789012123456789012345678901 2
2345678901234567890123456789012123456789012345678901
12345678901234567890123456789012123456789012345678901
Ten percent of 155 is 15.5, and so 20% is 31. Five percent of 155 is 7.75. 2
2
12345678901234567890123456789012123456789012345678901
If we add the 20% number to the 5% number, we should get (31 1 7.75 5 2
12345678901234567890123456789012123456789012345678901
12345678901234567890123456789012123456789012345678901 2
12345678901234567890123456789012123456789012345678901 2
12345678901234567890123456789012123456789012345678901 2
38.75). If 25% of 155 is 38.75, then 27.5% must be a little higher than
2
1 this. Choice (C) is in the right neighborhood.
12345678901234567890123456789012123456789012345678901 2
12345678901234567890123456789012123456789012345678901 2
12345678901234567890123456789012123456789012345678901 2
12345678901234567890123456789012123456789012345678901 2
2345678901234567890123456789012123456789012345678901
12345678901234567890123456789012123456789012345678901 2
12345678901234567890123456789012123456789012345678901 2
12345678901234567890123456789012123456789012345678901 2
12345678901234567890123456789012123456789012345678901 2
12345678901234567890123456789012123456789012345678901 2
12345678901234567890123456789012123456789012345678901 2
12345678901234567890123456789012123456789012345678901 2
12345678901234567890123456789012123456789012345678901 2 2
1
12345678901234567890123456789012123456789012345678901 2
12345678901234567890123456789012123456789012345678901 2
12345678901234567890123456789012123456789012345678901 2
12345678901234567890123456789012123456789012345678901 2
12345678901234567890123456789012123456789012345678901 2
12345678901234567890123456789012123456789012345678901 2
12345678901234567890123456789012123456789012345678901 2
12345678901234567890123456789012123456789012345678901 2
12345678901234567890123456789012123456789012345678901 2
12345678901234567890123456789012123456789012345678901 2
12345678901234567890123456789012123456789012345678901 2
2345678901234567890123456789012123456789012345678901
12345678901234567890123456789012123456789012345678901 2 2
1
12345678901234567890123456789012123456789012345678901 2
12345678901234567890123456789012123456789012345678901 2
12345678901234567890123456789012123456789012345678901 2
12345678901234567890123456789012123456789012345678901 2
12345678901234567890123456789012123456789012345678901 2
12345678901234567890123456789012123456789012345678901 2
12345678901234567890123456789012123456789012345678901 2
12345678901234567890123456789012123456789012345678901 2
2345678901234567890123456789012123456789012345678901
12345678901234567890123456789012123456789012345678901 2
12345678901234567890123456789012123456789012345678901 2
12345678901234567890123456789012123456789012345678901 2 2
1
12345678901234567890123456789012123456789012345678901 2
12345678901234567890123456789012123456789012345678901 2
12345678901234567890123456789012123456789012345678901 2
12345678901234567890123456789012123456789012345678901 2
12345678901234567890123456789012123456789012345678901 2
12345678901234567890123456789012123456789012345678901 2
2345678901234567890123456789012123456789012345678901
2
1
12345678901234567890123456789012123456789012345678901 2
12345678901234567890123456789012123456789012345678901 2
12345678901234567890123456789012123456789012345678901 2
12345678901234567890123456789012123456789012345678901 2
12345678901234567890123456789012123456789012345678901 2
12345678901234567890123456789012123456789012345678901 2 2
130 12345678901234567890123456789012123456789012345678901
123456789012345678901234567890121234567890123456789012

www.petersons.com
Chapter 4: Quantitative Review

123456789012345678901234567890121234567890123456789012
12345678901234567890123456789012123456789012345678901 2
12345678901234567890123456789012123456789012345678901 2
12345678901234567890123456789012123456789012345678901
You might also see a less conventional graph set than the previous one. The 2
12345678901234567890123456789012123456789012345678901 2
12345678901234567890123456789012123456789012345678901
test likes to see if you can think on your feet by presenting information in 2
12345678901234567890123456789012123456789012345678901 2
12345678901234567890123456789012123456789012345678901
a format that you might not be expecting. Don’t be surprised if you see 2
2345678901234567890123456789012123456789012345678901 2
1 something wacky like the following on test day.
12345678901234567890123456789012123456789012345678901 2
12345678901234567890123456789012123456789012345678901 2
12345678901234567890123456789012123456789012345678901 2
12345678901234567890123456789012123456789012345678901
2345678901234567890123456789012123456789012345678901
2
2
1
12345678901234567890123456789012123456789012345678901 2
12345678901234567890123456789012123456789012345678901 2
12345678901234567890123456789012123456789012345678901 2
12345678901234567890123456789012123456789012345678901
2345678901234567890123456789012123456789012345678901
2
2
1
12345678901234567890123456789012123456789012345678901 2
12345678901234567890123456789012123456789012345678901 2
12345678901234567890123456789012123456789012345678901 2
12345678901234567890123456789012123456789012345678901
2345678901234567890123456789012123456789012345678901
2
2
1
12345678901234567890123456789012123456789012345678901 2
12345678901234567890123456789012123456789012345678901 2
12345678901234567890123456789012123456789012345678901 2
12345678901234567890123456789012123456789012345678901 2
12345678901234567890123456789012123456789012345678901 22
2345678901234567890123456789012123456789012345678901
12345678901234567890123456789012123456789012345678901 2

Take It to the Next Level


12345678901234567890123456789012123456789012345678901 2
12345678901234567890123456789012123456789012345678901 2
12345678901234567890123456789012123456789012345678901 2
12345678901234567890123456789012123456789012345678901 2
1
12345678901234567890123456789012123456789012345678901 2
12345678901234567890123456789012123456789012345678901 2
12345678901234567890123456789012123456789012345678901 2
12345678901234567890123456789012123456789012345678901 22
2345678901234567890123456789012123456789012345678901
12345678901234567890123456789012123456789012345678901 2
12345678901234567890123456789012123456789012345678901 2
12345678901234567890123456789012123456789012345678901 2
12345678901234567890123456789012123456789012345678901 2
12345678901234567890123456789012123456789012345678901 2
12345678901234567890123456789012123456789012345678901 2
12345678901234567890123456789012123456789012345678901 2
12345678901234567890123456789012123456789012345678901 2
1
12345678901234567890123456789012123456789012345678901
How many miles is the section of Rt. 2 that runs through the city 2
12345678901234567890123456789012123456789012345678901 2
12345678901234567890123456789012123456789012345678901
limits of Douglass? 2
12345678901234567890123456789012123456789012345678901 2
12345678901234567890123456789012123456789012345678901 2
12345678901234567890123456789012123456789012345678901
A. 2 2
12345678901234567890123456789012123456789012345678901 2
12345678901234567890123456789012123456789012345678901
B. =6 2
12345678901234567890123456789012123456789012345678901 2
12345678901234567890123456789012123456789012345678901
C. =8 2
12345678901234567890123456789012123456789012345678901 22
2345678901234567890123456789012123456789012345678901
12345678901234567890123456789012123456789012345678901 2
1 D. 3
12345678901234567890123456789012123456789012345678901 2
12345678901234567890123456789012123456789012345678901
E. = 12 2
12345678901234567890123456789012123456789012345678901 22
2345678901234567890123456789012123456789012345678901
12345678901234567890123456789012123456789012345678901
Rt. 2 in the city limits works out to be the hypotenuse of two different right 2
12345678901234567890123456789012123456789012345678901 2
12345678901234567890123456789012123456789012345678901
1 triangles. You can determine the length of the section of Rt. 2 with either 2
2345678901234567890123456789012123456789012345678901
triangle, so pick one and go with it. Both of the other sides of the triangles 2
12345678901234567890123456789012123456789012345678901
12345678901234567890123456789012123456789012345678901 2
12345678901234567890123456789012123456789012345678901 2
12345678901234567890123456789012123456789012345678901 22
are 2 miles, so both triangles are 45-45-90 triangles. The length of the
1 hypotenuse can simply be read off, 2=2, which is equal to choice (C).
12345678901234567890123456789012123456789012345678901 2
12345678901234567890123456789012123456789012345678901 2
12345678901234567890123456789012123456789012345678901 2
12345678901234567890123456789012123456789012345678901 2
12345678901234567890123456789012123456789012345678901 2
12345678901234567890123456789012123456789012345678901 22
2345678901234567890123456789012123456789012345678901
1
12345678901234567890123456789012123456789012345678901 2
12345678901234567890123456789012123456789012345678901 2
12345678901234567890123456789012123456789012345678901 2
12345678901234567890123456789012123456789012345678901 2
12345678901234567890123456789012123456789012345678901 2
12345678901234567890123456789012123456789012345678901 2
12345678901234567890123456789012123456789012345678901 2 131
123456789012345678901234567890121234567890123456789012
Part III: Section Review

123456789012345678901234567890121234567890123456789012
12345678901234567890123456789012123456789012345678901 2
12345678901234567890123456789012123456789012345678901 2
12345678901234567890123456789012123456789012345678901
What approximate percentage of Douglass County is within the city 2
12345678901234567890123456789012123456789012345678901 2
12345678901234567890123456789012123456789012345678901
limits of Douglass? 2
12345678901234567890123456789012123456789012345678901 2
12345678901234567890123456789012123456789012345678901 2
12345678901234567890123456789012123456789012345678901
A. 20% 2
12345678901234567890123456789012123456789012345678901 2
12345678901234567890123456789012123456789012345678901
B. 33% 2
12345678901234567890123456789012123456789012345678901 2
12345678901234567890123456789012123456789012345678901
C. 42%
2345678901234567890123456789012123456789012345678901
2
2
1
12345678901234567890123456789012123456789012345678901
D. 50% 2
12345678901234567890123456789012123456789012345678901 2
12345678901234567890123456789012123456789012345678901
E. 66% 2
12345678901234567890123456789012123456789012345678901 2
1 The areas of the county and the city have to be determined to find this 2
2345678901234567890123456789012123456789012345678901
12345678901234567890123456789012123456789012345678901 2
12345678901234567890123456789012123456789012345678901
percentage. This is greatly simplified by the fact that both areas are 2
12345678901234567890123456789012123456789012345678901 2
12345678901234567890123456789012123456789012345678901
rectangles, which you can infer from the note that all the lines are parallel 2
2345678901234567890123456789012123456789012345678901 2
1
12345678901234567890123456789012123456789012345678901
and at right angles in the diagram. Examining the right length of the 2
12345678901234567890123456789012123456789012345678901 2
12345678901234567890123456789012123456789012345678901
diagram, the length of the diagram is 6 (4 miles 12 miles). The width is 3 2
12345678901234567890123456789012123456789012345678901 2
12345678901234567890123456789012123456789012345678901
(read off from the bottom width as 0.5 12 1 0.5). 2
12345678901234567890123456789012123456789012345678901 2
12345678901234567890123456789012123456789012345678901 2
12345678901234567890123456789012123456789012345678901
This makes the area of the county 3 3 6 5 18. The length of the city is 3 2
12345678901234567890123456789012123456789012345678901 2
12345678901234567890123456789012123456789012345678901 2
2345678901234567890123456789012123456789012345678901
and the width is 2, so the city‘s area is 6. The ratio of city area to county
2
1
12345678901234567890123456789012123456789012345678901
6 1 2
12345678901234567890123456789012123456789012345678901
area then is 5 ' 0.33. The correct answer is (B). 2
12345678901234567890123456789012123456789012345678901
18 3 2
12345678901234567890123456789012123456789012345678901 2
2345678901234567890123456789012123456789012345678901
12345678901234567890123456789012123456789012345678901 2
You can drop the units since all the units are the same and will cancel each 2
12345678901234567890123456789012123456789012345678901

Tip
12345678901234567890123456789012123456789012345678901 2
12345678901234567890123456789012123456789012345678901 2
12345678901234567890123456789012123456789012345678901 2
other out.
12345678901234567890123456789012123456789012345678901 2
12345678901234567890123456789012123456789012345678901 2
12345678901234567890123456789012123456789012345678901 2 2
1
12345678901234567890123456789012123456789012345678901
If you are 2 miles due southeast of the intersection of Rt. 7 and 2
12345678901234567890123456789012123456789012345678901 2
12345678901234567890123456789012123456789012345678901 2
Rt. 4, where are you?
12345678901234567890123456789012123456789012345678901 2
12345678901234567890123456789012123456789012345678901 2
12345678901234567890123456789012123456789012345678901
A. In the city 2
12345678901234567890123456789012123456789012345678901 2
12345678901234567890123456789012123456789012345678901 2
B. Outside the city but in the county
12345678901234567890123456789012123456789012345678901
C. Outside the county 2
12345678901234567890123456789012123456789012345678901 2
2345678901234567890123456789012123456789012345678901
12345678901234567890123456789012123456789012345678901
D. Along Rt. 10 2
12345678901234567890123456789012123456789012345678901 2
1 E. Along Rt. 2 2
12345678901234567890123456789012123456789012345678901 2
12345678901234567890123456789012123456789012345678901 2
12345678901234567890123456789012123456789012345678901
First find the intersection of Rt. 7 and Rt. 4. It is on the left side. According 2
12345678901234567890123456789012123456789012345678901 2
12345678901234567890123456789012123456789012345678901
to the compass rose, Rt. 4 runs north-south and Rt. 7 runs east-west. 2
12345678901234567890123456789012123456789012345678901 2
12345678901234567890123456789012123456789012345678901
Southeast then would be halfway between heading east on Rt. 7 and south 2
2345678901234567890123456789012123456789012345678901
on Rt. 4. If there were a line defining this direction, it would form a 2
12345678901234567890123456789012123456789012345678901
12345678901234567890123456789012123456789012345678901 2
12345678901234567890123456789012123456789012345678901 2
12345678901234567890123456789012123456789012345678901 2
45-degree angle with Rt. 7 and with Rt. 4. If that line is long enough, it will
1 form two 45-45-90 triangles like you saw that Rt. 2 did two questions 2
12345678901234567890123456789012123456789012345678901 2
12345678901234567890123456789012123456789012345678901 2
12345678901234567890123456789012123456789012345678901
back. However, in that instance you found that the length of Rt. 2 in the 2
12345678901234567890123456789012123456789012345678901 2
12345678901234567890123456789012123456789012345678901
city was 2= 2 miles, which is greater than 2 miles. So 2 miles southeast of 2
2345678901234567890123456789012123456789012345678901
12345678901234567890123456789012123456789012345678901 2
1 the intersection is not outside of the box defined by the city limits south of 2
12345678901234567890123456789012123456789012345678901 2
12345678901234567890123456789012123456789012345678901
Rt. 7. So you would be inside the city, choice (A). 2
12345678901234567890123456789012123456789012345678901
2345678901234567890123456789012123456789012345678901
2
2
1
12345678901234567890123456789012123456789012345678901 2
12345678901234567890123456789012123456789012345678901 2 2
132 12345678901234567890123456789012123456789012345678901
123456789012345678901234567890121234567890123456789012

www.petersons.com
Chapter 4: Quantitative Review

123456789012345678901234567890121234567890123456789012
12345678901234567890123456789012123456789012345678901 2
12345678901234567890123456789012123456789012345678901 22
A 112345678901234567890123456789012123456789012345678901
Final Word
2345678901234567890123456789012123456789012345678901 2
12345678901234567890123456789012123456789012345678901 2
12345678901234567890123456789012123456789012345678901 2
The Next Level section of this chapter covers many of the favorite ways to
12345678901234567890123456789012123456789012345678901
make a GRE problem hard, but not all of them. Invariably, if you are doing 2
12345678901234567890123456789012123456789012345678901 2
12345678901234567890123456789012123456789012345678901 2
12345678901234567890123456789012123456789012345678901
really well on the GRE Math section, you will encounter something that at 2
12345678901234567890123456789012123456789012345678901 2
12345678901234567890123456789012123456789012345678901
first glance looks impossibly complex. Ask yourself, “How did the 2
1 test-makers make this problem hard?” Is it a multi-step problem involving 2
2345678901234567890123456789012123456789012345678901
12345678901234567890123456789012123456789012345678901 2
12345678901234567890123456789012123456789012345678901 2
12345678901234567890123456789012123456789012345678901
a host of equations? If so, the goal is often to find terms that cancel each 2
12345678901234567890123456789012123456789012345678901
other out. Does the question have a bunch
2345678901234567890123456789012123456789012345678901of interconnected figures? If so, 2
2
1
12345678901234567890123456789012123456789012345678901
the key is usually to figure out facts about individual figures and then use 2
12345678901234567890123456789012123456789012345678901 2
12345678901234567890123456789012123456789012345678901
this knowledge to gain information about other figures. With an 2
12345678901234567890123456789012123456789012345678901 2
12345678901234567890123456789012123456789012345678901
aggressive attitude and a willingness to get creative and keep working, 2
12345678901234567890123456789012123456789012345678901 2
12345678901234567890123456789012123456789012345678901
many of the hardest GRE problems will be yours for the taking. 2
12345678901234567890123456789012123456789012345678901 2
12345678901234567890123456789012123456789012345678901 2
12345678901234567890123456789012123456789012345678901 22
2345678901234567890123456789012123456789012345678901
12345678901234567890123456789012123456789012345678901 2

Take It to the Next Level


12345678901234567890123456789012123456789012345678901 2
12345678901234567890123456789012123456789012345678901 2
12345678901234567890123456789012123456789012345678901 2
12345678901234567890123456789012123456789012345678901 2
1
12345678901234567890123456789012123456789012345678901 2
12345678901234567890123456789012123456789012345678901 2
12345678901234567890123456789012123456789012345678901 2
12345678901234567890123456789012123456789012345678901 22
2345678901234567890123456789012123456789012345678901
12345678901234567890123456789012123456789012345678901 2
12345678901234567890123456789012123456789012345678901 2
12345678901234567890123456789012123456789012345678901 2
12345678901234567890123456789012123456789012345678901 2
12345678901234567890123456789012123456789012345678901 2
12345678901234567890123456789012123456789012345678901 2
12345678901234567890123456789012123456789012345678901 2
12345678901234567890123456789012123456789012345678901 2
1
12345678901234567890123456789012123456789012345678901 2
12345678901234567890123456789012123456789012345678901 2
12345678901234567890123456789012123456789012345678901 2
12345678901234567890123456789012123456789012345678901 2
12345678901234567890123456789012123456789012345678901 2
12345678901234567890123456789012123456789012345678901 2
12345678901234567890123456789012123456789012345678901 2
12345678901234567890123456789012123456789012345678901 2
12345678901234567890123456789012123456789012345678901 2
12345678901234567890123456789012123456789012345678901 2
12345678901234567890123456789012123456789012345678901 22
2345678901234567890123456789012123456789012345678901
12345678901234567890123456789012123456789012345678901 2
1
12345678901234567890123456789012123456789012345678901 2
12345678901234567890123456789012123456789012345678901 2
12345678901234567890123456789012123456789012345678901 2
12345678901234567890123456789012123456789012345678901 2
12345678901234567890123456789012123456789012345678901 2
12345678901234567890123456789012123456789012345678901 2
12345678901234567890123456789012123456789012345678901 2
12345678901234567890123456789012123456789012345678901 22
2345678901234567890123456789012123456789012345678901
12345678901234567890123456789012123456789012345678901 2
12345678901234567890123456789012123456789012345678901 2
12345678901234567890123456789012123456789012345678901 2
1
12345678901234567890123456789012123456789012345678901 2
12345678901234567890123456789012123456789012345678901 2
12345678901234567890123456789012123456789012345678901 2
12345678901234567890123456789012123456789012345678901 2
12345678901234567890123456789012123456789012345678901 2
12345678901234567890123456789012123456789012345678901 22
2345678901234567890123456789012123456789012345678901
1
12345678901234567890123456789012123456789012345678901 2
12345678901234567890123456789012123456789012345678901 2
12345678901234567890123456789012123456789012345678901 2
12345678901234567890123456789012123456789012345678901 2
12345678901234567890123456789012123456789012345678901 2
12345678901234567890123456789012123456789012345678901 2
12345678901234567890123456789012123456789012345678901 2 133
123456789012345678901234567890121234567890123456789012
Chapter

5
Verbal Review
123456789012345678901234567890121234567890123456789012
12345678901234567890123456789012123456789012345678901 2
12345678901234567890123456789012123456789012345678901 2
12345678901234567890123456789012123456789012345678901
The Perfect Charm for Those Shooting to 2
12345678901234567890123456789012123456789012345678901 2
2345678901234567890123456789012123456789012345678901
1
Score above a 500 2
12345678901234567890123456789012123456789012345678901 2
12345678901234567890123456789012123456789012345678901 2
12345678901234567890123456789012123456789012345678901
When people talk about standardized tests, they often fall into one of two 2
2345678901234567890123456789012123456789012345678901
12345678901234567890123456789012123456789012345678901 2
1 camps. The first group believes that the tests are a reasonably accurate 2
12345678901234567890123456789012123456789012345678901
indication of a person’s “intelligence.” It stands to reason, then, that 2
12345678901234567890123456789012123456789012345678901 2
12345678901234567890123456789012123456789012345678901 2
12345678901234567890123456789012123456789012345678901 2
2345678901234567890123456789012123456789012345678901
someone who scores well on a standardized test must be “smart.” The
second camp does not hold standardized tests in such a high esteem light. 2
12345678901234567890123456789012123456789012345678901
12345678901234567890123456789012123456789012345678901 2
12345678901234567890123456789012123456789012345678901 2
12345678901234567890123456789012123456789012345678901 2
To people in the second camp, the only thing a standardized test measures
is how well someone does on that particular standardized test. To them, 2
12345678901234567890123456789012123456789012345678901
12345678901234567890123456789012123456789012345678901 2
12345678901234567890123456789012123456789012345678901 2
12345678901234567890123456789012123456789012345678901 2
standardized tests are too arbitrary and narrowly focused to be any kind of
2
1 accurate indicator of intelligence.
12345678901234567890123456789012123456789012345678901 2
12345678901234567890123456789012123456789012345678901 2
12345678901234567890123456789012123456789012345678901
By the time you are finished reading through this section (and this book), 2
12345678901234567890123456789012123456789012345678901 2
12345678901234567890123456789012123456789012345678901
many of you in the first camp will have migrated to the second camp. This 2
12345678901234567890123456789012123456789012345678901 2
12345678901234567890123456789012123456789012345678901
is because many of you will no longer think of the GRE Verbal section as a 2
12345678901234567890123456789012123456789012345678901 2
12345678901234567890123456789012123456789012345678901
comprehensive, multiple-choice exam that uses a variety of question 2
12345678901234567890123456789012123456789012345678901 2
12345678901234567890123456789012123456789012345678901
formats to accurately gauge a person’s level of English mastery. In place of 2
12345678901234567890123456789012123456789012345678901 2
12345678901234567890123456789012123456789012345678901
this idea, many of you will view the test in a much different way. 2
12345678901234567890123456789012123456789012345678901 2
12345678901234567890123456789012123456789012345678901 2
12345678901234567890123456789012123456789012345678901 2
12345678901234567890123456789012123456789012345678901 2
12345678901234567890123456789012123456789012345678901
The GRE Verbal Section Is a Vocabulary Quiz 2
12345678901234567890123456789012123456789012345678901 2
12345678901234567890123456789012123456789012345678901 2
12345678901234567890123456789012123456789012345678901 2
2345678901234567890123456789012123456789012345678901
Gone Wild
12345678901234567890123456789012123456789012345678901 2
For the Verbal section, you get 30 minutes to answer 30 questions that 2
12345678901234567890123456789012123456789012345678901
12345678901234567890123456789012123456789012345678901 2
1 come in four different types: 2
12345678901234567890123456789012123456789012345678901 2
2345678901234567890123456789012123456789012345678901
2
12345678901234567890123456789012123456789012345678901 2
12345678901234567890123456789012123456789012345678901
1. Antonyms
12345678901234567890123456789012123456789012345678901 2 2
1
12345678901234567890123456789012123456789012345678901
2. Analogies 2
12345678901234567890123456789012123456789012345678901 2
12345678901234567890123456789012123456789012345678901 2
12345678901234567890123456789012123456789012345678901
3. Sentence Completions (just SC if you’re in a hurry) 2
12345678901234567890123456789012123456789012345678901
2345678901234567890123456789012123456789012345678901
2
2
1 4. Reading Comprehension (RC for those on the go)
12345678901234567890123456789012123456789012345678901 2
12345678901234567890123456789012123456789012345678901 2
12345678901234567890123456789012123456789012345678901 2
123456789012345678901234567890121234567890123456789012
134
Chapter 5: Verbal Review

123456789012345678901234567890121234567890123456789012
12345678901234567890123456789012123456789012345678901 2
12345678901234567890123456789012123456789012345678901 2
12345678901234567890123456789012123456789012345678901
A chart describing these four question types—as well as a discussion about 2
12345678901234567890123456789012123456789012345678901 2
12345678901234567890123456789012123456789012345678901
the basic approach to the GRE Verbal test—can be found starting on page 2
12345678901234567890123456789012123456789012345678901 2
12345678901234567890123456789012123456789012345678901
25. Be sure to read this section if you haven’t already done so. The bulk of 2
1 this chapter will discuss each question type in detail, but you need to fit 2
2345678901234567890123456789012123456789012345678901
12345678901234567890123456789012123456789012345678901 2
12345678901234567890123456789012123456789012345678901 2
12345678901234567890123456789012123456789012345678901
these individual ideas into the overall Verbal strategy covered in the earlier 2
12345678901234567890123456789012123456789012345678901
2345678901234567890123456789012123456789012345678901
2
2
1 section.
12345678901234567890123456789012123456789012345678901 2
12345678901234567890123456789012123456789012345678901 2
12345678901234567890123456789012123456789012345678901
Like the Quantitative section, the GRE Verbal section is scored on a scale 2
12345678901234567890123456789012123456789012345678901
2345678901234567890123456789012123456789012345678901
2
2
1 from 200 to 800, with 800 a perfect score. Theoretically, a 500 would be
12345678901234567890123456789012123456789012345678901 2
12345678901234567890123456789012123456789012345678901
an average score, and this chapter is going to emphasize strategies to help 2
12345678901234567890123456789012123456789012345678901
you score above a 500 on the GRE Verbal section. A Verbal score in the 2
12345678901234567890123456789012123456789012345678901 2
1 high 500s to mid 600s can be considered a solid score, although if you are 2
2345678901234567890123456789012123456789012345678901
12345678901234567890123456789012123456789012345678901 2
12345678901234567890123456789012123456789012345678901
applying for an English program you will most likely want to stay above 2
12345678901234567890123456789012123456789012345678901 2
12345678901234567890123456789012123456789012345678901
600. The fact that the scores are scaled makes for some strange 2
12345678901234567890123456789012123456789012345678901 22
2345678901234567890123456789012123456789012345678901
12345678901234567890123456789012123456789012345678901
discrepancies between the percentile scores and scaled scores. For
12345678901234567890123456789012123456789012345678901
example, a 700 Verbal might translate to the 96th percentile, meaning that 2
2
12345678901234567890123456789012123456789012345678901
12345678901234567890123456789012123456789012345678901
this score was better than 95 out of 100 test-takers. However, a higher 2
2
12345678901234567890123456789012123456789012345678901
1 Quantitative scaled score might translate to a lower percentile score 2
12345678901234567890123456789012123456789012345678901 2
12345678901234567890123456789012123456789012345678901 2
12345678901234567890123456789012123456789012345678901
because of the results of all the other people taking the GRE CAT. On the 2
2345678901234567890123456789012123456789012345678901
same test where a 700 Verbal equals the 96th percentile, a 760 2
12345678901234567890123456789012123456789012345678901
12345678901234567890123456789012123456789012345678901 2
12345678901234567890123456789012123456789012345678901 2
12345678901234567890123456789012123456789012345678901 22
Quantitative scaled score might only rank in the 83 rd
percentile. Even
12345678901234567890123456789012123456789012345678901
though the scaled score is higher, the percentile ranking is lower. If this 2
12345678901234567890123456789012123456789012345678901
seems a little odd, that’s because it is. The end result is that it is usually 2
12345678901234567890123456789012123456789012345678901
12345678901234567890123456789012123456789012345678901 2
12345678901234567890123456789012123456789012345678901 2
1 tougher to get a high Verbal scaled score than it is to get a high 2
2345678901234567890123456789012123456789012345678901
Quantitative scaled score because the pool of GRE test-takers is skewed 2
12345678901234567890123456789012123456789012345678901
12345678901234567890123456789012123456789012345678901 2
1 with people who score better in Verbal than Math. 2
12345678901234567890123456789012123456789012345678901 2
12345678901234567890123456789012123456789012345678901 2
12345678901234567890123456789012123456789012345678901 2
12345678901234567890123456789012123456789012345678901 2
12345678901234567890123456789012123456789012345678901 2
You receive your scaled scores immediately after completing the entire
12345678901234567890123456789012123456789012345678901 2
12345678901234567890123456789012123456789012345678901
test. There are also percentile scores (0–99%) for both the Verbal and 2
2345678901234567890123456789012123456789012345678901
12345678901234567890123456789012123456789012345678901 2
Alert!

12345678901234567890123456789012123456789012345678901
Quantitative sections. These percentile scores can be found in the same 2
1 letter that gives you your Analytical score. The percentile score can be a 2
12345678901234567890123456789012123456789012345678901 2
12345678901234567890123456789012123456789012345678901
little strange, since it varies over time depending on all the people taking 2
12345678901234567890123456789012123456789012345678901 2
12345678901234567890123456789012123456789012345678901
the test. Most graduate programs are more interested in the scaled scores, 2
12345678901234567890123456789012123456789012345678901 2
12345678901234567890123456789012123456789012345678901
so don’t lose too much sleep trying to figure out the vagaries of percentile 2
12345678901234567890123456789012123456789012345678901 2
2345678901234567890123456789012123456789012345678901
12345678901234567890123456789012123456789012345678901
scores. 2
12345678901234567890123456789012123456789012345678901 2
12345678901234567890123456789012123456789012345678901 22
12345678901234567890123456789012123456789012345678901 2
1 Although there are four different question types, the first three all have a
12345678901234567890123456789012123456789012345678901 2
12345678901234567890123456789012123456789012345678901
heavy vocabulary component. The first question type, Antonyms, is 2
12345678901234567890123456789012123456789012345678901 2
12345678901234567890123456789012123456789012345678901
almost exclusively a vocabulary quiz, since you often need to know the 2
12345678901234567890123456789012123456789012345678901 2
12345678901234567890123456789012123456789012345678901
definition of a word in order to pick its opposite. There are some guessing 2
12345678901234567890123456789012123456789012345678901 2
12345678901234567890123456789012123456789012345678901
techniques that enable you to work on an antonym problem even if you 2
12345678901234567890123456789012123456789012345678901
don’t know the definition of the word, but for the most part many 2
12345678901234567890123456789012123456789012345678901
2345678901234567890123456789012123456789012345678901
2
2
1 antonym problems boil down to, “Do you know the word or not?”
12345678901234567890123456789012123456789012345678901 2
12345678901234567890123456789012123456789012345678901 2
12345678901234567890123456789012123456789012345678901 2 135
123456789012345678901234567890121234567890123456789012
Part III: Section Review

123456789012345678901234567890121234567890123456789012
12345678901234567890123456789012123456789012345678901 2
12345678901234567890123456789012123456789012345678901 2
12345678901234567890123456789012123456789012345678901
Both analogies and sentence completions have formats that are more 2
12345678901234567890123456789012123456789012345678901 2
12345678901234567890123456789012123456789012345678901
complicated than antonyms, and both formats can be approached with a 2
12345678901234567890123456789012123456789012345678901 2
12345678901234567890123456789012123456789012345678901
host of methods. However, the presence of difficult vocabulary words in 2
1 the answer choices can make things difficult. For instance, you might read 2
2345678901234567890123456789012123456789012345678901
12345678901234567890123456789012123456789012345678901 2
12345678901234567890123456789012123456789012345678901 2
12345678901234567890123456789012123456789012345678901
a SC problem and realize the word in the blank needs to mean something 2
12345678901234567890123456789012123456789012345678901
2345678901234567890123456789012123456789012345678901
2
2
1 akin to “small,” but then look down and see that your answer choices are:
12345678901234567890123456789012123456789012345678901 2
12345678901234567890123456789012123456789012345678901 2
12345678901234567890123456789012123456789012345678901
A. weighty 2
12345678901234567890123456789012123456789012345678901
2345678901234567890123456789012123456789012345678901
2
2
1 B. pulchritudinous
12345678901234567890123456789012123456789012345678901 2
12345678901234567890123456789012123456789012345678901
C. pusillanimous 2
12345678901234567890123456789012123456789012345678901
D. voluminous 2
12345678901234567890123456789012123456789012345678901
2345678901234567890123456789012123456789012345678901
2
2
1 E. angry
12345678901234567890123456789012123456789012345678901 2
12345678901234567890123456789012123456789012345678901 2
12345678901234567890123456789012123456789012345678901
Choices (A) and (E) are well-known words, but (B), (C), and (D) are not as 2
12345678901234567890123456789012123456789012345678901 2
12345678901234567890123456789012123456789012345678901
common. For this question, you could eliminate (A) and (E) since you 2
12345678901234567890123456789012123456789012345678901 2
12345678901234567890123456789012123456789012345678901
know that neither of these words means “small,” but you would then be 2
12345678901234567890123456789012123456789012345678901 2
12345678901234567890123456789012123456789012345678901
left with a choice of (B), (C), and (D). Many of you might be able to look at 2
2345678901234567890123456789012123456789012345678901
12345678901234567890123456789012123456789012345678901 2
1 voluminous and recognize the root volume in the word, making this choice 2
12345678901234567890123456789012123456789012345678901
unlikely to be the answer. This would leave you with two choices to guess 2
12345678901234567890123456789012123456789012345678901 2
12345678901234567890123456789012123456789012345678901 2
12345678901234567890123456789012123456789012345678901 2
2345678901234567890123456789012123456789012345678901
from, illustrating how good test strategies can take you a long way, but
sometimes not all the way to finding the answer outright. For the last push, 2
12345678901234567890123456789012123456789012345678901
12345678901234567890123456789012123456789012345678901 2
12345678901234567890123456789012123456789012345678901 2
12345678901234567890123456789012123456789012345678901 2
you would have to know that pulchritudinous is an ugly-sounding word
that surprisingly means “beautiful,” while pusillanimous is a very large 2
12345678901234567890123456789012123456789012345678901
12345678901234567890123456789012123456789012345678901 2
12345678901234567890123456789012123456789012345678901 2
12345678901234567890123456789012123456789012345678901 2
word that means “small.” How’s that for opposites?
2
1
12345678901234567890123456789012123456789012345678901
Three fourths of the GRE Verbal question types have a vocabulary 2
12345678901234567890123456789012123456789012345678901 2
12345678901234567890123456789012123456789012345678901
component to them. Combine this with the fact that the first 3–6 questions 2
12345678901234567890123456789012123456789012345678901 2
12345678901234567890123456789012123456789012345678901
you face will be both antonyms and analogies, and that the first questions 2
12345678901234567890123456789012123456789012345678901 2
12345678901234567890123456789012123456789012345678901
on a computer adaptive test section count more than the questions at the 2
12345678901234567890123456789012123456789012345678901 2
12345678901234567890123456789012123456789012345678901
end. The end result is a GRE Verbal exam that is a lot like a “souped-up” 2
12345678901234567890123456789012123456789012345678901 2
12345678901234567890123456789012123456789012345678901
vocabulary test, or a vocabulary quiz gone wild. 2
12345678901234567890123456789012123456789012345678901 2
12345678901234567890123456789012123456789012345678901 2
12345678901234567890123456789012123456789012345678901 2
If you wanted to practice your Analytical skills, you can write an essay
12345678901234567890123456789012123456789012345678901
showing why the GRE Verbal section is not like a vocabulary quiz on 2
12345678901234567890123456789012123456789012345678901 2
12345678901234567890123456789012123456789012345678901 2
12345678901234567890123456789012123456789012345678901
steroids. You could go through and find the flaws in the author’s 2
12345678901234567890123456789012123456789012345678901 2
12345678901234567890123456789012123456789012345678901 2
argument, which admittedly is not airtight. This might help you improve
2345678901234567890123456789012123456789012345678901
your Analytical score, but it won’t do much for the GRE Verbal score. For 2
12345678901234567890123456789012123456789012345678901
12345678901234567890123456789012123456789012345678901 2
12345678901234567890123456789012123456789012345678901 2
12345678901234567890123456789012123456789012345678901 2
help in this area, the first thing you should do is improve your vocabulary,
2
1 because it plays a huge part on the GRE Verbal.
12345678901234567890123456789012123456789012345678901 2
12345678901234567890123456789012123456789012345678901 2
12345678901234567890123456789012123456789012345678901 2
12345678901234567890123456789012123456789012345678901 2
12345678901234567890123456789012123456789012345678901 2
12345678901234567890123456789012123456789012345678901 2
2345678901234567890123456789012123456789012345678901
2
1
12345678901234567890123456789012123456789012345678901 2
12345678901234567890123456789012123456789012345678901 2
12345678901234567890123456789012123456789012345678901 2
12345678901234567890123456789012123456789012345678901 2
12345678901234567890123456789012123456789012345678901 2
12345678901234567890123456789012123456789012345678901 2 2
136 12345678901234567890123456789012123456789012345678901
123456789012345678901234567890121234567890123456789012

www.petersons.com
Chapter 5: Verbal Review

123456789012345678901234567890121234567890123456789012
12345678901234567890123456789012123456789012345678901 2
12345678901234567890123456789012123456789012345678901 2
12345678901234567890123456789012123456789012345678901
Upgrading Your Personal Lexicon 2
12345678901234567890123456789012123456789012345678901 2
12345678901234567890123456789012123456789012345678901 2
12345678901234567890123456789012123456789012345678901 2
In the ideal scenario, you are reading this book when you are 5 years old.
12345678901234567890123456789012123456789012345678901
It might be the first book you ever read. If this is the case, then the GRE 2
12345678901234567890123456789012123456789012345678901 2
12345678901234567890123456789012123456789012345678901
CAT is almost two decades away, and you have an ample amount of time 2
12345678901234567890123456789012123456789012345678901 2
12345678901234567890123456789012123456789012345678901
to learn a ton of tough vocabulary words. Put the lollipop down and crack 2
12345678901234567890123456789012123456789012345678901 2
12345678901234567890123456789012123456789012345678901
open that dictionary, youngster! 2
12345678901234567890123456789012123456789012345678901 2
12345678901234567890123456789012123456789012345678901 2
12345678901234567890123456789012123456789012345678901
Sadly, most of you reading this book do not have as much time as the wee 2
12345678901234567890123456789012123456789012345678901 2
12345678901234567890123456789012123456789012345678901
child described above. That’s okay, too! It means you have an actual life. 2
12345678901234567890123456789012123456789012345678901
The simple fact is that the more time you have before the test, the more 2
12345678901234567890123456789012123456789012345678901 2
12345678901234567890123456789012123456789012345678901 2
12345678901234567890123456789012123456789012345678901
vocabulary words you can learn.
2345678901234567890123456789012123456789012345678901
2
2
1
12345678901234567890123456789012123456789012345678901 2
12345678901234567890123456789012123456789012345678901 2
12345678901234567890123456789012123456789012345678901
Tips on Learning New Vocabulary 2
12345678901234567890123456789012123456789012345678901 2
2345678901234567890123456789012123456789012345678901
1. Strive for Quality over Quantity. Sort of learning the definition of 2
12345678901234567890123456789012123456789012345678901
12345678901234567890123456789012123456789012345678901 2
1,000 words is not as good as actually learning 100 words. Building 2
12345678901234567890123456789012123456789012345678901
12345678901234567890123456789012123456789012345678901
vocabulary for a single test is a bit problematic to begin with, since there’s 2
2
12345678901234567890123456789012123456789012345678901
12345678901234567890123456789012123456789012345678901 2
1 no way you can cover all the possible words that might appear. Therefore, 2
12345678901234567890123456789012123456789012345678901 2
12345678901234567890123456789012123456789012345678901
don’t blow a gasket trying to guarantee you’ll recognize every word you 2
12345678901234567890123456789012123456789012345678901 2
12345678901234567890123456789012123456789012345678901
see, since it just isn’t going to happen. Instead, make sure the new words 2
12345678901234567890123456789012123456789012345678901
you do learn are remembered. That way, if one of these words shows up as 2
12345678901234567890123456789012123456789012345678901 2
12345678901234567890123456789012123456789012345678901 2
12345678901234567890123456789012123456789012345678901
an antonym, you have a great chance to answer that question correctly. 2
12345678901234567890123456789012123456789012345678901 22
2345678901234567890123456789012123456789012345678901
12345678901234567890123456789012123456789012345678901
People learn at different rates. Learning about 30 new words a week is a 2
12345678901234567890123456789012123456789012345678901
12345678901234567890123456789012123456789012345678901 2
1 nice goal. If you start six weeks before the test, that gives you almost 200 2
2345678901234567890123456789012123456789012345678901
new words. Feel free to adjust the 30 words/week figure up or down 2
12345678901234567890123456789012123456789012345678901
12345678901234567890123456789012123456789012345678901 2
1 depending on your own ability and available time. Just make sure that you 2
12345678901234567890123456789012123456789012345678901 2
12345678901234567890123456789012123456789012345678901
actually learn—and remember—the definitions of the words you are 2
12345678901234567890123456789012123456789012345678901 2
12345678901234567890123456789012123456789012345678901
studying. 2
12345678901234567890123456789012123456789012345678901 2
12345678901234567890123456789012123456789012345678901 2
12345678901234567890123456789012123456789012345678901
2. Don’t Let an Unknown Word Cross Your Path Again. The best way to 2
2345678901234567890123456789012123456789012345678901
12345678901234567890123456789012123456789012345678901
learn new words is when a word appears in the context of other words. 2
2
12345678901234567890123456789012123456789012345678901
1 Suppose you happen to be reading a magazine, book, or newspaper when 2
12345678901234567890123456789012123456789012345678901 2
12345678901234567890123456789012123456789012345678901
you come across the word charivari. Looking at the context of the word 2
12345678901234567890123456789012123456789012345678901 2
12345678901234567890123456789012123456789012345678901 2
12345678901234567890123456789012123456789012345678901
(how the word is used in the sentence), you can sort of figure out its 2
12345678901234567890123456789012123456789012345678901 2
12345678901234567890123456789012123456789012345678901 2
meaning. This allows you to keep on reading, even if you don’t know
2345678901234567890123456789012123456789012345678901
12345678901234567890123456789012123456789012345678901 2
12345678901234567890123456789012123456789012345678901
exactly what charivari means. 2
12345678901234567890123456789012123456789012345678901 22
12345678901234567890123456789012123456789012345678901
1 Instead of settling for this quasi-comprehension, write the unknown word 2
12345678901234567890123456789012123456789012345678901
down and then look it up later. Or look it up right then, if you have a 2
12345678901234567890123456789012123456789012345678901 2
12345678901234567890123456789012123456789012345678901
dictionary nearby. The goal is to add that word to your mental stable of 2
12345678901234567890123456789012123456789012345678901 2
12345678901234567890123456789012123456789012345678901
words, and it’s much easier to do this with words that you actually 2
12345678901234567890123456789012123456789012345678901 22
2345678901234567890123456789012123456789012345678901
1 encounter in the course of a day. Encountering the word in context helps
12345678901234567890123456789012123456789012345678901 2
12345678901234567890123456789012123456789012345678901
anchor the word in your mind. 2
12345678901234567890123456789012123456789012345678901
2345678901234567890123456789012123456789012345678901
2
2
1
12345678901234567890123456789012123456789012345678901 2
12345678901234567890123456789012123456789012345678901 2
12345678901234567890123456789012123456789012345678901 2 137
123456789012345678901234567890121234567890123456789012
Part III: Section Review

123456789012345678901234567890121234567890123456789012
12345678901234567890123456789012123456789012345678901 2
12345678901234567890123456789012123456789012345678901 2
12345678901234567890123456789012123456789012345678901 2
12345678901234567890123456789012123456789012345678901
You can keep a small notebook with you to write unknown words down, 2
12345678901234567890123456789012123456789012345678901 2
12345678901234567890123456789012123456789012345678901 2
12345678901234567890123456789012123456789012345678901
or if you’re techno-savvy you can store the words in a PDA or some 2

Note
1 similar device. For optimal benefit, write down the unknown word and 2
2345678901234567890123456789012123456789012345678901
12345678901234567890123456789012123456789012345678901 2
12345678901234567890123456789012123456789012345678901
the sentence in which it appears. When you look up the definition, you 2
12345678901234567890123456789012123456789012345678901 2
12345678901234567890123456789012123456789012345678901
can then add the definition to the word and the sample sentence you 2
2345678901234567890123456789012123456789012345678901 2
1
12345678901234567890123456789012123456789012345678901
already have. It almost goes without saying that creating a list this way 2
12345678901234567890123456789012123456789012345678901 2
12345678901234567890123456789012123456789012345678901
gives you a vocabulary review sheet at the same time. 2
12345678901234567890123456789012123456789012345678901
2345678901234567890123456789012123456789012345678901
2
2
1
12345678901234567890123456789012123456789012345678901 2
12345678901234567890123456789012123456789012345678901
Writing down and learning the meaning of unknown words will help 2
12345678901234567890123456789012123456789012345678901 2
12345678901234567890123456789012123456789012345678901
improve your vocabulary, and it will have benefits beyond the GRE CAT 2
1 Verbal. You might be able to use some of these tough words on the 2
2345678901234567890123456789012123456789012345678901
12345678901234567890123456789012123456789012345678901 2
12345678901234567890123456789012123456789012345678901 2
12345678901234567890123456789012123456789012345678901
Analytical essay, and in general you’ll just have more words at your 2
12345678901234567890123456789012123456789012345678901 2
12345678901234567890123456789012123456789012345678901
disposal on a day-by-day basis. You will find that a large vocabulary can 2
12345678901234567890123456789012123456789012345678901 2
12345678901234567890123456789012123456789012345678901
be very helpful in social, educational, and business settings. 2
12345678901234567890123456789012123456789012345678901 2
12345678901234567890123456789012123456789012345678901 2
12345678901234567890123456789012123456789012345678901 2
2345678901234567890123456789012123456789012345678901
3. Do Some Test-Specific Vocabulary Reading. If context learning is the
1 best way to pump up your GRE vocabulary, the second-best way is to buy 2
12345678901234567890123456789012123456789012345678901 2
12345678901234567890123456789012123456789012345678901 2
12345678901234567890123456789012123456789012345678901
a “Boost Your GRE Vocabulary” book. There are also a slew of Web sites 2
2345678901234567890123456789012123456789012345678901
12345678901234567890123456789012123456789012345678901 2
12345678901234567890123456789012123456789012345678901
covering GRE vocabulary. 2
12345678901234567890123456789012123456789012345678901 2
Regardless of what source you use, a GRE-specific vocabulary list can help 2
12345678901234567890123456789012123456789012345678901
12345678901234567890123456789012123456789012345678901 2
focus your studies to cover words that have a good likelihood of appearing 2
12345678901234567890123456789012123456789012345678901
12345678901234567890123456789012123456789012345678901
on the GRE. However, this advantage is offset by two opposing factors. 2
2
12345678901234567890123456789012123456789012345678901
12345678901234567890123456789012123456789012345678901 2
1 The first factor is that learning words from a book does not guarantee that 2
2345678901234567890123456789012123456789012345678901
12345678901234567890123456789012123456789012345678901
you’ll remember the word in the future. Typically, the high density of new 2
2
12345678901234567890123456789012123456789012345678901
1 words you encounter and the artificial method in which you experience 2
12345678901234567890123456789012123456789012345678901 2
12345678901234567890123456789012123456789012345678901
them translates to a lower retention level. Instead of anchoring the word in 2
12345678901234567890123456789012123456789012345678901 2
12345678901234567890123456789012123456789012345678901
your mind, you might see it on the test and say, “Oh yeah, I remember that 2
12345678901234567890123456789012123456789012345678901 2
12345678901234567890123456789012123456789012345678901 2
12345678901234567890123456789012123456789012345678901
was one of the words I studied.” This recollection might lead you to recall 2
2345678901234567890123456789012123456789012345678901
the word’s definition, but often all you have is a vague idea of what the 2
12345678901234567890123456789012123456789012345678901
12345678901234567890123456789012123456789012345678901 2
1 word means. Still, this is better than nothing. 2
12345678901234567890123456789012123456789012345678901 2
12345678901234567890123456789012123456789012345678901 2
12345678901234567890123456789012123456789012345678901
The second problem is that even if you do memorize 200 GRE-specific 2
12345678901234567890123456789012123456789012345678901 2
12345678901234567890123456789012123456789012345678901
words, the chances you will encounter a bunch of them on the actual test is 2
12345678901234567890123456789012123456789012345678901 2
12345678901234567890123456789012123456789012345678901
still rather small. Picture the Gulf of Mexico in your mind. Even if you 2
12345678901234567890123456789012123456789012345678901 2
2345678901234567890123456789012123456789012345678901
know every fish off the coast of Florida, does this mean you know every 2
12345678901234567890123456789012123456789012345678901
12345678901234567890123456789012123456789012345678901 2
fish in the entire gulf? The answer is no, because the volume of water in the 2
12345678901234567890123456789012123456789012345678901
1 gulf is simply too big. The same holds true for the amount of vocabulary 2
12345678901234567890123456789012123456789012345678901 2
12345678901234567890123456789012123456789012345678901 2
12345678901234567890123456789012123456789012345678901
that might appear on the GRE. The number of possible words is too large 2
12345678901234567890123456789012123456789012345678901 2
12345678901234567890123456789012123456789012345678901
for any single book, or volume of books, to contain. 2
12345678901234567890123456789012123456789012345678901 2
2345678901234567890123456789012123456789012345678901
1 This doesn’t mean you must gnash your teeth and scream to the stars 2
12345678901234567890123456789012123456789012345678901 2
12345678901234567890123456789012123456789012345678901 2
12345678901234567890123456789012123456789012345678901
about the injustice of the GRE, although if that makes you feel better, go 2
1 right ahead. Instead, simply realize that you will not know the exact 2
2345678901234567890123456789012123456789012345678901
12345678901234567890123456789012123456789012345678901 2
12345678901234567890123456789012123456789012345678901 2 2
138 12345678901234567890123456789012123456789012345678901
123456789012345678901234567890121234567890123456789012

www.petersons.com
Chapter 5: Verbal Review

123456789012345678901234567890121234567890123456789012
12345678901234567890123456789012123456789012345678901 2
12345678901234567890123456789012123456789012345678901 2
12345678901234567890123456789012123456789012345678901
definition of every word you encounter. That’s fine. Everybody else taking 2
12345678901234567890123456789012123456789012345678901 2
12345678901234567890123456789012123456789012345678901
the GRE is in the same boat as you, to one extent or another. Studying 2
12345678901234567890123456789012123456789012345678901 2
12345678901234567890123456789012123456789012345678901
from a GRE-specific list might place 1–3 problems right in your lap. Since 2
1 the test is only 30 questions long, every question helps. If some recently 2
2345678901234567890123456789012123456789012345678901
12345678901234567890123456789012123456789012345678901 2
12345678901234567890123456789012123456789012345678901 2
12345678901234567890123456789012123456789012345678901
learned vocabulary helps you on the first 5 questions, then it will have 2
12345678901234567890123456789012123456789012345678901
2345678901234567890123456789012123456789012345678901
2
2
1 been worth it.
12345678901234567890123456789012123456789012345678901 2
12345678901234567890123456789012123456789012345678901 2
12345678901234567890123456789012123456789012345678901
The rest of the chapter will cover various question-specific techniques you 2
12345678901234567890123456789012123456789012345678901
2345678901234567890123456789012123456789012345678901
2
2
1 can employ. Vocabulary was covered first since some of these techniques
12345678901234567890123456789012123456789012345678901 2
12345678901234567890123456789012123456789012345678901
hinge upon whether or not you know the definition of the words used. 2
12345678901234567890123456789012123456789012345678901
Compiling new vocabulary words and learning questions strategies go 2
12345678901234567890123456789012123456789012345678901
2345678901234567890123456789012123456789012345678901
2
2
1 hand in hand, so let’s get started.
12345678901234567890123456789012123456789012345678901 2
12345678901234567890123456789012123456789012345678901 2
12345678901234567890123456789012123456789012345678901 2
12345678901234567890123456789012123456789012345678901 2
12345678901234567890123456789012123456789012345678901 22
2345678901234567890123456789012123456789012345678901
12345678901234567890123456789012123456789012345678901 2
Antonyms
12345678901234567890123456789012123456789012345678901 2
12345678901234567890123456789012123456789012345678901
In test-taking lingo, a GRE antonym question consists of a “stem word”
12345678901234567890123456789012123456789012345678901 2
12345678901234567890123456789012123456789012345678901 2
1 followed by 5 answer choices. For example: 2
12345678901234567890123456789012123456789012345678901 2
12345678901234567890123456789012123456789012345678901 2
12345678901234567890123456789012123456789012345678901
CIRCUITOUS : 2
12345678901234567890123456789012123456789012345678901 22
2345678901234567890123456789012123456789012345678901
12345678901234567890123456789012123456789012345678901
12345678901234567890123456789012123456789012345678901
A. direct 2
12345678901234567890123456789012123456789012345678901 2
12345678901234567890123456789012123456789012345678901 22
B. electrical
12345678901234567890123456789012123456789012345678901
12345678901234567890123456789012123456789012345678901
C. wooden 2
12345678901234567890123456789012123456789012345678901
D. oblique 2
12345678901234567890123456789012123456789012345678901 22
1 E. long-winded
12345678901234567890123456789012123456789012345678901 2
12345678901234567890123456789012123456789012345678901 2
12345678901234567890123456789012123456789012345678901
The stem word is circuitous, and one of the five answer choices below this 2
12345678901234567890123456789012123456789012345678901 2
12345678901234567890123456789012123456789012345678901
stem word has a meaning that is “most nearly opposite” in meaning to 2
2345678901234567890123456789012123456789012345678901 2
12345678901234567890123456789012123456789012345678901
circuitous. Don’t look yet at the answer choices, though. The best way to 2
12345678901234567890123456789012123456789012345678901
1 approach every antonym problem is to focus on the stem word first and 2
12345678901234567890123456789012123456789012345678901 2
12345678901234567890123456789012123456789012345678901 2
12345678901234567890123456789012123456789012345678901 22
2345678901234567890123456789012123456789012345678901
answer this question as honestly as possible: “Do I know the meaning of
12345678901234567890123456789012123456789012345678901 2
1 this word?”
12345678901234567890123456789012123456789012345678901 2
12345678901234567890123456789012123456789012345678901 2
12345678901234567890123456789012123456789012345678901 2
12345678901234567890123456789012123456789012345678901
Although these questions are commonly referred to as antonyms, not 2
12345678901234567890123456789012123456789012345678901 2
12345678901234567890123456789012123456789012345678901
every stem word/correct answer is an antonym pair in the purest sense. 2
12345678901234567890123456789012123456789012345678901 2
Tip

12345678901234567890123456789012123456789012345678901 22
2345678901234567890123456789012123456789012345678901
This is why the phrase “most nearly opposite” (or something similar) will
12345678901234567890123456789012123456789012345678901 2
12345678901234567890123456789012123456789012345678901
be used to describe these questions. The phrase is vague enough to give
2
12345678901234567890123456789012123456789012345678901 2
1 the test-writers wiggle room if they want to use two words that aren’t
2345678901234567890123456789012123456789012345678901
12345678901234567890123456789012123456789012345678901 2
12345678901234567890123456789012123456789012345678901
exactly antonyms, but do have meanings that are fairly opposite. 2
12345678901234567890123456789012123456789012345678901 22
12345678901234567890123456789012123456789012345678901 2
1 This question triggers your approach to every antonym problem. Looking
12345678901234567890123456789012123456789012345678901 2
12345678901234567890123456789012123456789012345678901 2
12345678901234567890123456789012123456789012345678901
at the answer choices before answering it might influence your thinking, as 2
12345678901234567890123456789012123456789012345678901 2
12345678901234567890123456789012123456789012345678901
you make an incorrect connection with the stem word and an answer 2
1 choice that is incorrect. The reason for this common mistake has to do 2
2345678901234567890123456789012123456789012345678901
12345678901234567890123456789012123456789012345678901 2
12345678901234567890123456789012123456789012345678901 2
12345678901234567890123456789012123456789012345678901 2 139
123456789012345678901234567890121234567890123456789012
Part III: Section Review

123456789012345678901234567890121234567890123456789012
12345678901234567890123456789012123456789012345678901 2
12345678901234567890123456789012123456789012345678901 2
12345678901234567890123456789012123456789012345678901
with the way wrong answer choices are created. In general, wrong answer 2
12345678901234567890123456789012123456789012345678901 2
12345678901234567890123456789012123456789012345678901
choices often take the form of words that can commonly be associated 2
12345678901234567890123456789012123456789012345678901 2
12345678901234567890123456789012123456789012345678901
with the stem word. Call this the “Salt-and-Pepper Trap.” The stem word 2
1 is salt, and directly below it is the answer choice pepper. Your mind 2
2345678901234567890123456789012123456789012345678901
12345678901234567890123456789012123456789012345678901 2
12345678901234567890123456789012123456789012345678901 2
12345678901234567890123456789012123456789012345678901
commonly associates these two items together, in kind of an opposite way. 2
12345678901234567890123456789012123456789012345678901
2345678901234567890123456789012123456789012345678901
2
2
1 So like the old tongue-twister you pick pepper, even though the two words
12345678901234567890123456789012123456789012345678901 2
12345678901234567890123456789012123456789012345678901
are not antonyms. They aren’t really related in any way except for the fact 2
12345678901234567890123456789012123456789012345678901 2
12345678901234567890123456789012123456789012345678901
that they are both common spices. Focusing
2345678901234567890123456789012123456789012345678901
on the stem word definition 2
2
1 first prevents you from falling into the Salt-and-Pepper Trap. Achoo!
12345678901234567890123456789012123456789012345678901 2
12345678901234567890123456789012123456789012345678901 2
12345678901234567890123456789012123456789012345678901
Answering the question, “Do I know the meaning of the stem word?” 2
12345678901234567890123456789012123456789012345678901 2
1 produces three outcomes, “Yes,” “No,” and “Maybe.” Each of these 2
2345678901234567890123456789012123456789012345678901
12345678901234567890123456789012123456789012345678901 2
12345678901234567890123456789012123456789012345678901
answers leads to a different technique. 2
12345678901234567890123456789012123456789012345678901 2
12345678901234567890123456789012123456789012345678901 2
12345678901234567890123456789012123456789012345678901 2
2345678901234567890123456789012123456789012345678901
12345678901234567890123456789012123456789012345678901 2
Outcome #1: Yes, I Know the Meaning of the Word 2
12345678901234567890123456789012123456789012345678901
12345678901234567890123456789012123456789012345678901 2
12345678901234567890123456789012123456789012345678901 2
12345678901234567890123456789012123456789012345678901 2
As you might expect, this is the answer you want the most. If you do know
1 the meaning of circuitous, define it mentally in a sentence like, “Circuitous 2
12345678901234567890123456789012123456789012345678901 2
12345678901234567890123456789012123456789012345678901 2
12345678901234567890123456789012123456789012345678901
means a long, round-about course or path.” Now, anticipate what the 2
2345678901234567890123456789012123456789012345678901
antonym of circuitous will be. By anticipating the antonym, you approach 2
12345678901234567890123456789012123456789012345678901
12345678901234567890123456789012123456789012345678901 2
12345678901234567890123456789012123456789012345678901 2
12345678901234567890123456789012123456789012345678901 2
the answer choices with a focus that will help you separate the wrong
answers from the better choices. People who don’t approach the answer 2
12345678901234567890123456789012123456789012345678901
12345678901234567890123456789012123456789012345678901 2
12345678901234567890123456789012123456789012345678901 2
12345678901234567890123456789012123456789012345678901 2
choices with such a clear focus often “shop” the answer choices, looking at
12345678901234567890123456789012123456789012345678901 2
1 each one and trying them on to see if they fit as the stem word’s antonyms. 2
2345678901234567890123456789012123456789012345678901
Since these wrong answers are designed to be tempting, this approach can 2
12345678901234567890123456789012123456789012345678901
12345678901234567890123456789012123456789012345678901 2
1 easily lead to a poor choice. 2
12345678901234567890123456789012123456789012345678901 2
12345678901234567890123456789012123456789012345678901 2
12345678901234567890123456789012123456789012345678901
By anticipating the antonym, you give yourself a clear idea of what you are 2
12345678901234567890123456789012123456789012345678901 2
12345678901234567890123456789012123456789012345678901
looking for. You don’t have to spend minutes crafting a perfect response, 2
12345678901234567890123456789012123456789012345678901 2
12345678901234567890123456789012123456789012345678901
just compose a mental sentence along the lines of, “the antonym of 2
2345678901234567890123456789012123456789012345678901
12345678901234567890123456789012123456789012345678901 2
12345678901234567890123456789012123456789012345678901
circuitous will be a word like straight or blunt.” 2
1 2
12345678901234567890123456789012123456789012345678901 2
12345678901234567890123456789012123456789012345678901 2
A. direct
2345678901234567890123456789012123456789012345678901
12345678901234567890123456789012123456789012345678901 2
12345678901234567890123456789012123456789012345678901
B. electrical 2
12345678901234567890123456789012123456789012345678901 2
12345678901234567890123456789012123456789012345678901 2
C. wooden
2
1
12345678901234567890123456789012123456789012345678901
D. oblique 2
12345678901234567890123456789012123456789012345678901 2
12345678901234567890123456789012123456789012345678901
E. long-winded 2
12345678901234567890123456789012123456789012345678901 2
12345678901234567890123456789012123456789012345678901
Using this mental sentence as a guide, choice (A) stands out as the correct 2
12345678901234567890123456789012123456789012345678901 2
2345678901234567890123456789012123456789012345678901
answer. You might not know exactly what oblique means, so you don’t 2
12345678901234567890123456789012123456789012345678901
12345678901234567890123456789012123456789012345678901 2
have to cross out that answer if you don’t want to. All other choices are 2
12345678901234567890123456789012123456789012345678901
1 clearly wrong, although if you went shopping in the answer choices, you 2
12345678901234567890123456789012123456789012345678901 2
12345678901234567890123456789012123456789012345678901 2
12345678901234567890123456789012123456789012345678901
might have liked electrical since the word circuitous has the word circuit 2
12345678901234567890123456789012123456789012345678901 2
12345678901234567890123456789012123456789012345678901
in it.
2345678901234567890123456789012123456789012345678901
2
2
1
12345678901234567890123456789012123456789012345678901 2
12345678901234567890123456789012123456789012345678901 2 2
140 12345678901234567890123456789012123456789012345678901
123456789012345678901234567890121234567890123456789012

www.petersons.com
Chapter 5: Verbal Review

123456789012345678901234567890121234567890123456789012
12345678901234567890123456789012123456789012345678901 2
12345678901234567890123456789012123456789012345678901 2
12345678901234567890123456789012123456789012345678901
If you know the meaning of the stem word, chances are good you would 2
12345678901234567890123456789012123456789012345678901 2
12345678901234567890123456789012123456789012345678901
get the answer correct even without anticipating the correct choice. Even 2
12345678901234567890123456789012123456789012345678901 2
12345678901234567890123456789012123456789012345678901
so, anticipating the correct answer virtually eliminates the chance of a 2
1 wrong answer. It’s a sound test-taking strategy, and one you should always 2
2345678901234567890123456789012123456789012345678901
12345678901234567890123456789012123456789012345678901 2
12345678901234567890123456789012123456789012345678901 2
12345678901234567890123456789012123456789012345678901
employ. 2
12345678901234567890123456789012123456789012345678901
2345678901234567890123456789012123456789012345678901
2
2
1
12345678901234567890123456789012123456789012345678901 2
12345678901234567890123456789012123456789012345678901
Outcome #2: I Sort of Know the Meaning of That 2
12345678901234567890123456789012123456789012345678901 2
12345678901234567890123456789012123456789012345678901
2345678901234567890123456789012123456789012345678901
2
2
1Word . . . Maybe
12345678901234567890123456789012123456789012345678901 2
12345678901234567890123456789012123456789012345678901
There are many words that fall into this category. They are like the 2
12345678901234567890123456789012123456789012345678901 2
12345678901234567890123456789012123456789012345678901
neighbors you occasionally see down the street or the people who work at 2
1 your local grocery store. You recognize them when you see them, but if 2
2345678901234567890123456789012123456789012345678901
12345678901234567890123456789012123456789012345678901 2
12345678901234567890123456789012123456789012345678901 2
12345678901234567890123456789012123456789012345678901
pressed, you have to admit that you don’t know that much about them. 2
12345678901234567890123456789012123456789012345678901 2
2345678901234567890123456789012123456789012345678901
People encounter these “passing acquaintance” words every day. If you are 2
12345678901234567890123456789012123456789012345678901
12345678901234567890123456789012123456789012345678901 2
following the suggestions listed earlier in this chapter, you are writing 2
12345678901234567890123456789012123456789012345678901
12345678901234567890123456789012123456789012345678901 22
12345678901234567890123456789012123456789012345678901
down these words and then looking up their definitions later. This allows
12345678901234567890123456789012123456789012345678901 2
1 you to change your answer from “Maybe I know this word” to “Yes, I 2
12345678901234567890123456789012123456789012345678901 2
12345678901234567890123456789012123456789012345678901
know this word,” something that greatly helps your GRE Verbal chances. 2
12345678901234567890123456789012123456789012345678901 2
12345678901234567890123456789012123456789012345678901
Still, there will always be some words that you recognize but can never 2
12345678901234567890123456789012123456789012345678901 2
12345678901234567890123456789012123456789012345678901
quite define. 2
12345678901234567890123456789012123456789012345678901 2
12345678901234567890123456789012123456789012345678901 2
2345678901234567890123456789012123456789012345678901
12345678901234567890123456789012123456789012345678901 2
12345678901234567890123456789012123456789012345678901
BRAVADO : 2
12345678901234567890123456789012123456789012345678901 22
12345678901234567890123456789012123456789012345678901
A. callowness
2
1
2345678901234567890123456789012123456789012345678901
12345678901234567890123456789012123456789012345678901
B. greed 2
12345678901234567890123456789012123456789012345678901 2
1 C. swagger 2
12345678901234567890123456789012123456789012345678901 2
12345678901234567890123456789012123456789012345678901
D. timorousness 2
12345678901234567890123456789012123456789012345678901 2
12345678901234567890123456789012123456789012345678901
E. hapless 2
12345678901234567890123456789012123456789012345678901 2
12345678901234567890123456789012123456789012345678901
Assume that you don’t quite know the definition of the stem word 2
12345678901234567890123456789012123456789012345678901 2
2345678901234567890123456789012123456789012345678901
bravado. However, you have seen the word in some novels, and you think 2
12345678901234567890123456789012123456789012345678901
12345678901234567890123456789012123456789012345678901 2
1 that brav- in bravado is the same as the brav- in brave or bravery. This 2
12345678901234567890123456789012123456789012345678901 2
12345678901234567890123456789012123456789012345678901
would mean that bravado means something similar to brave or courageous 2
12345678901234567890123456789012123456789012345678901 2
12345678901234567890123456789012123456789012345678901 2
12345678901234567890123456789012123456789012345678901
. . . if the brav- is the same for both words. 2
12345678901234567890123456789012123456789012345678901 2
12345678901234567890123456789012123456789012345678901 2
12345678901234567890123456789012123456789012345678901 22
2345678901234567890123456789012123456789012345678901
The above train of thought is commendable, and it shows someone
12345678901234567890123456789012123456789012345678901
working to decipher the meaning of a word with an educated guess. This is
2
12345678901234567890123456789012123456789012345678901
a good strategy. To determine whether or not the guess is correct, you need 2
12345678901234567890123456789012123456789012345678901
1 2
12345678901234567890123456789012123456789012345678901
to develop your awareness of how the computer adaptive GRE works. For 2
12345678901234567890123456789012123456789012345678901 2
12345678901234567890123456789012123456789012345678901
example, if bravado is the first question in the GRE Verbal section, which 2
12345678901234567890123456789012123456789012345678901 2
12345678901234567890123456789012123456789012345678901
would imply that it is a question of medium difficulty. This question 2
2345678901234567890123456789012123456789012345678901
12345678901234567890123456789012123456789012345678901 2
1 won’t be simple, but a little work—like figuring out the definition by 2
12345678901234567890123456789012123456789012345678901 2
12345678901234567890123456789012123456789012345678901
comparing the root brav—should place you on the right course. In this 2
12345678901234567890123456789012123456789012345678901 2
12345678901234567890123456789012123456789012345678901
scenario, bravado probably does share some similarity with the more 2
12345678901234567890123456789012123456789012345678901 2
12345678901234567890123456789012123456789012345678901
common word brave. 2
12345678901234567890123456789012123456789012345678901 2 141
123456789012345678901234567890121234567890123456789012
Part III: Section Review

123456789012345678901234567890121234567890123456789012
12345678901234567890123456789012123456789012345678901 2
12345678901234567890123456789012123456789012345678901 2
12345678901234567890123456789012123456789012345678901
However, if bravado is the fifth question, and you’re very confident you’ve 2
12345678901234567890123456789012123456789012345678901 2
12345678901234567890123456789012123456789012345678901
answered the previous four questions correctly, then the scenario is 2
12345678901234567890123456789012123456789012345678901 2
12345678901234567890123456789012123456789012345678901
different. You should realize that your correct answers have caused the test 2
1 to adapt and start feeding you harder problems, so the root brav- in 2
2345678901234567890123456789012123456789012345678901
12345678901234567890123456789012123456789012345678901 2
12345678901234567890123456789012123456789012345678901 2
12345678901234567890123456789012123456789012345678901
bravado is a decoy put there to catch unwary students. There’s no way a 2
12345678901234567890123456789012123456789012345678901
2345678901234567890123456789012123456789012345678901
2
2
1 hard GRE Verbal question would be so easy to decipher.
12345678901234567890123456789012123456789012345678901 2
12345678901234567890123456789012123456789012345678901 2
12345678901234567890123456789012123456789012345678901
If the previous paragraph makes sense to you, then you are well on your 2
12345678901234567890123456789012123456789012345678901
2345678901234567890123456789012123456789012345678901
2
2
1 way to becoming a great test-taker.
12345678901234567890123456789012123456789012345678901 2
12345678901234567890123456789012123456789012345678901
For this example, suppose bravado is the first question you encounter on 2
12345678901234567890123456789012123456789012345678901 2
12345678901234567890123456789012123456789012345678901
the GRE Verbal, which means the brav- root theory is valid. Since you sort 2
2345678901234567890123456789012123456789012345678901 2
1
12345678901234567890123456789012123456789012345678901
of know the definition, you can sort of anticipate an answer. The opposite 2
12345678901234567890123456789012123456789012345678901 2
12345678901234567890123456789012123456789012345678901
of bravery would be cowardice or something similar, so the opposite of 2
12345678901234567890123456789012123456789012345678901 2
12345678901234567890123456789012123456789012345678901
bravado should be something like cowardice as well. Glancing at the 2
12345678901234567890123456789012123456789012345678901 2
12345678901234567890123456789012123456789012345678901
answer choices, you have: 2
12345678901234567890123456789012123456789012345678901 2
12345678901234567890123456789012123456789012345678901 2
12345678901234567890123456789012123456789012345678901 2
2345678901234567890123456789012123456789012345678901
A. callowness
2
1
12345678901234567890123456789012123456789012345678901
B. greed 2
12345678901234567890123456789012123456789012345678901 2
12345678901234567890123456789012123456789012345678901
C. swagger 2
2345678901234567890123456789012123456789012345678901
12345678901234567890123456789012123456789012345678901 2
12345678901234567890123456789012123456789012345678901
D. timorousness 2
12345678901234567890123456789012123456789012345678901 2
12345678901234567890123456789012123456789012345678901 2
E. hapless
12345678901234567890123456789012123456789012345678901 2
12345678901234567890123456789012123456789012345678901 2
Some people like to think in terms of positive/negative words. Painting 2
12345678901234567890123456789012123456789012345678901
12345678901234567890123456789012123456789012345678901 2
12345678901234567890123456789012123456789012345678901 2
1 with broad strokes, positive words mean things like larger/happier/better/ 2
2345678901234567890123456789012123456789012345678901
good, while negative words mean the opposite. Bravado has the root 2
12345678901234567890123456789012123456789012345678901
12345678901234567890123456789012123456789012345678901 2
1 brav- in it, and bravery would be considered positive. You can could then 2
12345678901234567890123456789012123456789012345678901 2
12345678901234567890123456789012123456789012345678901
go into the answer choices and look for negative words. This technique 2
Note
12345678901234567890123456789012123456789012345678901 2
12345678901234567890123456789012123456789012345678901
does not always work, though, as many answer choices are negative 2
12345678901234567890123456789012123456789012345678901 2
12345678901234567890123456789012123456789012345678901
words but with different meanings. Nevertheless, thinking in terms of 2
12345678901234567890123456789012123456789012345678901 2
2345678901234567890123456789012123456789012345678901
positive/negative can help for some words in which the only thing you can 2
12345678901234567890123456789012123456789012345678901
12345678901234567890123456789012123456789012345678901 2
1 recall about them is whether or not they mean something good or bad. 2
12345678901234567890123456789012123456789012345678901 2
12345678901234567890123456789012123456789012345678901 2
12345678901234567890123456789012123456789012345678901 2
12345678901234567890123456789012123456789012345678901
These choices illustrate the influence that vocabulary has on the GRE 2
12345678901234567890123456789012123456789012345678901 2
12345678901234567890123456789012123456789012345678901
Verbal, since some of the answer choice words are as hard as or harder 2
12345678901234567890123456789012123456789012345678901 2
12345678901234567890123456789012123456789012345678901
than the stem word itself. Still, choices (B), (C), and (E) have nothing to do 2
12345678901234567890123456789012123456789012345678901 2
12345678901234567890123456789012123456789012345678901
with cowardice, so you can cross them out. This leaves you with a 50/50 2
12345678901234567890123456789012123456789012345678901 2
12345678901234567890123456789012123456789012345678901
chance, not bad considering you aren’t certain of the stem word definition. 2
2345678901234567890123456789012123456789012345678901
12345678901234567890123456789012123456789012345678901 2
12345678901234567890123456789012123456789012345678901
Therefore, the correct answer is (D). 2
12345678901234567890123456789012123456789012345678901 2
12345678901234567890123456789012123456789012345678901 2 2
1
12345678901234567890123456789012123456789012345678901 2
12345678901234567890123456789012123456789012345678901 2
12345678901234567890123456789012123456789012345678901 2
12345678901234567890123456789012123456789012345678901 2
12345678901234567890123456789012123456789012345678901 2
12345678901234567890123456789012123456789012345678901 2
12345678901234567890123456789012123456789012345678901 2
12345678901234567890123456789012123456789012345678901 2 2
142 12345678901234567890123456789012123456789012345678901
123456789012345678901234567890121234567890123456789012

www.petersons.com
Chapter 5: Verbal Review

123456789012345678901234567890121234567890123456789012
12345678901234567890123456789012123456789012345678901 2
12345678901234567890123456789012123456789012345678901 2
12345678901234567890123456789012123456789012345678901
Outcome #3: No, I Have No Idea What That 2
12345678901234567890123456789012123456789012345678901 2
12345678901234567890123456789012123456789012345678901
Word Means 2
12345678901234567890123456789012123456789012345678901 2
12345678901234567890123456789012123456789012345678901
For most people, the question is not if you will encounter a word you don’t 2
12345678901234567890123456789012123456789012345678901 2
12345678901234567890123456789012123456789012345678901 2
12345678901234567890123456789012123456789012345678901
know, but when you will run across that word. Don’t beat yourself up 2
12345678901234567890123456789012123456789012345678901 2
12345678901234567890123456789012123456789012345678901
about this, as the words used on the GRE
2345678901234567890123456789012123456789012345678901
Verbal section reflect a pretty 2
2
1 advanced vocabulary pool. For example:
12345678901234567890123456789012123456789012345678901 2
12345678901234567890123456789012123456789012345678901 2
12345678901234567890123456789012123456789012345678901 2
12345678901234567890123456789012123456789012345678901
WINSOME :
2345678901234567890123456789012123456789012345678901
2
2
1
12345678901234567890123456789012123456789012345678901 2
12345678901234567890123456789012123456789012345678901
A. unpleasant 2
12345678901234567890123456789012123456789012345678901 2
12345678901234567890123456789012123456789012345678901
B. defeated
2345678901234567890123456789012123456789012345678901
2
2
1
12345678901234567890123456789012123456789012345678901
C. garrulous 2
12345678901234567890123456789012123456789012345678901 2
12345678901234567890123456789012123456789012345678901
D. whiny 2
12345678901234567890123456789012123456789012345678901 2
12345678901234567890123456789012123456789012345678901 22
2345678901234567890123456789012123456789012345678901
E. spurious
12345678901234567890123456789012123456789012345678901 2
12345678901234567890123456789012123456789012345678901
If you don’t know what the word winsome means, and if your mind can
12345678901234567890123456789012123456789012345678901 2
not even summon up a passing memory of seeing the word in print, then 2
12345678901234567890123456789012123456789012345678901
12345678901234567890123456789012123456789012345678901 22
1 there’s no way to anticipate the answer. Instead, you need to attack the
12345678901234567890123456789012123456789012345678901 2
12345678901234567890123456789012123456789012345678901
answer choices, scrutinize them and try to eliminate unlikely answers, and 2
12345678901234567890123456789012123456789012345678901 2
12345678901234567890123456789012123456789012345678901
then take a guess. 2
12345678901234567890123456789012123456789012345678901 2
12345678901234567890123456789012123456789012345678901 2
12345678901234567890123456789012123456789012345678901
There are two primary ways to attack answer choices. The first one has 2
12345678901234567890123456789012123456789012345678901 2
12345678901234567890123456789012123456789012345678901 22
2345678901234567890123456789012123456789012345678901
been touched on in the previous section, where you took your knowledge
12345678901234567890123456789012123456789012345678901
of the adaptive test format and used it to your advantage. Suppose the 2
12345678901234567890123456789012123456789012345678901
12345678901234567890123456789012123456789012345678901 2
1 word winsome appears after you have answered several questions 2
2345678901234567890123456789012123456789012345678901
correctly. Personally, you think it’s a tough word (which is true), and that 2
12345678901234567890123456789012123456789012345678901
12345678901234567890123456789012123456789012345678901 2
1 you are in a hard portion of the GRE Verbal (a good assumption if you’ve 2
12345678901234567890123456789012123456789012345678901 2
12345678901234567890123456789012123456789012345678901
been answering questions correctly). This means there’s no easy shortcut 2
12345678901234567890123456789012123456789012345678901 2
12345678901234567890123456789012123456789012345678901
on this problem. In fact, anything that looks like a shortcut is probably too 2
12345678901234567890123456789012123456789012345678901 2
12345678901234567890123456789012123456789012345678901
good to be true, and can be eliminated. 2
12345678901234567890123456789012123456789012345678901 2
2345678901234567890123456789012123456789012345678901
12345678901234567890123456789012123456789012345678901
The root win- appears in the word winsome, which might lead some to 2
2
12345678901234567890123456789012123456789012345678901
1 think the word has something to do with victory or winning. Choice (B) 2
12345678901234567890123456789012123456789012345678901 2
12345678901234567890123456789012123456789012345678901 2
12345678901234567890123456789012123456789012345678901
then practically screams, “I must be the answer!” Knowing where you are 2
12345678901234567890123456789012123456789012345678901 2
12345678901234567890123456789012123456789012345678901
on the test, and how tough a word winsome is, you can therefore eliminate 2
12345678901234567890123456789012123456789012345678901
choice (B). You know it’s there to catch unwary test-takers looking for an 2
12345678901234567890123456789012123456789012345678901 2
2345678901234567890123456789012123456789012345678901
12345678901234567890123456789012123456789012345678901 2
12345678901234567890123456789012123456789012345678901
easy out when there isn’t one. 2
12345678901234567890123456789012123456789012345678901 22
12345678901234567890123456789012123456789012345678901
1 One answer choice is out, but four remain. The second elimination 2
12345678901234567890123456789012123456789012345678901
technique can now be employed. Call it the “Sandwich Technique.” Try to 2
12345678901234567890123456789012123456789012345678901 2
12345678901234567890123456789012123456789012345678901 2
12345678901234567890123456789012123456789012345678901
answer the question, “What is the opposite of sandwich?” Is it a pizza? 2
12345678901234567890123456789012123456789012345678901
Soup? A salad? The point is there is no opposite word for sandwich. Many 2
12345678901234567890123456789012123456789012345678901 22
2345678901234567890123456789012123456789012345678901
1 words have no opposite, or you have to get very creative to come up with
12345678901234567890123456789012123456789012345678901 2
12345678901234567890123456789012123456789012345678901
an opposite. Words without true opposites are very poor candidates to be 2
12345678901234567890123456789012123456789012345678901 2
12345678901234567890123456789012123456789012345678901
the correct answer on an Antonym problem, so you can cross them out on 2
12345678901234567890123456789012123456789012345678901 2
12345678901234567890123456789012123456789012345678901
hard problems. 2
12345678901234567890123456789012123456789012345678901 2 143
123456789012345678901234567890121234567890123456789012
Part III: Section Review

123456789012345678901234567890121234567890123456789012
12345678901234567890123456789012123456789012345678901 2
12345678901234567890123456789012123456789012345678901 2
12345678901234567890123456789012123456789012345678901
Looking over the four remaining choices, choice (A) has a very clear 2
12345678901234567890123456789012123456789012345678901 2
12345678901234567890123456789012123456789012345678901
opposite. You might not know what garrulous or spurious means, so you 2
12345678901234567890123456789012123456789012345678901 2
12345678901234567890123456789012123456789012345678901
can’t eliminate them if you don’t know what they mean. It turns out both 2
1 have valid opposites. Look at (D), though, whiny. What is the opposite of 2
2345678901234567890123456789012123456789012345678901
12345678901234567890123456789012123456789012345678901 2
12345678901234567890123456789012123456789012345678901 2
12345678901234567890123456789012123456789012345678901
whiny? Unwhiny? Polite? Not annoying? You can make a case that there is 2
12345678901234567890123456789012123456789012345678901
2345678901234567890123456789012123456789012345678901
2
2
1 a word that means the opposite of whiny . . . but you have to make a case
12345678901234567890123456789012123456789012345678901 2
12345678901234567890123456789012123456789012345678901
in order to do so. This makes it unlikely to be the right answer, so you can 2
12345678901234567890123456789012123456789012345678901 2
12345678901234567890123456789012123456789012345678901
eliminate choice (D).
2345678901234567890123456789012123456789012345678901
2
2
1
12345678901234567890123456789012123456789012345678901 2
12345678901234567890123456789012123456789012345678901
You are left with three choices. Take a guess, and keep in mind that 2
12345678901234567890123456789012123456789012345678901
one-in-three odds are not too bad when you are dealing with a completely 2
12345678901234567890123456789012123456789012345678901 2
1 unfamiliar word. The correct answer choice happens to be the one that 2
2345678901234567890123456789012123456789012345678901
12345678901234567890123456789012123456789012345678901 2
12345678901234567890123456789012123456789012345678901
could easily be an antonym. A winsome person is charming and pleasant, 2
12345678901234567890123456789012123456789012345678901 2
12345678901234567890123456789012123456789012345678901
the opposite of unpleasant, choice (A). 2
12345678901234567890123456789012123456789012345678901 2
2345678901234567890123456789012123456789012345678901
12345678901234567890123456789012123456789012345678901
There are several methods for dealing with antonym problems, but 2
2
12345678901234567890123456789012123456789012345678901
12345678901234567890123456789012123456789012345678901
vocabulary plays a large part in the overall strategy. The same can also be 2
2
12345678901234567890123456789012123456789012345678901
12345678901234567890123456789012123456789012345678901 2
1 said of another vocabulary-centric question type, the analogy questions. 2
12345678901234567890123456789012123456789012345678901 2
12345678901234567890123456789012123456789012345678901 2
12345678901234567890123456789012123456789012345678901 2
12345678901234567890123456789012123456789012345678901 2
2345678901234567890123456789012123456789012345678901
1
ANALOGY is to GRE as FEATHER is to OSTRICH 22
12345678901234567890123456789012123456789012345678901
12345678901234567890123456789012123456789012345678901
Analogies are similar in form to antonyms, and they also have a heavy 2
12345678901234567890123456789012123456789012345678901 2
2345678901234567890123456789012123456789012345678901
vocabulary component. However, where there is only one stem word for 2
12345678901234567890123456789012123456789012345678901
12345678901234567890123456789012123456789012345678901 2
12345678901234567890123456789012123456789012345678901 2
12345678901234567890123456789012123456789012345678901 2
an antonym, an analogy question starts off with two stem words that bear
1 some type of relationship to one another. As you will see, understanding 2
12345678901234567890123456789012123456789012345678901 2
12345678901234567890123456789012123456789012345678901 2
12345678901234567890123456789012123456789012345678901 2
the relationship between the two stem words is the critical skill to master
12345678901234567890123456789012123456789012345678901 2
12345678901234567890123456789012123456789012345678901
when handling analogy problems. 2
12345678901234567890123456789012123456789012345678901 2
2345678901234567890123456789012123456789012345678901
12345678901234567890123456789012123456789012345678901 2
1 FEATHER : OSTRICH :: 2
12345678901234567890123456789012123456789012345678901 2
12345678901234567890123456789012123456789012345678901 2
12345678901234567890123456789012123456789012345678901 2
2345678901234567890123456789012123456789012345678901
A. beak : raven
12345678901234567890123456789012123456789012345678901 2
1 B. scale : lizard 2
12345678901234567890123456789012123456789012345678901 2
12345678901234567890123456789012123456789012345678901 2
C. foot : chicken
2345678901234567890123456789012123456789012345678901
12345678901234567890123456789012123456789012345678901 2
12345678901234567890123456789012123456789012345678901
D. desert : cactus 2
12345678901234567890123456789012123456789012345678901 2
12345678901234567890123456789012123456789012345678901 2
E. tail : mammal
2
1
12345678901234567890123456789012123456789012345678901
There’s a colon between the two stem words, but there is also some kind of 2
12345678901234567890123456789012123456789012345678901 2
12345678901234567890123456789012123456789012345678901
link between the two words. One of the answer choices—the correct 2
12345678901234567890123456789012123456789012345678901 2
12345678901234567890123456789012123456789012345678901
one—will share this same link, or analogous relationship. Hence the term, 2
12345678901234567890123456789012123456789012345678901 2
2345678901234567890123456789012123456789012345678901
2
12345678901234567890123456789012123456789012345678901
analogies.
2
12345678901234567890123456789012123456789012345678901
12345678901234567890123456789012123456789012345678901 2 2
1
12345678901234567890123456789012123456789012345678901 2
12345678901234567890123456789012123456789012345678901 2
12345678901234567890123456789012123456789012345678901 2
12345678901234567890123456789012123456789012345678901 2
12345678901234567890123456789012123456789012345678901 2
12345678901234567890123456789012123456789012345678901 2
12345678901234567890123456789012123456789012345678901 2
12345678901234567890123456789012123456789012345678901 2 2
144 12345678901234567890123456789012123456789012345678901
123456789012345678901234567890121234567890123456789012

www.petersons.com
Chapter 5: Verbal Review

123456789012345678901234567890121234567890123456789012
12345678901234567890123456789012123456789012345678901 2
12345678901234567890123456789012123456789012345678901 2
12345678901234567890123456789012123456789012345678901
To get a clearer picture of what is needed on an analogy problem, it helps 2
12345678901234567890123456789012123456789012345678901 2
12345678901234567890123456789012123456789012345678901
to get rid of the colon and format the question in the following manner: 2
12345678901234567890123456789012123456789012345678901 2
12345678901234567890123456789012123456789012345678901 2
12345678901234567890123456789012123456789012345678901 2
12345678901234567890123456789012123456789012345678901
FEATHER _____________________________ OSTRICH :: 2
12345678901234567890123456789012123456789012345678901 2
12345678901234567890123456789012123456789012345678901 2
12345678901234567890123456789012123456789012345678901
A. beak ______________________________
2345678901234567890123456789012123456789012345678901
raven 2
2
1 B. scale ______________________________ lizard
12345678901234567890123456789012123456789012345678901 2
12345678901234567890123456789012123456789012345678901
C. foot _______________________________ chicken 2
12345678901234567890123456789012123456789012345678901 2
12345678901234567890123456789012123456789012345678901
D. desert ____________________________
2345678901234567890123456789012123456789012345678901 __ cactus 2
2
1
12345678901234567890123456789012123456789012345678901
E. tail ________________________________ mammal 2
12345678901234567890123456789012123456789012345678901 2
12345678901234567890123456789012123456789012345678901 2
12345678901234567890123456789012123456789012345678901
If the question were set up this way, it would be easy to see that what you 2
1 need to do is write some text that shows the link between the two stem 2
2345678901234567890123456789012123456789012345678901
12345678901234567890123456789012123456789012345678901 2
12345678901234567890123456789012123456789012345678901 2
12345678901234567890123456789012123456789012345678901
words. You can have something like 2
12345678901234567890123456789012123456789012345678901 2
2345678901234567890123456789012123456789012345678901
12345678901234567890123456789012123456789012345678901 2
12345678901234567890123456789012123456789012345678901
a FEATHER is part of an OSTRICH :: 2
12345678901234567890123456789012123456789012345678901 22
12345678901234567890123456789012123456789012345678901
Creating a link between the two stem words is the first step of every 2
12345678901234567890123456789012123456789012345678901
12345678901234567890123456789012123456789012345678901 2
1 analogy problem. If you can create a good link, then you can take that link 2
12345678901234567890123456789012123456789012345678901
down into the answer choices and see which answer choices share that 2
12345678901234567890123456789012123456789012345678901 2
12345678901234567890123456789012123456789012345678901
link. 2
12345678901234567890123456789012123456789012345678901 22
2345678901234567890123456789012123456789012345678901
12345678901234567890123456789012123456789012345678901 2
12345678901234567890123456789012123456789012345678901 2
12345678901234567890123456789012123456789012345678901
Granted, you can’t create a link between the stem words if you don’t
Tip

12345678901234567890123456789012123456789012345678901 2
12345678901234567890123456789012123456789012345678901
know what one or both of the words means. Strategies to deal with these 2
12345678901234567890123456789012123456789012345678901 2
12345678901234567890123456789012123456789012345678901
circumstances are covered later in the chapter. 2
12345678901234567890123456789012123456789012345678901 22
1
12345678901234567890123456789012123456789012345678901 2
12345678901234567890123456789012123456789012345678901
A. beak is part of a raven 2
12345678901234567890123456789012123456789012345678901 2
12345678901234567890123456789012123456789012345678901
B. scale is part of a lizard 2
12345678901234567890123456789012123456789012345678901 2
12345678901234567890123456789012123456789012345678901 22
2345678901234567890123456789012123456789012345678901
C. foot is part of a chicken
12345678901234567890123456789012123456789012345678901 2
1 D. desert is part of a cactus
12345678901234567890123456789012123456789012345678901 2
12345678901234567890123456789012123456789012345678901
E. tail is part of a mammal 2
12345678901234567890123456789012123456789012345678901 22
2345678901234567890123456789012123456789012345678901
12345678901234567890123456789012123456789012345678901
1 A desert is NOT part of a cactus, so choice (D) is not going to be the right 2
12345678901234567890123456789012123456789012345678901
answer on this problem since it does not contain the same link as the two 2
12345678901234567890123456789012123456789012345678901 2
2345678901234567890123456789012123456789012345678901
stem words (desert : cactus do share a link, it’s just not the right one for 2
12345678901234567890123456789012123456789012345678901
12345678901234567890123456789012123456789012345678901 2
12345678901234567890123456789012123456789012345678901 2
12345678901234567890123456789012123456789012345678901 22
this problem). Choices (A), (B), (C), and (E) all seem to share the same link.
1 This does not mean that all four choices are correct. Instead, we must
12345678901234567890123456789012123456789012345678901 2
12345678901234567890123456789012123456789012345678901 2
12345678901234567890123456789012123456789012345678901
refine our initial link and be a little more precise in describing how the 2
12345678901234567890123456789012123456789012345678901
words feather and ostrich are connected. A little bit of thinking here will 2
12345678901234567890123456789012123456789012345678901 2
2345678901234567890123456789012123456789012345678901
12345678901234567890123456789012123456789012345678901
make the problem easy to solve. 2
12345678901234567890123456789012123456789012345678901 2
12345678901234567890123456789012123456789012345678901
Some people might say that feathers are used by birds to fly. This is 2
2
12345678901234567890123456789012123456789012345678901
1 somewhat true, although it could be argued that the wings are what enable 2
12345678901234567890123456789012123456789012345678901 2
12345678901234567890123456789012123456789012345678901 2
12345678901234567890123456789012123456789012345678901
a bird to fly. In any case, the ostrich is a flightless bird, so creating a link 2
12345678901234567890123456789012123456789012345678901 2
12345678901234567890123456789012123456789012345678901
that states, “A feather is part of an ostrich that is used to fly” would be 2
2345678901234567890123456789012123456789012345678901 2
1 incorrect.
12345678901234567890123456789012123456789012345678901 2
12345678901234567890123456789012123456789012345678901 2
12345678901234567890123456789012123456789012345678901 2 145
123456789012345678901234567890121234567890123456789012
Part III: Section Review

123456789012345678901234567890121234567890123456789012
12345678901234567890123456789012123456789012345678901 2
12345678901234567890123456789012123456789012345678901 2
12345678901234567890123456789012123456789012345678901
What do the feathers do for an ostrich? Without over-thinking it, feathers 2
12345678901234567890123456789012123456789012345678901 2
12345678901234567890123456789012123456789012345678901
cover the outside of the bird and protect it in the same way that skin 2
12345678901234567890123456789012123456789012345678901 2
12345678901234567890123456789012123456789012345678901
protects us. You can then modify our initial linking sentence to read 2
2345678901234567890123456789012123456789012345678901 2
1 something like:
12345678901234567890123456789012123456789012345678901 2
12345678901234567890123456789012123456789012345678901 2
12345678901234567890123456789012123456789012345678901 2
12345678901234567890123456789012123456789012345678901
a FEATHER is part of the outer
2345678901234567890123456789012123456789012345678901
covering of an OSTRICH 2
2
1
12345678901234567890123456789012123456789012345678901
a FEATHER is the part of an OSTRICH found along the outside 2
12345678901234567890123456789012123456789012345678901 2
12345678901234567890123456789012123456789012345678901
of its body 2
12345678901234567890123456789012123456789012345678901
2345678901234567890123456789012123456789012345678901
2
1 a FEATHER is the part of an OSTRICH that covers it like skin 2
12345678901234567890123456789012123456789012345678901 2
12345678901234567890123456789012123456789012345678901 2
12345678901234567890123456789012123456789012345678901
All three of these sentences give a more precise definition of the connection 2
12345678901234567890123456789012123456789012345678901 2
1 between feather and ostrich. Use any of them above and see how they fit in 2
2345678901234567890123456789012123456789012345678901
12345678901234567890123456789012123456789012345678901 2
12345678901234567890123456789012123456789012345678901
with the remaining answer choices. 2
12345678901234567890123456789012123456789012345678901 2
12345678901234567890123456789012123456789012345678901 2
12345678901234567890123456789012123456789012345678901 2
2345678901234567890123456789012123456789012345678901
A. beak is part of the outer covering of a raven
12345678901234567890123456789012123456789012345678901 2
12345678901234567890123456789012123456789012345678901
B. scale is part of the outer covering of a lizard 2
12345678901234567890123456789012123456789012345678901 2
12345678901234567890123456789012123456789012345678901 2
C. foot is part of the outer covering of a chicken
12345678901234567890123456789012123456789012345678901 2
1 E. tail is part of the outer covering of a mammal 2
12345678901234567890123456789012123456789012345678901 2
12345678901234567890123456789012123456789012345678901
You could try to make the case that choices (A), (C), and (E) are still valid, 2
12345678901234567890123456789012123456789012345678901 2
12345678901234567890123456789012123456789012345678901
but it should be clear that choice (B) is the best answer for this analogy 2
12345678901234567890123456789012123456789012345678901 2
12345678901234567890123456789012123456789012345678901
problem. Scales cover the outside of lizards, and feathers cover the exterior 2
12345678901234567890123456789012123456789012345678901 2
12345678901234567890123456789012123456789012345678901
of ostriches. 2
12345678901234567890123456789012123456789012345678901 2
2345678901234567890123456789012123456789012345678901
If you know both the stem words for an analogy question, then you should 2
12345678901234567890123456789012123456789012345678901
12345678901234567890123456789012123456789012345678901 2
12345678901234567890123456789012123456789012345678901 2
1 focus on creating a link that describes the connection between the two 2
2345678901234567890123456789012123456789012345678901
words. Once this is done, you simply take that link and check it against all 2
12345678901234567890123456789012123456789012345678901
12345678901234567890123456789012123456789012345678901 2
1 the answer choices to find the one that shares the same relationship. That 2
12345678901234567890123456789012123456789012345678901 2
12345678901234567890123456789012123456789012345678901 2
12345678901234567890123456789012123456789012345678901
is your correct answer. 2
12345678901234567890123456789012123456789012345678901 2
12345678901234567890123456789012123456789012345678901
Creating good links can take some practice. Quite often, you might start 2
12345678901234567890123456789012123456789012345678901 2
12345678901234567890123456789012123456789012345678901
out with a general link and then have to modify it to be more specific. The 2
2345678901234567890123456789012123456789012345678901
12345678901234567890123456789012123456789012345678901
chart below discusses some of the more common types of Analogy links 2
2
12345678901234567890123456789012123456789012345678901 2
1 that will be encountered on the GRE.
12345678901234567890123456789012123456789012345678901 2
12345678901234567890123456789012123456789012345678901 2
12345678901234567890123456789012123456789012345678901 2
12345678901234567890123456789012123456789012345678901 2
12345678901234567890123456789012123456789012345678901
Common Types of Analogies 2
12345678901234567890123456789012123456789012345678901 2
12345678901234567890123456789012123456789012345678901 2
1. Synonyms: Synonyms are fairly simple analogies, since the two words
2345678901234567890123456789012123456789012345678901
mean the same thing. An example would be PREVARICATE : LIE, 2
12345678901234567890123456789012123456789012345678901
12345678901234567890123456789012123456789012345678901 2
12345678901234567890123456789012123456789012345678901 2
12345678901234567890123456789012123456789012345678901 2
since prevaricate is just a fancy word meaning “to lie.” Synonym
1 analogies like to combine one tough word with a simple one, leaving 2
12345678901234567890123456789012123456789012345678901 2
12345678901234567890123456789012123456789012345678901 2
12345678901234567890123456789012123456789012345678901
you to wonder what the relationship between the words are. The 2
12345678901234567890123456789012123456789012345678901 2
12345678901234567890123456789012123456789012345678901 2
words could be synonyms, or . . .
2345678901234567890123456789012123456789012345678901
12345678901234567890123456789012123456789012345678901 2
1 2. Antonyms: These analogies are exactly what you would expect them 2
12345678901234567890123456789012123456789012345678901
to be. PREVARICATION: TRUTH is an example of an antonym 2
12345678901234567890123456789012123456789012345678901 2
12345678901234567890123456789012123456789012345678901
2345678901234567890123456789012123456789012345678901
2
2
1 analogy. Many synonym and antonym analogies use adjectives as the
12345678901234567890123456789012123456789012345678901 2
12345678901234567890123456789012123456789012345678901 2
stem words.
2
146 12345678901234567890123456789012123456789012345678901
123456789012345678901234567890121234567890123456789012

www.petersons.com
Chapter 5: Verbal Review

123456789012345678901234567890121234567890123456789012
12345678901234567890123456789012123456789012345678901 2
12345678901234567890123456789012123456789012345678901 2
12345678901234567890123456789012123456789012345678901
3. Degree: Degree analogies combine two things that are similar in 2
12345678901234567890123456789012123456789012345678901 2
12345678901234567890123456789012123456789012345678901
nature but different in scale. BREEZE : GALE is a good example 2
12345678901234567890123456789012123456789012345678901 2
12345678901234567890123456789012123456789012345678901
since they are both winds, although a gale is much stronger.
2345678901234567890123456789012123456789012345678901
2
2
1
12345678901234567890123456789012123456789012345678901 2
12345678901234567890123456789012123456789012345678901
4. Set/Subset: Fans of categories will like these analogies. Subset 2
12345678901234567890123456789012123456789012345678901 2
12345678901234567890123456789012123456789012345678901
analogies often employ linking
2345678901234567890123456789012123456789012345678901
phrases such as “kind of” or “part 2
1 of.” ELM : TREE is a simple example, since an elm is a kind of tree. 2
12345678901234567890123456789012123456789012345678901 2
12345678901234567890123456789012123456789012345678901 2
12345678901234567890123456789012123456789012345678901
FEATHER : OSTRICH could also be considered one of these 2
12345678901234567890123456789012123456789012345678901
2345678901234567890123456789012123456789012345678901
2
2
1 analogies, but if you disagree, that’s fine, too. These categories
12345678901234567890123456789012123456789012345678901 2
12345678901234567890123456789012123456789012345678901
should be used if they help you come up with good links, but it 2
12345678901234567890123456789012123456789012345678901
doesn’t really matter whether or not each category is distinct from all 2
12345678901234567890123456789012123456789012345678901
2345678901234567890123456789012123456789012345678901
2
2
1 the others.
12345678901234567890123456789012123456789012345678901 2
12345678901234567890123456789012123456789012345678901 2
12345678901234567890123456789012123456789012345678901
5. Function: Certain things have been made with a specific purpose or 2
12345678901234567890123456789012123456789012345678901 2
12345678901234567890123456789012123456789012345678901
function. For instance, what do gloves do? GLOVES : HANDS is a 2
12345678901234567890123456789012123456789012345678901 2
12345678901234567890123456789012123456789012345678901
good example of a function relationship, since gloves are meant to be 2
12345678901234567890123456789012123456789012345678901 2
12345678901234567890123456789012123456789012345678901
worn on the hands. 2
12345678901234567890123456789012123456789012345678901 22
2345678901234567890123456789012123456789012345678901
1
12345678901234567890123456789012123456789012345678901 2
12345678901234567890123456789012123456789012345678901
You might also think that GLOVES : WARM is a good link, but this is not 2
12345678901234567890123456789012123456789012345678901 2
12345678901234567890123456789012123456789012345678901 22
2345678901234567890123456789012123456789012345678901
as strong. Gloves can be used to keep your hands warm, but gloves can
12345678901234567890123456789012123456789012345678901
12345678901234567890123456789012123456789012345678901 2
Note

also be worn for other purposes. A person wearing gloves while washing
12345678901234567890123456789012123456789012345678901 2
12345678901234567890123456789012123456789012345678901 22
dishes is wearing gloves for protection, not warmth. The two stem words
12345678901234567890123456789012123456789012345678901
12345678901234567890123456789012123456789012345678901
will always have a strong link, but the words in the answer choices do not 2
12345678901234567890123456789012123456789012345678901 2
12345678901234567890123456789012123456789012345678901 22
always contain strong links. If you saw GLOVES : WARM as an answer
1 choice, you could eliminate it as a likely answer choice because the link
12345678901234567890123456789012123456789012345678901 2
12345678901234567890123456789012123456789012345678901 2
12345678901234567890123456789012123456789012345678901 2
between the two words is not very strong.
12345678901234567890123456789012123456789012345678901 2
12345678901234567890123456789012123456789012345678901 2
12345678901234567890123456789012123456789012345678901
6. Definitional: This is a big catch-all category. One stem word often 2
12345678901234567890123456789012123456789012345678901 2
12345678901234567890123456789012123456789012345678901 2
12345678901234567890123456789012123456789012345678901
plays a big part in defining the other stem word. CANDLE : WAX is 2
12345678901234567890123456789012123456789012345678901 2
12345678901234567890123456789012123456789012345678901 22
2345678901234567890123456789012123456789012345678901
an example, since it is hard to describe a candle without using the
12345678901234567890123456789012123456789012345678901
term wax. While there are some candles that might not be made out 2
1
12345678901234567890123456789012123456789012345678901
of wax, the overwhelming majority are, so a link like “a candle is 2
12345678901234567890123456789012123456789012345678901 2
2345678901234567890123456789012123456789012345678901
made out of wax” might be all you need to answer a problem 2
12345678901234567890123456789012123456789012345678901
12345678901234567890123456789012123456789012345678901 2
correctly. There is the possibility you would have to get more specific, 2
12345678901234567890123456789012123456789012345678901
12345678901234567890123456789012123456789012345678901 22
1 so you could modify this first definition and change it to something
12345678901234567890123456789012123456789012345678901 2
12345678901234567890123456789012123456789012345678901
more precise such as, “A candle is made out of wax, which is then 2
12345678901234567890123456789012123456789012345678901 2
12345678901234567890123456789012123456789012345678901
burned away.” 2
12345678901234567890123456789012123456789012345678901 2
2345678901234567890123456789012123456789012345678901
This list could be given more categories, but the overall goal is to show you 2
12345678901234567890123456789012123456789012345678901
12345678901234567890123456789012123456789012345678901 2
12345678901234567890123456789012123456789012345678901 2
12345678901234567890123456789012123456789012345678901 22
some common routes you can take to create a link between the stem
1 words. Look at the two analogies questions below, and come up with
12345678901234567890123456789012123456789012345678901 2
12345678901234567890123456789012123456789012345678901 2
12345678901234567890123456789012123456789012345678901
strong links that describe a connection between the two words. See if your 2
12345678901234567890123456789012123456789012345678901 2
12345678901234567890123456789012123456789012345678901
links are good enough to enable you
2345678901234567890123456789012123456789012345678901
to answer both questions correctly. 2
2
1
12345678901234567890123456789012123456789012345678901 2
12345678901234567890123456789012123456789012345678901 2
12345678901234567890123456789012123456789012345678901 2 147
123456789012345678901234567890121234567890123456789012
Part III: Section Review

123456789012345678901234567890121234567890123456789012
12345678901234567890123456789012123456789012345678901 2
12345678901234567890123456789012123456789012345678901 2
12345678901234567890123456789012123456789012345678901
SHEPHERD : FLOCK :: 2
12345678901234567890123456789012123456789012345678901 2
12345678901234567890123456789012123456789012345678901 2
12345678901234567890123456789012123456789012345678901 2
A. apothecary : drugs
12345678901234567890123456789012123456789012345678901 2
12345678901234567890123456789012123456789012345678901
B. butcher : carcass 2
12345678901234567890123456789012123456789012345678901 2
12345678901234567890123456789012123456789012345678901
C. physician : patients 2
12345678901234567890123456789012123456789012345678901 2
12345678901234567890123456789012123456789012345678901
D. farmer : corn
2345678901234567890123456789012123456789012345678901
2
2
1
12345678901234567890123456789012123456789012345678901
E. businessperson : employer 2
12345678901234567890123456789012123456789012345678901 2
12345678901234567890123456789012123456789012345678901 2
12345678901234567890123456789012123456789012345678901
BANEFUL : BENEFIT ::
2345678901234567890123456789012123456789012345678901
2
2
1
12345678901234567890123456789012123456789012345678901 2
12345678901234567890123456789012123456789012345678901
A. helpful : cure 2
12345678901234567890123456789012123456789012345678901 2
12345678901234567890123456789012123456789012345678901
B. tragic : end
2345678901234567890123456789012123456789012345678901
2
2
1
12345678901234567890123456789012123456789012345678901
C. toxic : poison 2
12345678901234567890123456789012123456789012345678901 2
12345678901234567890123456789012123456789012345678901
D. flawless : aberration 2
12345678901234567890123456789012123456789012345678901 2
12345678901234567890123456789012123456789012345678901 2
2345678901234567890123456789012123456789012345678901
E. disguised : concealment
12345678901234567890123456789012123456789012345678901 2
You can describe the first two stem words in either functional or 2
12345678901234567890123456789012123456789012345678901
12345678901234567890123456789012123456789012345678901 2
12345678901234567890123456789012123456789012345678901 2
12345678901234567890123456789012123456789012345678901 2
definitional terms:
2
1
12345678901234567890123456789012123456789012345678901
Functional route: 2
12345678901234567890123456789012123456789012345678901 2
12345678901234567890123456789012123456789012345678901 2
12345678901234567890123456789012123456789012345678901 2
2345678901234567890123456789012123456789012345678901
he function of a SHEPHERD is to take care of his/her FLOCK.
12345678901234567890123456789012123456789012345678901 2
12345678901234567890123456789012123456789012345678901 2
12345678901234567890123456789012123456789012345678901
Definitional route: 2
12345678901234567890123456789012123456789012345678901 2
12345678901234567890123456789012123456789012345678901 2
12345678901234567890123456789012123456789012345678901
A SHEPHERD is someone who takes care of a FLOCK. 2
12345678901234567890123456789012123456789012345678901 2
12345678901234567890123456789012123456789012345678901 2
1 Both links do a good job of describing the connection between a shepherd 2
2345678901234567890123456789012123456789012345678901
and a flock. Taking either link and heading into the answer choices, you 2
12345678901234567890123456789012123456789012345678901
12345678901234567890123456789012123456789012345678901 2
1 should see that choice (C) works better than any other. A PHYSICIAN is 2
12345678901234567890123456789012123456789012345678901 2
12345678901234567890123456789012123456789012345678901
someone who takes care of his/her PATIENTS. Some people will balk at 2
12345678901234567890123456789012123456789012345678901 2
12345678901234567890123456789012123456789012345678901
this choice, mainly because they do not want to be compared with sheep. 2
12345678901234567890123456789012123456789012345678901 2
12345678901234567890123456789012123456789012345678901
Yet the whole point of analogous relationships is to show a shared 2
12345678901234567890123456789012123456789012345678901 2
12345678901234567890123456789012123456789012345678901
similarity between two seemingly dissimilar groups. 2
12345678901234567890123456789012123456789012345678901 2
12345678901234567890123456789012123456789012345678901 2
12345678901234567890123456789012123456789012345678901 2
For the second analogy problem, the words baneful and benefit have
12345678901234567890123456789012123456789012345678901
opposite connotations. Something that is BANEFUL has no BENEFIT, so 2
12345678901234567890123456789012123456789012345678901 2
12345678901234567890123456789012123456789012345678901 2
12345678901234567890123456789012123456789012345678901
you can use the link “has no” to check all the answer choices. Choices (C) 2
12345678901234567890123456789012123456789012345678901 2
12345678901234567890123456789012123456789012345678901 2
and (E) are mostly synonyms, so while there is a strong link between the
2345678901234567890123456789012123456789012345678901
words in these answer choices, they do not have the link you are looking 2
12345678901234567890123456789012123456789012345678901
12345678901234567890123456789012123456789012345678901 2
12345678901234567890123456789012123456789012345678901 2
12345678901234567890123456789012123456789012345678901 2
for. Something FLAWLESS has no ABERRATION, making (D) the correct
2
1 answer.
12345678901234567890123456789012123456789012345678901 2
12345678901234567890123456789012123456789012345678901 2
12345678901234567890123456789012123456789012345678901
The words used in the second problem were deliberately a lot tougher than 2
12345678901234567890123456789012123456789012345678901 2
12345678901234567890123456789012123456789012345678901
the words in the first problem. If you didn’t know what baneful meant, it 2
2345678901234567890123456789012123456789012345678901
12345678901234567890123456789012123456789012345678901 2
1 would be hard to make a link between the two stem words. There will be 2
12345678901234567890123456789012123456789012345678901 2
12345678901234567890123456789012123456789012345678901
some tough stem words on the GRE, so there are some strategies you can 2
12345678901234567890123456789012123456789012345678901 2
12345678901234567890123456789012123456789012345678901
use to approach these problems. 2
12345678901234567890123456789012123456789012345678901 2
12345678901234567890123456789012123456789012345678901 2 2
148 12345678901234567890123456789012123456789012345678901
123456789012345678901234567890121234567890123456789012

www.petersons.com
Chapter 5: Verbal Review

123456789012345678901234567890121234567890123456789012
12345678901234567890123456789012123456789012345678901 2
12345678901234567890123456789012123456789012345678901 2
12345678901234567890123456789012123456789012345678901
What to Do When the Stem Words Are Just 2
12345678901234567890123456789012123456789012345678901 2
12345678901234567890123456789012123456789012345678901
Too Freaky 2
12345678901234567890123456789012123456789012345678901 2
12345678901234567890123456789012123456789012345678901
There will always be a strong link between the two stem words. This is not 2
12345678901234567890123456789012123456789012345678901 2
12345678901234567890123456789012123456789012345678901 2
12345678901234567890123456789012123456789012345678901
the case with the answer choices. Some of these will have weak links, or 2
12345678901234567890123456789012123456789012345678901 2
12345678901234567890123456789012123456789012345678901
even no links at all. Since the correct answer must have the same, strong 2
1 link as the two stem words, any answer choices with weak links can be 2
2345678901234567890123456789012123456789012345678901
12345678901234567890123456789012123456789012345678901 2
12345678901234567890123456789012123456789012345678901
eliminated. There’s no way they could be the right answer, because the 2
12345678901234567890123456789012123456789012345678901 2
12345678901234567890123456789012123456789012345678901
right answer must have a strong link.
2345678901234567890123456789012123456789012345678901
2
2
1
12345678901234567890123456789012123456789012345678901 2
12345678901234567890123456789012123456789012345678901
This is the best elimination technique you have, so be prepared to use it on 2
12345678901234567890123456789012123456789012345678901 2
12345678901234567890123456789012123456789012345678901
lots of problems. Let’s look back at the very first analogy problem in this 2
2345678901234567890123456789012123456789012345678901 2
1 section.
12345678901234567890123456789012123456789012345678901 2
12345678901234567890123456789012123456789012345678901 2
12345678901234567890123456789012123456789012345678901 2
12345678901234567890123456789012123456789012345678901 2
12345678901234567890123456789012123456789012345678901 22
2345678901234567890123456789012123456789012345678901
FEATHER : OSTRICH ::
12345678901234567890123456789012123456789012345678901 2
12345678901234567890123456789012123456789012345678901
A. beak : raven
12345678901234567890123456789012123456789012345678901 2
12345678901234567890123456789012123456789012345678901
B. scale : lizard 2
12345678901234567890123456789012123456789012345678901 22
1 C. foot : chicken
12345678901234567890123456789012123456789012345678901 2
12345678901234567890123456789012123456789012345678901
D. desert : cactus 2
12345678901234567890123456789012123456789012345678901 2
12345678901234567890123456789012123456789012345678901 22
2345678901234567890123456789012123456789012345678901
E. tail : mammal
12345678901234567890123456789012123456789012345678901 2
12345678901234567890123456789012123456789012345678901
Assume you have no idea what the word feather means. Go down to your 2
12345678901234567890123456789012123456789012345678901
12345678901234567890123456789012123456789012345678901 2
12345678901234567890123456789012123456789012345678901
answer choices and try to make links between the two words. 2
12345678901234567890123456789012123456789012345678901 22
12345678901234567890123456789012123456789012345678901
12345678901234567890123456789012123456789012345678901
A. beak : raven—a BEAK is like the mouth of a RAVEN 2
1 2
12345678901234567890123456789012123456789012345678901 22
2345678901234567890123456789012123456789012345678901
B. scale : lizard—a LIZARD is covered in SCALES (It’s OK to
12345678901234567890123456789012123456789012345678901
reverse direction if you need to, and you can add a plural if 2
1
12345678901234567890123456789012123456789012345678901
you like) 2
12345678901234567890123456789012123456789012345678901
2345678901234567890123456789012123456789012345678901
2
2
12345678901234567890123456789012123456789012345678901
C. foot : chicken—a CHICKEN kinda has a FOOT
12345678901234567890123456789012123456789012345678901 2
1 D. desert : cactus—a DESERT is where you commonly find 2
12345678901234567890123456789012123456789012345678901 2
12345678901234567890123456789012123456789012345678901
CACTUS 2
2345678901234567890123456789012123456789012345678901
12345678901234567890123456789012123456789012345678901 2
12345678901234567890123456789012123456789012345678901
E. tail : mammal—a TAIL can often be found on a MAMMAL, 2
1 2
12345678901234567890123456789012123456789012345678901
but not always 2
12345678901234567890123456789012123456789012345678901 2
2345678901234567890123456789012123456789012345678901
Choice (C) is not really a strong link, and (E) is pretty shaky, too. 2
12345678901234567890123456789012123456789012345678901
12345678901234567890123456789012123456789012345678901 2
12345678901234567890123456789012123456789012345678901 2
12345678901234567890123456789012123456789012345678901 22
Mammals can have tails, but not every one does. Both (C) and (E) are
1 weak choices that can be crossed out as possible answers. Choices (A) and
12345678901234567890123456789012123456789012345678901 2
12345678901234567890123456789012123456789012345678901 2
12345678901234567890123456789012123456789012345678901
(D) have fine links. But if (D) is the right answer, then the stem words 2
12345678901234567890123456789012123456789012345678901 2
12345678901234567890123456789012123456789012345678901 2
would have to share the same link, meaning that a FEATHER “is where
2345678901234567890123456789012123456789012345678901
you commonly find” an OSTRICH. You would pick this if you thought 2
12345678901234567890123456789012123456789012345678901
12345678901234567890123456789012123456789012345678901 2
12345678901234567890123456789012123456789012345678901 2
12345678901234567890123456789012123456789012345678901 22
there was a specific word out there meaning, “the place where ostriches
1 are commonly found.” The existence of such a word is unlikely, so this
12345678901234567890123456789012123456789012345678901 2
12345678901234567890123456789012123456789012345678901
takes (D) out of the picture. You are left with (A) or (C), two solid choices. 2
12345678901234567890123456789012123456789012345678901 2
12345678901234567890123456789012123456789012345678901 2
12345678901234567890123456789012123456789012345678901
Let’s revisit choice (C) once more before bidding a fond farewell to this 2
1 problem. Since foot and chicken do not share any real connection, why is 2
2345678901234567890123456789012123456789012345678901
12345678901234567890123456789012123456789012345678901 2
12345678901234567890123456789012123456789012345678901 2
12345678901234567890123456789012123456789012345678901 2 149
123456789012345678901234567890121234567890123456789012
Part III: Section Review

123456789012345678901234567890121234567890123456789012
12345678901234567890123456789012123456789012345678901 2
12345678901234567890123456789012123456789012345678901 2
12345678901234567890123456789012123456789012345678901
this choice even here? The reason lies in the fact that most people like to 2
12345678901234567890123456789012123456789012345678901 2
12345678901234567890123456789012123456789012345678901
play the “Connect the Words” game when tackling tough analogies 2
12345678901234567890123456789012123456789012345678901 2
12345678901234567890123456789012123456789012345678901
problems. This means they might connect the second stem word with the 2
1 second word in an answer choice, or maybe they’ll take the first stem word 2
2345678901234567890123456789012123456789012345678901
12345678901234567890123456789012123456789012345678901 2
12345678901234567890123456789012123456789012345678901 2
12345678901234567890123456789012123456789012345678901
and make a connection with the second word of a different answer choice. 2
12345678901234567890123456789012123456789012345678901
2345678901234567890123456789012123456789012345678901
2
2
1 They’ll make links however they can, with no regard for the strict rules
12345678901234567890123456789012123456789012345678901 2
12345678901234567890123456789012123456789012345678901
regarding analogous relationships. In terms of this question, this means 2
12345678901234567890123456789012123456789012345678901 2
12345678901234567890123456789012123456789012345678901
people will link ostrich, a bird, with chicken or raven, two other birds. 2
1 Who cares what the first words are? Chickens and ostriches are both birds, 2
2345678901234567890123456789012123456789012345678901
12345678901234567890123456789012123456789012345678901 2
12345678901234567890123456789012123456789012345678901 2
12345678901234567890123456789012123456789012345678901
aren’t they? 2
12345678901234567890123456789012123456789012345678901 2
1 None of these “Connect the Words” links are valid. They are traps 2
2345678901234567890123456789012123456789012345678901
12345678901234567890123456789012123456789012345678901 2
12345678901234567890123456789012123456789012345678901
designed to lead you astray. It doesn’t matter that chicken and ostriches are 2
12345678901234567890123456789012123456789012345678901 2
12345678901234567890123456789012123456789012345678901
both birds. You can only make links between the two paired words, and 2
12345678901234567890123456789012123456789012345678901 2
2345678901234567890123456789012123456789012345678901
12345678901234567890123456789012123456789012345678901
then compare links to one another. 2
12345678901234567890123456789012123456789012345678901 2
12345678901234567890123456789012123456789012345678901 2
12345678901234567890123456789012123456789012345678901 2
The temptation to play “Connect the Words” gets stronger when you have
12345678901234567890123456789012123456789012345678901 2
1 some tough words that you might not know. 2
12345678901234567890123456789012123456789012345678901 2
12345678901234567890123456789012123456789012345678901 2
12345678901234567890123456789012123456789012345678901 2
12345678901234567890123456789012123456789012345678901 2
2345678901234567890123456789012123456789012345678901
Looking back at the shepherd and flock problem, you can see two answer
12345678901234567890123456789012123456789012345678901
choices designed to catch people making incorrect connections. The 2
2
12345678901234567890123456789012123456789012345678901
words farmer and corn do not really have a strong link, since farmers can 2
12345678901234567890123456789012123456789012345678901

Note
12345678901234567890123456789012123456789012345678901 2
grow many other things besides corn. It’s there to catch people combining 2
12345678901234567890123456789012123456789012345678901
12345678901234567890123456789012123456789012345678901 2
12345678901234567890123456789012123456789012345678901 2
12345678901234567890123456789012123456789012345678901 2
shepherd and farmer, two pastoral occupations. Some people might
1 equate shepherds with tending herd animals of some sort, and these 2
12345678901234567890123456789012123456789012345678901 2
12345678901234567890123456789012123456789012345678901 2
12345678901234567890123456789012123456789012345678901 2
animals often end up with a butcher, the first word in choice (B).
12345678901234567890123456789012123456789012345678901 2
12345678901234567890123456789012123456789012345678901 2
12345678901234567890123456789012123456789012345678901 2
12345678901234567890123456789012123456789012345678901
BANEFUL : BENEFIT :: 2
12345678901234567890123456789012123456789012345678901 2
12345678901234567890123456789012123456789012345678901
A. helpful : cure 2
12345678901234567890123456789012123456789012345678901 2
2345678901234567890123456789012123456789012345678901
12345678901234567890123456789012123456789012345678901
B. tragic : end 2
12345678901234567890123456789012123456789012345678901 2
1 C. toxic : poison 2
12345678901234567890123456789012123456789012345678901 2
12345678901234567890123456789012123456789012345678901
D. flawless : aberration 2
12345678901234567890123456789012123456789012345678901 2
12345678901234567890123456789012123456789012345678901
E. camouflaged : concealment 2
12345678901234567890123456789012123456789012345678901 2
12345678901234567890123456789012123456789012345678901 2
12345678901234567890123456789012123456789012345678901 2
If you don’t know what baneful means, attack this question by heading
2345678901234567890123456789012123456789012345678901
into the answer choices to test the links there. Choice (C) contains two 2
12345678901234567890123456789012123456789012345678901
12345678901234567890123456789012123456789012345678901 2
12345678901234567890123456789012123456789012345678901 2
12345678901234567890123456789012123456789012345678901 2
synonyms, so it remains a likely pick if you think baneful and benefit are
1 synonymous. When you try to make a link for choices (A) and (B), you will 2
12345678901234567890123456789012123456789012345678901 2
12345678901234567890123456789012123456789012345678901 2
12345678901234567890123456789012123456789012345678901
find that the words are not really related. Choice (A) is probably there for 2
12345678901234567890123456789012123456789012345678901 2
12345678901234567890123456789012123456789012345678901 2
people connecting cure and benefit, two positive words. You should realize
2345678901234567890123456789012123456789012345678901
12345678901234567890123456789012123456789012345678901 2
1 by now that choice (A) is wrong for two reasons: It has a weak link, and 2
12345678901234567890123456789012123456789012345678901 2
12345678901234567890123456789012123456789012345678901
it’s a trap for folks playing “Connect the Words.” 2
12345678901234567890123456789012123456789012345678901
2345678901234567890123456789012123456789012345678901
2
2
1
12345678901234567890123456789012123456789012345678901 2
12345678901234567890123456789012123456789012345678901 2 2
150 12345678901234567890123456789012123456789012345678901
123456789012345678901234567890121234567890123456789012

www.petersons.com
Chapter 5: Verbal Review

123456789012345678901234567890121234567890123456789012
12345678901234567890123456789012123456789012345678901 2
12345678901234567890123456789012123456789012345678901 2
12345678901234567890123456789012123456789012345678901
Choices (C), (D), and (E) all contain good links, so they should all remain. 2
12345678901234567890123456789012123456789012345678901 2
12345678901234567890123456789012123456789012345678901
A one-in-three chance may not be ideal, but it shows once again that you 2
12345678901234567890123456789012123456789012345678901 2
12345678901234567890123456789012123456789012345678901
can use techniques to give yourself a fighting chance even on problems 2
2345678901234567890123456789012123456789012345678901 2
1 where you don’t know the meaning of some of the words.
12345678901234567890123456789012123456789012345678901 2
12345678901234567890123456789012123456789012345678901 2
12345678901234567890123456789012123456789012345678901 2
12345678901234567890123456789012123456789012345678901
2345678901234567890123456789012123456789012345678901
2
2
1
12345678901234567890123456789012123456789012345678901
This __________ of the Book Deals with 2
12345678901234567890123456789012123456789012345678901 2
12345678901234567890123456789012123456789012345678901 2
12345678901234567890123456789012123456789012345678901
Sentence __________
2345678901234567890123456789012123456789012345678901
2
2
1
12345678901234567890123456789012123456789012345678901
If you could figure out that heading, you are well on your way to 2
12345678901234567890123456789012123456789012345678901 2
12345678901234567890123456789012123456789012345678901
succeeding on Sentence Completions (SC) problems. Like the heading, SC 2
12345678901234567890123456789012123456789012345678901 2
12345678901234567890123456789012123456789012345678901
problems feature a sentence—often a convoluted one filled with multiple 2
12345678901234567890123456789012123456789012345678901 2
12345678901234567890123456789012123456789012345678901
types of punctuation (commas, semicolons, and so forth), kind of like the 2
12345678901234567890123456789012123456789012345678901 2
12345678901234567890123456789012123456789012345678901
sentence you are currently reading—that has one or more words missing 2
2345678901234567890123456789012123456789012345678901
12345678901234567890123456789012123456789012345678901 2
12345678901234567890123456789012123456789012345678901
from it. A sample SC problem would look like: 2
12345678901234567890123456789012123456789012345678901 22
12345678901234567890123456789012123456789012345678901 2
12345678901234567890123456789012123456789012345678901
12345678901234567890123456789012123456789012345678901
SC problems feature a sentence—often a convoluted one filled with 2
1 2
12345678901234567890123456789012123456789012345678901
multiple types of __________ (commas, semicolons, and so forth), 2
12345678901234567890123456789012123456789012345678901
kind of like the sentence you are currently reading—that has one or 2
12345678901234567890123456789012123456789012345678901 2
2345678901234567890123456789012123456789012345678901
12345678901234567890123456789012123456789012345678901
more words missing from it. 2
12345678901234567890123456789012123456789012345678901 2
12345678901234567890123456789012123456789012345678901 2
12345678901234567890123456789012123456789012345678901
A. pauses 2
12345678901234567890123456789012123456789012345678901 2
12345678901234567890123456789012123456789012345678901 22
B. punctuation
12345678901234567890123456789012123456789012345678901
12345678901234567890123456789012123456789012345678901
C. grammar 2
12345678901234567890123456789012123456789012345678901 2
1 D. issues 2
2345678901234567890123456789012123456789012345678901
12345678901234567890123456789012123456789012345678901 2
12345678901234567890123456789012123456789012345678901
E. subjects 2
1 2
12345678901234567890123456789012123456789012345678901 2
12345678901234567890123456789012123456789012345678901
Using context is the critical element with
2345678901234567890123456789012123456789012345678901
SC problems. You are not 2
required to conjecture randomly to figure out what the missing word 2
12345678901234567890123456789012123456789012345678901
12345678901234567890123456789012123456789012345678901 2
1 should be. Instead, every sentence will contain hints and clues in the 2
12345678901234567890123456789012123456789012345678901 2
12345678901234567890123456789012123456789012345678901 2
12345678901234567890123456789012123456789012345678901 22
2345678901234567890123456789012123456789012345678901
context that should steer you toward the right word to place in the missing
12345678901234567890123456789012123456789012345678901
1 blank. In the sample sentence above, two words immediately following the 2
12345678901234567890123456789012123456789012345678901
blank are commas and semicolons, both forms of punctuation. These are 2
12345678901234567890123456789012123456789012345678901 2
2345678901234567890123456789012123456789012345678901
the main clues that should lead you to the correct answer of choice (B), 2
12345678901234567890123456789012123456789012345678901
12345678901234567890123456789012123456789012345678901 2
punctuation. Of course, having the entire sentence directly above the 2
12345678901234567890123456789012123456789012345678901
12345678901234567890123456789012123456789012345678901 22
1 problem also helps.
12345678901234567890123456789012123456789012345678901 2
12345678901234567890123456789012123456789012345678901 2
12345678901234567890123456789012123456789012345678901 2
12345678901234567890123456789012123456789012345678901 2
12345678901234567890123456789012123456789012345678901 2
12345678901234567890123456789012123456789012345678901 22
2345678901234567890123456789012123456789012345678901
12345678901234567890123456789012123456789012345678901 2
12345678901234567890123456789012123456789012345678901 2
12345678901234567890123456789012123456789012345678901 2
1
12345678901234567890123456789012123456789012345678901 2
12345678901234567890123456789012123456789012345678901 2
12345678901234567890123456789012123456789012345678901 2
12345678901234567890123456789012123456789012345678901 2
12345678901234567890123456789012123456789012345678901 2
12345678901234567890123456789012123456789012345678901 2
12345678901234567890123456789012123456789012345678901 2
12345678901234567890123456789012123456789012345678901 2
12345678901234567890123456789012123456789012345678901 2 151
123456789012345678901234567890121234567890123456789012
Part III: Section Review

123456789012345678901234567890121234567890123456789012
12345678901234567890123456789012123456789012345678901 2
12345678901234567890123456789012123456789012345678901 2
12345678901234567890123456789012123456789012345678901 2
12345678901234567890123456789012123456789012345678901
The idea of context should be familiar to everyone enhancing their 2
12345678901234567890123456789012123456789012345678901 2
12345678901234567890123456789012123456789012345678901 2
12345678901234567890123456789012123456789012345678901
vocabulary by grabbing unfamiliar words in the context of everyday 2
1 reading. (This topic was discussed on page 137 in this section.) When you 2
2345678901234567890123456789012123456789012345678901

X-Ref
12345678901234567890123456789012123456789012345678901 2
12345678901234567890123456789012123456789012345678901
encounter an unfamiliar word in context, you can often determine its 2
12345678901234567890123456789012123456789012345678901 2
12345678901234567890123456789012123456789012345678901
meaning from the way it is used in the sentence it is in. You figure out the 2
2345678901234567890123456789012123456789012345678901 2
1
12345678901234567890123456789012123456789012345678901
meaning of the word through its context, and this practice is very similar 2
12345678901234567890123456789012123456789012345678901 2
12345678901234567890123456789012123456789012345678901
to the strategy used to determine what word should appear in the blank 2
12345678901234567890123456789012123456789012345678901 2
12345678901234567890123456789012123456789012345678901
space of the Sentence Completion problem. 2
12345678901234567890123456789012123456789012345678901 2
12345678901234567890123456789012123456789012345678901 2
12345678901234567890123456789012123456789012345678901 2
12345678901234567890123456789012123456789012345678901
Just like antonym problems, anticipating the answer will help you on SC 2
1 problems. People who read the original sentence and then go shopping in 2
2345678901234567890123456789012123456789012345678901
12345678901234567890123456789012123456789012345678901 2
12345678901234567890123456789012123456789012345678901 2
12345678901234567890123456789012123456789012345678901
the answer choices increase their possibility of picking a wrong answer. 2
12345678901234567890123456789012123456789012345678901 2
12345678901234567890123456789012123456789012345678901
The incorrect answers are created to catch shoppers, so don’t follow this 2
12345678901234567890123456789012123456789012345678901
pattern of behavior. Instead, read the original sentence slowly, and try to 2
12345678901234567890123456789012123456789012345678901 2
12345678901234567890123456789012123456789012345678901 2
12345678901234567890123456789012123456789012345678901
decide what word would fit best in each blank. 2
12345678901234567890123456789012123456789012345678901 2
2345678901234567890123456789012123456789012345678901
1 Figuring out a word can be difficult, but remember that every sentence will 2
12345678901234567890123456789012123456789012345678901 2
12345678901234567890123456789012123456789012345678901 2
12345678901234567890123456789012123456789012345678901
contain context clues to help you determine the right answer. If the missing 2
2345678901234567890123456789012123456789012345678901
word doesn’t come to you right away, scan the sentence and look for words 2
12345678901234567890123456789012123456789012345678901
12345678901234567890123456789012123456789012345678901 2
12345678901234567890123456789012123456789012345678901
or phrases that provide a clue. There will always be some. If you come up 2
12345678901234567890123456789012123456789012345678901 2
with a word but are a bit uncertain it is the correct one, finding the context 2
12345678901234567890123456789012123456789012345678901
12345678901234567890123456789012123456789012345678901 2
12345678901234567890123456789012123456789012345678901
clues can help you decide whether or not you made the right choice. 2
12345678901234567890123456789012123456789012345678901 2
12345678901234567890123456789012123456789012345678901 2 2
1
12345678901234567890123456789012123456789012345678901
Determined to bring back an egg of the Emperor penguin, Apsley 2
12345678901234567890123456789012123456789012345678901 2
12345678901234567890123456789012123456789012345678901
and his companions struggled through ferocious blizzards, 2
12345678901234567890123456789012123456789012345678901 2
12345678901234567890123456789012123456789012345678901
__________ low temperatures, and across
2345678901234567890123456789012123456789012345678901
dangerously unknown 2
2
12345678901234567890123456789012123456789012345678901
12345678901234567890123456789012123456789012345678901
Antarctic __________ in order to complete their mission. 2
1 2
12345678901234567890123456789012123456789012345678901 2
12345678901234567890123456789012123456789012345678901
A. appallingly. .terrain 2
2345678901234567890123456789012123456789012345678901
12345678901234567890123456789012123456789012345678901
B. terrifyingly. .errors 2
12345678901234567890123456789012123456789012345678901 2
1 C. cryptically. .ice 2
12345678901234567890123456789012123456789012345678901 2
12345678901234567890123456789012123456789012345678901
D. bitingly. .forces 2
12345678901234567890123456789012123456789012345678901 2
12345678901234567890123456789012123456789012345678901
E. stingingly. .circumstances 2
12345678901234567890123456789012123456789012345678901 2
12345678901234567890123456789012123456789012345678901 2
12345678901234567890123456789012123456789012345678901 2
2345678901234567890123456789012123456789012345678901
The sample sentence also contains directional words. These are words 2
12345678901234567890123456789012123456789012345678901
12345678901234567890123456789012123456789012345678901 2
that change or continue the flow of a sentence. A conjunction like and 2
12345678901234567890123456789012123456789012345678901
12345678901234567890123456789012123456789012345678901 2
Tip

1 continues the flow of a sentence, while words like but, although, or 2


12345678901234567890123456789012123456789012345678901 2
12345678901234567890123456789012123456789012345678901
however change the flow of sentence. Since changing the flow makes the 2
12345678901234567890123456789012123456789012345678901 2
12345678901234567890123456789012123456789012345678901
meaning of a sentence harder to determine, you can expect to see at least 2
12345678901234567890123456789012123456789012345678901 2
12345678901234567890123456789012123456789012345678901
some changing directional words on your SC problems. 2
12345678901234567890123456789012123456789012345678901 2
12345678901234567890123456789012123456789012345678901 2
12345678901234567890123456789012123456789012345678901 2
12345678901234567890123456789012123456789012345678901
This two-blank SC problem has a host of
2345678901234567890123456789012123456789012345678901
context clues. We can break 2
2
1 down this entire sentence and discuss the relevance of all the words.
12345678901234567890123456789012123456789012345678901 2
12345678901234567890123456789012123456789012345678901 2 2
152 12345678901234567890123456789012123456789012345678901
123456789012345678901234567890121234567890123456789012

www.petersons.com
Chapter 5: Verbal Review

123456789012345678901234567890121234567890123456789012
12345678901234567890123456789012123456789012345678901 2
12345678901234567890123456789012123456789012345678901 2
12345678901234567890123456789012123456789012345678901 2
12345678901234567890123456789012123456789012345678901
Word or Phrase Significance 2
12345678901234567890123456789012123456789012345678901 2
12345678901234567890123456789012123456789012345678901 2
12345678901234567890123456789012123456789012345678901
“Determined to On many SC problems,
2345678901234567890123456789012123456789012345678901
the introductory phrase 2
2
1
12345678901234567890123456789012123456789012345678901
bring back an is employed to set up a change in the flow of the 2
12345678901234567890123456789012123456789012345678901 2
12345678901234567890123456789012123456789012345678901
egg of the Em- sentence. These introductory phrases start with 2
12345678901234567890123456789012123456789012345678901
2345678901234567890123456789012123456789012345678901
2
2
1 peror penguin” words like although, while, and something
12345678901234567890123456789012123456789012345678901
related. However, this phrase does not do this. 2
12345678901234567890123456789012123456789012345678901 2
12345678901234567890123456789012123456789012345678901
The key words in this phrase might be Deter- 2
12345678901234567890123456789012123456789012345678901
2345678901234567890123456789012123456789012345678901
2
2
1 mined—showing resolution—and Emperor pen-
12345678901234567890123456789012123456789012345678901 2
12345678901234567890123456789012123456789012345678901
guin, which would lead to start thinking about 2
12345678901234567890123456789012123456789012345678901 2
12345678901234567890123456789012123456789012345678901
cold weather, since penguins are associated
2345678901234567890123456789012123456789012345678901
2
2
1
12345678901234567890123456789012123456789012345678901
with cold weather. 2
12345678901234567890123456789012123456789012345678901 2
12345678901234567890123456789012123456789012345678901 2
12345678901234567890123456789012123456789012345678901
“Apsley and his Not much here except for the fact that Apsley’s 2
2345678901234567890123456789012123456789012345678901
12345678901234567890123456789012123456789012345678901 2
12345678901234567890123456789012123456789012345678901
companions” a decidely British name? In terms of answering 2
12345678901234567890123456789012123456789012345678901 2
12345678901234567890123456789012123456789012345678901 22
a SC question, the phrase and his companions
12345678901234567890123456789012123456789012345678901
is completely worthless. Whether or not Apsley
12345678901234567890123456789012123456789012345678901 2
1 acted alone or with an entire troupe of clowns 2
12345678901234567890123456789012123456789012345678901 2
12345678901234567890123456789012123456789012345678901
does not help you figure out the missing words. 2
12345678901234567890123456789012123456789012345678901 2
2345678901234567890123456789012123456789012345678901
12345678901234567890123456789012123456789012345678901
Think of these worthless phrases as sentence 2
12345678901234567890123456789012123456789012345678901 2
12345678901234567890123456789012123456789012345678901
completion chaff. 2
12345678901234567890123456789012123456789012345678901 22
12345678901234567890123456789012123456789012345678901
12345678901234567890123456789012123456789012345678901
“struggled Here’s a big clue that ties in well with the 2
12345678901234567890123456789012123456789012345678901 2
12345678901234567890123456789012123456789012345678901 22
through fero- penguin clue from earlier. The words penguin
12345678901234567890123456789012123456789012345678901 2
1 cious blizzards” and blizzards should evoke images of very cold
12345678901234567890123456789012123456789012345678901 2
12345678901234567890123456789012123456789012345678901
weather. The words struggled and ferocious 2
12345678901234567890123456789012123456789012345678901
reinforce the idea that the place Apsley is at is 2
12345678901234567890123456789012123456789012345678901 2
12345678901234567890123456789012123456789012345678901
very harsh and cold. 2
12345678901234567890123456789012123456789012345678901 2
12345678901234567890123456789012123456789012345678901 2
12345678901234567890123456789012123456789012345678901
“__________ low Given the way this sentence is going, you 2
12345678901234567890123456789012123456789012345678901 2
12345678901234567890123456789012123456789012345678901
temperatures” should conjecture that the missing word should 2
2345678901234567890123456789012123456789012345678901
12345678901234567890123456789012123456789012345678901 2
12345678901234567890123456789012123456789012345678901
be something like “extremely” or “terribly.” 2
1 2
12345678901234567890123456789012123456789012345678901 2
The phrase “pleasantly low temperatures”
12345678901234567890123456789012123456789012345678901 2
12345678901234567890123456789012123456789012345678901
simply would not jive with everything you’ve 2
12345678901234567890123456789012123456789012345678901 2
12345678901234567890123456789012123456789012345678901
read up to now. 2
12345678901234567890123456789012123456789012345678901 2
12345678901234567890123456789012123456789012345678901
“and across Apsley hasn’t gone to poach penguin eggs from 2
2345678901234567890123456789012123456789012345678901
12345678901234567890123456789012123456789012345678901 2
12345678901234567890123456789012123456789012345678901
dangerously un- a zoo. He’s obviously fighting to get eggs in the 2
12345678901234567890123456789012123456789012345678901 2
12345678901234567890123456789012123456789012345678901
known Antarctic Antarctic. Picturing Antarctica in your mind, 2
1 2
12345678901234567890123456789012123456789012345678901 22
2345678901234567890123456789012123456789012345678901
__________” you might think the ice would fit well into the
12345678901234567890123456789012123456789012345678901 2
12345678901234567890123456789012123456789012345678901
sentence. It would, but so would land or
2
12345678901234567890123456789012123456789012345678901 2
1 ground. Keep both options in mind.
12345678901234567890123456789012123456789012345678901 22
2345678901234567890123456789012123456789012345678901
1 “in order to SC filler, but three cheers for completing the
12345678901234567890123456789012123456789012345678901 2
12345678901234567890123456789012123456789012345678901
complete their mission. 2
12345678901234567890123456789012123456789012345678901
2345678901234567890123456789012123456789012345678901
2
2
1 mission”
12345678901234567890123456789012123456789012345678901 2
12345678901234567890123456789012123456789012345678901 2
12345678901234567890123456789012123456789012345678901 2 153
123456789012345678901234567890121234567890123456789012
Part III: Section Review

123456789012345678901234567890121234567890123456789012
12345678901234567890123456789012123456789012345678901 2
12345678901234567890123456789012123456789012345678901 2
12345678901234567890123456789012123456789012345678901
Now that you have potential words for each blank, look at the answer 2
12345678901234567890123456789012123456789012345678901 2
12345678901234567890123456789012123456789012345678901
choices. Start with only one word, and eliminate any answer choices that 2
12345678901234567890123456789012123456789012345678901 2
12345678901234567890123456789012123456789012345678901
do not fit the words you came up with. Start with the first blank and cross 2
2345678901234567890123456789012123456789012345678901 2
1 out any words that do not mean “terribly” or “extremely.”
12345678901234567890123456789012123456789012345678901 2
12345678901234567890123456789012123456789012345678901 2
12345678901234567890123456789012123456789012345678901 2
12345678901234567890123456789012123456789012345678901
A. appallingly. .terrain
2345678901234567890123456789012123456789012345678901
2
2
1
12345678901234567890123456789012123456789012345678901
B. terrifyingly. .errors 2
12345678901234567890123456789012123456789012345678901 2
12345678901234567890123456789012123456789012345678901
C cryptically. .ice 2
12345678901234567890123456789012123456789012345678901
2345678901234567890123456789012123456789012345678901
2
2
1 D. bitingly. .forces
12345678901234567890123456789012123456789012345678901 2
12345678901234567890123456789012123456789012345678901
E. stingingly. .circumstances 2
12345678901234567890123456789012123456789012345678901 2
12345678901234567890123456789012123456789012345678901
Choice (C) can be eliminated, since “cryptically low temperatures” does 2
2345678901234567890123456789012123456789012345678901 2
1
12345678901234567890123456789012123456789012345678901
not make sense, and it does not mean the same thing as what you are 2
12345678901234567890123456789012123456789012345678901 2
12345678901234567890123456789012123456789012345678901
looking for. Note that the second word in choice (C), ice, is a very good 2
12345678901234567890123456789012123456789012345678901 2
12345678901234567890123456789012123456789012345678901
word to use for that blank. People shopping might pick choice (C) because 2
12345678901234567890123456789012123456789012345678901 2
12345678901234567890123456789012123456789012345678901
the second word fits so well, even though the first word is a poor choice. If 2
12345678901234567890123456789012123456789012345678901 2
12345678901234567890123456789012123456789012345678901
you work one blank at a time and eliminate improbable answer choices, 2
2345678901234567890123456789012123456789012345678901
12345678901234567890123456789012123456789012345678901 2
1 you spare yourself this mistake. 2
12345678901234567890123456789012123456789012345678901 2
12345678901234567890123456789012123456789012345678901 2
12345678901234567890123456789012123456789012345678901
For the second blank, ice works nicely but it’s already gone. The word 2
2345678901234567890123456789012123456789012345678901
errors does not work very well, so (B) can be crossed out. This leaves (A), 2
12345678901234567890123456789012123456789012345678901
12345678901234567890123456789012123456789012345678901 2
12345678901234567890123456789012123456789012345678901 2
12345678901234567890123456789012123456789012345678901 2
(D), and (E). The word terrain is synonymous with ground, making it the
best choice from the ones left. (A) is your answer, then. You didn’t start out 2
12345678901234567890123456789012123456789012345678901
12345678901234567890123456789012123456789012345678901 2
12345678901234567890123456789012123456789012345678901 2
12345678901234567890123456789012123456789012345678901 2
thinking of the words appallingly and terrain, but you understood the
12345678901234567890123456789012123456789012345678901 2
1 context clues well enough to choose words that led you to these correct 2
2345678901234567890123456789012123456789012345678901
12345678901234567890123456789012123456789012345678901
answers. 2
12345678901234567890123456789012123456789012345678901 2
1 The approach outlined above should be used on all SC questions, one 2
12345678901234567890123456789012123456789012345678901 2
12345678901234567890123456789012123456789012345678901 2
12345678901234567890123456789012123456789012345678901
blank or two. As always, vocabulary plays a factor, since it is difficult to 2
12345678901234567890123456789012123456789012345678901 2
12345678901234567890123456789012123456789012345678901
eliminate a word like cryptically if you are uncertain of what it means. 2
12345678901234567890123456789012123456789012345678901
Even though vocabulary works as a limiting factor on SC problems, the 2
12345678901234567890123456789012123456789012345678901 2
2345678901234567890123456789012123456789012345678901
fact remains that sentence completion questions are very “techniquable.” 2
12345678901234567890123456789012123456789012345678901
12345678901234567890123456789012123456789012345678901 2
1 If you can understand the sentence, you have a shot at getting the answer 2
12345678901234567890123456789012123456789012345678901 2
12345678901234567890123456789012123456789012345678901 2
12345678901234567890123456789012123456789012345678901
right. 2
12345678901234567890123456789012123456789012345678901 2
12345678901234567890123456789012123456789012345678901 2
12345678901234567890123456789012123456789012345678901
This is the best thing about SC problems. The worst thing is that there are 2
12345678901234567890123456789012123456789012345678901 2
12345678901234567890123456789012123456789012345678901 2
2345678901234567890123456789012123456789012345678901
usually fewer SC problems than any other question type. You might see as
few as five, and these arrive near the back end of the test where their 2
12345678901234567890123456789012123456789012345678901
12345678901234567890123456789012123456789012345678901 2
impact on scoring is lessened. Isn’t it strange how the question type that’s 2
12345678901234567890123456789012123456789012345678901
1 2
12345678901234567890123456789012123456789012345678901
most solvable appears the least, while the question type that’s little more 2
12345678901234567890123456789012123456789012345678901 2
12345678901234567890123456789012123456789012345678901
than a “do-you-know-the-word-or-not?” appears the most? Hmm. . . . 2
12345678901234567890123456789012123456789012345678901 2
12345678901234567890123456789012123456789012345678901 2
12345678901234567890123456789012123456789012345678901 2
2345678901234567890123456789012123456789012345678901
2
1
12345678901234567890123456789012123456789012345678901 2
12345678901234567890123456789012123456789012345678901 2
12345678901234567890123456789012123456789012345678901 2
12345678901234567890123456789012123456789012345678901 2
12345678901234567890123456789012123456789012345678901 2
12345678901234567890123456789012123456789012345678901 2 2
154 12345678901234567890123456789012123456789012345678901
123456789012345678901234567890121234567890123456789012

www.petersons.com
Chapter 5: Verbal Review

123456789012345678901234567890121234567890123456789012
12345678901234567890123456789012123456789012345678901 2
12345678901234567890123456789012123456789012345678901 2
12345678901234567890123456789012123456789012345678901
Reading Comprehension, the Kevin Bacon of 2
12345678901234567890123456789012123456789012345678901 2
12345678901234567890123456789012123456789012345678901 2
12345678901234567890123456789012123456789012345678901 2
Standardized Tests (It’s in Every Test)
12345678901234567890123456789012123456789012345678901 2
12345678901234567890123456789012123456789012345678901
Odds are high that you have already taken scads of tests containing 2
12345678901234567890123456789012123456789012345678901 2
12345678901234567890123456789012123456789012345678901
Reading Comprehension (RC) problems. The format, then, should hold 2
12345678901234567890123456789012123456789012345678901 2
12345678901234567890123456789012123456789012345678901
no surprise: You get a pile of words to read and must answer questions 2
1 about these words to show that you understood said pile. RC on the GRE 2
2345678901234567890123456789012123456789012345678901
12345678901234567890123456789012123456789012345678901 2
12345678901234567890123456789012123456789012345678901 2
12345678901234567890123456789012123456789012345678901
Verbal section is no different, although the twenty-first century techno- 2
12345678901234567890123456789012123456789012345678901
2345678901234567890123456789012123456789012345678901
2
2
1 version of RC means that you must do your reading on a computer screen,
12345678901234567890123456789012123456789012345678901 2
12345678901234567890123456789012123456789012345678901
scrolling up and down to read the words that appear in an annoyingly 2
12345678901234567890123456789012123456789012345678901 2
12345678901234567890123456789012123456789012345678901
small text box.
2345678901234567890123456789012123456789012345678901
2
2
1
12345678901234567890123456789012123456789012345678901 2
12345678901234567890123456789012123456789012345678901
Since the format is so familiar, there won’t be any surprises when your first 2
12345678901234567890123456789012123456789012345678901
RC questions pop up on the screen. These questions do not focus on 2
12345678901234567890123456789012123456789012345678901 2
2345678901234567890123456789012123456789012345678901
vocabulary as much as the other three question types, providing a bit of 2
12345678901234567890123456789012123456789012345678901
12345678901234567890123456789012123456789012345678901 2
relief in that arena. These two aspects work to your advantage, but not 2
12345678901234567890123456789012123456789012345678901
12345678901234567890123456789012123456789012345678901 22
12345678901234567890123456789012123456789012345678901
everything about RC is rosy. The main drawback is that you have to spend
12345678901234567890123456789012123456789012345678901 2
1 an inordinate amount of time—compared to the other GRE problem 2
12345678901234567890123456789012123456789012345678901 2
12345678901234567890123456789012123456789012345678901
types—to answer a small amount of questions. This can lead to problems 2
12345678901234567890123456789012123456789012345678901 2
12345678901234567890123456789012123456789012345678901
if you are a very slow reader, as you might find yourself taking 4–6 minutes 2
12345678901234567890123456789012123456789012345678901 2
12345678901234567890123456789012123456789012345678901
of a 30-minute test just to answer two measly questions. The ratio of what 2
12345678901234567890123456789012123456789012345678901 2
12345678901234567890123456789012123456789012345678901
you put in to answer RC questions to what you get out of it is really lousy. 2
12345678901234567890123456789012123456789012345678901 22
2345678901234567890123456789012123456789012345678901
12345678901234567890123456789012123456789012345678901
You should always take whatever time is needed to get a question right, 2
12345678901234567890123456789012123456789012345678901
12345678901234567890123456789012123456789012345678901 2
1 even if this means having to guess on some questions at the very end. Many 2
2345678901234567890123456789012123456789012345678901
people tend to rush when taking a standardized test, and this is something 2
12345678901234567890123456789012123456789012345678901
12345678901234567890123456789012123456789012345678901 2
1 you never want to do as it leads to careless errors. Careless errors are 2
12345678901234567890123456789012123456789012345678901 2
12345678901234567890123456789012123456789012345678901
compounded by the adaptive nature of the GRE, so people flying through 2
12345678901234567890123456789012123456789012345678901 2
12345678901234567890123456789012123456789012345678901
RC problems find themselves at an even greater disadvantage. To 2
12345678901234567890123456789012123456789012345678901 2
12345678901234567890123456789012123456789012345678901
overcome any pacing problems before they appear, simply understand that 2
12345678901234567890123456789012123456789012345678901 2
2345678901234567890123456789012123456789012345678901
you will have to take more time on RC problems, which means you should 2
12345678901234567890123456789012123456789012345678901
12345678901234567890123456789012123456789012345678901 2
1 take less time on some other problems. For example, if you definitely 2
12345678901234567890123456789012123456789012345678901 2
12345678901234567890123456789012123456789012345678901
know the answer to an antonym problem, it should only take you about 2
12345678901234567890123456789012123456789012345678901 2
12345678901234567890123456789012123456789012345678901
15 seconds to mark the correct answer. 2
12345678901234567890123456789012123456789012345678901 2
12345678901234567890123456789012123456789012345678901 2
12345678901234567890123456789012123456789012345678901 2
2345678901234567890123456789012123456789012345678901
12345678901234567890123456789012123456789012345678901
A typical RC passage runs about 200 words in length and has 2–3 2
12345678901234567890123456789012123456789012345678901 2
12345678901234567890123456789012123456789012345678901
questions associated with it. Longer passages—those over and around 2
Tip

12345678901234567890123456789012123456789012345678901 2
1 300 words—might have as many as 4 questions attached. On a normal 2
2345678901234567890123456789012123456789012345678901
12345678901234567890123456789012123456789012345678901 2
12345678901234567890123456789012123456789012345678901
GRE, RC questions are shy, appearing for the first time only after you 2
12345678901234567890123456789012123456789012345678901 2
12345678901234567890123456789012123456789012345678901
have answered about 8–10 other questions. 2
1 2
12345678901234567890123456789012123456789012345678901 22
2345678901234567890123456789012123456789012345678901
1 Along with their different format, RC problems require a different
12345678901234567890123456789012123456789012345678901 2
12345678901234567890123456789012123456789012345678901 2
12345678901234567890123456789012123456789012345678901
approach. The overall goal is to make sure you answer the questions 2
1 correctly in the shortest amount of time possible. To help you do this, read 2
2345678901234567890123456789012123456789012345678901
12345678901234567890123456789012123456789012345678901 2
12345678901234567890123456789012123456789012345678901 2
12345678901234567890123456789012123456789012345678901 2 155
123456789012345678901234567890121234567890123456789012
Part III: Section Review

123456789012345678901234567890121234567890123456789012
12345678901234567890123456789012123456789012345678901 2
12345678901234567890123456789012123456789012345678901 2
12345678901234567890123456789012123456789012345678901
the question before you start reading the passage. Keep the question in the 2
12345678901234567890123456789012123456789012345678901 2
12345678901234567890123456789012123456789012345678901
back of your mind when tackling the passage. If you can answer the 2
12345678901234567890123456789012123456789012345678901 2
12345678901234567890123456789012123456789012345678901
question accurately before reading the entire passage, by all means do so. 2
1 The whole goal is to answer questions, not while away time reading a 2
2345678901234567890123456789012123456789012345678901
12345678901234567890123456789012123456789012345678901 2
12345678901234567890123456789012123456789012345678901 2
12345678901234567890123456789012123456789012345678901
bland passage. 2
12345678901234567890123456789012123456789012345678901 2
1 The content of the passages is not going to electrify you. The passages are 2
2345678901234567890123456789012123456789012345678901
12345678901234567890123456789012123456789012345678901 2
12345678901234567890123456789012123456789012345678901 2
12345678901234567890123456789012123456789012345678901
non-fiction and tackle a diverse range of scientific and non-scientific 2
12345678901234567890123456789012123456789012345678901
2345678901234567890123456789012123456789012345678901
2
2
1 topics. It should be noted that all the information presented has to be
12345678901234567890123456789012123456789012345678901 2
12345678901234567890123456789012123456789012345678901
accurate, and that the answers to the questions must also be factually 2
12345678901234567890123456789012123456789012345678901
accurate. Therefore, if you get a passage about the War of the Austrian 2
12345678901234567890123456789012123456789012345678901 2
1 Succession and you just happen to be a big fan of the Hapsburg dynasty, 2
2345678901234567890123456789012123456789012345678901
12345678901234567890123456789012123456789012345678901 2
12345678901234567890123456789012123456789012345678901
then you’re in luck! On a more general note, you can use common sense to 2
12345678901234567890123456789012123456789012345678901 2
12345678901234567890123456789012123456789012345678901
eliminate some answer choices that are unlikely to be real or true. 2
12345678901234567890123456789012123456789012345678901 2
2345678901234567890123456789012123456789012345678901
12345678901234567890123456789012123456789012345678901
When you start reading the passages, try to understand the main point of 2
2
12345678901234567890123456789012123456789012345678901
12345678901234567890123456789012123456789012345678901
the passage first. This is often found hiding out in the first paragraph. Your 2
2
12345678901234567890123456789012123456789012345678901
12345678901234567890123456789012123456789012345678901 2
1 overall goal is to understand the main points of the passage and each of its 2
12345678901234567890123456789012123456789012345678901
paragraphs, but you don’t need to waste time absorbing all the 2
12345678901234567890123456789012123456789012345678901 2
12345678901234567890123456789012123456789012345678901 2
12345678901234567890123456789012123456789012345678901 2
2345678901234567890123456789012123456789012345678901
information into your long-term memory. Have an idea of where all the
facts are, since you want to be able to go to the right part of the passage to 2
12345678901234567890123456789012123456789012345678901
12345678901234567890123456789012123456789012345678901 2
12345678901234567890123456789012123456789012345678901 2
12345678901234567890123456789012123456789012345678901 2
answer any questions posed. At the same time, you don’t want to waste
any time reading information that’s not essential to answering the 2
12345678901234567890123456789012123456789012345678901
12345678901234567890123456789012123456789012345678901 2
12345678901234567890123456789012123456789012345678901 2
12345678901234567890123456789012123456789012345678901 2
questions, since answering questions is the only reason you’re here.
1 Finding the right balance between passage reading and question answering 2
12345678901234567890123456789012123456789012345678901 2
12345678901234567890123456789012123456789012345678901
takes some practice, but that’s why there are three pencil-and-paper tests 2
12345678901234567890123456789012123456789012345678901 2
12345678901234567890123456789012123456789012345678901
in this book as well as others on the CD. 2
12345678901234567890123456789012123456789012345678901
2345678901234567890123456789012123456789012345678901
2
2
12345678901234567890123456789012123456789012345678901
12345678901234567890123456789012123456789012345678901
Here’s a short sample paragraph: 2
1 2
12345678901234567890123456789012123456789012345678901 2
12345678901234567890123456789012123456789012345678901 2
12345678901234567890123456789012123456789012345678901 2
2345678901234567890123456789012123456789012345678901
Passage 1
12345678901234567890123456789012123456789012345678901 2 2
1
12345678901234567890123456789012123456789012345678901
Line Viewing the underwater world through a land-based perspec-
2
12345678901234567890123456789012123456789012345678901 2
12345678901234567890123456789012123456789012345678901 2
2345678901234567890123456789012123456789012345678901
tive, it is easy to consider a colony of coral—a broad term used
2
12345678901234567890123456789012123456789012345678901
to describe the marine organisms, like those of the genus 2
12345678901234567890123456789012123456789012345678901 2
12345678901234567890123456789012123456789012345678901
Corallium, which inhabit many tropical waters—as something
2
1
2345678901234567890123456789012123456789012345678901
similar to a bed of flowers. Both corals and flowers have many 2
12345678901234567890123456789012123456789012345678901
(5)
12345678901234567890123456789012123456789012345678901 2
12345678901234567890123456789012123456789012345678901
species that display brilliant markings and have intricate 2
12345678901234567890123456789012123456789012345678901 2
1 shapes pleasing to the human eye. Yet the relationship between 2
12345678901234567890123456789012123456789012345678901 2
12345678901234567890123456789012123456789012345678901
flowers and corals is predominantly an aesthetic one, as the 2
12345678901234567890123456789012123456789012345678901 2
12345678901234567890123456789012123456789012345678901
basic composition of a coral polyp, an animal, differs mark- 2
12345678901234567890123456789012123456789012345678901 2
12345678901234567890123456789012123456789012345678901 2
2345678901234567890123456789012123456789012345678901
(10) edly from that of a self-photosynthesizing plant like a rose or
2
1
12345678901234567890123456789012123456789012345678901
zinnia. 2
12345678901234567890123456789012123456789012345678901 2
12345678901234567890123456789012123456789012345678901
It could even be argued that
2345678901234567890123456789012123456789012345678901
lichen, a fungus, bears more in 2
2
1
12345678901234567890123456789012123456789012345678901
common with corals than flowers, since both lichen and corals 2
12345678901234567890123456789012123456789012345678901 2 2
156 12345678901234567890123456789012123456789012345678901
123456789012345678901234567890121234567890123456789012

www.petersons.com
Chapter 5: Verbal Review

123456789012345678901234567890121234567890123456789012
12345678901234567890123456789012123456789012345678901 2
12345678901234567890123456789012123456789012345678901 2
12345678901234567890123456789012123456789012345678901
are typified by beneficial relationships with photosynthesizing 2
12345678901234567890123456789012123456789012345678901 2
12345678901234567890123456789012123456789012345678901
(15) algae. While there are some species of coral that can exist 2
12345678901234567890123456789012123456789012345678901 2
12345678901234567890123456789012123456789012345678901
solely by capturing food, most
2345678901234567890123456789012123456789012345678901
corals receive energy by hosting 2
2
1
12345678901234567890123456789012123456789012345678901
millions of single-celled organisms commonly referred to as 2
12345678901234567890123456789012123456789012345678901 2
12345678901234567890123456789012123456789012345678901
zooxanthellae. The zooxanthellae are provided safety from 2
12345678901234567890123456789012123456789012345678901
2345678901234567890123456789012123456789012345678901
2
2
1 predation as well as certain by-products of the coral’s
12345678901234567890123456789012123456789012345678901 2
12345678901234567890123456789012123456789012345678901
(20) metabolism, like ammonia, which zooxanthellae require to 2
12345678901234567890123456789012123456789012345678901 2
12345678901234567890123456789012123456789012345678901
grow and reproduce. In return,
2345678901234567890123456789012123456789012345678901
a portion of the energy 2
2
1
12345678901234567890123456789012123456789012345678901
produced by the zooxanthellae through photosynthesis is used 2
12345678901234567890123456789012123456789012345678901 2
12345678901234567890123456789012123456789012345678901
by the coral to further its own needs for sustenance, growth, 2
12345678901234567890123456789012123456789012345678901
2345678901234567890123456789012123456789012345678901
2
2
1 and reproduction.
12345678901234567890123456789012123456789012345678901 2
12345678901234567890123456789012123456789012345678901
The text is wordy, the sentences are long, the style is bookish, and the topic 2
12345678901234567890123456789012123456789012345678901 2
12345678901234567890123456789012123456789012345678901
somewhat dull. In short, it’s your typical RC passage on a standardized 2
12345678901234567890123456789012123456789012345678901 22
2345678901234567890123456789012123456789012345678901
12345678901234567890123456789012123456789012345678901
test. Instead of trying to gain some deep insight from the text, just get an
12345678901234567890123456789012123456789012345678901 2
12345678901234567890123456789012123456789012345678901
idea of what each paragraph is about and then dive into the questions. 2
12345678901234567890123456789012123456789012345678901 22
12345678901234567890123456789012123456789012345678901 2
1
12345678901234567890123456789012123456789012345678901
The author of the passage uses the visual comparison of coral to a 2
12345678901234567890123456789012123456789012345678901 2
12345678901234567890123456789012123456789012345678901
bed of flowers in order to illustrate 2
12345678901234567890123456789012123456789012345678901 22
2345678901234567890123456789012123456789012345678901
12345678901234567890123456789012123456789012345678901
12345678901234567890123456789012123456789012345678901
A. how human perceptions of other habitats are influenced by 2
12345678901234567890123456789012123456789012345678901
their own environment. 2
12345678901234567890123456789012123456789012345678901 22
12345678901234567890123456789012123456789012345678901
B. the zoological similarities that exist between corals and
12345678901234567890123456789012123456789012345678901 2
12345678901234567890123456789012123456789012345678901
flowers. 2
12345678901234567890123456789012123456789012345678901 22
1 C. why a better understanding of corals is needed in order to
12345678901234567890123456789012123456789012345678901 2
12345678901234567890123456789012123456789012345678901
preserve endangered forms. 2
12345678901234567890123456789012123456789012345678901 2
12345678901234567890123456789012123456789012345678901
D. the multiple ways that the perspective organisms are regarded. 2
12345678901234567890123456789012123456789012345678901 2
12345678901234567890123456789012123456789012345678901 22
2345678901234567890123456789012123456789012345678901
E. the ability of underwater life to mimic that of the land-based
12345678901234567890123456789012123456789012345678901 2
1 world.
12345678901234567890123456789012123456789012345678901 2
12345678901234567890123456789012123456789012345678901 2
12345678901234567890123456789012123456789012345678901 22
2345678901234567890123456789012123456789012345678901
12345678901234567890123456789012123456789012345678901
Note

If you were really cagy, you would have zipped down and read this
1 question before tackling the text of the passage. It doesn’t matter if you 2
12345678901234567890123456789012123456789012345678901 2
12345678901234567890123456789012123456789012345678901 2
12345678901234567890123456789012123456789012345678901
didn’t do this for this question, but make sure to get in the habit of 2
12345678901234567890123456789012123456789012345678901 2
12345678901234567890123456789012123456789012345678901
reviewing the questions before tackling the text. 2
12345678901234567890123456789012123456789012345678901 2
12345678901234567890123456789012123456789012345678901 2
2345678901234567890123456789012123456789012345678901
12345678901234567890123456789012123456789012345678901 2
This question covers a bit of text in the first paragraph, so don’t try to 2
12345678901234567890123456789012123456789012345678901
12345678901234567890123456789012123456789012345678901
answer the question without looking back at the first paragraph. There’s a 2
2
12345678901234567890123456789012123456789012345678901
1 tendency for students who feel rushed to try to answer a problem “from 2
12345678901234567890123456789012123456789012345678901 2
12345678901234567890123456789012123456789012345678901 2
12345678901234567890123456789012123456789012345678901
memory,” as if great amounts of time are saved by not referring back to the 2
12345678901234567890123456789012123456789012345678901
passage. Some seconds might be shaved off, it’s true, but these seconds 2
12345678901234567890123456789012123456789012345678901 2
2345678901234567890123456789012123456789012345678901
12345678901234567890123456789012123456789012345678901 2
1 won’t help much if you get the problem wrong. Go back and look over the 2
12345678901234567890123456789012123456789012345678901
first paragraph to give yourself the best shot possible to find the correct 2
12345678901234567890123456789012123456789012345678901 2
12345678901234567890123456789012123456789012345678901
2345678901234567890123456789012123456789012345678901
2
2
1 answer.
12345678901234567890123456789012123456789012345678901 2
12345678901234567890123456789012123456789012345678901 2
12345678901234567890123456789012123456789012345678901 2 157
123456789012345678901234567890121234567890123456789012
Part III: Section Review

123456789012345678901234567890121234567890123456789012
12345678901234567890123456789012123456789012345678901 2
12345678901234567890123456789012123456789012345678901 2
12345678901234567890123456789012123456789012345678901
Instead of jumping right to the correct answer, let’s review why some of the 2
12345678901234567890123456789012123456789012345678901 2
12345678901234567890123456789012123456789012345678901
other choices are not the right one. Choice (C) sounds good, and people 2
12345678901234567890123456789012123456789012345678901 2
12345678901234567890123456789012123456789012345678901
who are very passionate about the environment might agree with the 2
1 statement whole-heartedly. Yet the author of the passage never once 2
2345678901234567890123456789012123456789012345678901
12345678901234567890123456789012123456789012345678901 2
12345678901234567890123456789012123456789012345678901 2
12345678901234567890123456789012123456789012345678901
mentions the environmental angle, so the answer choice is just there to trip 2
12345678901234567890123456789012123456789012345678901
2345678901234567890123456789012123456789012345678901
2
2
1 up GRE-taking members of Greenpeace. Think of this as the “True, but So
12345678901234567890123456789012123456789012345678901 2
12345678901234567890123456789012123456789012345678901
What?” wrong answer choice. There will be answer choices that sound 2
12345678901234567890123456789012123456789012345678901 2
12345678901234567890123456789012123456789012345678901
really good but have nothing to do with the
2345678901234567890123456789012123456789012345678901
actual text you are working 2
2
1 with. Cross these out.
12345678901234567890123456789012123456789012345678901 2
12345678901234567890123456789012123456789012345678901 2
12345678901234567890123456789012123456789012345678901
Choices (B) and (E) both use words and phrases that can be found in the 2
12345678901234567890123456789012123456789012345678901 2
1 passage itself. Choice (B) has the words corals, flower, similarities, and the 2
2345678901234567890123456789012123456789012345678901
12345678901234567890123456789012123456789012345678901 2
12345678901234567890123456789012123456789012345678901
prefix zoo-, all of which appear in the passage. Choice (E) has underwater 2
12345678901234567890123456789012123456789012345678901 2
12345678901234567890123456789012123456789012345678901
and land-based set up in a nice, contrasting way. Both of these choices 2
12345678901234567890123456789012123456789012345678901 2
2345678901234567890123456789012123456789012345678901
offer looks over substance, since the actual meaning of each sentence 2
12345678901234567890123456789012123456789012345678901
12345678901234567890123456789012123456789012345678901
makes them incorrect answer choices. They are like a bracelet made out of 2
2
12345678901234567890123456789012123456789012345678901
12345678901234567890123456789012123456789012345678901
pearls and whipped cream. Even though the pearls look nice, the bracelet 2
2
12345678901234567890123456789012123456789012345678901
1 itself cannot be worn. Choice (B) actually states that there are deeper 2
12345678901234567890123456789012123456789012345678901 2
12345678901234567890123456789012123456789012345678901 2
12345678901234567890123456789012123456789012345678901
similarities between flowers and corals, when the whole paragraph is 2
2345678901234567890123456789012123456789012345678901
about how these similarities are superficial. The verb mimic in choice (E) 2
12345678901234567890123456789012123456789012345678901
12345678901234567890123456789012123456789012345678901 2
12345678901234567890123456789012123456789012345678901 2
12345678901234567890123456789012123456789012345678901 2
makes this statement totally incorrect, since there is nothing in the passage
12345678901234567890123456789012123456789012345678901 2
12345678901234567890123456789012123456789012345678901
to back up this claim. 2
12345678901234567890123456789012123456789012345678901 2
This leaves only two choices, (A) and (D). Quite often, the correct answer 2
12345678901234567890123456789012123456789012345678901
12345678901234567890123456789012123456789012345678901 2
1 for an RC question merely paraphrases, or restates, text from the passage 2
12345678901234567890123456789012123456789012345678901 2
2345678901234567890123456789012123456789012345678901
12345678901234567890123456789012123456789012345678901 2
1 using different words. The passage starts by showing how a land-based 2
12345678901234567890123456789012123456789012345678901
perspective can lead to the wrong conclusion about corals. In other words, 2
12345678901234567890123456789012123456789012345678901
2345678901234567890123456789012123456789012345678901
2
people’s perceptions of other habitats (the ocean) are influenced by their 2
12345678901234567890123456789012123456789012345678901
12345678901234567890123456789012123456789012345678901 2
1 own environment, the land. That’s choice (A), and that’s your paraphrased 2
12345678901234567890123456789012123456789012345678901 2
12345678901234567890123456789012123456789012345678901
correct answer. 2
12345678901234567890123456789012123456789012345678901 2
2345678901234567890123456789012123456789012345678901
12345678901234567890123456789012123456789012345678901 2 2
1
12345678901234567890123456789012123456789012345678901
From the second paragraph, it can be inferred that 2
12345678901234567890123456789012123456789012345678901 2
2345678901234567890123456789012123456789012345678901
12345678901234567890123456789012123456789012345678901 2
12345678901234567890123456789012123456789012345678901
A. lichen use zooxanthellae algae for use in photosynthesis. 2
12345678901234567890123456789012123456789012345678901 2
12345678901234567890123456789012123456789012345678901 2
B. lichen provides some form of photosynthesizing algae with the
2
1
12345678901234567890123456789012123456789012345678901
same basic nutrients that corals provide zooxanthellae. 2
12345678901234567890123456789012123456789012345678901
C. zooxanthellae serve a function with corals that is served by 2
12345678901234567890123456789012123456789012345678901 2
12345678901234567890123456789012123456789012345678901
some photosynthesizing organism in lichen. 2
12345678901234567890123456789012123456789012345678901 2
2345678901234567890123456789012123456789012345678901
D. corals are not as dependent on their photosynthesizing partners 2
12345678901234567890123456789012123456789012345678901
12345678901234567890123456789012123456789012345678901 2
12345678901234567890123456789012123456789012345678901
as lichen is. 2
12345678901234567890123456789012123456789012345678901 2
1 E. corals and lichen are both able to capture food for themselves 2
2345678901234567890123456789012123456789012345678901
12345678901234567890123456789012123456789012345678901 2
1 if necessary. 2
12345678901234567890123456789012123456789012345678901 2
12345678901234567890123456789012123456789012345678901 2
12345678901234567890123456789012123456789012345678901 2
12345678901234567890123456789012123456789012345678901 2
12345678901234567890123456789012123456789012345678901 2
12345678901234567890123456789012123456789012345678901 2 2
158 12345678901234567890123456789012123456789012345678901
123456789012345678901234567890121234567890123456789012

www.petersons.com
Chapter 5: Verbal Review

123456789012345678901234567890121234567890123456789012
12345678901234567890123456789012123456789012345678901 2
12345678901234567890123456789012123456789012345678901 2
12345678901234567890123456789012123456789012345678901 2
12345678901234567890123456789012123456789012345678901 2
12345678901234567890123456789012123456789012345678901
Most RC problems are not going to ask you to find information 2
12345678901234567890123456789012123456789012345678901 2
12345678901234567890123456789012123456789012345678901
specifically stated in the passage. Instead,
2345678901234567890123456789012123456789012345678901
you will be asked to infer or 2
2
1 conclude an answer, based on what was written. This means that while
12345678901234567890123456789012123456789012345678901 2
Tip 12345678901234567890123456789012123456789012345678901
the correct answer is not stated directly, the clues needed to find the right
12345678901234567890123456789012123456789012345678901
2
2
12345678901234567890123456789012123456789012345678901
answer are there. So, think like you did on the SC problems and search the
2345678901234567890123456789012123456789012345678901
2
2
1
12345678901234567890123456789012123456789012345678901
passage text for the clues that lead you to the right answer. The clues will 2
12345678901234567890123456789012123456789012345678901 2
12345678901234567890123456789012123456789012345678901
always be there; you need only find them and they will lead you to the 2
12345678901234567890123456789012123456789012345678901
2345678901234567890123456789012123456789012345678901
2
2
1 correct answer.
12345678901234567890123456789012123456789012345678901 2
12345678901234567890123456789012123456789012345678901 2
12345678901234567890123456789012123456789012345678901 2
12345678901234567890123456789012123456789012345678901
If this question seems confusing, it’s because it is. Many RC problems and 2
1 answer choices appear to have been penned by Theodore Dreiser. (For 2
2345678901234567890123456789012123456789012345678901
12345678901234567890123456789012123456789012345678901 2
12345678901234567890123456789012123456789012345678901 2
12345678901234567890123456789012123456789012345678901
those non-English majors, this joke means that they are very laboriously 2
12345678901234567890123456789012123456789012345678901 2
12345678901234567890123456789012123456789012345678901
written.) The whole point of these choices is to get you unnerved to the 2
12345678901234567890123456789012123456789012345678901
extent that you hurry and make a poor decision. If you understand that 2
12345678901234567890123456789012123456789012345678901 2
12345678901234567890123456789012123456789012345678901 2
12345678901234567890123456789012123456789012345678901
this is the trap, you can avoid it. 2
12345678901234567890123456789012123456789012345678901 22
2345678901234567890123456789012123456789012345678901
1 First, you know where to look for the right answer, since the question
12345678901234567890123456789012123456789012345678901 2
12345678901234567890123456789012123456789012345678901 2
12345678901234567890123456789012123456789012345678901
states, “in the second passage.” Accept any free information that’s given. 2
2345678901234567890123456789012123456789012345678901
The answer choices continue to make various claims about the analogous— 2
12345678901234567890123456789012123456789012345678901
12345678901234567890123456789012123456789012345678901 2
12345678901234567890123456789012123456789012345678901
remember this word?—relationship between corals and zooxanthellae and 2
12345678901234567890123456789012123456789012345678901 22
12345678901234567890123456789012123456789012345678901
lichen with some photosynthesizing organism. In other words,
12345678901234567890123456789012123456789012345678901 2
12345678901234567890123456789012123456789012345678901
CORAL : ZOOXANTHELLAE :: LICHEN : Some unnamed 2
2
12345678901234567890123456789012123456789012345678901
12345678901234567890123456789012123456789012345678901 2
1 photosynthesizing organism 2
12345678901234567890123456789012123456789012345678901 22
2345678901234567890123456789012123456789012345678901
12345678901234567890123456789012123456789012345678901 2
1 You can speak GRE, too!
12345678901234567890123456789012123456789012345678901 2
12345678901234567890123456789012123456789012345678901
Choices (B), (D), and (E) don’t work because you don’t know enough 2
12345678901234567890123456789012123456789012345678901 2
12345678901234567890123456789012123456789012345678901 2
12345678901234567890123456789012123456789012345678901 2
about lichen. There’s nothing in the passage that says how dependent
12345678901234567890123456789012123456789012345678901
lichen is, choice (D), or whether lichen can capture its own food, choice 2
12345678901234567890123456789012123456789012345678901 2
2345678901234567890123456789012123456789012345678901
(E). These answers wouldn’t be inferences, they would be unfounded 2
12345678901234567890123456789012123456789012345678901
12345678901234567890123456789012123456789012345678901 2
1 guesses. Furthermore, you don’t know if lichen uses zooxanthellae 2
12345678901234567890123456789012123456789012345678901 2
12345678901234567890123456789012123456789012345678901
algae—an unlikely choice (A) since lichen is not found underwater—or if 2
12345678901234567890123456789012123456789012345678901 2
12345678901234567890123456789012123456789012345678901
lichen provides its guest algae with basic nutrients, choice (B). All you 2
12345678901234567890123456789012123456789012345678901 2
12345678901234567890123456789012123456789012345678901
know is that there is an analogy between lichen and its photosynthesizing 2
12345678901234567890123456789012123456789012345678901 2
12345678901234567890123456789012123456789012345678901
organism and corals with zooxanthellae. The correct answer is (C). 2
12345678901234567890123456789012123456789012345678901 2
12345678901234567890123456789012123456789012345678901 2
12345678901234567890123456789012123456789012345678901
One last note: People with saltwater aquariums might have known about 2
12345678901234567890123456789012123456789012345678901 2
12345678901234567890123456789012123456789012345678901 22
2345678901234567890123456789012123456789012345678901
corals and their little sun-loving buddies, showing how outside knowledge
12345678901234567890123456789012123456789012345678901 2
12345678901234567890123456789012123456789012345678901
can come in handy depending on the subject you get.
12345678901234567890123456789012123456789012345678901 22
1
12345678901234567890123456789012123456789012345678901 2
12345678901234567890123456789012123456789012345678901 2
12345678901234567890123456789012123456789012345678901 2
12345678901234567890123456789012123456789012345678901 2
12345678901234567890123456789012123456789012345678901 2
12345678901234567890123456789012123456789012345678901 2
12345678901234567890123456789012123456789012345678901 2
12345678901234567890123456789012123456789012345678901 2
12345678901234567890123456789012123456789012345678901 2 159
123456789012345678901234567890121234567890123456789012
Part III: Section Review

123456789012345678901234567890121234567890123456789012
12345678901234567890123456789012123456789012345678901 2
12345678901234567890123456789012123456789012345678901 2
12345678901234567890123456789012123456789012345678901
One Last Note about Vocabulary 2
12345678901234567890123456789012123456789012345678901 2
12345678901234567890123456789012123456789012345678901 2
12345678901234567890123456789012123456789012345678901 2
And that note is, study more vocabulary! All the techniques discussed
12345678901234567890123456789012123456789012345678901
above are useful, but if you don’t know the words, their effectiveness will 2
12345678901234567890123456789012123456789012345678901 2
12345678901234567890123456789012123456789012345678901 2
12345678901234567890123456789012123456789012345678901
be greatly reduced. The other advantage to studying vocabulary is that you 2
12345678901234567890123456789012123456789012345678901 2
12345678901234567890123456789012123456789012345678901
can use these advanced words to make your Analytical essays sound more 2
1 urbane and educated. For more advice on how to improve your essays, 2
2345678901234567890123456789012123456789012345678901
12345678901234567890123456789012123456789012345678901 2
12345678901234567890123456789012123456789012345678901 2
12345678901234567890123456789012123456789012345678901
turn to Chapter 6 and start reading. 2
12345678901234567890123456789012123456789012345678901
2345678901234567890123456789012123456789012345678901
2
2
1
12345678901234567890123456789012123456789012345678901 2
12345678901234567890123456789012123456789012345678901 2
12345678901234567890123456789012123456789012345678901 2
12345678901234567890123456789012123456789012345678901
2345678901234567890123456789012123456789012345678901
2
2
1
12345678901234567890123456789012123456789012345678901 2
12345678901234567890123456789012123456789012345678901 2
12345678901234567890123456789012123456789012345678901 2
12345678901234567890123456789012123456789012345678901 2
12345678901234567890123456789012123456789012345678901 2
2345678901234567890123456789012123456789012345678901
12345678901234567890123456789012123456789012345678901 2
12345678901234567890123456789012123456789012345678901 2
12345678901234567890123456789012123456789012345678901 2
12345678901234567890123456789012123456789012345678901 2
12345678901234567890123456789012123456789012345678901 2 2
1
12345678901234567890123456789012123456789012345678901 2
12345678901234567890123456789012123456789012345678901 2
12345678901234567890123456789012123456789012345678901 2
12345678901234567890123456789012123456789012345678901 2
2345678901234567890123456789012123456789012345678901
12345678901234567890123456789012123456789012345678901 2
12345678901234567890123456789012123456789012345678901 2
12345678901234567890123456789012123456789012345678901 2
12345678901234567890123456789012123456789012345678901 2
12345678901234567890123456789012123456789012345678901 2
12345678901234567890123456789012123456789012345678901 2
12345678901234567890123456789012123456789012345678901 2
12345678901234567890123456789012123456789012345678901 2 2
1
12345678901234567890123456789012123456789012345678901 2
12345678901234567890123456789012123456789012345678901 2
12345678901234567890123456789012123456789012345678901 2
12345678901234567890123456789012123456789012345678901 2
12345678901234567890123456789012123456789012345678901 2
12345678901234567890123456789012123456789012345678901 2
12345678901234567890123456789012123456789012345678901 2
12345678901234567890123456789012123456789012345678901 2
12345678901234567890123456789012123456789012345678901 2
12345678901234567890123456789012123456789012345678901 2
12345678901234567890123456789012123456789012345678901 2
2345678901234567890123456789012123456789012345678901
12345678901234567890123456789012123456789012345678901 2 2
1
12345678901234567890123456789012123456789012345678901 2
12345678901234567890123456789012123456789012345678901 2
12345678901234567890123456789012123456789012345678901 2
12345678901234567890123456789012123456789012345678901 2
12345678901234567890123456789012123456789012345678901 2
12345678901234567890123456789012123456789012345678901 2
12345678901234567890123456789012123456789012345678901 2
12345678901234567890123456789012123456789012345678901 2
2345678901234567890123456789012123456789012345678901
12345678901234567890123456789012123456789012345678901 2
12345678901234567890123456789012123456789012345678901 2
12345678901234567890123456789012123456789012345678901 2 2
1
12345678901234567890123456789012123456789012345678901 2
12345678901234567890123456789012123456789012345678901 2
12345678901234567890123456789012123456789012345678901 2
12345678901234567890123456789012123456789012345678901 2
12345678901234567890123456789012123456789012345678901 2
12345678901234567890123456789012123456789012345678901 2
2345678901234567890123456789012123456789012345678901
2
1
12345678901234567890123456789012123456789012345678901 2
12345678901234567890123456789012123456789012345678901 2
12345678901234567890123456789012123456789012345678901 2
12345678901234567890123456789012123456789012345678901 2
12345678901234567890123456789012123456789012345678901 2
12345678901234567890123456789012123456789012345678901 2 2
160 12345678901234567890123456789012123456789012345678901
123456789012345678901234567890121234567890123456789012

www.petersons.com
Take It
to the Next Level
123456789012345678901234567890121234567890123456789012
12345678901234567890123456789012123456789012345678901 2
12345678901234567890123456789012123456789012345678901
2345678901234567890123456789012123456789012345678901
2
2
1
Here There Be Wyverns
12345678901234567890123456789012123456789012345678901 2
12345678901234567890123456789012123456789012345678901 2
12345678901234567890123456789012123456789012345678901
(and other Difficult Words) 2
12345678901234567890123456789012123456789012345678901 2
12345678901234567890123456789012123456789012345678901 22
2345678901234567890123456789012123456789012345678901
Graduate programs use GRE scores in a bewildering variety of ways, but
12345678901234567890123456789012123456789012345678901
in the end it all basically comes down to making piles. Graduate programs 2
12345678901234567890123456789012123456789012345678901
12345678901234567890123456789012123456789012345678901
start out with a huge pile of applicants, and they need to do things to make 2
2
12345678901234567890123456789012123456789012345678901
12345678901234567890123456789012123456789012345678901 2
1 this big pile smaller. The big pile gets whittled down one way or another, 2
12345678901234567890123456789012123456789012345678901
and when it gets down to a manageable size, then people start to get 2
12345678901234567890123456789012123456789012345678901 2
12345678901234567890123456789012123456789012345678901 2
12345678901234567890123456789012123456789012345678901 22
2345678901234567890123456789012123456789012345678901
seriously considered.
12345678901234567890123456789012123456789012345678901 2
12345678901234567890123456789012123456789012345678901
Your GRE score is one easy, easy way to whittle down a pile. Viewed this 2
12345678901234567890123456789012123456789012345678901
way, you can see that your goal is to have a GRE score good enough to 2
12345678901234567890123456789012123456789012345678901
12345678901234567890123456789012123456789012345678901 2
12345678901234567890123456789012123456789012345678901 2
12345678901234567890123456789012123456789012345678901 22
keep from getting placed on the wrong pile. Some programs employ the
12345678901234567890123456789012123456789012345678901
1 combined Quantitative and Verbal scores, which is good for you because it 2
12345678901234567890123456789012123456789012345678901
is easier to gain points on the math side than it is on the verbal side. If you 2
12345678901234567890123456789012123456789012345678901 2
12345678901234567890123456789012123456789012345678901
wanted to boost your score by about 100 points, you could shoot for a 2
12345678901234567890123456789012123456789012345678901 2
12345678901234567890123456789012123456789012345678901
70-point improvement in math and a 30-point increase in verbal. 2
12345678901234567890123456789012123456789012345678901 2
12345678901234567890123456789012123456789012345678901 2
12345678901234567890123456789012123456789012345678901 2
12345678901234567890123456789012123456789012345678901
A higher number of humanities programs require the GRE, so the major 2
12345678901234567890123456789012123456789012345678901 2
Alert!

2345678901234567890123456789012123456789012345678901
12345678901234567890123456789012123456789012345678901
proportion of people taking the test are better at the verbal portion than 2
12345678901234567890123456789012123456789012345678901 2
1 the math portion. This verbal-centric skew makes it harder to get a high 2
12345678901234567890123456789012123456789012345678901 2
12345678901234567890123456789012123456789012345678901
verbal score, since relatively speaking most people are doing better on 2
2345678901234567890123456789012123456789012345678901
12345678901234567890123456789012123456789012345678901 2
12345678901234567890123456789012123456789012345678901
that section than on the math section. 2
12345678901234567890123456789012123456789012345678901 22
12345678901234567890123456789012123456789012345678901 2
1 Other graduate programs—like those with the word “English” in
12345678901234567890123456789012123456789012345678901 2
12345678901234567890123456789012123456789012345678901 2
12345678901234567890123456789012123456789012345678901
them—are often not concerned about your Quantitative score. These 2
12345678901234567890123456789012123456789012345678901 2
12345678901234567890123456789012123456789012345678901
programs are going to whittle down their piles based on Verbal and 2
2345678901234567890123456789012123456789012345678901
perhaps Analytical scores. If this is the case for you, then gunning for a 2
12345678901234567890123456789012123456789012345678901
12345678901234567890123456789012123456789012345678901 2
12345678901234567890123456789012123456789012345678901 2
12345678901234567890123456789012123456789012345678901 22
high Verbal score is the right plan. However, if the program is more
1 concerned with overall score, keep in mind that it will be easier to gain
12345678901234567890123456789012123456789012345678901 2
12345678901234567890123456789012123456789012345678901 2
12345678901234567890123456789012123456789012345678901
extra points in the math portion. 2
12345678901234567890123456789012123456789012345678901 2
12345678901234567890123456789012123456789012345678901 2
12345678901234567890123456789012123456789012345678901 2
12345678901234567890123456789012123456789012345678901 2
12345678901234567890123456789012123456789012345678901 2
12345678901234567890123456789012123456789012345678901 2
123456789012345678901234567890121234567890123456789012
161
Part III: Section Review

123456789012345678901234567890121234567890123456789012
12345678901234567890123456789012123456789012345678901 2
12345678901234567890123456789012123456789012345678901 2
12345678901234567890123456789012123456789012345678901
Getting a high score in the GRE Verbal section—high 600s to low 700s—is 2
12345678901234567890123456789012123456789012345678901 2
12345678901234567890123456789012123456789012345678901
not very simple, since it often boils down to one word. To be more precise, 2
12345678901234567890123456789012123456789012345678901 2
12345678901234567890123456789012123456789012345678901
it comes down to one word that means all words: vocabulary.
2345678901234567890123456789012123456789012345678901
2
2
1
12345678901234567890123456789012123456789012345678901 2
12345678901234567890123456789012123456789012345678901 2
12345678901234567890123456789012123456789012345678901 2
A 112345678901234567890123456789012123456789012345678901
Simple Formula to Make Medium Verbal
2345678901234567890123456789012123456789012345678901
2
2
12345678901234567890123456789012123456789012345678901 2
12345678901234567890123456789012123456789012345678901
Problems Harder: Use Tougher Words 2
12345678901234567890123456789012123456789012345678901 2
12345678901234567890123456789012123456789012345678901
There are no different question types to separate medium problems from 2
2345678901234567890123456789012123456789012345678901 2
1
12345678901234567890123456789012123456789012345678901
hard ones. Antonyms, analogies, and sentence completions will all have 2
12345678901234567890123456789012123456789012345678901 2
12345678901234567890123456789012123456789012345678901
the same format, but instead of the antonym BULKY you’ll have the word 2
12345678901234567890123456789012123456789012345678901 2
12345678901234567890123456789012123456789012345678901
CORPULENT. Both words mean the same thing, but corpulent is a more 2
12345678901234567890123456789012123456789012345678901 2
12345678901234567890123456789012123456789012345678901
difficult word. Analogies will still be formed using the common 2
12345678901234567890123456789012123456789012345678901 2
12345678901234567890123456789012123456789012345678901
relationships described on page 144, but once again the words used will be 2
2345678901234567890123456789012123456789012345678901
12345678901234567890123456789012123456789012345678901
tougher and not as commonly known. Ditto for sentence completions, as 2
2
12345678901234567890123456789012123456789012345678901
answer choices will come from the same stockpile of advanced vocabulary. 2
12345678901234567890123456789012123456789012345678901
12345678901234567890123456789012123456789012345678901 2
12345678901234567890123456789012123456789012345678901 2
12345678901234567890123456789012123456789012345678901 2
The emphasis on tougher words will be slightest for reading comprehen-
1 sion, but even there passages may feature a large amount of technical and 2
12345678901234567890123456789012123456789012345678901 2
12345678901234567890123456789012123456789012345678901 2
12345678901234567890123456789012123456789012345678901
scientific jargon within the text. It will be like the coral/zooxanthellae 2
2345678901234567890123456789012123456789012345678901
passage on pages 156–157, but with even more terms such as “photosyn- 2
12345678901234567890123456789012123456789012345678901
12345678901234567890123456789012123456789012345678901 2
12345678901234567890123456789012123456789012345678901 2
12345678901234567890123456789012123456789012345678901 2
thesizing organism” bandied about.
12345678901234567890123456789012123456789012345678901 2
The adaptive format, and the manner in which the GRE Verbal starts, only 2
12345678901234567890123456789012123456789012345678901
12345678901234567890123456789012123456789012345678901 2
exacerbates the problem. The first 3–6 problems you’ll face will be either 2
12345678901234567890123456789012123456789012345678901
12345678901234567890123456789012123456789012345678901 2
1 antonyms or analogies, and if you’re shooting for a high score it’s going to 2
12345678901234567890123456789012123456789012345678901 2
12345678901234567890123456789012123456789012345678901
be important to get most of these questions right. Since these questions are 2
12345678901234567890123456789012123456789012345678901 2
12345678901234567890123456789012123456789012345678901
little more than vocabulary problems, knowing the words used on these 2
12345678901234567890123456789012123456789012345678901 2
12345678901234567890123456789012123456789012345678901
problems is going to take a combination of knowledge and luck. The 2
12345678901234567890123456789012123456789012345678901 2
12345678901234567890123456789012123456789012345678901
knowledge will come in the form of a good lexicon, and the luck will come 2
12345678901234567890123456789012123456789012345678901 2
12345678901234567890123456789012123456789012345678901
since not everybody knows the definition of every word. 2
12345678901234567890123456789012123456789012345678901 2
2345678901234567890123456789012123456789012345678901
12345678901234567890123456789012123456789012345678901 2 2
1
12345678901234567890123456789012123456789012345678901 2
Note

12345678901234567890123456789012123456789012345678901 2
12345678901234567890123456789012123456789012345678901
Notice the term exacerbates is used instead of increases. To help your 2
12345678901234567890123456789012123456789012345678901 2
12345678901234567890123456789012123456789012345678901
score, it makes sense to jump on the GRE Tough Words bandwagon. 2
12345678901234567890123456789012123456789012345678901 2
12345678901234567890123456789012123456789012345678901 2
12345678901234567890123456789012123456789012345678901 2
2345678901234567890123456789012123456789012345678901
12345678901234567890123456789012123456789012345678901 2 2
12345678901234567890123456789012123456789012345678901
Since vocabulary is the key to getting a high Verbal score, the bulk of this 2
12345678901234567890123456789012123456789012345678901
1 chapter will focus on expanding the list of tough words you know. Before 2
12345678901234567890123456789012123456789012345678901 2
12345678901234567890123456789012123456789012345678901 2
12345678901234567890123456789012123456789012345678901
this occurs, though, a brief note about the various question types will be 2
12345678901234567890123456789012123456789012345678901 2
12345678901234567890123456789012123456789012345678901
covered. 2
12345678901234567890123456789012123456789012345678901 2
2345678901234567890123456789012123456789012345678901
2
1
12345678901234567890123456789012123456789012345678901 2
12345678901234567890123456789012123456789012345678901 2
12345678901234567890123456789012123456789012345678901 2
12345678901234567890123456789012123456789012345678901 2
12345678901234567890123456789012123456789012345678901 2
12345678901234567890123456789012123456789012345678901 2 2
162 12345678901234567890123456789012123456789012345678901
123456789012345678901234567890121234567890123456789012

www.petersons.com
Chapter 5: Verbal Review

123456789012345678901234567890121234567890123456789012
12345678901234567890123456789012123456789012345678901 2
12345678901234567890123456789012123456789012345678901 2
12345678901234567890123456789012123456789012345678901
Similar Strategies, Harder Problems 2
12345678901234567890123456789012123456789012345678901 2
12345678901234567890123456789012123456789012345678901 2
12345678901234567890123456789012123456789012345678901 2
The question-specific verbal strategies—covered earlier in this chapter—
12345678901234567890123456789012123456789012345678901
are the same for antonyms, analogies, and sentence completions. You 2
12345678901234567890123456789012123456789012345678901 2
12345678901234567890123456789012123456789012345678901
always want to make a link between the stem words on an analogy, 2
12345678901234567890123456789012123456789012345678901 2
12345678901234567890123456789012123456789012345678901
regardless of when the question shows up and how difficult the words are. 2
12345678901234567890123456789012123456789012345678901 2
12345678901234567890123456789012123456789012345678901
Similarly, the approach to antonyms is the same: Focus on the stem word, 2
12345678901234567890123456789012123456789012345678901 2
12345678901234567890123456789012123456789012345678901
decide whether or not you know the meaning of this word, and then 2
12345678901234567890123456789012123456789012345678901 2
12345678901234567890123456789012123456789012345678901
proceed accordingly.
2345678901234567890123456789012123456789012345678901
2
2
1
12345678901234567890123456789012123456789012345678901
Difficult SC problems will typically have two blanks instead of just one, 2
12345678901234567890123456789012123456789012345678901 2
12345678901234567890123456789012123456789012345678901 2
12345678901234567890123456789012123456789012345678901
making you search for clues for both missing words. When the test decides 2
1 to get really clever, you get a situation where you can’t figure out either 2
2345678901234567890123456789012123456789012345678901
12345678901234567890123456789012123456789012345678901 2
12345678901234567890123456789012123456789012345678901 2
12345678901234567890123456789012123456789012345678901
blank because they are linked. The sample sentence below shows this in 2
12345678901234567890123456789012123456789012345678901 2
12345678901234567890123456789012123456789012345678901 22
2345678901234567890123456789012123456789012345678901
action:
12345678901234567890123456789012123456789012345678901 2

Take It to the Next Level


12345678901234567890123456789012123456789012345678901
The guests were __________ at the __________ state of the basement. 2
12345678901234567890123456789012123456789012345678901
12345678901234567890123456789012123456789012345678901 22
12345678901234567890123456789012123456789012345678901
1 The problem here isn’t that you can’t place any words. Rather, many 2
12345678901234567890123456789012123456789012345678901
different combinations of words work in the two places, since the clue to 2
12345678901234567890123456789012123456789012345678901 2
12345678901234567890123456789012123456789012345678901
the first blank (a missing verb) is the second blank (an adjective describing 2
2345678901234567890123456789012123456789012345678901
12345678901234567890123456789012123456789012345678901 2
12345678901234567890123456789012123456789012345678901
the basement). 2
12345678901234567890123456789012123456789012345678901 22
12345678901234567890123456789012123456789012345678901
12345678901234567890123456789012123456789012345678901
The guests could be: 2
12345678901234567890123456789012123456789012345678901 22
12345678901234567890123456789012123456789012345678901
12345678901234567890123456789012123456789012345678901
delighted at the magnificent state of the basement. 2
12345678901234567890123456789012123456789012345678901 22
1 disgusted at the slovenly state of the basement.
12345678901234567890123456789012123456789012345678901 2
12345678901234567890123456789012123456789012345678901 2
12345678901234567890123456789012123456789012345678901
bemused at the whimsical state of the basement. 2
12345678901234567890123456789012123456789012345678901 2
12345678901234567890123456789012123456789012345678901
It’s uncertain how you get a basement to be whimsical, but if you could, 2
12345678901234567890123456789012123456789012345678901 2
12345678901234567890123456789012123456789012345678901 2
12345678901234567890123456789012123456789012345678901
then the last choice works just as well as the others. 2
12345678901234567890123456789012123456789012345678901 2
12345678901234567890123456789012123456789012345678901 2
12345678901234567890123456789012123456789012345678901 22
2345678901234567890123456789012123456789012345678901
If you come across one of these two-blank SC questions, congratulate
12345678901234567890123456789012123456789012345678901
1 yourself for doing well on the test so far, since you wouldn’t get one of 2
12345678901234567890123456789012123456789012345678901
these puppies if you were answering questions poorly. After you are done 2
12345678901234567890123456789012123456789012345678901 2
2345678901234567890123456789012123456789012345678901
patting your back, determine what the link between the two missing words 2
12345678901234567890123456789012123456789012345678901
12345678901234567890123456789012123456789012345678901 2
is. This link is usually just “similar” (like in the example above) or 2
12345678901234567890123456789012123456789012345678901
12345678901234567890123456789012123456789012345678901 22
1 “opposite.” An opposite SC would resemble:
12345678901234567890123456789012123456789012345678901 2
12345678901234567890123456789012123456789012345678901 2
12345678901234567890123456789012123456789012345678901
The guests were not __________ at the __________ state of the 2
12345678901234567890123456789012123456789012345678901 2
12345678901234567890123456789012123456789012345678901
basement. 2
12345678901234567890123456789012123456789012345678901 22
2345678901234567890123456789012123456789012345678901
12345678901234567890123456789012123456789012345678901
The “not” flips the connection between the two words from similar to 2
12345678901234567890123456789012123456789012345678901 2
12345678901234567890123456789012123456789012345678901 2
1 opposite.
12345678901234567890123456789012123456789012345678901 22
2345678901234567890123456789012123456789012345678901
1
12345678901234567890123456789012123456789012345678901 2
12345678901234567890123456789012123456789012345678901 2
12345678901234567890123456789012123456789012345678901 2
12345678901234567890123456789012123456789012345678901 2
12345678901234567890123456789012123456789012345678901 2
12345678901234567890123456789012123456789012345678901 2
12345678901234567890123456789012123456789012345678901 2 163
123456789012345678901234567890121234567890123456789012
Part III: Section Review

123456789012345678901234567890121234567890123456789012
12345678901234567890123456789012123456789012345678901 2
12345678901234567890123456789012123456789012345678901 2
12345678901234567890123456789012123456789012345678901
The guests were __________ at the __________ state of the 2
12345678901234567890123456789012123456789012345678901 2
12345678901234567890123456789012123456789012345678901
basement. 2
12345678901234567890123456789012123456789012345678901 2
12345678901234567890123456789012123456789012345678901 2
12345678901234567890123456789012123456789012345678901
A. appalled. .tidy 2
12345678901234567890123456789012123456789012345678901 2
12345678901234567890123456789012123456789012345678901
B. appreciative. .flooded 2
12345678901234567890123456789012123456789012345678901 2
12345678901234567890123456789012123456789012345678901
C. befuddled. .paltry
2345678901234567890123456789012123456789012345678901
2
2
1
12345678901234567890123456789012123456789012345678901
D. enthralled. .singular 2
12345678901234567890123456789012123456789012345678901 2
12345678901234567890123456789012123456789012345678901
E. ambivalent. .mildewed 2
12345678901234567890123456789012123456789012345678901 2
1 Once you figure out the link, it’s time to hit the answer choices and start 2
2345678901234567890123456789012123456789012345678901
12345678901234567890123456789012123456789012345678901 2
12345678901234567890123456789012123456789012345678901
sifting through the vocabulary. Choice (A) has opposites, so that’s a wrong 2
12345678901234567890123456789012123456789012345678901 2
12345678901234567890123456789012123456789012345678901
link. Choices (B) and (E) don’t work very well, as the second term does not 2
2345678901234567890123456789012123456789012345678901 2
1
12345678901234567890123456789012123456789012345678901
do a very good job of justifying the first one. Also, flooded and mildewed 2
12345678901234567890123456789012123456789012345678901 2
12345678901234567890123456789012123456789012345678901
are two things that happen to real basements, so these answer choices are 2
12345678901234567890123456789012123456789012345678901 2
12345678901234567890123456789012123456789012345678901
probably there to distract you. Paring things down to two good choices, 2
12345678901234567890123456789012123456789012345678901 2
12345678901234567890123456789012123456789012345678901
(C) and (D), there is no reason to think people would be confused 2
12345678901234567890123456789012123456789012345678901 2
12345678901234567890123456789012123456789012345678901
(befuddled) by a spare (paltry) basement. Choice (D) is the best answer, as 2
2345678901234567890123456789012123456789012345678901
12345678901234567890123456789012123456789012345678901 2
1 something singular (wondrous) would cause people to be enthralled 2
12345678901234567890123456789012123456789012345678901 2
12345678901234567890123456789012123456789012345678901
(captivated). 2
12345678901234567890123456789012123456789012345678901 2
2345678901234567890123456789012123456789012345678901
After you survive the first eight or so questions on the GRE Verbal, you’ll 2
12345678901234567890123456789012123456789012345678901
12345678901234567890123456789012123456789012345678901 2
12345678901234567890123456789012123456789012345678901 2
12345678901234567890123456789012123456789012345678901 2
get your first shot at the Reading Comp passage. If you are scoring very
well, this passage should be a tough RC passage, which means a densely 2
12345678901234567890123456789012123456789012345678901
12345678901234567890123456789012123456789012345678901 2
12345678901234567890123456789012123456789012345678901 2
12345678901234567890123456789012123456789012345678901 2
worded passage on some arcane topic. One way to handle this type of
12345678901234567890123456789012123456789012345678901 2
1 dense passage is to jot down notes on a sheet of scratch paper. Note-taking 2
2345678901234567890123456789012123456789012345678901
can help you understand the passage better, although it will slow you 2
12345678901234567890123456789012123456789012345678901
12345678901234567890123456789012123456789012345678901 2
1 down a bit. However, imagine that you are in a classroom and listening to 2
12345678901234567890123456789012123456789012345678901 2
12345678901234567890123456789012123456789012345678901
a professor drone on about a boring subject that you know will be on the 2
12345678901234567890123456789012123456789012345678901 2
12345678901234567890123456789012123456789012345678901
mid-term. Will you understand the topic better if you take notes, or if you 2
12345678901234567890123456789012123456789012345678901 2
12345678901234567890123456789012123456789012345678901
don’t? Again, this takes additional time, but if you are gunning for a high 2
12345678901234567890123456789012123456789012345678901 2
12345678901234567890123456789012123456789012345678901
score, you should have the time if your wide knowledge of vocabulary 2
12345678901234567890123456789012123456789012345678901 2
12345678901234567890123456789012123456789012345678901
helped you sail through the antonym problems. 2
12345678901234567890123456789012123456789012345678901 2
12345678901234567890123456789012123456789012345678901
Jotting down a quick note about each paragraph will help you understand 2
12345678901234567890123456789012123456789012345678901 2
12345678901234567890123456789012123456789012345678901 2
12345678901234567890123456789012123456789012345678901
the content of each one. This will come in handy when you tackle the RC 2
12345678901234567890123456789012123456789012345678901 2
12345678901234567890123456789012123456789012345678901 2
problems. It is highly unlikely that any of these problems will be a question
2345678901234567890123456789012123456789012345678901
as straightforward as, “What would be a good title for this essay?” 2
12345678901234567890123456789012123456789012345678901
12345678901234567890123456789012123456789012345678901 2
12345678901234567890123456789012123456789012345678901 2
12345678901234567890123456789012123456789012345678901 2
Instead, expect to see convoluted problems like:
2
1
12345678901234567890123456789012123456789012345678901
“If Dr. Andersen changes her mind about the social behavior of 2
12345678901234567890123456789012123456789012345678901 2
12345678901234567890123456789012123456789012345678901
nudibranchs, then which of the following statements would she be most 2
12345678901234567890123456789012123456789012345678901 2
12345678901234567890123456789012123456789012345678901
likely to agree with?” 2
12345678901234567890123456789012123456789012345678901 2
2345678901234567890123456789012123456789012345678901
1 Faced with this twisted piece of writing, you would first have to go into the 2
12345678901234567890123456789012123456789012345678901 2
12345678901234567890123456789012123456789012345678901 2
12345678901234567890123456789012123456789012345678901
passage to the section where Dr. Andersen discusses nudibranchs and/or 2
1 social behavior. Then you would have to infer what she thinks about it, 2
2345678901234567890123456789012123456789012345678901
12345678901234567890123456789012123456789012345678901 2
12345678901234567890123456789012123456789012345678901 2 2
164 12345678901234567890123456789012123456789012345678901
123456789012345678901234567890121234567890123456789012

www.petersons.com
Chapter 5: Verbal Review

123456789012345678901234567890121234567890123456789012
12345678901234567890123456789012123456789012345678901 2
12345678901234567890123456789012123456789012345678901 2
12345678901234567890123456789012123456789012345678901
figure out what the opposite of that is, and then look through the answer 2
12345678901234567890123456789012123456789012345678901 2
12345678901234567890123456789012123456789012345678901
choices to find the paraphrased answer that matches this opposite 2
12345678901234567890123456789012123456789012345678901 2
12345678901234567890123456789012123456789012345678901
inference.
2345678901234567890123456789012123456789012345678901
2
2
1
12345678901234567890123456789012123456789012345678901 2
12345678901234567890123456789012123456789012345678901 2
12345678901234567890123456789012123456789012345678901
You would have to infer what Dr. Andersen thinks because it is very 2
12345678901234567890123456789012123456789012345678901 2
Tip
12345678901234567890123456789012123456789012345678901
doubtful anything will be given to you on a tough RC question. Nothing 2
12345678901234567890123456789012123456789012345678901 2
12345678901234567890123456789012123456789012345678901
will just be waiting in the passage; you will have to take the clues in the 2
12345678901234567890123456789012123456789012345678901 2
12345678901234567890123456789012123456789012345678901
passage and use them to deduce the information
2345678901234567890123456789012123456789012345678901
you need. 2
2
1
12345678901234567890123456789012123456789012345678901 2
12345678901234567890123456789012123456789012345678901 2
12345678901234567890123456789012123456789012345678901
The additional work required to answer a hard RC problem makes these 2
12345678901234567890123456789012123456789012345678901
2345678901234567890123456789012123456789012345678901
2
2
1 questions more involved and time-consuming. Yet they are still very
12345678901234567890123456789012123456789012345678901 2
12345678901234567890123456789012123456789012345678901
solvable if you take the time to look back into the passage and infer the 2
12345678901234567890123456789012123456789012345678901 2
12345678901234567890123456789012123456789012345678901
answer that you need. 2
12345678901234567890123456789012123456789012345678901 22
2345678901234567890123456789012123456789012345678901
12345678901234567890123456789012123456789012345678901 2

Take It to the Next Level


12345678901234567890123456789012123456789012345678901 2
12345678901234567890123456789012123456789012345678901 2
12345678901234567890123456789012123456789012345678901
Free Words! Free Words!
12345678901234567890123456789012123456789012345678901 2
1 To acquire a comprehensive list of all the words that might appear on the 2
12345678901234567890123456789012123456789012345678901 2
12345678901234567890123456789012123456789012345678901 2
12345678901234567890123456789012123456789012345678901
GRE, simply go down to your local bookstore and look for a dictionary of 2
2345678901234567890123456789012123456789012345678901
at least 1,500 pages. For a slightly smaller list of advanced words, try the 2
12345678901234567890123456789012123456789012345678901
12345678901234567890123456789012123456789012345678901 2
12345678901234567890123456789012123456789012345678901 2
12345678901234567890123456789012123456789012345678901 22
set below. There’s no way to tell which batch of words you will get on your
12345678901234567890123456789012123456789012345678901
particular GRE, but any increase in your stockpile of tough words 2
12345678901234567890123456789012123456789012345678901
12345678901234567890123456789012123456789012345678901
increases your odds of succeeding on the test. 2
12345678901234567890123456789012123456789012345678901 2
12345678901234567890123456789012123456789012345678901 2
1 To really get the most out of this list, write out each word and then provide 2
2345678901234567890123456789012123456789012345678901
12345678901234567890123456789012123456789012345678901
a definition and sample sentence. You can put this list of words in a 2
2
12345678901234567890123456789012123456789012345678901
1 notebook, handheld PDA, or plain old flash cards. Use whatever media 2
12345678901234567890123456789012123456789012345678901 2
12345678901234567890123456789012123456789012345678901 2
12345678901234567890123456789012123456789012345678901
you are most comfortable with, but make sure to compile the list in a way 2
12345678901234567890123456789012123456789012345678901 2
12345678901234567890123456789012123456789012345678901
that makes it easy for you to review the contents. 2
12345678901234567890123456789012123456789012345678901 2
12345678901234567890123456789012123456789012345678901 2
12345678901234567890123456789012123456789012345678901 22
2345678901234567890123456789012123456789012345678901
If you are willing to take the time, get a thesaurus and include a list of
12345678901234567890123456789012123456789012345678901
1 possible antonyms for each applicable word. Some words won’t have 2
12345678901234567890123456789012123456789012345678901
antonyms, but many of them will, and studying the antonyms of difficult 2
12345678901234567890123456789012123456789012345678901 2
2345678901234567890123456789012123456789012345678901
words has obvious, obvious advantages. Some of those antonyms will be 2
12345678901234567890123456789012123456789012345678901
12345678901234567890123456789012123456789012345678901 2
12345678901234567890123456789012123456789012345678901
hard words themselves, giving you an easy way to increase your 2
12345678901234567890123456789012123456789012345678901 22
1 vocabulary even further.
12345678901234567890123456789012123456789012345678901 2
12345678901234567890123456789012123456789012345678901 2
12345678901234567890123456789012123456789012345678901 2
12345678901234567890123456789012123456789012345678901 2
12345678901234567890123456789012123456789012345678901 2
12345678901234567890123456789012123456789012345678901 22
2345678901234567890123456789012123456789012345678901
12345678901234567890123456789012123456789012345678901 2
12345678901234567890123456789012123456789012345678901 2
12345678901234567890123456789012123456789012345678901 2
1
12345678901234567890123456789012123456789012345678901 2
12345678901234567890123456789012123456789012345678901 2
12345678901234567890123456789012123456789012345678901 2
12345678901234567890123456789012123456789012345678901 2
12345678901234567890123456789012123456789012345678901 2
12345678901234567890123456789012123456789012345678901 2
12345678901234567890123456789012123456789012345678901 2
12345678901234567890123456789012123456789012345678901 2
12345678901234567890123456789012123456789012345678901 2 165
123456789012345678901234567890121234567890123456789012
Part III: Section Review

123456789012345678901234567890121234567890123456789012
12345678901234567890123456789012123456789012345678901 2
12345678901234567890123456789012123456789012345678901 2
12345678901234567890123456789012123456789012345678901
The Word List 2
12345678901234567890123456789012123456789012345678901 2
12345678901234567890123456789012123456789012345678901 2
12345678901234567890123456789012123456789012345678901 2
12345678901234567890123456789012123456789012345678901 2
12345678901234567890123456789012123456789012345678901
Word Origin Latin brevis 5 short. Also found in English brevity. 2
12345678901234567890123456789012123456789012345678901 2
12345678901234567890123456789012123456789012345678901 2
12345678901234567890123456789012123456789012345678901
abbreviate (verb) To make briefer, to shorten. Because time 2
12345678901234567890123456789012123456789012345678901
2345678901234567890123456789012123456789012345678901
2
2
1 was running out, the speaker was forced to abbreviate
12345678901234567890123456789012123456789012345678901 2
12345678901234567890123456789012123456789012345678901
his remarks. abbreviation (noun). 2
12345678901234567890123456789012123456789012345678901 2
12345678901234567890123456789012123456789012345678901
2345678901234567890123456789012123456789012345678901
2
2
1 aberration (noun) A deviation from what is normal or
12345678901234567890123456789012123456789012345678901 2
12345678901234567890123456789012123456789012345678901
natural, an abnormality. Jack’s extravagant lunch at 2
12345678901234567890123456789012123456789012345678901
Lutece was an aberration from his usual meal, a peanut 2
12345678901234567890123456789012123456789012345678901
2345678901234567890123456789012123456789012345678901
2
2
1 butter sandwich and a diet soda. aberrant (adjective).
12345678901234567890123456789012123456789012345678901 2
12345678901234567890123456789012123456789012345678901 2
12345678901234567890123456789012123456789012345678901
abeyance (noun) A temporary lapse in activity; suspension. 2
12345678901234567890123456789012123456789012345678901 2
12345678901234567890123456789012123456789012345678901
In the aftermath of the bombing, all normal activities 2
12345678901234567890123456789012123456789012345678901 2
12345678901234567890123456789012123456789012345678901
were held in abeyance. 2
12345678901234567890123456789012123456789012345678901 2
12345678901234567890123456789012123456789012345678901 2
12345678901234567890123456789012123456789012345678901 2
2345678901234567890123456789012123456789012345678901
abjure (verb) To renounce or reject; to officially disclaim.
1 While being tried by the inquisition in 1633, Galileo 2
12345678901234567890123456789012123456789012345678901 2
12345678901234567890123456789012123456789012345678901 2
12345678901234567890123456789012123456789012345678901
abjured all his writings holding that Earth and other 2
2345678901234567890123456789012123456789012345678901
12345678901234567890123456789012123456789012345678901 2
12345678901234567890123456789012123456789012345678901
planets revolved around the sun. 2
12345678901234567890123456789012123456789012345678901 2
12345678901234567890123456789012123456789012345678901 2
12345678901234567890123456789012123456789012345678901 2
12345678901234567890123456789012123456789012345678901
Word Origin Latin abradare 5 to scrape. Also found in English abrasive. 2
12345678901234567890123456789012123456789012345678901 2
12345678901234567890123456789012123456789012345678901
abrade (verb) To irritate by rubbing; to wear down in spirit. 2
2
12345678901234567890123456789012123456789012345678901
1 Olga’s “conditioning facial” abraded Sabrina’s skin so 2
12345678901234567890123456789012123456789012345678901 2
12345678901234567890123456789012123456789012345678901
severely that she vowed never to let anyone’s hands 2
12345678901234567890123456789012123456789012345678901 2
12345678901234567890123456789012123456789012345678901 2
12345678901234567890123456789012123456789012345678901
touch her face again. abrasion (noun). 2
12345678901234567890123456789012123456789012345678901 2
2345678901234567890123456789012123456789012345678901
1 abridge (verb) To shorten, to reduce. The Bill of Rights is 2
12345678901234567890123456789012123456789012345678901 2
12345678901234567890123456789012123456789012345678901 2
12345678901234567890123456789012123456789012345678901
designed to prevent Congress from abridging the rights 2
2345678901234567890123456789012123456789012345678901
12345678901234567890123456789012123456789012345678901
of Americans. abridgment (noun). 2
12345678901234567890123456789012123456789012345678901 2
1 2
12345678901234567890123456789012123456789012345678901
abrogate (verb) To nullify, to abolish. During World War II, 2
12345678901234567890123456789012123456789012345678901 2
12345678901234567890123456789012123456789012345678901
the United States abrogated the rights of Japanese 2
12345678901234567890123456789012123456789012345678901 2
12345678901234567890123456789012123456789012345678901
Americans by detaining them in internment camps. 2
12345678901234567890123456789012123456789012345678901 2
12345678901234567890123456789012123456789012345678901
abrogation (noun). 2
12345678901234567890123456789012123456789012345678901 2
2345678901234567890123456789012123456789012345678901
abscond (verb) To make a secret departure, to elope. 2
12345678901234567890123456789012123456789012345678901
12345678901234567890123456789012123456789012345678901 2
12345678901234567890123456789012123456789012345678901 2
1 Theresa will never forgive her daughter, Elena, for 2
2345678901234567890123456789012123456789012345678901
absconding to Miami with Philip when they were 2
12345678901234567890123456789012123456789012345678901
12345678901234567890123456789012123456789012345678901 2
12345678901234567890123456789012123456789012345678901 2
12345678901234567890123456789012123456789012345678901 2
only 17.
2
1
12345678901234567890123456789012123456789012345678901
accretion (noun) A gradual build-up or enlargement. My 2
12345678901234567890123456789012123456789012345678901 2
12345678901234567890123456789012123456789012345678901
mother’s house is a mess due to her steady accretion of 2
12345678901234567890123456789012123456789012345678901 2
12345678901234567890123456789012123456789012345678901
bric-a-brac and her inability to throw anything away. 2
12345678901234567890123456789012123456789012345678901 2
12345678901234567890123456789012123456789012345678901 2
12345678901234567890123456789012123456789012345678901 2 2
166 12345678901234567890123456789012123456789012345678901
123456789012345678901234567890121234567890123456789012

www.petersons.com
Chapter 5: Verbal Review

123456789012345678901234567890121234567890123456789012
12345678901234567890123456789012123456789012345678901 2
12345678901234567890123456789012123456789012345678901 2
12345678901234567890123456789012123456789012345678901
adjunct (noun) Something added to another thing, but not a 2
12345678901234567890123456789012123456789012345678901 2
12345678901234567890123456789012123456789012345678901
part of it; an associate or assistant. While Felix and Fritz 2
12345678901234567890123456789012123456789012345678901 2
12345678901234567890123456789012123456789012345678901
were adjuncts to Professor Himmelman during his 2
2345678901234567890123456789012123456789012345678901 2
1
12345678901234567890123456789012123456789012345678901
experiments in electrodynamics, they did not receive 2
12345678901234567890123456789012123456789012345678901 2
12345678901234567890123456789012123456789012345678901
credit when the results were published. 2
12345678901234567890123456789012123456789012345678901
2345678901234567890123456789012123456789012345678901
2
1 adroit (adjective) Skillful, adept. The writer Laurie Colwin 2
12345678901234567890123456789012123456789012345678901 2
12345678901234567890123456789012123456789012345678901 2
12345678901234567890123456789012123456789012345678901
was particularly adroit at concocting love stories involv- 2
12345678901234567890123456789012123456789012345678901
2345678901234567890123456789012123456789012345678901
2
2
1 ing admirable and quirky female heroines and men who
12345678901234567890123456789012123456789012345678901 2
12345678901234567890123456789012123456789012345678901
deserve them. 2
12345678901234567890123456789012123456789012345678901 2
12345678901234567890123456789012123456789012345678901
adulterate (verb) To corrupt, to make impure. Unlike the 2
2345678901234567890123456789012123456789012345678901 2
1
12345678901234567890123456789012123456789012345678901
chickens from the large poultry companies, Murray’s 2
12345678901234567890123456789012123456789012345678901 2
12345678901234567890123456789012123456789012345678901
free-roaming chickens have not been adulterated with 2
12345678901234567890123456789012123456789012345678901 2
12345678901234567890123456789012123456789012345678901
hormones and other additives. 2
12345678901234567890123456789012123456789012345678901 2

Take It to the Next Level


12345678901234567890123456789012123456789012345678901 2
12345678901234567890123456789012123456789012345678901 2
12345678901234567890123456789012123456789012345678901 2
2345678901234567890123456789012123456789012345678901
12345678901234567890123456789012123456789012345678901
Word
Latin vertere 5 to turn. Also found in English adversary, adverse,
Origin reverse, vertical, and vertigo. 2
1 2
12345678901234567890123456789012123456789012345678901 2
12345678901234567890123456789012123456789012345678901 2
12345678901234567890123456789012123456789012345678901
adversary (noun) An enemy or opponent. When the former 2
2345678901234567890123456789012123456789012345678901
Soviet Union became an American ally, the United States 2
12345678901234567890123456789012123456789012345678901
12345678901234567890123456789012123456789012345678901 2
12345678901234567890123456789012123456789012345678901 2
12345678901234567890123456789012123456789012345678901 22
lost its last major international adversary. adverse
12345678901234567890123456789012123456789012345678901
12345678901234567890123456789012123456789012345678901
(adjective). 2
12345678901234567890123456789012123456789012345678901 22
12345678901234567890123456789012123456789012345678901
aesthete (noun) Someone devoted to beauty and to beautiful
12345678901234567890123456789012123456789012345678901 2
1 things. A renowned aesthete, Oscar Wilde was the 2
12345678901234567890123456789012123456789012345678901 22
2345678901234567890123456789012123456789012345678901
12345678901234567890123456789012123456789012345678901
center of a group that glorified beauty and adopted the
2
1
12345678901234567890123456789012123456789012345678901
slogan “art for art’s sake.” aesthetic (adjective). 2
12345678901234567890123456789012123456789012345678901
2345678901234567890123456789012123456789012345678901
2
affability (noun) The quality of being easy to talk to and 2
12345678901234567890123456789012123456789012345678901
12345678901234567890123456789012123456789012345678901 2
1 2
12345678901234567890123456789012123456789012345678901
gracious. Affability is a much-desired trait in any 2
12345678901234567890123456789012123456789012345678901 2
12345678901234567890123456789012123456789012345678901 22
2345678901234567890123456789012123456789012345678901
profession that involves dealing with many people on a
12345678901234567890123456789012123456789012345678901 2
1 daily basis. affable (adjective).
12345678901234567890123456789012123456789012345678901 2
12345678901234567890123456789012123456789012345678901
affected (adjective) False, artificial. At one time, Japanese 2
12345678901234567890123456789012123456789012345678901 2
12345678901234567890123456789012123456789012345678901
women were taught to speak in an affected high-pitched 2
12345678901234567890123456789012123456789012345678901 2
12345678901234567890123456789012123456789012345678901
voice, which was thought girlishly attractive. affect 2
12345678901234567890123456789012123456789012345678901 2
12345678901234567890123456789012123456789012345678901
(verb), affectation (noun). 2
12345678901234567890123456789012123456789012345678901 2
12345678901234567890123456789012123456789012345678901 2
12345678901234567890123456789012123456789012345678901
affinity (noun) A feeling of shared attraction, kinship; a 2
12345678901234567890123456789012123456789012345678901 2
12345678901234567890123456789012123456789012345678901
similarity. When they first fell in love, Andrew and 2
12345678901234567890123456789012123456789012345678901 2
12345678901234567890123456789012123456789012345678901
Tanya marveled over their affinity for bluegrass music, 2
12345678901234567890123456789012123456789012345678901
obscure French poetry, and beer taken with a squirt of 2
12345678901234567890123456789012123456789012345678901 2
2345678901234567890123456789012123456789012345678901
lemon juice. People often say there is a striking affinity 2
12345678901234567890123456789012123456789012345678901
1 2
12345678901234567890123456789012123456789012345678901 2
12345678901234567890123456789012123456789012345678901 2
between dogs and their owners (but please don’t tell
12345678901234567890123456789012123456789012345678901
Clara that she and her bassett hound are starting to 2
12345678901234567890123456789012123456789012345678901 2
12345678901234567890123456789012123456789012345678901 2
12345678901234567890123456789012123456789012345678901 22
resemble each other).
12345678901234567890123456789012123456789012345678901 167
123456789012345678901234567890121234567890123456789012
Part III: Section Review

123456789012345678901234567890121234567890123456789012
12345678901234567890123456789012123456789012345678901 2
12345678901234567890123456789012123456789012345678901 2
12345678901234567890123456789012123456789012345678901
aggrandize (verb) To make bigger or greater; to inflate. 2
12345678901234567890123456789012123456789012345678901 2
12345678901234567890123456789012123456789012345678901
When he was mayor of New York City, Ed Koch was 2
12345678901234567890123456789012123456789012345678901 2
12345678901234567890123456789012123456789012345678901
renowned for aggrandizing his accomplishments and 2
2345678901234567890123456789012123456789012345678901 2
1
12345678901234567890123456789012123456789012345678901
strolling through city events shouting, “How’m I 2
12345678901234567890123456789012123456789012345678901 2
12345678901234567890123456789012123456789012345678901
doing?” aggrandizement (noun). 2
12345678901234567890123456789012123456789012345678901
2345678901234567890123456789012123456789012345678901
2
1 agitation (noun) A disturbance; a disturbing feeling of 2
12345678901234567890123456789012123456789012345678901 2
12345678901234567890123456789012123456789012345678901 2
12345678901234567890123456789012123456789012345678901
upheaval and excitement. After the CEO announced the 2
12345678901234567890123456789012123456789012345678901
2345678901234567890123456789012123456789012345678901
2
2
1 coming layoffs, the employees’ agitation was evident as
12345678901234567890123456789012123456789012345678901 2
12345678901234567890123456789012123456789012345678901
they remained in the auditorium talking excitedly 2
12345678901234567890123456789012123456789012345678901
among themselves. agitated (adjective), agitate (verb). 2
12345678901234567890123456789012123456789012345678901
2345678901234567890123456789012123456789012345678901
2
2
1
12345678901234567890123456789012123456789012345678901
alias (noun) An assumed name. Determined not to reveal 2
12345678901234567890123456789012123456789012345678901 2
12345678901234567890123456789012123456789012345678901
his upper-class roots, Harold Steerforth Hetherington 2
12345678901234567890123456789012123456789012345678901 2
12345678901234567890123456789012123456789012345678901
III went under the alias of “Hound Dog” when playing 2
12345678901234567890123456789012123456789012345678901 2
12345678901234567890123456789012123456789012345678901
trumpet in his blues band. 2
12345678901234567890123456789012123456789012345678901 2
12345678901234567890123456789012123456789012345678901 2
12345678901234567890123456789012123456789012345678901 2
2345678901234567890123456789012123456789012345678901
allegiance (noun) Loyalty or devotion shown to one’s
1 government or to a person, group, or cause. At the 2
12345678901234567890123456789012123456789012345678901 2
12345678901234567890123456789012123456789012345678901 2
12345678901234567890123456789012123456789012345678901
moving naturalization ceremony, forty-three new Ameri- 2
2345678901234567890123456789012123456789012345678901
cans from twenty-five lands swore allegiance to the 2
12345678901234567890123456789012123456789012345678901
12345678901234567890123456789012123456789012345678901 2
12345678901234567890123456789012123456789012345678901 2
12345678901234567890123456789012123456789012345678901 2
United States.
12345678901234567890123456789012123456789012345678901 2
allocate (verb) To apportion for a specific purpose; to 2
12345678901234567890123456789012123456789012345678901
12345678901234567890123456789012123456789012345678901 2
distribute. The president talked about the importance of 2
12345678901234567890123456789012123456789012345678901
12345678901234567890123456789012123456789012345678901 2
1 education and health care in his State of the Union 2
12345678901234567890123456789012123456789012345678901 2
12345678901234567890123456789012123456789012345678901
address, but, in the end, the administration did not 2
12345678901234567890123456789012123456789012345678901 2
12345678901234567890123456789012123456789012345678901
allocate enough resources for these pressing concerns. 2
12345678901234567890123456789012123456789012345678901 2
12345678901234567890123456789012123456789012345678901
allocation (noun). 2
12345678901234567890123456789012123456789012345678901 2
12345678901234567890123456789012123456789012345678901 2
12345678901234567890123456789012123456789012345678901
amalgamate (verb) To blend thoroughly. The tendency of 2
12345678901234567890123456789012123456789012345678901 2
12345678901234567890123456789012123456789012345678901 2
2345678901234567890123456789012123456789012345678901
grains to sort when they should mix makes it difficult for
1 manufacturers to create powders that are amalgamated. 2
12345678901234567890123456789012123456789012345678901 2
12345678901234567890123456789012123456789012345678901 2
12345678901234567890123456789012123456789012345678901 2
amalgamation (noun).
12345678901234567890123456789012123456789012345678901 2
2345678901234567890123456789012123456789012345678901
ameliorate (verb) To make something better or more 2
12345678901234567890123456789012123456789012345678901 2
12345678901234567890123456789012123456789012345678901
tolerable. The living conditions of the tenants were 2
12345678901234567890123456789012123456789012345678901
1 2
12345678901234567890123456789012123456789012345678901
certainly ameliorated when the landlord finally installed 2
12345678901234567890123456789012123456789012345678901 2
12345678901234567890123456789012123456789012345678901
washing machines and dryers in the basement. ameliora- 2
12345678901234567890123456789012123456789012345678901 2
12345678901234567890123456789012123456789012345678901
tion (noun). 2
12345678901234567890123456789012123456789012345678901 2
2345678901234567890123456789012123456789012345678901
2
12345678901234567890123456789012123456789012345678901
amortize (verb) To pay off or reduce a debt gradually 2
12345678901234567890123456789012123456789012345678901
1 through periodic payments. If you don’t need to take a 2
12345678901234567890123456789012123456789012345678901 2
2345678901234567890123456789012123456789012345678901
lump sum tax deduction, it’s best to amortize large 2
12345678901234567890123456789012123456789012345678901
1 2
12345678901234567890123456789012123456789012345678901 2
12345678901234567890123456789012123456789012345678901 2
business expenditures by spreading the cost out over
12345678901234567890123456789012123456789012345678901 2
12345678901234567890123456789012123456789012345678901
several years. 2
12345678901234567890123456789012123456789012345678901 2
12345678901234567890123456789012123456789012345678901 2 2
168 12345678901234567890123456789012123456789012345678901
123456789012345678901234567890121234567890123456789012

www.petersons.com
Chapter 5: Verbal Review

123456789012345678901234567890121234567890123456789012
12345678901234567890123456789012123456789012345678901 2
12345678901234567890123456789012123456789012345678901 2
12345678901234567890123456789012123456789012345678901
amplify (verb) To enlarge, expand, or increase. Uncertain as 2
12345678901234567890123456789012123456789012345678901 2
12345678901234567890123456789012123456789012345678901
to whether they understood, the students asked the 2
12345678901234567890123456789012123456789012345678901 2
12345678901234567890123456789012123456789012345678901
teacher to amplify his explanation. amplification (noun). 2
2345678901234567890123456789012123456789012345678901 2
1
12345678901234567890123456789012123456789012345678901 2
12345678901234567890123456789012123456789012345678901 2
12345678901234567890123456789012123456789012345678901 2
12345678901234567890123456789012123456789012345678901
Word
Greek chronos 5 time. Also found in English chronic, chronicle,
Origin chronograph, chronology, and synchronize.
2345678901234567890123456789012123456789012345678901
2
2
1
12345678901234567890123456789012123456789012345678901 2
12345678901234567890123456789012123456789012345678901 2
12345678901234567890123456789012123456789012345678901
anachronistic (adjective) Out of the proper time. The 2
12345678901234567890123456789012123456789012345678901
2345678901234567890123456789012123456789012345678901
2
2
1 reference, in Shakespeare’s Julius Caesar, to “the clock
12345678901234567890123456789012123456789012345678901 2
12345678901234567890123456789012123456789012345678901
striking twelve” is anachronistic, since there were no 2
12345678901234567890123456789012123456789012345678901
striking timepieces in ancient Rome. anachronism (noun). 2
12345678901234567890123456789012123456789012345678901
2345678901234567890123456789012123456789012345678901
2
2
1
12345678901234567890123456789012123456789012345678901
anarchy (noun) Absence of law or order. For several months 2
12345678901234567890123456789012123456789012345678901 2
12345678901234567890123456789012123456789012345678901
after the Nazi government was destroyed, there was no 2
12345678901234567890123456789012123456789012345678901 2
12345678901234567890123456789012123456789012345678901
effective government in parts of Germany, and anarchy 2
12345678901234567890123456789012123456789012345678901 2

Take It to the Next Level


12345678901234567890123456789012123456789012345678901
ruled. anarchic (adjective). 2
12345678901234567890123456789012123456789012345678901 2
12345678901234567890123456789012123456789012345678901 2
12345678901234567890123456789012123456789012345678901 22
2345678901234567890123456789012123456789012345678901
animosity (noun) Hostility, resentment. During the last
1
12345678901234567890123456789012123456789012345678901
debate, the candidates could no longer disguise their 2
12345678901234567890123456789012123456789012345678901 2
12345678901234567890123456789012123456789012345678901
animosity and began to trade accusations and insults. 2
12345678901234567890123456789012123456789012345678901 22
2345678901234567890123456789012123456789012345678901
12345678901234567890123456789012123456789012345678901
anomaly (noun) Something different or irregular. The tiny 2
12345678901234567890123456789012123456789012345678901
planet Pluto, orbiting next to the giants Jupiter, Saturn, 2
12345678901234567890123456789012123456789012345678901
2
12345678901234567890123456789012123456789012345678901
and Neptune, has long appeared to be an anomaly. 2
12345678901234567890123456789012123456789012345678901
12345678901234567890123456789012123456789012345678901 2
12345678901234567890123456789012123456789012345678901
anomalous (adjective). 2
12345678901234567890123456789012123456789012345678901 22
1
12345678901234567890123456789012123456789012345678901
antagonism (noun) Hostility, conflict, opposition. As more 2
12345678901234567890123456789012123456789012345678901 2
12345678901234567890123456789012123456789012345678901
and more reporters investigated the Watergate scandal, 2
12345678901234567890123456789012123456789012345678901 2
12345678901234567890123456789012123456789012345678901
antagonism between the Nixon administration and the 2
2345678901234567890123456789012123456789012345678901
press increased. antagonistic (adjective), antagonize 2
12345678901234567890123456789012123456789012345678901
12345678901234567890123456789012123456789012345678901 2
1 2
12345678901234567890123456789012123456789012345678901
(verb). 2
12345678901234567890123456789012123456789012345678901 2
12345678901234567890123456789012123456789012345678901 22
2345678901234567890123456789012123456789012345678901
12345678901234567890123456789012123456789012345678901 2
1 Greek pathos 5 suffering. Also found in English apathy, empathy,
12345678901234567890123456789012123456789012345678901
Word Origin pathetic, pathos, and sympathy. 2
12345678901234567890123456789012123456789012345678901 2
12345678901234567890123456789012123456789012345678901 22
2345678901234567890123456789012123456789012345678901
12345678901234567890123456789012123456789012345678901
antipathy (noun) A long-held feeling of dislike or aversion. 2
12345678901234567890123456789012123456789012345678901
When asked why he didn’t call for help immediately 2
12345678901234567890123456789012123456789012345678901
1 2
2345678901234567890123456789012123456789012345678901
after his wife fell into a coma, the defendant emphasized 2
12345678901234567890123456789012123456789012345678901
12345678901234567890123456789012123456789012345678901 2
12345678901234567890123456789012123456789012345678901
his wife’s utter antipathy to doctors. 2
12345678901234567890123456789012123456789012345678901 22
1
12345678901234567890123456789012123456789012345678901
apprehension (noun) A feeling of fear or foreboding; an 2
12345678901234567890123456789012123456789012345678901 2
12345678901234567890123456789012123456789012345678901
arrest. The peculiar feeling of apprehension that Harold 2
12345678901234567890123456789012123456789012345678901
Pinter creates in his plays derives as much from the long 2
12345678901234567890123456789012123456789012345678901 2
2345678901234567890123456789012123456789012345678901
silences between speeches as from the speeches them- 2
12345678901234567890123456789012123456789012345678901
1 2
12345678901234567890123456789012123456789012345678901 2
12345678901234567890123456789012123456789012345678901 2
selves. The policewoman’s dramatic apprehension of the
12345678901234567890123456789012123456789012345678901
gunman took place in full view of the midtown lunch 2
12345678901234567890123456789012123456789012345678901 2
12345678901234567890123456789012123456789012345678901 2
12345678901234567890123456789012123456789012345678901 22
crowd. apprehend (verb).
12345678901234567890123456789012123456789012345678901 169
123456789012345678901234567890121234567890123456789012
Part III: Section Review

123456789012345678901234567890121234567890123456789012
12345678901234567890123456789012123456789012345678901 2
12345678901234567890123456789012123456789012345678901 2
12345678901234567890123456789012123456789012345678901
arabesque (noun) Intricate decorative patterns involving 2
12345678901234567890123456789012123456789012345678901 2
12345678901234567890123456789012123456789012345678901
intertwining lines and sometimes incorporating flowers, 2
12345678901234567890123456789012123456789012345678901 2
12345678901234567890123456789012123456789012345678901
animals, and fruits. Borders of gold and fanciful 2
2345678901234567890123456789012123456789012345678901 2
1
12345678901234567890123456789012123456789012345678901
arabesques surround the Arabic script on every page of 2
12345678901234567890123456789012123456789012345678901 2
12345678901234567890123456789012123456789012345678901
this ancient edition of the Koran. 2
12345678901234567890123456789012123456789012345678901
2345678901234567890123456789012123456789012345678901
2
2
1
12345678901234567890123456789012123456789012345678901
2345678901234567890123456789012123456789012345678901
2
2
1
12345678901234567890123456789012123456789012345678901
Word
Latin arbiter 5 judge. Also found in English arbiter, arbitrage, and
Origin arbitrate. 2
12345678901234567890123456789012123456789012345678901
2345678901234567890123456789012123456789012345678901
2
2
1
12345678901234567890123456789012123456789012345678901 2
12345678901234567890123456789012123456789012345678901
arbitrary (adjective) Based on random or merely personal 2
12345678901234567890123456789012123456789012345678901
preference. Both computers cost the same and had the 2
12345678901234567890123456789012123456789012345678901
2345678901234567890123456789012123456789012345678901
2
1 same features, so in the end I made an arbitrary decision 2
12345678901234567890123456789012123456789012345678901 2
12345678901234567890123456789012123456789012345678901
about which one to buy. 2
12345678901234567890123456789012123456789012345678901 2
12345678901234567890123456789012123456789012345678901 2
12345678901234567890123456789012123456789012345678901 2
2345678901234567890123456789012123456789012345678901
archaic (adjective) Old-fashioned, obsolete. Those who
12345678901234567890123456789012123456789012345678901
believe in “open marriage” often declare that they will 2
2
12345678901234567890123456789012123456789012345678901
12345678901234567890123456789012123456789012345678901
not be bound by archaic laws and religious rituals, but 2
2
12345678901234567890123456789012123456789012345678901
1 state instead that love alone should bring two people 2
12345678901234567890123456789012123456789012345678901 2
12345678901234567890123456789012123456789012345678901 2
12345678901234567890123456789012123456789012345678901
together. archaism (noun). 2
12345678901234567890123456789012123456789012345678901 2
2345678901234567890123456789012123456789012345678901
ardor (noun) A strong feeling of passion, energy, or zeal. 2
12345678901234567890123456789012123456789012345678901
12345678901234567890123456789012123456789012345678901 2
12345678901234567890123456789012123456789012345678901 2
12345678901234567890123456789012123456789012345678901 2
The young revolutionary proclaimed his convictions
with an ardor that excited the crowd. ardent (adjective). 2
12345678901234567890123456789012123456789012345678901
12345678901234567890123456789012123456789012345678901 2
12345678901234567890123456789012123456789012345678901 2
arid (adjective) Very dry; boring and meaningless. The arid 2
12345678901234567890123456789012123456789012345678901
12345678901234567890123456789012123456789012345678901 2
1 climate of Arizona makes farming difficult. Some find 2
12345678901234567890123456789012123456789012345678901 2
2345678901234567890123456789012123456789012345678901
the law a fascinating topic, but for me it is an arid 2
12345678901234567890123456789012123456789012345678901
1 2
12345678901234567890123456789012123456789012345678901
discipline. aridity (noun). 2
12345678901234567890123456789012123456789012345678901 2
12345678901234567890123456789012123456789012345678901 2
2345678901234567890123456789012123456789012345678901
12345678901234567890123456789012123456789012345678901 2 2
1
12345678901234567890123456789012123456789012345678901
Word
Latin articulus I 5 joint, division. Also found in English arthritis,
Origin article, and inarticulate. 2
12345678901234567890123456789012123456789012345678901 2
12345678901234567890123456789012123456789012345678901 2
2345678901234567890123456789012123456789012345678901
1 articulate (adjective) To express oneself clearly and effec- 2
12345678901234567890123456789012123456789012345678901 2
12345678901234567890123456789012123456789012345678901 2
12345678901234567890123456789012123456789012345678901 2
tively. Compared to George Bush, with his stammering
2345678901234567890123456789012123456789012345678901
and his frequently incomplete sentences, Bill Clinton 2
12345678901234567890123456789012123456789012345678901
12345678901234567890123456789012123456789012345678901 2
12345678901234567890123456789012123456789012345678901 2
12345678901234567890123456789012123456789012345678901 2
was considered a highly articulate president.
2
1
12345678901234567890123456789012123456789012345678901
asperity (noun) Harshness, severity. Total silence at the 2
12345678901234567890123456789012123456789012345678901 2
12345678901234567890123456789012123456789012345678901
dinner table, baths in icy water, prayers five times a 2
12345678901234567890123456789012123456789012345678901 2
12345678901234567890123456789012123456789012345678901
day—these practices all contributed to the asperity of 2
2345678901234567890123456789012123456789012345678901
12345678901234567890123456789012123456789012345678901 2
12345678901234567890123456789012123456789012345678901
life in the monastery. 2
12345678901234567890123456789012123456789012345678901 2
1 assail (verb) To attack with blows or words. When the 2
12345678901234567890123456789012123456789012345678901 2
2345678901234567890123456789012123456789012345678901
president’s cabinet members rose to justify the case for 2
12345678901234567890123456789012123456789012345678901
1 2
12345678901234567890123456789012123456789012345678901 2
12345678901234567890123456789012123456789012345678901 2
military intervention in Iraq, they were assailed by many
12345678901234567890123456789012123456789012345678901
audience members who were critical of U.S. policy. 2
12345678901234567890123456789012123456789012345678901 2
12345678901234567890123456789012123456789012345678901 2
12345678901234567890123456789012123456789012345678901 2
assailant (noun).
2
170 12345678901234567890123456789012123456789012345678901
123456789012345678901234567890121234567890123456789012

www.petersons.com
Chapter 5: Verbal Review

123456789012345678901234567890121234567890123456789012
12345678901234567890123456789012123456789012345678901 2
12345678901234567890123456789012123456789012345678901 2
12345678901234567890123456789012123456789012345678901
assay (verb) To analyze for particular components; to 2
12345678901234567890123456789012123456789012345678901 2
12345678901234567890123456789012123456789012345678901
determine weight, quality etc. The jeweler assayed the 2
12345678901234567890123456789012123456789012345678901 2
12345678901234567890123456789012123456789012345678901
stone pendant Gwyneth inherited from her mother and 2
2345678901234567890123456789012123456789012345678901 2
1
12345678901234567890123456789012123456789012345678901
found it to contain a topaz of high quality. 2
12345678901234567890123456789012123456789012345678901 2
12345678901234567890123456789012123456789012345678901 2
12345678901234567890123456789012123456789012345678901
assimilate (verb) To absorb into a
2345678901234567890123456789012123456789012345678901
system or culture. New 2
1 York City has assimilated one group of immigrants after 2
12345678901234567890123456789012123456789012345678901 2
12345678901234567890123456789012123456789012345678901 2
12345678901234567890123456789012123456789012345678901
another, from the Jewish, German, and Irish immigrants 2
12345678901234567890123456789012123456789012345678901
2345678901234567890123456789012123456789012345678901
2
2
1 who arrived at the turn of the last century to the waves
12345678901234567890123456789012123456789012345678901 2
12345678901234567890123456789012123456789012345678901
of Mexican and Latin American immigrants who 2
12345678901234567890123456789012123456789012345678901
arrived in the 1980s. assimilated (adjective). 2
12345678901234567890123456789012123456789012345678901
2345678901234567890123456789012123456789012345678901
2
2
1
12345678901234567890123456789012123456789012345678901
assuage (verb) To ease, to pacify. Knowing that the pilot’s 2
12345678901234567890123456789012123456789012345678901 2
12345678901234567890123456789012123456789012345678901
record was perfect did little to assuage Linnet’s fear of 2
12345678901234567890123456789012123456789012345678901 2
12345678901234567890123456789012123456789012345678901
flying in the two-seater airplane. 2
12345678901234567890123456789012123456789012345678901 2

Take It to the Next Level


12345678901234567890123456789012123456789012345678901 2
12345678901234567890123456789012123456789012345678901
audacious (adjective) Bold, daring, adventurous. Her plan 2
12345678901234567890123456789012123456789012345678901 2
12345678901234567890123456789012123456789012345678901 22
2345678901234567890123456789012123456789012345678901
to cross the Atlantic single-handed in a twelve-foot
1
12345678901234567890123456789012123456789012345678901
sailboat was an audacious, if not reckless, one. audacity 2
12345678901234567890123456789012123456789012345678901 2
12345678901234567890123456789012123456789012345678901
(noun). 2
12345678901234567890123456789012123456789012345678901 22
2345678901234567890123456789012123456789012345678901
12345678901234567890123456789012123456789012345678901
authoritarian (adjective) Favoring or demanding blind obe- 2
12345678901234567890123456789012123456789012345678901
dience to leaders. Despite most Americans’ strong belief 2
12345678901234567890123456789012123456789012345678901
2
12345678901234567890123456789012123456789012345678901
in democracy, the American government has sometimes 2
12345678901234567890123456789012123456789012345678901
12345678901234567890123456789012123456789012345678901 2
supported authoritarian regimes in other countries. 2
12345678901234567890123456789012123456789012345678901
12345678901234567890123456789012123456789012345678901 22
1 authoritarianism (noun).
12345678901234567890123456789012123456789012345678901 2
12345678901234567890123456789012123456789012345678901 2
12345678901234567890123456789012123456789012345678901
authoritative (adjective) Official, conclusive. For over five 2
12345678901234567890123456789012123456789012345678901 2
12345678901234567890123456789012123456789012345678901
decades, American parents regarded Doctor Benjamin 2
2345678901234567890123456789012123456789012345678901
Spock as the most authoritative voice on baby and child 2
12345678901234567890123456789012123456789012345678901
12345678901234567890123456789012123456789012345678901 2
1 2
12345678901234567890123456789012123456789012345678901
care. authority (noun), authorize (verb). 2
12345678901234567890123456789012123456789012345678901 2
2345678901234567890123456789012123456789012345678901
avenge (verb) To exact a punishment for or on behalf of 2
12345678901234567890123456789012123456789012345678901
12345678901234567890123456789012123456789012345678901 2
1 2
12345678901234567890123456789012123456789012345678901 2
someone. In Shakespeare’s tragedy Hamlet, the ghost of
12345678901234567890123456789012123456789012345678901
the dead king of Denmark visits his son, Prince Hamlet, 2
12345678901234567890123456789012123456789012345678901 2
12345678901234567890123456789012123456789012345678901
and urges him to avenge his murder. 2
12345678901234567890123456789012123456789012345678901 2
12345678901234567890123456789012123456789012345678901 2
12345678901234567890123456789012123456789012345678901
aver (verb) To claim to be true; to avouch. The fact that the 2
2345678901234567890123456789012123456789012345678901
12345678901234567890123456789012123456789012345678901
key witness averred the defendant’s innocence is what 2
2
12345678901234567890123456789012123456789012345678901 2
12345678901234567890123456789012123456789012345678901
ultimately swayed the jury to deliver a “not guilty” 2
12345678901234567890123456789012123456789012345678901 2
1
12345678901234567890123456789012123456789012345678901
verdict. 2
12345678901234567890123456789012123456789012345678901 2
12345678901234567890123456789012123456789012345678901 2
12345678901234567890123456789012123456789012345678901
avow (verb) To declare boldly. Immediately after Cyrus 2
12345678901234567890123456789012123456789012345678901 2
12345678901234567890123456789012123456789012345678901 22
2345678901234567890123456789012123456789012345678901
avowed his atheism at our church fund-raiser, there was
1
12345678901234567890123456789012123456789012345678901
a long, uncomfortable silence. avowal (noun), avowed 2
12345678901234567890123456789012123456789012345678901 2
12345678901234567890123456789012123456789012345678901
(adjective).
2345678901234567890123456789012123456789012345678901
2
2
1
12345678901234567890123456789012123456789012345678901 2
12345678901234567890123456789012123456789012345678901 2
12345678901234567890123456789012123456789012345678901 2 171
123456789012345678901234567890121234567890123456789012
Part III: Section Review

123456789012345678901234567890121234567890123456789012
12345678901234567890123456789012123456789012345678901 2
12345678901234567890123456789012123456789012345678901 2
12345678901234567890123456789012123456789012345678901
barren (adjective) Desolate; infertile. The subarctic tundra 2
12345678901234567890123456789012123456789012345678901 2
12345678901234567890123456789012123456789012345678901
is a barren wasteland inhabited only by lichens and 2
12345678901234567890123456789012123456789012345678901 2
12345678901234567890123456789012123456789012345678901
mosses. Women who try to conceive in their 40s are 2
2345678901234567890123456789012123456789012345678901 2
1
12345678901234567890123456789012123456789012345678901
often barren and must turn to artificial means of 2
12345678901234567890123456789012123456789012345678901 2
12345678901234567890123456789012123456789012345678901
producing a child. 2
12345678901234567890123456789012123456789012345678901
2345678901234567890123456789012123456789012345678901
2
1 belligerent (adjective) Quarrelsome, combative. Mrs. Juni- 2
12345678901234567890123456789012123456789012345678901 2
12345678901234567890123456789012123456789012345678901 2
12345678901234567890123456789012123456789012345678901
per was so belligerent toward the clerks at the local 2
12345678901234567890123456789012123456789012345678901
2345678901234567890123456789012123456789012345678901
2
2
1 stores that they cringed when they saw her coming.
12345678901234567890123456789012123456789012345678901 2
12345678901234567890123456789012123456789012345678901
belligerent (noun) An opposing army, a party waging war. 2
12345678901234567890123456789012123456789012345678901 2
12345678901234567890123456789012123456789012345678901
The Union and Confederate forces were the belligerents 2
2345678901234567890123456789012123456789012345678901 2
1
12345678901234567890123456789012123456789012345678901
in the American Civil War. 2
12345678901234567890123456789012123456789012345678901 2
12345678901234567890123456789012123456789012345678901 2
12345678901234567890123456789012123456789012345678901 2
2345678901234567890123456789012123456789012345678901
12345678901234567890123456789012123456789012345678901 2
1 Latin bene 5 well. Also found in English benediction, benefactor, 2
12345678901234567890123456789012123456789012345678901
Word Origin beneficent, beneficial, benefit, and benign. 2
12345678901234567890123456789012123456789012345678901 2
12345678901234567890123456789012123456789012345678901 2
12345678901234567890123456789012123456789012345678901 2
2345678901234567890123456789012123456789012345678901
benevolent (adjective) Wishing or doing good. In old age,
1 Carnegie used his wealth for benevolent purposes, 2
12345678901234567890123456789012123456789012345678901 2
12345678901234567890123456789012123456789012345678901 2
12345678901234567890123456789012123456789012345678901
donating large sums to found libraries and schools 2
2345678901234567890123456789012123456789012345678901
12345678901234567890123456789012123456789012345678901 2
12345678901234567890123456789012123456789012345678901
around the country. benevolence (noun). 2
12345678901234567890123456789012123456789012345678901 2
berate (verb) To scold or criticize harshly. The judge angrily 2
12345678901234567890123456789012123456789012345678901
2
12345678901234567890123456789012123456789012345678901
berated the two lawyers for their childish and unprofes- 2
12345678901234567890123456789012123456789012345678901
12345678901234567890123456789012123456789012345678901 2
12345678901234567890123456789012123456789012345678901
sional behavior. 2
12345678901234567890123456789012123456789012345678901 2 2
1
12345678901234567890123456789012123456789012345678901
boggle (verb) To overwhelm with amazement. The ability of 2
12345678901234567890123456789012123456789012345678901 2
12345678901234567890123456789012123456789012345678901
physicists to isolate the most infinitesimal particles of 2
12345678901234567890123456789012123456789012345678901 2
12345678901234567890123456789012123456789012345678901
matter truly boggles the mind. 2
12345678901234567890123456789012123456789012345678901 2
2345678901234567890123456789012123456789012345678901
1 bogus (adjective) Phony, a sham. Senior citizens are often 2
12345678901234567890123456789012123456789012345678901 2
12345678901234567890123456789012123456789012345678901 2
12345678901234567890123456789012123456789012345678901
the target of telemarketing scams pushing bogus invest- 2
2345678901234567890123456789012123456789012345678901
12345678901234567890123456789012123456789012345678901 2
12345678901234567890123456789012123456789012345678901
ment opportunities. 2
1 2
12345678901234567890123456789012123456789012345678901
bombastic (adjective) Inflated or pompous in style. Old- 2
12345678901234567890123456789012123456789012345678901 2
12345678901234567890123456789012123456789012345678901
fashioned bombastic political speeches don’t work on 2
12345678901234567890123456789012123456789012345678901 2
12345678901234567890123456789012123456789012345678901
television, which demands a more intimate, personal 2
12345678901234567890123456789012123456789012345678901 2
12345678901234567890123456789012123456789012345678901
style of communication. bombast (noun). 2
12345678901234567890123456789012123456789012345678901 2
2345678901234567890123456789012123456789012345678901
2
12345678901234567890123456789012123456789012345678901
boor (noun) Crude, insensitive and overbearing. Harold 2
12345678901234567890123456789012123456789012345678901
1 was well-known to be a boor; at parties he horrified 2
12345678901234567890123456789012123456789012345678901 2
2345678901234567890123456789012123456789012345678901
people with stories of his past sexual exploits and old, 2
12345678901234567890123456789012123456789012345678901
12345678901234567890123456789012123456789012345678901 2
12345678901234567890123456789012123456789012345678901 2
12345678901234567890123456789012123456789012345678901 2
off-color jokes. boorish (adjective).
2
1
12345678901234567890123456789012123456789012345678901
brazenly (adverb) Acting with disrespectful boldness. Some 2
12345678901234567890123456789012123456789012345678901 2
12345678901234567890123456789012123456789012345678901 2
12345678901234567890123456789012123456789012345678901 2
say that the former White House intern brazenly threw
12345678901234567890123456789012123456789012345678901
herself at the president, but the American public will 2
12345678901234567890123456789012123456789012345678901 2
12345678901234567890123456789012123456789012345678901
probably never know the full truth. brazen (adjective). 2
12345678901234567890123456789012123456789012345678901 2 2
172 12345678901234567890123456789012123456789012345678901
123456789012345678901234567890121234567890123456789012

www.petersons.com
Chapter 5: Verbal Review

123456789012345678901234567890121234567890123456789012
12345678901234567890123456789012123456789012345678901 2
12345678901234567890123456789012123456789012345678901 2
12345678901234567890123456789012123456789012345678901
broach (verb) To bring up an issue for discussion, to 2
12345678901234567890123456789012123456789012345678901 2
12345678901234567890123456789012123456789012345678901
propose. Knowing my father’s strictness about adhering 2
12345678901234567890123456789012123456789012345678901 2
12345678901234567890123456789012123456789012345678901
to a budget, I just can’t seem to broach the subject of my 2
2345678901234567890123456789012123456789012345678901 2
1
12345678901234567890123456789012123456789012345678901
massive credit-card debt. 2
12345678901234567890123456789012123456789012345678901 2
12345678901234567890123456789012123456789012345678901 2
12345678901234567890123456789012123456789012345678901
burgeon (verb) To bloom, literally
2345678901234567890123456789012123456789012345678901
or figuratively. Due to 2
1 the extremely mild winter, the forsythia burgeoned as 2
12345678901234567890123456789012123456789012345678901 2
12345678901234567890123456789012123456789012345678901 2
12345678901234567890123456789012123456789012345678901
early as March. The story of two prison inmates in 2
12345678901234567890123456789012123456789012345678901
2345678901234567890123456789012123456789012345678901
2
2
1 Manuel Puig’s play The Kiss of The Spiderwoman is
12345678901234567890123456789012123456789012345678901 2
12345678901234567890123456789012123456789012345678901
testimony that tenderness can burgeon in the most 2
12345678901234567890123456789012123456789012345678901
unlikely places. 2
12345678901234567890123456789012123456789012345678901
2345678901234567890123456789012123456789012345678901
2
2
1
12345678901234567890123456789012123456789012345678901
burnish (verb) To shine by polishing, literally or figura- 2
12345678901234567890123456789012123456789012345678901 2
12345678901234567890123456789012123456789012345678901
tively. After stripping seven layers of old paint off the 2
12345678901234567890123456789012123456789012345678901 2
12345678901234567890123456789012123456789012345678901
antique door, the carpenter stained the wood and 2
12345678901234567890123456789012123456789012345678901 2

Take It to the Next Level


12345678901234567890123456789012123456789012345678901
burnished it to a rich hue. When Bill Gates, the 2
12345678901234567890123456789012123456789012345678901 2
12345678901234567890123456789012123456789012345678901
wealthiest man in the country, decided to endorse the 2
2345678901234567890123456789012123456789012345678901
Big Bertha line of Golf Clubs, many suggested that he 2
12345678901234567890123456789012123456789012345678901
1 2
12345678901234567890123456789012123456789012345678901 2
12345678901234567890123456789012123456789012345678901
was trying to burnish his image as a “regular guy.” 2
12345678901234567890123456789012123456789012345678901 2
2345678901234567890123456789012123456789012345678901
buttress (noun) Something that supports or strengthens. 2
12345678901234567890123456789012123456789012345678901
12345678901234567890123456789012123456789012345678901 2
12345678901234567890123456789012123456789012345678901 2
12345678901234567890123456789012123456789012345678901 22
The endorsement of the American Medical Association
12345678901234567890123456789012123456789012345678901
is a powerful buttress for the claims made on behalf of 2
12345678901234567890123456789012123456789012345678901
12345678901234567890123456789012123456789012345678901 2
this new medicine. buttress (verb).
12345678901234567890123456789012123456789012345678901 2
12345678901234567890123456789012123456789012345678901 22
1 cacophony (noun) Discordant sounds; dissonance. In the
12345678901234567890123456789012123456789012345678901 2
12345678901234567890123456789012123456789012345678901
minutes before classes start, the high school’s halls are 2
12345678901234567890123456789012123456789012345678901 2
12345678901234567890123456789012123456789012345678901
filled with a cacophony of shrieks, shouts, banging 2
12345678901234567890123456789012123456789012345678901 2
12345678901234567890123456789012123456789012345678901
locker doors, and pounding feet. cacophonous (adjec- 2
12345678901234567890123456789012123456789012345678901 2
12345678901234567890123456789012123456789012345678901
tive). 2
12345678901234567890123456789012123456789012345678901 2
12345678901234567890123456789012123456789012345678901 2
12345678901234567890123456789012123456789012345678901 22
2345678901234567890123456789012123456789012345678901
cadge (verb) To beg for, to sponge. Few in our crowd want
12345678901234567890123456789012123456789012345678901
to go out on the town with Piper, since he routinely 2
1
12345678901234567890123456789012123456789012345678901 2
12345678901234567890123456789012123456789012345678901
cadges cigarettes, subway tokens, and drinks. 2
12345678901234567890123456789012123456789012345678901 22
2345678901234567890123456789012123456789012345678901
12345678901234567890123456789012123456789012345678901
calibrate (verb) To determine or mark graduations (of a 2
12345678901234567890123456789012123456789012345678901
measuring instrument); to adjust or finely tune. We tried 2
12345678901234567890123456789012123456789012345678901
1 2
2345678901234567890123456789012123456789012345678901
to calibrate the heating to Rufus’s liking, but he still 2
12345678901234567890123456789012123456789012345678901
12345678901234567890123456789012123456789012345678901 2
ended up shivering in our living room. calibration 2
12345678901234567890123456789012123456789012345678901
12345678901234567890123456789012123456789012345678901 22
1 (noun).
12345678901234567890123456789012123456789012345678901 2
12345678901234567890123456789012123456789012345678901 2
12345678901234567890123456789012123456789012345678901
castigate (verb) To chastise; to punish severely. The editor 2
12345678901234567890123456789012123456789012345678901
castigated Bob for repeatedly failing to meet his 2
12345678901234567890123456789012123456789012345678901 2
2345678901234567890123456789012123456789012345678901
12345678901234567890123456789012123456789012345678901 2
1 deadlines. castigation (noun). 2
12345678901234567890123456789012123456789012345678901 2
12345678901234567890123456789012123456789012345678901 2
12345678901234567890123456789012123456789012345678901 2
12345678901234567890123456789012123456789012345678901 2
12345678901234567890123456789012123456789012345678901 2
12345678901234567890123456789012123456789012345678901 2
12345678901234567890123456789012123456789012345678901 2 173
123456789012345678901234567890121234567890123456789012
Part III: Section Review

123456789012345678901234567890121234567890123456789012
12345678901234567890123456789012123456789012345678901 2
12345678901234567890123456789012123456789012345678901 2
12345678901234567890123456789012123456789012345678901
catalytic (adjective) Bringing about, causing, or producing 2
12345678901234567890123456789012123456789012345678901 2
12345678901234567890123456789012123456789012345678901
some result. The conditions for revolution existed in 2
12345678901234567890123456789012123456789012345678901 2
12345678901234567890123456789012123456789012345678901
America by 1765; the disputes about taxation that arose 2
2345678901234567890123456789012123456789012345678901 2
1
12345678901234567890123456789012123456789012345678901
during the following decade were the catalytic events 2
12345678901234567890123456789012123456789012345678901 2
12345678901234567890123456789012123456789012345678901
that sparked the rebellion. catalyze (verb). 2
12345678901234567890123456789012123456789012345678901
2345678901234567890123456789012123456789012345678901
2
2
1
12345678901234567890123456789012123456789012345678901 2
12345678901234567890123456789012123456789012345678901 2
12345678901234567890123456789012123456789012345678901
Word Origin Greek kaustikos 5 burning. Also found in English holocaust.
2
12345678901234567890123456789012123456789012345678901
2345678901234567890123456789012123456789012345678901
2
1 caustic (adjective) Burning, corrosive. No pretensions were 2
12345678901234567890123456789012123456789012345678901 2
12345678901234567890123456789012123456789012345678901
safe when the famous satirist H. L. Mencken unleashed 2
12345678901234567890123456789012123456789012345678901 2
12345678901234567890123456789012123456789012345678901
his caustic wit.
2345678901234567890123456789012123456789012345678901
2
2
1
12345678901234567890123456789012123456789012345678901
chaos (noun) Disorder, confusion, chance. The first few 2
12345678901234567890123456789012123456789012345678901 2
12345678901234567890123456789012123456789012345678901 2
12345678901234567890123456789012123456789012345678901
moments after the explosion were pure chaos: No one 2
2345678901234567890123456789012123456789012345678901
was sure what had happened, and the area was filled 2
12345678901234567890123456789012123456789012345678901
12345678901234567890123456789012123456789012345678901 2
12345678901234567890123456789012123456789012345678901 2
12345678901234567890123456789012123456789012345678901 2
with people running and yelling. chaotic (adjective).
12345678901234567890123456789012123456789012345678901 2
1 charisma (noun) Dynamic charm or appeal. Eva Peron was 2
12345678901234567890123456789012123456789012345678901 2
12345678901234567890123456789012123456789012345678901
such a fiery orator and had so much charisma that she 2
12345678901234567890123456789012123456789012345678901 2
12345678901234567890123456789012123456789012345678901
commanded an enormous political following. charis- 2
12345678901234567890123456789012123456789012345678901 2
2345678901234567890123456789012123456789012345678901
12345678901234567890123456789012123456789012345678901
matic (adjective). 2
12345678901234567890123456789012123456789012345678901 2
12345678901234567890123456789012123456789012345678901
chary (adjective) Slow to accept, cautious. Yuan was chary 2
2
12345678901234567890123456789012123456789012345678901
12345678901234567890123456789012123456789012345678901
about going out with Xinhua, since she had been badly 2
2
12345678901234567890123456789012123456789012345678901
12345678901234567890123456789012123456789012345678901 2
12345678901234567890123456789012123456789012345678901
hurt in her previous relationship. 2
1 2
2345678901234567890123456789012123456789012345678901
chronology (noun) An arrangement of events by order of 2
12345678901234567890123456789012123456789012345678901
12345678901234567890123456789012123456789012345678901 2
1 2
12345678901234567890123456789012123456789012345678901 2
occurrence, a list of dates; the science of time. If you ask
12345678901234567890123456789012123456789012345678901
Susan about her 2-year-old son, she will give you a 2
12345678901234567890123456789012123456789012345678901 2
12345678901234567890123456789012123456789012345678901 2
12345678901234567890123456789012123456789012345678901 2
chronology of his accomplishments and childhood
12345678901234567890123456789012123456789012345678901
illnesses, from the day he was born to the present. The 2
12345678901234567890123456789012123456789012345678901 2
2345678901234567890123456789012123456789012345678901
village of Copan was where Mayan astronomical 2
12345678901234567890123456789012123456789012345678901
12345678901234567890123456789012123456789012345678901 2
1 learning, as applied to chronology, achieved its most 2
12345678901234567890123456789012123456789012345678901 2
12345678901234567890123456789012123456789012345678901
accurate expression in the famous Mayan calendar. 2
12345678901234567890123456789012123456789012345678901 2
12345678901234567890123456789012123456789012345678901
chronological (adjective). 2
12345678901234567890123456789012123456789012345678901 2
12345678901234567890123456789012123456789012345678901 2
12345678901234567890123456789012123456789012345678901 2
churlish (adjective) Coarse and ill-mannered. Few journal-
2345678901234567890123456789012123456789012345678901
12345678901234567890123456789012123456789012345678901
ists were eager to interview the aging film star, since he 2
2
12345678901234567890123456789012123456789012345678901
was reputed to be a churlish, uncooperative subject. 2
12345678901234567890123456789012123456789012345678901
12345678901234567890123456789012123456789012345678901 2
1 2
12345678901234567890123456789012123456789012345678901 2
2345678901234567890123456789012123456789012345678901
churl (noun).
12345678901234567890123456789012123456789012345678901 2
12345678901234567890123456789012123456789012345678901 2
12345678901234567890123456789012123456789012345678901 2 2
1
12345678901234567890123456789012123456789012345678901 2
12345678901234567890123456789012123456789012345678901 2
12345678901234567890123456789012123456789012345678901 2
12345678901234567890123456789012123456789012345678901 2
12345678901234567890123456789012123456789012345678901 2
12345678901234567890123456789012123456789012345678901 2
12345678901234567890123456789012123456789012345678901 2
12345678901234567890123456789012123456789012345678901 2 2
174 12345678901234567890123456789012123456789012345678901
123456789012345678901234567890121234567890123456789012

www.petersons.com
Chapter 5: Verbal Review

123456789012345678901234567890121234567890123456789012
12345678901234567890123456789012123456789012345678901 2
12345678901234567890123456789012123456789012345678901 2
12345678901234567890123456789012123456789012345678901 2
12345678901234567890123456789012123456789012345678901
Latin circus 5 circle. Also found in English circumference, 2
12345678901234567890123456789012123456789012345678901
Word Origin 2
12345678901234567890123456789012123456789012345678901 2
circumnavigate, circumscribe, and circumvent.
12345678901234567890123456789012123456789012345678901 2
12345678901234567890123456789012123456789012345678901
circumspect (adjective) Prudent, cautious. After he had 2
12345678901234567890123456789012123456789012345678901 2
12345678901234567890123456789012123456789012345678901
been acquitted of the sexual harassment charge, the 2
12345678901234567890123456789012123456789012345678901 2
12345678901234567890123456789012123456789012345678901
sergeant realized he would have to be more circumspect 2
2345678901234567890123456789012123456789012345678901 2
1
12345678901234567890123456789012123456789012345678901
in his dealings with the female cadets. circumspection 2
12345678901234567890123456789012123456789012345678901 2
12345678901234567890123456789012123456789012345678901
(noun). 2
12345678901234567890123456789012123456789012345678901
2345678901234567890123456789012123456789012345678901
2
1 cleave (verb) NOTE: A tricky verb that can mean either to 2
12345678901234567890123456789012123456789012345678901 2
12345678901234567890123456789012123456789012345678901 2
12345678901234567890123456789012123456789012345678901
stick closely together or to split apart. (Pay attention to 2
12345678901234567890123456789012123456789012345678901
2345678901234567890123456789012123456789012345678901
2
2
1 context.) The more abusive his father became, the more
12345678901234567890123456789012123456789012345678901 2
12345678901234567890123456789012123456789012345678901
Timothy cleaved to his mother and refused to let her out 2
12345678901234567890123456789012123456789012345678901
of his sight. Sometimes a few words carelessly spoken 2
12345678901234567890123456789012123456789012345678901 2
2345678901234567890123456789012123456789012345678901
are enough to cleave a married couple and leave the 2
12345678901234567890123456789012123456789012345678901
12345678901234567890123456789012123456789012345678901 2

Take It to the Next Level


12345678901234567890123456789012123456789012345678901
relationship in shambles. cleavage (noun). 2
12345678901234567890123456789012123456789012345678901 22
12345678901234567890123456789012123456789012345678901
coagulant (noun) Any material that causes another to 2
12345678901234567890123456789012123456789012345678901 2
1
12345678901234567890123456789012123456789012345678901
thicken or clot. Hemophilia is characterized by excessive 2
12345678901234567890123456789012123456789012345678901 2
12345678901234567890123456789012123456789012345678901
bleeding from even the slightest cut and is caused by a 2
2345678901234567890123456789012123456789012345678901
lack of one of the coagulants necessary for blood 2
12345678901234567890123456789012123456789012345678901
12345678901234567890123456789012123456789012345678901 2
12345678901234567890123456789012123456789012345678901 2
12345678901234567890123456789012123456789012345678901 22
clotting. coagulate (verb).
12345678901234567890123456789012123456789012345678901 2
12345678901234567890123456789012123456789012345678901
coalesce (verb) To fuse, to unite. The music we know as jazz 2
12345678901234567890123456789012123456789012345678901
coalesced from diverse elements from many musical 2
12345678901234567890123456789012123456789012345678901
12345678901234567890123456789012123456789012345678901 2
1 2
12345678901234567890123456789012123456789012345678901 22
2345678901234567890123456789012123456789012345678901
cultures, including those of West Africa, America, and
12345678901234567890123456789012123456789012345678901
Europe. coalescence (noun). 2
1
12345678901234567890123456789012123456789012345678901 2
12345678901234567890123456789012123456789012345678901
coerce (verb) To force someone either to do something or to 2
12345678901234567890123456789012123456789012345678901 2
12345678901234567890123456789012123456789012345678901
refrain from doing something. The Miranda ruling 2
12345678901234567890123456789012123456789012345678901 2
12345678901234567890123456789012123456789012345678901
prevents police from coercing a confession by forcing 2
12345678901234567890123456789012123456789012345678901 2
12345678901234567890123456789012123456789012345678901
them to read criminals their rights. coercion (noun). 2
12345678901234567890123456789012123456789012345678901 2
12345678901234567890123456789012123456789012345678901 2
12345678901234567890123456789012123456789012345678901 2
cogent (adjective) Forceful and convincing. The committee
12345678901234567890123456789012123456789012345678901
members were won over to the project by the cogent 2
12345678901234567890123456789012123456789012345678901 2
12345678901234567890123456789012123456789012345678901 2
12345678901234567890123456789012123456789012345678901
arguments of the chairman. cogency (noun). 2
12345678901234567890123456789012123456789012345678901 2
12345678901234567890123456789012123456789012345678901 2
2345678901234567890123456789012123456789012345678901
12345678901234567890123456789012123456789012345678901 2
1 Latin mensura 5 to measure. Also found in English measure, 2
12345678901234567890123456789012123456789012345678901
Word Origin immeasurable, immense, and mensuration. 2
12345678901234567890123456789012123456789012345678901 2
12345678901234567890123456789012123456789012345678901 2
2345678901234567890123456789012123456789012345678901
commensurate (adjective) Aligned with, proportional. Many 2
12345678901234567890123456789012123456789012345678901
12345678901234567890123456789012123456789012345678901 2
Ph.D.s in the humanities do not feel their paltry salaries 2
12345678901234567890123456789012123456789012345678901
12345678901234567890123456789012123456789012345678901 2
1 are commensurate with their abilities, their experience, 2
2345678901234567890123456789012123456789012345678901
12345678901234567890123456789012123456789012345678901 2
1 or the heavy workload they are asked to bear. 2
12345678901234567890123456789012123456789012345678901 2
12345678901234567890123456789012123456789012345678901 2
12345678901234567890123456789012123456789012345678901 2
12345678901234567890123456789012123456789012345678901 2
12345678901234567890123456789012123456789012345678901 2
12345678901234567890123456789012123456789012345678901 2
12345678901234567890123456789012123456789012345678901 2 175
123456789012345678901234567890121234567890123456789012
Part III: Section Review

123456789012345678901234567890121234567890123456789012
12345678901234567890123456789012123456789012345678901 2
12345678901234567890123456789012123456789012345678901 2
12345678901234567890123456789012123456789012345678901
commingle (verb) To blend, to mix. Just as he had when he 2
12345678901234567890123456789012123456789012345678901 2
12345678901234567890123456789012123456789012345678901
was only five years old, Elmer did not allow any of the 2
12345678901234567890123456789012123456789012345678901 2
12345678901234567890123456789012123456789012345678901
foods on his plate to commingle: The beans must not 2
2345678901234567890123456789012123456789012345678901 2
1
12345678901234567890123456789012123456789012345678901
merge with the rice nor the chicken rub shoulders with 2
12345678901234567890123456789012123456789012345678901 2
12345678901234567890123456789012123456789012345678901
the broccoli! 2
12345678901234567890123456789012123456789012345678901
2345678901234567890123456789012123456789012345678901
2
1 complaisant (adjective) Tending to bow to others’ wishes; 2
12345678901234567890123456789012123456789012345678901 2
12345678901234567890123456789012123456789012345678901 2
12345678901234567890123456789012123456789012345678901
amiable. Of the two Dashwood sisters, Elinor was the 2
12345678901234567890123456789012123456789012345678901
more complaisant, often putting
2345678901234567890123456789012123456789012345678901 the strictures of society 2
2
1
12345678901234567890123456789012123456789012345678901
and family above her own desires. complaisance (noun). 2
12345678901234567890123456789012123456789012345678901 2
12345678901234567890123456789012123456789012345678901 2
12345678901234567890123456789012123456789012345678901
compound (verb) To intensify, to
2345678901234567890123456789012123456789012345678901
exacerbate. When you 2
2
1
12345678901234567890123456789012123456789012345678901
make a faux pas, my father advised me, don’t compound 2
12345678901234567890123456789012123456789012345678901
the problem by apologizing profusely; just say you’re 2
12345678901234567890123456789012123456789012345678901 2
12345678901234567890123456789012123456789012345678901 2
12345678901234567890123456789012123456789012345678901 2
2345678901234567890123456789012123456789012345678901
sorry and get on with life!
12345678901234567890123456789012123456789012345678901 2
conceivable (adjective) Possible, imaginable. It’s possible to 2
12345678901234567890123456789012123456789012345678901
12345678901234567890123456789012123456789012345678901 2
find people with every conceivable interest by surfing the 2
12345678901234567890123456789012123456789012345678901
12345678901234567890123456789012123456789012345678901 2
1 World Wide Web—from fans of minor film stars to 2
12345678901234567890123456789012123456789012345678901 2
12345678901234567890123456789012123456789012345678901
those who study the mating habits of crustaceans. 2
12345678901234567890123456789012123456789012345678901 2
12345678901234567890123456789012123456789012345678901
conception (noun). 2
12345678901234567890123456789012123456789012345678901 2
12345678901234567890123456789012123456789012345678901 2
12345678901234567890123456789012123456789012345678901
concur (verb) To agree, to approve. We concur that a 2
12345678901234567890123456789012123456789012345678901 2
12345678901234567890123456789012123456789012345678901 2
2345678901234567890123456789012123456789012345678901
toddler functions best on a fairly reliable schedule;
however, my husband tends to be a bit more rigid than I 2
12345678901234567890123456789012123456789012345678901
12345678901234567890123456789012123456789012345678901 2
12345678901234567890123456789012123456789012345678901 2
1 am. concurrence (noun). 2
12345678901234567890123456789012123456789012345678901 2
2345678901234567890123456789012123456789012345678901
condensation (noun) A reduction to a denser form (from steam 2
12345678901234567890123456789012123456789012345678901
1 2
12345678901234567890123456789012123456789012345678901
to water); an abridgment of a literary work. The conden- 2
12345678901234567890123456789012123456789012345678901 2
12345678901234567890123456789012123456789012345678901
sation of humidity on the car’s windshield made it difficult 2
12345678901234567890123456789012123456789012345678901 2
12345678901234567890123456789012123456789012345678901
for me to see the road. It seems as though every beach 2
12345678901234567890123456789012123456789012345678901 2
12345678901234567890123456789012123456789012345678901
house I’ve ever rented features a shelf full of Reader’s Di- 2
2345678901234567890123456789012123456789012345678901
12345678901234567890123456789012123456789012345678901 2
12345678901234567890123456789012123456789012345678901
gest condensations of b-grade novels. condense (verb). 2
1 2
12345678901234567890123456789012123456789012345678901 2
12345678901234567890123456789012123456789012345678901 2
condescending (adjective) Having an attitude of superiority
2345678901234567890123456789012123456789012345678901
toward another; patronizing. “What a cute little car!” 2
12345678901234567890123456789012123456789012345678901
12345678901234567890123456789012123456789012345678901 2
12345678901234567890123456789012123456789012345678901 2
12345678901234567890123456789012123456789012345678901 2
she remarked in a condescending fashion. “I suppose it’s
1 the nicest one someone like you could afford!” conde- 2
12345678901234567890123456789012123456789012345678901 2
12345678901234567890123456789012123456789012345678901
scension (noun). 2
12345678901234567890123456789012123456789012345678901 2
12345678901234567890123456789012123456789012345678901 2
12345678901234567890123456789012123456789012345678901 2
condone (verb) To overlook, to permit to happen. Schools
2345678901234567890123456789012123456789012345678901
12345678901234567890123456789012123456789012345678901
with Zero Tolerance policies do not condone alcohol, 2
2
12345678901234567890123456789012123456789012345678901 2
12345678901234567890123456789012123456789012345678901 2
12345678901234567890123456789012123456789012345678901
drugs, vandalism, or violence on school grounds.
2
1
2345678901234567890123456789012123456789012345678901
congruent (adjective) Coinciding; harmonious. Fortunately, 2
12345678901234567890123456789012123456789012345678901
1 2
12345678901234567890123456789012123456789012345678901 2
12345678901234567890123456789012123456789012345678901 2
the two employees who had been asked to organize the
12345678901234567890123456789012123456789012345678901
department had congruent views on the budget. congru- 2
12345678901234567890123456789012123456789012345678901 2
12345678901234567890123456789012123456789012345678901 2
12345678901234567890123456789012123456789012345678901 2
ence (noun).
2
176 12345678901234567890123456789012123456789012345678901
123456789012345678901234567890121234567890123456789012

www.petersons.com
Chapter 5: Verbal Review

123456789012345678901234567890121234567890123456789012
12345678901234567890123456789012123456789012345678901 2
12345678901234567890123456789012123456789012345678901 2
12345678901234567890123456789012123456789012345678901 2
12345678901234567890123456789012123456789012345678901
Latin jungere 5 to join. Also found in English injunction, junction, 2
12345678901234567890123456789012123456789012345678901
Word Origin 2
12345678901234567890123456789012123456789012345678901 2
and juncture.
12345678901234567890123456789012123456789012345678901 2
12345678901234567890123456789012123456789012345678901
conjunction (noun) The occurrence of two or more events 2
12345678901234567890123456789012123456789012345678901 2
12345678901234567890123456789012123456789012345678901
together in time or space; in astronomy, the point at 2
12345678901234567890123456789012123456789012345678901 2
12345678901234567890123456789012123456789012345678901
which two celestial bodies have the least separation. 2
2345678901234567890123456789012123456789012345678901 2
1
12345678901234567890123456789012123456789012345678901
Low inflation, occurring in conjunction with low 2
12345678901234567890123456789012123456789012345678901
unemployment and relatively low interest rates, has 2
12345678901234567890123456789012123456789012345678901 2
12345678901234567890123456789012123456789012345678901
2345678901234567890123456789012123456789012345678901
2
2
1 enabled the United States to enjoy a long period of
12345678901234567890123456789012123456789012345678901 2
12345678901234567890123456789012123456789012345678901
sustained economic growth. The moon is in conjunction 2
12345678901234567890123456789012123456789012345678901 2
12345678901234567890123456789012123456789012345678901
with the sun when it is new;
2345678901234567890123456789012123456789012345678901
if the conjunction is perfect, 2
2
1
12345678901234567890123456789012123456789012345678901
an eclipse of the sun will occur. conjoin (verb). 2
12345678901234567890123456789012123456789012345678901 2
12345678901234567890123456789012123456789012345678901
consolation (noun) Relief or comfort in sorrow or suffering. 2
12345678901234567890123456789012123456789012345678901 2
2345678901234567890123456789012123456789012345678901
Although we miss our dog very much, it is a consolation 2
12345678901234567890123456789012123456789012345678901
12345678901234567890123456789012123456789012345678901 2

Take It to the Next Level


to know that she died quickly, without much suffering. 2
12345678901234567890123456789012123456789012345678901
12345678901234567890123456789012123456789012345678901 22
12345678901234567890123456789012123456789012345678901
console (verb).
12345678901234567890123456789012123456789012345678901 2
1 2
12345678901234567890123456789012123456789012345678901
consternation (noun) Shock, amazement, dismay. When a 2
12345678901234567890123456789012123456789012345678901 2
12345678901234567890123456789012123456789012345678901
voice in the back of the church shouted out, “I know 2
2345678901234567890123456789012123456789012345678901
why they should not be married!” the entire gathering 2
12345678901234567890123456789012123456789012345678901
12345678901234567890123456789012123456789012345678901 2
12345678901234567890123456789012123456789012345678901 2
12345678901234567890123456789012123456789012345678901 22
was thrown into consternation.
12345678901234567890123456789012123456789012345678901 2
12345678901234567890123456789012123456789012345678901
convergence (noun) The act of coming together in unity or 2
12345678901234567890123456789012123456789012345678901
similarity. A remarkable example of evolutionary conver- 2
12345678901234567890123456789012123456789012345678901
12345678901234567890123456789012123456789012345678901 2
1 2
12345678901234567890123456789012123456789012345678901 22
2345678901234567890123456789012123456789012345678901
gence can be seen in the shark and the dolphin, two sea
12345678901234567890123456789012123456789012345678901
creatures that developed from different origins to 2
1
12345678901234567890123456789012123456789012345678901
become very similar in form and appearance. converge 2
12345678901234567890123456789012123456789012345678901
2345678901234567890123456789012123456789012345678901
2
2
12345678901234567890123456789012123456789012345678901
(verb).
12345678901234567890123456789012123456789012345678901 2
1 2
12345678901234567890123456789012123456789012345678901 2
12345678901234567890123456789012123456789012345678901 2
2345678901234567890123456789012123456789012345678901
12345678901234567890123456789012123456789012345678901
Latin vivere 5 to live. Also found in English revive, vital, vivid, and 2
12345678901234567890123456789012123456789012345678901
Word Origin vivisection. 2
1 2
12345678901234567890123456789012123456789012345678901 2
12345678901234567890123456789012123456789012345678901 2
conviviality (noun) Fond of good company and eating and
2345678901234567890123456789012123456789012345678901
drinking. The conviviality of my fellow employees 2
12345678901234567890123456789012123456789012345678901
12345678901234567890123456789012123456789012345678901 2
12345678901234567890123456789012123456789012345678901 2
12345678901234567890123456789012123456789012345678901 22
seemed to turn every staff meeting into a party, complete
1
12345678901234567890123456789012123456789012345678901
with snacks, drinks, and lots of hearty laughter. conviv- 2
12345678901234567890123456789012123456789012345678901 2
12345678901234567890123456789012123456789012345678901
ial (adjective). 2
12345678901234567890123456789012123456789012345678901 2
12345678901234567890123456789012123456789012345678901 2
2345678901234567890123456789012123456789012345678901
12345678901234567890123456789012123456789012345678901 2
1 Latin volvere 5 to roll. Also found in English devolve, involve, 2
12345678901234567890123456789012123456789012345678901
Word Origin revolution, revolve, and voluble. 2
12345678901234567890123456789012123456789012345678901 2
12345678901234567890123456789012123456789012345678901 2
2345678901234567890123456789012123456789012345678901
convoluted (adjective) Twisting, complicated, intricate. In- 2
12345678901234567890123456789012123456789012345678901
1 2
12345678901234567890123456789012123456789012345678901
come tax law has become so convoluted that it’s easy for 2
12345678901234567890123456789012123456789012345678901 2
12345678901234567890123456789012123456789012345678901
people to violate it completely by accident. convolute 2
2345678901234567890123456789012123456789012345678901 2
1
12345678901234567890123456789012123456789012345678901
(verb), convolution (noun). 2
12345678901234567890123456789012123456789012345678901 22
12345678901234567890123456789012123456789012345678901 177
123456789012345678901234567890121234567890123456789012
Part III: Section Review

123456789012345678901234567890121234567890123456789012
12345678901234567890123456789012123456789012345678901 2
12345678901234567890123456789012123456789012345678901 2
12345678901234567890123456789012123456789012345678901
cordon (verb) To form a protective or restrictive barrier. 2
12345678901234567890123456789012123456789012345678901 2
12345678901234567890123456789012123456789012345678901
Well before the Academy Awards ceremony began, the 2
12345678901234567890123456789012123456789012345678901 2
12345678901234567890123456789012123456789012345678901
police cordoned off the hordes of fans who were 2
2345678901234567890123456789012123456789012345678901 2
1
12345678901234567890123456789012123456789012345678901
desperate to ogle the arriving stars. cordon (noun). 2
12345678901234567890123456789012123456789012345678901 2
12345678901234567890123456789012123456789012345678901 2
12345678901234567890123456789012123456789012345678901
corral (verb) To enclose, to collect,
2345678901234567890123456789012123456789012345678901
to gather. Tyrone 2
1 couldn’t enjoy the wedding at all, since he spent most of 2
12345678901234567890123456789012123456789012345678901 2
12345678901234567890123456789012123456789012345678901 2
12345678901234567890123456789012123456789012345678901
his time corralling his two children into the reception 2
12345678901234567890123456789012123456789012345678901
2345678901234567890123456789012123456789012345678901
2
2
1 room and preventing them from running amok through
12345678901234567890123456789012123456789012345678901 2
12345678901234567890123456789012123456789012345678901
the Potters’ mansion. corral (noun). 2
12345678901234567890123456789012123456789012345678901 2
12345678901234567890123456789012123456789012345678901
corroborating (adjective) Supporting with evidence; confirm- 2
2345678901234567890123456789012123456789012345678901 2
1
12345678901234567890123456789012123456789012345678901
ing. A passerby who had witnessed the crime gave 2
12345678901234567890123456789012123456789012345678901 2
12345678901234567890123456789012123456789012345678901
corroborating testimony about the presence of the 2
12345678901234567890123456789012123456789012345678901 2
12345678901234567890123456789012123456789012345678901
accused person. corroborate (verb), corroboration (noun). 2
12345678901234567890123456789012123456789012345678901 2
12345678901234567890123456789012123456789012345678901 2
12345678901234567890123456789012123456789012345678901
corrosive (adjective) Eating away, gnawing, or destroying. 2
12345678901234567890123456789012123456789012345678901 2
12345678901234567890123456789012123456789012345678901 2
2345678901234567890123456789012123456789012345678901
Years of poverty and hard work had a corrosive effect on
1 her strength and beauty. corrode (verb), corrosion 2
12345678901234567890123456789012123456789012345678901 2
12345678901234567890123456789012123456789012345678901 2
12345678901234567890123456789012123456789012345678901
(noun). 2
12345678901234567890123456789012123456789012345678901 2
2345678901234567890123456789012123456789012345678901
cosmopolitanism (noun) International sophistication; world- 2
12345678901234567890123456789012123456789012345678901
12345678901234567890123456789012123456789012345678901 2
liness. Budapest is known for its cosmopolitanism, 2
12345678901234567890123456789012123456789012345678901
12345678901234567890123456789012123456789012345678901 2
perhaps because it was the first Eastern European city to 2
12345678901234567890123456789012123456789012345678901
12345678901234567890123456789012123456789012345678901 2
be more open to capitalism and influences from the 2
12345678901234567890123456789012123456789012345678901
12345678901234567890123456789012123456789012345678901 2 2
1 West. cosmopolitan (adjective).
12345678901234567890123456789012123456789012345678901 2
12345678901234567890123456789012123456789012345678901 2
12345678901234567890123456789012123456789012345678901
covert (adjective) Secret, clandestine. The CIA has often 2
12345678901234567890123456789012123456789012345678901 2
12345678901234567890123456789012123456789012345678901
been criticized for its covert operations in the domestic 2
2345678901234567890123456789012123456789012345678901
policies of foreign countries, such as the failed Bay of 2
12345678901234567890123456789012123456789012345678901
12345678901234567890123456789012123456789012345678901 2
1 2
12345678901234567890123456789012123456789012345678901
Pigs operation in Cuba. 2
12345678901234567890123456789012123456789012345678901 2
2345678901234567890123456789012123456789012345678901
covetous (adjective) Envious, particularly of another’s pos- 2
12345678901234567890123456789012123456789012345678901
12345678901234567890123456789012123456789012345678901 2
1 2
12345678901234567890123456789012123456789012345678901 2
sessions. Benita would never admit to being covetous of
12345678901234567890123456789012123456789012345678901
my new sable jacket, but I found it odd that she couldn’t 2
12345678901234567890123456789012123456789012345678901 2
12345678901234567890123456789012123456789012345678901
refrain from trying it on each time we met. covet (verb). 2
12345678901234567890123456789012123456789012345678901 2
12345678901234567890123456789012123456789012345678901 2
12345678901234567890123456789012123456789012345678901
craven (adjective) Cowardly. Local Gay and Lesbian activ- 2
2345678901234567890123456789012123456789012345678901
12345678901234567890123456789012123456789012345678901
ists were outraged by the craven behavior of a policeman 2
2
12345678901234567890123456789012123456789012345678901 2
12345678901234567890123456789012123456789012345678901
who refused to come to the aid of an HIV-positive 2
12345678901234567890123456789012123456789012345678901 2
1
12345678901234567890123456789012123456789012345678901
accident victim. 2
12345678901234567890123456789012123456789012345678901 2
12345678901234567890123456789012123456789012345678901 2
12345678901234567890123456789012123456789012345678901
credulous (adjective) Ready to believe; gullible. Elaine was 2
12345678901234567890123456789012123456789012345678901 2
12345678901234567890123456789012123456789012345678901 2
2345678901234567890123456789012123456789012345678901
not very credulous of the explanation Serge gave for his
2
1
12345678901234567890123456789012123456789012345678901
acquisition of the Matisse lithograph. credulity (noun). 2
12345678901234567890123456789012123456789012345678901 2
12345678901234567890123456789012123456789012345678901 2
12345678901234567890123456789012123456789012345678901 2
12345678901234567890123456789012123456789012345678901 2
12345678901234567890123456789012123456789012345678901 2 2
178 12345678901234567890123456789012123456789012345678901
123456789012345678901234567890121234567890123456789012

www.petersons.com
Chapter 5: Verbal Review

123456789012345678901234567890121234567890123456789012
12345678901234567890123456789012123456789012345678901 2
12345678901234567890123456789012123456789012345678901 2
12345678901234567890123456789012123456789012345678901
cryptic (adjective) Puzzling, ambiguous. I was puzzled by 2
12345678901234567890123456789012123456789012345678901 2
12345678901234567890123456789012123456789012345678901
the cryptic message left on my answering machine about 2
12345678901234567890123456789012123456789012345678901 2
12345678901234567890123456789012123456789012345678901
“a shipment of pomegranates from an anonymous 2
2345678901234567890123456789012123456789012345678901 2
1
12345678901234567890123456789012123456789012345678901
donor.” 2
12345678901234567890123456789012123456789012345678901 2
12345678901234567890123456789012123456789012345678901 2
12345678901234567890123456789012123456789012345678901
culmination (noun) The climax.
2345678901234567890123456789012123456789012345678901
The Los Angeles riots, in 2
1 the aftermath of the Rodney King verdict, were the 2
12345678901234567890123456789012123456789012345678901 2
12345678901234567890123456789012123456789012345678901 2
12345678901234567890123456789012123456789012345678901
culmination of long-standing racial tensions between the 2
12345678901234567890123456789012123456789012345678901
2345678901234567890123456789012123456789012345678901
2
2
1 residents of South Central LA and the police. culminate
12345678901234567890123456789012123456789012345678901 2
12345678901234567890123456789012123456789012345678901
(verb). 2
12345678901234567890123456789012123456789012345678901 2
12345678901234567890123456789012123456789012345678901
culpable (adjective) Deserving blame, guilty. Although he 2
2345678901234567890123456789012123456789012345678901 2
1
12345678901234567890123456789012123456789012345678901
committed the crime, because he was mentally ill he 2
12345678901234567890123456789012123456789012345678901 2
12345678901234567890123456789012123456789012345678901
should not be considered culpable for his actions. 2
12345678901234567890123456789012123456789012345678901 2
12345678901234567890123456789012123456789012345678901
culpability (noun). 2
12345678901234567890123456789012123456789012345678901 2

Take It to the Next Level


12345678901234567890123456789012123456789012345678901 2
12345678901234567890123456789012123456789012345678901
curmudgeon (noun) A crusty, ill-tempered person. Todd 2
12345678901234567890123456789012123456789012345678901 2
12345678901234567890123456789012123456789012345678901 22
2345678901234567890123456789012123456789012345678901
hated to drive with his Uncle Jasper, a notorious
1
12345678901234567890123456789012123456789012345678901
curmudgeon, who complained non-stop about the 2
12345678901234567890123456789012123456789012345678901
air-conditioning and Todd’s driving. curmudgeonly (ad- 2
12345678901234567890123456789012123456789012345678901 2
2345678901234567890123456789012123456789012345678901
12345678901234567890123456789012123456789012345678901 2
12345678901234567890123456789012123456789012345678901
jective). 2
12345678901234567890123456789012123456789012345678901 22
12345678901234567890123456789012123456789012345678901
cursory (adjective) Hasty and superficial. Detective Martinez 2
12345678901234567890123456789012123456789012345678901
was rebuked by his superior officer for drawing conclu- 2
12345678901234567890123456789012123456789012345678901
12345678901234567890123456789012123456789012345678901 2
sions about the murder after only a cursory examination 2
12345678901234567890123456789012123456789012345678901
12345678901234567890123456789012123456789012345678901 22
1 of the crime scene.
12345678901234567890123456789012123456789012345678901 2
12345678901234567890123456789012123456789012345678901 2
12345678901234567890123456789012123456789012345678901
debilitating (adjective) Weakening; sapping the strength of. 2
12345678901234567890123456789012123456789012345678901 2
12345678901234567890123456789012123456789012345678901
One can’t help but marvel at the courage Steven 2
2345678901234567890123456789012123456789012345678901
Hawking displays in the face of such a debilitating 2
12345678901234567890123456789012123456789012345678901
12345678901234567890123456789012123456789012345678901 2
1 2
12345678901234567890123456789012123456789012345678901
disease as ALS. debilitate (verb). 2
12345678901234567890123456789012123456789012345678901 2
12345678901234567890123456789012123456789012345678901 22
2345678901234567890123456789012123456789012345678901
12345678901234567890123456789012123456789012345678901 2
1
12345678901234567890123456789012123456789012345678901
Word Origin Latin celer 5 swift. Also found in English accelerate and celerity. 2
12345678901234567890123456789012123456789012345678901 2
12345678901234567890123456789012123456789012345678901 22
2345678901234567890123456789012123456789012345678901
decelerate (verb) To slow down. Randall didn’t decelerate
12345678901234567890123456789012123456789012345678901
enough on the winding roads, and he ended up smashing 2
12345678901234567890123456789012123456789012345678901 2
12345678901234567890123456789012123456789012345678901
his new sports utility vehicle into a guard rail. decelera- 2
1
2345678901234567890123456789012123456789012345678901
12345678901234567890123456789012123456789012345678901 2
12345678901234567890123456789012123456789012345678901
tion (noun). 2
12345678901234567890123456789012123456789012345678901 22
12345678901234567890123456789012123456789012345678901
decimation (noun) Almost complete destruction. Michael 2
1
12345678901234567890123456789012123456789012345678901
Moore’s documentary Roger and Me chronicles the 2
12345678901234567890123456789012123456789012345678901 2
12345678901234567890123456789012123456789012345678901 2
12345678901234567890123456789012123456789012345678901
decimation of the economy of Flint, Michigan, after the 2
12345678901234567890123456789012123456789012345678901 2
12345678901234567890123456789012123456789012345678901 22
2345678901234567890123456789012123456789012345678901
closing of a General Motors factory. decimate (verb).
1
12345678901234567890123456789012123456789012345678901
decry (verb) To criticize or condemn. Cigarette ads aimed at 2
12345678901234567890123456789012123456789012345678901 2
12345678901234567890123456789012123456789012345678901
youngsters have led many to decry the unfair marketing 2
2345678901234567890123456789012123456789012345678901 2
1
12345678901234567890123456789012123456789012345678901
tactics of the tobacco industry. 2
12345678901234567890123456789012123456789012345678901 22
12345678901234567890123456789012123456789012345678901 179
123456789012345678901234567890121234567890123456789012
Part III: Section Review

123456789012345678901234567890121234567890123456789012
12345678901234567890123456789012123456789012345678901 2
12345678901234567890123456789012123456789012345678901 2
12345678901234567890123456789012123456789012345678901
defamation (noun) Act of harming someone by libel or 2
12345678901234567890123456789012123456789012345678901 2
12345678901234567890123456789012123456789012345678901
slander. When the article in The National Enquirer 2
12345678901234567890123456789012123456789012345678901 2
12345678901234567890123456789012123456789012345678901
implied that she was somehow responsible for her 2
2345678901234567890123456789012123456789012345678901 2
1
12345678901234567890123456789012123456789012345678901
husband’s untimely death, Renata instructed her lawyer 2
12345678901234567890123456789012123456789012345678901
to sue the paper for defamation of character. defame 2
12345678901234567890123456789012123456789012345678901 2
12345678901234567890123456789012123456789012345678901
2345678901234567890123456789012123456789012345678901
2
2
1 (verb).
12345678901234567890123456789012123456789012345678901 2
12345678901234567890123456789012123456789012345678901 2
12345678901234567890123456789012123456789012345678901
defer (verb) To graciously submit to another’s will; to 2
12345678901234567890123456789012123456789012345678901
2345678901234567890123456789012123456789012345678901
2
2
1 delegate. In all matters relating to the children’s religious
12345678901234567890123456789012123456789012345678901 2
12345678901234567890123456789012123456789012345678901
education, Joy deferred to her husband, since he clearly 2
12345678901234567890123456789012123456789012345678901
cared more about giving them a solid grounding in 2
12345678901234567890123456789012123456789012345678901
2345678901234567890123456789012123456789012345678901
2
2
1 Judaism. deference (noun).
12345678901234567890123456789012123456789012345678901 2
12345678901234567890123456789012123456789012345678901 2
12345678901234567890123456789012123456789012345678901
deliberate (verb) To think about an issue before reaching a 2
12345678901234567890123456789012123456789012345678901 2
12345678901234567890123456789012123456789012345678901
decision. The legal pundits covering the O.J. Simpson 2
12345678901234567890123456789012123456789012345678901 2
12345678901234567890123456789012123456789012345678901
trial were shocked by the short time the jury took to 2
12345678901234567890123456789012123456789012345678901 2
12345678901234567890123456789012123456789012345678901
deliberate after a trial that lasted months. deliberation 2
2345678901234567890123456789012123456789012345678901
12345678901234567890123456789012123456789012345678901 2
1 (noun). 2
12345678901234567890123456789012123456789012345678901 2
12345678901234567890123456789012123456789012345678901 2
12345678901234567890123456789012123456789012345678901
demagogue (noun) A leader who plays dishonestly on the 2
2345678901234567890123456789012123456789012345678901
prejudices and emotions of his followers. Senator Joseph 2
12345678901234567890123456789012123456789012345678901
12345678901234567890123456789012123456789012345678901 2
12345678901234567890123456789012123456789012345678901 2
12345678901234567890123456789012123456789012345678901 2
McCarthy was a demagogue who used the paranoia and
biases of the anti-communist 1950s as a way of seizing 2
12345678901234567890123456789012123456789012345678901
12345678901234567890123456789012123456789012345678901 2
12345678901234567890123456789012123456789012345678901 2
12345678901234567890123456789012123456789012345678901 2
fame and considerable power in Washington. demagogu-
12345678901234567890123456789012123456789012345678901
ery (noun). 2
1 2
12345678901234567890123456789012123456789012345678901 2
2345678901234567890123456789012123456789012345678901
12345678901234567890123456789012123456789012345678901 2 2
1
12345678901234567890123456789012123456789012345678901
Greek demos 5 people. Also found in English democracy, 2
12345678901234567890123456789012123456789012345678901
Word Origin 2
12345678901234567890123456789012123456789012345678901 2
2345678901234567890123456789012123456789012345678901
demographic, and endemic.
12345678901234567890123456789012123456789012345678901
demographic (adjective) Relating to the statistical study of 2
2
1
12345678901234567890123456789012123456789012345678901 2
12345678901234567890123456789012123456789012345678901
population. Three demographic groups have been the 2
2345678901234567890123456789012123456789012345678901
intense focus of marketing strategy: baby boomers, born 2
12345678901234567890123456789012123456789012345678901
12345678901234567890123456789012123456789012345678901 2
1 2
12345678901234567890123456789012123456789012345678901 2
between 1946 and 1964; baby busters, or the youth
12345678901234567890123456789012123456789012345678901
market, born between 1965 and 1976; and a group 2
12345678901234567890123456789012123456789012345678901 2
12345678901234567890123456789012123456789012345678901 2
12345678901234567890123456789012123456789012345678901
referred to as tweens, those born between 1977 and 2
12345678901234567890123456789012123456789012345678901 2
12345678901234567890123456789012123456789012345678901 2
1983. demography (noun), demographics (noun).
12345678901234567890123456789012123456789012345678901 2
2345678901234567890123456789012123456789012345678901
demonstratively (adverb) Openly displaying feeling. The 2
12345678901234567890123456789012123456789012345678901
12345678901234567890123456789012123456789012345678901 2
young congressman demonstratively campaigned for 2
12345678901234567890123456789012123456789012345678901
1 2
12345678901234567890123456789012123456789012345678901
reelection, kissing every baby and hugging every senior 2
12345678901234567890123456789012123456789012345678901 2
12345678901234567890123456789012123456789012345678901
citizen at the Saugerties Chrysanthemum festival. demon- 2
12345678901234567890123456789012123456789012345678901 2
12345678901234567890123456789012123456789012345678901
strative (adjective). 2
12345678901234567890123456789012123456789012345678901 2
2345678901234567890123456789012123456789012345678901
2
1
12345678901234567890123456789012123456789012345678901 2
12345678901234567890123456789012123456789012345678901 2
12345678901234567890123456789012123456789012345678901 2
12345678901234567890123456789012123456789012345678901 2
12345678901234567890123456789012123456789012345678901 2
12345678901234567890123456789012123456789012345678901 2 2
180 12345678901234567890123456789012123456789012345678901
123456789012345678901234567890121234567890123456789012

www.petersons.com
Chapter 5: Verbal Review

123456789012345678901234567890121234567890123456789012
12345678901234567890123456789012123456789012345678901 2
12345678901234567890123456789012123456789012345678901 2
12345678901234567890123456789012123456789012345678901
derisive (adjective) Expressing ridicule or scorn. Many 2
12345678901234567890123456789012123456789012345678901 2
12345678901234567890123456789012123456789012345678901
women’s groups were derisive of Avon’s choice of a male 2
12345678901234567890123456789012123456789012345678901 2
12345678901234567890123456789012123456789012345678901
CEO, since the company derives its $5.1 billion in sales 2
2345678901234567890123456789012123456789012345678901 2
1
12345678901234567890123456789012123456789012345678901
from an army of female salespeople. derision (noun). 2
12345678901234567890123456789012123456789012345678901 2
12345678901234567890123456789012123456789012345678901 2
12345678901234567890123456789012123456789012345678901
derivative (adjective) Imitating or borrowed
2345678901234567890123456789012123456789012345678901
from a particu- 2
1 lar source. When a person first writes poetry, her poems 2
12345678901234567890123456789012123456789012345678901 2
12345678901234567890123456789012123456789012345678901 2
12345678901234567890123456789012123456789012345678901
are apt to be derivative of whatever poetry she most 2
12345678901234567890123456789012123456789012345678901
2345678901234567890123456789012123456789012345678901
2
2
1 enjoys reading. derivation (noun), derive (verb).
12345678901234567890123456789012123456789012345678901 2
12345678901234567890123456789012123456789012345678901
desiccate (verb) To dry out, to wither; to drain of vitality. 2
12345678901234567890123456789012123456789012345678901 2
12345678901234567890123456789012123456789012345678901
The long drought thoroughly desiccated our garden; 2
2345678901234567890123456789012123456789012345678901 2
1
12345678901234567890123456789012123456789012345678901
what was once a glorious Eden was now a scorched and 2
12345678901234567890123456789012123456789012345678901 2
12345678901234567890123456789012123456789012345678901
hellish wasteland. A recent spate of books has debunked 2
12345678901234567890123456789012123456789012345678901 2
12345678901234567890123456789012123456789012345678901
the myth that menopause desiccates women and af- 2
12345678901234567890123456789012123456789012345678901 2

Take It to the Next Level


12345678901234567890123456789012123456789012345678901
firmed, instead, that women often reach heights of 2
12345678901234567890123456789012123456789012345678901 2
12345678901234567890123456789012123456789012345678901
creativity in their later years. desiccant (noun), desicca- 2
2345678901234567890123456789012123456789012345678901
12345678901234567890123456789012123456789012345678901 2
1 tion (noun). 2
12345678901234567890123456789012123456789012345678901 2
12345678901234567890123456789012123456789012345678901 2
12345678901234567890123456789012123456789012345678901
despotic (adjective) Oppressive and tyrannical. During the 2
2345678901234567890123456789012123456789012345678901
despotic reign of Idi Amin in the 1970s, an estimated 2
12345678901234567890123456789012123456789012345678901
12345678901234567890123456789012123456789012345678901 2
12345678901234567890123456789012123456789012345678901 2
12345678901234567890123456789012123456789012345678901 22
200,000 Ugandans were killed. despot (noun).
12345678901234567890123456789012123456789012345678901 2
12345678901234567890123456789012123456789012345678901
desultory (adjective) Disconnected, aimless. Tina’s few des-
12345678901234567890123456789012123456789012345678901 2
ultory stabs at conversation fell flat as Guy just sat there, 2
12345678901234567890123456789012123456789012345678901
12345678901234567890123456789012123456789012345678901 22
1 stony-faced; it was a disastrous first date.
12345678901234567890123456789012123456789012345678901 2
12345678901234567890123456789012123456789012345678901 2
12345678901234567890123456789012123456789012345678901
deviate (verb) To depart from a standard or norm. Having 2
12345678901234567890123456789012123456789012345678901 2
12345678901234567890123456789012123456789012345678901
agreed upon a spending budget for the company, we 2
2345678901234567890123456789012123456789012345678901
mustn’t deviate from it; if we do, we may run out of 2
12345678901234567890123456789012123456789012345678901
12345678901234567890123456789012123456789012345678901 2
1 2
12345678901234567890123456789012123456789012345678901
money before the year ends. deviation (noun). 2
12345678901234567890123456789012123456789012345678901 2
2345678901234567890123456789012123456789012345678901
diatribe (noun) Abusive or bitter speech or writing. While 2
12345678901234567890123456789012123456789012345678901
12345678901234567890123456789012123456789012345678901 2
1 2
12345678901234567890123456789012123456789012345678901 2
angry conservatives dismissed Susan Faludi’s Backlash
12345678901234567890123456789012123456789012345678901
as a feminist diatribe, it is actually a meticulously 2
12345678901234567890123456789012123456789012345678901 2
12345678901234567890123456789012123456789012345678901
researched book. 2
12345678901234567890123456789012123456789012345678901 2
12345678901234567890123456789012123456789012345678901 2
12345678901234567890123456789012123456789012345678901
diffident (adjective) Hesitant, reserved, shy. Someone with a 2
2345678901234567890123456789012123456789012345678901
12345678901234567890123456789012123456789012345678901
diffident personality is most likely to succeed in a career 2
2
12345678901234567890123456789012123456789012345678901 2
12345678901234567890123456789012123456789012345678901 2
12345678901234567890123456789012123456789012345678901
that involves little public contact. diffidence (noun).
2
1
2345678901234567890123456789012123456789012345678901
digress (verb) To wander from the main path or the main 2
12345678901234567890123456789012123456789012345678901
12345678901234567890123456789012123456789012345678901 2
12345678901234567890123456789012123456789012345678901 2
12345678901234567890123456789012123456789012345678901 22
topic. My high school biology teacher loved to digress
1
12345678901234567890123456789012123456789012345678901
from science into personal anecdotes about his college 2
12345678901234567890123456789012123456789012345678901 2
12345678901234567890123456789012123456789012345678901
adventures. digression (noun), digressive (adjective). 2
12345678901234567890123456789012123456789012345678901 2
12345678901234567890123456789012123456789012345678901 2
12345678901234567890123456789012123456789012345678901 2
12345678901234567890123456789012123456789012345678901 2
12345678901234567890123456789012123456789012345678901 2
12345678901234567890123456789012123456789012345678901 2 181
123456789012345678901234567890121234567890123456789012
Part III: Section Review

123456789012345678901234567890121234567890123456789012
12345678901234567890123456789012123456789012345678901 2
12345678901234567890123456789012123456789012345678901 2
12345678901234567890123456789012123456789012345678901
dirge (noun) Song or hymn of grief. When Princess Diana 2
12345678901234567890123456789012123456789012345678901 2
12345678901234567890123456789012123456789012345678901
was killed in a car crash, Elton John resurrected his hit 2
12345678901234567890123456789012123456789012345678901 2
12345678901234567890123456789012123456789012345678901
song “Candle in the Wind,” rewrote it as “Good-bye 2
2345678901234567890123456789012123456789012345678901 2
1
12345678901234567890123456789012123456789012345678901
England’s Rose,” and created one of the most widely 2
12345678901234567890123456789012123456789012345678901 2
12345678901234567890123456789012123456789012345678901
heard funeral dirges of all time. 2
12345678901234567890123456789012123456789012345678901
2345678901234567890123456789012123456789012345678901
2
1 disabuse (verb) To correct a fallacy, to clarify. I hated to 2
12345678901234567890123456789012123456789012345678901 2
12345678901234567890123456789012123456789012345678901 2
12345678901234567890123456789012123456789012345678901
disabuse Filbert, who is a passionate collector of musical 2
12345678901234567890123456789012123456789012345678901
2345678901234567890123456789012123456789012345678901
2
2
1 trivia, but I had to tell him that the Monkees hadn’t
12345678901234567890123456789012123456789012345678901 2
12345678901234567890123456789012123456789012345678901
played their own instruments on almost all of their 2
12345678901234567890123456789012123456789012345678901
albums. 2
12345678901234567890123456789012123456789012345678901
2345678901234567890123456789012123456789012345678901
2
2
1
12345678901234567890123456789012123456789012345678901
disburse (verb) To pay out or distribute (funds or property). 2
12345678901234567890123456789012123456789012345678901 2
12345678901234567890123456789012123456789012345678901
Jaime was flabbergasted when his father’s will disbursed 2
12345678901234567890123456789012123456789012345678901 2
12345678901234567890123456789012123456789012345678901
all of the old man’s financial assets to Raymundo and 2
12345678901234567890123456789012123456789012345678901 2
12345678901234567890123456789012123456789012345678901
left him with only a few sticks of furniture. disbursement 2
12345678901234567890123456789012123456789012345678901 2
12345678901234567890123456789012123456789012345678901
(noun). 2
12345678901234567890123456789012123456789012345678901 2
2345678901234567890123456789012123456789012345678901
1 discern (verb) To detect, notice, or observe. With difficulty, 2
12345678901234567890123456789012123456789012345678901 2
12345678901234567890123456789012123456789012345678901 2
12345678901234567890123456789012123456789012345678901
I could discern the shape of a whale off the starboard 2
2345678901234567890123456789012123456789012345678901
bow, but it was too far away to determine its size or 2
12345678901234567890123456789012123456789012345678901
12345678901234567890123456789012123456789012345678901 2
12345678901234567890123456789012123456789012345678901 2
12345678901234567890123456789012123456789012345678901 2
species. discernment (noun).
12345678901234567890123456789012123456789012345678901 2
discordant (adjective) Characterized by conflict. Stories and 2
12345678901234567890123456789012123456789012345678901
12345678901234567890123456789012123456789012345678901 2
films about discordant relationships that resolve them- 2
12345678901234567890123456789012123456789012345678901
12345678901234567890123456789012123456789012345678901 2
1 selves happily are always more interesting than stories 2
12345678901234567890123456789012123456789012345678901 2
12345678901234567890123456789012123456789012345678901
about content couples who simply stay content. discor- 2
12345678901234567890123456789012123456789012345678901 2
12345678901234567890123456789012123456789012345678901
dance (noun). 2
12345678901234567890123456789012123456789012345678901
2345678901234567890123456789012123456789012345678901
2
discourse (noun) Formal and orderly exchange of ideas, a 2
12345678901234567890123456789012123456789012345678901
12345678901234567890123456789012123456789012345678901 2
1 2
12345678901234567890123456789012123456789012345678901
discussion. In the late twentieth century, cloning and 2
12345678901234567890123456789012123456789012345678901 2
12345678901234567890123456789012123456789012345678901 2
2345678901234567890123456789012123456789012345678901
other feats of genetic engineering became popular topics
12345678901234567890123456789012123456789012345678901 2
1 of public discourse. discursive (adjective). 2
12345678901234567890123456789012123456789012345678901 2
12345678901234567890123456789012123456789012345678901 2
12345678901234567890123456789012123456789012345678901
2345678901234567890123456789012123456789012345678901
2
2
1 Latin credere 5 to believe. Also found in English credential,
12345678901234567890123456789012123456789012345678901
Word Origin credible, credit, credo, credulous, and incredible. 2
12345678901234567890123456789012123456789012345678901 2
12345678901234567890123456789012123456789012345678901 2
2345678901234567890123456789012123456789012345678901
discredit (verb) To cause disbelief in the accuracy of some 2
12345678901234567890123456789012123456789012345678901
12345678901234567890123456789012123456789012345678901 2
statement or the reliability of a person. Although many 2
12345678901234567890123456789012123456789012345678901
12345678901234567890123456789012123456789012345678901 2
1 people still believe in UFOs, among scientists the reports 2
12345678901234567890123456789012123456789012345678901 2
12345678901234567890123456789012123456789012345678901
of “alien encounters” have been thoroughly discredited. 2
12345678901234567890123456789012123456789012345678901 2
12345678901234567890123456789012123456789012345678901 2
12345678901234567890123456789012123456789012345678901 2
discreet (adjective) Showing good judgment in speech and
2345678901234567890123456789012123456789012345678901
behavior. Be discreet when discussing confidential busi- 2
12345678901234567890123456789012123456789012345678901
1 2
12345678901234567890123456789012123456789012345678901 2
12345678901234567890123456789012123456789012345678901 2
ness matters—don’t talk among strangers on the eleva-
12345678901234567890123456789012123456789012345678901 2
12345678901234567890123456789012123456789012345678901
tor, for example. discretion (noun). 2
12345678901234567890123456789012123456789012345678901 2
12345678901234567890123456789012123456789012345678901 2 2
182 12345678901234567890123456789012123456789012345678901
123456789012345678901234567890121234567890123456789012

www.petersons.com
Chapter 5: Verbal Review

123456789012345678901234567890121234567890123456789012
12345678901234567890123456789012123456789012345678901 2
12345678901234567890123456789012123456789012345678901 2
12345678901234567890123456789012123456789012345678901
discrete (adjective) Separate, unconnected. Canadians get 2
12345678901234567890123456789012123456789012345678901 2
12345678901234567890123456789012123456789012345678901
peeved when people can’t seem to distinguish between 2
12345678901234567890123456789012123456789012345678901 2
12345678901234567890123456789012123456789012345678901
Canada and the United States, forgetting that Canada 2
2345678901234567890123456789012123456789012345678901 2
1
12345678901234567890123456789012123456789012345678901
has its own discrete heritage and culture. 2
12345678901234567890123456789012123456789012345678901 2
12345678901234567890123456789012123456789012345678901 2
12345678901234567890123456789012123456789012345678901
disparity (noun) Difference in
2345678901234567890123456789012123456789012345678901
quality or kind. There is often 2
1 a disparity between the kind of serious, high-quality 2
12345678901234567890123456789012123456789012345678901 2
12345678901234567890123456789012123456789012345678901 2
12345678901234567890123456789012123456789012345678901
television people say they want and the low-brow 2
12345678901234567890123456789012123456789012345678901
2345678901234567890123456789012123456789012345678901
2
2
1 programs they actually watch. disparate (adjective).
12345678901234567890123456789012123456789012345678901 2
12345678901234567890123456789012123456789012345678901 2
12345678901234567890123456789012123456789012345678901 2
12345678901234567890123456789012123456789012345678901
Latin simulare 5 to resemble. Also
2345678901234567890123456789012123456789012345678901
found in English semblance, 2
2
1
Word Origin
12345678901234567890123456789012123456789012345678901
similarity, simulacrum, simultaneous, and verisimilitude.
2
12345678901234567890123456789012123456789012345678901 2
12345678901234567890123456789012123456789012345678901
dissemble (verb) To pretend, to simulate. When the police 2
12345678901234567890123456789012123456789012345678901 2
12345678901234567890123456789012123456789012345678901
asked whether Nancy knew anything about the crime, 2
12345678901234567890123456789012123456789012345678901 2

Take It to the Next Level


12345678901234567890123456789012123456789012345678901
she dissembled innocence. 2
12345678901234567890123456789012123456789012345678901 2
12345678901234567890123456789012123456789012345678901 2
12345678901234567890123456789012123456789012345678901 22
2345678901234567890123456789012123456789012345678901
dissipate (verb) To spread out or scatter. The windows and
1
12345678901234567890123456789012123456789012345678901
doors were opened, allowing the smoke that had filled 2
12345678901234567890123456789012123456789012345678901 2
12345678901234567890123456789012123456789012345678901
the room to dissipate. dissipation (noun). 2
12345678901234567890123456789012123456789012345678901 22
2345678901234567890123456789012123456789012345678901
12345678901234567890123456789012123456789012345678901
dissonance (noun) Lack of music harmony; lack of agree- 2
12345678901234567890123456789012123456789012345678901
ment between ideas. Most modern music is character- 2
12345678901234567890123456789012123456789012345678901
2
12345678901234567890123456789012123456789012345678901
ized by dissonance, which many listeners find hard to 2
12345678901234567890123456789012123456789012345678901
12345678901234567890123456789012123456789012345678901 2
enjoy. There is a noticeable dissonance between two 2
12345678901234567890123456789012123456789012345678901
12345678901234567890123456789012123456789012345678901 22
1 common beliefs of most conservatives: their faith in
12345678901234567890123456789012123456789012345678901 2
12345678901234567890123456789012123456789012345678901
unfettered free markets and their preference for tradi- 2
12345678901234567890123456789012123456789012345678901 2
12345678901234567890123456789012123456789012345678901
tional social values. dissonant (adjective). 2
12345678901234567890123456789012123456789012345678901
2345678901234567890123456789012123456789012345678901
2
distillation (noun) Something distilled, an essence or ex- 2
12345678901234567890123456789012123456789012345678901
12345678901234567890123456789012123456789012345678901 2
1 2
12345678901234567890123456789012123456789012345678901
tract. In chemistry, a process that drives gas or vapor 2
12345678901234567890123456789012123456789012345678901 2
12345678901234567890123456789012123456789012345678901 22
2345678901234567890123456789012123456789012345678901
from liquids or solids. Sharon Olds’s poems are power-
12345678901234567890123456789012123456789012345678901
ful distillations of motherhood and other primal experi- 2
1
12345678901234567890123456789012123456789012345678901
ences. In Mrs. Hornmeister’s chemistry class, our first 2
12345678901234567890123456789012123456789012345678901 2
2345678901234567890123456789012123456789012345678901
experiment was to create a distillation of carbon gas 2
12345678901234567890123456789012123456789012345678901
12345678901234567890123456789012123456789012345678901 2
12345678901234567890123456789012123456789012345678901 2
12345678901234567890123456789012123456789012345678901 22
from wood. distill (verb).
1
12345678901234567890123456789012123456789012345678901
diverge (verb) To move in different directions. Frost’s poem 2
12345678901234567890123456789012123456789012345678901 2
12345678901234567890123456789012123456789012345678901
“The Road Not Taken” tells of the choice he made when 2
12345678901234567890123456789012123456789012345678901 2
12345678901234567890123456789012123456789012345678901
“Two roads diverged in a yellow wood.” divergence 2
2345678901234567890123456789012123456789012345678901
12345678901234567890123456789012123456789012345678901 2
12345678901234567890123456789012123456789012345678901
(noun), divergent (adjective). 2
12345678901234567890123456789012123456789012345678901 22
12345678901234567890123456789012123456789012345678901
diversify (verb) To balance by adding variety. Any financial 2
1
12345678901234567890123456789012123456789012345678901
manager will recommend that you diversify your stock 2
12345678901234567890123456789012123456789012345678901 2
12345678901234567890123456789012123456789012345678901 2
12345678901234567890123456789012123456789012345678901 2
portfolio by holding some less-volatile blue-chip stocks
12345678901234567890123456789012123456789012345678901
along with more growth-oriented technology issues. 2
12345678901234567890123456789012123456789012345678901 2
12345678901234567890123456789012123456789012345678901 2
12345678901234567890123456789012123456789012345678901 22
diversification (noun), diversified (adjective).
12345678901234567890123456789012123456789012345678901 183
123456789012345678901234567890121234567890123456789012
Part III: Section Review

123456789012345678901234567890121234567890123456789012
12345678901234567890123456789012123456789012345678901 2
12345678901234567890123456789012123456789012345678901 2
12345678901234567890123456789012123456789012345678901
divest (verb) To rid (oneself) or be freed of property, 2
12345678901234567890123456789012123456789012345678901 2
12345678901234567890123456789012123456789012345678901
authority, or title. In order to turn around its ailing 2
12345678901234567890123456789012123456789012345678901 2
12345678901234567890123456789012123456789012345678901
company and concentrate on imaging, Eastman Kodak 2
2345678901234567890123456789012123456789012345678901 2
1
12345678901234567890123456789012123456789012345678901
divested itself of peripheral businesses in the areas of 2
12345678901234567890123456789012123456789012345678901
household products, clinical diagnostics, and pharma- 2
12345678901234567890123456789012123456789012345678901 2
12345678901234567890123456789012123456789012345678901
2345678901234567890123456789012123456789012345678901
2
2
1 ceuticals. divestiture (noun).
12345678901234567890123456789012123456789012345678901 2
12345678901234567890123456789012123456789012345678901 2
12345678901234567890123456789012123456789012345678901
divulge (verb) To reveal. The people who count the votes for 2
12345678901234567890123456789012123456789012345678901
2345678901234567890123456789012123456789012345678901
2
2
1 the Oscar awards are under strict orders not to divulge
12345678901234567890123456789012123456789012345678901 2
12345678901234567890123456789012123456789012345678901
the names of the winners. 2
12345678901234567890123456789012123456789012345678901 2
12345678901234567890123456789012123456789012345678901
dogmatic (adjective) Holding firmly to a particular set of 2
2345678901234567890123456789012123456789012345678901 2
1
12345678901234567890123456789012123456789012345678901
beliefs with little or no basis. Believers in Marxist 2
12345678901234567890123456789012123456789012345678901 2
12345678901234567890123456789012123456789012345678901
doctrine tend to be dogmatic, ignoring evidence that 2
12345678901234567890123456789012123456789012345678901 2
12345678901234567890123456789012123456789012345678901
contradicts their beliefs or explaining it away. dogma 2
12345678901234567890123456789012123456789012345678901 2
12345678901234567890123456789012123456789012345678901
(noun), dogmatism (noun). 2
12345678901234567890123456789012123456789012345678901 2
12345678901234567890123456789012123456789012345678901 2
12345678901234567890123456789012123456789012345678901 2
2345678901234567890123456789012123456789012345678901
dolt (noun) A stupid or foolish person. Due to his frequent
1 verbal blunders, politician Dan Quayle was widely 2
12345678901234567890123456789012123456789012345678901 2
12345678901234567890123456789012123456789012345678901 2
12345678901234567890123456789012123456789012345678901
considered to be a dolt. 2
12345678901234567890123456789012123456789012345678901 2
2345678901234567890123456789012123456789012345678901
12345678901234567890123456789012123456789012345678901 2
12345678901234567890123456789012123456789012345678901 2
12345678901234567890123456789012123456789012345678901
Word Origin Latin dormire 5 to sleep. Also found in English dormitory. 2
12345678901234567890123456789012123456789012345678901 2
12345678901234567890123456789012123456789012345678901
dormant (adjective) Temporarily inactive, as if asleep. An 2
2
12345678901234567890123456789012123456789012345678901
12345678901234567890123456789012123456789012345678901
eruption of Mt. Rainier, a dormant volcano in Washing- 2
2
12345678901234567890123456789012123456789012345678901
1 ton state, would cause massive, life-threatening mud 2
12345678901234567890123456789012123456789012345678901 2
12345678901234567890123456789012123456789012345678901
slides in the surrounding area. Bill preferred to think 2
12345678901234567890123456789012123456789012345678901 2
12345678901234567890123456789012123456789012345678901 2
12345678901234567890123456789012123456789012345678901
that his sex drive was dormant
2345678901234567890123456789012123456789012345678901
rather than extinct. 2
2
12345678901234567890123456789012123456789012345678901
12345678901234567890123456789012123456789012345678901
dormancy (noun). 2
1 2
12345678901234567890123456789012123456789012345678901 2
12345678901234567890123456789012123456789012345678901
dross (noun) Something that is trivial or inferior; an 2
2345678901234567890123456789012123456789012345678901
impurity. As a reader for the Paris Review, Julia spent 2
12345678901234567890123456789012123456789012345678901
12345678901234567890123456789012123456789012345678901 2
1 most of her time sifting through piles of manuscripts to 2
12345678901234567890123456789012123456789012345678901 2
12345678901234567890123456789012123456789012345678901
separate the extraordinary poems from the dross. 2
12345678901234567890123456789012123456789012345678901 2
12345678901234567890123456789012123456789012345678901 2
12345678901234567890123456789012123456789012345678901
dubious (adjective) Doubtful, uncertain. Despite the chair- 2
12345678901234567890123456789012123456789012345678901 2
12345678901234567890123456789012123456789012345678901
man’s attempts to convince the committee members that 2
2345678901234567890123456789012123456789012345678901
12345678901234567890123456789012123456789012345678901
his plan would succeed, most of them remained dubious. 2
2
12345678901234567890123456789012123456789012345678901 2
12345678901234567890123456789012123456789012345678901 2
12345678901234567890123456789012123456789012345678901
dubiety (noun).
2
1
2345678901234567890123456789012123456789012345678901
dupe (noun) Someone who is easily cheated. My cousin 2
12345678901234567890123456789012123456789012345678901
12345678901234567890123456789012123456789012345678901 2
12345678901234567890123456789012123456789012345678901 2
12345678901234567890123456789012123456789012345678901 2
Ravi is such a dupe; he actually gets excited when he
1 receives those envelopes saying “Ravi Murtugudde, you 2
12345678901234567890123456789012123456789012345678901 2
12345678901234567890123456789012123456789012345678901 2
12345678901234567890123456789012123456789012345678901
may have won a million dollars,” and he even goes so far 2
12345678901234567890123456789012123456789012345678901 2
12345678901234567890123456789012123456789012345678901
as to try claiming his prize.
2345678901234567890123456789012123456789012345678901
2
2
1
12345678901234567890123456789012123456789012345678901 2
12345678901234567890123456789012123456789012345678901 2 2
184 12345678901234567890123456789012123456789012345678901
123456789012345678901234567890121234567890123456789012

www.petersons.com
Chapter 5: Verbal Review

123456789012345678901234567890121234567890123456789012
12345678901234567890123456789012123456789012345678901 2
12345678901234567890123456789012123456789012345678901 2
12345678901234567890123456789012123456789012345678901
eccentricity (noun) Odd or whimsical behavior. Rock star 2
12345678901234567890123456789012123456789012345678901 2
12345678901234567890123456789012123456789012345678901
Michael Jackson is now better known for his offstage 2
12345678901234567890123456789012123456789012345678901 2
12345678901234567890123456789012123456789012345678901
eccentricities—such as sleeping in an oxygen tank, 2
2345678901234567890123456789012123456789012345678901 2
1
12345678901234567890123456789012123456789012345678901
wearing a surgical mask, and building his own theme 2
12345678901234567890123456789012123456789012345678901
park—than for his onstage performances. eccentric 2
12345678901234567890123456789012123456789012345678901 2
12345678901234567890123456789012123456789012345678901
2345678901234567890123456789012123456789012345678901
2
2
1 (adjective).
12345678901234567890123456789012123456789012345678901 2
12345678901234567890123456789012123456789012345678901 2
12345678901234567890123456789012123456789012345678901
edifying (adjective) Instructive, enlightening. Ariel would 2
12345678901234567890123456789012123456789012345678901
2345678901234567890123456789012123456789012345678901
2
2
1 never admit it to her high-brow friends, but she found
12345678901234567890123456789012123456789012345678901 2
12345678901234567890123456789012123456789012345678901
the latest self-help best-seller edifying and actually 2
12345678901234567890123456789012123456789012345678901
helpful. edification (noun), edify (verb). 2
12345678901234567890123456789012123456789012345678901
2345678901234567890123456789012123456789012345678901
2
2
1
12345678901234567890123456789012123456789012345678901 2
12345678901234567890123456789012123456789012345678901 2
12345678901234567890123456789012123456789012345678901
Latin facere 5 to do. Also found in English facility, factor, facsimile, 2
12345678901234567890123456789012123456789012345678901
Word Origin 2
12345678901234567890123456789012123456789012345678901 22
2345678901234567890123456789012123456789012345678901
and faculty.
12345678901234567890123456789012123456789012345678901

Take It to the Next Level


efficacy (noun) The power to produce the desired effect. 2
12345678901234567890123456789012123456789012345678901
12345678901234567890123456789012123456789012345678901
While teams have been enormously popular in the 2
2
12345678901234567890123456789012123456789012345678901
1 workplace, there are some who now question their 2
12345678901234567890123456789012123456789012345678901 2
12345678901234567890123456789012123456789012345678901 2
12345678901234567890123456789012123456789012345678901
efficacy and say that “one head is better than ten.” 2
12345678901234567890123456789012123456789012345678901 2
12345678901234567890123456789012123456789012345678901 22
2345678901234567890123456789012123456789012345678901
efficacious (noun).
12345678901234567890123456789012123456789012345678901 2
12345678901234567890123456789012123456789012345678901
effrontery (noun) Shameless boldness. The sports world 2
12345678901234567890123456789012123456789012345678901
was shocked when a pro basketball player had the 2
12345678901234567890123456789012123456789012345678901
12345678901234567890123456789012123456789012345678901 2
12345678901234567890123456789012123456789012345678901 2
12345678901234567890123456789012123456789012345678901 22
effrontery to choke the head coach of his team during a
12345678901234567890123456789012123456789012345678901
practice session.
2
1
12345678901234567890123456789012123456789012345678901 22
2345678901234567890123456789012123456789012345678901
12345678901234567890123456789012123456789012345678901
elaborate (verb) To expand upon something; develop. One
2
1
12345678901234567890123456789012123456789012345678901
characteristic of the best essayists is their ability to 2
12345678901234567890123456789012123456789012345678901 2
12345678901234567890123456789012123456789012345678901
elaborate ideas through examples, lists, similes, small 2
12345678901234567890123456789012123456789012345678901 2
12345678901234567890123456789012123456789012345678901
variations, and even exaggerations. elaborate (adjec- 2
12345678901234567890123456789012123456789012345678901 2
12345678901234567890123456789012123456789012345678901
tive), elaboration (noun). 2
12345678901234567890123456789012123456789012345678901 22
2345678901234567890123456789012123456789012345678901
12345678901234567890123456789012123456789012345678901
elegy (noun) A song or poem expressing sorrow. Thomas 2
1
12345678901234567890123456789012123456789012345678901
Gray’s “Elegy Written in a Country Churchyard,” one 2
12345678901234567890123456789012123456789012345678901 2
2345678901234567890123456789012123456789012345678901
of the most famous elegies in Western literature, mourns 2
12345678901234567890123456789012123456789012345678901
12345678901234567890123456789012123456789012345678901 2
12345678901234567890123456789012123456789012345678901 2
12345678901234567890123456789012123456789012345678901 22
the unsung, inglorious lives of the souls buried in an
1
12345678901234567890123456789012123456789012345678901
obscure, rustic graveyard. elegaic (adjective). 2
12345678901234567890123456789012123456789012345678901 2
12345678901234567890123456789012123456789012345678901
embellish (verb) To enhance or exaggerate; to decorate. The 2
12345678901234567890123456789012123456789012345678901 2
12345678901234567890123456789012123456789012345678901
long-married couple told their stories in tandem, with 2
2345678901234567890123456789012123456789012345678901
12345678901234567890123456789012123456789012345678901
the husband outlining the plot and the wife embellishing 2
2
12345678901234567890123456789012123456789012345678901 2
12345678901234567890123456789012123456789012345678901 2
12345678901234567890123456789012123456789012345678901
it with colorful details. embellished (adjective).
2
1
12345678901234567890123456789012123456789012345678901 22
2345678901234567890123456789012123456789012345678901
1
12345678901234567890123456789012123456789012345678901 2
12345678901234567890123456789012123456789012345678901 2
12345678901234567890123456789012123456789012345678901 2
12345678901234567890123456789012123456789012345678901 2
12345678901234567890123456789012123456789012345678901 2
12345678901234567890123456789012123456789012345678901 2
12345678901234567890123456789012123456789012345678901 2 185
123456789012345678901234567890121234567890123456789012
Part III: Section Review

123456789012345678901234567890121234567890123456789012
12345678901234567890123456789012123456789012345678901 2
12345678901234567890123456789012123456789012345678901 2
12345678901234567890123456789012123456789012345678901
embezzle (verb) To steal money or property that has been 2
12345678901234567890123456789012123456789012345678901 2
12345678901234567890123456789012123456789012345678901
entrusted to your care. The church treasurer was found 2
12345678901234567890123456789012123456789012345678901 2
12345678901234567890123456789012123456789012345678901
to have embezzled thousands of dollars by writing 2
2345678901234567890123456789012123456789012345678901 2
1
12345678901234567890123456789012123456789012345678901
phony checks on the church bank account. embezzle- 2
12345678901234567890123456789012123456789012345678901 2
12345678901234567890123456789012123456789012345678901
ment (noun). 2
12345678901234567890123456789012123456789012345678901
2345678901234567890123456789012123456789012345678901
2
1 emollient (noun) Something that softens or soothes. She 2
12345678901234567890123456789012123456789012345678901 2
12345678901234567890123456789012123456789012345678901 2
12345678901234567890123456789012123456789012345678901
used a hand cream as an emollient on her dry, 2
12345678901234567890123456789012123456789012345678901
2345678901234567890123456789012123456789012345678901
2
2
1 work-roughened hands. emollient (adjective).
12345678901234567890123456789012123456789012345678901 2
12345678901234567890123456789012123456789012345678901
empirical (adjective) Based on experience or personal obser- 2
12345678901234567890123456789012123456789012345678901 2
12345678901234567890123456789012123456789012345678901
vation. Although many people believe in ESP, scientists 2
2345678901234567890123456789012123456789012345678901 2
1
12345678901234567890123456789012123456789012345678901
have found no empirical evidence of its existence. 2
12345678901234567890123456789012123456789012345678901 2
12345678901234567890123456789012123456789012345678901
empiricism (noun). 2
12345678901234567890123456789012123456789012345678901 2
2345678901234567890123456789012123456789012345678901
12345678901234567890123456789012123456789012345678901
emulate (verb) To imitate or copy. The British band Oasis is 2
2
12345678901234567890123456789012123456789012345678901
quite open about their desire to emulate their idols, the 2
12345678901234567890123456789012123456789012345678901
12345678901234567890123456789012123456789012345678901 2
12345678901234567890123456789012123456789012345678901 2
12345678901234567890123456789012123456789012345678901 2
Beatles. emulation (noun).
2
1
12345678901234567890123456789012123456789012345678901 2
12345678901234567890123456789012123456789012345678901
encomium (noun) A formal expression of praise. For many 2
12345678901234567890123456789012123456789012345678901 2
12345678901234567890123456789012123456789012345678901 2
2345678901234567890123456789012123456789012345678901
filmmakers, winning the Palm d’Or at the Cannes Film
12345678901234567890123456789012123456789012345678901 2
12345678901234567890123456789012123456789012345678901
Festival is considered the highest encomium. 2
12345678901234567890123456789012123456789012345678901 2
enervate (verb) To reduce the energy or strength of someone 2
12345678901234567890123456789012123456789012345678901
12345678901234567890123456789012123456789012345678901 2
or something. The stress of the operation left her feeling 2
12345678901234567890123456789012123456789012345678901
12345678901234567890123456789012123456789012345678901 2
12345678901234567890123456789012123456789012345678901
enervated for about two weeks. enervation (noun). 2
1 2
2345678901234567890123456789012123456789012345678901
engender (verb) To produce, to cause. Countless disagree- 2
12345678901234567890123456789012123456789012345678901
12345678901234567890123456789012123456789012345678901 2
1 2
12345678901234567890123456789012123456789012345678901 2
ments over the proper use of national forests and
12345678901234567890123456789012123456789012345678901
parklands have engendered feelings of hostility between 2
12345678901234567890123456789012123456789012345678901 2
12345678901234567890123456789012123456789012345678901 2
12345678901234567890123456789012123456789012345678901 2
ranchers and environmentalists.
12345678901234567890123456789012123456789012345678901 2
12345678901234567890123456789012123456789012345678901 2
12345678901234567890123456789012123456789012345678901 2
2345678901234567890123456789012123456789012345678901
enhance (verb) To improve in value or quality. New kitchen
1 appliances will enhance your house and increase the 2
12345678901234567890123456789012123456789012345678901 2
12345678901234567890123456789012123456789012345678901
amount of money you’ll make when you sell it. 2
12345678901234567890123456789012123456789012345678901 2
12345678901234567890123456789012123456789012345678901
enhancement (noun). 2
12345678901234567890123456789012123456789012345678901 2
12345678901234567890123456789012123456789012345678901 2
12345678901234567890123456789012123456789012345678901
enigmatic (adjective) Puzzling, mysterious. Alain Resnais’ 2
12345678901234567890123456789012123456789012345678901 2
12345678901234567890123456789012123456789012345678901
enigmatic film Last Year at Marienbad sets up a puzzle 2
12345678901234567890123456789012123456789012345678901 2
12345678901234567890123456789012123456789012345678901
that is never resolved: A man meets a woman at a hotel 2
12345678901234567890123456789012123456789012345678901
and believes he once had an affair with her—-or did he? 2
12345678901234567890123456789012123456789012345678901 2
2345678901234567890123456789012123456789012345678901
12345678901234567890123456789012123456789012345678901 2
12345678901234567890123456789012123456789012345678901
enigma (noun). 2
12345678901234567890123456789012123456789012345678901 2
1 enmity (noun) Hatred, hostility, ill will. Long-standing 2
12345678901234567890123456789012123456789012345678901 2
2345678901234567890123456789012123456789012345678901
enmity, like that between the Protestants and Catholics 2
12345678901234567890123456789012123456789012345678901
1 2
12345678901234567890123456789012123456789012345678901 2
12345678901234567890123456789012123456789012345678901 2
in Northern Ireland, is difficult to overcome.
12345678901234567890123456789012123456789012345678901 2
12345678901234567890123456789012123456789012345678901 2
12345678901234567890123456789012123456789012345678901 2
12345678901234567890123456789012123456789012345678901 2 2
186 12345678901234567890123456789012123456789012345678901
123456789012345678901234567890121234567890123456789012

www.petersons.com
Chapter 5: Verbal Review

123456789012345678901234567890121234567890123456789012
12345678901234567890123456789012123456789012345678901 2
12345678901234567890123456789012123456789012345678901 2
12345678901234567890123456789012123456789012345678901
ensure (verb) To make certain; to guarantee. In order to 2
12345678901234567890123456789012123456789012345678901 2
12345678901234567890123456789012123456789012345678901
ensure a sufficient crop of programmers and engineers 2
12345678901234567890123456789012123456789012345678901 2
12345678901234567890123456789012123456789012345678901
for the future, the United States needs to raise the quality 2
2345678901234567890123456789012123456789012345678901 2
1
12345678901234567890123456789012123456789012345678901
of its math and science schooling. 2
12345678901234567890123456789012123456789012345678901 2
12345678901234567890123456789012123456789012345678901 2
12345678901234567890123456789012123456789012345678901
2345678901234567890123456789012123456789012345678901
2
2
1
12345678901234567890123456789012123456789012345678901
Word
Latin aequus 5 equal. Also found in English equality, equanimity,
Origin and equation. 2
12345678901234567890123456789012123456789012345678901 2
12345678901234567890123456789012123456789012345678901 2
12345678901234567890123456789012123456789012345678901
2345678901234567890123456789012123456789012345678901
2
2
1 equable (adjective) Steady, uniform. While many people
12345678901234567890123456789012123456789012345678901 2
12345678901234567890123456789012123456789012345678901
can’t see how Helena could possibly be attracted to 2
12345678901234567890123456789012123456789012345678901
“Boring Bruno,” his equable nature is the perfect 2
12345678901234567890123456789012123456789012345678901
2345678901234567890123456789012123456789012345678901
2
2
1 complement to her volatile personality.
12345678901234567890123456789012123456789012345678901 2
12345678901234567890123456789012123456789012345678901 2
12345678901234567890123456789012123456789012345678901
equivocate (verb) To use misleading or intentionally confus- 2
12345678901234567890123456789012123456789012345678901 2
12345678901234567890123456789012123456789012345678901
ing language. When Pedro pressed Renee for an answer 2
12345678901234567890123456789012123456789012345678901 2

Take It to the Next Level


12345678901234567890123456789012123456789012345678901
to his marriage proposal, she equivocated by saying, 2
12345678901234567890123456789012123456789012345678901 2
12345678901234567890123456789012123456789012345678901
“I’ve just got to know when your Mercedes will be out 2
2345678901234567890123456789012123456789012345678901
of the shop!” equivocal (adjective), equivocation (noun). 2
12345678901234567890123456789012123456789012345678901
1 2
12345678901234567890123456789012123456789012345678901 2
12345678901234567890123456789012123456789012345678901 2
12345678901234567890123456789012123456789012345678901
epicure (noun) Someone who appreciates fine wine and fine 2
2345678901234567890123456789012123456789012345678901
food, a gourmand. M.F.K. Fisher, a famous epicure, 2
12345678901234567890123456789012123456789012345678901
12345678901234567890123456789012123456789012345678901 2
12345678901234567890123456789012123456789012345678901 2
12345678901234567890123456789012123456789012345678901 22
begins her book The Gastronomical Me by saying,
12345678901234567890123456789012123456789012345678901
“There is a communion of more than bodies when bread 2
12345678901234567890123456789012123456789012345678901
12345678901234567890123456789012123456789012345678901 2
is broken and wine is drunk.” epicurean (adjective).
12345678901234567890123456789012123456789012345678901 2
12345678901234567890123456789012123456789012345678901 22
1 epithet (noun) Term or words used to characterize a person
12345678901234567890123456789012123456789012345678901 2
12345678901234567890123456789012123456789012345678901
or thing, often in a disparaging way. In her recorded 2
12345678901234567890123456789012123456789012345678901 2
12345678901234567890123456789012123456789012345678901
phone conversations with Linda Tripp, Monica Lewinsky 2
12345678901234567890123456789012123456789012345678901 2
12345678901234567890123456789012123456789012345678901
is said to have referred to President Clinton by a number 2
12345678901234567890123456789012123456789012345678901 2
12345678901234567890123456789012123456789012345678901
of epithets, including “The Creep” and “The Big He.” 2
12345678901234567890123456789012123456789012345678901 2
12345678901234567890123456789012123456789012345678901
epithetical (adjective). 2
12345678901234567890123456789012123456789012345678901 22
2345678901234567890123456789012123456789012345678901
12345678901234567890123456789012123456789012345678901 2
1
12345678901234567890123456789012123456789012345678901 2
12345678901234567890123456789012123456789012345678901
Word Origin Latin radix 5 root. Also found in English radical. 2
12345678901234567890123456789012123456789012345678901 22
2345678901234567890123456789012123456789012345678901
12345678901234567890123456789012123456789012345678901
eradicate (verb) To destroy completely. American society
12345678901234567890123456789012123456789012345678901 22
12345678901234567890123456789012123456789012345678901
has failed to eradicate racism, although some of its worst
2
1
2345678901234567890123456789012123456789012345678901
12345678901234567890123456789012123456789012345678901
effects have been reduced. eradication (noun). 2
12345678901234567890123456789012123456789012345678901 22
12345678901234567890123456789012123456789012345678901
erudition (noun) Extensive knowledge, usually acquired 2
12345678901234567890123456789012123456789012345678901
1 from books. When Dorothea first saw Mr. Casaubon’s 2
12345678901234567890123456789012123456789012345678901 2
12345678901234567890123456789012123456789012345678901 2
12345678901234567890123456789012123456789012345678901
voluminous library she was awed, but after their 2
12345678901234567890123456789012123456789012345678901 2
12345678901234567890123456789012123456789012345678901 2
marriage she quickly realized that erudition is no
2345678901234567890123456789012123456789012345678901
12345678901234567890123456789012123456789012345678901 2
1 substitute for originality. erudite (adjective). 2
12345678901234567890123456789012123456789012345678901 2
12345678901234567890123456789012123456789012345678901 2
12345678901234567890123456789012123456789012345678901 2
12345678901234567890123456789012123456789012345678901 2
12345678901234567890123456789012123456789012345678901 2
12345678901234567890123456789012123456789012345678901 2
12345678901234567890123456789012123456789012345678901 2 187
123456789012345678901234567890121234567890123456789012
Part III: Section Review

123456789012345678901234567890121234567890123456789012
12345678901234567890123456789012123456789012345678901 2
12345678901234567890123456789012123456789012345678901 2
12345678901234567890123456789012123456789012345678901
esoterica (noun) Items of interest to a select group. The fish 2
12345678901234567890123456789012123456789012345678901 2
12345678901234567890123456789012123456789012345678901
symposium at St. Antony’s College in Oxford explored 2
12345678901234567890123456789012123456789012345678901 2
12345678901234567890123456789012123456789012345678901
all manner of esoterica relating to fish, as is evidenced in 2
2345678901234567890123456789012123456789012345678901 2
1
12345678901234567890123456789012123456789012345678901
presentations such as “The Buoyant Slippery Lipids of 2
12345678901234567890123456789012123456789012345678901
the Escolar and Orange Roughy” or “Food on Board 2
12345678901234567890123456789012123456789012345678901 2
12345678901234567890123456789012123456789012345678901
2345678901234567890123456789012123456789012345678901
2
2
1 Whale Ships—from the Inedible to the Incredible.”
12345678901234567890123456789012123456789012345678901 2
12345678901234567890123456789012123456789012345678901
esoteric (adjective). 2
12345678901234567890123456789012123456789012345678901 2
12345678901234567890123456789012123456789012345678901
2345678901234567890123456789012123456789012345678901
2
2
1 espouse (verb) To take up as a cause; to adopt. No politician
12345678901234567890123456789012123456789012345678901 2
12345678901234567890123456789012123456789012345678901
in America today will openly espouse racism, although 2
12345678901234567890123456789012123456789012345678901
some behave and speak in racially prejudiced ways. 2
12345678901234567890123456789012123456789012345678901
2345678901234567890123456789012123456789012345678901
2
2
1
12345678901234567890123456789012123456789012345678901
estimable (adjective) Worthy of esteem and admiration. 2
12345678901234567890123456789012123456789012345678901 2
12345678901234567890123456789012123456789012345678901
After a tragic fire raged through Malden Mills, the 2
12345678901234567890123456789012123456789012345678901 2
12345678901234567890123456789012123456789012345678901
estimable mill owner, Aaron Feuerstein, restarted opera- 2
12345678901234567890123456789012123456789012345678901 2
12345678901234567890123456789012123456789012345678901
tions and rebuilt the company within just one month. 2
12345678901234567890123456789012123456789012345678901 2
12345678901234567890123456789012123456789012345678901
esteem (noun). 2
12345678901234567890123456789012123456789012345678901 2
2345678901234567890123456789012123456789012345678901
1 eulogy (noun) A formal tribute usually delivered at a 2
12345678901234567890123456789012123456789012345678901 2
12345678901234567890123456789012123456789012345678901 2
12345678901234567890123456789012123456789012345678901
funeral. Most people in Britain applauded Lord Earl 2
2345678901234567890123456789012123456789012345678901
Spencer’s eulogy for Princess Diana, not only as a warm 2
12345678901234567890123456789012123456789012345678901
12345678901234567890123456789012123456789012345678901 2
12345678901234567890123456789012123456789012345678901 2
12345678901234567890123456789012123456789012345678901 2
tribute to his sister Diana but also as a biting indictment
12345678901234567890123456789012123456789012345678901 2
12345678901234567890123456789012123456789012345678901
of the Royal Family. eulogize (verb). 2
12345678901234567890123456789012123456789012345678901 2
euphemism (noun) An agreeable expression that is substi- 2
12345678901234567890123456789012123456789012345678901
12345678901234567890123456789012123456789012345678901 2
1 tuted for an offensive one. Some of the more creative 2
12345678901234567890123456789012123456789012345678901 2
2345678901234567890123456789012123456789012345678901
euphemisms for “layoffs” in current use are: “release of 2
12345678901234567890123456789012123456789012345678901
1 2
12345678901234567890123456789012123456789012345678901
resources,” “involuntary severance,” “strengthening 2
12345678901234567890123456789012123456789012345678901 2
12345678901234567890123456789012123456789012345678901
global effectiveness,” and “career transition program.” 2
12345678901234567890123456789012123456789012345678901 2
12345678901234567890123456789012123456789012345678901
euphemistic (adjective). 2
12345678901234567890123456789012123456789012345678901 2
12345678901234567890123456789012123456789012345678901 2
12345678901234567890123456789012123456789012345678901 2
2345678901234567890123456789012123456789012345678901
12345678901234567890123456789012123456789012345678901 2
1 Origin acrimonious.
Word
Latin acer 5 sharp. Also found in English acerbity, acrid, and
2
12345678901234567890123456789012123456789012345678901 2
12345678901234567890123456789012123456789012345678901 2
12345678901234567890123456789012123456789012345678901
exacerbate (verb) To make worse or more severe. The roads 2
12345678901234567890123456789012123456789012345678901 2
12345678901234567890123456789012123456789012345678901 2
12345678901234567890123456789012123456789012345678901
in our town already have too much traffic; building a 2
12345678901234567890123456789012123456789012345678901 2
12345678901234567890123456789012123456789012345678901 2
2345678901234567890123456789012123456789012345678901
new shopping mall will exacerbate the problem.
12345678901234567890123456789012123456789012345678901 2
excoriation (noun) The act of condemning someone with 2
12345678901234567890123456789012123456789012345678901
12345678901234567890123456789012123456789012345678901 2
1 harsh words. In the small office we shared, it was 2
2345678901234567890123456789012123456789012345678901
12345678901234567890123456789012123456789012345678901
painful to hear my boss’s constant excoriation of his 2
2
12345678901234567890123456789012123456789012345678901 2
12345678901234567890123456789012123456789012345678901
assistant for the smallest faults—a misdirected letter, an 2
12345678901234567890123456789012123456789012345678901 2
1
12345678901234567890123456789012123456789012345678901
unclear phone message, or even a tepid cup of coffee. 2
12345678901234567890123456789012123456789012345678901 2
12345678901234567890123456789012123456789012345678901
excoriate (verb). 2
12345678901234567890123456789012123456789012345678901 2
12345678901234567890123456789012123456789012345678901 2
12345678901234567890123456789012123456789012345678901 2
12345678901234567890123456789012123456789012345678901 2
12345678901234567890123456789012123456789012345678901 2 2
188 12345678901234567890123456789012123456789012345678901
123456789012345678901234567890121234567890123456789012

www.petersons.com
Chapter 5: Verbal Review

123456789012345678901234567890121234567890123456789012
12345678901234567890123456789012123456789012345678901 2
12345678901234567890123456789012123456789012345678901 2
12345678901234567890123456789012123456789012345678901
exculpate (verb) To free from blame or guilt. When 2
12345678901234567890123456789012123456789012345678901 2
12345678901234567890123456789012123456789012345678901
someone else confessed to the crime, the previous 2
12345678901234567890123456789012123456789012345678901 2
12345678901234567890123456789012123456789012345678901
suspect was exculpated. exculpation (noun), exculpa- 2
2345678901234567890123456789012123456789012345678901 2
1
12345678901234567890123456789012123456789012345678901
tory (adjective). 2
12345678901234567890123456789012123456789012345678901 2
12345678901234567890123456789012123456789012345678901 2
12345678901234567890123456789012123456789012345678901
executor (noun) The person appointed
2345678901234567890123456789012123456789012345678901
to execute some- 2
1 one’s will. As the executor of his Aunt Ida’s will, Phil 2
12345678901234567890123456789012123456789012345678901 2
12345678901234567890123456789012123456789012345678901 2
12345678901234567890123456789012123456789012345678901
must deal with squabbling relatives, conniving lawyers, 2
12345678901234567890123456789012123456789012345678901
2345678901234567890123456789012123456789012345678901
2
2
1 and the ruinous state of Ida’s house.
12345678901234567890123456789012123456789012345678901 2
12345678901234567890123456789012123456789012345678901
exigent (adjective) Urgent, requiring immediate attention. A 2
12345678901234567890123456789012123456789012345678901 2
12345678901234567890123456789012123456789012345678901
two-year-old is likely to behave as if her every demand is 2
2345678901234567890123456789012123456789012345678901 2
1
12345678901234567890123456789012123456789012345678901
exigent, even if it involves simply retrieving a beloved 2
12345678901234567890123456789012123456789012345678901 2
12345678901234567890123456789012123456789012345678901
teddy bear from under the couch. exigency (noun). 2
12345678901234567890123456789012123456789012345678901 2
2345678901234567890123456789012123456789012345678901
12345678901234567890123456789012123456789012345678901
expedient (adjective) Providing an immediate advantage or 2
2
12345678901234567890123456789012123456789012345678901

Take It to the Next Level


serving one’s immediate self-interest. When the passen- 2
12345678901234567890123456789012123456789012345678901
12345678901234567890123456789012123456789012345678901 2
12345678901234567890123456789012123456789012345678901 2
12345678901234567890123456789012123456789012345678901 22
ger next to her was strafed by a bullet, Sharon chose the
1
12345678901234567890123456789012123456789012345678901
most expedient means to stop the bleeding; she whipped 2
12345678901234567890123456789012123456789012345678901
off her pantyhose and made an impromptu, but effec- 2
12345678901234567890123456789012123456789012345678901 2
2345678901234567890123456789012123456789012345678901
12345678901234567890123456789012123456789012345678901 2
12345678901234567890123456789012123456789012345678901
tive, tourniquet. expediency (noun). 2
12345678901234567890123456789012123456789012345678901 22
12345678901234567890123456789012123456789012345678901
extant (adjective) Currently in existence. Of the seven 2
12345678901234567890123456789012123456789012345678901
ancient “Wonders of the World,” only the pyramids of 2
12345678901234567890123456789012123456789012345678901
12345678901234567890123456789012123456789012345678901 2
12345678901234567890123456789012123456789012345678901
Egypt are still extant. 2
12345678901234567890123456789012123456789012345678901 22
1
12345678901234567890123456789012123456789012345678901 2
12345678901234567890123456789012123456789012345678901 2
12345678901234567890123456789012123456789012345678901
Latin tenere 5 to hold. Also found in English retain, tenable, 2
12345678901234567890123456789012123456789012345678901
Word Origin tenant, tenet, and tenure. 2
12345678901234567890123456789012123456789012345678901
2345678901234567890123456789012123456789012345678901
2
extenuate (verb) To make less serious. Karen’s guilt is 2
12345678901234567890123456789012123456789012345678901
12345678901234567890123456789012123456789012345678901 2
1 2
12345678901234567890123456789012123456789012345678901
extenuated by the fact that she was only 12 when she 2
12345678901234567890123456789012123456789012345678901 2
12345678901234567890123456789012123456789012345678901 22
2345678901234567890123456789012123456789012345678901
committed the theft. extenuating (adjective), extenua-
12345678901234567890123456789012123456789012345678901 2
1 tion (noun).
12345678901234567890123456789012123456789012345678901 2
12345678901234567890123456789012123456789012345678901
extol (verb) To greatly praise. At the party convention, one 2
12345678901234567890123456789012123456789012345678901 2
12345678901234567890123456789012123456789012345678901
speaker after another took to the podium to extol the 2
12345678901234567890123456789012123456789012345678901 2
12345678901234567890123456789012123456789012345678901
virtues of their candidate for the presidency. 2
12345678901234567890123456789012123456789012345678901 2
2345678901234567890123456789012123456789012345678901
12345678901234567890123456789012123456789012345678901
extraneous (adjective) Irrelevant, nonessential. One review 2
2
12345678901234567890123456789012123456789012345678901 2
12345678901234567890123456789012123456789012345678901
of the new Chekhov biography said the author had 2
12345678901234567890123456789012123456789012345678901 2
1
12345678901234567890123456789012123456789012345678901
bogged down the book with far too many extraneous 2
12345678901234567890123456789012123456789012345678901 2
12345678901234567890123456789012123456789012345678901
details, such as the dates of Chekhov’s bouts of diarrhea. 2
12345678901234567890123456789012123456789012345678901 2
12345678901234567890123456789012123456789012345678901 2
12345678901234567890123456789012123456789012345678901 22
2345678901234567890123456789012123456789012345678901
1
12345678901234567890123456789012123456789012345678901 2
12345678901234567890123456789012123456789012345678901 2
12345678901234567890123456789012123456789012345678901 2
12345678901234567890123456789012123456789012345678901 2
12345678901234567890123456789012123456789012345678901 2
12345678901234567890123456789012123456789012345678901 2
12345678901234567890123456789012123456789012345678901 2 189
123456789012345678901234567890121234567890123456789012
Part III: Section Review

123456789012345678901234567890121234567890123456789012
12345678901234567890123456789012123456789012345678901 2
12345678901234567890123456789012123456789012345678901 2
12345678901234567890123456789012123456789012345678901
extrapolate (verb) To deduce from something known, to 2
12345678901234567890123456789012123456789012345678901 2
12345678901234567890123456789012123456789012345678901
infer. Meteorologists were able to use old weather 2
12345678901234567890123456789012123456789012345678901 2
12345678901234567890123456789012123456789012345678901
records to extrapolate backward and compile lists of El 2
2345678901234567890123456789012123456789012345678901 2
1
12345678901234567890123456789012123456789012345678901
Niño years and their effects over the last century. 2
12345678901234567890123456789012123456789012345678901 2
12345678901234567890123456789012123456789012345678901
extrapolation (noun). 2
12345678901234567890123456789012123456789012345678901
2345678901234567890123456789012123456789012345678901
2
1 extricate (verb) To free from a difficult or complicated 2
12345678901234567890123456789012123456789012345678901 2
12345678901234567890123456789012123456789012345678901 2
12345678901234567890123456789012123456789012345678901
situation. Much of the humor in the TV show I Love 2
12345678901234567890123456789012123456789012345678901
2345678901234567890123456789012123456789012345678901
2
2
1 Lucy comes in watching Lucy try to extricate herself
12345678901234567890123456789012123456789012345678901 2
12345678901234567890123456789012123456789012345678901
from the problems she creates by fibbing or trickery. 2
12345678901234567890123456789012123456789012345678901
extricable (adjective). 2
12345678901234567890123456789012123456789012345678901
2345678901234567890123456789012123456789012345678901
2
2
1
12345678901234567890123456789012123456789012345678901
facetious (adjective) Humorous in a mocking way; not 2
12345678901234567890123456789012123456789012345678901 2
12345678901234567890123456789012123456789012345678901
serious. French composer Erik Satie often concealed his 2
12345678901234567890123456789012123456789012345678901 2
12345678901234567890123456789012123456789012345678901
serious artistic intent by giving his works facetious titles 2
12345678901234567890123456789012123456789012345678901 2
12345678901234567890123456789012123456789012345678901
such as “Three Pieces in the Shape of a Pear.” 2
12345678901234567890123456789012123456789012345678901 2
12345678901234567890123456789012123456789012345678901 2
12345678901234567890123456789012123456789012345678901 2
2345678901234567890123456789012123456789012345678901
facilitate (verb) To make easier or to moderate. When the
1 issue of racism reared its ugly head, the company 2
12345678901234567890123456789012123456789012345678901 2
12345678901234567890123456789012123456789012345678901 2
12345678901234567890123456789012123456789012345678901
brought in a consultant to facilitate a discussion of 2
2345678901234567890123456789012123456789012345678901
diversity in the workplace. facile (adjective), facility 2
12345678901234567890123456789012123456789012345678901
12345678901234567890123456789012123456789012345678901 2
12345678901234567890123456789012123456789012345678901 2
12345678901234567890123456789012123456789012345678901 2
(noun).
12345678901234567890123456789012123456789012345678901 2
fallacy (noun) An error in fact or logic. It’s a fallacy to think 2
12345678901234567890123456789012123456789012345678901
12345678901234567890123456789012123456789012345678901 2
that “natural” means “healthful”; after all, the deadly 2
12345678901234567890123456789012123456789012345678901
12345678901234567890123456789012123456789012345678901 2
1 poison arsenic is completely natural. fallacious (adjec- 2
12345678901234567890123456789012123456789012345678901 2
12345678901234567890123456789012123456789012345678901
tive). 2
12345678901234567890123456789012123456789012345678901 2
12345678901234567890123456789012123456789012345678901 2
12345678901234567890123456789012123456789012345678901
fatuous (adjective) Inanely foolish; silly. Once backstage, 2
2345678901234567890123456789012123456789012345678901
Elizabeth showered the opera singer with fatuous praise 2
12345678901234567890123456789012123456789012345678901
12345678901234567890123456789012123456789012345678901 2
1 2
12345678901234567890123456789012123456789012345678901
and embarrassing confessions, which he clearly had no 2
12345678901234567890123456789012123456789012345678901 2
12345678901234567890123456789012123456789012345678901 2
2345678901234567890123456789012123456789012345678901
interest in hearing.
12345678901234567890123456789012123456789012345678901 2
1 fawn (verb) To flatter in a particularly subservient manner. 2
12345678901234567890123456789012123456789012345678901 2
12345678901234567890123456789012123456789012345678901 2
12345678901234567890123456789012123456789012345678901 2
2345678901234567890123456789012123456789012345678901
Mildly disgusted, Pedro stood alone at the bar and
watched Renee fawn over the heir to the Fabco Surgical 2
12345678901234567890123456789012123456789012345678901 2
12345678901234567890123456789012123456789012345678901 2
12345678901234567890123456789012123456789012345678901
Appliances fortune.
2
1
2345678901234567890123456789012123456789012345678901
12345678901234567890123456789012123456789012345678901
feckless (adjective) Weak and ineffective; irresponsible. Our 2
2
12345678901234567890123456789012123456789012345678901 2
12345678901234567890123456789012123456789012345678901
co-op board president is a feckless fellow who has let 2
12345678901234567890123456789012123456789012345678901 2
1
12345678901234567890123456789012123456789012345678901
much-needed repairs go unattended while our mainte- 2
12345678901234567890123456789012123456789012345678901 2
12345678901234567890123456789012123456789012345678901
nance fees continue to rise. 2
12345678901234567890123456789012123456789012345678901 2
12345678901234567890123456789012123456789012345678901 2
12345678901234567890123456789012123456789012345678901 2
2345678901234567890123456789012123456789012345678901
feint (noun) A bluff; a mock blow. It didn’t take us long to
1 realize that Gaby’s tears and stomachaches were all a 2
12345678901234567890123456789012123456789012345678901 2
12345678901234567890123456789012123456789012345678901 2
12345678901234567890123456789012123456789012345678901
feint, since they appeared so regularly
2345678901234567890123456789012123456789012345678901
at her bedtime. 2
2
1
12345678901234567890123456789012123456789012345678901 2
12345678901234567890123456789012123456789012345678901 2 2
190 12345678901234567890123456789012123456789012345678901
123456789012345678901234567890121234567890123456789012

www.petersons.com
Chapter 5: Verbal Review

123456789012345678901234567890121234567890123456789012
12345678901234567890123456789012123456789012345678901 2
12345678901234567890123456789012123456789012345678901 2
12345678901234567890123456789012123456789012345678901
ferret (verb) To bring to light by an extensive search. With 2
12345678901234567890123456789012123456789012345678901 2
12345678901234567890123456789012123456789012345678901
his repeated probing and questions, Fritz was able to 2
12345678901234567890123456789012123456789012345678901 2
12345678901234567890123456789012123456789012345678901
ferret out the location of Myrna’s safe deposit box.
2345678901234567890123456789012123456789012345678901
2
2
1
12345678901234567890123456789012123456789012345678901 2
12345678901234567890123456789012123456789012345678901
finesse (noun) Skillful maneuvering; delicate workmanship. 2
12345678901234567890123456789012123456789012345678901 2
12345678901234567890123456789012123456789012345678901
With her usual finesse,
2345678901234567890123456789012123456789012345678901
Charmaine gently persuaded the 2
1 Duncans not to install a motorized Santa and sleigh on 2
12345678901234567890123456789012123456789012345678901 2
12345678901234567890123456789012123456789012345678901 2
12345678901234567890123456789012123456789012345678901
their front lawn. 2
12345678901234567890123456789012123456789012345678901
2345678901234567890123456789012123456789012345678901
2
1 florid (adjective) Flowery, fancy; reddish. The grand ball- 2
12345678901234567890123456789012123456789012345678901 2
12345678901234567890123456789012123456789012345678901
room was decorated in a florid style. Years of heavy 2
12345678901234567890123456789012123456789012345678901 2
12345678901234567890123456789012123456789012345678901
drinking had given him a florid complexion.
2345678901234567890123456789012123456789012345678901
2
2
1
12345678901234567890123456789012123456789012345678901 2
12345678901234567890123456789012123456789012345678901 2
12345678901234567890123456789012123456789012345678901 2
12345678901234567890123456789012123456789012345678901
Latin fluere 5 to flow. Also found in English affluent, effluvia, fluid, 2
12345678901234567890123456789012123456789012345678901 22
2345678901234567890123456789012123456789012345678901
Word Origin and influx.
12345678901234567890123456789012123456789012345678901 2

Take It to the Next Level


12345678901234567890123456789012123456789012345678901
flourish (noun) An extraneous embellishment; a dramatic 2
12345678901234567890123456789012123456789012345678901
gesture. The napkin rings made out of intertwined ferns 2
12345678901234567890123456789012123456789012345678901
12345678901234567890123456789012123456789012345678901 2
1 2
12345678901234567890123456789012123456789012345678901
and flowers were just the kind of flourish one would 2
12345678901234567890123456789012123456789012345678901 2
12345678901234567890123456789012123456789012345678901
expect from Carol, a slavish follower of Martha Stewart. 2
12345678901234567890123456789012123456789012345678901 22
2345678901234567890123456789012123456789012345678901
12345678901234567890123456789012123456789012345678901
fluctuation (noun) A shifting back and forth. Investment 2
12345678901234567890123456789012123456789012345678901
analysts predict fluctuations in the Dow Jones Industrial 2
12345678901234567890123456789012123456789012345678901
2
12345678901234567890123456789012123456789012345678901
Average due to the instability of the value of the dollar. 2
12345678901234567890123456789012123456789012345678901
12345678901234567890123456789012123456789012345678901 2
12345678901234567890123456789012123456789012345678901
fluctuate (verb). 2
12345678901234567890123456789012123456789012345678901 22
1
12345678901234567890123456789012123456789012345678901
foil (verb) To thwart or frustrate. I was certain that Jerry’s 2
12345678901234567890123456789012123456789012345678901 2
12345678901234567890123456789012123456789012345678901
tendency to insert himself into everyone’s conversations 2
12345678901234567890123456789012123456789012345678901 2
12345678901234567890123456789012123456789012345678901
would foil my chances to have a private word with 2
2345678901234567890123456789012123456789012345678901 2
12345678901234567890123456789012123456789012345678901
12345678901234567890123456789012123456789012345678901
Helen. 2
1 2
12345678901234567890123456789012123456789012345678901 2
12345678901234567890123456789012123456789012345678901
foment (verb) To rouse or incite. The petty tyrannies and 2
2345678901234567890123456789012123456789012345678901
indignities inflicted on the workers by upper manage- 2
12345678901234567890123456789012123456789012345678901
12345678901234567890123456789012123456789012345678901 2
1 2
12345678901234567890123456789012123456789012345678901 2
ment helped foment the walkout at the meat-processing
12345678901234567890123456789012123456789012345678901 2
12345678901234567890123456789012123456789012345678901
plant. 2
12345678901234567890123456789012123456789012345678901 2
12345678901234567890123456789012123456789012345678901
forestall (verb) To hinder or prevent by taking action in 2
12345678901234567890123456789012123456789012345678901 2
12345678901234567890123456789012123456789012345678901
advance. The pilot’s calm, levelheaded demeanor during 2
2345678901234567890123456789012123456789012345678901
12345678901234567890123456789012123456789012345678901
the turbulence forestalled any hysteria among the 2
2
12345678901234567890123456789012123456789012345678901 2
12345678901234567890123456789012123456789012345678901 2
12345678901234567890123456789012123456789012345678901
passengers of Flight 268.
2
1
2345678901234567890123456789012123456789012345678901
fortuitous (adjective) Lucky, fortunate. Although the mayor 2
12345678901234567890123456789012123456789012345678901
12345678901234567890123456789012123456789012345678901 2
12345678901234567890123456789012123456789012345678901 2
12345678901234567890123456789012123456789012345678901 22
claimed credit for the falling crime rate, it was really
1
12345678901234567890123456789012123456789012345678901
caused by a series of fortuitous accidents. 2
12345678901234567890123456789012123456789012345678901 2
12345678901234567890123456789012123456789012345678901 2
12345678901234567890123456789012123456789012345678901 2
12345678901234567890123456789012123456789012345678901 2
12345678901234567890123456789012123456789012345678901 2
12345678901234567890123456789012123456789012345678901 2
12345678901234567890123456789012123456789012345678901 2
12345678901234567890123456789012123456789012345678901 2 191
123456789012345678901234567890121234567890123456789012
Part III: Section Review

123456789012345678901234567890121234567890123456789012
12345678901234567890123456789012123456789012345678901 2
12345678901234567890123456789012123456789012345678901 2
12345678901234567890123456789012123456789012345678901
foster (verb) To nurture or encourage. The white-water 2
12345678901234567890123456789012123456789012345678901 2
12345678901234567890123456789012123456789012345678901
rafting trip was supposed to foster creative problem 2
12345678901234567890123456789012123456789012345678901 2
12345678901234567890123456789012123456789012345678901
solving and teamwork between the account executives 2
2345678901234567890123456789012123456789012345678901 2
1
12345678901234567890123456789012123456789012345678901
and the creative staff at Apex Advertising Agency. 2
12345678901234567890123456789012123456789012345678901 2
12345678901234567890123456789012123456789012345678901 2
12345678901234567890123456789012123456789012345678901
fracas (noun) A noisy fight; a brawl.
2345678901234567890123456789012123456789012345678901
As Bill approached the 2
1 stadium ticket window, he was alarmed to see the fracas 2
12345678901234567890123456789012123456789012345678901 2
12345678901234567890123456789012123456789012345678901 2
12345678901234567890123456789012123456789012345678901
that had broken out between a group of Giants fans and 2
12345678901234567890123456789012123456789012345678901
2345678901234567890123456789012123456789012345678901
2
2
1 a man wearing a Cowboys jersey and helmet.
12345678901234567890123456789012123456789012345678901 2
12345678901234567890123456789012123456789012345678901
functionary (noun) Someone holding office in a political 2
12345678901234567890123456789012123456789012345678901 2
12345678901234567890123456789012123456789012345678901
party or government. The man shaking hands with the 2
2345678901234567890123456789012123456789012345678901 2
1
12345678901234567890123456789012123456789012345678901
governor was a low-ranking Democratic Party function- 2
12345678901234567890123456789012123456789012345678901 2
12345678901234567890123456789012123456789012345678901
ary who had worked to garner the Hispanic vote. 2
12345678901234567890123456789012123456789012345678901 2
2345678901234567890123456789012123456789012345678901
12345678901234567890123456789012123456789012345678901
gainsay (verb) To contradict or oppose; deny, dispute. Dot 2
2
12345678901234567890123456789012123456789012345678901
would gainsay her married sister’s efforts to introduce 2
12345678901234567890123456789012123456789012345678901
12345678901234567890123456789012123456789012345678901 2
12345678901234567890123456789012123456789012345678901 2
12345678901234567890123456789012123456789012345678901 2
her to eligible men by refusing to either leave her ailing
2
1
12345678901234567890123456789012123456789012345678901
canary or give up her thrice-weekly bingo nights. 2
12345678901234567890123456789012123456789012345678901 2
12345678901234567890123456789012123456789012345678901 2
12345678901234567890123456789012123456789012345678901 2
2345678901234567890123456789012123456789012345678901
garble (verb) To distort or slur. No matter how much money
the Metropolitan Transit Authority spends on improv- 2
12345678901234567890123456789012123456789012345678901
12345678901234567890123456789012123456789012345678901 2
ing the subway trains, the public address system in 2
12345678901234567890123456789012123456789012345678901
12345678901234567890123456789012123456789012345678901 2
almost every station seems to garble each announce- 2
12345678901234567890123456789012123456789012345678901
12345678901234567890123456789012123456789012345678901 2
12345678901234567890123456789012123456789012345678901
ment. garbled (adjective). 2
12345678901234567890123456789012123456789012345678901 2 2
1
12345678901234567890123456789012123456789012345678901
garrulous (adjective) Annoyingly talkative. Claude pre- 2
12345678901234567890123456789012123456789012345678901 2
12345678901234567890123456789012123456789012345678901
tended to be asleep so he could avoid his garrulous 2
12345678901234567890123456789012123456789012345678901 2
12345678901234567890123456789012123456789012345678901
seatmate, a self-proclaimed expert on bonsai cultivation. 2
12345678901234567890123456789012123456789012345678901 2
2345678901234567890123456789012123456789012345678901
12345678901234567890123456789012123456789012345678901 2 2
1
12345678901234567890123456789012123456789012345678901 2
12345678901234567890123456789012123456789012345678901
Word
Latin genus 5 type or kind; birth. Also found in English congenital,
Origin genetic, genital, genre, genuine, and genus. 2
12345678901234567890123456789012123456789012345678901 2
2345678901234567890123456789012123456789012345678901
12345678901234567890123456789012123456789012345678901 2
1 generic (adjective) General; having no brand name. Connie 2
12345678901234567890123456789012123456789012345678901 2
12345678901234567890123456789012123456789012345678901 2
12345678901234567890123456789012123456789012345678901 2
2345678901234567890123456789012123456789012345678901
tried to reduce her grocery bills by religiously clipping
2
12345678901234567890123456789012123456789012345678901
coupons and buying generic brands of most products. 2
12345678901234567890123456789012123456789012345678901
12345678901234567890123456789012123456789012345678901 2
1 gist (noun) The main point, the essence. Although they felt 2
12345678901234567890123456789012123456789012345678901 2
12345678901234567890123456789012123456789012345678901
sympathy for the victim’s family, the jurors were won 2
12345678901234567890123456789012123456789012345678901 2
12345678901234567890123456789012123456789012345678901
over by the gist of the defense’s argument: There was 2
12345678901234567890123456789012123456789012345678901 2
12345678901234567890123456789012123456789012345678901
insufficient evidence to convict. 2
12345678901234567890123456789012123456789012345678901 2
12345678901234567890123456789012123456789012345678901 2
12345678901234567890123456789012123456789012345678901 2
12345678901234567890123456789012123456789012345678901 2
12345678901234567890123456789012123456789012345678901 2
2345678901234567890123456789012123456789012345678901
2
1
12345678901234567890123456789012123456789012345678901 2
12345678901234567890123456789012123456789012345678901 2
12345678901234567890123456789012123456789012345678901 2
12345678901234567890123456789012123456789012345678901 2
12345678901234567890123456789012123456789012345678901 2
12345678901234567890123456789012123456789012345678901 2 2
192 12345678901234567890123456789012123456789012345678901
123456789012345678901234567890121234567890123456789012

www.petersons.com
Chapter 5: Verbal Review

123456789012345678901234567890121234567890123456789012
12345678901234567890123456789012123456789012345678901 2
12345678901234567890123456789012123456789012345678901 2
12345678901234567890123456789012123456789012345678901
gouge (verb) To cut out, to scoop out with one’s thumbs or 2
12345678901234567890123456789012123456789012345678901 2
12345678901234567890123456789012123456789012345678901
a sharp instrument; to overcharge, to cheat. Instead of 2
12345678901234567890123456789012123456789012345678901 2
12345678901234567890123456789012123456789012345678901
picking the lock with a credit card, the clumsy thieves 2
2345678901234567890123456789012123456789012345678901 2
1
12345678901234567890123456789012123456789012345678901
gouged a hole in my door. The consumer watchdog 2
12345678901234567890123456789012123456789012345678901
group accused the clothing stores of gouging customers 2
12345678901234567890123456789012123456789012345678901 2
12345678901234567890123456789012123456789012345678901
2345678901234567890123456789012123456789012345678901
2
2
1 with high prices.
12345678901234567890123456789012123456789012345678901 2
12345678901234567890123456789012123456789012345678901 2
12345678901234567890123456789012123456789012345678901
guile (noun) Deceit, duplicity. In Margaret Mitchell’s Gone 2
12345678901234567890123456789012123456789012345678901
2345678901234567890123456789012123456789012345678901
2
2
1 With the Wind, Scarlett O’Hara uses her guile to
12345678901234567890123456789012123456789012345678901 2
12345678901234567890123456789012123456789012345678901
manipulate two men and then is matched for wits by a 2
12345678901234567890123456789012123456789012345678901
third, Rhett Butler. guileful (adjective). 2
12345678901234567890123456789012123456789012345678901
2345678901234567890123456789012123456789012345678901
2
2
1
12345678901234567890123456789012123456789012345678901
gullible (adjective) Easily fooled. Terry was so gullible she 2
12345678901234567890123456789012123456789012345678901 2
12345678901234567890123456789012123456789012345678901
actually believed Robert’s stories of his connections to 2
12345678901234567890123456789012123456789012345678901 2
12345678901234567890123456789012123456789012345678901
the Czar and Czarina. gullibility (noun). 2
12345678901234567890123456789012123456789012345678901 2

Take It to the Next Level


12345678901234567890123456789012123456789012345678901 2
12345678901234567890123456789012123456789012345678901
hackneyed (adjective) Without originality, trite. When some- 2
12345678901234567890123456789012123456789012345678901 2
12345678901234567890123456789012123456789012345678901 22
2345678901234567890123456789012123456789012345678901
one invented the phrase, “No pain, no gain,” it was
1
12345678901234567890123456789012123456789012345678901
clever and witty, but now it is so commonly heard that it 2
12345678901234567890123456789012123456789012345678901 2
12345678901234567890123456789012123456789012345678901
seems hackneyed. 2
12345678901234567890123456789012123456789012345678901 22
2345678901234567890123456789012123456789012345678901
12345678901234567890123456789012123456789012345678901
harrow (verb) To cultivate with a harrow; to torment or 2
12345678901234567890123456789012123456789012345678901
vex. During grade school, my sister was harrowed 2
12345678901234567890123456789012123456789012345678901
2
12345678901234567890123456789012123456789012345678901
mercilessly for being overweight. harrowing (adjective) 2
12345678901234567890123456789012123456789012345678901
12345678901234567890123456789012123456789012345678901 2
nerve-wracking, traumatic. Jon Krakauer’s harrowing 2
12345678901234567890123456789012123456789012345678901
12345678901234567890123456789012123456789012345678901 22
1 book Into Thin Air chronicles the tragic consequences of
12345678901234567890123456789012123456789012345678901 2
12345678901234567890123456789012123456789012345678901
leading groups of untrained climbers up Mt. Everest. 2
12345678901234567890123456789012123456789012345678901 2
12345678901234567890123456789012123456789012345678901 2
12345678901234567890123456789012123456789012345678901
haughty (adjective) Overly proud. The fashion model strode 2
2345678901234567890123456789012123456789012345678901
down the runway, her hips thrust forward and a haughty 2
12345678901234567890123456789012123456789012345678901
12345678901234567890123456789012123456789012345678901 2
1 2
12345678901234567890123456789012123456789012345678901
expression, something like a sneer, on her face. haughti- 2
12345678901234567890123456789012123456789012345678901 2
12345678901234567890123456789012123456789012345678901 22
2345678901234567890123456789012123456789012345678901
ness (noun).
12345678901234567890123456789012123456789012345678901 2
1
12345678901234567890123456789012123456789012345678901
hierarchy (noun) A ranking of people, things, or ideas from 2
12345678901234567890123456789012123456789012345678901
highest to lowest. A cabinet secretary ranks just below 2
12345678901234567890123456789012123456789012345678901 2
12345678901234567890123456789012123456789012345678901
the president and vice president in the hierarchy of the 2
12345678901234567890123456789012123456789012345678901 2
12345678901234567890123456789012123456789012345678901
government’s executive branch. hierarchical (adjective). 2
12345678901234567890123456789012123456789012345678901 2
12345678901234567890123456789012123456789012345678901 2
12345678901234567890123456789012123456789012345678901 2
12345678901234567890123456789012123456789012345678901 2
12345678901234567890123456789012123456789012345678901
Greek homos 5 same. Also found in English homologous, 2
12345678901234567890123456789012123456789012345678901
Word Origin 2
12345678901234567890123456789012123456789012345678901 22
2345678901234567890123456789012123456789012345678901
homonym, and homosexual.
12345678901234567890123456789012123456789012345678901
homogeneous (adjective) Uniform, made entirely of one 2
12345678901234567890123456789012123456789012345678901
1 thing. It’s hard to think of a more homogenous group 2
12345678901234567890123456789012123456789012345678901 2
2345678901234567890123456789012123456789012345678901
than those eerie children in Village of the Damned, who 2
12345678901234567890123456789012123456789012345678901
1 2
12345678901234567890123456789012123456789012345678901 2
12345678901234567890123456789012123456789012345678901 2
all had perfect features, white-blond hair, and silver,
12345678901234567890123456789012123456789012345678901 2
12345678901234567890123456789012123456789012345678901
penetrating eyes. 2
12345678901234567890123456789012123456789012345678901 2
12345678901234567890123456789012123456789012345678901 2
12345678901234567890123456789012123456789012345678901 2 193
123456789012345678901234567890121234567890123456789012
Part III: Section Review

123456789012345678901234567890121234567890123456789012
12345678901234567890123456789012123456789012345678901 2
12345678901234567890123456789012123456789012345678901 2
12345678901234567890123456789012123456789012345678901
hoodwink (verb) To deceive by trickery or false appear- 2
12345678901234567890123456789012123456789012345678901 2
12345678901234567890123456789012123456789012345678901
ances; to dupe. That was my cousin Ravi calling to say 2
12345678901234567890123456789012123456789012345678901 2
12345678901234567890123456789012123456789012345678901
that he’s been hoodwinked again, this time by some 2
2345678901234567890123456789012123456789012345678901 2
1
12345678901234567890123456789012123456789012345678901
outfit offering time shares in a desolate tract of land in 2
12345678901234567890123456789012123456789012345678901 2
12345678901234567890123456789012123456789012345678901
central Florida. 2
12345678901234567890123456789012123456789012345678901
2345678901234567890123456789012123456789012345678901
2
1 hone (verb) To improve and make more acute or affective. 2
12345678901234567890123456789012123456789012345678901 2
12345678901234567890123456789012123456789012345678901 2
12345678901234567890123456789012123456789012345678901
While she was a receptionist, Norma honed her skills as 2
12345678901234567890123456789012123456789012345678901
2345678901234567890123456789012123456789012345678901
2
2
1 a stand-up comic by trying out jokes on the tense crowd
12345678901234567890123456789012123456789012345678901 2
12345678901234567890123456789012123456789012345678901
in the waiting room. 2
12345678901234567890123456789012123456789012345678901 2
12345678901234567890123456789012123456789012345678901
iconoclast (noun) Someone who attacks traditional beliefs 2
2345678901234567890123456789012123456789012345678901 2
1
12345678901234567890123456789012123456789012345678901
or institutions. Comedian Dennis Miller relishes his 2
12345678901234567890123456789012123456789012345678901 2
12345678901234567890123456789012123456789012345678901
reputation as an iconoclast, though people in power 2
12345678901234567890123456789012123456789012345678901 2
12345678901234567890123456789012123456789012345678901
often resent his satirical jabs. iconoclasm (noun), icono- 2
12345678901234567890123456789012123456789012345678901 2
12345678901234567890123456789012123456789012345678901
clastic (adjective). 2
12345678901234567890123456789012123456789012345678901 2
12345678901234567890123456789012123456789012345678901 2
12345678901234567890123456789012123456789012345678901 2
2345678901234567890123456789012123456789012345678901
idolatry (noun) The worship of a person, thing, or institu-
1 tion as a god. In communist China, admiration for Mao 2
12345678901234567890123456789012123456789012345678901 2
12345678901234567890123456789012123456789012345678901 2
12345678901234567890123456789012123456789012345678901
resembled idolatry; his picture was displayed every- 2
2345678901234567890123456789012123456789012345678901
where, and millions of Chinese memorized his sayings 2
12345678901234567890123456789012123456789012345678901
12345678901234567890123456789012123456789012345678901 2
12345678901234567890123456789012123456789012345678901 2
12345678901234567890123456789012123456789012345678901 2
and repeated them endlessly. idolatrous (adjective).
12345678901234567890123456789012123456789012345678901 2
idyll (noun) A rustic, romantic interlude; poetry or prose 2
12345678901234567890123456789012123456789012345678901
12345678901234567890123456789012123456789012345678901 2
that celebrates simple pastoral life. Her picnic with Max 2
12345678901234567890123456789012123456789012345678901
12345678901234567890123456789012123456789012345678901 2
1 at Fahnstock Lake was not the serene idyll she had 2
12345678901234567890123456789012123456789012345678901 2
12345678901234567890123456789012123456789012345678901
envisioned; instead, they were surrounded by hundreds 2
12345678901234567890123456789012123456789012345678901 2
12345678901234567890123456789012123456789012345678901
of other picnickers blaring music from their boom boxes 2
12345678901234567890123456789012123456789012345678901 2
12345678901234567890123456789012123456789012345678901
and cracking open soda cans. idyllic (adjective). 2
12345678901234567890123456789012123456789012345678901 2
12345678901234567890123456789012123456789012345678901 2
12345678901234567890123456789012123456789012345678901
illicit (adjective) Illegal, wrongful. When Janet caught her 2
12345678901234567890123456789012123456789012345678901 2
12345678901234567890123456789012123456789012345678901 2
2345678901234567890123456789012123456789012345678901
13-year-old son and his friend downloading illicit
1 pornographic photos from the World Wide Web, she 2
12345678901234567890123456789012123456789012345678901 2
12345678901234567890123456789012123456789012345678901 2
12345678901234567890123456789012123456789012345678901 2
promptly pulled the plug on his computer.
12345678901234567890123456789012123456789012345678901 2
2345678901234567890123456789012123456789012345678901
illuminate (verb) To brighten with light; to enlighten or 2
12345678901234567890123456789012123456789012345678901 2
12345678901234567890123456789012123456789012345678901
elucidate; to decorate (a manuscript). The frosted-glass 2
12345678901234567890123456789012123456789012345678901
1 2
12345678901234567890123456789012123456789012345678901
sconces in the dressing rooms at Le Cirque not only 2
12345678901234567890123456789012123456789012345678901 2
12345678901234567890123456789012123456789012345678901
illuminate the rooms but make everyone look like a 2
12345678901234567890123456789012123456789012345678901 2
12345678901234567890123456789012123456789012345678901
movie star. Alice Munro is a writer who can illuminate 2
2345678901234567890123456789012123456789012345678901
12345678901234567890123456789012123456789012345678901 2
12345678901234567890123456789012123456789012345678901
an entire character with a few deft sentences. 2
12345678901234567890123456789012123456789012345678901 2
1 immaterial (adjective) Of no consequence, unimportant. 2
12345678901234567890123456789012123456789012345678901 2
2345678901234567890123456789012123456789012345678901
“The fact that your travel agent is your best friend’s son 2
12345678901234567890123456789012123456789012345678901
1 2
12345678901234567890123456789012123456789012345678901 2
12345678901234567890123456789012123456789012345678901 2
should be immaterial,” I told Rosa; “If he keeps putting
12345678901234567890123456789012123456789012345678901
you on hold and acting nasty, just take your business 2
12345678901234567890123456789012123456789012345678901 2
12345678901234567890123456789012123456789012345678901 2
12345678901234567890123456789012123456789012345678901 2
elsewhere.”
2
194 12345678901234567890123456789012123456789012345678901
123456789012345678901234567890121234567890123456789012

www.petersons.com
Chapter 5: Verbal Review

123456789012345678901234567890121234567890123456789012
12345678901234567890123456789012123456789012345678901 2
12345678901234567890123456789012123456789012345678901 2
12345678901234567890123456789012123456789012345678901
immaculate (adjective) Totally unblemished, spotlessly clean. 2
12345678901234567890123456789012123456789012345678901 2
12345678901234567890123456789012123456789012345678901
The cream-colored upholstery in my new Porsche was 2
12345678901234567890123456789012123456789012345678901 2
12345678901234567890123456789012123456789012345678901
immaculate—that is, until a raccoon came in through 2
2345678901234567890123456789012123456789012345678901 2
1
12345678901234567890123456789012123456789012345678901
the window and tracked mud across the seats. 2
12345678901234567890123456789012123456789012345678901 2
12345678901234567890123456789012123456789012345678901 2
12345678901234567890123456789012123456789012345678901
2345678901234567890123456789012123456789012345678901
2
2
1
12345678901234567890123456789012123456789012345678901
Word
Latin mutare 5 to change. Also found in English immutable,
Origin mutant, and mutation. 2
12345678901234567890123456789012123456789012345678901 2
12345678901234567890123456789012123456789012345678901 2
12345678901234567890123456789012123456789012345678901
2345678901234567890123456789012123456789012345678901
2
2
1 immutable (adjective) Incapable of change. Does there ever
12345678901234567890123456789012123456789012345678901 2
12345678901234567890123456789012123456789012345678901
come an age when we realize that our parents’ person- 2
12345678901234567890123456789012123456789012345678901
alities are immutable, when we can relax and stop trying 2
12345678901234567890123456789012123456789012345678901
2345678901234567890123456789012123456789012345678901
2
2
1 to make them change?
12345678901234567890123456789012123456789012345678901 2
12345678901234567890123456789012123456789012345678901 2
12345678901234567890123456789012123456789012345678901
impartial (adjective) Fair, equal, unbiased. If a judge is not 2
12345678901234567890123456789012123456789012345678901 2
12345678901234567890123456789012123456789012345678901
impartial, then all of her rulings are questionable. 2
12345678901234567890123456789012123456789012345678901 2

Take It to the Next Level


12345678901234567890123456789012123456789012345678901
impartiality (noun). 2
12345678901234567890123456789012123456789012345678901 2
12345678901234567890123456789012123456789012345678901 2
12345678901234567890123456789012123456789012345678901 22
2345678901234567890123456789012123456789012345678901
impassivity (noun) Apathy, unresponsiveness. Dot truly
1
12345678901234567890123456789012123456789012345678901
thinks that Mr. Right will magically show up on her 2
12345678901234567890123456789012123456789012345678901
door step, and her utter impassivity regarding her social 2
12345678901234567890123456789012123456789012345678901 2
2345678901234567890123456789012123456789012345678901
12345678901234567890123456789012123456789012345678901 2
12345678901234567890123456789012123456789012345678901
life makes me want to shake her! impassive (adjective). 2
12345678901234567890123456789012123456789012345678901 22
12345678901234567890123456789012123456789012345678901
imperceptible (adjective) Impossible to perceive, inaudible, 2
12345678901234567890123456789012123456789012345678901
or incomprehensible. The sound of footsteps was almost 2
12345678901234567890123456789012123456789012345678901
12345678901234567890123456789012123456789012345678901 2
imperceptible, but Donald’s paranoia had reached such 2
12345678901234567890123456789012123456789012345678901
12345678901234567890123456789012123456789012345678901 22
1 a pitch that he immediately assumed he was being
12345678901234567890123456789012123456789012345678901 2
12345678901234567890123456789012123456789012345678901
followed. 2
12345678901234567890123456789012123456789012345678901 2
12345678901234567890123456789012123456789012345678901 2
12345678901234567890123456789012123456789012345678901
imperturbable (adjective) Marked by extreme calm, impas- 2
2345678901234567890123456789012123456789012345678901
sivity, and steadiness. The proper English butler in 2
12345678901234567890123456789012123456789012345678901
12345678901234567890123456789012123456789012345678901 2
1 2
12345678901234567890123456789012123456789012345678901
Kazuo Ishiguro’s novel The Remains of the Day appears 2
12345678901234567890123456789012123456789012345678901 2
12345678901234567890123456789012123456789012345678901 22
2345678901234567890123456789012123456789012345678901
completely imperturbable even when his father dies or
12345678901234567890123456789012123456789012345678901 2
1 when his own heart is breaking.
12345678901234567890123456789012123456789012345678901 2
12345678901234567890123456789012123456789012345678901
impetuous (adjective) Acting hastily or impulsively. Ben’s 2
12345678901234567890123456789012123456789012345678901 2
12345678901234567890123456789012123456789012345678901
resignation was an impetuous act; he did it without 2
12345678901234567890123456789012123456789012345678901 2
12345678901234567890123456789012123456789012345678901
thinking, and he soon regretted it. impetuosity (noun). 2
12345678901234567890123456789012123456789012345678901 2
12345678901234567890123456789012123456789012345678901 22
2345678901234567890123456789012123456789012345678901
12345678901234567890123456789012123456789012345678901 2
12345678901234567890123456789012123456789012345678901 2
12345678901234567890123456789012123456789012345678901
Latin placare 5 to please. Also found in English complacent,
2
1
Word Origin placate, and placid.
12345678901234567890123456789012123456789012345678901 22
2345678901234567890123456789012123456789012345678901
12345678901234567890123456789012123456789012345678901
implacable (adjective) Unbending, resolute. The state of 2
12345678901234567890123456789012123456789012345678901
1 Israel is implacable in its policy of never negotiating 2
12345678901234567890123456789012123456789012345678901 2
2345678901234567890123456789012123456789012345678901
12345678901234567890123456789012123456789012345678901 2
1 with criminals. 2
12345678901234567890123456789012123456789012345678901 2
12345678901234567890123456789012123456789012345678901 2
12345678901234567890123456789012123456789012345678901 2
12345678901234567890123456789012123456789012345678901 2
12345678901234567890123456789012123456789012345678901 2
12345678901234567890123456789012123456789012345678901 2
12345678901234567890123456789012123456789012345678901 2 195
123456789012345678901234567890121234567890123456789012
Part III: Section Review

123456789012345678901234567890121234567890123456789012
12345678901234567890123456789012123456789012345678901 2
12345678901234567890123456789012123456789012345678901 2
12345678901234567890123456789012123456789012345678901
implosion (noun) To collapse inward from outside pressure. 2
12345678901234567890123456789012123456789012345678901 2
12345678901234567890123456789012123456789012345678901
While it is difficult to know what is going on in North 2
12345678901234567890123456789012123456789012345678901 2
12345678901234567890123456789012123456789012345678901
Korea, no one can rule out a violent implosion of the 2
2345678901234567890123456789012123456789012345678901 2
1
12345678901234567890123456789012123456789012345678901
North Korean regime and a subsequent flood of refugees 2
12345678901234567890123456789012123456789012345678901 2
12345678901234567890123456789012123456789012345678901
across its borders. implode (verb). 2
12345678901234567890123456789012123456789012345678901
2345678901234567890123456789012123456789012345678901
2
1 incessant (adjective) Unceasing. The incessant blaring of the 2
12345678901234567890123456789012123456789012345678901 2
12345678901234567890123456789012123456789012345678901 2
12345678901234567890123456789012123456789012345678901
neighbor’s car alarm made it impossible for me to 2
12345678901234567890123456789012123456789012345678901
2345678901234567890123456789012123456789012345678901
2
2
1 concentrate on my upcoming Bar exam.
12345678901234567890123456789012123456789012345678901 2
12345678901234567890123456789012123456789012345678901
inchoate (adjective) Only partly formed or formulated. At 2
12345678901234567890123456789012123456789012345678901 2
12345678901234567890123456789012123456789012345678901
editorial meetings, Nancy had a habit of presenting her 2
2345678901234567890123456789012123456789012345678901 2
1
12345678901234567890123456789012123456789012345678901
inchoate book ideas before she had a chance to fully 2
12345678901234567890123456789012123456789012345678901 2
12345678901234567890123456789012123456789012345678901
determine their feasibility. 2
12345678901234567890123456789012123456789012345678901 2
12345678901234567890123456789012123456789012345678901 2
2345678901234567890123456789012123456789012345678901
12345678901234567890123456789012123456789012345678901 2
12345678901234567890123456789012123456789012345678901 2
12345678901234567890123456789012123456789012345678901
Latin caedere 5 to cut. Also found in English concise, decide,
2
12345678901234567890123456789012123456789012345678901 2
Word Origin excise, incision, and precise.
12345678901234567890123456789012123456789012345678901 2
1 incise (verb) To carve into, to engrave. My wife felt 2
12345678901234567890123456789012123456789012345678901 2
12345678901234567890123456789012123456789012345678901 2
12345678901234567890123456789012123456789012345678901
nostalgic about the old elm tree since we had incised our 2
2345678901234567890123456789012123456789012345678901
12345678901234567890123456789012123456789012345678901 2
12345678901234567890123456789012123456789012345678901
initials in it when we were both in high school. 2
12345678901234567890123456789012123456789012345678901 2
incisive (adjective) Admirably direct and decisive. Ted 2
12345678901234567890123456789012123456789012345678901
2
12345678901234567890123456789012123456789012345678901
Koppel’s incisive questions have made many politicians 2
12345678901234567890123456789012123456789012345678901
12345678901234567890123456789012123456789012345678901 2
12345678901234567890123456789012123456789012345678901
squirm and stammer. 2
12345678901234567890123456789012123456789012345678901 2 2
1
12345678901234567890123456789012123456789012345678901
incongruous (adjective) Unlikely. Art makes incongruous 2
12345678901234567890123456789012123456789012345678901 2
12345678901234567890123456789012123456789012345678901
alliances, as when punk rockers, Tibetan folk musicians, 2
12345678901234567890123456789012123456789012345678901 2
12345678901234567890123456789012123456789012345678901
gospel singers, and beat poets shared the stage at the 2
2345678901234567890123456789012123456789012345678901 2
12345678901234567890123456789012123456789012345678901
12345678901234567890123456789012123456789012345678901
Tibet House benefit concert. incongruity (noun). 2
1 2
12345678901234567890123456789012123456789012345678901 2
12345678901234567890123456789012123456789012345678901
incorrigible (adjective) Impossible to manage or reform. 2
2345678901234567890123456789012123456789012345678901
Lou is an incorrigible trickster, constantly playing 2
12345678901234567890123456789012123456789012345678901
12345678901234567890123456789012123456789012345678901 2
1 2
12345678901234567890123456789012123456789012345678901 2
practical jokes no matter how much his friends com-
12345678901234567890123456789012123456789012345678901 2
12345678901234567890123456789012123456789012345678901
plain. 2
12345678901234567890123456789012123456789012345678901 2
12345678901234567890123456789012123456789012345678901
incursion (noun) A hostile entrance into a territory; a foray 2
12345678901234567890123456789012123456789012345678901 2
12345678901234567890123456789012123456789012345678901
into an activity or venture. It is a little-known fact that 2
2345678901234567890123456789012123456789012345678901
12345678901234567890123456789012123456789012345678901
the Central Intelligence Agency organized military 2
2
12345678901234567890123456789012123456789012345678901 2
12345678901234567890123456789012123456789012345678901
incursions into China during the 1950s. The ComicCon 2
12345678901234567890123456789012123456789012345678901 2
1
12345678901234567890123456789012123456789012345678901
was Barbara’s first incursion into the world of comic 2
12345678901234567890123456789012123456789012345678901 2
12345678901234567890123456789012123456789012345678901
strip artists. 2
12345678901234567890123456789012123456789012345678901 2
12345678901234567890123456789012123456789012345678901 2
12345678901234567890123456789012123456789012345678901 2
2345678901234567890123456789012123456789012345678901
indefatigable (adjective) Tireless. Eleanor Roosevelt’s inde-
1 fatigable dedication to the cause of human welfare won 2
12345678901234567890123456789012123456789012345678901 2
12345678901234567890123456789012123456789012345678901 2
12345678901234567890123456789012123456789012345678901
her affection and honor throughout
2345678901234567890123456789012123456789012345678901
the world. indefati- 2
2
1
12345678901234567890123456789012123456789012345678901
gability (noun). 2
12345678901234567890123456789012123456789012345678901 2 2
196 12345678901234567890123456789012123456789012345678901
123456789012345678901234567890121234567890123456789012

www.petersons.com
Chapter 5: Verbal Review

123456789012345678901234567890121234567890123456789012
12345678901234567890123456789012123456789012345678901 2
12345678901234567890123456789012123456789012345678901 2
12345678901234567890123456789012123456789012345678901
indelicate (adjective) Blunt, undisguised. No sooner had we 2
12345678901234567890123456789012123456789012345678901 2
12345678901234567890123456789012123456789012345678901
sat down to eat than Mark made an indelicate remark 2
12345678901234567890123456789012123456789012345678901 2
12345678901234567890123456789012123456789012345678901
about my high salary.
2345678901234567890123456789012123456789012345678901
2
2
1
12345678901234567890123456789012123456789012345678901 2
12345678901234567890123456789012123456789012345678901
inevitable (adjective) Unable to be avoided. Once the 2
12345678901234567890123456789012123456789012345678901 2
12345678901234567890123456789012123456789012345678901
Japanese attacked Pearl Harbor,
2345678901234567890123456789012123456789012345678901
U.S. involvement in 2
2
1
12345678901234567890123456789012123456789012345678901
World War II was inevitable. inevitability (noun). 2
12345678901234567890123456789012123456789012345678901 2
12345678901234567890123456789012123456789012345678901 2
12345678901234567890123456789012123456789012345678901
infer (verb) To conclude, to deduce.
2345678901234567890123456789012123456789012345678901
Can I infer from your 2
1 hostile tone of voice that you are still angry about 2
12345678901234567890123456789012123456789012345678901 2
12345678901234567890123456789012123456789012345678901
yesterday’s incident? inference (noun). 2
12345678901234567890123456789012123456789012345678901 2
12345678901234567890123456789012123456789012345678901
2345678901234567890123456789012123456789012345678901
2
2
1 inimical (adjective) Unfriendly, hostile; adverse or difficult.
12345678901234567890123456789012123456789012345678901 2
12345678901234567890123456789012123456789012345678901
Relations between Greece and Turkey have been inimi- 2
12345678901234567890123456789012123456789012345678901 2
12345678901234567890123456789012123456789012345678901
cal for centuries. 2
12345678901234567890123456789012123456789012345678901 22
2345678901234567890123456789012123456789012345678901
12345678901234567890123456789012123456789012345678901

Take It to the Next Level


inimitable (adjective) Incapable of being imitated, match- 2
12345678901234567890123456789012123456789012345678901
less. John F. Kennedy’s administration dazzled the 2
12345678901234567890123456789012123456789012345678901
12345678901234567890123456789012123456789012345678901 2
1 public, partly because of the inimitable style and 2
12345678901234567890123456789012123456789012345678901 2
12345678901234567890123456789012123456789012345678901 2
12345678901234567890123456789012123456789012345678901
elegance of his wife, Jacqueline. 2
12345678901234567890123456789012123456789012345678901 2
2345678901234567890123456789012123456789012345678901
inopportune (adjective) Awkward, untimely. When Gus 2
12345678901234567890123456789012123456789012345678901
12345678901234567890123456789012123456789012345678901 2
heard raised voices and the crash of breaking china 2
12345678901234567890123456789012123456789012345678901
12345678901234567890123456789012123456789012345678901 22
12345678901234567890123456789012123456789012345678901
behind the kitchen door, he realized that he’d picked an
12345678901234567890123456789012123456789012345678901 2
12345678901234567890123456789012123456789012345678901
inopportune moment to visit the Fairlights. 2
12345678901234567890123456789012123456789012345678901 22
12345678901234567890123456789012123456789012345678901
inscrutability (noun) Quality of being extremely difficult to 2
1
12345678901234567890123456789012123456789012345678901
interpret or understand, mysteriousness. I am still 2
12345678901234567890123456789012123456789012345678901 2
12345678901234567890123456789012123456789012345678901 2
12345678901234567890123456789012123456789012345678901 2
puzzling over the inscrutability of the package I received
12345678901234567890123456789012123456789012345678901
yesterday, which contained twenty pomegranates and a 2
12345678901234567890123456789012123456789012345678901 2
12345678901234567890123456789012123456789012345678901 2
12345678901234567890123456789012123456789012345678901 2
note that said simply “Yours.” inscrutable (adjective).
12345678901234567890123456789012123456789012345678901 2
12345678901234567890123456789012123456789012345678901 2
12345678901234567890123456789012123456789012345678901 22
2345678901234567890123456789012123456789012345678901
insensible (adjective) Unaware, incognizant; unconscious,
12345678901234567890123456789012123456789012345678901
out cold. It’s a good thing that Marty was insensible to 2
1
12345678901234567890123456789012123456789012345678901
the titters and laughter that greeted his arrival in the 2
12345678901234567890123456789012123456789012345678901 2
2345678901234567890123456789012123456789012345678901
ballroom. In the latest episode of police brutality, an 2
12345678901234567890123456789012123456789012345678901
12345678901234567890123456789012123456789012345678901 2
innocent young man was beaten insensible after two 2
12345678901234567890123456789012123456789012345678901
12345678901234567890123456789012123456789012345678901 22
1 cops stormed his apartment.
12345678901234567890123456789012123456789012345678901 2
12345678901234567890123456789012123456789012345678901 2
12345678901234567890123456789012123456789012345678901
insipid (adjective) Flavorless, uninteresting. Most TV shows 2
12345678901234567890123456789012123456789012345678901
are so insipid that you can watch them while reading or 2
12345678901234567890123456789012123456789012345678901 2
2345678901234567890123456789012123456789012345678901
12345678901234567890123456789012123456789012345678901 2
12345678901234567890123456789012123456789012345678901
chatting without missing a thing. insipidity (noun). 2
12345678901234567890123456789012123456789012345678901 22
12345678901234567890123456789012123456789012345678901 2
1
12345678901234567890123456789012123456789012345678901 2
12345678901234567890123456789012123456789012345678901 2
12345678901234567890123456789012123456789012345678901 2
12345678901234567890123456789012123456789012345678901 2
12345678901234567890123456789012123456789012345678901 2
12345678901234567890123456789012123456789012345678901 2
12345678901234567890123456789012123456789012345678901 2
12345678901234567890123456789012123456789012345678901 2
12345678901234567890123456789012123456789012345678901 2 197
123456789012345678901234567890121234567890123456789012
Part III: Section Review

123456789012345678901234567890121234567890123456789012
12345678901234567890123456789012123456789012345678901 2
12345678901234567890123456789012123456789012345678901 2
12345678901234567890123456789012123456789012345678901
insinuate (verb) Hint or intimate; to creep in. During an 2
12345678901234567890123456789012123456789012345678901 2
12345678901234567890123456789012123456789012345678901
extremely unusual broadcast, the anchor man insinu- 2
12345678901234567890123456789012123456789012345678901 2
12345678901234567890123456789012123456789012345678901
ated that the Washington bureau chief was having a 2
2345678901234567890123456789012123456789012345678901 2
1
12345678901234567890123456789012123456789012345678901
nervous breakdown. Marla managed to insinuate herself 2
12345678901234567890123456789012123456789012345678901
into the Duchess of York’s conversation during the 2
12345678901234567890123456789012123456789012345678901 2
12345678901234567890123456789012123456789012345678901
2345678901234567890123456789012123456789012345678901
2
2
1 “Weight Watchers” promotion event. insinuation (noun).
12345678901234567890123456789012123456789012345678901 2
12345678901234567890123456789012123456789012345678901 2
12345678901234567890123456789012123456789012345678901
insolence (noun) An attitude or behavior that is bold and 2
12345678901234567890123456789012123456789012345678901
2345678901234567890123456789012123456789012345678901
2
2
1 disrespectful. Some feel that news reporters who shout
12345678901234567890123456789012123456789012345678901 2
12345678901234567890123456789012123456789012345678901
accusatory questions at the president are behaving with 2
12345678901234567890123456789012123456789012345678901
insolence toward his high office. insolent (adjective). 2
12345678901234567890123456789012123456789012345678901
2345678901234567890123456789012123456789012345678901
2
2
1
12345678901234567890123456789012123456789012345678901
insoluble (adjective) Unable to be solved, irresolvable; 2
12345678901234567890123456789012123456789012345678901 2
12345678901234567890123456789012123456789012345678901
indissoluble. Fermat’s last theorem remained insoluble 2
12345678901234567890123456789012123456789012345678901 2
12345678901234567890123456789012123456789012345678901
for over 300 years until a young mathematician from 2
12345678901234567890123456789012123456789012345678901 2
12345678901234567890123456789012123456789012345678901
Princeton solved it in 1995. If you are a gum chewer, 2
12345678901234567890123456789012123456789012345678901 2
12345678901234567890123456789012123456789012345678901
you probably wouldn’t like to know that insoluble 2
2345678901234567890123456789012123456789012345678901
plastics are a common ingredient of most popular gums. 2
12345678901234567890123456789012123456789012345678901
1 2
12345678901234567890123456789012123456789012345678901 2
12345678901234567890123456789012123456789012345678901 2
12345678901234567890123456789012123456789012345678901
insular (adjective) Narrow or isolated in attitude or view- 2
2345678901234567890123456789012123456789012345678901
point. New Yorkers are famous for their insular atti- 2
12345678901234567890123456789012123456789012345678901
12345678901234567890123456789012123456789012345678901 2
12345678901234567890123456789012123456789012345678901 2
12345678901234567890123456789012123456789012345678901 2
tudes; they seem to think that nothing important has
12345678901234567890123456789012123456789012345678901 2
12345678901234567890123456789012123456789012345678901
ever happened outside of their city. insularity (noun). 2
12345678901234567890123456789012123456789012345678901 2
intercede (verb) To step in, to moderate; to mediate or 2
12345678901234567890123456789012123456789012345678901
12345678901234567890123456789012123456789012345678901 2
1 negotiate on behalf of someone else. After their rejection 2
12345678901234567890123456789012123456789012345678901 2
2345678901234567890123456789012123456789012345678901
by the co-op board, Kevin and Sol asked Rachel, another 2
12345678901234567890123456789012123456789012345678901
1 2
12345678901234567890123456789012123456789012345678901
tenant, to intercede for them at the next board meeting. 2
12345678901234567890123456789012123456789012345678901 2
12345678901234567890123456789012123456789012345678901
intercession (noun). 2
12345678901234567890123456789012123456789012345678901 2
12345678901234567890123456789012123456789012345678901 2
12345678901234567890123456789012123456789012345678901
interim (noun) A break or interlude. In the interim between 2
12345678901234567890123456789012123456789012345678901 2
12345678901234567890123456789012123456789012345678901 2
2345678901234567890123456789012123456789012345678901
figure-skating programs, the exhausted skaters retreat to
12345678901234567890123456789012123456789012345678901 2
1 the “kiss and cry” room to wait for their scores. 2
12345678901234567890123456789012123456789012345678901 2
12345678901234567890123456789012123456789012345678901
interpolate (verb) To interject. The director’s decision to 2
12345678901234567890123456789012123456789012345678901 2
12345678901234567890123456789012123456789012345678901
interpolate topical political jokes into his production of 2
12345678901234567890123456789012123456789012345678901 2
12345678901234567890123456789012123456789012345678901
Shakespeare’s Twelfth Night was not viewed kindly by 2
12345678901234567890123456789012123456789012345678901 2
12345678901234567890123456789012123456789012345678901
the critics. interpolation (noun). 2
12345678901234567890123456789012123456789012345678901 2
12345678901234567890123456789012123456789012345678901 2
12345678901234567890123456789012123456789012345678901
intransigent (adjective) Unwilling to compromise. Despite 2
12345678901234567890123456789012123456789012345678901 2
12345678901234567890123456789012123456789012345678901
the mediator’s attempts to suggest a fair solution to the 2
12345678901234567890123456789012123456789012345678901 2
12345678901234567890123456789012123456789012345678901
disagreement, the two parties were intransigent, forcing 2
12345678901234567890123456789012123456789012345678901 2
12345678901234567890123456789012123456789012345678901
a showdown. intransigence (noun). 2
12345678901234567890123456789012123456789012345678901 2
2345678901234567890123456789012123456789012345678901
2
1
12345678901234567890123456789012123456789012345678901 2
12345678901234567890123456789012123456789012345678901 2
12345678901234567890123456789012123456789012345678901 2
12345678901234567890123456789012123456789012345678901 2
12345678901234567890123456789012123456789012345678901 2
12345678901234567890123456789012123456789012345678901 2 2
198 12345678901234567890123456789012123456789012345678901
123456789012345678901234567890121234567890123456789012

www.petersons.com
Chapter 5: Verbal Review

123456789012345678901234567890121234567890123456789012
12345678901234567890123456789012123456789012345678901 2
12345678901234567890123456789012123456789012345678901 2
12345678901234567890123456789012123456789012345678901
intrinsically (adverb) Essentially, inherently. There is noth- 2
12345678901234567890123456789012123456789012345678901 2
12345678901234567890123456789012123456789012345678901
ing intrinsically difficult about upgrading a computer’s 2
12345678901234567890123456789012123456789012345678901 2
12345678901234567890123456789012123456789012345678901
microprocessor, yet Al was afraid to even open up the 2
2345678901234567890123456789012123456789012345678901 2
1
12345678901234567890123456789012123456789012345678901
hard drive. intrinsic (adjective). 2
12345678901234567890123456789012123456789012345678901 2
12345678901234567890123456789012123456789012345678901 2
12345678901234567890123456789012123456789012345678901
2345678901234567890123456789012123456789012345678901
2
2
1
12345678901234567890123456789012123456789012345678901
Word Origin Latin unda 5 wave. Also found in English undulate. 2
12345678901234567890123456789012123456789012345678901 2
12345678901234567890123456789012123456789012345678901
inundate (verb) To overwhelm; to flood. When America 2
12345678901234567890123456789012123456789012345678901 2
12345678901234567890123456789012123456789012345678901
Online first announced its flat-rate pricing, the company 2
12345678901234567890123456789012123456789012345678901 2
12345678901234567890123456789012123456789012345678901
was inundated with new customers, and thus began the 2
12345678901234567890123456789012123456789012345678901 2
12345678901234567890123456789012123456789012345678901
annoying delays in service. inundation (noun).
2345678901234567890123456789012123456789012345678901
2
2
1
12345678901234567890123456789012123456789012345678901
invective (noun) Insulting, abusive language. I remained 2
12345678901234567890123456789012123456789012345678901 2
12345678901234567890123456789012123456789012345678901 2
12345678901234567890123456789012123456789012345678901
unscathed by his blistering invective because in my heart 2
2345678901234567890123456789012123456789012345678901
12345678901234567890123456789012123456789012345678901 2
12345678901234567890123456789012123456789012345678901 2

Take It to the Next Level


I knew I had done the right thing.
12345678901234567890123456789012123456789012345678901 22
12345678901234567890123456789012123456789012345678901
invigorate (verb) To give energy to, to stimulate. As her car 2
12345678901234567890123456789012123456789012345678901
1 climbed the mountain road, Lucinda felt herself invigo- 2
12345678901234567890123456789012123456789012345678901 2
12345678901234567890123456789012123456789012345678901
rated by the clear air and the cool breezes. invigoration 2
12345678901234567890123456789012123456789012345678901 2
12345678901234567890123456789012123456789012345678901
(noun). 2
12345678901234567890123456789012123456789012345678901 22
2345678901234567890123456789012123456789012345678901
12345678901234567890123456789012123456789012345678901
irascible (adjective) Easily provoked into anger, hot-headed. 2
12345678901234567890123456789012123456789012345678901
12345678901234567890123456789012123456789012345678901
Soup chef Al Yeganah, the model for Seinfeld’s “Soup 2
2
12345678901234567890123456789012123456789012345678901
12345678901234567890123456789012123456789012345678901
Nazi,” is an irascible man who flies into a temper if his 2
2
12345678901234567890123456789012123456789012345678901
12345678901234567890123456789012123456789012345678901
customers don’t follow his rigid procedure for purchas- 2
2
12345678901234567890123456789012123456789012345678901 2
1
12345678901234567890123456789012123456789012345678901
ing soup. irascibility (noun). 2
12345678901234567890123456789012123456789012345678901 2
12345678901234567890123456789012123456789012345678901 2
12345678901234567890123456789012123456789012345678901 2
jeopardize (verb) To put in danger. Terrorist attacks on
12345678901234567890123456789012123456789012345678901
civilians jeopardize the fragile peace in the Middle East. 2
12345678901234567890123456789012123456789012345678901 2
12345678901234567890123456789012123456789012345678901 2
12345678901234567890123456789012123456789012345678901 2
jeopardy (noun).
12345678901234567890123456789012123456789012345678901 2
12345678901234567890123456789012123456789012345678901
jocular (adjective) Humorous, amusing. Listening to the 2
12345678901234567890123456789012123456789012345678901 22
2345678901234567890123456789012123456789012345678901
12345678901234567890123456789012123456789012345678901
CEO launch into yet another uproarious anecdote, Ted
2
1
12345678901234567890123456789012123456789012345678901
was frankly surprised by the jocular nature of the 2
12345678901234567890123456789012123456789012345678901 2
12345678901234567890123456789012123456789012345678901
“emergency” board meeting. jocularity (noun). 2
12345678901234567890123456789012123456789012345678901 2
12345678901234567890123456789012123456789012345678901 2
12345678901234567890123456789012123456789012345678901
labyrinthine (adjective) Extremely intricate or involved; 2
12345678901234567890123456789012123456789012345678901 2
12345678901234567890123456789012123456789012345678901 22
2345678901234567890123456789012123456789012345678901
circuitous. Was I the only one who couldn’t follow the
12345678901234567890123456789012123456789012345678901
labyrinthine plot of the movie L.A. Confidential? I was 2
12345678901234567890123456789012123456789012345678901
1 so confused I had to watch it twice to see “who did it.” 2
12345678901234567890123456789012123456789012345678901 2
12345678901234567890123456789012123456789012345678901 22
2345678901234567890123456789012123456789012345678901
12345678901234567890123456789012123456789012345678901
laconic (adjective) Concise to the point of terseness; taci- 2
12345678901234567890123456789012123456789012345678901
1 turn. Tall, handsome, and laconic, the actor Gary 2
12345678901234567890123456789012123456789012345678901 2
2345678901234567890123456789012123456789012345678901
Cooper came to personify the strong, silent American, a 2
12345678901234567890123456789012123456789012345678901
1 2
12345678901234567890123456789012123456789012345678901
man of action and few words. 2
12345678901234567890123456789012123456789012345678901 2
12345678901234567890123456789012123456789012345678901 2
12345678901234567890123456789012123456789012345678901 2
12345678901234567890123456789012123456789012345678901 2
12345678901234567890123456789012123456789012345678901 2
12345678901234567890123456789012123456789012345678901 2 199
123456789012345678901234567890121234567890123456789012
Part III: Section Review

123456789012345678901234567890121234567890123456789012
12345678901234567890123456789012123456789012345678901 2
12345678901234567890123456789012123456789012345678901 2
12345678901234567890123456789012123456789012345678901
lambaste (verb) To give someone a dressing-down; to attack 2
12345678901234567890123456789012123456789012345678901 2
12345678901234567890123456789012123456789012345678901
someone verbally; to whip. Once inside the locker room, 2
12345678901234567890123456789012123456789012345678901 2
12345678901234567890123456789012123456789012345678901
the coach thoroughly lambasted the team for their 2
2345678901234567890123456789012123456789012345678901 2
1
12345678901234567890123456789012123456789012345678901
incompetent performance on the football field. 2
12345678901234567890123456789012123456789012345678901 2
12345678901234567890123456789012123456789012345678901 2
12345678901234567890123456789012123456789012345678901
2345678901234567890123456789012123456789012345678901
2
2
1
12345678901234567890123456789012123456789012345678901
Word
Latin laus 5 praise. Also found in English applaud, laud, laudatory,
Origin and plaudit. 2
12345678901234567890123456789012123456789012345678901 2
12345678901234567890123456789012123456789012345678901 2
12345678901234567890123456789012123456789012345678901
2345678901234567890123456789012123456789012345678901
2
2
1 laudable (adjective) Commendable, praiseworthy. The
12345678901234567890123456789012123456789012345678901 2
12345678901234567890123456789012123456789012345678901
Hunt’s Point nonprofit organization has embarked on a 2
12345678901234567890123456789012123456789012345678901
series of laudable ventures pairing businesses and 2
12345678901234567890123456789012123456789012345678901
2345678901234567890123456789012123456789012345678901
2
2
1 disadvantaged youth.
12345678901234567890123456789012123456789012345678901 2
12345678901234567890123456789012123456789012345678901 2
12345678901234567890123456789012123456789012345678901
lethargic (adjective) Lacking energy; sluggish. Visitors to 2
12345678901234567890123456789012123456789012345678901 2
12345678901234567890123456789012123456789012345678901
the zoo are surprised that the lions appear so lethargic, 2
12345678901234567890123456789012123456789012345678901 2
12345678901234567890123456789012123456789012345678901
but, in the wild, lions sleep up to 18 hours a day. 2
12345678901234567890123456789012123456789012345678901 2
12345678901234567890123456789012123456789012345678901
lethargy (noun). 2
12345678901234567890123456789012123456789012345678901 2
2345678901234567890123456789012123456789012345678901
1 levy (verb) To demand payment or collection of a tax or fee. 2
12345678901234567890123456789012123456789012345678901 2
12345678901234567890123456789012123456789012345678901 2
12345678901234567890123456789012123456789012345678901
The environmental activists pushed Congress to levy 2
2345678901234567890123456789012123456789012345678901
higher taxes on gasoline, but the automakers’ lobbyists 2
12345678901234567890123456789012123456789012345678901
12345678901234567890123456789012123456789012345678901 2
12345678901234567890123456789012123456789012345678901 2
12345678901234567890123456789012123456789012345678901 2
quashed their plans.
12345678901234567890123456789012123456789012345678901 2
lien (noun) A claim against a property for the satisfaction of 2
12345678901234567890123456789012123456789012345678901
12345678901234567890123456789012123456789012345678901 2
a debt. Nat was in such financial straits when he died 2
12345678901234567890123456789012123456789012345678901
12345678901234567890123456789012123456789012345678901 2
1 that his Fishkill property had several liens against it and 2
12345678901234567890123456789012123456789012345678901 2
12345678901234567890123456789012123456789012345678901
all of his furniture was being repossessed. 2
12345678901234567890123456789012123456789012345678901 2
12345678901234567890123456789012123456789012345678901 2
12345678901234567890123456789012123456789012345678901
limn (verb) To outline in distinct detail; to delineate. Like 2
2345678901234567890123456789012123456789012345678901
many of her novels, Edith Wharton’s The Age of 2
12345678901234567890123456789012123456789012345678901
12345678901234567890123456789012123456789012345678901 2
1 2
12345678901234567890123456789012123456789012345678901
Innocence expertly limns the tyranny of New York’s 2
12345678901234567890123456789012123456789012345678901 2
12345678901234567890123456789012123456789012345678901 2
2345678901234567890123456789012123456789012345678901
upper-class society in the 1800s.
12345678901234567890123456789012123456789012345678901 2
1 loquacity (noun) Talkativeness, wordiness. While some 2
12345678901234567890123456789012123456789012345678901 2
12345678901234567890123456789012123456789012345678901 2
12345678901234567890123456789012123456789012345678901 2
2345678901234567890123456789012123456789012345678901
people deride his loquacity and his tendency to use
outrageous rhymes, no one can doubt that Jesse Jackson 2
12345678901234567890123456789012123456789012345678901 2
12345678901234567890123456789012123456789012345678901 2
12345678901234567890123456789012123456789012345678901
is a powerful orator. loquacious (adjective).
2
1
12345678901234567890123456789012123456789012345678901 2
2345678901234567890123456789012123456789012345678901
12345678901234567890123456789012123456789012345678901 2 2
12345678901234567890123456789012123456789012345678901 2
12345678901234567890123456789012123456789012345678901
Latin lux 5 light. Also found in English elucidate, pellucid, and
2
1
Word Origin
12345678901234567890123456789012123456789012345678901 2
2345678901234567890123456789012123456789012345678901
translucent.
2
12345678901234567890123456789012123456789012345678901
lucid (adjective) Clear and understandable. Hawking’s A 2
12345678901234567890123456789012123456789012345678901
1 Short History of the Universe is a lucid explanation of a 2
12345678901234567890123456789012123456789012345678901 2
2345678901234567890123456789012123456789012345678901
difficult topic, modern scientific theories of the origin of 2
12345678901234567890123456789012123456789012345678901
1 2
12345678901234567890123456789012123456789012345678901 2
12345678901234567890123456789012123456789012345678901 2
the universe. lucidity (noun).
12345678901234567890123456789012123456789012345678901 2
12345678901234567890123456789012123456789012345678901 2
12345678901234567890123456789012123456789012345678901 2
12345678901234567890123456789012123456789012345678901 2 2
200 12345678901234567890123456789012123456789012345678901
123456789012345678901234567890121234567890123456789012

www.petersons.com
Chapter 5: Verbal Review

123456789012345678901234567890121234567890123456789012
12345678901234567890123456789012123456789012345678901 2
12345678901234567890123456789012123456789012345678901 2
12345678901234567890123456789012123456789012345678901
magnanimous (adjective) Noble, generous. When media 2
12345678901234567890123456789012123456789012345678901 2
12345678901234567890123456789012123456789012345678901
titan Ted Turner pledged a gift of $1 billion to the 2
12345678901234567890123456789012123456789012345678901 2
12345678901234567890123456789012123456789012345678901
United Nations, he challenged other wealthy people to 2
2345678901234567890123456789012123456789012345678901 2
1
12345678901234567890123456789012123456789012345678901
be equally magnanimous. magnanimity (noun). 2
12345678901234567890123456789012123456789012345678901 2
12345678901234567890123456789012123456789012345678901 2
12345678901234567890123456789012123456789012345678901
maladroit (adjective) Inept, awkward.
2345678901234567890123456789012123456789012345678901
It was painful to 2
1 watch the young congressman’s maladroit delivery of 2
12345678901234567890123456789012123456789012345678901 2
12345678901234567890123456789012123456789012345678901
the nominating speech. 2
12345678901234567890123456789012123456789012345678901 2
12345678901234567890123456789012123456789012345678901
2345678901234567890123456789012123456789012345678901
2
2
1 malinger (verb) To pretend illness to avoid work. During
12345678901234567890123456789012123456789012345678901 2
12345678901234567890123456789012123456789012345678901
the labor dispute, hundreds of employees malingered, 2
12345678901234567890123456789012123456789012345678901 2
12345678901234567890123456789012123456789012345678901
forcing the company to slow production and costing it 2
2345678901234567890123456789012123456789012345678901 2
1
12345678901234567890123456789012123456789012345678901
millions in profits. 2
12345678901234567890123456789012123456789012345678901 2
12345678901234567890123456789012123456789012345678901 2
12345678901234567890123456789012123456789012345678901
malleable (adjective) Able to be changed, shaped, or formed 2
2345678901234567890123456789012123456789012345678901
by outside pressures. Gold is a very useful metal because 2
12345678901234567890123456789012123456789012345678901
12345678901234567890123456789012123456789012345678901 2

Take It to the Next Level


12345678901234567890123456789012123456789012345678901
it is so malleable. A child’s personality is malleable, and 2
12345678901234567890123456789012123456789012345678901 22
12345678901234567890123456789012123456789012345678901
is often deeply influenced by things her parents say and
12345678901234567890123456789012123456789012345678901 2
1 do. malleability (noun). 2
12345678901234567890123456789012123456789012345678901 2
12345678901234567890123456789012123456789012345678901 2
12345678901234567890123456789012123456789012345678901 2
2345678901234567890123456789012123456789012345678901
12345678901234567890123456789012123456789012345678901 2
1 Latin mandare 5 entrust, order. Also found in English command,
2
12345678901234567890123456789012123456789012345678901
Word Origin demand, and remand. 2
12345678901234567890123456789012123456789012345678901 2
12345678901234567890123456789012123456789012345678901 2
12345678901234567890123456789012123456789012345678901 22
2345678901234567890123456789012123456789012345678901
mandate (noun) Order, command. The new policy on gays
12345678901234567890123456789012123456789012345678901
in the military went into effect as soon as the president 2
12345678901234567890123456789012123456789012345678901
1 issued his mandate about it. mandate (verb), mandatory 2
12345678901234567890123456789012123456789012345678901 2
2345678901234567890123456789012123456789012345678901
12345678901234567890123456789012123456789012345678901 2
12345678901234567890123456789012123456789012345678901
(adjective). 2
1 2
12345678901234567890123456789012123456789012345678901
marginal (adjective) At the outer edge or fringe; of minimal 2
12345678901234567890123456789012123456789012345678901 2
12345678901234567890123456789012123456789012345678901
quality or acceptability. In spite of the trend toward 2
12345678901234567890123456789012123456789012345678901 2
12345678901234567890123456789012123456789012345678901
greater paternal involvement in child-rearing, most 2
12345678901234567890123456789012123456789012345678901 2
12345678901234567890123456789012123456789012345678901
fathers still have a marginal role in their children’s lives. 2
2345678901234567890123456789012123456789012345678901
12345678901234567890123456789012123456789012345678901
Jerry’s GRE scores were so marginal that he didn’t get 2
2
12345678901234567890123456789012123456789012345678901 2
1
12345678901234567890123456789012123456789012345678901
accepted into the graduate schools of his choice. 2
12345678901234567890123456789012123456789012345678901 2
2345678901234567890123456789012123456789012345678901
marginalize (verb) To push toward the fringes; to make less 2
12345678901234567890123456789012123456789012345678901
12345678901234567890123456789012123456789012345678901 2
12345678901234567890123456789012123456789012345678901 2
12345678901234567890123456789012123456789012345678901 22
consequential. Hannah argued that the designation of a
1
12345678901234567890123456789012123456789012345678901
certain month as “Black History Month” or “Gay and 2
12345678901234567890123456789012123456789012345678901
Lesbian Book Month” actually does a disservice to 2
12345678901234567890123456789012123456789012345678901 2
12345678901234567890123456789012123456789012345678901
minorities by marginalizing them. 2
12345678901234567890123456789012123456789012345678901 2
2345678901234567890123456789012123456789012345678901
12345678901234567890123456789012123456789012345678901
martial (adjective) Of, relating to, or suited to military life. 2
2
12345678901234567890123456789012123456789012345678901
My old teacher, Miss Woody, had such a martial 2
12345678901234567890123456789012123456789012345678901
12345678901234567890123456789012123456789012345678901 2
1 2
12345678901234567890123456789012123456789012345678901 22
2345678901234567890123456789012123456789012345678901
demeanor that you’d think she was running a boot camp
1
12345678901234567890123456789012123456789012345678901
instead of teaching fifth grade. The military seized 2
12345678901234567890123456789012123456789012345678901
control of Myanmar in 1988, and this embattled country 2
12345678901234567890123456789012123456789012345678901
2345678901234567890123456789012123456789012345678901
2
2
1
12345678901234567890123456789012123456789012345678901
has been ruled by martial law since then. 2
12345678901234567890123456789012123456789012345678901 22
12345678901234567890123456789012123456789012345678901 201
123456789012345678901234567890121234567890123456789012
Part III: Section Review

123456789012345678901234567890121234567890123456789012
12345678901234567890123456789012123456789012345678901 2
12345678901234567890123456789012123456789012345678901 2
12345678901234567890123456789012123456789012345678901 2
12345678901234567890123456789012123456789012345678901
Latin medius 5 middle. Also found in English intermediate, media, 2
12345678901234567890123456789012123456789012345678901
Word Origin 2
12345678901234567890123456789012123456789012345678901 2
and medium.
12345678901234567890123456789012123456789012345678901 2
12345678901234567890123456789012123456789012345678901
mediate (verb) To reconcile differences between two parties. 2
12345678901234567890123456789012123456789012345678901 2
12345678901234567890123456789012123456789012345678901
During the baseball strike, both the players and the club 2
12345678901234567890123456789012123456789012345678901 2
12345678901234567890123456789012123456789012345678901
owners expressed willingness to have the president 2
2345678901234567890123456789012123456789012345678901 2
1
12345678901234567890123456789012123456789012345678901
mediate the dispute. mediation (noun). 2
12345678901234567890123456789012123456789012345678901 2
12345678901234567890123456789012123456789012345678901 2
12345678901234567890123456789012123456789012345678901
mercenary (adjective) Doing something
2345678901234567890123456789012123456789012345678901
only for pay or for 2
1 personal advantage. People have criticized the U.S. 2
12345678901234567890123456789012123456789012345678901 2
12345678901234567890123456789012123456789012345678901 2
12345678901234567890123456789012123456789012345678901
motives in the Persian Gulf War as mercenary, pointing 2
12345678901234567890123456789012123456789012345678901
2345678901234567890123456789012123456789012345678901
2
2
1 out that the U.S. would not have come to Kuwait’s
12345678901234567890123456789012123456789012345678901 2
12345678901234567890123456789012123456789012345678901
defense had it grown carrots rather than produced oil. 2
12345678901234567890123456789012123456789012345678901
mercenary (noun). 2
12345678901234567890123456789012123456789012345678901 2
12345678901234567890123456789012123456789012345678901 2
2345678901234567890123456789012123456789012345678901
mercurial (adjective) Changing quickly and unpredictably. 2
12345678901234567890123456789012123456789012345678901
12345678901234567890123456789012123456789012345678901
The mercurial personality of Robin Williams, with his 2
2
12345678901234567890123456789012123456789012345678901
12345678901234567890123456789012123456789012345678901
many voices and styles, made him a natural choice to 2
2
12345678901234567890123456789012123456789012345678901 2
1
12345678901234567890123456789012123456789012345678901
play the part of the ever-changing genie in Aladdin. 2
12345678901234567890123456789012123456789012345678901 2
12345678901234567890123456789012123456789012345678901 2
12345678901234567890123456789012123456789012345678901 2
2345678901234567890123456789012123456789012345678901
metamorphose (verb) To undergo a striking transforma-
tion. In just a century, book publishers have metamor- 2
12345678901234567890123456789012123456789012345678901
12345678901234567890123456789012123456789012345678901 2
12345678901234567890123456789012123456789012345678901 2
12345678901234567890123456789012123456789012345678901 2
phosed from independent, exclusively literary businesses
to minor divisions in multimedia entertainment conglom- 2
12345678901234567890123456789012123456789012345678901
12345678901234567890123456789012123456789012345678901 2
12345678901234567890123456789012123456789012345678901 2
12345678901234567890123456789012123456789012345678901 2
erates. metamorphosis (noun).
2
1
12345678901234567890123456789012123456789012345678901
meticulous (adjective) Very careful with details. Watch 2
12345678901234567890123456789012123456789012345678901 2
12345678901234567890123456789012123456789012345678901
repair calls for a craftsperson who is patient and 2
12345678901234567890123456789012123456789012345678901 2
12345678901234567890123456789012123456789012345678901
meticulous. 2
12345678901234567890123456789012123456789012345678901 2
12345678901234567890123456789012123456789012345678901 2
12345678901234567890123456789012123456789012345678901
mettle (noun) Strength of spirit; stamina. Linda’s mettle 2
12345678901234567890123456789012123456789012345678901
was severely tested while she served as the only female 2
12345678901234567890123456789012123456789012345678901 2
2345678901234567890123456789012123456789012345678901
attorney at Smith, Futterweitt, Houghton, and Dobbs. 2
12345678901234567890123456789012123456789012345678901
12345678901234567890123456789012123456789012345678901 2
1 2
12345678901234567890123456789012123456789012345678901 2
mettlesome (adjective).
12345678901234567890123456789012123456789012345678901 2
12345678901234567890123456789012123456789012345678901
mimicry (noun) Imitation, aping. The continued popularity 2
12345678901234567890123456789012123456789012345678901 2
12345678901234567890123456789012123456789012345678901 2
12345678901234567890123456789012123456789012345678901
of Elvis Presley has given rise to a class of entertainers 2
12345678901234567890123456789012123456789012345678901 2
12345678901234567890123456789012123456789012345678901 2
2345678901234567890123456789012123456789012345678901
who make a living through mimicry of “The King.”
2
12345678901234567890123456789012123456789012345678901
mimic (noun and verb).
2
12345678901234567890123456789012123456789012345678901
12345678901234567890123456789012123456789012345678901 2
1 minatory (adjective) Menacing, threatening. As soon as she 2
12345678901234567890123456789012123456789012345678901 2
12345678901234567890123456789012123456789012345678901
met Mrs. Danforth, the head housemaid at Manderlay, 2
12345678901234567890123456789012123456789012345678901 2
12345678901234567890123456789012123456789012345678901
the young bride was cowed by her minatory manner and 2
12345678901234567890123456789012123456789012345678901 2
12345678901234567890123456789012123456789012345678901
quickly retreated to the morning room. 2
12345678901234567890123456789012123456789012345678901 2
12345678901234567890123456789012123456789012345678901 2
12345678901234567890123456789012123456789012345678901 2
12345678901234567890123456789012123456789012345678901 2
12345678901234567890123456789012123456789012345678901 2
12345678901234567890123456789012123456789012345678901 2
12345678901234567890123456789012123456789012345678901 2 2
202 12345678901234567890123456789012123456789012345678901
123456789012345678901234567890121234567890123456789012

www.petersons.com
Chapter 5: Verbal Review

123456789012345678901234567890121234567890123456789012
12345678901234567890123456789012123456789012345678901 2
12345678901234567890123456789012123456789012345678901 2
12345678901234567890123456789012123456789012345678901
mince (verb) To chop into small pieces; to speak with 2
12345678901234567890123456789012123456789012345678901 2
12345678901234567890123456789012123456789012345678901
decorum and restraint. Malaysia’s prime minister, 2
12345678901234567890123456789012123456789012345678901 2
12345678901234567890123456789012123456789012345678901
Mahathir Mohamad, is not a man known to mince 2
2345678901234567890123456789012123456789012345678901 2
1
12345678901234567890123456789012123456789012345678901
words; he has accused satellite TV of poisoning Asia and 2
12345678901234567890123456789012123456789012345678901
has denounced the Australian press as “congenital 2
12345678901234567890123456789012123456789012345678901 2
12345678901234567890123456789012123456789012345678901
2345678901234567890123456789012123456789012345678901
2
2
1 liars.”
12345678901234567890123456789012123456789012345678901 2
12345678901234567890123456789012123456789012345678901 2
12345678901234567890123456789012123456789012345678901 2
12345678901234567890123456789012123456789012345678901
2345678901234567890123456789012123456789012345678901
Greek anthropos 5 human. Also found in English anthropology, 2
2
1
Word Origin
12345678901234567890123456789012123456789012345678901
anthropoid, anthropomorphic, and philanthropy. 2
12345678901234567890123456789012123456789012345678901 2
12345678901234567890123456789012123456789012345678901
misanthrope (noun) Someone who hates or distrusts all 2
12345678901234567890123456789012123456789012345678901
2345678901234567890123456789012123456789012345678901
2
1 people. In the beloved Christmas classic, It’s a Wonder- 2
12345678901234567890123456789012123456789012345678901 2
12345678901234567890123456789012123456789012345678901
ful Life, Lionel Barrymore plays Potter, the wealthy 2
12345678901234567890123456789012123456789012345678901 2
12345678901234567890123456789012123456789012345678901
misanthrope who is determined to make life miserable 2
12345678901234567890123456789012123456789012345678901 22
2345678901234567890123456789012123456789012345678901
12345678901234567890123456789012123456789012345678901

Take It to the Next Level


for everyone, and particularly for the young, idealistic
12345678901234567890123456789012123456789012345678901
George Bailey. misanthropic (adjective), misanthropy 2
2
12345678901234567890123456789012123456789012345678901
12345678901234567890123456789012123456789012345678901 2
12345678901234567890123456789012123456789012345678901
(noun). 2
1 2
12345678901234567890123456789012123456789012345678901 2
12345678901234567890123456789012123456789012345678901
miscreant (adjective) Unbelieving, heretical; evil, villainous. 2
12345678901234567890123456789012123456789012345678901 2
12345678901234567890123456789012123456789012345678901 22
2345678901234567890123456789012123456789012345678901
After a one-year run playing Iago in Othello, and then
12345678901234567890123456789012123456789012345678901
two years playing Bill Sikes in Oliver, Sean was tired of 2
12345678901234567890123456789012123456789012345678901
12345678901234567890123456789012123456789012345678901 2
12345678901234567890123456789012123456789012345678901
being typecast in miscreant roles. miscreant (noun). 2
12345678901234567890123456789012123456789012345678901 22
12345678901234567890123456789012123456789012345678901
mitigate (verb) To make less severe; to relieve. There’s no 2
12345678901234567890123456789012123456789012345678901
doubt that Wallace committed the assault, but the verbal 2
12345678901234567890123456789012123456789012345678901
1 2
2345678901234567890123456789012123456789012345678901
abuse Wallace had received helps to explain his behavior 2
12345678901234567890123456789012123456789012345678901
12345678901234567890123456789012123456789012345678901 2
1 and somewhat mitigates his guilt. mitigation (noun). 2
12345678901234567890123456789012123456789012345678901 2
12345678901234567890123456789012123456789012345678901 2
12345678901234567890123456789012123456789012345678901
monopoly (noun) A condition in which there is only one 2
12345678901234567890123456789012123456789012345678901 2
12345678901234567890123456789012123456789012345678901
seller of a certain commodity. Wary of Microsoft’s 2
12345678901234567890123456789012123456789012345678901
seeming monopoly of the computer operating-system 2
12345678901234567890123456789012123456789012345678901 2
2345678901234567890123456789012123456789012345678901
business, rivals are asking for government intervention. 2
12345678901234567890123456789012123456789012345678901
12345678901234567890123456789012123456789012345678901 2
1 2
12345678901234567890123456789012123456789012345678901 2
monopolistic (adjective).
12345678901234567890123456789012123456789012345678901 2
12345678901234567890123456789012123456789012345678901
monotonous (adjective) Tediously uniform, unchanging. 2
12345678901234567890123456789012123456789012345678901 2
12345678901234567890123456789012123456789012345678901 2
12345678901234567890123456789012123456789012345678901
Brian Eno’s “Music for Airports” is characterized by 2
12345678901234567890123456789012123456789012345678901 2
12345678901234567890123456789012123456789012345678901 22
2345678901234567890123456789012123456789012345678901
minimal melodies, subtle textures, and variable repeti-
12345678901234567890123456789012123456789012345678901
tion, which I find rather bland and monotonous.
2
12345678901234567890123456789012123456789012345678901 2
12345678901234567890123456789012123456789012345678901
monotony (noun).
2
1
2345678901234567890123456789012123456789012345678901
12345678901234567890123456789012123456789012345678901
morose (adjective) Gloomy, sullen. After Chuck’s girlfriend 2
2
12345678901234567890123456789012123456789012345678901 2
12345678901234567890123456789012123456789012345678901
dumped him, he lay around the house for a couple of 2
12345678901234567890123456789012123456789012345678901 2
1
12345678901234567890123456789012123456789012345678901
days, refusing to come to the phone and feeling morose. 2
12345678901234567890123456789012123456789012345678901 2
12345678901234567890123456789012123456789012345678901 2
12345678901234567890123456789012123456789012345678901 2
12345678901234567890123456789012123456789012345678901 2
12345678901234567890123456789012123456789012345678901 2
12345678901234567890123456789012123456789012345678901 2
12345678901234567890123456789012123456789012345678901 2
12345678901234567890123456789012123456789012345678901 2 203
123456789012345678901234567890121234567890123456789012
Part III: Section Review

123456789012345678901234567890121234567890123456789012
12345678901234567890123456789012123456789012345678901 2
12345678901234567890123456789012123456789012345678901 2
12345678901234567890123456789012123456789012345678901
mutation (noun) A significant change; in biology, a perma- 2
12345678901234567890123456789012123456789012345678901 2
12345678901234567890123456789012123456789012345678901
nent change in hereditary material. Most genetic muta- 2
12345678901234567890123456789012123456789012345678901 2
12345678901234567890123456789012123456789012345678901
tions are not beneficial, since any change in the delicate 2
2345678901234567890123456789012123456789012345678901 2
1
12345678901234567890123456789012123456789012345678901
balance of an organism tends to be disruptive. mutate 2
12345678901234567890123456789012123456789012345678901 2
12345678901234567890123456789012123456789012345678901
(verb). 2
12345678901234567890123456789012123456789012345678901
2345678901234567890123456789012123456789012345678901
2
2
1
12345678901234567890123456789012123456789012345678901
2345678901234567890123456789012123456789012345678901
2
2
1
12345678901234567890123456789012123456789012345678901
Word
Latin frangere 5 to break. Also found in English fraction, fractious,
Origin fracture, frangible, infraction, and refract. 2
12345678901234567890123456789012123456789012345678901
2345678901234567890123456789012123456789012345678901
2
2
1
12345678901234567890123456789012123456789012345678901 2
12345678901234567890123456789012123456789012345678901
nadir (noun) Lowest point. Pedro and Renee’s marriage 2
12345678901234567890123456789012123456789012345678901
reached a new nadir last Christmas Eve when Pedro 2
12345678901234567890123456789012123456789012345678901
2345678901234567890123456789012123456789012345678901
2
1 locked Renee out of the house upon her return from the 2
12345678901234567890123456789012123456789012345678901 2
12345678901234567890123456789012123456789012345678901
supposed “business trip.” 2
12345678901234567890123456789012123456789012345678901 2
12345678901234567890123456789012123456789012345678901 2
12345678901234567890123456789012123456789012345678901 2
2345678901234567890123456789012123456789012345678901
nascent (adjective) Newly born, just beginning. While her
12345678901234567890123456789012123456789012345678901
artistry was still nascent, it was 15-year-old Tara 2
2
12345678901234567890123456789012123456789012345678901
12345678901234567890123456789012123456789012345678901
Lipinski’s technical wizardry that enabled her to win a 2
2
12345678901234567890123456789012123456789012345678901
1 gold medal in the 1998 Winter Olympics. nascence 2
12345678901234567890123456789012123456789012345678901 2
12345678901234567890123456789012123456789012345678901 2
12345678901234567890123456789012123456789012345678901
(noun). 2
12345678901234567890123456789012123456789012345678901 2
2345678901234567890123456789012123456789012345678901
noisome (adjective) Putrid, fetid, noxious. We were con- 2
12345678901234567890123456789012123456789012345678901
12345678901234567890123456789012123456789012345678901 2
12345678901234567890123456789012123456789012345678901 2
12345678901234567890123456789012123456789012345678901 2
vinced that the noisome odor infiltrating every corner of
12345678901234567890123456789012123456789012345678901 2
12345678901234567890123456789012123456789012345678901
our building was evidence of a mouldering corpse. 2
12345678901234567890123456789012123456789012345678901 2
notorious (adjective) Famous, especially for evil actions or 2
12345678901234567890123456789012123456789012345678901
12345678901234567890123456789012123456789012345678901 2
1 qualities. Warner Brothers produced a series of movies 2
12345678901234567890123456789012123456789012345678901 2
2345678901234567890123456789012123456789012345678901
about notorious gangsters such as John Dillinger and Al 2
12345678901234567890123456789012123456789012345678901
1 2
12345678901234567890123456789012123456789012345678901
Capone. notoriety (noun). 2
12345678901234567890123456789012123456789012345678901 2
12345678901234567890123456789012123456789012345678901 2
2345678901234567890123456789012123456789012345678901
12345678901234567890123456789012123456789012345678901 2 2
1 Origin Latin durus 5 hard. Also found in English durable and endure.
12345678901234567890123456789012123456789012345678901
Word 2
12345678901234567890123456789012123456789012345678901 2
2345678901234567890123456789012123456789012345678901
obdurate (adjective) Unwilling to change; stubborn, inflex- 2
12345678901234567890123456789012123456789012345678901
12345678901234567890123456789012123456789012345678901 2
1 ible. Despite the many pleas he received, the governor 2
12345678901234567890123456789012123456789012345678901 2
12345678901234567890123456789012123456789012345678901
was obdurate in his refusal to grant clemency to the 2
12345678901234567890123456789012123456789012345678901 2
12345678901234567890123456789012123456789012345678901
convicted murderer. 2
12345678901234567890123456789012123456789012345678901 2
12345678901234567890123456789012123456789012345678901 2
12345678901234567890123456789012123456789012345678901 2
oblivious (adjective) Unaware, unconscious. Karen prac-
2345678901234567890123456789012123456789012345678901
12345678901234567890123456789012123456789012345678901
ticed her oboe solo with complete concentration, oblivi- 2
2
12345678901234567890123456789012123456789012345678901
ous to the noise and activity around her. oblivion (noun), 2
12345678901234567890123456789012123456789012345678901
12345678901234567890123456789012123456789012345678901 2
1 2
12345678901234567890123456789012123456789012345678901 2
2345678901234567890123456789012123456789012345678901
obviousness (noun).
12345678901234567890123456789012123456789012345678901 2
12345678901234567890123456789012123456789012345678901 2
12345678901234567890123456789012123456789012345678901 2 2
1
12345678901234567890123456789012123456789012345678901 2
12345678901234567890123456789012123456789012345678901 2
12345678901234567890123456789012123456789012345678901 2
12345678901234567890123456789012123456789012345678901 2
12345678901234567890123456789012123456789012345678901 2
12345678901234567890123456789012123456789012345678901 2
12345678901234567890123456789012123456789012345678901 2
12345678901234567890123456789012123456789012345678901 2 2
204 12345678901234567890123456789012123456789012345678901
123456789012345678901234567890121234567890123456789012

www.petersons.com
Chapter 5: Verbal Review

123456789012345678901234567890121234567890123456789012
12345678901234567890123456789012123456789012345678901 2
12345678901234567890123456789012123456789012345678901 2
12345678901234567890123456789012123456789012345678901
obscure (adjective) Little known; hard to understand. Mendel 2
12345678901234567890123456789012123456789012345678901 2
12345678901234567890123456789012123456789012345678901
was an obscure monk until decades after his death, when 2
12345678901234567890123456789012123456789012345678901 2
12345678901234567890123456789012123456789012345678901
his scientific work was finally discovered. Most people 2
2345678901234567890123456789012123456789012345678901 2
1
12345678901234567890123456789012123456789012345678901
find the writings of James Joyce obscure; hence the 2
12345678901234567890123456789012123456789012345678901
popularity of books that explain the many odd refer- 2
12345678901234567890123456789012123456789012345678901 2
12345678901234567890123456789012123456789012345678901
2345678901234567890123456789012123456789012345678901
2
2
1 ences and tricks of language in his work. obscure (verb),
12345678901234567890123456789012123456789012345678901 2
12345678901234567890123456789012123456789012345678901
obscurity (noun). 2
12345678901234567890123456789012123456789012345678901 2
12345678901234567890123456789012123456789012345678901
2345678901234567890123456789012123456789012345678901
2
2
1 obsolete (adjective) No longer current; old-fashioned. W. H.
12345678901234567890123456789012123456789012345678901 2
12345678901234567890123456789012123456789012345678901
Auden said that his ideal landscape would contain water 2
12345678901234567890123456789012123456789012345678901
wheels, grain mills, and other forms of obsolete machin- 2
12345678901234567890123456789012123456789012345678901
2345678901234567890123456789012123456789012345678901
2
2
1 ery. obsolescence (noun).
12345678901234567890123456789012123456789012345678901 2
12345678901234567890123456789012123456789012345678901 2
12345678901234567890123456789012123456789012345678901
obstinate (adjective) Stubborn, unyielding. Despite years of 2
12345678901234567890123456789012123456789012345678901 2
12345678901234567890123456789012123456789012345678901
government effort, the problem of drug abuse remains 2
12345678901234567890123456789012123456789012345678901 2

Take It to the Next Level


12345678901234567890123456789012123456789012345678901
obstinate. obstinacy (noun). 2
12345678901234567890123456789012123456789012345678901 2
12345678901234567890123456789012123456789012345678901 2
12345678901234567890123456789012123456789012345678901 22
2345678901234567890123456789012123456789012345678901
obtuse (adjective) Dull-witted, insensitive; incomprehen-
1
12345678901234567890123456789012123456789012345678901
sible, unclear, or imprecise. Amy was so obtuse she 2
12345678901234567890123456789012123456789012345678901
didn’t realize that Alexi had proposed marriage to her. 2
12345678901234567890123456789012123456789012345678901 2
2345678901234567890123456789012123456789012345678901
French psychoanalyst Jacques Lacan’s collection of 2
12345678901234567890123456789012123456789012345678901
12345678901234567890123456789012123456789012345678901 2
12345678901234567890123456789012123456789012345678901 2
12345678901234567890123456789012123456789012345678901 22
papers, Ecrits, is notoriously obtuse, yet it has still been
12345678901234567890123456789012123456789012345678901
highly influential in linguistics, film theory, and literary 2
12345678901234567890123456789012123456789012345678901
12345678901234567890123456789012123456789012345678901 2
criticism.
12345678901234567890123456789012123456789012345678901 2
12345678901234567890123456789012123456789012345678901 22
1 obviate (verb) Preclude, make unnecessary. Truman Capote’s
12345678901234567890123456789012123456789012345678901 2
12345678901234567890123456789012123456789012345678901
meticulous accuracy and total recall obviated the need 2
12345678901234567890123456789012123456789012345678901 2
12345678901234567890123456789012123456789012345678901
for note-taking when he wrote his account of a 1959 2
12345678901234567890123456789012123456789012345678901 2
12345678901234567890123456789012123456789012345678901
murder, In Cold Blood. 2
12345678901234567890123456789012123456789012345678901 2
12345678901234567890123456789012123456789012345678901 2
12345678901234567890123456789012123456789012345678901
odium (noun) Intense feeling of hatred, abhorrence. When 2
12345678901234567890123456789012123456789012345678901 2
12345678901234567890123456789012123456789012345678901 22
2345678901234567890123456789012123456789012345678901
the neighbors learned that a convicted sex offender was
12345678901234567890123456789012123456789012345678901
now living in their midst, they could not restrain their 2
1
12345678901234567890123456789012123456789012345678901
odium and began harassing the man whenever he left his 2
12345678901234567890123456789012123456789012345678901 2
2345678901234567890123456789012123456789012345678901
12345678901234567890123456789012123456789012345678901 2
12345678901234567890123456789012123456789012345678901
house. odious (adjective). 2
12345678901234567890123456789012123456789012345678901 22
12345678901234567890123456789012123456789012345678901
opprobrium (noun) Dishonor, disapproval. Switzerland re-
2
1
2345678901234567890123456789012123456789012345678901
cently came under public opprobrium when it was 2
12345678901234567890123456789012123456789012345678901
12345678901234567890123456789012123456789012345678901 2
revealed that Swiss bankers had hoarded the gold the 2
12345678901234567890123456789012123456789012345678901
12345678901234567890123456789012123456789012345678901 22
1 Nazis had confiscated from their victims. opprobrious
12345678901234567890123456789012123456789012345678901 2
12345678901234567890123456789012123456789012345678901
(adjective). 2
12345678901234567890123456789012123456789012345678901 2
12345678901234567890123456789012123456789012345678901 2
12345678901234567890123456789012123456789012345678901 2
orthodox (adjective) In religion, conforming to a certain
2345678901234567890123456789012123456789012345678901
doctrine; conventional. George Eliot’s relationship with 2
12345678901234567890123456789012123456789012345678901
1 2
12345678901234567890123456789012123456789012345678901 2
12345678901234567890123456789012123456789012345678901 2
George Lewes, a married journalist, offended the
12345678901234567890123456789012123456789012345678901
sensibilities of her more orthodox peers. orthodoxy 2
12345678901234567890123456789012123456789012345678901 2
12345678901234567890123456789012123456789012345678901 2
12345678901234567890123456789012123456789012345678901 22
(noun).
12345678901234567890123456789012123456789012345678901 205
123456789012345678901234567890121234567890123456789012
Part III: Section Review

123456789012345678901234567890121234567890123456789012
12345678901234567890123456789012123456789012345678901 2
12345678901234567890123456789012123456789012345678901 2
12345678901234567890123456789012123456789012345678901
ossified (adjective) In biology, to turn into bone; to become 2
12345678901234567890123456789012123456789012345678901 2
12345678901234567890123456789012123456789012345678901
rigidly conventional and opposed to change. His harsh 2
12345678901234567890123456789012123456789012345678901 2
12345678901234567890123456789012123456789012345678901
view of co-education had ossified over the years so that 2
2345678901234567890123456789012123456789012345678901 2
1
12345678901234567890123456789012123456789012345678901
he was now the only teacher who sought to bar girls 2
12345678901234567890123456789012123456789012345678901 2
12345678901234567890123456789012123456789012345678901
from the venerable boys’ school. ossification (noun). 2
12345678901234567890123456789012123456789012345678901
2345678901234567890123456789012123456789012345678901
2
1 ostentatious (adjective) Overly showy, pretentious. To show 2
12345678901234567890123456789012123456789012345678901 2
12345678901234567890123456789012123456789012345678901 2
12345678901234567890123456789012123456789012345678901
off his new wealth, the financier threw an ostentatious 2
12345678901234567890123456789012123456789012345678901
2345678901234567890123456789012123456789012345678901
2
2
1 party featuring a full orchestra, a famous singer, and tens
12345678901234567890123456789012123456789012345678901 2
12345678901234567890123456789012123456789012345678901
of thousands of dollars’ worth of food. ostentation 2
12345678901234567890123456789012123456789012345678901
(noun). 2
12345678901234567890123456789012123456789012345678901
2345678901234567890123456789012123456789012345678901
2
2
1
12345678901234567890123456789012123456789012345678901
ostracize (verb) To exclude from a group. In Biblical times, 2
12345678901234567890123456789012123456789012345678901 2
12345678901234567890123456789012123456789012345678901
those who suffered from the disease of leprosy were 2
12345678901234567890123456789012123456789012345678901 2
12345678901234567890123456789012123456789012345678901
ostracized and forced to live alone. ostracism (noun). 2
12345678901234567890123456789012123456789012345678901 2
12345678901234567890123456789012123456789012345678901 2
12345678901234567890123456789012123456789012345678901
paean (adjective) A joyous expression of praise, gratitude, 2
12345678901234567890123456789012123456789012345678901 2
12345678901234567890123456789012123456789012345678901 2
2345678901234567890123456789012123456789012345678901
or triumph. Choreographer Paul Taylor’s dance “Even-
1 tide” is a sublime paean to remembered love, with 2
12345678901234567890123456789012123456789012345678901 2
12345678901234567890123456789012123456789012345678901 2
12345678901234567890123456789012123456789012345678901
couple after loving couple looking back as they embrace 2
2345678901234567890123456789012123456789012345678901
12345678901234567890123456789012123456789012345678901 2
12345678901234567890123456789012123456789012345678901
an unknown future. 2
12345678901234567890123456789012123456789012345678901 2
parody (noun) An imitation created for comic effect; a 2
12345678901234567890123456789012123456789012345678901
2
12345678901234567890123456789012123456789012345678901
caricature. While the creators of the 1970s comedy 2
12345678901234567890123456789012123456789012345678901
12345678901234567890123456789012123456789012345678901 2
series All in the Family intended Archie Bunker to be a 2
12345678901234567890123456789012123456789012345678901
12345678901234567890123456789012123456789012345678901 2
1 parody of closed-mindedeness in Americans, large num- 2
12345678901234567890123456789012123456789012345678901 2
12345678901234567890123456789012123456789012345678901
bers of people adopted Bunker as a working-class hero. 2
12345678901234567890123456789012123456789012345678901 2
12345678901234567890123456789012123456789012345678901 2
12345678901234567890123456789012123456789012345678901
parse (verb) To break a sentence down into grammatical 2
2345678901234567890123456789012123456789012345678901
components; to analyze bit by bit. In the wake of the sex 2
12345678901234567890123456789012123456789012345678901
12345678901234567890123456789012123456789012345678901 2
1 2
12345678901234567890123456789012123456789012345678901
scandal, journalists parsed every utterance by adminis- 2
12345678901234567890123456789012123456789012345678901 2
12345678901234567890123456789012123456789012345678901 2
2345678901234567890123456789012123456789012345678901
tration officials regarding the president’s alleged promis-
1 cuity. At $1.25 million a day, Titanic is the most 2
12345678901234567890123456789012123456789012345678901 2
12345678901234567890123456789012123456789012345678901 2
12345678901234567890123456789012123456789012345678901 2
expensive movie ever made, but director James Cameron
2345678901234567890123456789012123456789012345678901
refused to parse the film’s enormous budget for inquisi- 2
12345678901234567890123456789012123456789012345678901
12345678901234567890123456789012123456789012345678901 2
12345678901234567890123456789012123456789012345678901 2
12345678901234567890123456789012123456789012345678901 2
tive reporters.
2
1
12345678901234567890123456789012123456789012345678901
partisan (adjective) Reflecting strong allegiance to a particu- 2
12345678901234567890123456789012123456789012345678901 2
12345678901234567890123456789012123456789012345678901
lar party or cause. The vote on the president’s budget 2
12345678901234567890123456789012123456789012345678901 2
12345678901234567890123456789012123456789012345678901
was strictly partisan: Every member of the president’s 2
2345678901234567890123456789012123456789012345678901
12345678901234567890123456789012123456789012345678901
party voted yes, and all others voted no. partisan (noun). 2
2
12345678901234567890123456789012123456789012345678901
12345678901234567890123456789012123456789012345678901 2
1 pastoral (adjective) Simple and rustic, bucolic, rural. While 2
12345678901234567890123456789012123456789012345678901 2
2345678901234567890123456789012123456789012345678901
industry grew and the country expanded westward, the 2
12345678901234567890123456789012123456789012345678901
1 2
12345678901234567890123456789012123456789012345678901 2
12345678901234567890123456789012123456789012345678901 2
Hudson River School of painters depicted the landscape
12345678901234567890123456789012123456789012345678901
as a pastoral setting where humans and nature could 2
12345678901234567890123456789012123456789012345678901 2
12345678901234567890123456789012123456789012345678901 2
12345678901234567890123456789012123456789012345678901 2
coexist.
2
206 12345678901234567890123456789012123456789012345678901
123456789012345678901234567890121234567890123456789012

www.petersons.com
Chapter 5: Verbal Review

123456789012345678901234567890121234567890123456789012
12345678901234567890123456789012123456789012345678901 2
12345678901234567890123456789012123456789012345678901 2
12345678901234567890123456789012123456789012345678901
patron (noun) A special guardian or protector; a wealthy or 2
12345678901234567890123456789012123456789012345678901 2
12345678901234567890123456789012123456789012345678901
influential supporter of the arts. Dominique de Menil 2
12345678901234567890123456789012123456789012345678901 2
12345678901234567890123456789012123456789012345678901
used her considerable wealth to become a well-known 2
2345678901234567890123456789012123456789012345678901 2
1
12345678901234567890123456789012123456789012345678901
patron of the arts; she and her husband owned a 2
12345678901234567890123456789012123456789012345678901
collection of more than 10,000 pieces ranging from 2
12345678901234567890123456789012123456789012345678901 2
12345678901234567890123456789012123456789012345678901
2345678901234567890123456789012123456789012345678901
2
2
1 cubist paintings to tribal artifacts. patronize (verb).
12345678901234567890123456789012123456789012345678901 2
12345678901234567890123456789012123456789012345678901 2
12345678901234567890123456789012123456789012345678901
peccadillo (noun) A minor offense, a lapse. What Dr. Sykes 2
12345678901234567890123456789012123456789012345678901
2345678901234567890123456789012123456789012345678901
2
2
1 saw as a major offense—being addressed as Marge
12345678901234567890123456789012123456789012345678901 2
12345678901234567890123456789012123456789012345678901
rather than Doctor—Tina saw as a mere peccadillo and 2
12345678901234567890123456789012123456789012345678901
one that certainly should not have lost her the job. 2
12345678901234567890123456789012123456789012345678901
2345678901234567890123456789012123456789012345678901
2
2
1
12345678901234567890123456789012123456789012345678901
pedantic (adjective) Academic, bookish. The men Hillary 2
12345678901234567890123456789012123456789012345678901 2
12345678901234567890123456789012123456789012345678901
met through personal ads in the New York Review of 2
12345678901234567890123456789012123456789012345678901 2
12345678901234567890123456789012123456789012345678901
Books were invariably pasty-skinned pedantic types 2
12345678901234567890123456789012123456789012345678901 2

Take It to the Next Level


12345678901234567890123456789012123456789012345678901
who dropped the names of nineteenth-century writers in 2
12345678901234567890123456789012123456789012345678901 2
12345678901234567890123456789012123456789012345678901
every sentence. pedantry (noun). 2
12345678901234567890123456789012123456789012345678901 22
2345678901234567890123456789012123456789012345678901
1
12345678901234567890123456789012123456789012345678901
pedestrian (adjective) Unimaginative, ordinary. The new 2
12345678901234567890123456789012123456789012345678901
Italian restaurant received a bad review due to its 2
12345678901234567890123456789012123456789012345678901 2
2345678901234567890123456789012123456789012345678901
reliance on pedestrian dishes such as pasta with marinara 2
12345678901234567890123456789012123456789012345678901
12345678901234567890123456789012123456789012345678901 2
12345678901234567890123456789012123456789012345678901 2
12345678901234567890123456789012123456789012345678901 22
sauce or chicken parmigiana.
12345678901234567890123456789012123456789012345678901 2
12345678901234567890123456789012123456789012345678901 2
12345678901234567890123456789012123456789012345678901
12345678901234567890123456789012123456789012345678901
Latin fides 5 faith. Also found in English confide, confidence, 2
12345678901234567890123456789012123456789012345678901
Word Origin fidelity, and infidel. 2
1 2
12345678901234567890123456789012123456789012345678901 22
2345678901234567890123456789012123456789012345678901
12345678901234567890123456789012123456789012345678901
perfidious (adjective) Disloyal, treacherous. Although he
2
1
12345678901234567890123456789012123456789012345678901
was one of the most talented generals of the American 2
12345678901234567890123456789012123456789012345678901 2
12345678901234567890123456789012123456789012345678901
Revolution, Benedict Arnold is remembered today as a 2
12345678901234567890123456789012123456789012345678901 2
12345678901234567890123456789012123456789012345678901
perfidious betrayer of the patriot cause. perfidy (noun). 2
12345678901234567890123456789012123456789012345678901 2
12345678901234567890123456789012123456789012345678901 2
12345678901234567890123456789012123456789012345678901 22
2345678901234567890123456789012123456789012345678901
peripatetic (adjective) Moving or traveling from place to
12345678901234567890123456789012123456789012345678901
place; always on the go. In Barbara Wilson’s Trouble in 2
1
12345678901234567890123456789012123456789012345678901
Transylvania, peripatetic translator Cassandra Reilly is 2
12345678901234567890123456789012123456789012345678901 2
2345678901234567890123456789012123456789012345678901
on the road again, this time to China by way of 2
12345678901234567890123456789012123456789012345678901
12345678901234567890123456789012123456789012345678901 2
12345678901234567890123456789012123456789012345678901 2
12345678901234567890123456789012123456789012345678901 22
Budapest, where she plans to catch the TransMongolian
1
12345678901234567890123456789012123456789012345678901
Express. 2
12345678901234567890123456789012123456789012345678901 2
12345678901234567890123456789012123456789012345678901
permeate (verb) To spread through or penetrate. Little by 2
12345678901234567890123456789012123456789012345678901 2
12345678901234567890123456789012123456789012345678901
little, the smell of gas from the broken pipe permeated 2
2345678901234567890123456789012123456789012345678901
12345678901234567890123456789012123456789012345678901 2
12345678901234567890123456789012123456789012345678901
the house. 2
12345678901234567890123456789012123456789012345678901 22
12345678901234567890123456789012123456789012345678901
personification (noun) The embodiment of a thing or an 2
1
12345678901234567890123456789012123456789012345678901
abstract idea in human form. Many people view 2
12345678901234567890123456789012123456789012345678901 2
12345678901234567890123456789012123456789012345678901 2
12345678901234567890123456789012123456789012345678901 2
Theodore Kaczynski, the killer known as the Unabomber,
12345678901234567890123456789012123456789012345678901 2
12345678901234567890123456789012123456789012345678901
as the very personification of evil. personify (verb). 2
12345678901234567890123456789012123456789012345678901 2
12345678901234567890123456789012123456789012345678901 2
12345678901234567890123456789012123456789012345678901 2 207
123456789012345678901234567890121234567890123456789012
Part III: Section Review

123456789012345678901234567890121234567890123456789012
12345678901234567890123456789012123456789012345678901 2
12345678901234567890123456789012123456789012345678901 2
12345678901234567890123456789012123456789012345678901
pervasive (adjective) Spreading throughout. As news of the 2
12345678901234567890123456789012123456789012345678901 2
12345678901234567890123456789012123456789012345678901
disaster reached the town, a pervasive sense of gloom 2
12345678901234567890123456789012123456789012345678901 2
12345678901234567890123456789012123456789012345678901
could be felt everywhere. pervade (verb).
2345678901234567890123456789012123456789012345678901
2
2
1
12345678901234567890123456789012123456789012345678901 2
12345678901234567890123456789012123456789012345678901
philistine (noun) Someone who is smugly ignorant and 2
12345678901234567890123456789012123456789012345678901 2
12345678901234567890123456789012123456789012345678901
uncultured. A true philistine,
2345678901234567890123456789012123456789012345678901
Meg claimed she didn’t 2
1 read any book that wasn’t either recommended by 2
12345678901234567890123456789012123456789012345678901 2
12345678901234567890123456789012123456789012345678901 2
12345678901234567890123456789012123456789012345678901
Oprah Winfrey or on the best-seller list. philistine 2
12345678901234567890123456789012123456789012345678901
2345678901234567890123456789012123456789012345678901
2
2
1 (adjective).
12345678901234567890123456789012123456789012345678901 2
12345678901234567890123456789012123456789012345678901
pith (noun) The core, the essential part; in biology, the 2
12345678901234567890123456789012123456789012345678901 2
12345678901234567890123456789012123456789012345678901
central strand of tissue in the stems of most vascular 2
2345678901234567890123456789012123456789012345678901 2
1
12345678901234567890123456789012123456789012345678901
plants. After spending seventeen years in psychoanaly- 2
12345678901234567890123456789012123456789012345678901 2
12345678901234567890123456789012123456789012345678901
sis, Frieda had finally come face to face with the pith of 2
12345678901234567890123456789012123456789012345678901 2
12345678901234567890123456789012123456789012345678901
her deep-seated anxiety. pithy (adjective). 2
12345678901234567890123456789012123456789012345678901 2
12345678901234567890123456789012123456789012345678901 2
12345678901234567890123456789012123456789012345678901
placate (verb) To soothe or appease. The waiter tried to 2
12345678901234567890123456789012123456789012345678901 2
12345678901234567890123456789012123456789012345678901 2
2345678901234567890123456789012123456789012345678901
placate the angry customer with the offer of a free
2
1
12345678901234567890123456789012123456789012345678901
dessert. placatory (adjective). 2
12345678901234567890123456789012123456789012345678901 2
12345678901234567890123456789012123456789012345678901 2
12345678901234567890123456789012123456789012345678901 2
2345678901234567890123456789012123456789012345678901
placid (adjective) Unmarked by disturbance; complacent.
Dr. Kahn was convinced that the placid exterior 2
12345678901234567890123456789012123456789012345678901
12345678901234567890123456789012123456789012345678901 2
presented by Frieda in her early analysis sessions masked 2
12345678901234567890123456789012123456789012345678901
12345678901234567890123456789012123456789012345678901 2
12345678901234567890123456789012123456789012345678901
a deeply disturbed psyche. placidity (noun). 2
12345678901234567890123456789012123456789012345678901 2
12345678901234567890123456789012123456789012345678901
plaintive (adjective) Expressing suffering or melancholy. In 2
2
12345678901234567890123456789012123456789012345678901 2
1
12345678901234567890123456789012123456789012345678901
the beloved children’s book The Secret Garden, Mary is 2
12345678901234567890123456789012123456789012345678901 2
12345678901234567890123456789012123456789012345678901
disturbed by plaintive cries echoing in the corridors of 2
12345678901234567890123456789012123456789012345678901 2
12345678901234567890123456789012123456789012345678901
gloomy Misselthwaite Manor. 2
12345678901234567890123456789012123456789012345678901 2
2345678901234567890123456789012123456789012345678901
1 plastic (adjective) Able to be molded or reshaped. Because it 2
12345678901234567890123456789012123456789012345678901 2
12345678901234567890123456789012123456789012345678901 2
12345678901234567890123456789012123456789012345678901
is highly plastic, clay is an easy material for beginning 2
2345678901234567890123456789012123456789012345678901
12345678901234567890123456789012123456789012345678901 2
12345678901234567890123456789012123456789012345678901
sculptors to use. plasticity (noun). 2
1 2
12345678901234567890123456789012123456789012345678901
plausible (adjective) Apparently believable. The idea that a 2
12345678901234567890123456789012123456789012345678901 2
12345678901234567890123456789012123456789012345678901
widespread conspiracy to kill the president has been 2
12345678901234567890123456789012123456789012345678901 2
12345678901234567890123456789012123456789012345678901
kept secret by all the participants for over thirty years 2
12345678901234567890123456789012123456789012345678901 2
12345678901234567890123456789012123456789012345678901
hardly seems plausible. plausibility (noun). 2
12345678901234567890123456789012123456789012345678901 2
2345678901234567890123456789012123456789012345678901
2
12345678901234567890123456789012123456789012345678901
platitude (noun) A trite remark or saying; a cliché. How 2
12345678901234567890123456789012123456789012345678901
1 typical of June to send a sympathy card filled with 2
12345678901234567890123456789012123456789012345678901 2
2345678901234567890123456789012123456789012345678901
mindless platitudes like “One day at a time,” rather than 2
12345678901234567890123456789012123456789012345678901
12345678901234567890123456789012123456789012345678901 2
12345678901234567890123456789012123456789012345678901 2
12345678901234567890123456789012123456789012345678901 2
calling the grieving widow on the phone. platitudinous
2
1
12345678901234567890123456789012123456789012345678901
(adjective). 2
12345678901234567890123456789012123456789012345678901 2
12345678901234567890123456789012123456789012345678901
plummet (verb) To dive or plunge. On October 27, 1997, 2
12345678901234567890123456789012123456789012345678901 2
12345678901234567890123456789012123456789012345678901
the stock market plummeted by
2345678901234567890123456789012123456789012345678901 554 points and left us all 2
2
1
12345678901234567890123456789012123456789012345678901
wondering if the bull market was finally over. 2
12345678901234567890123456789012123456789012345678901 2 2
208 12345678901234567890123456789012123456789012345678901
123456789012345678901234567890121234567890123456789012

www.petersons.com
Chapter 5: Verbal Review

123456789012345678901234567890121234567890123456789012
12345678901234567890123456789012123456789012345678901 2
12345678901234567890123456789012123456789012345678901 2
12345678901234567890123456789012123456789012345678901
polarize (adjective) To separate into opposing groups or 2
12345678901234567890123456789012123456789012345678901 2
12345678901234567890123456789012123456789012345678901
forces. For years, the abortion debate polarized the 2
12345678901234567890123456789012123456789012345678901 2
12345678901234567890123456789012123456789012345678901
American people, with many people voicing views at 2
2345678901234567890123456789012123456789012345678901 2
1
12345678901234567890123456789012123456789012345678901
either extreme and few people trying to find a middle 2
12345678901234567890123456789012123456789012345678901 2
12345678901234567890123456789012123456789012345678901
ground. polarization (noun). 2
12345678901234567890123456789012123456789012345678901
2345678901234567890123456789012123456789012345678901
2
1 ponderous (adjective) Unwieldy and bulky; oppressively 2
12345678901234567890123456789012123456789012345678901 2
12345678901234567890123456789012123456789012345678901 2
12345678901234567890123456789012123456789012345678901
dull. Unfortunately, the film director weighed the movie 2
12345678901234567890123456789012123456789012345678901
2345678901234567890123456789012123456789012345678901
2
2
1 down with a ponderous voice-over narrated by the
12345678901234567890123456789012123456789012345678901 2
12345678901234567890123456789012123456789012345678901
protagonist as an old man. 2
12345678901234567890123456789012123456789012345678901 2
12345678901234567890123456789012123456789012345678901
poseur (noun) Someone who pretends to be what he isn’t. 2
2345678901234567890123456789012123456789012345678901 2
1
12345678901234567890123456789012123456789012345678901
Gerald had pretensions for literary stardom with his 2
12345678901234567890123456789012123456789012345678901 2
12345678901234567890123456789012123456789012345678901
book proposal on an obscure World War II battle, yet 2
12345678901234567890123456789012123456789012345678901 2
12345678901234567890123456789012123456789012345678901
most agents soon realized that the book would never be 2
12345678901234567890123456789012123456789012345678901 2

Take It to the Next Level


12345678901234567890123456789012123456789012345678901
written and categorized him as a poseur. 2
12345678901234567890123456789012123456789012345678901 2
12345678901234567890123456789012123456789012345678901 2
12345678901234567890123456789012123456789012345678901 22
2345678901234567890123456789012123456789012345678901
positivism (noun) A philosophy that denies speculation and
1
12345678901234567890123456789012123456789012345678901
assumes that the only knowledge is scientific knowledge. 2
12345678901234567890123456789012123456789012345678901
David Hume carried his positivism to an extreme when 2
12345678901234567890123456789012123456789012345678901 2
2345678901234567890123456789012123456789012345678901
he argued that our expectation that the sun will rise 2
12345678901234567890123456789012123456789012345678901
12345678901234567890123456789012123456789012345678901 2
12345678901234567890123456789012123456789012345678901 2
12345678901234567890123456789012123456789012345678901 22
tomorrow has no basis in reason and is purely a matter
12345678901234567890123456789012123456789012345678901
12345678901234567890123456789012123456789012345678901
of belief. positivistic (adjective). 2
12345678901234567890123456789012123456789012345678901 22
12345678901234567890123456789012123456789012345678901
pragmatism (noun) A belief in approaching problems
12345678901234567890123456789012123456789012345678901 2
1 through practical rather than theoretical means. 2
12345678901234567890123456789012123456789012345678901 22
2345678901234567890123456789012123456789012345678901
12345678901234567890123456789012123456789012345678901
Roosevelt’s attitude toward the economic troubles of the
2
1
12345678901234567890123456789012123456789012345678901
Depression was based on pragmatism: “Try some- 2
12345678901234567890123456789012123456789012345678901 2
12345678901234567890123456789012123456789012345678901
thing,” he said; “If it doesn’t work, try something else.” 2
12345678901234567890123456789012123456789012345678901 2
12345678901234567890123456789012123456789012345678901
pragmatic (adjective). 2
12345678901234567890123456789012123456789012345678901 2
12345678901234567890123456789012123456789012345678901 2
12345678901234567890123456789012123456789012345678901 22
2345678901234567890123456789012123456789012345678901
precedent (noun) An earlier occurrence that serves as an
12345678901234567890123456789012123456789012345678901
example for a decision. In a legal system that reveres 2
1
12345678901234567890123456789012123456789012345678901
precedent, even defining the nature of a completely new 2
12345678901234567890123456789012123456789012345678901 2
2345678901234567890123456789012123456789012345678901
12345678901234567890123456789012123456789012345678901 2
12345678901234567890123456789012123456789012345678901
type of dispute can seem impossible. precede (verb). 2
12345678901234567890123456789012123456789012345678901 22
12345678901234567890123456789012123456789012345678901
precept (noun) A general principle or law. One of the
2
1
2345678901234567890123456789012123456789012345678901
central precepts of Tai Chi Ch’uan is the necessity of 2
12345678901234567890123456789012123456789012345678901
12345678901234567890123456789012123456789012345678901 2
allowing ki (cosmic energy) to flow through one’s body 2
12345678901234567890123456789012123456789012345678901
12345678901234567890123456789012123456789012345678901 22
1 in slow, graceful movements.
12345678901234567890123456789012123456789012345678901 2
12345678901234567890123456789012123456789012345678901 2
12345678901234567890123456789012123456789012345678901
precipitate (verb) To spur or activate. In the summer of 2
12345678901234567890123456789012123456789012345678901
1997, the selling off of the Thai baht precipitated a 2
12345678901234567890123456789012123456789012345678901 2
2345678901234567890123456789012123456789012345678901
12345678901234567890123456789012123456789012345678901 2
1 currency crisis that spread throughout Asia. 2
12345678901234567890123456789012123456789012345678901 2
12345678901234567890123456789012123456789012345678901 2
12345678901234567890123456789012123456789012345678901 2
12345678901234567890123456789012123456789012345678901 2
12345678901234567890123456789012123456789012345678901 2
12345678901234567890123456789012123456789012345678901 2
12345678901234567890123456789012123456789012345678901 2 209
123456789012345678901234567890121234567890123456789012
Part III: Section Review

123456789012345678901234567890121234567890123456789012
12345678901234567890123456789012123456789012345678901 2
12345678901234567890123456789012123456789012345678901 2
12345678901234567890123456789012123456789012345678901 2
12345678901234567890123456789012123456789012345678901
Latin claudere 5 to close. Also found in English conclude, include, 2
12345678901234567890123456789012123456789012345678901
Word Origin 2
12345678901234567890123456789012123456789012345678901 2
recluse, and seclude.
12345678901234567890123456789012123456789012345678901 2
12345678901234567890123456789012123456789012345678901
preclude (verb) To prevent, to hinder. Unfortunately, Jas- 2
12345678901234567890123456789012123456789012345678901 2
12345678901234567890123456789012123456789012345678901
mine’s appointment at the New Age Expo precluded her 2
12345678901234567890123456789012123456789012345678901 2
12345678901234567890123456789012123456789012345678901
attendance at our weekend Workshop for Shamans and 2
2345678901234567890123456789012123456789012345678901 2
1
12345678901234567890123456789012123456789012345678901
Psychics. preclusive (adjective), preclusion (noun). 2
12345678901234567890123456789012123456789012345678901 2
12345678901234567890123456789012123456789012345678901 2
12345678901234567890123456789012123456789012345678901
precursor (noun) A forerunner,
2345678901234567890123456789012123456789012345678901
a predecessor. The Kodak 2
1 Brownie camera, a small boxy camera made of jute 2
12345678901234567890123456789012123456789012345678901 2
12345678901234567890123456789012123456789012345678901 2
12345678901234567890123456789012123456789012345678901
board and wood, was the precursor to today’s sleek 2
12345678901234567890123456789012123456789012345678901
2345678901234567890123456789012123456789012345678901
2
2
1 35mm cameras. precursory (adjective).
12345678901234567890123456789012123456789012345678901 2
12345678901234567890123456789012123456789012345678901
preponderance (noun) A superiority in weight, size, or 2
12345678901234567890123456789012123456789012345678901 2
12345678901234567890123456789012123456789012345678901
quantity; a majority. In Seattle, there is a great prepon- 2
12345678901234567890123456789012123456789012345678901 2
2345678901234567890123456789012123456789012345678901
derance of seasonal affective disorder, or SAD, a malady 2
12345678901234567890123456789012123456789012345678901
12345678901234567890123456789012123456789012345678901
brought on by light starvation during the dark North- 2
2
12345678901234567890123456789012123456789012345678901
12345678901234567890123456789012123456789012345678901 2
12345678901234567890123456789012123456789012345678901
west winter. preponderate (verb). 2
1 2
12345678901234567890123456789012123456789012345678901 2
12345678901234567890123456789012123456789012345678901
presage (verb) To foretell, to anticipate. According to 2
12345678901234567890123456789012123456789012345678901 2
12345678901234567890123456789012123456789012345678901 2
2345678901234567890123456789012123456789012345678901
folklore, a red sky at dawn presages a day of stormy
12345678901234567890123456789012123456789012345678901 2
12345678901234567890123456789012123456789012345678901
weather. 2
12345678901234567890123456789012123456789012345678901 2
prescience (noun) Foreknowledge or foresight. When she 2
12345678901234567890123456789012123456789012345678901
12345678901234567890123456789012123456789012345678901 2
12345678901234567890123456789012123456789012345678901 2
12345678901234567890123456789012123456789012345678901 2
saw the characteristic eerie yellowish-black light in the
sky, Dorothy had the prescience to seek shelter in the 2
12345678901234567890123456789012123456789012345678901
1 2
2345678901234567890123456789012123456789012345678901
12345678901234567890123456789012123456789012345678901
storm cellar. prescient (adjective). 2
12345678901234567890123456789012123456789012345678901 2
1 2
12345678901234567890123456789012123456789012345678901
presumptuous (adjective) Going beyond the limits of cour- 2
12345678901234567890123456789012123456789012345678901 2
12345678901234567890123456789012123456789012345678901
tesy or appropriateness. The senator winced when the 2
12345678901234567890123456789012123456789012345678901 2
12345678901234567890123456789012123456789012345678901
presumptuous young staffer addressed him as “Ted.” 2
12345678901234567890123456789012123456789012345678901 2
12345678901234567890123456789012123456789012345678901
presume (verb), presumption (noun). 2
12345678901234567890123456789012123456789012345678901 2
2345678901234567890123456789012123456789012345678901
1 prevaricate (verb) To lie, to equivocate. When it became 2
12345678901234567890123456789012123456789012345678901 2
12345678901234567890123456789012123456789012345678901 2
12345678901234567890123456789012123456789012345678901 2
clear to the FBI that the mobster had threatened the
2345678901234567890123456789012123456789012345678901
12-year-old witness, they could well understand why he 2
12345678901234567890123456789012123456789012345678901
12345678901234567890123456789012123456789012345678901 2
12345678901234567890123456789012123456789012345678901 2
12345678901234567890123456789012123456789012345678901 2
had prevaricated during the hearing.
2
1
12345678901234567890123456789012123456789012345678901 2
12345678901234567890123456789012123456789012345678901 2
12345678901234567890123456789012123456789012345678901
Latin primus 5 first. Also found in English primate, primitive, 2
12345678901234567890123456789012123456789012345678901
Word Origin 2
12345678901234567890123456789012123456789012345678901 2
primogeniture, and primodial.
2345678901234567890123456789012123456789012345678901
12345678901234567890123456789012123456789012345678901
primacy (noun) State of being the utmost in importance; 2
2
12345678901234567890123456789012123456789012345678901 2
12345678901234567890123456789012123456789012345678901
preeminence. The anthropologist Ruth Benedict was an 2
12345678901234567890123456789012123456789012345678901 2
1
12345678901234567890123456789012123456789012345678901
inspiration to Margaret Mead for her emphasis on the 2
12345678901234567890123456789012123456789012345678901 2
12345678901234567890123456789012123456789012345678901
primacy of culture in the formation of an individual’s 2
12345678901234567890123456789012123456789012345678901 2
12345678901234567890123456789012123456789012345678901
personality. primal (adjective).
2345678901234567890123456789012123456789012345678901
2
2
1
12345678901234567890123456789012123456789012345678901 2
12345678901234567890123456789012123456789012345678901 2 2
210 12345678901234567890123456789012123456789012345678901
123456789012345678901234567890121234567890123456789012

www.petersons.com
Chapter 5: Verbal Review

123456789012345678901234567890121234567890123456789012
12345678901234567890123456789012123456789012345678901 2
12345678901234567890123456789012123456789012345678901 2
12345678901234567890123456789012123456789012345678901
pristine (adjective) Pure, undefiled. As climbers who have 2
12345678901234567890123456789012123456789012345678901 2
12345678901234567890123456789012123456789012345678901
scaled Mt. Everest can attest, the trails to the summit are 2
12345678901234567890123456789012123456789012345678901 2
12345678901234567890123456789012123456789012345678901
hardly in pristine condition and are actually strewn with 2
2345678901234567890123456789012123456789012345678901 2
1
12345678901234567890123456789012123456789012345678901
trash. 2
12345678901234567890123456789012123456789012345678901 2
12345678901234567890123456789012123456789012345678901 2
12345678901234567890123456789012123456789012345678901
probity (noun) Goodness, integrity.
2345678901234567890123456789012123456789012345678901
The vicious editorial 2
1 attacked the moral probity of the senatorial candidate, 2
12345678901234567890123456789012123456789012345678901 2
12345678901234567890123456789012123456789012345678901 2
12345678901234567890123456789012123456789012345678901
saying he had profited handsomely from his pet project, 2
12345678901234567890123456789012123456789012345678901
2345678901234567890123456789012123456789012345678901
2
2
1 the senior-citizen housing project.
12345678901234567890123456789012123456789012345678901 2
12345678901234567890123456789012123456789012345678901
procure (verb) To obtain by using particular care and effort. 2
12345678901234567890123456789012123456789012345678901 2
12345678901234567890123456789012123456789012345678901
Through partnerships with a large number of specialty 2
2345678901234567890123456789012123456789012345678901 2
1
12345678901234567890123456789012123456789012345678901
wholesalers, W.W. Grainger is able to procure a startling 2
12345678901234567890123456789012123456789012345678901 2
12345678901234567890123456789012123456789012345678901
array of products for its customers, from bear repellent 2
12345678901234567890123456789012123456789012345678901 2
12345678901234567890123456789012123456789012345678901
for Alaska pipeline workers to fork-lift trucks and toilet 2
12345678901234567890123456789012123456789012345678901 2

Take It to the Next Level


12345678901234567890123456789012123456789012345678901
paper. procurement (noun). 2
12345678901234567890123456789012123456789012345678901 2
12345678901234567890123456789012123456789012345678901 2
12345678901234567890123456789012123456789012345678901 22
2345678901234567890123456789012123456789012345678901
prodigality (noun) The condition of being wastefully extrava-
1
12345678901234567890123456789012123456789012345678901
gant. Richard was ashamed of the prodigality of his 2
12345678901234567890123456789012123456789012345678901
bride’s parents when he realized that the cost of the 2
12345678901234567890123456789012123456789012345678901 2
2345678901234567890123456789012123456789012345678901
wedding reception alone was more than his father 2
12345678901234567890123456789012123456789012345678901
12345678901234567890123456789012123456789012345678901 2
12345678901234567890123456789012123456789012345678901 2
12345678901234567890123456789012123456789012345678901 22
earned in one year. prodigal (adjective).
12345678901234567890123456789012123456789012345678901 2
12345678901234567890123456789012123456789012345678901
proliferate (verb) To increase or multiply. Over the past
12345678901234567890123456789012123456789012345678901 2
fifteen years, high-tech companies have proliferated in 2
12345678901234567890123456789012123456789012345678901
12345678901234567890123456789012123456789012345678901 22
1 northern California, Massachusetts, and other regions.
12345678901234567890123456789012123456789012345678901 2
12345678901234567890123456789012123456789012345678901
proliferation (noun). 2
12345678901234567890123456789012123456789012345678901 2
12345678901234567890123456789012123456789012345678901 2
12345678901234567890123456789012123456789012345678901
prolixity (noun) A diffuseness; a rambling and verbose 2
2345678901234567890123456789012123456789012345678901
quality. The prolixity of Sarah’s dissertation on Otto- 2
12345678901234567890123456789012123456789012345678901
12345678901234567890123456789012123456789012345678901 2
1 2
12345678901234567890123456789012123456789012345678901
man history defied even her adviser’s attempts to read it. 2
12345678901234567890123456789012123456789012345678901 2
12345678901234567890123456789012123456789012345678901 22
2345678901234567890123456789012123456789012345678901
prolix (adjective).
12345678901234567890123456789012123456789012345678901 2
1
12345678901234567890123456789012123456789012345678901
prophetic (adjective) Auspicious, predictive of what’s to 2
12345678901234567890123456789012123456789012345678901
come. We often look at every event leading up to a new 2
12345678901234567890123456789012123456789012345678901 2
12345678901234567890123456789012123456789012345678901
love affair as prophetic—the flat tire that caused us to be 2
12345678901234567890123456789012123456789012345678901 2
12345678901234567890123456789012123456789012345678901
late for work, the chance meeting in the elevator, the 2
12345678901234567890123456789012123456789012345678901 2
12345678901234567890123456789012123456789012345678901
horoscope that argued “a new beginning.” prophecy 2
12345678901234567890123456789012123456789012345678901 2
12345678901234567890123456789012123456789012345678901
(noun), prophesy (verb). 2
12345678901234567890123456789012123456789012345678901 2
12345678901234567890123456789012123456789012345678901 2
12345678901234567890123456789012123456789012345678901 22
2345678901234567890123456789012123456789012345678901
propagate (verb) To cause to grow; to foster. John Smithson’s
12345678901234567890123456789012123456789012345678901
will left his fortune for the founding of an institution to 2
12345678901234567890123456789012123456789012345678901
1 propagate knowledge, leaving open whether that meant 2
12345678901234567890123456789012123456789012345678901 2
2345678901234567890123456789012123456789012345678901
a university, a library, or a museum. propagation (noun). 2
12345678901234567890123456789012123456789012345678901
1 2
12345678901234567890123456789012123456789012345678901 2
12345678901234567890123456789012123456789012345678901 2
12345678901234567890123456789012123456789012345678901 2
12345678901234567890123456789012123456789012345678901 2
12345678901234567890123456789012123456789012345678901 2
12345678901234567890123456789012123456789012345678901 2
12345678901234567890123456789012123456789012345678901 2 211
123456789012345678901234567890121234567890123456789012
Part III: Section Review

123456789012345678901234567890121234567890123456789012
12345678901234567890123456789012123456789012345678901 2
12345678901234567890123456789012123456789012345678901 2
12345678901234567890123456789012123456789012345678901
propitiating (adjective) Conciliatory, mollifying or appeas- 2
12345678901234567890123456789012123456789012345678901 2
12345678901234567890123456789012123456789012345678901
ing. Management’s offer of a five-percent raise was 2
12345678901234567890123456789012123456789012345678901 2
12345678901234567890123456789012123456789012345678901
meant as a propitiating gesture, yet the striking workers 2
2345678901234567890123456789012123456789012345678901 2
1
12345678901234567890123456789012123456789012345678901
were unimpressed. propitiate (verb). 2
12345678901234567890123456789012123456789012345678901 2
12345678901234567890123456789012123456789012345678901 2
12345678901234567890123456789012123456789012345678901
propriety (noun) Appropriateness.
2345678901234567890123456789012123456789012345678901
Some people expressed 2
1 doubts about the propriety of Clinton’s discussing his 2
12345678901234567890123456789012123456789012345678901 2
12345678901234567890123456789012123456789012345678901 2
12345678901234567890123456789012123456789012345678901
underwear on MTV. 2
12345678901234567890123456789012123456789012345678901
2345678901234567890123456789012123456789012345678901
2
1 proximity (noun) Closeness, nearness. Neighborhood resi- 2
12345678901234567890123456789012123456789012345678901 2
12345678901234567890123456789012123456789012345678901
dents were angry over the proximity of the proposed 2
12345678901234567890123456789012123456789012345678901 2
12345678901234567890123456789012123456789012345678901
sewage plant to the local elementary school. proximate 2
2345678901234567890123456789012123456789012345678901 2
1
12345678901234567890123456789012123456789012345678901
(adjective). 2
12345678901234567890123456789012123456789012345678901 2
12345678901234567890123456789012123456789012345678901 2
12345678901234567890123456789012123456789012345678901
pundit (noun) Someone who offers opinions in an authori- 2
2345678901234567890123456789012123456789012345678901
12345678901234567890123456789012123456789012345678901
tative style. The Sunday morning talk shows are filled 2
2
12345678901234567890123456789012123456789012345678901
with pundits, each with his or her own theory about this 2
12345678901234567890123456789012123456789012345678901
12345678901234567890123456789012123456789012345678901 2
12345678901234567890123456789012123456789012345678901 2
12345678901234567890123456789012123456789012345678901 2
week’s political news.
2
1
12345678901234567890123456789012123456789012345678901 2
12345678901234567890123456789012123456789012345678901 2
12345678901234567890123456789012123456789012345678901
Latin pungere = to jab, to prick. Also found in English pugilist, 2
2345678901234567890123456789012123456789012345678901
12345678901234567890123456789012123456789012345678901
Word Origin 2
12345678901234567890123456789012123456789012345678901
punctuate, and puncture. 2
12345678901234567890123456789012123456789012345678901 2
pungency (noun) Marked by having a sharp, biting quality. 2
12345678901234567890123456789012123456789012345678901
2
12345678901234567890123456789012123456789012345678901
Unfortunately, the pungency of the fresh cilantro over- 2
12345678901234567890123456789012123456789012345678901
12345678901234567890123456789012123456789012345678901 2
whelmed the delicate flavor of the poached flounder. 2
12345678901234567890123456789012123456789012345678901
12345678901234567890123456789012123456789012345678901 2 2
1 pungent (adjective).
12345678901234567890123456789012123456789012345678901 2
12345678901234567890123456789012123456789012345678901 2
12345678901234567890123456789012123456789012345678901
purify (verb) To make pure, clean, or perfect. The new 2
12345678901234567890123456789012123456789012345678901 2
12345678901234567890123456789012123456789012345678901
water-treatment plant is supposed to purify the drinking 2
2345678901234567890123456789012123456789012345678901
water provided to everyone in the nearby towns. 2
12345678901234567890123456789012123456789012345678901
12345678901234567890123456789012123456789012345678901 2
1 2
12345678901234567890123456789012123456789012345678901
purification (noun). 2
12345678901234567890123456789012123456789012345678901 2
2345678901234567890123456789012123456789012345678901
quiescent (adjective) In a state of rest or inactivity; latent. 2
12345678901234567890123456789012123456789012345678901
12345678901234567890123456789012123456789012345678901 2
1 2
12345678901234567890123456789012123456789012345678901 2
Polly’s ulcer has been quiescent ever since her mother-in-
12345678901234567890123456789012123456789012345678901
law moved out of the condo, which was well over a year 2
12345678901234567890123456789012123456789012345678901 2
12345678901234567890123456789012123456789012345678901
ago. quiescence (noun). 2
12345678901234567890123456789012123456789012345678901 2
12345678901234567890123456789012123456789012345678901 2
12345678901234567890123456789012123456789012345678901
quixotic (adjective) Foolishly romantic, idealistic to an 2
2345678901234567890123456789012123456789012345678901
12345678901234567890123456789012123456789012345678901
impractical degree. In the novel Shoeless Joe, Ray 2
2
12345678901234567890123456789012123456789012345678901 2
12345678901234567890123456789012123456789012345678901
Kinsella carries out a quixotic plan to build a baseball 2
12345678901234567890123456789012123456789012345678901 2
1
12345678901234567890123456789012123456789012345678901
field in the hopes that past baseball greats will come to 2
12345678901234567890123456789012123456789012345678901 2
12345678901234567890123456789012123456789012345678901
play there. 2
12345678901234567890123456789012123456789012345678901 2
12345678901234567890123456789012123456789012345678901 2
12345678901234567890123456789012123456789012345678901 2
2345678901234567890123456789012123456789012345678901
2
1
12345678901234567890123456789012123456789012345678901 2
12345678901234567890123456789012123456789012345678901 2
12345678901234567890123456789012123456789012345678901 2
12345678901234567890123456789012123456789012345678901 2
12345678901234567890123456789012123456789012345678901 2
12345678901234567890123456789012123456789012345678901 2 2
212 12345678901234567890123456789012123456789012345678901
123456789012345678901234567890121234567890123456789012

www.petersons.com
Chapter 5: Verbal Review

123456789012345678901234567890121234567890123456789012
12345678901234567890123456789012123456789012345678901 2
12345678901234567890123456789012123456789012345678901 2
12345678901234567890123456789012123456789012345678901
quotidian (adjective) Occurring every day; commonplace 2
12345678901234567890123456789012123456789012345678901 2
12345678901234567890123456789012123456789012345678901
and ordinary. Most of the time, we long to escape from 2
12345678901234567890123456789012123456789012345678901 2
12345678901234567890123456789012123456789012345678901
quotidian concerns, but in the midst of a crisis we want 2
2345678901234567890123456789012123456789012345678901 2
1
12345678901234567890123456789012123456789012345678901
nothing more than to be plagued by such simple 2
12345678901234567890123456789012123456789012345678901 2
12345678901234567890123456789012123456789012345678901
problems as a leaky faucet or a whining child. 2
12345678901234567890123456789012123456789012345678901
2345678901234567890123456789012123456789012345678901
2
1 raconteur (noun) An excellent storyteller. A member of the 2
12345678901234567890123456789012123456789012345678901 2
12345678901234567890123456789012123456789012345678901 2
12345678901234567890123456789012123456789012345678901
Algonquin Round Table, Robert Benchley was a natural 2
12345678901234567890123456789012123456789012345678901
2345678901234567890123456789012123456789012345678901
2
2
1 raconteur with a seemingly endless ability to turn daily
12345678901234567890123456789012123456789012345678901 2
12345678901234567890123456789012123456789012345678901
life and its irritations into entertaining commentary. 2
12345678901234567890123456789012123456789012345678901 2
12345678901234567890123456789012123456789012345678901
rancorous (adjective) Marked by deeply embedded bitter- 2
2345678901234567890123456789012123456789012345678901 2
1
12345678901234567890123456789012123456789012345678901
ness or animosity. While Ralph and Kishu have been 2
12345678901234567890123456789012123456789012345678901 2
12345678901234567890123456789012123456789012345678901
separated for three years, their relationship is so 2
12345678901234567890123456789012123456789012345678901 2
12345678901234567890123456789012123456789012345678901
rancorous that they had to hire a professional mediator 2
12345678901234567890123456789012123456789012345678901 2

Take It to the Next Level


12345678901234567890123456789012123456789012345678901
just to discuss divorce arrangements. rancor (noun). 2
12345678901234567890123456789012123456789012345678901 2
12345678901234567890123456789012123456789012345678901 2
12345678901234567890123456789012123456789012345678901 22
2345678901234567890123456789012123456789012345678901
rapacious (adjective) Excessively grasping or greedy. Some
1
12345678901234567890123456789012123456789012345678901
see global currency speculators like George Soros as 2
12345678901234567890123456789012123456789012345678901
rapacious parasites who destroy economies and then 2
12345678901234567890123456789012123456789012345678901 2
2345678901234567890123456789012123456789012345678901
12345678901234567890123456789012123456789012345678901 2
12345678901234567890123456789012123456789012345678901
line their pockets with the profits. rapacity (noun). 2
12345678901234567890123456789012123456789012345678901 22
12345678901234567890123456789012123456789012345678901
rarefied (adjective) Of interest or relating to a small, refined 2
12345678901234567890123456789012123456789012345678901
circle; less dense, thinner. Those whose names dot the 2
12345678901234567890123456789012123456789012345678901
12345678901234567890123456789012123456789012345678901 2
society pages live in a rarefied world where it’s entirely 2
12345678901234567890123456789012123456789012345678901
12345678901234567890123456789012123456789012345678901 22
1 normal to dine on caviar for breakfast or order a $2,000
12345678901234567890123456789012123456789012345678901 2
12345678901234567890123456789012123456789012345678901
bottle of wine at Le Cirque. When she reached the 2
12345678901234567890123456789012123456789012345678901 2
12345678901234567890123456789012123456789012345678901
summit of Mt. McKinley, Deborah could hardly breathe 2
12345678901234567890123456789012123456789012345678901 2
12345678901234567890123456789012123456789012345678901
in the rarefied air. 2
12345678901234567890123456789012123456789012345678901 2
12345678901234567890123456789012123456789012345678901 2
12345678901234567890123456789012123456789012345678901
raucous (adjective) Boisterous, unruly and wild. Sounds of 2
12345678901234567890123456789012123456789012345678901 2
12345678901234567890123456789012123456789012345678901 22
2345678901234567890123456789012123456789012345678901
shouts and raucous laughter drifted out of the hotel
12345678901234567890123456789012123456789012345678901 2
1 room where Felipe’s bachelor party was being held.
12345678901234567890123456789012123456789012345678901 2
12345678901234567890123456789012123456789012345678901
reactionary (adjective) Ultra conservative. Every day, over 2
12345678901234567890123456789012123456789012345678901 2
12345678901234567890123456789012123456789012345678901
twenty million listeners tune in to hear Rush Limbaugh 2
12345678901234567890123456789012123456789012345678901 2
12345678901234567890123456789012123456789012345678901
spew his reactionary opinions about “feminazis” and 2
12345678901234567890123456789012123456789012345678901 2
12345678901234567890123456789012123456789012345678901
environmental “fanatics.” reactionary (noun). 2
12345678901234567890123456789012123456789012345678901 2
12345678901234567890123456789012123456789012345678901 2
12345678901234567890123456789012123456789012345678901
recede (verb) To draw back, to ebb, to abate. Once his 2
12345678901234567890123456789012123456789012345678901 2
12345678901234567890123456789012123456789012345678901
hairline began to recede, Hap took to wearing bizarre 2
12345678901234567890123456789012123456789012345678901 2
12345678901234567890123456789012123456789012345678901
accessories, like velvet ascots, to divert attention from it. 2
12345678901234567890123456789012123456789012345678901 2
12345678901234567890123456789012123456789012345678901
recession (noun). 2
12345678901234567890123456789012123456789012345678901 22
2345678901234567890123456789012123456789012345678901
1
12345678901234567890123456789012123456789012345678901
reclusive (adjective) Withdrawn from society. During the 2
12345678901234567890123456789012123456789012345678901
last years of her life, Garbo led a reclusive existence, 2
12345678901234567890123456789012123456789012345678901
2345678901234567890123456789012123456789012345678901
2
2
1
12345678901234567890123456789012123456789012345678901
rarely appearing in public. recluse (noun). 2
12345678901234567890123456789012123456789012345678901 22
12345678901234567890123456789012123456789012345678901 213
123456789012345678901234567890121234567890123456789012
Part III: Section Review

123456789012345678901234567890121234567890123456789012
12345678901234567890123456789012123456789012345678901 2
12345678901234567890123456789012123456789012345678901 2
12345678901234567890123456789012123456789012345678901
reconcile (verb) To make consistent or harmonious. 2
12345678901234567890123456789012123456789012345678901 2
12345678901234567890123456789012123456789012345678901
Roosevelt’s greatness as a leader can be seen in his 2
12345678901234567890123456789012123456789012345678901 2
12345678901234567890123456789012123456789012345678901
ability to reconcile the differing demands and values of 2
2345678901234567890123456789012123456789012345678901 2
1
12345678901234567890123456789012123456789012345678901
the varied groups that supported him. reconciliation 2
12345678901234567890123456789012123456789012345678901 2
12345678901234567890123456789012123456789012345678901
(noun). 2
12345678901234567890123456789012123456789012345678901
2345678901234567890123456789012123456789012345678901
2
1 recompense (noun) Compensation for a service rendered or 2
12345678901234567890123456789012123456789012345678901 2
12345678901234567890123456789012123456789012345678901 2
12345678901234567890123456789012123456789012345678901
to pay for damages. The 5% of the estate that Phil 2
12345678901234567890123456789012123456789012345678901
2345678901234567890123456789012123456789012345678901
2
2
1 received as executor of his Aunt Ida’s will is small
12345678901234567890123456789012123456789012345678901 2
12345678901234567890123456789012123456789012345678901
recompense for the headaches he endured in settling her 2
12345678901234567890123456789012123456789012345678901
affairs. recompense (verb). 2
12345678901234567890123456789012123456789012345678901
2345678901234567890123456789012123456789012345678901
2
2
1
12345678901234567890123456789012123456789012345678901
recondite (adjective) Profound, deep, abstruse. Professor 2
12345678901234567890123456789012123456789012345678901 2
12345678901234567890123456789012123456789012345678901
Miyaki’s recondite knowledge of seventeenth-century 2
12345678901234567890123456789012123456789012345678901 2
12345678901234567890123456789012123456789012345678901
Flemish painters made him a prized—if barely under- 2
12345678901234567890123456789012123456789012345678901 2
12345678901234567890123456789012123456789012345678901
stood—member of the art history department. 2
12345678901234567890123456789012123456789012345678901 2
12345678901234567890123456789012123456789012345678901 2
12345678901234567890123456789012123456789012345678901 2
2345678901234567890123456789012123456789012345678901
redemptive (adjective) Liberating and reforming. While she
1 doesn’t attend formal church services, Carrie is a firm 2
12345678901234567890123456789012123456789012345678901 2
12345678901234567890123456789012123456789012345678901 2
12345678901234567890123456789012123456789012345678901
believer in the redemptive power of prayer. redeem 2
2345678901234567890123456789012123456789012345678901
12345678901234567890123456789012123456789012345678901 2
12345678901234567890123456789012123456789012345678901
(verb), redemption (noun). 2
12345678901234567890123456789012123456789012345678901 2
12345678901234567890123456789012123456789012345678901 2
12345678901234567890123456789012123456789012345678901 2
12345678901234567890123456789012123456789012345678901
Latin frangere = to break. Also found in English fraction, fractions, 2
12345678901234567890123456789012123456789012345678901
Word Origin fracture, frangible, infraction, and refract. 2
12345678901234567890123456789012123456789012345678901 2
12345678901234567890123456789012123456789012345678901 2
1 refractory (adjective) Stubbornly resisting control or author- 2
12345678901234567890123456789012123456789012345678901 2
12345678901234567890123456789012123456789012345678901
ity. Like a refractory child, Jill stomped out of the car, 2
12345678901234567890123456789012123456789012345678901 2
12345678901234567890123456789012123456789012345678901
slammed the door, and said she would walk home, even 2
12345678901234567890123456789012123456789012345678901 2
12345678901234567890123456789012123456789012345678901
though her house was 10 miles away. 2
12345678901234567890123456789012123456789012345678901 2
12345678901234567890123456789012123456789012345678901 2
12345678901234567890123456789012123456789012345678901
relevance (noun) Connection to the matter at hand; perti- 2
12345678901234567890123456789012123456789012345678901 2
12345678901234567890123456789012123456789012345678901 2
2345678901234567890123456789012123456789012345678901
nence. Testimony in a criminal trial may only be
1 admitted to the extent that it has clear relevance to the 2
12345678901234567890123456789012123456789012345678901 2
12345678901234567890123456789012123456789012345678901 2
12345678901234567890123456789012123456789012345678901 2
question of guilt or innocence. relevant (adjective).
12345678901234567890123456789012123456789012345678901 2
2345678901234567890123456789012123456789012345678901
reparation (noun) The act of making amends; payment of 2
12345678901234567890123456789012123456789012345678901 2
12345678901234567890123456789012123456789012345678901
damages by a defeated nation to the victors. The Treaty 2
12345678901234567890123456789012123456789012345678901
1 2
12345678901234567890123456789012123456789012345678901
of Versailles, signed in 1919, formally asserted Germa- 2
12345678901234567890123456789012123456789012345678901 2
12345678901234567890123456789012123456789012345678901
ny’s war guilt and ordered it to pay reparations to the 2
12345678901234567890123456789012123456789012345678901 2
12345678901234567890123456789012123456789012345678901
allies. 2
12345678901234567890123456789012123456789012345678901 2
2345678901234567890123456789012123456789012345678901
2
12345678901234567890123456789012123456789012345678901
reproof (noun) A reprimand, a reproach or castigation. Joe 2
12345678901234567890123456789012123456789012345678901
1 thought being grounded for one month was a harsh 2
12345678901234567890123456789012123456789012345678901 2
2345678901234567890123456789012123456789012345678901
reproof for coming home late only once. reprove (verb). 2
12345678901234567890123456789012123456789012345678901
1 2
12345678901234567890123456789012123456789012345678901 2
12345678901234567890123456789012123456789012345678901 2
12345678901234567890123456789012123456789012345678901 2
12345678901234567890123456789012123456789012345678901 2
12345678901234567890123456789012123456789012345678901 2
12345678901234567890123456789012123456789012345678901 2 2
214 12345678901234567890123456789012123456789012345678901
123456789012345678901234567890121234567890123456789012

www.petersons.com
Chapter 5: Verbal Review

123456789012345678901234567890121234567890123456789012
12345678901234567890123456789012123456789012345678901 2
12345678901234567890123456789012123456789012345678901 2
12345678901234567890123456789012123456789012345678901
repugnant (adjective) Causing dislike or disgust. After the 2
12345678901234567890123456789012123456789012345678901 2
12345678901234567890123456789012123456789012345678901
news broke about Mad Cow Disease, much of the 2
12345678901234567890123456789012123456789012345678901 2
12345678901234567890123456789012123456789012345678901
beef-loving British public began to find the thought of a 2
2345678901234567890123456789012123456789012345678901 2
1
12345678901234567890123456789012123456789012345678901
Sunday roast repugnant. 2
12345678901234567890123456789012123456789012345678901 2
12345678901234567890123456789012123456789012345678901 2
12345678901234567890123456789012123456789012345678901
requiem (noun) A musical composition
2345678901234567890123456789012123456789012345678901
or poem written to 2
1 honor the dead. Many financial analysts think that the 2
12345678901234567890123456789012123456789012345678901 2
12345678901234567890123456789012123456789012345678901 2
12345678901234567890123456789012123456789012345678901
ailing typewriter company should simply say a requiem 2
12345678901234567890123456789012123456789012345678901
2345678901234567890123456789012123456789012345678901
2
2
1 for itself and shut down; however, the CEO has other plans.
12345678901234567890123456789012123456789012345678901 2
12345678901234567890123456789012123456789012345678901
repudiate (verb) To reject, to renounce. After it became 2
12345678901234567890123456789012123456789012345678901 2
12345678901234567890123456789012123456789012345678901
known that Duke had been a leader of the Ku Klux 2
2345678901234567890123456789012123456789012345678901 2
1
12345678901234567890123456789012123456789012345678901
Klan, most Republican leaders repudiated him. repudia- 2
12345678901234567890123456789012123456789012345678901 2
12345678901234567890123456789012123456789012345678901
tion (noun). 2
12345678901234567890123456789012123456789012345678901 2
2345678901234567890123456789012123456789012345678901
12345678901234567890123456789012123456789012345678901
resilient (adjective) Able to recover from difficulty. A pro 2
2
12345678901234567890123456789012123456789012345678901

Take It to the Next Level


athlete must be mentally resilient, able to lose a game 2
12345678901234567890123456789012123456789012345678901
12345678901234567890123456789012123456789012345678901 2
12345678901234567890123456789012123456789012345678901 2
12345678901234567890123456789012123456789012345678901 22
one day and come back the next with renewed enthusi-
1
12345678901234567890123456789012123456789012345678901
asm and confidence. resilience (noun). 2
12345678901234567890123456789012123456789012345678901 2
12345678901234567890123456789012123456789012345678901 2
12345678901234567890123456789012123456789012345678901 22
2345678901234567890123456789012123456789012345678901
resonant (adjective) Full of special import or meaning. I
12345678901234567890123456789012123456789012345678901
found the speaker’s words particularly resonant because 2
12345678901234567890123456789012123456789012345678901
I, too, had served in Vietnam and felt the same mixture 2
12345678901234567890123456789012123456789012345678901
2
12345678901234567890123456789012123456789012345678901
12345678901234567890123456789012123456789012345678901
of shame and pride. resonance (noun). 2
12345678901234567890123456789012123456789012345678901 2
12345678901234567890123456789012123456789012345678901
resplendent (adjective) Glowing, shining. In late December, 2
2
12345678901234567890123456789012123456789012345678901 2
1
12345678901234567890123456789012123456789012345678901
midtown New York is resplendent with holiday lights 2
12345678901234567890123456789012123456789012345678901 2
12345678901234567890123456789012123456789012345678901
and decorations. resplendence (noun). 2
12345678901234567890123456789012123456789012345678901 2
12345678901234567890123456789012123456789012345678901
rite (noun) Ceremony. From October to May, the Patwin 2
12345678901234567890123456789012123456789012345678901 2
12345678901234567890123456789012123456789012345678901 2
12345678901234567890123456789012123456789012345678901 2
Indians of California’s Sacramento Valley held a series
12345678901234567890123456789012123456789012345678901
of rites and dances designed to bring the tribe health and 2
12345678901234567890123456789012123456789012345678901 2
2345678901234567890123456789012123456789012345678901
12345678901234567890123456789012123456789012345678901 2
12345678901234567890123456789012123456789012345678901
prosperity. 2
1 2
12345678901234567890123456789012123456789012345678901
rogue (noun) A mischievously dishonest person; a scamp. In 2
12345678901234567890123456789012123456789012345678901 2
12345678901234567890123456789012123456789012345678901
Jane Austen’s Pride and Prejudice, Wickham, a charm- 2
12345678901234567890123456789012123456789012345678901 2
12345678901234567890123456789012123456789012345678901
ing rogue, seduces Darcy’s young sister Georgiana and 2
12345678901234567890123456789012123456789012345678901 2
12345678901234567890123456789012123456789012345678901
later does the same thing with Kitty Bennett. 2
12345678901234567890123456789012123456789012345678901 22
2345678901234567890123456789012123456789012345678901
12345678901234567890123456789012123456789012345678901
ruffian (noun) A brute, roughneck or bully. In Dickens’ 2
12345678901234567890123456789012123456789012345678901
1 Oliver Twist, Fagin instructs his gang of orphaned 2
12345678901234567890123456789012123456789012345678901 2
2345678901234567890123456789012123456789012345678901
12345678901234567890123456789012123456789012345678901 2
12345678901234567890123456789012123456789012345678901
ruffians on the arts of picking pockets and shoplifting. 2
12345678901234567890123456789012123456789012345678901 22
12345678901234567890123456789012123456789012345678901
rumination (noun) The act of engaging in contemplation. 2
1
12345678901234567890123456789012123456789012345678901
Marcel Proust’s semi-autobiographical novel cycle, Re- 2
12345678901234567890123456789012123456789012345678901 2
12345678901234567890123456789012123456789012345678901 2
12345678901234567890123456789012123456789012345678901 2
membrance of Things Past, is less a narrative than an
12345678901234567890123456789012123456789012345678901
extended rumination on the nature of memory. ruminate 2
12345678901234567890123456789012123456789012345678901 2
12345678901234567890123456789012123456789012345678901
(verb). 2
12345678901234567890123456789012123456789012345678901 22
12345678901234567890123456789012123456789012345678901 215
123456789012345678901234567890121234567890123456789012
Part III: Section Review

123456789012345678901234567890121234567890123456789012
12345678901234567890123456789012123456789012345678901 2
12345678901234567890123456789012123456789012345678901 2
12345678901234567890123456789012123456789012345678901
sage (noun) A person of great wisdom, a sagacious philoso- 2
12345678901234567890123456789012123456789012345678901 2
12345678901234567890123456789012123456789012345678901
pher. It was the Chinese sage Confucius who first taught 2
12345678901234567890123456789012123456789012345678901 2
12345678901234567890123456789012123456789012345678901
what is now known the world over as “The Golden 2
2345678901234567890123456789012123456789012345678901 2
1
12345678901234567890123456789012123456789012345678901
Rule.” sagacious (adjective), sagacity (noun). 2
12345678901234567890123456789012123456789012345678901 2
12345678901234567890123456789012123456789012345678901 2
12345678901234567890123456789012123456789012345678901
2345678901234567890123456789012123456789012345678901
2
2
1
12345678901234567890123456789012123456789012345678901
Word
Latin salus 5 health. Also found in English salubrious, salutation,
Origin and salute. 2
12345678901234567890123456789012123456789012345678901 2
12345678901234567890123456789012123456789012345678901 2
12345678901234567890123456789012123456789012345678901
2345678901234567890123456789012123456789012345678901
2
2
1 salutary (adjective) Restorative, healthful. I find a short dip
12345678901234567890123456789012123456789012345678901 2
12345678901234567890123456789012123456789012345678901
in an icy stream to be extremely salutary, although the 2
12345678901234567890123456789012123456789012345678901
health benefits of my bracing swims are, as yet, unclear. 2
12345678901234567890123456789012123456789012345678901
2345678901234567890123456789012123456789012345678901
2
2
1
12345678901234567890123456789012123456789012345678901
sanction (verb) Support or authorize. Even after a bomb 2
12345678901234567890123456789012123456789012345678901 2
12345678901234567890123456789012123456789012345678901
exploded on the front porch of his home, the Reverend 2
12345678901234567890123456789012123456789012345678901 2
12345678901234567890123456789012123456789012345678901
Martin Luther King Jr. refused to sanction any violent 2
12345678901234567890123456789012123456789012345678901 2
12345678901234567890123456789012123456789012345678901
response and urged his angry followers to love their 2
12345678901234567890123456789012123456789012345678901 2
12345678901234567890123456789012123456789012345678901
enemies. sanctify (verb), sanction (noun). 2
12345678901234567890123456789012123456789012345678901 2
2345678901234567890123456789012123456789012345678901
1 sap (verb) To exhaust, to deplete. The exhaustive twelve-city 2
12345678901234567890123456789012123456789012345678901 2
12345678901234567890123456789012123456789012345678901 2
12345678901234567890123456789012123456789012345678901
reading tour so sapped the novelist’s strength that she 2
2345678901234567890123456789012123456789012345678901
told her publicist that she hoped her next book would be 2
12345678901234567890123456789012123456789012345678901
12345678901234567890123456789012123456789012345678901 2
12345678901234567890123456789012123456789012345678901 2
12345678901234567890123456789012123456789012345678901 2
a flop! While Uganda is making enormous economic
strides under President Yoweri Musevini, rebel fighting 2
12345678901234567890123456789012123456789012345678901
12345678901234567890123456789012123456789012345678901 2
12345678901234567890123456789012123456789012345678901 2
12345678901234567890123456789012123456789012345678901 2
has sapped much of the country’s resources.
12345678901234567890123456789012123456789012345678901 2
1 satiate (verb) To fulfill to or beyond capacity. Judging by the 2
12345678901234567890123456789012123456789012345678901 2
12345678901234567890123456789012123456789012345678901
current crop of films featuring serial killers, rape, ritual 2
12345678901234567890123456789012123456789012345678901 2
12345678901234567890123456789012123456789012345678901
murder, gunslinging, and plain old-fashioned slugfests, 2
12345678901234567890123456789012123456789012345678901 2
12345678901234567890123456789012123456789012345678901
the public appetite for violence has not yet been satiated. 2
12345678901234567890123456789012123456789012345678901 2
12345678901234567890123456789012123456789012345678901
satiation (noun), satiety (noun). 2
12345678901234567890123456789012123456789012345678901 2
12345678901234567890123456789012123456789012345678901 2
12345678901234567890123456789012123456789012345678901 2
2345678901234567890123456789012123456789012345678901
saturate (verb) To drench or suffuse with liquid or anything
1 that permeates or invades. The hostess’s furious dabbing 2
12345678901234567890123456789012123456789012345678901 2
12345678901234567890123456789012123456789012345678901 2
12345678901234567890123456789012123456789012345678901 2
at the tablecloth was in vain, since the spilt wine had
2345678901234567890123456789012123456789012345678901
already saturated the damask cloth. saturation (noun), 2
12345678901234567890123456789012123456789012345678901
12345678901234567890123456789012123456789012345678901 2
12345678901234567890123456789012123456789012345678901 2
12345678901234567890123456789012123456789012345678901 2
saturated (adjective).
2
1
12345678901234567890123456789012123456789012345678901
scrutinize (verb) To study closely. The lawyer scrutinized 2
12345678901234567890123456789012123456789012345678901 2
12345678901234567890123456789012123456789012345678901
the contract, searching for any detail that could pose a 2
12345678901234567890123456789012123456789012345678901 2
12345678901234567890123456789012123456789012345678901
risk for her client. scrutiny (noun). 2
12345678901234567890123456789012123456789012345678901 2
2345678901234567890123456789012123456789012345678901
2
12345678901234567890123456789012123456789012345678901
scurvy (adjective) Shabby, low. I couldn’t believe that 2
12345678901234567890123456789012123456789012345678901
1 Farouk was so scurvy as to open up my computer files 2
12345678901234567890123456789012123456789012345678901 2
2345678901234567890123456789012123456789012345678901
12345678901234567890123456789012123456789012345678901 2
1 and read my e-mail. 2
12345678901234567890123456789012123456789012345678901 2
12345678901234567890123456789012123456789012345678901 2
12345678901234567890123456789012123456789012345678901 2
12345678901234567890123456789012123456789012345678901 2
12345678901234567890123456789012123456789012345678901 2
12345678901234567890123456789012123456789012345678901 2 2
216 12345678901234567890123456789012123456789012345678901
123456789012345678901234567890121234567890123456789012

www.petersons.com
Chapter 5: Verbal Review

123456789012345678901234567890121234567890123456789012
12345678901234567890123456789012123456789012345678901 2
12345678901234567890123456789012123456789012345678901 2
12345678901234567890123456789012123456789012345678901 2
12345678901234567890123456789012123456789012345678901
Latin sequi 5 to follow. Also found in English consequence, 2
12345678901234567890123456789012123456789012345678901
Word Origin 2
12345678901234567890123456789012123456789012345678901 2
sequel, and subsequent.
12345678901234567890123456789012123456789012345678901 2
12345678901234567890123456789012123456789012345678901
sequential (adjective) Arranged in an order or series. The 2
12345678901234567890123456789012123456789012345678901 2
12345678901234567890123456789012123456789012345678901
courses required for the chemistry major are sequential; 2
12345678901234567890123456789012123456789012345678901 2
12345678901234567890123456789012123456789012345678901
you must take them in the prescribed order since each 2
2345678901234567890123456789012123456789012345678901 2
1
12345678901234567890123456789012123456789012345678901
course builds on the previous ones. sequence (noun). 2
12345678901234567890123456789012123456789012345678901 2
12345678901234567890123456789012123456789012345678901 2
12345678901234567890123456789012123456789012345678901
sedulous (adjective) Diligent, industrious.
2345678901234567890123456789012123456789012345678901
Those who are 2
1 most sedulous about studying this vocabulary list are 2
12345678901234567890123456789012123456789012345678901 2
12345678901234567890123456789012123456789012345678901 2
12345678901234567890123456789012123456789012345678901
likely to breeze through the antonyms sections of their 2
12345678901234567890123456789012123456789012345678901
2345678901234567890123456789012123456789012345678901
2
2
1 GRE exam.
12345678901234567890123456789012123456789012345678901 2
12345678901234567890123456789012123456789012345678901
sidereal (adjective) Relating to the stars or the constella- 2
12345678901234567890123456789012123456789012345678901 2
12345678901234567890123456789012123456789012345678901
tions. Jacqueline was interested in matters sidereal and 2
12345678901234567890123456789012123456789012345678901 22
2345678901234567890123456789012123456789012345678901
12345678901234567890123456789012123456789012345678901

Take It to the Next Level


was always begging my father to take the dusty old
12345678901234567890123456789012123456789012345678901 2
12345678901234567890123456789012123456789012345678901
telescope out of our garage. 2
12345678901234567890123456789012123456789012345678901 22
12345678901234567890123456789012123456789012345678901
signatory (noun) Someone who signs an official document 2
1
12345678901234567890123456789012123456789012345678901
or petition along with others. Alex urged me to join the 2
12345678901234567890123456789012123456789012345678901 2
12345678901234567890123456789012123456789012345678901 2
12345678901234567890123456789012123456789012345678901 22
2345678901234567890123456789012123456789012345678901
other signatories and add my name to the petition
12345678901234567890123456789012123456789012345678901
against toxic sludge in organic foods, but I simply did 2
12345678901234567890123456789012123456789012345678901
not care enough about the issue. The signatories of the 2
12345678901234567890123456789012123456789012345678901
12345678901234567890123456789012123456789012345678901 2
Declaration of Independence included John Adams, 2
12345678901234567890123456789012123456789012345678901
12345678901234567890123456789012123456789012345678901 2
12345678901234567890123456789012123456789012345678901 2
12345678901234567890123456789012123456789012345678901 22
Benjamin Franklin, John Hancock, and Thomas
1
12345678901234567890123456789012123456789012345678901
Jefferson. 2
12345678901234567890123456789012123456789012345678901 2
12345678901234567890123456789012123456789012345678901
sinuous (noun) Winding, circuitous, serpentine. Frank 2
12345678901234567890123456789012123456789012345678901 2
12345678901234567890123456789012123456789012345678901
Gehry’s sinuous design for the Guggenheim Museum in 2
12345678901234567890123456789012123456789012345678901 2
12345678901234567890123456789012123456789012345678901
Bilbao, Spain, has led people to hail the museum as the 2
12345678901234567890123456789012123456789012345678901 2
12345678901234567890123456789012123456789012345678901
first great building of the twenty-first century. sinuosity 2
12345678901234567890123456789012123456789012345678901 2
12345678901234567890123456789012123456789012345678901
(noun). 2
12345678901234567890123456789012123456789012345678901 2
12345678901234567890123456789012123456789012345678901 2
12345678901234567890123456789012123456789012345678901 2
specious (adjective) Deceptively plausible or attractive. The
12345678901234567890123456789012123456789012345678901
infomercial for “Fat-Away” offered mainly specious 2
12345678901234567890123456789012123456789012345678901 2
12345678901234567890123456789012123456789012345678901 2
12345678901234567890123456789012123456789012345678901
arguments for a product that is, essentially, a heavy-duty 2
12345678901234567890123456789012123456789012345678901 2
12345678901234567890123456789012123456789012345678901 2
girdle.
12345678901234567890123456789012123456789012345678901 22
2345678901234567890123456789012123456789012345678901
12345678901234567890123456789012123456789012345678901
spontaneous (adjective) Happening without plan or outside
2
12345678901234567890123456789012123456789012345678901
cause. When the news of Kennedy’s assassination hit the 2
12345678901234567890123456789012123456789012345678901
1 2
12345678901234567890123456789012123456789012345678901
airwaves, people everywhere gathered in a spontaneous 2
12345678901234567890123456789012123456789012345678901 2
12345678901234567890123456789012123456789012345678901
effort to express their shock and grief. spontaneity 2
12345678901234567890123456789012123456789012345678901 2
12345678901234567890123456789012123456789012345678901
(noun). 2
12345678901234567890123456789012123456789012345678901 22
2345678901234567890123456789012123456789012345678901
1
12345678901234567890123456789012123456789012345678901 2
12345678901234567890123456789012123456789012345678901 2
12345678901234567890123456789012123456789012345678901 2
12345678901234567890123456789012123456789012345678901 2
12345678901234567890123456789012123456789012345678901 2
12345678901234567890123456789012123456789012345678901 2
12345678901234567890123456789012123456789012345678901 2 217
123456789012345678901234567890121234567890123456789012
Part III: Section Review

123456789012345678901234567890121234567890123456789012
12345678901234567890123456789012123456789012345678901 2
12345678901234567890123456789012123456789012345678901 2
12345678901234567890123456789012123456789012345678901
spurious (adjective) False, fake. The so-called Piltdown 2
12345678901234567890123456789012123456789012345678901 2
12345678901234567890123456789012123456789012345678901
Man, supposed to be the fossil of a primitive human, 2
12345678901234567890123456789012123456789012345678901 2
12345678901234567890123456789012123456789012345678901
turned out to be spurious, though who created the hoax 2
2345678901234567890123456789012123456789012345678901 2
1
12345678901234567890123456789012123456789012345678901
is still uncertain. 2
12345678901234567890123456789012123456789012345678901 2
12345678901234567890123456789012123456789012345678901 2
12345678901234567890123456789012123456789012345678901
splice (verb) To unite by interweaving
2345678901234567890123456789012123456789012345678901
separate strands or 2
1 parts. Amateur filmmaker Duddy Kravitz shocked and 2
12345678901234567890123456789012123456789012345678901 2
12345678901234567890123456789012123456789012345678901 2
12345678901234567890123456789012123456789012345678901
angered his clients by splicing footage of tribal rituals 2
12345678901234567890123456789012123456789012345678901
2345678901234567890123456789012123456789012345678901
2
2
1 into his films of their weddings and bar mitzvahs.
12345678901234567890123456789012123456789012345678901 2
12345678901234567890123456789012123456789012345678901
squander (verb) To use up carelessly, to waste. Those who 2
12345678901234567890123456789012123456789012345678901 2
12345678901234567890123456789012123456789012345678901
had made donations to the charity were outraged to 2
2345678901234567890123456789012123456789012345678901 2
1
12345678901234567890123456789012123456789012345678901
learn that its director had squandered millions on fancy 2
12345678901234567890123456789012123456789012345678901 2
12345678901234567890123456789012123456789012345678901
dinners, first-class travel, and an expensive apartment 2
12345678901234567890123456789012123456789012345678901 2
12345678901234567890123456789012123456789012345678901
for entertaining. 2
12345678901234567890123456789012123456789012345678901 2
12345678901234567890123456789012123456789012345678901 2
12345678901234567890123456789012123456789012345678901
stanch (verb) To stop the flow. When Edison began to bleed 2
12345678901234567890123456789012123456789012345678901 2
12345678901234567890123456789012123456789012345678901 2
2345678901234567890123456789012123456789012345678901
profusely, Dr. Munger stanched the blood flow by
2
1
12345678901234567890123456789012123456789012345678901
applying direct pressure to the wound. 2
12345678901234567890123456789012123456789012345678901 2
12345678901234567890123456789012123456789012345678901 2
12345678901234567890123456789012123456789012345678901 2
2345678901234567890123456789012123456789012345678901
stint (verb) To limit, to restrain. The British bed and
breakfast certainly did not stint on the breakfast part of 2
12345678901234567890123456789012123456789012345678901
12345678901234567890123456789012123456789012345678901 2
the equation; they provided us with fried tomatoes, fried 2
12345678901234567890123456789012123456789012345678901
12345678901234567890123456789012123456789012345678901 2
sausages, fried eggs, smoked kippers, fried bread, fried 2
12345678901234567890123456789012123456789012345678901
12345678901234567890123456789012123456789012345678901 2
mushrooms, and bowls of a cereal called “Wheatabix” 2
12345678901234567890123456789012123456789012345678901
12345678901234567890123456789012123456789012345678901 2 2
1 (which tasted like cardboard). stinting (adjective).
12345678901234567890123456789012123456789012345678901 2
12345678901234567890123456789012123456789012345678901 2
12345678901234567890123456789012123456789012345678901
stolid (adjective) Impassive, unemotional. The popular ani- 2
12345678901234567890123456789012123456789012345678901 2
12345678901234567890123456789012123456789012345678901
mated television series King of the Hill chronicles the 2
2345678901234567890123456789012123456789012345678901
woes of a stolid, conservative Texan confronting chang- 2
12345678901234567890123456789012123456789012345678901
12345678901234567890123456789012123456789012345678901 2
1 2
12345678901234567890123456789012123456789012345678901
ing times. stolidity (noun). 2
12345678901234567890123456789012123456789012345678901 2
2345678901234567890123456789012123456789012345678901
subordination (noun) The state of being subservient or 2
12345678901234567890123456789012123456789012345678901
12345678901234567890123456789012123456789012345678901 2
1 2
12345678901234567890123456789012123456789012345678901 2
treated as less valuable. Heather left the naval academy
12345678901234567890123456789012123456789012345678901
because she could no longer stand the subordination of 2
12345678901234567890123456789012123456789012345678901 2
12345678901234567890123456789012123456789012345678901
every personal whim or desire to the rigorous demands 2
12345678901234567890123456789012123456789012345678901 2
12345678901234567890123456789012123456789012345678901
of military life. subordinate (verb). 2
12345678901234567890123456789012123456789012345678901 2
12345678901234567890123456789012123456789012345678901 2
2345678901234567890123456789012123456789012345678901
12345678901234567890123456789012123456789012345678901 2
12345678901234567890123456789012123456789012345678901 2
12345678901234567890123456789012123456789012345678901 2 2
1
12345678901234567890123456789012123456789012345678901 2
12345678901234567890123456789012123456789012345678901 2
12345678901234567890123456789012123456789012345678901 2
12345678901234567890123456789012123456789012345678901 2
12345678901234567890123456789012123456789012345678901 2
12345678901234567890123456789012123456789012345678901 2
2345678901234567890123456789012123456789012345678901
2
1
12345678901234567890123456789012123456789012345678901 2
12345678901234567890123456789012123456789012345678901 2
12345678901234567890123456789012123456789012345678901 2
12345678901234567890123456789012123456789012345678901 2
12345678901234567890123456789012123456789012345678901 2
12345678901234567890123456789012123456789012345678901 2 2
218 12345678901234567890123456789012123456789012345678901
123456789012345678901234567890121234567890123456789012

www.petersons.com
Chapter 5: Verbal Review

123456789012345678901234567890121234567890123456789012
12345678901234567890123456789012123456789012345678901 2
12345678901234567890123456789012123456789012345678901 2
12345678901234567890123456789012123456789012345678901 2
12345678901234567890123456789012123456789012345678901
Latin poena 5 pain. Also found in English impunity, penal, penalty, 2
12345678901234567890123456789012123456789012345678901
Word Origin 2
12345678901234567890123456789012123456789012345678901 2
and punishment.
12345678901234567890123456789012123456789012345678901 2
12345678901234567890123456789012123456789012345678901
subpoena (noun) An order of a court, legislation or grand 2
12345678901234567890123456789012123456789012345678901 2
12345678901234567890123456789012123456789012345678901
jury that compels a witness to be present at a trial or 2
12345678901234567890123456789012123456789012345678901 2
12345678901234567890123456789012123456789012345678901
hearing. The young man’s lawyer asked the judge to 2
2345678901234567890123456789012123456789012345678901 2
1
12345678901234567890123456789012123456789012345678901
subpoena a boa constrictor into court on the grounds 2
12345678901234567890123456789012123456789012345678901
that the police had used the snake as an “instrument of 2
12345678901234567890123456789012123456789012345678901 2
12345678901234567890123456789012123456789012345678901
2345678901234567890123456789012123456789012345678901
2
2
1 terror” to coerce his confession.
12345678901234567890123456789012123456789012345678901 2
12345678901234567890123456789012123456789012345678901 2
12345678901234567890123456789012123456789012345678901
subside (verb) To settle or die down. The celebrated lecturer 2
12345678901234567890123456789012123456789012345678901
2345678901234567890123456789012123456789012345678901
2
2
1 had to wait ten minutes for the applause to subside
12345678901234567890123456789012123456789012345678901 2
12345678901234567890123456789012123456789012345678901
before he began his speech. 2
12345678901234567890123456789012123456789012345678901 2
12345678901234567890123456789012123456789012345678901
subsidization (noun) The state of being financed by a grant 2
12345678901234567890123456789012123456789012345678901 22
2345678901234567890123456789012123456789012345678901
12345678901234567890123456789012123456789012345678901

Take It to the Next Level


from a government or other agency. Without subsidiza-
12345678901234567890123456789012123456789012345678901
tion, the nation’s passenger rail system would probably 2
2
12345678901234567890123456789012123456789012345678901
12345678901234567890123456789012123456789012345678901 2
12345678901234567890123456789012123456789012345678901
go bankrupt. subsidize (verb). 2
1 2
12345678901234567890123456789012123456789012345678901 2
12345678901234567890123456789012123456789012345678901
substantiated (adjective) Verified or supported by evidence. 2
12345678901234567890123456789012123456789012345678901 2
12345678901234567890123456789012123456789012345678901 22
2345678901234567890123456789012123456789012345678901
The charge that Nixon had helped to cover up crimes
12345678901234567890123456789012123456789012345678901
was substantiated by his comments about it on a series 2
12345678901234567890123456789012123456789012345678901
of audiotapes. substantiate (verb), substantiation (noun). 2
12345678901234567890123456789012123456789012345678901
12345678901234567890123456789012123456789012345678901 2
12345678901234567890123456789012123456789012345678901 22
12345678901234567890123456789012123456789012345678901
subsume (verb) To encompass or engulf within something 2
12345678901234567890123456789012123456789012345678901
larger. In Alan Dershowitz’s Reversal of Fortune, he 2
12345678901234567890123456789012123456789012345678901
1 2
2345678901234567890123456789012123456789012345678901
makes it clear that his work as a lawyer subsumes his 2
12345678901234567890123456789012123456789012345678901
12345678901234567890123456789012123456789012345678901 2
1 personal life. 2
12345678901234567890123456789012123456789012345678901 2
12345678901234567890123456789012123456789012345678901 2
12345678901234567890123456789012123456789012345678901
subterranean (adjective) Under the surface of the earth. 2
12345678901234567890123456789012123456789012345678901 2
12345678901234567890123456789012123456789012345678901
Subterranean testing of nuclear weapons was permitted 2
12345678901234567890123456789012123456789012345678901 2
12345678901234567890123456789012123456789012345678901
under the Nuclear Test Ban Treaty of 1963. 2
12345678901234567890123456789012123456789012345678901 22
2345678901234567890123456789012123456789012345678901
12345678901234567890123456789012123456789012345678901
summarily (adverb) Quickly and concisely. No sooner had I 2
1
12345678901234567890123456789012123456789012345678901
voiced my concerns about the new ad campaign than my 2
12345678901234567890123456789012123456789012345678901 2
2345678901234567890123456789012123456789012345678901
boss put her hand on my elbow and summarily ushered 2
12345678901234567890123456789012123456789012345678901
12345678901234567890123456789012123456789012345678901 2
12345678901234567890123456789012123456789012345678901 2
12345678901234567890123456789012123456789012345678901 22
me out of her office.
1
12345678901234567890123456789012123456789012345678901
superficial (adjective) On the surface only; without depth or 2
12345678901234567890123456789012123456789012345678901 2
12345678901234567890123456789012123456789012345678901
substance. Her wound was only superficial and required 2
12345678901234567890123456789012123456789012345678901 2
12345678901234567890123456789012123456789012345678901
no treatment except a light bandage. His superficial 2
2345678901234567890123456789012123456789012345678901
12345678901234567890123456789012123456789012345678901
attractiveness hides the fact that his personality is lifeless 2
2
12345678901234567890123456789012123456789012345678901 2
12345678901234567890123456789012123456789012345678901 2
12345678901234567890123456789012123456789012345678901
and his mind is dull. superficiality (noun).
2
1
12345678901234567890123456789012123456789012345678901 22
2345678901234567890123456789012123456789012345678901
1
12345678901234567890123456789012123456789012345678901 2
12345678901234567890123456789012123456789012345678901 2
12345678901234567890123456789012123456789012345678901 2
12345678901234567890123456789012123456789012345678901 2
12345678901234567890123456789012123456789012345678901 2
12345678901234567890123456789012123456789012345678901 2
12345678901234567890123456789012123456789012345678901 2 219
123456789012345678901234567890121234567890123456789012
Part III: Section Review

123456789012345678901234567890121234567890123456789012
12345678901234567890123456789012123456789012345678901 2
12345678901234567890123456789012123456789012345678901 2
12345678901234567890123456789012123456789012345678901
superimpose (verb) To place or lay over or above some- 2
12345678901234567890123456789012123456789012345678901 2
12345678901234567890123456789012123456789012345678901
thing. The artist stirred controversy by superimposing 2
12345678901234567890123456789012123456789012345678901 2
12345678901234567890123456789012123456789012345678901
portraits of certain contemporary politicians over im- 2
2345678901234567890123456789012123456789012345678901 2
1
12345678901234567890123456789012123456789012345678901
ages of such reviled historical figures as Hitler and 2
12345678901234567890123456789012123456789012345678901 2
12345678901234567890123456789012123456789012345678901
Stalin. 2
12345678901234567890123456789012123456789012345678901
2345678901234567890123456789012123456789012345678901
2
1 supersede (verb) To displace, to substitute or supplant. “I’m 2
12345678901234567890123456789012123456789012345678901 2
12345678901234567890123456789012123456789012345678901 2
12345678901234567890123456789012123456789012345678901
sorry,” the principal announced, “but today’s afternoon 2
12345678901234567890123456789012123456789012345678901
2345678901234567890123456789012123456789012345678901
2
2
1 classes will be superseded by an assembly on drug and
12345678901234567890123456789012123456789012345678901 2
12345678901234567890123456789012123456789012345678901
alcohol abuse.” 2
12345678901234567890123456789012123456789012345678901 2
12345678901234567890123456789012123456789012345678901
supine (adjective) Prone. One always feels rather vulnerable 2
2345678901234567890123456789012123456789012345678901 2
1
12345678901234567890123456789012123456789012345678901
when wearing a flimsy paper gown and lying supine on a 2
12345678901234567890123456789012123456789012345678901 2
12345678901234567890123456789012123456789012345678901
doctor’s examining table. 2
12345678901234567890123456789012123456789012345678901 2
2345678901234567890123456789012123456789012345678901
supposition (noun) Assumption, conjecture. While most 2
12345678901234567890123456789012123456789012345678901
12345678901234567890123456789012123456789012345678901 2
12345678901234567890123456789012123456789012345678901 2
12345678901234567890123456789012123456789012345678901 2
climate researchers believe that increasing levels of
greenhouse gases will warm the planet, skeptics claim 2
12345678901234567890123456789012123456789012345678901
12345678901234567890123456789012123456789012345678901 2
1 2
12345678901234567890123456789012123456789012345678901
that this theory is mere supposition. suppose (verb). 2
12345678901234567890123456789012123456789012345678901 2
12345678901234567890123456789012123456789012345678901
surge (noun) A gush; a swelling or sweeping forward. When 2
2345678901234567890123456789012123456789012345678901
12345678901234567890123456789012123456789012345678901
Mattel gave the Barbie Doll a makeover in the late 2
2
12345678901234567890123456789012123456789012345678901
12345678901234567890123456789012123456789012345678901
eighties, coming out with dolls like doctor Barbie and 2
2
12345678901234567890123456789012123456789012345678901
12345678901234567890123456789012123456789012345678901
astronaut Barbie, the company experienced a surge in 2
2
12345678901234567890123456789012123456789012345678901
12345678901234567890123456789012123456789012345678901 2
12345678901234567890123456789012123456789012345678901
sales. 2
12345678901234567890123456789012123456789012345678901 2 2
1
12345678901234567890123456789012123456789012345678901 2
12345678901234567890123456789012123456789012345678901 2
12345678901234567890123456789012123456789012345678901
Word
Latin tangere 5 to touch. Also found in English contact,
Origin contiguous, tactile, tangent, and tangible. 2
12345678901234567890123456789012123456789012345678901 2
12345678901234567890123456789012123456789012345678901 2
12345678901234567890123456789012123456789012345678901
tangential (adjective) Touching lightly; only slightly con- 2
12345678901234567890123456789012123456789012345678901 2
12345678901234567890123456789012123456789012345678901
nected or related. Having enrolled in a class on 2
12345678901234567890123456789012123456789012345678901 2
12345678901234567890123456789012123456789012345678901
African-American history, the students found the teach- 2
12345678901234567890123456789012123456789012345678901 2
2345678901234567890123456789012123456789012345678901
er’s stories about his travels in South America only of 2
12345678901234567890123456789012123456789012345678901
1 2
12345678901234567890123456789012123456789012345678901
tangential interest. tangent (noun). 2
12345678901234567890123456789012123456789012345678901 2
2345678901234567890123456789012123456789012345678901
12345678901234567890123456789012123456789012345678901
tedium (noun) Boredom. For most people, watching even a 2
2
12345678901234567890123456789012123456789012345678901 2
12345678901234567890123456789012123456789012345678901
fifteen-minute broadcast of Earth as seen from space 2
12345678901234567890123456789012123456789012345678901
1 would be an exercise in sheer tedium. tedious (adjective). 2
12345678901234567890123456789012123456789012345678901 2
12345678901234567890123456789012123456789012345678901 2
12345678901234567890123456789012123456789012345678901 2
12345678901234567890123456789012123456789012345678901
temperance (noun) Moderation or restraint in feelings and 2
12345678901234567890123456789012123456789012345678901 2
12345678901234567890123456789012123456789012345678901 2
2345678901234567890123456789012123456789012345678901
behavior. Most professional athletes practice temper-
ance in their personal habits; too much eating or 2
12345678901234567890123456789012123456789012345678901
12345678901234567890123456789012123456789012345678901 2
12345678901234567890123456789012123456789012345678901 2
1 drinking and too many late nights, they know, can harm 2
2345678901234567890123456789012123456789012345678901
12345678901234567890123456789012123456789012345678901
their performance. 2
1 2
12345678901234567890123456789012123456789012345678901 2
12345678901234567890123456789012123456789012345678901
temperate (adjective) Moderate, calm. The warm gulf 2
12345678901234567890123456789012123456789012345678901 2
12345678901234567890123456789012123456789012345678901
streams are largely responsible for the temperate climate 2
12345678901234567890123456789012123456789012345678901 2
12345678901234567890123456789012123456789012345678901
of the British Isles. 2
2
220 12345678901234567890123456789012123456789012345678901
123456789012345678901234567890121234567890123456789012

www.petersons.com
Chapter 5: Verbal Review

123456789012345678901234567890121234567890123456789012
12345678901234567890123456789012123456789012345678901 2
12345678901234567890123456789012123456789012345678901 2
12345678901234567890123456789012123456789012345678901
tenuous (adjective) Lacking in substance; weak, flimsy, very 2
12345678901234567890123456789012123456789012345678901 2
12345678901234567890123456789012123456789012345678901
thin. His tenuous grasp of the Spanish language was 2
12345678901234567890123456789012123456789012345678901 2
12345678901234567890123456789012123456789012345678901
evident when he addressed Señor Chavez as “Señora.”
2345678901234567890123456789012123456789012345678901
2
2
1
12345678901234567890123456789012123456789012345678901 2
12345678901234567890123456789012123456789012345678901
terrestrial (adjective) Of the earth. The movie Close Encoun- 2
12345678901234567890123456789012123456789012345678901 2
12345678901234567890123456789012123456789012345678901
ters of the Third Kind tells
2345678901234567890123456789012123456789012345678901
the story of the first contact 2
1 between beings from outer space and terrestrial crea- 2
12345678901234567890123456789012123456789012345678901 2
12345678901234567890123456789012123456789012345678901
tures. 2
12345678901234567890123456789012123456789012345678901 2
12345678901234567890123456789012123456789012345678901
2345678901234567890123456789012123456789012345678901
2
2
1 throwback (noun) A reversion to an earlier type; an
12345678901234567890123456789012123456789012345678901
atavism. The new Volkswagen Beetle, with its familiar 2
12345678901234567890123456789012123456789012345678901 2
12345678901234567890123456789012123456789012345678901 2
12345678901234567890123456789012123456789012345678901
bubble shape, looks like a throwback
2345678901234567890123456789012123456789012345678901
to the sixties, but 2
1 it is actually packed with modern high-tech equipment. 2
12345678901234567890123456789012123456789012345678901 2
12345678901234567890123456789012123456789012345678901 2
12345678901234567890123456789012123456789012345678901
tiff (noun) A small, almost inconsequential quarrel or 2
12345678901234567890123456789012123456789012345678901 2
12345678901234567890123456789012123456789012345678901
disagreement. Megan and Bruce got into a tiff when 2
12345678901234567890123456789012123456789012345678901 2

Take It to the Next Level


12345678901234567890123456789012123456789012345678901
Bruce criticized her smoking. 2
12345678901234567890123456789012123456789012345678901 2
12345678901234567890123456789012123456789012345678901 2
12345678901234567890123456789012123456789012345678901 22
2345678901234567890123456789012123456789012345678901
tirade (noun) A long, harshly critical speech. Reformed
1
12345678901234567890123456789012123456789012345678901
smokers, like Bruce, are prone to delivering tirades on 2
12345678901234567890123456789012123456789012345678901 2
12345678901234567890123456789012123456789012345678901
the evils of smoking. 2
12345678901234567890123456789012123456789012345678901 22
2345678901234567890123456789012123456789012345678901
12345678901234567890123456789012123456789012345678901
torpor (noun) Apathy, sluggishness. Stranded in an airless 2
12345678901234567890123456789012123456789012345678901
hotel room in Madras after a 27-hour train ride, I felt 2
12345678901234567890123456789012123456789012345678901
12345678901234567890123456789012123456789012345678901 2
such overwhelming torpor that I doubted I would make 2
12345678901234567890123456789012123456789012345678901
12345678901234567890123456789012123456789012345678901 2
12345678901234567890123456789012123456789012345678901
it to Bangalore, the next leg of my journey. torpid 2
12345678901234567890123456789012123456789012345678901 22
1 (adjective).
12345678901234567890123456789012123456789012345678901 2
12345678901234567890123456789012123456789012345678901 2
12345678901234567890123456789012123456789012345678901
tout (verb) To praise highly, to brag publicly. A much 2
12345678901234567890123456789012123456789012345678901 2
12345678901234567890123456789012123456789012345678901
happier Eileen is now touting the benefits of Prozac, but, 2
2345678901234567890123456789012123456789012345678901 2
12345678901234567890123456789012123456789012345678901
to tell you the truth, I miss her witty, self-lacerating 2
12345678901234567890123456789012123456789012345678901 2
1
12345678901234567890123456789012123456789012345678901
commentaries. 2
12345678901234567890123456789012123456789012345678901 2
12345678901234567890123456789012123456789012345678901 22
2345678901234567890123456789012123456789012345678901
12345678901234567890123456789012123456789012345678901 2
1
12345678901234567890123456789012123456789012345678901
Word
Latin tractare 5 to handle. Also found in English intractable,
Origin tractate, and traction. 2
12345678901234567890123456789012123456789012345678901 2
12345678901234567890123456789012123456789012345678901 22
2345678901234567890123456789012123456789012345678901
12345678901234567890123456789012123456789012345678901
tractable (adjective) Obedient, manageable. When he turned 2
12345678901234567890123456789012123456789012345678901
1 3, Harrison suddenly became a tractable, well-mannered 2
12345678901234567890123456789012123456789012345678901 2
2345678901234567890123456789012123456789012345678901
little boy after being, quite frankly, an unruly little 2
12345678901234567890123456789012123456789012345678901
12345678901234567890123456789012123456789012345678901 2
12345678901234567890123456789012123456789012345678901
monster! 2
12345678901234567890123456789012123456789012345678901 22
1
12345678901234567890123456789012123456789012345678901
tranquillity (noun) Freedom from disturbance or turmoil; calm. 2
12345678901234567890123456789012123456789012345678901 2
12345678901234567890123456789012123456789012345678901
Seeking the tranquillity of country life, she moved from 2
12345678901234567890123456789012123456789012345678901 2
12345678901234567890123456789012123456789012345678901
New York City to rural Vermont. tranquil (adjective). 2
12345678901234567890123456789012123456789012345678901 22
2345678901234567890123456789012123456789012345678901
1
12345678901234567890123456789012123456789012345678901 2
12345678901234567890123456789012123456789012345678901 2
12345678901234567890123456789012123456789012345678901 2
12345678901234567890123456789012123456789012345678901 2
12345678901234567890123456789012123456789012345678901 2
12345678901234567890123456789012123456789012345678901 2
12345678901234567890123456789012123456789012345678901 2 221
123456789012345678901234567890121234567890123456789012
Part III: Section Review

123456789012345678901234567890121234567890123456789012
12345678901234567890123456789012123456789012345678901 2
12345678901234567890123456789012123456789012345678901 2
12345678901234567890123456789012123456789012345678901
transgress (verb) To go past limits; to violate. If Iraq has 2
12345678901234567890123456789012123456789012345678901 2
12345678901234567890123456789012123456789012345678901
developed biological weapons, then it has transgressed 2
12345678901234567890123456789012123456789012345678901 2
12345678901234567890123456789012123456789012345678901
the UN’s rules against manufacturing weapons of mass 2
2345678901234567890123456789012123456789012345678901 2
1
12345678901234567890123456789012123456789012345678901
destruction. transgression (noun). 2
12345678901234567890123456789012123456789012345678901 2
12345678901234567890123456789012123456789012345678901 2
12345678901234567890123456789012123456789012345678901
transmute (verb) To change in form
2345678901234567890123456789012123456789012345678901
or substance. Practitio- 2
1 ners of alchemy, a forebear of modern chemistry, tried to 2
12345678901234567890123456789012123456789012345678901 2
12345678901234567890123456789012123456789012345678901 2
12345678901234567890123456789012123456789012345678901
discover ways to transmute metals such as iron into 2
12345678901234567890123456789012123456789012345678901
2345678901234567890123456789012123456789012345678901
2
2
1 gold. transmutation (noun).
12345678901234567890123456789012123456789012345678901 2
12345678901234567890123456789012123456789012345678901
treacherous (adjective) Untrustworthy or disloyal; danger- 2
12345678901234567890123456789012123456789012345678901 2
12345678901234567890123456789012123456789012345678901
ous or unreliable. Nazi Germany proved to be a 2
2345678901234567890123456789012123456789012345678901 2
1
12345678901234567890123456789012123456789012345678901
treacherous ally, first signing a peace pact with the 2
12345678901234567890123456789012123456789012345678901 2
12345678901234567890123456789012123456789012345678901
Soviet Union, then invading. Be careful crossing the rope 2
12345678901234567890123456789012123456789012345678901 2
12345678901234567890123456789012123456789012345678901
bridge; parts of the span are badly frayed and treacher- 2
12345678901234567890123456789012123456789012345678901 2
12345678901234567890123456789012123456789012345678901
ous. treachery (noun). 2
12345678901234567890123456789012123456789012345678901 2
12345678901234567890123456789012123456789012345678901 2
12345678901234567890123456789012123456789012345678901 2
2345678901234567890123456789012123456789012345678901
tremor (noun) An involuntary shaking or trembling. Michael
1 still manages to appear calm and at ease despite the 2
12345678901234567890123456789012123456789012345678901 2
12345678901234567890123456789012123456789012345678901 2
12345678901234567890123456789012123456789012345678901
tremors caused by Parkinson’s disease. Brooke felt the 2
2345678901234567890123456789012123456789012345678901
first tremors of the 1989 San Francisco earthquake while 2
12345678901234567890123456789012123456789012345678901
12345678901234567890123456789012123456789012345678901 2
12345678901234567890123456789012123456789012345678901 2
12345678901234567890123456789012123456789012345678901 2
she was sitting in Candlestick Park watching a Giants
12345678901234567890123456789012123456789012345678901 2
12345678901234567890123456789012123456789012345678901
baseball game. 2
12345678901234567890123456789012123456789012345678901 2
trenchant (adjective) Caustic and incisive. Essayist H. L. 2
12345678901234567890123456789012123456789012345678901
12345678901234567890123456789012123456789012345678901 2
1 Mencken was known for his trenchant wit and was 2
12345678901234567890123456789012123456789012345678901 2
2345678901234567890123456789012123456789012345678901
famed for mercilessly puncturing the American middle 2
12345678901234567890123456789012123456789012345678901
1 2
12345678901234567890123456789012123456789012345678901
class (which he called the “booboisie”). 2
12345678901234567890123456789012123456789012345678901 2
12345678901234567890123456789012123456789012345678901 2
2345678901234567890123456789012123456789012345678901
12345678901234567890123456789012123456789012345678901 2 2
1 Origin Latin trepidus 5 alarmed. Also found in English intrepid.
12345678901234567890123456789012123456789012345678901
Word 2
12345678901234567890123456789012123456789012345678901 2
2345678901234567890123456789012123456789012345678901
trepidation (noun) Fear and anxiety. After the tragedy of 2
12345678901234567890123456789012123456789012345678901
12345678901234567890123456789012123456789012345678901 2
1 TWA flight 800, many previously fearless flyers were 2
12345678901234567890123456789012123456789012345678901 2
12345678901234567890123456789012123456789012345678901
filled with trepidation whenever they stepped into an 2
12345678901234567890123456789012123456789012345678901 2
12345678901234567890123456789012123456789012345678901
airplane. 2
12345678901234567890123456789012123456789012345678901 2
12345678901234567890123456789012123456789012345678901 2
12345678901234567890123456789012123456789012345678901 2
turbulent (adjective) Agitated or disturbed. The night be-
2345678901234567890123456789012123456789012345678901
12345678901234567890123456789012123456789012345678901
fore the championship match, Martina was unable to 2
2
12345678901234567890123456789012123456789012345678901
sleep, her mind turbulent with fears and hopes. turbu- 2
12345678901234567890123456789012123456789012345678901
12345678901234567890123456789012123456789012345678901 2
1 2
12345678901234567890123456789012123456789012345678901 2
2345678901234567890123456789012123456789012345678901
lence (noun).
12345678901234567890123456789012123456789012345678901 2
turpitude (noun) Depravity, wickedness. Radical feminists 2
12345678901234567890123456789012123456789012345678901
12345678901234567890123456789012123456789012345678901 2
1 2
12345678901234567890123456789012123456789012345678901 2
2345678901234567890123456789012123456789012345678901
who contrast women’s essential goodness with men’s
1 moral turpitude can be likened to religious fundamental- 2
12345678901234567890123456789012123456789012345678901 2
12345678901234567890123456789012123456789012345678901 2
12345678901234567890123456789012123456789012345678901
ists who make a clear distinction
2345678901234567890123456789012123456789012345678901
between the saved and 2
2
1 the damned.
12345678901234567890123456789012123456789012345678901 2
12345678901234567890123456789012123456789012345678901 2 2
222 12345678901234567890123456789012123456789012345678901
123456789012345678901234567890121234567890123456789012

www.petersons.com
Chapter 5: Verbal Review

123456789012345678901234567890121234567890123456789012
12345678901234567890123456789012123456789012345678901 2
12345678901234567890123456789012123456789012345678901 2
12345678901234567890123456789012123456789012345678901
tyro (noun) Novice, amateur. For an absolute tyro on the 2
12345678901234567890123456789012123456789012345678901 2
12345678901234567890123456789012123456789012345678901
ski slopes, Gina was surprisingly agile at taking the 2
12345678901234567890123456789012123456789012345678901 2
12345678901234567890123456789012123456789012345678901
moguls.
2345678901234567890123456789012123456789012345678901
2
2
1
12345678901234567890123456789012123456789012345678901 2
12345678901234567890123456789012123456789012345678901
unalloyed (adjective) Unqualified, pure. Holding his new- 2
12345678901234567890123456789012123456789012345678901 2
12345678901234567890123456789012123456789012345678901
born son for the first time,
2345678901234567890123456789012123456789012345678901
Malik felt an unalloyed 2
1 happiness that was unlike anything he had ever experi- 2
12345678901234567890123456789012123456789012345678901 2
12345678901234567890123456789012123456789012345678901 2
12345678901234567890123456789012123456789012345678901
enced in his 45 years. 2
12345678901234567890123456789012123456789012345678901
2345678901234567890123456789012123456789012345678901
2
1 undermine (verb) To excavate beneath; to subvert, to 2
12345678901234567890123456789012123456789012345678901 2
12345678901234567890123456789012123456789012345678901
weaken. Dot continued to undermine my efforts to find 2
12345678901234567890123456789012123456789012345678901 2
12345678901234567890123456789012123456789012345678901
her a date by showing up at our dinner parties in her 2
2345678901234567890123456789012123456789012345678901 2
1
12345678901234567890123456789012123456789012345678901
ratty old sweat suit. 2
12345678901234567890123456789012123456789012345678901 2
12345678901234567890123456789012123456789012345678901 2
12345678901234567890123456789012123456789012345678901
unfeigned (adjective) Genuine, sincere. Lashawn responded 2
2345678901234567890123456789012123456789012345678901
12345678901234567890123456789012123456789012345678901
with such unfeigned astonishment when we all leapt out 2
2
12345678901234567890123456789012123456789012345678901

Take It to the Next Level


of the kitchen that I think she had had no inkling of the 2
12345678901234567890123456789012123456789012345678901
12345678901234567890123456789012123456789012345678901 2
12345678901234567890123456789012123456789012345678901 2
12345678901234567890123456789012123456789012345678901 22
surprise party.
1
12345678901234567890123456789012123456789012345678901
univocal (adjective) With a single voice. While they came 2
12345678901234567890123456789012123456789012345678901 2
12345678901234567890123456789012123456789012345678901 2
12345678901234567890123456789012123456789012345678901 22
2345678901234567890123456789012123456789012345678901
from different backgrounds and classes, the employees
12345678901234567890123456789012123456789012345678901
were univocal in their demands that the corrupt CEO 2
12345678901234567890123456789012123456789012345678901
12345678901234567890123456789012123456789012345678901
resign immediately. 2
12345678901234567890123456789012123456789012345678901 2
12345678901234567890123456789012123456789012345678901 22
12345678901234567890123456789012123456789012345678901
unstinting (adjective) Giving with unrestrained generosity.
12345678901234567890123456789012123456789012345678901
Few people will be able to match the unstinting 2
2
12345678901234567890123456789012123456789012345678901 2
1
12345678901234567890123456789012123456789012345678901
dedication and care that Mother Theresa lavished on the 2
12345678901234567890123456789012123456789012345678901 2
12345678901234567890123456789012123456789012345678901
poor people of Calcutta. 2
12345678901234567890123456789012123456789012345678901 2
12345678901234567890123456789012123456789012345678901 2
12345678901234567890123456789012123456789012345678901 2
12345678901234567890123456789012123456789012345678901 2
12345678901234567890123456789012123456789012345678901 2
Word Origin Latin urbs 5 city. Also found in English suburb and urban.
12345678901234567890123456789012123456789012345678901 2
12345678901234567890123456789012123456789012345678901
urbanity (noun) Sophistication, suaveness and polish. Part 2
12345678901234567890123456789012123456789012345678901 22
2345678901234567890123456789012123456789012345678901
12345678901234567890123456789012123456789012345678901
of the fun in a Cary Grant movie lies in seeing whether
2
1
12345678901234567890123456789012123456789012345678901
the star can be made to lose his urbanity and elegance in 2
12345678901234567890123456789012123456789012345678901 2
12345678901234567890123456789012123456789012345678901
the midst of chaotic or kooky situations. urbane 2
12345678901234567890123456789012123456789012345678901 2
12345678901234567890123456789012123456789012345678901
(adjective). 2
12345678901234567890123456789012123456789012345678901 2
12345678901234567890123456789012123456789012345678901 2
12345678901234567890123456789012123456789012345678901 22
2345678901234567890123456789012123456789012345678901
usurious (adjective) Lending money at an unconscionably
12345678901234567890123456789012123456789012345678901
high interest rate. Some people feel that Shakespeare’s 2
12345678901234567890123456789012123456789012345678901
1 portrayal of the Jew, Shylock, the usurious money lender 2
12345678901234567890123456789012123456789012345678901 2
2345678901234567890123456789012123456789012345678901
in The Merchant of Venice, has enflamed prejudice 2
12345678901234567890123456789012123456789012345678901
12345678901234567890123456789012123456789012345678901 2
12345678901234567890123456789012123456789012345678901 2
12345678901234567890123456789012123456789012345678901 22
against the Jews. usury (adjective).
1
12345678901234567890123456789012123456789012345678901 2
12345678901234567890123456789012123456789012345678901 2
12345678901234567890123456789012123456789012345678901 2
12345678901234567890123456789012123456789012345678901 2
12345678901234567890123456789012123456789012345678901 2
12345678901234567890123456789012123456789012345678901 2
12345678901234567890123456789012123456789012345678901 2
12345678901234567890123456789012123456789012345678901 2
12345678901234567890123456789012123456789012345678901 2 223
123456789012345678901234567890121234567890123456789012
Part III: Section Review

123456789012345678901234567890121234567890123456789012
12345678901234567890123456789012123456789012345678901 2
12345678901234567890123456789012123456789012345678901 2
12345678901234567890123456789012123456789012345678901 2
12345678901234567890123456789012123456789012345678901
Latin validus 5 strong. Also found in English invalid, invaluable, 2
12345678901234567890123456789012123456789012345678901
Word Origin 2
12345678901234567890123456789012123456789012345678901 2
prevail, and value.
12345678901234567890123456789012123456789012345678901 2
12345678901234567890123456789012123456789012345678901
validate (verb) To officially approve or confirm. The 2
12345678901234567890123456789012123456789012345678901 2
12345678901234567890123456789012123456789012345678901
election of the president is formally validated when the 2
12345678901234567890123456789012123456789012345678901 2
12345678901234567890123456789012123456789012345678901
members of the Electoral College meet to confirm the 2
2345678901234567890123456789012123456789012345678901 2
1
12345678901234567890123456789012123456789012345678901
verdict of the voters. valid (adjective), validity (noun). 2
12345678901234567890123456789012123456789012345678901 2
12345678901234567890123456789012123456789012345678901 2
12345678901234567890123456789012123456789012345678901
vapid (adjective) Flat, flavorless.
2345678901234567890123456789012123456789012345678901
Whenever I have insom- 2
1 nia, I just tune the clock radio to Lite FM, and soon 2
12345678901234567890123456789012123456789012345678901 2
12345678901234567890123456789012123456789012345678901 2
12345678901234567890123456789012123456789012345678901
those vapid songs from the seventies have me floating 2
12345678901234567890123456789012123456789012345678901
2345678901234567890123456789012123456789012345678901
2
2
1 away to dreamland. vapidity (noun).
12345678901234567890123456789012123456789012345678901 2
12345678901234567890123456789012123456789012345678901
venal (adjective) Corrupt, mercenary. Sese Seko Mobuto 2
12345678901234567890123456789012123456789012345678901 2
12345678901234567890123456789012123456789012345678901
was the venal dictator of Zaire who reportedly diverted 2
12345678901234567890123456789012123456789012345678901 2
2345678901234567890123456789012123456789012345678901
millions of dollars in foreign aid to his own personal 2
12345678901234567890123456789012123456789012345678901
12345678901234567890123456789012123456789012345678901 2
12345678901234567890123456789012123456789012345678901
fortune. venality (noun). 2
12345678901234567890123456789012123456789012345678901 2
1 veneer (noun) A superficial or deceptive covering. Beneath 2
12345678901234567890123456789012123456789012345678901 2
12345678901234567890123456789012123456789012345678901 2
12345678901234567890123456789012123456789012345678901
her folksy veneer, Samantha is a shrewd and calculating 2
12345678901234567890123456789012123456789012345678901 2
12345678901234567890123456789012123456789012345678901 2
2345678901234567890123456789012123456789012345678901
businessperson just waiting for the right moment to
12345678901234567890123456789012123456789012345678901 2
12345678901234567890123456789012123456789012345678901
pounce. 2
12345678901234567890123456789012123456789012345678901 2
venerate (verb) To admire or honor. In Communist China, 2
12345678901234567890123456789012123456789012345678901
12345678901234567890123456789012123456789012345678901 2
12345678901234567890123456789012123456789012345678901 2
12345678901234567890123456789012123456789012345678901 2
Mao Tse-Tung is venerated as an almost godlike figure.
12345678901234567890123456789012123456789012345678901
venerable (adjective), veneration (noun). 2
1 2
12345678901234567890123456789012123456789012345678901 2
2345678901234567890123456789012123456789012345678901
veracious (adjective) Truthful, earnest. Many people still 2
12345678901234567890123456789012123456789012345678901
1 2
12345678901234567890123456789012123456789012345678901
feel that Anita Hill was entirely veracious in her 2
12345678901234567890123456789012123456789012345678901 2
12345678901234567890123456789012123456789012345678901
allegations of sexual harassment during the Clarence 2
12345678901234567890123456789012123456789012345678901 2
12345678901234567890123456789012123456789012345678901
Thomas confirmation hearings. veracity (noun). 2
12345678901234567890123456789012123456789012345678901 2
12345678901234567890123456789012123456789012345678901 2
12345678901234567890123456789012123456789012345678901 2
2345678901234567890123456789012123456789012345678901
verify (verb) To prove to be true. The contents of Robert L.
1 Ripley’s syndicated “Believe it or Not” cartoons could 2
12345678901234567890123456789012123456789012345678901 2
12345678901234567890123456789012123456789012345678901 2
12345678901234567890123456789012123456789012345678901 2
not be verified, yet the public still thrilled to reports of
2345678901234567890123456789012123456789012345678901
“the man with two pupils in each eye,” “the human 2
12345678901234567890123456789012123456789012345678901
12345678901234567890123456789012123456789012345678901 2
12345678901234567890123456789012123456789012345678901 2
12345678901234567890123456789012123456789012345678901 2
unicorn,” and other amazing oddities. verification
2
1
12345678901234567890123456789012123456789012345678901
(noun). 2
12345678901234567890123456789012123456789012345678901 2
12345678901234567890123456789012123456789012345678901 2
12345678901234567890123456789012123456789012345678901 2
12345678901234567890123456789012123456789012345678901
Latin verus 5 true. Also found in English verisimilitude, veritable, 2
2345678901234567890123456789012123456789012345678901
12345678901234567890123456789012123456789012345678901
Word Origin 2
12345678901234567890123456789012123456789012345678901
and verity.
2
12345678901234567890123456789012123456789012345678901
veritable (adjective) Authentic. A French antiques dealer 2
2
12345678901234567890123456789012123456789012345678901 2
1
12345678901234567890123456789012123456789012345678901
recently claimed that a fifteenth-century child-sized suit 2
12345678901234567890123456789012123456789012345678901 2
12345678901234567890123456789012123456789012345678901
of armor that he purchased in 1994 is the veritable suit 2
12345678901234567890123456789012123456789012345678901 2
12345678901234567890123456789012123456789012345678901
of armor worn by heroine Joan of Arc.
2345678901234567890123456789012123456789012345678901
2
2
1
12345678901234567890123456789012123456789012345678901 2
12345678901234567890123456789012123456789012345678901 2 2
224 12345678901234567890123456789012123456789012345678901
123456789012345678901234567890121234567890123456789012

www.petersons.com
Chapter 5: Verbal Review

123456789012345678901234567890121234567890123456789012
12345678901234567890123456789012123456789012345678901 2
12345678901234567890123456789012123456789012345678901 2
12345678901234567890123456789012123456789012345678901
vindictive (adjective) Spiteful. Paula embarked on a string 2
12345678901234567890123456789012123456789012345678901 2
12345678901234567890123456789012123456789012345678901
of petty, vindictive acts against her philandering boy- 2
12345678901234567890123456789012123456789012345678901 2
12345678901234567890123456789012123456789012345678901
friend, such as mixing dry cat food with his cereal and 2
2345678901234567890123456789012123456789012345678901 2
1
12345678901234567890123456789012123456789012345678901
snipping the blooms off his prize African violets. 2
12345678901234567890123456789012123456789012345678901 2
12345678901234567890123456789012123456789012345678901 2
12345678901234567890123456789012123456789012345678901
viscid (adjective) Sticky. The 3M
2345678901234567890123456789012123456789012345678901
company’s “Post-It,” a 2
1 simple piece of paper with one viscid side, has become as 2
12345678901234567890123456789012123456789012345678901 2
12345678901234567890123456789012123456789012345678901 2
12345678901234567890123456789012123456789012345678901
commonplace—and as indispensable—as the paper clip. 2
12345678901234567890123456789012123456789012345678901
2345678901234567890123456789012123456789012345678901
2
1 viscous (adjective) Having a gelatinous or gooey quality. I 2
12345678901234567890123456789012123456789012345678901 2
12345678901234567890123456789012123456789012345678901
put too much liquid in the batter, and so my Black Forest 2
12345678901234567890123456789012123456789012345678901 2
12345678901234567890123456789012123456789012345678901
cake turned out to be a viscous, inedible mass.
2345678901234567890123456789012123456789012345678901
2
2
1
12345678901234567890123456789012123456789012345678901 2
12345678901234567890123456789012123456789012345678901
vitiate (verb) To pollute, to impair. When they voted to ban 2
12345678901234567890123456789012123456789012345678901 2
12345678901234567890123456789012123456789012345678901
smoking from all bars in California, the public affirmed 2
2345678901234567890123456789012123456789012345678901
12345678901234567890123456789012123456789012345678901
their belief that smoking vitiates the health of all people, 2
2
12345678901234567890123456789012123456789012345678901

Take It to the Next Level


12345678901234567890123456789012123456789012345678901 2
12345678901234567890123456789012123456789012345678901
not just smokers. 2
12345678901234567890123456789012123456789012345678901 22
12345678901234567890123456789012123456789012345678901
vituperative (adjective) Verbally abusive, insulting. Eliza- 2
1
12345678901234567890123456789012123456789012345678901
beth Taylor won an award for her harrowing portrayal 2
12345678901234567890123456789012123456789012345678901 2
12345678901234567890123456789012123456789012345678901 2
12345678901234567890123456789012123456789012345678901 22
2345678901234567890123456789012123456789012345678901
of Martha, the bitter, vituperative wife of a college
12345678901234567890123456789012123456789012345678901
professor in Edward Albee’s Who’s Afraid of Virginia 2
12345678901234567890123456789012123456789012345678901
12345678901234567890123456789012123456789012345678901
Woolf? vituperate (verb). 2
12345678901234567890123456789012123456789012345678901 2
12345678901234567890123456789012123456789012345678901 22
12345678901234567890123456789012123456789012345678901
volatile (adjective) Quickly changing; fleeting, transitory;
12345678901234567890123456789012123456789012345678901
prone to violence. Public opinion is notoriously volatile; 2
2
12345678901234567890123456789012123456789012345678901 2
1
12345678901234567890123456789012123456789012345678901
a politician who is very popular one month may be 2
12345678901234567890123456789012123456789012345678901 2
12345678901234567890123456789012123456789012345678901
voted out of office the next. volatility (noun). 2
12345678901234567890123456789012123456789012345678901 2
12345678901234567890123456789012123456789012345678901
volubility (noun) Quality of being overly talkative, glib. As 2
12345678901234567890123456789012123456789012345678901 2
12345678901234567890123456789012123456789012345678901 2
12345678901234567890123456789012123456789012345678901 2
Lorraine’s anxiety increased, her volubility increased in
12345678901234567890123456789012123456789012345678901
direct proportion, so during her job interview the poor 2
12345678901234567890123456789012123456789012345678901 2
2345678901234567890123456789012123456789012345678901
interviewer couldn’t get a word in edgewise. voluble 2
12345678901234567890123456789012123456789012345678901
12345678901234567890123456789012123456789012345678901 2
1 2
12345678901234567890123456789012123456789012345678901 2
(adjective).
12345678901234567890123456789012123456789012345678901 2
12345678901234567890123456789012123456789012345678901 2
12345678901234567890123456789012123456789012345678901 2
12345678901234567890123456789012123456789012345678901
Latin vorare 5 to eat. Also found in English carnivorous, devour, 2
12345678901234567890123456789012123456789012345678901
Word Origin 2
12345678901234567890123456789012123456789012345678901 2
and omnivorous.
2345678901234567890123456789012123456789012345678901
12345678901234567890123456789012123456789012345678901
voracious (adjective) Gluttonous, ravenous. “Are all your 2
2
12345678901234567890123456789012123456789012345678901 2
12345678901234567890123456789012123456789012345678901
appetites so voracious?” Wesley asked Nina as he 2
12345678901234567890123456789012123456789012345678901 2
1
12345678901234567890123456789012123456789012345678901
watched her finish off seven miniature sandwiches and 2
12345678901234567890123456789012123456789012345678901 2
12345678901234567890123456789012123456789012345678901
two lamb kabob skewers in a matter of minutes. 2
12345678901234567890123456789012123456789012345678901 2
12345678901234567890123456789012123456789012345678901
voracity (noun). 2
12345678901234567890123456789012123456789012345678901 22
2345678901234567890123456789012123456789012345678901
1
12345678901234567890123456789012123456789012345678901
wag (noun) Wit, joker. Tom was getting tired of his role as 2
12345678901234567890123456789012123456789012345678901
the comical wag who injected life into Kathy’s otherwise 2
12345678901234567890123456789012123456789012345678901
2345678901234567890123456789012123456789012345678901
2
2
1
12345678901234567890123456789012123456789012345678901
tedious parties. waggish (adjective). 2
12345678901234567890123456789012123456789012345678901 22
12345678901234567890123456789012123456789012345678901 225
123456789012345678901234567890121234567890123456789012
Part III: Section Review

123456789012345678901234567890121234567890123456789012
12345678901234567890123456789012123456789012345678901 2
12345678901234567890123456789012123456789012345678901 2
12345678901234567890123456789012123456789012345678901
whimsical (adjective) Based on a capricious, carefree, or 2
12345678901234567890123456789012123456789012345678901 2
12345678901234567890123456789012123456789012345678901
sudden impulse or idea; fanciful, playful. Dave Barry’s 2
12345678901234567890123456789012123456789012345678901 2
12345678901234567890123456789012123456789012345678901
Book of Bad Songs is filled with the kind of goofy jokes 2
2345678901234567890123456789012123456789012345678901 2
1
12345678901234567890123456789012123456789012345678901
that are typical of his whimsical sense of humor. whim 2
12345678901234567890123456789012123456789012345678901 2
12345678901234567890123456789012123456789012345678901
(noun). 2
12345678901234567890123456789012123456789012345678901
2345678901234567890123456789012123456789012345678901
2
1 xenophobia (noun) Fear of foreigners or outsiders. Slobodan 2
12345678901234567890123456789012123456789012345678901 2
12345678901234567890123456789012123456789012345678901 2
12345678901234567890123456789012123456789012345678901
Milosevic’s nationalistic talk played on the deep xeno- 2
12345678901234567890123456789012123456789012345678901
2345678901234567890123456789012123456789012345678901
2
2
1 phobia of the Serbs, who after 500 years of brutal
12345678901234567890123456789012123456789012345678901 2
12345678901234567890123456789012123456789012345678901
Ottoman occupation had come to distrust all outsiders. 2
12345678901234567890123456789012123456789012345678901 2
12345678901234567890123456789012123456789012345678901
zenith (noun) Highest point. Compiling the vocabulary list 2
2345678901234567890123456789012123456789012345678901 2
1
12345678901234567890123456789012123456789012345678901
for the Insider’s Guide to the GRE was the zenith of my 2
12345678901234567890123456789012123456789012345678901 2
12345678901234567890123456789012123456789012345678901
literary career: after this, there was nowhere to go but 2
12345678901234567890123456789012123456789012345678901 2
12345678901234567890123456789012123456789012345678901
downhill. 2
12345678901234567890123456789012123456789012345678901 2
12345678901234567890123456789012123456789012345678901 2
12345678901234567890123456789012123456789012345678901 2
12345678901234567890123456789012123456789012345678901 2
12345678901234567890123456789012123456789012345678901 2
2345678901234567890123456789012123456789012345678901
2
1
12345678901234567890123456789012123456789012345678901 2
12345678901234567890123456789012123456789012345678901 2
12345678901234567890123456789012123456789012345678901 2
12345678901234567890123456789012123456789012345678901 2
2345678901234567890123456789012123456789012345678901
12345678901234567890123456789012123456789012345678901 2
12345678901234567890123456789012123456789012345678901 2
12345678901234567890123456789012123456789012345678901 2
12345678901234567890123456789012123456789012345678901 2
12345678901234567890123456789012123456789012345678901 2
12345678901234567890123456789012123456789012345678901 2
12345678901234567890123456789012123456789012345678901 2
12345678901234567890123456789012123456789012345678901 2 2
1
12345678901234567890123456789012123456789012345678901 2
12345678901234567890123456789012123456789012345678901 2
12345678901234567890123456789012123456789012345678901 2
12345678901234567890123456789012123456789012345678901 2
12345678901234567890123456789012123456789012345678901 2
12345678901234567890123456789012123456789012345678901 2
12345678901234567890123456789012123456789012345678901 2
12345678901234567890123456789012123456789012345678901 2
12345678901234567890123456789012123456789012345678901 2
12345678901234567890123456789012123456789012345678901 2
12345678901234567890123456789012123456789012345678901 2
2345678901234567890123456789012123456789012345678901
12345678901234567890123456789012123456789012345678901 2 2
1
12345678901234567890123456789012123456789012345678901 2
12345678901234567890123456789012123456789012345678901 2
12345678901234567890123456789012123456789012345678901 2
12345678901234567890123456789012123456789012345678901 2
12345678901234567890123456789012123456789012345678901 2
12345678901234567890123456789012123456789012345678901 2
12345678901234567890123456789012123456789012345678901 2
12345678901234567890123456789012123456789012345678901 2
2345678901234567890123456789012123456789012345678901
12345678901234567890123456789012123456789012345678901 2
12345678901234567890123456789012123456789012345678901 2
12345678901234567890123456789012123456789012345678901 2 2
1
12345678901234567890123456789012123456789012345678901 2
12345678901234567890123456789012123456789012345678901 2
12345678901234567890123456789012123456789012345678901 2
12345678901234567890123456789012123456789012345678901 2
12345678901234567890123456789012123456789012345678901 2
12345678901234567890123456789012123456789012345678901 2
2345678901234567890123456789012123456789012345678901
2
1
12345678901234567890123456789012123456789012345678901 2
12345678901234567890123456789012123456789012345678901 2
12345678901234567890123456789012123456789012345678901 2
12345678901234567890123456789012123456789012345678901 2
12345678901234567890123456789012123456789012345678901 2
12345678901234567890123456789012123456789012345678901 2 2
226 12345678901234567890123456789012123456789012345678901
123456789012345678901234567890121234567890123456789012

www.petersons.com
Chapter

6
Analytical Writing Review
123456789012345678901234567890121234567890123456789012
12345678901234567890123456789012123456789012345678901 2
12345678901234567890123456789012123456789012345678901 2
12345678901234567890123456789012123456789012345678901
Analytical 101: Fundamental Essay Writing Skills 2
12345678901234567890123456789012123456789012345678901 22
2345678901234567890123456789012123456789012345678901
12345678901234567890123456789012123456789012345678901
The two-essay Analytical section is the first section you encounter after 2
12345678901234567890123456789012123456789012345678901
surviving the interminable form-filling-out pre-GRE shenanigans. It does 2
12345678901234567890123456789012123456789012345678901
12345678901234567890123456789012123456789012345678901 2
12345678901234567890123456789012123456789012345678901 2
1 not have much in common with its Math and Verbal siblings. The 2
12345678901234567890123456789012123456789012345678901
Analytical section is scored differently, structured differently, and takes 2
12345678901234567890123456789012123456789012345678901 2
12345678901234567890123456789012123456789012345678901
longer to complete than both other sections combined. This last point can 2
12345678901234567890123456789012123456789012345678901 22
2345678901234567890123456789012123456789012345678901
12345678901234567890123456789012123456789012345678901
be somewhat frustrating if your Analytical score is not relevant to your
12345678901234567890123456789012123456789012345678901 2
particular graduate program, since you will have to spend the first 75 2
12345678901234567890123456789012123456789012345678901
12345678901234567890123456789012123456789012345678901 22
12345678901234567890123456789012123456789012345678901
minutes of the test mucking about and wasting valuable brainpower units
12345678901234567890123456789012123456789012345678901
in a section that’s not important to you. If this is the case, conserve your 2
2
12345678901234567890123456789012123456789012345678901
12345678901234567890123456789012123456789012345678901 2
1 energy for the sections that matter, but don’t blow off the section either. 2
12345678901234567890123456789012123456789012345678901 22
2345678901234567890123456789012123456789012345678901
12345678901234567890123456789012123456789012345678901 2
1
12345678901234567890123456789012123456789012345678901 2
Note

12345678901234567890123456789012123456789012345678901
If you are an international student or if English is your second language, 2
12345678901234567890123456789012123456789012345678901 2
12345678901234567890123456789012123456789012345678901
your Analytical score might be scrutinized by universities to determine 2
12345678901234567890123456789012123456789012345678901 2
12345678901234567890123456789012123456789012345678901
how fluent you are in English. 2
12345678901234567890123456789012123456789012345678901 2
12345678901234567890123456789012123456789012345678901 22
2345678901234567890123456789012123456789012345678901
12345678901234567890123456789012123456789012345678901 2
1
12345678901234567890123456789012123456789012345678901
Immediately after you complete your GRE CAT, your Quantitative and 2
12345678901234567890123456789012123456789012345678901 2
2345678901234567890123456789012123456789012345678901
Verbal scores appear on the screen. The Analytical score takes two to six 2
12345678901234567890123456789012123456789012345678901
12345678901234567890123456789012123456789012345678901
weeks longer to get, since your essays must be sent off to “GRE-Land,” 2
2
12345678901234567890123456789012123456789012345678901 2
12345678901234567890123456789012123456789012345678901
1 where they are read and graded by professional readers. These readers 2
2345678901234567890123456789012123456789012345678901
12345678901234567890123456789012123456789012345678901
might be English graduate students, former English teachers, or educa- 2
2
12345678901234567890123456789012123456789012345678901 2
12345678901234567890123456789012123456789012345678901
tional professionals of one ilk or another. Whatever their background, the 2
12345678901234567890123456789012123456789012345678901 2
1 professional readers share some traits in common:
12345678901234567890123456789012123456789012345678901 2
12345678901234567890123456789012123456789012345678901 2
12345678901234567890123456789012123456789012345678901 2
12345678901234567890123456789012123456789012345678901
1. They are experts in the essay-writing format and are well- 2
12345678901234567890123456789012123456789012345678901 2
12345678901234567890123456789012123456789012345678901 22
2345678901234567890123456789012123456789012345678901
qualified to make judgments about the effectiveness of a
1
12345678901234567890123456789012123456789012345678901
particular essay. 2
12345678901234567890123456789012123456789012345678901 2
12345678901234567890123456789012123456789012345678901
2. They use a shared rubric to evaluate both essays and decide upon 2
12345678901234567890123456789012123456789012345678901 2
12345678901234567890123456789012123456789012345678901
a single score for each test-taker. 2
12345678901234567890123456789012123456789012345678901 22
12345678901234567890123456789012123456789012345678901
123456789012345678901234567890121234567890123456789012
227
Part III: Section Review

123456789012345678901234567890121234567890123456789012
12345678901234567890123456789012123456789012345678901 2
12345678901234567890123456789012123456789012345678901 2
12345678901234567890123456789012123456789012345678901
3. All the ones who give out the lowest scores are members of Mrs. 2
12345678901234567890123456789012123456789012345678901 2
12345678901234567890123456789012123456789012345678901
Smith’s 8th grade class at Lawrence Middle School. 2
12345678901234567890123456789012123456789012345678901 2
12345678901234567890123456789012123456789012345678901 2
12345678901234567890123456789012123456789012345678901 2
12345678901234567890123456789012123456789012345678901
The ugly-sounding term rubric often rears its head when the topic of essay 2
12345678901234567890123456789012123456789012345678901 2
12345678901234567890123456789012123456789012345678901
grading comes up. A rubric is little more than a set of rules, or standards, 2
12345678901234567890123456789012123456789012345678901 2
12345678901234567890123456789012123456789012345678901
regarding a topic like essay-writing. Since essay-grading is inherently 2
12345678901234567890123456789012123456789012345678901 2
12345678901234567890123456789012123456789012345678901
subjective, a rubric helps codify certain general points that every good 2
12345678901234567890123456789012123456789012345678901 2

Tip
12345678901234567890123456789012123456789012345678901
essay should have. Standards like “Every essay should have a clear, 2
1 well-articulated point of view” and “A well-written essay will contain a 2
2345678901234567890123456789012123456789012345678901
12345678901234567890123456789012123456789012345678901 2
12345678901234567890123456789012123456789012345678901 2
12345678901234567890123456789012123456789012345678901
series of concise supporting details that back up the argument” are two 2
12345678901234567890123456789012123456789012345678901
2345678901234567890123456789012123456789012345678901
2
2
1 examples that usually appear on an essay-writing rubric. Although a
12345678901234567890123456789012123456789012345678901 2
12345678901234567890123456789012123456789012345678901
rubric helps professional readers standardize the essay-grading process, 2
12345678901234567890123456789012123456789012345678901 2
12345678901234567890123456789012123456789012345678901
there is still room for interpretation and differences of opinion. 2
12345678901234567890123456789012123456789012345678901 2
2345678901234567890123456789012123456789012345678901
12345678901234567890123456789012123456789012345678901 2
The third point might not be factually accurate (it’s more of an outright lie), 2
12345678901234567890123456789012123456789012345678901
12345678901234567890123456789012123456789012345678901 2
12345678901234567890123456789012123456789012345678901 2
12345678901234567890123456789012123456789012345678901 2
but the other two points are true. Both essays are read, evaluated, and given
1 a score from 0 to 6 by two different readers. A 6 is the highest score, while a 2
12345678901234567890123456789012123456789012345678901 2
12345678901234567890123456789012123456789012345678901 2
0 means you didn’t make much sense or use
12345678901234567890123456789012123456789012345678901 real words. You can get half- 2
2345678901234567890123456789012123456789012345678901
point scores if one reader gives you a 4 while another gives you a 5; if that’s 2
12345678901234567890123456789012123456789012345678901
12345678901234567890123456789012123456789012345678901 2
the case, you end up with a 4.5 on that test. If two readers disagree by a lot, 2
12345678901234567890123456789012123456789012345678901
12345678901234567890123456789012123456789012345678901 2
12345678901234567890123456789012123456789012345678901
then a third reader is brought in to help resolve the disagreement. 2
12345678901234567890123456789012123456789012345678901 2
12345678901234567890123456789012123456789012345678901
A 4 or 5 are both good scores, a 3’s a bit dodgy, and a 1 or 2 show you have 2
2
12345678901234567890123456789012123456789012345678901
12345678901234567890123456789012123456789012345678901 2
1 some deficiencies in the essay-writing arena. The goal of this section is to 2
2345678901234567890123456789012123456789012345678901
give you the knowledge needed to score at least a 4 on the essay. A 5 or 6 is 2
12345678901234567890123456789012123456789012345678901
12345678901234567890123456789012123456789012345678901 2
1 also a possibility, but the primary goal is to keep your score above a 3. 2
12345678901234567890123456789012123456789012345678901 2
12345678901234567890123456789012123456789012345678901 2
12345678901234567890123456789012123456789012345678901
Level 2 Analytical, which begins on page 245, focuses on scoring a 6. 2
12345678901234567890123456789012123456789012345678901 2
12345678901234567890123456789012123456789012345678901 2
12345678901234567890123456789012123456789012345678901 2
12345678901234567890123456789012123456789012345678901
The Issue Essay 2
12345678901234567890123456789012123456789012345678901 2
2345678901234567890123456789012123456789012345678901
12345678901234567890123456789012123456789012345678901 2
1 For an idea of what the issue will be, look no further than the official GRE 2
12345678901234567890123456789012123456789012345678901
Web site, www.gre.org. Every possible issue topic is listed there (the 2
12345678901234567890123456789012123456789012345678901 2
2345678901234567890123456789012123456789012345678901
current link is www.gre.org/issuetop.html, but this might change if the site 2
12345678901234567890123456789012123456789012345678901
12345678901234567890123456789012123456789012345678901 2
12345678901234567890123456789012123456789012345678901 2
12345678901234567890123456789012123456789012345678901 2
gets redesigned). There are more than 200 topics listed, so writing sample
1 essays for each one would not be the best use of your time. Then again, it 2
12345678901234567890123456789012123456789012345678901 2
12345678901234567890123456789012123456789012345678901 2
12345678901234567890123456789012123456789012345678901
wouldn’t hurt to practice writing a couple of sample essays from this list if 2
12345678901234567890123456789012123456789012345678901 2
12345678901234567890123456789012123456789012345678901 2
you feel your essay-writing skills need work.
12345678901234567890123456789012123456789012345678901 2
2345678901234567890123456789012123456789012345678901
Looking over the list, you will see that most topics are broad statements 2
12345678901234567890123456789012123456789012345678901
12345678901234567890123456789012123456789012345678901 2
that leave themselves open for interpretation. A statement like, “A nation 2
12345678901234567890123456789012123456789012345678901
1 2
12345678901234567890123456789012123456789012345678901
should be ruled by its telemarketers” is the kind of open-ended topic that 2
12345678901234567890123456789012123456789012345678901 2
12345678901234567890123456789012123456789012345678901
just dares you to agree or disagree with it. 2
12345678901234567890123456789012123456789012345678901 2
12345678901234567890123456789012123456789012345678901 2
12345678901234567890123456789012123456789012345678901 2
12345678901234567890123456789012123456789012345678901 2
12345678901234567890123456789012123456789012345678901 2 2
228 12345678901234567890123456789012123456789012345678901
123456789012345678901234567890121234567890123456789012

www.petersons.com
Chapter 6: Analytical Writing Review

123456789012345678901234567890121234567890123456789012
12345678901234567890123456789012123456789012345678901 2
12345678901234567890123456789012123456789012345678901 2
12345678901234567890123456789012123456789012345678901
The first essay begins by presenting you with two issues, not just one. You 2
12345678901234567890123456789012123456789012345678901 2
12345678901234567890123456789012123456789012345678901
get to choose which issue you would like to gab about. You might get two 2
12345678901234567890123456789012123456789012345678901 2
12345678901234567890123456789012123456789012345678901
topics similar to the ones below:
2345678901234567890123456789012123456789012345678901
2
2
1
12345678901234567890123456789012123456789012345678901 2
12345678901234567890123456789012123456789012345678901
1. The creation of a theater company, however small, increases the 2
12345678901234567890123456789012123456789012345678901 2
12345678901234567890123456789012123456789012345678901
attractiveness of a city to visitors.
2345678901234567890123456789012123456789012345678901
The theater company demon- 2
1 strates that a city is committed to artistic endeavors, and such a 2
12345678901234567890123456789012123456789012345678901 2
12345678901234567890123456789012123456789012345678901 2
12345678901234567890123456789012123456789012345678901
commitment is usually associated with good governance. 2
12345678901234567890123456789012123456789012345678901
2345678901234567890123456789012123456789012345678901
2
2
1 2. Quick decisions lead to success in today’s business world.
12345678901234567890123456789012123456789012345678901 2
12345678901234567890123456789012123456789012345678901 2
12345678901234567890123456789012123456789012345678901 2
12345678901234567890123456789012123456789012345678901
2345678901234567890123456789012123456789012345678901
2
2
1 Since the idea of being sued is always unappealing, the two sample topics
12345678901234567890123456789012123456789012345678901 2
Alert!

listed here are NOT from the official list.


12345678901234567890123456789012123456789012345678901 They’re not even close to any of 2
12345678901234567890123456789012123456789012345678901
the topics listed there, thank goodness. Rest assured that the methods and 2
12345678901234567890123456789012123456789012345678901 2
2345678901234567890123456789012123456789012345678901
12345678901234567890123456789012123456789012345678901
strategies discussed throughout the section will work on all the topics 2
12345678901234567890123456789012123456789012345678901 2
12345678901234567890123456789012123456789012345678901
from the official list, even though these two sample topics are 2
12345678901234567890123456789012123456789012345678901 22
12345678901234567890123456789012123456789012345678901
NOT—N.O.T.—from the official list.
12345678901234567890123456789012123456789012345678901 2
1 2
12345678901234567890123456789012123456789012345678901 2
12345678901234567890123456789012123456789012345678901
It’s nice to have a choice of two topics to write about, but don’t waste too 2
12345678901234567890123456789012123456789012345678901 2
12345678901234567890123456789012123456789012345678901
much time deciding which one you want to discuss. The clock starts 2
12345678901234567890123456789012123456789012345678901 2
12345678901234567890123456789012123456789012345678901
counting down once the topics appear, so you want to decide as quickly as 2
12345678901234567890123456789012123456789012345678901 2
12345678901234567890123456789012123456789012345678901
possible. Every second you spend thinking about the topic you don’t 2
2345678901234567890123456789012123456789012345678901
12345678901234567890123456789012123456789012345678901
choose is time completely wasted. You want to maximize the amount of 2
2
12345678901234567890123456789012123456789012345678901
time you have for the topic you do choose, so read the two choices and 2
12345678901234567890123456789012123456789012345678901
12345678901234567890123456789012123456789012345678901 2
1 then ask yourself, “Which topic can I write about more?” Getting words 2
12345678901234567890123456789012123456789012345678901 22
2345678901234567890123456789012123456789012345678901
12345678901234567890123456789012123456789012345678901
1 onto the screen is key, so make a decision about which topic you feel more 2
12345678901234567890123456789012123456789012345678901
loquacious about. If you’re the kind of person who reads the Wall Street 2
12345678901234567890123456789012123456789012345678901
2345678901234567890123456789012123456789012345678901
2
Journal, choose the second topic. If you’re someone who flips through the 2
12345678901234567890123456789012123456789012345678901
12345678901234567890123456789012123456789012345678901 2
1 Wall Street Journal searching for comics and movie reviews, the first topic 2
12345678901234567890123456789012123456789012345678901 2
12345678901234567890123456789012123456789012345678901 2
12345678901234567890123456789012123456789012345678901 22
2345678901234567890123456789012123456789012345678901
is probably better for you.
12345678901234567890123456789012123456789012345678901 2
1 Once you pick a topic, you need to decide whether you want to agree or
12345678901234567890123456789012123456789012345678901 2
12345678901234567890123456789012123456789012345678901
disagree with the initial statement. Each topic sentence presents one side of 2
12345678901234567890123456789012123456789012345678901 2
12345678901234567890123456789012123456789012345678901
an issue, and it’s up to you to “present your perspective on an issue” 2
12345678901234567890123456789012123456789012345678901 2
12345678901234567890123456789012123456789012345678901
(that’s the official moniker for this part of the Analytical section). It 2
12345678901234567890123456789012123456789012345678901 2
12345678901234567890123456789012123456789012345678901
doesn’t matter whether you choose to agree or disagree, since there’s no 2
12345678901234567890123456789012123456789012345678901 2
12345678901234567890123456789012123456789012345678901
single “right” answer to these topics. All that matters is how well you 2
12345678901234567890123456789012123456789012345678901 2
12345678901234567890123456789012123456789012345678901
explain your side of the argument with supporting details arranged in a 2
2345678901234567890123456789012123456789012345678901
12345678901234567890123456789012123456789012345678901 2
12345678901234567890123456789012123456789012345678901
logical manner. 2
12345678901234567890123456789012123456789012345678901 22
12345678901234567890123456789012123456789012345678901
1 Let’s say you choose the first topic sentence, and feeling cantankerous you 2
12345678901234567890123456789012123456789012345678901
decide to disagree with the opinion that theater companies increase the 2
12345678901234567890123456789012123456789012345678901 2
12345678901234567890123456789012123456789012345678901 2
12345678901234567890123456789012123456789012345678901 2
attractiveness of a city. With about 43 minutes remaining, take about 5
12345678901234567890123456789012123456789012345678901
minutes to brainstorm some ideas that will help support this viewpoint. Jot 2
12345678901234567890123456789012123456789012345678901 2
12345678901234567890123456789012123456789012345678901
these points down on scratch paper, and try to decide where you want to 2
12345678901234567890123456789012123456789012345678901 2
12345678901234567890123456789012123456789012345678901 2 229
123456789012345678901234567890121234567890123456789012
Part III: Section Review

123456789012345678901234567890121234567890123456789012
12345678901234567890123456789012123456789012345678901 2
12345678901234567890123456789012123456789012345678901 2
12345678901234567890123456789012123456789012345678901
place them in the essay. There is a fine balance between spending time 2
12345678901234567890123456789012123456789012345678901 2
12345678901234567890123456789012123456789012345678901
formulating your essay outline and getting down to actually writing it. The 2
12345678901234567890123456789012123456789012345678901 2
12345678901234567890123456789012123456789012345678901
more planning you do, the better your essay will appear in terms of 2
1 organization. Countering this attribute is the fact that a well-planned essay 2
2345678901234567890123456789012123456789012345678901
12345678901234567890123456789012123456789012345678901 2
12345678901234567890123456789012123456789012345678901 2
12345678901234567890123456789012123456789012345678901
of only two paragraphs is not as good as a four-paragraph essay with some 2
12345678901234567890123456789012123456789012345678901
2345678901234567890123456789012123456789012345678901
2
2
1 continuity problems. Five minutes is a good amount of time to plan. After
12345678901234567890123456789012123456789012345678901 2
12345678901234567890123456789012123456789012345678901
that, start clacking those keys. 2
12345678901234567890123456789012123456789012345678901 2
12345678901234567890123456789012123456789012345678901 2
1 As stated on page 28, your typing speed is a factor in the Analytical 2
2345678901234567890123456789012123456789012345678901
12345678901234567890123456789012123456789012345678901 2
12345678901234567890123456789012123456789012345678901 2
12345678901234567890123456789012123456789012345678901
section, since getting words into the computer can be a time-consuming 2
12345678901234567890123456789012123456789012345678901
2345678901234567890123456789012123456789012345678901
2
2
1 process. Take the test on page 28 to see how many words you should be
12345678901234567890123456789012123456789012345678901 2
12345678901234567890123456789012123456789012345678901
able to type for the first essay. If you are a speedy typist, you can give 2
12345678901234567890123456789012123456789012345678901 2

X-Ref
12345678901234567890123456789012123456789012345678901
yourself a little more time to brainstorm ideas and organize your 2
2345678901234567890123456789012123456789012345678901
thoughts. If you are an average typist, shoot for a four- to five-paragraph 2
12345678901234567890123456789012123456789012345678901
12345678901234567890123456789012123456789012345678901 2
12345678901234567890123456789012123456789012345678901 2
12345678901234567890123456789012123456789012345678901 2
essay. If you are a very slow, hunt-and-peck, are-these-letters-moving-
around-on-me? kind of typist, you’ll need to improve your speed. You 2
12345678901234567890123456789012123456789012345678901
12345678901234567890123456789012123456789012345678901 2
1 want to be able to write at least 400 words for the first essay. Start 2
12345678901234567890123456789012123456789012345678901 2
12345678901234567890123456789012123456789012345678901
keeping a journal on your computer, write letters to your friends, type out 2
12345678901234567890123456789012123456789012345678901 2
12345678901234567890123456789012123456789012345678901
a manifesto, but do something to increase your speed so that your slow 2
12345678901234567890123456789012123456789012345678901 2
12345678901234567890123456789012123456789012345678901
typing no longer becomes an issue that can affect your score. 2
12345678901234567890123456789012123456789012345678901 2
12345678901234567890123456789012123456789012345678901 2
12345678901234567890123456789012123456789012345678901 2
2345678901234567890123456789012123456789012345678901
Take the time now and jot down some ideas about how theaters do not 2
12345678901234567890123456789012123456789012345678901
12345678901234567890123456789012123456789012345678901
make a city more attractive to visitors. You might have some ideas similar 2
2
12345678901234567890123456789012123456789012345678901
1 to the ones below, but the topic is broad enough that there are many 2
12345678901234567890123456789012123456789012345678901 2
12345678901234567890123456789012123456789012345678901 2
12345678901234567890123456789012123456789012345678901
different ways to craft an essay arguing that this is not true. 2
12345678901234567890123456789012123456789012345678901 2
12345678901234567890123456789012123456789012345678901
1. The introduction of a local theater company has little or 2
12345678901234567890123456789012123456789012345678901 2
12345678901234567890123456789012123456789012345678901
no impact at all on a cityÕs attractiveness to visitors. (Main 2
12345678901234567890123456789012123456789012345678901 2
12345678901234567890123456789012123456789012345678901 2
12345678901234567890123456789012123456789012345678901
idea, first sentence) 2
12345678901234567890123456789012123456789012345678901 2
2345678901234567890123456789012123456789012345678901
1 2. Local theater is not something that most visitors think 2
12345678901234567890123456789012123456789012345678901 2
12345678901234567890123456789012123456789012345678901 2
12345678901234567890123456789012123456789012345678901 2
about when considering a city—pollution, crime, economic
2345678901234567890123456789012123456789012345678901
12345678901234567890123456789012123456789012345678901 2
12345678901234567890123456789012123456789012345678901
vitality all bigger factors. 2
12345678901234567890123456789012123456789012345678901 2
1 3. Tourist attractions can increase a cityÕs attractiveness, 2
12345678901234567890123456789012123456789012345678901 2
2345678901234567890123456789012123456789012345678901
12345678901234567890123456789012123456789012345678901
BUT 2
12345678901234567890123456789012123456789012345678901 2
12345678901234567890123456789012123456789012345678901 2
12345678901234567890123456789012123456789012345678901
—How much of a draw is a ÒlocalÓ theater? 2
1 2
2345678901234567890123456789012123456789012345678901
—Except for NYC Broadway, what place is known for its 2
12345678901234567890123456789012123456789012345678901
12345678901234567890123456789012123456789012345678901 2
12345678901234567890123456789012123456789012345678901 2
12345678901234567890123456789012123456789012345678901 2
plays?
2
1
12345678901234567890123456789012123456789012345678901
—One-of-a-kind natural wonders, or museums of one 2
12345678901234567890123456789012123456789012345678901 2
12345678901234567890123456789012123456789012345678901 2
12345678901234567890123456789012123456789012345678901 2
type of another, these are things that increase a cityÕs
12345678901234567890123456789012123456789012345678901 2
12345678901234567890123456789012123456789012345678901
attractiveness. 2
12345678901234567890123456789012123456789012345678901 2
12345678901234567890123456789012123456789012345678901 2 2
230 12345678901234567890123456789012123456789012345678901
123456789012345678901234567890121234567890123456789012

www.petersons.com
Chapter 6: Analytical Writing Review

123456789012345678901234567890121234567890123456789012
12345678901234567890123456789012123456789012345678901 2
12345678901234567890123456789012123456789012345678901 2
12345678901234567890123456789012123456789012345678901
—In this light, local theater is about as important as a 2
12345678901234567890123456789012123456789012345678901 2
12345678901234567890123456789012123456789012345678901
local grocer. Everyone likes having one, but itÕs not some 2
12345678901234567890123456789012123456789012345678901 2
12345678901234567890123456789012123456789012345678901
kind of tourist draw or city enhancement. 2
12345678901234567890123456789012123456789012345678901 2
12345678901234567890123456789012123456789012345678901
4. Also, some lousy governments have presided over great art, 2
12345678901234567890123456789012123456789012345678901 2
12345678901234567890123456789012123456789012345678901 2
12345678901234567890123456789012123456789012345678901
so connection between governance
2345678901234567890123456789012123456789012345678901
and pursuit of artistic 2
2
1
12345678901234567890123456789012123456789012345678901
excellence pretty flimsy. 2
12345678901234567890123456789012123456789012345678901 2
12345678901234567890123456789012123456789012345678901 2
12345678901234567890123456789012123456789012345678901
The last statement attacks the link connecting artistic endeavor with good 2
1 governance, but the other statements don’t have anything to do with it. 2
2345678901234567890123456789012123456789012345678901
12345678901234567890123456789012123456789012345678901 2
12345678901234567890123456789012123456789012345678901
Statement 4 might make it into the essay, but it’s not crucial that it does. If 2
12345678901234567890123456789012123456789012345678901 2
12345678901234567890123456789012123456789012345678901
this essay were untimed, it would help to mount a comprehensive attack 2
2345678901234567890123456789012123456789012345678901 2
1
12345678901234567890123456789012123456789012345678901
on all salient points that you want to disagree with, but hey! This is Essay 2
12345678901234567890123456789012123456789012345678901 2
12345678901234567890123456789012123456789012345678901
Writing under Pressure, and no one expects you to be able to cover all 2
12345678901234567890123456789012123456789012345678901 2
12345678901234567890123456789012123456789012345678901
bases with the paltry amount of time available. 2
12345678901234567890123456789012123456789012345678901 2
12345678901234567890123456789012123456789012345678901 2
12345678901234567890123456789012123456789012345678901
Statement 1 provides your perspective on the issue, placing you firmly in 2
12345678901234567890123456789012123456789012345678901 2
12345678901234567890123456789012123456789012345678901 22
2345678901234567890123456789012123456789012345678901
the “I’ve listened to the original statement and I disagree” camp. Make
1 sure you very clearly state whether you are for or against the original issue,
12345678901234567890123456789012123456789012345678901 2
12345678901234567890123456789012123456789012345678901 2
12345678901234567890123456789012123456789012345678901
and do so in the first one or two sentences. The readers are not going to 2
2345678901234567890123456789012123456789012345678901
spend four hours on every essay. It’s more accurate to say that they will 2
12345678901234567890123456789012123456789012345678901
12345678901234567890123456789012123456789012345678901 2
12345678901234567890123456789012123456789012345678901 2
12345678901234567890123456789012123456789012345678901 22
spend less than 10 minutes on both essays combined, in which case you
12345678901234567890123456789012123456789012345678901
want to have your position very clearly stated right away, in plain view and 2
12345678901234567890123456789012123456789012345678901
for everyone to see. Mentally, a reader can then place a check alongside the 2
12345678901234567890123456789012123456789012345678901
12345678901234567890123456789012123456789012345678901 2
rubric standard that says, “Every essay should have a clear, well- 2
12345678901234567890123456789012123456789012345678901
1 2
12345678901234567890123456789012123456789012345678901
articulated point of view.” 2
12345678901234567890123456789012123456789012345678901 2
12345678901234567890123456789012123456789012345678901 2
12345678901234567890123456789012123456789012345678901
Your first paragraph could look like something like this: 2
12345678901234567890123456789012123456789012345678901
2345678901234567890123456789012123456789012345678901
2
2
12345678901234567890123456789012123456789012345678901
Although a local theater company can have many positive 2
12345678901234567890123456789012123456789012345678901
1 effects on a community, it is unlikely that such a company will 2
12345678901234567890123456789012123456789012345678901 2
12345678901234567890123456789012123456789012345678901 2
12345678901234567890123456789012123456789012345678901 22
2345678901234567890123456789012123456789012345678901
have any impact on how visitors view that city. There are many
12345678901234567890123456789012123456789012345678901
different ways people consider a city not their own. While there 2
1
12345678901234567890123456789012123456789012345678901
are certainly some people who use theater as part of their 2
12345678901234567890123456789012123456789012345678901 2
2345678901234567890123456789012123456789012345678901
criteria, this number is probably very small. It is much more 2
12345678901234567890123456789012123456789012345678901
12345678901234567890123456789012123456789012345678901 2
12345678901234567890123456789012123456789012345678901 2
12345678901234567890123456789012123456789012345678901 22
likely that most visitors will use other factors to decide
1
12345678901234567890123456789012123456789012345678901
whether or not a city is attractive to them, and the existence 2
12345678901234567890123456789012123456789012345678901 2
12345678901234567890123456789012123456789012345678901
of a local theater will not even be an issue. 2
12345678901234567890123456789012123456789012345678901 2
12345678901234567890123456789012123456789012345678901
One paragraph down, three to go. 2
12345678901234567890123456789012123456789012345678901 22
2345678901234567890123456789012123456789012345678901
12345678901234567890123456789012123456789012345678901
Having stated your position, you must now back it up. The readers will 2
12345678901234567890123456789012123456789012345678901 2
12345678901234567890123456789012123456789012345678901
1 expect your second paragraph to back up your position using specific 2
2345678901234567890123456789012123456789012345678901
12345678901234567890123456789012123456789012345678901 2
1 details, so don’t disappoint them. Flesh out Statement 2, and use precise 2
12345678901234567890123456789012123456789012345678901
examples whenever possible. Cite specific examples that work in your 2
12345678901234567890123456789012123456789012345678901 2
12345678901234567890123456789012123456789012345678901
2345678901234567890123456789012123456789012345678901
2
2
1 favor. The point is to be convincing by using facts to state your point. You
12345678901234567890123456789012123456789012345678901 2
12345678901234567890123456789012123456789012345678901 22
can’t just say, “C’mon! Believe me! I know what I’m talkin’ about!”
12345678901234567890123456789012123456789012345678901 231
123456789012345678901234567890121234567890123456789012
Part III: Section Review

123456789012345678901234567890121234567890123456789012
12345678901234567890123456789012123456789012345678901 2
12345678901234567890123456789012123456789012345678901 2
12345678901234567890123456789012123456789012345678901
Sample Second Paragraph 2
12345678901234567890123456789012123456789012345678901 2
12345678901234567890123456789012123456789012345678901 2
12345678901234567890123456789012123456789012345678901 2
(A) Looking back over the past ten years, many of the Most
12345678901234567890123456789012123456789012345678901
popular cities in the U.S. became popular for reasons that had 2
12345678901234567890123456789012123456789012345678901 2
12345678901234567890123456789012123456789012345678901 2
12345678901234567890123456789012123456789012345678901
nothing to do with theater. (B) Seattle became known as a 2
12345678901234567890123456789012123456789012345678901 2
12345678901234567890123456789012123456789012345678901
place where the trendiest rock-and-roll
2345678901234567890123456789012123456789012345678901
music was created. 2
2
1
12345678901234567890123456789012123456789012345678901
(C) That was good enough for a lot of young people. (D) Silicon 2
12345678901234567890123456789012123456789012345678901 2
12345678901234567890123456789012123456789012345678901
Valley was the center of the Internet explosion, and people 2
12345678901234567890123456789012123456789012345678901
2345678901234567890123456789012123456789012345678901
2
2
1 flocked there in search of great jobs, not great Shakespeare.
12345678901234567890123456789012123456789012345678901 2
12345678901234567890123456789012123456789012345678901
(E) Several older cities revitalized their image with large 2
12345678901234567890123456789012123456789012345678901 2
12345678901234567890123456789012123456789012345678901
public renovations like the Harbor
2345678901234567890123456789012123456789012345678901
project in Baltimore. 2
2
1 (F) BaltimoreÕs harbor area now contains restaurants, malls,
12345678901234567890123456789012123456789012345678901 2
12345678901234567890123456789012123456789012345678901
an aquarium, and various other tourist attractions. (G) No 2
12345678901234567890123456789012123456789012345678901 2
12345678901234567890123456789012123456789012345678901
local theater, though, showing how it was not factor. 2
12345678901234567890123456789012123456789012345678901 2
2345678901234567890123456789012123456789012345678901
12345678901234567890123456789012123456789012345678901 2
12345678901234567890123456789012123456789012345678901 2
Admittedly, this is not the greatest second paragraph the world has ever 2
12345678901234567890123456789012123456789012345678901
12345678901234567890123456789012123456789012345678901 2
12345678901234567890123456789012123456789012345678901 2
1 seen. Sentence A capitalizes the word Most for no reason. Sentences B and 2
12345678901234567890123456789012123456789012345678901
C could probably be combined to form one single good sentence instead of 2
12345678901234567890123456789012123456789012345678901 2
12345678901234567890123456789012123456789012345678901 2
12345678901234567890123456789012123456789012345678901 2
2345678901234567890123456789012123456789012345678901
two choppy sentences. Sentence D is a bit vague on details, with phrases
like Silicon Valley and Internet explosion tossed around with little 2
12345678901234567890123456789012123456789012345678901
12345678901234567890123456789012123456789012345678901 2
12345678901234567890123456789012123456789012345678901
explanation. Sentence E has the word Harbor capitalized incorrectly. 2
12345678901234567890123456789012123456789012345678901 2
12345678901234567890123456789012123456789012345678901 2
Sentences E, F, and G are all culled from the author’s brief visit to 2
12345678901234567890123456789012123456789012345678901
12345678901234567890123456789012123456789012345678901
Baltimore last year, so a bold statement like G is not known to be 2
2
12345678901234567890123456789012123456789012345678901
1 definitively true. They are just the author’s recollections from a brief 2
12345678901234567890123456789012123456789012345678901 2
12345678901234567890123456789012123456789012345678901 2
12345678901234567890123456789012123456789012345678901
three-day visit two years ago that were dredged up from memory to help 2
12345678901234567890123456789012123456789012345678901 2
12345678901234567890123456789012123456789012345678901
bolster a point on the standardized test. Someone could go to Baltimore or 2
2345678901234567890123456789012123456789012345678901
do research in a library to determine whether or not local theater was a 2
12345678901234567890123456789012123456789012345678901
12345678901234567890123456789012123456789012345678901 2
1 factor in the Baltimore harbor renaissance. However, that someone is not 2
12345678901234567890123456789012123456789012345678901 2
12345678901234567890123456789012123456789012345678901 2
12345678901234567890123456789012123456789012345678901 2
2345678901234567890123456789012123456789012345678901
going to be the reader of your essay, since this individual has only a couple
12345678901234567890123456789012123456789012345678901 2
1 of minutes to read through your essays, assign a grade, and then move on 2
12345678901234567890123456789012123456789012345678901
to the next. In other words, while you can’t—and shouldn’t—lie outright, 2
12345678901234567890123456789012123456789012345678901 2
2345678901234567890123456789012123456789012345678901
you can use broad, plausible facts to prove your point. Looking over the 2
12345678901234567890123456789012123456789012345678901
12345678901234567890123456789012123456789012345678901 2
12345678901234567890123456789012123456789012345678901 2
12345678901234567890123456789012123456789012345678901 2
statements made in the second paragraph, you can see that all of them are
1 a nice blend of specificity (naming the actual city) and generalization 2
12345678901234567890123456789012123456789012345678901 2
12345678901234567890123456789012123456789012345678901
(“people flocked there in search of great jobs”). 2
12345678901234567890123456789012123456789012345678901 2
12345678901234567890123456789012123456789012345678901 2
12345678901234567890123456789012123456789012345678901
For all its faults, this second paragraph uses a host of supporting details to 2
2345678901234567890123456789012123456789012345678901
12345678901234567890123456789012123456789012345678901
back up the initial perspective stated in the first paragraph. That is exactly 2
2
12345678901234567890123456789012123456789012345678901 2
12345678901234567890123456789012123456789012345678901
what you want a second paragraph to do in order to get a good score on 2
12345678901234567890123456789012123456789012345678901
1 this essay. If this were an untimed essay, you would be expected to correct 2
12345678901234567890123456789012123456789012345678901 2
12345678901234567890123456789012123456789012345678901 2
12345678901234567890123456789012123456789012345678901
mistakes like improperly capitalized words, and you might also be 2
12345678901234567890123456789012123456789012345678901
expected to provide actual data. For instance, an untimed essay should 2
12345678901234567890123456789012123456789012345678901 2
1 provide actual census figures for the growth rate (and unemployment rate) 2
2345678901234567890123456789012123456789012345678901
12345678901234567890123456789012123456789012345678901 2
12345678901234567890123456789012123456789012345678901 2 2
232 12345678901234567890123456789012123456789012345678901
123456789012345678901234567890121234567890123456789012

www.petersons.com
Chapter 6: Analytical Writing Review

123456789012345678901234567890121234567890123456789012
12345678901234567890123456789012123456789012345678901 2
12345678901234567890123456789012123456789012345678901 2
12345678901234567890123456789012123456789012345678901
in Silicon Valley between 1990 and 2000, and it should probably give a 2
12345678901234567890123456789012123456789012345678901 2
12345678901234567890123456789012123456789012345678901
more precise geographical location than just “Silicon Valley.” 2
12345678901234567890123456789012123456789012345678901 2
12345678901234567890123456789012123456789012345678901
But this isn’t an untimed essay, is it? The people reading this essay don’t 2
12345678901234567890123456789012123456789012345678901 2
12345678901234567890123456789012123456789012345678901 2
12345678901234567890123456789012123456789012345678901
expect exact facts. They are looking for a slew of supporting details that 2
12345678901234567890123456789012123456789012345678901 2
12345678901234567890123456789012123456789012345678901
back up the original position, and that’s just what the second paragraph 2
1 gives them. The paragraph covers three different areas of the country and 2
2345678901234567890123456789012123456789012345678901
12345678901234567890123456789012123456789012345678901 2
12345678901234567890123456789012123456789012345678901 2
12345678901234567890123456789012123456789012345678901
shows how local theater was not a factor in making them attractive to 2
12345678901234567890123456789012123456789012345678901
2345678901234567890123456789012123456789012345678901
2
2
1 visitors.
12345678901234567890123456789012123456789012345678901 2
12345678901234567890123456789012123456789012345678901
With the first paragraph firmly supported by the second, it’s time to 2
12345678901234567890123456789012123456789012345678901 2
12345678901234567890123456789012123456789012345678901
change tack.
2345678901234567890123456789012123456789012345678901
2
2
1
12345678901234567890123456789012123456789012345678901 2
12345678901234567890123456789012123456789012345678901
Your third paragraph should be the “I understand the other side of the 2
12345678901234567890123456789012123456789012345678901 2
12345678901234567890123456789012123456789012345678901
argument, I just don’t agree with it” paragraph. Somewhere in your essay 2
2345678901234567890123456789012123456789012345678901
12345678901234567890123456789012123456789012345678901
the readers want to see that you understand both sides of the issue, and the 2
2
12345678901234567890123456789012123456789012345678901
third paragraph’s as good a place as any. So throw the opposition a bone, 2
12345678901234567890123456789012123456789012345678901
12345678901234567890123456789012123456789012345678901 2
12345678901234567890123456789012123456789012345678901 2
12345678901234567890123456789012123456789012345678901 22
but then show how this little point still does not alter the fact that your
1 initial position is the superior one.
12345678901234567890123456789012123456789012345678901 2
12345678901234567890123456789012123456789012345678901 2
12345678901234567890123456789012123456789012345678901 2
2345678901234567890123456789012123456789012345678901
12345678901234567890123456789012123456789012345678901
Sample Third Paragraph 2
12345678901234567890123456789012123456789012345678901 2
12345678901234567890123456789012123456789012345678901 2
12345678901234567890123456789012123456789012345678901
(H) Of course people do enjoy going to the theater occasion- 2
12345678901234567890123456789012123456789012345678901 2
12345678901234567890123456789012123456789012345678901 22
ally, so having a local theater company would make a city
12345678901234567890123456789012123456789012345678901
12345678901234567890123456789012123456789012345678901
more attractive for some. (I) But who, exactly? (J) You would 2
12345678901234567890123456789012123456789012345678901 2
1 think a ÔlocalÕ theater would benefit the local population the 2
2345678901234567890123456789012123456789012345678901
12345678901234567890123456789012123456789012345678901 2
12345678901234567890123456789012123456789012345678901
most, and that visitors would hardly notice it. (K) Most people 2
1 2
12345678901234567890123456789012123456789012345678901 2
who come in from out of town want to go see large, unique
12345678901234567890123456789012123456789012345678901 2
12345678901234567890123456789012123456789012345678901
tourist attractions like Graceland or an exceptional natural 2
12345678901234567890123456789012123456789012345678901 2
12345678901234567890123456789012123456789012345678901 2
wonder like PikeÕs Peak. (L) The local theater is there to raise
12345678901234567890123456789012123456789012345678901 2
12345678901234567890123456789012123456789012345678901
the artistic consciousness of the community, not attract 2
2345678901234567890123456789012123456789012345678901
visitors from another city who may well have a Local theater 2
12345678901234567890123456789012123456789012345678901
12345678901234567890123456789012123456789012345678901 2
1 2
12345678901234567890123456789012123456789012345678901 2
scene of their own.
12345678901234567890123456789012123456789012345678901 2
12345678901234567890123456789012123456789012345678901 2
12345678901234567890123456789012123456789012345678901 2
12345678901234567890123456789012123456789012345678901
Again, this paragraph is not flawless, nor does it have to be. All it must do 2
12345678901234567890123456789012123456789012345678901
is show that you are a balanced person able to see both sides of an issue, 2
12345678901234567890123456789012123456789012345678901 2
2345678901234567890123456789012123456789012345678901
and that you have reasons for rejecting the opposing side in favor of your 2
12345678901234567890123456789012123456789012345678901
12345678901234567890123456789012123456789012345678901 2
12345678901234567890123456789012123456789012345678901 2
12345678901234567890123456789012123456789012345678901 22
own. Sentence H is the proverbial bone, and the rest of the paragraph does
1 its best to explain why this viewpoint is not as strong as your own. This is
12345678901234567890123456789012123456789012345678901 2
12345678901234567890123456789012123456789012345678901 2
12345678901234567890123456789012123456789012345678901
done by making broad, plausible statements with a dash of specificity if 2
12345678901234567890123456789012123456789012345678901 2
12345678901234567890123456789012123456789012345678901 2
possible. Sentence L works fine as, “Most people who come in from out of
2345678901234567890123456789012123456789012345678901
12345678901234567890123456789012123456789012345678901 2
1 town want to go see large, unique tourist attractions or exceptional 2
12345678901234567890123456789012123456789012345678901
natural wonders.” It works better by including precise examples, since this 2
12345678901234567890123456789012123456789012345678901 2
12345678901234567890123456789012123456789012345678901
helps reinforce the point and it shows the
2345678901234567890123456789012123456789012345678901 reader that you can cite specific 2
2
1
12345678901234567890123456789012123456789012345678901
examples if you have to. Think of it as your way of saying, “If I had time, 2
12345678901234567890123456789012123456789012345678901 2
12345678901234567890123456789012123456789012345678901 2 233
123456789012345678901234567890121234567890123456789012
Part III: Section Review

123456789012345678901234567890121234567890123456789012
12345678901234567890123456789012123456789012345678901 2
12345678901234567890123456789012123456789012345678901 2
12345678901234567890123456789012123456789012345678901
I would bore you with precise details, but I only have time for a few right 2
12345678901234567890123456789012123456789012345678901 2
12345678901234567890123456789012123456789012345678901
now.” 2
12345678901234567890123456789012123456789012345678901 2
12345678901234567890123456789012123456789012345678901
Specific examples like this help mask some other, vaguer claims. For 2
12345678901234567890123456789012123456789012345678901 2
12345678901234567890123456789012123456789012345678901 2
12345678901234567890123456789012123456789012345678901
instance, what exactly does the “artistic consciousness of the community” 2
12345678901234567890123456789012123456789012345678901 2
12345678901234567890123456789012123456789012345678901
in Sentence E mean? The precise meaning is up for grabs, but it sure sounds 2
1 good, doesn’t it? Readers skimming their way through this essay won’t 2
2345678901234567890123456789012123456789012345678901
12345678901234567890123456789012123456789012345678901 2
12345678901234567890123456789012123456789012345678901 2
12345678901234567890123456789012123456789012345678901
pause, place their chin in their hand, and deeply reflect upon the exact 2
12345678901234567890123456789012123456789012345678901
2345678901234567890123456789012123456789012345678901
2
2
1 meaning of that phrase. Instead, they’ll just go “uh-huh” absently and
12345678901234567890123456789012123456789012345678901 2
12345678901234567890123456789012123456789012345678901
keep on blazing through the text. You will get the benefit of the doubt due 2
12345678901234567890123456789012123456789012345678901
to the time constraints of the format. 2
12345678901234567890123456789012123456789012345678901
2345678901234567890123456789012123456789012345678901
2
2
1
12345678901234567890123456789012123456789012345678901
With three paragraphs in the bag, it’s time to check the clock. If you have 2
12345678901234567890123456789012123456789012345678901 2
12345678901234567890123456789012123456789012345678901
less than 5 minutes left, you should hammer out a concluding sentence or 2
12345678901234567890123456789012123456789012345678901 2
12345678901234567890123456789012123456789012345678901
two for paragraph four and then glance over the rest of your essay. Try to 2
12345678901234567890123456789012123456789012345678901 2
12345678901234567890123456789012123456789012345678901
catch any obvious grammatical errors, and try to work on the overall flow 2
12345678901234567890123456789012123456789012345678901 2
12345678901234567890123456789012123456789012345678901
of the essay, making sure that one paragraph flows well into another. You 2
2345678901234567890123456789012123456789012345678901
12345678901234567890123456789012123456789012345678901 2
1 could use this time to combine Sentences B and C in the second paragraph, 2
12345678901234567890123456789012123456789012345678901 2
12345678901234567890123456789012123456789012345678901
for instance. 2
12345678901234567890123456789012123456789012345678901 2
2345678901234567890123456789012123456789012345678901
If you finish three paragraphs and have around 10 minutes left, then you 2
12345678901234567890123456789012123456789012345678901
12345678901234567890123456789012123456789012345678901 2
12345678901234567890123456789012123456789012345678901 2
12345678901234567890123456789012123456789012345678901 2
can use the word-processing format to your advantage in the following
12345678901234567890123456789012123456789012345678901 2
12345678901234567890123456789012123456789012345678901
ways: 2
12345678901234567890123456789012123456789012345678901 2
1. Write a 2- to 3-sentence final paragraph that restates your take on 2
12345678901234567890123456789012123456789012345678901
12345678901234567890123456789012123456789012345678901 2
1 the initial proposition. 2
12345678901234567890123456789012123456789012345678901 2
2345678901234567890123456789012123456789012345678901
12345678901234567890123456789012123456789012345678901
2. Look through everything you’ve written and catch any blatant 2
2
1
12345678901234567890123456789012123456789012345678901 2
12345678901234567890123456789012123456789012345678901
grammatical mistakes. 2
12345678901234567890123456789012123456789012345678901 2
2345678901234567890123456789012123456789012345678901
1 3. Now, try to slip in another paragraph between the third 2
12345678901234567890123456789012123456789012345678901 2
12345678901234567890123456789012123456789012345678901 2
12345678901234567890123456789012123456789012345678901
paragraph and the final one. Your conclusion is already in place, 2
2345678901234567890123456789012123456789012345678901
so you don’t have to worry about ending your essay properly. 2
12345678901234567890123456789012123456789012345678901
12345678901234567890123456789012123456789012345678901 2
1 2
12345678901234567890123456789012123456789012345678901 2
You can use the remaining time to create a fourth paragraph that
12345678901234567890123456789012123456789012345678901
piles on more detail to support your belief. You might attack the 2
12345678901234567890123456789012123456789012345678901 2
12345678901234567890123456789012123456789012345678901
“good governance” angle at this point, which is something you 2
12345678901234567890123456789012123456789012345678901 2
12345678901234567890123456789012123456789012345678901
haven’t really addressed in the rest of your essay. 2
12345678901234567890123456789012123456789012345678901 2
2345678901234567890123456789012123456789012345678901
12345678901234567890123456789012123456789012345678901
4. When the clock hits 1 minute to go, stop work on the fourth 2
2
12345678901234567890123456789012123456789012345678901 2
12345678901234567890123456789012123456789012345678901
paragraph and make sure the transition between it and your final 2
12345678901234567890123456789012123456789012345678901 2
1
12345678901234567890123456789012123456789012345678901
paragraph is smooth. If you only had a chance to write two 2
12345678901234567890123456789012123456789012345678901 2
12345678901234567890123456789012123456789012345678901
sentences in the fourth paragraph, then just tack your two- 2
12345678901234567890123456789012123456789012345678901
sentence conclusion to the end of it. However, if you were able to 2
12345678901234567890123456789012123456789012345678901 2
2345678901234567890123456789012123456789012345678901
write 3–5 sentences in the fourth paragraph, don’t combine the 2
12345678901234567890123456789012123456789012345678901
1 2
12345678901234567890123456789012123456789012345678901 2
12345678901234567890123456789012123456789012345678901 2
final paragraph with the fourth.
12345678901234567890123456789012123456789012345678901 2
12345678901234567890123456789012123456789012345678901 2
12345678901234567890123456789012123456789012345678901 2
12345678901234567890123456789012123456789012345678901 2 2
234 12345678901234567890123456789012123456789012345678901
123456789012345678901234567890121234567890123456789012

www.petersons.com
Chapter 6: Analytical Writing Review

123456789012345678901234567890121234567890123456789012
12345678901234567890123456789012123456789012345678901 2
12345678901234567890123456789012123456789012345678901 2
12345678901234567890123456789012123456789012345678901
If you had to write out your essay, you wouldn’t be able to go back and 2
12345678901234567890123456789012123456789012345678901 2
12345678901234567890123456789012123456789012345678901
insert an entire paragraph into the rest of the text. Yet the computer format 2
12345678901234567890123456789012123456789012345678901 2
12345678901234567890123456789012123456789012345678901
allows you to do that, so you should take advantage of it. Writing the final 2
1 paragraph first fulfills the rubric standard, “restates original position 2
2345678901234567890123456789012123456789012345678901
12345678901234567890123456789012123456789012345678901 2
12345678901234567890123456789012123456789012345678901 2
12345678901234567890123456789012123456789012345678901
clearly,” a basic requirement in essay-writing. You can then use the 2
12345678901234567890123456789012123456789012345678901
2345678901234567890123456789012123456789012345678901
2
2
1 remaining time to expound upon your perspective on the issue. No one
12345678901234567890123456789012123456789012345678901 2
12345678901234567890123456789012123456789012345678901
except you will know that you didn’t write the entire essay out in perfect 2
12345678901234567890123456789012123456789012345678901 2
12345678901234567890123456789012123456789012345678901
linear fashion. All they will see is a well-organized
2345678901234567890123456789012123456789012345678901
five-paragraph essay. 2
2
1
12345678901234567890123456789012123456789012345678901 2
12345678901234567890123456789012123456789012345678901
Sample Last Paragraph to Write First 2
12345678901234567890123456789012123456789012345678901 2
12345678901234567890123456789012123456789012345678901
2345678901234567890123456789012123456789012345678901
2
2
1 Local theater has benefits, but these benefits serve mainly
12345678901234567890123456789012123456789012345678901 2
12345678901234567890123456789012123456789012345678901
the community itself and remain unknown or of no concern to 2
12345678901234567890123456789012123456789012345678901 2
12345678901234567890123456789012123456789012345678901
outsiders. Therefore, the creation of a local theater 2
2345678901234567890123456789012123456789012345678901
12345678901234567890123456789012123456789012345678901 2
12345678901234567890123456789012123456789012345678901
company is unlikely to have any impact on whether or not a 2
12345678901234567890123456789012123456789012345678901 2
12345678901234567890123456789012123456789012345678901 22
city is attractive to outside visitors.
12345678901234567890123456789012123456789012345678901 2
12345678901234567890123456789012123456789012345678901 2
1
12345678901234567890123456789012123456789012345678901
Then, squeeze in a fourth paragraph above that can go something like: 2
12345678901234567890123456789012123456789012345678901 2
12345678901234567890123456789012123456789012345678901 2
2345678901234567890123456789012123456789012345678901
12345678901234567890123456789012123456789012345678901 2
12345678901234567890123456789012123456789012345678901
(M) Furthermore, while the connection between theater and 2
12345678901234567890123456789012123456789012345678901 2
12345678901234567890123456789012123456789012345678901 22
city attractiveness is fairly slim, the connection between art
12345678901234567890123456789012123456789012345678901
12345678901234567890123456789012123456789012345678901
and good government is even more so. (N) A city might provide 2
12345678901234567890123456789012123456789012345678901 2
12345678901234567890123456789012123456789012345678901 22
public funds to help a local theater get started. (O) Although
12345678901234567890123456789012123456789012345678901 2
1 some would like this, others would call it bad government,
12345678901234567890123456789012123456789012345678901
since that was tax money that could be spent elsewhere, like 2
12345678901234567890123456789012123456789012345678901 2
12345678901234567890123456789012123456789012345678901 2
12345678901234567890123456789012123456789012345678901 2
in a public school or to provide more police officers. (P) In this
12345678901234567890123456789012123456789012345678901
light, the connection between theater and government would 2
12345678901234567890123456789012123456789012345678901 2
12345678901234567890123456789012123456789012345678901 2
12345678901234567890123456789012123456789012345678901 2
be considered bad government since public funds are being
12345678901234567890123456789012123456789012345678901 2
12345678901234567890123456789012123456789012345678901
spent poorly. 2
12345678901234567890123456789012123456789012345678901 22
2345678901234567890123456789012123456789012345678901
12345678901234567890123456789012123456789012345678901 2
1 This fourth paragraph has mistakes just like the others before it, but it still
12345678901234567890123456789012123456789012345678901 2
12345678901234567890123456789012123456789012345678901
achieves its purpose, which is to show your ability to approach the issue 2
12345678901234567890123456789012123456789012345678901 2
12345678901234567890123456789012123456789012345678901 2
12345678901234567890123456789012123456789012345678901
from more than one angle. Sentence N sets up a “what-if” hypothesis, and 2
12345678901234567890123456789012123456789012345678901 2
12345678901234567890123456789012123456789012345678901 2
then Sentence O shows how this could be considered poor government.
2345678901234567890123456789012123456789012345678901
Sentence O is a bit unwieldy, but that’s OK, since the phrase “like in a 2
12345678901234567890123456789012123456789012345678901
12345678901234567890123456789012123456789012345678901 2
12345678901234567890123456789012123456789012345678901 2
12345678901234567890123456789012123456789012345678901 22
public school or to provide more police officers” shows that you could be
1 more specific . . . if only you had more time (fake sigh). Since you don’t
12345678901234567890123456789012123456789012345678901 2
12345678901234567890123456789012123456789012345678901
have enough time, the essay has to stay as is. 2
12345678901234567890123456789012123456789012345678901 2
12345678901234567890123456789012123456789012345678901 2
12345678901234567890123456789012123456789012345678901
Grouping all the sample paragraphs together gives you a good blueprint 2
2345678901234567890123456789012123456789012345678901
12345678901234567890123456789012123456789012345678901 2
1 on how to achieve a solid score on the first Analytical essay. You don’t 2
12345678901234567890123456789012123456789012345678901 2
12345678901234567890123456789012123456789012345678901
have to follow this blueprint exactly, but it wouldn’t hurt you if you did. 2
12345678901234567890123456789012123456789012345678901 2
12345678901234567890123456789012123456789012345678901
Whether or not you add any variations, make sure that your first essay 2
12345678901234567890123456789012123456789012345678901 2
12345678901234567890123456789012123456789012345678901
clearly contains the following points: 2
12345678901234567890123456789012123456789012345678901 2 235
123456789012345678901234567890121234567890123456789012
Part III: Section Review

123456789012345678901234567890121234567890123456789012
12345678901234567890123456789012123456789012345678901 2
12345678901234567890123456789012123456789012345678901 2
12345678901234567890123456789012123456789012345678901 2
12345678901234567890123456789012123456789012345678901 2
12345678901234567890123456789012123456789012345678901
Key Points to Include on the First Essay 2
12345678901234567890123456789012123456789012345678901 2
12345678901234567890123456789012123456789012345678901 2
12345678901234567890123456789012123456789012345678901
1. A Clearly Stated Position. Don’t send the readers on a scavenger 2
12345678901234567890123456789012123456789012345678901 2
12345678901234567890123456789012123456789012345678901
hunt to see how you feel about the issue. Make sure your point of 2
12345678901234567890123456789012123456789012345678901 2
12345678901234567890123456789012123456789012345678901
view is clearly stated. Basically, you will either disagree or agree
2345678901234567890123456789012123456789012345678901
2
2
1
12345678901234567890123456789012123456789012345678901
with the initial topic sentence. The safest place to state your point 2
12345678901234567890123456789012123456789012345678901 2
12345678901234567890123456789012123456789012345678901
of view is the first paragraph. 2
12345678901234567890123456789012123456789012345678901
2345678901234567890123456789012123456789012345678901
2
2
1 2. A Cartload of Plausible Supporting Details. You can’t use the
12345678901234567890123456789012123456789012345678901 2
12345678901234567890123456789012123456789012345678901 2
12345678901234567890123456789012123456789012345678901
phrase “because I say so” to back up your argument. You need to 2
12345678901234567890123456789012123456789012345678901
2345678901234567890123456789012123456789012345678901
2
2
1 come up with believable reasons that justify your position. The
12345678901234567890123456789012123456789012345678901 2
12345678901234567890123456789012123456789012345678901
more specific they are, the better, but the format of the 2
12345678901234567890123456789012123456789012345678901 2
12345678901234567890123456789012123456789012345678901
time-intensive essay will prevent you from using hard data. You 2
2345678901234567890123456789012123456789012345678901
12345678901234567890123456789012123456789012345678901 2
12345678901234567890123456789012123456789012345678901
could memorize an almanac or census book if you like, but this is 2
12345678901234567890123456789012123456789012345678901 2
12345678901234567890123456789012123456789012345678901 2
probably not a good use of your time. Instead, paint with broad
12345678901234567890123456789012123456789012345678901
strokes with phrases like, “many people enjoy fine seafood” and 2
12345678901234567890123456789012123456789012345678901 2
1 “computer skills are becoming increasingly important in the 2
12345678901234567890123456789012123456789012345678901 2
12345678901234567890123456789012123456789012345678901
modern world.” Both of these statements are believable, even 2
12345678901234567890123456789012123456789012345678901 2
2345678901234567890123456789012123456789012345678901
12345678901234567890123456789012123456789012345678901
though they contain no hard facts. 2
12345678901234567890123456789012123456789012345678901 2
12345678901234567890123456789012123456789012345678901 2
12345678901234567890123456789012123456789012345678901
3. Demonstrate That You Can See Other Side’s Point of View, Even 2
12345678901234567890123456789012123456789012345678901 2
12345678901234567890123456789012123456789012345678901 2
Though You Disagree with It. Sadly, actual debate is very rare, at
12345678901234567890123456789012123456789012345678901 2
12345678901234567890123456789012123456789012345678901
least on television. On TV, most shows put two implacable 2
12345678901234567890123456789012123456789012345678901 2
1 enemies on screen together, and then watch both sides use every 2
2345678901234567890123456789012123456789012345678901
12345678901234567890123456789012123456789012345678901 2
12345678901234567890123456789012123456789012345678901
sophistic trick in the book to browbeat their opponent and win 2
1 2
12345678901234567890123456789012123456789012345678901 2
the argument. It might make for entertaining TV, but it’s a lousy
12345678901234567890123456789012123456789012345678901 2
12345678901234567890123456789012123456789012345678901
discourse. Such an extreme, unbending position is the exact 2
12345678901234567890123456789012123456789012345678901 2
12345678901234567890123456789012123456789012345678901 2
opposite of what you want in your essay. Take a sentence or two
12345678901234567890123456789012123456789012345678901 2
12345678901234567890123456789012123456789012345678901
to show how you understand the opponent’s point of view, and 2
2345678901234567890123456789012123456789012345678901
12345678901234567890123456789012123456789012345678901 2
12345678901234567890123456789012123456789012345678901
then use the rest of the paragraph to explain why you feel this 2
1 position is not as strong as the one you believe. 2
12345678901234567890123456789012123456789012345678901 2
12345678901234567890123456789012123456789012345678901 2
12345678901234567890123456789012123456789012345678901
4. Have a Clear Conclusion. It’s a basic point of every essay, and the 2
12345678901234567890123456789012123456789012345678901 2
12345678901234567890123456789012123456789012345678901
readers will be looking for it. If possible, try to rephrase your 2
12345678901234567890123456789012123456789012345678901 2
12345678901234567890123456789012123456789012345678901
original position. Don’t use the exact same words as you did at the 2
2345678901234567890123456789012123456789012345678901
12345678901234567890123456789012123456789012345678901 2
12345678901234567890123456789012123456789012345678901
beginning of the essay. 2
12345678901234567890123456789012123456789012345678901 2
12345678901234567890123456789012123456789012345678901 2 2
1
12345678901234567890123456789012123456789012345678901 2
12345678901234567890123456789012123456789012345678901
Look at the some of the sample issues at the GRE Web site and decide how 2
12345678901234567890123456789012123456789012345678901 2
12345678901234567890123456789012123456789012345678901
you would approach them using the information in the chart above. 2
12345678901234567890123456789012123456789012345678901 2
12345678901234567890123456789012123456789012345678901
Would you agree or disagree? What facts would you use to support your 2
12345678901234567890123456789012123456789012345678901 2
12345678901234567890123456789012123456789012345678901
case? How would you show that you understood the other side’s view? If 2
12345678901234567890123456789012123456789012345678901 2
12345678901234567890123456789012123456789012345678901
the Analytical score is important to you, give yourself 45 minutes and 2
1 practice writing some sample essays. Use a computer to type out your 2
2345678901234567890123456789012123456789012345678901
12345678901234567890123456789012123456789012345678901 2
12345678901234567890123456789012123456789012345678901 2 2
236 12345678901234567890123456789012123456789012345678901
123456789012345678901234567890121234567890123456789012

www.petersons.com
Chapter 6: Analytical Writing Review

123456789012345678901234567890121234567890123456789012
12345678901234567890123456789012123456789012345678901 2
12345678901234567890123456789012123456789012345678901 2
12345678901234567890123456789012123456789012345678901
response, since you want to practice your typing skills as well. Since there 2
12345678901234567890123456789012123456789012345678901 2
12345678901234567890123456789012123456789012345678901
are about 240 topics listed, it is not very likely that a topic you practice on 2
12345678901234567890123456789012123456789012345678901 2
12345678901234567890123456789012123456789012345678901
will be one of the ones you get for your actual GRE. Then again, stranger 2
1 things have happened, and people do get lucky. If you glance over a bunch 2
2345678901234567890123456789012123456789012345678901
12345678901234567890123456789012123456789012345678901 2
12345678901234567890123456789012123456789012345678901 2
12345678901234567890123456789012123456789012345678901
of the issue topics—say, about half of them—and think about how you 2
12345678901234567890123456789012123456789012345678901
2345678901234567890123456789012123456789012345678901
2
2
1 would respond to them, you will be making your own luck.
12345678901234567890123456789012123456789012345678901 2
12345678901234567890123456789012123456789012345678901 2
12345678901234567890123456789012123456789012345678901
The last thing to note is that you should take the entire 45 minutes on the 2
12345678901234567890123456789012123456789012345678901
2345678901234567890123456789012123456789012345678901
2
2
1 essay. When it’s over, stretch your fingers and get prepared for some more
12345678901234567890123456789012123456789012345678901 2
12345678901234567890123456789012123456789012345678901
typing. The Argument essay is next on the menu. 2
12345678901234567890123456789012123456789012345678901 2
12345678901234567890123456789012123456789012345678901
2345678901234567890123456789012123456789012345678901
2
2
1
12345678901234567890123456789012123456789012345678901 2
12345678901234567890123456789012123456789012345678901
The Argument Essay 2
12345678901234567890123456789012123456789012345678901 2
12345678901234567890123456789012123456789012345678901
The second essay is 15 minutes shorter than the first, and you are only 2
12345678901234567890123456789012123456789012345678901 22
2345678901234567890123456789012123456789012345678901
12345678901234567890123456789012123456789012345678901
given one topic. A lengthy list of possible topics can again be found at the
12345678901234567890123456789012123456789012345678901 2
GRE Web site, specifically at www.gre.org/argutop.html. Most topics are 2
12345678901234567890123456789012123456789012345678901
12345678901234567890123456789012123456789012345678901 22
12345678901234567890123456789012123456789012345678901
1 around 100 words in length and are written to resemble editorials, memos, 2
12345678901234567890123456789012123456789012345678901
letters, and reports from fictional people or businesses. 2
12345678901234567890123456789012123456789012345678901 2
12345678901234567890123456789012123456789012345678901 2
12345678901234567890123456789012123456789012345678901 22
2345678901234567890123456789012123456789012345678901
Read a bunch of these “arguments” to get a feel for what they are like. The
12345678901234567890123456789012123456789012345678901
first thing you should notice is that every argument has a point. Quite 2
12345678901234567890123456789012123456789012345678901
often, this point is the first sentence, although in other instances it is 2
12345678901234567890123456789012123456789012345678901
12345678901234567890123456789012123456789012345678901 2
farther into the text. The point—which can also be called the topic 2
12345678901234567890123456789012123456789012345678901
12345678901234567890123456789012123456789012345678901 2
12345678901234567890123456789012123456789012345678901 2
12345678901234567890123456789012123456789012345678901 22
statement, main idea of the argument, or conclusion—is a judgment or
1 belief made by the author of the topic. This point can be something like,
12345678901234567890123456789012123456789012345678901 2
12345678901234567890123456789012123456789012345678901 2
12345678901234567890123456789012123456789012345678901 2
“Therefore, people should try to reduce their intake of salmon if they are
12345678901234567890123456789012123456789012345678901
concerned about mercury poisoning” or “It is obvious that higher salaries 2
12345678901234567890123456789012123456789012345678901
2345678901234567890123456789012123456789012345678901
2
2
12345678901234567890123456789012123456789012345678901
will lead to a higher number of new products.”
12345678901234567890123456789012123456789012345678901 2
1 2
12345678901234567890123456789012123456789012345678901 2
12345678901234567890123456789012123456789012345678901 2
12345678901234567890123456789012123456789012345678901 22
2345678901234567890123456789012123456789012345678901
Words like therefore, clearly, and thus are big signposts that scream,
12345678901234567890123456789012123456789012345678901
Note

1 “Main point up ahead!” These words are often used after a series of 2
12345678901234567890123456789012123456789012345678901
supporting details, and they act as a verbal transition between the 2
12345678901234567890123456789012123456789012345678901 2
2345678901234567890123456789012123456789012345678901
12345678901234567890123456789012123456789012345678901
supporting details and the main idea supported by the details. You should 2
12345678901234567890123456789012123456789012345678901 2
12345678901234567890123456789012123456789012345678901
be familiar with them, as it is likely you used one of these exact words 2
12345678901234567890123456789012123456789012345678901 22
1 when restating your main point at the end of the first essay.
12345678901234567890123456789012123456789012345678901 2
12345678901234567890123456789012123456789012345678901 2
12345678901234567890123456789012123456789012345678901 2
12345678901234567890123456789012123456789012345678901
The main point of the argument is something that can be disputed. To help 2
12345678901234567890123456789012123456789012345678901 2
12345678901234567890123456789012123456789012345678901
convince you of the main idea, the author surrounds the main idea with 2
12345678901234567890123456789012123456789012345678901 2
12345678901234567890123456789012123456789012345678901
supporting details that back up the initial claim. These supporting details 2
12345678901234567890123456789012123456789012345678901 2
12345678901234567890123456789012123456789012345678901
serve the same function here as the supporting details you created for the 2
2345678901234567890123456789012123456789012345678901
12345678901234567890123456789012123456789012345678901 2
1 first essay: They provide facts that support the author’s claim. Some 2
12345678901234567890123456789012123456789012345678901 2
12345678901234567890123456789012123456789012345678901
arguments consist of one supporting idea, which is fleshed out and 2
12345678901234567890123456789012123456789012345678901
described in detail for the rest of the paragraph. Other arguments make 2
12345678901234567890123456789012123456789012345678901 2
12345678901234567890123456789012123456789012345678901 2
12345678901234567890123456789012123456789012345678901 22
3–4 different points, each of which supports the main point. If you scan
12345678901234567890123456789012123456789012345678901 237
123456789012345678901234567890121234567890123456789012
Part III: Section Review

123456789012345678901234567890121234567890123456789012
12345678901234567890123456789012123456789012345678901 2
12345678901234567890123456789012123456789012345678901 2
12345678901234567890123456789012123456789012345678901
through the sample passages provided online, you can find examples of 2
12345678901234567890123456789012123456789012345678901 2
12345678901234567890123456789012123456789012345678901
both types, as well as many more that fall between these two poles. 2
12345678901234567890123456789012123456789012345678901 2
12345678901234567890123456789012123456789012345678901
The ability to dissect each argument into a main idea and its supporting 2
12345678901234567890123456789012123456789012345678901 2
12345678901234567890123456789012123456789012345678901 2
12345678901234567890123456789012123456789012345678901
details is the first step to doing well on the second essay. If you feel you 2
12345678901234567890123456789012123456789012345678901 2
12345678901234567890123456789012123456789012345678901
need some practice at this, take a sheet
2345678901234567890123456789012123456789012345678901
of lined paper and write the 2
2
1 following:
12345678901234567890123456789012123456789012345678901 2
12345678901234567890123456789012123456789012345678901 2
12345678901234567890123456789012123456789012345678901 2
12345678901234567890123456789012123456789012345678901
The main idea of this argument is:
2345678901234567890123456789012123456789012345678901
2
2
1
12345678901234567890123456789012123456789012345678901 2
12345678901234567890123456789012123456789012345678901 2
12345678901234567890123456789012123456789012345678901 2
12345678901234567890123456789012123456789012345678901
2345678901234567890123456789012123456789012345678901
2
2
1
12345678901234567890123456789012123456789012345678901 2
12345678901234567890123456789012123456789012345678901
The supporting details used to make this claim are: 2
12345678901234567890123456789012123456789012345678901 2
12345678901234567890123456789012123456789012345678901 2
12345678901234567890123456789012123456789012345678901 2
2345678901234567890123456789012123456789012345678901
1.
12345678901234567890123456789012123456789012345678901 2
12345678901234567890123456789012123456789012345678901 2
12345678901234567890123456789012123456789012345678901 2
12345678901234567890123456789012123456789012345678901 2
12345678901234567890123456789012123456789012345678901
2. 2
1 2
12345678901234567890123456789012123456789012345678901 2
12345678901234567890123456789012123456789012345678901 2
12345678901234567890123456789012123456789012345678901 2
2345678901234567890123456789012123456789012345678901
12345678901234567890123456789012123456789012345678901 2
12345678901234567890123456789012123456789012345678901
3. 2
12345678901234567890123456789012123456789012345678901 2
12345678901234567890123456789012123456789012345678901 2
12345678901234567890123456789012123456789012345678901 2
12345678901234567890123456789012123456789012345678901 2
Scroll through the GRE argument passages, pick one at random, and place 2
12345678901234567890123456789012123456789012345678901
12345678901234567890123456789012123456789012345678901
it into the above template. Determine the main idea, and write out that 2
2
12345678901234567890123456789012123456789012345678901
1 sentence in the space provided. Then list the supporting details that back 2
12345678901234567890123456789012123456789012345678901 2
12345678901234567890123456789012123456789012345678901 2
12345678901234567890123456789012123456789012345678901
up the claim. Since some details might be connected, make a note like ↔ to 2
12345678901234567890123456789012123456789012345678901 2
12345678901234567890123456789012123456789012345678901
show that two facts are linked. Repeat this exercise several times until you 2
2345678901234567890123456789012123456789012345678901
are able to spot a conclusion and its supporting details with little difficulty. 2
12345678901234567890123456789012123456789012345678901
12345678901234567890123456789012123456789012345678901 2
1 It should help that you spent the first essay deciding on your own point of 2
12345678901234567890123456789012123456789012345678901 2
12345678901234567890123456789012123456789012345678901 2
12345678901234567890123456789012123456789012345678901 2
2345678901234567890123456789012123456789012345678901
view and then backing it up with supporting details. For the second essay,
12345678901234567890123456789012123456789012345678901 2
1 the main point and supporting details have been given to you. You just 2
12345678901234567890123456789012123456789012345678901 2
12345678901234567890123456789012123456789012345678901
have to determine which is which. 2
12345678901234567890123456789012123456789012345678901 2
2345678901234567890123456789012123456789012345678901
When you’ve sliced apart a bunch of essays, look with a skeptical eye at the 2
12345678901234567890123456789012123456789012345678901 2
12345678901234567890123456789012123456789012345678901
conclusions and the supporting details used to prove them. How airtight 2
12345678901234567890123456789012123456789012345678901
1 2
12345678901234567890123456789012123456789012345678901
are these arguments? Are any of these arguments going to be eligible for 2
12345678901234567890123456789012123456789012345678901 2
12345678901234567890123456789012123456789012345678901
the Nobel prize in logic? 2
12345678901234567890123456789012123456789012345678901 2
12345678901234567890123456789012123456789012345678901 2
12345678901234567890123456789012123456789012345678901 2
2345678901234567890123456789012123456789012345678901
The answer is “No.” A thousand times, no! Every single one of these
2
12345678901234567890123456789012123456789012345678901
arguments can be attacked and discredited in one way or another. To 2
12345678901234567890123456789012123456789012345678901 2
12345678901234567890123456789012123456789012345678901
1 succeed on the second essay, your primary mission is to recognize these 2
2345678901234567890123456789012123456789012345678901
12345678901234567890123456789012123456789012345678901 2
1 errors and write about them. 2
12345678901234567890123456789012123456789012345678901 2
12345678901234567890123456789012123456789012345678901 2
12345678901234567890123456789012123456789012345678901 2
12345678901234567890123456789012123456789012345678901 2
12345678901234567890123456789012123456789012345678901 2
12345678901234567890123456789012123456789012345678901 2 2
238 12345678901234567890123456789012123456789012345678901
123456789012345678901234567890121234567890123456789012

www.petersons.com
Chapter 6: Analytical Writing Review

123456789012345678901234567890121234567890123456789012
12345678901234567890123456789012123456789012345678901 2
12345678901234567890123456789012123456789012345678901 2
12345678901234567890123456789012123456789012345678901
Find the Flaw 2
12345678901234567890123456789012123456789012345678901 2
12345678901234567890123456789012123456789012345678901 2
12345678901234567890123456789012123456789012345678901 2
The second essay is officially called “Analyze an Argument,” but you
12345678901234567890123456789012123456789012345678901
should think of it as a game called “Find the Flaw.” Using this name will 2
12345678901234567890123456789012123456789012345678901 2
12345678901234567890123456789012123456789012345678901
get you in the proper mindset. When you read the argument essay, you 2
12345678901234567890123456789012123456789012345678901 2
12345678901234567890123456789012123456789012345678901
want to be much more critical than you usually are. When doing regular 2
12345678901234567890123456789012123456789012345678901 2
12345678901234567890123456789012123456789012345678901
reading, people often give an author the benefit of the doubt and keep an 2
12345678901234567890123456789012123456789012345678901 2
12345678901234567890123456789012123456789012345678901
open mind that what is being written could be valid. This means you might 2
12345678901234567890123456789012123456789012345678901 2
12345678901234567890123456789012123456789012345678901
read an editorial, mentally shrug, and think, “Yes, I suppose that could be 2
2345678901234567890123456789012123456789012345678901 2
1 true.”
12345678901234567890123456789012123456789012345678901 2
12345678901234567890123456789012123456789012345678901 2
12345678901234567890123456789012123456789012345678901 2
12345678901234567890123456789012123456789012345678901
You don’t want to be so kind on the second
2345678901234567890123456789012123456789012345678901
essay. Once the clock starts 2
2
1 ticking, your goal is to:
12345678901234567890123456789012123456789012345678901 2
12345678901234567890123456789012123456789012345678901 2
12345678901234567890123456789012123456789012345678901 2
12345678901234567890123456789012123456789012345678901
1. Read the argument 2
12345678901234567890123456789012123456789012345678901 22
2345678901234567890123456789012123456789012345678901
12345678901234567890123456789012123456789012345678901
2. Determine the main point
12345678901234567890123456789012123456789012345678901 2
12345678901234567890123456789012123456789012345678901 2
12345678901234567890123456789012123456789012345678901
3. Determine the supporting details 2
12345678901234567890123456789012123456789012345678901 22
1
12345678901234567890123456789012123456789012345678901
4. Brainstorm alternatives that attack either the main idea or a 2
12345678901234567890123456789012123456789012345678901 2
12345678901234567890123456789012123456789012345678901
supporting detail 2
12345678901234567890123456789012123456789012345678901 22
2345678901234567890123456789012123456789012345678901
12345678901234567890123456789012123456789012345678901
12345678901234567890123456789012123456789012345678901
5. Write about these flaws in the argument 2
12345678901234567890123456789012123456789012345678901 22
12345678901234567890123456789012123456789012345678901
A run-through of these steps can be shown using this sample argument
12345678901234567890123456789012123456789012345678901 2
12345678901234567890123456789012123456789012345678901
(which once again has NOT been plagiarized from the official site). 2
12345678901234567890123456789012123456789012345678901 22
12345678901234567890123456789012123456789012345678901
The following memorandum was written by the President of 2
1
12345678901234567890123456789012123456789012345678901 2
12345678901234567890123456789012123456789012345678901
Dextrose Bank. 2
12345678901234567890123456789012123456789012345678901 2
12345678901234567890123456789012123456789012345678901
“The President of Sucrose Bank, our main competitor, 2
12345678901234567890123456789012123456789012345678901
2345678901234567890123456789012123456789012345678901
2
recently mentioned that over half of Sucrose’s new 2
12345678901234567890123456789012123456789012345678901
12345678901234567890123456789012123456789012345678901 2
1 2
12345678901234567890123456789012123456789012345678901
customers were people passing by who simply walked in 2
12345678901234567890123456789012123456789012345678901 2
12345678901234567890123456789012123456789012345678901 22
2345678901234567890123456789012123456789012345678901
the front door. With this in mind, a large portion of this
12345678901234567890123456789012123456789012345678901
year’s marketing budget will be shifted to improving the 2
1
12345678901234567890123456789012123456789012345678901
exterior look of our bank. This will make our bank look 2
12345678901234567890123456789012123456789012345678901 2
2345678901234567890123456789012123456789012345678901
more attractive to people passing by, and we should be 2
12345678901234567890123456789012123456789012345678901
12345678901234567890123456789012123456789012345678901 2
able to attract new customers the same way Sucrose 2
12345678901234567890123456789012123456789012345678901
12345678901234567890123456789012123456789012345678901 22
1 Bank did.”
12345678901234567890123456789012123456789012345678901 2
12345678901234567890123456789012123456789012345678901 2
12345678901234567890123456789012123456789012345678901
Having read the paragraph, Step 1 is complete. Now it’s time to determine 2
12345678901234567890123456789012123456789012345678901 2
12345678901234567890123456789012123456789012345678901
what the main point is. Some of you might think the main point is, “a large 2
2345678901234567890123456789012123456789012345678901
12345678901234567890123456789012123456789012345678901
portion of this year’s marketing budget will be shifted to improving the 2
2
12345678901234567890123456789012123456789012345678901
exterior look of our bank,” since this is the big change at Dextrose Bank. 2
12345678901234567890123456789012123456789012345678901
12345678901234567890123456789012123456789012345678901 2
1 However, keep in mind that a main point is a point of view. It’s something 2
12345678901234567890123456789012123456789012345678901 22
2345678901234567890123456789012123456789012345678901
1 that a particular person believes, but it’s not a fact. The statement, “A
12345678901234567890123456789012123456789012345678901 2
12345678901234567890123456789012123456789012345678901 2
12345678901234567890123456789012123456789012345678901
large portion of this year’s marketing budget .
2345678901234567890123456789012123456789012345678901
. .” is a fact. You can’t agree 2
2
1 or disagree, so it’s not a point of view.
12345678901234567890123456789012123456789012345678901 2
12345678901234567890123456789012123456789012345678901 2
12345678901234567890123456789012123456789012345678901 2 239
123456789012345678901234567890121234567890123456789012
Part III: Section Review

123456789012345678901234567890121234567890123456789012
12345678901234567890123456789012123456789012345678901 2
12345678901234567890123456789012123456789012345678901 2
12345678901234567890123456789012123456789012345678901 2
12345678901234567890123456789012123456789012345678901
Again, thinking about the first essay will help you determine the main 2
12345678901234567890123456789012123456789012345678901 2
12345678901234567890123456789012123456789012345678901 2
12345678901234567890123456789012123456789012345678901
point of the second essay. Recall how you decided whether to agree or 2
1 disagree with the initial statement, since this subjective statement could be 2
2345678901234567890123456789012123456789012345678901

Tip
12345678901234567890123456789012123456789012345678901 2
12345678901234567890123456789012123456789012345678901
interpreted either way. The main idea of the argument essay will be a 2
12345678901234567890123456789012123456789012345678901 2
12345678901234567890123456789012123456789012345678901
statement you could either agree or disagree with, although you want to 2
2345678901234567890123456789012123456789012345678901 2
1
12345678901234567890123456789012123456789012345678901
do more disagreeing than agreeing. 2
12345678901234567890123456789012123456789012345678901 2
12345678901234567890123456789012123456789012345678901 2
12345678901234567890123456789012123456789012345678901 2
1 The conclusion reached by the President of Dextrose Bank is that spending 2
2345678901234567890123456789012123456789012345678901
12345678901234567890123456789012123456789012345678901 2
12345678901234567890123456789012123456789012345678901
money on the exterior “will make our bank look more attractive to people 2
12345678901234567890123456789012123456789012345678901 2
12345678901234567890123456789012123456789012345678901
passing by, and we should be able to attract new customers the same way 2
1 Sucrose Bank did.” This is the President’s belief, but it’s not a fact like “the 2
2345678901234567890123456789012123456789012345678901
12345678901234567890123456789012123456789012345678901 2
12345678901234567890123456789012123456789012345678901 2
12345678901234567890123456789012123456789012345678901
temperature is currently 80 degrees.” 2
12345678901234567890123456789012123456789012345678901 2
2345678901234567890123456789012123456789012345678901
Just for starters, some people might think the bank looks less attractive, 2
12345678901234567890123456789012123456789012345678901
12345678901234567890123456789012123456789012345678901 2
12345678901234567890123456789012123456789012345678901 2
12345678901234567890123456789012123456789012345678901 2
and others might like the way the bank looks but not go in and start a new
account because of it. Both of these scenarios call the President’s 2
12345678901234567890123456789012123456789012345678901
12345678901234567890123456789012123456789012345678901 2
1 conclusion into question. While the President’s point may or may not be 2
12345678901234567890123456789012123456789012345678901 2
12345678901234567890123456789012123456789012345678901 2
12345678901234567890123456789012123456789012345678901
valid, it is still the conclusion, so Step 2 is complete. Once you have the 2
2345678901234567890123456789012123456789012345678901
conclusion, pretty much everything else becomes the supporting details 2
12345678901234567890123456789012123456789012345678901
12345678901234567890123456789012123456789012345678901 2
12345678901234567890123456789012123456789012345678901
used to justify the claim, so Step 3 falls into place as well. 2
12345678901234567890123456789012123456789012345678901 2
12345678901234567890123456789012123456789012345678901 2
Reading the argument and determining what’s what should take no more 2
12345678901234567890123456789012123456789012345678901
12345678901234567890123456789012123456789012345678901
than 3 minutes, leaving you with 27 minutes for the last two steps. With 2
2
12345678901234567890123456789012123456789012345678901
12345678901234567890123456789012123456789012345678901 2
1 any essay (even a timed one), it’s always a good idea to do a bit of 2
2345678901234567890123456789012123456789012345678901
12345678901234567890123456789012123456789012345678901
brainstorming before you start writing. This essay’s no exception, so spend 2
2
12345678901234567890123456789012123456789012345678901
1 about 3–5 minutes coming up with ways to show why the President’s 2
12345678901234567890123456789012123456789012345678901 2
12345678901234567890123456789012123456789012345678901 2
12345678901234567890123456789012123456789012345678901
conclusion might not hold true. 2
12345678901234567890123456789012123456789012345678901 2
12345678901234567890123456789012123456789012345678901 2
12345678901234567890123456789012123456789012345678901
You wouldn’t have to write out every alternative. Just take quick notes to 2
12345678901234567890123456789012123456789012345678901 2
12345678901234567890123456789012123456789012345678901 2
2345678901234567890123456789012123456789012345678901
remind yourself. Once the clock reads 23 minutes to go, you should stop
12345678901234567890123456789012123456789012345678901 2
1 brainstorming and start writing. Your first sentence can be something like, 2
12345678901234567890123456789012123456789012345678901
“Although [Main Point . . . Blah Blah Blah] has some validity, it does not 2
12345678901234567890123456789012123456789012345678901 2
2345678901234567890123456789012123456789012345678901
hold up to intense scrutiny. There are several ways that this conclusion can 2
12345678901234567890123456789012123456789012345678901
12345678901234567890123456789012123456789012345678901 2
be proven wrong, such as. . . .” Then you put your essay-writing cap on 2
12345678901234567890123456789012123456789012345678901
12345678901234567890123456789012123456789012345678901 2 2
1 and light the sucker up for all its flaws.
12345678901234567890123456789012123456789012345678901 2
12345678901234567890123456789012123456789012345678901 2
12345678901234567890123456789012123456789012345678901
To do this, you need to come up with alternatives to what is assumed in the 2
12345678901234567890123456789012123456789012345678901 2
12345678901234567890123456789012123456789012345678901
text. These alternatives require a little bit of creativity on your part, but 2
2345678901234567890123456789012123456789012345678901
12345678901234567890123456789012123456789012345678901
once you practice on some sample passages, you’ll get the hang of it. These 2
2
12345678901234567890123456789012123456789012345678901
alternatives—you can call them alternative reasons, outcomes, explana- 2
12345678901234567890123456789012123456789012345678901
12345678901234567890123456789012123456789012345678901 2
1 tions, scenarios, and possibilities—are all designed to undercut a specific 2
12345678901234567890123456789012123456789012345678901 2
2345678901234567890123456789012123456789012345678901
1 part of the original text. Look at the previous chart that shows the 2
12345678901234567890123456789012123456789012345678901 2
12345678901234567890123456789012123456789012345678901 2
12345678901234567890123456789012123456789012345678901
conclusion. The bank president assumes that the new look “will make our 2
1 bank look more attractive,” so Alternative #1 is created by simply stating 2
2345678901234567890123456789012123456789012345678901
12345678901234567890123456789012123456789012345678901 2
12345678901234567890123456789012123456789012345678901 2 2
240 12345678901234567890123456789012123456789012345678901
123456789012345678901234567890121234567890123456789012

www.petersons.com
Chapter 6: Analytical Writing Review

123456789012345678901234567890121234567890123456789012
12345678901234567890123456789012123456789012345678901 2
12345678901234567890123456789012123456789012345678901 2
12345678901234567890123456789012123456789012345678901 2
12345678901234567890123456789012123456789012345678901 2
12345678901234567890123456789012123456789012345678901
Finding Flaws in the Argument 2
12345678901234567890123456789012123456789012345678901 2
12345678901234567890123456789012123456789012345678901 2
12345678901234567890123456789012123456789012345678901
Conclusion: “This will make our bank look more attractive to people 2
12345678901234567890123456789012123456789012345678901 2
12345678901234567890123456789012123456789012345678901
passing by, and we should be able to attract new customers the same 2
12345678901234567890123456789012123456789012345678901 2
12345678901234567890123456789012123456789012345678901
way Sucrose Bank did.”
2345678901234567890123456789012123456789012345678901
2
2
1
12345678901234567890123456789012123456789012345678901 2
12345678901234567890123456789012123456789012345678901
Alternative #1: What if people don’t like the new look? This 2
12345678901234567890123456789012123456789012345678901 2
12345678901234567890123456789012123456789012345678901
would discourage them from entering,
2345678901234567890123456789012123456789012345678901
the exact opposite of what 2
2
1
12345678901234567890123456789012123456789012345678901
is wanted. 2
12345678901234567890123456789012123456789012345678901 2
12345678901234567890123456789012123456789012345678901 2
12345678901234567890123456789012123456789012345678901
Alternative #2: What if exterior appearance
2345678901234567890123456789012123456789012345678901
had nothing to do 2
2
1
12345678901234567890123456789012123456789012345678901
with Sucrose’s success with walk-ins? If Sucrose was located near 2
12345678901234567890123456789012123456789012345678901 2
12345678901234567890123456789012123456789012345678901
a busy, trendy supermarket, people might walk in and start an 2
12345678901234567890123456789012123456789012345678901
account there because it was convenient. 2
12345678901234567890123456789012123456789012345678901 22
2345678901234567890123456789012123456789012345678901
12345678901234567890123456789012123456789012345678901 2
12345678901234567890123456789012123456789012345678901
Supporting Detail: “With this in mind, a large portion of this year’s
12345678901234567890123456789012123456789012345678901 2
12345678901234567890123456789012123456789012345678901
marketing budget will be shifted to improving the exterior look of 2
12345678901234567890123456789012123456789012345678901 2
1 our bank.” 2
12345678901234567890123456789012123456789012345678901 2
12345678901234567890123456789012123456789012345678901 2
12345678901234567890123456789012123456789012345678901
Alternative #3: What if Dextrose has had success with radio and 2
2345678901234567890123456789012123456789012345678901
12345678901234567890123456789012123456789012345678901 2
12345678901234567890123456789012123456789012345678901
print ads? Taking money in the budget away from these 2
12345678901234567890123456789012123456789012345678901 2
12345678901234567890123456789012123456789012345678901 22
successful marketing strategies might hurt the overall number of
12345678901234567890123456789012123456789012345678901
12345678901234567890123456789012123456789012345678901
new customers. Walk-ins might increase, but they wouldn’t 2
12345678901234567890123456789012123456789012345678901 2
12345678901234567890123456789012123456789012345678901 22
counteract the number of customers lost by abandoning or
12345678901234567890123456789012123456789012345678901 2
1 cutting back on the proven methods of radio and print ads.
12345678901234567890123456789012123456789012345678901 2
12345678901234567890123456789012123456789012345678901
Alternative #4: What if there are better ways to spend money and 2
12345678901234567890123456789012123456789012345678901 2
12345678901234567890123456789012123456789012345678901
attract walk ins? Dextrose Bank could leave the exterior as is, but 2
12345678901234567890123456789012123456789012345678901
2345678901234567890123456789012123456789012345678901
2
2
12345678901234567890123456789012123456789012345678901
throw a benefit concert featuring local musicians. This might
12345678901234567890123456789012123456789012345678901 2
1 generate a large crowd of people who then sign up as new 2
12345678901234567890123456789012123456789012345678901 2
12345678901234567890123456789012123456789012345678901
customers. 2
12345678901234567890123456789012123456789012345678901 22
2345678901234567890123456789012123456789012345678901
12345678901234567890123456789012123456789012345678901 2
1 Supporting Detail: “The President of Sucrose Bank, our main
12345678901234567890123456789012123456789012345678901 2
12345678901234567890123456789012123456789012345678901 2
competitor, recently mentioned that over half of Sucrose’s new
2345678901234567890123456789012123456789012345678901
12345678901234567890123456789012123456789012345678901 2
12345678901234567890123456789012123456789012345678901
customers were people passing by who simply walked in the front 2
12345678901234567890123456789012123456789012345678901 2
12345678901234567890123456789012123456789012345678901 22
door.”
1
12345678901234567890123456789012123456789012345678901 2
12345678901234567890123456789012123456789012345678901
Alternative #5: Putting it bluntly, what if the President of Sucrose 2
12345678901234567890123456789012123456789012345678901
Bank is lying? Suppose the Sucrose Bank president made those 2
12345678901234567890123456789012123456789012345678901 2
12345678901234567890123456789012123456789012345678901
remarks knowing that his/her counterpart at Dextrose Bank 2
12345678901234567890123456789012123456789012345678901 22
2345678901234567890123456789012123456789012345678901
12345678901234567890123456789012123456789012345678901
would react strongly to them? If this is the case, then the Dextrose
12345678901234567890123456789012123456789012345678901 22
12345678901234567890123456789012123456789012345678901
President has been played like a violin, and large amounts of
2
1
2345678901234567890123456789012123456789012345678901
12345678901234567890123456789012123456789012345678901
Dextrose Bank marketing money are being spent to counter a 2
1 2
12345678901234567890123456789012123456789012345678901
claim that is not even true. 2
12345678901234567890123456789012123456789012345678901 2
12345678901234567890123456789012123456789012345678901 2
12345678901234567890123456789012123456789012345678901 2
12345678901234567890123456789012123456789012345678901 2
12345678901234567890123456789012123456789012345678901 2
12345678901234567890123456789012123456789012345678901 2 241
123456789012345678901234567890121234567890123456789012
Part III: Section Review

123456789012345678901234567890121234567890123456789012
12345678901234567890123456789012123456789012345678901 2
12345678901234567890123456789012123456789012345678901 2
12345678901234567890123456789012123456789012345678901
a different outcome. To flesh out Alternative #1, you could have something 2
12345678901234567890123456789012123456789012345678901 2
12345678901234567890123456789012123456789012345678901
like the following paragraph. 2
12345678901234567890123456789012123456789012345678901 2
12345678901234567890123456789012123456789012345678901 2
12345678901234567890123456789012123456789012345678901 2
12345678901234567890123456789012123456789012345678901
(A) The President of Dextrose Bank believes that the new 2
12345678901234567890123456789012123456789012345678901 2
12345678901234567890123456789012123456789012345678901
look will make the bank more attractive to passers by, but 2
12345678901234567890123456789012123456789012345678901
2345678901234567890123456789012123456789012345678901
2
2
1 this is not necessarily true. (B) Some people might not like the
12345678901234567890123456789012123456789012345678901 2
12345678901234567890123456789012123456789012345678901
new decor, and avoid the bank for that reason. (C) For 2
12345678901234567890123456789012123456789012345678901 2
12345678901234567890123456789012123456789012345678901
example, suppose the bank spends a
2345678901234567890123456789012123456789012345678901
lot of money and plants a 2
2
1
12345678901234567890123456789012123456789012345678901
large number of flowers and bushes around the parking lot. 2
12345678901234567890123456789012123456789012345678901
(D) However, the state is in the midst of a severe drought. 2
12345678901234567890123456789012123456789012345678901 2
12345678901234567890123456789012123456789012345678901
(E) Some people might think the bank looks nice, but others
2345678901234567890123456789012123456789012345678901
2
2
1
12345678901234567890123456789012123456789012345678901
would believe that the bank is foolishly wasting water and 2
12345678901234567890123456789012123456789012345678901 2
12345678901234567890123456789012123456789012345678901
money during a drought. (F) These people would be turned 2
12345678901234567890123456789012123456789012345678901 2
12345678901234567890123456789012123456789012345678901 2
2345678901234567890123456789012123456789012345678901
off by the expensive remodeling, considering it wasteful
12345678901234567890123456789012123456789012345678901 2
12345678901234567890123456789012123456789012345678901
and boorish. 2
12345678901234567890123456789012123456789012345678901 2
12345678901234567890123456789012123456789012345678901 2
12345678901234567890123456789012123456789012345678901 2
1 Sentence A is the set-up sentence. It clearly states which part of the 2
12345678901234567890123456789012123456789012345678901 2
12345678901234567890123456789012123456789012345678901
argument you are going to attack. As you might expect, starting every 2
12345678901234567890123456789012123456789012345678901 2
2345678901234567890123456789012123456789012345678901
paragraph with a simple set-up sentence is a good game plan. Sentence B 2
12345678901234567890123456789012123456789012345678901
12345678901234567890123456789012123456789012345678901 2
states an alternative outcome, showing how this alternative leads to 2
12345678901234567890123456789012123456789012345678901
12345678901234567890123456789012123456789012345678901 2
actions opposite of what the bank president assumes will happen. 2
12345678901234567890123456789012123456789012345678901
12345678901234567890123456789012123456789012345678901 2
Sentences C and D are where you have to get a little creative. Basically, you 2
12345678901234567890123456789012123456789012345678901
12345678901234567890123456789012123456789012345678901
need to think of a reason why people would not like a new exterior for a 2
2
12345678901234567890123456789012123456789012345678901
1 bank. The sample above goes the “ecological conservationist” route, 2
12345678901234567890123456789012123456789012345678901 2
12345678901234567890123456789012123456789012345678901 2
12345678901234567890123456789012123456789012345678901
setting up a mythical drought that the cads at the bank care nothing about. 2
12345678901234567890123456789012123456789012345678901 2
12345678901234567890123456789012123456789012345678901
Sentences E and F explain why the eco-conservationists are now less likely 2
2345678901234567890123456789012123456789012345678901
to open an account at Dextrose Bank now that the exterior has been 2
12345678901234567890123456789012123456789012345678901
12345678901234567890123456789012123456789012345678901 2
1 expensively remodeled. 2
12345678901234567890123456789012123456789012345678901 2
12345678901234567890123456789012123456789012345678901 2
12345678901234567890123456789012123456789012345678901 2
2345678901234567890123456789012123456789012345678901
12345678901234567890123456789012123456789012345678901 2
1 The “eco-conservationist” route is not the only explanation you can give. 2
12345678901234567890123456789012123456789012345678901
You could also say, “Suppose the renovations to the exterior require large 2
12345678901234567890123456789012123456789012345678901 2
2345678901234567890123456789012123456789012345678901
parts of the bank building (including the bank sign) to be covered in 2
12345678901234567890123456789012123456789012345678901
12345678901234567890123456789012123456789012345678901 2
12345678901234567890123456789012123456789012345678901 2
Alert!

12345678901234567890123456789012123456789012345678901 2
scaffolding and other construction materials. This might make it hard for
1 people new to the area to realize that the building under construction was 2
12345678901234567890123456789012123456789012345678901 2
12345678901234567890123456789012123456789012345678901 2
12345678901234567890123456789012123456789012345678901
a bank. Since newly moved people are more likely to need a new account, 2
12345678901234567890123456789012123456789012345678901 2
12345678901234567890123456789012123456789012345678901 2
the work on the exterior would hurt Dextrose’s ability to sign these
2345678901234567890123456789012123456789012345678901
people up.” This isn’t perfect, but perfection isn’t required for a 2
12345678901234567890123456789012123456789012345678901
12345678901234567890123456789012123456789012345678901 2
12345678901234567890123456789012123456789012345678901
30-minute essay. You just need to show that you can break down an 2
12345678901234567890123456789012123456789012345678901 2 2
1 argument and come up with alternatives.
12345678901234567890123456789012123456789012345678901 2
12345678901234567890123456789012123456789012345678901 2
12345678901234567890123456789012123456789012345678901 2
12345678901234567890123456789012123456789012345678901
The essay does not require you to go out and actually find an 2
12345678901234567890123456789012123456789012345678901 2
12345678901234567890123456789012123456789012345678901
eco-conservationist who supports your opinion. You do not need to have a 2
12345678901234567890123456789012123456789012345678901 2
12345678901234567890123456789012123456789012345678901
sentence that shows the national membership figures for the Sierra Club. 2
2
242 12345678901234567890123456789012123456789012345678901
123456789012345678901234567890121234567890123456789012

www.petersons.com
Chapter 6: Analytical Writing Review

123456789012345678901234567890121234567890123456789012
12345678901234567890123456789012123456789012345678901 2
12345678901234567890123456789012123456789012345678901 2
12345678901234567890123456789012123456789012345678901
All you have to do is come up with a plausible (there’s that word again!) 2
12345678901234567890123456789012123456789012345678901 2
12345678901234567890123456789012123456789012345678901
alternative that shows how the initial point might be wrong. Once you 2
12345678901234567890123456789012123456789012345678901 2
12345678901234567890123456789012123456789012345678901
explain your alternative, you have done what is needed for the second 2
1 essay. Now you need some more of it, so it’s time to start another 2
2345678901234567890123456789012123456789012345678901
12345678901234567890123456789012123456789012345678901 2
12345678901234567890123456789012123456789012345678901 2
12345678901234567890123456789012123456789012345678901
paragraph and attack something else from the original argument. 2
12345678901234567890123456789012123456789012345678901 2
1 If you are wondering why the emphasis is on finding flaws in the 2
2345678901234567890123456789012123456789012345678901
12345678901234567890123456789012123456789012345678901 2
12345678901234567890123456789012123456789012345678901 2
12345678901234567890123456789012123456789012345678901
argument, the answer is simple. The GRE readers aren’t going to be kind 2
12345678901234567890123456789012123456789012345678901
2345678901234567890123456789012123456789012345678901
2
2
1 to an essay that states, “After looking over the argument, everything the
12345678901234567890123456789012123456789012345678901 2
12345678901234567890123456789012123456789012345678901
speaker claims is true.” Anyone who doesn’t understand the argument can 2
12345678901234567890123456789012123456789012345678901
write this, so it doesn’t really show that you did any thinking. The readers 2
12345678901234567890123456789012123456789012345678901 2
1 want evidence of some mental work on your part. This is why you must 2
2345678901234567890123456789012123456789012345678901
12345678901234567890123456789012123456789012345678901 2
12345678901234567890123456789012123456789012345678901
attack the argument, pulling it apart and coming up with alternative 2
12345678901234567890123456789012123456789012345678901 2
12345678901234567890123456789012123456789012345678901
possibilities that subvert the main point. 2
12345678901234567890123456789012123456789012345678901 22
2345678901234567890123456789012123456789012345678901
12345678901234567890123456789012123456789012345678901
However, you can recycle the old “I understand both sides of the issue, but 2
12345678901234567890123456789012123456789012345678901
12345678901234567890123456789012123456789012345678901
they’re wrong” concept from the first essay. This is done below with 2
2
12345678901234567890123456789012123456789012345678901
12345678901234567890123456789012123456789012345678901 2
1 Alternative #3. 2
12345678901234567890123456789012123456789012345678901 2
12345678901234567890123456789012123456789012345678901 2
12345678901234567890123456789012123456789012345678901 2
12345678901234567890123456789012123456789012345678901 22
2345678901234567890123456789012123456789012345678901
(G) The Dextrose Bank President has decided to shift Òa
12345678901234567890123456789012123456789012345678901
12345678901234567890123456789012123456789012345678901
large portion of this yearÕs marketing budget . . . to improving 2
(H) A new exterior might indeed 2
12345678901234567890123456789012123456789012345678901
12345678901234567890123456789012123456789012345678901 22
the exterior look of our bank.Ó
12345678901234567890123456789012123456789012345678901
bring in some new customers, since a bold new exterior might
12345678901234567890123456789012123456789012345678901 2
12345678901234567890123456789012123456789012345678901
increase walk-in interest. (I) However, the shift in marketing 2
12345678901234567890123456789012123456789012345678901 22
1 money might not be the best for the overall number of new
12345678901234567890123456789012123456789012345678901 2
12345678901234567890123456789012123456789012345678901
customers. (J) Suppose the shift in money means that the bank 2
12345678901234567890123456789012123456789012345678901 2
12345678901234567890123456789012123456789012345678901
can no longer do many print and radio ads? (K) If these ads 2
12345678901234567890123456789012123456789012345678901 2
12345678901234567890123456789012123456789012345678901 22
2345678901234567890123456789012123456789012345678901
were very successful, their decline might mean fewer new
12345678901234567890123456789012123456789012345678901 2
1 customers come in after reading or hearing an ad. (L) This
12345678901234567890123456789012123456789012345678901 2
12345678901234567890123456789012123456789012345678901
means that the number of walk-in new customers increases, 2
2345678901234567890123456789012123456789012345678901
12345678901234567890123456789012123456789012345678901 2
12345678901234567890123456789012123456789012345678901
but the number of radio and print new customers drops. 2
1 2
12345678901234567890123456789012123456789012345678901 2
(M) Overall, this might lead to a decline in the total number
12345678901234567890123456789012123456789012345678901 2
12345678901234567890123456789012123456789012345678901
of new customers, and Dextrose Bank will be worse off. 2
12345678901234567890123456789012123456789012345678901 2
12345678901234567890123456789012123456789012345678901 2
12345678901234567890123456789012123456789012345678901 2
12345678901234567890123456789012123456789012345678901
Sentence G is a set-up sentence that uses quotes to show exactly what part 2
2345678901234567890123456789012123456789012345678901
12345678901234567890123456789012123456789012345678901
of the argument is going to be scrutinized. Sentence H shows, “Hey, dude, 2
2
12345678901234567890123456789012123456789012345678901 2
12345678901234567890123456789012123456789012345678901
I can totally see where you’re coming from.” The other side of the issue has 2
12345678901234567890123456789012123456789012345678901
1 now been given a little credit. The rest of the paragraph proceeds to show 2
12345678901234567890123456789012123456789012345678901 2
12345678901234567890123456789012123456789012345678901 2
12345678901234567890123456789012123456789012345678901
why this point of view is flawed. Sentence I is a transition sentence, as you 2
12345678901234567890123456789012123456789012345678901
go from agreeing to disagreeing. The next four sentences follow the same 2
12345678901234567890123456789012123456789012345678901 2
2345678901234567890123456789012123456789012345678901
12345678901234567890123456789012123456789012345678901 2
1 blueprint as the first paragraph. Sentences J and K create a hypothetical 2
12345678901234567890123456789012123456789012345678901
situation—which you can totally make up so long as it’s somewhat believ- 2
12345678901234567890123456789012123456789012345678901 2
12345678901234567890123456789012123456789012345678901
2345678901234567890123456789012123456789012345678901
2
2
1 able—and Sentences L and M show how this hypothetical situation will
12345678901234567890123456789012123456789012345678901 2
12345678901234567890123456789012123456789012345678901 22
lead to an outcome exactly opposite of what the Dextrose Bank President
12345678901234567890123456789012123456789012345678901 243
123456789012345678901234567890121234567890123456789012
Part III: Section Review

123456789012345678901234567890121234567890123456789012
12345678901234567890123456789012123456789012345678901 2
12345678901234567890123456789012123456789012345678901 2
12345678901234567890123456789012123456789012345678901
assumes will happen. Not every sentence in this second paragraph is stellar, 2
12345678901234567890123456789012123456789012345678901 2
12345678901234567890123456789012123456789012345678901
and if you had time, you might not want to use both radio and print ads. 2
12345678901234567890123456789012123456789012345678901 2
12345678901234567890123456789012123456789012345678901
Using only one would simplify the paragraph and still get the point across. 2
1 But you don’t have time, so it doesn’t matter. The second paragraph is fine 2
2345678901234567890123456789012123456789012345678901
12345678901234567890123456789012123456789012345678901 2
12345678901234567890123456789012123456789012345678901 2
12345678901234567890123456789012123456789012345678901
as it is for a 30-minute “Analyze the Argument” essay. 2
12345678901234567890123456789012123456789012345678901 2
1 After two paragraphs, you might not have much time left. Following the 2
2345678901234567890123456789012123456789012345678901
12345678901234567890123456789012123456789012345678901 2
12345678901234567890123456789012123456789012345678901 2
12345678901234567890123456789012123456789012345678901
template of the first essay, write a brief closing and then use any remaining 2
12345678901234567890123456789012123456789012345678901
2345678901234567890123456789012123456789012345678901
2
2
1 time to hammer out more alternatives. Your closing might be something
12345678901234567890123456789012123456789012345678901 2
12345678901234567890123456789012123456789012345678901
like the following: 2
12345678901234567890123456789012123456789012345678901 2
12345678901234567890123456789012123456789012345678901
2345678901234567890123456789012123456789012345678901
2
2
1 The Dextrose Bank PresidentÕs plan to gain more new walk-in
12345678901234567890123456789012123456789012345678901 2
12345678901234567890123456789012123456789012345678901
customers might work. However, if it doesnÕt, it could be 2
12345678901234567890123456789012123456789012345678901 2
12345678901234567890123456789012123456789012345678901
because of one of the reasons listed above, since the 2
2345678901234567890123456789012123456789012345678901
12345678901234567890123456789012123456789012345678901 2
12345678901234567890123456789012123456789012345678901
PresidentÕs plan is by no means foolproof. 2
12345678901234567890123456789012123456789012345678901 2
12345678901234567890123456789012123456789012345678901 2
12345678901234567890123456789012123456789012345678901 2
12345678901234567890123456789012123456789012345678901 2
1 The above closes out the essay nicely. The vague phrase “it could be 2
12345678901234567890123456789012123456789012345678901
because of one of the reasons listed” works quite well, because you 2
12345678901234567890123456789012123456789012345678901 2
12345678901234567890123456789012123456789012345678901
wouldn’t be expected to list every one of the reasons in the closing. You 2
12345678901234567890123456789012123456789012345678901 2
2345678901234567890123456789012123456789012345678901
can now use all the time remaining to add as many reasons as you can type. 2
12345678901234567890123456789012123456789012345678901
12345678901234567890123456789012123456789012345678901 2
12345678901234567890123456789012123456789012345678901
When the clock starts flashing 3 minutes, finish up the sentence you are 2
2
12345678901234567890123456789012123456789012345678901
writing and then look over the essay for any grammatical mistakes. 2
12345678901234567890123456789012123456789012345678901
12345678901234567890123456789012123456789012345678901 2
12345678901234567890123456789012123456789012345678901 2
12345678901234567890123456789012123456789012345678901 2
Correct any mistakes you can find until the clock winds down. Once the
1 time is up, the essay will vanish, and the Analytical portion of the GRE will 2
12345678901234567890123456789012123456789012345678901 2
12345678901234567890123456789012123456789012345678901 2
12345678901234567890123456789012123456789012345678901 2
be over.
12345678901234567890123456789012123456789012345678901 2
12345678901234567890123456789012123456789012345678901 2
12345678901234567890123456789012123456789012345678901 2
12345678901234567890123456789012123456789012345678901 2
12345678901234567890123456789012123456789012345678901
. . . And One More Thing 2
12345678901234567890123456789012123456789012345678901 2
12345678901234567890123456789012123456789012345678901
The following words were used in this chapter. They are all college-level 2
12345678901234567890123456789012123456789012345678901 2
2345678901234567890123456789012123456789012345678901
12345678901234567890123456789012123456789012345678901 2
1 vocabulary words that might appear on the Verbal portion of the GRE. If 2
12345678901234567890123456789012123456789012345678901
you didn’t know the meaning of these words, add them to your vocabulary 2
12345678901234567890123456789012123456789012345678901 2
2345678901234567890123456789012123456789012345678901
lists now. The best way to improve your vocabulary is to learn new words 2
12345678901234567890123456789012123456789012345678901
12345678901234567890123456789012123456789012345678901
in the context of reading, so find the definitions of all the words below and 2
2
12345678901234567890123456789012123456789012345678901 2
12345678901234567890123456789012123456789012345678901 2
1 then look back and find them throughout the chapter:
12345678901234567890123456789012123456789012345678901 2
2345678901234567890123456789012123456789012345678901
12345678901234567890123456789012123456789012345678901 2 2
12345678901234567890123456789012123456789012345678901 2
12345678901234567890123456789012123456789012345678901
cad proverbial
2
1
12345678901234567890123456789012123456789012345678901 2
2345678901234567890123456789012123456789012345678901
cantankerous renaissance
2
12345678901234567890123456789012123456789012345678901 2
12345678901234567890123456789012123456789012345678901
endeavor rubric
2
12345678901234567890123456789012123456789012345678901 2
1 interminable scrutiny (three different times!)
2345678901234567890123456789012123456789012345678901
12345678901234567890123456789012123456789012345678901 2
1 loquacious sophistic 2
12345678901234567890123456789012123456789012345678901 2
12345678901234567890123456789012123456789012345678901 2
moniker
12345678901234567890123456789012123456789012345678901 2
12345678901234567890123456789012123456789012345678901 2
12345678901234567890123456789012123456789012345678901 2
12345678901234567890123456789012123456789012345678901 2 2
244 12345678901234567890123456789012123456789012345678901
123456789012345678901234567890121234567890123456789012

www.petersons.com
Take It to
the Next Level
123456789012345678901234567890121234567890123456789012
12345678901234567890123456789012123456789012345678901 2
12345678901234567890123456789012123456789012345678901
2345678901234567890123456789012123456789012345678901
2
2
1
Acing the Essays
12345678901234567890123456789012123456789012345678901 2
12345678901234567890123456789012123456789012345678901 2
12345678901234567890123456789012123456789012345678901
The fundamentals of sound essay writing are covered earlier in this 2
12345678901234567890123456789012123456789012345678901 2
12345678901234567890123456789012123456789012345678901 22
2345678901234567890123456789012123456789012345678901
chapter. If you follow those basic steps—clearly stated main idea in first
12345678901234567890123456789012123456789012345678901
paragraph of first essay, finding the flaw in the argument for the second 2
12345678901234567890123456789012123456789012345678901
essay, and so forth—you should get a decent score on the Analytical 2
12345678901234567890123456789012123456789012345678901
12345678901234567890123456789012123456789012345678901 2
12345678901234567890123456789012123456789012345678901 2
1 portion of the GRE. However, if you are an English major or someone 2
12345678901234567890123456789012123456789012345678901
shooting to get into a highly competitive program that stresses writing 2
12345678901234567890123456789012123456789012345678901 2
12345678901234567890123456789012123456789012345678901
skills, a decent score might not be all that you want. Anything less then a 5 2
12345678901234567890123456789012123456789012345678901 22
2345678901234567890123456789012123456789012345678901
12345678901234567890123456789012123456789012345678901
might not be enough; you want a 5.5 or ideally a 6.
12345678901234567890123456789012123456789012345678901 2
12345678901234567890123456789012123456789012345678901
There’s no way to guarantee you a perfect score. Essays are just too 2
2
12345678901234567890123456789012123456789012345678901
12345678901234567890123456789012123456789012345678901
subjective, and one reader’s idea of an excellent essay might differ from 2
2
12345678901234567890123456789012123456789012345678901
12345678901234567890123456789012123456789012345678901
what another reader thinks a perfect essay should look like. But there are a 2
2
12345678901234567890123456789012123456789012345678901
1 host of different strategies you can use to transform a good essay into a 2
12345678901234567890123456789012123456789012345678901 2
12345678901234567890123456789012123456789012345678901 2
12345678901234567890123456789012123456789012345678901
superior essay, and these strategies will be covered in this chapter. 2
12345678901234567890123456789012123456789012345678901 2
12345678901234567890123456789012123456789012345678901 2
12345678901234567890123456789012123456789012345678901 2
12345678901234567890123456789012123456789012345678901 2
12345678901234567890123456789012123456789012345678901
Mixing It up on the First Essay 2
12345678901234567890123456789012123456789012345678901 2
12345678901234567890123456789012123456789012345678901 2
12345678901234567890123456789012123456789012345678901 22
2345678901234567890123456789012123456789012345678901
Since the first essay is longer and requires more mental work than the
12345678901234567890123456789012123456789012345678901
1 second, it stands to reason that this essay affects your overall Analytical 2
12345678901234567890123456789012123456789012345678901
score more than the second essay. The first essay is also more open-ended, 2
12345678901234567890123456789012123456789012345678901 2
2345678901234567890123456789012123456789012345678901
which gives you a greater range of creativity on this essay than on the 2
12345678901234567890123456789012123456789012345678901
12345678901234567890123456789012123456789012345678901 2
12345678901234567890123456789012123456789012345678901 2
12345678901234567890123456789012123456789012345678901 22
second. For the second essay, the goal is to “analyze the argument,” and
1 this limits your responses since you must stay focused on the argument
12345678901234567890123456789012123456789012345678901 2
12345678901234567890123456789012123456789012345678901
given. This set-up is good news for you, because it means that you can 2
12345678901234567890123456789012123456789012345678901 2
12345678901234567890123456789012123456789012345678901
demonstrate your snazzy writing skills on the essay that counts more 2
12345678901234567890123456789012123456789012345678901 2
12345678901234567890123456789012123456789012345678901
toward your overall score. 2
12345678901234567890123456789012123456789012345678901 2
12345678901234567890123456789012123456789012345678901 2
12345678901234567890123456789012123456789012345678901
Before delving into specific techniques to improve your essay, the general 2
12345678901234567890123456789012123456789012345678901 2
12345678901234567890123456789012123456789012345678901
theme you should shoot for is to write a “standard essay, but with frills.” 2
12345678901234567890123456789012123456789012345678901 2
12345678901234567890123456789012123456789012345678901
Think of a perfect score of 6 as a 4 1 2. The 4 comes from having all the 2
12345678901234567890123456789012123456789012345678901
basic requirements needed in an essay, such as a main idea supported by 2
12345678901234567890123456789012123456789012345678901 2
1 details. Every essay needs the basics, and yours is no exception. Yet most 2
2345678901234567890123456789012123456789012345678901
12345678901234567890123456789012123456789012345678901 2
12345678901234567890123456789012123456789012345678901 2
12345678901234567890123456789012123456789012345678901 2
123456789012345678901234567890121234567890123456789012
245
Part III: Section Review

123456789012345678901234567890121234567890123456789012
12345678901234567890123456789012123456789012345678901 2
12345678901234567890123456789012123456789012345678901 2
12345678901234567890123456789012123456789012345678901
students are familiar with the basic essay format, so just writing a standard 2
12345678901234567890123456789012123456789012345678901 2
12345678901234567890123456789012123456789012345678901
essay might not be enough to get you a 6. That’s where the “12” idea 2
12345678901234567890123456789012123456789012345678901 2
12345678901234567890123456789012123456789012345678901
comes in. The extra 2 points comes from distinguishing your essay in some 2
1 way. Your overall goal is to make your essay different in a way that shows 2
2345678901234567890123456789012123456789012345678901
12345678901234567890123456789012123456789012345678901 2
12345678901234567890123456789012123456789012345678901 2
12345678901234567890123456789012123456789012345678901
you are capable of writing more than a standard essay. 2
12345678901234567890123456789012123456789012345678901
2345678901234567890123456789012123456789012345678901
2
2
1
12345678901234567890123456789012123456789012345678901 2
12345678901234567890123456789012123456789012345678901
The “4 1 2” concept seems to imply that simply writing a basic essay will 2
12345678901234567890123456789012123456789012345678901 2
12345678901234567890123456789012123456789012345678901
never get you a score higher than a 4. This is not its purpose, nor is the 2
1 implication true. If you totally nail a basic essay—a clear main idea, great 2
2345678901234567890123456789012123456789012345678901
12345678901234567890123456789012123456789012345678901 2
12345678901234567890123456789012123456789012345678901 2
12345678901234567890123456789012123456789012345678901 2

Note
set of supporting details, excellent organization on the paragraph and
12345678901234567890123456789012123456789012345678901
2345678901234567890123456789012123456789012345678901
2
2
1 sentence level, and good overall grammar—you could easily earn a 6 that
12345678901234567890123456789012123456789012345678901 2
12345678901234567890123456789012123456789012345678901
way. In the end, it all depends on the reader. The point behind the “4 1 2” 2
12345678901234567890123456789012123456789012345678901 2
12345678901234567890123456789012123456789012345678901
concept is to emphasize that adding advanced writing flourishes will help 2
2345678901234567890123456789012123456789012345678901
separate your essay from the standard essays. If you add these flourishes 2
12345678901234567890123456789012123456789012345678901
12345678901234567890123456789012123456789012345678901 2
12345678901234567890123456789012123456789012345678901 2
12345678901234567890123456789012123456789012345678901 2
well, it will greatly increase your chance of getting a 5 or 6 since you will
12345678901234567890123456789012123456789012345678901 2
12345678901234567890123456789012123456789012345678901
be demonstrating your advanced writing ability. 2
1 2
12345678901234567890123456789012123456789012345678901 2
12345678901234567890123456789012123456789012345678901 2
How you decide to make your essay stand
12345678901234567890123456789012123456789012345678901 out from the pack is up to you. 2
2345678901234567890123456789012123456789012345678901
12345678901234567890123456789012123456789012345678901
Here are three different methods that you can use: 2
12345678901234567890123456789012123456789012345678901 2
12345678901234567890123456789012123456789012345678901 2
12345678901234567890123456789012123456789012345678901 2
12345678901234567890123456789012123456789012345678901 2
12345678901234567890123456789012123456789012345678901 2
12345678901234567890123456789012123456789012345678901
1. Tweak the Basic Format 2
12345678901234567890123456789012123456789012345678901
It’s a simple fact that repetition leads to boredom for most people. You 2
2
12345678901234567890123456789012123456789012345678901
1 have to think that GRE readers get tired of seeing the same essays over and 2
12345678901234567890123456789012123456789012345678901 2
12345678901234567890123456789012123456789012345678901
over again. This doesn’t mean they’ll start giving out bad grades to 2
12345678901234567890123456789012123456789012345678901 2
12345678901234567890123456789012123456789012345678901 2
12345678901234567890123456789012123456789012345678901
standard essays. It means that they might
2345678901234567890123456789012123456789012345678901
perk up a bit if your essay plays 2
2
12345678901234567890123456789012123456789012345678901
12345678901234567890123456789012123456789012345678901
with the basic format a bit. 2
1 2
12345678901234567890123456789012123456789012345678901 2
12345678901234567890123456789012123456789012345678901
For instance, the basic essay starts with the main idea clearly stated in the 2
2345678901234567890123456789012123456789012345678901
first paragraph. So tweak the format by placing your main idea in the 2
12345678901234567890123456789012123456789012345678901
12345678901234567890123456789012123456789012345678901 2
1 second paragraph. Use the first paragraph to “set the scene,” vividly 2
12345678901234567890123456789012123456789012345678901 2
12345678901234567890123456789012123456789012345678901
describing an imaginary scenario that will back up your claim. Or start the 2
12345678901234567890123456789012123456789012345678901 2
12345678901234567890123456789012123456789012345678901
paragraph off with a good quote, provided the quote could then be used to 2
12345678901234567890123456789012123456789012345678901 2
12345678901234567890123456789012123456789012345678901
back up your point. Do something that captures the reader’s attention 2
12345678901234567890123456789012123456789012345678901 2
12345678901234567890123456789012123456789012345678901
while remaining relevant to the discussion at hand. 2
12345678901234567890123456789012123456789012345678901 2
12345678901234567890123456789012123456789012345678901 2
12345678901234567890123456789012123456789012345678901
Page 229 earlier in this chapter listed two fictitious essay topics. The first 2
12345678901234567890123456789012123456789012345678901
one has already been discussed, so we will use the second topic to take it to 2
12345678901234567890123456789012123456789012345678901 2
12345678901234567890123456789012123456789012345678901 2
12345678901234567890123456789012123456789012345678901
the next level. 2
12345678901234567890123456789012123456789012345678901 2
12345678901234567890123456789012123456789012345678901 2
12345678901234567890123456789012123456789012345678901 2
2345678901234567890123456789012123456789012345678901
2
1
12345678901234567890123456789012123456789012345678901 2
12345678901234567890123456789012123456789012345678901 2
12345678901234567890123456789012123456789012345678901 2
12345678901234567890123456789012123456789012345678901 2
12345678901234567890123456789012123456789012345678901 2
12345678901234567890123456789012123456789012345678901 2 2
246 12345678901234567890123456789012123456789012345678901
123456789012345678901234567890121234567890123456789012

www.petersons.com
Chapter 6: Analytical Writing Review

123456789012345678901234567890121234567890123456789012
12345678901234567890123456789012123456789012345678901 2
12345678901234567890123456789012123456789012345678901 2
12345678901234567890123456789012123456789012345678901
Sample Topic: 2
12345678901234567890123456789012123456789012345678901 2
12345678901234567890123456789012123456789012345678901
Quick decisions lead to success in today’s business world. 2
12345678901234567890123456789012123456789012345678901 2
12345678901234567890123456789012123456789012345678901 2
12345678901234567890123456789012123456789012345678901
Setting the scene with an imaginary scenario. (agrees with the topic) 2
12345678901234567890123456789012123456789012345678901 2
12345678901234567890123456789012123456789012345678901 2
12345678901234567890123456789012123456789012345678901
The owners of Murpu Labs face a quandary. Over the past 2
12345678901234567890123456789012123456789012345678901
2345678901234567890123456789012123456789012345678901
2
2
1 six months, theyÕve worked long hours developing a new
12345678901234567890123456789012123456789012345678901 2
12345678901234567890123456789012123456789012345678901
software program that makes it easier to transfer video 2
12345678901234567890123456789012123456789012345678901 2
12345678901234567890123456789012123456789012345678901
images over the Internet.
2345678901234567890123456789012123456789012345678901
This market is currently 2
1 underserved, and large profits are available for a company 2
12345678901234567890123456789012123456789012345678901 2
12345678901234567890123456789012123456789012345678901
that fulfills customer demands in this area. Yet the Murpu 2
12345678901234567890123456789012123456789012345678901 2
12345678901234567890123456789012123456789012345678901
software still has some bugs to
2345678901234567890123456789012123456789012345678901 be worked out. With start-up 2
2
1
12345678901234567890123456789012123456789012345678901
money drying up, the company owners find themselves 2
12345678901234567890123456789012123456789012345678901 2
12345678901234567890123456789012123456789012345678901
pondering the question, ÔDo we release the flawed software 2
12345678901234567890123456789012123456789012345678901 2
12345678901234567890123456789012123456789012345678901
or not?Õ 2
12345678901234567890123456789012123456789012345678901 2

Take It to the Next Level


12345678901234567890123456789012123456789012345678901 2
12345678901234567890123456789012123456789012345678901
MurpuÕs owners should stop deliberating and start selling, 2
12345678901234567890123456789012123456789012345678901 2
12345678901234567890123456789012123456789012345678901 22
2345678901234567890123456789012123456789012345678901
since speedy decisions are crucial in todayÕs business world.
1
12345678901234567890123456789012123456789012345678901
If you have knowledge of an actual business that hesitated and lost, by all 2
12345678901234567890123456789012123456789012345678901 2
12345678901234567890123456789012123456789012345678901 2
12345678901234567890123456789012123456789012345678901 22
2345678901234567890123456789012123456789012345678901
means use it. Using a historical anecdote here would be even better, since it
12345678901234567890123456789012123456789012345678901
would give your argument the weight of some real historical events behind 2
12345678901234567890123456789012123456789012345678901
it. However, a lack of precise business history will not hurt the stylistic 2
12345678901234567890123456789012123456789012345678901
2
12345678901234567890123456789012123456789012345678901
effects of the anecdotal first paragraph. You just create a fictitious 2
12345678901234567890123456789012123456789012345678901
12345678901234567890123456789012123456789012345678901 2
company and place them in the situation you want them to be stuck in. 2
12345678901234567890123456789012123456789012345678901
12345678901234567890123456789012123456789012345678901 22
1 The point is to give the first paragraph some punch by creating a specific
12345678901234567890123456789012123456789012345678901 2
12345678901234567890123456789012123456789012345678901
situation, one that draws the readers in and has them wondering how they 2
12345678901234567890123456789012123456789012345678901 2
12345678901234567890123456789012123456789012345678901
would act if they were running Murpu Labs. 2
12345678901234567890123456789012123456789012345678901
2345678901234567890123456789012123456789012345678901
2
The sentence that starts the second paragraph, “Murpu’s owners should 2
12345678901234567890123456789012123456789012345678901
12345678901234567890123456789012123456789012345678901 2
1 stop deliberating and start selling, since speedy decisions are crucial in 2
12345678901234567890123456789012123456789012345678901 2
12345678901234567890123456789012123456789012345678901 2
12345678901234567890123456789012123456789012345678901 22
2345678901234567890123456789012123456789012345678901
today’s business world” ties into the first paragraph and shows just where
12345678901234567890123456789012123456789012345678901
1 you stand on the topic. It’s your viewpoint, cleverly placed in the second 2
12345678901234567890123456789012123456789012345678901
paragraph, not the first. You can then expound upon this theme for the 2
12345678901234567890123456789012123456789012345678901 2
2345678901234567890123456789012123456789012345678901
12345678901234567890123456789012123456789012345678901 2
12345678901234567890123456789012123456789012345678901
rest of the second paragraph. 2
12345678901234567890123456789012123456789012345678901 22
12345678901234567890123456789012123456789012345678901
1 Since you started out with software as your business model, you can make 2
12345678901234567890123456789012123456789012345678901
that the I-see-the-other-side-of-the-coin angle for the third paragraph, too. 2
12345678901234567890123456789012123456789012345678901 2
12345678901234567890123456789012123456789012345678901
Speedy decisions are critical in the fast-paced computer 2
12345678901234567890123456789012123456789012345678901 2
12345678901234567890123456789012123456789012345678901 2
12345678901234567890123456789012123456789012345678901 22
2345678901234567890123456789012123456789012345678901
software world, but itÕs true that other business areas are not
12345678901234567890123456789012123456789012345678901
so time-intensive. Banking, and institutional lending in general, 2
12345678901234567890123456789012123456789012345678901
1 are two areas in which deliberate decision making are well 2
12345678901234567890123456789012123456789012345678901 2
2345678901234567890123456789012123456789012345678901
established. Yet technology has a way of speeding up the pace 2
12345678901234567890123456789012123456789012345678901
1 2
12345678901234567890123456789012123456789012345678901 2
12345678901234567890123456789012123456789012345678901 2
of the world, and even such ancient sectors as banking might
12345678901234567890123456789012123456789012345678901
find themselves pushed to make decisions more rapidly than 2
12345678901234567890123456789012123456789012345678901 2
12345678901234567890123456789012123456789012345678901 2
12345678901234567890123456789012123456789012345678901 22
they would have in the past.
12345678901234567890123456789012123456789012345678901 247
123456789012345678901234567890121234567890123456789012
Part III: Section Review

123456789012345678901234567890121234567890123456789012
12345678901234567890123456789012123456789012345678901 2
12345678901234567890123456789012123456789012345678901 2
12345678901234567890123456789012123456789012345678901 2
12345678901234567890123456789012123456789012345678901
In the sample essay outlined earlier, only one paragraph is devoted to 2
12345678901234567890123456789012123456789012345678901 2
12345678901234567890123456789012123456789012345678901 2
12345678901234567890123456789012123456789012345678901
showing the other side’s point and then shooting it down. You can alter 2
1 this standard essay format by creating every paragraph in the “I see your 2
2345678901234567890123456789012123456789012345678901
12345678901234567890123456789012123456789012345678901 2

Tip
12345678901234567890123456789012123456789012345678901
side, but mine’s stronger” mold. Each paragraph would start off with you 2
12345678901234567890123456789012123456789012345678901 2
12345678901234567890123456789012123456789012345678901
stating your opponent’s viewpoint, but would then explain why your side 2
2345678901234567890123456789012123456789012345678901 2
1
12345678901234567890123456789012123456789012345678901
of the argument is better. This is a little tougher to pull off in a short 2
12345678901234567890123456789012123456789012345678901 2
12345678901234567890123456789012123456789012345678901
amount of time, which is why it would make your essay stand out. 2
12345678901234567890123456789012123456789012345678901
2345678901234567890123456789012123456789012345678901
2
2
1
12345678901234567890123456789012123456789012345678901 2
12345678901234567890123456789012123456789012345678901
The above paragraph shows how you can alter the initial outline of an 2
12345678901234567890123456789012123456789012345678901 2
12345678901234567890123456789012123456789012345678901
essay just a bit, and still keep the rest of it the same. As always, you use 2
1 vague yet plausible facts that cannot be disputed. The phrase “institutional 2
2345678901234567890123456789012123456789012345678901
12345678901234567890123456789012123456789012345678901 2
12345678901234567890123456789012123456789012345678901 2
12345678901234567890123456789012123456789012345678901
lending in general” sounds very good and could mean such a huge range of 2
12345678901234567890123456789012123456789012345678901 2
12345678901234567890123456789012123456789012345678901
things it’s not even funny. In a time-intensive essay where readers can’t 2
12345678901234567890123456789012123456789012345678901 2
12345678901234567890123456789012123456789012345678901
pause and deeply ponder, it fits wonderfully. 2
12345678901234567890123456789012123456789012345678901 2
12345678901234567890123456789012123456789012345678901 2
12345678901234567890123456789012123456789012345678901 2
2345678901234567890123456789012123456789012345678901
Instead of starting out with a scenario, you could also begin with a quote.
2
1
12345678901234567890123456789012123456789012345678901
Starting things off with a quote. (disagrees with the topic) 2
12345678901234567890123456789012123456789012345678901 2
12345678901234567890123456789012123456789012345678901 2
2345678901234567890123456789012123456789012345678901
Edward Young once wrote, ÒBe wise with speed; a fool at forty 2
12345678901234567890123456789012123456789012345678901
12345678901234567890123456789012123456789012345678901 2
is a fool indeed.Ó Although the English poet and dramatist lived 2
12345678901234567890123456789012123456789012345678901
12345678901234567890123456789012123456789012345678901
over three hundred years ago, YoungÕs words are still relevant 2
2
12345678901234567890123456789012123456789012345678901
today. They highlight the fact that rash decisions are often 2
12345678901234567890123456789012123456789012345678901
12345678901234567890123456789012123456789012345678901 2
bad decisions, and such foolishness leaves people open to 2
12345678901234567890123456789012123456789012345678901
12345678901234567890123456789012123456789012345678901 2
1 2
12345678901234567890123456789012123456789012345678901 2
2345678901234567890123456789012123456789012345678901
embarrassment and other unwanted consequences.
12345678901234567890123456789012123456789012345678901 2
1 Quick decisions can also translate to bad business, which is 2
12345678901234567890123456789012123456789012345678901 2
12345678901234567890123456789012123456789012345678901 2
12345678901234567890123456789012123456789012345678901 2
2345678901234567890123456789012123456789012345678901
why they should be avoided in todayÕs business world.
12345678901234567890123456789012123456789012345678901 2
1 Now, the main problem with this set-up is that you have to have the quote 2
12345678901234567890123456789012123456789012345678901 2
12345678901234567890123456789012123456789012345678901
from Edward Young floating around in your head in order to use it on the 2
12345678901234567890123456789012123456789012345678901 2
2345678901234567890123456789012123456789012345678901
12345678901234567890123456789012123456789012345678901 2
1 Analytical section. There are three ways to go about achieving this. 2
12345678901234567890123456789012123456789012345678901 2
12345678901234567890123456789012123456789012345678901 2
Starting with a quote or adding fictitious scenarios are just two ways to
2345678901234567890123456789012123456789012345678901
12345678901234567890123456789012123456789012345678901
tweak the basic essay format. Anything you do that demonstrates you can 2
2
12345678901234567890123456789012123456789012345678901 2
12345678901234567890123456789012123456789012345678901
write more than just a standard essay will improve your chances of 2
12345678901234567890123456789012123456789012345678901
1 garnering a high score on the Analytical portion of the GRE. Admittedly, 2
12345678901234567890123456789012123456789012345678901 2
12345678901234567890123456789012123456789012345678901 2
12345678901234567890123456789012123456789012345678901
you have to be able to pull off the altered essay format successfully, so 2
12345678901234567890123456789012123456789012345678901 2
12345678901234567890123456789012123456789012345678901 2
don’t add change just for change’s sake. Adopting a writing style that is
2345678901234567890123456789012123456789012345678901
obviously artificial will not help your cause, so never strain or stretch to 2
12345678901234567890123456789012123456789012345678901
12345678901234567890123456789012123456789012345678901 2
12345678901234567890123456789012123456789012345678901 2
12345678901234567890123456789012123456789012345678901 2
place a quote just to give your essay a quote. If tweaking the essay format
1 in some way feels good to you, do it. Your instincts are better than you 2
12345678901234567890123456789012123456789012345678901 2
12345678901234567890123456789012123456789012345678901 2
12345678901234567890123456789012123456789012345678901
might think, and a more natural writing style can greatly benefit your 2
12345678901234567890123456789012123456789012345678901 2
12345678901234567890123456789012123456789012345678901
essay, as you will see in the following section.
2345678901234567890123456789012123456789012345678901
2
2
1
12345678901234567890123456789012123456789012345678901 2
12345678901234567890123456789012123456789012345678901 2 2
248 12345678901234567890123456789012123456789012345678901
123456789012345678901234567890121234567890123456789012

www.petersons.com
Chapter 6: Analytical Writing Review

123456789012345678901234567890121234567890123456789012
12345678901234567890123456789012123456789012345678901 2
12345678901234567890123456789012123456789012345678901 2
12345678901234567890123456789012123456789012345678901 2
12345678901234567890123456789012123456789012345678901 2
12345678901234567890123456789012123456789012345678901
Three Ways to Have Snappy Quotes Floating around in 2
12345678901234567890123456789012123456789012345678901 2
12345678901234567890123456789012123456789012345678901
Your Head
2345678901234567890123456789012123456789012345678901
2
2
1
12345678901234567890123456789012123456789012345678901 2
12345678901234567890123456789012123456789012345678901
1. Be the Kind of Person Who Likes to Memorize Quotes. Many 2
12345678901234567890123456789012123456789012345678901 2
12345678901234567890123456789012123456789012345678901
English majors like nothing more than to snap off a few lines
2345678901234567890123456789012123456789012345678901
2
2
1
12345678901234567890123456789012123456789012345678901
of Shakespeare to prove their point. You might find this habit 2
12345678901234567890123456789012123456789012345678901 2
12345678901234567890123456789012123456789012345678901
annoying in the real world, but in GRE-Land it’s quite a useful 2
12345678901234567890123456789012123456789012345678901
2345678901234567890123456789012123456789012345678901
2
2
1 habit. Even if you’re not an English major, many people have
12345678901234567890123456789012123456789012345678901 2
12345678901234567890123456789012123456789012345678901
proverbs (like “Haste makes waste”), Biblical quotes, and 2
12345678901234567890123456789012123456789012345678901 2
12345678901234567890123456789012123456789012345678901
snippets of famous speeches floating
2345678901234567890123456789012123456789012345678901
around in their grey mat- 2
2
1
12345678901234567890123456789012123456789012345678901
ter. These quotes can all be used if the situation warrants it. 2
12345678901234567890123456789012123456789012345678901 2
12345678901234567890123456789012123456789012345678901 2
12345678901234567890123456789012123456789012345678901
One caveat is that you want to use quotes that are as “literary” 2
2345678901234567890123456789012123456789012345678901
12345678901234567890123456789012123456789012345678901 2
12345678901234567890123456789012123456789012345678901
as possible. Shakespeare is good but John Dryden is better, 2

Take It to the Next Level


12345678901234567890123456789012123456789012345678901 2
12345678901234567890123456789012123456789012345678901 22
since a Dryden quote shows that you have knowledge of
12345678901234567890123456789012123456789012345678901
famous literature that goes beyond the basics. If you need help
12345678901234567890123456789012123456789012345678901 2
1 remembering this idea, think of the acronym FIBTS: Faulkner 2
12345678901234567890123456789012123456789012345678901 2
12345678901234567890123456789012123456789012345678901
Is Better Than Seuss. 2
12345678901234567890123456789012123456789012345678901 2
2345678901234567890123456789012123456789012345678901
12345678901234567890123456789012123456789012345678901 2
12345678901234567890123456789012123456789012345678901
2. Start a List of Cool Quotes You Like. This method works best 2
12345678901234567890123456789012123456789012345678901 2
12345678901234567890123456789012123456789012345678901
if you are someone who reads literature on a regular basis. If 2
12345678901234567890123456789012123456789012345678901 2
12345678901234567890123456789012123456789012345678901 22
you do, you should come across neat phrases that make you
12345678901234567890123456789012123456789012345678901
12345678901234567890123456789012123456789012345678901
think to yourself, “Hey! That sentence rocks!” Start a quote 2
12345678901234567890123456789012123456789012345678901 2
1 list where you write down these good phrases. The list will 2
2345678901234567890123456789012123456789012345678901
12345678901234567890123456789012123456789012345678901 2
12345678901234567890123456789012123456789012345678901
help you remember the quote, and you can always refresh 2
1 2
12345678901234567890123456789012123456789012345678901 2
your memory by rereading the list from time to time.
12345678901234567890123456789012123456789012345678901 2
12345678901234567890123456789012123456789012345678901 2
12345678901234567890123456789012123456789012345678901
3. Memorize a Bunch of Quotes for the GRE. This might not be 2
12345678901234567890123456789012123456789012345678901
the best use of your time, but to achieve it, all you need is a 2
12345678901234567890123456789012123456789012345678901 2
12345678901234567890123456789012123456789012345678901
book of famous quotes and a computer connected to the 2
12345678901234567890123456789012123456789012345678901 22
2345678901234567890123456789012123456789012345678901
12345678901234567890123456789012123456789012345678901
Internet. Use the computer to scroll around and view all the
2
1
12345678901234567890123456789012123456789012345678901
different GRE essay topics, and use the quote book to 2
12345678901234567890123456789012123456789012345678901 2
2345678901234567890123456789012123456789012345678901
12345678901234567890123456789012123456789012345678901
rummage for quotes that you think would work well for each 2
12345678901234567890123456789012123456789012345678901 2
12345678901234567890123456789012123456789012345678901
topic. This idea is part of a larger plan to prep for the essay 2
12345678901234567890123456789012123456789012345678901 2
1 section, and it will be discussed in greater detail starting on 2
2345678901234567890123456789012123456789012345678901
12345678901234567890123456789012123456789012345678901 2
12345678901234567890123456789012123456789012345678901
page 251. 2
12345678901234567890123456789012123456789012345678901 22
12345678901234567890123456789012123456789012345678901 2
1
12345678901234567890123456789012123456789012345678901 2
12345678901234567890123456789012123456789012345678901
2. Use Your Voice 2
12345678901234567890123456789012123456789012345678901 2
12345678901234567890123456789012123456789012345678901
Writing in your voice is the best way to help distinguish your essay from 2
12345678901234567890123456789012123456789012345678901 2
12345678901234567890123456789012123456789012345678901 22
2345678901234567890123456789012123456789012345678901
others, but it’s the hardest one to achieve. There’s no way to really teach
1 voice, either, since a person’s voice must come from the individual.
12345678901234567890123456789012123456789012345678901 2
12345678901234567890123456789012123456789012345678901 2
12345678901234567890123456789012123456789012345678901
However, the idea of voice can be discussed
2345678901234567890123456789012123456789012345678901
and described, and you can 2
2
1 take it from there and run with it as long as you can.
12345678901234567890123456789012123456789012345678901 2
12345678901234567890123456789012123456789012345678901 2
12345678901234567890123456789012123456789012345678901 2 249
123456789012345678901234567890121234567890123456789012
Part III: Section Review

123456789012345678901234567890121234567890123456789012
12345678901234567890123456789012123456789012345678901 2
12345678901234567890123456789012123456789012345678901 2
12345678901234567890123456789012123456789012345678901 2
12345678901234567890123456789012123456789012345678901
In writing, “voice” refers to an author’s distinctive style of using words. 2
12345678901234567890123456789012123456789012345678901 2
12345678901234567890123456789012123456789012345678901 2
12345678901234567890123456789012123456789012345678901
Many established writers have such a unique voice that you can 2
1 determine when a piece of writing is theirs after a couple of sentences. 2
2345678901234567890123456789012123456789012345678901
12345678901234567890123456789012123456789012345678901 2
12345678901234567890123456789012123456789012345678901
Suppose you read the words, “Round about there/In the Valley of 2

Tip
12345678901234567890123456789012123456789012345678901 2
12345678901234567890123456789012123456789012345678901
Crair/Lived an old Hoobavax/By the name of Snare” and were asked to 2
2345678901234567890123456789012123456789012345678901 2
1
12345678901234567890123456789012123456789012345678901
determine the writer. You would probably guess the well-known 2
12345678901234567890123456789012123456789012345678901 2
12345678901234567890123456789012123456789012345678901
children’s author Dr. Seuss, since these words are written in the voice of 2
12345678901234567890123456789012123456789012345678901 2
12345678901234567890123456789012123456789012345678901
Dr. Seuss. 2
12345678901234567890123456789012123456789012345678901 2
12345678901234567890123456789012123456789012345678901 2
12345678901234567890123456789012123456789012345678901 2
12345678901234567890123456789012123456789012345678901
Take a moment and think about your own writing. Look over some old 2
1 journal entries, term papers, poems, or short stories, if you have any laying 2
2345678901234567890123456789012123456789012345678901
12345678901234567890123456789012123456789012345678901 2
12345678901234567890123456789012123456789012345678901 2
12345678901234567890123456789012123456789012345678901
around. Is your writing very serious, or do you include humor whenever 2
12345678901234567890123456789012123456789012345678901 2
12345678901234567890123456789012123456789012345678901
possible? Do you prefer a short, terse journalistic style full of short 2
12345678901234567890123456789012123456789012345678901
sentences in small paragraphs, or do you tend toward lengthy paragraphs 2
12345678901234567890123456789012123456789012345678901 2
12345678901234567890123456789012123456789012345678901 2
12345678901234567890123456789012123456789012345678901
with long, highly developed sentences? No answer is better than another. 2
2345678901234567890123456789012123456789012345678901
12345678901234567890123456789012123456789012345678901 2
1 The overarching goal is to differentiate your essay from the pack (in a 2
12345678901234567890123456789012123456789012345678901
good way), and using a style that’s your own is an excellent way to do this. 2
12345678901234567890123456789012123456789012345678901 2
12345678901234567890123456789012123456789012345678901 2
2345678901234567890123456789012123456789012345678901
Like everything, using your voice has some qualifiers attached to it. Not 2
12345678901234567890123456789012123456789012345678901
12345678901234567890123456789012123456789012345678901 2
12345678901234567890123456789012123456789012345678901
every type of voice can be used for an essay. For an example of an inap- 2
12345678901234567890123456789012123456789012345678901 2
propriate voice, suppose you like to write in a style similar to the French 2
12345678901234567890123456789012123456789012345678901
12345678901234567890123456789012123456789012345678901 2
writer Celine. Your essay would be a mishmash of thoughts connected by 2
12345678901234567890123456789012123456789012345678901
12345678901234567890123456789012123456789012345678901 2
12345678901234567890123456789012123456789012345678901 2
1 ellipses . . . like hearing a story through a strobe light . . . every other phrase 2
2345678901234567890123456789012123456789012345678901
making its way through—others, blacked out . . . Mon Dieu! That’s a 2
12345678901234567890123456789012123456789012345678901
12345678901234567890123456789012123456789012345678901 2
1 terrible style to use for an essay. The reader will not say,”Hmm, an homage 2
12345678901234567890123456789012123456789012345678901 2
12345678901234567890123456789012123456789012345678901
to Celine!” The response will be more along the lines of, “This clown doesn’t 2
2345678901234567890123456789012123456789012345678901 2
12345678901234567890123456789012123456789012345678901
know when to use a period.” As you can see, some styles are just too radical 2
12345678901234567890123456789012123456789012345678901
1 to use on the essay format. If you are gunning for a 6, you probably have 2
12345678901234567890123456789012123456789012345678901 2
12345678901234567890123456789012123456789012345678901 2
12345678901234567890123456789012123456789012345678901 2
2345678901234567890123456789012123456789012345678901
enough of a writing background to know when you’ve crossed the line
12345678901234567890123456789012123456789012345678901 2
1 from strong voice into experimental text creation. 2
12345678901234567890123456789012123456789012345678901 2
12345678901234567890123456789012123456789012345678901
On a personal note, I like to make jokes while writing. (I won’t go so far 2
12345678901234567890123456789012123456789012345678901 2
12345678901234567890123456789012123456789012345678901 2
12345678901234567890123456789012123456789012345678901
and claim they are funny.) I could have written an essay without humor, 2
12345678901234567890123456789012123456789012345678901 2
12345678901234567890123456789012123456789012345678901 2
but I didn’t see the point in doing so. While the entire text wasn’t a
2345678901234567890123456789012123456789012345678901
stand-up routine, I told about one joke per paragraph, for both essays. My 2
12345678901234567890123456789012123456789012345678901
12345678901234567890123456789012123456789012345678901 2
conclusion on the first essay was even a joke. I did this because that’s the 2
12345678901234567890123456789012123456789012345678901
12345678901234567890123456789012123456789012345678901 2
1 kind of writer I am. My score on the Analytical portion was a 6, which is 2
12345678901234567890123456789012123456789012345678901 2
12345678901234567890123456789012123456789012345678901
good news for you because you probably wouldn’t want to be reading a 2
12345678901234567890123456789012123456789012345678901 2
12345678901234567890123456789012123456789012345678901
book from someone who scored a 3. You can say I received a 6 because: 2
12345678901234567890123456789012123456789012345678901 2
2345678901234567890123456789012123456789012345678901
12345678901234567890123456789012123456789012345678901 2
1 1. I’m a professional writer. 2
12345678901234567890123456789012123456789012345678901 2
12345678901234567890123456789012123456789012345678901 2
12345678901234567890123456789012123456789012345678901
2. I used my own voice, and this helped
2345678901234567890123456789012123456789012345678901
distinguish my essay from 2
2
1
12345678901234567890123456789012123456789012345678901
others. 2
12345678901234567890123456789012123456789012345678901 2 2
250 12345678901234567890123456789012123456789012345678901
123456789012345678901234567890121234567890123456789012

www.petersons.com
Chapter 6: Analytical Writing Review

123456789012345678901234567890121234567890123456789012
12345678901234567890123456789012123456789012345678901 2
12345678901234567890123456789012123456789012345678901 2
12345678901234567890123456789012123456789012345678901
3. I told jokes and the bored readers thought my jokes were funny, 2
12345678901234567890123456789012123456789012345678901 2
12345678901234567890123456789012123456789012345678901
or at least helped break up the monotony of their task. 2
12345678901234567890123456789012123456789012345678901 2
12345678901234567890123456789012123456789012345678901
There might an element of truth to all three points, but the second point is 2
12345678901234567890123456789012123456789012345678901 2
12345678901234567890123456789012123456789012345678901 2
12345678901234567890123456789012123456789012345678901
the one you should concern yourself with. Don’t feel you have to shackle 2
12345678901234567890123456789012123456789012345678901 2
12345678901234567890123456789012123456789012345678901
the way you write in order to conform to the standard essay. Make all the 2
1 standard essay points as necessary, and then garnish them with sentences 2
2345678901234567890123456789012123456789012345678901
12345678901234567890123456789012123456789012345678901 2
12345678901234567890123456789012123456789012345678901 2
12345678901234567890123456789012123456789012345678901
that are your own. 2
12345678901234567890123456789012123456789012345678901 2
1 While the strategy of using your own voice is a good one, it’s not 2
2345678901234567890123456789012123456789012345678901
12345678901234567890123456789012123456789012345678901 2
12345678901234567890123456789012123456789012345678901
something that is easily taught. For more concrete assistance, look no 2
12345678901234567890123456789012123456789012345678901 2
12345678901234567890123456789012123456789012345678901
further than the last technique, which pretty much reeks of realpolitik.
2345678901234567890123456789012123456789012345678901
2
2
1
12345678901234567890123456789012123456789012345678901 2
12345678901234567890123456789012123456789012345678901 2
12345678901234567890123456789012123456789012345678901 2
12345678901234567890123456789012123456789012345678901 2
12345678901234567890123456789012123456789012345678901 22
3.2345678901234567890123456789012123456789012345678901
Just Prepare Ahead of Time
12345678901234567890123456789012123456789012345678901

Take It to the Next Level


You know you have to write 2 essays, and you know where all the sample 2
12345678901234567890123456789012123456789012345678901
topics are located. It doesn’t take a rocket scientist—or person aspiring to 2
12345678901234567890123456789012123456789012345678901
12345678901234567890123456789012123456789012345678901 2
12345678901234567890123456789012123456789012345678901 2
1 go to rocket scientist graduate school—to put two and two together and 2
12345678901234567890123456789012123456789012345678901 2
12345678901234567890123456789012123456789012345678901
realize you can look over the topics and prepare ahead of time. 2
12345678901234567890123456789012123456789012345678901 2
2345678901234567890123456789012123456789012345678901
The question is not whether you should prepare ahead of time, but how 2
12345678901234567890123456789012123456789012345678901
12345678901234567890123456789012123456789012345678901 2
12345678901234567890123456789012123456789012345678901
much preparation do you wish to do? Writing out roughly 500 sample 2
12345678901234567890123456789012123456789012345678901 22
12345678901234567890123456789012123456789012345678901
essays would not be the best use of your time. Besides, other people have
12345678901234567890123456789012123456789012345678901 2
already done this for you. If you’re interested in seeing what some 2
12345678901234567890123456789012123456789012345678901
12345678901234567890123456789012123456789012345678901 22
12345678901234567890123456789012123456789012345678901
1 sample essays look like, you check out ARCO GRE CAT Answers to the 2
12345678901234567890123456789012123456789012345678901
Real Essay Questions. You don’t want to use the essays as they appear in 2
12345678901234567890123456789012123456789012345678901 2
12345678901234567890123456789012123456789012345678901
the book, since this is wrong for many different reasons. Instead, looking 2
12345678901234567890123456789012123456789012345678901 2
12345678901234567890123456789012123456789012345678901
over the sample essays will give you an idea of what kinds of supporting 2
2345678901234567890123456789012123456789012345678901 2
12345678901234567890123456789012123456789012345678901
details you could use for each essay. It will also give you a chance to 2
12345678901234567890123456789012123456789012345678901
1 look over different essay formats. Seeing different ways that supporting 2
12345678901234567890123456789012123456789012345678901 2
12345678901234567890123456789012123456789012345678901 2
12345678901234567890123456789012123456789012345678901 22
2345678901234567890123456789012123456789012345678901
details are used should help you brainstorm ideas of your own, while
12345678901234567890123456789012123456789012345678901
1 reviewing different formats should make it easier for you to organize 2
12345678901234567890123456789012123456789012345678901 2
12345678901234567890123456789012123456789012345678901
your own essay. 2
12345678901234567890123456789012123456789012345678901 22
2345678901234567890123456789012123456789012345678901
12345678901234567890123456789012123456789012345678901 2
12345678901234567890123456789012123456789012345678901 2
12345678901234567890123456789012123456789012345678901
Note

1 These books are by Peterson’s as well, although there might be similar 2


12345678901234567890123456789012123456789012345678901 2
12345678901234567890123456789012123456789012345678901
titles by other publishers. The books listed here are the ones rated highest 2
12345678901234567890123456789012123456789012345678901 2
12345678901234567890123456789012123456789012345678901
by the author and publisher, but that’s merely a coincidence. 2
12345678901234567890123456789012123456789012345678901 2
12345678901234567890123456789012123456789012345678901 22
2345678901234567890123456789012123456789012345678901
12345678901234567890123456789012123456789012345678901 2
12345678901234567890123456789012123456789012345678901
The importance of spending about 5 minutes brainstorming before writing 2
12345678901234567890123456789012123456789012345678901
1 your essay is covered on pages 229–230 earlier in this chapter. This 2
12345678901234567890123456789012123456789012345678901 2
12345678901234567890123456789012123456789012345678901 2
12345678901234567890123456789012123456789012345678901
brainstorming will be a lot better and take less time if you have already 2
12345678901234567890123456789012123456789012345678901 2
12345678901234567890123456789012123456789012345678901
thought about the topic previously. So even if you don’t want to use the 2
1 sample essay book, you should still look over some sample topics and 2
2345678901234567890123456789012123456789012345678901
12345678901234567890123456789012123456789012345678901 2
12345678901234567890123456789012123456789012345678901 2
12345678901234567890123456789012123456789012345678901 2 251
123456789012345678901234567890121234567890123456789012
Part III: Section Review

123456789012345678901234567890121234567890123456789012
12345678901234567890123456789012123456789012345678901 2
12345678901234567890123456789012123456789012345678901 2
12345678901234567890123456789012123456789012345678901
mentally sketch out how you would approach them. Take about 5 minutes 2
12345678901234567890123456789012123456789012345678901 2
12345678901234567890123456789012123456789012345678901
for each topic and ask yourself, “Would I agree or disagree with the topic 2
12345678901234567890123456789012123456789012345678901 2
12345678901234567890123456789012123456789012345678901
sentence? What type of supporting details would I like to use?” You don’t 2
1 have to write your answers down, although you can if you like. The main 2
2345678901234567890123456789012123456789012345678901
12345678901234567890123456789012123456789012345678901 2
12345678901234567890123456789012123456789012345678901 2
12345678901234567890123456789012123456789012345678901
goal is to acquaint yourself with the topic beforehand, and come up with a 2
12345678901234567890123456789012123456789012345678901
2345678901234567890123456789012123456789012345678901
2
2
1 rudimentary idea about how you would want to approach an essay on the
12345678901234567890123456789012123456789012345678901 2
12345678901234567890123456789012123456789012345678901
subject. This can help you in several ways: 2
12345678901234567890123456789012123456789012345678901 2
12345678901234567890123456789012123456789012345678901
2345678901234567890123456789012123456789012345678901
2
2
1 1. If you have a choice between a topic you’ve thought about and
12345678901234567890123456789012123456789012345678901 2
12345678901234567890123456789012123456789012345678901
one that you haven’t, the decision to choose the one you’ve 2
12345678901234567890123456789012123456789012345678901
thought about should come very rapidly. 2
12345678901234567890123456789012123456789012345678901
2345678901234567890123456789012123456789012345678901
2
2
1
12345678901234567890123456789012123456789012345678901
2. If you get a topic you’ve seen before, you won’t waste any time 2
12345678901234567890123456789012123456789012345678901 2
12345678901234567890123456789012123456789012345678901
deciding whether to agree or disagree since you already thought 2
12345678901234567890123456789012123456789012345678901 2
12345678901234567890123456789012123456789012345678901
about how to approach that topic. This should also make coming 2
12345678901234567890123456789012123456789012345678901 2
12345678901234567890123456789012123456789012345678901
up with supporting details much quicker and easier, leaving you 2
12345678901234567890123456789012123456789012345678901 2
12345678901234567890123456789012123456789012345678901
with more time to get words into the computer. 2
12345678901234567890123456789012123456789012345678901 2
2345678901234567890123456789012123456789012345678901
1 3. Even if you get two topics that you haven’t thought about 2
12345678901234567890123456789012123456789012345678901 2
12345678901234567890123456789012123456789012345678901 2
12345678901234567890123456789012123456789012345678901
already, your brainstorming skills will be greatly improved with 2
2345678901234567890123456789012123456789012345678901
all the practice. This should translate to less time needed to decide 2
12345678901234567890123456789012123456789012345678901
12345678901234567890123456789012123456789012345678901 2
12345678901234567890123456789012123456789012345678901 2
12345678901234567890123456789012123456789012345678901 2
whether to agree or disagree, as well as greater ease at coming up
12345678901234567890123456789012123456789012345678901 2
12345678901234567890123456789012123456789012345678901
with good supporting details. 2
12345678901234567890123456789012123456789012345678901 2
Doing the 5-minute brainstorm exercise on several topics is something 2
12345678901234567890123456789012123456789012345678901
12345678901234567890123456789012123456789012345678901 2
1 everyone should do. If you want to be more gung-ho, there are other things 2
12345678901234567890123456789012123456789012345678901 2
2345678901234567890123456789012123456789012345678901
12345678901234567890123456789012123456789012345678901
you can do, but these might not be the optimal use of your time. 2
1 2
12345678901234567890123456789012123456789012345678901 2
12345678901234567890123456789012123456789012345678901 2
12345678901234567890123456789012123456789012345678901
You could do research to come up with hard facts to use on every essay, 2
12345678901234567890123456789012123456789012345678901 2
12345678901234567890123456789012123456789012345678901 2
12345678901234567890123456789012123456789012345678901
but it’s not a good idea for two reasons. For one thing, it would take a 2
12345678901234567890123456789012123456789012345678901 2
12345678901234567890123456789012123456789012345678901 2
2345678901234567890123456789012123456789012345678901
massive amount of time for very little payoff. Also, if you did include
12345678901234567890123456789012123456789012345678901 2
1 something very specific, it might tip the reader off to the fact that you 2
12345678901234567890123456789012123456789012345678901
prepared beforehand. There’s nothing illegal about this, but it’s not going 2
12345678901234567890123456789012123456789012345678901 2
2345678901234567890123456789012123456789012345678901
to earn you any Brownie points. It’s much better to appear like you just 2
12345678901234567890123456789012123456789012345678901
12345678901234567890123456789012123456789012345678901 2
happened to get that particular topic, and you just happened to come up 2
12345678901234567890123456789012123456789012345678901
12345678901234567890123456789012123456789012345678901 2 2
1 with the set of supporting details that you did.
12345678901234567890123456789012123456789012345678901 2
12345678901234567890123456789012123456789012345678901 2
12345678901234567890123456789012123456789012345678901 2
12345678901234567890123456789012123456789012345678901 2
12345678901234567890123456789012123456789012345678901 2
12345678901234567890123456789012123456789012345678901 2
The2345678901234567890123456789012123456789012345678901
Second Essay
2
12345678901234567890123456789012123456789012345678901
The second essay doesn’t give you as much writing wiggle room as the 2
12345678901234567890123456789012123456789012345678901 2
12345678901234567890123456789012123456789012345678901
1 first. That essay is more of a writing assignment, while the second essay is 2
2345678901234567890123456789012123456789012345678901
12345678901234567890123456789012123456789012345678901 2
1 closer to an exercise in logical thinking. Sure, you still have to write, but 2
12345678901234567890123456789012123456789012345678901
you don’t have to formulate an argument from scratch. You just have to 2
12345678901234567890123456789012123456789012345678901 2
12345678901234567890123456789012123456789012345678901
2345678901234567890123456789012123456789012345678901
2
2
1 comment on an argument given to you.
12345678901234567890123456789012123456789012345678901 2
12345678901234567890123456789012123456789012345678901 2 2
252 12345678901234567890123456789012123456789012345678901
123456789012345678901234567890121234567890123456789012

www.petersons.com
Chapter 6: Analytical Writing Review

123456789012345678901234567890121234567890123456789012
12345678901234567890123456789012123456789012345678901 2
12345678901234567890123456789012123456789012345678901 2
12345678901234567890123456789012123456789012345678901 2
12345678901234567890123456789012123456789012345678901 2
12345678901234567890123456789012123456789012345678901
Other Ways to Prep for the Essay Beforehand 2
12345678901234567890123456789012123456789012345678901 2
12345678901234567890123456789012123456789012345678901 2
12345678901234567890123456789012123456789012345678901
1. Prepare a Quote List. As stated earlier on page 248, placing a 2
12345678901234567890123456789012123456789012345678901 2
12345678901234567890123456789012123456789012345678901
relevant quote is a nice addition to any essay. When 2
12345678901234567890123456789012123456789012345678901 2
12345678901234567890123456789012123456789012345678901
brainstorming on a sample topic, ask yourself, “Is there a
2345678901234567890123456789012123456789012345678901
2
2
1
12345678901234567890123456789012123456789012345678901
quote I can think of that would help prove my point?” If you 2
12345678901234567890123456789012123456789012345678901 2
12345678901234567890123456789012123456789012345678901
can’t think of one right away, you could always do some 2
12345678901234567890123456789012123456789012345678901
2345678901234567890123456789012123456789012345678901
2
2
1 searching in a book of quotations like Bartlett’s. Even if this
12345678901234567890123456789012123456789012345678901 2
12345678901234567890123456789012123456789012345678901
exercise doesn’t land you the quote you want, looking 2
12345678901234567890123456789012123456789012345678901 2
12345678901234567890123456789012123456789012345678901
through a book of quotes is often
2345678901234567890123456789012123456789012345678901
pretty cool. 2
2
1
12345678901234567890123456789012123456789012345678901 2
12345678901234567890123456789012123456789012345678901
2. Have Some Snazzy Vocabulary Words Ready to Go. You can 2
12345678901234567890123456789012123456789012345678901 2
12345678901234567890123456789012123456789012345678901
compile a list of advanced words that could be brought into 2
2345678901234567890123456789012123456789012345678901
12345678901234567890123456789012123456789012345678901 2
12345678901234567890123456789012123456789012345678901
play on your essay. For example, in the paragraph where you 2

Take It to the Next Level


12345678901234567890123456789012123456789012345678901 2
12345678901234567890123456789012123456789012345678901 22
show that you can acknowledge the other side’s point of view,
12345678901234567890123456789012123456789012345678901
you might have a sentence like, “Although this might happen,
12345678901234567890123456789012123456789012345678901 2
1 the chances of this occurring are actually very small.” Instead 2
12345678901234567890123456789012123456789012345678901 2
12345678901234567890123456789012123456789012345678901
of small, use infinitesimal. It means the same thing, but the 2
12345678901234567890123456789012123456789012345678901 2
2345678901234567890123456789012123456789012345678901
12345678901234567890123456789012123456789012345678901
GRE readers will note your use of advanced vocabulary. 2
12345678901234567890123456789012123456789012345678901 2
12345678901234567890123456789012123456789012345678901
Other good words like specious, verify, quandary (seen earlier 2
12345678901234567890123456789012123456789012345678901 2
12345678901234567890123456789012123456789012345678901
in this chapter), ratiocination, and blinkered can crop up in 2
12345678901234567890123456789012123456789012345678901 2
12345678901234567890123456789012123456789012345678901
many point-of-view essays. 2
12345678901234567890123456789012123456789012345678901 22
12345678901234567890123456789012123456789012345678901 2
1 If you are already compiling a list of vocabulary words for the
12345678901234567890123456789012123456789012345678901 2
12345678901234567890123456789012123456789012345678901
GRE Verbal section, you don’t need to make two lists. Simply 2
12345678901234567890123456789012123456789012345678901 2
12345678901234567890123456789012123456789012345678901 2
keep those advanced words in mind when writing your GRE
12345678901234567890123456789012123456789012345678901 2
12345678901234567890123456789012123456789012345678901
essays. 2
12345678901234567890123456789012123456789012345678901 2
12345678901234567890123456789012123456789012345678901 2
12345678901234567890123456789012123456789012345678901 2
12345678901234567890123456789012123456789012345678901
Two of the three strategies you used on the first essay work on the second 2
2345678901234567890123456789012123456789012345678901
12345678901234567890123456789012123456789012345678901
one, but the “Tweaking the Essay Format” technique doesn’t really apply 2
2
12345678901234567890123456789012123456789012345678901
1 to the second essay. With only 30 minutes, you need to attack the 2
12345678901234567890123456789012123456789012345678901 2
12345678901234567890123456789012123456789012345678901
argument and uncover the flaws. A basic essay with distinct paragraphs 2
12345678901234567890123456789012123456789012345678901 2
12345678901234567890123456789012123456789012345678901 2
12345678901234567890123456789012123456789012345678901
attacking specific points is the best way to do this. If you can add a quote 2
12345678901234567890123456789012123456789012345678901 2
12345678901234567890123456789012123456789012345678901 2
to help make your point, bully for you. However, adding a complex
2345678901234567890123456789012123456789012345678901
organizational layer might obscure the points you are trying to make, so 2
12345678901234567890123456789012123456789012345678901
12345678901234567890123456789012123456789012345678901 2
12345678901234567890123456789012123456789012345678901 2
12345678901234567890123456789012123456789012345678901 22
keeping it simple works best. If you want to take a risk and write the entire
1 second essay as a Socratic dialogue, go right ahead, but this approach will
12345678901234567890123456789012123456789012345678901 2
12345678901234567890123456789012123456789012345678901 2
12345678901234567890123456789012123456789012345678901
likely be too radical for most readers. A simple format showing complex 2
12345678901234567890123456789012123456789012345678901 2
12345678901234567890123456789012123456789012345678901 2
thoughts is better than a complex format obscuring complex thoughts.
12345678901234567890123456789012123456789012345678901 22
2345678901234567890123456789012123456789012345678901
1 Using your own voice can work on the second paragraph, but in a limited
12345678901234567890123456789012123456789012345678901 2
12345678901234567890123456789012123456789012345678901
manner. Again, for the second essay your overall goal is to expose flaws in 2
12345678901234567890123456789012123456789012345678901 2
12345678901234567890123456789012123456789012345678901
the original argument, not display your writing chops. If you can show a 2
12345678901234567890123456789012123456789012345678901 2
12345678901234567890123456789012123456789012345678901
little personality, by all means do so. It would certainly look good if 2
12345678901234567890123456789012123456789012345678901 2 253
123456789012345678901234567890121234567890123456789012
Part III: Section Review

123456789012345678901234567890121234567890123456789012
12345678901234567890123456789012123456789012345678901 2
12345678901234567890123456789012123456789012345678901 2
12345678901234567890123456789012123456789012345678901
elements of your writing style displayed in the first essay appear in the 2
12345678901234567890123456789012123456789012345678901 2
12345678901234567890123456789012123456789012345678901
second essay as well. That will show the GRE readers that the writing is 2
12345678901234567890123456789012123456789012345678901 2
12345678901234567890123456789012123456789012345678901
definitely yours, and not something you crafted ahead of time with the 2
2345678901234567890123456789012123456789012345678901 2
1 help of others.
12345678901234567890123456789012123456789012345678901 2
12345678901234567890123456789012123456789012345678901 2
12345678901234567890123456789012123456789012345678901 2
12345678901234567890123456789012123456789012345678901
The third strategy, preparing beforehand, is the one that works best for the 2
1 second essay. It is easily the best thing you can do to write a better second 2
2345678901234567890123456789012123456789012345678901
12345678901234567890123456789012123456789012345678901 2
12345678901234567890123456789012123456789012345678901 2
12345678901234567890123456789012123456789012345678901
essay. The second essay requires you to acquire and display a certain 2
12345678901234567890123456789012123456789012345678901
2345678901234567890123456789012123456789012345678901
2
2
1 skill—determining the flaws of a short argument—and the foremost way
12345678901234567890123456789012123456789012345678901 2
12345678901234567890123456789012123456789012345678901
to get better at this skill is through practice on real topics. 2
12345678901234567890123456789012123456789012345678901 2
12345678901234567890123456789012123456789012345678901 2
1 To help you analyze arguments, you can peruse the book ARCO GRE 2
2345678901234567890123456789012123456789012345678901
12345678901234567890123456789012123456789012345678901 2
12345678901234567890123456789012123456789012345678901 2

Tip
12345678901234567890123456789012123456789012345678901
CAT Answers to the Real Essay Questions. This will give you an idea of 2
12345678901234567890123456789012123456789012345678901 2
12345678901234567890123456789012123456789012345678901 2
2345678901234567890123456789012123456789012345678901
how to create and write about alternative scenarios that undermine the
12345678901234567890123456789012123456789012345678901
conclusions and supporting details of an argument. 2
12345678901234567890123456789012123456789012345678901 2
12345678901234567890123456789012123456789012345678901 2
12345678901234567890123456789012123456789012345678901 2
12345678901234567890123456789012123456789012345678901 2
1 You do not need to write out entire essays, although you can write out 2
12345678901234567890123456789012123456789012345678901
some if you feel like it. Focus instead on honing your brainstorming skills 2
12345678901234567890123456789012123456789012345678901 2
12345678901234567890123456789012123456789012345678901
to the point where you can take any 100-word argument, easily spot the 2
12345678901234567890123456789012123456789012345678901 2
2345678901234567890123456789012123456789012345678901
conclusion and supporting details, and then start churning out alternatives 2
12345678901234567890123456789012123456789012345678901
12345678901234567890123456789012123456789012345678901 2
that show the logical flaws. This can be a completely mental exercise 2
12345678901234567890123456789012123456789012345678901
12345678901234567890123456789012123456789012345678901 2
12345678901234567890123456789012123456789012345678901
where you spend roughly 5 minutes just thinking about every argument. 2
12345678901234567890123456789012123456789012345678901 2
You have 240 arguments to work on, but you probably won’t need that 2
12345678901234567890123456789012123456789012345678901
12345678901234567890123456789012123456789012345678901 2
1 many to get proficient at finding the flaws in an argument. Once you have 2
12345678901234567890123456789012123456789012345678901 2
2345678901234567890123456789012123456789012345678901
12345678901234567890123456789012123456789012345678901 2
1 this skill under your belt, the hardest part of the second essay is done. 2
12345678901234567890123456789012123456789012345678901
When test day comes, you might get lucky and encounter an argument 2
12345678901234567890123456789012123456789012345678901
2345678901234567890123456789012123456789012345678901
2
you’ve worked on, but even if you don’t, it should be easy for you to read 2
12345678901234567890123456789012123456789012345678901
12345678901234567890123456789012123456789012345678901 2
1 the text and begin formulating alternatives. You then place as many of 2
12345678901234567890123456789012123456789012345678901 2
12345678901234567890123456789012123456789012345678901 2
12345678901234567890123456789012123456789012345678901 2
2345678901234567890123456789012123456789012345678901
these alternatives into a basic essay format as time allows, and give
12345678901234567890123456789012123456789012345678901 2
1 yourself a few minutes at the very end to check over your grammar and 2
12345678901234567890123456789012123456789012345678901
perhaps tighten up a few sentences. 2
12345678901234567890123456789012123456789012345678901 2
12345678901234567890123456789012123456789012345678901 2
2345678901234567890123456789012123456789012345678901
A second essay with a host of strong alternatives is an excellent 2
12345678901234567890123456789012123456789012345678901
12345678901234567890123456789012123456789012345678901
complement to a first essay that shows some originality in content and 2
2
12345678901234567890123456789012123456789012345678901
1 structure. While this combo can’t guarantee you a 6—nothing can, for that 2
12345678901234567890123456789012123456789012345678901 2
12345678901234567890123456789012123456789012345678901 2
12345678901234567890123456789012123456789012345678901
matter—it does present a strong case to the GRE readers that you are 2
12345678901234567890123456789012123456789012345678901 2
12345678901234567890123456789012123456789012345678901
someone who really knows her stuff on the GRE Analytical section. 2
12345678901234567890123456789012123456789012345678901 2
2345678901234567890123456789012123456789012345678901
12345678901234567890123456789012123456789012345678901 2
12345678901234567890123456789012123456789012345678901 2
12345678901234567890123456789012123456789012345678901 2 2
1
12345678901234567890123456789012123456789012345678901 2
12345678901234567890123456789012123456789012345678901 2
12345678901234567890123456789012123456789012345678901 2
12345678901234567890123456789012123456789012345678901 2
12345678901234567890123456789012123456789012345678901 2
12345678901234567890123456789012123456789012345678901 2
12345678901234567890123456789012123456789012345678901 2
12345678901234567890123456789012123456789012345678901 2 2
254 12345678901234567890123456789012123456789012345678901
123456789012345678901234567890121234567890123456789012

www.petersons.com
PART

IV
Three Practice GREs
Preface to Practice Tests 256

Practice Test 1 258

Practice Test 2 282

Practice Test 3 305

Closing Remarks 329

PART IV
Chapter

7
Preface to Practice Tests
123456789012345678901234567890121234567890123456789012
12345678901234567890123456789012123456789012345678901 2
12345678901234567890123456789012123456789012345678901 2
12345678901234567890123456789012123456789012345678901
Putting Ideas and Concepts into Practice 2
12345678901234567890123456789012123456789012345678901 2
2345678901234567890123456789012123456789012345678901
NOTE: Don’t start on any of the following three tests until you have read 2
12345678901234567890123456789012123456789012345678901
12345678901234567890123456789012123456789012345678901 2
12345678901234567890123456789012123456789012345678901
this section. 2
12345678901234567890123456789012123456789012345678901 2
12345678901234567890123456789012123456789012345678901 2
1 The study materials in this book should have your head swirling with 2
12345678901234567890123456789012123456789012345678901 2
12345678901234567890123456789012123456789012345678901
information. Although this might lead to strange dreams featuring QC 2
12345678901234567890123456789012123456789012345678901 2
12345678901234567890123456789012123456789012345678901
problems, on the whole it’s a positive thing. The following 3 practice tests 2
12345678901234567890123456789012123456789012345678901
will give you a chance to put some of the knowledge flitting around in your 2
12345678901234567890123456789012123456789012345678901 2
12345678901234567890123456789012123456789012345678901 2
12345678901234567890123456789012123456789012345678901
noggin to good use. 2
12345678901234567890123456789012123456789012345678901 2
2345678901234567890123456789012123456789012345678901
These tests should be used to give you a chance to test-drive the new 2
12345678901234567890123456789012123456789012345678901
12345678901234567890123456789012123456789012345678901 2
12345678901234567890123456789012123456789012345678901 2
1 strategies and ideas previously presented within this book. It’s important 2
2345678901234567890123456789012123456789012345678901
you don’t just jump to these questions and start answering them willy-nilly 2
12345678901234567890123456789012123456789012345678901
12345678901234567890123456789012123456789012345678901 2
1 before reading all that came before. If you do this, odds are good you will 2
12345678901234567890123456789012123456789012345678901 2
12345678901234567890123456789012123456789012345678901
only help reinforce the faults you had before you started studying for the 2
12345678901234567890123456789012123456789012345678901 2
12345678901234567890123456789012123456789012345678901
GRE. To truly maximize your GRE score, these tests must be approached 2
12345678901234567890123456789012123456789012345678901 2
12345678901234567890123456789012123456789012345678901
with the attitude that you will use them to try out the various different 2
12345678901234567890123456789012123456789012345678901 2
2345678901234567890123456789012123456789012345678901
points and ideas covered so far. Doing this will help you absorb the 2
12345678901234567890123456789012123456789012345678901
12345678901234567890123456789012123456789012345678901 2
1 material discussed and make you a better GRE test-taker. 2
12345678901234567890123456789012123456789012345678901 2
12345678901234567890123456789012123456789012345678901
Make sure you take only one test at a time, and then look over the 2
12345678901234567890123456789012123456789012345678901 2
12345678901234567890123456789012123456789012345678901 2
12345678901234567890123456789012123456789012345678901
explanations to the answers. Everyone will want to read the explanations 2
12345678901234567890123456789012123456789012345678901 2
12345678901234567890123456789012123456789012345678901 2
for the questions that they miss, but it’s also a good idea to look the
2345678901234567890123456789012123456789012345678901
answers for the other questions as well. The explanations can give you 2
12345678901234567890123456789012123456789012345678901
12345678901234567890123456789012123456789012345678901 2
12345678901234567890123456789012123456789012345678901 2
12345678901234567890123456789012123456789012345678901 2
ideas about different ways you can approach a problem; learning about
1 these different approaches will earn you points toward your Superior 2
12345678901234567890123456789012123456789012345678901 2
12345678901234567890123456789012123456789012345678901 2
12345678901234567890123456789012123456789012345678901
Test-Taker merit badge. Many people who score poorly on standardized 2
12345678901234567890123456789012123456789012345678901 2
12345678901234567890123456789012123456789012345678901 2
tests do so because they are locked into the mindset that there is only one
2345678901234567890123456789012123456789012345678901
12345678901234567890123456789012123456789012345678901 2
1 way to answer every problem. It’s as if every problem is a locked door, and 2
12345678901234567890123456789012123456789012345678901
only a single correct key can unlock it. 2
12345678901234567890123456789012123456789012345678901 2
12345678901234567890123456789012123456789012345678901
2345678901234567890123456789012123456789012345678901
2
2
1
12345678901234567890123456789012123456789012345678901 2
12345678901234567890123456789012123456789012345678901 2
12345678901234567890123456789012123456789012345678901 2
123456789012345678901234567890121234567890123456789012
256
Chapter 7: Preface to Practice Tests

123456789012345678901234567890121234567890123456789012
12345678901234567890123456789012123456789012345678901 2
12345678901234567890123456789012123456789012345678901 2
12345678901234567890123456789012123456789012345678901 2
12345678901234567890123456789012123456789012345678901 2
12345678901234567890123456789012123456789012345678901 2

Note
It’s doesn’t matter if you take these tests under timed conditions or not,
12345678901234567890123456789012123456789012345678901 2
12345678901234567890123456789012123456789012345678901
although a good plan would be to take the first
2345678901234567890123456789012123456789012345678901
two untimed and the third 2
2
1 one timed. Your real practice with timed GRE sections will come with the
12345678901234567890123456789012123456789012345678901 2
12345678901234567890123456789012123456789012345678901 2
12345678901234567890123456789012123456789012345678901
exams, not the CD, since these mimic the adaptive format. 2
12345678901234567890123456789012123456789012345678901
2345678901234567890123456789012123456789012345678901
2
2
1
12345678901234567890123456789012123456789012345678901 2
12345678901234567890123456789012123456789012345678901
If you get nothing else from this book, you should learn that this mode of 2
12345678901234567890123456789012123456789012345678901 2
12345678901234567890123456789012123456789012345678901
thinking is incorrect. There are skeleton keys (general facts you can use to 2
1 come to the right answer), lock-picking equipment (strategies like 2
2345678901234567890123456789012123456789012345678901
12345678901234567890123456789012123456789012345678901 2
12345678901234567890123456789012123456789012345678901 2
12345678901234567890123456789012123456789012345678901
estimating and process of elimination), and if worse comes to worse, you 2
12345678901234567890123456789012123456789012345678901
can just try kicking the door down with
2345678901234567890123456789012123456789012345678901your boot (guessing as time runs 2
2
1
12345678901234567890123456789012123456789012345678901
out). These won’t work every time, but they’ll work more times than you 2
12345678901234567890123456789012123456789012345678901 2
12345678901234567890123456789012123456789012345678901
might think. 2
12345678901234567890123456789012123456789012345678901 2
2345678901234567890123456789012123456789012345678901
12345678901234567890123456789012123456789012345678901
Do your best, and remember that the goal is to learn something from every 2
2
12345678901234567890123456789012123456789012345678901
12345678901234567890123456789012123456789012345678901 2
12345678901234567890123456789012123456789012345678901
missed question, and even from some that you answered correctly. 2
12345678901234567890123456789012123456789012345678901 22
12345678901234567890123456789012123456789012345678901 2
1
12345678901234567890123456789012123456789012345678901 2
12345678901234567890123456789012123456789012345678901 2
12345678901234567890123456789012123456789012345678901 2
12345678901234567890123456789012123456789012345678901 22
2345678901234567890123456789012123456789012345678901
12345678901234567890123456789012123456789012345678901 2
12345678901234567890123456789012123456789012345678901 2
12345678901234567890123456789012123456789012345678901 2
12345678901234567890123456789012123456789012345678901 2
12345678901234567890123456789012123456789012345678901 2
12345678901234567890123456789012123456789012345678901 2
12345678901234567890123456789012123456789012345678901 2
12345678901234567890123456789012123456789012345678901 2
1
12345678901234567890123456789012123456789012345678901 2
12345678901234567890123456789012123456789012345678901 2
12345678901234567890123456789012123456789012345678901 2
12345678901234567890123456789012123456789012345678901 2
12345678901234567890123456789012123456789012345678901 2
12345678901234567890123456789012123456789012345678901 2
12345678901234567890123456789012123456789012345678901 2
12345678901234567890123456789012123456789012345678901 2
12345678901234567890123456789012123456789012345678901 2
12345678901234567890123456789012123456789012345678901 2
12345678901234567890123456789012123456789012345678901 22
2345678901234567890123456789012123456789012345678901
12345678901234567890123456789012123456789012345678901 2
1
12345678901234567890123456789012123456789012345678901 2
12345678901234567890123456789012123456789012345678901 2
12345678901234567890123456789012123456789012345678901 2
12345678901234567890123456789012123456789012345678901 2
12345678901234567890123456789012123456789012345678901 2
12345678901234567890123456789012123456789012345678901 2
12345678901234567890123456789012123456789012345678901 2
12345678901234567890123456789012123456789012345678901 22
2345678901234567890123456789012123456789012345678901
12345678901234567890123456789012123456789012345678901 2
12345678901234567890123456789012123456789012345678901 2
12345678901234567890123456789012123456789012345678901 2
1
12345678901234567890123456789012123456789012345678901 2
12345678901234567890123456789012123456789012345678901 2
12345678901234567890123456789012123456789012345678901 2
12345678901234567890123456789012123456789012345678901 2
12345678901234567890123456789012123456789012345678901 2
12345678901234567890123456789012123456789012345678901 22
2345678901234567890123456789012123456789012345678901
1
12345678901234567890123456789012123456789012345678901 2
12345678901234567890123456789012123456789012345678901 2
12345678901234567890123456789012123456789012345678901 2
12345678901234567890123456789012123456789012345678901 2
12345678901234567890123456789012123456789012345678901 2
12345678901234567890123456789012123456789012345678901 2
12345678901234567890123456789012123456789012345678901 2 257
123456789012345678901234567890121234567890123456789012
Practice Test

1
1234567890123456789012345678901212345678901234567890123456789012123456
12 3456789012345678901234567890121234567890123456789012345678901212345 6
12 3456789012345678901234567890121234567890123456789012345678901212345 6
12 3456789012345678901234567890121234567890123456789012345678901212345
Quantitative 6
123456789012345678901234567890121234567890123456789012345678901212345
2 6
123456789012345678901234567890121234567890123456789012345678901212345 6
123456789012345678901234567890121234567890123456789012345678901212345
28 Questions—45 Minutes 6
123456789012345678901234567890121234567890123456789012345678901212345 6
123456789012345678901234567890121234567890123456789012345678901212345 6
123456789012345678901234567890121234567890123456789012345678901212345 66
3456789012345678901234567890121234567890123456789012345678901212345
General Information
1
12 3456789012345678901234567890121234567890123456789012345678901212345
1. The test has 28 questions. If you decide to take this exam under timed conditions, you have 6
12 3456789012345678901234567890121234567890123456789012345678901212345 6
12 3456789012345678901234567890121234567890123456789012345678901212345
45 minutes to complete the section. 6
123456789012345678901234567890121234567890123456789012345678901212345 66
2 3456789012345678901234567890121234567890123456789012345678901212345
123456789012345678901234567890121234567890123456789012345678901212345
123456789012345678901234567890121234567890123456789012345678901212345
2. All numbers used are real numbers. 6
123456789012345678901234567890121234567890123456789012345678901212345 66
123456789012345678901234567890121234567890123456789012345678901212345
123456789012345678901234567890121234567890123456789012345678901212345
3. All angle measurements can be assumed to be positive unless otherwise noted. 6
123456789012345678901234567890121234567890123456789012345678901212345 66
123456789012345678901234567890121234567890123456789012345678901212345
123456789012345678901234567890121234567890123456789012345678901212345
4. All figures lie in the same plane unless otherwise noted. 6
1 6
2 3456789012345678901234567890121234567890123456789012345678901212345
5. Drawings that accompany questions are intended to provide information useful in 6
123456789012345678901234567890121234567890123456789012345678901212345
123456789012345678901234567890121234567890123456789012345678901212345 6
1 answering the question. The figures are drawn closely to scale unless otherwise noted. 6
123456789012345678901234567890121234567890123456789012345678901212345 6
12 3456789012345678901234567890121234567890123456789012345678901212345 6
12 3456789012345678901234567890121234567890123456789012345678901212345 6
12 3456789012345678901234567890121234567890123456789012345678901212345
Directions: For questions 1–14, Column A and Column B will have one quantity each. 6
123456789012345678901234567890121234567890123456789012345678901212345 6
12 3456789012345678901234567890121234567890123456789012345678901212345
Your goal is to compare the quantities in each column and choose 6
12 3456789012345678901234567890121234567890123456789012345678901212345 6
2 3456789012345678901234567890121234567890123456789012345678901212345
123456789012345678901234567890121234567890123456789012345678901212345 6
123456789012345678901234567890121234567890123456789012345678901212345
A if the quantity in Column A is greater 6
1 6
123456789012345678901234567890121234567890123456789012345678901212345
B if the quantity in Column B is greater 6
123456789012345678901234567890121234567890123456789012345678901212345 6
123456789012345678901234567890121234567890123456789012345678901212345 66
2 3456789012345678901234567890121234567890123456789012345678901212345
C if the two quantities are equal
123456789012345678901234567890121234567890123456789012345678901212345 6
123456789012345678901234567890121234567890123456789012345678901212345
D if the relationship between the two quantities cannot be determined from the
6
123456789012345678901234567890121234567890123456789012345678901212345 6
1 information in the problem
123456789012345678901234567890121234567890123456789012345678901212345 66
2 3456789012345678901234567890121234567890123456789012345678901212345
123456789012345678901234567890121234567890123456789012345678901212345 6
123456789012345678901234567890121234567890123456789012345678901212345
On some questions, information about one or both quantities will be given. This informa-
6
123456789012345678901234567890121234567890123456789012345678901212345 6
1 tion will be centered above the two columns.
123456789012345678901234567890121234567890123456789012345678901212345 66
2 3456789012345678901234567890121234567890123456789012345678901212345
123456789012345678901234567890121234567890123456789012345678901212345 6
123456789012345678901234567890121234567890123456789012345678901212345 6
123456789012345678901234567890121234567890123456789012345678901212345 6
1
12 3456789012345678901234567890121234567890123456789012345678901212345 6
123456789012345678901234567890121234567890123456789012345678901212345 6
12 3456789012345678901234567890121234567890123456789012345678901212345 6
123456789012345678901234567890121234567890123456789012345678901212345 6
12 3456789012345678901234567890121234567890123456789012345678901212345 6
123456789012345678901234567890121234567890123456789012345678901212345 6
123456789012345678901234567890121234567890123456789012345678901212345 6
123456789012345678901234567890121234567890123456789012345678901212345 6
123456789012345678901234567890121234567890123456789012345678901212345 6
1234567890123456789012345678901212345678901234567890123456789012123456
258
TEST 1
1234567890123456789012345678901212345678901234567890123456789012123456
123456789012345678901234567890121234567890123456789012345678901212345 6
12 3456789012345678901234567890121234567890123456789012345678901212345 6
123456789012345678901234567890121234567890123456789012345678901212345
Column A Column B Column A Column B 6
12 3456789012345678901234567890121234567890123456789012345678901212345 6
12 3456789012345678901234567890121234567890123456789012345678901212345 6
123456789012345678901234567890121234567890123456789012345678901212345
1. h56 4. |x| 5 3, |y| 5 4 6
12
2
3456789012345678901234567890121234567890123456789012345678901212345 6
3456789012345678901234567890121234567890123456789012345678901212345 6
1
12 3456789012345678901234567890121234567890123456789012345678901212345
The number of The number of y2x 1 6
12 3456789012345678901234567890121234567890123456789012345678901212345 6
12 3456789012345678901234567890121234567890123456789012345678901212345 6
minutes in h hours degrees in the sum
12 3456789012345678901234567890121234567890123456789012345678901212345 6
123456789012345678901234567890121234567890123456789012345678901212345
of the angles of 6
12 3456789012345678901234567890121234567890123456789012345678901212345 6
12 3456789012345678901234567890121234567890123456789012345678901212345 6
a square
12 3456789012345678901234567890121234567890123456789012345678901212345 6
12 3456789012345678901234567890121234567890123456789012345678901212345 6
1 3456789012345678901234567890121234567890123456789012345678901212345 6
2
12 3456789012345678901234567890121234567890123456789012345678901212345 6
12 3456789012345678901234567890121234567890123456789012345678901212345 6
12 3456789012345678901234567890121234567890123456789012345678901212345 6
12
2
3456789012345678901234567890121234567890123456789012345678901212345
5. The sum of the
3456789012345678901234567890121234567890123456789012345678901212345 = 49 1 =8 6
6
1
12 3456789012345678901234567890121234567890123456789012345678901212345
three smallest 6
12 3456789012345678901234567890121234567890123456789012345678901212345 6
12 3456789012345678901234567890121234567890123456789012345678901212345 6
prime numbers
12 3456789012345678901234567890121234567890123456789012345678901212345 6
12 3456789012345678901234567890121234567890123456789012345678901212345
2. 6
12 3456789012345678901234567890121234567890123456789012345678901212345 6
12 3456789012345678901234567890121234567890123456789012345678901212345 6
12 3456789012345678901234567890121234567890123456789012345678901212345 6
12 3456789012345678901234567890121234567890123456789012345678901212345 6
123456789012345678901234567890121234567890123456789012345678901212345
2 6
1 3456789012345678901234567890121234567890123456789012345678901212345 6
12 3456789012345678901234567890121234567890123456789012345678901212345 6
12 3456789012345678901234567890121234567890123456789012345678901212345 6
12 3456789012345678901234567890121234567890123456789012345678901212345 6
123456789012345678901234567890121234567890123456789012345678901212345
2 6
123456789012345678901234567890121234567890123456789012345678901212345
6. 6
123456789012345678901234567890121234567890123456789012345678901212345 6
123456789012345678901234567890121234567890123456789012345678901212345 6
123456789012345678901234567890121234567890123456789012345678901212345 6
123456789012345678901234567890121234567890123456789012345678901212345 66
3456789012345678901234567890121234567890123456789012345678901212345
123456789012345678901234567890121234567890123456789012345678901212345
123456789012345678901234567890121234567890123456789012345678901212345
a2 6
123456789012345678901234567890121234567890123456789012345678901212345
The hypotenuse 6
1 6
123456789012345678901234567890121234567890123456789012345678901212345 66
2 3456789012345678901234567890121234567890123456789012345678901212345
squared minus
123456789012345678901234567890121234567890123456789012345678901212345 6
1 itself
123456789012345678901234567890121234567890123456789012345678901212345 6
12 3456789012345678901234567890121234567890123456789012345678901212345 6
12 3456789012345678901234567890121234567890123456789012345678901212345 6
12 3456789012345678901234567890121234567890123456789012345678901212345 6
123456789012345678901234567890121234567890123456789012345678901212345
a2 1 ab 1 a 180 6
12 3456789012345678901234567890121234567890123456789012345678901212345 6
12 3456789012345678901234567890121234567890123456789012345678901212345
a 6
123456789012345678901234567890121234567890123456789012345678901212345 66
2 3456789012345678901234567890121234567890123456789012345678901212345
123456789012345678901234567890121234567890123456789012345678901212345 6
1
123456789012345678901234567890121234567890123456789012345678901212345 6
123456789012345678901234567890121234567890123456789012345678901212345 6
12 3456789012345678901234567890121234567890123456789012345678901212345
3. A square has a side that is h inches long. 6
12 3456789012345678901234567890121234567890123456789012345678901212345 6
12 3456789012345678901234567890121234567890123456789012345678901212345
h2
The number of 6
12 3456789012345678901234567890121234567890123456789012345678901212345 6
123456789012345678901234567890121234567890123456789012345678901212345
inches along the 6
2 3456789012345678901234567890121234567890123456789012345678901212345
123456789012345678901234567890121234567890123456789012345678901212345 6
123456789012345678901234567890121234567890123456789012345678901212345
perimeter of the 6
123456789012345678901234567890121234567890123456789012345678901212345 6
123456789012345678901234567890121234567890123456789012345678901212345
square 6
1 6
123456789012345678901234567890121234567890123456789012345678901212345 66
2 3456789012345678901234567890121234567890123456789012345678901212345
123456789012345678901234567890121234567890123456789012345678901212345 6
123456789012345678901234567890121234567890123456789012345678901212345 6
123456789012345678901234567890121234567890123456789012345678901212345 6
1
12 3456789012345678901234567890121234567890123456789012345678901212345 6
123456789012345678901234567890121234567890123456789012345678901212345 6
12 3456789012345678901234567890121234567890123456789012345678901212345 6
123456789012345678901234567890121234567890123456789012345678901212345 6
12 3456789012345678901234567890121234567890123456789012345678901212345 6
123456789012345678901234567890121234567890123456789012345678901212345 6
123456789012345678901234567890121234567890123456789012345678901212345 6
123456789012345678901234567890121234567890123456789012345678901212345 6
123456789012345678901234567890121234567890123456789012345678901212345 6 259
1234567890123456789012345678901212345678901234567890123456789012123456
Part IV: Three Practice GREs

1234567890123456789012345678901212345678901234567890123456789012123456
123456789012345678901234567890121234567890123456789012345678901212345 6
12 3456789012345678901234567890121234567890123456789012345678901212345 6
123456789012345678901234567890121234567890123456789012345678901212345
Column A Column B Column A Column B 6
12 3456789012345678901234567890121234567890123456789012345678901212345 6
12 3456789012345678901234567890121234567890123456789012345678901212345 6
123456789012345678901234567890121234567890123456789012345678901212345
7. Jackie, Beth, and Allison each have 10. A triangle has one side of length 4 and 6
12
2
3456789012345678901234567890121234567890123456789012345678901212345
3456789012345678901234567890121234567890123456789012345678901212345
6
6
1 different cars and park in a three spot another of length 10.
12 3456789012345678901234567890121234567890123456789012345678901212345 6
12 3456789012345678901234567890121234567890123456789012345678901212345
parking lot behind their house. 6
12 3456789012345678901234567890121234567890123456789012345678901212345 6
5 The length of the
12 3456789012345678901234567890121234567890123456789012345678901212345 6
123456789012345678901234567890121234567890123456789012345678901212345
The number of 7 third side of the 6
12 3456789012345678901234567890121234567890123456789012345678901212345 6
12 3456789012345678901234567890121234567890123456789012345678901212345 6
possible different triangle
12 3456789012345678901234567890121234567890123456789012345678901212345 6
12
2
3456789012345678901234567890121234567890123456789012345678901212345
arrangements of
3456789012345678901234567890121234567890123456789012345678901212345
6
6
1
12 3456789012345678901234567890121234567890123456789012345678901212345
their cars in the 6
12 3456789012345678901234567890121234567890123456789012345678901212345 6
12 3456789012345678901234567890121234567890123456789012345678901212345 6
three spots
12 3456789012345678901234567890121234567890123456789012345678901212345 6
123456789012345678901234567890121234567890123456789012345678901212345 6
12 3456789012345678901234567890121234567890123456789012345678901212345 6
12 3456789012345678901234567890121234567890123456789012345678901212345 6
12 3456789012345678901234567890121234567890123456789012345678901212345 6
12 3456789012345678901234567890121234567890123456789012345678901212345 6
123456789012345678901234567890121234567890123456789012345678901212345
2 11. The median of the The mean of the 6
123456789012345678901234567890121234567890123456789012345678901212345 6
123456789012345678901234567890121234567890123456789012345678901212345
even integers 10 even integers 10 6
123456789012345678901234567890121234567890123456789012345678901212345 6
123456789012345678901234567890121234567890123456789012345678901212345
through 18 through 18 6
3456789012345678901234567890121234567890123456789012345678901212345
123456789012345678901234567890121234567890123456789012345678901212345 6
1 8. Twice the circum- p inclusive inclusive 6
12 3456789012345678901234567890121234567890123456789012345678901212345 6
12 3456789012345678901234567890121234567890123456789012345678901212345 6
ference of a circle
12 3456789012345678901234567890121234567890123456789012345678901212345 6
12 3456789012345678901234567890121234567890123456789012345678901212345
divided by two 6
12 3456789012345678901234567890121234567890123456789012345678901212345 6
12 3456789012345678901234567890121234567890123456789012345678901212345 6
times its diameter
12 3456789012345678901234567890121234567890123456789012345678901212345 6
12 3456789012345678901234567890121234567890123456789012345678901212345 6
123456789012345678901234567890121234567890123456789012345678901212345
2 6
123456789012345678901234567890121234567890123456789012345678901212345 6
123456789012345678901234567890121234567890123456789012345678901212345 6
123456789012345678901234567890121234567890123456789012345678901212345 6
1 3456789012345678901234567890121234567890123456789012345678901212345 6
12.
123456789012345678901234567890121234567890123456789012345678901212345 66
2 3456789012345678901234567890121234567890123456789012345678901212345
123456789012345678901234567890121234567890123456789012345678901212345 6
1
123456789012345678901234567890121234567890123456789012345678901212345 6
12 3456789012345678901234567890121234567890123456789012345678901212345 6
12 3456789012345678901234567890121234567890123456789012345678901212345
9. One is less than a, and a is less than b. 6
12 3456789012345678901234567890121234567890123456789012345678901212345 6
12
12
2
F
a
2
2
b
2 12aGS D F S D G
123456789012345678901234567890121234567890123456789012345678901212345
3456789012345678901234567890121234567890123456789012345678901212345
1
a
3 b
a
6
2
1
3456789012345678901234567890121234567890123456789012345678901212345
3b
2 4
123456789012345678901234567890121234567890123456789012345678901212345 66
3456789012345678901234567890121234567890123456789012345678901212345
6
6
6
123456789012345678901234567890121234567890123456789012345678901212345 6
1
123456789012345678901234567890121234567890123456789012345678901212345 6
123456789012345678901234567890121234567890123456789012345678901212345 6
12 3456789012345678901234567890121234567890123456789012345678901212345 6
12 3456789012345678901234567890121234567890123456789012345678901212345 6
12 3456789012345678901234567890121234567890123456789012345678901212345 6
12 3456789012345678901234567890121234567890123456789012345678901212345 6
123456789012345678901234567890121234567890123456789012345678901212345 6
123456789012345678901234567890121234567890123456789012345678901212345
2 6
123456789012345678901234567890121234567890123456789012345678901212345
The length of GH The slope of GH 6
123456789012345678901234567890121234567890123456789012345678901212345 6
123456789012345678901234567890121234567890123456789012345678901212345 times 24 6
1 3456789012345678901234567890121234567890123456789012345678901212345 6
123456789012345678901234567890121234567890123456789012345678901212345 66
2 3456789012345678901234567890121234567890123456789012345678901212345
123456789012345678901234567890121234567890123456789012345678901212345 6
123456789012345678901234567890121234567890123456789012345678901212345 6
123456789012345678901234567890121234567890123456789012345678901212345 6
1
12 3456789012345678901234567890121234567890123456789012345678901212345 6
123456789012345678901234567890121234567890123456789012345678901212345 6
12 3456789012345678901234567890121234567890123456789012345678901212345 6
123456789012345678901234567890121234567890123456789012345678901212345 6
12 3456789012345678901234567890121234567890123456789012345678901212345 6
123456789012345678901234567890121234567890123456789012345678901212345 6
123456789012345678901234567890121234567890123456789012345678901212345 6
123456789012345678901234567890121234567890123456789012345678901212345 66
260 123456789012345678901234567890121234567890123456789012345678901212345
1234567890123456789012345678901212345678901234567890123456789012123456

www.petersons.com
TEST 1
1234567890123456789012345678901212345678901234567890123456789012123456
123456789012345678901234567890121234567890123456789012345678901212345 6
12 3456789012345678901234567890121234567890123456789012345678901212345 6
123456789012345678901234567890121234567890123456789012345678901212345
Column A Column B 16. 6
12 3456789012345678901234567890121234567890123456789012345678901212345 y
6
12 3456789012345678901234567890121234567890123456789012345678901212345 6
123456789012345678901234567890121234567890123456789012345678901212345
13. A and B are distinct digits. 6
12
2
3456789012345678901234567890121234567890123456789012345678901212345 6
3456789012345678901234567890121234567890123456789012345678901212345 6
1
12 3456789012345678901234567890121234567890123456789012345678901212345
|A 1 A 1 B| 28 6
12 3456789012345678901234567890121234567890123456789012345678901212345 6
12 3456789012345678901234567890121234567890123456789012345678901212345 6
12 3456789012345678901234567890121234567890123456789012345678901212345 6
1 3456789012345678901234567890121234567890123456789012345678901212345 6
2
12 3456789012345678901234567890121234567890123456789012345678901212345 6
12 3456789012345678901234567890121234567890123456789012345678901212345 x 6
12 3456789012345678901234567890121234567890123456789012345678901212345 60
12 3456789012345678901234567890121234567890123456789012345678901212345 6
123456789012345678901234567890121234567890123456789012345678901212345 6
12 3456789012345678901234567890121234567890123456789012345678901212345 6
12 3456789012345678901234567890121234567890123456789012345678901212345 6
12 3456789012345678901234567890121234567890123456789012345678901212345 6
12
2
3456789012345678901234567890121234567890123456789012345678901212345
14. The local pet store has 73 cats and dogs.
3456789012345678901234567890121234567890123456789012345678901212345
6
6
1
12 3456789012345678901234567890121234567890123456789012345678901212345
40 of them are female, and 35 are 6
12 3456789012345678901234567890121234567890123456789012345678901212345
The linear equation that accurately 6
12 3456789012345678901234567890121234567890123456789012345678901212345
dogs. There are 9 less male cats 6
12 3456789012345678901234567890121234567890123456789012345678901212345
corresponds to line r is 6
123456789012345678901234567890121234567890123456789012345678901212345 66
2 3456789012345678901234567890121234567890123456789012345678901212345
than male dogs.
123456789012345678901234567890121234567890123456789012345678901212345
123456789012345678901234567890121234567890123456789012345678901212345
A. y 1 2 5 22x 6
123456789012345678901234567890121234567890123456789012345678901212345
The number of The number of 6
123456789012345678901234567890121234567890123456789012345678901212345
B. y 2 2 5 22x 6
123456789012345678901234567890121234567890123456789012345678901212345
female cats male dogs 6
1 C. y 1 2 5 2x 6
12 3456789012345678901234567890121234567890123456789012345678901212345 6
12 3456789012345678901234567890121234567890123456789012345678901212345 6
D. y 2 2 5 2x
12 3456789012345678901234567890121234567890123456789012345678901212345 6
12 3456789012345678901234567890121234567890123456789012345678901212345
E. y 1 2 5 x 6
12 3456789012345678901234567890121234567890123456789012345678901212345 6
12 3456789012345678901234567890121234567890123456789012345678901212345 6
12 3456789012345678901234567890121234567890123456789012345678901212345
Directions: Select the best answer for each 6
12 3456789012345678901234567890121234567890123456789012345678901212345
Questions 17–18 refer to the following graph. 6
2 3456789012345678901234567890121234567890123456789012345678901212345
123456789012345678901234567890121234567890123456789012345678901212345
of the following questions. 6
123456789012345678901234567890121234567890123456789012345678901212345 6
123456789012345678901234567890121234567890123456789012345678901212345 66
123456789012345678901234567890121234567890123456789012345678901212345 6
1 15. If 2x2 5 2x 2 15, then what does x
12 3456789012345678901234567890121234567890123456789012345678901212345 6
12 3456789012345678901234567890121234567890123456789012345678901212345 6
123456789012345678901234567890121234567890123456789012345678901212345
equal? 6
123456789012345678901234567890121234567890123456789012345678901212345 6
12 3456789012345678901234567890121234567890123456789012345678901212345
I. 23 6
12 3456789012345678901234567890121234567890123456789012345678901212345 6
12 3456789012345678901234567890121234567890123456789012345678901212345 6
123456789012345678901234567890121234567890123456789012345678901212345
II. 3 6
12 3456789012345678901234567890121234567890123456789012345678901212345 6
12 3456789012345678901234567890121234567890123456789012345678901212345 6
123456789012345678901234567890121234567890123456789012345678901212345 66
2 3456789012345678901234567890121234567890123456789012345678901212345
III. 5
123456789012345678901234567890121234567890123456789012345678901212345 6
1
123456789012345678901234567890121234567890123456789012345678901212345
A. I only 6
123456789012345678901234567890121234567890123456789012345678901212345 6
123456789012345678901234567890121234567890123456789012345678901212345 66
2 3456789012345678901234567890121234567890123456789012345678901212345
B. II only
123456789012345678901234567890121234567890123456789012345678901212345 6
123456789012345678901234567890121234567890123456789012345678901212345
C. I and III only
6
123456789012345678901234567890121234567890123456789012345678901212345
17. Which month had the fewest reported
6
1 D. II and III only
2 3456789012345678901234567890121234567890123456789012345678901212345
123456789012345678901234567890121234567890123456789012345678901212345
incidences of flu? 6
123456789012345678901234567890121234567890123456789012345678901212345
E. I, II, and III 6
123456789012345678901234567890121234567890123456789012345678901212345 6
123456789012345678901234567890121234567890123456789012345678901212345
A. January 1962 6
1 6
123456789012345678901234567890121234567890123456789012345678901212345 66
2 3456789012345678901234567890121234567890123456789012345678901212345
B. June 1962
123456789012345678901234567890121234567890123456789012345678901212345 6
123456789012345678901234567890121234567890123456789012345678901212345
C. January 1963
6
123456789012345678901234567890121234567890123456789012345678901212345 6
1 D. February 1963
2 3456789012345678901234567890121234567890123456789012345678901212345
123456789012345678901234567890121234567890123456789012345678901212345 6
1 E. April 1963 6
12 3456789012345678901234567890121234567890123456789012345678901212345 6
123456789012345678901234567890121234567890123456789012345678901212345 6
12 3456789012345678901234567890121234567890123456789012345678901212345 6
123456789012345678901234567890121234567890123456789012345678901212345 6
123456789012345678901234567890121234567890123456789012345678901212345 6
123456789012345678901234567890121234567890123456789012345678901212345 6
123456789012345678901234567890121234567890123456789012345678901212345 6 261
1234567890123456789012345678901212345678901234567890123456789012123456
Part IV: Three Practice GREs

1234567890123456789012345678901212345678901234567890123456789012123456
123456789012345678901234567890121234567890123456789012345678901212345 6
12 3456789012345678901234567890121234567890123456789012345678901212345 6
123456789012345678901234567890121234567890123456789012345678901212345
18. Which two-month period had the greatest Questions 21–23 are based on the following 6
12 3456789012345678901234567890121234567890123456789012345678901212345 6
12 3456789012345678901234567890121234567890123456789012345678901212345
number of reported child incidences of flu? graphs. 6
123456789012345678901234567890121234567890123456789012345678901212345 6
12 3456789012345678901234567890121234567890123456789012345678901212345 6
123456789012345678901234567890121234567890123456789012345678901212345
A. February–March 1962 6
12 3456789012345678901234567890121234567890123456789012345678901212345 6
12 3456789012345678901234567890121234567890123456789012345678901212345 6
B. March–April 1962
12 3456789012345678901234567890121234567890123456789012345678901212345 6
12
2
3456789012345678901234567890121234567890123456789012345678901212345
C. April–May 1962
3456789012345678901234567890121234567890123456789012345678901212345
6
6
1
12 3456789012345678901234567890121234567890123456789012345678901212345
D. April–May 1963 6
12 3456789012345678901234567890121234567890123456789012345678901212345 6
12 3456789012345678901234567890121234567890123456789012345678901212345 6
E. May–June 1963
12 3456789012345678901234567890121234567890123456789012345678901212345 6
123456789012345678901234567890121234567890123456789012345678901212345 6
12 3456789012345678901234567890121234567890123456789012345678901212345
19. 6
12 3456789012345678901234567890121234567890123456789012345678901212345 6
12 3456789012345678901234567890121234567890123456789012345678901212345 6
12 3456789012345678901234567890121234567890123456789012345678901212345 6
123456789012345678901234567890121234567890123456789012345678901212345 6
12 3456789012345678901234567890121234567890123456789012345678901212345 6
12 3456789012345678901234567890121234567890123456789012345678901212345 6
12 3456789012345678901234567890121234567890123456789012345678901212345 6
12 3456789012345678901234567890121234567890123456789012345678901212345 6
123456789012345678901234567890121234567890123456789012345678901212345
2 6
123456789012345678901234567890121234567890123456789012345678901212345 6
123456789012345678901234567890121234567890123456789012345678901212345 6
123456789012345678901234567890121234567890123456789012345678901212345 6
123456789012345678901234567890121234567890123456789012345678901212345
If the area of the top of the rectangular 6
123456789012345678901234567890121234567890123456789012345678901212345 66
3456789012345678901234567890121234567890123456789012345678901212345
1 box is 15, what is the surface area of the
12 3456789012345678901234567890121234567890123456789012345678901212345
21. In which quarter was the ratio of criminal 6
12 3456789012345678901234567890121234567890123456789012345678901212345
entire figure? 6
12 3456789012345678901234567890121234567890123456789012345678901212345 6
2 3456789012345678901234567890121234567890123456789012345678901212345
123456789012345678901234567890121234567890123456789012345678901212345
to civil cases the greatest? 6
123456789012345678901234567890121234567890123456789012345678901212345
A. 69 6
123456789012345678901234567890121234567890123456789012345678901212345
st 6
B. 72 A. 1 quarter,
123456789012345678901234567890121234567890123456789012345678901212345 1970 6
123456789012345678901234567890121234567890123456789012345678901212345
nd
B. 2 quarter, 1970 6
123456789012345678901234567890121234567890123456789012345678901212345
C. 78 6
123456789012345678901234567890121234567890123456789012345678901212345
rd
C. 3 quarter, 1970 6
123456789012345678901234567890121234567890123456789012345678901212345
D. 85 6
123456789012345678901234567890121234567890123456789012345678901212345
th
D. 4 quarter, 1970 6
1 E. 90 6
2 3456789012345678901234567890121234567890123456789012345678901212345
123456789012345678901234567890121234567890123456789012345678901212345
st
E. 1 quarter, 1971 6
123456789012345678901234567890121234567890123456789012345678901212345 6
1 20. What is the median minus the mode of the 6
123456789012345678901234567890121234567890123456789012345678901212345 6
12
2
3456789012345678901234567890121234567890123456789012345678901212345
following number set {4, 16, 7, 2, 7, 10}? 22. Which of the
3456789012345678901234567890121234567890123456789012345678901212345
following best approximates 6
6
123456789012345678901234567890121234567890123456789012345678901212345
123456789012345678901234567890121234567890123456789012345678901212345
the ratio of female lawyers in Canada in 6
1 A. 23 6
12 3456789012345678901234567890121234567890123456789012345678901212345
1970 practicing Civil Law to Criminal 6
12 3456789012345678901234567890121234567890123456789012345678901212345
B. 0 6
2 3456789012345678901234567890121234567890123456789012345678901212345
123456789012345678901234567890121234567890123456789012345678901212345
Law? 6
123456789012345678901234567890121234567890123456789012345678901212345
C. 3 6
1 A. 1:2 6
123456789012345678901234567890121234567890123456789012345678901212345 6
D. 5
123456789012345678901234567890121234567890123456789012345678901212345
B. 4:1 6
12 3456789012345678901234567890121234567890123456789012345678901212345
E. 9 6
12 3456789012345678901234567890121234567890123456789012345678901212345
C. 1:3 6
12 3456789012345678901234567890121234567890123456789012345678901212345 6
12 3456789012345678901234567890121234567890123456789012345678901212345
D. 5:1 6
123456789012345678901234567890121234567890123456789012345678901212345 6
2 3456789012345678901234567890121234567890123456789012345678901212345
123456789012345678901234567890121234567890123456789012345678901212345
E. 1:4 6
123456789012345678901234567890121234567890123456789012345678901212345 66
123456789012345678901234567890121234567890123456789012345678901212345 6
123456789012345678901234567890121234567890123456789012345678901212345 6
1
12 3456789012345678901234567890121234567890123456789012345678901212345 6
12 3456789012345678901234567890121234567890123456789012345678901212345 6
12 3456789012345678901234567890121234567890123456789012345678901212345 6
12 3456789012345678901234567890121234567890123456789012345678901212345 6
123456789012345678901234567890121234567890123456789012345678901212345 6
123456789012345678901234567890121234567890123456789012345678901212345
2 6
1 3456789012345678901234567890121234567890123456789012345678901212345 6
12 3456789012345678901234567890121234567890123456789012345678901212345 6
123456789012345678901234567890121234567890123456789012345678901212345 6
12 3456789012345678901234567890121234567890123456789012345678901212345 6
123456789012345678901234567890121234567890123456789012345678901212345 6
123456789012345678901234567890121234567890123456789012345678901212345 6
123456789012345678901234567890121234567890123456789012345678901212345 66
262 123456789012345678901234567890121234567890123456789012345678901212345
1234567890123456789012345678901212345678901234567890123456789012123456

www.petersons.com
TEST 1
1234567890123456789012345678901212345678901234567890123456789012123456
123456789012345678901234567890121234567890123456789012345678901212345 6
12 3456789012345678901234567890121234567890123456789012345678901212345 6
123456789012345678901234567890121234567890123456789012345678901212345
23. Which field of law in Canada had the 26. The average fifth grader scored a 15 on 6
12 3456789012345678901234567890121234567890123456789012345678901212345 6
12 3456789012345678901234567890121234567890123456789012345678901212345
greatest number of practitioners in 1970? the GST test. The average sixth grader 6
123456789012345678901234567890121234567890123456789012345678901212345 6
12
2
3456789012345678901234567890121234567890123456789012345678901212345
scored 18 on
3456789012345678901234567890121234567890123456789012345678901212345
the GST test. There are half 6
6
1 A. Criminal Law
12 3456789012345678901234567890121234567890123456789012345678901212345
as many sixth graders as fifth graders. 6
12 3456789012345678901234567890121234567890123456789012345678901212345
B. Civil Law 6
12 3456789012345678901234567890121234567890123456789012345678901212345
What is the overall average score? 6
12
2
3456789012345678901234567890121234567890123456789012345678901212345 6
C. Corporate Law
3456789012345678901234567890121234567890123456789012345678901212345 6
1
12 3456789012345678901234567890121234567890123456789012345678901212345
D. Real Estate Law A. 15.5 6
12 3456789012345678901234567890121234567890123456789012345678901212345 6
12 3456789012345678901234567890121234567890123456789012345678901212345 6
E. Public Interest Law B. 16
12 3456789012345678901234567890121234567890123456789012345678901212345 6
123456789012345678901234567890121234567890123456789012345678901212345
C. 16.5 6
12 3456789012345678901234567890121234567890123456789012345678901212345
24. 6
12 3456789012345678901234567890121234567890123456789012345678901212345 6
D. 17
12 3456789012345678901234567890121234567890123456789012345678901212345
E. 17.5 6
12
2
3456789012345678901234567890121234567890123456789012345678901212345
3456789012345678901234567890121234567890123456789012345678901212345
6
6
1
12 3456789012345678901234567890121234567890123456789012345678901212345 6
12 3456789012345678901234567890121234567890123456789012345678901212345
27. 6
12 3456789012345678901234567890121234567890123456789012345678901212345 6
12 3456789012345678901234567890121234567890123456789012345678901212345 6
12 3456789012345678901234567890121234567890123456789012345678901212345 6
12 3456789012345678901234567890121234567890123456789012345678901212345 6
12 3456789012345678901234567890121234567890123456789012345678901212345 6
12 3456789012345678901234567890121234567890123456789012345678901212345 6
12 3456789012345678901234567890121234567890123456789012345678901212345 6
123456789012345678901234567890121234567890123456789012345678901212345
2 6
1 3456789012345678901234567890121234567890123456789012345678901212345 6
12 3456789012345678901234567890121234567890123456789012345678901212345 6
In the figure above the non-shaded region
12 3456789012345678901234567890121234567890123456789012345678901212345
is a quarter circle. If the area of rectangle 6
12 3456789012345678901234567890121234567890123456789012345678901212345 6
2 3456789012345678901234567890121234567890123456789012345678901212345
123456789012345678901234567890121234567890123456789012345678901212345
ABCD is the sum of 12 and AB 5 4, what 6
123456789012345678901234567890121234567890123456789012345678901212345 6
123456789012345678901234567890121234567890123456789012345678901212345
is the length of arc BE 1 ED? 6
123456789012345678901234567890121234567890123456789012345678901212345 66
123456789012345678901234567890121234567890123456789012345678901212345
123456789012345678901234567890121234567890123456789012345678901212345
3 6
123456789012345678901234567890121234567890123456789012345678901212345
A. p 1 1 6
123456789012345678901234567890121234567890123456789012345678901212345
2 Which function is graphed above? 6
123456789012345678901234567890121234567890123456789012345678901212345 6
1 2 6
2 3456789012345678901234567890121234567890123456789012345678901212345
123456789012345678901234567890121234567890123456789012345678901212345
B. 2p 1 1 A. y 5 x 2 1 6
1234567890123456789012345678901212345678901234567890123456789012123452 6
1 5
B. x 5 y 2 1 6
123456789012345678901234567890121234567890123456789012345678901212345
C. p 1 1 2
C. y 5 x 1 1 6
12
2
3456789012345678901234567890121234567890123456789012345678901212345
2
3456789012345678901234567890121234567890123456789012345678901212345
6
6
1234567890123456789012345678901212345678901234567890123456789012123452
D. x 5 y 1 1
123456789012345678901234567890121234567890123456789012345678901212345 6
1 D. 6p 1 2 2
E. y 5 x 1 1 2
6
12 3456789012345678901234567890121234567890123456789012345678901212345 6
12 3456789012345678901234567890121234567890123456789012345678901212345 6
E. 8p 1 2
123456789012345678901234567890121234567890123456789012345678901212345 66
2 3456789012345678901234567890121234567890123456789012345678901212345
123456789012345678901234567890121234567890123456789012345678901212345 6
1 25. What is the length of the longest distance
123456789012345678901234567890121234567890123456789012345678901212345 6
123456789012345678901234567890121234567890123456789012345678901212345 6
12 3456789012345678901234567890121234567890123456789012345678901212345
between two corners of a rectangular box 6
12 3456789012345678901234567890121234567890123456789012345678901212345 6
12 3456789012345678901234567890121234567890123456789012345678901212345 6
with sides 2, 3, and 4?
12 3456789012345678901234567890121234567890123456789012345678901212345 6
123456789012345678901234567890121234567890123456789012345678901212345
A. 2=7 6
123456789012345678901234567890121234567890123456789012345678901212345
2 6
123456789012345678901234567890121234567890123456789012345678901212345 6
123456789012345678901234567890121234567890123456789012345678901212345
B. 5 6
123456789012345678901234567890121234567890123456789012345678901212345 6
1 3456789012345678901234567890121234567890123456789012345678901212345
C. =29 6
123456789012345678901234567890121234567890123456789012345678901212345 66
2 3456789012345678901234567890121234567890123456789012345678901212345
123456789012345678901234567890121234567890123456789012345678901212345 6
123456789012345678901234567890121234567890123456789012345678901212345 6
123456789012345678901234567890121234567890123456789012345678901212345
D. =33
6
1
2 3456789012345678901234567890121234567890123456789012345678901212345
123456789012345678901234567890121234567890123456789012345678901212345 6
1 E. 6 6
12 3456789012345678901234567890121234567890123456789012345678901212345 6
123456789012345678901234567890121234567890123456789012345678901212345 6
12 3456789012345678901234567890121234567890123456789012345678901212345 6
123456789012345678901234567890121234567890123456789012345678901212345 6
123456789012345678901234567890121234567890123456789012345678901212345 6
123456789012345678901234567890121234567890123456789012345678901212345 6
123456789012345678901234567890121234567890123456789012345678901212345 6 263
1234567890123456789012345678901212345678901234567890123456789012123456
Part IV: Three Practice GREs

1234567890123456789012345678901212345678901234567890123456789012123456
123456789012345678901234567890121234567890123456789012345678901212345 6
12 3456789012345678901234567890121234567890123456789012345678901212345 6
123456789012345678901234567890121234567890123456789012345678901212345
28. 6
12 3456789012345678901234567890121234567890123456789012345678901212345 6
12 3456789012345678901234567890121234567890123456789012345678901212345 6
123456789012345678901234567890121234567890123456789012345678901212345 6
12 3456789012345678901234567890121234567890123456789012345678901212345 6
123456789012345678901234567890121234567890123456789012345678901212345 6
12 3456789012345678901234567890121234567890123456789012345678901212345 6
12 3456789012345678901234567890121234567890123456789012345678901212345 6
12 3456789012345678901234567890121234567890123456789012345678901212345 6
12 3456789012345678901234567890121234567890123456789012345678901212345 6
1 3456789012345678901234567890121234567890123456789012345678901212345 6
2
12 3456789012345678901234567890121234567890123456789012345678901212345 6
12 3456789012345678901234567890121234567890123456789012345678901212345 6
12 3456789012345678901234567890121234567890123456789012345678901212345 6
12 3456789012345678901234567890121234567890123456789012345678901212345 6
1 3456789012345678901234567890121234567890123456789012345678901212345 6
2
12 3456789012345678901234567890121234567890123456789012345678901212345 6
12 3456789012345678901234567890121234567890123456789012345678901212345 6
12 3456789012345678901234567890121234567890123456789012345678901212345 6
12 3456789012345678901234567890121234567890123456789012345678901212345 6
1 3456789012345678901234567890121234567890123456789012345678901212345 6
2
12 3456789012345678901234567890121234567890123456789012345678901212345 6
12 3456789012345678901234567890121234567890123456789012345678901212345
The two figures in the circle are squares, 6
12 3456789012345678901234567890121234567890123456789012345678901212345 6
12 3456789012345678901234567890121234567890123456789012345678901212345 6
each with one corner at the center of the
2 3456789012345678901234567890121234567890123456789012345678901212345
123456789012345678901234567890121234567890123456789012345678901212345 6
123456789012345678901234567890121234567890123456789012345678901212345
circle and one corner touching the 6
123456789012345678901234567890121234567890123456789012345678901212345 6
123456789012345678901234567890121234567890123456789012345678901212345 66
circumference. If the radius of the circle is
123456789012345678901234567890121234567890123456789012345678901212345
123456789012345678901234567890121234567890123456789012345678901212345
4, what is the area of the shaded region? 6
1 6
12 3456789012345678901234567890121234567890123456789012345678901212345 6
12 3456789012345678901234567890121234567890123456789012345678901212345 6
A. 2(p 2 1)
12 3456789012345678901234567890121234567890123456789012345678901212345
B. 6(p 2 1) 6
12 3456789012345678901234567890121234567890123456789012345678901212345 6
12 3456789012345678901234567890121234567890123456789012345678901212345
C. 8(p 2 1) 6
12 3456789012345678901234567890121234567890123456789012345678901212345 6
12 3456789012345678901234567890121234567890123456789012345678901212345
D. 12(p 2 1) 6
12 3456789012345678901234567890121234567890123456789012345678901212345 6
2 3456789012345678901234567890121234567890123456789012345678901212345
123456789012345678901234567890121234567890123456789012345678901212345
E. 16(p 2 1) 6
123456789012345678901234567890121234567890123456789012345678901212345 6
123456789012345678901234567890121234567890123456789012345678901212345 66
123456789012345678901234567890121234567890123456789012345678901212345 6
1 Verbal
12 3456789012345678901234567890121234567890123456789012345678901212345 6
12 3456789012345678901234567890121234567890123456789012345678901212345 6
123456789012345678901234567890121234567890123456789012345678901212345 6
123456789012345678901234567890121234567890123456789012345678901212345
30 Questions—30 Minutes 6
12
2
3456789012345678901234567890121234567890123456789012345678901212345 6
3456789012345678901234567890121234567890123456789012345678901212345 6
123456789012345678901234567890121234567890123456789012345678901212345
123456789012345678901234567890121234567890123456789012345678901212345
Antonyms 6
1 6
12 3456789012345678901234567890121234567890123456789012345678901212345 6
12 3456789012345678901234567890121234567890123456789012345678901212345 6
12 3456789012345678901234567890121234567890123456789012345678901212345
Directions: Each question contains a word printed in capital letters, followed by five 6
12 3456789012345678901234567890121234567890123456789012345678901212345 6
123456789012345678901234567890121234567890123456789012345678901212345 6
answer choices. Choose the answer choice that contains the word or phrase most nearly
123456789012345678901234567890121234567890123456789012345678901212345 6
123456789012345678901234567890121234567890123456789012345678901212345
OPPOSITE in meaning to the word in capital letters. 6
123456789012345678901234567890121234567890123456789012345678901212345 66
2 3456789012345678901234567890121234567890123456789012345678901212345
123456789012345678901234567890121234567890123456789012345678901212345 6
123456789012345678901234567890121234567890123456789012345678901212345 6
123456789012345678901234567890121234567890123456789012345678901212345 6
1 1. ZEAL :
12 3456789012345678901234567890121234567890123456789012345678901212345
2. HETEROGENEOUS : 6
12 3456789012345678901234567890121234567890123456789012345678901212345 6
12 3456789012345678901234567890121234567890123456789012345678901212345 6
12 3456789012345678901234567890121234567890123456789012345678901212345
A. enthusiasm A. orthodox 6
123456789012345678901234567890121234567890123456789012345678901212345
B. apathy B. abnormal 6
123456789012345678901234567890121234567890123456789012345678901212345 66
2 3456789012345678901234567890121234567890123456789012345678901212345
123456789012345678901234567890121234567890123456789012345678901212345
C. antagonism C. uniform
6
123456789012345678901234567890121234567890123456789012345678901212345 6
123456789012345678901234567890121234567890123456789012345678901212345
D. pride D. translucent
6
1
2 3456789012345678901234567890121234567890123456789012345678901212345
123456789012345678901234567890121234567890123456789012345678901212345
E. neutrality E. deficient 6
1 6
12 3456789012345678901234567890121234567890123456789012345678901212345 6
123456789012345678901234567890121234567890123456789012345678901212345 6
12 3456789012345678901234567890121234567890123456789012345678901212345 6
123456789012345678901234567890121234567890123456789012345678901212345 6
123456789012345678901234567890121234567890123456789012345678901212345 6
123456789012345678901234567890121234567890123456789012345678901212345 66
264 123456789012345678901234567890121234567890123456789012345678901212345
1234567890123456789012345678901212345678901234567890123456789012123456

www.petersons.com
TEST 1
1234567890123456789012345678901212345678901234567890123456789012123456
123456789012345678901234567890121234567890123456789012345678901212345 6
12 3456789012345678901234567890121234567890123456789012345678901212345 6
123456789012345678901234567890121234567890123456789012345678901212345
3. FEALTY : 7. OPPROBRIUM : 6
12 3456789012345678901234567890121234567890123456789012345678901212345 6
12 3456789012345678901234567890121234567890123456789012345678901212345 6
123456789012345678901234567890121234567890123456789012345678901212345
A. disloyalty A. contempt 6
12
2
3456789012345678901234567890121234567890123456789012345678901212345
3456789012345678901234567890121234567890123456789012345678901212345
6
6
1 B. turbidity B. platitude
12 3456789012345678901234567890121234567890123456789012345678901212345 6
12 3456789012345678901234567890121234567890123456789012345678901212345 6
C. probity C. commendation
12 3456789012345678901234567890121234567890123456789012345678901212345 6
12
2
3456789012345678901234567890121234567890123456789012345678901212345
D. fidelity D. prescience
3456789012345678901234567890121234567890123456789012345678901212345
6
6
1
12 3456789012345678901234567890121234567890123456789012345678901212345
E. substantiality E. ennui 6
12 3456789012345678901234567890121234567890123456789012345678901212345 6
12 3456789012345678901234567890121234567890123456789012345678901212345 6
12 3456789012345678901234567890121234567890123456789012345678901212345
4. ARCHAIC : 8. TYRO : 6
1 3456789012345678901234567890121234567890123456789012345678901212345
2 6
12 3456789012345678901234567890121234567890123456789012345678901212345 6
12 3456789012345678901234567890121234567890123456789012345678901212345
A. lusterless A. miscreant 6
12 3456789012345678901234567890121234567890123456789012345678901212345 6
12
2
3456789012345678901234567890121234567890123456789012345678901212345 6
B. odious B. curmudgeon
3456789012345678901234567890121234567890123456789012345678901212345 6
1
12 3456789012345678901234567890121234567890123456789012345678901212345
C. clichéd C. virtuoso 6
12 3456789012345678901234567890121234567890123456789012345678901212345 6
12 3456789012345678901234567890121234567890123456789012345678901212345 6
D. customary D. dilettante
12 3456789012345678901234567890121234567890123456789012345678901212345 6
12 3456789012345678901234567890121234567890123456789012345678901212345
E. novel E. misogynist 6
12 3456789012345678901234567890121234567890123456789012345678901212345 6
12 3456789012345678901234567890121234567890123456789012345678901212345 6
12 3456789012345678901234567890121234567890123456789012345678901212345
5. ASSUAGE : 9. SOPORIFIC : 6
12 3456789012345678901234567890121234567890123456789012345678901212345 6
123456789012345678901234567890121234567890123456789012345678901212345 66
2 3456789012345678901234567890121234567890123456789012345678901212345
1 A. pillory A. stimulant
12 3456789012345678901234567890121234567890123456789012345678901212345 6
12 3456789012345678901234567890121234567890123456789012345678901212345
B. satiate B. exigent 6
12 3456789012345678901234567890121234567890123456789012345678901212345 6
123456789012345678901234567890121234567890123456789012345678901212345 66
2 3456789012345678901234567890121234567890123456789012345678901212345
C. blandish C. torpor
123456789012345678901234567890121234567890123456789012345678901212345
123456789012345678901234567890121234567890123456789012345678901212345
D. ameliorate D. sophomoric 6
123456789012345678901234567890121234567890123456789012345678901212345 6
123456789012345678901234567890121234567890123456789012345678901212345 66
E. provoke E. jejune
123456789012345678901234567890121234567890123456789012345678901212345 6
123456789012345678901234567890121234567890123456789012345678901212345 6
123456789012345678901234567890121234567890123456789012345678901212345
123456789012345678901234567890121234567890123456789012345678901212345
6. MERCURIAL : 6
1 6
2 3456789012345678901234567890121234567890123456789012345678901212345
123456789012345678901234567890121234567890123456789012345678901212345
A. emotionally stable 6
123456789012345678901234567890121234567890123456789012345678901212345 6
1 B. bitterly rancorous 6
123456789012345678901234567890121234567890123456789012345678901212345 6
12 3456789012345678901234567890121234567890123456789012345678901212345
C. embarrassingly sordid 6
12 3456789012345678901234567890121234567890123456789012345678901212345 6
12 3456789012345678901234567890121234567890123456789012345678901212345
D. constantly morose 6
123456789012345678901234567890121234567890123456789012345678901212345 6
12 3456789012345678901234567890121234567890123456789012345678901212345
E. extremely melodramatic 6
12 3456789012345678901234567890121234567890123456789012345678901212345 6
123456789012345678901234567890121234567890123456789012345678901212345 66
2 3456789012345678901234567890121234567890123456789012345678901212345
123456789012345678901234567890121234567890123456789012345678901212345 6
1
123456789012345678901234567890121234567890123456789012345678901212345
Analogies 6
123456789012345678901234567890121234567890123456789012345678901212345 6
123456789012345678901234567890121234567890123456789012345678901212345 66
2 3456789012345678901234567890121234567890123456789012345678901212345
123456789012345678901234567890121234567890123456789012345678901212345 6
123456789012345678901234567890121234567890123456789012345678901212345
Directions: In each question, there is an initial pair of words or phrases that are related in
6
123456789012345678901234567890121234567890123456789012345678901212345 6
1 some manner. Select the answer choice containing the lettered pair that best expresses a
2 3456789012345678901234567890121234567890123456789012345678901212345
123456789012345678901234567890121234567890123456789012345678901212345
relationship similar to the initial pair. 6
123456789012345678901234567890121234567890123456789012345678901212345 6
123456789012345678901234567890121234567890123456789012345678901212345 66
123456789012345678901234567890121234567890123456789012345678901212345 6
1
12 3456789012345678901234567890121234567890123456789012345678901212345 6
12 3456789012345678901234567890121234567890123456789012345678901212345
10. PRATTLE : BABBLE :: 11. FAWN : FLATTER :: 6
12 3456789012345678901234567890121234567890123456789012345678901212345 6
12 3456789012345678901234567890121234567890123456789012345678901212345 6
123456789012345678901234567890121234567890123456789012345678901212345
A. battle : fight A. praise : criticize 6
12 3456789012345678901234567890121234567890123456789012345678901212345 6
123456789012345678901234567890121234567890123456789012345678901212345
B. digest : eat B. divulge : tell 6
12 3456789012345678901234567890121234567890123456789012345678901212345 6
123456789012345678901234567890121234567890123456789012345678901212345
C. engage : withdraw C. burgeon : grow 6
12
2
3456789012345678901234567890121234567890123456789012345678901212345
3456789012345678901234567890121234567890123456789012345678901212345
6
6
1 D. misconstrue : grasp D. bolster : disapprove
123456789012345678901234567890121234567890123456789012345678901212345 6
123456789012345678901234567890121234567890123456789012345678901212345
E. mettle : disassociate E. misquote : lie 6
123456789012345678901234567890121234567890123456789012345678901212345 6 265
1234567890123456789012345678901212345678901234567890123456789012123456
Part IV: Three Practice GREs

1234567890123456789012345678901212345678901234567890123456789012123456
123456789012345678901234567890121234567890123456789012345678901212345 6
12 3456789012345678901234567890121234567890123456789012345678901212345 6
123456789012345678901234567890121234567890123456789012345678901212345
12. DENOUEMENT: STORY :: 15. ERSATZ : ARTIFICAL :: 6
12 3456789012345678901234567890121234567890123456789012345678901212345 6
12 3456789012345678901234567890121234567890123456789012345678901212345 6
123456789012345678901234567890121234567890123456789012345678901212345
A. prologue : novel A. natural : polluted 6
12
2
3456789012345678901234567890121234567890123456789012345678901212345
3456789012345678901234567890121234567890123456789012345678901212345
6
6
1 B. sunset : day B. abstruse : facile
12 3456789012345678901234567890121234567890123456789012345678901212345 6
12 3456789012345678901234567890121234567890123456789012345678901212345 6
C. intermission : play C. caustic : astute
12 3456789012345678901234567890121234567890123456789012345678901212345 6
12
2
3456789012345678901234567890121234567890123456789012345678901212345
D. sidewalk : bistro D. nascent : incipient
3456789012345678901234567890121234567890123456789012345678901212345
6
6
1
12 3456789012345678901234567890121234567890123456789012345678901212345
E. traffic : thoroughfare E. ribald : naïve 6
12 3456789012345678901234567890121234567890123456789012345678901212345 6
12 3456789012345678901234567890121234567890123456789012345678901212345 6
12 3456789012345678901234567890121234567890123456789012345678901212345
13. FEDERALISM : POLITICAL ORDER :: 16. DIATRIBE : EULOGY :: 6
1 3456789012345678901234567890121234567890123456789012345678901212345
2 6
12 3456789012345678901234567890121234567890123456789012345678901212345 6
12 3456789012345678901234567890121234567890123456789012345678901212345
A. class : student A. recommendation : commendation 6
12 3456789012345678901234567890121234567890123456789012345678901212345 6
12
2
3456789012345678901234567890121234567890123456789012345678901212345 6
B. dictatorship : anarchy B. impugnation
3456789012345678901234567890121234567890123456789012345678901212345
: chastisement
6
1
12 3456789012345678901234567890121234567890123456789012345678901212345
C. colleague : cohort C. rebuke : reproof 6
12 3456789012345678901234567890121234567890123456789012345678901212345 6
12 3456789012345678901234567890121234567890123456789012345678901212345 6
D. insomnia : sleep disorder D. verity : axiomatic
12 3456789012345678901234567890121234567890123456789012345678901212345 6
12 3456789012345678901234567890121234567890123456789012345678901212345
E. extrinsic : external E. remonstration : encomium 6
12 3456789012345678901234567890121234567890123456789012345678901212345 6
12 3456789012345678901234567890121234567890123456789012345678901212345 6
12 3456789012345678901234567890121234567890123456789012345678901212345
14. SPURIOUS : TRUE :: 17. HAGIOGRAPHY: VENERATES :: 6
12 3456789012345678901234567890121234567890123456789012345678901212345 6
123456789012345678901234567890121234567890123456789012345678901212345 66
2 3456789012345678901234567890121234567890123456789012345678901212345
1 A. sporadic : anomalous A. polemic : attacks
12 3456789012345678901234567890121234567890123456789012345678901212345 6
12 3456789012345678901234567890121234567890123456789012345678901212345
B. ingenious : innovative B. vendetta : scares 6
12 3456789012345678901234567890121234567890123456789012345678901212345 6
123456789012345678901234567890121234567890123456789012345678901212345 66
2 3456789012345678901234567890121234567890123456789012345678901212345
C. hilarious : witty C. complaint : angers
123456789012345678901234567890121234567890123456789012345678901212345
123456789012345678901234567890121234567890123456789012345678901212345
D. insipid : flavorful D. report : obscures 6
123456789012345678901234567890121234567890123456789012345678901212345 6
123456789012345678901234567890121234567890123456789012345678901212345 66
E. base : dishonest E. truism : enunciates
123456789012345678901234567890121234567890123456789012345678901212345 6
123456789012345678901234567890121234567890123456789012345678901212345 6
123456789012345678901234567890121234567890123456789012345678901212345 6
123456789012345678901234567890121234567890123456789012345678901212345
Sentence Completions
6
1
123456789012345678901234567890121234567890123456789012345678901212345 66
2 3456789012345678901234567890121234567890123456789012345678901212345
123456789012345678901234567890121234567890123456789012345678901212345 6
1 Directions: Each sentence below contains one or two blanks, with each blank corre-
123456789012345678901234567890121234567890123456789012345678901212345 6
12 3456789012345678901234567890121234567890123456789012345678901212345
sponding to an omitted word. Find the answer choice that has the word or set or words 6
123456789012345678901234567890121234567890123456789012345678901212345 66
2 3456789012345678901234567890121234567890123456789012345678901212345
123456789012345678901234567890121234567890123456789012345678901212345
that best fits the meaning of the sentence as a whole.
6
1
12 3456789012345678901234567890121234567890123456789012345678901212345 6
12 3456789012345678901234567890121234567890123456789012345678901212345 6
12 3456789012345678901234567890121234567890123456789012345678901212345 6
12 3456789012345678901234567890121234567890123456789012345678901212345 6
123456789012345678901234567890121234567890123456789012345678901212345
18. Although __________ continues on minor 19. Recent advances in research methodolo- 6
123456789012345678901234567890121234567890123456789012345678901212345 6
123456789012345678901234567890121234567890123456789012345678901212345
points, the __________ themes of the gies in neuroscience have created excite- 6
123456789012345678901234567890121234567890123456789012345678901212345 66
2 3456789012345678901234567890121234567890123456789012345678901212345
123456789012345678901234567890121234567890123456789012345678901212345
General’s plan have been accepted. ment in academic circles and have
6
123456789012345678901234567890121234567890123456789012345678901212345 6
123456789012345678901234567890121234567890123456789012345678901212345
__________ greater interest in funding
6
1 A. discussion. .peripheral
2 3456789012345678901234567890121234567890123456789012345678901212345
123456789012345678901234567890121234567890123456789012345678901212345
new research projects in the field. 6
123456789012345678901234567890121234567890123456789012345678901212345
B. debate. .central 6
123456789012345678901234567890121234567890123456789012345678901212345 6
123456789012345678901234567890121234567890123456789012345678901212345 66
C. dissension. .cultural A. stymied
1
12 3456789012345678901234567890121234567890123456789012345678901212345
D. discovery. .main B. intensified 6
12 3456789012345678901234567890121234567890123456789012345678901212345 6
12 3456789012345678901234567890121234567890123456789012345678901212345 6
E. agreement. .basic C. magnified
12 3456789012345678901234567890121234567890123456789012345678901212345 6
123456789012345678901234567890121234567890123456789012345678901212345
D. engendered 6
2 3456789012345678901234567890121234567890123456789012345678901212345
123456789012345678901234567890121234567890123456789012345678901212345 6
1 E. multiplied 6
12 3456789012345678901234567890121234567890123456789012345678901212345 6
123456789012345678901234567890121234567890123456789012345678901212345 6
12 3456789012345678901234567890121234567890123456789012345678901212345 6
123456789012345678901234567890121234567890123456789012345678901212345 6
123456789012345678901234567890121234567890123456789012345678901212345 6
123456789012345678901234567890121234567890123456789012345678901212345 66
266 123456789012345678901234567890121234567890123456789012345678901212345
1234567890123456789012345678901212345678901234567890123456789012123456

www.petersons.com
TEST 1
1234567890123456789012345678901212345678901234567890123456789012123456
123456789012345678901234567890121234567890123456789012345678901212345 6
12 3456789012345678901234567890121234567890123456789012345678901212345 6
123456789012345678901234567890121234567890123456789012345678901212345
20. Although the boom appeared to still be in 23. The professor became __________ after 6
12 3456789012345678901234567890121234567890123456789012345678901212345 6
12 3456789012345678901234567890121234567890123456789012345678901212345
full swing, in the latter years of the attempting to explain the important 6
123456789012345678901234567890121234567890123456789012345678901212345 6
12
2
3456789012345678901234567890121234567890123456789012345678901212345 6
decade, most economic indicators pointed concept
3456789012345678901234567890121234567890123456789012345678901212345
multiple different ways, but her
6
1
12 3456789012345678901234567890121234567890123456789012345678901212345
to __________ times ahead. visible frustration only made the student 6
12 3456789012345678901234567890121234567890123456789012345678901212345 6
12 3456789012345678901234567890121234567890123456789012345678901212345
less likely to __________ the material. 6
12
2
3456789012345678901234567890121234567890123456789012345678901212345 6
A. leaner
3456789012345678901234567890121234567890123456789012345678901212345 6
1
12 3456789012345678901234567890121234567890123456789012345678901212345
B. profitable A. mystified. .assimilate 6
12 3456789012345678901234567890121234567890123456789012345678901212345 6
12 3456789012345678901234567890121234567890123456789012345678901212345 6
C. healthy B. belligerent. .understand
12 3456789012345678901234567890121234567890123456789012345678901212345 6
123456789012345678901234567890121234567890123456789012345678901212345
D. daring C. stupefied. .comprehend 6
12 3456789012345678901234567890121234567890123456789012345678901212345 6
12 3456789012345678901234567890121234567890123456789012345678901212345 6
E. aberrant D. vexed. .grasp
12 3456789012345678901234567890121234567890123456789012345678901212345
E. agitated. .eschew 6
12
2
3456789012345678901234567890121234567890123456789012345678901212345
3456789012345678901234567890121234567890123456789012345678901212345
6
6
1 21. Though often crude and overly
12 3456789012345678901234567890121234567890123456789012345678901212345 6
12 3456789012345678901234567890121234567890123456789012345678901212345
__________, early theories of biogenesis 6
12 3456789012345678901234567890121234567890123456789012345678901212345 6
12 3456789012345678901234567890121234567890123456789012345678901212345 6
were important for the __________ of the
2 3456789012345678901234567890121234567890123456789012345678901212345
123456789012345678901234567890121234567890123456789012345678901212345 6
123456789012345678901234567890121234567890123456789012345678901212345
question of life’s origin. 6
123456789012345678901234567890121234567890123456789012345678901212345 66
123456789012345678901234567890121234567890123456789012345678901212345
123456789012345678901234567890121234567890123456789012345678901212345
A. dramatic. .institutionalization 6
123456789012345678901234567890121234567890123456789012345678901212345 6
1 B. hypothetical. .clarification 6
12 3456789012345678901234567890121234567890123456789012345678901212345 6
12 3456789012345678901234567890121234567890123456789012345678901212345 6
C. inscrutable. .illumination
12 3456789012345678901234567890121234567890123456789012345678901212345 6
12 3456789012345678901234567890121234567890123456789012345678901212345
D. schematic. .extermination 6
12 3456789012345678901234567890121234567890123456789012345678901212345 6
12 3456789012345678901234567890121234567890123456789012345678901212345 6
E. simplistic. .advancement
12 3456789012345678901234567890121234567890123456789012345678901212345 6
12 3456789012345678901234567890121234567890123456789012345678901212345 6
123456789012345678901234567890121234567890123456789012345678901212345
2 22. The lawyer’s argumentation was 6
123456789012345678901234567890121234567890123456789012345678901212345 6
123456789012345678901234567890121234567890123456789012345678901212345
__________ and factually misleading, but 6
123456789012345678901234567890121234567890123456789012345678901212345 6
1 3456789012345678901234567890121234567890123456789012345678901212345 6
these problems in his closing argument
2 3456789012345678901234567890121234567890123456789012345678901212345
123456789012345678901234567890121234567890123456789012345678901212345 6
123456789012345678901234567890121234567890123456789012345678901212345
were masked by his __________ and 6
1 6
123456789012345678901234567890121234567890123456789012345678901212345 6
superior mastery of language.
12 3456789012345678901234567890121234567890123456789012345678901212345 6
12 3456789012345678901234567890121234567890123456789012345678901212345 6
12 3456789012345678901234567890121234567890123456789012345678901212345
A. spurious. .elusiveness 6
123456789012345678901234567890121234567890123456789012345678901212345 6
12 3456789012345678901234567890121234567890123456789012345678901212345 6
B. debonair. .equivocations
12 3456789012345678901234567890121234567890123456789012345678901212345 6
12 3456789012345678901234567890121234567890123456789012345678901212345
C. duplicitous. .dissembling 6
12 3456789012345678901234567890121234567890123456789012345678901212345
D. specious. .eloquence 6
123456789012345678901234567890121234567890123456789012345678901212345 6
123456789012345678901234567890121234567890123456789012345678901212345
E. spectacular. .suaveness 6
123456789012345678901234567890121234567890123456789012345678901212345 6
123456789012345678901234567890121234567890123456789012345678901212345 66
2 3456789012345678901234567890121234567890123456789012345678901212345
123456789012345678901234567890121234567890123456789012345678901212345 6
123456789012345678901234567890121234567890123456789012345678901212345 6
123456789012345678901234567890121234567890123456789012345678901212345 6
1
12 3456789012345678901234567890121234567890123456789012345678901212345 6
12 3456789012345678901234567890121234567890123456789012345678901212345 6
12 3456789012345678901234567890121234567890123456789012345678901212345 6
12 3456789012345678901234567890121234567890123456789012345678901212345 6
123456789012345678901234567890121234567890123456789012345678901212345 6
123456789012345678901234567890121234567890123456789012345678901212345
2 6
123456789012345678901234567890121234567890123456789012345678901212345 6
123456789012345678901234567890121234567890123456789012345678901212345 6
123456789012345678901234567890121234567890123456789012345678901212345 6
1 3456789012345678901234567890121234567890123456789012345678901212345 6
123456789012345678901234567890121234567890123456789012345678901212345 66
2 3456789012345678901234567890121234567890123456789012345678901212345
1
12 3456789012345678901234567890121234567890123456789012345678901212345 6
123456789012345678901234567890121234567890123456789012345678901212345 6
12 3456789012345678901234567890121234567890123456789012345678901212345 6
123456789012345678901234567890121234567890123456789012345678901212345 6
123456789012345678901234567890121234567890123456789012345678901212345 6
123456789012345678901234567890121234567890123456789012345678901212345 6
123456789012345678901234567890121234567890123456789012345678901212345 6 267
1234567890123456789012345678901212345678901234567890123456789012123456
Part IV: Three Practice GREs

1234567890123456789012345678901212345678901234567890123456789012123456
123456789012345678901234567890121234567890123456789012345678901212345 6
12 3456789012345678901234567890121234567890123456789012345678901212345 6
123456789012345678901234567890121234567890123456789012345678901212345
Reading Comprehension 6
12 3456789012345678901234567890121234567890123456789012345678901212345 6
12 3456789012345678901234567890121234567890123456789012345678901212345 6
123456789012345678901234567890121234567890123456789012345678901212345 6
12 3456789012345678901234567890121234567890123456789012345678901212345
Directions: Questions follow each passage. After reading the passage, determine the best 6
123456789012345678901234567890121234567890123456789012345678901212345 6
12 3456789012345678901234567890121234567890123456789012345678901212345
answer for each question based on the passage’s content. 6
12 3456789012345678901234567890121234567890123456789012345678901212345 6
12 3456789012345678901234567890121234567890123456789012345678901212345 6
12 3456789012345678901234567890121234567890123456789012345678901212345 6
1 3456789012345678901234567890121234567890123456789012345678901212345 6
2
12 3456789012345678901234567890121234567890123456789012345678901212345 6
12 3456789012345678901234567890121234567890123456789012345678901212345
Passage 1 presence of dark matter if the galaxy’s 6
12 3456789012345678901234567890121234567890123456789012345678901212345 6
12
2
3456789012345678901234567890121234567890123456789012345678901212345
motion requires gravitational interactions 6
3456789012345678901234567890121234567890123456789012345678901212345 6
1 Line The nature and structure of dark matter
12 3456789012345678901234567890121234567890123456789012345678901212345
greater than those afforded by the 6
12 3456789012345678901234567890121234567890123456789012345678901212345
is a primary topic in contemporary 6
12 3456789012345678901234567890121234567890123456789012345678901212345
galaxy’s light matter. 6
12
2
3456789012345678901234567890121234567890123456789012345678901212345
cosmology, the sub-discipline of as-
3456789012345678901234567890121234567890123456789012345678901212345
6
6
1 (40) Our understanding of dark matter
12 3456789012345678901234567890121234567890123456789012345678901212345
tronomy in which the universe is studied 6
12 3456789012345678901234567890121234567890123456789012345678901212345
does not extend much beyond this 6
12 3456789012345678901234567890121234567890123456789012345678901212345
(5) as a whole. Dark matter has aroused 6
12 3456789012345678901234567890121234567890123456789012345678901212345
rudimentary model of matter that is 6
2 3456789012345678901234567890121234567890123456789012345678901212345
123456789012345678901234567890121234567890123456789012345678901212345
great interest in cosmological circles, 6
123456789012345678901234567890121234567890123456789012345678901212345
gravitationally interactive but electro- 6
123456789012345678901234567890121234567890123456789012345678901212345
because current theories of the evolution 6
123456789012345678901234567890121234567890123456789012345678901212345
magnetically non-interactive. Since most 6
123456789012345678901234567890121234567890123456789012345678901212345
of the universe require the hypothesis 6
123456789012345678901234567890121234567890123456789012345678901212345
(45) astronomical probing of the universe 6
1 that dark matter constitutes nearly 6
12 3456789012345678901234567890121234567890123456789012345678901212345
three-fourths of the matter in the
employs light-sensitive instruments, 6
12 3456789012345678901234567890121234567890123456789012345678901212345
(10)
6
12 3456789012345678901234567890121234567890123456789012345678901212345
universe. This hypothesis, if true, makes
knowledge of dark matter comes only 6
2 3456789012345678901234567890121234567890123456789012345678901212345
indirectly. Thus, we are left to theoretical 6
123456789012345678901234567890121234567890123456789012345678901212345
123456789012345678901234567890121234567890123456789012345678901212345
understanding dark matter of signal 6
123456789012345678901234567890121234567890123456789012345678901212345
hypotheses about dark matter until more 6
123456789012345678901234567890121234567890123456789012345678901212345
importance to understanding the 6
123456789012345678901234567890121234567890123456789012345678901212345
fecund methods of investigating dark 6
(50)
123456789012345678901234567890121234567890123456789012345678901212345
universe. 6
123456789012345678901234567890121234567890123456789012345678901212345
matter are developed. 6
123456789012345678901234567890121234567890123456789012345678901212345
(15) Dark matter is matter that does not 6
123456789012345678901234567890121234567890123456789012345678901212345 6
1 interact with electromagnetic waves— 6
2 3456789012345678901234567890121234567890123456789012345678901212345
123456789012345678901234567890121234567890123456789012345678901212345 6
123456789012345678901234567890121234567890123456789012345678901212345
that is, it does not emit or absorb light. 24. The author’s tone toward the future 6
1 6
123456789012345678901234567890121234567890123456789012345678901212345
In contrast, light or regular matter does advancement of our understanding of 6
12 3456789012345678901234567890121234567890123456789012345678901212345 6
123456789012345678901234567890121234567890123456789012345678901212345 66
2 3456789012345678901234567890121234567890123456789012345678901212345
interact with electromagnetic waves and dark matter is
123456789012345678901234567890121234567890123456789012345678901212345 6
1 (20) therefore does emit and absorb light. A. deeply pessimistic.
12 3456789012345678901234567890121234567890123456789012345678901212345 6
12 3456789012345678901234567890121234567890123456789012345678901212345
When astronomers look into space with B. moderately skeptical. 6
2 3456789012345678901234567890121234567890123456789012345678901212345
123456789012345678901234567890121234567890123456789012345678901212345 6
123456789012345678901234567890121234567890123456789012345678901212345
electromagnetic sensitive telescopes (or C. guardedly hopeful. 6
1 6
123456789012345678901234567890121234567890123456789012345678901212345
other forms of light sensitive telescopes), D. tenaciously idealistic. 6
123456789012345678901234567890121234567890123456789012345678901212345 6
123456789012345678901234567890121234567890123456789012345678901212345 66
2 3456789012345678901234567890121234567890123456789012345678901212345
they can detect light matter but not dark E. basically theoretical.
123456789012345678901234567890121234567890123456789012345678901212345 6
123456789012345678901234567890121234567890123456789012345678901212345
(25) matter. This is how dark matter earned
6
123456789012345678901234567890121234567890123456789012345678901212345 6
1 its moniker. Astronomers are not able to
2 3456789012345678901234567890121234567890123456789012345678901212345
123456789012345678901234567890121234567890123456789012345678901212345
“see” it with conventional telescopes. 6
123456789012345678901234567890121234567890123456789012345678901212345 6
123456789012345678901234567890121234567890123456789012345678901212345
The existence of dark matter in a 6
123456789012345678901234567890121234567890123456789012345678901212345 66
1 particular region of space must be
12 3456789012345678901234567890121234567890123456789012345678901212345 6
12 3456789012345678901234567890121234567890123456789012345678901212345
(30) inferred from the dark matter’s gravita- 6
12 3456789012345678901234567890121234567890123456789012345678901212345 6
12 3456789012345678901234567890121234567890123456789012345678901212345
tional interaction with light matter. Like 6
123456789012345678901234567890121234567890123456789012345678901212345 6
123456789012345678901234567890121234567890123456789012345678901212345 66
2 3456789012345678901234567890121234567890123456789012345678901212345
light matter, dark matter gravitationally
1
12 3456789012345678901234567890121234567890123456789012345678901212345
attracts all matter and so sustains a 6
123456789012345678901234567890121234567890123456789012345678901212345 6
12
2
3456789012345678901234567890121234567890123456789012345678901212345 6
gravitational field. As an astronomer
3456789012345678901234567890121234567890123456789012345678901212345 6
1 (35) studies a galaxy, he or she can infer the
123456789012345678901234567890121234567890123456789012345678901212345 6
123456789012345678901234567890121234567890123456789012345678901212345 66
268 123456789012345678901234567890121234567890123456789012345678901212345
1234567890123456789012345678901212345678901234567890123456789012123456

www.petersons.com
TEST 1
1234567890123456789012345678901212345678901234567890123456789012123456
123456789012345678901234567890121234567890123456789012345678901212345 6
12 3456789012345678901234567890121234567890123456789012345678901212345 6
123456789012345678901234567890121234567890123456789012345678901212345
25. According to the passage, dark matter crowd of narratives the causes, inner 6
12 3456789012345678901234567890121234567890123456789012345678901212345 6
12 3456789012345678901234567890121234567890123456789012345678901212345
workings, and final consequences of the 6
123456789012345678901234567890121234567890123456789012345678901212345
I. gravitationally interacts with light 6
12
2
3456789012345678901234567890121234567890123456789012345678901212345
(15) Revolution all have differing assign-
3456789012345678901234567890121234567890123456789012345678901212345
6
6
1 matter.
12 3456789012345678901234567890121234567890123456789012345678901212345
ments. 6
12 3456789012345678901234567890121234567890123456789012345678901212345
II. does interact with electromagnetic 6
12 3456789012345678901234567890121234567890123456789012345678901212345
To the flagship conservative, Edmund 6
12
2
3456789012345678901234567890121234567890123456789012345678901212345
waves.
3456789012345678901234567890121234567890123456789012345678901212345
6
6
1 Burke, writing contemporaneously with
12 3456789012345678901234567890121234567890123456789012345678901212345
III. is similar to light matter in its 6
12 3456789012345678901234567890121234567890123456789012345678901212345
the events, the Revolution was a sudden 6
12 3456789012345678901234567890121234567890123456789012345678901212345
gravitational properties. 6
12
2
3456789012345678901234567890121234567890123456789012345678901212345 6
(20) and
3456789012345678901234567890121234567890123456789012345678901212345
radical manipulation of the French
6
1 A. I only
12 3456789012345678901234567890121234567890123456789012345678901212345
nation by a small class of philosophes, 6
12 3456789012345678901234567890121234567890123456789012345678901212345
B. III only 6
12 3456789012345678901234567890121234567890123456789012345678901212345 6
literati, and the newly-monied. In Burke’s
12 3456789012345678901234567890121234567890123456789012345678901212345
C. I and II only view, these so-termed aspirants to power 6
123456789012345678901234567890121234567890123456789012345678901212345 6
12 3456789012345678901234567890121234567890123456789012345678901212345
D. I and III only 6
12 3456789012345678901234567890121234567890123456789012345678901212345 6
successively harnessed methods of mass
12 3456789012345678901234567890121234567890123456789012345678901212345
E. I, II, and III persuasion to incite revolt and to 6
12 3456789012345678901234567890121234567890123456789012345678901212345
(25)
6
2 3456789012345678901234567890121234567890123456789012345678901212345
123456789012345678901234567890121234567890123456789012345678901212345
ultimately destroy the basic institutions 6
123456789012345678901234567890121234567890123456789012345678901212345 6
26. It can be inferred from the third para- of French society and in their place erect 6
123456789012345678901234567890121234567890123456789012345678901212345
123456789012345678901234567890121234567890123456789012345678901212345 6
a historical novelty, the newly christened 6
123456789012345678901234567890121234567890123456789012345678901212345
graph that
123456789012345678901234567890121234567890123456789012345678901212345 66
1 French Republic. In an instructive
12 3456789012345678901234567890121234567890123456789012345678901212345
A. light matter sustains a gravitational 6
12 3456789012345678901234567890121234567890123456789012345678901212345
field. (30) contrast, Alex de Tocqueville, a French 6
12 3456789012345678901234567890121234567890123456789012345678901212345 6
2 3456789012345678901234567890121234567890123456789012345678901212345
123456789012345678901234567890121234567890123456789012345678901212345
B. regular matter is more stable than provincial aristocrat, writing a few 6
123456789012345678901234567890121234567890123456789012345678901212345
decades after Burke, saw the Revolution 6
6
123456789012345678901234567890121234567890123456789012345678901212345
dark matter.
123456789012345678901234567890121234567890123456789012345678901212345
as a logical consequence and unfolding of 6
6
123456789012345678901234567890121234567890123456789012345678901212345
C. astronomers infer the presence of light
123456789012345678901234567890121234567890123456789012345678901212345 6
123456789012345678901234567890121234567890123456789012345678901212345
matter if a galaxy’s dark matter does the reforms set in motion by the ancien 6
123456789012345678901234567890121234567890123456789012345678901212345 6
123456789012345678901234567890121234567890123456789012345678901212345
not fully account for gravitational (35) regime. On this account, the Revolution 6
1 6
123456789012345678901234567890121234567890123456789012345678901212345 66
2 3456789012345678901234567890121234567890123456789012345678901212345
interactions. was a continuation of the “administrative
123456789012345678901234567890121234567890123456789012345678901212345
revolution” begun by Louis XVI. In this 6
1 D. dark matter is of primary importance
123456789012345678901234567890121234567890123456789012345678901212345 6
12
2
3456789012345678901234567890121234567890123456789012345678901212345
in contemporary cosmology. light, the French Republic was the
3456789012345678901234567890121234567890123456789012345678901212345
6
reasonable outgrowth of the evolution of 6
123456789012345678901234567890121234567890123456789012345678901212345
123456789012345678901234567890121234567890123456789012345678901212345
E. theoretical hypotheses about dark 6
1 6
12 3456789012345678901234567890121234567890123456789012345678901212345 6
matter are becoming more refined. (40) French society that stretched far back in
12 3456789012345678901234567890121234567890123456789012345678901212345 6
12 3456789012345678901234567890121234567890123456789012345678901212345
the eighteenth century, as opposed to a 6
12 3456789012345678901234567890121234567890123456789012345678901212345
Passage 2 monstrous and cataclysmic break with 6
123456789012345678901234567890121234567890123456789012345678901212345 6
123456789012345678901234567890121234567890123456789012345678901212345
France’s past. 6
123456789012345678901234567890121234567890123456789012345678901212345
Line Though the discipline of historiography 6
2 3456789012345678901234567890121234567890123456789012345678901212345
123456789012345678901234567890121234567890123456789012345678901212345Both of these figures, through their 6
123456789012345678901234567890121234567890123456789012345678901212345
purports to be an empirical endeavor free 6
123456789012345678901234567890121234567890123456789012345678901212345
(45) respective historical fame, have come to 6
123456789012345678901234567890121234567890123456789012345678901212345
of the biases of political persuasion and 6
1 embody loaded historical mythoi 6
2 3456789012345678901234567890121234567890123456789012345678901212345
123456789012345678901234567890121234567890123456789012345678901212345
temperament, the actual writing of 6
123456789012345678901234567890121234567890123456789012345678901212345
themselves, but the contrast between 6
123456789012345678901234567890121234567890123456789012345678901212345
(5) history is often reduced to the role of
their understanding and interpretation of 6
6
123456789012345678901234567890121234567890123456789012345678901212345 6
1 ideology’s handmaiden. Histories of the
12 3456789012345678901234567890121234567890123456789012345678901212345
the events of the French Revolution is 6
12 3456789012345678901234567890121234567890123456789012345678901212345
French Revolution serve as a prime 6
12 3456789012345678901234567890121234567890123456789012345678901212345
(50) still illuminating with respect to histori- 6
12 3456789012345678901234567890121234567890123456789012345678901212345
example of this historiographic tendency. 6
123456789012345678901234567890121234567890123456789012345678901212345
ography. The complexity of large-scale 6
2 3456789012345678901234567890121234567890123456789012345678901212345
of the histories written on the
historical events resists simplification into 6
123456789012345678901234567890121234567890123456789012345678901212345
A survey
1 (10) French Revolution produces an impres- 6
12 3456789012345678901234567890121234567890123456789012345678901212345 6
123456789012345678901234567890121234567890123456789012345678901212345
incontrovertible datum in the service of 6
12
2
3456789012345678901234567890121234567890123456789012345678901212345
sive array of narratives of late eighteenth
3456789012345678901234567890121234567890123456789012345678901212345
6
6
1 one narrative over another. This obtuse
123456789012345678901234567890121234567890123456789012345678901212345
century France. Within this diverse 6
123456789012345678901234567890121234567890123456789012345678901212345 66
123456789012345678901234567890121234567890123456789012345678901212345 269
1234567890123456789012345678901212345678901234567890123456789012123456
Part IV: Three Practice GREs

1234567890123456789012345678901212345678901234567890123456789012123456
123456789012345678901234567890121234567890123456789012345678901212345 6
12 3456789012345678901234567890121234567890123456789012345678901212345 6
123456789012345678901234567890121234567890123456789012345678901212345
(55) reality complicates the historian’s 29. In line 28, the author states that in Burke’s 6
12 3456789012345678901234567890121234567890123456789012345678901212345 6
12 3456789012345678901234567890121234567890123456789012345678901212345
attempt to simply ply her trade, but view, the French Republic was a “histori- 6
123456789012345678901234567890121234567890123456789012345678901212345 6
12
2
3456789012345678901234567890121234567890123456789012345678901212345
nonetheless is a facet of historiography cal novelty.” This implies that Burke
3456789012345678901234567890121234567890123456789012345678901212345
6
6
1
12 3456789012345678901234567890121234567890123456789012345678901212345
that must be addressed if history is to be believed the French Republic 6
12 3456789012345678901234567890121234567890123456789012345678901212345 6
12 3456789012345678901234567890121234567890123456789012345678901212345
written with intellectual honesty. 6
12
2
3456789012345678901234567890121234567890123456789012345678901212345 6
A. bore close
3456789012345678901234567890121234567890123456789012345678901212345
resemblance to its historical
6
1
12 3456789012345678901234567890121234567890123456789012345678901212345
predecessor. 6
12 3456789012345678901234567890121234567890123456789012345678901212345
27. The passage would most appropriately be 6
12 3456789012345678901234567890121234567890123456789012345678901212345 6
B. shared some resemblance to its
12 3456789012345678901234567890121234567890123456789012345678901212345
titled 6
123456789012345678901234567890121234567890123456789012345678901212345
historical predecessor. 6
12 3456789012345678901234567890121234567890123456789012345678901212345 6
12 3456789012345678901234567890121234567890123456789012345678901212345
A. “The French Revolution: Two C. bore no resemblance to its historical 6
12 3456789012345678901234567890121234567890123456789012345678901212345
predecessor. 6
12
2
3456789012345678901234567890121234567890123456789012345678901212345 6
Competing Views.”
3456789012345678901234567890121234567890123456789012345678901212345 6
1 D. was the logical outgrowth of its
12 3456789012345678901234567890121234567890123456789012345678901212345
B. “Contemporary Issues in Historiogra- 6
12 3456789012345678901234567890121234567890123456789012345678901212345
historical predecessor. 6
12 3456789012345678901234567890121234567890123456789012345678901212345 6
phy.”
12 3456789012345678901234567890121234567890123456789012345678901212345
E. had minimal institutional continuity 6
12 3456789012345678901234567890121234567890123456789012345678901212345
C. “Challenges in Intellectually Honest 6
12 3456789012345678901234567890121234567890123456789012345678901212345
with its historical predecessor. 6
12 3456789012345678901234567890121234567890123456789012345678901212345 6
Historiography.”
12 3456789012345678901234567890121234567890123456789012345678901212345 6
12 3456789012345678901234567890121234567890123456789012345678901212345
D. “Methodologies in Intellectual 6
2 3456789012345678901234567890121234567890123456789012345678901212345
123456789012345678901234567890121234567890123456789012345678901212345
30. The author employs the idea that there are 6
1 Historiography.” 6
12 3456789012345678901234567890121234567890123456789012345678901212345
“diverse narratives” (line 12–13) of the 6
12 3456789012345678901234567890121234567890123456789012345678901212345
E. “Historiography’s Dark Secret.” 6
12 3456789012345678901234567890121234567890123456789012345678901212345 6
French Revolution to support the conten-
2 3456789012345678901234567890121234567890123456789012345678901212345
123456789012345678901234567890121234567890123456789012345678901212345 6
123456789012345678901234567890121234567890123456789012345678901212345
28. The second paragraph’s role in the tion that historiography 6
123456789012345678901234567890121234567890123456789012345678901212345 6
A. is merely a function of the historian’s 6
123456789012345678901234567890121234567890123456789012345678901212345
passage as a whole is to
123456789012345678901234567890121234567890123456789012345678901212345 66
123456789012345678901234567890121234567890123456789012345678901212345
biases.
123456789012345678901234567890121234567890123456789012345678901212345
A. present a view contrary to the one 6
123456789012345678901234567890121234567890123456789012345678901212345
B. can be influenced by the historian’s 6
123456789012345678901234567890121234567890123456789012345678901212345 66
presented in the introductory para-
1 ideology.
12 3456789012345678901234567890121234567890123456789012345678901212345
graph. 6
12 3456789012345678901234567890121234567890123456789012345678901212345
C. is not a science. 6
123456789012345678901234567890121234567890123456789012345678901212345
B. conceptually link the first and third 6
123456789012345678901234567890121234567890123456789012345678901212345
D. is the discipline of sophisticated story 6
12 3456789012345678901234567890121234567890123456789012345678901212345
paragraphs. 6
12 3456789012345678901234567890121234567890123456789012345678901212345
telling. 6
12 3456789012345678901234567890121234567890123456789012345678901212345
C. illustrate with examples the thesis of 6
123456789012345678901234567890121234567890123456789012345678901212345
E. can be manipulated by diverse 6
12 3456789012345678901234567890121234567890123456789012345678901212345
the passage. 6
12 3456789012345678901234567890121234567890123456789012345678901212345
interests. 6
123456789012345678901234567890121234567890123456789012345678901212345 66
2 3456789012345678901234567890121234567890123456789012345678901212345
D. offer a complimentary interpretation
123456789012345678901234567890121234567890123456789012345678901212345
to the one given in the first paragraph. 6
1
123456789012345678901234567890121234567890123456789012345678901212345
E. attack the topic sentence of the first 6
123456789012345678901234567890121234567890123456789012345678901212345 6
2 3456789012345678901234567890121234567890123456789012345678901212345
123456789012345678901234567890121234567890123456789012345678901212345
paragraph. 6
123456789012345678901234567890121234567890123456789012345678901212345 6
123456789012345678901234567890121234567890123456789012345678901212345 66
123456789012345678901234567890121234567890123456789012345678901212345 6
1
12 3456789012345678901234567890121234567890123456789012345678901212345 6
12 3456789012345678901234567890121234567890123456789012345678901212345 6
12 3456789012345678901234567890121234567890123456789012345678901212345 6
12 3456789012345678901234567890121234567890123456789012345678901212345 6
123456789012345678901234567890121234567890123456789012345678901212345 6
123456789012345678901234567890121234567890123456789012345678901212345
2 6
123456789012345678901234567890121234567890123456789012345678901212345 6
123456789012345678901234567890121234567890123456789012345678901212345 6
123456789012345678901234567890121234567890123456789012345678901212345 6
1 3456789012345678901234567890121234567890123456789012345678901212345 6
123456789012345678901234567890121234567890123456789012345678901212345 66
2 3456789012345678901234567890121234567890123456789012345678901212345
1
12 3456789012345678901234567890121234567890123456789012345678901212345 6
123456789012345678901234567890121234567890123456789012345678901212345 6
12 3456789012345678901234567890121234567890123456789012345678901212345 6
123456789012345678901234567890121234567890123456789012345678901212345 6
123456789012345678901234567890121234567890123456789012345678901212345 6
123456789012345678901234567890121234567890123456789012345678901212345 66
270 123456789012345678901234567890121234567890123456789012345678901212345
1234567890123456789012345678901212345678901234567890123456789012123456

www.petersons.com
Answers and Explanations

123456789012345678901234567890121234567890123456789012
12345678901234567890123456789012123456789012345678901 2
12345678901234567890123456789012123456789012345678901
2345678901234567890123456789012123456789012345678901
2
2
1 The following notes are intended to show you one way in which the
12345678901234567890123456789012123456789012345678901 2
12345678901234567890123456789012123456789012345678901
correct answer can be determined. If you found the right answer using 2
12345678901234567890123456789012123456789012345678901 2
12345678901234567890123456789012123456789012345678901
another method, good for you! The more strategies you have at your 2
2345678901234567890123456789012123456789012345678901
12345678901234567890123456789012123456789012345678901 2
12345678901234567890123456789012123456789012345678901
disposal, the better off you will be. 2
12345678901234567890123456789012123456789012345678901 22
12345678901234567890123456789012123456789012345678901 2
12345678901234567890123456789012123456789012345678901 2
12345678901234567890123456789012123456789012345678901 2
1Quantitative
12345678901234567890123456789012123456789012345678901 2
12345678901234567890123456789012123456789012345678901 2
12345678901234567890123456789012123456789012345678901
1. C For Column A, the number of minutes in 6 hours (since h 5 6) 2
2345678901234567890123456789012123456789012345678901
12345678901234567890123456789012123456789012345678901
can be found by multiplying: (6)(60 minutes) 5 360 minutes, or 2
2
12345678901234567890123456789012123456789012345678901
12345678901234567890123456789012123456789012345678901
360. For Column B, you must know that a square has four right 2
2
12345678901234567890123456789012123456789012345678901
12345678901234567890123456789012123456789012345678901
angles, all of which equal 90 degrees. This means Column B 2
2
12345678901234567890123456789012123456789012345678901
12345678901234567890123456789012123456789012345678901 2
12345678901234567890123456789012123456789012345678901
works out to: (4)(90) 5 360. 2
12345678901234567890123456789012123456789012345678901 22
1
12345678901234567890123456789012123456789012345678901
This first problem is fairly straightforward, and even though 2
12345678901234567890123456789012123456789012345678901
there was a variable involved, definite real numbers were found 2
12345678901234567890123456789012123456789012345678901 2
12345678901234567890123456789012123456789012345678901 2
12345678901234567890123456789012123456789012345678901
for both columns. Be aware
2345678901234567890123456789012123456789012345678901
that this will change as the 2
questions get harder. On later problems, the use of variables will 2
12345678901234567890123456789012123456789012345678901
12345678901234567890123456789012123456789012345678901 2
1 lead to column quantities that have more than one possible 2
12345678901234567890123456789012123456789012345678901 2
12345678901234567890123456789012123456789012345678901
answer, so even though these problems will resemble this earlier 2
12345678901234567890123456789012123456789012345678901 22
2345678901234567890123456789012123456789012345678901
12345678901234567890123456789012123456789012345678901
one, finding the correct answer will not be as straightforward.
2
1
12345678901234567890123456789012123456789012345678901 2
12345678901234567890123456789012123456789012345678901 2
2. D Since they are a simple shape with a ton of special cases and
2345678901234567890123456789012123456789012345678901
12345678901234567890123456789012123456789012345678901
associated rules, triangles make many appearances on the 2
2
12345678901234567890123456789012123456789012345678901 2
12345678901234567890123456789012123456789012345678901
standard GRE. On question 2, the triangle has a right angle, 2
12345678901234567890123456789012123456789012345678901
1 meaning the Pythagorean theorem can be applied. Start with the 2
12345678901234567890123456789012123456789012345678901 2
12345678901234567890123456789012123456789012345678901 2
12345678901234567890123456789012123456789012345678901
basic Pythagorean theorem: 2
12345678901234567890123456789012123456789012345678901 2
12345678901234567890123456789012123456789012345678901
2 2 2 2
12345678901234567890123456789012123456789012345678901 22
2345678901234567890123456789012123456789012345678901
a 1 b 5 c
12345678901234567890123456789012123456789012345678901 2
12345678901234567890123456789012123456789012345678901
a2 1 b2 2 b2 5 c2 2 b2 (This subtracts b2from both sides)
2
12345678901234567890123456789012123456789012345678901 2
1
12345678901234567890123456789012123456789012345678901 22
2345678901234567890123456789012123456789012345678901
a2
5 c2
2 b2
1
12345678901234567890123456789012123456789012345678901
Column A can be written as either a2 or c2 2 b2. As for Column 2
12345678901234567890123456789012123456789012345678901 2
12345678901234567890123456789012123456789012345678901
2345678901234567890123456789012123456789012345678901
2
2
1 B, the hypotenuse is length c, so the square of the hypotenuse
12345678901234567890123456789012123456789012345678901 2
12345678901234567890123456789012123456789012345678901 22
2
minus itself would be c 2 c. Comparing this to Column A, you
12345678901234567890123456789012123456789012345678901
123456789012345678901234567890121234567890123456789012
271
Part IV: Three Practice GREs

123456789012345678901234567890121234567890123456789012
12345678901234567890123456789012123456789012345678901 2
12345678901234567890123456789012123456789012345678901 2
12345678901234567890123456789012123456789012345678901
can see that both have an initial c2 that is then subtracted by 2
12345678901234567890123456789012123456789012345678901 2
12345678901234567890123456789012123456789012345678901
either b2(Column A) or c (Column B). The problem boils down 2
12345678901234567890123456789012123456789012345678901 2
12345678901234567890123456789012123456789012345678901
to how c compares with b2. Frankly, there’s no way to know. 2
2345678901234567890123456789012123456789012345678901 2
1
12345678901234567890123456789012123456789012345678901
The variable b could be smaller than one, in which case its 2
12345678901234567890123456789012123456789012345678901
square would be even smaller. Depending on which numbers 2
12345678901234567890123456789012123456789012345678901 2
12345678901234567890123456789012123456789012345678901
2345678901234567890123456789012123456789012345678901
2
2
1 you plug in, Column A or Column B might be greater.
12345678901234567890123456789012123456789012345678901 2
12345678901234567890123456789012123456789012345678901 2
12345678901234567890123456789012123456789012345678901
3. D The perimeter of the square is 4h, but you do not know how this 2
12345678901234567890123456789012123456789012345678901
2
2345678901234567890123456789012123456789012345678901
2
2
1 relates to h . If, h 5 1 then Column B is greater, but if h 5 10,
12345678901234567890123456789012123456789012345678901 2
12345678901234567890123456789012123456789012345678901
then Column A is greater. Note that if you confuse the area of a 2
12345678901234567890123456789012123456789012345678901
square with its perimeter, you will find the two columns are 2
12345678901234567890123456789012123456789012345678901
2345678901234567890123456789012123456789012345678901
2
2
1 equal, leading you to pick the incorrect answer (C).
12345678901234567890123456789012123456789012345678901 2
12345678901234567890123456789012123456789012345678901 2
12345678901234567890123456789012123456789012345678901
4. D You might jump to say that the answer is (C), but that would be a 2
12345678901234567890123456789012123456789012345678901 2
12345678901234567890123456789012123456789012345678901
little hasty. The absolute value sign (those two bars on either side) 2
12345678901234567890123456789012123456789012345678901 2
12345678901234567890123456789012123456789012345678901
means that x could be either 3 or 23, and that y could equal either 2
12345678901234567890123456789012123456789012345678901 2
12345678901234567890123456789012123456789012345678901
4 or 24. Remember that |y 2 x| Þ |y| 2 |x| 5 1. We actually do not 2
2345678901234567890123456789012123456789012345678901
know what |y 2 x| equals. It could be seven, or it could be one. The 2
12345678901234567890123456789012123456789012345678901
1 2
12345678901234567890123456789012123456789012345678901 2
12345678901234567890123456789012123456789012345678901
answer, then, is (D). 2
12345678901234567890123456789012123456789012345678901 2
2345678901234567890123456789012123456789012345678901
5. A For people rusty with their math terminology, this question 2
12345678901234567890123456789012123456789012345678901
12345678901234567890123456789012123456789012345678901 2
12345678901234567890123456789012123456789012345678901 2
12345678901234567890123456789012123456789012345678901 2
looks much more difficult that it really is. The three smallest
prime numbers are 2, 3, and 5, and their sum is 10. As for 2
12345678901234567890123456789012123456789012345678901
12345678901234567890123456789012123456789012345678901 2
12345678901234567890123456789012123456789012345678901 2
12345678901234567890123456789012123456789012345678901 2
Column B, the square root of 49 is 7. You probably don’t know
the square root of 8 off the top of your head, but you do know 2
12345678901234567890123456789012123456789012345678901
1 2
2345678901234567890123456789012123456789012345678901
that the square root of 9 is 3. This means the square root of 8 2
12345678901234567890123456789012123456789012345678901
12345678901234567890123456789012123456789012345678901 2
1 must be something less than 3. Seven 1 something less than 3 5 2
12345678901234567890123456789012123456789012345678901 2
12345678901234567890123456789012123456789012345678901
something a little less than 10. Therefore, Column A is greater. 2
12345678901234567890123456789012123456789012345678901 2
12345678901234567890123456789012123456789012345678901 2
12345678901234567890123456789012123456789012345678901
6. A In looking at the figure, we can reason that a 1 b 5 180 since 2
12345678901234567890123456789012123456789012345678901
the lines are parallel. Thus, we need to manipulate Column A so 2
12345678901234567890123456789012123456789012345678901 2
2345678901234567890123456789012123456789012345678901
that we can compare it to a 1 b. In the numerator, we can factor 2
12345678901234567890123456789012123456789012345678901
12345678901234567890123456789012123456789012345678901 2
1 2
12345678901234567890123456789012123456789012345678901 2
out an a and then eliminate it in the following manner:
12345678901234567890123456789012123456789012345678901 2
12345678901234567890123456789012123456789012345678901 2
12345678901234567890123456789012123456789012345678901
a2 1 ab 1 a a(a 1 b 1 1) a(a 1 b 1 1) 2
12345678901234567890123456789012123456789012345678901
5 5 5a1b11 2
12345678901234567890123456789012123456789012345678901
a a a 2
12345678901234567890123456789012123456789012345678901 2
2345678901234567890123456789012123456789012345678901
12345678901234567890123456789012123456789012345678901
This leaves us with a 1 b 1 1, which is 1 greater than Column 2
2
12345678901234567890123456789012123456789012345678901 2
12345678901234567890123456789012123456789012345678901 2
12345678901234567890123456789012123456789012345678901
B’s a 1 b 5 180. Therefore, Column A is greater.
2
1
2345678901234567890123456789012123456789012345678901
7. B A good way to attack a problem like this is to sketch it out. If J is 2
12345678901234567890123456789012123456789012345678901
12345678901234567890123456789012123456789012345678901 2
12345678901234567890123456789012123456789012345678901 2
12345678901234567890123456789012123456789012345678901 2
space 1 then there are two possibilities: JBA and JAB. If (B) is in
1 space 1, BJA and BAJ. If (A) is in space 1, AJB and ABJ. That is 2
12345678901234567890123456789012123456789012345678901 2
12345678901234567890123456789012123456789012345678901 2
12345678901234567890123456789012123456789012345678901
six distinct arrangements. Can you get anymore? Try to put J in 2
12345678901234567890123456789012123456789012345678901 2
12345678901234567890123456789012123456789012345678901
space 2, AJB or BJA. You already
2345678901234567890123456789012123456789012345678901
have both of those counted. 2
2
1
12345678901234567890123456789012123456789012345678901 2
12345678901234567890123456789012123456789012345678901 2 2
272 12345678901234567890123456789012123456789012345678901
123456789012345678901234567890121234567890123456789012

www.petersons.com
ANSWERS
123456789012345678901234567890121234567890123456789012
12345678901234567890123456789012123456789012345678901 2
12345678901234567890123456789012123456789012345678901 2
12345678901234567890123456789012123456789012345678901
Try any other arrangement, and you will see that it is a repeat. 2
12345678901234567890123456789012123456789012345678901 2
12345678901234567890123456789012123456789012345678901
(B) is greater. 2
12345678901234567890123456789012123456789012345678901 2
12345678901234567890123456789012123456789012345678901
8. C This one is tricky, but not if you recall that p is defined as the 2
12345678901234567890123456789012123456789012345678901 2
12345678901234567890123456789012123456789012345678901 2
12345678901234567890123456789012123456789012345678901
ratio of the circumference divided by the diameter. If you didn’t 2
12345678901234567890123456789012123456789012345678901 2
12345678901234567890123456789012123456789012345678901
recall this, you still could have
2345678901234567890123456789012123456789012345678901
tried to find an equation that 2
2
1
12345678901234567890123456789012123456789012345678901 C 2
12345678901234567890123456789012123456789012345678901
related all three entities, such as C 5 2pr 5 pd → p 5 . As for 2
12345678901234567890123456789012123456789012345678901 d 2
12345678901234567890123456789012123456789012345678901
2345678901234567890123456789012123456789012345678901 2C C 2
1 Column B it can be manipulated like this, 5 5 p. The 2
12345678901234567890123456789012123456789012345678901 2d d 2
12345678901234567890123456789012123456789012345678901 2
12345678901234567890123456789012123456789012345678901
columns are equal, so the answer is (C). 2
12345678901234567890123456789012123456789012345678901
2345678901234567890123456789012123456789012345678901
2
2
1 9. D This requires straightforward arithmetic work.
12345678901234567890123456789012123456789012345678901 2
12345678901234567890123456789012123456789012345678901 2
12345678901234567890123456789012123456789012345678901 2
12345678901234567890123456789012123456789012345678901
Column A Column B 2
2345678901234567890123456789012123456789012345678901
12345678901234567890123456789012123456789012345678901 2
F GS D F S
12345678901234567890123456789012123456789012345678901
2
12345678901234567890123456789012123456789012345678901
a b 1
12345678901234567890123456789012123456789012345678901
2 12a 3 b
a
12345678901234567890123456789012123456789012345678901 22
2 a 6
2 D G
1
3b
2 4
2
2
2
12345678901234567890123456789012123456789012345678901
1 5
ab
2 12 S
12345678901234567890123456789012123456789012345678901
5 3b 2
12345678901234567890123456789012123456789012345678901
2
a
6 3b
D 1
2 12
2
2
2
12345678901234567890123456789012123456789012345678901 2
2345678901234567890123456789012123456789012345678901
12345678901234567890123456789012123456789012345678901
ab 2
12345678901234567890123456789012123456789012345678901 2 1 2 12 2
12345678901234567890123456789012123456789012345678901
5
2 2
12345678901234567890123456789012123456789012345678901 22
12345678901234567890123456789012123456789012345678901 2
12345678901234567890123456789012123456789012345678901
You can cancel out the 212, since both columns share it. 2
12345678901234567890123456789012123456789012345678901
Column A and B share the first term, but Column B is the first 2
12345678901234567890123456789012123456789012345678901
12345678901234567890123456789012123456789012345678901 2
1 term 21. Since we are dealing with positive numbers here, that 2
12345678901234567890123456789012123456789012345678901 22
2345678901234567890123456789012123456789012345678901
12345678901234567890123456789012123456789012345678901
makes Column A greater.
2
1
12345678901234567890123456789012123456789012345678901 2
12345678901234567890123456789012123456789012345678901
10. B The only way that your triangle
2345678901234567890123456789012123456789012345678901
could be a triangle is if the third 2
2
12345678901234567890123456789012123456789012345678901
side is greater than 6. If it is not, then the three sides would not 2
12345678901234567890123456789012123456789012345678901 2
1
12345678901234567890123456789012123456789012345678901
connect, making an enclosed figure with 3 angles and 3 sides. 2
12345678901234567890123456789012123456789012345678901
This is readily apparent if you attempt to make a triangle with 2
12345678901234567890123456789012123456789012345678901 2
12345678901234567890123456789012123456789012345678901 2
12345678901234567890123456789012123456789012345678901 2
sides 1, 4, and 10. Thus, (B) is greater.
12345678901234567890123456789012123456789012345678901 2
12345678901234567890123456789012123456789012345678901
11. C The number set is {10, 12, 14, 16, 18}, so, the median is 14. The 2
12345678901234567890123456789012123456789012345678901 2
12345678901234567890123456789012123456789012345678901 2
12345678901234567890123456789012123456789012345678901
mean of any evenly spaced number set is always the median, so 2
12345678901234567890123456789012123456789012345678901 2
12345678901234567890123456789012123456789012345678901 2
the mean is also 14 (you could also just sum the numbers and
2345678901234567890123456789012123456789012345678901
12345678901234567890123456789012123456789012345678901
divide by 5). 2
12345678901234567890123456789012123456789012345678901 2
12345678901234567890123456789012123456789012345678901 22
12345678901234567890123456789012123456789012345678901
12. B This one is greatly simplified if you realize that you are
2
1
2345678901234567890123456789012123456789012345678901
looking at a right triangle here, and you know two of its sides, 2
12345678901234567890123456789012123456789012345678901
12345678901234567890123456789012123456789012345678901
namely 3 and 4. Using the Pythagorean theorem or recognizing 2
2
12345678901234567890123456789012123456789012345678901 2
12345678901234567890123456789012123456789012345678901
it is a 3:4:5 triangle, you can determine the third side is 5. That 2
1
2345678901234567890123456789012123456789012345678901
12345678901234567890123456789012123456789012345678901
is the value of Column A. For Column B, the slope of GH can 2
2
1
12345678901234567890123456789012123456789012345678901
be determined by simply counting the rise (4) and the run (3) 2
12345678901234567890123456789012123456789012345678901 2
12345678901234567890123456789012123456789012345678901
2345678901234567890123456789012123456789012345678901
2
2
1
12345678901234567890123456789012123456789012345678901 2
12345678901234567890123456789012123456789012345678901 2
12345678901234567890123456789012123456789012345678901 2 273
123456789012345678901234567890121234567890123456789012
Part IV: Three Practice GREs

123456789012345678901234567890121234567890123456789012
12345678901234567890123456789012123456789012345678901 2
12345678901234567890123456789012123456789012345678901 2
12345678901234567890123456789012123456789012345678901 4 2
12345678901234567890123456789012123456789012345678901
and putting the former over the latter, 2 . Now multiply it, 2
12345678901234567890123456789012123456789012345678901 3 2
12345678901234567890123456789012123456789012345678901 2
1 2
4
3
S D
12345678901234567890123456789012123456789012345678901
24 5
16
3
5
1
2345678901234567890123456789012123456789012345678901
~ ! 5 , which is
12345678901234567890123456789012123456789012345678901
3
greater than 5. 2
2
2
12345678901234567890123456789012123456789012345678901 2
12345678901234567890123456789012123456789012345678901
13. B The greatest that Column A could be is 9 1 9 1 8 5 26, since 2
12345678901234567890123456789012123456789012345678901 2
12345678901234567890123456789012123456789012345678901
A and B are distinct digits, so the answer then is (B). 2
12345678901234567890123456789012123456789012345678901 2
12345678901234567890123456789012123456789012345678901 2
12345678901234567890123456789012123456789012345678901
14. A A good way to approach this problem is make a table filling in 2
12345678901234567890123456789012123456789012345678901
2345678901234567890123456789012123456789012345678901
2
2
1 the information you do know and from that trying to determine
12345678901234567890123456789012123456789012345678901 2
12345678901234567890123456789012123456789012345678901
the information you need to answer the question. 2
12345678901234567890123456789012123456789012345678901 2
12345678901234567890123456789012123456789012345678901
2345678901234567890123456789012123456789012345678901
2
2
1 If there are 73 total animals and 35 dogs, then there are 38 cats.
12345678901234567890123456789012123456789012345678901
If there are 40 females, then there are 33 males. If x is the 2
12345678901234567890123456789012123456789012345678901 2
12345678901234567890123456789012123456789012345678901
number of male cats, then the number of male dogs is x 19, 2
12345678901234567890123456789012123456789012345678901 2
12345678901234567890123456789012123456789012345678901
since it states there are 9 less male cats than dogs. 2
12345678901234567890123456789012123456789012345678901 2
12345678901234567890123456789012123456789012345678901 2
12345678901234567890123456789012123456789012345678901
The total number of male animals is 33, so x 1 (x 1 9) 5 33 → 2
12345678901234567890123456789012123456789012345678901 2
12345678901234567890123456789012123456789012345678901
2x 5 24 → x 5 12. There are 12 male cats and 21 (x 1 9) male 2
12345678901234567890123456789012123456789012345678901 2
12345678901234567890123456789012123456789012345678901
dogs. The total number of cats is 38, so that means there are 26 2
12345678901234567890123456789012123456789012345678901 2
12345678901234567890123456789012123456789012345678901
female cats. 2
12345678901234567890123456789012123456789012345678901 2
2345678901234567890123456789012123456789012345678901
15. C There are two ways to find the answer. One way is to plug in the 2
12345678901234567890123456789012123456789012345678901
12345678901234567890123456789012123456789012345678901 2
12345678901234567890123456789012123456789012345678901 2
12345678901234567890123456789012123456789012345678901 2
answer choices and see which ones work or solve the binomial
12345678901234567890123456789012123456789012345678901 2
12345678901234567890123456789012123456789012345678901
expression. You can also solve the equation: 2
12345678901234567890123456789012123456789012345678901 2
12345678901234567890123456789012123456789012345678901
2
→ 2
1 2 x 5 22x 2 15 2
2345678901234567890123456789012123456789012345678901
12345678901234567890123456789012123456789012345678901 2
12345678901234567890123456789012123456789012345678901
2 x2 1 x2 5 22x 2 15 1 x2 → 2
1 2
12345678901234567890123456789012123456789012345678901
2
0 5 x 2 2x 2 15 → 2
12345678901234567890123456789012123456789012345678901
2345678901234567890123456789012123456789012345678901
2
2
12345678901234567890123456789012123456789012345678901
0 5 ~x 2 5!~x 1 3! →
12345678901234567890123456789012123456789012345678901 2
1 2
12345678901234567890123456789012123456789012345678901
x 5 23, 5 2
12345678901234567890123456789012123456789012345678901 2
2345678901234567890123456789012123456789012345678901
16. E The first thing to do is to convert each answer choice into 2
12345678901234567890123456789012123456789012345678901
12345678901234567890123456789012123456789012345678901 2
1 2
12345678901234567890123456789012123456789012345678901 2
y 5 mx 1 b form. From the graph, you can figure that the slope
12345678901234567890123456789012123456789012345678901
is 21, and the y-intercept is 22. Now you can simply look for 2
12345678901234567890123456789012123456789012345678901 2
12345678901234567890123456789012123456789012345678901
the equation with these characteristics, and (E) is it. 2
12345678901234567890123456789012123456789012345678901 2
12345678901234567890123456789012123456789012345678901 2
12345678901234567890123456789012123456789012345678901
17. C Eyeball the graph to see which months have both adult and 2
2345678901234567890123456789012123456789012345678901
12345678901234567890123456789012123456789012345678901
children graphs near the zero line in the middle. Both Januaries 2
2
12345678901234567890123456789012123456789012345678901 2
12345678901234567890123456789012123456789012345678901
do, but 1963 is clearly a smaller total number. Thus, (C) is 2
12345678901234567890123456789012123456789012345678901 2
1
12345678901234567890123456789012123456789012345678901
correct. 2
12345678901234567890123456789012123456789012345678901 2
12345678901234567890123456789012123456789012345678901 2
12345678901234567890123456789012123456789012345678901
18. B If you notice that March–April 1962 is the only time the child 2
12345678901234567890123456789012123456789012345678901 2
12345678901234567890123456789012123456789012345678901 2
2345678901234567890123456789012123456789012345678901
flu incidence exceeds 100, this one will be straightforward:
2
1
12345678901234567890123456789012123456789012345678901
answer choice (B). 2
12345678901234567890123456789012123456789012345678901 2
12345678901234567890123456789012123456789012345678901
19. C If the area of the top of the box is 15, then the third side is of 2
12345678901234567890123456789012123456789012345678901 2
12345678901234567890123456789012123456789012345678901
length 5 since this length times 3 must yield an area of 15. Now 2
12345678901234567890123456789012123456789012345678901 2 2
274 12345678901234567890123456789012123456789012345678901
123456789012345678901234567890121234567890123456789012

www.petersons.com
ANSWERS
123456789012345678901234567890121234567890123456789012
12345678901234567890123456789012123456789012345678901 2
12345678901234567890123456789012123456789012345678901 2
12345678901234567890123456789012123456789012345678901
that you know all three lengths you just need to add up the areas 2
12345678901234567890123456789012123456789012345678901 2
12345678901234567890123456789012123456789012345678901
of all six sides. Four of the sides have an area of 15, which sums 2
12345678901234567890123456789012123456789012345678901 2
12345678901234567890123456789012123456789012345678901
to 60. The two end sides have an area of 9, which sums to 18. 2
2345678901234567890123456789012123456789012345678901 2
1
12345678901234567890123456789012123456789012345678901
Add these together and you get (C). 2
12345678901234567890123456789012123456789012345678901 2
12345678901234567890123456789012123456789012345678901 2
12345678901234567890123456789012123456789012345678901
20. B Begin by determining the median
2345678901234567890123456789012123456789012345678901
and the mode. The mode is 2
1 simply the number that occurs the most times in the set. That is 2
12345678901234567890123456789012123456789012345678901 2
12345678901234567890123456789012123456789012345678901 2
12345678901234567890123456789012123456789012345678901
clearly 7. The median is the number that is in the middle if you 2
12345678901234567890123456789012123456789012345678901
2345678901234567890123456789012123456789012345678901
2
2
1 line them up from least to greatest. The two middle numbers are
12345678901234567890123456789012123456789012345678901 2
12345678901234567890123456789012123456789012345678901
7, so halfway between them is still just 7. The median minus the 2
12345678901234567890123456789012123456789012345678901
mode then is 7 7 0. 2
12345678901234567890123456789012123456789012345678901
2 5
2345678901234567890123456789012123456789012345678901
2
2
1
12345678901234567890123456789012123456789012345678901
21. B Look to the number chart to find this one. The ratio we are 2
12345678901234567890123456789012123456789012345678901 2
12345678901234567890123456789012123456789012345678901
looking for is criminal/civil. Remember that you are looking for 2
12345678901234567890123456789012123456789012345678901 2
12345678901234567890123456789012123456789012345678901
the quarter that has the greatest ratio and not the quarter with 2
12345678901234567890123456789012123456789012345678901 2
12345678901234567890123456789012123456789012345678901
the largest numbers. Look for numbers whose difference is great, 2
12345678901234567890123456789012123456789012345678901 2
12345678901234567890123456789012123456789012345678901
and then compare. The 2nd quarter in 1970 is the answer, 2
2345678901234567890123456789012123456789012345678901
12345678901234567890123456789012123456789012345678901 2
1 choice (B). 2
12345678901234567890123456789012123456789012345678901 2
12345678901234567890123456789012123456789012345678901 2
12345678901234567890123456789012123456789012345678901
22. B You need to be careful on this one. It has a number of steps, but 2
2345678901234567890123456789012123456789012345678901
none of them are hard. First, estimate how many female 2
12345678901234567890123456789012123456789012345678901
12345678901234567890123456789012123456789012345678901 2
12345678901234567890123456789012123456789012345678901 2
12345678901234567890123456789012123456789012345678901 22
criminal lawyers there are (approximately 10,000). Now
12345678901234567890123456789012123456789012345678901
estimate how many female civil lawyers there are (approxi- 2
12345678901234567890123456789012123456789012345678901
mately 20,000). So the ratio is, 20,000 to 10,000, which reduces 2
12345678901234567890123456789012123456789012345678901
12345678901234567890123456789012123456789012345678901 2
12345678901234567890123456789012123456789012345678901
to 2:1 or (B). 2
1 2
12345678901234567890123456789012123456789012345678901 22
2345678901234567890123456789012123456789012345678901
12345678901234567890123456789012123456789012345678901
23. C As the graph headings make clear, (A) and (B) are not fields of
2
1
12345678901234567890123456789012123456789012345678901
law but are types of law. With that obstacle out of the way, you 2
12345678901234567890123456789012123456789012345678901 2
12345678901234567890123456789012123456789012345678901
merely need to add up the bar graph pairs to see which is the 2
12345678901234567890123456789012123456789012345678901 2
12345678901234567890123456789012123456789012345678901
greatest. 2
12345678901234567890123456789012123456789012345678901 2
12345678901234567890123456789012123456789012345678901 2
12345678901234567890123456789012123456789012345678901 22
2345678901234567890123456789012123456789012345678901
24. A Since you know the area of the rectangle and one of the sides,
12345678901234567890123456789012123456789012345678901
you can deduce the length of the second side ~BC 5 3!. This 2
1
12345678901234567890123456789012123456789012345678901
side also happens to be the radius of the quarter circle. Thus the 2
12345678901234567890123456789012123456789012345678901 2
2345678901234567890123456789012123456789012345678901
circumference of the full circle is C 5 2p(3) 5 6p. But you only 2
12345678901234567890123456789012123456789012345678901
12345678901234567890123456789012123456789012345678901 2
12345678901234567890123456789012123456789012345678901 2
12345678901234567890123456789012123456789012345678901
1 need a quarter of the circumference,
2345678901234567890123456789012123456789012345678901
12345678901234567890123456789012123456789012345678901
which SD
is 6p
1
4
5
2
3
p. Now 2
2
2
12345678901234567890123456789012123456789012345678901
CD 5 4 and EC 5 3, so ED 5 1. So, (A) is the answer. 2
12345678901234567890123456789012123456789012345678901 22
12345678901234567890123456789012123456789012345678901 2
1
12345678901234567890123456789012123456789012345678901 2
12345678901234567890123456789012123456789012345678901 2
12345678901234567890123456789012123456789012345678901 2
12345678901234567890123456789012123456789012345678901 2
12345678901234567890123456789012123456789012345678901 2
12345678901234567890123456789012123456789012345678901 22
2345678901234567890123456789012123456789012345678901
1
12345678901234567890123456789012123456789012345678901 2
12345678901234567890123456789012123456789012345678901 2
12345678901234567890123456789012123456789012345678901 2
12345678901234567890123456789012123456789012345678901 2
12345678901234567890123456789012123456789012345678901 2
12345678901234567890123456789012123456789012345678901 2
12345678901234567890123456789012123456789012345678901 2 275
123456789012345678901234567890121234567890123456789012
Part IV: Three Practice GREs

123456789012345678901234567890121234567890123456789012
12345678901234567890123456789012123456789012345678901 2
12345678901234567890123456789012123456789012345678901 2
12345678901234567890123456789012123456789012345678901
25. C The longest distance between two corners in a rectangular 2
12345678901234567890123456789012123456789012345678901 2
12345678901234567890123456789012123456789012345678901
box—though sounding difficult to find—is actually rather easy 2
12345678901234567890123456789012123456789012345678901 2
12345678901234567890123456789012123456789012345678901
to compute. Simply square all three sides, sum them, then take 2
2345678901234567890123456789012123456789012345678901 2
1
12345678901234567890123456789012123456789012345678901
the square root of this sum and you have it. Here it would be: 2
12345678901234567890123456789012123456789012345678901 2
12345678901234567890123456789012123456789012345678901 2
12345678901234567890123456789012123456789012345678901
2
2 13 14 52 2
2345678901234567890123456789012123456789012345678901
2
2
1
12345678901234567890123456789012123456789012345678901
4 9 16 29 2
12345678901234567890123456789012123456789012345678901
1 1 5 2
12345678901234567890123456789012123456789012345678901 2
12345678901234567890123456789012123456789012345678901
whose square root is (C). The other way to do this problem is to 2
2345678901234567890123456789012123456789012345678901 2
1
12345678901234567890123456789012123456789012345678901
draw the figure and start using the Pythagorean theorem. You 2
12345678901234567890123456789012123456789012345678901 2
12345678901234567890123456789012123456789012345678901
need to find the diagonal of one of the rectangular sides, and 2
12345678901234567890123456789012123456789012345678901 2
12345678901234567890123456789012123456789012345678901
then create another triangle using that diagonal, the remaining 2
12345678901234567890123456789012123456789012345678901
side, and a hypotenuse, which just happens to be the diagonal 2
12345678901234567890123456789012123456789012345678901 2
12345678901234567890123456789012123456789012345678901 2
12345678901234567890123456789012123456789012345678901
from one corner of the box to another. You end up with the 2
2345678901234567890123456789012123456789012345678901
12345678901234567890123456789012123456789012345678901 2
12345678901234567890123456789012123456789012345678901
same answer, (C). 2
12345678901234567890123456789012123456789012345678901 2
26. B The overall average is the sum of all the scores of all the classes 2
12345678901234567890123456789012123456789012345678901
12345678901234567890123456789012123456789012345678901 2
1 divided by the total number of students. You do not have either 2
12345678901234567890123456789012123456789012345678901 2
12345678901234567890123456789012123456789012345678901 2
12345678901234567890123456789012123456789012345678901
of those numbers, so you have to arrive at the overall average in 2
12345678901234567890123456789012123456789012345678901
some other fashion. A weighted average would do the trick here. 2
12345678901234567890123456789012123456789012345678901 2
2345678901234567890123456789012123456789012345678901
Since there are twice as many fifth graders, you weight their 2
12345678901234567890123456789012123456789012345678901
12345678901234567890123456789012123456789012345678901 2
average with a coefficient of two and the sixth grade average 2
12345678901234567890123456789012123456789012345678901
12345678901234567890123456789012123456789012345678901 2
12345678901234567890123456789012123456789012345678901
with a coefficient of one. 2
12345678901234567890123456789012123456789012345678901 2
12345678901234567890123456789012123456789012345678901 2
12345678901234567890123456789012123456789012345678901
~2!~15! 1 ~1!~18! 5 48 5 16 2
1 2
2345678901234567890123456789012123456789012345678901
12345678901234567890123456789012123456789012345678901 2
3 3
12345678901234567890123456789012123456789012345678901 2
1 2
12345678901234567890123456789012123456789012345678901
27. C On this graph there are two x values for every y value; thus, it is 2
12345678901234567890123456789012123456789012345678901 2
12345678901234567890123456789012123456789012345678901
the x variable that should be squared. Furthermore, when x 5 0 2
12345678901234567890123456789012123456789012345678901 2
12345678901234567890123456789012123456789012345678901
on this graph, y 5 1, so you need an equation that reflects this. 2
12345678901234567890123456789012123456789012345678901 2
12345678901234567890123456789012123456789012345678901
That is true of (C). 2
12345678901234567890123456789012123456789012345678901 2
2345678901234567890123456789012123456789012345678901
1 28. E You can find the area of the circle since you know the radius (a 2
12345678901234567890123456789012123456789012345678901 2
12345678901234567890123456789012123456789012345678901 2
12345678901234567890123456789012123456789012345678901 2
2
5 gpr 5 16p). If you find the area of the two squares then you
2345678901234567890123456789012123456789012345678901
can subtract from the area of the circle to find the area of the 2
12345678901234567890123456789012123456789012345678901
12345678901234567890123456789012123456789012345678901 2
12345678901234567890123456789012123456789012345678901 2
12345678901234567890123456789012123456789012345678901 2
shaded region. The diagonal of each square is the radius of the
1 circle, so this number is 4. From this you can deduce that the 2
12345678901234567890123456789012123456789012345678901 2
12345678901234567890123456789012123456789012345678901
sides of the square are 2=2. Thus the area of each square is 8. 2
12345678901234567890123456789012123456789012345678901 2
12345678901234567890123456789012123456789012345678901 2
12345678901234567890123456789012123456789012345678901 2
Incidentally, you know that the squares are congruent because
2345678901234567890123456789012123456789012345678901
the length of their diagonal is the same. This makes the area in 2
12345678901234567890123456789012123456789012345678901
12345678901234567890123456789012123456789012345678901 2
12345678901234567890123456789012123456789012345678901 2
12345678901234567890123456789012123456789012345678901 2
the two squares 16. The area of the circle minus the area of the
2
1
12345678901234567890123456789012123456789012345678901
squares is 16p 2 16, which is equivalent to (E). 2
12345678901234567890123456789012123456789012345678901 2
12345678901234567890123456789012123456789012345678901 2
12345678901234567890123456789012123456789012345678901 2
12345678901234567890123456789012123456789012345678901 2
12345678901234567890123456789012123456789012345678901 2
12345678901234567890123456789012123456789012345678901 2
12345678901234567890123456789012123456789012345678901 2 2
276 12345678901234567890123456789012123456789012345678901
123456789012345678901234567890121234567890123456789012

www.petersons.com
ANSWERS
123456789012345678901234567890121234567890123456789012
12345678901234567890123456789012123456789012345678901 2
12345678901234567890123456789012123456789012345678901 2
12345678901234567890123456789012123456789012345678901
Verbal 2
12345678901234567890123456789012123456789012345678901 2
12345678901234567890123456789012123456789012345678901 2
12345678901234567890123456789012123456789012345678901
Antonyms 2
12345678901234567890123456789012123456789012345678901 2
1 1. B To have zeal is to have great enthusiasm. The opposite of zeal is 2
2345678901234567890123456789012123456789012345678901
12345678901234567890123456789012123456789012345678901 2
12345678901234567890123456789012123456789012345678901 2
12345678901234567890123456789012123456789012345678901
apathy, choice (B). 2
12345678901234567890123456789012123456789012345678901 2
1 2. C Heterogeneous is the antonym of homogeneous. Homogeneous 2
2345678901234567890123456789012123456789012345678901
12345678901234567890123456789012123456789012345678901 2
12345678901234567890123456789012123456789012345678901 2
12345678901234567890123456789012123456789012345678901
means “to be uniform in kind or quality; similar.” Choice (C) 2
12345678901234567890123456789012123456789012345678901
comes closest to this.
2345678901234567890123456789012123456789012345678901
2
2
1
12345678901234567890123456789012123456789012345678901 2
12345678901234567890123456789012123456789012345678901
3. A Fealty is loyalty. Thus, if you know the definition, this antonym 2
12345678901234567890123456789012123456789012345678901 2
12345678901234567890123456789012123456789012345678901
is straightforward, disloyalty, choice (A).
2345678901234567890123456789012123456789012345678901
2
2
1
12345678901234567890123456789012123456789012345678901
4. E Something archaic is old or antiquated. A good pre-guess then 2
12345678901234567890123456789012123456789012345678901 2
12345678901234567890123456789012123456789012345678901 2
12345678901234567890123456789012123456789012345678901
would be new. In looking down the list of answer choices, only 2
2345678901234567890123456789012123456789012345678901
(E), novel, fits our pre-guess. In this context, novel is not a noun, 2
12345678901234567890123456789012123456789012345678901
12345678901234567890123456789012123456789012345678901 2
12345678901234567890123456789012123456789012345678901 2
12345678901234567890123456789012123456789012345678901 22
which you can tell since all choices and the stem word are
12345678901234567890123456789012123456789012345678901
12345678901234567890123456789012123456789012345678901
adjectives. The adjective novel means new, original, or unique. 2
1 2
12345678901234567890123456789012123456789012345678901
E To assuage is to pacify or to satiate. Choices (B) and (D) are 2
12345678901234567890123456789012123456789012345678901
5. 2
12345678901234567890123456789012123456789012345678901
synonyms, not antonyms. The opposite of assuaging an issue is 2
12345678901234567890123456789012123456789012345678901 22
2345678901234567890123456789012123456789012345678901
12345678901234567890123456789012123456789012345678901
to provoke it, answer choice (E). Note that even if you didn’t
12345678901234567890123456789012123456789012345678901 2
know what the stem word meant, if you had two answer choices 2
12345678901234567890123456789012123456789012345678901
12345678901234567890123456789012123456789012345678901 22
12345678901234567890123456789012123456789012345678901
that mean almost the same thing, (B) and (D), it’s a good guess
12345678901234567890123456789012123456789012345678901
that neither of them will be right since there’s no way they could 2
2
12345678901234567890123456789012123456789012345678901
12345678901234567890123456789012123456789012345678901 2
1 both be right. 2
12345678901234567890123456789012123456789012345678901 22
2345678901234567890123456789012123456789012345678901
12345678901234567890123456789012123456789012345678901
1 6. A If you did not know the definition of mercurial, you might have 2
12345678901234567890123456789012123456789012345678901
recalled that mercury was once used in thermometers to gauge 2
12345678901234567890123456789012123456789012345678901
2345678901234567890123456789012123456789012345678901
2
temperatures. The mercury goes up and down in a thermometer, 2
12345678901234567890123456789012123456789012345678901
12345678901234567890123456789012123456789012345678901 2
1 2
12345678901234567890123456789012123456789012345678901
and in similar fashion, mercurial means to have an unstable or 2
12345678901234567890123456789012123456789012345678901 2
12345678901234567890123456789012123456789012345678901 22
2345678901234567890123456789012123456789012345678901
fast-changing mood. The opposite of this is (A).
12345678901234567890123456789012123456789012345678901 2
1 7. C Opprobrium means severe scorn, so you are looking for a word
12345678901234567890123456789012123456789012345678901 2
12345678901234567890123456789012123456789012345678901
that means something like approval or praise. Choice (C), 2
12345678901234567890123456789012123456789012345678901 2
12345678901234567890123456789012123456789012345678901
commendation, fits the bill. 2
12345678901234567890123456789012123456789012345678901 2
12345678901234567890123456789012123456789012345678901 2
12345678901234567890123456789012123456789012345678901 2
8. C This is a tough word. Tyro means beginner or novice. Thus a
2345678901234567890123456789012123456789012345678901
12345678901234567890123456789012123456789012345678901
good pre-guess would be experienced. None of the words fit 2
2
12345678901234567890123456789012123456789012345678901
with our pre-guess except (C), virtuoso. A virtuoso is a master of 2
12345678901234567890123456789012123456789012345678901
12345678901234567890123456789012123456789012345678901 2
1 2
12345678901234567890123456789012123456789012345678901 22
2345678901234567890123456789012123456789012345678901
a medium or instrument.
12345678901234567890123456789012123456789012345678901 2
12345678901234567890123456789012123456789012345678901
9. A Something that is soporific is sleep inducing. The opposite of 2
12345678901234567890123456789012123456789012345678901
1 this then would be something that would wake you up. 2
12345678901234567890123456789012123456789012345678901 2
12345678901234567890123456789012123456789012345678901 2
12345678901234567890123456789012123456789012345678901
Stimulant, choice (A), fits. 2
12345678901234567890123456789012123456789012345678901 2
12345678901234567890123456789012123456789012345678901 2
12345678901234567890123456789012123456789012345678901 2
12345678901234567890123456789012123456789012345678901 2
12345678901234567890123456789012123456789012345678901 2
12345678901234567890123456789012123456789012345678901 2 277
123456789012345678901234567890121234567890123456789012
Part IV: Three Practice GREs

123456789012345678901234567890121234567890123456789012
12345678901234567890123456789012123456789012345678901 2
12345678901234567890123456789012123456789012345678901 2
12345678901234567890123456789012123456789012345678901
Analogies 2
12345678901234567890123456789012123456789012345678901 2
12345678901234567890123456789012123456789012345678901 2
12345678901234567890123456789012123456789012345678901 2
10. A The stem relationship is definitional: To prattle is to babble. To
12345678901234567890123456789012123456789012345678901
battle is to fight, so (A) fits. Do any of the others? No. “To digest 2
12345678901234567890123456789012123456789012345678901 2
12345678901234567890123456789012123456789012345678901 2
12345678901234567890123456789012123456789012345678901
is to eat” does not work because after you eat something, you 2
12345678901234567890123456789012123456789012345678901 2
12345678901234567890123456789012123456789012345678901
then digest it. These events come
2345678901234567890123456789012123456789012345678901
in sequence, but they don’t 2
2
1
12345678901234567890123456789012123456789012345678901
mean the same thing. 2
12345678901234567890123456789012123456789012345678901 2
12345678901234567890123456789012123456789012345678901 2
12345678901234567890123456789012123456789012345678901
11. C Here the relationship between the stem
2345678901234567890123456789012123456789012345678901
words is one of degree. To 2
1 fawn is to flatter excessively. In similar fashion, to burgeon is to 2
12345678901234567890123456789012123456789012345678901 2
12345678901234567890123456789012123456789012345678901
grow quickly. Choice (B) does not share the same relationship of 2
12345678901234567890123456789012123456789012345678901 2
12345678901234567890123456789012123456789012345678901
degree because it is a relationship of definition (to divulge some- 2
2345678901234567890123456789012123456789012345678901 2
1
12345678901234567890123456789012123456789012345678901
thing is to tell it). Choice (E) might have also seemed attractive, 2
12345678901234567890123456789012123456789012345678901 2
12345678901234567890123456789012123456789012345678901
but remember to misquote is not the same thing as lying. 2
12345678901234567890123456789012123456789012345678901 2
2345678901234567890123456789012123456789012345678901
12345678901234567890123456789012123456789012345678901
12. B The denouement is the end/falling action of the story. Which 2
2
12345678901234567890123456789012123456789012345678901
answer pair has the same relationship? Choice (B), a sunset, is 2
12345678901234567890123456789012123456789012345678901
12345678901234567890123456789012123456789012345678901 2
12345678901234567890123456789012123456789012345678901 2
12345678901234567890123456789012123456789012345678901 2
the end of a day. If you didn’t know the meaning of
1 denouement, you should be very wary of answer choices (A) and 2
12345678901234567890123456789012123456789012345678901 2
12345678901234567890123456789012123456789012345678901 2
12345678901234567890123456789012123456789012345678901
(C), which have second words meant to catch people shopping 2
2345678901234567890123456789012123456789012345678901
12345678901234567890123456789012123456789012345678901 2
12345678901234567890123456789012123456789012345678901
for connections. 2
12345678901234567890123456789012123456789012345678901 2
13. D Federalism is a form/kind of political order. Going through the 2
12345678901234567890123456789012123456789012345678901
2
12345678901234567890123456789012123456789012345678901
answer choices, a class is composed of students, a dictatorship is 2
12345678901234567890123456789012123456789012345678901
12345678901234567890123456789012123456789012345678901 2
not a kind of anarchy, a colleague might be in a cohort, 2
12345678901234567890123456789012123456789012345678901
12345678901234567890123456789012123456789012345678901 2
1 insomnia is a kind of sleep disorder, and something extrinsic is 2
12345678901234567890123456789012123456789012345678901 2
12345678901234567890123456789012123456789012345678901
external to a thing. Clearly, (D) is the only pair that shares the 2
12345678901234567890123456789012123456789012345678901 2
12345678901234567890123456789012123456789012345678901
same relationship. 2
12345678901234567890123456789012123456789012345678901
2345678901234567890123456789012123456789012345678901
2
14. D What is the stem relationship? To be spurious is to not be true. 2
12345678901234567890123456789012123456789012345678901
12345678901234567890123456789012123456789012345678901 2
1 2
12345678901234567890123456789012123456789012345678901
Thus, we have a relationship of opposites. The only answer 2
12345678901234567890123456789012123456789012345678901 2
12345678901234567890123456789012123456789012345678901 2
2345678901234567890123456789012123456789012345678901
choice that shares this relationship is (D), since to be insipid is to
12345678901234567890123456789012123456789012345678901 2
1 not be flavorful. 2
12345678901234567890123456789012123456789012345678901 2
12345678901234567890123456789012123456789012345678901
15. D Remember, always begin with the stem relationship. It is 2
12345678901234567890123456789012123456789012345678901 2
12345678901234567890123456789012123456789012345678901
definitional, as something ersatz is artificial. If you did not know 2
12345678901234567890123456789012123456789012345678901 2
12345678901234567890123456789012123456789012345678901
the definition of ersatz, you could have eliminated (A) and (C) 2
12345678901234567890123456789012123456789012345678901 2
12345678901234567890123456789012123456789012345678901
since neither has a necessary relationship. Of choices (B), (D), 2
12345678901234567890123456789012123456789012345678901 2
12345678901234567890123456789012123456789012345678901
and (E), only (D)—to be nascent is to be incipient—has the 2
12345678901234567890123456789012123456789012345678901 2
12345678901234567890123456789012123456789012345678901
definitional relationship. 2
12345678901234567890123456789012123456789012345678901 2
2345678901234567890123456789012123456789012345678901
2
12345678901234567890123456789012123456789012345678901
16. E A diatribe is harsh verbal attack, and a eulogy is a speech of 2
12345678901234567890123456789012123456789012345678901
1 praise or remembrance. Thus, the words are antonyms. The 2
12345678901234567890123456789012123456789012345678901 2
2345678901234567890123456789012123456789012345678901
stem relationship is a diatribe is the opposite of a eulogy. To 2
12345678901234567890123456789012123456789012345678901
1 2
12345678901234567890123456789012123456789012345678901 2
12345678901234567890123456789012123456789012345678901 2
remonstrate is to reprove, or stand in opposition to, while an
12345678901234567890123456789012123456789012345678901
encomium is warm praise. Thus a remonstration is the opposite 2
12345678901234567890123456789012123456789012345678901 2
12345678901234567890123456789012123456789012345678901 2
12345678901234567890123456789012123456789012345678901 2
of an encomium.
2
278 12345678901234567890123456789012123456789012345678901
123456789012345678901234567890121234567890123456789012

www.petersons.com
ANSWERS
123456789012345678901234567890121234567890123456789012
12345678901234567890123456789012123456789012345678901 2
12345678901234567890123456789012123456789012345678901 2
12345678901234567890123456789012123456789012345678901
17. A The stem relationship is one of definition, a hagiography 2
12345678901234567890123456789012123456789012345678901 2
12345678901234567890123456789012123456789012345678901
venerates its subject. In (A), a polemic attacks its subject. A 2
12345678901234567890123456789012123456789012345678901 2
12345678901234567890123456789012123456789012345678901
vendetta may or may not scare its object. The same is true for a 2
2345678901234567890123456789012123456789012345678901 2
1
12345678901234567890123456789012123456789012345678901
complaint. The other choices do not have a strong relationship. 2
12345678901234567890123456789012123456789012345678901
Choice (A) is the answer. If you have trouble with the 2
12345678901234567890123456789012123456789012345678901 2
12345678901234567890123456789012123456789012345678901
2345678901234567890123456789012123456789012345678901
2
2
1 vocabulary, you could look at (C) and (D) and eliminate these
12345678901234567890123456789012123456789012345678901 2
12345678901234567890123456789012123456789012345678901
choices for having weak links. 2
12345678901234567890123456789012123456789012345678901 2
12345678901234567890123456789012123456789012345678901
2345678901234567890123456789012123456789012345678901
2
2
1
12345678901234567890123456789012123456789012345678901
Sentence Completions 2
12345678901234567890123456789012123456789012345678901 2
12345678901234567890123456789012123456789012345678901 2
12345678901234567890123456789012123456789012345678901
18. B On two blank questions, it’s easiest
2345678901234567890123456789012123456789012345678901
to consider one blank at a 2
2
1
12345678901234567890123456789012123456789012345678901
time. Attacking the second blank first makes most sense since 2
12345678901234567890123456789012123456789012345678901
you know that it is in contrast to minor points (the trigger word 2
12345678901234567890123456789012123456789012345678901 2
12345678901234567890123456789012123456789012345678901 2
12345678901234567890123456789012123456789012345678901 22
2345678901234567890123456789012123456789012345678901
Although should tip your hand). What is a contrast to a minor
12345678901234567890123456789012123456789012345678901
point? A major one. You should go look for something like 2
12345678901234567890123456789012123456789012345678901
major or important. That eliminates (A) and (C). Into the first 2
12345678901234567890123456789012123456789012345678901
12345678901234567890123456789012123456789012345678901 2
1 blank, debate clearly fits best of the remaining choices, as a 2
12345678901234567890123456789012123456789012345678901 2
12345678901234567890123456789012123456789012345678901 2
12345678901234567890123456789012123456789012345678901
debate is more likely to have themes than an agreement or 2
12345678901234567890123456789012123456789012345678901 2
12345678901234567890123456789012123456789012345678901 22
2345678901234567890123456789012123456789012345678901
discovery. So, (B) is the answer.
12345678901234567890123456789012123456789012345678901 2
12345678901234567890123456789012123456789012345678901
19. D The blank is a word that goes along with created excitement, and
12345678901234567890123456789012123456789012345678901 2
so it is definitely positive. What is a positive verb that flows with 2
12345678901234567890123456789012123456789012345678901
12345678901234567890123456789012123456789012345678901 22
12345678901234567890123456789012123456789012345678901
greater interest? Created is a good pre-guess. Check the answer
12345678901234567890123456789012123456789012345678901 2
12345678901234567890123456789012123456789012345678901
choices, and (D), engendered, fits your pre-guess quite well. 2
1 2
2345678901234567890123456789012123456789012345678901
20. A The “although” at the beginning of the sentence clues you into 2
12345678901234567890123456789012123456789012345678901
12345678901234567890123456789012123456789012345678901 2
1 2
12345678901234567890123456789012123456789012345678901 2
the fact that this is a contrast sentence. The good times spoken of
12345678901234567890123456789012123456789012345678901
in the present are contrasted with what is in the blank. What is a 2
12345678901234567890123456789012123456789012345678901 2
12345678901234567890123456789012123456789012345678901 2
12345678901234567890123456789012123456789012345678901 2
contrast with good times? Tough times. Choice (A) works well.
12345678901234567890123456789012123456789012345678901 2
12345678901234567890123456789012123456789012345678901 2
12345678901234567890123456789012123456789012345678901 22
2345678901234567890123456789012123456789012345678901
21. E It should be easy to eliminate some answer choices with the first
12345678901234567890123456789012123456789012345678901
blank since it has to fit with crude. Choice (C) certainly does not 2
1
12345678901234567890123456789012123456789012345678901
fit and possibly (A) also. The second blank is a positive word 2
12345678901234567890123456789012123456789012345678901 2
2345678901234567890123456789012123456789012345678901
since the logic of the sentence is that though the early theories 2
12345678901234567890123456789012123456789012345678901
12345678901234567890123456789012123456789012345678901 2
were crude, they were still helpful. Thus we can eliminate (A) 2
12345678901234567890123456789012123456789012345678901
12345678901234567890123456789012123456789012345678901 22
1 and (D). Choice (E) works best.
12345678901234567890123456789012123456789012345678901 2
12345678901234567890123456789012123456789012345678901 2
12345678901234567890123456789012123456789012345678901
22. D Here the first blank seems more assailable. You know that it has 2
12345678901234567890123456789012123456789012345678901
to pair with factually misleading. Choices (B) and (E) do not, so 2
12345678901234567890123456789012123456789012345678901 2
2345678901234567890123456789012123456789012345678901
eliminate them. As for the second blank, you know two things: 2
12345678901234567890123456789012123456789012345678901
12345678901234567890123456789012123456789012345678901 2
12345678901234567890123456789012123456789012345678901 2
12345678901234567890123456789012123456789012345678901 22
It pairs with superior mastery of language, and it masks the
1
12345678901234567890123456789012123456789012345678901
problems mentioned in the first part of the sentence. Thus, it 2
12345678901234567890123456789012123456789012345678901
must be something fairly positive. Choices (A) and (C) do not 2
12345678901234567890123456789012123456789012345678901 2
12345678901234567890123456789012123456789012345678901 2
12345678901234567890123456789012123456789012345678901
meet these requirements, but (D)
2345678901234567890123456789012123456789012345678901
does. 2
2
1
12345678901234567890123456789012123456789012345678901 2
12345678901234567890123456789012123456789012345678901 2
12345678901234567890123456789012123456789012345678901 2 279
123456789012345678901234567890121234567890123456789012
Part IV: Three Practice GREs

123456789012345678901234567890121234567890123456789012
12345678901234567890123456789012123456789012345678901 2
12345678901234567890123456789012123456789012345678901 2
12345678901234567890123456789012123456789012345678901
23. D A pre-guess on the second blank would be a good place to start. 2
12345678901234567890123456789012123456789012345678901 2
12345678901234567890123456789012123456789012345678901
Something like understand works. Checking the answer choices, 2
12345678901234567890123456789012123456789012345678901 2
12345678901234567890123456789012123456789012345678901
we can eliminate (E). The first blank goes along with the 2
2345678901234567890123456789012123456789012345678901 2
1
12345678901234567890123456789012123456789012345678901
professor being visibly frustrated. Choice (B) is out because it 2
12345678901234567890123456789012123456789012345678901
does not fit the context. Choices (A) and (C) are okay choices, 2
12345678901234567890123456789012123456789012345678901 2
12345678901234567890123456789012123456789012345678901
2345678901234567890123456789012123456789012345678901
2
2
1 but (D) definitely works best in the first blank.
12345678901234567890123456789012123456789012345678901 2
12345678901234567890123456789012123456789012345678901 2
12345678901234567890123456789012123456789012345678901 2
12345678901234567890123456789012123456789012345678901
Reading Comprehension
2345678901234567890123456789012123456789012345678901
2
2
1
12345678901234567890123456789012123456789012345678901 2
12345678901234567890123456789012123456789012345678901
24. C The author’s tone toward the advancement of our understand- 2
12345678901234567890123456789012123456789012345678901 2
12345678901234567890123456789012123456789012345678901
ing of dark matter is most clearly seen in the last sentence of the 2
2345678901234567890123456789012123456789012345678901 2
1
12345678901234567890123456789012123456789012345678901
passage. There, the author looks forward to a time when “more 2
12345678901234567890123456789012123456789012345678901
fecund methods of investigating dark matter are developed.” 2
12345678901234567890123456789012123456789012345678901 2
12345678901234567890123456789012123456789012345678901 2
12345678901234567890123456789012123456789012345678901 2
2345678901234567890123456789012123456789012345678901
That is an expression of hope, but not unqualifiedly so (just read
the first part of that sentence to see that). Thus, (C) is the best 2
12345678901234567890123456789012123456789012345678901
12345678901234567890123456789012123456789012345678901 2
12345678901234567890123456789012123456789012345678901 2
12345678901234567890123456789012123456789012345678901 2
answer.
12345678901234567890123456789012123456789012345678901 2
1 25. D Here, you just have to go through each proposition and see 2
12345678901234567890123456789012123456789012345678901 2
12345678901234567890123456789012123456789012345678901
which ones the passage relates to. 2
12345678901234567890123456789012123456789012345678901 2
12345678901234567890123456789012123456789012345678901 2
2345678901234567890123456789012123456789012345678901
Option I—the passage makes clear that this is true. Check 2
12345678901234567890123456789012123456789012345678901
12345678901234567890123456789012123456789012345678901 2
12345678901234567890123456789012123456789012345678901
the third and fourth paragraph for verification of this. 2
12345678901234567890123456789012123456789012345678901 2
12345678901234567890123456789012123456789012345678901 2
12345678901234567890123456789012123456789012345678901
Option II—the passage states the exact opposite of this. 2
12345678901234567890123456789012123456789012345678901 2
1 Option III—lines 31–33, “Like light matter, dark matter 2
12345678901234567890123456789012123456789012345678901 2
2345678901234567890123456789012123456789012345678901
gravitationally attracts all matter.” III then is true. Choice 2
12345678901234567890123456789012123456789012345678901
12345678901234567890123456789012123456789012345678901 2
1 2
12345678901234567890123456789012123456789012345678901 2
(D) then is correct.
12345678901234567890123456789012123456789012345678901 2
12345678901234567890123456789012123456789012345678901
26. A To start, reread the entire third paragraph. You have to go 2
12345678901234567890123456789012123456789012345678901 2
12345678901234567890123456789012123456789012345678901
through the answer choices to see which is a correct inference 2
12345678901234567890123456789012123456789012345678901 2
12345678901234567890123456789012123456789012345678901
from the third paragraph. For choice (A), the third paragraph 2
2345678901234567890123456789012123456789012345678901
12345678901234567890123456789012123456789012345678901
reads, “Like light matter, dark matter gravitationally attracts all 2
2
12345678901234567890123456789012123456789012345678901 2
1
12345678901234567890123456789012123456789012345678901
matter and so sustains a gravitational field.” If dark matter 2
12345678901234567890123456789012123456789012345678901 2
12345678901234567890123456789012123456789012345678901
sustains a gravitational field and dark matter is like light matter 2
12345678901234567890123456789012123456789012345678901 2
12345678901234567890123456789012123456789012345678901
in this respect, then it can be inferred that light matter sustains a 2
12345678901234567890123456789012123456789012345678901
gravitational field. For choice (B), the third paragraph does not 2
12345678901234567890123456789012123456789012345678901 2
2345678901234567890123456789012123456789012345678901
mention anything about the comparative stability of light and 2
12345678901234567890123456789012123456789012345678901
12345678901234567890123456789012123456789012345678901 2
12345678901234567890123456789012123456789012345678901 2
12345678901234567890123456789012123456789012345678901 2
dark matter. Choice (C) has it backward, as astronomers infer
1 the existence of dark matter from insufficient light matter. 2
12345678901234567890123456789012123456789012345678901 2
12345678901234567890123456789012123456789012345678901 2
12345678901234567890123456789012123456789012345678901
Choice (D) is true, but is stated in the first paragraph, not the 2
12345678901234567890123456789012123456789012345678901 2
12345678901234567890123456789012123456789012345678901 2
third. For (E), the third paragraph does not discuss theoretical
2345678901234567890123456789012123456789012345678901
hypotheses about dark matter. Choice (A) is the correct answer. 2
12345678901234567890123456789012123456789012345678901
1 2
12345678901234567890123456789012123456789012345678901 2
12345678901234567890123456789012123456789012345678901
27. C On a question like this, you have to go through the answer 2
12345678901234567890123456789012123456789012345678901 2
12345678901234567890123456789012123456789012345678901
choices to see which is best. The title, of course, should reflect 2
12345678901234567890123456789012123456789012345678901 2
12345678901234567890123456789012123456789012345678901
the tone and content of the passage, so you are looking for the 2
2
280 12345678901234567890123456789012123456789012345678901
123456789012345678901234567890121234567890123456789012

www.petersons.com
ANSWERS
123456789012345678901234567890121234567890123456789012
12345678901234567890123456789012123456789012345678901 2
12345678901234567890123456789012123456789012345678901 2
12345678901234567890123456789012123456789012345678901
title that best does this. Choice (A) could be the title of the 2
12345678901234567890123456789012123456789012345678901 2
12345678901234567890123456789012123456789012345678901
second paragraph, but not of the entire passage. Choice (B) 2
12345678901234567890123456789012123456789012345678901 2
12345678901234567890123456789012123456789012345678901
sounds appealing but there is no indication in the passage that 2
2345678901234567890123456789012123456789012345678901 2
1
12345678901234567890123456789012123456789012345678901
the issues discussed are just ones for the present time. Choice (C) 2
12345678901234567890123456789012123456789012345678901
captures the tone and content of the passage and so is a likely 2
12345678901234567890123456789012123456789012345678901 2
12345678901234567890123456789012123456789012345678901
2345678901234567890123456789012123456789012345678901
2
2
1 candidate. Choice (D) is also appealing except for the phrase
12345678901234567890123456789012123456789012345678901 2
12345678901234567890123456789012123456789012345678901
Intellectual Historiography, which does not appear in the 2
12345678901234567890123456789012123456789012345678901 2
12345678901234567890123456789012123456789012345678901
passage. Choice (E) is much too extreme
2345678901234567890123456789012123456789012345678901
in its tone. Choice (C) 2
2
1
12345678901234567890123456789012123456789012345678901
is the best choice. 2
12345678901234567890123456789012123456789012345678901 2
12345678901234567890123456789012123456789012345678901
C Reread the second paragraph (or at least skim it) and consider 2
12345678901234567890123456789012123456789012345678901
28.
2345678901234567890123456789012123456789012345678901
2
1 its relation to the rest of the passage. Then, proceed to the 2
12345678901234567890123456789012123456789012345678901 2
12345678901234567890123456789012123456789012345678901
question. Notice that the second paragraph contains two 2
12345678901234567890123456789012123456789012345678901 2
12345678901234567890123456789012123456789012345678901
contrasting examples. The answer choice should reflect this. The 2
12345678901234567890123456789012123456789012345678901 22
2345678901234567890123456789012123456789012345678901
12345678901234567890123456789012123456789012345678901
basic thesis of the passage is that there are multiple narratives in
12345678901234567890123456789012123456789012345678901
historiography of the same events. The content of the second 2
2
12345678901234567890123456789012123456789012345678901
12345678901234567890123456789012123456789012345678901 2
12345678901234567890123456789012123456789012345678901
paragraph illustrates this thesis. Thus, (C) is correct. 2
1 2
12345678901234567890123456789012123456789012345678901 2
12345678901234567890123456789012123456789012345678901
29. C Go and read the relevant text and be careful to have in mind 2
12345678901234567890123456789012123456789012345678901 2
12345678901234567890123456789012123456789012345678901 22
2345678901234567890123456789012123456789012345678901
what Burke’s view is, which is that the Revolution produced a
12345678901234567890123456789012123456789012345678901
result that was a radical departure from the past. On this view 2
12345678901234567890123456789012123456789012345678901
the phrase historical novelty clearly implies the newness and 2
12345678901234567890123456789012123456789012345678901
12345678901234567890123456789012123456789012345678901 2
uniqueness of the French Republic. Choice (C) fits this well. 2
12345678901234567890123456789012123456789012345678901
12345678901234567890123456789012123456789012345678901 2
12345678901234567890123456789012123456789012345678901 2
12345678901234567890123456789012123456789012345678901 22
Choice (E) might have seemed attractive but recall that
1
12345678901234567890123456789012123456789012345678901
institutional continuity was never mentioned in the passage. 2
12345678901234567890123456789012123456789012345678901
That should make you wary of that choice. Choice (D) is the 2
12345678901234567890123456789012123456789012345678901 2
12345678901234567890123456789012123456789012345678901
view of de Tocqueville, not Burke. So, (C) is the best choice. 2
12345678901234567890123456789012123456789012345678901
2345678901234567890123456789012123456789012345678901
2
2
12345678901234567890123456789012123456789012345678901
30. B This question relates to the main idea and tone of the passage. 2
12345678901234567890123456789012123456789012345678901 2
1
12345678901234567890123456789012123456789012345678901
The correct answer will reflect both. 2
12345678901234567890123456789012123456789012345678901 2
2345678901234567890123456789012123456789012345678901
Main idea: Historiography can be influenced by ideology 2
12345678901234567890123456789012123456789012345678901
12345678901234567890123456789012123456789012345678901 2
1 2
12345678901234567890123456789012123456789012345678901 2
and so intellectually honest historiography should be
12345678901234567890123456789012123456789012345678901 2
12345678901234567890123456789012123456789012345678901
mindful of this. 2
12345678901234567890123456789012123456789012345678901 2
12345678901234567890123456789012123456789012345678901 2
12345678901234567890123456789012123456789012345678901
Tone: Guarded optimism that with care historiography can 2
12345678901234567890123456789012123456789012345678901 2
12345678901234567890123456789012123456789012345678901 22
2345678901234567890123456789012123456789012345678901
be intellectually honest
12345678901234567890123456789012123456789012345678901 2
12345678901234567890123456789012123456789012345678901
Choice (A) is too extreme. Choice (B) fits well with both. As for
2
12345678901234567890123456789012123456789012345678901
1 (C), though the passage states, “historiography purports to be 2
2345678901234567890123456789012123456789012345678901
12345678901234567890123456789012123456789012345678901
an empirical endeavor free of . . . biases” the passage neither 2
2
12345678901234567890123456789012123456789012345678901 2
12345678901234567890123456789012123456789012345678901
defines what it is to be a science nor states that historiography is 2
12345678901234567890123456789012123456789012345678901 2
1
12345678901234567890123456789012123456789012345678901
not one. Choice (D) is cynical, and the tone of the passage is not 2
12345678901234567890123456789012123456789012345678901 2
12345678901234567890123456789012123456789012345678901
(just read the last sentence of the passage). Choice (E) is also 2
12345678901234567890123456789012123456789012345678901 2
12345678901234567890123456789012123456789012345678901
cynical, more so than the passage is. So, (B) is the best choice.
2345678901234567890123456789012123456789012345678901
2
2
1
12345678901234567890123456789012123456789012345678901 2
12345678901234567890123456789012123456789012345678901 2
12345678901234567890123456789012123456789012345678901 2 281
123456789012345678901234567890121234567890123456789012
Practice Test

2
1234567890123456789012345678901212345678901234567890123456789012123456
12 3456789012345678901234567890121234567890123456789012345678901212345 6
12 3456789012345678901234567890121234567890123456789012345678901212345 6
12 3456789012345678901234567890121234567890123456789012345678901212345
Quantitative 6
123456789012345678901234567890121234567890123456789012345678901212345
2 6
123456789012345678901234567890121234567890123456789012345678901212345 6
123456789012345678901234567890121234567890123456789012345678901212345
28 Questions—45 Minutes 6
123456789012345678901234567890121234567890123456789012345678901212345 6
123456789012345678901234567890121234567890123456789012345678901212345 6
123456789012345678901234567890121234567890123456789012345678901212345 66
3456789012345678901234567890121234567890123456789012345678901212345
General Information
1
12 3456789012345678901234567890121234567890123456789012345678901212345
1. The test has 28 questions. If you decide to take this exam under timed conditions, you have 6
12 3456789012345678901234567890121234567890123456789012345678901212345 6
12 3456789012345678901234567890121234567890123456789012345678901212345
45 minutes to complete the section. 6
123456789012345678901234567890121234567890123456789012345678901212345 66
2 3456789012345678901234567890121234567890123456789012345678901212345
123456789012345678901234567890121234567890123456789012345678901212345
123456789012345678901234567890121234567890123456789012345678901212345
2. All numbers used are real numbers. 6
123456789012345678901234567890121234567890123456789012345678901212345 66
123456789012345678901234567890121234567890123456789012345678901212345
123456789012345678901234567890121234567890123456789012345678901212345
3. All angle measurements can be assumed to be positive unless otherwise noted. 6
123456789012345678901234567890121234567890123456789012345678901212345 66
123456789012345678901234567890121234567890123456789012345678901212345
123456789012345678901234567890121234567890123456789012345678901212345
4. All figures lie in the same plane unless otherwise noted. 6
1 6
2 3456789012345678901234567890121234567890123456789012345678901212345
5. Drawings that accompany questions are intended to provide information useful in 6
123456789012345678901234567890121234567890123456789012345678901212345
123456789012345678901234567890121234567890123456789012345678901212345 6
1 answering the question. The figures are drawn closely to scale unless otherwise noted. 6
123456789012345678901234567890121234567890123456789012345678901212345 6
12 3456789012345678901234567890121234567890123456789012345678901212345 6
12 3456789012345678901234567890121234567890123456789012345678901212345 6
12 3456789012345678901234567890121234567890123456789012345678901212345
Directions: For questions 16–28, Column A and Column B will have one quantity each. 6
123456789012345678901234567890121234567890123456789012345678901212345 6
12 3456789012345678901234567890121234567890123456789012345678901212345
Your goal is to compare the quantities in each column and choose 6
12 3456789012345678901234567890121234567890123456789012345678901212345 6
2 3456789012345678901234567890121234567890123456789012345678901212345
123456789012345678901234567890121234567890123456789012345678901212345 6
123456789012345678901234567890121234567890123456789012345678901212345
A if the quantity in Column A is greater 6
1 6
123456789012345678901234567890121234567890123456789012345678901212345
B if the quantity in Column B is greater 6
123456789012345678901234567890121234567890123456789012345678901212345 6
123456789012345678901234567890121234567890123456789012345678901212345 66
2 3456789012345678901234567890121234567890123456789012345678901212345
C if the two quantities are equal
123456789012345678901234567890121234567890123456789012345678901212345 6
123456789012345678901234567890121234567890123456789012345678901212345
D if the relationship between the two quantities cannot be determined from the
6
123456789012345678901234567890121234567890123456789012345678901212345 6
1 information in the problem
123456789012345678901234567890121234567890123456789012345678901212345 66
2 3456789012345678901234567890121234567890123456789012345678901212345
123456789012345678901234567890121234567890123456789012345678901212345 6
123456789012345678901234567890121234567890123456789012345678901212345
On some questions, information about one or both quantities will be given. This informa-
6
123456789012345678901234567890121234567890123456789012345678901212345 6
1 tion will be centered above the two columns.
123456789012345678901234567890121234567890123456789012345678901212345 66
2 3456789012345678901234567890121234567890123456789012345678901212345
123456789012345678901234567890121234567890123456789012345678901212345 6
123456789012345678901234567890121234567890123456789012345678901212345 6
123456789012345678901234567890121234567890123456789012345678901212345 6
1
12 3456789012345678901234567890121234567890123456789012345678901212345 6
123456789012345678901234567890121234567890123456789012345678901212345 6
12 3456789012345678901234567890121234567890123456789012345678901212345 6
123456789012345678901234567890121234567890123456789012345678901212345 6
12 3456789012345678901234567890121234567890123456789012345678901212345 6
123456789012345678901234567890121234567890123456789012345678901212345 6
123456789012345678901234567890121234567890123456789012345678901212345 6
123456789012345678901234567890121234567890123456789012345678901212345 6
123456789012345678901234567890121234567890123456789012345678901212345 6
1234567890123456789012345678901212345678901234567890123456789012123456
282
TEST 2
1234567890123456789012345678901212345678901234567890123456789012123456
123456789012345678901234567890121234567890123456789012345678901212345 6
12 3456789012345678901234567890121234567890123456789012345678901212345 6
123456789012345678901234567890121234567890123456789012345678901212345
1. 4. If 3c 1 6d 5 24 and 4a 13d 5 18, what 6
12 3456789012345678901234567890121234567890123456789012345678901212345 6
12 3456789012345678901234567890121234567890123456789012345678901212345
does a equal in terms of c? 6
123456789012345678901234567890121234567890123456789012345678901212345 6
12 3456789012345678901234567890121234567890123456789012345678901212345 6
123456789012345678901234567890121234567890123456789012345678901212345
9 3 6
12 3456789012345678901234567890121234567890123456789012345678901212345
A. 2 c 6
12 3456789012345678901234567890121234567890123456789012345678901212345
2 4 6
12 3456789012345678901234567890121234567890123456789012345678901212345 6
12 3456789012345678901234567890121234567890123456789012345678901212345
3 3 6
123456789012345678901234567890121234567890123456789012345678901212345
B. c1 6
12 3456789012345678901234567890121234567890123456789012345678901212345
BCDE is a parallelogram. What is its 8 2 6
12 3456789012345678901234567890121234567890123456789012345678901212345 6
12 3456789012345678901234567890121234567890123456789012345678901212345
area? 6
12
2
3456789012345678901234567890121234567890123456789012345678901212345
3 12
3456789012345678901234567890121234567890123456789012345678901212345
6
6
1 C. c 1
12 3456789012345678901234567890121234567890123456789012345678901212345
A. =10 11 11 6
12 3456789012345678901234567890121234567890123456789012345678901212345 6
12 3456789012345678901234567890121234567890123456789012345678901212345
B. 10 1 6
12
2
3456789012345678901234567890121234567890123456789012345678901212345
D. c 1 4
3456789012345678901234567890121234567890123456789012345678901212345
6
6
1 C. 10=2 2
12 3456789012345678901234567890121234567890123456789012345678901212345 6
12 3456789012345678901234567890121234567890123456789012345678901212345
10=5 3 6
12 3456789012345678901234567890121234567890123456789012345678901212345
D. 6
12 3456789012345678901234567890121234567890123456789012345678901212345
E. c 1 5 6
2 3456789012345678901234567890121234567890123456789012345678901212345
123456789012345678901234567890121234567890123456789012345678901212345
E. 10=10
4 6
123456789012345678901234567890121234567890123456789012345678901212345 6
123456789012345678901234567890121234567890123456789012345678901212345 66
123456789012345678901234567890121234567890123456789012345678901212345
123456789012345678901234567890121234567890123456789012345678901212345
2. If &x 5 [x 2 (x 2 1)](x 1 2) then Questions 5–6 refer to the following graph. 6
123456789012345678901234567890121234567890123456789012345678901212345 6
1 &(&(2)) 5 6
12 3456789012345678901234567890121234567890123456789012345678901212345 6
12 3456789012345678901234567890121234567890123456789012345678901212345 6
12 3456789012345678901234567890121234567890123456789012345678901212345
A. 4 6
2 3456789012345678901234567890121234567890123456789012345678901212345
123456789012345678901234567890121234567890123456789012345678901212345 6
123456789012345678901234567890121234567890123456789012345678901212345
B. 6 6
123456789012345678901234567890121234567890123456789012345678901212345 6
123456789012345678901234567890121234567890123456789012345678901212345 66
C. 12
123456789012345678901234567890121234567890123456789012345678901212345
D. 18
123456789012345678901234567890121234567890123456789012345678901212345 6
123456789012345678901234567890121234567890123456789012345678901212345
E. 24 6
123456789012345678901234567890121234567890123456789012345678901212345 66
123456789012345678901234567890121234567890123456789012345678901212345 6
1 3. The price of Company X’s stock started
12 3456789012345678901234567890121234567890123456789012345678901212345 6
12 3456789012345678901234567890121234567890123456789012345678901212345 6
123456789012345678901234567890121234567890123456789012345678901212345
the day at $40 per share, rose to $90 per 6
123456789012345678901234567890121234567890123456789012345678901212345 6
12
2
3456789012345678901234567890121234567890123456789012345678901212345 6
share, and then fell to $45 per share.
3456789012345678901234567890121234567890123456789012345678901212345 6
123456789012345678901234567890121234567890123456789012345678901212345
123456789012345678901234567890121234567890123456789012345678901212345
Compared to the price at the beginning of 6
1 6
12 3456789012345678901234567890121234567890123456789012345678901212345 6
the day, the final price of the stock was a
12 3456789012345678901234567890121234567890123456789012345678901212345 6
12 3456789012345678901234567890121234567890123456789012345678901212345 6
12 3456789012345678901234567890121234567890123456789012345678901212345
A. 10% increase. 6
123456789012345678901234567890121234567890123456789012345678901212345 6
123456789012345678901234567890121234567890123456789012345678901212345
B. 10% decrease.
5. How many millions of years ago did the 6
123456789012345678901234567890121234567890123456789012345678901212345
C. 12.5% increase. 6
2 3456789012345678901234567890121234567890123456789012345678901212345
123456789012345678901234567890121234567890123456789012345678901212345
extinction rate reach its highest value? 6
123456789012345678901234567890121234567890123456789012345678901212345
D. 12.5% decrease. 6
123456789012345678901234567890121234567890123456789012345678901212345 6
123456789012345678901234567890121234567890123456789012345678901212345
E. 50% decrease. A. 200 6
1 6
123456789012345678901234567890121234567890123456789012345678901212345 66
2 3456789012345678901234567890121234567890123456789012345678901212345
B. 245
123456789012345678901234567890121234567890123456789012345678901212345 6
123456789012345678901234567890121234567890123456789012345678901212345
C. 500
6
123456789012345678901234567890121234567890123456789012345678901212345 6
1 D. 560
2 3456789012345678901234567890121234567890123456789012345678901212345
123456789012345678901234567890121234567890123456789012345678901212345 6
123456789012345678901234567890121234567890123456789012345678901212345
E. 600 6
123456789012345678901234567890121234567890123456789012345678901212345 66
123456789012345678901234567890121234567890123456789012345678901212345 6
1
12 3456789012345678901234567890121234567890123456789012345678901212345 6
123456789012345678901234567890121234567890123456789012345678901212345 6
12 3456789012345678901234567890121234567890123456789012345678901212345 6
123456789012345678901234567890121234567890123456789012345678901212345 6
12 3456789012345678901234567890121234567890123456789012345678901212345 6
123456789012345678901234567890121234567890123456789012345678901212345 6
123456789012345678901234567890121234567890123456789012345678901212345 6
123456789012345678901234567890121234567890123456789012345678901212345 6
123456789012345678901234567890121234567890123456789012345678901212345 6 283
1234567890123456789012345678901212345678901234567890123456789012123456
Part IV: Three Practice GREs

1234567890123456789012345678901212345678901234567890123456789012123456
123456789012345678901234567890121234567890123456789012345678901212345 6
12 3456789012345678901234567890121234567890123456789012345678901212345 6
123456789012345678901234567890121234567890123456789012345678901212345
6. In which era did the extinction rate and 9. 6
12 3456789012345678901234567890121234567890123456789012345678901212345 6
12 3456789012345678901234567890121234567890123456789012345678901212345
number of families have the same value
G
6
123456789012345678901234567890121234567890123456789012345678901212345 6
12
2
3456789012345678901234567890121234567890123456789012345678901212345 6
the most number of times?
3456789012345678901234567890121234567890123456789012345678901212345 6
1
12 3456789012345678901234567890121234567890123456789012345678901212345 6
12 3456789012345678901234567890121234567890123456789012345678901212345 6
A. Pre-Cambrian
12 3456789012345678901234567890121234567890123456789012345678901212345 6
12
2
3456789012345678901234567890121234567890123456789012345678901212345
B. Paleozoic
3456789012345678901234567890121234567890123456789012345678901212345
6
6
1
12 3456789012345678901234567890121234567890123456789012345678901212345
C. Mesozoic 6
12 3456789012345678901234567890121234567890123456789012345678901212345 F
6
12 3456789012345678901234567890121234567890123456789012345678901212345 6
D. Cenozoic
12 3456789012345678901234567890121234567890123456789012345678901212345 6
123456789012345678901234567890121234567890123456789012345678901212345
E. Silurian 6
12 3456789012345678901234567890121234567890123456789012345678901212345 6
12 3456789012345678901234567890121234567890123456789012345678901212345 6
D E
12 3456789012345678901234567890121234567890123456789012345678901212345
7. Which of the following numbers would 6
12
2
3456789012345678901234567890121234567890123456789012345678901212345
3456789012345678901234567890121234567890123456789012345678901212345
6
6
1 have a remainder of zero if it were divided The area of square B is 16 meters squared
12 3456789012345678901234567890121234567890123456789012345678901212345 6
12 3456789012345678901234567890121234567890123456789012345678901212345
by 3 or 6 and 9? and DE 5 EG What is the area of square 6
12 3456789012345678901234567890121234567890123456789012345678901212345 6
12 3456789012345678901234567890121234567890123456789012345678901212345
C in meters squared? 6
123456789012345678901234567890121234567890123456789012345678901212345 66
2 3456789012345678901234567890121234567890123456789012345678901212345
A. 1,024
123456789012345678901234567890121234567890123456789012345678901212345
A. 1
123456789012345678901234567890121234567890123456789012345678901212345
B. 1,323 6
123456789012345678901234567890121234567890123456789012345678901212345
B. 2 6
123456789012345678901234567890121234567890123456789012345678901212345 66
C. 1,506
123456789012345678901234567890121234567890123456789012345678901212345
C. 4
6
1 D. 1,632
12 3456789012345678901234567890121234567890123456789012345678901212345
D. 6 6
12 3456789012345678901234567890121234567890123456789012345678901212345
E. 1,764 6
12 3456789012345678901234567890121234567890123456789012345678901212345
E. 9 6
123456789012345678901234567890121234567890123456789012345678901212345 66
2 3456789012345678901234567890121234567890123456789012345678901212345
123456789012345678901234567890121234567890123456789012345678901212345
8. In the number set {15, 10, 7, 12, 10, 18} a
123456789012345678901234567890121234567890123456789012345678901212345 6
123456789012345678901234567890121234567890123456789012345678901212345
equals the mean, b equals the mode, c 6
123456789012345678901234567890121234567890123456789012345678901212345
Questions 10–12 refer to the following graphs. 6
123456789012345678901234567890121234567890123456789012345678901212345 66
equals the median, and d equals the range.
123456789012345678901234567890121234567890123456789012345678901212345
123456789012345678901234567890121234567890123456789012345678901212345
Which of the following statements is true? 6
123456789012345678901234567890121234567890123456789012345678901212345 66
1
12 3456789012345678901234567890121234567890123456789012345678901212345
A. a , b 5 c , d 6
12 3456789012345678901234567890121234567890123456789012345678901212345 6
123456789012345678901234567890121234567890123456789012345678901212345 6
B. a . b , c . d
123456789012345678901234567890121234567890123456789012345678901212345 6
12
2
3456789012345678901234567890121234567890123456789012345678901212345
C. a 5 b 5 c . d
3456789012345678901234567890121234567890123456789012345678901212345
6
6
123456789012345678901234567890121234567890123456789012345678901212345
123456789012345678901234567890121234567890123456789012345678901212345
D. a . b , c 5 d 6
1 6
12 3456789012345678901234567890121234567890123456789012345678901212345 6
E. a , b , c , d
12 3456789012345678901234567890121234567890123456789012345678901212345 6
12 3456789012345678901234567890121234567890123456789012345678901212345 6
12 3456789012345678901234567890121234567890123456789012345678901212345 6
123456789012345678901234567890121234567890123456789012345678901212345 6
123456789012345678901234567890121234567890123456789012345678901212345 6
123456789012345678901234567890121234567890123456789012345678901212345 6
12 3456789012345678901234567890121234567890123456789012345678901212345 6
12 3456789012345678901234567890121234567890123456789012345678901212345 6
12 3456789012345678901234567890121234567890123456789012345678901212345 6
12 3456789012345678901234567890121234567890123456789012345678901212345 6
123456789012345678901234567890121234567890123456789012345678901212345 6
123456789012345678901234567890121234567890123456789012345678901212345
2 10. Which county and crop had the smallest 6
123456789012345678901234567890121234567890123456789012345678901212345 6
123456789012345678901234567890121234567890123456789012345678901212345
percentage change from 1970 to 1990? 6
123456789012345678901234567890121234567890123456789012345678901212345 6
1 3456789012345678901234567890121234567890123456789012345678901212345 6
123456789012345678901234567890121234567890123456789012345678901212345 66
2 3456789012345678901234567890121234567890123456789012345678901212345
A. County X, Corn
123456789012345678901234567890121234567890123456789012345678901212345 6
123456789012345678901234567890121234567890123456789012345678901212345
B. County Y, Corn
6
123456789012345678901234567890121234567890123456789012345678901212345
C. County X, Wheat 6
1
2 3456789012345678901234567890121234567890123456789012345678901212345
123456789012345678901234567890121234567890123456789012345678901212345
D. County Y, Wheat 6
1 6
12 3456789012345678901234567890121234567890123456789012345678901212345
E. None of the above 6
123456789012345678901234567890121234567890123456789012345678901212345 6
12 3456789012345678901234567890121234567890123456789012345678901212345 6
123456789012345678901234567890121234567890123456789012345678901212345 6
123456789012345678901234567890121234567890123456789012345678901212345 6
123456789012345678901234567890121234567890123456789012345678901212345 66
284 123456789012345678901234567890121234567890123456789012345678901212345
1234567890123456789012345678901212345678901234567890123456789012123456

www.petersons.com
TEST 2
1234567890123456789012345678901212345678901234567890123456789012123456
123456789012345678901234567890121234567890123456789012345678901212345 6
12 3456789012345678901234567890121234567890123456789012345678901212345 6
123456789012345678901234567890121234567890123456789012345678901212345
11. How much on average would ten acres of 14. 6
12 3456789012345678901234567890121234567890123456789012345678901212345 6
12 3456789012345678901234567890121234567890123456789012345678901212345
corn in County Y in 1980 produce? 6
123456789012345678901234567890121234567890123456789012345678901212345 6
12 3456789012345678901234567890121234567890123456789012345678901212345 6
123456789012345678901234567890121234567890123456789012345678901212345
A. 100 bushels 6
12 3456789012345678901234567890121234567890123456789012345678901212345 6
12 3456789012345678901234567890121234567890123456789012345678901212345 6
B. 120 bushels
12 3456789012345678901234567890121234567890123456789012345678901212345 6
12
2
3456789012345678901234567890121234567890123456789012345678901212345
C. 150 bushels
3456789012345678901234567890121234567890123456789012345678901212345
6
6
1
12 3456789012345678901234567890121234567890123456789012345678901212345
D. 200 bushels 6
12 3456789012345678901234567890121234567890123456789012345678901212345 6
12 3456789012345678901234567890121234567890123456789012345678901212345 6
E. 220 bushels
12 3456789012345678901234567890121234567890123456789012345678901212345 6
123456789012345678901234567890121234567890123456789012345678901212345 6
12 3456789012345678901234567890121234567890123456789012345678901212345
12. What is the ratio of bushels of corn 6
12 3456789012345678901234567890121234567890123456789012345678901212345 6
12 3456789012345678901234567890121234567890123456789012345678901212345
produced per acre to bushels of wheat 6
12
2
3456789012345678901234567890121234567890123456789012345678901212345
3456789012345678901234567890121234567890123456789012345678901212345
6
6
1 produced per acre in County Y in 1980? For the 5-sided polygon above a 1 b 5
12 3456789012345678901234567890121234567890123456789012345678901212345 6
12 3456789012345678901234567890121234567890123456789012345678901212345
160 degrees and d 2 b 5 30. How many 6
12 3456789012345678901234567890121234567890123456789012345678901212345
A. 1:1 degrees is c e b? 6
12 34567890123456789012345678901212345678901234567890123456789012123451 1 6
123456789012345678901234567890121234567890123456789012345678901212345 66
2 3456789012345678901234567890121234567890123456789012345678901212345
B. 2:1
123456789012345678901234567890121234567890123456789012345678901212345
A. 70
123456789012345678901234567890121234567890123456789012345678901212345
C. 3:1 6
123456789012345678901234567890121234567890123456789012345678901212345
B. 125 6
123456789012345678901234567890121234567890123456789012345678901212345
D. 3:2 6
123456789012345678901234567890121234567890123456789012345678901212345
C. 350 6
1 E. 3:4 6
12 3456789012345678901234567890121234567890123456789012345678901212345
D. 300 6
12 3456789012345678901234567890121234567890123456789012345678901212345 6
12 3456789012345678901234567890121234567890123456789012345678901212345
13. How many distinct integers less than 100 can E. 325 6
123456789012345678901234567890121234567890123456789012345678901212345 66
2 3456789012345678901234567890121234567890123456789012345678901212345
123456789012345678901234567890121234567890123456789012345678901212345
be formulated with the digits 2, 4, and 6?
123456789012345678901234567890121234567890123456789012345678901212345 6
123456789012345678901234567890121234567890123456789012345678901212345
15. What is the sum of the consecutive 6
123456789012345678901234567890121234567890123456789012345678901212345
A. 9 6
123456789012345678901234567890121234567890123456789012345678901212345
integers from 15 to 55 (inclusive)? 6
123456789012345678901234567890121234567890123456789012345678901212345
B. 11 6
123456789012345678901234567890121234567890123456789012345678901212345
A. 1405 6
123456789012345678901234567890121234567890123456789012345678901212345 66
C. 12
1 B. 1435
12 3456789012345678901234567890121234567890123456789012345678901212345
D. 14 6
12 3456789012345678901234567890121234567890123456789012345678901212345
C. 1465 6
123456789012345678901234567890121234567890123456789012345678901212345
E. 15 6
123456789012345678901234567890121234567890123456789012345678901212345
D. 1500 6
12 3456789012345678901234567890121234567890123456789012345678901212345 6
123456789012345678901234567890121234567890123456789012345678901212345 66
2 3456789012345678901234567890121234567890123456789012345678901212345
E. 1587
123456789012345678901234567890121234567890123456789012345678901212345 6
1
12 3456789012345678901234567890121234567890123456789012345678901212345 6
12 3456789012345678901234567890121234567890123456789012345678901212345 6
123456789012345678901234567890121234567890123456789012345678901212345
2 6
123456789012345678901234567890121234567890123456789012345678901212345 6
1 3456789012345678901234567890121234567890123456789012345678901212345 6
123456789012345678901234567890121234567890123456789012345678901212345 6
123456789012345678901234567890121234567890123456789012345678901212345 6
12 3456789012345678901234567890121234567890123456789012345678901212345 6
12 3456789012345678901234567890121234567890123456789012345678901212345 6
12 3456789012345678901234567890121234567890123456789012345678901212345 6
12 3456789012345678901234567890121234567890123456789012345678901212345 6
123456789012345678901234567890121234567890123456789012345678901212345 6
123456789012345678901234567890121234567890123456789012345678901212345
2 6
123456789012345678901234567890121234567890123456789012345678901212345 6
123456789012345678901234567890121234567890123456789012345678901212345 6
123456789012345678901234567890121234567890123456789012345678901212345 6
1 3456789012345678901234567890121234567890123456789012345678901212345 6
123456789012345678901234567890121234567890123456789012345678901212345 66
2 3456789012345678901234567890121234567890123456789012345678901212345
123456789012345678901234567890121234567890123456789012345678901212345 6
123456789012345678901234567890121234567890123456789012345678901212345 6
123456789012345678901234567890121234567890123456789012345678901212345 6
1
12 3456789012345678901234567890121234567890123456789012345678901212345 6
123456789012345678901234567890121234567890123456789012345678901212345 6
12 3456789012345678901234567890121234567890123456789012345678901212345 6
123456789012345678901234567890121234567890123456789012345678901212345 6
12 3456789012345678901234567890121234567890123456789012345678901212345 6
123456789012345678901234567890121234567890123456789012345678901212345 6
123456789012345678901234567890121234567890123456789012345678901212345 6
123456789012345678901234567890121234567890123456789012345678901212345 6
123456789012345678901234567890121234567890123456789012345678901212345 6 285
1234567890123456789012345678901212345678901234567890123456789012123456
Part IV: Three Practice GREs

1234567890123456789012345678901212345678901234567890123456789012123456
123456789012345678901234567890121234567890123456789012345678901212345 6
12 3456789012345678901234567890121234567890123456789012345678901212345 6
123456789012345678901234567890121234567890123456789012345678901212345
Column A Column B Column A Column B 6
12 3456789012345678901234567890121234567890123456789012345678901212345 6
12 3456789012345678901234567890121234567890123456789012345678901212345 6
123456789012345678901234567890121234567890123456789012345678901212345
16. The circumference The perimeter of a 20. The circumference The perimeter of a 6
12 3456789012345678901234567890121234567890123456789012345678901212345 6
123456789012345678901234567890121234567890123456789012345678901212345
of a circle with square that has one of a circle with a rectangle with an 6
12 3456789012345678901234567890121234567890123456789012345678901212345 6
12 3456789012345678901234567890121234567890123456789012345678901212345
a diameter 1 diameter of 7 area of 24 6
12 3456789012345678901234567890121234567890123456789012345678901212345
side equal to 1 6
12
2
3456789012345678901234567890121234567890123456789012345678901212345 6
equal to 2 2
3456789012345678901234567890121234567890123456789012345678901212345 6
1
12 3456789012345678901234567890121234567890123456789012345678901212345 6
12 3456789012345678901234567890121234567890123456789012345678901212345 6
12 3456789012345678901234567890121234567890123456789012345678901212345 6
12 3456789012345678901234567890121234567890123456789012345678901212345 6
1 3456789012345678901234567890121234567890123456789012345678901212345 6
2
12 3456789012345678901234567890121234567890123456789012345678901212345 6
12 3456789012345678901234567890121234567890123456789012345678901212345 6
12 3456789012345678901234567890121234567890123456789012345678901212345 6
12 3456789012345678901234567890121234567890123456789012345678901212345 6
1 3456789012345678901234567890121234567890123456789012345678901212345
2 21. z is a one-digit even integer. The square 6
12 3456789012345678901234567890121234567890123456789012345678901212345 6
12 3456789012345678901234567890121234567890123456789012345678901212345 6
12
12
2
17.
S DS D
1 1
3 4
S DS D
2 1
3 5
root of z is also an integer.
3456789012345678901234567890121234567890123456789012345678901212345
3456789012345678901234567890121234567890123456789012345678901212345
123456789012345678901234567890121234567890123456789012345678901212345 66
3456789012345678901234567890121234567890123456789012345678901212345
z 1 z 1 z z 2
6
6
123456789012345678901234567890121234567890123456789012345678901212345
S DS D S DS D
123456789012345678901234567890121234567890123456789012345678901212345
2 2 2 1
123456789012345678901234567890121234567890123456789012345678901212345
123456789012345678901234567890121234567890123456789012345678901212345 66
3 4 5 3
6
6
123456789012345678901234567890121234567890123456789012345678901212345 6
1
12 3456789012345678901234567890121234567890123456789012345678901212345 6
12 3456789012345678901234567890121234567890123456789012345678901212345 6
12 3456789012345678901234567890121234567890123456789012345678901212345 6
123456789012345678901234567890121234567890123456789012345678901212345
2 6
123456789012345678901234567890121234567890123456789012345678901212345 6
123456789012345678901234567890121234567890123456789012345678901212345 6
123456789012345678901234567890121234567890123456789012345678901212345
22. $16,000 is invested at 5% simple 6
123456789012345678901234567890121234567890123456789012345678901212345 6
3456789012345678901234567890121234567890123456789012345678901212345
123456789012345678901234567890121234567890123456789012345678901212345
annual interest. 6
123456789012345678901234567890121234567890123456789012345678901212345
18. AB 5 CD 6
123456789012345678901234567890121234567890123456789012345678901212345 6
123456789012345678901234567890121234567890123456789012345678901212345 66
Interest accrued Six hundred dollars
1
12 3456789012345678901234567890121234567890123456789012345678901212345
after eight months 6
12 3456789012345678901234567890121234567890123456789012345678901212345 6
123456789012345678901234567890121234567890123456789012345678901212345 6
123456789012345678901234567890121234567890123456789012345678901212345
BC 10t 6
12 3456789012345678901234567890121234567890123456789012345678901212345 6
123456789012345678901234567890121234567890123456789012345678901212345 66
2 3456789012345678901234567890121234567890123456789012345678901212345
123456789012345678901234567890121234567890123456789012345678901212345 6
1
12 3456789012345678901234567890121234567890123456789012345678901212345 6
12 3456789012345678901234567890121234567890123456789012345678901212345 6
123456789012345678901234567890121234567890123456789012345678901212345
2 6
123456789012345678901234567890121234567890123456789012345678901212345 6
1 3456789012345678901234567890121234567890123456789012345678901212345 6
123456789012345678901234567890121234567890123456789012345678901212345 6
123456789012345678901234567890121234567890123456789012345678901212345 6
12 3456789012345678901234567890121234567890123456789012345678901212345 6
12 3456789012345678901234567890121234567890123456789012345678901212345
19. 3 2 2x . 5 6
12 3456789012345678901234567890121234567890123456789012345678901212345 6
12 3456789012345678901234567890121234567890123456789012345678901212345 6
123456789012345678901234567890121234567890123456789012345678901212345 6
x2
2
123456789012345678901234567890121234567890123456789012345678901212345 66
2 3456789012345678901234567890121234567890123456789012345678901212345
123456789012345678901234567890121234567890123456789012345678901212345 6
123456789012345678901234567890121234567890123456789012345678901212345 6
123456789012345678901234567890121234567890123456789012345678901212345 6
1
12 3456789012345678901234567890121234567890123456789012345678901212345 6
12 3456789012345678901234567890121234567890123456789012345678901212345 6
12 3456789012345678901234567890121234567890123456789012345678901212345 6
12 3456789012345678901234567890121234567890123456789012345678901212345 6
123456789012345678901234567890121234567890123456789012345678901212345 6
123456789012345678901234567890121234567890123456789012345678901212345
2 6
1 3456789012345678901234567890121234567890123456789012345678901212345 6
12 3456789012345678901234567890121234567890123456789012345678901212345 6
123456789012345678901234567890121234567890123456789012345678901212345 6
12 3456789012345678901234567890121234567890123456789012345678901212345 6
123456789012345678901234567890121234567890123456789012345678901212345 6
123456789012345678901234567890121234567890123456789012345678901212345 6
123456789012345678901234567890121234567890123456789012345678901212345 66
286 123456789012345678901234567890121234567890123456789012345678901212345
1234567890123456789012345678901212345678901234567890123456789012123456

www.petersons.com
TEST 2
1234567890123456789012345678901212345678901234567890123456789012123456
123456789012345678901234567890121234567890123456789012345678901212345 6
12 3456789012345678901234567890121234567890123456789012345678901212345 6
123456789012345678901234567890121234567890123456789012345678901212345
Column A Column B Column A Column B 6
12 3456789012345678901234567890121234567890123456789012345678901212345 6
12 3456789012345678901234567890121234567890123456789012345678901212345 6
123456789012345678901234567890121234567890123456789012345678901212345
23. The ratio of a to b is three fourths. 26. 2 2
x 2 4y 1 7 5 7 6
12
2
3456789012345678901234567890121234567890123456789012345678901212345 6
3456789012345678901234567890121234567890123456789012345678901212345 6
1
12 3456789012345678901234567890121234567890123456789012345678901212345 6
12 3456789012345678901234567890121234567890123456789012345678901212345 6
12 3456789012345678901234567890121234567890123456789012345678901212345 6
12 3456789012345678901234567890121234567890123456789012345678901212345 6
1 3456789012345678901234567890121234567890123456789012345678901212345 6
2
12 3456789012345678901234567890121234567890123456789012345678901212345 6
12 3456789012345678901234567890121234567890123456789012345678901212345 6
12 3456789012345678901234567890121234567890123456789012345678901212345 6
12 3456789012345678901234567890121234567890123456789012345678901212345 6
1 3456789012345678901234567890121234567890123456789012345678901212345 6
2
12 3456789012345678901234567890121234567890123456789012345678901212345 6
12 3456789012345678901234567890121234567890123456789012345678901212345 6
12 3456789012345678901234567890121234567890123456789012345678901212345 6
12
2
3456789012345678901234567890121234567890123456789012345678901212345
a˚ = b˚
3456789012345678901234567890121234567890123456789012345678901212345
6
6
1
12 3456789012345678901234567890121234567890123456789012345678901212345 6
12 3456789012345678901234567890121234567890123456789012345678901212345 6
12 3456789012345678901234567890121234567890123456789012345678901212345 6
12 3456789012345678901234567890121234567890123456789012345678901212345 6
123456789012345678901234567890121234567890123456789012345678901212345
2 6
123456789012345678901234567890121234567890123456789012345678901212345 6
123456789012345678901234567890121234567890123456789012345678901212345 6
123456789012345678901234567890121234567890123456789012345678901212345
The area of Seventy-five c 2
1 a 2
a 2
1 b 2
6
123456789012345678901234567890121234567890123456789012345678901212345 6
3456789012345678901234567890121234567890123456789012345678901212345
123456789012345678901234567890121234567890123456789012345678901212345
sector 1 percent of sector 2 6
1 6
12 3456789012345678901234567890121234567890123456789012345678901212345 6
12 3456789012345678901234567890121234567890123456789012345678901212345 6
12 3456789012345678901234567890121234567890123456789012345678901212345 6
123456789012345678901234567890121234567890123456789012345678901212345
2 6
123456789012345678901234567890121234567890123456789012345678901212345 6
123456789012345678901234567890121234567890123456789012345678901212345 6
123456789012345678901234567890121234567890123456789012345678901212345 6
123456789012345678901234567890121234567890123456789012345678901212345 6
3456789012345678901234567890121234567890123456789012345678901212345
123456789012345678901234567890121234567890123456789012345678901212345 6
123456789012345678901234567890121234567890123456789012345678901212345
27. C and D are integers, and C . D. 6
123456789012345678901234567890121234567890123456789012345678901212345 66
123456789012345678901234567890121234567890123456789012345678901212345
1 24. The greatest The least common The number of C2 2 CD 1 2C 6
2 3456789012345678901234567890121234567890123456789012345678901212345
123456789012345678901234567890121234567890123456789012345678901212345
common factor of multiple of 8 and 6 6
123456789012345678901234567890121234567890123456789012345678901212345
integers from D to 4C 2 3C 6
1 24 and 120 6
123456789012345678901234567890121234567890123456789012345678901212345 6
C inclusive
12 3456789012345678901234567890121234567890123456789012345678901212345 6
12 3456789012345678901234567890121234567890123456789012345678901212345 6
12 3456789012345678901234567890121234567890123456789012345678901212345 6
123456789012345678901234567890121234567890123456789012345678901212345 6
12 3456789012345678901234567890121234567890123456789012345678901212345 6
12 3456789012345678901234567890121234567890123456789012345678901212345 6
123456789012345678901234567890121234567890123456789012345678901212345
2 6
123456789012345678901234567890121234567890123456789012345678901212345 6
1 3456789012345678901234567890121234567890123456789012345678901212345 6
123456789012345678901234567890121234567890123456789012345678901212345 6
123456789012345678901234567890121234567890123456789012345678901212345 6
12 3456789012345678901234567890121234567890123456789012345678901212345
25. m and n are integers. 6
12 3456789012345678901234567890121234567890123456789012345678901212345
28. {x, y, x 1 4, z, y 1 3} 6
12 3456789012345678901234567890121234567890123456789012345678901212345 6
12 3456789012345678901234567890121234567890123456789012345678901212345
(m 1 n)(m 1 n) (m 1 2n)(m 2 n) This number set is ordered from least to 6
123456789012345678901234567890121234567890123456789012345678901212345 6
123456789012345678901234567890121234567890123456789012345678901212345 66
2 3456789012345678901234567890121234567890123456789012345678901212345
greatest, and x, y, and z are distinct
123456789012345678901234567890121234567890123456789012345678901212345 6
123456789012345678901234567890121234567890123456789012345678901212345 integers.
123456789012345678901234567890121234567890123456789012345678901212345 66
1
12 3456789012345678901234567890121234567890123456789012345678901212345
z25 x
6
12 3456789012345678901234567890121234567890123456789012345678901212345 6
12 3456789012345678901234567890121234567890123456789012345678901212345 6
12 3456789012345678901234567890121234567890123456789012345678901212345 6
123456789012345678901234567890121234567890123456789012345678901212345 6
123456789012345678901234567890121234567890123456789012345678901212345
2 6
1 3456789012345678901234567890121234567890123456789012345678901212345 6
12 3456789012345678901234567890121234567890123456789012345678901212345 6
123456789012345678901234567890121234567890123456789012345678901212345 6
12 3456789012345678901234567890121234567890123456789012345678901212345 6
123456789012345678901234567890121234567890123456789012345678901212345 6
123456789012345678901234567890121234567890123456789012345678901212345 6
123456789012345678901234567890121234567890123456789012345678901212345 6
123456789012345678901234567890121234567890123456789012345678901212345 6 287
1234567890123456789012345678901212345678901234567890123456789012123456
Part IV: Three Practice GREs

1234567890123456789012345678901212345678901234567890123456789012123456
123456789012345678901234567890121234567890123456789012345678901212345 6
12 3456789012345678901234567890121234567890123456789012345678901212345 6
123456789012345678901234567890121234567890123456789012345678901212345
Verbal 6
12 3456789012345678901234567890121234567890123456789012345678901212345 6
12 3456789012345678901234567890121234567890123456789012345678901212345 6
123456789012345678901234567890121234567890123456789012345678901212345
30 Questions—30 Minutes 6
12
2
3456789012345678901234567890121234567890123456789012345678901212345
3456789012345678901234567890121234567890123456789012345678901212345
6
6
1
12 3456789012345678901234567890121234567890123456789012345678901212345
Antonyms 6
12 3456789012345678901234567890121234567890123456789012345678901212345 6
12 3456789012345678901234567890121234567890123456789012345678901212345 6
12 3456789012345678901234567890121234567890123456789012345678901212345 6
1 3456789012345678901234567890121234567890123456789012345678901212345
2 Directions: Each question contains a word printed in capital letters, followed by five 6
12 3456789012345678901234567890121234567890123456789012345678901212345 6
12 3456789012345678901234567890121234567890123456789012345678901212345
answer choices. Choose the answer choice that contains the word or phrase most nearly 6
12 3456789012345678901234567890121234567890123456789012345678901212345 6
12 3456789012345678901234567890121234567890123456789012345678901212345
OPPOSITE in meaning to the word in capital letters. 6
123456789012345678901234567890121234567890123456789012345678901212345 6
12 3456789012345678901234567890121234567890123456789012345678901212345 6
12 3456789012345678901234567890121234567890123456789012345678901212345 6
12 3456789012345678901234567890121234567890123456789012345678901212345 6
12 3456789012345678901234567890121234567890123456789012345678901212345 6
1 3456789012345678901234567890121234567890123456789012345678901212345
2 1. PRAGMATIC : 5. GARRULOUS : 6
12 3456789012345678901234567890121234567890123456789012345678901212345 6
12 3456789012345678901234567890121234567890123456789012345678901212345 6
12 3456789012345678901234567890121234567890123456789012345678901212345
A. reticent A. trepidatious 6
12 3456789012345678901234567890121234567890123456789012345678901212345 6
123456789012345678901234567890121234567890123456789012345678901212345 66
2 3456789012345678901234567890121234567890123456789012345678901212345
B. quixotic B. decorious
123456789012345678901234567890121234567890123456789012345678901212345
123456789012345678901234567890121234567890123456789012345678901212345
C. lavish C. irksome 6
123456789012345678901234567890121234567890123456789012345678901212345 6
123456789012345678901234567890121234567890123456789012345678901212345 66
D. practical D. gregarious
123456789012345678901234567890121234567890123456789012345678901212345 6
1 E. hopeful E. taciturn
12 3456789012345678901234567890121234567890123456789012345678901212345 6
12 3456789012345678901234567890121234567890123456789012345678901212345 6
12 3456789012345678901234567890121234567890123456789012345678901212345
2. COGENT : 6. ENERVATE : 6
123456789012345678901234567890121234567890123456789012345678901212345
2 6
123456789012345678901234567890121234567890123456789012345678901212345 6
123456789012345678901234567890121234567890123456789012345678901212345
A. unintelligent A. disabuse 6
123456789012345678901234567890121234567890123456789012345678901212345 6
123456789012345678901234567890121234567890123456789012345678901212345
B. culpable B. strengthen 6
3456789012345678901234567890121234567890123456789012345678901212345
123456789012345678901234567890121234567890123456789012345678901212345 6
123456789012345678901234567890121234567890123456789012345678901212345
C. thoughtful C. maximize 6
123456789012345678901234567890121234567890123456789012345678901212345 6
123456789012345678901234567890121234567890123456789012345678901212345
D. unconvincing D. maltreat 6
1 6
123456789012345678901234567890121234567890123456789012345678901212345 66
2 3456789012345678901234567890121234567890123456789012345678901212345
E. mysterious E. articulate
123456789012345678901234567890121234567890123456789012345678901212345 6
1
123456789012345678901234567890121234567890123456789012345678901212345 6
12 3456789012345678901234567890121234567890123456789012345678901212345
3. OBDURATE : 7. MANUMIT : 6
123456789012345678901234567890121234567890123456789012345678901212345 66
2 3456789012345678901234567890121234567890123456789012345678901212345
123456789012345678901234567890121234567890123456789012345678901212345
A. persuadable A. enslave
6
1
12 3456789012345678901234567890121234567890123456789012345678901212345
B. gullible B. indemnify 6
12 3456789012345678901234567890121234567890123456789012345678901212345 6
2 3456789012345678901234567890121234567890123456789012345678901212345
123456789012345678901234567890121234567890123456789012345678901212345
C. litigious C. ostracize 6
123456789012345678901234567890121234567890123456789012345678901212345 6
1 D. manageable D. simplify 6
123456789012345678901234567890121234567890123456789012345678901212345 6
123456789012345678901234567890121234567890123456789012345678901212345 6
E. stubborn E. delete
123456789012345678901234567890121234567890123456789012345678901212345 66
2 3456789012345678901234567890121234567890123456789012345678901212345
123456789012345678901234567890121234567890123456789012345678901212345 6
123456789012345678901234567890121234567890123456789012345678901212345 6
123456789012345678901234567890121234567890123456789012345678901212345
4. EBULLIENT :
6
1
2 3456789012345678901234567890121234567890123456789012345678901212345
123456789012345678901234567890121234567890123456789012345678901212345
A. stoic 6
123456789012345678901234567890121234567890123456789012345678901212345 6
123456789012345678901234567890121234567890123456789012345678901212345
B. stationary 6
123456789012345678901234567890121234567890123456789012345678901212345 66
1 C. stern
12 3456789012345678901234567890121234567890123456789012345678901212345 6
12 3456789012345678901234567890121234567890123456789012345678901212345
D. turgid 6
12 3456789012345678901234567890121234567890123456789012345678901212345 6
12 3456789012345678901234567890121234567890123456789012345678901212345
E. static 6
123456789012345678901234567890121234567890123456789012345678901212345 6
123456789012345678901234567890121234567890123456789012345678901212345 66
2 3456789012345678901234567890121234567890123456789012345678901212345
1
12 3456789012345678901234567890121234567890123456789012345678901212345 6
123456789012345678901234567890121234567890123456789012345678901212345 6
12 3456789012345678901234567890121234567890123456789012345678901212345 6
123456789012345678901234567890121234567890123456789012345678901212345 6
123456789012345678901234567890121234567890123456789012345678901212345 6
123456789012345678901234567890121234567890123456789012345678901212345 66
288 123456789012345678901234567890121234567890123456789012345678901212345
1234567890123456789012345678901212345678901234567890123456789012123456

www.petersons.com
TEST 2
1234567890123456789012345678901212345678901234567890123456789012123456
123456789012345678901234567890121234567890123456789012345678901212345 6
12 3456789012345678901234567890121234567890123456789012345678901212345 6
123456789012345678901234567890121234567890123456789012345678901212345
8. ADUMBRATE : 9. PANEGYRIC : 6
12 3456789012345678901234567890121234567890123456789012345678901212345 6
12 3456789012345678901234567890121234567890123456789012345678901212345 6
123456789012345678901234567890121234567890123456789012345678901212345
A. to cease A. assertion 6
12
2
3456789012345678901234567890121234567890123456789012345678901212345
3456789012345678901234567890121234567890123456789012345678901212345
6
6
1 B. to blandish B. elegy
12 3456789012345678901234567890121234567890123456789012345678901212345 6
12 3456789012345678901234567890121234567890123456789012345678901212345 6
C. to foreshadow C. dirge
12 3456789012345678901234567890121234567890123456789012345678901212345 6
12
2
3456789012345678901234567890121234567890123456789012345678901212345
D. to make explicit D. dissertation
3456789012345678901234567890121234567890123456789012345678901212345
6
6
1
12 3456789012345678901234567890121234567890123456789012345678901212345
E. to loosen E. obloquy 6
12 3456789012345678901234567890121234567890123456789012345678901212345 6
12 3456789012345678901234567890121234567890123456789012345678901212345 6
12 3456789012345678901234567890121234567890123456789012345678901212345 6
1 3456789012345678901234567890121234567890123456789012345678901212345
2 Analogies 6
12 3456789012345678901234567890121234567890123456789012345678901212345 6
12 3456789012345678901234567890121234567890123456789012345678901212345 6
12 3456789012345678901234567890121234567890123456789012345678901212345 6
12
2
3456789012345678901234567890121234567890123456789012345678901212345
Directions: In each question, there is an initial pair of words or phrases that are related in
3456789012345678901234567890121234567890123456789012345678901212345
6
6
1
12 3456789012345678901234567890121234567890123456789012345678901212345
some manner. Select the answer choice containing the lettered pair that best expresses a 6
12 3456789012345678901234567890121234567890123456789012345678901212345 6
12 3456789012345678901234567890121234567890123456789012345678901212345
relationship similar to the initial pair. 6
12 3456789012345678901234567890121234567890123456789012345678901212345 6
123456789012345678901234567890121234567890123456789012345678901212345 66
2 3456789012345678901234567890121234567890123456789012345678901212345
123456789012345678901234567890121234567890123456789012345678901212345 6
123456789012345678901234567890121234567890123456789012345678901212345 6
123456789012345678901234567890121234567890123456789012345678901212345
123456789012345678901234567890121234567890123456789012345678901212345
10. ECLIPSE : OVERSHADOW :: 14. HALCYON : PEACEFUL :: 6
123456789012345678901234567890121234567890123456789012345678901212345 66
1
12 3456789012345678901234567890121234567890123456789012345678901212345
A. set : rise A. tectonic : magnetic 6
12 3456789012345678901234567890121234567890123456789012345678901212345 6
12 3456789012345678901234567890121234567890123456789012345678901212345 6
B. outstrip : flail B. tempestuous : placid
2 3456789012345678901234567890121234567890123456789012345678901212345
123456789012345678901234567890121234567890123456789012345678901212345 6
123456789012345678901234567890121234567890123456789012345678901212345
C. debauch : debase C. surreptitious : clandestine 6
123456789012345678901234567890121234567890123456789012345678901212345 6
123456789012345678901234567890121234567890123456789012345678901212345 66
D. overcome : outstretch D. boisterous : rueful
123456789012345678901234567890121234567890123456789012345678901212345
123456789012345678901234567890121234567890123456789012345678901212345
E. pervert : correct E. terrifying : relaxing 6
123456789012345678901234567890121234567890123456789012345678901212345 66
123456789012345678901234567890121234567890123456789012345678901212345
123456789012345678901234567890121234567890123456789012345678901212345
11. EMPHATIC : INSIST :: 15. DESOLATE : VERDANT :: 6
1 6
2 3456789012345678901234567890121234567890123456789012345678901212345
123456789012345678901234567890121234567890123456789012345678901212345 6
123456789012345678901234567890121234567890123456789012345678901212345
A. contentious : argue A. destitute : opulent 6
1 6
123456789012345678901234567890121234567890123456789012345678901212345 6
B. absorbed : engrossed B. rococo : ornate
12 3456789012345678901234567890121234567890123456789012345678901212345 6
12 3456789012345678901234567890121234567890123456789012345678901212345
C. raucous : mitigate C. astringent : severe 6
12 3456789012345678901234567890121234567890123456789012345678901212345 6
123456789012345678901234567890121234567890123456789012345678901212345 6
D. delusional : rage D. tropical : humid
12 3456789012345678901234567890121234567890123456789012345678901212345 6
12 3456789012345678901234567890121234567890123456789012345678901212345
E. intentional : forget E. technical : detailed 6
123456789012345678901234567890121234567890123456789012345678901212345 66
2 3456789012345678901234567890121234567890123456789012345678901212345
123456789012345678901234567890121234567890123456789012345678901212345 6
1 12. ASTRONOMER : STARS :: 16. HARBINGER : PRESAGE ::
123456789012345678901234567890121234567890123456789012345678901212345 6
123456789012345678901234567890121234567890123456789012345678901212345 6
123456789012345678901234567890121234567890123456789012345678901212345 66
2 3456789012345678901234567890121234567890123456789012345678901212345
A. account : financial records A. computer : calculates
123456789012345678901234567890121234567890123456789012345678901212345 6
123456789012345678901234567890121234567890123456789012345678901212345
B. librarian : books B. television : broadcasts
6
123456789012345678901234567890121234567890123456789012345678901212345 6
1 C. historian : past C. liaison : interrogates
2 3456789012345678901234567890121234567890123456789012345678901212345
123456789012345678901234567890121234567890123456789012345678901212345 6
123456789012345678901234567890121234567890123456789012345678901212345
D. police officer : laws D. messenger : relates 6
123456789012345678901234567890121234567890123456789012345678901212345 6
123456789012345678901234567890121234567890123456789012345678901212345 66
E. magician : tricks E. broker : absconds
1
12 3456789012345678901234567890121234567890123456789012345678901212345 6
12 3456789012345678901234567890121234567890123456789012345678901212345
13. INOCULATE : DISEASE :: 17. ASSEVERATE : AVER :: 6
12 3456789012345678901234567890121234567890123456789012345678901212345 6
12 3456789012345678901234567890121234567890123456789012345678901212345 6
123456789012345678901234567890121234567890123456789012345678901212345
A. defraud : conspirator A. beseech : request 6
12 3456789012345678901234567890121234567890123456789012345678901212345 6
123456789012345678901234567890121234567890123456789012345678901212345
B. detest : agitator B. reject : install 6
12 3456789012345678901234567890121234567890123456789012345678901212345 6
123456789012345678901234567890121234567890123456789012345678901212345
C. delight : friend C. answer : reply 6
12
2
3456789012345678901234567890121234567890123456789012345678901212345
3456789012345678901234567890121234567890123456789012345678901212345
6
6
1 D. defend : enemy D. enrage : mollify
123456789012345678901234567890121234567890123456789012345678901212345 6
123456789012345678901234567890121234567890123456789012345678901212345
E. defame : title E. approach : accost 6
123456789012345678901234567890121234567890123456789012345678901212345 6 289
1234567890123456789012345678901212345678901234567890123456789012123456
Part IV: Three Practice GREs

1234567890123456789012345678901212345678901234567890123456789012123456
123456789012345678901234567890121234567890123456789012345678901212345 6
12 3456789012345678901234567890121234567890123456789012345678901212345 6
123456789012345678901234567890121234567890123456789012345678901212345
Sentence Completions 6
12 3456789012345678901234567890121234567890123456789012345678901212345 6
12 3456789012345678901234567890121234567890123456789012345678901212345 6
123456789012345678901234567890121234567890123456789012345678901212345 6
12 3456789012345678901234567890121234567890123456789012345678901212345
Directions: Each sentence below contains one or two blanks, with each blank corre- 6
123456789012345678901234567890121234567890123456789012345678901212345 6
12 3456789012345678901234567890121234567890123456789012345678901212345
sponding to an omitted word. Find the answer choice that has the word or set or words 6
12 3456789012345678901234567890121234567890123456789012345678901212345 6
12 3456789012345678901234567890121234567890123456789012345678901212345
that best fits the meaning of the sentence as a whole. 6
12
2
3456789012345678901234567890121234567890123456789012345678901212345 6
3456789012345678901234567890121234567890123456789012345678901212345 6
1
12 3456789012345678901234567890121234567890123456789012345678901212345 6
12 3456789012345678901234567890121234567890123456789012345678901212345 6
12 3456789012345678901234567890121234567890123456789012345678901212345 6
12 3456789012345678901234567890121234567890123456789012345678901212345
18. Mr. Thomas had, in his local community, 21. The __________ feuds between rival 6
1 3456789012345678901234567890121234567890123456789012345678901212345
2 6
12 3456789012345678901234567890121234567890123456789012345678901212345
earned the reputation as being somewhat factions of the company’s board destroyed 6
12 3456789012345678901234567890121234567890123456789012345678901212345 6
12 3456789012345678901234567890121234567890123456789012345678901212345
of a miser, but a review of his financial the effectiveness of the company’s 6
12
2
3456789012345678901234567890121234567890123456789012345678901212345
3456789012345678901234567890121234567890123456789012345678901212345
6
6
1 records revealed he was actually a leadership.
12 3456789012345678901234567890121234567890123456789012345678901212345 6
12 3456789012345678901234567890121234567890123456789012345678901212345
__________. 6
12 3456789012345678901234567890121234567890123456789012345678901212345 6
A. fatalistic
12 3456789012345678901234567890121234567890123456789012345678901212345 6
12 3456789012345678901234567890121234567890123456789012345678901212345
A. maverick B. infernal 6
12 3456789012345678901234567890121234567890123456789012345678901212345 6
12 3456789012345678901234567890121234567890123456789012345678901212345 6
B. hooligan C. hegemonic
12 3456789012345678901234567890121234567890123456789012345678901212345 6
12 3456789012345678901234567890121234567890123456789012345678901212345
C. maestro D. internecine 6
2 3456789012345678901234567890121234567890123456789012345678901212345
123456789012345678901234567890121234567890123456789012345678901212345 6
1 D. spendthrift E. flagging 6
12 3456789012345678901234567890121234567890123456789012345678901212345
E. micro-manager 6
12 3456789012345678901234567890121234567890123456789012345678901212345 6
12 3456789012345678901234567890121234567890123456789012345678901212345
22. Unlike his most recent predecessor, the 6
2 3456789012345678901234567890121234567890123456789012345678901212345
123456789012345678901234567890121234567890123456789012345678901212345 6
123456789012345678901234567890121234567890123456789012345678901212345
19. Recent demographic shifts in the popula- newly elected mayor did not appoint 6
123456789012345678901234567890121234567890123456789012345678901212345
friends and associates to high posts in the 6
6
123456789012345678901234567890121234567890123456789012345678901212345
tion of Country X have resulted in many
123456789012345678901234567890121234567890123456789012345678901212345 6
123456789012345678901234567890121234567890123456789012345678901212345
traditional political alliances becoming city government, but eschewed such 6
__________ in favor of a __________-based 6
123456789012345678901234567890121234567890123456789012345678901212345
123456789012345678901234567890121234567890123456789012345678901212345
__________ as nascent political parties 6
123456789012345678901234567890121234567890123456789012345678901212345 6
1 __________ large swathes of the electoral appointment criteria. 6
2 3456789012345678901234567890121234567890123456789012345678901212345
123456789012345678901234567890121234567890123456789012345678901212345 6
123456789012345678901234567890121234567890123456789012345678901212345
landscape. 6
1 A. nepotism. .value 6
123456789012345678901234567890121234567890123456789012345678901212345 6
12
2
3456789012345678901234567890121234567890123456789012345678901212345 6
A. obsolete. .secure B. cronyism.
3456789012345678901234567890121234567890123456789012345678901212345
.merit
6
123456789012345678901234567890121234567890123456789012345678901212345
B. dislodged. .provide C. favoritism. .work
123456789012345678901234567890121234567890123456789012345678901212345 6
1 C. unhinged. .procure D. jingoism. .resumé 6
12 3456789012345678901234567890121234567890123456789012345678901212345 6
12 3456789012345678901234567890121234567890123456789012345678901212345
D. solvent. .release E. recidivism. .experience 6
12 3456789012345678901234567890121234567890123456789012345678901212345 6
12 3456789012345678901234567890121234567890123456789012345678901212345
E. elusive. .gain 6
123456789012345678901234567890121234567890123456789012345678901212345 6
123456789012345678901234567890121234567890123456789012345678901212345 6
23. Though customarily marked by deliberate
123456789012345678901234567890121234567890123456789012345678901212345 6
12 3456789012345678901234567890121234567890123456789012345678901212345
20. Many see the attempt to sway public and __________ prose, the author’s most 6
12 3456789012345678901234567890121234567890123456789012345678901212345 6
12 3456789012345678901234567890121234567890123456789012345678901212345 6
__________, though ostensibly a process recent correspondence was convoluted
12 3456789012345678901234567890121234567890123456789012345678901212345 6
123456789012345678901234567890121234567890123456789012345678901212345
of popular persuasion, to be __________ and __________. 6
2 3456789012345678901234567890121234567890123456789012345678901212345
123456789012345678901234567890121234567890123456789012345678901212345 6
123456789012345678901234567890121234567890123456789012345678901212345
since it has been undertaken employing 6
123456789012345678901234567890121234567890123456789012345678901212345
A. terse. .tiresome 6
123456789012345678901234567890121234567890123456789012345678901212345
questionable pretences. 6
1 B. genuine. .superfluous 6
2 3456789012345678901234567890121234567890123456789012345678901212345
123456789012345678901234567890121234567890123456789012345678901212345
A. consent. .disreputable C. acidic. .prosaic 6
123456789012345678901234567890121234567890123456789012345678901212345 6
123456789012345678901234567890121234567890123456789012345678901212345
B. opinion. .undemocratic D. urbane. .ubiquitous 6
123456789012345678901234567890121234567890123456789012345678901212345 66
1 C. disfavor. .impractical E. succinc. .prolix
12 3456789012345678901234567890121234567890123456789012345678901212345 6
123456789012345678901234567890121234567890123456789012345678901212345
D. ill-will. .treacherous 6
12 3456789012345678901234567890121234567890123456789012345678901212345 6
123456789012345678901234567890121234567890123456789012345678901212345
E. disinterest. .crude 6
12
2
3456789012345678901234567890121234567890123456789012345678901212345
3456789012345678901234567890121234567890123456789012345678901212345
6
6
1
123456789012345678901234567890121234567890123456789012345678901212345 6
123456789012345678901234567890121234567890123456789012345678901212345 66
290 123456789012345678901234567890121234567890123456789012345678901212345
1234567890123456789012345678901212345678901234567890123456789012123456

www.petersons.com
TEST 2
1234567890123456789012345678901212345678901234567890123456789012123456
123456789012345678901234567890121234567890123456789012345678901212345 6
12 3456789012345678901234567890121234567890123456789012345678901212345 6
123456789012345678901234567890121234567890123456789012345678901212345
Reading Comprehension 6
12 3456789012345678901234567890121234567890123456789012345678901212345 6
12 3456789012345678901234567890121234567890123456789012345678901212345 6
123456789012345678901234567890121234567890123456789012345678901212345 6
12 3456789012345678901234567890121234567890123456789012345678901212345
Directions: Questions follow each passage. After reading the passage, determine the best 6
123456789012345678901234567890121234567890123456789012345678901212345 6
12 3456789012345678901234567890121234567890123456789012345678901212345
answer for each question based on the passage’s content. 6
12 3456789012345678901234567890121234567890123456789012345678901212345 6
12 3456789012345678901234567890121234567890123456789012345678901212345 6
12 3456789012345678901234567890121234567890123456789012345678901212345 6
1 3456789012345678901234567890121234567890123456789012345678901212345 6
2
12 3456789012345678901234567890121234567890123456789012345678901212345 6
12 3456789012345678901234567890121234567890123456789012345678901212345
Passage 1 cellular life, it was believed, was a fixture 6
12 3456789012345678901234567890121234567890123456789012345678901212345 6
12
2
3456789012345678901234567890121234567890123456789012345678901212345
across all biological organisms. Archae-
3456789012345678901234567890121234567890123456789012345678901212345
6
6
1 Line The existence and novelty of the domain
12 3456789012345678901234567890121234567890123456789012345678901212345
ans, in recent decades, have repeatedly 6
12 3456789012345678901234567890121234567890123456789012345678901212345
Archaean has demanded many long- 6
12 3456789012345678901234567890121234567890123456789012345678901212345
demonstrated that the previously 6
12
2
3456789012345678901234567890121234567890123456789012345678901212345
standing hypotheses in biology to be
3456789012345678901234567890121234567890123456789012345678901212345
6
6
1 (40) maintained thermal threshold for life was
12 3456789012345678901234567890121234567890123456789012345678901212345
reconfigured. The most basic challenge 6
12 3456789012345678901234567890121234567890123456789012345678901212345
far too low. So-called extremophilic 6
12 3456789012345678901234567890121234567890123456789012345678901212345
(5) that the discovery of Archaeans presented 6
12 3456789012345678901234567890121234567890123456789012345678901212345
Archaeans have been discovered to thrive 6
2 3456789012345678901234567890121234567890123456789012345678901212345
123456789012345678901234567890121234567890123456789012345678901212345
to biologists was classification. Prior to 6
123456789012345678901234567890121234567890123456789012345678901212345
in temperatures as high as 160 degrees 6
123456789012345678901234567890121234567890123456789012345678901212345
the discovery of the existence of Archae- 6
123456789012345678901234567890121234567890123456789012345678901212345
centigrade. Such discoveries have 6
123456789012345678901234567890121234567890123456789012345678901212345
ans, life was classified into five king- 6
123456789012345678901234567890121234567890123456789012345678901212345
(45) required a broadening of biology’s 6
1 doms: Animalia, Plantae, Fungi, Proto- 6
12 3456789012345678901234567890121234567890123456789012345678901212345
zoa, and Bacteria. But Archaeans, being
conceptions concerning what environ- 6
12 3456789012345678901234567890121234567890123456789012345678901212345
(10)
6
12 3456789012345678901234567890121234567890123456789012345678901212345
such a biological novelty, did not readily
ments are hospitable to life. 6
123456789012345678901234567890121234567890123456789012345678901212345 66
2 3456789012345678901234567890121234567890123456789012345678901212345
123456789012345678901234567890121234567890123456789012345678901212345
submit to this system of classification. In
123456789012345678901234567890121234567890123456789012345678901212345 6
123456789012345678901234567890121234567890123456789012345678901212345
fact, an innovation of biological classifi- 24. This passage could best be titled 6
123456789012345678901234567890121234567890123456789012345678901212345 66
123456789012345678901234567890121234567890123456789012345678901212345
cation was needed to properly place A. “Biological Designations in Transfor- 6
123456789012345678901234567890121234567890123456789012345678901212345
123456789012345678901234567890121234567890123456789012345678901212345
(15) Archaeans in the panorama of life. 6
123456789012345678901234567890121234567890123456789012345678901212345 mation.” 6
1 Previously, kingdom had been the most 6
2 3456789012345678901234567890121234567890123456789012345678901212345
123456789012345678901234567890121234567890123456789012345678901212345
B. “The Archaean Diversity of Life.” 6
123456789012345678901234567890121234567890123456789012345678901212345
general biological designation, but to 6
1 C. “The Archaean Challenge to Biological 6
123456789012345678901234567890121234567890123456789012345678901212345
properly include the Archaeans a new 6
12
2
3456789012345678901234567890121234567890123456789012345678901212345
3456789012345678901234567890121234567890123456789012345678901212345
Paradigms.” 6
biological classification, the domain, was D. “Discovering Thermophilic Archaeans.” 6
123456789012345678901234567890121234567890123456789012345678901212345
123456789012345678901234567890121234567890123456789012345678901212345 6
1 (20) formulated. Now kingdoms are classified E. “Classifying Thermophiles.” 6
12 3456789012345678901234567890121234567890123456789012345678901212345 6
12 3456789012345678901234567890121234567890123456789012345678901212345 6
as a sub-group of a particular domain.
2 3456789012345678901234567890121234567890123456789012345678901212345
123456789012345678901234567890121234567890123456789012345678901212345 6
123456789012345678901234567890121234567890123456789012345678901212345
Thus, domain has become the most 6
1 6
123456789012345678901234567890121234567890123456789012345678901212345 6
general biological designation for life.
123456789012345678901234567890121234567890123456789012345678901212345 6
12 3456789012345678901234567890121234567890123456789012345678901212345
A second arena in which Archaeans 6
12 3456789012345678901234567890121234567890123456789012345678901212345 6
12 3456789012345678901234567890121234567890123456789012345678901212345 6
(25) have challenged accepted hypotheses and
12 3456789012345678901234567890121234567890123456789012345678901212345 6
123456789012345678901234567890121234567890123456789012345678901212345
broadened biological horizons is in the 6
2 3456789012345678901234567890121234567890123456789012345678901212345
123456789012345678901234567890121234567890123456789012345678901212345
realm of environmental extremes. It was 6
123456789012345678901234567890121234567890123456789012345678901212345 6
123456789012345678901234567890121234567890123456789012345678901212345
generally accepted not more than a few 6
123456789012345678901234567890121234567890123456789012345678901212345 66
1 decades ago that no life could exist in
12 3456789012345678901234567890121234567890123456789012345678901212345 6
12 3456789012345678901234567890121234567890123456789012345678901212345
(30) temperatures much hotter than 60 6
12 3456789012345678901234567890121234567890123456789012345678901212345 6
12 3456789012345678901234567890121234567890123456789012345678901212345
degrees centigrade. This limit was set 6
123456789012345678901234567890121234567890123456789012345678901212345 6
123456789012345678901234567890121234567890123456789012345678901212345 66
2 3456789012345678901234567890121234567890123456789012345678901212345
because it was thought that the molecular
1
12 3456789012345678901234567890121234567890123456789012345678901212345
integrity of vital cellular components 6
123456789012345678901234567890121234567890123456789012345678901212345 6
12
2
3456789012345678901234567890121234567890123456789012345678901212345 6
could not be maintained beyond such
3456789012345678901234567890121234567890123456789012345678901212345 6
1 (35) temperatures. The thermal capacity of
123456789012345678901234567890121234567890123456789012345678901212345 6
123456789012345678901234567890121234567890123456789012345678901212345 6
123456789012345678901234567890121234567890123456789012345678901212345 6 291
1234567890123456789012345678901212345678901234567890123456789012123456
Part IV: Three Practice GREs

1234567890123456789012345678901212345678901234567890123456789012123456
123456789012345678901234567890121234567890123456789012345678901212345 6
12 3456789012345678901234567890121234567890123456789012345678901212345 6
123456789012345678901234567890121234567890123456789012345678901212345
25. The passage implies that extremophilic Passage 2 6
12 3456789012345678901234567890121234567890123456789012345678901212345 6
12 3456789012345678901234567890121234567890123456789012345678901212345
Archaeans 6
123456789012345678901234567890121234567890123456789012345678901212345 6
Line Theoretical Economics faces considerable
12 3456789012345678901234567890121234567890123456789012345678901212345
challenges in developing reliable models 6
123456789012345678901234567890121234567890123456789012345678901212345
I. are able to maintain molecular 6
12 3456789012345678901234567890121234567890123456789012345678901212345 6
12 3456789012345678901234567890121234567890123456789012345678901212345 6
integrity of cellular components past for public choice scenarios. These
12 3456789012345678901234567890121234567890123456789012345678901212345
scenarios involve aggregating individual 6
12
2
3456789012345678901234567890121234567890123456789012345678901212345
the formerly accepted thermal
3456789012345678901234567890121234567890123456789012345678901212345
6
1
12 preferences into a public choice outcome. 6
3456789012345678901234567890121234567890123456789012345678901212345
threshold of life. (5) 6
12 3456789012345678901234567890121234567890123456789012345678901212345 6
12 3456789012345678901234567890121234567890123456789012345678901212345 6
II. have been known to exist in moderate An election in a democratic society is a
12 3456789012345678901234567890121234567890123456789012345678901212345 6
123456789012345678901234567890121234567890123456789012345678901212345
environments for some years, but well-considered example of a public 6
12 3456789012345678901234567890121234567890123456789012345678901212345 6
12 3456789012345678901234567890121234567890123456789012345678901212345 6
their extremophilic properties have choice scenario. Formulating helpful and
12 3456789012345678901234567890121234567890123456789012345678901212345
only recently been discovered. insightful models of political elections is 6
12
2
3456789012345678901234567890121234567890123456789012345678901212345
3456789012345678901234567890121234567890123456789012345678901212345
6
6
1 III. are able to live and thrive in tempera- a perpetual issue in politico-economic
12 3456789012345678901234567890121234567890123456789012345678901212345
(10)
6
12 3456789012345678901234567890121234567890123456789012345678901212345
tures as hot as 150 degrees centigrade. discourse. 6
12 3456789012345678901234567890121234567890123456789012345678901212345 6
12 3456789012345678901234567890121234567890123456789012345678901212345 Many public-choice election scenario 6
12 3456789012345678901234567890121234567890123456789012345678901212345
A. I only 6
12 3456789012345678901234567890121234567890123456789012345678901212345
models begin with the individual as the 6
12 3456789012345678901234567890121234567890123456789012345678901212345
B. II only 6
12 3456789012345678901234567890121234567890123456789012345678901212345
basic building block of the aggregate 6
12 3456789012345678901234567890121234567890123456789012345678901212345
C. III only 6
2 3456789012345678901234567890121234567890123456789012345678901212345
123456789012345678901234567890121234567890123456789012345678901212345
(15) outcome. On this methodology, the 6
1 D. I and II only 6
12 3456789012345678901234567890121234567890123456789012345678901212345
primary issue is determining a proper 6
12 3456789012345678901234567890121234567890123456789012345678901212345
E. I and III only 6
12 3456789012345678901234567890121234567890123456789012345678901212345 6
decision-matrix for the individual in the
2 3456789012345678901234567890121234567890123456789012345678901212345
123456789012345678901234567890121234567890123456789012345678901212345 6
123456789012345678901234567890121234567890123456789012345678901212345
society under consideration. The deci- 6
123456789012345678901234567890121234567890123456789012345678901212345
26. The passage states that a kingdom is 6
123456789012345678901234567890121234567890123456789012345678901212345 66
sion-matrix is a theoretical construct
123456789012345678901234567890121234567890123456789012345678901212345
A. a sub-group within a domain. which—when given the appropriate input 6
123456789012345678901234567890121234567890123456789012345678901212345
(20)
123456789012345678901234567890121234567890123456789012345678901212345
B. the most general biological designation. 6
123456789012345678901234567890121234567890123456789012345678901212345
for a particular situation—outputs the 6
123456789012345678901234567890121234567890123456789012345678901212345
C. composed of sub-groups of domains. 6
1 choice of a rational individual in the 6
2 3456789012345678901234567890121234567890123456789012345678901212345
123456789012345678901234567890121234567890123456789012345678901212345
D. a newly formulated biological 6
123456789012345678901234567890121234567890123456789012345678901212345
specified situation. The integrity of the 6
1 classification. 6
123456789012345678901234567890121234567890123456789012345678901212345
decision-matrix is the central concern of 6
12 3456789012345678901234567890121234567890123456789012345678901212345
E. an Archaean domain. 6
123456789012345678901234567890121234567890123456789012345678901212345 66
2 3456789012345678901234567890121234567890123456789012345678901212345
(25) the model-maker. If the decision-matrix
123456789012345678901234567890121234567890123456789012345678901212345
appropriately produces the choices of 6
1 27. It can be inferred from the passage that the
12 3456789012345678901234567890121234567890123456789012345678901212345
individuals in specific situations, then the 6
12 3456789012345678901234567890121234567890123456789012345678901212345 6
12 3456789012345678901234567890121234567890123456789012345678901212345
phrase “thermal threshold” in line 40 is
aggregate model will be useable. How- 6
12 3456789012345678901234567890121234567890123456789012345678901212345 6
123456789012345678901234567890121234567890123456789012345678901212345
A. the critical point temperature at which ever, if the decision-matrix inadequately 6
123456789012345678901234567890121234567890123456789012345678901212345 6
123456789012345678901234567890121234567890123456789012345678901212345
the metabolic pathways of extremo- (30) reflects the decision-making process of 6
12 3456789012345678901234567890121234567890123456789012345678901212345 6
12 3456789012345678901234567890121234567890123456789012345678901212345
philes become functional. individuals, the aggregate model will be 6
12 3456789012345678901234567890121234567890123456789012345678901212345 6
12 3456789012345678901234567890121234567890123456789012345678901212345
B. the temperature at which the molecu- unreliable. 6
123456789012345678901234567890121234567890123456789012345678901212345 6
123456789012345678901234567890121234567890123456789012345678901212345 66
2 3456789012345678901234567890121234567890123456789012345678901212345
lar integrity of cellular components of Decision-matrices are often con-
123456789012345678901234567890121234567890123456789012345678901212345
structed on the assumption that individu- 6
123456789012345678901234567890121234567890123456789012345678901212345
an organism are compromised.
6
123456789012345678901234567890121234567890123456789012345678901212345 6
1 C. the thermal capacity of extremophiles. (35) als are rational in decision-making and
2 3456789012345678901234567890121234567890123456789012345678901212345
123456789012345678901234567890121234567890123456789012345678901212345 6
123456789012345678901234567890121234567890123456789012345678901212345
D. an environmental extremity indicator. that they predicate decisions upon 6
123456789012345678901234567890121234567890123456789012345678901212345 6
123456789012345678901234567890121234567890123456789012345678901212345 66
E. a biological constant across all of self-interest. Furthermore, self-interest is
1
12 3456789012345678901234567890121234567890123456789012345678901212345
nature. generally construed to mean selfish 6
123456789012345678901234567890121234567890123456789012345678901212345 6
12 3456789012345678901234567890121234567890123456789012345678901212345 6
interests. A serious flaw in such a
123456789012345678901234567890121234567890123456789012345678901212345 6
12
2
3456789012345678901234567890121234567890123456789012345678901212345
(40) constructional is its disallowal of
3456789012345678901234567890121234567890123456789012345678901212345
6
6
1
123456789012345678901234567890121234567890123456789012345678901212345
altruistic motivation in the decision- 6
123456789012345678901234567890121234567890123456789012345678901212345 66
292 123456789012345678901234567890121234567890123456789012345678901212345
1234567890123456789012345678901212345678901234567890123456789012123456

www.petersons.com
TEST 2
1234567890123456789012345678901212345678901234567890123456789012123456
123456789012345678901234567890121234567890123456789012345678901212345 6
12 3456789012345678901234567890121234567890123456789012345678901212345 6
123456789012345678901234567890121234567890123456789012345678901212345
matrix. It can easily be empirically 29. What is a common assumption of 6
12 3456789012345678901234567890121234567890123456789012345678901212345 6
12 3456789012345678901234567890121234567890123456789012345678901212345
verified that human choice is not decision-matrix construction that the 6
123456789012345678901234567890121234567890123456789012345678901212345 6
12
2
3456789012345678901234567890121234567890123456789012345678901212345 6
predominated by altruism, but such author does
3456789012345678901234567890121234567890123456789012345678901212345
not dispute?
6
1 (45) verification does not warrant the
12 3456789012345678901234567890121234567890123456789012345678901212345 6
12 3456789012345678901234567890121234567890123456789012345678901212345
A. Self-interest should be construed as 6
12 3456789012345678901234567890121234567890123456789012345678901212345
complete exclusion of altruism from 6
12
2
3456789012345678901234567890121234567890123456789012345678901212345
selfish
3456789012345678901234567890121234567890123456789012345678901212345
interest. 6
6
1 decision-matrices. Therefore, altruistic
12 3456789012345678901234567890121234567890123456789012345678901212345
B. Altruism is a basic motivator of 6
12 3456789012345678901234567890121234567890123456789012345678901212345
motivators should be possible inputs for 6
12 3456789012345678901234567890121234567890123456789012345678901212345
human preference. 6
12
2
3456789012345678901234567890121234567890123456789012345678901212345
any decision-matrix schema that attempts
3456789012345678901234567890121234567890123456789012345678901212345
6
6
1 (50) to model human preference C. Individuals are rational in decision-
12 3456789012345678901234567890121234567890123456789012345678901212345 6
12 3456789012345678901234567890121234567890123456789012345678901212345
making. 6
12 3456789012345678901234567890121234567890123456789012345678901212345
determination.
D. Altruism is an irrational predicate. 6
12
2
3456789012345678901234567890121234567890123456789012345678901212345 6
3456789012345678901234567890121234567890123456789012345678901212345
1 E. Inputs should be expanded to include 6
12 3456789012345678901234567890121234567890123456789012345678901212345 6
12 3456789012345678901234567890121234567890123456789012345678901212345
self-interest motivators. 6
12 3456789012345678901234567890121234567890123456789012345678901212345 6
12 3456789012345678901234567890121234567890123456789012345678901212345 6
12 3456789012345678901234567890121234567890123456789012345678901212345
28. The intent of the author in writing this 6
12 3456789012345678901234567890121234567890123456789012345678901212345
30. The second paragraph in the passage 6
12 3456789012345678901234567890121234567890123456789012345678901212345 6
passage is to
12 3456789012345678901234567890121234567890123456789012345678901212345
A. elaborates on the issues introduced in 6
12 3456789012345678901234567890121234567890123456789012345678901212345
A. offer a radical critique of contempo- 6
2 3456789012345678901234567890121234567890123456789012345678901212345
123456789012345678901234567890121234567890123456789012345678901212345 6
1 rary Theoretical Economics. the first paragraph. 6
12 3456789012345678901234567890121234567890123456789012345678901212345 6
12 3456789012345678901234567890121234567890123456789012345678901212345
B. explain the basic mechanisms of B. presents views in contrast to those 6
12 3456789012345678901234567890121234567890123456789012345678901212345 6
123456789012345678901234567890121234567890123456789012345678901212345 66
2 3456789012345678901234567890121234567890123456789012345678901212345
modeling public choice scenarios. introduced in the first paragraph.
123456789012345678901234567890121234567890123456789012345678901212345
123456789012345678901234567890121234567890123456789012345678901212345
C. contrast two competing views on C. presents the thesis of the passage. 6
D. argues for the validity of the thesis of 6
123456789012345678901234567890121234567890123456789012345678901212345
123456789012345678901234567890121234567890123456789012345678901212345 66
decision-matrix construction.
123456789012345678901234567890121234567890123456789012345678901212345
the passage.
123456789012345678901234567890121234567890123456789012345678901212345
D. explain why in public choice scenarios 6
123456789012345678901234567890121234567890123456789012345678901212345
E. describes the intellectual issues 6
123456789012345678901234567890121234567890123456789012345678901212345 66
decision-matrices should include
1 pertaining to the thesis of the passage.
12 3456789012345678901234567890121234567890123456789012345678901212345
altruistic motivators. 6
12 3456789012345678901234567890121234567890123456789012345678901212345 6
123456789012345678901234567890121234567890123456789012345678901212345 6
E. criticize the assumption that society is
123456789012345678901234567890121234567890123456789012345678901212345 6
12
2
3456789012345678901234567890121234567890123456789012345678901212345
composed of aggregates of rational
3456789012345678901234567890121234567890123456789012345678901212345
6
6
123456789012345678901234567890121234567890123456789012345678901212345
123456789012345678901234567890121234567890123456789012345678901212345
decision-makers. 6
1 6
12 3456789012345678901234567890121234567890123456789012345678901212345 6
12 3456789012345678901234567890121234567890123456789012345678901212345 6
12 3456789012345678901234567890121234567890123456789012345678901212345 6
12 3456789012345678901234567890121234567890123456789012345678901212345 6
123456789012345678901234567890121234567890123456789012345678901212345 6
123456789012345678901234567890121234567890123456789012345678901212345 6
123456789012345678901234567890121234567890123456789012345678901212345 6
12 3456789012345678901234567890121234567890123456789012345678901212345 6
12 3456789012345678901234567890121234567890123456789012345678901212345 6
12 3456789012345678901234567890121234567890123456789012345678901212345 6
12 3456789012345678901234567890121234567890123456789012345678901212345 6
123456789012345678901234567890121234567890123456789012345678901212345 6
123456789012345678901234567890121234567890123456789012345678901212345
2 6
123456789012345678901234567890121234567890123456789012345678901212345 6
123456789012345678901234567890121234567890123456789012345678901212345 6
123456789012345678901234567890121234567890123456789012345678901212345 6
1 3456789012345678901234567890121234567890123456789012345678901212345 6
123456789012345678901234567890121234567890123456789012345678901212345 66
2 3456789012345678901234567890121234567890123456789012345678901212345
123456789012345678901234567890121234567890123456789012345678901212345 6
123456789012345678901234567890121234567890123456789012345678901212345 6
123456789012345678901234567890121234567890123456789012345678901212345 6
1
12 3456789012345678901234567890121234567890123456789012345678901212345 6
123456789012345678901234567890121234567890123456789012345678901212345 6
12 3456789012345678901234567890121234567890123456789012345678901212345 6
123456789012345678901234567890121234567890123456789012345678901212345 6
12 3456789012345678901234567890121234567890123456789012345678901212345 6
123456789012345678901234567890121234567890123456789012345678901212345 6
123456789012345678901234567890121234567890123456789012345678901212345 6
123456789012345678901234567890121234567890123456789012345678901212345 6
123456789012345678901234567890121234567890123456789012345678901212345 6 293
1234567890123456789012345678901212345678901234567890123456789012123456
Answers and Explanations

123456789012345678901234567890121234567890123456789012
12345678901234567890123456789012123456789012345678901 2
12345678901234567890123456789012123456789012345678901
2345678901234567890123456789012123456789012345678901
2
2
1 As before, the following notes are intended to show you one way in which
12345678901234567890123456789012123456789012345678901 2
12345678901234567890123456789012123456789012345678901
the correct answer can be determined. Try to concern yourself with the 2
12345678901234567890123456789012123456789012345678901 2
12345678901234567890123456789012123456789012345678901
methods used to arrive at the right answer, and not focus all your attention 2
2345678901234567890123456789012123456789012345678901
12345678901234567890123456789012123456789012345678901 2
12345678901234567890123456789012123456789012345678901
on whether you got the question right or wrong. 2
12345678901234567890123456789012123456789012345678901 2
12345678901234567890123456789012123456789012345678901 2
12345678901234567890123456789012123456789012345678901 2
12345678901234567890123456789012123456789012345678901 2 2
1Quantitative
12345678901234567890123456789012123456789012345678901 2
12345678901234567890123456789012123456789012345678901 2
12345678901234567890123456789012123456789012345678901
1. B The area of a parallelogram is base times height (like with a 2
2345678901234567890123456789012123456789012345678901
12345678901234567890123456789012123456789012345678901
triangle). The obvious base to choose is ED and then attempt to 2
2
12345678901234567890123456789012123456789012345678901
12345678901234567890123456789012123456789012345678901
find its height, or AE. The triangle is a 45-45-90 triangle, and 2
2
12345678901234567890123456789012123456789012345678901
12345678901234567890123456789012123456789012345678901 2
since the hypotenuse is 2=2, the other two sides are 2. So the 2
12345678901234567890123456789012123456789012345678901
12345678901234567890123456789012123456789012345678901 2
12345678901234567890123456789012123456789012345678901
height is 2 and the base is 5, which makes the area 10, or (B). 2
12345678901234567890123456789012123456789012345678901 2
1 2. B Here, you just need to be careful and work through the problem 2
12345678901234567890123456789012123456789012345678901 2
12345678901234567890123456789012123456789012345678901 2
12345678901234567890123456789012123456789012345678901
step by step. 2
12345678901234567890123456789012123456789012345678901 2
12345678901234567890123456789012123456789012345678901 2
12345678901234567890123456789012123456789012345678901
&(2) 5 [2 2 (2 21)](2 1 2) 5 (2 2 1)(4) 5 (1)(4) 5 4, 2
12345678901234567890123456789012123456789012345678901 2
12345678901234567890123456789012123456789012345678901 2
12345678901234567890123456789012123456789012345678901
and now plug this result in: 2
12345678901234567890123456789012123456789012345678901 2
2345678901234567890123456789012123456789012345678901
12345678901234567890123456789012123456789012345678901 2
12345678901234567890123456789012123456789012345678901
&(&(2)) 5 &(4) 5 [4 2 (4 2 1)](4 1 2) 2
1 2
12345678901234567890123456789012123456789012345678901
5 (4 2 3)(6) 2
12345678901234567890123456789012123456789012345678901 2
2345678901234567890123456789012123456789012345678901
12345678901234567890123456789012123456789012345678901
5 (1)(6) 2
12345678901234567890123456789012123456789012345678901 2
12345678901234567890123456789012123456789012345678901 2
12345678901234567890123456789012123456789012345678901 2
5 6
2
1
12345678901234567890123456789012123456789012345678901
Choice (B) is the answer. 2
12345678901234567890123456789012123456789012345678901 2
12345678901234567890123456789012123456789012345678901 2
12345678901234567890123456789012123456789012345678901
3. C You can solve this one quickly if you realize that the stock price 2
12345678901234567890123456789012123456789012345678901 2
12345678901234567890123456789012123456789012345678901
increased and that it did so more than 10 percent. If you do not 2
12345678901234567890123456789012123456789012345678901 2
12345678901234567890123456789012123456789012345678901
see this right off the bat, you can always just work the equation 2
12345678901234567890123456789012123456789012345678901 2
12345678901234567890123456789012123456789012345678901
for percentage change to get the answer: 2
12345678901234567890123456789012123456789012345678901 2
2345678901234567890123456789012123456789012345678901
2
1
S D40
~ 100% !SD
12345678901234567890123456789012123456789012345678901
5
5
1
12345678901234567890123456789012123456789012345678901
8
~100% ! 5
12345678901234567890123456789012123456789012345678901
12.5% 2
2
2
12345678901234567890123456789012123456789012345678901 2
12345678901234567890123456789012123456789012345678901
The answer is (C). 2
12345678901234567890123456789012123456789012345678901 2
12345678901234567890123456789012123456789012345678901 2
123456789012345678901234567890121234567890123456789012
294
ANSWERS
123456789012345678901234567890121234567890123456789012
12345678901234567890123456789012123456789012345678901 2
12345678901234567890123456789012123456789012345678901 2
12345678901234567890123456789012123456789012345678901
4. B You need to have an equation that has only the variables a and c 2
12345678901234567890123456789012123456789012345678901 2
12345678901234567890123456789012123456789012345678901
in it. If you multiply both sides of 4a 1 3d 5 18 by two we get 2
12345678901234567890123456789012123456789012345678901 2
12345678901234567890123456789012123456789012345678901
8a 1 6d 5 36. You can subtract this from the other equation to 2
2345678901234567890123456789012123456789012345678901 2
1
12345678901234567890123456789012123456789012345678901
eliminate the d variable; (3c 1 6d 5 24) 2 (8a 1 6d 5 36) yields 2
12345678901234567890123456789012123456789012345678901 2
12345678901234567890123456789012123456789012345678901
3c 2 8a 5 212. Now you only need to solve for a: 2
12345678901234567890123456789012123456789012345678901
2345678901234567890123456789012123456789012345678901
2
2
1
12345678901234567890123456789012123456789012345678901 3 3 2
12345678901234567890123456789012123456789012345678901
3c 2 8a 5 212 → 8a 5 3c 1 12 → a 5 c 1 , which is (B). 2
12345678901234567890123456789012123456789012345678901 8 2 2
12345678901234567890123456789012123456789012345678901
2345678901234567890123456789012123456789012345678901
2
2
1 5. D The extinction rate is the solid line, and its highest value is its
12345678901234567890123456789012123456789012345678901 2
12345678901234567890123456789012123456789012345678901
highest peak. The highest peak is in the early Paleozoic era 2
12345678901234567890123456789012123456789012345678901 2
12345678901234567890123456789012123456789012345678901
approximately 560 million years ago.
2345678901234567890123456789012123456789012345678901
2
2
1
12345678901234567890123456789012123456789012345678901 2
12345678901234567890123456789012123456789012345678901
6. B The extinction rate and the number of families have the same 2
12345678901234567890123456789012123456789012345678901 2
12345678901234567890123456789012123456789012345678901
value when the two lines cross. You are looking, then, for the era 2
2345678901234567890123456789012123456789012345678901
in which the lines cross the most number of times. They do so 2
12345678901234567890123456789012123456789012345678901
12345678901234567890123456789012123456789012345678901 2
12345678901234567890123456789012123456789012345678901 2
12345678901234567890123456789012123456789012345678901 22
four times in the Paleozoic era, which is more than any other,
12345678901234567890123456789012123456789012345678901
12345678901234567890123456789012123456789012345678901
making (B) the answer. 2
1 2
12345678901234567890123456789012123456789012345678901
7. E You can eliminate the odd number right off the bat, because a 2
12345678901234567890123456789012123456789012345678901 2
12345678901234567890123456789012123456789012345678901
number has to be even in order to be divisible by 6. Now you 2
2345678901234567890123456789012123456789012345678901
12345678901234567890123456789012123456789012345678901 2
12345678901234567890123456789012123456789012345678901
just have to go through the remaining choices one by one: 2
12345678901234567890123456789012123456789012345678901 22
12345678901234567890123456789012123456789012345678901
12345678901234567890123456789012123456789012345678901
1,024: Digits sum to 7, which is not a multiple of 3. 2
12345678901234567890123456789012123456789012345678901 22
12345678901234567890123456789012123456789012345678901
1,506: Digits sum to 12, which is a multiple of 3 but not 9. 2
12345678901234567890123456789012123456789012345678901
12345678901234567890123456789012123456789012345678901 22
1
12345678901234567890123456789012123456789012345678901
1,632: Digits sum to 12, and so same as previous. 2
12345678901234567890123456789012123456789012345678901 2
12345678901234567890123456789012123456789012345678901
1,764: Digits sum to 18, which is a multiple of 3 and 9. For 2
12345678901234567890123456789012123456789012345678901 2
12345678901234567890123456789012123456789012345678901
6, the digits must sum to a multiple of 3 and the original 2
12345678901234567890123456789012123456789012345678901 2
12345678901234567890123456789012123456789012345678901
number must be even. 2
12345678901234567890123456789012123456789012345678901 2
12345678901234567890123456789012123456789012345678901 2
12345678901234567890123456789012123456789012345678901
10 1 10 1 15 1 7 1 12 1 18 72 2
2345678901234567890123456789012123456789012345678901
12345678901234567890123456789012123456789012345678901
8. D The mean is 5 5 12, and so 2
12345678901234567890123456789012123456789012345678901
6 6 2
1 2
12345678901234567890123456789012123456789012345678901 2
a 5 12. The mode is 10, and so b 5 10. As for the median, order
12345678901234567890123456789012123456789012345678901
the numbers least to greatest {7, 10, 10, 12, 15, 18}. Since you 2
12345678901234567890123456789012123456789012345678901 2
12345678901234567890123456789012123456789012345678901
have an even number of numbers, the median is the average of 2
12345678901234567890123456789012123456789012345678901 2
12345678901234567890123456789012123456789012345678901
the two middle numbers. Thus, the median is 11, so c 5 11. The 2
12345678901234567890123456789012123456789012345678901 2
12345678901234567890123456789012123456789012345678901
range is simply the greatest number minus the least, which 2
12345678901234567890123456789012123456789012345678901 2
12345678901234567890123456789012123456789012345678901
equals 11, so d 5 11. So, a . b , c 5 d, which is (D). 2
12345678901234567890123456789012123456789012345678901 2
12345678901234567890123456789012123456789012345678901 2
12345678901234567890123456789012123456789012345678901 22
2345678901234567890123456789012123456789012345678901
12345678901234567890123456789012123456789012345678901 2
12345678901234567890123456789012123456789012345678901 2
12345678901234567890123456789012123456789012345678901 2
1
12345678901234567890123456789012123456789012345678901 2
12345678901234567890123456789012123456789012345678901 2
12345678901234567890123456789012123456789012345678901 2
12345678901234567890123456789012123456789012345678901 2
12345678901234567890123456789012123456789012345678901 2
12345678901234567890123456789012123456789012345678901 2
12345678901234567890123456789012123456789012345678901 2
12345678901234567890123456789012123456789012345678901 2
12345678901234567890123456789012123456789012345678901 2 295
123456789012345678901234567890121234567890123456789012
Part IV: Three Practice GREs

123456789012345678901234567890121234567890123456789012
12345678901234567890123456789012123456789012345678901 2
12345678901234567890123456789012123456789012345678901 2
12345678901234567890123456789012123456789012345678901
9. C To begin with, you know that the sides of square B are equal to 2
12345678901234567890123456789012123456789012345678901 2
12345678901234567890123456789012123456789012345678901
4 since its area is 16. If you can find the length of the 2
12345678901234567890123456789012123456789012345678901 2
12345678901234567890123456789012123456789012345678901
non-hypotenuse sides of triangle A, then you could find the 2
2345678901234567890123456789012123456789012345678901 2
1
12345678901234567890123456789012123456789012345678901
answer. Since you know the hypotenuse you can use the 2
12345678901234567890123456789012123456789012345678901
Pythagorean theorem to find the other sides of the triangle; 2
12345678901234567890123456789012123456789012345678901 2
12345678901234567890123456789012123456789012345678901
2345678901234567890123456789012123456789012345678901
2
1 a 1 b 5 ~=72! → 2a 5 72 → a 5 36 → a 5 6. The 2
2 2 2 2 2
12345678901234567890123456789012123456789012345678901 2
12345678901234567890123456789012123456789012345678901
length of the triangle side that B and C share is 6, which means 2
12345678901234567890123456789012123456789012345678901 2
12345678901234567890123456789012123456789012345678901
that one of the sides of square C is 2 (since the B side is 4). 2
2345678901234567890123456789012123456789012345678901 2
1
12345678901234567890123456789012123456789012345678901
Therefore, the area of C is 4, or (C). 2
12345678901234567890123456789012123456789012345678901 2
12345678901234567890123456789012123456789012345678901 2
12345678901234567890123456789012123456789012345678901
10. B Eye the graph to see which of
2345678901234567890123456789012123456789012345678901
the four options changed little 2
1 from its 1970 average to its 1990 average. Looking closely, you 2
12345678901234567890123456789012123456789012345678901 2
12345678901234567890123456789012123456789012345678901 2
12345678901234567890123456789012123456789012345678901
can see that the average production of corn in County Y was the 2
12345678901234567890123456789012123456789012345678901 2
12345678901234567890123456789012123456789012345678901 2
2345678901234567890123456789012123456789012345678901
same in 1970 and 1990. Thus, the percentage change is 0. That
12345678901234567890123456789012123456789012345678901 2
12345678901234567890123456789012123456789012345678901
must be the smallest, so the answer is (B). 2
12345678901234567890123456789012123456789012345678901 2
11. C First note that the graph is given in terms of bushels per acre. 2
12345678901234567890123456789012123456789012345678901
12345678901234567890123456789012123456789012345678901 2
1 Reading the graph then, County Y in 1980 produced 15 bushels 2
12345678901234567890123456789012123456789012345678901 2
12345678901234567890123456789012123456789012345678901
of corn per acre. Thus, 10 acres on average would produce 150 2
12345678901234567890123456789012123456789012345678901 2
12345678901234567890123456789012123456789012345678901
bushels, so (C) is correct. 2
12345678901234567890123456789012123456789012345678901 2
12345678901234567890123456789012123456789012345678901 2
12345678901234567890123456789012123456789012345678901
12. D In 1980 County Y produced 15 bushels of corn per acre on 2
12345678901234567890123456789012123456789012345678901
average, and it produced 10 bushels of wheat on average. Thus, 2
12345678901234567890123456789012123456789012345678901 2
12345678901234567890123456789012123456789012345678901 2
12345678901234567890123456789012123456789012345678901
the ratio is 15:10, or 3:2, which is answer choice (D). 2
12345678901234567890123456789012123456789012345678901 2
12345678901234567890123456789012123456789012345678901 2
12345678901234567890123456789012123456789012345678901 2
2345678901234567890123456789012123456789012345678901
13. C It is best to write the possibilities out and count them up. In the
1 ones column the possibilities are: 2, 4, 6. In the tens, the 2
12345678901234567890123456789012123456789012345678901 2
12345678901234567890123456789012123456789012345678901 2
12345678901234567890123456789012123456789012345678901
possibilities are: 22, 24, 26, 42,
2345678901234567890123456789012123456789012345678901
44, 46, 62, 64, 66. That 2
exhausts the possibilities. You get 12 when you count them up, 2
12345678901234567890123456789012123456789012345678901
12345678901234567890123456789012123456789012345678901 2
1 so (C) is the answer. 2
12345678901234567890123456789012123456789012345678901 2
12345678901234567890123456789012123456789012345678901 2
12345678901234567890123456789012123456789012345678901 2
2345678901234567890123456789012123456789012345678901
14. C First, the sum of the interior angles of an n-sided polygon is
12345678901234567890123456789012123456789012345678901
(n 2 2)(180), which makes the sum of the interior angles of a 2
2
1
12345678901234567890123456789012123456789012345678901 2
12345678901234567890123456789012123456789012345678901
5-sided polygon 540 degrees. You can find the sum of a 1 d by 2
2345678901234567890123456789012123456789012345678901
12345678901234567890123456789012123456789012345678901
summing the two equations a 1 b 5 160 and d 2 b 5 30. The 2
2
12345678901234567890123456789012123456789012345678901
end result is a 1 d 5 190. The remaining angles must sum to 2
12345678901234567890123456789012123456789012345678901
12345678901234567890123456789012123456789012345678901 2
1 2
12345678901234567890123456789012123456789012345678901 2
2345678901234567890123456789012123456789012345678901
350 for the sum of the interior angles to be 540. The answer,
2
12345678901234567890123456789012123456789012345678901 2
12345678901234567890123456789012123456789012345678901
then, is (C).
12345678901234567890123456789012123456789012345678901 2
1 15. B The sum of a set of consecutive integers is the average times the 2
12345678901234567890123456789012123456789012345678901 2
12345678901234567890123456789012123456789012345678901 2
12345678901234567890123456789012123456789012345678901 55 1 15 2
12345678901234567890123456789012123456789012345678901
number of terms. The average is 5 35. The number of 2
12345678901234567890123456789012123456789012345678901 2 2
12345678901234567890123456789012123456789012345678901 2
2345678901234567890123456789012123456789012345678901
terms is (remember, it is inclusive) 55 2 15 1 1 5 41. Now you
2
1
12345678901234567890123456789012123456789012345678901
just have to multiply these together. Their product is 1,435, 2
12345678901234567890123456789012123456789012345678901 2
12345678901234567890123456789012123456789012345678901
or (B).
2345678901234567890123456789012123456789012345678901
2
2
1
12345678901234567890123456789012123456789012345678901 2
12345678901234567890123456789012123456789012345678901 2 2
296 12345678901234567890123456789012123456789012345678901
123456789012345678901234567890121234567890123456789012

www.petersons.com
ANSWERS
123456789012345678901234567890121234567890123456789012
12345678901234567890123456789012123456789012345678901 2
12345678901234567890123456789012123456789012345678901 2
12345678901234567890123456789012123456789012345678901
16. D The circumference of the circle is 2p ' 6.28. The perimeter of 2
12345678901234567890123456789012123456789012345678901 2
12345678901234567890123456789012123456789012345678901
the square is just 4 times the length of the side, which is equal to 2
12345678901234567890123456789012123456789012345678901 2
12345678901234567890123456789012123456789012345678901
6. Column A is greater.
2345678901234567890123456789012123456789012345678901
2
2
1
12345678901234567890123456789012123456789012345678901 2
12345678901234567890123456789012123456789012345678901
17. B Here you just need to work through simplifying each fraction. 2
12345678901234567890123456789012123456789012345678901 2
12345678901234567890123456789012123456789012345678901
2345678901234567890123456789012123456789012345678901
2
2
1 Column A Column B
12345678901234567890123456789012123456789012345678901 2
12345678901234567890123456789012123456789012345678901 2
S DS D1 1
S DS D
12345678901234567890123456789012123456789012345678901
1 2 1
12345678901234567890123456789012123456789012345678901
12345678901234567890123456789012123456789012345678901
3 4 12 3 5
2
15
2
2
2
12345678901234567890123456789012123456789012345678901
5 5 2
S DS D
2 2 4
S DS D
12345678901234567890123456789012123456789012345678901
2 1
12345678901234567890123456789012123456789012345678901
12345678901234567890123456789012123456789012345678901
3 4 12 5 3
2
15
2
2
2
12345678901234567890123456789012123456789012345678901 2
12345678901234567890123456789012123456789012345678901
1 12 1 2 15 2 2
12345678901234567890123456789012123456789012345678901 2
12345678901234567890123456789012123456789012345678901
5 5 5 5 5 1 2
12345678901234567890123456789012123456789012345678901
12 4 4 15 2 2 2
12345678901234567890123456789012123456789012345678901 22
2345678901234567890123456789012123456789012345678901
12345678901234567890123456789012123456789012345678901
12345678901234567890123456789012123456789012345678901
As you can see, Column B is greater. 2
12345678901234567890123456789012123456789012345678901 22
12345678901234567890123456789012123456789012345678901
18. A Use the information given about the line segments to write down
12345678901234567890123456789012123456789012345678901 2
1 equations that might be helpful in comparing the two columns. 2
12345678901234567890123456789012123456789012345678901 2
12345678901234567890123456789012123456789012345678901
AB 5 CD yields: r 1 s 5 s 1 3t → r 5 3t. You can plug this 2
12345678901234567890123456789012123456789012345678901 2
12345678901234567890123456789012123456789012345678901
directly into BC 5 2t 1 3r 5 2t 1 3(3t) 5 11t. You can infer 2
12345678901234567890123456789012123456789012345678901 2
12345678901234567890123456789012123456789012345678901
that both r and t are positive numbers since they only differ by a 2
12345678901234567890123456789012123456789012345678901 2
12345678901234567890123456789012123456789012345678901
multiple of three and at least one of them has to be positive since 2
2345678901234567890123456789012123456789012345678901
12345678901234567890123456789012123456789012345678901 2
12345678901234567890123456789012123456789012345678901
BC, a distance, is positive. Column A is greater. 2
12345678901234567890123456789012123456789012345678901 22
12345678901234567890123456789012123456789012345678901 2
1 19. D To begin, you need to solve the inequality for x, 22x . 2 → x , 21.
12345678901234567890123456789012123456789012345678901
You know x is less than 21. What does that tell you about x2? 2
12345678901234567890123456789012123456789012345678901 2
12345678901234567890123456789012123456789012345678901 2
12345678901234567890123456789012123456789012345678901 2
Not much, certainly nothing definite in relation to 2 since x could
12345678901234567890123456789012123456789012345678901 2
12345678901234567890123456789012123456789012345678901
be 21.1 or it could be 210. Thus, (D) is correct. 2
12345678901234567890123456789012123456789012345678901 2
12345678901234567890123456789012123456789012345678901
20. D The circumference of the circle has to be something around 7p 2
12345678901234567890123456789012123456789012345678901 2
12345678901234567890123456789012123456789012345678901
' 22. The perimeter of the rectangle, on the other hand, is not 2
12345678901234567890123456789012123456789012345678901 22
2345678901234567890123456789012123456789012345678901
12345678901234567890123456789012123456789012345678901
definite. The sides could be 2 and 12, and so the perimeter
2
1
12345678901234567890123456789012123456789012345678901
would be 28, or the sides could be 4 and 6 and the perimeter 2
12345678901234567890123456789012123456789012345678901 2
12345678901234567890123456789012123456789012345678901
would be 20. In one instance Column A is greater and in the 2
12345678901234567890123456789012123456789012345678901 2
12345678901234567890123456789012123456789012345678901
other Column B is greater. 2
12345678901234567890123456789012123456789012345678901 2
12345678901234567890123456789012123456789012345678901 2
12345678901234567890123456789012123456789012345678901 22
2345678901234567890123456789012123456789012345678901
21. B The first thing to do is determine the value of z. The 1-digit even
12345678901234567890123456789012123456789012345678901
integers are 2, 4, 6, and 8. The square root of z is also an integer. 2
12345678901234567890123456789012123456789012345678901
1 That is only true of 4. Thus, z equals 4. Four squared is 16, and 2
12345678901234567890123456789012123456789012345678901 2
2345678901234567890123456789012123456789012345678901
12345678901234567890123456789012123456789012345678901 2
12345678901234567890123456789012123456789012345678901
three times 4 is 12. Thus, Column B is greater. 2
12345678901234567890123456789012123456789012345678901 22
12345678901234567890123456789012123456789012345678901 2
1
12345678901234567890123456789012123456789012345678901 2
12345678901234567890123456789012123456789012345678901 2
12345678901234567890123456789012123456789012345678901 2
12345678901234567890123456789012123456789012345678901 2
12345678901234567890123456789012123456789012345678901 2
12345678901234567890123456789012123456789012345678901 2
12345678901234567890123456789012123456789012345678901 2
12345678901234567890123456789012123456789012345678901 2
12345678901234567890123456789012123456789012345678901 2 297
123456789012345678901234567890121234567890123456789012
Part IV: Three Practice GREs

123456789012345678901234567890121234567890123456789012
12345678901234567890123456789012123456789012345678901 2
12345678901234567890123456789012123456789012345678901 2
12345678901234567890123456789012123456789012345678901
22. B Five percent simple annual interest means that after one year’s 2
12345678901234567890123456789012123456789012345678901 2
12345678901234567890123456789012123456789012345678901
time, 5 percent of interest will have accrued. After eight months 2
12345678901234567890123456789012123456789012345678901 2
12345678901234567890123456789012123456789012345678901
(which is two thirds of a year) three fourths of the interest will 2
2345678901234567890123456789012123456789012345678901 2
1
12345678901234567890123456789012123456789012345678901
have accrued. You need to multiply the 16,000 times 5 percent 2
12345678901234567890123456789012123456789012345678901 2
12345678901234567890123456789012123456789012345678901
12345678901234567890123456789012123456789012345678901
1
times three fourths, which is ~
2345678901234567890123456789012123456789012345678901
16,000 !~ 0.05 SD
!
2
3
5 533.33.
2
2
2
12345678901234567890123456789012123456789012345678901 2
12345678901234567890123456789012123456789012345678901
Column B is greater, so the answer is (B). 2
12345678901234567890123456789012123456789012345678901 2
12345678901234567890123456789012123456789012345678901
a 3 3 2
1 23. C You know that 5 → a 5 b, and that the area of a sector is 2
2345678901234567890123456789012123456789012345678901
12345678901234567890123456789012123456789012345678901
b 4 4 2
12345678901234567890123456789012123456789012345678901 2
12345678901234567890123456789012123456789012345678901 2
2
the sector angle measure divided by 360 times pr . For Column
12345678901234567890123456789012123456789012345678901
2345678901234567890123456789012123456789012345678901
2
2
1 A, the area of sector 1 is.
12345678901234567890123456789012123456789012345678901 2
S D
~ pr ! 5
3
S DS D
12345678901234567890123456789012123456789012345678901
a b
12345678901234567890123456789012123456789012345678901
2
~ 2
pr !
12345678901234567890123456789012123456789012345678901
360 4 360
12345678901234567890123456789012123456789012345678901 2
2345678901234567890123456789012123456789012345678901
2
2
2
12345678901234567890123456789012123456789012345678901
For Column B, you need the area of the sector 2 times 0.75, 2
2
12345678901234567890123456789012123456789012345678901
12345678901234567890123456789012123456789012345678901 2
12345678901234567890123456789012123456789012345678901
which is 2
12345678901234567890123456789012123456789012345678901 2 2
1
~0.75!
b
360
S D
~ pr2!5
3
S DS D
b
12345678901234567890123456789012123456789012345678901
12345678901234567890123456789012123456789012345678901
4 360
12345678901234567890123456789012123456789012345678901
~ pr2! 2
2
2
2345678901234567890123456789012123456789012345678901
12345678901234567890123456789012123456789012345678901 2
12345678901234567890123456789012123456789012345678901
Column A and B are equal and (C) is the answer. 2
12345678901234567890123456789012123456789012345678901 2
24. C Remember the factors of a number multiply to give that number 2
12345678901234567890123456789012123456789012345678901
12345678901234567890123456789012123456789012345678901 2
and the multiples of a number are that number times the 2
12345678901234567890123456789012123456789012345678901
12345678901234567890123456789012123456789012345678901 2
12345678901234567890123456789012123456789012345678901 2
12345678901234567890123456789012123456789012345678901 2
integers. For Column A, begin by checking to see if 120 is evenly
1 divisible by 24 because that would clearly make 24 the greatest 2
12345678901234567890123456789012123456789012345678901 2
12345678901234567890123456789012123456789012345678901 2
12345678901234567890123456789012123456789012345678901 2
common factor. Five times 24 gives 120 and so 24 is the greatest
12345678901234567890123456789012123456789012345678901
common factor. For Column B, if the number does not quickly 2
12345678901234567890123456789012123456789012345678901
2345678901234567890123456789012123456789012345678901
2
come to mind then, write out multiples of each to find the least 2
12345678901234567890123456789012123456789012345678901
12345678901234567890123456789012123456789012345678901 2
1 common multiple. The LCM is 24 so the answer is again (C). 2
12345678901234567890123456789012123456789012345678901 2
12345678901234567890123456789012123456789012345678901 2
2345678901234567890123456789012123456789012345678901
25. D Column A is equivalent to m 1 2mn 1 n and Column B is 2
12345678901234567890123456789012123456789012345678901
2 2
12345678901234567890123456789012123456789012345678901
equivalent to m2 1 mn 2 2n2. You have to compare these two 2
2
1
12345678901234567890123456789012123456789012345678901 2
12345678901234567890123456789012123456789012345678901
expressions. Since the first term in each is the same, you can 2
2345678901234567890123456789012123456789012345678901
12345678901234567890123456789012123456789012345678901
eliminate them from consideration. If n equals zero then both 2
2
12345678901234567890123456789012123456789012345678901 2
12345678901234567890123456789012123456789012345678901
expressions are zero and so equal, but if n is non-zero then there 2
12345678901234567890123456789012123456789012345678901
1 certainly are situations when the two expressions are unequal. 2
12345678901234567890123456789012123456789012345678901 2
12345678901234567890123456789012123456789012345678901 2
12345678901234567890123456789012123456789012345678901
Thus, the answer is (D). 2
12345678901234567890123456789012123456789012345678901 2
12345678901234567890123456789012123456789012345678901 2
12345678901234567890123456789012123456789012345678901 2
2345678901234567890123456789012123456789012345678901
26. C First solve the equation to see if you can glean any useful
2
12345678901234567890123456789012123456789012345678901 2
12345678901234567890123456789012123456789012345678901
information about the relationship of x and y:
2
12345678901234567890123456789012123456789012345678901 2
1 x 2 4y 1 7 5 7 → x 5 4y → x 5 2y
2 2 2 2
12345678901234567890123456789012123456789012345678901 2
2345678901234567890123456789012123456789012345678901
2
1
12345678901234567890123456789012123456789012345678901
If x 5 2y, then side b is equal to side c. This means that we can 2
12345678901234567890123456789012123456789012345678901 2
12345678901234567890123456789012123456789012345678901
eliminate c2 and b2 from the columns, and all we are left with is 2
2345678901234567890123456789012123456789012345678901 2
1
12345678901234567890123456789012123456789012345678901
a2 in each. Ergo, the answer is (C). 2
12345678901234567890123456789012123456789012345678901 2 2
298 12345678901234567890123456789012123456789012345678901
123456789012345678901234567890121234567890123456789012

www.petersons.com
ANSWERS
123456789012345678901234567890121234567890123456789012
12345678901234567890123456789012123456789012345678901 2
12345678901234567890123456789012123456789012345678901 2
12345678901234567890123456789012123456789012345678901
27. B The number of integers from D to C is simply C 2 D, but since 2
12345678901234567890123456789012123456789012345678901 2
12345678901234567890123456789012123456789012345678901
it is inclusive we must include D and so add one, C 2 D 1 1. 2
12345678901234567890123456789012123456789012345678901 2
12345678901234567890123456789012123456789012345678901
Now you just need to see if we can manipulate Column B so that 2
2345678901234567890123456789012123456789012345678901 2
1
12345678901234567890123456789012123456789012345678901
you can compare it to Column A. 2
12345678901234567890123456789012123456789012345678901 2
12345678901234567890123456789012123456789012345678901 2
12345678901234567890123456789012123456789012345678901
2
C 2 CD 1 2C C(C 2 D 1 2)
2345678901234567890123456789012123456789012345678901
2
2
1 5 5C2D12
12345678901234567890123456789012123456789012345678901
4C 2 3C C 2
12345678901234567890123456789012123456789012345678901 2
12345678901234567890123456789012123456789012345678901 2
12345678901234567890123456789012123456789012345678901
You can now directly compare Column
2345678901234567890123456789012123456789012345678901
A and B, and Column B 2
1 is greater since they share the C minus D term but Column B 2
12345678901234567890123456789012123456789012345678901 2
12345678901234567890123456789012123456789012345678901
is 2
12345678901234567890123456789012123456789012345678901
12. 2
12345678901234567890123456789012123456789012345678901 2
1 28. C You might suspect (D) here, but look closely at the number set. 2
2345678901234567890123456789012123456789012345678901
12345678901234567890123456789012123456789012345678901 2
12345678901234567890123456789012123456789012345678901
The only way that y and y 1 3 could have the two intervening 2
12345678901234567890123456789012123456789012345678901 2
12345678901234567890123456789012123456789012345678901
numbers in the set is if they are all consecutive integers: 2
12345678901234567890123456789012123456789012345678901 22
2345678901234567890123456789012123456789012345678901
12345678901234567890123456789012123456789012345678901
12345678901234567890123456789012123456789012345678901
(x 1 4) 5 (y) 1 1 2
12345678901234567890123456789012123456789012345678901 2
12345678901234567890123456789012123456789012345678901
(z) 5 (y) 1 2 2
12345678901234567890123456789012123456789012345678901 22
1
12345678901234567890123456789012123456789012345678901
Solve for x and z in terms of y: 2
12345678901234567890123456789012123456789012345678901 2
12345678901234567890123456789012123456789012345678901 2
12345678901234567890123456789012123456789012345678901 22
2345678901234567890123456789012123456789012345678901
x 5 y 1 1 2 4 5 y 2 3
12345678901234567890123456789012123456789012345678901
12345678901234567890123456789012123456789012345678901
z5y12 2
12345678901234567890123456789012123456789012345678901 22
12345678901234567890123456789012123456789012345678901
So, Column A, z 2 5, must equal (y 1 2) 2 5, or y 2 3. Since 2
12345678901234567890123456789012123456789012345678901
x 5 y 2 3, both columns are equal, and the correct answer 2
12345678901234567890123456789012123456789012345678901
12345678901234567890123456789012123456789012345678901 2
12345678901234567890123456789012123456789012345678901 2
1 is (C). 2
12345678901234567890123456789012123456789012345678901 22
2345678901234567890123456789012123456789012345678901
12345678901234567890123456789012123456789012345678901 2
1
12345678901234567890123456789012123456789012345678901 2
12345678901234567890123456789012123456789012345678901
Verbal 2
12345678901234567890123456789012123456789012345678901 22
2345678901234567890123456789012123456789012345678901
12345678901234567890123456789012123456789012345678901 2
1 Antonyms
12345678901234567890123456789012123456789012345678901 2
12345678901234567890123456789012123456789012345678901 2
12345678901234567890123456789012123456789012345678901 22
2345678901234567890123456789012123456789012345678901
1. B To be pragmatic is to focus on the practical. The opposite of this
12345678901234567890123456789012123456789012345678901
is something like idealistic. Looking through the answer choices,
2
1
12345678901234567890123456789012123456789012345678901
(B), quixotic, fits the pre-guess. If you are unfamiliar with this 2
12345678901234567890123456789012123456789012345678901 2
12345678901234567890123456789012123456789012345678901
term, pick up a copy of Cervantes’ literary classic Don Quixote. 2
12345678901234567890123456789012123456789012345678901 2
12345678901234567890123456789012123456789012345678901 2
12345678901234567890123456789012123456789012345678901
2. D A cogent argument is compelling and well-constructed. You are 2
12345678901234567890123456789012123456789012345678901 2
12345678901234567890123456789012123456789012345678901
looking for the opposite of this. Unintelligent might attract you, 2
12345678901234567890123456789012123456789012345678901 2
12345678901234567890123456789012123456789012345678901
but unconvincing is much nearer the mark. 2
12345678901234567890123456789012123456789012345678901 2
12345678901234567890123456789012123456789012345678901 2
12345678901234567890123456789012123456789012345678901 22
2345678901234567890123456789012123456789012345678901
3. A To be obdurate about an issue is to stubbornly maintain your
12345678901234567890123456789012123456789012345678901
position. Flexible or changeable is a good pre-guess for the 2
12345678901234567890123456789012123456789012345678901
1 antonym. Choice (A) fits the pre-guess well because to be 2
12345678901234567890123456789012123456789012345678901 2
2345678901234567890123456789012123456789012345678901
12345678901234567890123456789012123456789012345678901 2
1 persuadable is to be capable of change. 2
12345678901234567890123456789012123456789012345678901 2
12345678901234567890123456789012123456789012345678901
4. A Ebullient means to be boiling over with emotion. The opposite 2
12345678901234567890123456789012123456789012345678901 2
12345678901234567890123456789012123456789012345678901
of this is emotionless. Stern approximates our pre-guess, but 2
12345678901234567890123456789012123456789012345678901 2
12345678901234567890123456789012123456789012345678901
stoic hits it on the head. 2
12345678901234567890123456789012123456789012345678901 2 299
123456789012345678901234567890121234567890123456789012
Part IV: Three Practice GREs

123456789012345678901234567890121234567890123456789012
12345678901234567890123456789012123456789012345678901 2
12345678901234567890123456789012123456789012345678901 2
12345678901234567890123456789012123456789012345678901
5. E A garrulous person is extremely talkative, even inconsiderately 2
12345678901234567890123456789012123456789012345678901 2
12345678901234567890123456789012123456789012345678901
so. Thus, you are looking for an adjective that means “not 2
12345678901234567890123456789012123456789012345678901 2
12345678901234567890123456789012123456789012345678901
talkative.” Decorious means to act with decorum, or good 2
2345678901234567890123456789012123456789012345678901 2
1
12345678901234567890123456789012123456789012345678901
manners, and so might seem attractive, but taciturn is a better 2
12345678901234567890123456789012123456789012345678901 2
12345678901234567890123456789012123456789012345678901
choice. It means “tight-lipped.” 2
12345678901234567890123456789012123456789012345678901 2
1 6. B To enervate is to weaken the strength of something. The 2
2345678901234567890123456789012123456789012345678901
12345678901234567890123456789012123456789012345678901 2
12345678901234567890123456789012123456789012345678901 2
12345678901234567890123456789012123456789012345678901
opposite is to make stronger. Looking through the answer 2
12345678901234567890123456789012123456789012345678901
2345678901234567890123456789012123456789012345678901
2
2
1 choices, (B) obviously jumps out. It is correct.
12345678901234567890123456789012123456789012345678901 2
12345678901234567890123456789012123456789012345678901
7. A To manumit someone is to free them from slavery. The antonym 2
12345678901234567890123456789012123456789012345678901 2
12345678901234567890123456789012123456789012345678901
is to make someone a slave. Enslave, choice (A), clearly fits the 2
2345678901234567890123456789012123456789012345678901 2
1
12345678901234567890123456789012123456789012345678901
pre-guess and it is correct. 2
12345678901234567890123456789012123456789012345678901 2
12345678901234567890123456789012123456789012345678901 2
12345678901234567890123456789012123456789012345678901
8. D Adumbrate means to foreshadow vaguely, and so its opposite 2
2345678901234567890123456789012123456789012345678901
12345678901234567890123456789012123456789012345678901
would be something like “to clearly state.” To make explicit, 2
2
12345678901234567890123456789012123456789012345678901
12345678901234567890123456789012123456789012345678901 2
12345678901234567890123456789012123456789012345678901
choice (D), fits the pre-guess and is correct. 2
12345678901234567890123456789012123456789012345678901 2
12345678901234567890123456789012123456789012345678901 2
1 9. E A panegyric is a speech of effusive praise. You are therefore 2
12345678901234567890123456789012123456789012345678901
looking for something like “criticize.” Obloquy means “abusive 2
12345678901234567890123456789012123456789012345678901 2
12345678901234567890123456789012123456789012345678901 2
12345678901234567890123456789012123456789012345678901 2
2345678901234567890123456789012123456789012345678901
or condemnatory language.” That certainly is the opposite of
12345678901234567890123456789012123456789012345678901 2
12345678901234567890123456789012123456789012345678901
praise, and so (E) is the answer. 2
12345678901234567890123456789012123456789012345678901 2
12345678901234567890123456789012123456789012345678901 2
12345678901234567890123456789012123456789012345678901 2
12345678901234567890123456789012123456789012345678901
Analogies 2
12345678901234567890123456789012123456789012345678901 2
12345678901234567890123456789012123456789012345678901 2
1 10. C The relationship between the stem words is one of definition, as 2
2345678901234567890123456789012123456789012345678901
to eclipse is to overshadow. Going through the answer choices, 2
12345678901234567890123456789012123456789012345678901
12345678901234567890123456789012123456789012345678901 2
1 2
12345678901234567890123456789012123456789012345678901 2
to set is not to rise; to outstrip is not to flail; to debauch is to
12345678901234567890123456789012123456789012345678901
debase; to overcome is not necessarily to outstretch; to pervert is 2
12345678901234567890123456789012123456789012345678901 2
12345678901234567890123456789012123456789012345678901
certainly not to correct. 2
12345678901234567890123456789012123456789012345678901 2
12345678901234567890123456789012123456789012345678901 2
12345678901234567890123456789012123456789012345678901
11. A Here the stem relationship is one of degree: To be emphatic is to 2
2345678901234567890123456789012123456789012345678901
12345678901234567890123456789012123456789012345678901
insist strongly. Choice (B) might seem appealing here, but be 2
2
12345678901234567890123456789012123456789012345678901 2
1
12345678901234567890123456789012123456789012345678901
careful because it has the order of degree reversed—to be very 2
12345678901234567890123456789012123456789012345678901 2
12345678901234567890123456789012123456789012345678901
absorbed is to be engrossed. (A) is the correct choice—to be 2
12345678901234567890123456789012123456789012345678901 2
12345678901234567890123456789012123456789012345678901
contentious is to argue excessively. 2
12345678901234567890123456789012123456789012345678901 2
12345678901234567890123456789012123456789012345678901 2
12345678901234567890123456789012123456789012345678901 2
2345678901234567890123456789012123456789012345678901
12. C An astronomer studies the stars. That is a good relationship.
Applying it to the answer choices, an accountant keeps the 2
12345678901234567890123456789012123456789012345678901
12345678901234567890123456789012123456789012345678901 2
12345678901234567890123456789012123456789012345678901 2
1 financial records; a librarian manages books; a historian studies 2
2345678901234567890123456789012123456789012345678901
history; a police officer administers the laws; and a magician 2
12345678901234567890123456789012123456789012345678901
12345678901234567890123456789012123456789012345678901 2
12345678901234567890123456789012123456789012345678901 2
12345678901234567890123456789012123456789012345678901 2
performs tricks.
2
1
12345678901234567890123456789012123456789012345678901
13. D The relationship between the stem pair is that you inoculate to 2
12345678901234567890123456789012123456789012345678901 2
12345678901234567890123456789012123456789012345678901
protect against a disease. Of the answer choices, only (D), you 2
12345678901234567890123456789012123456789012345678901 2
12345678901234567890123456789012123456789012345678901
defend to protect against an enemy, shares this relationship.
2345678901234567890123456789012123456789012345678901
2
2
1
12345678901234567890123456789012123456789012345678901 2
12345678901234567890123456789012123456789012345678901 2 2
300 12345678901234567890123456789012123456789012345678901
123456789012345678901234567890121234567890123456789012

www.petersons.com
ANSWERS
123456789012345678901234567890121234567890123456789012
12345678901234567890123456789012123456789012345678901 2
12345678901234567890123456789012123456789012345678901 2
12345678901234567890123456789012123456789012345678901
14. C Halcyon means peaceful and calm, and so the relationship 2
12345678901234567890123456789012123456789012345678901 2
12345678901234567890123456789012123456789012345678901
between the stem words is one of definition—halcyon means 2
12345678901234567890123456789012123456789012345678901 2
12345678901234567890123456789012123456789012345678901
peaceful. Choices (B) and (E) are antonyms not synonyms, and 2
2345678901234567890123456789012123456789012345678901 2
1
12345678901234567890123456789012123456789012345678901
(A) and (D) do not share a necessary relationship. Choice (C) 2
12345678901234567890123456789012123456789012345678901
does share a necessary relationship—surreptitious means clan- 2
12345678901234567890123456789012123456789012345678901 2
12345678901234567890123456789012123456789012345678901
2345678901234567890123456789012123456789012345678901
2
2
1 destine. Choice (B) is there to catch people shopping for words
12345678901234567890123456789012123456789012345678901 2
12345678901234567890123456789012123456789012345678901
like peaceful. 2
12345678901234567890123456789012123456789012345678901 2
12345678901234567890123456789012123456789012345678901
2345678901234567890123456789012123456789012345678901
2
2
1 15. A The stem words are opposites—a desolate place is the opposite
12345678901234567890123456789012123456789012345678901 2
12345678901234567890123456789012123456789012345678901
of a verdant place. Choices (B) and (C) are synonyms and so do 2
12345678901234567890123456789012123456789012345678901
not fit the stem relationship, and (D) and (E) do not have a 2
12345678901234567890123456789012123456789012345678901
2345678901234567890123456789012123456789012345678901
2
1 strong relationship (a tropical place is often humid, a technical 2
12345678901234567890123456789012123456789012345678901 2
12345678901234567890123456789012123456789012345678901
manual may or may not be detailed.) Choice (A) shares the stem 2
12345678901234567890123456789012123456789012345678901 2
12345678901234567890123456789012123456789012345678901
relationship—a destitute area is the opposite of an opulent area. 2
12345678901234567890123456789012123456789012345678901 22
2345678901234567890123456789012123456789012345678901
12345678901234567890123456789012123456789012345678901
16. D Here, the relationship is again one of definition—a harbinger 2
12345678901234567890123456789012123456789012345678901
12345678901234567890123456789012123456789012345678901
presages something (presage means to foretell, or portend). A 2
2
12345678901234567890123456789012123456789012345678901
1 computer can calculate something; a television shows things 2
12345678901234567890123456789012123456789012345678901 2
12345678901234567890123456789012123456789012345678901 2
12345678901234567890123456789012123456789012345678901
that have been broadcast; a liaison does not necessarily 2
12345678901234567890123456789012123456789012345678901 2
12345678901234567890123456789012123456789012345678901 22
2345678901234567890123456789012123456789012345678901
interrogate someone; a messenger relates something; and a
12345678901234567890123456789012123456789012345678901
12345678901234567890123456789012123456789012345678901
broker does not necessarily abscond with something. 2
12345678901234567890123456789012123456789012345678901 22
12345678901234567890123456789012123456789012345678901
17. A Asseverate means to aver or affirm earnestly, and so the stem 2
12345678901234567890123456789012123456789012345678901
relationship is to asseverate is to aver earnestly. Choice (A) 2
12345678901234567890123456789012123456789012345678901
12345678901234567890123456789012123456789012345678901 2
shares this relationship—to beseech is to request earnestly. Do 2
12345678901234567890123456789012123456789012345678901
1 2
12345678901234567890123456789012123456789012345678901
any of the other answer pairs share it? Approach and accost 2
12345678901234567890123456789012123456789012345678901 2
12345678901234567890123456789012123456789012345678901
might, but the order of degree is reversed—to accost is to 2
12345678901234567890123456789012123456789012345678901 2
12345678901234567890123456789012123456789012345678901
approach forcefully. 2
12345678901234567890123456789012123456789012345678901 2
12345678901234567890123456789012123456789012345678901 2
12345678901234567890123456789012123456789012345678901 2
12345678901234567890123456789012123456789012345678901 2
12345678901234567890123456789012123456789012345678901
Sentence Completions 2
12345678901234567890123456789012123456789012345678901 22
2345678901234567890123456789012123456789012345678901
12345678901234567890123456789012123456789012345678901
1 18. D The word but in the sentence is the key clue. It tells you that the 2
12345678901234567890123456789012123456789012345678901
first and second parts of the sentence are in contrast to each 2
12345678901234567890123456789012123456789012345678901 2
2345678901234567890123456789012123456789012345678901
other. In the first part we are told that Mr. Thomas had earned 2
12345678901234567890123456789012123456789012345678901
12345678901234567890123456789012123456789012345678901 2
the reputation as a miser, and so in the second part you should 2
12345678901234567890123456789012123456789012345678901
12345678901234567890123456789012123456789012345678901 22
1 expect to find a strong contrast to this. Which answer choice
12345678901234567890123456789012123456789012345678901 2
12345678901234567890123456789012123456789012345678901
makes a strong contrast to miser? Choice (D), spendthrift, 2
12345678901234567890123456789012123456789012345678901 2
12345678901234567890123456789012123456789012345678901
which means one who spends money wastefully, fits the bill. 2
12345678901234567890123456789012123456789012345678901 2
2345678901234567890123456789012123456789012345678901
19. A Consider the first blank first. Shifts in the demographics of a 2
12345678901234567890123456789012123456789012345678901
12345678901234567890123456789012123456789012345678901 2
12345678901234567890123456789012123456789012345678901 2
12345678901234567890123456789012123456789012345678901 22
country would at least challenge traditional political alliances
1
12345678901234567890123456789012123456789012345678901
and at most destroy them. Since the sentence speaks of nascent 2
12345678901234567890123456789012123456789012345678901
political parties, you can infer that the traditional ones are at 2
12345678901234567890123456789012123456789012345678901 2
12345678901234567890123456789012123456789012345678901 2
12345678901234567890123456789012123456789012345678901
least being challenged. Solvent and elusive
2345678901234567890123456789012123456789012345678901
are not good answer 2
1 choices for the first blank. As the old political parties are being 2
12345678901234567890123456789012123456789012345678901 2
12345678901234567890123456789012123456789012345678901 22
12345678901234567890123456789012123456789012345678901 301
123456789012345678901234567890121234567890123456789012
Part IV: Three Practice GREs

123456789012345678901234567890121234567890123456789012
12345678901234567890123456789012123456789012345678901 2
12345678901234567890123456789012123456789012345678901 2
12345678901234567890123456789012123456789012345678901
challenged, the new ones then are gaining ground. Gain then is a 2
12345678901234567890123456789012123456789012345678901 2
12345678901234567890123456789012123456789012345678901
good pre-guess to the second blank. Provide does not match this 2
12345678901234567890123456789012123456789012345678901 2
12345678901234567890123456789012123456789012345678901
pre-guess. Between procure and secure, secure is the better 2
2345678901234567890123456789012123456789012345678901 2
1
12345678901234567890123456789012123456789012345678901
choice since procure intimates buying, and one does not 2
12345678901234567890123456789012123456789012345678901 2
12345678901234567890123456789012123456789012345678901
purchase large swathes of the electoral landscape. 2
12345678901234567890123456789012123456789012345678901 2
1 20. B Begin with a pre-guess on the first blank. What is a word that goes 2
2345678901234567890123456789012123456789012345678901
12345678901234567890123456789012123456789012345678901 2
12345678901234567890123456789012123456789012345678901 2
12345678901234567890123456789012123456789012345678901
with public and “can be persuaded”? Opinion. Looking down the 2
12345678901234567890123456789012123456789012345678901
2345678901234567890123456789012123456789012345678901
2
2
1 first blank answer choices, (B) obviously matches this pre-guess,
12345678901234567890123456789012123456789012345678901 2
12345678901234567890123456789012123456789012345678901
but (A) and possibly (C) and (D) do as well. The word ostensibly 2
12345678901234567890123456789012123456789012345678901
clues you to the fact that the second blank is a strong contrast to 2
12345678901234567890123456789012123456789012345678901
2345678901234567890123456789012123456789012345678901
2
1 process of popular persuasion. Which of the answer choices 2
12345678901234567890123456789012123456789012345678901 2
12345678901234567890123456789012123456789012345678901
matches this conception? Undemocratic does. Thus, (B) is a strong 2
12345678901234567890123456789012123456789012345678901 2
12345678901234567890123456789012123456789012345678901
candidate for the best answer choice. Reading the other answer 2
12345678901234567890123456789012123456789012345678901 2
2345678901234567890123456789012123456789012345678901
choices, (A) is the reasonable selection, but (B) is better since both 2
12345678901234567890123456789012123456789012345678901
12345678901234567890123456789012123456789012345678901 2
12345678901234567890123456789012123456789012345678901
answer blanks more specifically fit the logic of the sentence. 2
12345678901234567890123456789012123456789012345678901 2
12345678901234567890123456789012123456789012345678901 2
1 21. D You are looking for a word that modifies feuds between rival 2
12345678901234567890123456789012123456789012345678901
factions of a company. Internal is a good pre-guess. Internecine 2
12345678901234567890123456789012123456789012345678901 2
12345678901234567890123456789012123456789012345678901 2
12345678901234567890123456789012123456789012345678901 2
2345678901234567890123456789012123456789012345678901
means fighting within a group, and so matches the pre-guess. It
12345678901234567890123456789012123456789012345678901 2
12345678901234567890123456789012123456789012345678901
is the best choice, (D). 2
12345678901234567890123456789012123456789012345678901 2
22. B The sentence tells us that the first blank can be described as the 2
12345678901234567890123456789012123456789012345678901
12345678901234567890123456789012123456789012345678901 2
12345678901234567890123456789012123456789012345678901 2
12345678901234567890123456789012123456789012345678901 2
practice of appointing friends and associates to high posts. A
good pre-guess would be favoritism. Looking at the answer 2
12345678901234567890123456789012123456789012345678901
1 2
2345678901234567890123456789012123456789012345678901
choices, (D) and (E) can definitely be eliminated. Choice (A) also 2
12345678901234567890123456789012123456789012345678901
12345678901234567890123456789012123456789012345678901 2
1 can be eliminated because nepotism is showing favor to one’s 2
12345678901234567890123456789012123456789012345678901 2
12345678901234567890123456789012123456789012345678901
own family, not to friends and associates. The second blank is a 2
12345678901234567890123456789012123456789012345678901 2
12345678901234567890123456789012123456789012345678901
contrast to the first since the favoritism of the predecessors is 2
12345678901234567890123456789012123456789012345678901 2
12345678901234567890123456789012123456789012345678901
being eschewed in favor of the new way. Both (B) and (C) are 2
12345678901234567890123456789012123456789012345678901 2
12345678901234567890123456789012123456789012345678901
contrasts to favoritism, and so you must ask which one better 2
12345678901234567890123456789012123456789012345678901 2
12345678901234567890123456789012123456789012345678901
fits the flow of the sentence. Merit, based makes more sense than 2
12345678901234567890123456789012123456789012345678901 2
12345678901234567890123456789012123456789012345678901
work, based, and so (B) is a better choice. 2
12345678901234567890123456789012123456789012345678901 2
2345678901234567890123456789012123456789012345678901
23. E In this sentence, the author’s customary style of correspondence 2
12345678901234567890123456789012123456789012345678901
12345678901234567890123456789012123456789012345678901 2
12345678901234567890123456789012123456789012345678901 2
1 is being contrasted with the style in her most recent letter. 2
2345678901234567890123456789012123456789012345678901
Customarily her style is deliberate, but in this letter it is 2
12345678901234567890123456789012123456789012345678901
12345678901234567890123456789012123456789012345678901 2
12345678901234567890123456789012123456789012345678901 2
12345678901234567890123456789012123456789012345678901 2
convoluted. Therefore, in the first blank, you are looking for a
1 word that pairs well with deliberate and contrasts well with 2
12345678901234567890123456789012123456789012345678901 2
12345678901234567890123456789012123456789012345678901
convoluted. In the second blank, you are looking for a word that 2
12345678901234567890123456789012123456789012345678901 2
12345678901234567890123456789012123456789012345678901
pairs well with convoluted and contrasts well with deliberate. 2
12345678901234567890123456789012123456789012345678901 2
12345678901234567890123456789012123456789012345678901
Genuine, acidic, and urbane are not good pairs with deliberate 2
12345678901234567890123456789012123456789012345678901 2
12345678901234567890123456789012123456789012345678901
or contrasts with convoluted. As for the second blank, tiresome 2
12345678901234567890123456789012123456789012345678901 2
12345678901234567890123456789012123456789012345678901
does not pair as well with convoluted as prolix does (which 2
2345678901234567890123456789012123456789012345678901 2
1
12345678901234567890123456789012123456789012345678901
means to be wordy). Choice (E) is best. 2
12345678901234567890123456789012123456789012345678901 2 2
302 12345678901234567890123456789012123456789012345678901
123456789012345678901234567890121234567890123456789012

www.petersons.com
ANSWERS
123456789012345678901234567890121234567890123456789012
12345678901234567890123456789012123456789012345678901 2
12345678901234567890123456789012123456789012345678901 2
12345678901234567890123456789012123456789012345678901
Reading Comprehension 2
12345678901234567890123456789012123456789012345678901 2
12345678901234567890123456789012123456789012345678901 2
12345678901234567890123456789012123456789012345678901 2
24. C A question about the proper title of the passage is looking for
12345678901234567890123456789012123456789012345678901
the basic point or thrust of the passage. What is the point of this 2
12345678901234567890123456789012123456789012345678901 2
12345678901234567890123456789012123456789012345678901 2
12345678901234567890123456789012123456789012345678901
passage? It relates how the discovery and study of the Archaeans 2
12345678901234567890123456789012123456789012345678901 2
12345678901234567890123456789012123456789012345678901
has led to the reformulation of
2345678901234567890123456789012123456789012345678901
central concerns in the field of 2
1 biology. Choice (A) is too narrow because it excludes the 2
12345678901234567890123456789012123456789012345678901 2
12345678901234567890123456789012123456789012345678901 2
12345678901234567890123456789012123456789012345678901
discussion of the extremophiles. Choice (B), although better 2
12345678901234567890123456789012123456789012345678901
2345678901234567890123456789012123456789012345678901
2
2
1 than (A), is still too narrow. Choice (C) is a good restatement of
12345678901234567890123456789012123456789012345678901 2
12345678901234567890123456789012123456789012345678901
the thrust of the passage, since reformulation of central concerns 2
12345678901234567890123456789012123456789012345678901
in biology is a good approximation of Challenge to Biological 2
12345678901234567890123456789012123456789012345678901
2345678901234567890123456789012123456789012345678901
2
1 Paradigms. Choices (D) and (E) are too narrow in that they 2
12345678901234567890123456789012123456789012345678901 2
12345678901234567890123456789012123456789012345678901
exclude the classification issues, and notice that the passage 2
12345678901234567890123456789012123456789012345678901 2
12345678901234567890123456789012123456789012345678901
discusses extremophiles and not thermophiles. (C)’s the one. 2
12345678901234567890123456789012123456789012345678901 22
2345678901234567890123456789012123456789012345678901
12345678901234567890123456789012123456789012345678901
25. E On questions of this form you must go through each proposition 2
12345678901234567890123456789012123456789012345678901
12345678901234567890123456789012123456789012345678901 2
12345678901234567890123456789012123456789012345678901
individually: 2
12345678901234567890123456789012123456789012345678901 22
1
12345678901234567890123456789012123456789012345678901
Option I—The passage states that prior to the discovery of 2
12345678901234567890123456789012123456789012345678901
extremophiles the thermal threshold was set at 60 degrees 2
12345678901234567890123456789012123456789012345678901 2
2345678901234567890123456789012123456789012345678901
centigrade “because it was thought that the molecular 2
12345678901234567890123456789012123456789012345678901
12345678901234567890123456789012123456789012345678901 2
12345678901234567890123456789012123456789012345678901 2
12345678901234567890123456789012123456789012345678901 22
integrity of vital cellular components could not be main-
12345678901234567890123456789012123456789012345678901
tained beyond such temperatures.” It can be inferred that if 2
12345678901234567890123456789012123456789012345678901
extremophiles can live and thrive past 60 degrees centigrade, 2
12345678901234567890123456789012123456789012345678901
12345678901234567890123456789012123456789012345678901 2
they can maintain molecular integrity past the previous 2
12345678901234567890123456789012123456789012345678901
1 2
12345678901234567890123456789012123456789012345678901
thermal threshold. 2
12345678901234567890123456789012123456789012345678901 2
12345678901234567890123456789012123456789012345678901 2
12345678901234567890123456789012123456789012345678901
Option II—The passage does not mention extremophiles 2
12345678901234567890123456789012123456789012345678901 2
12345678901234567890123456789012123456789012345678901
existing in moderate environments, so II cannot be inferred 2
12345678901234567890123456789012123456789012345678901 2
12345678901234567890123456789012123456789012345678901
from the passage. 2
12345678901234567890123456789012123456789012345678901 2
12345678901234567890123456789012123456789012345678901 2
12345678901234567890123456789012123456789012345678901 22
2345678901234567890123456789012123456789012345678901
Option III—The passage states that extremophiles can live
12345678901234567890123456789012123456789012345678901
and thrive at 160 degrees and so it can be inferred that they 2
1
12345678901234567890123456789012123456789012345678901 2
12345678901234567890123456789012123456789012345678901
can live and thrive in temperatures as hot as 150 degrees. 2
12345678901234567890123456789012123456789012345678901 22
2345678901234567890123456789012123456789012345678901
12345678901234567890123456789012123456789012345678901
Since I and III are inferable, the answer is (E). 2
12345678901234567890123456789012123456789012345678901
12345678901234567890123456789012123456789012345678901 22
1 26. A The passage states that, “kingdoms are classified to be a
12345678901234567890123456789012123456789012345678901 2
12345678901234567890123456789012123456789012345678901
sub-group of a particular domain” (lines 20–21). Choice (A) 2
12345678901234567890123456789012123456789012345678901 2
12345678901234567890123456789012123456789012345678901
states exactly that. 2
12345678901234567890123456789012123456789012345678901 2
2345678901234567890123456789012123456789012345678901
27. B The passage uses thermal threshold to mean “the temperature at 2
12345678901234567890123456789012123456789012345678901
12345678901234567890123456789012123456789012345678901 2
12345678901234567890123456789012123456789012345678901 2
12345678901234567890123456789012123456789012345678901 22
which biological organisms begin to break down due to heat.”
1
12345678901234567890123456789012123456789012345678901
Which of the answer choices matches this usage? Choice (A) 2
12345678901234567890123456789012123456789012345678901
does not, and it uses terms not in the passage. Choice (B) 2
12345678901234567890123456789012123456789012345678901 2
12345678901234567890123456789012123456789012345678901 2
12345678901234567890123456789012123456789012345678901
matches up. As for (C), the passage
2345678901234567890123456789012123456789012345678901
clearly uses the term thermal 2
1 threshold to apply to all kinds of life and not just extremophiles, 2
12345678901234567890123456789012123456789012345678901 2
12345678901234567890123456789012123456789012345678901 22
12345678901234567890123456789012123456789012345678901 303
123456789012345678901234567890121234567890123456789012
Part IV: Three Practice GREs

123456789012345678901234567890121234567890123456789012
12345678901234567890123456789012123456789012345678901 2
12345678901234567890123456789012123456789012345678901 2
12345678901234567890123456789012123456789012345678901
and so this choice is out. Choice (D) uses terms not in the 2
12345678901234567890123456789012123456789012345678901 2
12345678901234567890123456789012123456789012345678901
passage and that should make you skeptical. Choice (E) is tricky 2
12345678901234567890123456789012123456789012345678901 2
12345678901234567890123456789012123456789012345678901
because the passage does speak of the former thermal threshold 2
2345678901234567890123456789012123456789012345678901 2
1
12345678901234567890123456789012123456789012345678901
as being thought of as a constant across nature. Yet, the 2
12345678901234567890123456789012123456789012345678901
threshold changed when extremophiles were discovered. Choice 2
12345678901234567890123456789012123456789012345678901 2
12345678901234567890123456789012123456789012345678901
2345678901234567890123456789012123456789012345678901
2
2
1 (B) is unquestionably a strong answer, and (E) is problematic
12345678901234567890123456789012123456789012345678901 2
12345678901234567890123456789012123456789012345678901
because the threshold has fluctuated. Therefore, (B) is a better 2
12345678901234567890123456789012123456789012345678901 2
12345678901234567890123456789012123456789012345678901
choice.
2345678901234567890123456789012123456789012345678901
2
2
1
12345678901234567890123456789012123456789012345678901 2
12345678901234567890123456789012123456789012345678901
28. D The entire passage culminates in the conclusion of the author in 2
12345678901234567890123456789012123456789012345678901
the last sentence of the passage, “Therefore, altruistic motiva- 2
12345678901234567890123456789012123456789012345678901
2345678901234567890123456789012123456789012345678901
2
1 tors should be possible inputs for any decision-matrix schema 2
12345678901234567890123456789012123456789012345678901 2
12345678901234567890123456789012123456789012345678901
that attempts to model human preference determination.” The 2
12345678901234567890123456789012123456789012345678901 2
12345678901234567890123456789012123456789012345678901
passage builds to this conclusion and so the clear intent of the 2
12345678901234567890123456789012123456789012345678901 2
2345678901234567890123456789012123456789012345678901
author is to make this point. Which answer choice reflects this 2
12345678901234567890123456789012123456789012345678901
12345678901234567890123456789012123456789012345678901 2
12345678901234567890123456789012123456789012345678901
intent? Choice (D) does. 2
12345678901234567890123456789012123456789012345678901 2
12345678901234567890123456789012123456789012345678901 2
1 29. C To answer this question you have to go to the place in the 2
12345678901234567890123456789012123456789012345678901
passage where the assumptions of decision-matrices are dis- 2
12345678901234567890123456789012123456789012345678901 2
12345678901234567890123456789012123456789012345678901 2
12345678901234567890123456789012123456789012345678901 2
2345678901234567890123456789012123456789012345678901
cussed (lines 31–37). The passage clearly states that in the
construction of decision-matrices it is assumed that individuals 2
12345678901234567890123456789012123456789012345678901
12345678901234567890123456789012123456789012345678901 2
12345678901234567890123456789012123456789012345678901 2
12345678901234567890123456789012123456789012345678901 2
are rational in decision-making. The author does not dispute
12345678901234567890123456789012123456789012345678901 2
12345678901234567890123456789012123456789012345678901
this assumption. That is answer choice (C), and it is correct. 2
12345678901234567890123456789012123456789012345678901 2
12345678901234567890123456789012123456789012345678901 2
1 30. A Looking at the structure of the passage, the first paragraph 2
2345678901234567890123456789012123456789012345678901
presents the challenge of modeling public choice scenarios, the 2
12345678901234567890123456789012123456789012345678901
12345678901234567890123456789012123456789012345678901 2
1 second paragraph explains the details of a particular methodol- 2
12345678901234567890123456789012123456789012345678901 2
12345678901234567890123456789012123456789012345678901
ogy for such modeling, and the third paragraph discusses 2
12345678901234567890123456789012123456789012345678901 2
12345678901234567890123456789012123456789012345678901
problems with the methodology presented in the third para- 2
12345678901234567890123456789012123456789012345678901 2
12345678901234567890123456789012123456789012345678901
graph. Given this basic structure, we can inspect the answer 2
12345678901234567890123456789012123456789012345678901 2
12345678901234567890123456789012123456789012345678901
choices to see which best describes the role of the second 2
12345678901234567890123456789012123456789012345678901 2
12345678901234567890123456789012123456789012345678901
paragraph. Choice (A) matches the tone and content of our 2
12345678901234567890123456789012123456789012345678901 2
12345678901234567890123456789012123456789012345678901
description of the second passage, and it is correct. 2
12345678901234567890123456789012123456789012345678901 2
2345678901234567890123456789012123456789012345678901
12345678901234567890123456789012123456789012345678901 2
12345678901234567890123456789012123456789012345678901 2
12345678901234567890123456789012123456789012345678901 2 2
1
12345678901234567890123456789012123456789012345678901 2
12345678901234567890123456789012123456789012345678901 2
12345678901234567890123456789012123456789012345678901 2
12345678901234567890123456789012123456789012345678901 2
12345678901234567890123456789012123456789012345678901 2
12345678901234567890123456789012123456789012345678901 2
2345678901234567890123456789012123456789012345678901
12345678901234567890123456789012123456789012345678901 2
12345678901234567890123456789012123456789012345678901 2
12345678901234567890123456789012123456789012345678901 2 2
1
12345678901234567890123456789012123456789012345678901 2
12345678901234567890123456789012123456789012345678901 2
12345678901234567890123456789012123456789012345678901 2
12345678901234567890123456789012123456789012345678901 2
12345678901234567890123456789012123456789012345678901 2
12345678901234567890123456789012123456789012345678901 2
12345678901234567890123456789012123456789012345678901 2
12345678901234567890123456789012123456789012345678901 2 2
304 12345678901234567890123456789012123456789012345678901
123456789012345678901234567890121234567890123456789012

www.petersons.com
Practice Test

3
1234567890123456789012345678901212345678901234567890123456789012123456
12 3456789012345678901234567890121234567890123456789012345678901212345 6
12 3456789012345678901234567890121234567890123456789012345678901212345 6
12 3456789012345678901234567890121234567890123456789012345678901212345
Quantitative 6
123456789012345678901234567890121234567890123456789012345678901212345
2 6
123456789012345678901234567890121234567890123456789012345678901212345 6
123456789012345678901234567890121234567890123456789012345678901212345
28 Questions—45 Minutes 6
123456789012345678901234567890121234567890123456789012345678901212345 6
123456789012345678901234567890121234567890123456789012345678901212345 6
123456789012345678901234567890121234567890123456789012345678901212345 66
3456789012345678901234567890121234567890123456789012345678901212345
General Information
1
12 3456789012345678901234567890121234567890123456789012345678901212345
1. The test has 28 questions. If you decide to take this exam under timed conditions, you have 6
12 3456789012345678901234567890121234567890123456789012345678901212345 6
12 3456789012345678901234567890121234567890123456789012345678901212345
45 minutes to complete the section. 6
123456789012345678901234567890121234567890123456789012345678901212345 66
2 3456789012345678901234567890121234567890123456789012345678901212345
123456789012345678901234567890121234567890123456789012345678901212345
123456789012345678901234567890121234567890123456789012345678901212345
2. All numbers used are real numbers. 6
123456789012345678901234567890121234567890123456789012345678901212345 66
123456789012345678901234567890121234567890123456789012345678901212345
123456789012345678901234567890121234567890123456789012345678901212345
3. All angle measurements can be assumed to be positive unless otherwise noted. 6
123456789012345678901234567890121234567890123456789012345678901212345 66
123456789012345678901234567890121234567890123456789012345678901212345
123456789012345678901234567890121234567890123456789012345678901212345
4. All figures lie in the same plane unless otherwise noted. 6
1 6
2 3456789012345678901234567890121234567890123456789012345678901212345
5. Drawings that accompany questions are intended to provide information useful in 6
123456789012345678901234567890121234567890123456789012345678901212345
123456789012345678901234567890121234567890123456789012345678901212345 6
1 answering the question. The figures are drawn closely to scale unless otherwise noted. 6
123456789012345678901234567890121234567890123456789012345678901212345 6
12 3456789012345678901234567890121234567890123456789012345678901212345 6
12 3456789012345678901234567890121234567890123456789012345678901212345 6
12 3456789012345678901234567890121234567890123456789012345678901212345
Directions: For questions 1–14, Column A and Column B will have one quantity each. 6
123456789012345678901234567890121234567890123456789012345678901212345 6
12 3456789012345678901234567890121234567890123456789012345678901212345
Your goal is to compare the quantities in each column and choose 6
12 3456789012345678901234567890121234567890123456789012345678901212345 6
2 3456789012345678901234567890121234567890123456789012345678901212345
123456789012345678901234567890121234567890123456789012345678901212345 6
123456789012345678901234567890121234567890123456789012345678901212345
A if the quantity in Column A is greater 6
1 6
123456789012345678901234567890121234567890123456789012345678901212345
B if the quantity in Column B is greater 6
123456789012345678901234567890121234567890123456789012345678901212345 6
123456789012345678901234567890121234567890123456789012345678901212345 66
2 3456789012345678901234567890121234567890123456789012345678901212345
C if the two quantities are equal
123456789012345678901234567890121234567890123456789012345678901212345 6
123456789012345678901234567890121234567890123456789012345678901212345
D if the relationship between the two quantities cannot be determined from the
6
123456789012345678901234567890121234567890123456789012345678901212345 6
1 information in the problem
123456789012345678901234567890121234567890123456789012345678901212345 66
2 3456789012345678901234567890121234567890123456789012345678901212345
123456789012345678901234567890121234567890123456789012345678901212345 6
123456789012345678901234567890121234567890123456789012345678901212345
On some questions, information about one or both quantities will be given. This informa-
6
123456789012345678901234567890121234567890123456789012345678901212345 6
1 tion will be centered above the two columns.
123456789012345678901234567890121234567890123456789012345678901212345 66
2 3456789012345678901234567890121234567890123456789012345678901212345
123456789012345678901234567890121234567890123456789012345678901212345 6
123456789012345678901234567890121234567890123456789012345678901212345 6
123456789012345678901234567890121234567890123456789012345678901212345 6
1
12 3456789012345678901234567890121234567890123456789012345678901212345 6
123456789012345678901234567890121234567890123456789012345678901212345 6
12 3456789012345678901234567890121234567890123456789012345678901212345 6
123456789012345678901234567890121234567890123456789012345678901212345 6
12 3456789012345678901234567890121234567890123456789012345678901212345 6
123456789012345678901234567890121234567890123456789012345678901212345 6
123456789012345678901234567890121234567890123456789012345678901212345 6
123456789012345678901234567890121234567890123456789012345678901212345 6
123456789012345678901234567890121234567890123456789012345678901212345 6
1234567890123456789012345678901212345678901234567890123456789012123456
305
Part IV: Three Practice GREs

1234567890123456789012345678901212345678901234567890123456789012123456
123456789012345678901234567890121234567890123456789012345678901212345 6
12 3456789012345678901234567890121234567890123456789012345678901212345 6
123456789012345678901234567890121234567890123456789012345678901212345
Column A Column B Column A Column B 6
12 3456789012345678901234567890121234567890123456789012345678901212345 6
12 3456789012345678901234567890121234567890123456789012345678901212345 6
123456789012345678901234567890121234567890123456789012345678901212345
1. = 2 2 3 2
6
12
2
3456789012345678901234567890121234567890123456789012345678901212345 6
3 1 7
3456789012345678901234567890121234567890123456789012345678901212345 6
1
12 3456789012345678901234567890121234567890123456789012345678901212345 6
12 3456789012345678901234567890121234567890123456789012345678901212345 6
12 3456789012345678901234567890121234567890123456789012345678901212345 6
12 3456789012345678901234567890121234567890123456789012345678901212345 6
1 3456789012345678901234567890121234567890123456789012345678901212345 6
2
12 3456789012345678901234567890121234567890123456789012345678901212345 6
12 3456789012345678901234567890121234567890123456789012345678901212345 6
12 3456789012345678901234567890121234567890123456789012345678901212345 6
12 3456789012345678901234567890121234567890123456789012345678901212345 6
1 3456789012345678901234567890121234567890123456789012345678901212345 6
2
12 3456789012345678901234567890121234567890123456789012345678901212345 6
12 3456789012345678901234567890121234567890123456789012345678901212345
2. x 1 2y 5 9 6
12 3456789012345678901234567890121234567890123456789012345678901212345 6
12 3456789012345678901234567890121234567890123456789012345678901212345
x5y 6
123456789012345678901234567890121234567890123456789012345678901212345
5. The figure above is an isosceles triangle. 6
12 3456789012345678901234567890121234567890123456789012345678901212345 6
12 3456789012345678901234567890121234567890123456789012345678901212345
2y 6 =2b 6
12 3456789012345678901234567890121234567890123456789012345678901212345
c 6
12 3456789012345678901234567890121234567890123456789012345678901212345 6
123456789012345678901234567890121234567890123456789012345678901212345 66
2 3456789012345678901234567890121234567890123456789012345678901212345
123456789012345678901234567890121234567890123456789012345678901212345 6
123456789012345678901234567890121234567890123456789012345678901212345 6
123456789012345678901234567890121234567890123456789012345678901212345 6
123456789012345678901234567890121234567890123456789012345678901212345 6
123456789012345678901234567890121234567890123456789012345678901212345 6
1
12 3456789012345678901234567890121234567890123456789012345678901212345 6
12 3456789012345678901234567890121234567890123456789012345678901212345 6
12 3456789012345678901234567890121234567890123456789012345678901212345 6
123456789012345678901234567890121234567890123456789012345678901212345
2 3. x is a positive integer. 6
123456789012345678901234567890121234567890123456789012345678901212345
6. b2a.1 6
123456789012345678901234567890121234567890123456789012345678901212345 6
123456789012345678901234567890121234567890123456789012345678901212345 6
12 SD
1 x 0.90
123456789012345678901234567890121234567890123456789012345678901212345
xa
xb

3456789012345678901234567890121234567890123456789012345678901212345
123456789012345678901234567890121234567890123456789012345678901212345
3
123456789012345678901234567890121234567890123456789012345678901212345 x
xa

b
6
6
6
123456789012345678901234567890121234567890123456789012345678901212345 66
123456789012345678901234567890121234567890123456789012345678901212345 6
1
12 3456789012345678901234567890121234567890123456789012345678901212345 6
12 3456789012345678901234567890121234567890123456789012345678901212345 6
123456789012345678901234567890121234567890123456789012345678901212345 6
123456789012345678901234567890121234567890123456789012345678901212345 6
12 3456789012345678901234567890121234567890123456789012345678901212345 6
12 3456789012345678901234567890121234567890123456789012345678901212345 6
12 3456789012345678901234567890121234567890123456789012345678901212345 6
123456789012345678901234567890121234567890123456789012345678901212345
|x| . 1 6
12 3456789012345678901234567890121234567890123456789012345678901212345
4. 7. 6
12 3456789012345678901234567890121234567890123456789012345678901212345 6
2 3456789012345678901234567890121234567890123456789012345678901212345
123456789012345678901234567890121234567890123456789012345678901212345 6
123456789012345678901234567890121234567890123456789012345678901212345
x(x 2 1) 0 6
1 6
123456789012345678901234567890121234567890123456789012345678901212345 6
123456789012345678901234567890121234567890123456789012345678901212345 6
12 3456789012345678901234567890121234567890123456789012345678901212345 6
12 3456789012345678901234567890121234567890123456789012345678901212345 6
12 3456789012345678901234567890121234567890123456789012345678901212345 6
12 3456789012345678901234567890121234567890123456789012345678901212345 6
123456789012345678901234567890121234567890123456789012345678901212345 6
123456789012345678901234567890121234567890123456789012345678901212345
2 1
6
123456789012345678901234567890121234567890123456789012345678901212345
(b2 1 2bc 1 c2)2 d1c 6
123456789012345678901234567890121234567890123456789012345678901212345 6
123456789012345678901234567890121234567890123456789012345678901212345 6
1 3456789012345678901234567890121234567890123456789012345678901212345 6
123456789012345678901234567890121234567890123456789012345678901212345 66
2 3456789012345678901234567890121234567890123456789012345678901212345
123456789012345678901234567890121234567890123456789012345678901212345 6
123456789012345678901234567890121234567890123456789012345678901212345 6
123456789012345678901234567890121234567890123456789012345678901212345 6
1
12 3456789012345678901234567890121234567890123456789012345678901212345 6
123456789012345678901234567890121234567890123456789012345678901212345 6
12 3456789012345678901234567890121234567890123456789012345678901212345 6
123456789012345678901234567890121234567890123456789012345678901212345 6
12 3456789012345678901234567890121234567890123456789012345678901212345 6
123456789012345678901234567890121234567890123456789012345678901212345 6
123456789012345678901234567890121234567890123456789012345678901212345 6
123456789012345678901234567890121234567890123456789012345678901212345 66
306 123456789012345678901234567890121234567890123456789012345678901212345
1234567890123456789012345678901212345678901234567890123456789012123456

www.petersons.com
TEST 3
1234567890123456789012345678901212345678901234567890123456789012123456
123456789012345678901234567890121234567890123456789012345678901212345 6
12 3456789012345678901234567890121234567890123456789012345678901212345 6
123456789012345678901234567890121234567890123456789012345678901212345
Column A Column B Column A Column B 6
12 3456789012345678901234567890121234567890123456789012345678901212345 6
12 3456789012345678901234567890121234567890123456789012345678901212345 6
123456789012345678901234567890121234567890123456789012345678901212345
8. A certain swim-team is composed of 11. 4 15
4 14 14 1 6
11 12 13
12 3456789012345678901234567890121234567890123456789012345678901212345 6
123456789012345678901234567890121234567890123456789012345678901212345
divers, individual swimmers, and relay 414 6
12 3456789012345678901234567890121234567890123456789012345678901212345 6
12 3456789012345678901234567890121234567890123456789012345678901212345
swimmers, and each member only 6
12 3456789012345678901234567890121234567890123456789012345678901212345 6
12
2
3456789012345678901234567890121234567890123456789012345678901212345 6
participates in one event. The team has
3456789012345678901234567890121234567890123456789012345678901212345 6
1
12 3456789012345678901234567890121234567890123456789012345678901212345
50 members, 20 percent of which are 6
12 3456789012345678901234567890121234567890123456789012345678901212345 6
12 3456789012345678901234567890121234567890123456789012345678901212345 6
divers.
12 3456789012345678901234567890121234567890123456789012345678901212345 6
123456789012345678901234567890121234567890123456789012345678901212345 6
12 3456789012345678901234567890121234567890123456789012345678901212345
Four times the The number of 6
12 3456789012345678901234567890121234567890123456789012345678901212345 6
12 3456789012345678901234567890121234567890123456789012345678901212345
number of divers relay racers 12. Six students compete for first, second, 6
12
2
3456789012345678901234567890121234567890123456789012345678901212345
3456789012345678901234567890121234567890123456789012345678901212345
6
6
1 and third place finishes in a ski race.
12 3456789012345678901234567890121234567890123456789012345678901212345 6
12 3456789012345678901234567890121234567890123456789012345678901212345 6
12 3456789012345678901234567890121234567890123456789012345678901212345
120 Number of 6
12 3456789012345678901234567890121234567890123456789012345678901212345 6
123456789012345678901234567890121234567890123456789012345678901212345 66
2 3456789012345678901234567890121234567890123456789012345678901212345 possible outcomes
123456789012345678901234567890121234567890123456789012345678901212345
123456789012345678901234567890121234567890123456789012345678901212345 for first, second, 6
123456789012345678901234567890121234567890123456789012345678901212345 6
123456789012345678901234567890121234567890123456789012345678901212345 66 and third place
123456789012345678901234567890121234567890123456789012345678901212345 6
1 9. Kathy drives 20 kilometers due north
12 3456789012345678901234567890121234567890123456789012345678901212345 6
12 3456789012345678901234567890121234567890123456789012345678901212345 6
from town A to town B. Then she drives
12 3456789012345678901234567890121234567890123456789012345678901212345 6
12 3456789012345678901234567890121234567890123456789012345678901212345
30 kilometers due west to town C. 6
12 3456789012345678901234567890121234567890123456789012345678901212345 6
12 3456789012345678901234567890121234567890123456789012345678901212345
Distance between 40 6
12 3456789012345678901234567890121234567890123456789012345678901212345 6
12 3456789012345678901234567890121234567890123456789012345678901212345
town A and C in 6
2 3456789012345678901234567890121234567890123456789012345678901212345
123456789012345678901234567890121234567890123456789012345678901212345 6
123456789012345678901234567890121234567890123456789012345678901212345
kilometers 6
123456789012345678901234567890121234567890123456789012345678901212345 66
13. C and D are digits and C . D.
123456789012345678901234567890121234567890123456789012345678901212345 6
1
12 3456789012345678901234567890121234567890123456789012345678901212345 CD 6
12 3456789012345678901234567890121234567890123456789012345678901212345 6
123456789012345678901234567890121234567890123456789012345678901212345 2 DC 6
123456789012345678901234567890121234567890123456789012345678901212345 6
12 3456789012345678901234567890121234567890123456789012345678901212345 EF 6
12 3456789012345678901234567890121234567890123456789012345678901212345 6
12 3456789012345678901234567890121234567890123456789012345678901212345 6
123456789012345678901234567890121234567890123456789012345678901212345
E1F 9 6
12 3456789012345678901234567890121234567890123456789012345678901212345 6
12 3456789012345678901234567890121234567890123456789012345678901212345
10. A sock drawer contains 75 socks. There 6
12 3456789012345678901234567890121234567890123456789012345678901212345 6
12 3456789012345678901234567890121234567890123456789012345678901212345
are an equal number of unpaired black, 6
123456789012345678901234567890121234567890123456789012345678901212345 6
123456789012345678901234567890121234567890123456789012345678901212345
white, and gray socks in the drawer. 6
123456789012345678901234567890121234567890123456789012345678901212345 6
2 3456789012345678901234567890121234567890123456789012345678901212345
123456789012345678901234567890121234567890123456789012345678901212345 6
123456789012345678901234567890121234567890123456789012345678901212345
The probability of One sixth 6
123456789012345678901234567890121234567890123456789012345678901212345 6
123456789012345678901234567890121234567890123456789012345678901212345 66
grabbing two gray
1
12 3456789012345678901234567890121234567890123456789012345678901212345
socks in two 14. n is a positive integer. 6
12 3456789012345678901234567890121234567890123456789012345678901212345 6
12 3456789012345678901234567890121234567890123456789012345678901212345
attempts 6
12 3456789012345678901234567890121234567890123456789012345678901212345 6
~ n 1 1! ! n! 1 1
123456789012345678901234567890121234567890123456789012345678901212345 6
12 3456789012345678901234567890121234567890123456789012345678901212345
n! 1 1 ~n 1 1!! 6
12 3456789012345678901234567890121234567890123456789012345678901212345 6
12 3456789012345678901234567890121234567890123456789012345678901212345 6
12 3456789012345678901234567890121234567890123456789012345678901212345 6
123456789012345678901234567890121234567890123456789012345678901212345 6
123456789012345678901234567890121234567890123456789012345678901212345
2 6
1 3456789012345678901234567890121234567890123456789012345678901212345 6
12 3456789012345678901234567890121234567890123456789012345678901212345 6
123456789012345678901234567890121234567890123456789012345678901212345 6
12 3456789012345678901234567890121234567890123456789012345678901212345 6
123456789012345678901234567890121234567890123456789012345678901212345 6
123456789012345678901234567890121234567890123456789012345678901212345 6
123456789012345678901234567890121234567890123456789012345678901212345 6
123456789012345678901234567890121234567890123456789012345678901212345 6 307
1234567890123456789012345678901212345678901234567890123456789012123456
Part IV: Three Practice GREs

1234567890123456789012345678901212345678901234567890123456789012123456
123456789012345678901234567890121234567890123456789012345678901212345 6
12 3456789012345678901234567890121234567890123456789012345678901212345 6
123456789012345678901234567890121234567890123456789012345678901212345
17. Which two y-intercepts could sum to zero? 6
12 3456789012345678901234567890121234567890123456789012345678901212345 6
12 3456789012345678901234567890121234567890123456789012345678901212345
Directions: Select the best answer for each 6
123456789012345678901234567890121234567890123456789012345678901212345
of the following questions. A. Line 1 and 4 6
12
2
3456789012345678901234567890121234567890123456789012345678901212345
3456789012345678901234567890121234567890123456789012345678901212345
6
6
1 B. Line 1 and 3
12 3456789012345678901234567890121234567890123456789012345678901212345 6
12 3456789012345678901234567890121234567890123456789012345678901212345
C. Line 2 and 3 6
12 3456789012345678901234567890121234567890123456789012345678901212345
15. 6
12
2
3456789012345678901234567890121234567890123456789012345678901212345 6
D. Line 2
3456789012345678901234567890121234567890123456789012345678901212345
and 4
6
1
12 3456789012345678901234567890121234567890123456789012345678901212345
E. Line 4 and 5 6
12 3456789012345678901234567890121234567890123456789012345678901212345 6
12 3456789012345678901234567890121234567890123456789012345678901212345 6
12 3456789012345678901234567890121234567890123456789012345678901212345
18. 6
1 3456789012345678901234567890121234567890123456789012345678901212345
2 6
12 3456789012345678901234567890121234567890123456789012345678901212345 6
12 3456789012345678901234567890121234567890123456789012345678901212345 6
12 3456789012345678901234567890121234567890123456789012345678901212345 6
12 3456789012345678901234567890121234567890123456789012345678901212345 6
1 3456789012345678901234567890121234567890123456789012345678901212345
2 The figures above are similar triangles. 6
12 3456789012345678901234567890121234567890123456789012345678901212345 6
12 3456789012345678901234567890121234567890123456789012345678901212345 6
What is the value of a?
12 3456789012345678901234567890121234567890123456789012345678901212345 6
12 3456789012345678901234567890121234567890123456789012345678901212345
5 6
123456789012345678901234567890121234567890123456789012345678901212345
2 6
123456789012345678901234567890121234567890123456789012345678901212345
A. 6
123456789012345678901234567890121234567890123456789012345678901212345
3 The two circles above have the same 6
123456789012345678901234567890121234567890123456789012345678901212345 6
123456789012345678901234567890121234567890123456789012345678901212345
B. 2 radius. Their circumference is 6p each. 6
3456789012345678901234567890121234567890123456789012345678901212345
123456789012345678901234567890121234567890123456789012345678901212345 6
1 C. 3 What is the distance between opposite 6
12 3456789012345678901234567890121234567890123456789012345678901212345 6
12 3456789012345678901234567890121234567890123456789012345678901212345
corners of the rectangle? 6
12 3456789012345678901234567890121234567890123456789012345678901212345
9 6
2 3456789012345678901234567890121234567890123456789012345678901212345
123456789012345678901234567890121234567890123456789012345678901212345
D. A. =160 6
123456789012345678901234567890121234567890123456789012345678901212345
2 6
123456789012345678901234567890121234567890123456789012345678901212345 66
123456789012345678901234567890121234567890123456789012345678901212345
E. 5
123456789012345678901234567890121234567890123456789012345678901212345
B. =165 6
123456789012345678901234567890121234567890123456789012345678901212345 66
123456789012345678901234567890121234567890123456789012345678901212345
123456789012345678901234567890121234567890123456789012345678901212345
C. =180 6
123456789012345678901234567890121234567890123456789012345678901212345
Questions 16–17 refer to the following graph. 6
1 6
2 3456789012345678901234567890121234567890123456789012345678901212345
123456789012345678901234567890121234567890123456789012345678901212345
D. =210 6
123456789012345678901234567890121234567890123456789012345678901212345 6
1 6
123456789012345678901234567890121234567890123456789012345678901212345
E. =220 6
12 3456789012345678901234567890121234567890123456789012345678901212345 6
123456789012345678901234567890121234567890123456789012345678901212345 66
2 3456789012345678901234567890121234567890123456789012345678901212345
123456789012345678901234567890121234567890123456789012345678901212345 6
1 19. A class has x number of students in it.
12 3456789012345678901234567890121234567890123456789012345678901212345 6
12 3456789012345678901234567890121234567890123456789012345678901212345
Three students join the class, and the class 6
2 3456789012345678901234567890121234567890123456789012345678901212345
123456789012345678901234567890121234567890123456789012345678901212345 6
123456789012345678901234567890121234567890123456789012345678901212345
size increases by 20 percent. How many 6
1 6
123456789012345678901234567890121234567890123456789012345678901212345
students are in the class now? 6
123456789012345678901234567890121234567890123456789012345678901212345 6
2 3456789012345678901234567890121234567890123456789012345678901212345
123456789012345678901234567890121234567890123456789012345678901212345 6
123456789012345678901234567890121234567890123456789012345678901212345
A. 12 6
123456789012345678901234567890121234567890123456789012345678901212345 6
123456789012345678901234567890121234567890123456789012345678901212345 66
B. 15
1
12 3456789012345678901234567890121234567890123456789012345678901212345
C. 18 6
12 3456789012345678901234567890121234567890123456789012345678901212345 6
12 3456789012345678901234567890121234567890123456789012345678901212345 6
D. 20
12 3456789012345678901234567890121234567890123456789012345678901212345 6
123456789012345678901234567890121234567890123456789012345678901212345
E. 21 6
123456789012345678901234567890121234567890123456789012345678901212345 66
2 3456789012345678901234567890121234567890123456789012345678901212345
16. Which line has the greatest slope?
123456789012345678901234567890121234567890123456789012345678901212345 6
123456789012345678901234567890121234567890123456789012345678901212345
A. Line 1
6
123456789012345678901234567890121234567890123456789012345678901212345 6
1 B. Line 2
123456789012345678901234567890121234567890123456789012345678901212345 66
2 3456789012345678901234567890121234567890123456789012345678901212345
1 C. Line 3
12 3456789012345678901234567890121234567890123456789012345678901212345 6
123456789012345678901234567890121234567890123456789012345678901212345
D. Line 4 6
12
2
3456789012345678901234567890121234567890123456789012345678901212345
3456789012345678901234567890121234567890123456789012345678901212345
6
6
1 E. Line 5
123456789012345678901234567890121234567890123456789012345678901212345 6
123456789012345678901234567890121234567890123456789012345678901212345 66
308 123456789012345678901234567890121234567890123456789012345678901212345
1234567890123456789012345678901212345678901234567890123456789012123456

www.petersons.com
TEST 3
1234567890123456789012345678901212345678901234567890123456789012123456
123456789012345678901234567890121234567890123456789012345678901212345 6
12 3456789012345678901234567890121234567890123456789012345678901212345 6
123456789012345678901234567890121234567890123456789012345678901212345
20. A square has a side of length 4. What is Questions 23–25 refers to the following graph. 6
12 3456789012345678901234567890121234567890123456789012345678901212345 6
12 3456789012345678901234567890121234567890123456789012345678901212345
the distance from the center of the square 6
123456789012345678901234567890121234567890123456789012345678901212345 6
12
2
3456789012345678901234567890121234567890123456789012345678901212345 6
to one of its corners?
3456789012345678901234567890121234567890123456789012345678901212345 6
1
12 3456789012345678901234567890121234567890123456789012345678901212345 6
12 3456789012345678901234567890121234567890123456789012345678901212345
A. = 2 6
12 3456789012345678901234567890121234567890123456789012345678901212345 6
z
12 3456789012345678901234567890121234567890123456789012345678901212345 6
123456789012345678901234567890121234567890123456789012345678901212345
B. 2=2 6
12 3456789012345678901234567890121234567890123456789012345678901212345 6
12 3456789012345678901234567890121234567890123456789012345678901212345 6
12 3456789012345678901234567890121234567890123456789012345678901212345
C. 4=2 6
12
2
3456789012345678901234567890121234567890123456789012345678901212345
3456789012345678901234567890121234567890123456789012345678901212345
6
6
1
12 3456789012345678901234567890121234567890123456789012345678901212345
D. 6=2 6
12 3456789012345678901234567890121234567890123456789012345678901212345 6
12 3456789012345678901234567890121234567890123456789012345678901212345 6
12 3456789012345678901234567890121234567890123456789012345678901212345
E. 8=2 6
1 3456789012345678901234567890121234567890123456789012345678901212345
2 6
12 3456789012345678901234567890121234567890123456789012345678901212345 6
12 3456789012345678901234567890121234567890123456789012345678901212345 6
12 3456789012345678901234567890121234567890123456789012345678901212345 6
12 3456789012345678901234567890121234567890123456789012345678901212345
21. If 3 pounds of peanuts, which cost 34 6
2 3456789012345678901234567890121234567890123456789012345678901212345
123456789012345678901234567890121234567890123456789012345678901212345 6
123456789012345678901234567890121234567890123456789012345678901212345
cents per pound, are mixed with 5 pounds 6
123456789012345678901234567890121234567890123456789012345678901212345
of cashews, which cost 90 cents per 6
123456789012345678901234567890121234567890123456789012345678901212345 66
123456789012345678901234567890121234567890123456789012345678901212345
pound, how many cents per pound is the
123456789012345678901234567890121234567890123456789012345678901212345 6
1 mixture of the two? 6
12 3456789012345678901234567890121234567890123456789012345678901212345 6
12 3456789012345678901234567890121234567890123456789012345678901212345 6
12 3456789012345678901234567890121234567890123456789012345678901212345
A. 67 23. According to the table, what is the 6
2 3456789012345678901234567890121234567890123456789012345678901212345
123456789012345678901234567890121234567890123456789012345678901212345 6
123456789012345678901234567890121234567890123456789012345678901212345
B. 69 relationship between the frequency and 6
123456789012345678901234567890121234567890123456789012345678901212345 6
123456789012345678901234567890121234567890123456789012345678901212345
C. 71 the energy level of electromagnetic 6
123456789012345678901234567890121234567890123456789012345678901212345 6
123456789012345678901234567890121234567890123456789012345678901212345
D. 73 radiation? 6
123456789012345678901234567890121234567890123456789012345678901212345 6
123456789012345678901234567890121234567890123456789012345678901212345
E. 75
A. As one increases, the other increases. 6
6
123456789012345678901234567890121234567890123456789012345678901212345
1
12 B. As one increases, the other decreases. 6
3456789012345678901234567890121234567890123456789012345678901212345
22. A board is 6 feet long, 6 inches wide, and 6
12 3456789012345678901234567890121234567890123456789012345678901212345 6
123456789012345678901234567890121234567890123456789012345678901212345
2 inches tall. What is its surface area in C. There is no direct relationship. 6
123456789012345678901234567890121234567890123456789012345678901212345 6
12
2
3456789012345678901234567890121234567890123456789012345678901212345
feet squared? (One foot is 12 inches.) D. The energy
3456789012345678901234567890121234567890123456789012345678901212345
level is twice as much as 6
6
123456789012345678901234567890121234567890123456789012345678901212345
123456789012345678901234567890121234567890123456789012345678901212345
the frequency level. 6
1 11 6
12 3456789012345678901234567890121234567890123456789012345678901212345
A. E. The frequency level is twice as much 6
12 3456789012345678901234567890121234567890123456789012345678901212345
2 6
123456789012345678901234567890121234567890123456789012345678901212345 66
2 3456789012345678901234567890121234567890123456789012345678901212345
as the wavelength level.
123456789012345678901234567890121234567890123456789012345678901212345
1 6
1
123456789012345678901234567890121234567890123456789012345678901212345
B. 6 24. Which kind of radiation has the shortest 6
123456789012345678901234567890121234567890123456789012345678901212345
2 6
2 3456789012345678901234567890121234567890123456789012345678901212345
123456789012345678901234567890121234567890123456789012345678901212345
wavelength? 6
123456789012345678901234567890121234567890123456789012345678901212345
C. 7 6
123456789012345678901234567890121234567890123456789012345678901212345 6
123456789012345678901234567890121234567890123456789012345678901212345
A. Gamma Rays 6
1 49 6
2 3456789012345678901234567890121234567890123456789012345678901212345
123456789012345678901234567890121234567890123456789012345678901212345
D. B. X-rays 6
123456789012345678901234567890121234567890123456789012345678901212345
6 6
123456789012345678901234567890121234567890123456789012345678901212345
C. Ultra-violet 6
123456789012345678901234567890121234567890123456789012345678901212345
E. 9 6
1 D. Infrared 6
2 3456789012345678901234567890121234567890123456789012345678901212345
123456789012345678901234567890121234567890123456789012345678901212345 6
123456789012345678901234567890121234567890123456789012345678901212345
E. Radio 6
123456789012345678901234567890121234567890123456789012345678901212345 66
123456789012345678901234567890121234567890123456789012345678901212345 6
1
12 3456789012345678901234567890121234567890123456789012345678901212345 6
123456789012345678901234567890121234567890123456789012345678901212345 6
12 3456789012345678901234567890121234567890123456789012345678901212345 6
123456789012345678901234567890121234567890123456789012345678901212345 6
12 3456789012345678901234567890121234567890123456789012345678901212345 6
123456789012345678901234567890121234567890123456789012345678901212345 6
123456789012345678901234567890121234567890123456789012345678901212345 6
123456789012345678901234567890121234567890123456789012345678901212345 6
123456789012345678901234567890121234567890123456789012345678901212345 6 309
1234567890123456789012345678901212345678901234567890123456789012123456
Part IV: Three Practice GREs

1234567890123456789012345678901212345678901234567890123456789012123456
123456789012345678901234567890121234567890123456789012345678901212345 6
12 3456789012345678901234567890121234567890123456789012345678901212345 6
123456789012345678901234567890121234567890123456789012345678901212345
25. If electromagnetic radiation has a fre- 27. How many liters of a solution that is 20 6
12 3456789012345678901234567890121234567890123456789012345678901212345 6
12 3456789012345678901234567890121234567890123456789012345678901212345
22
quency of 10 Hz, what kind of radiation percent water by volume must be added to 6
123456789012345678901234567890121234567890123456789012345678901212345 6
12
2
3456789012345678901234567890121234567890123456789012345678901212345
is it? 3 liters of a solution that is 40 percent
3456789012345678901234567890121234567890123456789012345678901212345
6
6
1
12 3456789012345678901234567890121234567890123456789012345678901212345
water by volume to create a solution that 6
12 3456789012345678901234567890121234567890123456789012345678901212345
A. Gamma Rays 6
12 3456789012345678901234567890121234567890123456789012345678901212345
is 25 percent water by volume? 6
12
2
3456789012345678901234567890121234567890123456789012345678901212345 6
B. X-rays
3456789012345678901234567890121234567890123456789012345678901212345 6
1
12 3456789012345678901234567890121234567890123456789012345678901212345
C. Ultra-violet A. 5 6
12 3456789012345678901234567890121234567890123456789012345678901212345 6
12 3456789012345678901234567890121234567890123456789012345678901212345 6
D. Infrared B. 6
12 3456789012345678901234567890121234567890123456789012345678901212345 6
123456789012345678901234567890121234567890123456789012345678901212345
E. Radio C. 7 6
12 3456789012345678901234567890121234567890123456789012345678901212345 6
12 3456789012345678901234567890121234567890123456789012345678901212345 6
D. 8
12 3456789012345678901234567890121234567890123456789012345678901212345
26. If a, b, and c are all positive odd integers, E. 9 6
12
2
3456789012345678901234567890121234567890123456789012345678901212345
3456789012345678901234567890121234567890123456789012345678901212345
6
6
1 then which of the following must be odd?
12 3456789012345678901234567890121234567890123456789012345678901212345 6
12 3456789012345678901234567890121234567890123456789012345678901212345
28. There are 150 students in math courses at 6
12 3456789012345678901234567890121234567890123456789012345678901212345
I. ab 2 bc 6
12 3456789012345678901234567890121234567890123456789012345678901212345
Kronhorst High School: 73 are in geom- 6
2 3456789012345678901234567890121234567890123456789012345678901212345
123456789012345678901234567890121234567890123456789012345678901212345
II. abc 6
123456789012345678901234567890121234567890123456789012345678901212345
etry, 62 are in algebra, and 52 are in 6
123456789012345678901234567890121234567890123456789012345678901212345 6
2
III. c 1 c
123456789012345678901234567890121234567890123456789012345678901212345 66
neither. How many students are in both
123456789012345678901234567890121234567890123456789012345678901212345
123456789012345678901234567890121234567890123456789012345678901212345
A. I only geometry and algebra? 6
1 6
12 3456789012345678901234567890121234567890123456789012345678901212345
B. II only 6
12 3456789012345678901234567890121234567890123456789012345678901212345
C. I and II only
A. 32 6
12 3456789012345678901234567890121234567890123456789012345678901212345 6
2 3456789012345678901234567890121234567890123456789012345678901212345
123456789012345678901234567890121234567890123456789012345678901212345
D. I and III only
B. 33 6
123456789012345678901234567890121234567890123456789012345678901212345 6
123456789012345678901234567890121234567890123456789012345678901212345
E. I, II, and III
C. 35 6
123456789012345678901234567890121234567890123456789012345678901212345
D. 36 6
123456789012345678901234567890121234567890123456789012345678901212345 66
123456789012345678901234567890121234567890123456789012345678901212345
E. 37
123456789012345678901234567890121234567890123456789012345678901212345 6
123456789012345678901234567890121234567890123456789012345678901212345 66
123456789012345678901234567890121234567890123456789012345678901212345 6
1 Verbal
12 3456789012345678901234567890121234567890123456789012345678901212345 6
12 3456789012345678901234567890121234567890123456789012345678901212345 6
123456789012345678901234567890121234567890123456789012345678901212345 6
123456789012345678901234567890121234567890123456789012345678901212345
30 Questions—30 Minutes 6
12
2
3456789012345678901234567890121234567890123456789012345678901212345 6
3456789012345678901234567890121234567890123456789012345678901212345 6
123456789012345678901234567890121234567890123456789012345678901212345
123456789012345678901234567890121234567890123456789012345678901212345
Antonyms 6
1 6
12 3456789012345678901234567890121234567890123456789012345678901212345 6
12 3456789012345678901234567890121234567890123456789012345678901212345 6
12 3456789012345678901234567890121234567890123456789012345678901212345
Directions: Each question contains a word printed in capital letters, followed by five 6
12 3456789012345678901234567890121234567890123456789012345678901212345 6
123456789012345678901234567890121234567890123456789012345678901212345 6
answer choices. Choose the answer choice that contains the word or phrase most nearly
123456789012345678901234567890121234567890123456789012345678901212345 6
123456789012345678901234567890121234567890123456789012345678901212345
OPPOSITE in meaning to the word in capital letters. 6
123456789012345678901234567890121234567890123456789012345678901212345 66
2 3456789012345678901234567890121234567890123456789012345678901212345
123456789012345678901234567890121234567890123456789012345678901212345 6
123456789012345678901234567890121234567890123456789012345678901212345 6
123456789012345678901234567890121234567890123456789012345678901212345 6
1 1. CORROBORATE :
12 3456789012345678901234567890121234567890123456789012345678901212345
2. INEXACTITUDE : 6
12 3456789012345678901234567890121234567890123456789012345678901212345 6
12 3456789012345678901234567890121234567890123456789012345678901212345 6
12 3456789012345678901234567890121234567890123456789012345678901212345
A. pledge A. crude 6
123456789012345678901234567890121234567890123456789012345678901212345 6
123456789012345678901234567890121234567890123456789012345678901212345 66
2 3456789012345678901234567890121234567890123456789012345678901212345
B. deny B. precise
123456789012345678901234567890121234567890123456789012345678901212345
C. harbor C. inexpert 6
123456789012345678901234567890121234567890123456789012345678901212345 6
123456789012345678901234567890121234567890123456789012345678901212345
D. detain D. industrious
6
1
2 3456789012345678901234567890121234567890123456789012345678901212345
123456789012345678901234567890121234567890123456789012345678901212345
E. affirm E. articulate 6
1 6
12 3456789012345678901234567890121234567890123456789012345678901212345 6
123456789012345678901234567890121234567890123456789012345678901212345 6
12 3456789012345678901234567890121234567890123456789012345678901212345 6
123456789012345678901234567890121234567890123456789012345678901212345 6
123456789012345678901234567890121234567890123456789012345678901212345 6
123456789012345678901234567890121234567890123456789012345678901212345 66
310 123456789012345678901234567890121234567890123456789012345678901212345
1234567890123456789012345678901212345678901234567890123456789012123456

www.petersons.com
TEST 3
1234567890123456789012345678901212345678901234567890123456789012123456
123456789012345678901234567890121234567890123456789012345678901212345 6
12 3456789012345678901234567890121234567890123456789012345678901212345 6
123456789012345678901234567890121234567890123456789012345678901212345
3. FOMENT : 6. PULCHRITUDE : 6
12 3456789012345678901234567890121234567890123456789012345678901212345 6
12 3456789012345678901234567890121234567890123456789012345678901212345 6
123456789012345678901234567890121234567890123456789012345678901212345
A. quell A. ostentatious 6
12
2
3456789012345678901234567890121234567890123456789012345678901212345
3456789012345678901234567890121234567890123456789012345678901212345
6
6
1 B. agitate B. elegant
12 3456789012345678901234567890121234567890123456789012345678901212345 6
12 3456789012345678901234567890121234567890123456789012345678901212345 6
C. appropriate C. estranged
12 3456789012345678901234567890121234567890123456789012345678901212345 6
12
2
3456789012345678901234567890121234567890123456789012345678901212345
D. estimate D. comedic
3456789012345678901234567890121234567890123456789012345678901212345
6
6
1
12 3456789012345678901234567890121234567890123456789012345678901212345
E. exonerate E. uncomely 6
12 3456789012345678901234567890121234567890123456789012345678901212345 6
12 3456789012345678901234567890121234567890123456789012345678901212345 6
12 3456789012345678901234567890121234567890123456789012345678901212345
4. CALLOW : 7. STOLID : 6
1 3456789012345678901234567890121234567890123456789012345678901212345
2 6
12 3456789012345678901234567890121234567890123456789012345678901212345 6
12 3456789012345678901234567890121234567890123456789012345678901212345
A. picturesque A. imaginative 6
12 3456789012345678901234567890121234567890123456789012345678901212345 6
12
2
3456789012345678901234567890121234567890123456789012345678901212345 6
B. mature B. sterile
3456789012345678901234567890121234567890123456789012345678901212345 6
1
12 3456789012345678901234567890121234567890123456789012345678901212345
C. timid C. venal 6
12 3456789012345678901234567890121234567890123456789012345678901212345 6
12 3456789012345678901234567890121234567890123456789012345678901212345 6
D. canny D. emotional
12 3456789012345678901234567890121234567890123456789012345678901212345 6
12 3456789012345678901234567890121234567890123456789012345678901212345
E. particular E. abrasive 6
12 3456789012345678901234567890121234567890123456789012345678901212345 6
12 3456789012345678901234567890121234567890123456789012345678901212345 6
12 3456789012345678901234567890121234567890123456789012345678901212345
5. BACCHANALIAN : 8. PREVARICATE : 6
12 3456789012345678901234567890121234567890123456789012345678901212345 6
123456789012345678901234567890121234567890123456789012345678901212345 66
2 3456789012345678901234567890121234567890123456789012345678901212345
1 A. petulant A. speak honestly
12 3456789012345678901234567890121234567890123456789012345678901212345 6
12 3456789012345678901234567890121234567890123456789012345678901212345
B. orgiastic B. speak hastily 6
12 3456789012345678901234567890121234567890123456789012345678901212345 6
123456789012345678901234567890121234567890123456789012345678901212345 66
2 3456789012345678901234567890121234567890123456789012345678901212345
C. analgesia C. speak slowly
123456789012345678901234567890121234567890123456789012345678901212345
123456789012345678901234567890121234567890123456789012345678901212345
D. pious D. speak with vulgarities 6
123456789012345678901234567890121234567890123456789012345678901212345 6
123456789012345678901234567890121234567890123456789012345678901212345 66
E. shrewd E. speak with pathos
123456789012345678901234567890121234567890123456789012345678901212345 6
123456789012345678901234567890121234567890123456789012345678901212345 6
123456789012345678901234567890121234567890123456789012345678901212345 6
123456789012345678901234567890121234567890123456789012345678901212345
Analogies
6
1
123456789012345678901234567890121234567890123456789012345678901212345 66
2 3456789012345678901234567890121234567890123456789012345678901212345
123456789012345678901234567890121234567890123456789012345678901212345 6
1 Directions: In each question, there is an initial pair of words or phrases that are related in
123456789012345678901234567890121234567890123456789012345678901212345 6
12 3456789012345678901234567890121234567890123456789012345678901212345
some manner. Select the answer choice containing the lettered pair that best expresses a 6
123456789012345678901234567890121234567890123456789012345678901212345 66
2 3456789012345678901234567890121234567890123456789012345678901212345
123456789012345678901234567890121234567890123456789012345678901212345
relationship similar to the initial pair.
6
1
12 3456789012345678901234567890121234567890123456789012345678901212345 6
12 3456789012345678901234567890121234567890123456789012345678901212345 6
12 3456789012345678901234567890121234567890123456789012345678901212345 6
12 3456789012345678901234567890121234567890123456789012345678901212345 6
123456789012345678901234567890121234567890123456789012345678901212345
9. LONG-WINDED : BREVITY :: 11. CANDID : FRANK 6
123456789012345678901234567890121234567890123456789012345678901212345 6
123456789012345678901234567890121234567890123456789012345678901212345 6
123456789012345678901234567890121234567890123456789012345678901212345 66
2 3456789012345678901234567890121234567890123456789012345678901212345
A. oversized : large A. sincere : kind
123456789012345678901234567890121234567890123456789012345678901212345 6
123456789012345678901234567890121234567890123456789012345678901212345
B. attractive : repulsive B. erudite : learned
6
123456789012345678901234567890121234567890123456789012345678901212345 6
1 C. lean : excess C. duplicitous: timorous
2 3456789012345678901234567890121234567890123456789012345678901212345
123456789012345678901234567890121234567890123456789012345678901212345 6
123456789012345678901234567890121234567890123456789012345678901212345
D. wealthy : money D. pernicious : hopeful 6
123456789012345678901234567890121234567890123456789012345678901212345 6
123456789012345678901234567890121234567890123456789012345678901212345 66
E. spent : used E. measured : mercurial
1
12 3456789012345678901234567890121234567890123456789012345678901212345 6
12 3456789012345678901234567890121234567890123456789012345678901212345
10. LAWYER : COURT :: 12. AUSTERE : DECORATED :: 6
12 3456789012345678901234567890121234567890123456789012345678901212345 6
12 3456789012345678901234567890121234567890123456789012345678901212345 6
123456789012345678901234567890121234567890123456789012345678901212345
A. actor : play A. paltry : exorbitant 6
12 3456789012345678901234567890121234567890123456789012345678901212345 6
123456789012345678901234567890121234567890123456789012345678901212345
B. janitor : school B. offensive : tactful 6
12 3456789012345678901234567890121234567890123456789012345678901212345 6
123456789012345678901234567890121234567890123456789012345678901212345
C. manager : team C. ingenious : creative 6
12
2
3456789012345678901234567890121234567890123456789012345678901212345
3456789012345678901234567890121234567890123456789012345678901212345
6
6
1 D. prison : guard D. lackluster : ominous
123456789012345678901234567890121234567890123456789012345678901212345 6
123456789012345678901234567890121234567890123456789012345678901212345
E. bureaucrat : bureaucracy E. powerful : elusive 6
123456789012345678901234567890121234567890123456789012345678901212345 6 311
1234567890123456789012345678901212345678901234567890123456789012123456
Part IV: Three Practice GREs

1234567890123456789012345678901212345678901234567890123456789012123456
123456789012345678901234567890121234567890123456789012345678901212345 6
12 3456789012345678901234567890121234567890123456789012345678901212345 6
123456789012345678901234567890121234567890123456789012345678901212345
13. PHYSICIAN : HUMAN BODY :: 15. TURGID : BOMBASTIC :: 6
12 3456789012345678901234567890121234567890123456789012345678901212345 6
12 3456789012345678901234567890121234567890123456789012345678901212345 6
123456789012345678901234567890121234567890123456789012345678901212345
A. horticulturist : plant A. lazy : lethargic 6
12
2
3456789012345678901234567890121234567890123456789012345678901212345
3456789012345678901234567890121234567890123456789012345678901212345
6
6
1 B. coach : player B. rotund : portly
12 3456789012345678901234567890121234567890123456789012345678901212345 6
12 3456789012345678901234567890121234567890123456789012345678901212345 6
C. teacher : student C. bemused : distract
12 3456789012345678901234567890121234567890123456789012345678901212345 6
12
2
3456789012345678901234567890121234567890123456789012345678901212345
D. veterinarian : cow D. catatonic : cataclysmic
3456789012345678901234567890121234567890123456789012345678901212345
6
6
1
12 3456789012345678901234567890121234567890123456789012345678901212345
E. biologist : evolution E. histrionic : sordid 6
12 3456789012345678901234567890121234567890123456789012345678901212345 6
12 3456789012345678901234567890121234567890123456789012345678901212345 6
12 3456789012345678901234567890121234567890123456789012345678901212345
14. CRAVEN : TREPIDATIOUS :: 6
1 3456789012345678901234567890121234567890123456789012345678901212345
2 6
12 3456789012345678901234567890121234567890123456789012345678901212345 6
12 3456789012345678901234567890121234567890123456789012345678901212345
A. miniscule : minute 6
12 3456789012345678901234567890121234567890123456789012345678901212345 6
12
2
3456789012345678901234567890121234567890123456789012345678901212345 6
B. migratory : transitory
3456789012345678901234567890121234567890123456789012345678901212345 6
1
12 3456789012345678901234567890121234567890123456789012345678901212345
C. ageless : ancient 6
12 3456789012345678901234567890121234567890123456789012345678901212345 6
12 3456789012345678901234567890121234567890123456789012345678901212345 6
D. agile : acute
12 3456789012345678901234567890121234567890123456789012345678901212345 6
12 3456789012345678901234567890121234567890123456789012345678901212345
E. exultant : joyful 6
12 3456789012345678901234567890121234567890123456789012345678901212345 6
12 3456789012345678901234567890121234567890123456789012345678901212345 6
12 3456789012345678901234567890121234567890123456789012345678901212345 6
12 3456789012345678901234567890121234567890123456789012345678901212345
Sentence Completions 6
123456789012345678901234567890121234567890123456789012345678901212345
2 6
1 3456789012345678901234567890121234567890123456789012345678901212345 6
12 3456789012345678901234567890121234567890123456789012345678901212345 6
12 3456789012345678901234567890121234567890123456789012345678901212345
Directions: Each sentence below contains one or two blanks, with each blank corre- 6
12 3456789012345678901234567890121234567890123456789012345678901212345 6
2 3456789012345678901234567890121234567890123456789012345678901212345
123456789012345678901234567890121234567890123456789012345678901212345
sponding to an omitted word. Find the answer choice that has the word or set or words 6
123456789012345678901234567890121234567890123456789012345678901212345 6
123456789012345678901234567890121234567890123456789012345678901212345
that best fits the meaning of the sentence as a whole. 6
123456789012345678901234567890121234567890123456789012345678901212345 66
123456789012345678901234567890121234567890123456789012345678901212345 6
123456789012345678901234567890121234567890123456789012345678901212345 6
123456789012345678901234567890121234567890123456789012345678901212345 6
123456789012345678901234567890121234567890123456789012345678901212345
18. The publicity surrounding the mismanage- 6
123456789012345678901234567890121234567890123456789012345678901212345
16. The demolition crew did not successfully
6
1
2 3456789012345678901234567890121234567890123456789012345678901212345
the building since two walls ment of the company’s capital __________ 6
123456789012345678901234567890121234567890123456789012345678901212345
__________
123456789012345678901234567890121234567890123456789012345678901212345 6
1 remained intact. the company’s credibility with investors, 6
123456789012345678901234567890121234567890123456789012345678901212345 6
12
2
3456789012345678901234567890121234567890123456789012345678901212345
but the company
3456789012345678901234567890121234567890123456789012345678901212345
remained solvent largely 6
6
123456789012345678901234567890121234567890123456789012345678901212345
A. raze
123456789012345678901234567890121234567890123456789012345678901212345
on account of its __________ market 6
1 B. construct 6
12 3456789012345678901234567890121234567890123456789012345678901212345
share. 6
12 3456789012345678901234567890121234567890123456789012345678901212345 6
C. erect
2 3456789012345678901234567890121234567890123456789012345678901212345
123456789012345678901234567890121234567890123456789012345678901212345 6
123456789012345678901234567890121234567890123456789012345678901212345
D. incapacitate A. belabored. .steady 6
1 6
123456789012345678901234567890121234567890123456789012345678901212345 6
E. deactivate B. weakened. .evolving
123456789012345678901234567890121234567890123456789012345678901212345 6
12 3456789012345678901234567890121234567890123456789012345678901212345
C. aided. .climbing 6
12 3456789012345678901234567890121234567890123456789012345678901212345
17. The ethicist argued that the person in D. lessened. .volatile 6
12 3456789012345678901234567890121234567890123456789012345678901212345 6
12 3456789012345678901234567890121234567890123456789012345678901212345
question was not __________ for the E. damaged. .stable 6
123456789012345678901234567890121234567890123456789012345678901212345 6
123456789012345678901234567890121234567890123456789012345678901212345 66
2 3456789012345678901234567890121234567890123456789012345678901212345
man’s injury since it was an accident.
123456789012345678901234567890121234567890123456789012345678901212345 6
123456789012345678901234567890121234567890123456789012345678901212345 6
123456789012345678901234567890121234567890123456789012345678901212345
A. culpable
6
1
123456789012345678901234567890121234567890123456789012345678901212345 66
2 3456789012345678901234567890121234567890123456789012345678901212345
B. attributable
123456789012345678901234567890121234567890123456789012345678901212345 6
123456789012345678901234567890121234567890123456789012345678901212345
C. capable
6
123456789012345678901234567890121234567890123456789012345678901212345 6
1 D. reasonable
2 3456789012345678901234567890121234567890123456789012345678901212345
123456789012345678901234567890121234567890123456789012345678901212345 6
1 E. actionable 6
12 3456789012345678901234567890121234567890123456789012345678901212345 6
123456789012345678901234567890121234567890123456789012345678901212345 6
12 3456789012345678901234567890121234567890123456789012345678901212345 6
123456789012345678901234567890121234567890123456789012345678901212345 6
123456789012345678901234567890121234567890123456789012345678901212345 6
123456789012345678901234567890121234567890123456789012345678901212345 66
312 123456789012345678901234567890121234567890123456789012345678901212345
1234567890123456789012345678901212345678901234567890123456789012123456

www.petersons.com
TEST 3
1234567890123456789012345678901212345678901234567890123456789012123456
123456789012345678901234567890121234567890123456789012345678901212345 6
12 3456789012345678901234567890121234567890123456789012345678901212345 6
123456789012345678901234567890121234567890123456789012345678901212345
19. Though the official had hoped his speech 22. Though the glass was __________ when 6
12 3456789012345678901234567890121234567890123456789012345678901212345 6
12 3456789012345678901234567890121234567890123456789012345678901212345
would calm the citizenry, his assurances bought, years of poor maintenance had 6
123456789012345678901234567890121234567890123456789012345678901212345 6
12
2
3456789012345678901234567890121234567890123456789012345678901212345 6
did not ________ the crowd’s fears rendered
3456789012345678901234567890121234567890123456789012345678901212345
it nearly __________, stifling
6
1
12 3456789012345678901234567890121234567890123456789012345678901212345
concerning the likelihood of the natural almost all of the sunlight that strained to 6
12 3456789012345678901234567890121234567890123456789012345678901212345 6
12 3456789012345678901234567890121234567890123456789012345678901212345 6
disaster recurring. cross its interior.
12 3456789012345678901234567890121234567890123456789012345678901212345 6
123456789012345678901234567890121234567890123456789012345678901212345 6
12 3456789012345678901234567890121234567890123456789012345678901212345
A. alter A. permeable. .dark 6
12 3456789012345678901234567890121234567890123456789012345678901212345 6
12 3456789012345678901234567890121234567890123456789012345678901212345 6
B. assuage B. clear. .obscure
12 3456789012345678901234567890121234567890123456789012345678901212345 6
123456789012345678901234567890121234567890123456789012345678901212345
C. increase C. lucid. .obstructed 6
12 3456789012345678901234567890121234567890123456789012345678901212345 6
12 3456789012345678901234567890121234567890123456789012345678901212345 6
D. bolster D. penetrable. .impregnable
12 3456789012345678901234567890121234567890123456789012345678901212345
E. gauge E. translucent. .opaque 6
12
2
3456789012345678901234567890121234567890123456789012345678901212345
3456789012345678901234567890121234567890123456789012345678901212345
6
6
1
12 3456789012345678901234567890121234567890123456789012345678901212345 6
12 3456789012345678901234567890121234567890123456789012345678901212345
20. It is a commonly noted behavior for a 23. Historiography, at least of historic 6
12 3456789012345678901234567890121234567890123456789012345678901212345 6
12 3456789012345678901234567890121234567890123456789012345678901212345 6
younger sibling to ________ the behavior personages, in the last century tended
2 3456789012345678901234567890121234567890123456789012345678901212345
123456789012345678901234567890121234567890123456789012345678901212345 6
123456789012345678901234567890121234567890123456789012345678901212345
of an older sibling, taking on their toward hagiography, but today 6
123456789012345678901234567890121234567890123456789012345678901212345 6
123456789012345678901234567890121234567890123456789012345678901212345 66
mannerisms or sense of style. __________ distance and a __________
123456789012345678901234567890121234567890123456789012345678901212345
123456789012345678901234567890121234567890123456789012345678901212345
tone are basic requirements of profes- 6
1 A. antagonize 6
12 3456789012345678901234567890121234567890123456789012345678901212345
sional history writing. 6
12 3456789012345678901234567890121234567890123456789012345678901212345 6
B. ridicule
12 3456789012345678901234567890121234567890123456789012345678901212345 6
12 3456789012345678901234567890121234567890123456789012345678901212345
C. improve A. critical. .reserved 6
12 3456789012345678901234567890121234567890123456789012345678901212345 6
12 3456789012345678901234567890121234567890123456789012345678901212345 6
D. emulate B. minute. .extravagant
12 3456789012345678901234567890121234567890123456789012345678901212345
E. vex C. impartial. .subdued 6
12 3456789012345678901234567890121234567890123456789012345678901212345 6
2 3456789012345678901234567890121234567890123456789012345678901212345
123456789012345678901234567890121234567890123456789012345678901212345
D. holistic. .daring 6
123456789012345678901234567890121234567890123456789012345678901212345 6
123456789012345678901234567890121234567890123456789012345678901212345
21. Since the visitor was unfamiliar with the E. even. .careful 6
123456789012345678901234567890121234567890123456789012345678901212345 6
1 region’s customs he was unaware of the 6
2 3456789012345678901234567890121234567890123456789012345678901212345
123456789012345678901234567890121234567890123456789012345678901212345 6
123456789012345678901234567890121234567890123456789012345678901212345
__________ of his actions, and so he was 6
1 6
123456789012345678901234567890121234567890123456789012345678901212345 6
__________ to the fact that he had
12 3456789012345678901234567890121234567890123456789012345678901212345 6
12 3456789012345678901234567890121234567890123456789012345678901212345
offended his host. 6
12 3456789012345678901234567890121234567890123456789012345678901212345 6
123456789012345678901234567890121234567890123456789012345678901212345 6
12 3456789012345678901234567890121234567890123456789012345678901212345
A. impropriety. .oblivious 6
12 3456789012345678901234567890121234567890123456789012345678901212345 6
123456789012345678901234567890121234567890123456789012345678901212345 66
2 3456789012345678901234567890121234567890123456789012345678901212345
B. curtness. .ignorant
123456789012345678901234567890121234567890123456789012345678901212345
C. blandness. .blithe 6
1
123456789012345678901234567890121234567890123456789012345678901212345
D. boldness. .unaware 6
123456789012345678901234567890121234567890123456789012345678901212345 6
2 3456789012345678901234567890121234567890123456789012345678901212345
123456789012345678901234567890121234567890123456789012345678901212345
E. audacity. .blind 6
123456789012345678901234567890121234567890123456789012345678901212345 6
123456789012345678901234567890121234567890123456789012345678901212345 66
123456789012345678901234567890121234567890123456789012345678901212345 6
1
12 3456789012345678901234567890121234567890123456789012345678901212345 6
12 3456789012345678901234567890121234567890123456789012345678901212345 6
12 3456789012345678901234567890121234567890123456789012345678901212345 6
12 3456789012345678901234567890121234567890123456789012345678901212345 6
123456789012345678901234567890121234567890123456789012345678901212345 6
123456789012345678901234567890121234567890123456789012345678901212345
2 6
123456789012345678901234567890121234567890123456789012345678901212345 6
123456789012345678901234567890121234567890123456789012345678901212345 6
123456789012345678901234567890121234567890123456789012345678901212345 6
1 3456789012345678901234567890121234567890123456789012345678901212345 6
123456789012345678901234567890121234567890123456789012345678901212345 66
2 3456789012345678901234567890121234567890123456789012345678901212345
1
12 3456789012345678901234567890121234567890123456789012345678901212345 6
123456789012345678901234567890121234567890123456789012345678901212345 6
12 3456789012345678901234567890121234567890123456789012345678901212345 6
123456789012345678901234567890121234567890123456789012345678901212345 6
123456789012345678901234567890121234567890123456789012345678901212345 6
123456789012345678901234567890121234567890123456789012345678901212345 6
123456789012345678901234567890121234567890123456789012345678901212345 6 313
1234567890123456789012345678901212345678901234567890123456789012123456
Part IV: Three Practice GREs

1234567890123456789012345678901212345678901234567890123456789012123456
123456789012345678901234567890121234567890123456789012345678901212345 6
12 3456789012345678901234567890121234567890123456789012345678901212345 6
123456789012345678901234567890121234567890123456789012345678901212345
Reading Comprehension 6
12 3456789012345678901234567890121234567890123456789012345678901212345 6
12 3456789012345678901234567890121234567890123456789012345678901212345 6
123456789012345678901234567890121234567890123456789012345678901212345 6
12 3456789012345678901234567890121234567890123456789012345678901212345
Directions: Questions follow each passage. After reading the passage, determine the best 6
123456789012345678901234567890121234567890123456789012345678901212345 6
12 3456789012345678901234567890121234567890123456789012345678901212345
answer for each question based on the passage’s content. 6
12 3456789012345678901234567890121234567890123456789012345678901212345 6
12 3456789012345678901234567890121234567890123456789012345678901212345 6
12 3456789012345678901234567890121234567890123456789012345678901212345 6
1 3456789012345678901234567890121234567890123456789012345678901212345 6
2
12 3456789012345678901234567890121234567890123456789012345678901212345 6
12 3456789012345678901234567890121234567890123456789012345678901212345
Passage 1 tance of the components neglected by the 6
12 3456789012345678901234567890121234567890123456789012345678901212345 6
12
2
3456789012345678901234567890121234567890123456789012345678901212345
semi-empirical method. Furthermore, the 6
3456789012345678901234567890121234567890123456789012345678901212345 6
1 Line The introduction of electronic computers
12 3456789012345678901234567890121234567890123456789012345678901212345
ab initio practitioners criticized the 6
12 3456789012345678901234567890121234567890123456789012345678901212345
into the field of quantum chemistry in the 6
12 3456789012345678901234567890121234567890123456789012345678901212345
semi-empirical method as only able to 6
12
2
3456789012345678901234567890121234567890123456789012345678901212345
1950s had an immediate and ultimately
3456789012345678901234567890121234567890123456789012345678901212345
6
6
1 (40) sustain the current theoretical paradigm
12 3456789012345678901234567890121234567890123456789012345678901212345
polarizing effect on the discipline. 6
12 3456789012345678901234567890121234567890123456789012345678901212345
and not able to explore possible inad- 6
12 3456789012345678901234567890121234567890123456789012345678901212345
(5) Quantum chemistry is primarily con- 6
12 3456789012345678901234567890121234567890123456789012345678901212345
equacies in the current framework. The 6
2 3456789012345678901234567890121234567890123456789012345678901212345
cerned with solving Schrödinger’s
semi-empirical practitioners, on the other 6
123456789012345678901234567890121234567890123456789012345678901212345
123456789012345678901234567890121234567890123456789012345678901212345 6
123456789012345678901234567890121234567890123456789012345678901212345
Equation—the fundamental equation of 6
123456789012345678901234567890121234567890123456789012345678901212345
hand, criticized the ab initio methodol- 6
123456789012345678901234567890121234567890123456789012345678901212345
the quantum realm—for different 6
123456789012345678901234567890121234567890123456789012345678901212345
(45) ogy as overly laborious and ultimately 6
1 theoretical scenarios. The difficulty in 6
12 3456789012345678901234567890121234567890123456789012345678901212345
this endeavor is that Schrödinger’s
less productive. Since the ab initio 6
12 3456789012345678901234567890121234567890123456789012345678901212345
(10)
6
12 3456789012345678901234567890121234567890123456789012345678901212345
Equation does not readily submit to
methodology solved all computational 6
2 3456789012345678901234567890121234567890123456789012345678901212345
components to Schrödinger’s Equation, it 6
123456789012345678901234567890121234567890123456789012345678901212345
123456789012345678901234567890121234567890123456789012345678901212345
straightforward solutions. Elaborate and 6
123456789012345678901234567890121234567890123456789012345678901212345
demanded considerable amounts of time 6
123456789012345678901234567890121234567890123456789012345678901212345
extensive calculations are often necessary 6
123456789012345678901234567890121234567890123456789012345678901212345
and effort, so fewer research projects 6
(50)
123456789012345678901234567890121234567890123456789012345678901212345
for solving Schrödinger’s Equation, 6
could be pursued with this methodology. 6
123456789012345678901234567890121234567890123456789012345678901212345
123456789012345678901234567890121234567890123456789012345678901212345
(15) which explains why the introduction of 6
123456789012345678901234567890121234567890123456789012345678901212345 Ultimately, both methodologies have 6
1 electronic computing held great promise 6
12 3456789012345678901234567890121234567890123456789012345678901212345
proved important to quantum chemistry. 6
12 3456789012345678901234567890121234567890123456789012345678901212345
as a research tool in theoretical quantum 6
123456789012345678901234567890121234567890123456789012345678901212345
The semi-empirical method has been 6
123456789012345678901234567890121234567890123456789012345678901212345
chemistry. 6
12
2
3456789012345678901234567890121234567890123456789012345678901212345
(55) essential for exploring questions within
3456789012345678901234567890121234567890123456789012345678901212345
6
6
123456789012345678901234567890121234567890123456789012345678901212345
As the discipline incorporated the use
the standard paradigm while the ab initio 6
123456789012345678901234567890121234567890123456789012345678901212345 6
1 (20) of electronic computers, two research
12 3456789012345678901234567890121234567890123456789012345678901212345
method has probed the boundaries of our 6
12 3456789012345678901234567890121234567890123456789012345678901212345
methodologies emerged. The first, termed 6
12 3456789012345678901234567890121234567890123456789012345678901212345
understanding of quantum chemistry. 6
12 3456789012345678901234567890121234567890123456789012345678901212345
ab initio, relied entirely upon the 6
123456789012345678901234567890121234567890123456789012345678901212345 6
123456789012345678901234567890121234567890123456789012345678901212345 6
electronic computer to solve the knotty
123456789012345678901234567890121234567890123456789012345678901212345 6
12 3456789012345678901234567890121234567890123456789012345678901212345
mathematics of Schrödinger’s Equation, 6
12 3456789012345678901234567890121234567890123456789012345678901212345 6
12 3456789012345678901234567890121234567890123456789012345678901212345 6
(25) and it employed no empirical data save
12 3456789012345678901234567890121234567890123456789012345678901212345 6
123456789012345678901234567890121234567890123456789012345678901212345
fundamental constants of nature. The 6
2 3456789012345678901234567890121234567890123456789012345678901212345
123456789012345678901234567890121234567890123456789012345678901212345
second, the so-called semi-empirical 6
123456789012345678901234567890121234567890123456789012345678901212345 6
123456789012345678901234567890121234567890123456789012345678901212345
method, simplified the computational 6
123456789012345678901234567890121234567890123456789012345678901212345 66
1 hurdles by neglecting possibly negligible
12 3456789012345678901234567890121234567890123456789012345678901212345 6
12 3456789012345678901234567890121234567890123456789012345678901212345
(30) components of the calculations and by 6
12 3456789012345678901234567890121234567890123456789012345678901212345 6
12 3456789012345678901234567890121234567890123456789012345678901212345
extensively employing empirical data. 6
123456789012345678901234567890121234567890123456789012345678901212345 6
123456789012345678901234567890121234567890123456789012345678901212345 66
2 3456789012345678901234567890121234567890123456789012345678901212345
The semi-empirical method quickly
1
12 3456789012345678901234567890121234567890123456789012345678901212345
became very fruitful, but the ab initio 6
123456789012345678901234567890121234567890123456789012345678901212345 6
12
2
3456789012345678901234567890121234567890123456789012345678901212345 6
practitioners were skeptical of this
3456789012345678901234567890121234567890123456789012345678901212345 6
1 (35) research because of the possible impor-
123456789012345678901234567890121234567890123456789012345678901212345 6
123456789012345678901234567890121234567890123456789012345678901212345 66
314 123456789012345678901234567890121234567890123456789012345678901212345
1234567890123456789012345678901212345678901234567890123456789012123456

www.petersons.com
TEST 3
1234567890123456789012345678901212345678901234567890123456789012123456
123456789012345678901234567890121234567890123456789012345678901212345 6
12 3456789012345678901234567890121234567890123456789012345678901212345 6
123456789012345678901234567890121234567890123456789012345678901212345
24. The ab initio method was criticized for 26. The semi-empirical method was empirical 6
12 3456789012345678901234567890121234567890123456789012345678901212345 6
12 3456789012345678901234567890121234567890123456789012345678901212345
in that it employed 6
123456789012345678901234567890121234567890123456789012345678901212345 6
I. being too demanding with regard to
12 3456789012345678901234567890121234567890123456789012345678901212345
A. strictly empirical means in obtaining a 6
123456789012345678901234567890121234567890123456789012345678901212345
labor. 6
12 3456789012345678901234567890121234567890123456789012345678901212345 6
12 3456789012345678901234567890121234567890123456789012345678901212345 6
II. its inability to critique the current result.
12 3456789012345678901234567890121234567890123456789012345678901212345 6
12
2
3456789012345678901234567890121234567890123456789012345678901212345
paradigm. B. the fundamental constants of nature
3456789012345678901234567890121234567890123456789012345678901212345
6
6
1
12 3456789012345678901234567890121234567890123456789012345678901212345
III. being a slower producer of new in obtaining a result. 6
12 3456789012345678901234567890121234567890123456789012345678901212345 6
12 3456789012345678901234567890121234567890123456789012345678901212345 6
research. C. extensive empirical data in obtaining
12 3456789012345678901234567890121234567890123456789012345678901212345 6
123456789012345678901234567890121234567890123456789012345678901212345
A. I only
a result. 6
12 3456789012345678901234567890121234567890123456789012345678901212345 6
12 3456789012345678901234567890121234567890123456789012345678901212345
B. II only
D. only empirically verifiable scenarios in 6
12 3456789012345678901234567890121234567890123456789012345678901212345
modeling quantum outcomes. 6
12
2
3456789012345678901234567890121234567890123456789012345678901212345
C. III only
3456789012345678901234567890121234567890123456789012345678901212345
6
6
1 E. restricted amounts of empirical data
12 3456789012345678901234567890121234567890123456789012345678901212345
D. I and II only 6
12 3456789012345678901234567890121234567890123456789012345678901212345
in obtaining results. 6
12 3456789012345678901234567890121234567890123456789012345678901212345
E. I and III only 6
12 3456789012345678901234567890121234567890123456789012345678901212345 6
2 3456789012345678901234567890121234567890123456789012345678901212345
123456789012345678901234567890121234567890123456789012345678901212345 6
123456789012345678901234567890121234567890123456789012345678901212345
27. The author’s intent was to 6
123456789012345678901234567890121234567890123456789012345678901212345 66
25. According to the passage, what was the
123456789012345678901234567890121234567890123456789012345678901212345
A. explain the effects of the introduction 6
123456789012345678901234567890121234567890123456789012345678901212345
primary methodological challenge in
123456789012345678901234567890121234567890123456789012345678901212345 6
1 working with Schrödinger’s Equation? of electronic computers on quantum 6
12 3456789012345678901234567890121234567890123456789012345678901212345 6
12 3456789012345678901234567890121234567890123456789012345678901212345
A. The difficulty of correctly applying
chemistry. 6
12 3456789012345678901234567890121234567890123456789012345678901212345 6
2 3456789012345678901234567890121234567890123456789012345678901212345
123456789012345678901234567890121234567890123456789012345678901212345
Schrödenger’s Equation in the
B. point out the harmfulness of scientific 6
123456789012345678901234567890121234567890123456789012345678901212345 6
123456789012345678901234567890121234567890123456789012345678901212345
quantum world
rivalries. 6
C. argue that the semi-empirical method 6
123456789012345678901234567890121234567890123456789012345678901212345
6
123456789012345678901234567890121234567890123456789012345678901212345
B. The complexity of the calculations
123456789012345678901234567890121234567890123456789012345678901212345
was more productive. 6
123456789012345678901234567890121234567890123456789012345678901212345
required to solve Schrödinger’s 6
123456789012345678901234567890121234567890123456789012345678901212345
D. illustrate the importance of coopera- 6
123456789012345678901234567890121234567890123456789012345678901212345
Equation 6
1 tion in scientific disciplines. 6
123456789012345678901234567890121234567890123456789012345678901212345 66
2 3456789012345678901234567890121234567890123456789012345678901212345
C. The inability of electronic computers
123456789012345678901234567890121234567890123456789012345678901212345
E. lay out the differences between the ab
6
1 to solve Schrödinger’s Equation
123456789012345678901234567890121234567890123456789012345678901212345
initio and semi-empirical methods. 6
12
2
3456789012345678901234567890121234567890123456789012345678901212345 6
D. The rivalry between the ab initio
3456789012345678901234567890121234567890123456789012345678901212345 6
123456789012345678901234567890121234567890123456789012345678901212345
123456789012345678901234567890121234567890123456789012345678901212345
practitioners and the semi-empirical 6
1 Passage 2 6
12 3456789012345678901234567890121234567890123456789012345678901212345 6
practitioners
12 3456789012345678901234567890121234567890123456789012345678901212345
Line Though nearly fifty years of research has 6
12 3456789012345678901234567890121234567890123456789012345678901212345
E. The difficulty of obtaining reliable 6
12 3456789012345678901234567890121234567890123456789012345678901212345
been done since the Khartoum Hospital 6
123456789012345678901234567890121234567890123456789012345678901212345 6
empirical data
123456789012345678901234567890121234567890123456789012345678901212345
and Shaheinab excavations in Central 6
123456789012345678901234567890121234567890123456789012345678901212345 6
2 3456789012345678901234567890121234567890123456789012345678901212345
123456789012345678901234567890121234567890123456789012345678901212345
Sudan, a coherent and rigorous frame- 6
123456789012345678901234567890121234567890123456789012345678901212345 6
123456789012345678901234567890121234567890123456789012345678901212345
(5) work for understanding the Mesolithic- 6
123456789012345678901234567890121234567890123456789012345678901212345 66
1 Neolithic (8000 B.C.E.–4000 B.C.E.)
12 3456789012345678901234567890121234567890123456789012345678901212345 6
12 3456789012345678901234567890121234567890123456789012345678901212345
transition in the Central Nile has not 6
12 3456789012345678901234567890121234567890123456789012345678901212345 6
12 3456789012345678901234567890121234567890123456789012345678901212345
been achieved. A. J. Arkell, the primary 6
123456789012345678901234567890121234567890123456789012345678901212345 6
123456789012345678901234567890121234567890123456789012345678901212345 66
2 3456789012345678901234567890121234567890123456789012345678901212345
excavator at these sites, developed a
123456789012345678901234567890121234567890123456789012345678901212345
model of cultural evolution based on two 6
123456789012345678901234567890121234567890123456789012345678901212345
(10)
1 basic ceramic typologies—the wavy and 6
123456789012345678901234567890121234567890123456789012345678901212345 6
2 3456789012345678901234567890121234567890123456789012345678901212345
123456789012345678901234567890121234567890123456789012345678901212345 6
1 dotted line—found at these sites. 6
12 3456789012345678901234567890121234567890123456789012345678901212345 6
123456789012345678901234567890121234567890123456789012345678901212345 6
Predictably, the wavy line type are
12 3456789012345678901234567890121234567890123456789012345678901212345 6
123456789012345678901234567890121234567890123456789012345678901212345
ceramics whose outer surface are 6
123456789012345678901234567890121234567890123456789012345678901212345 6
123456789012345678901234567890121234567890123456789012345678901212345 6
123456789012345678901234567890121234567890123456789012345678901212345 6 315
1234567890123456789012345678901212345678901234567890123456789012123456
Part IV: Three Practice GREs

1234567890123456789012345678901212345678901234567890123456789012123456
123456789012345678901234567890121234567890123456789012345678901212345 6
12 3456789012345678901234567890121234567890123456789012345678901212345 6
123456789012345678901234567890121234567890123456789012345678901212345
(15) decorated with wavy lines, while the (55) chronological index for the region? Does 6
12 3456789012345678901234567890121234567890123456789012345678901212345 6
12 3456789012345678901234567890121234567890123456789012345678901212345
dotted line ceramics are decorated with their geographical dispersion tell us of 6
123456789012345678901234567890121234567890123456789012345678901212345 6
12
2
3456789012345678901234567890121234567890123456789012345678901212345 6
the same grooves of the wavy line but possible
3456789012345678901234567890121234567890123456789012345678901212345
cultural uniformities across these
6
1
12 3456789012345678901234567890121234567890123456789012345678901212345
with indentions, or dots, further lining diverse regions? The answers these to 6
12 3456789012345678901234567890121234567890123456789012345678901212345 6
12 3456789012345678901234567890121234567890123456789012345678901212345 6
the grooves. questions about the past hopefully lie
12 3456789012345678901234567890121234567890123456789012345678901212345
somewhere in the not-too-distant future. 6
123456789012345678901234567890121234567890123456789012345678901212345
(20) Arkell’s model designates the wavy (60) 6
12 3456789012345678901234567890121234567890123456789012345678901212345 6
12 3456789012345678901234567890121234567890123456789012345678901212345 6
line type as markers for Khartoum
12 3456789012345678901234567890121234567890123456789012345678901212345 6
12
2
3456789012345678901234567890121234567890123456789012345678901212345
Mesolithic culture and the dotted line 28. The main idea of the passage is
3456789012345678901234567890121234567890123456789012345678901212345
6
6
1
12 3456789012345678901234567890121234567890123456789012345678901212345
type as markers for Khartoum Neolithic 6
12 3456789012345678901234567890121234567890123456789012345678901212345
A. Nilotic and Saharan archaeology has 6
12 3456789012345678901234567890121234567890123456789012345678901212345
culture. Yet this model became problem- 6
12
2
3456789012345678901234567890121234567890123456789012345678901212345
3456789012345678901234567890121234567890123456789012345678901212345
failed to provide an adequate frame- 6
6
1 (25) atic as excavations beyond the Central
12 3456789012345678901234567890121234567890123456789012345678901212345 work for understanding the Me- 6
12 3456789012345678901234567890121234567890123456789012345678901212345
Nile began to unearth wavy and dotted 6
12 3456789012345678901234567890121234567890123456789012345678901212345
line pottery across the Sahara-Sahel belt solithic-Neolithic transition. 6
12 3456789012345678901234567890121234567890123456789012345678901212345 6
2 3456789012345678901234567890121234567890123456789012345678901212345
123456789012345678901234567890121234567890123456789012345678901212345
in strata and in sequences inconsistent B. Arkell and Canerva were incorrect in 6
123456789012345678901234567890121234567890123456789012345678901212345 6
123456789012345678901234567890121234567890123456789012345678901212345
with Arkell’s designations. It appeared a their respective attempts to system- 6
123456789012345678901234567890121234567890123456789012345678901212345 6
123456789012345678901234567890121234567890123456789012345678901212345
(30) new model was needed to account for atize their research. 6
1 not only chronological development but C. wavy line ceramics clearly are not the 6
123456789012345678901234567890121234567890123456789012345678901212345 6
12 3456789012345678901234567890121234567890123456789012345678901212345 6
12 3456789012345678901234567890121234567890123456789012345678901212345
geographical dispersal as well. chronological predecessors of dotted 6
12 3456789012345678901234567890121234567890123456789012345678901212345 6
123456789012345678901234567890121234567890123456789012345678901212345 66
2 3456789012345678901234567890121234567890123456789012345678901212345
To address these issues, I. Canerva line ceramics.
123456789012345678901234567890121234567890123456789012345678901212345
D. dotted and wavy line pottery has not 6
123456789012345678901234567890121234567890123456789012345678901212345
put forth a classification strategy for
123456789012345678901234567890121234567890123456789012345678901212345 been successfully incorporated into a 6 6
123456789012345678901234567890121234567890123456789012345678901212345
(35) Nilotic and Saharan ceramics based on
123456789012345678901234567890121234567890123456789012345678901212345 larger theory that explains their 6
123456789012345678901234567890121234567890123456789012345678901212345
technique. Arkell’s model classified by 6
123456789012345678901234567890121234567890123456789012345678901212345 importance, thus leaving important 6
123456789012345678901234567890121234567890123456789012345678901212345 66
motif, the ceramic’s outer decorations,
1 further research open.
12 3456789012345678901234567890121234567890123456789012345678901212345
but Canerva focused on the techniques 6
12 3456789012345678901234567890121234567890123456789012345678901212345
E. research on Nilotic and Saharan 6
123456789012345678901234567890121234567890123456789012345678901212345
employed, such as method of hardening, 6
123456789012345678901234567890121234567890123456789012345678901212345 ceramics has matured over the past 6
12
2
3456789012345678901234567890121234567890123456789012345678901212345
(40) and technologies needed, such as tools, in
3456789012345678901234567890121234567890123456789012345678901212345
6
6
123456789012345678901234567890121234567890123456789012345678901212345 five decades.
123456789012345678901234567890121234567890123456789012345678901212345
making the ceramics. This strategy 6
1 6
12 3456789012345678901234567890121234567890123456789012345678901212345
initially promised to provide artifact 6
12 3456789012345678901234567890121234567890123456789012345678901212345
29. Arkell’s model was problematic in that 6
123456789012345678901234567890121234567890123456789012345678901212345 66
2 3456789012345678901234567890121234567890123456789012345678901212345
markers for the cultural evolution from
123456789012345678901234567890121234567890123456789012345678901212345
the Nile basin to the western edges of the A. the discovery of wavy and dotted line 6
1
123456789012345678901234567890121234567890123456789012345678901212345
(45) Sahara in the Mesolithic-Neolithic. This pottery in the Central Nile conflicted 6
123456789012345678901234567890121234567890123456789012345678901212345 6
2 3456789012345678901234567890121234567890123456789012345678901212345
123456789012345678901234567890121234567890123456789012345678901212345
approach, though, became mired in with his findings. 6
123456789012345678901234567890121234567890123456789012345678901212345 6
B. dating discrepancies were discovered 6
123456789012345678901234567890121234567890123456789012345678901212345
questions related to determination of
123456789012345678901234567890121234567890123456789012345678901212345 66
1 technique, as the reverse inference from within his account.
12 3456789012345678901234567890121234567890123456789012345678901212345 6
12 3456789012345678901234567890121234567890123456789012345678901212345
motif to technique became dubious. C. wavy and dotted line pottery were 6
12 3456789012345678901234567890121234567890123456789012345678901212345 6
12 3456789012345678901234567890121234567890123456789012345678901212345
(50) At this point major questions remain discovered outside the Central Nile 6
123456789012345678901234567890121234567890123456789012345678901212345 6
123456789012345678901234567890121234567890123456789012345678901212345 66
2 3456789012345678901234567890121234567890123456789012345678901212345
unanswered concerning wavy and dotted that did not conform to his model.
123456789012345678901234567890121234567890123456789012345678901212345 6
123456789012345678901234567890121234567890123456789012345678901212345
line ceramics in Mesolithic-Neolithic D. pottery fragments were discovered
6
123456789012345678901234567890121234567890123456789012345678901212345 6
1 Nilotic and Saharan regions. Do these outside the Central Nile as far west as
2 3456789012345678901234567890121234567890123456789012345678901212345
123456789012345678901234567890121234567890123456789012345678901212345 6
1 ceramic typologies provide any sort of the Western Sahara. 6
12 3456789012345678901234567890121234567890123456789012345678901212345 6
123456789012345678901234567890121234567890123456789012345678901212345 6
E. ceramics that chronologically pre-
12 3456789012345678901234567890121234567890123456789012345678901212345 6
123456789012345678901234567890121234567890123456789012345678901212345 ceded the wavy line pottery were 6
123456789012345678901234567890121234567890123456789012345678901212345 6
123456789012345678901234567890121234567890123456789012345678901212345 66
discovered in the Central Nile.
316 123456789012345678901234567890121234567890123456789012345678901212345
1234567890123456789012345678901212345678901234567890123456789012123456

www.petersons.com
TEST 3
1234567890123456789012345678901212345678901234567890123456789012123456
123456789012345678901234567890121234567890123456789012345678901212345 6
12 3456789012345678901234567890121234567890123456789012345678901212345 6
123456789012345678901234567890121234567890123456789012345678901212345
30. It can be inferred from the passage that 6
12 3456789012345678901234567890121234567890123456789012345678901212345 6
12 3456789012345678901234567890121234567890123456789012345678901212345
the author would agree with which one of 6
123456789012345678901234567890121234567890123456789012345678901212345 6
12
2
3456789012345678901234567890121234567890123456789012345678901212345 6
the following?
3456789012345678901234567890121234567890123456789012345678901212345 6
1
12 3456789012345678901234567890121234567890123456789012345678901212345 6
12 3456789012345678901234567890121234567890123456789012345678901212345 6
A. The work of Arkell and Canerva has
12 3456789012345678901234567890121234567890123456789012345678901212345 6
12
2
3456789012345678901234567890121234567890123456789012345678901212345
been thoroughly debunked.
3456789012345678901234567890121234567890123456789012345678901212345
6
6
1
12 3456789012345678901234567890121234567890123456789012345678901212345
B. Future research into Mesolithic- 6
12 3456789012345678901234567890121234567890123456789012345678901212345 6
12 3456789012345678901234567890121234567890123456789012345678901212345 6
Neolithic Khartoum should be done.
12 3456789012345678901234567890121234567890123456789012345678901212345 6
123456789012345678901234567890121234567890123456789012345678901212345
C. Motif should be considered over 6
12 3456789012345678901234567890121234567890123456789012345678901212345 6
12 3456789012345678901234567890121234567890123456789012345678901212345 6
technique when researching ceramics.
12 3456789012345678901234567890121234567890123456789012345678901212345
D. The material composition of a 6
12
2
3456789012345678901234567890121234567890123456789012345678901212345
3456789012345678901234567890121234567890123456789012345678901212345
6
6
1 ceramic is an important clue in
12 3456789012345678901234567890121234567890123456789012345678901212345 6
12 3456789012345678901234567890121234567890123456789012345678901212345
archaeological research. 6
12 3456789012345678901234567890121234567890123456789012345678901212345 6
12 3456789012345678901234567890121234567890123456789012345678901212345
E. Material culture is more readily 6
12 3456789012345678901234567890121234567890123456789012345678901212345 6
12 3456789012345678901234567890121234567890123456789012345678901212345
studied than non-material culture. 6
12 3456789012345678901234567890121234567890123456789012345678901212345 6
12 3456789012345678901234567890121234567890123456789012345678901212345 6
12 3456789012345678901234567890121234567890123456789012345678901212345 6
123456789012345678901234567890121234567890123456789012345678901212345
2 6
1 3456789012345678901234567890121234567890123456789012345678901212345 6
12 3456789012345678901234567890121234567890123456789012345678901212345 6
12 3456789012345678901234567890121234567890123456789012345678901212345 6
12 3456789012345678901234567890121234567890123456789012345678901212345 6
123456789012345678901234567890121234567890123456789012345678901212345
2 6
123456789012345678901234567890121234567890123456789012345678901212345 6
123456789012345678901234567890121234567890123456789012345678901212345 6
123456789012345678901234567890121234567890123456789012345678901212345 6
123456789012345678901234567890121234567890123456789012345678901212345 6
123456789012345678901234567890121234567890123456789012345678901212345 66
3456789012345678901234567890121234567890123456789012345678901212345
123456789012345678901234567890121234567890123456789012345678901212345 6
123456789012345678901234567890121234567890123456789012345678901212345 6
123456789012345678901234567890121234567890123456789012345678901212345 6
1
12 3456789012345678901234567890121234567890123456789012345678901212345 6
12 3456789012345678901234567890121234567890123456789012345678901212345 6
123456789012345678901234567890121234567890123456789012345678901212345 6
123456789012345678901234567890121234567890123456789012345678901212345 6
12 3456789012345678901234567890121234567890123456789012345678901212345 6
12 3456789012345678901234567890121234567890123456789012345678901212345 6
12 3456789012345678901234567890121234567890123456789012345678901212345 6
123456789012345678901234567890121234567890123456789012345678901212345 6
12 3456789012345678901234567890121234567890123456789012345678901212345 6
12 3456789012345678901234567890121234567890123456789012345678901212345 6
123456789012345678901234567890121234567890123456789012345678901212345
2 6
123456789012345678901234567890121234567890123456789012345678901212345 6
1 3456789012345678901234567890121234567890123456789012345678901212345 6
123456789012345678901234567890121234567890123456789012345678901212345 6
123456789012345678901234567890121234567890123456789012345678901212345 6
12 3456789012345678901234567890121234567890123456789012345678901212345 6
12 3456789012345678901234567890121234567890123456789012345678901212345 6
12 3456789012345678901234567890121234567890123456789012345678901212345 6
12 3456789012345678901234567890121234567890123456789012345678901212345 6
123456789012345678901234567890121234567890123456789012345678901212345 6
123456789012345678901234567890121234567890123456789012345678901212345
2 6
123456789012345678901234567890121234567890123456789012345678901212345 6
123456789012345678901234567890121234567890123456789012345678901212345 6
123456789012345678901234567890121234567890123456789012345678901212345 6
1 3456789012345678901234567890121234567890123456789012345678901212345 6
123456789012345678901234567890121234567890123456789012345678901212345 66
2 3456789012345678901234567890121234567890123456789012345678901212345
123456789012345678901234567890121234567890123456789012345678901212345 6
123456789012345678901234567890121234567890123456789012345678901212345 6
123456789012345678901234567890121234567890123456789012345678901212345 6
1
12 3456789012345678901234567890121234567890123456789012345678901212345 6
123456789012345678901234567890121234567890123456789012345678901212345 6
12 3456789012345678901234567890121234567890123456789012345678901212345 6
123456789012345678901234567890121234567890123456789012345678901212345 6
12 3456789012345678901234567890121234567890123456789012345678901212345 6
123456789012345678901234567890121234567890123456789012345678901212345 6
123456789012345678901234567890121234567890123456789012345678901212345 6
123456789012345678901234567890121234567890123456789012345678901212345 6
123456789012345678901234567890121234567890123456789012345678901212345 6 317
1234567890123456789012345678901212345678901234567890123456789012123456
Answers and Explanations

123456789012345678901234567890121234567890123456789012
12345678901234567890123456789012123456789012345678901 2
12345678901234567890123456789012123456789012345678901
2345678901234567890123456789012123456789012345678901
2
2
1 This is the last sample test you will take before moving onto the computer
12345678901234567890123456789012123456789012345678901 2
12345678901234567890123456789012123456789012345678901
adaptive exams. Hopefully, you have tested out some new strategies and 2
12345678901234567890123456789012123456789012345678901 2
12345678901234567890123456789012123456789012345678901
ideas on the previous two tests and are now comfortable with them. 2
12345678901234567890123456789012123456789012345678901 2
2345678901234567890123456789012123456789012345678901
12345678901234567890123456789012123456789012345678901 2
12345678901234567890123456789012123456789012345678901 2
12345678901234567890123456789012123456789012345678901 2
12345678901234567890123456789012123456789012345678901
Quantitative 2
12345678901234567890123456789012123456789012345678901 2
1 2
12345678901234567890123456789012123456789012345678901
1. B The only way to do a problem like this is to simplify the 2
12345678901234567890123456789012123456789012345678901 2
12345678901234567890123456789012123456789012345678901
expressions. 2
12345678901234567890123456789012123456789012345678901 2
2345678901234567890123456789012123456789012345678901
12345678901234567890123456789012123456789012345678901 2
12345678901234567890123456789012123456789012345678901
2
For Column B, 3 5 9. 2
12345678901234567890123456789012123456789012345678901 2
12345678901234567890123456789012123456789012345678901
For Column A, =32 1 72 5 =9 1 49 5 =58 , 8 since 2
2
12345678901234567890123456789012123456789012345678901
12345678901234567890123456789012123456789012345678901 2
12345678901234567890123456789012123456789012345678901
=64 5 8. 2
12345678901234567890123456789012123456789012345678901 2
1 2
2345678901234567890123456789012123456789012345678901
12345678901234567890123456789012123456789012345678901
Column B is greater. 2
12345678901234567890123456789012123456789012345678901 2
1 2. C To be able to compare 2y to 6, you need to solve for y. You can 2
12345678901234567890123456789012123456789012345678901 2
12345678901234567890123456789012123456789012345678901 2
12345678901234567890123456789012123456789012345678901
do this by substituting y for x in the first equation. This yields: 2
12345678901234567890123456789012123456789012345678901 2
12345678901234567890123456789012123456789012345678901 2
12345678901234567890123456789012123456789012345678901
x 1 2y 5 9 → 2
12345678901234567890123456789012123456789012345678901 2
2345678901234567890123456789012123456789012345678901
12345678901234567890123456789012123456789012345678901
y 1 2y 5 9 → 2
12345678901234567890123456789012123456789012345678901 2
1 → 2
12345678901234567890123456789012123456789012345678901 2
3y 5 9
12345678901234567890123456789012123456789012345678901 2
12345678901234567890123456789012123456789012345678901
y53 2
12345678901234567890123456789012123456789012345678901 2
12345678901234567890123456789012123456789012345678901 2
12345678901234567890123456789012123456789012345678901
This means 2y 5 6. Choice (C) then is the answer. 2
12345678901234567890123456789012123456789012345678901 2
2345678901234567890123456789012123456789012345678901
3. D A good way to attack a problem like this is try a few values for 2
12345678901234567890123456789012123456789012345678901
12345678901234567890123456789012123456789012345678901 2
x. At its smallest x 5 1, and Column A would equal two thirds, 2
12345678901234567890123456789012123456789012345678901
12345678901234567890123456789012123456789012345678901 2
1 or 0.67. If x 5 2, Column A equals eight ninths, and if x 5 3, 2
12345678901234567890123456789012123456789012345678901 2
12345678901234567890123456789012123456789012345678901
Column A equals twenty-six twenty-sevenths. As you can see, 2
12345678901234567890123456789012123456789012345678901 2
12345678901234567890123456789012123456789012345678901
Column A increases as x increases. In the first case, Column B 2
12345678901234567890123456789012123456789012345678901 2
12345678901234567890123456789012123456789012345678901
was greater; in the third, Column A was. Thus the answer is (D). 2
12345678901234567890123456789012123456789012345678901 2
12345678901234567890123456789012123456789012345678901 2
12345678901234567890123456789012123456789012345678901 2
12345678901234567890123456789012123456789012345678901 2
12345678901234567890123456789012123456789012345678901 2
12345678901234567890123456789012123456789012345678901 2
12345678901234567890123456789012123456789012345678901 2
12345678901234567890123456789012123456789012345678901 2
123456789012345678901234567890121234567890123456789012
318
ANSWERS
123456789012345678901234567890121234567890123456789012
12345678901234567890123456789012123456789012345678901 2
12345678901234567890123456789012123456789012345678901 2
12345678901234567890123456789012123456789012345678901
4. A First, simplify the expression in Column A, 2
12345678901234567890123456789012123456789012345678901 2
12345678901234567890123456789012123456789012345678901 2
12345678901234567890123456789012123456789012345678901
1 1
2
12345678901234567890123456789012123456789012345678901
~ 2 2 2
b 1 2bc 1 c ! 5 @~b 1 c! # 5 b 1 c.
2345678901234567890123456789012123456789012345678901
2 2
2
2
1
12345678901234567890123456789012123456789012345678901 2
12345678901234567890123456789012123456789012345678901
b 1 c 5 180° . 0.90 2
12345678901234567890123456789012123456789012345678901 2
12345678901234567890123456789012123456789012345678901
2345678901234567890123456789012123456789012345678901
2
2
1 5. D Remember that an isosceles triangle is not necessarily a right
12345678901234567890123456789012123456789012345678901 2
12345678901234567890123456789012123456789012345678901
triangle. Since the triangle is not a right triangle, you have no 2
12345678901234567890123456789012123456789012345678901 2
12345678901234567890123456789012123456789012345678901
way of comparing b with c. Thus,
2345678901234567890123456789012123456789012345678901
the answer is (D). 2
2
1
12345678901234567890123456789012123456789012345678901
6. D Begin by manipulating the exponents to more readily compare 2
12345678901234567890123456789012123456789012345678901 2
12345678901234567890123456789012123456789012345678901
the columns: 2
12345678901234567890123456789012123456789012345678901
2345678901234567890123456789012123456789012345678901
2
2
1
12345678901234567890123456789012123456789012345678901
Column A: x x 5 xa b a1b 2
12345678901234567890123456789012123456789012345678901 2
12345678901234567890123456789012123456789012345678901 2
12345678901234567890123456789012123456789012345678901
x a 2
2345678901234567890123456789012123456789012345678901
12345678901234567890123456789012123456789012345678901
a 2 b 2
12345678901234567890123456789012123456789012345678901
Column B: b 5 x 2
12345678901234567890123456789012123456789012345678901 22
x
12345678901234567890123456789012123456789012345678901 2
12345678901234567890123456789012123456789012345678901
and you know that b 2 a . 1 → a 2 b , 21. This means that
12345678901234567890123456789012123456789012345678901 2
1 the x in Column B is raised to a negative power, but how much 2
12345678901234567890123456789012123456789012345678901 2
12345678901234567890123456789012123456789012345678901
does this tell us about its value in comparison to Column A? 2
12345678901234567890123456789012123456789012345678901 2
12345678901234567890123456789012123456789012345678901
a 1 b could also be negative, which would mean that Column A 2
12345678901234567890123456789012123456789012345678901 2
12345678901234567890123456789012123456789012345678901
was also raised to a negative, and a greater negative at that. Or 2
12345678901234567890123456789012123456789012345678901 2
12345678901234567890123456789012123456789012345678901
a 1 b could be positive, which would mean that the x in Column 2
2345678901234567890123456789012123456789012345678901
12345678901234567890123456789012123456789012345678901 2
12345678901234567890123456789012123456789012345678901
A would be raised to a positive. 2
12345678901234567890123456789012123456789012345678901 22
12345678901234567890123456789012123456789012345678901
In the former case, Column B could be greater, depending on the 2
1
12345678901234567890123456789012123456789012345678901
value of x, and in the latter case, Column A would be greater, 2
12345678901234567890123456789012123456789012345678901 2
12345678901234567890123456789012123456789012345678901 2
12345678901234567890123456789012123456789012345678901 2
again depending on the value of x. As you can see, there are a
12345678901234567890123456789012123456789012345678901 2
12345678901234567890123456789012123456789012345678901
variety of possible outcomes. 2
12345678901234567890123456789012123456789012345678901 2
12345678901234567890123456789012123456789012345678901
7. A Distribute the x in Column A, which yields x 2 x. You can 2 2
12345678901234567890123456789012123456789012345678901 2
12345678901234567890123456789012123456789012345678901
reason that this difference is greater than zero. x’s absolute value 2
2345678901234567890123456789012123456789012345678901
12345678901234567890123456789012123456789012345678901 2
is greater than 1, and x2 is positive and greater than x. Column 2
12345678901234567890123456789012123456789012345678901
1 2
12345678901234567890123456789012123456789012345678901
A, then, is greater. 2
12345678901234567890123456789012123456789012345678901 2
2345678901234567890123456789012123456789012345678901
8. A Twenty percent of 50 is 10, and so there are 10 divers. Four times 2
12345678901234567890123456789012123456789012345678901
12345678901234567890123456789012123456789012345678901 2
12345678901234567890123456789012123456789012345678901 2
12345678901234567890123456789012123456789012345678901 22
this number is 40. How many relay racers are there? Since each
1
12345678901234567890123456789012123456789012345678901
member only participates in 1 event, and since there are 10 divers, 2
12345678901234567890123456789012123456789012345678901
there are 40 members who are either divers or individual swim- 2
12345678901234567890123456789012123456789012345678901 2
12345678901234567890123456789012123456789012345678901 2
12345678901234567890123456789012123456789012345678901 2
mers. We know there must be at least 1 individual swimmer, or
2345678901234567890123456789012123456789012345678901
the team would not be composed of individual swimmers; that 2
12345678901234567890123456789012123456789012345678901
12345678901234567890123456789012123456789012345678901 2
12345678901234567890123456789012123456789012345678901 2
12345678901234567890123456789012123456789012345678901 22
means there can at most be 39 relay racers. Thus, Column A
1
12345678901234567890123456789012123456789012345678901
is greater. 2
12345678901234567890123456789012123456789012345678901 2
12345678901234567890123456789012123456789012345678901 2
12345678901234567890123456789012123456789012345678901 2
12345678901234567890123456789012123456789012345678901 2
12345678901234567890123456789012123456789012345678901 2
12345678901234567890123456789012123456789012345678901 2
12345678901234567890123456789012123456789012345678901 2
12345678901234567890123456789012123456789012345678901 2 319
123456789012345678901234567890121234567890123456789012
Part IV: Three Practice GREs

123456789012345678901234567890121234567890123456789012
12345678901234567890123456789012123456789012345678901 2
12345678901234567890123456789012123456789012345678901 2
12345678901234567890123456789012123456789012345678901
9. B Since the directions are due north and due west, the three towns 2
12345678901234567890123456789012123456789012345678901 2
12345678901234567890123456789012123456789012345678901
form a right triangle, and we can use the Pythagorean theorem 2
12345678901234567890123456789012123456789012345678901 2
12345678901234567890123456789012123456789012345678901
to determine the distance between A and C.
2345678901234567890123456789012123456789012345678901
2
2
1
12345678901234567890123456789012123456789012345678901 2
12345678901234567890123456789012123456789012345678901
c 5 400 1 900 5 1,300 → c 5 =1,300 5 10=13
2
2
12345678901234567890123456789012123456789012345678901 2
12345678901234567890123456789012123456789012345678901
2345678901234567890123456789012123456789012345678901
2
2
1 The question now becomes, “How do you compare the value of
12345678901234567890123456789012123456789012345678901 2
12345678901234567890123456789012123456789012345678901
c with 40?” Notice that =13 , =16 5 4 and so ten times 2
12345678901234567890123456789012123456789012345678901 2
12345678901234567890123456789012123456789012345678901
the square root of 13 is less than 40. Ergo, Column B is greater. 2
2345678901234567890123456789012123456789012345678901 2
1
12345678901234567890123456789012123456789012345678901 2
12345678901234567890123456789012123456789012345678901
10. B The probability of the first event, grabbing the first gray sock, is 2
12345678901234567890123456789012123456789012345678901 2
12345678901234567890123456789012123456789012345678901
one third. The probability of grabbing the second gray sock is 2
2345678901234567890123456789012123456789012345678901 2
1
12345678901234567890123456789012123456789012345678901
not as straightforward. Now there are only 24 gray socks and 2
12345678901234567890123456789012123456789012345678901
still 25 black and white socks. Thus, the probability is 2
12345678901234567890123456789012123456789012345678901 2
12345678901234567890123456789012123456789012345678901 2
12345678901234567890123456789012123456789012345678901 2
2345678901234567890123456789012123456789012345678901
something a little bit less than one third. To get the probability
of both events occurring, we have to multiply the two 2
12345678901234567890123456789012123456789012345678901
12345678901234567890123456789012123456789012345678901 2
12345678901234567890123456789012123456789012345678901 2
12345678901234567890123456789012123456789012345678901 2
probabilities together. One third times something a little smaller
1 than one third is going to be smaller than one sixth, the value in 2
12345678901234567890123456789012123456789012345678901 2
12345678901234567890123456789012123456789012345678901 2
12345678901234567890123456789012123456789012345678901
Column B. 2
12345678901234567890123456789012123456789012345678901 2
2345678901234567890123456789012123456789012345678901
11. A Obviously it is going to be difficult to compare these by 2
12345678901234567890123456789012123456789012345678901
12345678901234567890123456789012123456789012345678901 2
multiplying out the exponents. There must be another way, and 2
12345678901234567890123456789012123456789012345678901
12345678901234567890123456789012123456789012345678901 2
there is 415 5 4 3 4 14 5 414 1 414 1 414 1 414, which gives us 2
12345678901234567890123456789012123456789012345678901
12345678901234567890123456789012123456789012345678901 2
four terms in Column A to compare with four terms in Column 2
12345678901234567890123456789012123456789012345678901
12345678901234567890123456789012123456789012345678901
B. The terms in Column B are less than all the respective terms in 2
2
12345678901234567890123456789012123456789012345678901 2
1
12345678901234567890123456789012123456789012345678901
Column A, and so (A) is the answer. 2
12345678901234567890123456789012123456789012345678901 2
12345678901234567890123456789012123456789012345678901 2
12345678901234567890123456789012123456789012345678901 2
12. C Six students are competing for first place, and so there are six
12345678901234567890123456789012123456789012345678901
possibilities for first. But since there will always be a first before 2
12345678901234567890123456789012123456789012345678901 2
12345678901234567890123456789012123456789012345678901 2
12345678901234567890123456789012123456789012345678901 2
a second, only five can be second. And since there will be a first
12345678901234567890123456789012123456789012345678901
and a second before a third, only four can be third. Multiply 2
12345678901234567890123456789012123456789012345678901 2
2345678901234567890123456789012123456789012345678901
these together to get the total number of possibilities, (6)(5)(4) 2
12345678901234567890123456789012123456789012345678901
12345678901234567890123456789012123456789012345678901 2
1 2
12345678901234567890123456789012123456789012345678901 2
5 120. The answer is (C). You may recall the formula for these
12345678901234567890123456789012123456789012345678901 n! 2
12345678901234567890123456789012123456789012345678901 where P is the 2
12345678901234567890123456789012123456789012345678901
kinds of permutation problems as P 5
~n 2 c!! 2
12345678901234567890123456789012123456789012345678901 2
12345678901234567890123456789012123456789012345678901
total number of possible outcomes, n is the number of 2
12345678901234567890123456789012123456789012345678901 2
12345678901234567890123456789012123456789012345678901
participants, and c is the number of places. 2
12345678901234567890123456789012123456789012345678901 2
12345678901234567890123456789012123456789012345678901 2
12345678901234567890123456789012123456789012345678901
13. C Consider the subtraction in the ones column and tens column, 2
12345678901234567890123456789012123456789012345678901 2
12345678901234567890123456789012123456789012345678901 2
2345678901234567890123456789012123456789012345678901
respectively. In the ones, since C . D, you have to borrow ten
from the tens column. This means F 5 D 1 10 2 C. As for the 2
12345678901234567890123456789012123456789012345678901
12345678901234567890123456789012123456789012345678901 2
12345678901234567890123456789012123456789012345678901 2
1 tens column, E 5 C 2 1 2 D since you borrowed one from the 2
2345678901234567890123456789012123456789012345678901
column. Putting these two results together, E 1 F 5 (C 2 1 2 D) 2
12345678901234567890123456789012123456789012345678901
1 2
12345678901234567890123456789012123456789012345678901
(D 10 C) 9. The answer then is (C). 2
12345678901234567890123456789012123456789012345678901 2
1 1 2 5
12345678901234567890123456789012123456789012345678901 2
12345678901234567890123456789012123456789012345678901 2
12345678901234567890123456789012123456789012345678901 2
12345678901234567890123456789012123456789012345678901 2 2
320 12345678901234567890123456789012123456789012345678901
123456789012345678901234567890121234567890123456789012

www.petersons.com
ANSWERS
123456789012345678901234567890121234567890123456789012
12345678901234567890123456789012123456789012345678901 2
12345678901234567890123456789012123456789012345678901 2
12345678901234567890123456789012123456789012345678901
14. D The numerator in Column A might at first blush appear greater, 2
12345678901234567890123456789012123456789012345678901 2
12345678901234567890123456789012123456789012345678901
but be careful. If n 5 1 then n! 1 1 5 (n 1 1)!, in which case 2
12345678901234567890123456789012123456789012345678901 2
12345678901234567890123456789012123456789012345678901
Column A and B are equal. If n is a large number, though, 2
2345678901234567890123456789012123456789012345678901 2
1
12345678901234567890123456789012123456789012345678901
n! 1 1 , (n 11)!, in which case Column A would be greater. 2
12345678901234567890123456789012123456789012345678901 2
12345678901234567890123456789012123456789012345678901
Thus the answer is (D). 2
12345678901234567890123456789012123456789012345678901 2
1 15. D Since the triangles are similar the ratios of the side lengths are 2
2345678901234567890123456789012123456789012345678901
12345678901234567890123456789012123456789012345678901 2
12345678901234567890123456789012123456789012345678901 2
12345678901234567890123456789012123456789012345678901
proportional, and you can use this to find a. You do not know 2
12345678901234567890123456789012123456789012345678901
2345678901234567890123456789012123456789012345678901
2
2
1 the length of the side on the larger triangle that corresponds
12345678901234567890123456789012123456789012345678901 2
12345678901234567890123456789012123456789012345678901
with side a, and so you need to determine the third side of the 2
12345678901234567890123456789012123456789012345678901
larger triangle. Using the Pythagorean theorem, you can 2
12345678901234567890123456789012123456789012345678901
2345678901234567890123456789012123456789012345678901
2
1 determine that the third side is equal to 6. Using similar triangles 2
12345678901234567890123456789012123456789012345678901 2
12345678901234567890123456789012123456789012345678901
then, 2
12345678901234567890123456789012123456789012345678901 2
12345678901234567890123456789012123456789012345678901 2
2345678901234567890123456789012123456789012345678901
12345678901234567890123456789012123456789012345678901 2
12345678901234567890123456789012123456789012345678901
6 a
→ →
36 9 2
12345678901234567890123456789012123456789012345678901
5 36 5 8a a 5 5 , 2
12345678901234567890123456789012123456789012345678901 22
8 6 8 2
12345678901234567890123456789012123456789012345678901 2
12345678901234567890123456789012123456789012345678901
which is answer choice (D).
2
1
12345678901234567890123456789012123456789012345678901 2
12345678901234567890123456789012123456789012345678901
16. D The greatest slope means the line that has the steepest ascent left 2
12345678901234567890123456789012123456789012345678901 2
12345678901234567890123456789012123456789012345678901
to right. If a line ascends right to left, then its slope is negative. 2
12345678901234567890123456789012123456789012345678901 2
12345678901234567890123456789012123456789012345678901
Line 4, choice (D), has the steepest slope left to right. 2
12345678901234567890123456789012123456789012345678901 2
12345678901234567890123456789012123456789012345678901 2
12345678901234567890123456789012123456789012345678901 22
2345678901234567890123456789012123456789012345678901
17. C Recall that the y-intercept is simply the y-value where a line
12345678901234567890123456789012123456789012345678901
crosses the y-axis. If the y-intercepts of two lines sum to zero, 2
12345678901234567890123456789012123456789012345678901
1 then they either both must have y-intercepts of zero, or they 2
12345678901234567890123456789012123456789012345678901 2
2345678901234567890123456789012123456789012345678901
must have y-intercepts equidistant from the x-axis, one with a 2
12345678901234567890123456789012123456789012345678901
12345678901234567890123456789012123456789012345678901 2
1 2
12345678901234567890123456789012123456789012345678901 2
positive value and the other a negative value. Line 2 and line 3
12345678901234567890123456789012123456789012345678901 2
12345678901234567890123456789012123456789012345678901
cross the y-axis equidistant from the x-axis. 2
12345678901234567890123456789012123456789012345678901 2
12345678901234567890123456789012123456789012345678901
18. C To begin with, since you know the circumference of the circles, 2
12345678901234567890123456789012123456789012345678901 2
12345678901234567890123456789012123456789012345678901
you can determine the radius of the circles, which is 3 (C 5 2pr). 2
2345678901234567890123456789012123456789012345678901
12345678901234567890123456789012123456789012345678901
If the radius is 3, then the height of the rectangle is 6, and the 2
2
12345678901234567890123456789012123456789012345678901 2
1
12345678901234567890123456789012123456789012345678901
length of the rectangle is 12. The distance between two opposite 2
12345678901234567890123456789012123456789012345678901 2
12345678901234567890123456789012123456789012345678901
corners then is the hypotenuse of a right triangle with sides 6 2
12345678901234567890123456789012123456789012345678901 2
12345678901234567890123456789012123456789012345678901
and 12. Using the Pythagorean theorem we can determine the 2
12345678901234567890123456789012123456789012345678901 2
12345678901234567890123456789012123456789012345678901
hypotenuse, =36 1 144 5 =180. Choice (C) then is the 2
12345678901234567890123456789012123456789012345678901 22
2345678901234567890123456789012123456789012345678901
12345678901234567890123456789012123456789012345678901
answer.
2
12345678901234567890123456789012123456789012345678901 2
12345678901234567890123456789012123456789012345678901 2
1 19. C Three students constitute a 20-percent increase for the class.
2345678901234567890123456789012123456789012345678901
12345678901234567890123456789012123456789012345678901 2
12345678901234567890123456789012123456789012345678901
This translates mathematically to 2
12345678901234567890123456789012123456789012345678901 22
12345678901234567890123456789012123456789012345678901 2
1 3 3
12345678901234567890123456789012123456789012345678901
0.20 5 → x 5 5 15, 2
12345678901234567890123456789012123456789012345678901
x 0.20 2
12345678901234567890123456789012123456789012345678901 2
12345678901234567890123456789012123456789012345678901 2
12345678901234567890123456789012123456789012345678901
which gives you the number of
2345678901234567890123456789012123456789012345678901
original students. Add 3 to 2
2
1
12345678901234567890123456789012123456789012345678901
get 18. 2
12345678901234567890123456789012123456789012345678901 22
12345678901234567890123456789012123456789012345678901 321
123456789012345678901234567890121234567890123456789012
Part IV: Three Practice GREs

123456789012345678901234567890121234567890123456789012
12345678901234567890123456789012123456789012345678901 2
12345678901234567890123456789012123456789012345678901 2
12345678901234567890123456789012123456789012345678901
20. B Drawing a square with a diagonal creates a 45-45-90 triangle, 2
12345678901234567890123456789012123456789012345678901 2
12345678901234567890123456789012123456789012345678901
since the diagonal bisects the 90 degree angles. This means that 2
12345678901234567890123456789012123456789012345678901 2
12345678901234567890123456789012123456789012345678901
the diagonal is 4=2. But you are not looking for the length 2
12345678901234567890123456789012123456789012345678901 2
12345678901234567890123456789012123456789012345678901
from corner to corner. You are looking for the length from 2
12345678901234567890123456789012123456789012345678901 2
12345678901234567890123456789012123456789012345678901
center to corner, which would be half the length of the diagonal, 2
12345678901234567890123456789012123456789012345678901 2
12345678901234567890123456789012123456789012345678901
or 2=2, choice (B). 2
12345678901234567890123456789012123456789012345678901 2
12345678901234567890123456789012123456789012345678901 2
12345678901234567890123456789012123456789012345678901
B Three pounds of peanuts at 34 cents/pound costs 102 cents. Five 2
12345678901234567890123456789012123456789012345678901
21.
2345678901234567890123456789012123456789012345678901
2
1 pounds of cashews at 90 cents/pound costs 450 cents. Added 2
12345678901234567890123456789012123456789012345678901 2
12345678901234567890123456789012123456789012345678901
together they cost 552 cents, and there are 8 pounds total. 552 2
12345678901234567890123456789012123456789012345678901 2
12345678901234567890123456789012123456789012345678901
cents divided by 8 pounds is 69 cents/pound, or choice (B).
2345678901234567890123456789012123456789012345678901
2
2
1
12345678901234567890123456789012123456789012345678901 2
12345678901234567890123456789012123456789012345678901
22. D The first thing to do is to convert the measurements in inches 2
12345678901234567890123456789012123456789012345678901 2
12345678901234567890123456789012123456789012345678901
1 1 2
2345678901234567890123456789012123456789012345678901
12345678901234567890123456789012123456789012345678901
into feet. Six inches is a foot. Two inches is a foot. The 2
12345678901234567890123456789012123456789012345678901
2 6 2
12345678901234567890123456789012123456789012345678901 2
12345678901234567890123456789012123456789012345678901 2
surface area of a three-dimensional rectangle is the sum of areas
12345678901234567890123456789012123456789012345678901 2
12345678901234567890123456789012123456789012345678901
of the sides. 2
1 2
12345678901234567890123456789012123456789012345678901
Top and bottom: 2
12345678901234567890123456789012123456789012345678901 2
12345678901234567890123456789012123456789012345678901 2
2345678901234567890123456789012123456789012345678901
12345678901234567890123456789012123456789012345678901 2
2 6•
1
S D
12345678901234567890123456789012123456789012345678901
12345678901234567890123456789012123456789012345678901
5 6,
12345678901234567890123456789012123456789012345678901 2
2
2
2
12345678901234567890123456789012123456789012345678901 2
12345678901234567890123456789012123456789012345678901
two ends: 2
12345678901234567890123456789012123456789012345678901 2
12345678901234567890123456789012123456789012345678901 2
12345678901234567890123456789012123456789012345678901 2
1 2
1 1
• S D 5
1
2345678901234567890123456789012123456789012345678901
12345678901234567890123456789012123456789012345678901
,
12345678901234567890123456789012123456789012345678901
6 2 6
2
2
2
1 2
12345678901234567890123456789012123456789012345678901
two sides: 2
12345678901234567890123456789012123456789012345678901 2
12345678901234567890123456789012123456789012345678901 2
2345678901234567890123456789012123456789012345678901
12345678901234567890123456789012123456789012345678901 2
1 1
6
• S D
12345678901234567890123456789012123456789012345678901
2 6 5 2
12345678901234567890123456789012123456789012345678901
2
2
2
12345678901234567890123456789012123456789012345678901 2
2345678901234567890123456789012123456789012345678901
12345678901234567890123456789012123456789012345678901
The sum of these is choice (D). 2
1 2
12345678901234567890123456789012123456789012345678901 2
12345678901234567890123456789012123456789012345678901 2
23. A Looking at the frequency column, the frequency increases as you
2345678901234567890123456789012123456789012345678901
12345678901234567890123456789012123456789012345678901
ascend the column. The same thing is true for the energy level 2
2
12345678901234567890123456789012123456789012345678901 2
12345678901234567890123456789012123456789012345678901
column—as you ascend the column the energy level increases. 2
12345678901234567890123456789012123456789012345678901
1 Thus, (A) is the answer. If you want further confirmation of 2
12345678901234567890123456789012123456789012345678901 2
12345678901234567890123456789012123456789012345678901 2
12345678901234567890123456789012123456789012345678901
this—simply pick a certain frequency and read the energy level 2
12345678901234567890123456789012123456789012345678901 2
12345678901234567890123456789012123456789012345678901 2
at that frequency. Then pick another frequency and read the
2345678901234567890123456789012123456789012345678901
energy level at that frequency. If the frequency increased—then 2
12345678901234567890123456789012123456789012345678901
12345678901234567890123456789012123456789012345678901 2
12345678901234567890123456789012123456789012345678901 2
12345678901234567890123456789012123456789012345678901 2
so did the energy and vice versa.
2
1
12345678901234567890123456789012123456789012345678901 2
12345678901234567890123456789012123456789012345678901 2
12345678901234567890123456789012123456789012345678901 2
12345678901234567890123456789012123456789012345678901 2
12345678901234567890123456789012123456789012345678901 2
12345678901234567890123456789012123456789012345678901 2
12345678901234567890123456789012123456789012345678901 2
12345678901234567890123456789012123456789012345678901 2 2
322 12345678901234567890123456789012123456789012345678901
123456789012345678901234567890121234567890123456789012

www.petersons.com
ANSWERS
123456789012345678901234567890121234567890123456789012
12345678901234567890123456789012123456789012345678901 2
12345678901234567890123456789012123456789012345678901 2
12345678901234567890123456789012123456789012345678901
24. A On this question, you have to be careful. Notice that the 2
12345678901234567890123456789012123456789012345678901 2
12345678901234567890123456789012123456789012345678901
wavelengths get shorter as they ascend the column (e.g., 10213 2
12345678901234567890123456789012123456789012345678901 2
12345678901234567890123456789012123456789012345678901
meters is shorter than 105 meters). This means that the shortest 2
2345678901234567890123456789012123456789012345678901 2
1
12345678901234567890123456789012123456789012345678901
wavelengths are at the top of the column. Which kind of 2
12345678901234567890123456789012123456789012345678901
radiation corresponds to this part of the column? Choice (A), 2
12345678901234567890123456789012123456789012345678901 2
12345678901234567890123456789012123456789012345678901
2345678901234567890123456789012123456789012345678901
2
2
1 gamma rays, is correct.
12345678901234567890123456789012123456789012345678901 2
12345678901234567890123456789012123456789012345678901
22 2
12345678901234567890123456789012123456789012345678901
25. A First find 10 Hz on the frequency column on the far left. Then 2
12345678901234567890123456789012123456789012345678901
2345678901234567890123456789012123456789012345678901
2
2
1 slide over to see which kind of radiation this is. It is gamma rays,
12345678901234567890123456789012123456789012345678901 2
12345678901234567890123456789012123456789012345678901
choice (A). 2
12345678901234567890123456789012123456789012345678901 2
12345678901234567890123456789012123456789012345678901
26. B On a problem like this, you have to go through the three choices 2
2345678901234567890123456789012123456789012345678901 2
1
12345678901234567890123456789012123456789012345678901
and determine which must be odd as follows: 2
12345678901234567890123456789012123456789012345678901 2
12345678901234567890123456789012123456789012345678901 2
12345678901234567890123456789012123456789012345678901
Option I—Is not odd since the two products are even and an 2
2345678901234567890123456789012123456789012345678901
12345678901234567890123456789012123456789012345678901 2
12345678901234567890123456789012123456789012345678901
even minus an even is even. 2
12345678901234567890123456789012123456789012345678901 22
12345678901234567890123456789012123456789012345678901
Option II—Is odd since an odd times an odd is even and an 2
12345678901234567890123456789012123456789012345678901
12345678901234567890123456789012123456789012345678901 2
1 even times an odd is odd. 2
12345678901234567890123456789012123456789012345678901 2
12345678901234567890123456789012123456789012345678901
Option III—Is even since an odd squared is odd and an odd 2
12345678901234567890123456789012123456789012345678901 2
2345678901234567890123456789012123456789012345678901
12345678901234567890123456789012123456789012345678901
plus an odd is even. 2
12345678901234567890123456789012123456789012345678901 2
12345678901234567890123456789012123456789012345678901
So only option II is odd, which is answer choice (B). If all that 2
2
12345678901234567890123456789012123456789012345678901
12345678901234567890123456789012123456789012345678901
seemed a bit confusing, just assign odd values to the three 2
2
12345678901234567890123456789012123456789012345678901
12345678901234567890123456789012123456789012345678901 2
12345678901234567890123456789012123456789012345678901
variables, plug them in, and see if the results are odd or even. 2
12345678901234567890123456789012123456789012345678901 22
1 27. E There is a helpful formula for a problem of balancing weaker and
12345678901234567890123456789012123456789012345678901 2
12345678901234567890123456789012123456789012345678901 2
12345678901234567890123456789012123456789012345678901 2
stronger solutions, (amount of weaker solution) times (percent-
12345678901234567890123456789012123456789012345678901
age difference of desired and weaker solution) 5 (amount of 2
12345678901234567890123456789012123456789012345678901
2345678901234567890123456789012123456789012345678901
2
stronger solution) times (percentage difference of stronger and 2
12345678901234567890123456789012123456789012345678901
12345678901234567890123456789012123456789012345678901 2
1 2
12345678901234567890123456789012123456789012345678901
desired solution). In this situation, the formula reads, n(25 2 20) 2
12345678901234567890123456789012123456789012345678901
→ → 2
12345678901234567890123456789012123456789012345678901 22
2345678901234567890123456789012123456789012345678901
5 3(40 2 25) 5n 5 45 n 5 9. Therefore, (E) is the answer.
12345678901234567890123456789012123456789012345678901 2
1 28. E There is a formula for neither or both problems like this,
12345678901234567890123456789012123456789012345678901 2
12345678901234567890123456789012123456789012345678901
G1 1 G2 1 Neither 1 Both 5 Total. You can use this equation 2
12345678901234567890123456789012123456789012345678901 2
12345678901234567890123456789012123456789012345678901
or reason your way through this problem. If 52 students are in 2
12345678901234567890123456789012123456789012345678901 2
12345678901234567890123456789012123456789012345678901
neither class, then 98 students are in algebra or geometry. This 2
12345678901234567890123456789012123456789012345678901 2
12345678901234567890123456789012123456789012345678901
means that 25 are in algebra but not geometry and 36 are in 2
12345678901234567890123456789012123456789012345678901 2
12345678901234567890123456789012123456789012345678901
geometry but not algebra. So, there are 73 in geometry and 36 of 2
12345678901234567890123456789012123456789012345678901
them are not in algebra, which means that 37 of those in 2
12345678901234567890123456789012123456789012345678901 2
2345678901234567890123456789012123456789012345678901
geometry are in algebra. You can obtain the same result using 2
12345678901234567890123456789012123456789012345678901
12345678901234567890123456789012123456789012345678901 2
12345678901234567890123456789012123456789012345678901 2
12345678901234567890123456789012123456789012345678901 22
the formula.
1
12345678901234567890123456789012123456789012345678901 2
12345678901234567890123456789012123456789012345678901 2
12345678901234567890123456789012123456789012345678901 2
12345678901234567890123456789012123456789012345678901 2
12345678901234567890123456789012123456789012345678901 2
12345678901234567890123456789012123456789012345678901 2
12345678901234567890123456789012123456789012345678901 2
12345678901234567890123456789012123456789012345678901 2
12345678901234567890123456789012123456789012345678901 2 323
123456789012345678901234567890121234567890123456789012
Part IV: Three Practice GREs

123456789012345678901234567890121234567890123456789012
12345678901234567890123456789012123456789012345678901 2
12345678901234567890123456789012123456789012345678901 2
12345678901234567890123456789012123456789012345678901
Verbal 2
12345678901234567890123456789012123456789012345678901 2
12345678901234567890123456789012123456789012345678901 2
12345678901234567890123456789012123456789012345678901
Antonyms 2
12345678901234567890123456789012123456789012345678901 2
1 1. B To corroborate someone’s claim is to back it up or validate it, so 2
2345678901234567890123456789012123456789012345678901
12345678901234567890123456789012123456789012345678901 2
12345678901234567890123456789012123456789012345678901 2
12345678901234567890123456789012123456789012345678901
you are looking for a word that means something like “not 2
12345678901234567890123456789012123456789012345678901
2345678901234567890123456789012123456789012345678901
2
2
1 support.” Scanning the answer choices, only (B) fits the
12345678901234567890123456789012123456789012345678901 2
12345678901234567890123456789012123456789012345678901
pre-guess. 2
12345678901234567890123456789012123456789012345678901 2
12345678901234567890123456789012123456789012345678901
B Inexactitude is the opposite of exactitude,
2345678901234567890123456789012123456789012345678901 or being exact, so you 2
2
1 2.
12345678901234567890123456789012123456789012345678901
are looking for a word that means exact. Precise, choice (B), 2
12345678901234567890123456789012123456789012345678901 2
12345678901234567890123456789012123456789012345678901
certainly means exact, and none of the other answer choices do. 2
12345678901234567890123456789012123456789012345678901 2
12345678901234567890123456789012123456789012345678901 2
12345678901234567890123456789012123456789012345678901
3. A To foment means to incite, and so its antonym would be 2
12345678901234567890123456789012123456789012345678901 2
12345678901234567890123456789012123456789012345678901
something like “un-incite.” Not a real word, but a fine pre-guess 2
12345678901234567890123456789012123456789012345678901 2
12345678901234567890123456789012123456789012345678901
since it puts you on the right path. Answer choice (A), quell, 2
12345678901234567890123456789012123456789012345678901 2
12345678901234567890123456789012123456789012345678901
matches this pre-guess. 2
12345678901234567890123456789012123456789012345678901 2
12345678901234567890123456789012123456789012345678901 2
12345678901234567890123456789012123456789012345678901 2
2345678901234567890123456789012123456789012345678901
4. B Callow has a negative-sounding connotation, and so the answer
1 will have a positive-sounding connotation. You can, at least, 2
12345678901234567890123456789012123456789012345678901 2
12345678901234567890123456789012123456789012345678901 2
12345678901234567890123456789012123456789012345678901
eliminate choice (C), timid, on these grounds. Callow means 2
2345678901234567890123456789012123456789012345678901
12345678901234567890123456789012123456789012345678901 2
12345678901234567890123456789012123456789012345678901
immature. Choice (B), mature, then, is a good antonym. 2
12345678901234567890123456789012123456789012345678901 2
5. D Bacchanalian derives from Bacchus, the Greek god of wine. It 2
12345678901234567890123456789012123456789012345678901
12345678901234567890123456789012123456789012345678901 2
means “unbridled revelry.” The opposite is something like 2
12345678901234567890123456789012123456789012345678901
12345678901234567890123456789012123456789012345678901
sobriety, or a tame activity. Of the answer choices, pious best 2
2
12345678901234567890123456789012123456789012345678901
12345678901234567890123456789012123456789012345678901 2
1 approximates this pre-guess. 2
12345678901234567890123456789012123456789012345678901 2
2345678901234567890123456789012123456789012345678901
12345678901234567890123456789012123456789012345678901 2
1 6. E Pulchritude means beauty. This word was used in the first part 2
12345678901234567890123456789012123456789012345678901 2
12345678901234567890123456789012123456789012345678901
of the book, so hopefully you remembered it. 2
12345678901234567890123456789012123456789012345678901 2
2345678901234567890123456789012123456789012345678901
12345678901234567890123456789012123456789012345678901 2
1 7. D To be stolid is to lack emotion. Thus, the antonym here is 2
12345678901234567890123456789012123456789012345678901 2
12345678901234567890123456789012123456789012345678901
something like “emotional.” Choice (D) is exactly that. 2
12345678901234567890123456789012123456789012345678901 2
2345678901234567890123456789012123456789012345678901
8. A If you do not know the definition of prevaricate, you might 2
12345678901234567890123456789012123456789012345678901
1 2
12345678901234567890123456789012123456789012345678901
guess that it has a negative-sounding connotation, and so the 2
12345678901234567890123456789012123456789012345678901 2
12345678901234567890123456789012123456789012345678901
answer would be positive. Choices (A) and (E) are the strong 2
12345678901234567890123456789012123456789012345678901 2
12345678901234567890123456789012123456789012345678901
positives. Prevaricate means to lie, and so (A) is the correct 2
12345678901234567890123456789012123456789012345678901 2
12345678901234567890123456789012123456789012345678901
answer. 2
12345678901234567890123456789012123456789012345678901 2
2345678901234567890123456789012123456789012345678901
12345678901234567890123456789012123456789012345678901 2
12345678901234567890123456789012123456789012345678901 2 2
12345678901234567890123456789012123456789012345678901 2
1 Analogies
12345678901234567890123456789012123456789012345678901 2
2345678901234567890123456789012123456789012345678901
9. C Someone who is long-winded lacks brevity. Choices (B) and (C) 2
12345678901234567890123456789012123456789012345678901
12345678901234567890123456789012123456789012345678901 2
appear to be possibilities. Attractive is the opposite of repulsive. 2
12345678901234567890123456789012123456789012345678901
1 2
2345678901234567890123456789012123456789012345678901
Something lean lacks excess. Choice (C) is the better of the two 2
12345678901234567890123456789012123456789012345678901
1 2
12345678901234567890123456789012123456789012345678901
choices. 2
12345678901234567890123456789012123456789012345678901 2
12345678901234567890123456789012123456789012345678901 2
12345678901234567890123456789012123456789012345678901 2
12345678901234567890123456789012123456789012345678901 2
12345678901234567890123456789012123456789012345678901 2 2
324 12345678901234567890123456789012123456789012345678901
123456789012345678901234567890121234567890123456789012

www.petersons.com
ANSWERS
123456789012345678901234567890121234567890123456789012
12345678901234567890123456789012123456789012345678901 2
12345678901234567890123456789012123456789012345678901 2
12345678901234567890123456789012123456789012345678901
10. E A lawyer is an officer of the court, or a lawyer works in the 2
12345678901234567890123456789012123456789012345678901 2
12345678901234567890123456789012123456789012345678901
court. An actor acts in a play, but a play is an activity whereas a 2
12345678901234567890123456789012123456789012345678901 2
12345678901234567890123456789012123456789012345678901
court is a location or part of a system. A janitor might work at a 2
2345678901234567890123456789012123456789012345678901 2
1
12345678901234567890123456789012123456789012345678901
school. A manager manages a team, a prison has guards, and a 2
12345678901234567890123456789012123456789012345678901 2
12345678901234567890123456789012123456789012345678901
bureaucrat works in a bureaucracy. 2
12345678901234567890123456789012123456789012345678901 2
1 11. B The relationship between the stem pair is one of definition; to be 2
2345678901234567890123456789012123456789012345678901
12345678901234567890123456789012123456789012345678901 2
12345678901234567890123456789012123456789012345678901 2
12345678901234567890123456789012123456789012345678901
candid is to be frank. Which of the answer choices shares this 2
12345678901234567890123456789012123456789012345678901
2345678901234567890123456789012123456789012345678901
2
2
1 relationship? Only (B)—to be erudite is to be learned. Choice
12345678901234567890123456789012123456789012345678901 2
12345678901234567890123456789012123456789012345678901
(A) might have seemed appealing but realize that a sincere 2
12345678901234567890123456789012123456789012345678901
person might be kind or he or she might be sincerely unkind. 2
12345678901234567890123456789012123456789012345678901
2345678901234567890123456789012123456789012345678901
2
2
1
12345678901234567890123456789012123456789012345678901
12. B Here, the stem relationship is of lack or negation—to be austere 2
12345678901234567890123456789012123456789012345678901 2
12345678901234567890123456789012123456789012345678901
is to not be decorated. In a similar fashion, to be offensive is to 2
12345678901234567890123456789012123456789012345678901 2
12345678901234567890123456789012123456789012345678901
not be tactful. None of the other stem pairs share the 2
12345678901234567890123456789012123456789012345678901 2
12345678901234567890123456789012123456789012345678901
relationship of negation. 2
12345678901234567890123456789012123456789012345678901 2
12345678901234567890123456789012123456789012345678901 2
12345678901234567890123456789012123456789012345678901 22
2345678901234567890123456789012123456789012345678901
13. A Physician studies/cares for the human body. In the same way, a
1
12345678901234567890123456789012123456789012345678901
horticulturist studies/cares for a plant. But it is also true that 2
12345678901234567890123456789012123456789012345678901
veterinarians study/care for cows. And certainly biologists study 2
12345678901234567890123456789012123456789012345678901 2
2345678901234567890123456789012123456789012345678901
evolution. But notice that biologists do not care for evolution. 2
12345678901234567890123456789012123456789012345678901
12345678901234567890123456789012123456789012345678901 2
12345678901234567890123456789012123456789012345678901 2
12345678901234567890123456789012123456789012345678901 22
Also notice that a cow is a specific kind of animal that a
12345678901234567890123456789012123456789012345678901
veterinarian cares for while human body and plant are general 2
12345678901234567890123456789012123456789012345678901
12345678901234567890123456789012123456789012345678901 2
terms. So, (A) is the best choice.
12345678901234567890123456789012123456789012345678901 2
12345678901234567890123456789012123456789012345678901 22
1 14. B Craven means “cowardly” and “completely subdued by fear.”
12345678901234567890123456789012123456789012345678901 2
12345678901234567890123456789012123456789012345678901
Trepidatious means “to be apprehensive or fearful.” Thus— 2
12345678901234567890123456789012123456789012345678901 2
12345678901234567890123456789012123456789012345678901
craven is an extreme of trepidatiousness. This is a relationship of 2
12345678901234567890123456789012123456789012345678901 2
12345678901234567890123456789012123456789012345678901
degree. Only choice (E) shares this relationship of degree— 2
12345678901234567890123456789012123456789012345678901 2
12345678901234567890123456789012123456789012345678901
exultant is an extreme of joyfulness. Choice (A) is not as strong 2
12345678901234567890123456789012123456789012345678901 2
12345678901234567890123456789012123456789012345678901
a choice because miniscule and minute both mean very small, 2
2345678901234567890123456789012123456789012345678901
and so there is no difference in degree. The same is true for 2
12345678901234567890123456789012123456789012345678901
12345678901234567890123456789012123456789012345678901 2
1 2
12345678901234567890123456789012123456789012345678901 2
migratory and transitory, choice (B). As for (C), ageless means
12345678901234567890123456789012123456789012345678901
without age while ancient means to by very old. They do not 2
12345678901234567890123456789012123456789012345678901 2
12345678901234567890123456789012123456789012345678901 2
12345678901234567890123456789012123456789012345678901
have a relationship of degree. 2
12345678901234567890123456789012123456789012345678901 2
12345678901234567890123456789012123456789012345678901 2
12345678901234567890123456789012123456789012345678901 22
2345678901234567890123456789012123456789012345678901
15. B If you do not know the definition of either of these words, you
12345678901234567890123456789012123456789012345678901
can eliminate some of the answer pairs that have a weak
2
12345678901234567890123456789012123456789012345678901
relationship. Catatonic and cataclysmic do not have a necessary 2
12345678901234567890123456789012123456789012345678901
1 2
12345678901234567890123456789012123456789012345678901
relationship, nor do histrionic (to be overly theatrical) and 2
12345678901234567890123456789012123456789012345678901 2
12345678901234567890123456789012123456789012345678901
sordid. That leaves (A), (B), and (C). Turgid actually means 2
12345678901234567890123456789012123456789012345678901 2
12345678901234567890123456789012123456789012345678901
pompous or bombastic, and so the stem relationship is one of 2
2345678901234567890123456789012123456789012345678901
12345678901234567890123456789012123456789012345678901
definition. To be turgid is to be bombastic. Is to be lazy to be 2
2
1
12345678901234567890123456789012123456789012345678901 2
12345678901234567890123456789012123456789012345678901
lethargic? Maybe. To be rotund is to be portly. And to bemuse is 2
12345678901234567890123456789012123456789012345678901 2
12345678901234567890123456789012123456789012345678901
to bewilder, not to distract. 2
12345678901234567890123456789012123456789012345678901 2
12345678901234567890123456789012123456789012345678901 2
12345678901234567890123456789012123456789012345678901 2 325
123456789012345678901234567890121234567890123456789012
Part IV: Three Practice GREs

123456789012345678901234567890121234567890123456789012
12345678901234567890123456789012123456789012345678901 2
12345678901234567890123456789012123456789012345678901 2
12345678901234567890123456789012123456789012345678901
Sentence Completions 2
12345678901234567890123456789012123456789012345678901 2
12345678901234567890123456789012123456789012345678901 2
12345678901234567890123456789012123456789012345678901 2
16. A What does a successful demolition crew do? It demolishes
12345678901234567890123456789012123456789012345678901
buildings, and so you are looking for a word like demolish. 2
12345678901234567890123456789012123456789012345678901 2
12345678901234567890123456789012123456789012345678901 2
12345678901234567890123456789012123456789012345678901
Which of the answer choices fits this pre-guess? Certainly not 2
12345678901234567890123456789012123456789012345678901 2
12345678901234567890123456789012123456789012345678901
construct or erect. Incapacitate and
2345678901234567890123456789012123456789012345678901
deactivate do not fit the 2
1 context. Raze means to destroy completely, so it completes the 2
12345678901234567890123456789012123456789012345678901 2
12345678901234567890123456789012123456789012345678901 2
12345678901234567890123456789012123456789012345678901
sentence well. 2
12345678901234567890123456789012123456789012345678901 2
1 17. A Begin with a pre-guess. Something like punishable/responsible 2
2345678901234567890123456789012123456789012345678901
12345678901234567890123456789012123456789012345678901 2
12345678901234567890123456789012123456789012345678901
works, since it is generally thought that if something is an 2
12345678901234567890123456789012123456789012345678901 2
12345678901234567890123456789012123456789012345678901
accident then one is not fully punishable/responsible. Looking 2
2345678901234567890123456789012123456789012345678901 2
1
12345678901234567890123456789012123456789012345678901
down the list, culpable matches our pre-guess. Reading it back 2
12345678901234567890123456789012123456789012345678901 2
12345678901234567890123456789012123456789012345678901
into the sentence, it works well. 2
12345678901234567890123456789012123456789012345678901 2
2345678901234567890123456789012123456789012345678901
12345678901234567890123456789012123456789012345678901
18. E Remember to do a two blank sentence one blank at a time. The 2
2
12345678901234567890123456789012123456789012345678901
first blank is clearly a negative term since publicity surrounding 2
12345678901234567890123456789012123456789012345678901
12345678901234567890123456789012123456789012345678901 2
12345678901234567890123456789012123456789012345678901 2
12345678901234567890123456789012123456789012345678901 2
mismanagement would hurt a company’s credibility. Thus, we
1 can eliminate choice (C), aided. Belabored, choice (A), also does 2
12345678901234567890123456789012123456789012345678901 2
12345678901234567890123456789012123456789012345678901 2
12345678901234567890123456789012123456789012345678901
not fit with the notion of hurt. On the second blank, we can 2
2345678901234567890123456789012123456789012345678901
infer that it must be a positive since it is on its account that the 2
12345678901234567890123456789012123456789012345678901
12345678901234567890123456789012123456789012345678901 2
12345678901234567890123456789012123456789012345678901 2
12345678901234567890123456789012123456789012345678901 2
company remained solvent. This eliminates (D) and (B) also,
12345678901234567890123456789012123456789012345678901 2
12345678901234567890123456789012123456789012345678901
since evolving is a more neutral than positive term. 2
12345678901234567890123456789012123456789012345678901 2
19. B The though at the beginning of the sentence clues you into the 2
12345678901234567890123456789012123456789012345678901
12345678901234567890123456789012123456789012345678901 2
1 fact that the official’s hopes were not achieved. Thus, his 2
12345678901234567890123456789012123456789012345678901 2
2345678901234567890123456789012123456789012345678901
assurances must not have had their desired effect. Assurances 2
12345678901234567890123456789012123456789012345678901
1 2
12345678901234567890123456789012123456789012345678901
are meant to calm, and that is a good pre-guess. Looking at the 2
12345678901234567890123456789012123456789012345678901 2
12345678901234567890123456789012123456789012345678901
answer choices, assuage, choice (B), is the only choice that 2
12345678901234567890123456789012123456789012345678901 2
12345678901234567890123456789012123456789012345678901
matches the pre-guess. 2
12345678901234567890123456789012123456789012345678901 2
12345678901234567890123456789012123456789012345678901 2
12345678901234567890123456789012123456789012345678901 2
2345678901234567890123456789012123456789012345678901
20. D The sentence structure indicates that the word in the blank is
1 described by the younger sibling . . . taking on mannerisms . . . 2
12345678901234567890123456789012123456789012345678901 2
12345678901234567890123456789012123456789012345678901 2
12345678901234567890123456789012123456789012345678901 2
What are the younger siblings doing? They are copying their
2345678901234567890123456789012123456789012345678901
older siblings. Which answer choice matches with copy? 2
12345678901234567890123456789012123456789012345678901
12345678901234567890123456789012123456789012345678901 2
12345678901234567890123456789012123456789012345678901 2
12345678901234567890123456789012123456789012345678901 2
Emulate, choice (D), means to copy, and it is the best answer
2
1
12345678901234567890123456789012123456789012345678901
choice. 2
12345678901234567890123456789012123456789012345678901 2
12345678901234567890123456789012123456789012345678901
21. A Start with the first blank here. Since the visitor was unfamiliar 2
12345678901234567890123456789012123456789012345678901 2
12345678901234567890123456789012123456789012345678901
with the region’s customs and you know that he offended his 2
2345678901234567890123456789012123456789012345678901
12345678901234567890123456789012123456789012345678901
host, you should suspect that his actions were culturally 2
2
12345678901234567890123456789012123456789012345678901 2
12345678901234567890123456789012123456789012345678901
improper. Looking at the answer choices, we can eliminate (C). 2
12345678901234567890123456789012123456789012345678901 2
1
12345678901234567890123456789012123456789012345678901
Moving to the second blank, since he was unaware of his poor 2
12345678901234567890123456789012123456789012345678901 2
12345678901234567890123456789012123456789012345678901
actions, it makes sense that he did not realize he had offended 2
12345678901234567890123456789012123456789012345678901
his hosts. Unaware does not fit the context, and blind does not 2
12345678901234567890123456789012123456789012345678901
2345678901234567890123456789012123456789012345678901
2
1 fit as well as oblivious or ignorant. That leaves choices (A) and 2
12345678901234567890123456789012123456789012345678901 2
12345678901234567890123456789012123456789012345678901 2 2
326 12345678901234567890123456789012123456789012345678901
123456789012345678901234567890121234567890123456789012

www.petersons.com
ANSWERS
123456789012345678901234567890121234567890123456789012
12345678901234567890123456789012123456789012345678901 2
12345678901234567890123456789012123456789012345678901 2
12345678901234567890123456789012123456789012345678901
(B). Reading both back into the sentence, (A) works better. 2
12345678901234567890123456789012123456789012345678901 2
12345678901234567890123456789012123456789012345678901
Curtness does not capture the idea of culture improperness like 2
12345678901234567890123456789012123456789012345678901 2
12345678901234567890123456789012123456789012345678901
impropriety does.
2345678901234567890123456789012123456789012345678901
2
2
1
12345678901234567890123456789012123456789012345678901 2
12345678901234567890123456789012123456789012345678901
22. E The second blank is quite approachable since you know that it 2
12345678901234567890123456789012123456789012345678901 2
12345678901234567890123456789012123456789012345678901
means “allowing no light through”
2345678901234567890123456789012123456789012345678901
or something like that. 2
1 Which answer choices fit this? Opaque, choice (E), really is the 2
12345678901234567890123456789012123456789012345678901 2
12345678901234567890123456789012123456789012345678901 2
12345678901234567890123456789012123456789012345678901
only good fit. Translucent works well in the first blank, and so 2
12345678901234567890123456789012123456789012345678901
2345678901234567890123456789012123456789012345678901
2
2
1 (E) is the best choice.
12345678901234567890123456789012123456789012345678901 2
12345678901234567890123456789012123456789012345678901
23. A Here the sentence is contrasting the way that history was written 2
12345678901234567890123456789012123456789012345678901 2
12345678901234567890123456789012123456789012345678901
in the past with the professional standards of history writing 2
2345678901234567890123456789012123456789012345678901 2
1
12345678901234567890123456789012123456789012345678901
today. In the past, the sentence reads, historiography . . . tended 2
12345678901234567890123456789012123456789012345678901 2
12345678901234567890123456789012123456789012345678901
to hagiography. In this usage, hagiography means overly 2
12345678901234567890123456789012123456789012345678901 2
12345678901234567890123456789012123456789012345678901
sympathetic and full of praise. Thus, today’s requirements 2
12345678901234567890123456789012123456789012345678901 2
12345678901234567890123456789012123456789012345678901
should be in strong contrast to this. With this in mind, which of 2
12345678901234567890123456789012123456789012345678901 2
12345678901234567890123456789012123456789012345678901
the answer choices is a good modifier for tone in the second 2
2345678901234567890123456789012123456789012345678901
blank? Reserved and careful are good choices. Looking at the 2
12345678901234567890123456789012123456789012345678901
1 2
12345678901234567890123456789012123456789012345678901 2
12345678901234567890123456789012123456789012345678901
first blank, even, choice (E), does not fit with distance, while 2
12345678901234567890123456789012123456789012345678901 2
12345678901234567890123456789012123456789012345678901 22
2345678901234567890123456789012123456789012345678901
critical does fit well. Therefore, choice (A) is the best choice.
12345678901234567890123456789012123456789012345678901 2
12345678901234567890123456789012123456789012345678901 2
12345678901234567890123456789012123456789012345678901 2
12345678901234567890123456789012123456789012345678901
Reading Comprehension
12345678901234567890123456789012123456789012345678901 2
12345678901234567890123456789012123456789012345678901
24. E The passage reads, “The semi-empirical practitioners . . . 2
2
12345678901234567890123456789012123456789012345678901
12345678901234567890123456789012123456789012345678901
criticized the ab initio methodology as overly laborious and 2
2
1
2345678901234567890123456789012123456789012345678901
ultimately less productive.” That is I and III. II, on the other 2
12345678901234567890123456789012123456789012345678901
12345678901234567890123456789012123456789012345678901 2
1 2
12345678901234567890123456789012123456789012345678901 2
hand, is actually a criticism of the semi-empirical method.
12345678901234567890123456789012123456789012345678901 2
12345678901234567890123456789012123456789012345678901
25. B The second half of the first paragraph clearly explains that the 2
12345678901234567890123456789012123456789012345678901 2
12345678901234567890123456789012123456789012345678901 2
12345678901234567890123456789012123456789012345678901
major hurdle in working with Schrödinger’s Equation was that 2
12345678901234567890123456789012123456789012345678901 2
12345678901234567890123456789012123456789012345678901 22
2345678901234567890123456789012123456789012345678901
it was difficult to solve. If you have been keeping track of the
12345678901234567890123456789012123456789012345678901
flow of ideas in the passage, it will be easy to locate the pertinent 2
1
12345678901234567890123456789012123456789012345678901
part of the text and answer the question. Choice (B) is correct. 2
12345678901234567890123456789012123456789012345678901 2
2345678901234567890123456789012123456789012345678901
12345678901234567890123456789012123456789012345678901
26. C Lines 27–31 read, “. . . the so-called semi-empirical method, 2
2
12345678901234567890123456789012123456789012345678901 2
12345678901234567890123456789012123456789012345678901
simplified the computational hurdles by neglecting possibly 2
12345678901234567890123456789012123456789012345678901 2
1
12345678901234567890123456789012123456789012345678901
negligible components of the calculations and by extensively 2
12345678901234567890123456789012123456789012345678901 2
12345678901234567890123456789012123456789012345678901
employed empirical data.” Thus, it was empirical in that it 2
12345678901234567890123456789012123456789012345678901
employed extensive empirical data in obtaining a result, which is 2
12345678901234567890123456789012123456789012345678901 2
2345678901234567890123456789012123456789012345678901
12345678901234567890123456789012123456789012345678901 2
12345678901234567890123456789012123456789012345678901
choice (C). 2
12345678901234567890123456789012123456789012345678901 22
12345678901234567890123456789012123456789012345678901
1 27. A A question about the author’s intent will include both the main 2
12345678901234567890123456789012123456789012345678901
idea of the passage and the author’s tone in expressing the main 2
12345678901234567890123456789012123456789012345678901 2
12345678901234567890123456789012123456789012345678901 2
12345678901234567890123456789012123456789012345678901 2
idea. What was the main idea of the passage? The introduction
12345678901234567890123456789012123456789012345678901
of electronic computers produced a bifurcation in methodolo- 2
12345678901234567890123456789012123456789012345678901 2
12345678901234567890123456789012123456789012345678901
gies in quantum chemistry. What was the author’s tone in 2
12345678901234567890123456789012123456789012345678901 2
12345678901234567890123456789012123456789012345678901 2 327
123456789012345678901234567890121234567890123456789012
Part IV: Three Practice GREs

123456789012345678901234567890121234567890123456789012
12345678901234567890123456789012123456789012345678901 2
12345678901234567890123456789012123456789012345678901 2
12345678901234567890123456789012123456789012345678901
expressing this idea? Primarily explanatory as opposed to 2
12345678901234567890123456789012123456789012345678901 2
12345678901234567890123456789012123456789012345678901
argumentative. Which answer choice captures both of these 2
12345678901234567890123456789012123456789012345678901 2
12345678901234567890123456789012123456789012345678901
elements? Choice (A) does. All the other answer choices are 2
2345678901234567890123456789012123456789012345678901 2
1
12345678901234567890123456789012123456789012345678901
either wrong in tone or wrong in content. 2
12345678901234567890123456789012123456789012345678901 2
12345678901234567890123456789012123456789012345678901 2
12345678901234567890123456789012123456789012345678901
28. D You should read with this question
2345678901234567890123456789012123456789012345678901
in mind, because questions 2
1 like this are standard. The passage discusses the inadequacies of 2
12345678901234567890123456789012123456789012345678901 2
12345678901234567890123456789012123456789012345678901 2
12345678901234567890123456789012123456789012345678901
current theories of wavy and dotted line pottery in the 2
12345678901234567890123456789012123456789012345678901
2345678901234567890123456789012123456789012345678901
2
2
1 Mesolithic-Neolithic transition, and points to the need for
12345678901234567890123456789012123456789012345678901 2
12345678901234567890123456789012123456789012345678901
further research in this area. The answer should include both of 2
12345678901234567890123456789012123456789012345678901
these aspects. Choice (A) includes the former but not the latter. 2
12345678901234567890123456789012123456789012345678901
2345678901234567890123456789012123456789012345678901
2
1 Choices (B) and (C) are too specific, while choice (E) is too 2
12345678901234567890123456789012123456789012345678901 2
12345678901234567890123456789012123456789012345678901
general. Only choice (D) contains both aspects of the passage, 2
12345678901234567890123456789012123456789012345678901 2
12345678901234567890123456789012123456789012345678901
the shortcomings of contemporary theories and the need for 2
12345678901234567890123456789012123456789012345678901 2
2345678901234567890123456789012123456789012345678901
12345678901234567890123456789012123456789012345678901
more research. 2
12345678901234567890123456789012123456789012345678901 2
12345678901234567890123456789012123456789012345678901 2
12345678901234567890123456789012123456789012345678901 2
29. C The passage clearly states that Arkell’s model was problematized
1 by the discovery of wavy and dotted line pottery outside of the 2
12345678901234567890123456789012123456789012345678901 2
12345678901234567890123456789012123456789012345678901 2
12345678901234567890123456789012123456789012345678901
Central Nile in strata and progressions that conflicted with his 2
12345678901234567890123456789012123456789012345678901 2
12345678901234567890123456789012123456789012345678901 2
2345678901234567890123456789012123456789012345678901
account. That is answer choice (C).
12345678901234567890123456789012123456789012345678901 2
30. B On inference questions like this, you have to bear down and 2
12345678901234567890123456789012123456789012345678901
12345678901234567890123456789012123456789012345678901 2
12345678901234567890123456789012123456789012345678901 2
12345678901234567890123456789012123456789012345678901 2
read through each answer choice and compare it with what you
know of the author through reading the passage. In choice (A), 2
12345678901234567890123456789012123456789012345678901
12345678901234567890123456789012123456789012345678901 2
the word thoroughly sounds extreme in comparison to the tone 2
12345678901234567890123456789012123456789012345678901
1 2
12345678901234567890123456789012123456789012345678901
of the passage and so should give you caution. Choice (B) 2
12345678901234567890123456789012123456789012345678901 2
12345678901234567890123456789012123456789012345678901
sounds like something the author would recommend. As for (C), 2
12345678901234567890123456789012123456789012345678901 2
12345678901234567890123456789012123456789012345678901
the author never makes any specific methodological suggestions 2
12345678901234567890123456789012123456789012345678901 2
12345678901234567890123456789012123456789012345678901
of this kind. Choice (D) might be something that the author 2
12345678901234567890123456789012123456789012345678901 2
12345678901234567890123456789012123456789012345678901
would agree with but the author never mentions composition in 2
12345678901234567890123456789012123456789012345678901 2
12345678901234567890123456789012123456789012345678901
the passage, and the same could be said for choice (E) and 2
12345678901234567890123456789012123456789012345678901 2
12345678901234567890123456789012123456789012345678901
material vs. non-material culture. Choice (B) is best. 2
12345678901234567890123456789012123456789012345678901 2
12345678901234567890123456789012123456789012345678901 2
12345678901234567890123456789012123456789012345678901 2
12345678901234567890123456789012123456789012345678901 2
12345678901234567890123456789012123456789012345678901 2
12345678901234567890123456789012123456789012345678901 2
12345678901234567890123456789012123456789012345678901 2
12345678901234567890123456789012123456789012345678901 2
2345678901234567890123456789012123456789012345678901
12345678901234567890123456789012123456789012345678901 2
12345678901234567890123456789012123456789012345678901 2
12345678901234567890123456789012123456789012345678901 2 2
1
12345678901234567890123456789012123456789012345678901 2
12345678901234567890123456789012123456789012345678901 2
12345678901234567890123456789012123456789012345678901 2
12345678901234567890123456789012123456789012345678901 2
12345678901234567890123456789012123456789012345678901 2
12345678901234567890123456789012123456789012345678901 2
2345678901234567890123456789012123456789012345678901
2
1
12345678901234567890123456789012123456789012345678901 2
12345678901234567890123456789012123456789012345678901 2
12345678901234567890123456789012123456789012345678901 2
12345678901234567890123456789012123456789012345678901 2
12345678901234567890123456789012123456789012345678901 2
12345678901234567890123456789012123456789012345678901 2 2
328 12345678901234567890123456789012123456789012345678901
123456789012345678901234567890121234567890123456789012

www.petersons.com
Chapter

8
Closing Remarks
123456789012345678901234567890121234567890123456789012
12345678901234567890123456789012123456789012345678901 2
12345678901234567890123456789012123456789012345678901 2
12345678901234567890123456789012123456789012345678901
Congratulations! You have just read through a mountain of GRE-related 2
2345678901234567890123456789012123456789012345678901
12345678901234567890123456789012123456789012345678901
information! Consider yourself an expert on a subject, but don’t go 2
2
12345678901234567890123456789012123456789012345678901
bragging about it at social functions because it’s not likely to impress 2
12345678901234567890123456789012123456789012345678901
12345678901234567890123456789012123456789012345678901 2
12345678901234567890123456789012123456789012345678901 2
12345678901234567890123456789012123456789012345678901 22
people. Believe me; I speak from experience.
1
12345678901234567890123456789012123456789012345678901
If you have studied all the strategies and techniques, learned the formats 2
12345678901234567890123456789012123456789012345678901 2
12345678901234567890123456789012123456789012345678901 2
12345678901234567890123456789012123456789012345678901 22
2345678901234567890123456789012123456789012345678901
for each section, and practiced on the sample questions, then you should
12345678901234567890123456789012123456789012345678901
feel pretty good about your chances on the real test. There’s only so much 2
12345678901234567890123456789012123456789012345678901
preparation you can do, and if you’ve finished this book, you’ve done 2
12345678901234567890123456789012123456789012345678901
2
12345678901234567890123456789012123456789012345678901
more than most people taking the test. Be confident about your chances on 2
12345678901234567890123456789012123456789012345678901
12345678901234567890123456789012123456789012345678901 2
the real test, because at the very least, this book has shown you what to 2
12345678901234567890123456789012123456789012345678901
12345678901234567890123456789012123456789012345678901 22
1 expect on the GRE. You would be surprised at how people’s scores
12345678901234567890123456789012123456789012345678901 2
12345678901234567890123456789012123456789012345678901
improve when a test leaves the realm of “mysterious, unexpected exam” 2
12345678901234567890123456789012123456789012345678901 2
12345678901234567890123456789012123456789012345678901
and becomes “just another standardized test.” 2
12345678901234567890123456789012123456789012345678901
2345678901234567890123456789012123456789012345678901
2
That’s all the GRE is, really. By now, you should realize that the GRE says 2
12345678901234567890123456789012123456789012345678901
12345678901234567890123456789012123456789012345678901 2
1 nothing about your potential as a graduate student or your abilities as a 2
12345678901234567890123456789012123456789012345678901 2
12345678901234567890123456789012123456789012345678901 2
12345678901234567890123456789012123456789012345678901 22
2345678901234567890123456789012123456789012345678901
scholar. It’s little more than a hoop you must pass through to get into
12345678901234567890123456789012123456789012345678901
1 graduate school, testing you in subjects that have a bearing on higher 2
12345678901234567890123456789012123456789012345678901
education in only the vaguest, most general sense. If you do well on the 2
12345678901234567890123456789012123456789012345678901 2
2345678901234567890123456789012123456789012345678901
12345678901234567890123456789012123456789012345678901 2
12345678901234567890123456789012123456789012345678901
test, then you’re a good standardized test-taker, nothing more. 2
12345678901234567890123456789012123456789012345678901 22
12345678901234567890123456789012123456789012345678901
1 I would wish you luck on the GRE, but if you’ve read this book, then you 2
12345678901234567890123456789012123456789012345678901
will have made your own luck. Your GRE score will reflect it. 2
12345678901234567890123456789012123456789012345678901 2
12345678901234567890123456789012123456789012345678901 2
12345678901234567890123456789012123456789012345678901 2
12345678901234567890123456789012123456789012345678901 2
12345678901234567890123456789012123456789012345678901 22
2345678901234567890123456789012123456789012345678901
12345678901234567890123456789012123456789012345678901 2
12345678901234567890123456789012123456789012345678901 2
12345678901234567890123456789012123456789012345678901 2
1
12345678901234567890123456789012123456789012345678901 2
12345678901234567890123456789012123456789012345678901 2
12345678901234567890123456789012123456789012345678901 2
12345678901234567890123456789012123456789012345678901 2
12345678901234567890123456789012123456789012345678901 2
12345678901234567890123456789012123456789012345678901 2
12345678901234567890123456789012123456789012345678901 2
12345678901234567890123456789012123456789012345678901 2
12345678901234567890123456789012123456789012345678901 2
123456789012345678901234567890121234567890123456789012
329
PART

V
Appendix
About the CAT 332

PART V
About the CAT

123456789012345678901234567890121234567890123456789012
12345678901234567890123456789012123456789012345678901 2
12345678901234567890123456789012123456789012345678901
2345678901234567890123456789012123456789012345678901
2
2
1 to Apply for the GRE
How
12345678901234567890123456789012123456789012345678901 2
12345678901234567890123456789012123456789012345678901 2
12345678901234567890123456789012123456789012345678901
The best way to learn all the relevant facts about the GRE is to go on line 2
12345678901234567890123456789012123456789012345678901 2
12345678901234567890123456789012123456789012345678901 2
2345678901234567890123456789012123456789012345678901
to the official GRE Web site, www.gre.org. There, you will find a list of
testing centers in your area, registration information, stuff about score 2
12345678901234567890123456789012123456789012345678901
12345678901234567890123456789012123456789012345678901 2
12345678901234567890123456789012123456789012345678901 2
12345678901234567890123456789012123456789012345678901 2
reporting, and scads of other GRE-related trivia. If you don’t want to go
12345678901234567890123456789012123456789012345678901 2
1 on line because you don’t like computers, you should keep in mind that it’s 2
12345678901234567890123456789012123456789012345678901
a computer-based test you are about to take. It’s time to start familiarizing 2
12345678901234567890123456789012123456789012345678901 2
12345678901234567890123456789012123456789012345678901
yourself. There is a phone number to call during business hours, 2
12345678901234567890123456789012123456789012345678901 2
2345678901234567890123456789012123456789012345678901
609-771-7670, but the recorded voice there will harangue you to use the 2
12345678901234567890123456789012123456789012345678901
12345678901234567890123456789012123456789012345678901 2
12345678901234567890123456789012123456789012345678901
Web site. You can eventually get a person, but it normally takes a while. 2
12345678901234567890123456789012123456789012345678901 2
12345678901234567890123456789012123456789012345678901
If you do want to speak to a person, a better approach is to call someone at 2
2
12345678901234567890123456789012123456789012345678901
12345678901234567890123456789012123456789012345678901
a local testing center. Since you may potentially give them your money, 2
2
12345678901234567890123456789012123456789012345678901
1 they are often more amenable to answering any questions you might have. 2
12345678901234567890123456789012123456789012345678901 2
12345678901234567890123456789012123456789012345678901 2
12345678901234567890123456789012123456789012345678901 2
12345678901234567890123456789012123456789012345678901 2
12345678901234567890123456789012123456789012345678901
The GRE CAT Interface 2
12345678901234567890123456789012123456789012345678901 2
12345678901234567890123456789012123456789012345678901 2
12345678901234567890123456789012123456789012345678901
The three simulated screen shots on pages 333, 335, and 336 show the 2
12345678901234567890123456789012123456789012345678901 2
12345678901234567890123456789012123456789012345678901
GRE CAT interface for the Analytical Writing sections, the Quantitative 2
2345678901234567890123456789012123456789012345678901
section, and the Verbal section. Let’s first examine the features of the 2
12345678901234567890123456789012123456789012345678901
12345678901234567890123456789012123456789012345678901 2
1 interface that are common to all exam sections. 2
12345678901234567890123456789012123456789012345678901 2
12345678901234567890123456789012123456789012345678901 2
12345678901234567890123456789012123456789012345678901 2
12345678901234567890123456789012123456789012345678901 2
12345678901234567890123456789012123456789012345678901
The CAT Title Bar 2
12345678901234567890123456789012123456789012345678901 2
12345678901234567890123456789012123456789012345678901 2
12345678901234567890123456789012123456789012345678901 2
2345678901234567890123456789012123456789012345678901
A dark title bar will appear across the top of the computer screen at all
times during all test sections. (You cannot hide this bar.) The CAT title bar 2
12345678901234567890123456789012123456789012345678901
12345678901234567890123456789012123456789012345678901 2
12345678901234567890123456789012123456789012345678901 2
1 displays three items: 2
12345678901234567890123456789012123456789012345678901 2
2345678901234567890123456789012123456789012345678901
• Left corner: The time remaining for the current section (hours and 2
12345678901234567890123456789012123456789012345678901
12345678901234567890123456789012123456789012345678901 2 2
12345678901234567890123456789012123456789012345678901
minutes)
2
1
2345678901234567890123456789012123456789012345678901
12345678901234567890123456789012123456789012345678901 2
1 • Middle: The name of the test (GRE) and current section number 2
12345678901234567890123456789012123456789012345678901 2
12345678901234567890123456789012123456789012345678901
• 2
12345678901234567890123456789012123456789012345678901
Right corner: The current question
2345678901234567890123456789012123456789012345678901
number and total number of 2
2
1
12345678901234567890123456789012123456789012345678901
questions in the current section 2
12345678901234567890123456789012123456789012345678901 2
12345678901234567890123456789012123456789012345678901 2
123456789012345678901234567890121234567890123456789012
332
About the CAT

123456789012345678901234567890121234567890123456789012
12345678901234567890123456789012123456789012345678901 2
12345678901234567890123456789012123456789012345678901 2
12345678901234567890123456789012123456789012345678901 2
12345678901234567890123456789012123456789012345678901 2
12345678901234567890123456789012123456789012345678901 2
Name and Number

12345678901234567890123456789012123456789012345678901 2
Time elapsed CAT Title Bar of Text Section

12345678901234567890123456789012123456789012345678901 2
12345678901234567890123456789012123456789012345678901 2
12345678901234567890123456789012123456789012345678901
00:28 Computer-Adaptive GRE-Section 2:Analytical Writing 2
2
12345678901234567890123456789012123456789012345678901 2
12345678901234567890123456789012123456789012345678901 2
12345678901234567890123456789012123456789012345678901
2345678901234567890123456789012123456789012345678901
2
Beginning

2
1
12345678901234567890123456789012123456789012345678901 2
The following appeared in a memo from the manager of UpperCuts, a hair salon

12345678901234567890123456789012123456789012345678901
located in a suburb of the city of Apton, to the salon's owner:
2
12345678901234567890123456789012123456789012345678901 2
12345678901234567890123456789012123456789012345678901
"According to a nationwide
2345678901234567890123456789012123456789012345678901
demographic study, more and more people today are 2
2
1 AWA Topic
12345678901234567890123456789012123456789012345678901 2
moving from suburbs to downtown areas. So in order to boost sagging profits at

12345678901234567890123456789012123456789012345678901 2
UpperCuts we should relocate the salon from its current location in Apton's

12345678901234567890123456789012123456789012345678901
suburban mall to downtown Apton, while retaining the salon's decidedly upscale
2
12345678901234567890123456789012123456789012345678901
approach in terms of services, products and pricing. After all, HairDooz, our chief
2345678901234567890123456789012123456789012345678901
2
2
1 competitor at the mall, has just relocated downtown and is populr among
12345678901234567890123456789012123456789012345678901 2
12345678901234567890123456789012123456789012345678901 2
12345678901234567890123456789012123456789012345678901
The manager's argument relies on a series of unproven assumptions and is
2
12345678901234567890123456789012123456789012345678901 2
The AWA Cut
therefore unconvincing as it stands. To begin with, the argument assumes that
2345678901234567890123456789012123456789012345678901
12345678901234567890123456789012123456789012345678901
Editing
2
12345678901234567890123456789012123456789012345678901 2
Apton's demographic trend reflects the national trend. Yet, the mere fact that
Screen
Paste
12345678901234567890123456789012123456789012345678901 22
one hair salon has moved downtown hardly suffices to |

12345678901234567890123456789012123456789012345678901 2
12345678901234567890123456789012123456789012345678901 2 Undo

12345678901234567890123456789012123456789012345678901 2
1
12345678901234567890123456789012123456789012345678901 2
12345678901234567890123456789012123456789012345678901 2
Test Section ? Answer

12345678901234567890123456789012123456789012345678901 2
Quit Exit Time Help Confirm Next

2345678901234567890123456789012123456789012345678901
12345678901234567890123456789012123456789012345678901 2
12345678901234567890123456789012123456789012345678901 Help Confirm Next 2
12345678901234567890123456789012123456789012345678901 2
Quit Exit Time
button Answer button
12345678901234567890123456789012123456789012345678901 2
Test Selection button

12345678901234567890123456789012123456789012345678901 22
button button button

12345678901234567890123456789012123456789012345678901 2
12345678901234567890123456789012123456789012345678901 2
12345678901234567890123456789012123456789012345678901 2
12345678901234567890123456789012123456789012345678901 2
1The CAT Toolbar
12345678901234567890123456789012123456789012345678901 2
12345678901234567890123456789012123456789012345678901
A series of six buttons appear in a toolbar across the bottom of the 2
12345678901234567890123456789012123456789012345678901
computer screen at all times during all test sections. (You cannot hide the 2
12345678901234567890123456789012123456789012345678901 2
12345678901234567890123456789012123456789012345678901 2
12345678901234567890123456789012123456789012345678901 22
2345678901234567890123456789012123456789012345678901
toolbar.) Here’s a description of each button’s function:
12345678901234567890123456789012123456789012345678901 2
1
12345678901234567890123456789012123456789012345678901
QUIT TEST 2
12345678901234567890123456789012123456789012345678901 2
12345678901234567890123456789012123456789012345678901 22
2345678901234567890123456789012123456789012345678901
Click on this button to stop the test and cancel your scores for the
12345678901234567890123456789012123456789012345678901
entire test. (Partial score cancellation is not allowed in any event.) 2
1
12345678901234567890123456789012123456789012345678901
If you click here, a dialog box will appear on the screen, asking you 2
12345678901234567890123456789012123456789012345678901 2
2345678901234567890123456789012123456789012345678901
to confirm this operation. Stay away from this button unless you’re 2
12345678901234567890123456789012123456789012345678901
12345678901234567890123456789012123456789012345678901 2
absolutely sure you wish your GRE score for the day to vaporize 2
12345678901234567890123456789012123456789012345678901
12345678901234567890123456789012123456789012345678901 22
1 and you’re willing to throw away your GRE registration fee.
12345678901234567890123456789012123456789012345678901 2
12345678901234567890123456789012123456789012345678901 2
12345678901234567890123456789012123456789012345678901
EXIT SECTION 2
12345678901234567890123456789012123456789012345678901 2
12345678901234567890123456789012123456789012345678901 2
Click on this button if you finish the section before the allotted
2345678901234567890123456789012123456789012345678901
time expires and wish to proceed immediately to the next section. 2
12345678901234567890123456789012123456789012345678901
12345678901234567890123456789012123456789012345678901 2
12345678901234567890123456789012123456789012345678901 2
12345678901234567890123456789012123456789012345678901 22
A dialog box will appear on the screen asking you to confirm this
1
12345678901234567890123456789012123456789012345678901
operation. Stay away from this button unless you’ve already 2
12345678901234567890123456789012123456789012345678901
answered every question in the current section and don’t feel you 2
12345678901234567890123456789012123456789012345678901 2
12345678901234567890123456789012123456789012345678901 2
12345678901234567890123456789012123456789012345678901
need a breather before starting
2345678901234567890123456789012123456789012345678901
the next one! 2
2
1
12345678901234567890123456789012123456789012345678901 2
12345678901234567890123456789012123456789012345678901 2
12345678901234567890123456789012123456789012345678901 2 333
123456789012345678901234567890121234567890123456789012
Part V: Appendix

123456789012345678901234567890121234567890123456789012
12345678901234567890123456789012123456789012345678901 2
12345678901234567890123456789012123456789012345678901 2
12345678901234567890123456789012123456789012345678901
TIME 2
12345678901234567890123456789012123456789012345678901 2
12345678901234567890123456789012123456789012345678901
Click on this button to display the time remaining to the nearest 2
12345678901234567890123456789012123456789012345678901 2
12345678901234567890123456789012123456789012345678901
second. By default, the time remaining is displayed (in the upper 2
2345678901234567890123456789012123456789012345678901 2
1
12345678901234567890123456789012123456789012345678901
left corner) in hours and minutes, but not to the nearest second. 2
12345678901234567890123456789012123456789012345678901 2
12345678901234567890123456789012123456789012345678901 2
12345678901234567890123456789012123456789012345678901
HELP
2345678901234567890123456789012123456789012345678901
2
1 Click on this button to access the directions for the current 2
12345678901234567890123456789012123456789012345678901 2
12345678901234567890123456789012123456789012345678901 2
12345678901234567890123456789012123456789012345678901
question type (for example, Data Sufficiency or Sentence Comple- 2
12345678901234567890123456789012123456789012345678901
2345678901234567890123456789012123456789012345678901
2
2
1 tion), as well as the general test directions and the instructions for
12345678901234567890123456789012123456789012345678901 2
12345678901234567890123456789012123456789012345678901
using the toolbar items. 2
12345678901234567890123456789012123456789012345678901 2
12345678901234567890123456789012123456789012345678901
NEXT and CONFIRM ANSWER
2345678901234567890123456789012123456789012345678901
2
2
1
12345678901234567890123456789012123456789012345678901
Click on the NEXT button when you’re finished with the current 2
12345678901234567890123456789012123456789012345678901 2
12345678901234567890123456789012123456789012345678901
question. When you click on NEXT, the current question will 2
12345678901234567890123456789012123456789012345678901 2
12345678901234567890123456789012123456789012345678901
remain on the screen until you click on CONFIRM ANSWER. 2
12345678901234567890123456789012123456789012345678901 2
12345678901234567890123456789012123456789012345678901
Until you confirm, you can change your answer as often as you 2
12345678901234567890123456789012123456789012345678901 2
12345678901234567890123456789012123456789012345678901
wish (by clicking on a different oval). But once you confirm, the 2
2345678901234567890123456789012123456789012345678901
question disappears forever and the next one appears in its place. 2
12345678901234567890123456789012123456789012345678901
1 2
12345678901234567890123456789012123456789012345678901 2
12345678901234567890123456789012123456789012345678901
Whenever the NEXT button is enabled (appearing dark gray), the 2
12345678901234567890123456789012123456789012345678901 2
12345678901234567890123456789012123456789012345678901 2
2345678901234567890123456789012123456789012345678901
CONFIRM ANSWER button is disabled (appearing light gray),
12345678901234567890123456789012123456789012345678901 2
12345678901234567890123456789012123456789012345678901
and vice versa. 2
12345678901234567890123456789012123456789012345678901 2
12345678901234567890123456789012123456789012345678901 2
12345678901234567890123456789012123456789012345678901 2
12345678901234567890123456789012123456789012345678901 2
12345678901234567890123456789012123456789012345678901
The Analytical Writing Screen 2
12345678901234567890123456789012123456789012345678901 2
1 As illustrated in the screen shot on page 333, the Analytical Writing 2
12345678901234567890123456789012123456789012345678901 2
2345678901234567890123456789012123456789012345678901
12345678901234567890123456789012123456789012345678901 2
1 prompt appears at the top of your screen, and your essay response appears 2
12345678901234567890123456789012123456789012345678901
below it as you type your response. (The screen in the figure includes the 2
12345678901234567890123456789012123456789012345678901
2345678901234567890123456789012123456789012345678901
2
first several lines of a response.) Notice that you have to scroll down to 2
12345678901234567890123456789012123456789012345678901
12345678901234567890123456789012123456789012345678901 2
1 read the entire topic and question. You compose your essays using the 2
12345678901234567890123456789012123456789012345678901 2
12345678901234567890123456789012123456789012345678901
CAT word processor. (Just ahead, you’ll look closely at its features and 2
2345678901234567890123456789012123456789012345678901
12345678901234567890123456789012123456789012345678901 2
12345678901234567890123456789012123456789012345678901
limitations.) 2
1 2
12345678901234567890123456789012123456789012345678901 2
12345678901234567890123456789012123456789012345678901 2
12345678901234567890123456789012123456789012345678901 2
12345678901234567890123456789012123456789012345678901
The Quantitative and Verbal Screens 2
12345678901234567890123456789012123456789012345678901 2
12345678901234567890123456789012123456789012345678901 2
12345678901234567890123456789012123456789012345678901 2
To respond to multiple-choice questions, click on one of the ovals to
2345678901234567890123456789012123456789012345678901
the left of the answer choices. You can’t use the keyboard to select 2
12345678901234567890123456789012123456789012345678901
12345678901234567890123456789012123456789012345678901 2
12345678901234567890123456789012123456789012345678901 2
12345678901234567890123456789012123456789012345678901 2
answers. Notice that the answer choices are not lettered; you’ll click on
2
1 blank ovals.
12345678901234567890123456789012123456789012345678901 2
12345678901234567890123456789012123456789012345678901 2
12345678901234567890123456789012123456789012345678901 2
12345678901234567890123456789012123456789012345678901 2
12345678901234567890123456789012123456789012345678901 2
12345678901234567890123456789012123456789012345678901 2
2345678901234567890123456789012123456789012345678901
2
1
12345678901234567890123456789012123456789012345678901 2
12345678901234567890123456789012123456789012345678901 2
12345678901234567890123456789012123456789012345678901 2
12345678901234567890123456789012123456789012345678901 2
12345678901234567890123456789012123456789012345678901 2
12345678901234567890123456789012123456789012345678901 2 2
334 12345678901234567890123456789012123456789012345678901
123456789012345678901234567890121234567890123456789012

www.petersons.com
About the CAT

123456789012345678901234567890121234567890123456789012
12345678901234567890123456789012123456789012345678901 2
12345678901234567890123456789012123456789012345678901 2
12345678901234567890123456789012123456789012345678901 2
12345678901234567890123456789012123456789012345678901 2
12345678901234567890123456789012123456789012345678901 2

X-Ref
12345678901234567890123456789012123456789012345678901 2
12345678901234567890123456789012123456789012345678901
In the sample questions throughout this book, the answer choices were 2
12345678901234567890123456789012123456789012345678901 2
12345678901234567890123456789012123456789012345678901
lettered for easy reference to corresponding explanations. 2
12345678901234567890123456789012123456789012345678901 2
12345678901234567890123456789012123456789012345678901 2
12345678901234567890123456789012123456789012345678901
2345678901234567890123456789012123456789012345678901
2
2
1
12345678901234567890123456789012123456789012345678901 2
12345678901234567890123456789012123456789012345678901 2
12345678901234567890123456789012123456789012345678901
Split screens. For some multiple-choice questions, the screen splits 2
12345678901234567890123456789012123456789012345678901 2
12345678901234567890123456789012123456789012345678901
either horizontally or vertically. 2
12345678901234567890123456789012123456789012345678901 2
12345678901234567890123456789012123456789012345678901 2
12345678901234567890123456789012123456789012345678901
Reading Comprehension: The screen splits vertically. The left side 2
12345678901234567890123456789012123456789012345678901
2345678901234567890123456789012123456789012345678901
2
2
1 displays the passage; the right side displays the question and
12345678901234567890123456789012123456789012345678901 2
12345678901234567890123456789012123456789012345678901
answer choices. 2
12345678901234567890123456789012123456789012345678901 2
12345678901234567890123456789012123456789012345678901 2
12345678901234567890123456789012123456789012345678901 22
2345678901234567890123456789012123456789012345678901
Quantitative questions that include figures: The screen splits
12345678901234567890123456789012123456789012345678901
horizontally. The figures appears at the top; the question and 2
12345678901234567890123456789012123456789012345678901
12345678901234567890123456789012123456789012345678901
answer choices appear at the bottom. 2
12345678901234567890123456789012123456789012345678901 2
12345678901234567890123456789012123456789012345678901
Vertical Scrolling. For some multiple-choice questions, you’ll have to 2
2
1
12345678901234567890123456789012123456789012345678901 2
12345678901234567890123456789012123456789012345678901
scroll up and down (using the vertical scroll bar) to view all the 2
12345678901234567890123456789012123456789012345678901 2
12345678901234567890123456789012123456789012345678901
material that pertains to the current question. 2
12345678901234567890123456789012123456789012345678901 2
12345678901234567890123456789012123456789012345678901 2
12345678901234567890123456789012123456789012345678901
Reading Comprehension: Passages are too long for you to see on 2
12345678901234567890123456789012123456789012345678901 2
12345678901234567890123456789012123456789012345678901 22
2345678901234567890123456789012123456789012345678901
the screen in their entirety; you’ll need to scroll.
12345678901234567890123456789012123456789012345678901 2
12345678901234567890123456789012123456789012345678901
Quantitative questions that include figures: Some figures— 2
12345678901234567890123456789012123456789012345678901
1 especially charts and graphs—won’t fit on the screen in their 2
12345678901234567890123456789012123456789012345678901 2
12345678901234567890123456789012123456789012345678901
entirety; you might need to scroll. 2
12345678901234567890123456789012123456789012345678901 2
12345678901234567890123456789012123456789012345678901 2
12345678901234567890123456789012123456789012345678901 2
12345678901234567890123456789012123456789012345678901
01:06 Computer-Adaptive GRE-Section 3:Quantitative 21 of 37
2
12345678901234567890123456789012123456789012345678901 2
12345678901234567890123456789012123456789012345678901 2
12345678901234567890123456789012123456789012345678901 2
12345678901234567890123456789012123456789012345678901 2
12345678901234567890123456789012123456789012345678901 22
2345678901234567890123456789012123456789012345678901
12345678901234567890123456789012123456789012345678901 2
1
12345678901234567890123456789012123456789012345678901 2
12345678901234567890123456789012123456789012345678901 2
Richard began driving from home on a trip averaging 30 miles per hour. How
12345678901234567890123456789012123456789012345678901 2
many miles per hour must Carla drive on average to catch up to him in exactly

12345678901234567890123456789012123456789012345678901
3 hours if she leaves 30 minutes after Richard?
2
12345678901234567890123456789012123456789012345678901 2
12345678901234567890123456789012123456789012345678901 2
12345678901234567890123456789012123456789012345678901 2
35

12345678901234567890123456789012123456789012345678901 22
2345678901234567890123456789012123456789012345678901 55

12345678901234567890123456789012123456789012345678901 2 39

12345678901234567890123456789012123456789012345678901 2 40
12345678901234567890123456789012123456789012345678901 2
1
60

12345678901234567890123456789012123456789012345678901 2
12345678901234567890123456789012123456789012345678901 2
12345678901234567890123456789012123456789012345678901 2
12345678901234567890123456789012123456789012345678901 2
12345678901234567890123456789012123456789012345678901 2
12345678901234567890123456789012123456789012345678901 22
2345678901234567890123456789012123456789012345678901
1
12345678901234567890123456789012123456789012345678901 2
12345678901234567890123456789012123456789012345678901 2
12345678901234567890123456789012123456789012345678901 ? Answer
2
Test Section

12345678901234567890123456789012123456789012345678901
Quit Exit Time
2
Help Confirm Next

12345678901234567890123456789012123456789012345678901 2
12345678901234567890123456789012123456789012345678901 2
12345678901234567890123456789012123456789012345678901 2 335
123456789012345678901234567890121234567890123456789012
Part V: Appendix

123456789012345678901234567890121234567890123456789012
12345678901234567890123456789012123456789012345678901 2
12345678901234567890123456789012123456789012345678901 2
12345678901234567890123456789012123456789012345678901 2
12345678901234567890123456789012123456789012345678901 2
12345678901234567890123456789012123456789012345678901
01:09 Computer-Adaptive GRE-Section 4:Verbal 6 of 41
2
12345678901234567890123456789012123456789012345678901 2
12345678901234567890123456789012123456789012345678901
2345678901234567890123456789012123456789012345678901
2
2
1
Questions 6 to 8 Beginning

12345678901234567890123456789012123456789012345678901
The Andean cordillera is made up of 2
In the passage, the author's primary concern is
12345678901234567890123456789012123456789012345678901
many interwoven mountain ranges, which to 2
12345678901234567890123456789012123456789012345678901
include high intermountain plateaus,
2
12345678901234567890123456789012123456789012345678901
basins and valleys. The Northern Andes
2345678901234567890123456789012123456789012345678901
2
describe the climate and topography of
2
1 (5) contain several broad ecosystems falling various regions of the Andean cordillera
12345678901234567890123456789012123456789012345678901
into four altitudinal belts. Its northern sub-
2
12345678901234567890123456789012123456789012345678901 2
discuss the factors affecting the climate of
region is distinguished from the rest of the

12345678901234567890123456789012123456789012345678901 2
the Andean cordillera
region by higher relative humidity and

12345678901234567890123456789012123456789012345678901 2
suggest various alternative explanations for
greater climatic symmetry between the

12345678901234567890123456789012123456789012345678901 2
the diversity of climate among the
(10) eastern and western flanks of the range.

12345678901234567890123456789012123456789012345678901 2
various regions of the Andean cordillera
The Central Andes are characterized by a

12345678901234567890123456789012123456789012345678901 2
examine the effects of topograpy on the
succession of agricultural zones with

12345678901234567890123456789012123456789012345678901 2
climate and vegetation of the Andean
varied climatic conditions along the

12345678901234567890123456789012123456789012345678901 2
cordillera
mountains' flanks and by large, high-

12345678901234567890123456789012123456789012345678901 2
compare and contrast the climate and
(15) altitude plateaus, variously called puna or

12345678901234567890123456789012123456789012345678901 2
topography of the Northern Andes to that
altiplano, which do not occur in the

12345678901234567890123456789012123456789012345678901 2
the Central Andes
Northern Andes. The soil fertility of the

12345678901234567890123456789012123456789012345678901
northern altiplano is generally good. The
2
12345678901234567890123456789012123456789012345678901
western Central Andean ranges are
2
12345678901234567890123456789012123456789012345678901 2
2345678901234567890123456789012123456789012345678901
(20) relatively arid with desert-like soils,

12345678901234567890123456789012123456789012345678901 2
whereas the eastern ranges are more

12345678901234567890123456789012123456789012345678901 2
humid and have more diverse soils. The

12345678901234567890123456789012123456789012345678901 2
eastern slopes of the Central Andes in

12345678901234567890123456789012123456789012345678901 2
many ways are similar to the wet forests

12345678901234567890123456789012123456789012345678901
Test Section ? Answer 2
1 2
12345678901234567890123456789012123456789012345678901 2
Quit Exit Time Help Confirm Next

12345678901234567890123456789012123456789012345678901 2
12345678901234567890123456789012123456789012345678901 2
12345678901234567890123456789012123456789012345678901 2
2345678901234567890123456789012123456789012345678901
12345678901234567890123456789012123456789012345678901 2
12345678901234567890123456789012123456789012345678901
The CAT’s Word Processor 2
12345678901234567890123456789012123456789012345678901 2
12345678901234567890123456789012123456789012345678901 2
During the two GRE essays section, you’ll use the simple word processor
built into the CAT system. While the word processor includes some 2
12345678901234567890123456789012123456789012345678901
12345678901234567890123456789012123456789012345678901 2
12345678901234567890123456789012123456789012345678901 2
12345678901234567890123456789012123456789012345678901 2
features standard in programs like Word and WordPerfect, it also lacks
2
1 many of these programs’ features.
12345678901234567890123456789012123456789012345678901 2
12345678901234567890123456789012123456789012345678901 2
12345678901234567890123456789012123456789012345678901 2
12345678901234567890123456789012123456789012345678901 2
12345678901234567890123456789012123456789012345678901 2
12345678901234567890123456789012123456789012345678901
Keyboard Commands for Navigation and Editing 2
12345678901234567890123456789012123456789012345678901 2
12345678901234567890123456789012123456789012345678901
Here are the navigational and editing keys available in the CAT word 2
12345678901234567890123456789012123456789012345678901 2
12345678901234567890123456789012123456789012345678901
processor: 2
12345678901234567890123456789012123456789012345678901 2
2345678901234567890123456789012123456789012345678901
12345678901234567890123456789012123456789012345678901 2
1 • Backspace removes the character to the left of the cursor. 2
12345678901234567890123456789012123456789012345678901 2
12345678901234567890123456789012123456789012345678901 2
12345678901234567890123456789012123456789012345678901
• Delete removes the character to the right of the cursor. 2
12345678901234567890123456789012123456789012345678901 2
12345678901234567890123456789012123456789012345678901
• 2
12345678901234567890123456789012123456789012345678901
Home moves the cursor to the beginning of the line. 2
12345678901234567890123456789012123456789012345678901 2
2345678901234567890123456789012123456789012345678901
12345678901234567890123456789012123456789012345678901
• End moves the cursor to the end of the line. 2
12345678901234567890123456789012123456789012345678901 2
12345678901234567890123456789012123456789012345678901 2 2
12345678901234567890123456789012123456789012345678901
• Arrow Keys move the cursor up, down, left, or right.
2
1
2345678901234567890123456789012123456789012345678901
12345678901234567890123456789012123456789012345678901 2
12345678901234567890123456789012123456789012345678901
• Enter inserts a paragraph break (starts a new line). 2
12345678901234567890123456789012123456789012345678901 2 2
12345678901234567890123456789012123456789012345678901
• Page Up moves the cursor up one page (screen). 2
1
12345678901234567890123456789012123456789012345678901 2
2345678901234567890123456789012123456789012345678901
2
1 • Page Down moves the cursor down one page (screen).
12345678901234567890123456789012123456789012345678901 2
12345678901234567890123456789012123456789012345678901 2
12345678901234567890123456789012123456789012345678901 2
12345678901234567890123456789012123456789012345678901 2
12345678901234567890123456789012123456789012345678901 2
12345678901234567890123456789012123456789012345678901 2 2
336 12345678901234567890123456789012123456789012345678901
123456789012345678901234567890121234567890123456789012

www.petersons.com
About the CAT

123456789012345678901234567890121234567890123456789012
12345678901234567890123456789012123456789012345678901 2
12345678901234567890123456789012123456789012345678901 2
12345678901234567890123456789012123456789012345678901
Certain often-used features of standard word processing programs are not 2
12345678901234567890123456789012123456789012345678901 2
12345678901234567890123456789012123456789012345678901
available in the CAT word processor. For example, no keyboard 2
12345678901234567890123456789012123456789012345678901 2
12345678901234567890123456789012123456789012345678901
commands are available for:
2345678901234567890123456789012123456789012345678901
2
2
1
12345678901234567890123456789012123456789012345678901 2
12345678901234567890123456789012123456789012345678901
• TAB—disabled (does not function) 2
12345678901234567890123456789012123456789012345678901 2
12345678901234567890123456789012123456789012345678901

2345678901234567890123456789012123456789012345678901
2
2
1 Beginning/end of paragraph (not available)
12345678901234567890123456789012123456789012345678901 2
12345678901234567890123456789012123456789012345678901
• Beginning/end of document (not available) 2
12345678901234567890123456789012123456789012345678901 2
12345678901234567890123456789012123456789012345678901
2345678901234567890123456789012123456789012345678901
2
2
1 • No key combinations (using the CTRL, ALT, or SHIFT key) or other
12345678901234567890123456789012123456789012345678901 2
12345678901234567890123456789012123456789012345678901
so-called “macros” are available for editing functions. (You’ll use 2
12345678901234567890123456789012123456789012345678901 2
12345678901234567890123456789012123456789012345678901
your mouse for cutting and pasting text.)
2345678901234567890123456789012123456789012345678901
2
2
1
12345678901234567890123456789012123456789012345678901 2
12345678901234567890123456789012123456789012345678901 2
12345678901234567890123456789012123456789012345678901 2
12345678901234567890123456789012123456789012345678901
Mouse-Driven Navigation and Editing Functions 2
12345678901234567890123456789012123456789012345678901 22
2345678901234567890123456789012123456789012345678901
12345678901234567890123456789012123456789012345678901
Just as with other word processors, to navigate the editing screen you can 2
12345678901234567890123456789012123456789012345678901
simply point the cursor to the position at which you wish to begin typing, 2
12345678901234567890123456789012123456789012345678901
12345678901234567890123456789012123456789012345678901 2
12345678901234567890123456789012123456789012345678901 2
1 then click. The CAT word processor also includes mouse-driven CUT, 2
12345678901234567890123456789012123456789012345678901
PASTE, and UNDO. 2
12345678901234567890123456789012123456789012345678901 2
12345678901234567890123456789012123456789012345678901 2
2345678901234567890123456789012123456789012345678901
Selecting text you wish to cut. You select text the same way as with 2
12345678901234567890123456789012123456789012345678901
12345678901234567890123456789012123456789012345678901 2
standard word processing programs: either (1) hold down your mouse 2
12345678901234567890123456789012123456789012345678901
12345678901234567890123456789012123456789012345678901
button while sweeping the I-beam on the screen over the desired text, or 2
2
12345678901234567890123456789012123456789012345678901
12345678901234567890123456789012123456789012345678901 2
12345678901234567890123456789012123456789012345678901
(2) hold down the SHIFT key and use the navigation keys to select text. 2
12345678901234567890123456789012123456789012345678901 22
12345678901234567890123456789012123456789012345678901
1 The CUT Button. If you wish to delete text but want to save it to a 2
12345678901234567890123456789012123456789012345678901
temporary clipboard for pasting elsewhere, select that text (see above) then 2
12345678901234567890123456789012123456789012345678901 2
12345678901234567890123456789012123456789012345678901 2
12345678901234567890123456789012123456789012345678901 2
click on the CUT button. Cutting text is not the same as deleting it. When
12345678901234567890123456789012123456789012345678901
you delete text (using the DELETE key), you cannot paste it elsewhere in 2
12345678901234567890123456789012123456789012345678901 2
12345678901234567890123456789012123456789012345678901 2
12345678901234567890123456789012123456789012345678901 2
your document (but see UNDO below).
12345678901234567890123456789012123456789012345678901 2
12345678901234567890123456789012123456789012345678901
The PASTE button. If you wish to move text from one position to another, 2
12345678901234567890123456789012123456789012345678901 22
2345678901234567890123456789012123456789012345678901
12345678901234567890123456789012123456789012345678901
1 select and cut the text, then reposition your cursor where you want the text 2
12345678901234567890123456789012123456789012345678901
to go and click on the PASTE button. 2
12345678901234567890123456789012123456789012345678901 2
2345678901234567890123456789012123456789012345678901
12345678901234567890123456789012123456789012345678901
The UNDO button. Click on this button to undo the most recent delete, 2
2
12345678901234567890123456789012123456789012345678901 2
12345678901234567890123456789012123456789012345678901 2
12345678901234567890123456789012123456789012345678901
cut, or paste that you performed.
2
1
2345678901234567890123456789012123456789012345678901
Limitations of CUT and UNDO. The following mouse-driven features are 2
12345678901234567890123456789012123456789012345678901
12345678901234567890123456789012123456789012345678901 2
12345678901234567890123456789012123456789012345678901 2
12345678901234567890123456789012123456789012345678901 22
not available:
1
12345678901234567890123456789012123456789012345678901
• DRAG-AND-DROP cut-and-paste (not available) 2
12345678901234567890123456789012123456789012345678901 2
12345678901234567890123456789012123456789012345678901 2
12345678901234567890123456789012123456789012345678901
• COPY (not available; to copy you need to cut, then paste, in the 2
12345678901234567890123456789012123456789012345678901 2
12345678901234567890123456789012123456789012345678901
same spot) 2
12345678901234567890123456789012123456789012345678901 2
12345678901234567890123456789012123456789012345678901
• MULTIPLE UNDO (the CAT word processor stores only your most 2
12345678901234567890123456789012123456789012345678901 2
12345678901234567890123456789012123456789012345678901
2345678901234567890123456789012123456789012345678901
2
2
1 recent delete, cut, or paste, or keyboard entry.)
12345678901234567890123456789012123456789012345678901 2
12345678901234567890123456789012123456789012345678901 2
12345678901234567890123456789012123456789012345678901 2 337
123456789012345678901234567890121234567890123456789012
Part V: Appendix

123456789012345678901234567890121234567890123456789012
12345678901234567890123456789012123456789012345678901 2
12345678901234567890123456789012123456789012345678901 2
12345678901234567890123456789012123456789012345678901
The vertical scroll bar. Once you key in ten lines or so, you’ll have to scroll 2
12345678901234567890123456789012123456789012345678901 2
12345678901234567890123456789012123456789012345678901
to view your entire response. A vertical scroll bar also appears to the right 2
12345678901234567890123456789012123456789012345678901 2
12345678901234567890123456789012123456789012345678901
of the AWA prompt. Be sure to scroll all the way down to make sure you’ve 2
2345678901234567890123456789012123456789012345678901 2
1 read the entire prompt.
12345678901234567890123456789012123456789012345678901 2
12345678901234567890123456789012123456789012345678901 2
12345678901234567890123456789012123456789012345678901 2
12345678901234567890123456789012123456789012345678901
Spell-checking, grammar-checking, fonts, attributes, hyphenation. The 2
1 CAT word processor does not include a spell-checker or grammar-checker, 2
2345678901234567890123456789012123456789012345678901
12345678901234567890123456789012123456789012345678901 2
12345678901234567890123456789012123456789012345678901 2
12345678901234567890123456789012123456789012345678901
nor does it allow you to choose typeface or point size. Neither manual nor 2
12345678901234567890123456789012123456789012345678901
2345678901234567890123456789012123456789012345678901
2
2
1 automatic hyphenation is available. Attributes such as bold, italics, and
12345678901234567890123456789012123456789012345678901 2
12345678901234567890123456789012123456789012345678901
underlining are not available. 2
12345678901234567890123456789012123456789012345678901 2
12345678901234567890123456789012123456789012345678901
2345678901234567890123456789012123456789012345678901
2
2
1
12345678901234567890123456789012123456789012345678901 2

Note
12345678901234567890123456789012123456789012345678901
As for words that you would otherwise italicize or underline (such as 2
12345678901234567890123456789012123456789012345678901 2
12345678901234567890123456789012123456789012345678901
titles or non-English words), it’s okay to leave them as is. The readers 2
2345678901234567890123456789012123456789012345678901
12345678901234567890123456789012123456789012345678901 2
12345678901234567890123456789012123456789012345678901
understand the limitations of the CAT word processor. 2
12345678901234567890123456789012123456789012345678901 2
12345678901234567890123456789012123456789012345678901 2
12345678901234567890123456789012123456789012345678901 2
12345678901234567890123456789012123456789012345678901 2
1 GRE CAT Test-Taking Experience
The 2
12345678901234567890123456789012123456789012345678901 2
12345678901234567890123456789012123456789012345678901
When you take a test as important as the GRE, it’s a good idea to minimize 2
12345678901234567890123456789012123456789012345678901 2
12345678901234567890123456789012123456789012345678901
test anxiety by knowing exactly what to expect on exam day—aside from 2
12345678901234567890123456789012123456789012345678901 2
12345678901234567890123456789012123456789012345678901
the timed test itself. Let’s walk you through the various pre-test and 2
12345678901234567890123456789012123456789012345678901 2
12345678901234567890123456789012123456789012345678901
post-test procedures and describe the physical testing environment. 2
12345678901234567890123456789012123456789012345678901 2
2345678901234567890123456789012123456789012345678901
12345678901234567890123456789012123456789012345678901 2
12345678901234567890123456789012123456789012345678901 2
12345678901234567890123456789012123456789012345678901 2 2
1When You Arrive at the Test Center
12345678901234567890123456789012123456789012345678901 2
12345678901234567890123456789012123456789012345678901 2
12345678901234567890123456789012123456789012345678901 2
Here’s what you can expect when you arrive at the test center:
12345678901234567890123456789012123456789012345678901 2
12345678901234567890123456789012123456789012345678901
• The supervisor will show you a roster, which includes the names 2
12345678901234567890123456789012123456789012345678901 2
12345678901234567890123456789012123456789012345678901 2
12345678901234567890123456789012123456789012345678901 2
of test-takers scheduled for that day, and will ask you to initial
12345678901234567890123456789012123456789012345678901
the roster next to your name, and indicate on the roster your 2
12345678901234567890123456789012123456789012345678901 2
2345678901234567890123456789012123456789012345678901
12345678901234567890123456789012123456789012345678901
arrival time. 2
12345678901234567890123456789012123456789012345678901 2
1 2
12345678901234567890123456789012123456789012345678901
• The supervisor will ask you to read a two-page list of testing 2
12345678901234567890123456789012123456789012345678901 2
12345678901234567890123456789012123456789012345678901
procedures and rules. (I’ll cover all these rules in the pages 2
12345678901234567890123456789012123456789012345678901 2
12345678901234567890123456789012123456789012345678901
immediately ahead.) 2
12345678901234567890123456789012123456789012345678901 2
12345678901234567890123456789012123456789012345678901 2
12345678901234567890123456789012123456789012345678901 2
2345678901234567890123456789012123456789012345678901
• The supervisor will give you a “Nondisclosure Statement.” You’re
to read the printed statement, then write the statement (in the space 2
12345678901234567890123456789012123456789012345678901
12345678901234567890123456789012123456789012345678901 2
12345678901234567890123456789012123456789012345678901 2
1 provided on the form) and sign it. In the statement, you agree to the 2
2345678901234567890123456789012123456789012345678901
testing policies and rules, and you agree not to reproduce or disclose 2
12345678901234567890123456789012123456789012345678901
12345678901234567890123456789012123456789012345678901 2
12345678901234567890123456789012123456789012345678901 2
12345678901234567890123456789012123456789012345678901 2
any of the actual test questions. The supervisor will not permit you
1 to enter the exam room until you’ve written and signed the 2
12345678901234567890123456789012123456789012345678901 2
12345678901234567890123456789012123456789012345678901
statement. 2
12345678901234567890123456789012123456789012345678901 2
12345678901234567890123456789012123456789012345678901 2
12345678901234567890123456789012123456789012345678901 2
12345678901234567890123456789012123456789012345678901 2
12345678901234567890123456789012123456789012345678901 2
12345678901234567890123456789012123456789012345678901 2 2
338 12345678901234567890123456789012123456789012345678901
123456789012345678901234567890121234567890123456789012

www.petersons.com
About the CAT

123456789012345678901234567890121234567890123456789012
12345678901234567890123456789012123456789012345678901 2
12345678901234567890123456789012123456789012345678901 2
12345678901234567890123456789012123456789012345678901
• You’ll probably have to sit in a waiting room until the supervisor 2
12345678901234567890123456789012123456789012345678901 2
12345678901234567890123456789012123456789012345678901
calls your name. A 5- to 10-minute wait beyond your scheduled 2
12345678901234567890123456789012123456789012345678901 2
12345678901234567890123456789012123456789012345678901
testing time is not uncommon. (Taking the GRE is like going to the 2
2345678901234567890123456789012123456789012345678901 2
1
12345678901234567890123456789012123456789012345678901
dentist—in more than one respect!) 2
12345678901234567890123456789012123456789012345678901 2
12345678901234567890123456789012123456789012345678901 2
12345678901234567890123456789012123456789012345678901
• The supervisor will check your photo
2345678901234567890123456789012123456789012345678901
identification. (You won’t be 2
1 permitted to take the test unless you have one acceptable form of 2
12345678901234567890123456789012123456789012345678901 2
12345678901234567890123456789012123456789012345678901 2
12345678901234567890123456789012123456789012345678901
photo identification with you.) 2
12345678901234567890123456789012123456789012345678901
2345678901234567890123456789012123456789012345678901
2
1 • The test center will provide a secure locker (free of charge) for 2
12345678901234567890123456789012123456789012345678901 2
12345678901234567890123456789012123456789012345678901
stowing your personal belongings during the test. 2
12345678901234567890123456789012123456789012345678901 2
12345678901234567890123456789012123456789012345678901
2345678901234567890123456789012123456789012345678901
2
2
1 • To help ensure that nobody else takes any part of the exam in your
12345678901234567890123456789012123456789012345678901 2
12345678901234567890123456789012123456789012345678901
place, the supervisor will take a photograph of you. 2
12345678901234567890123456789012123456789012345678901 2
12345678901234567890123456789012123456789012345678901 2
12345678901234567890123456789012123456789012345678901 22
2345678901234567890123456789012123456789012345678901
• The supervisor might give you some rudimentary tips about
12345678901234567890123456789012123456789012345678901
managing your time during the exam. Just ignore the supervisor’s 2
12345678901234567890123456789012123456789012345678901
12345678901234567890123456789012123456789012345678901 2
12345678901234567890123456789012123456789012345678901
tips, because they might not be good advice for you! 2
12345678901234567890123456789012123456789012345678901 22
1 • Before you enter the testing room, you must remove everything from 2
12345678901234567890123456789012123456789012345678901
12345678901234567890123456789012123456789012345678901
your pockets except your photo I.D. and locker key. 2
12345678901234567890123456789012123456789012345678901 2
12345678901234567890123456789012123456789012345678901 22
2345678901234567890123456789012123456789012345678901
12345678901234567890123456789012123456789012345678901
• The supervisor will provide you with several pieces of scratch paper
12345678901234567890123456789012123456789012345678901
(stapled together), along with two pencils. These are the only items 2
2
12345678901234567890123456789012123456789012345678901
12345678901234567890123456789012123456789012345678901 2
12345678901234567890123456789012123456789012345678901
you’ll have in hand as you enter the testing room. 2
12345678901234567890123456789012123456789012345678901 22
12345678901234567890123456789012123456789012345678901 2
12345678901234567890123456789012123456789012345678901 2
1
12345678901234567890123456789012123456789012345678901
Testing Procedures and Rules 2
12345678901234567890123456789012123456789012345678901 2
12345678901234567890123456789012123456789012345678901
• 2
12345678901234567890123456789012123456789012345678901 2
If you want to exit the testing room for any reason, you must raise your
12345678901234567890123456789012123456789012345678901
hand and wait for the supervisor to come in and escort you from the 2
12345678901234567890123456789012123456789012345678901 2
12345678901234567890123456789012123456789012345678901 2
12345678901234567890123456789012123456789012345678901 2
room. (You won’t be able to pause the testing clock for any reason.)
12345678901234567890123456789012123456789012345678901 2
12345678901234567890123456789012123456789012345678901
• No guests are allowed in the waiting room during your test. 2
12345678901234567890123456789012123456789012345678901 22
2345678901234567890123456789012123456789012345678901
12345678901234567890123456789012123456789012345678901 2
1 • No food or drink is allowed in the testing room.
12345678901234567890123456789012123456789012345678901 2
12345678901234567890123456789012123456789012345678901 2
12345678901234567890123456789012123456789012345678901
• No hats are allowed. 2
12345678901234567890123456789012123456789012345678901 2
12345678901234567890123456789012123456789012345678901
• 2
12345678901234567890123456789012123456789012345678901
You must sign out whenever you exit the testing room. 2
12345678901234567890123456789012123456789012345678901 2
2345678901234567890123456789012123456789012345678901
• You must sign in whenever you re-enter the testing room (the supervisor 2
12345678901234567890123456789012123456789012345678901
12345678901234567890123456789012123456789012345678901 2
12345678901234567890123456789012123456789012345678901
will ask to see your photo I.D. each time). 2
12345678901234567890123456789012123456789012345678901 22
1 • If you need more scratch paper during the exam, just raise your hand
12345678901234567890123456789012123456789012345678901 2
12345678901234567890123456789012123456789012345678901 2
12345678901234567890123456789012123456789012345678901
and ask for it. The supervisor will happily replace your bundle with a 2
12345678901234567890123456789012123456789012345678901 2
12345678901234567890123456789012123456789012345678901
fresh one. 2
12345678901234567890123456789012123456789012345678901 22
2345678901234567890123456789012123456789012345678901
1 • The supervisor will replace your tired pencils with fresh, sharp ones
12345678901234567890123456789012123456789012345678901 2
12345678901234567890123456789012123456789012345678901 2
12345678901234567890123456789012123456789012345678901
upon your request anytime during the exam
2345678901234567890123456789012123456789012345678901
(just raise your hand). 2
2
1
12345678901234567890123456789012123456789012345678901 2
12345678901234567890123456789012123456789012345678901 2
12345678901234567890123456789012123456789012345678901 2 339
123456789012345678901234567890121234567890123456789012
Part V: Appendix

123456789012345678901234567890121234567890123456789012
12345678901234567890123456789012123456789012345678901 2
12345678901234567890123456789012123456789012345678901 2
12345678901234567890123456789012123456789012345678901
What You Should Know about the CAT Testing 2
12345678901234567890123456789012123456789012345678901 2
12345678901234567890123456789012123456789012345678901
Environment 2
12345678901234567890123456789012123456789012345678901 2
12345678901234567890123456789012123456789012345678901
• Individual testing stations are like library carrels; they’re separated by 2
12345678901234567890123456789012123456789012345678901 2
12345678901234567890123456789012123456789012345678901
half-walls. 2
12345678901234567890123456789012123456789012345678901 2
12345678901234567890123456789012123456789012345678901 2
12345678901234567890123456789012123456789012345678901
• The height of your chair’s seat will be
2345678901234567890123456789012123456789012345678901
adjustable and the chair will 2
2
1 swivel. Chairs at most testing centers have arms.
12345678901234567890123456789012123456789012345678901 2
12345678901234567890123456789012123456789012345678901 2
12345678901234567890123456789012123456789012345678901 2
12345678901234567890123456789012123456789012345678901
• Computer monitors are generally of the 15-inch variety. You can adjust 2
1 contrast. If you notice any flickering, ask the supervisor to move you to 2
2345678901234567890123456789012123456789012345678901
12345678901234567890123456789012123456789012345678901 2
12345678901234567890123456789012123456789012345678901 2
12345678901234567890123456789012123456789012345678901
another station. (You won’t be able to tell if you monitor has color 2
12345678901234567890123456789012123456789012345678901
2345678901234567890123456789012123456789012345678901
2
2
1 capability, because the GRE is strictly a black-and-white affair.)
12345678901234567890123456789012123456789012345678901 2
12345678901234567890123456789012123456789012345678901 2
12345678901234567890123456789012123456789012345678901 2
12345678901234567890123456789012123456789012345678901 2

Alert!
2345678901234567890123456789012123456789012345678901
You can’t change the size of the font on the screen, unless you specifically 2
12345678901234567890123456789012123456789012345678901
12345678901234567890123456789012123456789012345678901 2
request before the exam begins that a special ZOOMTEXT function be 2
12345678901234567890123456789012123456789012345678901
12345678901234567890123456789012123456789012345678901 2
12345678901234567890123456789012123456789012345678901
made available to you. 2
12345678901234567890123456789012123456789012345678901 2
1 2
12345678901234567890123456789012123456789012345678901 2
12345678901234567890123456789012123456789012345678901 2
12345678901234567890123456789012123456789012345678901 2
12345678901234567890123456789012123456789012345678901 2
2345678901234567890123456789012123456789012345678901
• If your mouse has two buttons, you can use either button to click your
12345678901234567890123456789012123456789012345678901
way through the exam (both buttons serve the same function). Don’t 2
2
12345678901234567890123456789012123456789012345678901
12345678901234567890123456789012123456789012345678901
expect that nifty wheel between buttons for easy scrolling, because 2
2
12345678901234567890123456789012123456789012345678901
12345678901234567890123456789012123456789012345678901
you’re not going to get it. For all you gamers and laptop users, trackballs 2
2
12345678901234567890123456789012123456789012345678901
12345678901234567890123456789012123456789012345678901 2
12345678901234567890123456789012123456789012345678901
are available, but only if you request one before you begin the test. 2
1 2
2345678901234567890123456789012123456789012345678901
• Testing rooms are not soundproof. During your test, you might hear 2
12345678901234567890123456789012123456789012345678901
12345678901234567890123456789012123456789012345678901 2
1 talking and other noise from outside the room. 2
12345678901234567890123456789012123456789012345678901 2
12345678901234567890123456789012123456789012345678901 2
12345678901234567890123456789012123456789012345678901
• Expect the supervisor to escort other test-takers in and out of the room 2
12345678901234567890123456789012123456789012345678901 2
12345678901234567890123456789012123456789012345678901
during your test. Do your best to ignore this potential distraction. 2
12345678901234567890123456789012123456789012345678901 2
12345678901234567890123456789012123456789012345678901 2
12345678901234567890123456789012123456789012345678901 2
2345678901234567890123456789012123456789012345678901
• If the testing room is busy, expect to hear lots of mouse-clicking during
12345678901234567890123456789012123456789012345678901 2
1 your test. Because the room is otherwise fairly quiet, the incessant 2
12345678901234567890123456789012123456789012345678901 2
12345678901234567890123456789012123456789012345678901
mouse-clicking can become annoying! 2
12345678901234567890123456789012123456789012345678901 2
2345678901234567890123456789012123456789012345678901
2
12345678901234567890123456789012123456789012345678901
• Earplugs are available upon request. 2
12345678901234567890123456789012123456789012345678901
12345678901234567890123456789012123456789012345678901 2 2
1 • Expect anything in terms of room temperature, so dress in layers.
12345678901234567890123456789012123456789012345678901 2
12345678901234567890123456789012123456789012345678901 2
12345678901234567890123456789012123456789012345678901
• You’ll be under continual audio and video surveillance. To guard 2
12345678901234567890123456789012123456789012345678901 2
12345678901234567890123456789012123456789012345678901
against cheating, and to record any irregularities or problems in the 2
2345678901234567890123456789012123456789012345678901
12345678901234567890123456789012123456789012345678901
testing room as they occur, the room is continually audiotaped and 2
2
12345678901234567890123456789012123456789012345678901 2
12345678901234567890123456789012123456789012345678901
videotaped. (Look for the cameras or two-way mirrors, then smile and 2
12345678901234567890123456789012123456789012345678901 2
1 wave!)
12345678901234567890123456789012123456789012345678901 2
12345678901234567890123456789012123456789012345678901 2
12345678901234567890123456789012123456789012345678901 2
12345678901234567890123456789012123456789012345678901 2
12345678901234567890123456789012123456789012345678901 2
12345678901234567890123456789012123456789012345678901 2
12345678901234567890123456789012123456789012345678901 2
12345678901234567890123456789012123456789012345678901 2 2
340 12345678901234567890123456789012123456789012345678901
123456789012345678901234567890121234567890123456789012

www.petersons.com
About the CAT

123456789012345678901234567890121234567890123456789012
12345678901234567890123456789012123456789012345678901 2
12345678901234567890123456789012123456789012345678901 2
12345678901234567890123456789012123456789012345678901
Before You Begin the Test—The Computer Tutorial 2
12345678901234567890123456789012123456789012345678901 2
12345678901234567890123456789012123456789012345678901 2
12345678901234567890123456789012123456789012345678901 2
Okay, the supervisor has just escorted you into the inner sanctum and to
12345678901234567890123456789012123456789012345678901
your station, and has wished you luck. (Some supervisors have been 2
12345678901234567890123456789012123456789012345678901 2
12345678901234567890123456789012123456789012345678901
known to encourage test-takers to “have fun!”) Before you begin the test, 2
12345678901234567890123456789012123456789012345678901 2
12345678901234567890123456789012123456789012345678901
the CAT System will lead you through a tutorial which includes five 2
12345678901234567890123456789012123456789012345678901 2
12345678901234567890123456789012123456789012345678901
sections (each section steps you through a series of “screens”): 2
12345678901234567890123456789012123456789012345678901 2
12345678901234567890123456789012123456789012345678901 2
12345678901234567890123456789012123456789012345678901
1. How to use the mouse (6 screens) 2
12345678901234567890123456789012123456789012345678901
2345678901234567890123456789012123456789012345678901
2
2
1
12345678901234567890123456789012123456789012345678901
2. How to select and change an answer (6 screens) 2
12345678901234567890123456789012123456789012345678901 2
12345678901234567890123456789012123456789012345678901 2
12345678901234567890123456789012123456789012345678901
3. How to scroll the screen display up
2345678901234567890123456789012123456789012345678901
and down (6 screens) 2
2
1
12345678901234567890123456789012123456789012345678901
4. How to use the toolbars (21 screens); here you’ll learn how to: 2
12345678901234567890123456789012123456789012345678901 2
12345678901234567890123456789012123456789012345678901 2
12345678901234567890123456789012123456789012345678901
• Quit the test. 2
12345678901234567890123456789012123456789012345678901 22
2345678901234567890123456789012123456789012345678901
12345678901234567890123456789012123456789012345678901
12345678901234567890123456789012123456789012345678901
• Exit the current section. 2
12345678901234567890123456789012123456789012345678901 22
12345678901234567890123456789012123456789012345678901
• Access the directions.
12345678901234567890123456789012123456789012345678901 2
1 2
12345678901234567890123456789012123456789012345678901
• Confirm your response and move to the next question. 2
12345678901234567890123456789012123456789012345678901 2
12345678901234567890123456789012123456789012345678901 2
2345678901234567890123456789012123456789012345678901
12345678901234567890123456789012123456789012345678901
5. How to use the AWA word processor features (14 screens) 2
12345678901234567890123456789012123456789012345678901 2
12345678901234567890123456789012123456789012345678901 2
12345678901234567890123456789012123456789012345678901
Here’s what you need to know about the CAT tutorial: 2
12345678901234567890123456789012123456789012345678901 22
12345678901234567890123456789012123456789012345678901
• You won’t be able to skip any section or any screen during 2
12345678901234567890123456789012123456789012345678901
12345678901234567890123456789012123456789012345678901 2
12345678901234567890123456789012123456789012345678901
the tutorial. 2
1 2
2345678901234567890123456789012123456789012345678901
• As you progress, the system requires that you demonstrate 2
12345678901234567890123456789012123456789012345678901
12345678901234567890123456789012123456789012345678901 2
1 competency in using the mouse, selecting and confirming answer 2
12345678901234567890123456789012123456789012345678901 2
12345678901234567890123456789012123456789012345678901
choices, and accessing the directions. So you can’t begin taking the 2
12345678901234567890123456789012123456789012345678901 2
12345678901234567890123456789012123456789012345678901
actual test unless you’ve shown that you know how to use the 2
12345678901234567890123456789012123456789012345678901 2
12345678901234567890123456789012123456789012345678901
system. (Don’t worry: no test-taker has ever flunked the CAT system 2
12345678901234567890123456789012123456789012345678901 2
2345678901234567890123456789012123456789012345678901
12345678901234567890123456789012123456789012345678901
competency test.) 2
12345678901234567890123456789012123456789012345678901 22
1
12345678901234567890123456789012123456789012345678901
• At the end of each tutorial section (series of screens), you can repeat 2
12345678901234567890123456789012123456789012345678901
that section, at your option. But once you leave a section you can’t 2
12345678901234567890123456789012123456789012345678901 2
12345678901234567890123456789012123456789012345678901 2
12345678901234567890123456789012123456789012345678901
return to it. 2
12345678901234567890123456789012123456789012345678901 2
12345678901234567890123456789012123456789012345678901 2
12345678901234567890123456789012123456789012345678901 22
2345678901234567890123456789012123456789012345678901
12345678901234567890123456789012123456789012345678901 2
Alert!

12345678901234567890123456789012123456789012345678901 2
12345678901234567890123456789012123456789012345678901
Don’t choose to repeat any tutorial section. Why not? If you do, you’ll be
2
1 forced to step through the entire sequence of screens in that section again
12345678901234567890123456789012123456789012345678901 22
2345678901234567890123456789012123456789012345678901
12345678901234567890123456789012123456789012345678901 2
12345678901234567890123456789012123456789012345678901
(an aggravating time-waster, especially for the 21-screen section!)
12345678901234567890123456789012123456789012345678901 22
1
12345678901234567890123456789012123456789012345678901 2
12345678901234567890123456789012123456789012345678901 2
12345678901234567890123456789012123456789012345678901 2
12345678901234567890123456789012123456789012345678901 2
12345678901234567890123456789012123456789012345678901 2
12345678901234567890123456789012123456789012345678901 2
12345678901234567890123456789012123456789012345678901 2
12345678901234567890123456789012123456789012345678901 2
12345678901234567890123456789012123456789012345678901 2 341
123456789012345678901234567890121234567890123456789012
Part V: Appendix

123456789012345678901234567890121234567890123456789012
12345678901234567890123456789012123456789012345678901 2
12345678901234567890123456789012123456789012345678901 2
12345678901234567890123456789012123456789012345678901
• The Analytical Writing section of the tutorial allows you to practice 2
12345678901234567890123456789012123456789012345678901 2
12345678901234567890123456789012123456789012345678901
using the word processor. 2
12345678901234567890123456789012123456789012345678901 2
12345678901234567890123456789012123456789012345678901
• If you carefully read all the information presented to you, expect to 2
12345678901234567890123456789012123456789012345678901 2
12345678901234567890123456789012123456789012345678901 2
12345678901234567890123456789012123456789012345678901
spend about 20 minutes on the tutorial. 2
12345678901234567890123456789012123456789012345678901 2
12345678901234567890123456789012123456789012345678901 2
1 On test day, you’ll already know how the CAT system works. So step 2
2345678901234567890123456789012123456789012345678901
12345678901234567890123456789012123456789012345678901 2
12345678901234567890123456789012123456789012345678901 2
12345678901234567890123456789012123456789012345678901
through the tutorial as quickly as you can, reading as little as possible. 2

Tip
12345678901234567890123456789012123456789012345678901
2345678901234567890123456789012123456789012345678901
2
2
1 You can easily dispense with the tutorial in 5–10 minutes this way.
12345678901234567890123456789012123456789012345678901 2
12345678901234567890123456789012123456789012345678901
Remember: The less time you spend with the tutorial, the less fatigued 2
12345678901234567890123456789012123456789012345678901 2
12345678901234567890123456789012123456789012345678901
you’ll be during the exam itself.
2345678901234567890123456789012123456789012345678901
2
2
1
12345678901234567890123456789012123456789012345678901 2
12345678901234567890123456789012123456789012345678901 2
12345678901234567890123456789012123456789012345678901
Post-Test GRE CAT Procedures 2
12345678901234567890123456789012123456789012345678901 2
12345678901234567890123456789012123456789012345678901 2
2345678901234567890123456789012123456789012345678901
Okay, it’s been about 4 hours since you first entered the testing center, and
12345678901234567890123456789012123456789012345678901
you’ve just completed the second of two multiple-choice GRE sections. You 2
2
12345678901234567890123456789012123456789012345678901
may think you’ve finished the CAT, but the CAT has not quite finished with 2
12345678901234567890123456789012123456789012345678901
12345678901234567890123456789012123456789012345678901 2
12345678901234567890123456789012123456789012345678901 2
1 you yet! There are a few more hoops to jump through before you’re done. 2
12345678901234567890123456789012123456789012345678901 2
12345678901234567890123456789012123456789012345678901 2
12345678901234567890123456789012123456789012345678901
1. Respond to a brief questionnaire. The CAT will impose on you a 2
2345678901234567890123456789012123456789012345678901
brief questionnaire (presented in a series of screens) about your 2
12345678901234567890123456789012123456789012345678901
12345678901234567890123456789012123456789012345678901 2
12345678901234567890123456789012123456789012345678901 2
12345678901234567890123456789012123456789012345678901 2
test-taking experience (believe it or not, these questions are
multiple-choice, just like the exam itself). The questionnaire 2
12345678901234567890123456789012123456789012345678901
2
12345678901234567890123456789012123456789012345678901
12345678901234567890123456789012123456789012345678901
might ask you, for example: 2
12345678901234567890123456789012123456789012345678901 2
12345678901234567890123456789012123456789012345678901 2
1 • Whether your supervisor was knowledgeable and helpful 2
12345678901234567890123456789012123456789012345678901 2
2345678901234567890123456789012123456789012345678901
12345678901234567890123456789012123456789012345678901 2
1 • Whether the testing environment was comfortable 2
12345678901234567890123456789012123456789012345678901 2
12345678901234567890123456789012123456789012345678901
• How long you waited after you arrived at the testing site to 2
12345678901234567890123456789012123456789012345678901 2
12345678901234567890123456789012123456789012345678901 2
12345678901234567890123456789012123456789012345678901 2
begin the test
12345678901234567890123456789012123456789012345678901 2
12345678901234567890123456789012123456789012345678901
• Whether you were distracted by noise during your exam 2
12345678901234567890123456789012123456789012345678901 2
2345678901234567890123456789012123456789012345678901
12345678901234567890123456789012123456789012345678901
2. Cancel your test, at your option. The most important question 2
2
1
12345678901234567890123456789012123456789012345678901 2
12345678901234567890123456789012123456789012345678901
you’ll answer while seated at your testing station is this next one. 2
2345678901234567890123456789012123456789012345678901
12345678901234567890123456789012123456789012345678901 2
12345678901234567890123456789012123456789012345678901
The CAT will ask you to choose whether to: 2
12345678901234567890123456789012123456789012345678901 2
1 • Cancel your scores (no scores are recorded; partial 2
12345678901234567890123456789012123456789012345678901 2
2345678901234567890123456789012123456789012345678901
cancellation is not provided for) or see your scores 2
12345678901234567890123456789012123456789012345678901
12345678901234567890123456789012123456789012345678901 2
12345678901234567890123456789012123456789012345678901 2
12345678901234567890123456789012123456789012345678901 2
immediately.
2
1
12345678901234567890123456789012123456789012345678901 2
12345678901234567890123456789012123456789012345678901 2
12345678901234567890123456789012123456789012345678901 2
12345678901234567890123456789012123456789012345678901 2
12345678901234567890123456789012123456789012345678901 2
Alert!

2345678901234567890123456789012123456789012345678901
12345678901234567890123456789012123456789012345678901 2
1 If you click on the CANCEL SCORES button, the CAT will then give you 2
12345678901234567890123456789012123456789012345678901
yet another 5 minutes to think over your decision. So you really have 10 2
12345678901234567890123456789012123456789012345678901 2
12345678901234567890123456789012123456789012345678901
minutes altogether to make up your mind.
2345678901234567890123456789012123456789012345678901
2
2
1
12345678901234567890123456789012123456789012345678901 2
12345678901234567890123456789012123456789012345678901 2 2
342 12345678901234567890123456789012123456789012345678901
123456789012345678901234567890121234567890123456789012

www.petersons.com
About the CAT

123456789012345678901234567890121234567890123456789012
12345678901234567890123456789012123456789012345678901 2
12345678901234567890123456789012123456789012345678901 2
12345678901234567890123456789012123456789012345678901
Once you elect to see your scores, you can no longer cancel them—ever! So 2
12345678901234567890123456789012123456789012345678901 2
12345678901234567890123456789012123456789012345678901
you should take a few minutes to think it over. The CAT gives you 5 2
12345678901234567890123456789012123456789012345678901 2
12345678901234567890123456789012123456789012345678901
minutes to choose. If you haven’t decided within 5 minutes, the CAT will 2
1 automatically show you your scores (and you forfeit your option to 2
2345678901234567890123456789012123456789012345678901
12345678901234567890123456789012123456789012345678901 2
12345678901234567890123456789012123456789012345678901 2
12345678901234567890123456789012123456789012345678901
cancel). 2
12345678901234567890123456789012123456789012345678901
2345678901234567890123456789012123456789012345678901
2
1 3. View and record your scores. If you elect to see your scores, you 2
12345678901234567890123456789012123456789012345678901 2
12345678901234567890123456789012123456789012345678901 2
12345678901234567890123456789012123456789012345678901
should write them down on your scratch paper. When you leave 2
12345678901234567890123456789012123456789012345678901
2345678901234567890123456789012123456789012345678901
2
2
1 the testing room, the supervisor will allow you to transcribe them
12345678901234567890123456789012123456789012345678901 2
12345678901234567890123456789012123456789012345678901
onto another sheet of paper (one that you can take home with 2
12345678901234567890123456789012123456789012345678901
you), so that you don’t have to memorize them. 2
12345678901234567890123456789012123456789012345678901
2345678901234567890123456789012123456789012345678901
2
2
1
12345678901234567890123456789012123456789012345678901
4. Direct your scores to the schools of your choice. Once you’ve 2
12345678901234567890123456789012123456789012345678901 2
12345678901234567890123456789012123456789012345678901
elected to see your scores, the CAT will ask you to select the 2
12345678901234567890123456789012123456789012345678901 2
12345678901234567890123456789012123456789012345678901
schools you wish to receive your score report (the CAT provides a 2
12345678901234567890123456789012123456789012345678901 2
12345678901234567890123456789012123456789012345678901
complete list of schools). 2
12345678901234567890123456789012123456789012345678901 2
12345678901234567890123456789012123456789012345678901 2
2345678901234567890123456789012123456789012345678901
12345678901234567890123456789012123456789012345678901 2
1 You can select as many as five schools at this time—without incurring an 2
12345678901234567890123456789012123456789012345678901 2
Tip

12345678901234567890123456789012123456789012345678901
additional fee. This is your last chance for a freebie, so you should take 2
12345678901234567890123456789012123456789012345678901 2
12345678901234567890123456789012123456789012345678901 22
2345678901234567890123456789012123456789012345678901
full advantage of it. Be sure to compile your list of five schools before
12345678901234567890123456789012123456789012345678901
12345678901234567890123456789012123456789012345678901
exam day. 2
12345678901234567890123456789012123456789012345678901 22
12345678901234567890123456789012123456789012345678901 2
12345678901234567890123456789012123456789012345678901
12345678901234567890123456789012123456789012345678901
Before You Leave the Testing Center 2
12345678901234567890123456789012123456789012345678901 22
12345678901234567890123456789012123456789012345678901
1 Upon exiting the testing room for the final time, the following will happen: 2
12345678901234567890123456789012123456789012345678901 2
12345678901234567890123456789012123456789012345678901
1. The supervisor will collect your pencils and scratch paper, and 2
12345678901234567890123456789012123456789012345678901 2
12345678901234567890123456789012123456789012345678901
will count the number of sheets of paper to make sure you aren’t 2
12345678901234567890123456789012123456789012345678901 2
12345678901234567890123456789012123456789012345678901
trying to sneak out with any. (Then, if you’re lucky, you’ll be 2
12345678901234567890123456789012123456789012345678901
allowed to watch while the supervisor ceremoniously rips up your 2
12345678901234567890123456789012123456789012345678901 2
12345678901234567890123456789012123456789012345678901 2
12345678901234567890123456789012123456789012345678901
scratch paper and drops it in the trash basket!) 2
12345678901234567890123456789012123456789012345678901 22
2345678901234567890123456789012123456789012345678901
12345678901234567890123456789012123456789012345678901
2. The supervisor will remind you to collect your belongings from 2
1
12345678901234567890123456789012123456789012345678901 2
12345678901234567890123456789012123456789012345678901
your locker (if you used one), and turn in your locker key. 2
12345678901234567890123456789012123456789012345678901 22
2345678901234567890123456789012123456789012345678901
12345678901234567890123456789012123456789012345678901
3. The supervisor will provide you with an ETS pamphlet that
2
12345678901234567890123456789012123456789012345678901
explains how to interpret your test scores. (You can take this 2
12345678901234567890123456789012123456789012345678901
1 2
12345678901234567890123456789012123456789012345678901
home with you.) 2
12345678901234567890123456789012123456789012345678901 2
12345678901234567890123456789012123456789012345678901 2
12345678901234567890123456789012123456789012345678901 2
12345678901234567890123456789012123456789012345678901 2
Note

2345678901234567890123456789012123456789012345678901
12345678901234567890123456789012123456789012345678901
The supervisor might also provide you with a postcard-sized invitation to 2
12345678901234567890123456789012123456789012345678901 2
12345678901234567890123456789012123456789012345678901
“blow the whistle” on anybody you suspect of cheating on the exam (the 2
12345678901234567890123456789012123456789012345678901 22
1 invitation ends with the assurance: “Confidentiality guaranteed”).
12345678901234567890123456789012123456789012345678901 2
12345678901234567890123456789012123456789012345678901 2
12345678901234567890123456789012123456789012345678901 2
12345678901234567890123456789012123456789012345678901 2
12345678901234567890123456789012123456789012345678901 2
12345678901234567890123456789012123456789012345678901 2
12345678901234567890123456789012123456789012345678901 2
12345678901234567890123456789012123456789012345678901 2
12345678901234567890123456789012123456789012345678901 2 343
123456789012345678901234567890121234567890123456789012

You might also like